Skip to main content

Full text of "2 Elzohry MRCP Questions On Examination"

See other formats


Part 1 



(18351 Questions) 


MRCPass 


OnExamination 


PassMedicine 


ReviseMRCP 


MRCPstudy 




1 st Edition 



(2475 Questions) 


Khalid Yusuf El-Zohry 

Sohag Teaching Hospital - Egypt 

elzohryxp@yahoo.com 

https://www.facebook.com/elzohryxp 










































El-zohry MRCP Questions Bank (Part 1) - 2013 


(For my personal use) 


Contents 


iLoJ&jO .8 

dfijLj olbo .10 

How to register for MRCP part 1.28 

Reference ranges.31 

MRCPass.33 

[ Q: 1 ] MRCPass - Gastroenterology.35 

[ Q: 141 ] MRCPass - Rheumatology.91 

[ Q: 268 ] MRCPass - Respiratory.141 

[ Q: 389 ] MRCPass - Nephrology.189 

[ Q: 529 ] MRCPass - Neurology.243 

[ Q: 682 ] MRCPass - Infectious disease.299 

[ Q: 840 ] MRCPass - Haematology.365 

[ Q: 999 ] MRCPass - Endocrinology.429 

[ Q: 1155 ] MRCPass - Clinical pharmacology.487 

[ Q: 1287 ] MRCPass - Cardiology.529 

[ Q: 1412 ] MRCPass - Basic Science.573 

[ Q: 1582 ] MRCPass - Dermatology.633 

[ Q: 1611 ] MRCPass - Ophthalmology.645 

[ Q: 1670 ] MRCPass - Psychiatry.667 

[ Q: 1729 ] MRCPass - Statistics.687 

[ Q: 1777 ] MRCPass - 2010 January.703 

[ Q: 1876 ] MRCPass - 2010 May.745 

[ Q: 1976 ] MRCPass - 2010 September.783 

[ Q: 2076 ] MRCPass - 2011 January.825 

[ Q: 2176 ] MRCPass - 2011 May.867 

[ Q: 2275 ] MRCPass - 2011 September.913 

[ Q: 2375 ] MRCPass - 2012 January.955 


Dr. Khalid Yusuf El-Zohry - Sohag Teaching Hospital (01118391123) 

Ref MRCPass OE OE 2012 PasTest 2009 PassMedicine 2009 PasTest Exam ReviseMRCP 



3 













































El-zohry MRCP Questions Bank (Part 1) - 2013 


(For my personal use) 


Dedications 


To my father, 
my mother, 
my wife, 
my sons: 

Abd El-Rahman, 
Muhammed, 
and Amr 

To president Muhammad Mursi 


Dr. Kholid Yusuf El-Zohry - Sohog Teaching Hospital (01118391123) 

Ref MRCPoss OE OE 2012 PosTest 2009 PassMedicine 2009 PosTest Exom ReviseMRCP 















El-zohry MRCP Questions Bank (Part 1) - 2013 


(For my personal use) 





c c c c 

fOu&AlA_Alq fOufirUI 01 UnLul foJ.g.jJ 
fo_ i kS I I Qg LlQjAJ i ULaj lq_JL±> 

oa _LLLfiJI v5U yjJjC. IajI QqJjJj Uq 

" # £ , t 

fo_d^Lbq 0-o IajI cuJaJI Qg Ko i\]g 

C m C 

fO_AlliA ql fO_ nVi i r j iiigl 

f qjJqj f - cuj<)n^o 7 i)l \ju j 1j o^J-o :io-2lo 



li) 4l)l jAp <(J I C-O Ij (J U15 

44 «« 4 44 ♦♦ 


Dr. Kholid Yusuf El-Zohry - Sohog Teaching Hospital (01118391123) 

Ref MRCPoss OE OE 2012 PosTest 2009 PassMedicine 2009 PosTest Exom ReviseMRCP 




























El-zohry MRCP Questions Bank (Part 1) - 2013 


(For my personal use) 



Take the first step, and your mind will mobilize all 

its forces to your aid. 

But 

The first essential is that you begin 

Once the battle is startled, all that is within and 
without you will come to your assistance 



Dr. Kholid Yusuf El-Zohry - Sohog Teaching Hospital (01118391123) 

Ref MRCPoss OE OE 2012 PosTest 2009 PassMedicine 2009 PosTest Exom ReviseMRCP 
















El-zohry MRCP Questions Bank (Part 1) - 2013 


(For my personal use) 


li_o> £lj_ >)) ol o_^ 0_^A_LJ 3_o>JI 

ul5 —Lc v—I Lb J —^ <X —Lo -Uftl —uxj ul o— iAi'b ‘J— 

u —o J—£J ^ I —> j g .—o —s£i <s—>_ c)J I—^aJL> d—La>j 



JUULoJ I J l 



clcji_IL 

■ 



. u/jpJI j£>Uhj 



C 

o i_gj J_ olsJI I j_e) _> I_o ,0^3_C 03->_ >3 ‘cL 9 j _uoMI 

_ *&£i} I3 _ juuJI J-S* |A gJ <y ~^3 I ol j^l I a I tOj^ i^JI 

. JolsJI lift) ^9 

J ull I olj _ !> O _ o JULftji _ uui ul <-Q _LoJI |j _ 0 ) ^ _9 o _ I[ 9 b* 

V—£>^3 ^_T— 9 ul—^ vS->3—6<°° J—*3 ‘ C ^°J— II ,J—O 

c c 

1^9—4 < H0 olj—M> ,_f—9 09 —£4 0)1 o iAi'l —uiiJ[9 

.ioUJI 

■ ■ 


vSjJI lAjulK^j jJL> /3 

JJ^LO ~ — ^y jOuJLgjJ I ^IgD^JUJ JUUUJUUUO 

https://www.facebook.com/elzohryxp 


Dr. Kholid Yusuf El-Zohry - Sohog Teaching Hospital (01118391123) 

Ref MRCPoss OE OE 2012 PosTest 2009 PassMedicine 2009 PosTest Exom ReviseMRCP 




























El-zohry MRCP Questions Bank (Part 1) - 2013 


(For my personal use) 


C 




(Juab jJjQjJ 


Inas 

Mohamed 

Alassar 


Ayman 

Shahin 


Ahmed 

Gabr 


Aqua 

Mariqe 







Reem Ali 






Shiny 

Moon 


Riyadh 

Shalabi 


Black 

House 





IV 

Allah 


Heba 

Mohammed 


Amira 

Hefney 




Faisal 

Hemeda 


Aburas 

Ab 




Dr. Kholid Yusuf El-Zohry - Sohog Teaching Hospital (01118391123) 

Ref MRCPass OE OE 2012 PasTest 2009 PassMedicine 2009 PasTest Exam ReviseMRCP 


9 































































El-zohry MRCP Questions Bank (Part 1) - 2013 


(For my personal use) 


<\su Luj 




<j\ <>.. 4^LJI ja 4JU^I 


1 mi ^1 (U^l ^LJI ^ ajUJJt 


Ojis JjO N^kJI ^ dm>^l djuujAjQJI ^k. gJUol Jl 03 jl Uo_u£ n j^n ll oIaj 
j_c ^yuj^J JQa/qI jl o.>jlg Loaqj jjl£d^I JgjJI yb oikiJI ojg> j.c oalqjuu Lo 
ClsoJI di>j OlJjg ♦♦ dulaJI CXAOjuuJI Olb (Uu>i/I Oblftunll A>U Oll£aI>i/l ^jjJo 

♦♦ ouuuUUall ^gjjkiJI 

ouuuu^l yi> djjjdbJ^I CU^akUl dil ^ 11 djQ.>>r o^lnuuu jl OA>g JjQ.lo Claj AI2j g 

i J I 

^ cvjjdl^Ji/l c\juL»j_kxxll a^l>oJ ■ 
^^yallnJI Ogliuuuall CXJL^aJjOJI djdiaJU OJUL^JI dfljQjuuJI ■ 
.6>o>lnJI ^6 dlLoLJI dJLoj oULdol gjua^i jdmJI ^1 gll^J 2 ■ 
juflj gl v±JLj Lj> ^0 JjoIS oljlax<bl jJaj jl jgjj lo^ilr Jj^-vll giLalmj ■ 

,c\.6^l>g oj-\fo)U jluuuo 

j-uuuul>IjqJI djlctu ^ yO} J^olo ->g>gj ^sr^ni J_£uuuu dujuUuO OjlS ■ 

(}£. J>h\ jl jg3 dJLojJI rlcciil OAikmjul g yj ^yLiLog „ oljgX£>jJI culjjg 

£.*. ■ ♦♦ ♦♦ 

.^blajcuflj 

jjO 1 jjjuuu 0 ssu Ojtwuuuulg lnulc Jg^ojJI jJLc dlioU y L^i>l O^Lquuj JjuQj ■ 

.dilnill 

♦ 

. (Ujjflll JgjJI J\ jnrnJI J> 6-uax) ■ 
Jjl^juulj gil^Tun djjiKi^l dill JI yb jjullL^I O^lcbjuj ^J.£ a.Qj ■ 

d^lnnll ^b viullbg ^JLc jQjAnUlg djulkijjJI xLlLo^I OjLqj yb cXjuudi 

♦ oa^JuqJI 

Jgil g ♦♦ oLoglojoJI ga> dJL >jjo ^fbg dJL >jjo oal^oI oIaj jljdLuui/l xsu 
ClajI dij gi^i olI annuli ptol dbgjo Oljld GwuJL> g CUiLuo dJL> jjqJI 0A£b Oul£> 
flftnO fOUidl JKoJl lA£b yb ^ygimj ^jOjO dJLojJI ^JLc ^jiJLaol^JI {)£. 


\ 


Dr. Kholid Yusuf El-Zohry - Sohog Teaching Hospital (01118391123) 

Ref MRCPoss OE OE 2012 PosTest 2009 PassMedicine 2009 PosTest Exom ReviseMRCP 






































El-zohry MRCP Questions Bank (Part 1) - 2013 


(For my personal use) 




lUuaJi j* .. a^ui ja aju^i 


fQj U) (XOLuuuOJ jjjuuu jl JJjkiJI OAlb JjuO ^6 jjbJI JjOX) ♦ ♦ gpuuoJI ^jjJoJI 
jLjljuj^I g rt-vi-v.^>11 oLoginoJI cJJilD ♦ ♦ lki> ^jjJo ^0 viUI nOujMJ sflniW 
^6 *Lk>MJ ajoloJI djlnUI ^1 Ouibb 9 „ cUJI jbU ghiJI Jjolgc /irbi Lan> J^LaJI 
c\JL>jJI oIaj g obogjALaJI ^jo ajJjo go^l (jjoIujuoII ^*1^)1 jCuiflUl j^jjo) ojiblnJI 

♦* abgWI 


^ ajjulli Jlimil Jib cuibgjl jisl pi a> ^Jl c\JL>jJI OAib ^6 ojJI ^yLn .69 9 

♦♦ 

Cu> */, * Jlnb-I CUjuuuu ^yuuliO jjOj jlgjujudJI rlftul Oflkiialujlg JnoJI 

.♦ ^)g>/l OjjOJI JX> £|J>I CXjMaJI )Lul>I - 0 A >9 CUJI J*ii£U 9 - ^Phuu) 

t ^ aJLojJI £l j>l Jl gikmjulg 




£j >9 /Iflifl £j> Iflub .axb-uc cuJoLJl) ak>l£> albUJI tfyo ifrQiiaJI 

CXjuazxC Lo^JulOjLc UULS ^JUI ggjAJI + c\jaJL> ; nmn £j>g OjLLoUJIj ajOj pJhO 

9 9 9^9 9 9 9 9 

(UujUjjJ/I jlgxUI £ 9 >o uiaflj ^lc Jg^l £_pJI Jaluy 9 (owiiiul 9 ^gJgisLojlfl 

(....9 £Lo>I 9 ^jjiujuLQijgjy 9 ^gJgjuuu 6 9 ^yoglll) 


jxoIajuj 9 gjlo g jjUj j^iiiI ^6 pLoJI oIjjo r aihlol ahiu ♦J* 

- ojg^oJI j^>>aJI 

lo>orn Jxiab^l) ^yuJjjuuul CUi> 0\/^ „ Y * ^ Y (\juuu ^6 JLajuD^I Ju)1£ij ^ 

( 6 jwLx) ^yiujjJI gogxxJI ^ ajI £5 I jib; 
OUL>l o (yX) C\jI>I ouuuJI jIul>I C\JLLulp^I gAO> ♦$♦ 
jjuni ^6 Jlgjuu \ * * aiijg g oLcluu V cxbjg Jis ^jjilojg v _ T b jl^Juoi/l ^ 

(Idle (uilDI aBjgJI pli) V ^1 £ CP9 ^ cr 1 ' " CP P94« 



Dr. Kholid Yusuf El-Zohry - Sohog Teaching Hospital (01118391123) 

Ref MRCPass OE OE 2012 PasTest 2009 PassMedicine 2009 PasTest Exam ReviseMRCP 



11 



















































El-zohry MRCP Questions Bank (Part 1) - 2013 


(For my personal use) 




yij .. a^ui j a-iuajjji aju^t 


: 6j£>IajqJI jjLuojo 

Essential revision notes for MRCP for Philip karla 
Or Oxford handbook of clinical medicine 


Cull i _ 7 JLc jinngrt ^x> dJLuuul £ujjlg jL^JuO^I ^Uoi | _ 7 JLc diuul run*-> jx^l J>+ 

Onexamination and passmedicine 

(<\jl>yi jjjLug Ijjl> AjI> 2I Oiljcg oljo 6 sc dirndl J>) 


jgnuuj l : oujjJU c\jjuuUuqJI ojInJI 


( ^3*3) 


£j>9 *SjjI£ijjju fUhO cj> LQuO ,Q£1>jJlC C\JLLoLJl) dJLolis dilsLJI EQj -0 :gxtuucjl 
(uuaC Irhiiiijlr UJL^ (J JUI £gjnJI + c\jaL> ( aouO £j>g dlLoLdli u^itdJI ajOj ,11010 

9 PJP^9J 9 ^=> 9^9 9 jlQi> 9^9 ja~q 9 

g JgJol dluu^l jl ^6 Jgifl rjiJI Q£ ^oJBj q£iJ g (ji t i ^n il g ^gjgislojlo 

g 0i£ £lo jg^o g 0"j 9 cuokmo g oolc olauil dluui/l ^0 juh ^b jg*o ^ 

.Cua 1> 9 odLo fdJuuj jg^o 

^yb J 4 XLD 9 J g gJgj g JjjjI jouul ^ <oLs 2 JI i _ T 6 oljo r cxihiol aoaj *X* 

.OjgaJiJI ^ilkyjJI i^>aJI 
Ifhoiii J.>in ^l) ^yLdjjuul Cui> .. Y * \ Y duu 1 _ y 6 j Ki'n ^l .nillA'i 

( 6 jujLo ^yQuijJI gdgxiJI ^ I jifli 

OUl>l 0 QjO dil>l ouuuuI jlu>l dluui/l gjua> *X* 

^b dflIDI ddjgJI g ,ogj ^b 0 j£oj9 JI&uj * dojg olijg X jliJuo^l *X* 

. Jlill pgJI 



Dr. Khalid Yusuf El-Zohry - Sohag Teaching Hospital (01118391123) 

Ref MRCPoss OE OE 2012 PosTest 2009 PassMedicine 2009 PosTest Exom ReviseMRCP 









































El-zohry MRCP Questions Bank (Part 1) - 2013 


(For my personal use) 




jo.. a^ui js a-iuajjjji ajujii 


: OjisljjQJI _pL<ajO 


Essential revision notes for MRCP for San jay Sharma 

+ JgHI £jiJI ul£. 

Essential revision notes for MRCP for Philip karla 
Or Oxford handbook of clinical medicine 


CuJI i _ 7 JLc jiOfi grt Qjo dJjuujI Jjujjlg ,oUaj | _ r i£ cULuul r un^ + 

Onexamination and passmedicine 

( fob-HI £jujg Ijo> <ul>2l odjag oljo oxc almjHI J>) 

odlnJI (Qjuuj g rujJbJI g qjlqJI £lo g dnuj^l jg^s > _ 7 lc vjjjaUI + 


.. acgJI U£>g U> ibgls jli= jhlo^l jl g 


J9LHUJ 1 : ojj-uU auuUnJI ojInJI 


(J Su^JLi J^lj jt£s) g>)t 


jjluD-j xfn /->9 jL^Juol jgi^j .rin/-> cub O-Qlflj Ji^-unl^-iJI 

£bgjai v^kij jl ik yuQj| pi^J) ^yakl ,jjutuul odllnJI pjLfij saJIkJI Olj-U) 

( oJgjiuuuJI ^o>iin jnLnTII oLJI 9 UjJLc OjJLSJI ^ Jl^nil^ll jKTnl 

sSjasiO A>lg jii V/nn oajLuuI Y OjuaJ J_£d jl Ingnr jKTn^l pLLai 
£.±j Ulao jjjj^JI j>j jl ^>^cu oUl ^ jjo j^IanJI dUrnlmj „ ,yUajjj ^^9 
^6 fOanj gxAjg dUlgjuuu Laibjol <0911 jg ji>^l j^TnoJI aJLuoj cJU pAibg oJbJUl 
fCunj ji>^l g Ia> J^Lc a^j>g qjqJL^ ^JLc pjoallJ loLoj ojlC Ldaj dbjg 

„ 1aob ^ylillg fCuajg JLuuy >21 ,£>>21 cxJhJI > jlg.>2l ^UL ^ laab 


t 


Dr. Kholid Yusuf El-Zohry - Sohog Teaching Hospital (01118391123) 

Ref MRCPoss OE OE 2012 PosTest 2009 PassMedicine 2009 PosTest Exom ReviseMRCP 

































El-zohry MRCP Questions Bank (Part 1) - 2013 


(For my personal use) 




yij .. a^ui j a-iuajjji aju^t 


uUa^M 0 


g j±*jo j^Jgi/l ■ 


jlu>^l ( _ r O OQjIj (\Qjj1o tSjgjuuuuiaJI ( _ T 0 4 rirv.I g) Uinn ^Sjgjuuuuib :Qjuli)l ■ 
£jl> gjjlluuu libj d£jg a>U Culd UjJLc ^Sjgluufhll (}C. Uja> vdil\7 
uks\j.c}l\ ^b (jixuo) u /3ial; ,jjuioUU J>Aj <Qj oljiij ddjiiJI 

^Jl Jll^jucuall gjoluuug ddjgJI ^b djgl^nll 
u ^ji\un)l o^Lcu>l j-C dJIgjuug ^iajjjall ^iolj_cl JgUu ^b ^JUbuudbuuug 

(ld> gilj ail) IV^mg jjoi/l gll>l Ibl xiDtuulgjog 


djuuzxc g odomDlill ■ 

♦♦♦ ^ ♦ 


up*/) vAogjo ^6 gxbD rujldlg ,jiajjjalU JLaj^I oljlctuo :d£j|jJI ■ 

g A* CjI u^.P 9 l a >^ ^>1 ^IJi 2 O' uiflg ^ 5 Uogj cv jai 

^b ajulg IkiiJU L>^Lc a>I ^>>jjn g| a>Mc JloX^I Jjd ^nmnnoll JjO 
cJTnrho ^>jjog %nogjgll Uib ^iuS ^£>jj ^qj ouior dJL> 

( JlOiJI I Alb Lo) uiflgxJI gilc J.c dljA^g 

fu ^SjgXujuLib a>(j Ould jlVio^l ^b UjJLc dJbglaJI oJIaJU auLI :dmjoldJI ■ 
l£tijA>g ^yCJI oLotoJI ^b (yxIoxiW jajiBUlj ^Qj jLilU j^j.o u ^-‘\0 JjQA2j 

♦♦ aULLuul ^JLc *jjg 

gji> ojjuuuJ0 Ia> gLacUl J l^iiil^l l jL^juol ^JLc oujaILI adjjJo J^b_6l g 

go J ^>l£imj| ^UJ ji>Aj|) Oiiu^uko^lg ^>jgIimQj| jlmi JjJ ojjaUI 
ail JuCuJIg j 1 lii^iiing^IIg ^SjgjuuuuuiJI ^JLc oujaDI ^b dogomll ^b ^djA^ao 

(Ryder aojuul jjlouju oli£> ,jjo Ugjuu Ua^J 



Dr. Kholid Yusuf El-Zohry - Sohog Teaching Hospital (01118391123) 

Ref MRCPoss OE OE 2012 PosTest 2009 PassMedicine 2009 PosTest Exom ReviseMRCP 














































El-zohry MRCP Questions Bank (Part 1) - 2013 


(For my personal use) 




jo.. a^ui js a-iuajjjji ajujii 


ojj-kUlg o^bJI CM> P± <V9J 9 o.LL=L^aJI cud, ^b g 

1 jg*-»i Lo UJLeg ^nil oijgj! t _ 7 o dJLJU gi~\..-il l p >vnjJI g gi-\.^il l u /-r\ni l 
Oxford speciality training (OST) aajuul >>l jjlquuj oII£j ^>jo otl j.bg JiUb 
JjujiJI .. CiJliJI £jiJI ( _ 7 X) kiflil oluujgiij dilQluu^l Odfld Lais j^nnl l jjag 

.. IfliUol jt>Cbj H ^ilillg Jg^l 


jgxtuuu ^ oyjjUJ ojjuuUuqJI ojIdJI 


: ^ j^aJLJJ 4^) 

y g QlkiuaJI ^1 OfllD x g ^gHI oglaiJI a> g o*l IjjI g ,o>aJI arcI • 
£ l^iJI oJI^IluI Qfl jibl^l Imic ocouj Adld CHj>WI P^ J* 8°^ 

J.C jjlLdLuu^I £lQjuu IjjI £.j> JX) CuJluil lai^s ♦♦ V §-Jj_LdJI JgJo g 

g gln^iJI g ji^l Un r W^J l jjo jlc ail OA>g g ajdb *£>jJI £j^JI 

♦ ♦ OiJI Jbb AjJl^JI fOj^JI 

IjjlOjuO jgiij jl ^ln u^j> I ♦♦ ^ol>jJI kuuug ^nlVnnn IJjlO.It> Ol£ij p} ^1 • 

jgjUo ^n -flgigJb H g j^lu jl u^j>lg dbljiil kuug 

jjh> Jiaj JjocI g *» jjualall aja^JI j_c Cjl 2 uI „ laib >fJjg.> jkilfi jllalii/l 

)j libA>g ai m - J l UlgJJLo 

H g kino jgjj^Jj rMnJJI jjo I Sjlo ♦♦ A£h_^JI An^JI g jl uaJI jl uaJI • 

Lo jLcjjuu flj Uhji^hjuo kJ7 q ouinUI g oul^xII jcIjjuj oJL>jjo jjuLa^JI Ssnb 
^Jl gll\j jLS IjixS v_6_vibJI 6 ooluu U jun\\6 ♦♦ flnrhll jlni 

♦ ♦ J&\ >taO g Afll> 

ojj a^juuJI vfjjjl g AH 12 JI J^> JjjI w j^oi ^g allb j-Qljuul djlnuJI ^og • 

.. ( Qjl^> 


JU>^U g UJ g ^ 0 9^1 O^ 1 cr 0 ^ fl** Cf° 0 1 ^^9 flhc>gZuj\ 

♦♦ aJJI jbU Luji ( aoi>bJ Q£ 



Dr. Kholid Yusuf El-Zohry - Sohog Teaching Hospital (01118391123) 

Ref MRCPoss OE OE 2012 PosTest 2009 PassMedicine 2009 PosTest Exom ReviseMRCP 








































El-zohry MRCP Questions Bank (Part 1) - 2013 


(For my personal use) 


^jLj ijc- Ua c_l&I j 4 r^ MRCP part 1 ls* ^ ^ 

.* jSIjuI! 

nti aiii ^Ic. jl aIc. y i tr“ <3 cJjSj 

Basic science j MCQ u^-^ 1 • 

4-^.1 j (jUcLu Jl^-a 100 CjIc-Lj 3 <s jj (J^ <jjj3 jj 

(jl 4-L^JA ^.9 U.1 ^_1a .1^.1 j Cln£ liij (JJjtil! C__lio! (jV AijSUb Ul (JA C_J^jujaUA j (_£jl L_J^x!LaUJ (Jjjlil! 

ftjjsJI <Ll9 S^^IaaII 4_ia£ (jl ^jjdj ^ j! i—jjlla^i distil ^j!j S^^I-IaII ^£> u^j \*>.s-i 

AlLaiVi (Ja. ^)j£l ^ a (JjmA cilii ^ixj AlLujVi (J^- (Ja ^)4^l ^ (3f^ c_fl^)*jj UJ^ 

L^jai^j S^£!.1 a]| ^ja 

(J j! ^jjjuaJ^£ ^) jg t*ti 4 CllA Jojuj^IaII *t3j^Iaj .1^.1^ cJ^ 1 ^ > Uti'N C_S^)j3 

SjSIJLaII jJLu^a • 

<j j' *^Uj J^Isj t^ia ^.ija! ja ^ jl aiKJ^a jIIaa l_jU£ Karla essential revision notes for MRCP -1 

A ^jjJal^Aj 

Oxford handbook of clinical medicine -2 

the o^jSaa ^a 1 j^)£! 2 a li! (JjVi ajuaIIIj IaI ^ic- u " k]A £ <jl ,Vq’ic.i (Jjj 

lU UUa j only mrcp notes u will ever need 

(jL£ ^jUII j J/il «■ jaJI !.i*. aJI£ The only MRCP notes u will ever need 4 th edition -3 

CiLii! jJI U <ea.liajA <jLs CJ j**al! 4-Jc.l Basic science for mrcp -4 

tiiifll J^.1 a£l U1 Aa.1 jaA\ ^ Get through j Last minute , ^ j«4i' l$ j> 

AllujV^ ^3i^A (jn^C. ^Ac. Cjl.V iaVI ^ij ^jc. S-lpUj AjLXA-a 


jl C^LlluuSfl Jjli CjUl K (JjJJ-4 |Jl^ 4 A^A 4liujVi J 42luiVI j-a • 

4jli£ JL^Ijfl tiSjluu c^njv 


Dr. Khalid Yusuf El-Zohry - Sohag Teaching Hospital (01118391123) 

Ref MRCPoss OE OE 2012 PosTest 2009 PassMedicine 2009 PosTest Exom ReviseMRCP 



16 












































El-zohry MRCP Questions Bank (Part 1) - 2013 


(For my personal use) 


www.Passmedicine.com 

www.Onexamination.com 

www.Pastest.co.uk 


<jUs 7 l Basic science • 

3'>- 20 cA'_>=- pharmacology ejUuljJI 
genetics ,immunology statistics ®-A 

clinical <jc. SjUc. ^iiu ^j <Lla ^jlijs^l j ^ jj'iVI <jj CjLijljJI 4 jL 

I C ■ o \jjai ~ I j 4 j. ul . 31 djLo^lx^J] ^_S_] haslC 

j' karla t> basic 34 ^LlSI * jaJ' jl Basic science for MRCP i> f*_A'A a 

MRCP notes 

<GlaUl) • 

Jj'il * j=2' Is- MS the only MRCP notes u will ever »jAA oS J3 sJa Ul U <_$ j 

jaSI ^ um jl 4k=-3 (_i&]l j basic science M jl CiU]l ^ 

SjSliall • 

(jl 4aa.U. ^1 j 4aa.U. 3j (j£j$ia CjiIjSj jjjS jjJa ^ij -Cilijla 4jj ,li.lj J£ 

a iv ni djj jjUir, (Jsj s^)£lij < _ s JJl «-3?2' 

passmedieine 3=- <3^ ®A u 1 ^ passmedicine j=A 3A-s e^uios MRCP notes 

(3jV! c. j^J' jjl a3As 3.)..^'^ w '*.' s* 3 c ‘ SjAiAl S^AI3 q a*j 

. _ SjuA ^CoG ( (jSljJ (jal jSI A y -n 4 V' . ..I ^a jlaA j SaJ^ J A y < ^4 I Ual (^a3aA 4 Vi . ..VI j^'i 3_s' 


Is- (_5 J '- C - M 


aj 


£ . £ 


( 3 * U^xjj ^^ic. 6j£!i« MRCP notes sa* l£^ 3 I j U! 

d* <y>-^ * J* jl^VI I ciui AhJ^\ CjUJxj I jijjj passmedicine 

S^£jla]| 3 ^ 63* 3 ^^ 3 ^ 3 ^ 

3laio j C > 1* x -a!j 6, S^ JAal onexamination j' pastest i> u^'s“l 3=2 «■£ ^ 

passmedicine J2- 

oxford j' karla ^ o^t s' tA jj^' -ss^l jl 

3^' (3^ 3 < 1 ^'. 1 ^h*-3' (3^ ^g 1 eliUiuljj V_5 ela^lj ^^S2I (js-° ®3^ lA®- 14 3^ 

passmedicine ‘'-Aa' 

3c ‘ p^aaa^ai]' (3^=A 6^4 3^ ~A S° 3^ 3^^ 4 '' ‘ ' 3^= 3 

3 I 1 I ^Lal 3 I 1 Ia 2 ^) 


Dr. Kholid Yusuf El-Zohry - Sohog Teaching Hospital (01118391123) . 

Ref MRCPoss OE OE 2012 PosTest 2009 PassMedicine 2009 PosTest Exam ReviseMRCP 17 





































El-zohry MRCP Questions Bank (Part 1) - 2013 


(For my personal use) 


Experience of Dr. Mohamed Sabagh for MRCP part 1&2 


MRCP parti, 2 written and ^ Mahmoud Abdel-Raheem Fathy j Heba Mohammed 

Ci]aU]l i_ya\y. jj^jjjPACES 


AIjIjlIIj j-o ^juajlj 1^-1 j JjVl ^ jV ^laJ! jA aJIaJII jlnsJ ^a 4^.1^. ^aI 

4 iKH ^3 4^.La. < _aI^Ia 1 Ia-iIxj Ua.1 Lj jl i£jfi SAaL^ A ^ 1\ V JtUl ^^ic. 

*•& ^ Vji ^11 ^Ijj WhUj ^ jV ^ ^ 6JJ I^LSIoia ^ cilljL jl ^ li«ij UJ^ t$l j] 

L-llUa ciljV dllxA (jl (Jj |j dJ'sj -la* didj Jjuo3j JjLa* Lai L 4 'si *s 4 JIa jll jc. ^^juoa^xJA LaIL 4_i3^H2k. 

La daJ Jalc. LI Idu La Ja3 4 .a^.a <La^1xa1I ^ *um 4_i3l)L jLudic. 4 jI ^Ic. ^)jdL djil c_fl^)xj jV LjL ^aic- 

^llllj JjVt j^j?^ cgi f JJ^ 4ll -IaisJI CjaAlj l^j^ajj ^j^all S^)£!-1 a]! 4j9J^)la ^Ic. ^a jW a -l^.lj dual 


j)A jj£| 4iLsJl £a JaLx!!] 4. <3 Vi's, a 4jaj^}laj 4jlVii-A ^Jaj ^.f\k'\W 4jL^}ia 4 JLa jll S^)£a ^Ic. Jl£jj]£Al (_£^laj 

(jd dli3 dj|^)l^A j/K jj 4jj ^IaC. J (_£^)Ja3 ^iLl! $. j^JI ^jJalj Lj CIjIa^Ixa 4j^juj 1 g U-LVn SjLgjui l^jj£ 

djL ^^3 ^'s Vi's La 4-lajsJ ^ jyj ^JjuJI djL^.^p Jjxx^a^ 4 JLa jll jl c_a^)xj jLa£ ^a jV (J-^-^-^J 4^^1*^l <—fl jlc- ^jaJ 

jjj djLl Ja Jja dLl -1^1 j j^ 

jpj 4_llc. j^)dj ciijl a l^j (jda 4,\ UjjI ^ ^ lj jLdLaVI jV jlaillj (jd (Jljjuallj Idl S^)£I-1 a]| 4A3^ia 

ClljJ \jj\j -Laic. L-JjLskj j^A-a 4uJa^)j l2jj^) 3 cilj^)k-^vs 6 ^)a (JjI -Laic. C-JjLaJb \jl ^gij)VW 4-Iaraill 

(Jl jjuj jl JIja 4jI j£JJJ 4 jI l^)ij ^jda jaII ^ Jl jjuJI djj£3 ciljl d3 ij C^nn^ 

side effects of corticosteroids ed 


tiljLa^lxA ^l‘d ^ (^lj| dun's jl ^_y jl dtun'S^j ^-^)jaJI l^)3l jj^JUj 4-Jjl^. j^^*-J 4 _i 3 JjV! 

,^-11 4ia i jSIj <1 ^Ujj (jlll i_j^' kumar or karla or handbook of clincal medicine ; oxoford J 1 ^] 

4 i. A .^1 ^jc- Iji CLiS^jC-Ia ^xj I^aa ^^111 j^LlaxaII 


MRCP part 1 


passmedicine q J' d eiuS 


guidelines J' ^ iP&j lWj^j ts^^J c>^' <^V 

kxa.ljAl ^ jA i_iUS£ ( JxmajXA <jV jUj£ ^iai ,xa.jj kxU. ^li (jLi j] dJjS La <_£ jj 1^-kaAJ ^1 

(j^al'sj L«a ^xj Jj£l J j x «olaj ^al jLuolc. I j3l Vi's a L* u£a ^)d-£a]| c**l*s*i j)A dlAj -1 ^-Ij Ul j jU^yi Ja3 CjiLo^1xa]I 

jtnp lAtnlj jjA *j=d! 


J!3 t> ts^ cjIc-LaII J=uaj 1 a j £jjj jj^J. 5 Cj jSli Ul <_>ajj£ JjUjj onexamination jSlil d 

CH^^LaII ®/q~1 5 ^4^)^ A ^g-iA^)£l Lij^) -lAiaJlj dll^)A 3 j-J—'<1 jjlj jjuj-llAjujlj C*Ural's dlc-Luj 


Dr. Kholid Yusuf El-Zohry - Sohog Teaching Hospital (01118391123) 

Ref MRCPoss OE OE 2012 PosTest 2009 PassMedicine 2009 PosTest Exom ReviseMRCP 



18 























































El-zohry MRCP Questions Bank (Port 1) - 2013 


(For my personal use) 


onexamination mandatory 
passmedicine mandatory 

kumar or karla or oxoford handbook any of them is helpful 
4 monthes enough to pass 


MRCP Part 2 


onexamination JjVl pastest Cj jSli 


google 4X-A ^j LajLui 


atlas clincal medicine cA ^^ ^jtj 


Ldlj a_i3 Ia^^I-IL lA 4 eiu£ aj 

^jjIa Ullc. ^iat J Altai I ^^3 AJL^jlaj <C.^)juj jnS n 

^ic- , Wjs J J jj djl^)U IaIxa A^LuoJa]! Jj t. flails J Lg_l c4a^ 3 ^Ic. 'IaIsjj <a^1xa 31 S^^ljUa ^ic- ■ Wl*jj 

differential diagnosis J 


jA CjjU ^)1 ^ lx j UjSIjSj A^^Ixa (_]£: ^aAti ^aJ)V j <—Utu^al ^jjoJ ^j-bulgxujj£' (j - * 1 A.\ u^Vl 

fiJlA dj^)£|j (AV (J^Aj^JA (JjjUll ^jLujlc. L_iAa. ^JjLg £^aJl ^3 ^JJ) Cljjlj S^)£IJLa 1 L^lxJjuJ^Mj) .Ljj'N.Ml 

(Jji ^ja Att ,^a^3I S^)£IAa]| (Jaa <^3 CjI^jL^a ^Ic. ,Wiajj Ajj Ul (JaJ ^baJA^I (_£^3 S^ix^-sa 

(_jaa i uX^li ft JalLuAl ^)^3 A^l -Ix^aall 425 ^AAI "SA' ^ J*-* 569 jAA? *j* 

S^)jt£jl £a (Jjjuoll A_1J Cll3tjJa La t4 AtlinkA £&jj C e4 1Lgjj t^. Ujj5a ^jjoJ S^l^-juall ^ic- (Jjx^aali ^3 ^jaJ 

jlkijl <jl tilSU Ja^ UjI^j ^1x31 Ijjjl jjillj I jA Aj u! 

IjSu Ld Clu£ Ijl (■ _ ^3 (JjIa ^3 A. Q 19^.31 AxIaII ^j! ^I Ajoiij Aixll ,\l* 31 

Jc. C QttJj ( X r lAaJl (33jJ UjJJ ^.Ic-Jill ^£1LujI Ij^l j IAjJjuo3J ^ jtl^A Cllj£j l^jSjjai Alla. J JjolilJ Aj3^lJ CIua j£ 

ytLVI 


Dr. Kholid Yusuf El-Zohry - Sohog Teaching Hospital (01118391123) 

Ref MRCPoss OE OE 2012 PosTest 2009 PassMedicine 2009 PosTest Exom ReviseMRCP 



19 










































El-zohry MRCP Questions Bank (Part 1) - 2013 


(For my personal use) 


Experience of Dr Salem Omar for part 1 


Egyptians for MRCP part 1 Heba Mohammed 


; all! put 


jjl' OH ^<JS1 Cj1x.Lu 3 SaJIj 100 (J^ JJ (jLaj^n (jl I— aj*J I t -1 


< _tlllN 


Ajuj!^)^1! ^9 ^auuaS (_]£ ^UaC-i c ^ \ ciUil AiLc^l! ^ic. 4.\ UjjVI -2 

si* cSUil AkJl 4 Ilk J!^ 15 ^ 


!^ Allis a\ujjV!j tiliiSj QjJn) dillj !-l^ ^pli -3 

^ooi! j-a j!&! (joj jJjV dilil djaj J ^aQl Slj l^Lii!jjj <jL^IaV! ^S " uJ^ ; Jlia 

^k! (jj-^ ^^Ic- j! Ajlujjj 

The only MRCP notes you will ever need -4 

tillLujI^p (JjV! -laid! AjtjJa cdlil (J^uVn 4.\ UjjVI ®/q $0 (j-a ^)j£! ^-lax ) 

:£> cJ£a^ AhJ^\ % 70 c> J&\ \ jK l_sU£ -5 

diSjl! ua^k cidjj aAIxjjV! ®/q 90 -0 

ajUIU 4+Ai o^ 3 ' (jUa^V! (j* JjV! * jaJI £* ^j!>Lk ^ <ji! 

nice c> guidelines ^ ^ j JjS'j yi J!>J! Ua j 

^L^ailujj! j! <v!j^ JjJaS!J A_ij^aikjaa]!j 4_i^)lx]! ClAj^jULd! ^j-ajJajjj 

The only MRCP notes J' u' 


edl j 4_ila*j ,J 9 U. 

** * ** W V 


1^-aA ! 4.1ujjVI cJ^d 4 n»j\lj- 


Onexamination -1 


Ailxld! <1 lu^\ <^Uj 5U£ fLAH lU 


Dr. Kholid Yusuf El-Zohry - Sohag Teaching Hospital (01118391123) 

Ref MRCPoss OE OE 2012 PasTest 2009 PassMedicine 2009 PosTest Exom ReviseMRCP 





















































El-zohry MRCP Questions Bank (Port 1) - 2013 


(For my personal use) 


-JO 4-Ia^.^La ^JuLadl s-lj-lllj j ^-aJ^uuallll j g.t x«oVnnl ^aJblj 

^j£Jj (jjoiij A VntiV' {jA ^)dj£ <sLjal - (Jj^ju (jj^ £3j-d! ^j>a qjjIj (JI^juj 15 

<c.Lix^aJ 

Passmedicine -2 

(j-<4 ^^70 <1^1.2*. ^jV 1*1^ C_j|^sJl ^glc. J 6 ^£lS]Ij A^jLuIa A.\ uJ ^^jtjJ 1*1^ cdijlS A^Cl^Xl 

A \ IjujV ^)tSslj ^idLa^Lt-a A.\ UjjV 

Pastest -3 

!.i^» j^. ^ILujVI (j-a c_jIjS djLo^lx-d aS^ AdUd! <^.^).dlj 

Revisemrcp -4 


^ISsVI a.\ ujjV <iLjal IjjL^. a. lx (jdlj^juj ^L^lLoV 1 ^3 dij 


a.VujjVI ^iaxj (JjSk, <juj!^)^ ^jj^-juj - 3 cJ^-^- Li^jj djtoLuj 4 4_iL<a^J! SaaI! 

L VK* (JSuudJ 4 .\\jujVI (Jj*. ^)£juj 1 J 4jull_J.d] 4-L-a^J (jjjSll 


• 4_JU]| aJIgVI cilJlJ V JlSj-a C_j|J^dij <JjjuJLaiiij AilaiVi l£ cJ^ j^Sjdl 1 u'c. Ja 

A 30-year-old femalepresents with a one year history of galactorrhoea. She has been 
receivingtreatment for hay fever, depression, obesity anddyspepsia. 

Her investigationsreveal: 

Full blood count Normal 

Urea and electrolytes Normal 

Prolactin 820 mU/l(<360) 

Free thyroxine (T4) 18.3 pmol/l (10-22) 

TSH concentration 2.1 mU/l (0.4-5) 

Which one of thefollowing drugs is most likely to explain these findings? 

(Please select loption) 

1- Astemizole 

2- Metoclopramide 

3- Orlistat 

4- Paroxetine 

5- Ranitidine 


Dr. Kholid Yusuf El-Zohry - Sohag Teaching Hospital (01118391123) 


Ref MRCPoss OE OE 2012 


PassMedicine 2009 PosTest Exom 



21 


PasTest 2009 


ReviseMRCP 

























El-zohry MRCP Questions Bank (Part 1) - 2013 


(For my personal use) 


A ill ill .ul <liuj| ^jV (^C- U (jSaua I^A (J!Lq 

: i>^- 

C Ji^ ^ -1 


e.\ 5 aU 2Ua.Lu.VI <JVI -2 
** ** 


O^jp JJJ ^AVt -3 


^jJaLd! ^U> (Jljjaill 1 JlA ; ^a.! (JHa 

MRCP PART 1 15-1-2012 


Subacute combined degeneration of the cord 


(j juI <jV J!>J| (J^, C_j\jUjdll aJax-aj) 


An 80-year-old woman has a three month history of progressive numbness and unsteadiness of her 
gait. 

On examination, thereis a mild spastic paraparesis, with brisk knee reflexes, ankle reflexes 
arepresent with reinforcement, extensor plantars, sensory loss in the legs with asensory level at 
T10, impaired joint position sense in the toes, and loss of vibration sense below the iliac crests. 

Investigations wereas follows: 

Haemoglobin 12.2 g/dL(12-16) 

MCV 95 fL (80-96) 

What is the mostlikely diagnosis ? 

(Please select loption) 

1- Anterior spinal artery occlusion 

2- Dorsal meningioma 

3- Multiple sclerosis 

4- Subacute combined degeneration of the cord 

5- Tabes dorsalis 


J&J JI>JI ^aJ A 1.1a. Ja-ajll ^a A.\ n^Vi 

* A-JUli CjVLa^VI 


Viol's I ^ALa -1 


jiLj^.V'V^a j\ LiC-lxjuj gA V! jaL, -2 


(JiLaVI jALa -3 


Dr. Khalid Yusuf El-Zohry - Sohag Teaching Hospital (01118391123) 

Ref MRCPoss OE OE 2012 PosTest 2009 PassMedicine 2009 PosTest Exom ReviseMRCP 



22 



























El-zohry MRCP Questions Bank (Port 1) - 2013 


(For my personal use) 


(Jijjaill IjlA liUj (JLLa La w M ^ A.\ uaVl C> 

Which clinical feature is consistent with a diagnosis of VIPoma ? 

(Please select loption) 

1- Alkalosis 

2- Hypoglycaemia 

3- Hypokalaemia 

4- Increased gastric acid seceretion 

5- Provocation of VIP release by somatostatin 

j Laj nq'l La-lic. Aii^)l3 ‘La^^lc- ^a<3 \ (j! l** l CllLalj j Ja-olll (JLa 

(J£ ^3 AJjjUll AJaVi-li ^ic- 


cdlLjj ^^-c. ^ >>il ^aIj ^Ja^-al] <L<Qlc-jl ^aI AjujI^)^ 1I ^jc. IL1 


; JLLaAjail J.JI JUC. Ijjj£ 






;Al3^)*^a tililc. La 


AjAJ^aII (J^ai^iJI ^a! -f 


B (J^ jc-l -2 




juj jl -lia. djj^xjoua -4 


_$A Laj ^ALa —Aii^lc« AJ ^g-a>Jjaull c_a-m-adll -5 


Ja^3 ^ jjll ^fX*l\ -(3 


CliL^Aj^a S^C- ^jJa^all ^ic. f Q^Lal 1 , W’hti (J^JjujLa AjLifLj dLaii Ja^V -2 


A-ilLil' c__a\_L-aVI .la.1 (j-a ^^Aj lAJj^jj ^yt> ^a aILuj! dilliA 


j jU t^yajA j£ JJ -1 


JLLa Jjjj ^-S jjuj AJajljlali jJC. CljVLa!ia.VI Cljli aIIujVI -2 


La ^_ya_y a ^3 (jC- C, aL J£ l^.jW < 3 < 3‘l>a ^yt- Aj^l-a C ^ n ti *1 ^Ic- j£^)j A LhuVI -3 


Dr. Khalid Yusuf El-Zohry - Sohag Teaching Hospital (01118391123) 

Ref MRCPoss OE OE 2012 PosTest 2009 PassMedicine 2009 PosTest Exom ReviseMRCP 






























El-zohry MRCP Questions Bank (Part 1) - 2013 


(For my personal use) 


-j AjI^jJIj aILujI ^3 Aj^-aLi. IAs* 


^ -4 

basic J' 5-uil ja 




1 Aa. Aa^Lla ^ic- AAj£ ^jl^A aVI ^3 aAj^jj £3 jj! ^ (Jlj^ud! i AA (_Jja J (JIja 

A 55-year-old man with Type 2 Diabetes Mellitus andlschaemic Heart Disease has been researching 
the Internet! He asks your opinionon Laser Transmyocardial Revascularisation. 

Which of the following statementsabout this technique is true? 

A) avoids the need for major surgery 

B) damages the endocardium 

C) involves destruction of coronary stenoses 

D) is of particular use in severe proximal coronary arterydisease 

E) stimulatescollateral vessel formation 


PCR 

Ullc. V (jL^ILq^U Aj^j^joJI ^jj^S aj y ^All cAjLa^IxaII (jl.Vqlc.i ^)jA cA11a£j <aIa 

The polymerase chain reaction (PCR) is used to amplify smallamounts of deoxyribonucleic acid 
(DNA) for further analysis. First the DNAdouble helix must be split into two strands. 

By which of the following is thisachieved? 

(Please select 1 option) 

6- Alkali solution 

7- Centrifugation 

8- DNA polymerase 

9- Heating to nearly 100°C 

10- Viral reverse transcriptase 


Which one of the following statements concerning T lymphocytes is correct ? 
(Please select 1 option) 

1- Are infected byEpstein-Barr virus in infectious mononucleosis 

2- Are the primary host response in bacterial infection 

3- Compose the majority of lymphocytes in plasma 

4- Produce IgG 

5- T cell lymphoma has a better prognosis than B cell lymphoma 


^^3 LaLaj Aj^}S3 (AjA^jj AA^ULaI! (AjjAaJ! (Jljjjuill lAA (Jja La! n 


u 




A 75-year-old man has a history of chronic lymphocyticleukaemia. He has had treatment with 
several courses of chemotherapy and hasnow been admitted to hospital with pneumonia. 


Dr. Khalid Yusuf El-Zohry - Sohag Teaching Hospital (01118391123) 

Ref MRCPoss OE OE 2012 PosTest 2009 PassMedicine 2009 PosTest Exom 



“ i 

ReviseMRCP 


24 


... J 


















El-zohry MRCP Questions Bank (Port 1) - 2013 


(For my personal use) 


His medical historyrevealed that he had suffered several previous upper respiratory tractinfections 
over the previous six months. 

Which ofthefollowing components of his immune system is likely to be deficient? 

(Please select 1 option) 

1- Complement 

2- Immunoglobulin G 

3- Macrophages 

4- Mast cells 

5- T lymphocytes 


In which one of the following conditions is deoxyribonucleicocid (DNA) analysis the most useful 
diagnostic test? 

(Please select 1 option) 

1- Adult polycystic kidney disease 

2- Down's syndrome 

3- Huntington's chorea 

4- Hypertrophic obstructive cardiomyopathy 

5- Klinefelter's syndrome 


3 


V jUi-J ^ jc. i ,W*nn (Ja. Xsc- Ja.'SkV 


Concerning immune cell antigen receptors, which ofthefollowing statements is false? 

(Please select 1 option) 

1- Affinity maturationof the B cell receptor is an important process initiated during the 
primaryimmune response 

2- IgD are surface receptors of B lymphocytes 

3- In normal individuals T lymphocytes with T cell receptors(TCR) that recognise autoantigens 
are all deleted to preventautoimmunity 

4- TCRs with different antigen specificities can beco-expressed on a single T lymphocytes 

5- The antigen specificity of the T cell receptor is generated during development 


cil j_n]| <jujI A-LouIIj 

V** * V * ♦ 


Lx£J 1. qa qa - \ 

CllLujjj (jA -1^3^ Cj? * 1Jt J LaLoj 4.\ UjjVI V -2 

LjLoj ; AjIjjIS A-AjouIU -3 

Luj£ <_Aj£ ■VI -4 


Dr. Khalid Yusuf El-Zohry - Sohag Teaching Hospital (01118391123) . 

Ref MRCPoss OE OE 2012 PosTest 2009 PassMedicine 2009 PosTest Exam ReviseMRCP 25 




























El-zohry MRCP Questions Bank (Part 1) - 2013 


(For my personal use) 


Aj^-ata* C* \ (j£ A^aV^aj AiV ^Jj^aq 'ia (J£juoj Ajoi^^V j ^jjjuq]| -5 


L_jIj£ Ajuj^plj A ^ }jJa*iV Ijil ^3 1.1a. Ac-LIa!! -^) 


^)J^I CllLajV Cll3^]l C* )"\\a !i! Aj^ala. aISjuIa il A.IUjNI ^axjS^)Lui^j AaIxJ! ^j^aVI ^glc. (^j-aajall A 1 nul -7 

A_iaA ! 


I j*!ill <ja ^giaij aJIojVI -8 


ReviceMRCP Jl *1U -9 


ONEX . PASSMED .. etc J <sL^\ W^ 3 — si* <jajj! lil cjUjIxa jijjj Sjla* ^ 


^aj^£j| A^_a.^ tx^alLa. A.lx9*1 1 a\* 1 <jlj Uiaa. (jjjjjll ^ IjJ*.I -10 


Dr. Khalid Yusuf El-Zohry - Sohag Teaching Hospital (01118391123) 

Ref MRCPass OE OE 2012 PasTest 2009 PassMedicine 2009 PasTest Exam ReviseMRCP 




26 


























El-zohry MRCP Questions Bank (Part 1) - 2013 


(For my personal use) 


(j!j djjb 

Egyptians for MRCP part 1 ^ Heba Mohammed 

^al 1 g trO j)x-oaj CIjV jLouli ^pa*J VlC. ^gaj)A*qaJ CliLaLa Aj^juj ^jO 

^jIj g‘i* \ ell! ^11! dll j’i* nj J)*n Vn L«a Ixj (JjV^ Vi ^ ^^! S^a 

k*J*l ^j^c- ^ JjxuI gr^J^ ^ £ j^jaI! 

Aj3 c_jjKa (jjfL tilAJUjj Aj^-S li dig! .Vim a ^\j I jSjjxjj _ dVij,>* Ig.ji C*n*n (jLuaC. (j\aloV! ^»jJ dltxA 1 g jjaj 

Admission W ajuj! A ‘^“=■4 jj%? cWil o-ij cjjjj^Ull jA lA&j... ^ cSj ^uAh^' ^ill? 

t»jj lilio Uj^kUjj cdl*^ Uai-tu 4^-bj^jjljl' <ua t_u£ij aJc. ^joj (jll^ Ljjsj \ g». 1 3- » ~i; Document 

. jlaisVI 

(jLaa^V! Jjc.'>0 iiliuac.U Ijjj£u JaajVI is* is*? f jJ 

<c.Luaj (jlajAV! JiIxia 3^ 

dloL, 3 JjV I 5ijjl! 

jjj Vg^)3 Ala. (_£! ^3 J£tj j! dl^l £A L j.)ajj La! U 111! I Jic. dllj^SjAJ ^JjLo ^aAj (J-dj AX-IjoJ !lxil dL^)J 

die-L a 3 Vll^! A3 jjJ! 

^^A Aj! ^jJi ^)a! lal j£jj jl£j (jLuaC. !la (JaJj£i All ^111 ^jjl! ^jU <jj£j ^ j^/ls Al£ ^ jJ! laid ell! d^S 

^j! j CIj^^jujjLiI! Ills L«a Agjk-aajuall CIjUj! tlllx-Q ^all A-ata ^Jb!_A-ala Vj W?(jlL jLx^g tA^a! ^^11! djlalaJ! 

Admission Document J' 


AjI^IIj A^jjjoiV^j i A^-H (Ja^Ju c, 5^^ A-^ta ^llajA 

^ c** 1^ n ^ Ja^Iaa>a AjJa^)j A-ala. clljLaiLal clLoS^jj (IIajujIj Agio ^jalxUJ tlLogJill ^ cilL^-lilaLa 

Ia^ajuoJ ! ! ! !A-aijL^a A-ala. ^jo bj^JC- Agjllll j! I j3^lj Ld ^jjj^jJaaliui 1^11 

(: (: <_£* tjt JjVl ujS!I 

vj- 1 ij.... >■ . <_>"> ‘-“t <_w CjVUJ! c?a.... c> oj^t j'- a: “^ i j' uAj't gj' (_W 

AgjU Aijj Igjjlai Clll ^131 AgtaVi (jl^ (JUa*ij 

c_jjl£ jj^ij IxjL jAnswer Sheet^^Vi 4ga Aijjllj aMJ)!\ 4g3 a! jjll qujVI ^ I ^j 

^aic- ^3 ^jujLg lH! Aiilj (jttia j Hll S^)£!3 a 1I Aala ^JblAJ ijjxai^iJ j ^ tlljliLlJ 

s-Hj (ji AgS clLi^)£jA \1 j_j 


Dr. Khalid Yusuf El-Zohry - Sohag Teaching Hospital (01118391123) . 

Ref MRCPoss OE OE 2012 PosTest 2009 PassMedicine 2009 PosTest Exam ReviseMRCP 27 



















































El-zohry MRCP Questions Bank (Part 1) - 2013 


(For my personal use) 


How to register for MRCP part 1 

Egyptians for MRCP part 1 ^ farek Altohamy j Heba Mohammed 


lie- 

\“ C 


a iKII ^ A^La dPW 


. l$ja ^-131 ^)S1\ I jsj ^ jV AasjL^a J£ <jl ; JjVl ^ <^:uc« 3j| l>^ 

Jsais CLuxj 3I ^jjjX<ux!L<a ^)Jj£ JJ Ajjdkjl CjU3£ <J3 j] .UiJl 

Ajjllll S^)J3 ^q ^js’n ^j£-a-q djjjioAJi (jLa^j La .la. ^JVl>a ^)lxxaJ^J S^)LlC. 


cdllll! (J jl\ 4jxaLa.j 4iLaJ}3! ^ lA'A 1 4-^La. (Jj^ 


http: //www. mrcpuk. org/p art 1 /Pages/_Home. aspx 


^Luj (j! JuilLau^ .la. l$\j (J-u^aaiilLj JjVl £.^^31 ^131 djLa^I*xa3l (J£ 4_lie. jA 1 * j)o 


(jl.VuaVi ^axa. U - *^ ^al£lA liA LI ^jjdJ 

Application 4 w; a c> c_±iaJI ^ <*jIS <ja 

Applying for MRCP part 1 examination ^ ^1 <£ L ^ tilLk^iA j Ia jLlla 

IaI jaj CllJ^ L AjLj^Ja CljLa^Ix-a l^_l3 

Apply online ^ jt ^ 4 ^' £ 

Create an 4JLa^3l j-a ^^ic. c l^a Jjl <^131j JaJaSL CjI jiaaJI Ig-ia a 1.\1Viqja 

CjI jlai. 6 J-S3 ^ jV ^A^io L-jtxxa. ^iaj tihl l.ia <k>j^l] UL-aj $.£ online account 

Jj 3 CliialLal jl C*iA m Ja tillc-LL CjLLJI ^£j3 JiL.lL 1.1a. ^Jajjan 

A_a>uii31 Ja3allj k-Jj) , i < \^ ^A Lq iSj) -3a3a3lj A _u (ji (j-a 

cdc-liL cIijjjjujUI! ^3 jA Lq ^jLa£j a jKH ^j-Q 1 ^*IWiA (_5-33^ ^ jVl 




<JS1! ^U! 


Dr. Kholid Yusuf El-Zohry - Sohog Teaching Hospital (01118391123) 

Ref MRCPoss OE OE 2012 PosTest 2009 PassMedicine 2009 PosTest Exom ReviseMRCP 

























































El-zohry MRCP Questions Bank (Port 1) - 2013 


(For my personal use) 


£ Date of issue 


£-3j}a]I ^ C'.iUc. tilAilu AjIj£ Qisuualll ^^JajAl 


Auk. JaxjJau tiliil IaI ja. Auu£ CIajI ^111 tiktu JjajVI ^Lujj tilljuxuA j 


liLll tn"s\ gjjj'S, ,lll ^_3 A AAd-l ^uJ gjijuaC' q^joAu till a 


Aullull AJ^jaII ^k J^Jb 

(j\ diA^l jaaj jbflj dlfl j ^1 J £Z jaII c lMaC. dul SjS 

liA ^ JjV 1 ^-lu tiliill! g>j»i&3 <Jdd 

Sign in J**j j 


Application—Apply online—Upcoming exams IaUIac Cjljiai. Jjt jj jjSij 


Aa^.a CjLa^Ixa l*3£ gjV lA '^idl g jk A. , Vi ) Audx^a (J£ eljjj lu dlls La (_£j)j 


^U3jk (Jlijuj A^-tu ^lll (JldlAVI ^ (ilkAjJ *1*J 


Apply 


IjJai. 7 jjSliJ jlaa^iU Jja. 


1 g K UHaj (O^ O^J 


dul Jll ejUbJI 1 $js i an> » 'ii Confirm personal details 


tilk.Hl dUlull laQVu d^S A*_l o 

* *** * * V 

Vj dbjk ji dl^l^ul d^kl (Ja ^Adc. dilsduil du£ lilj V Vj GMC dk m du£ <j! c^\l\ joUJ Sb£ *l*u 

£ 

V 


ldllaJ^}J ^3 ^laIu^J ^jAjJa Aj^a 1 j\l ~s du! gjA 


Aj 3 ^pHaj ^jjtc. du! jlluJI SA^ Aju tilj^)jJa^ ^)llduA j 


dull gjC- Vj) tildiJ Vj) gpld JA £9Al! AAl^la ^)Ldd AxJ 




Dr. Khalid Yusuf El-Zohry - Sohag Teaching Hospital (01118391123) 

Ref MRCPoss OE OE 2012 PosTest 2009 PassMedicine 2009 PosTest Exom ReviseMRCP 















































El-zohry MRCP Questions Bank (Part 1) - 2013 


(For my personal use) 


Credit or debit card ikjk up- <^' kajjL 

lilc-Uj J> jlfUl dlUjJ 


g-form ijl iillai - Jjj ^ Uu 

a a) .. v jI (jV ((jtiLa ^3 -ixJ j ■ ^ j' ^ y- ^ ' Ajajlla ^jLa ^1) ' -x 'tt’i 

jl^aJI £- liJ-lic. e-fomi 4 -aK ^'<~- Ut ■ Wi ia LjUoJ^d idjc-Lu cd. * -tit ^ I g *v» 


i a 


laooi dul Jjl litjUloJ (J£ l^jS i_jjS<z' 4 . >■ j' ^iijA j 4j3 £3jj iilj| ijlla i_jjI^j3 (jl^a La 3 ju 

jAj 4jk- <1 ±*jjA < _ s lli lgj3 (jla£ i_Jj^aj Ig-U 


Overseas examinations office 
11 St Andrews place 
Regent’s park 
London 

United Kingdom 
NW1 4LE 




*4bljj (ijjl' u' . ^jdb e-form 'aU^j idjc-Uj <_sjdaiVI si^ddl <uk clwjja ids ju^ 

•* 

.f Jj ^ ^ (JJjH (J$ L-jLai^ lU*-^ ^ jV gJLilbj ^ja]! A\)*\ ml ^ JJC- jfuil j!^k 

jl 120 cdilliA (S i < 1 *^ 4 jx^ ^ chxSi& j] y-i*^ 

(j-a ^)J*^; ^)j£l ^jfl] ^^)jujI 4'\x nft DHL 

^JJ (*i i-ajjA Ia] A ^a 1! (Jj^aj tilc-ljj (. tiH]jjli&j *15^)2! i> ciul ^ 

rf£A\ ^i!)li]! ^Lx_ia]I ( ^2 ^jaIiA <4Ijlj cilic-tlj jV i Ija VujjI ^j! till jit. Lu (JjajI cd]j!l*JuA 

J^liill (J^ AaS cJ^a-jl cilllat^jjc>^ jL^jaVI c_)^J 


(jVWl La tiijl ^£> (JjJali (JAA ^jI^jaVI 


Lia ^g-kju clljl .ti]j^)JA 


^-lAail c_3jjj jj jj jLllj j 


Dr. Kholid Yusuf El-Zohry - Sohog Teaching Hospital (01118391123) 

Ref MRCPoss OE OE 2012 PosTest 2009 PassMedicine 2009 PosTest Exom ReviseMRCP 



30 

















































El-zohry MRCP Questions Bank (Part 1) - 2013 


(For my personal use) 


Reference ranges 


Reference ranges vary according to individual labs. 
All values are for adults unless otherwise stated 

Full blood count 

Haemoglobin Men: 13.5-18 g/dl 

Women: 11.5-16 g/dl 
Mean cell volume 82-100 fl 
Platelets 150-400 x 10 9 /l 
White blood cells 4-11 x 10 9 /l | 

Urea and electrolytes 

Sodium 135-145 mmol/I 
Potassium 3.5 - 5.0 mmol/I 
Urea 2.0-7 mmol/I 
Creatinine 55-120 umol/l 
Bicarbonate 22-28 mmol/I 

Liver function tests 

Bilirubin 3-17 umol/l 
Alanine transferase (ALT) 3-40 iu/l 
Aspartate transaminase (AST) 3-30 iu/l 
Alkaline phosphatase (ALP) 30-100 umol/l 
Gamma glutamyl transferase (yGT) 8-60 u/l 
Albumin 35-50 g/l 
Total protein 60-80 g/l 

Other haematology 

Erythrocyte sedimentation rate (ESR) 

Men: < (age / 2) mm/hr 

Women: < ((age + 10) / 2) mm/hr 

Prothrombin time (PT) 10-14 secs 

Activated partial thromboplastin time (APTT) 

25-35 secs 

Ferritin 20-230 ng/ml 


Vitamin Bi 2 200-900 ng/l 
Folate 3.0 nmol/I 
Reticulocytes 0.5-1.5% 

Other biochemistry 

Calcium 2.1-2.6 mmol/I 
Phosphate 0.8-1.4 mmol/I 
CRP < 10 mg/I 

Thyroid stimulating hormone (TSH) 0.5-5.5 
mu/I 

Free thyroxine (T4) 9-18 pmol/l 
Total thyroxine (T4) 70-140 nmol/I 
Amylase 70-300 u/l 
Uric acid 0.18-0.48 mmol/l 

Arterial blood gases 

pH 7.35-7.45 
pC0 2 4.5 - 6.0 kPa 
p0 2 10 -14 kPa 

Lipids 

Desirable lipid values depend on other risk 
factors for cardiovascular disease, below is 
just a guide: 

Total cholesterol < 5 mmol/l 
Triglycerides < 2 mmol/l 
HDL cholesterol > 1 mmol/l 
LDL cholesterol < 3 mmol/l 


Dr. Khalid Yusuf El-Zohry - Sohag Teaching Hospital (01118391123) . 

Ref MRCPass OE OE 2012 PasTest 2009 PassMedicine 2009 PasTest Exam ReviseMRCP 31 
























El-zohry MRCP Questions Bank (Part 1) - 2013 


(For my personal use) 


MRCPass 

WebSite: www.MRCPass.com 
Total number of Questions: 2475 



From Question number 

Page 

Gastroenterology 

1-140 


Rheumatology 

141 - 267 


Respiratory 

268 - 388 


Nephrology 

389-528 


Neurology 

529-681 


Infectious disease 

682-839 


Haematology 

840 - 998 


Endocrinology 

999-1154 


Clinical pharmacology 

1155-1286 


Cardiology 

1287 -1410 


Basic Science 

1412-1581 


Dermatology 

1582 -1610 


Ophthalmology 

1611-1669 


Psychiatry 

1670-1728 


Statistics 

1729-1776 


2010 January 

1777-1875 


2010 May 

1876-1975 


2010 September 

1976-2075 


2011 January 

2076-2175 


2011 May 

2176-2274 


2011 September 

2275-2374 


2012 January 

2375 - 2475 



Dr. Kholid Yusuf El-Zohry - Sohog Teaching Hospital (01118391123) 

Ref MRCPass OE OE 2012 PasTest 2009 PassMedicine 2009 PasTest Exam ReviseMRCP 


































































El-zohry MRCP Questions Bank (Port 1) - 2013 


(For my personal use) 



[ Q: 1 ] MRCPass - Gastroenterology 

A 45 year old man presents with 
acute, profuse, watery diarrhoea some after 
returning from a holiday in Namibia. 


Which one of the following is the most 
appropriate treatment? 


1- Metronidazole 


2- Ciprofloxacin 

3- Vancomycin 

4- Prednisolone 

5- Cefuroxime 


Answer & Comments 

Answer: 2- Ciprofloxacin 

The most likely cause of travellers diarrhoea is 
E. coli. Ciprofloxacin would cover for this as 
well as shigella, salmonella and 
Campylobacter. However, if giardiasis was 
cultured in the stool then metronidazole is 
recommended. 


[ Q: 2 ] MRCPass - Gastroenterology 

A 45 year old man has had long 
standing jaundice, malaise and poor appetite. 
Over the past 6 months he has lost 2 stones in 
weight. 

On examination he has palmar erythema, 
jaundiced sclerae, spider naevi, hepatomegaly 
and ascites. 

His bloods reveal: 

Bilirubin 50 mmol/I 

ALT 150 U/l 

ALP 240 U/l 

Hep C core antibody Positive 

Hep BsAg Negative 

Hep A antibody Negative 



3- Hepatocellular carcinoma 

4- Hepatitis A infection 

5- Infectious mononucleosis 

Answer & Comments 

Answer: 3- Hepatocellular carcinoma 

30% of patients with hepatitis C develop 
hepatocellular carcinoma over 30 years. 20% 
develop cirrhosis over 20 years. 



Hepatocellular carcinoma 


[ Q: 3 ] MRCPass - Gastroenterology 

A 72 year old woman with 
longstanding hip osteoarthritis presents 
complains of constipation and loose stool. She 
undergoes a sigmoidoscopy. A rectal biopsy 
shows pigment-laden macrophages in the 
lamina propria. 

What is the most likely cause? 

1- Non-steroidal anti-inflammatory drugs 

2- Crohn's disease 

3- Diverticular disease 

4- Whipple's disease 

5- Laxative abuse 

Answer & Comments 

Answer: 5- Laxative abuse 



What is the likely diagnosis? 

1- Abnormal variant hepatitis C 

2- Superimposed hepatitis E infection 


The pigment laden macrophages suggest that 
there is melanosis coli. The most common 
association is with laxatives. To a lesser 



Dr. Khalid Yusuf El-Zohry - Sohag Teaching Hospital (01118391123) 

Ref MRCPass OE OE 2012 PasTest 2009 PassMedicine 2009 PasTest Exam ReviseMRCP 































El-zohry MRCP Questions Bank (Port 1) - 2013 


(For my personal use) 


extent, melanosis coli is also associated with 
ulcerative colitis. 



Melanosis coli 


[ Q: 4 ] MRCPass - Gastroenterology 

A 40 year old man has symptoms of 
lethargy, joint pains and jaundice which have 
occurred over the past 8 months. Four years 
later he became diabetic and was referred to 
our hospital clinic. He was noted to be 
pigmented. 

Haemochromatosis was confirmed by an iron 
saturation of 93.4%, a ferritin concentration of 
1050 pg/l, and typical pre-cirrhotic changes in 
a liver biopsy specimen. 

What is the recommended management? 

1- Venesection 

2- Haemodialysis 

3- Liver transplant 

4- Phenoxybenzamine 

5- Glucagon 



Venesection is recommended when the serum 
ferritin reading is over 1000?g/l. Venesection 
can restore hypothalamic-pituitary-gonadal 
and can reduce liver fibrosis. 

[ Q: 5 ] MRCPass - Gastroenterology 

A 50 year old patient has a 2 year 
history of weight loss and diarrhoea. He also 
gives a history of episodes of flushing. VIP 
syndrome is considered by the admitting 
physician. 

Which one of the following is a feature of 
VIPoma syndrome? 

1- Hypoglycaemia 

2- Hypokalaemia 

3- Induction of VIP release by somatostatin 

4- Anaemia 

5- Increased gastric acid seceretion 



Answer & Comments 

Answer: 2- Hypokalaemia 

VIPomas [vasoactive intestinal peptide (VIP)] 
originate in amine precursor uptake and 
decarboxylation (APUD) cells of the 
gastroenteropancreatic endocrine system and 
in adrenal or extra-adrenal neurogenic sites. 

Features of VIP syndrome include watery 
diarrhea (100%), hypochlorhydria (70% in 
adults), hyperglycemia (20-50% in adults), 
hypercalcemia (20-50% in adults), flushing 
(20% in adults) and hypokalaemia due to 
diarrhoea. 


Answer & Comments 

Answer: 1- Venesection 

In haemochromatosis, the defect is due to 
increased iron absorption, hence 
hydroxypyridone orally helps chelate iron in 
the gut. However, venesection is preferred 
therapy and desferrioxamine infusion 
(another iron chelator) can also be used. 


Initial treatment is directed toward correcting 
volume and electrolyte abnormalities by using 
potassium chloride and sodium bicarbonate. 
Octreotide controls diarrhea in 80% of cases. 
Glucocorticoids reduce symptoms in 50% of 
patients with VIPoma. 


Dr. Khalid Yusuf El-Zohry - Sohag Teaching Hospital (01118391123) 

Ref MRCPass OE OE 2012 PasTest 2009 PassMedicine 2009 PasTest Exam ReviseMRCP 



36 



























El-zohry MRCP Questions Bank (Port 1) - 2013 


(For my personal use) 



Vipoma 


[ Q: 6 ] MRCPass - Gastroenterology 

A 45 year old man has returned from 
holiday in Italy following several episodes of 
bloody diarrhoea which had lasted over two 
weeks. He has lost 2.5 kg in weight and has 
occasional lower abdominal cramping 
discomfort. 

He also a painful swelling of his left elbow and 
right knee. 

What is the likely diagnosis? 

1- Campylobacter infection 

2- Coeliac disease 

3- Tuberculosis 

4- Ulcerative colitis 

5- Gonococcal sepsis 



Answer & Comments 

Answer: 1- Campylobacter infection 

Campylobacter infection is one of commomest 
causes of infective diarrhoea. 

Abdominal pain is often a feature of the 
illness. Diarrhoea is often associated with 
blood. Other causes of bloody diarrhoea are 
salmonella & shigella. 



[ Q: 7 ] MRCPass - Gastroenterology 


A 22 year old man has recently 


returned from India. He complains of fever, 
rigors and headache. 

On examination he had a temperature of 
38°C, a blood pressure of 120/70 mmHg, a 
pulse of 110 bpm. His abdomen was tender in 
right upper quadrant. 

Investigations showed: 

Hb 10.5 g/dL 

WBC 13.5 x 10 9 /L 

Neutrophils 11.2 x 109/1 

Platelets 360 x 10 9 /L 

Blood film No malaria parasites seen 

Aik Phos 420 U/L 

AST 60 U/L 

CRP 110 mg/L 

Stool culture Negative 

Chest x-ray: Small right pleural effusion 

Which of the following investigations would be 
of diagnostic value? 

1- Ultrasound scan of abdomen 

2- Anti endomysial antibody 

3- Typhoid serology 

4- Stool for ova, cysts parasites 

5- Colonosocopy 


Answer & Comments 

Answer: 1- Ultrasound scan of abdomen 

The ultrasound would determine if there is a 
pyogenic liver abscess or amoebic liver 
abscess. The clinical history with associated 
pleural effusion suggests that an abscess 
needs to be excluded (and drained if 
necessary). 



[ Q: 8 ] MRCPass - Gastroenterology 

A 60 year old woman presents with 
diarrhoea. She had a past history of 
radiotherapy for ovarian cancer. Small 
intestine biopsy reveals villous atrophy, crypt 


Dr. Khalid Yusuf El-Zohry - Sohag Teaching Hospital (01118391123) 

Ref MRCPass OE OE 2012 PasTest 2009 PassMedicine 2009 PasTest Exam ReviseMRCP 



37 


























El-zohry MRCP Questions Bank (Port 1) - 2013 


(For my personal use) 


hypertrophy, chronic inflammatory cell 
infiltrate of lamina propria and increase in 
intraepithelial lymphocytes. 

What is the likely diagnosis? 

1- Radiation enteropathy 

2- Coeliac disease 

3- Ischaemic colitis 

4- Crohn's disease 

5- Tropical Sprue 


Answer & Comments 

Answer: 2- Coeliac disease 

Histology of small bowel biopsy specimens 
remains the "gold standard" for diagnosis. 

Features recognised include villus atrophy, 
crypt hyperplasia, degenerate surface 
epithelial cells, and an increase in 
intraepithelial lymphocytes. 

[ Q: 9 ] MRCPass - Gastroenterology 

A 60 year old man has a 5 day history 
of abdominal pains and bloody diarrhoea. He 
is unwell on admission. Blood pressure is 
90/50 mmHg and he has a tender abdomen on 
palpation. 

His Hb is 9.0 g/dl, white cell count 11.0 x 10 9 /L 
and platelet count is 80 x 10 9 /L Urea is 18 
mmol/l and creatinine 250 pmol/l, sodium 137 
mmol/l and potassium 5.5 mmol/l/ Blood film 
shows red cell fragmentation and 
thrombocytopenia. 

Which of the following is most likely to confirm 
the unifying diagnosis? 

1- CT scan of the abdomen 

2- Amoxycillin and metronidazole 

3- Stool sample for E coli 0157 

4- Mesenteric angiography to exclude 
ischaemic colitis 

5- Surgical laparotomy 



Answer & Comments 

Answer: 3- Stool sample for E coli 0157 

The diagnosis is HUS (haemolytic uraemic 
syndrome). The commonest causes are E coli 
0157, but other precipitants are 
Campylobacter, shigella and Clostridium. There 
is classical renal failure, thrombocytopenia 
(HUS-TTP) and evidence of microangiopathic 
haemolysis on the blood film. 

[ Q: 10 ] MRCPass - Gastroenterology 

A 50 year old man has diabetes. He 
has the following results: 

Alanine aminotransferase 35 U/L 

Aspartate aminotransferase 40 U/L 

Fasting plasma glucose 7.4 

Ferritin 500 ug/L, (15-300) 

Which one of the following is an appropriate 
investigation? 

1- Transferrin saturation 

2- Serum electrophoresis 

3- Serum transferrin receptors 

4- Liver biopsy 

5- Urinary PBG 



Answer & Comments 

Answer: 1- Transferrin saturation 

In hemochromatosis, the serum Fe is elevated 
(> 300 mg/dL). The serumtransferrin 

saturation is a sensitive parameter of 
increased Fe and merits evaluation when > 
50%. The serum ferritin is increased. Urinary 
Fe excretion is markedly increased (> 2 mg/24 
h) by the chelating drug deferoxamine (500 to 
1000 mg IM based on the size of the patient), 
and this has been used as a diagnostic test. 

In addition, when the Fe content in the liver is 
significantly increased, an MRI may reflect this 
change. Liver biopsy had been the gold 
standard in diagnosis; it now serves only to 


Dr. Khalid Yusuf El-Zohry - Sohag Teaching Hospital (01118391123) 

Ref MRCPass OE OE 2012 PasTest 2009 PassMedicine 2009 PasTest Exam ReviseMRCP 



38 




























El-zohry MRCP Questions Bank (Port 1) - 2013 


(For my personal use) 


provide evidence of fibrosis (cirrhosis). Gene 
assay (Homozygosity C282y mutations) is also 
an excellent diagnostic test. 



[ Q: 11 ] MRCPass - Gastroenterology 

A 65 year old woman presents with 
dysphagia and intermittent vomiting. 
Endoscopy shows a tight lower oesophageal 
sphincter suggestive of achalasia. 


Which of the following medical therapies is 
most effective? 


1- Diltiazem 


2- Bismuth 

3- Glyceryl trinitrate 

4- Botulinum toxin 

5- Glypressin 


Answer & Comments 

Answer: 4- Botulinum toxin 

Botulinum injections are most effective of all 
the options for relieving a lower oesophageal 
sphincter restriction which leads to achalasia. 
Nifedipine, nitrates or sildenafil can also be 
used, but are less effective. 


[ Q: 13 ] MRCPass - Gastroenterology 

A 50 year old presents with tiredness 
and heavy periods. She is known to drink large 
amounts of alcohol. 



Her investigations reveal: 

Haemoglobin 7.3 g/dl 
MCV 72 fL 

white cell count 7.5 x 10 9 /L 

platelet count 250 x 10 9 /L 

serum ferritin 7 pg/L, (15-300) 

She was commenced on oral iron therapy one 
month later but her haemoglobin 
concentration was 7.8 g/dl. 

What is the likely cause of the failure of her 
haemoglobin to respond? 

1- Folate deficiency 

2- Poor compliance therapy 

3- Sideroblastic anaemia 


4- Alcoholism 

5- Irreversible cause of iron deficiency 


Answer & Comments 


Answer: 2- Poor compliance therapy 


[ Q: 12 ] MRCPass - Gastroenterology 

Which of the following factors 
decreases large intestinal motility? 

1- Lactulose 

2- Parasympathetic activity 

3- CCKPZ 

4- Gastric Distension 

5- Anticholinergic agents 



Answer & Comments 

Answer: 5- Anticholinergic agents 

Anticholinergic agents, e.g. atropine, reduce 
intestinal motility. All the other agents 
increase intestinal motility. 


The likely explanation failure of an iron 
deficiency anaemia to respond iron therapy in 
a patient with heavy periods is poor 
compliance. 


[ Q: 14 ] MRCPass - Gastroenterology 

A 55 year old man with a history of 
heavy alcohol intake presents with acute 
confusion. A diagnosis of hepatic 
encephalopathy is made and treatment with 
lactulose is commenced. 

What is its mode of action in this context? 

1- Reduces absorption of chlordiazepoxide 

2- Inhibits proliferation of ammonia forming 
organisms in the gut 

3- Absorbed from gut 



Dr. Khalid Yusuf El-Zohry - Sohag Teaching Hospital (01118391123) 

Ref MRCPass OE OE 2012 PasTest 2009 PassMedicine 2009 PasTest Exam 




ReviseMRCP 

39 































El-zohry MRCP Questions Bank (Port 1) - 2013 


(For my personal use) 


4- Contraindicated in diabetes mellitus 

5- Causes hypermagnesaemia 

Answer & Comments 

Answer: 2- Inhibits proliferation of ammonia 
forming organisms in the gut 


The syndrome most often occurs in patients 
with underlying thrombotic diathesis, 
including myeloproliferative disorders such as 
polycythemia vera and paroxysmal nocturnal 
hemoglobinuria, pregnancy, tumors, chronic 
inflammatory diseases, clotting disorders, and 
infections. 


Lactulose is used in patients with 
cirrhosis/hepatic encephalopathy. It limits the 
proliferation of ammonia forming gut 
organisms and increases clearance of protein 
load in gut. It causes hypomagnesaemia. 
Chlordiazepoxide absorption is not affected. 

[ Q: 15 ] MRCPass - Gastroenterology 

A 35 year old lady has sudden onset 
right upper quadrant pain and abdominal 
distension. She was well until 5 weeks ago, 
when over several days she rapidly developed 
abdominal distension and pain. She was also 
nauseous and vomiting. 

On examination, temperature was 37.1°C, her 
JVP was not raised and breath sounds were 
clear. Abdominal examination revealed tender 
hepatomegaly, jaundice and gross ascites. 
There was also bilateral ankle oedema. 

Whot is the likely diagnosis? 

1- Dubin Johnson syndrome 

2- Gilbert's syndrome 

3- Budd Chiari syndrome 

4- Lymphoma 

5- Myeloma 



Doppler ultrasonography, as was used in this 
case, is the most effective primary initial 
screening method. CT and magnetic 
resonance angiography are both more 
sensitive than ultrasonography. The gold 
standard for diagnosis is hepatic venography, 
which should be performed when there is a 
high index of clinical suspicion and the results 
of noninvasive testing are either equivocal or 
negative. 

[ Q: 16 ] MRCPass - Gastroenterology 

A 40 year old man who usually drinks 
only 2 units of alcohol a day went on an 
alcohol binge with his friends. On that day, he 
vomited 10 times and was brought to hospital 
feeling very unwell. He has not previously had 
any symptoms of dyspepsia or abdominal 
pains. During physical assessment, he vomits a 
large bowlful of blood. 

Whot is the likely couse of his hoemetemesis? 

1- Oesophageal varices 

2- Duodenal ulcer 

3- Mallory Weiss tear 

4- Gastritis 

5- Gastric outlet obstruction 



Answer & Comments 

Answer: 3- Budd Chiari syndrome 

Budd-Chiari syndrome is a condition induced 
by thrombotic or nonthrombotic obstruction 
to hepatic venous outflow. The classic clinical 
triad of abdominal pain, hepatomegaly, and 
ascites was described by Budd in 1845, and 
the histopathological features were described 
by Chiari at the turn of the 20th century. 


Answer & Comments 

Answer: 3- Mallory Weiss tear 

A Mallory-Weiss tear occurs in the mucous 
membrane typically in the lower oesophagus. 
Mallory-Weiss tears are usually caused by 
forceful or prolonged vomiting or coughing. 
They may also be caused by epileptic 
convulsions. 


Dr. Kholid Yusuf El-Zohry - Sohog Teaching Hospital (01118391123) 

Ref MRCPass OE OE 2012 PasTest 2009 PassMedicine 2009 PasTest Exam ReviseMRCP 






























El-zohry MRCP Questions Bank (Port 1) - 2013 


(For my personal use) 


The tear may be followed by vomiting bright 
red blood or by passing blood in the stool. The 
incidence is 4 in 100,000 people. 



Mallory Weiss tear 


Investigations reveal: 

Hb 8.5 g/dl 

MCV 85 fl 

WCC 6 x 10 9 /l 

Iron 11 (14-29) pmol/l 

Ferritin 20 (15-200) pmol/l 

Folate 2 (3-20) ?g/l 

What investigation should be done? 

1- Ultrasound of abdomen 

2- Small bowel biopsy 

3- Smooth muscle antibodies 

4- ERCP 

5- Rigid sigmoidoscopy 


[ Q: 17 ] MRCPass - Gastroenterology 

A 30 year old lady has altered bowel 
habit. At the gastroenterology clinic, her 
symptoms are reviewed. She has a 2 year 
history of bloating and abdominal pains. Some 
weeks she is constipated and during others 
she has diarrhoea. 

What is the likely diagnosis? 

1- Whipple's disease 

2- Tropical sprue 

3- Coeliac disease 

4- Irritable bowel syndrome 

5- Ulcerative colitis 



Answer & Comments 

Answer: 4- Irritable bowel syndrome 

Weight loss, fevers and blood in the stool are 
features which suggest a different organic 
cause other than irritable bowel syndrome. 


[ Q: 18 ] MRCPass - Gastroenterology 

fit 

# A 60 lady has symptoms of 
intermittent abdominal pain and loose stool 
which have occurred over 1 year. 


Answer & Comments 

Answer: 2- Small bowel biopsy 

The combined iron and folate deficiency 
anaemia as well as symptoms suggestive of 
malabsorption makes celiac disease a likely 
diagnosis. Small bowel biopsy may show 
partial or subtotal villous atrophy. Anti 
endomysial antibodies will also be helpful. 



[ Q: 19 ] MRCPass - Gastroenterology 

A 25 year old bartender has had 
abdominal pains and loose stools for 3 years. 
He also has symptoms of myalgia and 
profound fatigue. He mentions that the 
abdominal pains are often worse after he has 
bread. 


Anti endomysial antibody is positive. An 
enzyme-linked immunosorbent assay test was 
conducted and it showed positive reactions to 
gluten, albumin, lactose, barley, and rye. 

What is the diagnosis? 

1- Diverticulosis 

2- Tropical sprue 

3- Ulcerative colitis 


4- Crohn's disease 


Dr. Khalid Yusuf El-Zohry - Sohag Teaching Hospital (01118391123) 

Ref MRCPass OE OE 2012 PasTest 2009 PassMedicine 2009 PasTest Exam 




ReviseMRCP 

41 




























El-zohry MRCP Questions Bank (Part 1) - 2013 


(For my personal use) 


5- Coeliac disease 


Answer & Comments 

Answer: 5- Coeliac disease 

Signs of coeliac disease include fatigue, weight 
loss, diarrhoea, arthralgia and myalgia. The 
"gold standard" for diagnosing celiac disease is 
through a jejunal mucosal biopsy sample to 
measure the extent of the damage. Another 
method is the enzyme-linked immunosorbant 
assay (ELISA). 

Celiac disease patients cannot tolerate gluten, 
a protein found in the grains wheat, rye, and 
barley. In order to preserve their 
gastrointestinal integrity, patients with celiac 
disease need to take extra precaution when 
selecting foods. 


[ Q: 20 ] MRCPass - Gastroenterology 

A 55 year old man is suspected of 
having a duodenal ulcer recurrence despite 
being on omeprazole. 

Which of the following is the most sensitive 
test in detecting ongoing infection with 
Helicobacter pylori? 

1- The (13C) urea breath test 

2- The urease test on a gastric biopsy 

3- A gastric fundal biopsy culture 

4- Helicobacter pylori serology 

5- Stool culture 



Answer & Comments 

Answer: 1- The (13C) urea breath test 

The urea breath test is expensive but has up 
to 98% sensitivity. The gastric biopsy culture 
has high specificity but sensitivity of 90%. 
Histology of gastric biopsy (not listed above) 
has both high sensitivity and specificity. 


[ Q: 21 ] MRCPass - Gastroenterology 

A 35 year old lady presents with 
abdominal pain to the GP who suspects 
irritable bowel syndrome. 



Which of the following is a recognised feature 
of irritable bowel syndrome? 


1- Lactase deficiency 


2- Bloating 

3- A past history of dysentery 


4- Late development of carcinoma of the 
colon 


5- Diarrhoea but not constipation 


Answer & Comments 

Answer: 2- Bloating 

Abdominal pain relieved by defecation, 
bloating, as well as alternating bowel habits is 
common. 



[ Q: 22 ] MRCPass - Gastroenterology 

A 28 year old intravenous drug user 
complains about severe epigastric pains, 
nausea and vomiting. He has upper Gl 
endoscopy which shows small areas of 
ulceration and white plaques. 


Which of the following is the best treatment 
option ? 


1- Metronidazole 


2- Amoxycillin 

3- Ranitidine 


4- Fluconazole 

5- Aciclovir 


Answer & Comments 

Answer: 4- Fluconazole 

This is a patient with possible HIV who has 
oesophageal candidiasis. Fluconazole, 
ketoconazole and itraconazole can be used. 


Dr. Khalid Yusuf El-Zohry - Sohag Teaching Hospital (01118391123) 

Ref MRCPass OE OE 2012 PasTest 2009 PassMedicine 2009 PasTest Exam ReviseMRCP 



42 































El-zohry MRCP Questions Bank (Port 1) - 2013 


(For my personal use) 



r 


% 


[ Q: 23 ] MRCPass - Gastroenterology 


A couple develops profuse vomiting 
after attending a dinner in a Chinese 
restaurant. They ate at 7 pm but became ill 
early in the next morning. 


Whot is the likely infective organism? 

1- Bacillus cereus 


2- Salmonella enteriditis 

3- Bacillus anthracis 

4- Clostridium perfringens 

5- E. Coli 


moist cooked protein foods and rice (hence its 
association with Chinese takeaways). 



[ Q: 24 ] MRCPass - Gastroenterology 

A 36 year old lady has noticed mild 
jaundice, worsening joint pains and is 
complaining of itching on her skin for the past 
8 months. On examination, she has palpable 
hepatomegaly and a bronze pigmentation on 
her skin. 


Her liver function tests show a bilirubin of 25 
pmol/l, ALT 100 U/l, ALP 480 U/l. ANA is 
negative, anti-mitochondrial antibody is 
positive at 1/320. 

Which of the following medications is helpful? 

1- Desferrioxamine 

2- Hydrocortisone 

3- Propanolol 

4- Ursodeoxycholic acid 

5- Tranexemic acid 


Answer & Comments 


Answer: 4- Ursodeoxycholic acid 


Answer & Comments 

Answer: 1- Bacillus cereus 

Bacillus Cereus food poisoning is a 
gastrointestinal intoxication caused by toxins 
produced by the Bacillus Cereus bacteria. 
There are two types of toxin, - the Diarrhoeal 
and the Emetic toxins. 

Symptoms with the diarrhoeal toxin are 
nausea, cramplike abdominal pains and 
watery diarrhoea. 

The diagnosis is confirmed by a laboratory test 
on a faecal specimen. Bacillus cereus exists in 
normal bacterial and spore forms in foods. 
The normal form is inactivated by cooking, but 
most illness is a result of the multiplication of 
spores during inadequate refrigeration of 


Primary biliary cirrhosis is described. Liver 
transplantation does not cure the condition. 
Histology shows white cell damage to the 
biliary epithelium with non necrotising 
granuloma formation in the portal triad. 

Ursodeoxycholic acid lowers serum bilirubin 
and symptoms of itching, and prolongs the 
progression tow ards requirement for liver 
transplantation. IgM levels are particularly 
high in PBC. 

[ Q: 25 ] MRCPass - Gastroenterology 

•Sf - 

# A 45 year old man has a diagnosis of 
coeliac disease. He presents with a one month 
history of intermittent, colicky, central 
abdominal pain and weight loss of 5 kg. There 
is positive faecal occult blood. 

What is the most appropriate investigation? 


Dr. Khalid Yusuf El-Zohry - Sohag Teaching Hospital (01118391123) 

Ref MRCPass OE OE 2012 PasTest 2009 PassMedicine 2009 PasTest Exam ReviseMRCP 




























El-zohry MRCP Questions Bank (Port 1) - 2013 


(For my personal use) 


1- Duodenal biopsy 

2- Barium enema 

3- Surgical exploration 

4- CT scan of abdomen 


episode may have been precipitated by 
intravenous dextrose administration which 
exhausted his vitamin B reserves. B vitamins 
should be administered to all alcoholic 
patients requiring dextrose. 


5- Colonoscopy 


Answer & Comments 

Answer: 5- Colonoscopy 

Colonoscopy is necessary to exclude a colonic 
carcinoma. There is a relatively high 
prevalence of colorectal neoplasia among 
older patients with coeliac disease who can 
present with iron deficiency or altered bowel 
habit. 


[ Q: 26 ] MRCPass - Gastroenterology 

A 60 year old man has symptoms of 
lethargy and vomiting. He drinks 10 pints of 
beer a day. He was started on an intravenous 
glucose infusion and chlordiazepoxide. For a 
day, he symptomatically improved. However, 
the next day, he became confused and started 
vomiting several times. He also had diplopia 
and was unable to stand. 

What is the likely diagnosis? 



r 


[ Q: 27 ] MRCPass - Gastroenterology 

A 30 year old psychology lecturer has 
deranged liver function tests. She also has 
jaundice, pruritus and xanthelasmata. 


Blood tests reveal elevated levels of 
conjugated bilirubin, alkaline phosphatase, 
gamma-glutamyltranspeptidase and positive 
anti-mitochondrial antibody. She seeks advice 
about the associations of the disease. 


Which one of the following is likely to be 
associated? 


1- Raised IgA 

2- Osteomalacia 

3- Hyperparathyroidism 

4- Nephrotic syndrome 

5- Vitamin A deficiency 


Answer & Comments 


Answer: 2- Osteomalacia 


1- Delirium tremens 

2- Hepatic encephalopathy 

3- Cerebellar stroke 

4- Vitamin B deficiency 

5- Pancreatitis 

Answer & Comments 

Answer: 4- Vitamin B deficiency 

Wernicke's encephalopathy is a neurologic 
disorder of acute onset caused by a thiamine 
deficiency. The condition is characterized by 
ocular abnormalities, ataxia, and a global 
confusional state. 

Wernicke's encephalopathy results from a 
deficiency in vitamin B-l (ie, thiamine). The 


The diagnosis is primary biliary cirrhosis (PBC). 
It is based on a combination of findings 
including cholestatic liver enzymes, a positive 
antimitochondrial antibody (AMA), and 
characteristic liver biopsy findings. Elevated 
serum alkaline phosphatase of liver origin is 
the most common laboratory finding. Fatigue, 
jaundice, pruritus and xanthelasmata are 
other features of primary biliary cirrhosis. 

In the disease, IgM is raised. Primary biliary 
cirrosis is associated with autoimmune 
conditions such as scleroderma and Sjogren's 
syndrome. These conditions are also 
associated with distal RTA (type 1). There is 
xanthelasma formation due to impaired 
cholesterol excretion and also osteomalacia 
due to impaired Vitamin D absorption. 


Dr. Khalid Yusuf El-Zohry - Sohag Teaching Hospital (01118391123) 

Ref MRCPass OE OE 2012 PasTest 2009 PassMedicine 2009 PasTest Exam ReviseMRCP 



44 




























El-zohry MRCP Questions Bank (Port 1) - 2013 


(For my personal use) 



[ Q: 28 ] MRCPass - Gastroenterology 

A 25 year old man presents with pain 
over the right side of the abdomen, diarrhoea, 
poor appetite and weight loss. He is pyrexial 
with a temperature of 39°C. He has oral 
apthous ulcers and a tender right lower 
quadrant in the abdomen. Rectal examination 
is normal. 


His Hb is 12.5g/dl, WCC 14 x 10 9 /L and 
platelets 550 x 10 9 /L Urea is 8 pmol/l and 
creatinine is 90 pmol/l, CRP is 105 mg/I. 

Which of the following is the best test to 
confirm the diagnosis? 

1- Stool cultures 


2- Barium meal and follow through 

3- Ultrasound of abdomen 


4- Colonoscopy 

5- Surgical laparotomy 



Strictures seen on the Barium Follow through 

in Crohn's disease 


Answer & Comments 

Answer: 2- Barium meal and follow through 

The likely diagnosis is Crohn's disease and a 
barium follow through is the best test to 
confirm this. Behcet's disease and Yersinia 
colitis can also present with raised 
inflammatory markers, oral ulceration and 
right sided abdominal pathology. 


^ [ Q: 29 ] MRCPass - Gastroenterology 

ii - 

# A 52 year old male presents with 
general weakness. He drinks approximately 20 
units of alcohol each week and smokes 10 
cigarettes daily. Examination reveals jaundice, 
numerous spider naevi and he has a 
temperature of 37.5°C. Abdominal 
examination reveals hepatosplenomegaly. 

Investigations show : 

Bilirubin 140 micromol/L (1-22) 

Alkaline phosphatase 525 iu/l (45-105) 

AST 178 iu/l (1-31) 

Albumin 28 g/L (37-49) 

Hepatitis B virus surface antigen - negative 

Hepatitis B virus e antigen - negative 

Hepatitis B virus e Antibody- positive 

Hepatitis B core Antigen (anti-HBc) - positive 

Hepatitis B virus DNA - undetectable 

What is the likely diagnosis? 

1- Chronic hepatitis D (delta) infection 

2- New hepatitis A infection 


Dr. Khalid Yusuf El-Zohry - Sohag Teaching Hospital (01118391123) 

Ref MRCPass OE OE 2012 PasTest 2009 PassMedicine 2009 PasTest Exam ReviseMRCP 






















El-zohry MRCP Questions Bank (Port 1) - 2013 


(For my personal use) 


3- Alcoholic liver disease 

4- Chronic hepatitis B infection 

5- New hepatitis C infection 


Answer & Comments 


Answer: 4- Chronic hepatitis B infection 

A negative HbsAg and HbeAg, along with 
positive hepatitis B antibodies (anti-HBc) 
would suggest past or chronic infection. 



^ [ Q: 30 ] MRCPass - Gastroenterology 

fh - 

# A 35 year old man presents with 2 
bowls full of haemetemesis. He drinks 10 pints 
of beer a day and has done so for 10 years. 
Upper Gl endoscopy reveals oesophageal 
varices. 


Which of the following is effective in reducing 
the rotes of rebleeding in the future? 

1- Lisinopril 

2- Propanolol 

3- Simvastatin 

4- Ranitidine 


5- Naproxen 


Answer & Comments 

Answer: 2- Propanolol 

Beta blockers (propanolol, nadolol), nitrates, 
vasopressin analogues and somatostatin 
analogues can be used for reducing rebleeding 
in oesophageal varices. 


[ Q: 31 ] MRCPass - Gastroenterology 

A 30 year old woman who has been 
on the contraceptive pill presents with 
abdominal pain and distension of 5 days 
duration. On examination she has no stigmata 
of chronic liver disease. She has distended 
veins over the anterior abdominal wall. She 
also has ascites, an enlarged tender palpable 
liver with absent hepato-jugular reflux. 

Her ankles are oedematous. 

Whot is the diagnosis? 

1- Congestive cardiac failure 

2- Antiphospholipid syndrome 

3- Fatty liver 

4- Budd Chiari syndrome 

5- Pulmonary hypertension 



Answer & Comments 

Answer: 4- Budd Chiari syndrome 

Budd-Chiari syndrome is thrombosis of the 
hepatic vein, the major vein that leaves the 
liver. Most patients have an underlying 
thrombotic tendency. About 10% have 
polycythemia vera, and about 10% have been 
on the OCP. The most common symptoms in 
Budd-Chiari syndrome are ascites and 
jaundice. 


[ Q: 32 ] MRCPass - Gastroenterology 


A 25 year old woman with cystic 
fibrosis presents with abdominal pain. The 
abdominal pain is colicky and localised in the 
lower abdomen. 


On examination, she was pyrexial and 
tachycardic. Her abdomen was distended. 
There was guarding and bowel sounds were 
present. 

Which of the following is likely to be the 
couse? 


1- Renal Calculi 

2- Ulcerative colitis 



Dr. Kholid Yusuf El-Zohry - Sohog Teaching Hospital (01118391123) 

Ref MRCPass OE OE 2012 PasTest 2009 PassMedicine 2009 PasTest Exam 


ReviseMRCP 








































El-zohry MRCP Questions Bank (Port 1) - 2013 


(For my personal use) 


3- Meconium Ileus Equivalent Syndrome 

4- Pyelonephritis 

5- Irritable Bowel Syndrome 

Answer & Comments 

Answer: 3- Meconium Ileus Equivalent 
Syndrome 


Answer & Comments 

Answer: 4- Cholestyramine 

The patient has had resection of the terminal 
ileum and the cause of the diarrhoea is likely 
to be Bile Acid Malabsorption (BAM). Bile acid 
sequestrants such as cholestyramine should 
help the symptoms of persistent diarrhoea. 


Meconium Ileus is the earliest clinical 
manifestation of cystic fibrosis (CF) and occurs 
in approximately 16% of patients with CF. 
Meconium in patients with ileus has higher 
protein and lower carbohydrate concentration 
than that in control populations. 

Signs of peritonitis include tenderness, 
abdominal wall edema, distension, and clinical 
evidence of sepsis. A palpable mass may 
indicate pseudocyst formation. Surgical 
exploration is indicated for patients with 
progressive distension, signs of peritonitis, or 
clinical deterioration. 

Complicated Meconium Ileus requires 
resection more often than simple cases and 
always requires temporary stomas. 


^ [ Q: 34 ] MRCPass - Gastroenterology 

A -— 

# A 45 year old man has had a 5 year 

history of severe sharp, epigastric pains and 

diarrhoea. He gets 2-3 episodes of these 

symptoms a day. His GP has prescribed proton 

pump inhibitors which has helped partly, but 

he still complains that the symptoms are 

severe. 

Which one of the following might confirm the 
diagnosis? 

1- Amylase 

2- ERCP 

3- C-peptide 

4- Insulin 

5- Gastrin level 



[ Q: 33 ] MRCPass - Gastroenterology 

A 40 year old woman has previously 
had a right hemicolectomy and resection of 30 
cms of terminal ileum for ileocaecal Crohn's 
disease. She has persistent diarrhoea, which is 
not explosive. She does not have abdominal 
pain, bloating, or loss of weight. Investigations 
have failed to demonstrate evidence of 
recurrent Crohn's disease. 


Which is the best therapy for the symptoms? 
1- Prednisolone 


Answer & Comments 

Answer: 5- Gastrin level 

The diagnosis is likely to be Zollinger Ellison 
syndrome which is frequently secondary to a 
gastrinoma. Gastrin levels are significantly 
elevated. The secretin test can help to confirm 
the diagnosis. There is increased gastrin 
elevation (greater than 200 pg/mL) in a 
positive test after secretin is given 
intravenously. 


2- Mesalazine 

3- Azathioprine 

4- Cholestyramine 

5- Loperamide 


There is an association with the MEN 1 
syndrome, so calcium levels should be 
checked (to screen for a parathyroid 
adenoma). 


Dr. Khalid Yusuf El-Zohry - Sohag Teaching Hospital (01118391123) 

Ref MRCPass OE OE 2012 PasTest 2009 PassMedicine 2009 PasTest Exam ReviseMRCP 




























El-zohry MRCP Questions Bank (Port 1) - 2013 


(For my personal use) 


[ Q: 35 ] MRCPass - Gastroenterology 

A 42 year old man has a diagnosis of 
Ulcerative Colitis. He was incidentally found to 
have positive anti smooth muscle antibodies 
by the GP who sent an autoimmune screen. 

Which is the next recommended test for this 
patient? 

1- CT of the abdomen 

2- Colonoscopy 

3- ESR 

4- Endoscopy 

5- Liver function tests 



Answer & Comments 

Answer: 5- Liver function tests 

The features are suggestive of autoimmune 
hepatitis. Liver function tests may 
demonstrated eleveated levels of bilirubin, 
AST and ALT. A liver biopsy may then be 
warranted. 

[ Q: 36 ] MRCPass - Gastroenterology 

f - 

# A 35 year old man has chronic liver 
disease secondary to hepatitis. He comes on 
having had a depressive episode but also feels 
tired and unw ell. He reveals that he has had a 
bottle of wine a day for a week. 

On examination he has gross abdominal 
distension. A peritoneal tap is done. Results 
from the tap show that it has albumin of 25 
g/l, LDH of 320 U/l, glucose 3.5 mmol/I (serum 
glucose 6.5) and a white cell count of 700 per 
mm * 1 2 3 (90% neutrophils). 

What is the diagnosis? 

1- Acute reactivation of hepatitis B 

2- Tuberculous peritonitis 

3- Alcoholic liver disease decompensation 

4- Chylous ascites 

5- Spontaneous bacterial peritiontis 


Answer & Comments 

Answer: 5- Spontaneous bacterial peritiontis 

A white cell count of > 350 mm 3 5 is diagnostic 
of spontaneous bacterial peritonitis. There is 
underlying cirrhotic liver disease and this 
should always be considered related to 
decompensation. 



[ Q: 37 ] MRCPass - Gastroenterology 

A 60 year old male presents with a 
two month history of shorness of breath, 
weight loss and lethargy. He looks pale and is 
jaundiced. 


Investigations show : 
Haemoglobin 6 g/dL 


MCV 106 fL 


White cell count 2.2 x 10 9 /L 
Platelets 60 x 10 9 /L 
Urinalysis: Increased urobilinogen. 
What is the next appropriate test? 

1- Coomb's test 

2- Colonoscopy 

3- Vitamin Bi 2 concentration 

4- Bone marrow aspirate 

5- Reticulocyte count 


Answer & Comments 

Answer: 3- Vitamin Bi 2 concentration 

The clinical picture is one of megaloblastic 
anaemia. B12 and folate measurement is the 
first test to confirm this, and then other tests 
such as blood film and marrow aspirates can 
be done to investigate the cause. 



[ Q: 38 ] MRCPass - Gastroenterology 

A 30 year old man had previous 
bowel resection for acute abdomen. He 
continues to have frequent episodes of bloody 
diarrhea and abdominal pain. 


Dr. Khalid Yusuf El-Zohry - Sohag Teaching Hospital (01118391123) 

Ref MRCPass OE OE 2012 PasTest 2009 PassMedicine 2009 PasTest Exam ReviseMRCP 



48 


































El-zohry MRCP Questions Bank (Part 1) - 2013 


(For my personal use) 


Which one of the following is an X-ray change 
which suggests Ulcerative Colitis? 

1- Cobblestones 

2- Skip lesions 

3- Loss of haustral pattern 

4- Rose thorn ulcers 


3- Gastric biopsy 

4- 24 hour oesophageal pH study 

5- Amylase 

Answer & Comments 

Answer 4- 24 hour oesophageal pH study 


5- Strictures 


Answer & Comments 

Answer: 5- Strictures 

Loss of haustral pattern, lead pipe, and 
shortened colon on the X ray, suggests 
ulcerative colitis. 

In Crohn's disease, transmural inflammation 
with formation of fissures, ulcers and 
granulomata, and cobblestone appearance are 
seen. 



Loss of haustral pattern in Ulcerative Colitis (X 

ray) 


[ Q: 39 ] MRCPass - Gastroenterology 

A 45 year old asian man presents 
with epigastric pains of burning nature. This 
was worst at night. 

He was prescribed Gaviscon for a year and but 
this did not relieve his symptoms entirely. He 
had an endoscopy which did not show any 
significant abnormalities 3 months ago. 

Which is the best test for father investigation? 

1- Repeat endoscopy 

2- Urease breath test 



24 hour oesophageal pH study is the 
investigation of choice for severe symptoms 
suggestive of oesophageal reflux. 


[ Q: 40 ] MRCPass - Gastroenterology 

A 30 year old man has had a 10 year 
history of chronic diarrhea. He has developed 
episodes of abdominal pain after eating wheat 
products, and is suspected of having celiac 
disease. 

Which one of the following is most likely to be 
associated? 

1- Increased serum ferritin 

2- Osteomalacia 

3- Anti double stranded DNA antibodies 

4- Anal ulcers 

5- Anti-Ro antibodies 



Answer & Comments 

Answer: 2- Osteomalacia 

Celiac disease usually presents with iron 
deficiency. Poor calcium absorption can result 
in osteomalacia. Celiac disease can cause 
mouth ulcers. Anti gliadin and anti endomysial 
antibodies are associated. 


[ Q: 41 ] MRCPass - Gastroenterology 

A 45 year old lady has rectal bleeding 
during bowel movements for 10 weeks. She 
has severe lower abdominal pains. Her 
appetite is poor and she has also lost 7 kg in 
weight. On examination she has a tender left 
lower abdomen and loud bowel sounds. Her 



Dr. Khalid Yusuf El-Zohry - Sohag Teaching Hospital (01118391123) 

Ref MRCPass OE OE 2012 PasTest 2009 PassMedicine 2009 PasTest Exam 




ReviseMRCP 

49 
































El-zohry MRCP Questions Bank (Port 1) - 2013 


(For my personal use) 


abdomen is not distended. Rectal examination 
reveals small streak of blood. 

Investigations: 

Hb is ll.lg/dl WCC 13.5 x 10 9 /L 

platelets 600 x 10 9 /L urea 6 mmol/l 

creatinine 100 pmol/l CRP 80 mg/I 

A rigid sigmoidoscopy shows inflammatory 
changes with ulceration and areas of bleeding. 

Which of the following treatment options is 
the best at present? 

1- Intravenous hydrocortisone 

2- Amoxycillin and metronidazole 

3- 5-aminosalicylate given intravenously 

4- Anti-TNFa antibody infusion 

5- Intravenous gamma globulin 


Answer & Comments 

Answer: 1- Intravenous hydrocortisone 

The features are consistent with colitis, 
probably on a background of likely ulcerative 
colitis. Steroids are used in severe cases of 
colitis. If there are mild episodes of colitis then 
5-aminosalicylates such as sulfasalazine can be 
used. Anti TNF a antibody is used for severe 
Crohn's disease. 



Ulcerative Colitis 



[ Q: 42 ] MRCPass - Gastroenterology 

A 45 year old man is referred to the 
hospital for assessment. Over the past year he 
has been complaining of joint paints and was 
thought to have osteoarthritis. 6 months 
earlier, he noted gradual onset of fatigue, 
decreased libido, and erectile dysfunction. He 
has also been progressively breathless. He did 
not have a cough, a fever, night sw eats, or 
visual changes. 


On physical examination, the patient was a 
thin but well-developed man who was not in 
distress. The blood pressure was 100/60 mm 
Hg, and the heart rate 88 beats per minute. 
The skin was tanned, with no spider angiomas 
or palmar erythema. The abdominal 
examination revealed palpable hepatomegaly 
and he was mildly jaundiced. The testicles 
were each estimated to be 18 ml without 
masses, and the prostate examination was 
normal. 


Blood results show : 
sodium 135 mmol/l 
potassium 4.5 mmol/l 
urea 5 mmol/l 
creatinine 100 pmol/l 
ALT 120 (5-35) U/l 
AST 135 (1-31) U/l 
ALP 132(20-120) U/l 
Bilirubin 36 (1-22) pmol/l 
Albumin 38 (37-49) g/l 
What is the diagnosis? 

1- Multiple Endocrine Neoplasia type II 

2- Carcinoid syndrome 

3- Primary biliary cirrhosis 

4- Haemochromatosis 

5- Amyloidosis 


Answer & Comments 


Answer: 4- Haemochromatosis 



Dr. Khalid Yusuf El-Zohry - Sohag Teaching Hospital (01118391123) 

Ref MRCPass OE OE 2012 PasTest 2009 PassMedicine 2009 PasTest Exam 


ReviseMRCP 























El-zohry MRCP Questions Bank (Port 1) - 2013 


(For my personal use) 


A combination of hepatomegaly, diabetes, and 
hyperpigmentation, reflecting parenchymal 
iron loading of the liver, pancreas, and skin is 
suggestive of haemochromstosis. 

In haemochromatosis , joint deposition of iron 
occurs, causing arthropathy. Increased iron 
deposition in the skin stimulates increased 
melanin production, and may cause a bronze / 
tan skin. There is an increased incidence in 
males. Haemochromatosis is a recognised 
cause of restrictive cardiomyopathy which has 
caused this patient to be breathless. 



Myocardial cells with excessive iron 
deposition (stained blue) 


[ Q: 43 ] MRCPass - Gastroenterology 

An obese 55 year old woman with 
Type II diabetes is referred as she has been 
found to have raised ALT and AST levels. She 
does not have a history of pruritus or 
jaundice. She is not on any medication and 
does not take alcohol. On examination, she 
has a smooth hepatomegaly of 3 cm edge. 
There are no xanthelasmata and she does not 
have signs of chronic liver disease. 

What is the likely diagnosis? 

1- Non-alcoholic fatty liver disease 

2- Neuroacantocythosis 

3- Chronic hepatitis 

4- Hepatocellular carcinoma 

5- Hepatorenal syndrome 



Answer & Comments 

Answer: 1- Non-alcoholic fatty liver disease 

This diagnosis is non-alcoholic fatty liver 
disease. The mildest type is simple fatty liver 
(steatosis), an accumulation of fat within the 
liver that usually causes no liver damage. This 
disease is usually nonprogressive and rarely 
causes liver cirrhosis. A potentially more 
serious type, nonalcoholic steatohepatitis 
(NASH), is associated with liver fibrosis. 
Nonalcoholic fatty liver disease affects more 
women than men, and is associated with 
insulin resistance/diabetes and obesity. 


[ Q: 44 ] MRCPass - Gastroenterology 

A 46 year old man is known to have 
alcoholic liver cirrhosis. He drinks 60 units of 
alcohol per week. He presents unwell and 
confused. Temperature is 38 C and blood 
pressure is 96/50 mmHg. Abdominal palpation 
reveals hepatomegaly and ascites. 

What should be done next? 

1- Albumin infusion 

2- Blood cultures 

3- Diagnostic ascitic tap 

4- Liver biopsy 

5- Transjugular intrahepatic shunting 



Answer & Comments 

Answer: 3- Diagnostic ascitic tap 

There is a high chance of spontaneous 
bacterial peritonitis in this patient with 
cirrhotic liver disease. When analysis of ascitic 
fluid reveals a white blood cell count of more 
than 250 cells/cc, SBP is likely. Cefotaxime 
should be commenced after a tap is done and 
blood cultures are sent. 

[ Q: 45 ] MRCPass - Gastroenterology 

A 60 year old man complains of a 6 
month history of lethargy, fever, weight loss, 



Dr. Khalid Yusuf El-Zohry - Sohag Teaching Hospital (01118391123) 

Ref MRCPass OE OE 2012 PasTest 2009 PassMedicine 2009 PasTest Exam 




ReviseMRCP 

51 




























El-zohry MRCP Questions Bank (Port 1) - 2013 


(For my personal use) 


arthralgia and diarrhoea. Jejunal biopsy 
reveals flattened mucosa with evidence of 
periodic acid Schiff (PAS) positive 
macrophages. 

What is the diagnosis? 

1- Tuberculosis 

2- Tropical sprue 

3- Whipple's disease 

4- Helminthic infection 

5- Ulcerative colitis 


Answer & Comments 


Answer: 3- Whipple's disease 


Whipple's disease can affect any organ, but 
predominantly affects the small bow el, 
causing a malabsorption syndrome. The 
organism (Tropheryma whippeii) can be 
identified both between within abnormal 
macrophages, which stain with PAS. 
Treatment is with a prolonged course of 
antibiotics eg. Iv penicillin and streptomycin 2 
weeks, followed by 1 year of doxycycline. 



Histology showing Whipple's disease 


[ Q: 46 ] MRCPass - Gastroenterology 

A 45 year old man has painless 
jaundice. He has pale stools and dark urine. 
On examination he is deeply jaundiced and 
has scratch marks all over his body. On 



palpation of the abdomen Courvoisier's sign is 
positive. 

Where is the level of biliary obstruction? 

1- Bile canaliculi 

2- Intrahepatic ducts 

3- Hepatic duct 

4- Accessory duct 

5- Common bile duct 


Answer & Comments 

Answer: 5- Common bile duct 

Courvoisier's sign is a palpable distended gall 
bladder on examination of the abdomen. 
Obstruction below the level of the cystic duct 
causes this, probably due to carcinoma of the 
pancreas. 



[ Q: 47 ] MRCPass - Gastroenterology 

A 35 year old woman presents with 
abdominal pain, weight loss, diarrhoea and 
mouth ulcers. Full blood count reveals 
normocytic normochromic anaemia, 
inflammatory markers are raised, and 
biochemical investigation reveals a raised 
alkaline phosphatase. Barium followthrough 
shows terminal ileal inflammatory disease. 

Long term remission may be maintained by 
the use of which of the following drugs? 

1- Mesalazine 

2- Prednisolone 


3- Azathioprine 

4- Ciclosporin 

5- Infliximab 


Answer & Comments 

Answer: 3- Azathioprine 

Remission of Crohn's disease may be achieved 
by the use of steroids, elemental diet, surgery 
or Infliximab. Mesalazine may maintain 
remission if induction has been achieved by 


Dr. Khalid Yusuf El-Zohry - Sohag Teaching Hospital (01118391123) 

Ref MRCPass OE OE 2012 PasTest 2009 PassMedicine 2009 PasTest Exam ReviseMRCP 






























El-zohry MRCP Questions Bank (Port 1) - 2013 


(For my personal use) 


surgery and if an 8-w eek course of 
metronidazole is administered. In other 
scenarios, the most useful drug to maintain 
remission is azathioprine. 



[ Q: 48 ] MRCPass - Gastroenterology 

A 55 year old woman presents with 
an 2 month history of diarrhoea with no blood 
or mucous, colicky abdominal pain, vomiting 
and 1 stone weight loss. 


She has a history of arthritis for which she 
takes diclofenac regularly. On examination she 
was afebrile and looked pale. The abdomen 
was diffusely tender but soft. Bowel sounds 
were normal and rectal examination was 
normal. 


Investigations showed: Hb 8.5 g/dL, MCV 68 fl, 
CRP 160, coeliac antibodies negative. 
Colonoscopy revealed several areas of 
superficial ulceration in the ascending colon. 
Biopsies from these areas showed non-specific 
inflammation. 


What is the likely diagnosis? 

1- Ulcerative colitis 


2- Crohn's disease 


3- Non-steroidal anti-inflammatory drug 
colopathy 

4- Coeliac disease 

5- Tropical sprue 


Answer & Comments 

Answer: 3- Non-steroidal anti-inflammatory 
drug colopathy 

NSAID-colopathy can mimic Crohn's and 
ulcerative colitis. Presenting symptoms and 
signs include iron-deficiency anaemia and 
crampy abdominal pain, alteration of bowel 
habit, weight loss, nausea and vomiting. 
Symptoms usually resolve with 
discontinuation of the drugs. 



[ Q: 49 ] MRCPass - Gastroenterology 

A 65 year old ex-publican presented 
to his GP complaining of difficulty 
swallowing for the past two months. The 
difficulty started several weeks ago with food 
sticking. He also complained more recently of 
pain on swallowing. 


An OGD and biopsy confirms adenocarcinoma 
of the oesophagus. 


Which underlying condition is associated? 

1- Alcoholism 


2- Oesophageal candidiasis 

3- Ulcerative colitis 


4- Plummer vinson syndrome 

5- Tropical sprue 


Answer & Comments 

Answer: 4- Plummer vinson syndrome 

Plummer Vinson syndrome causes iron 
deficiency anaemia and post cricoid 
oesophageal web and achalasia. 

There is an increased incidence of 
oesophageal cancer. 



Barium Swallow showing a stricture indicating 

oesophageal cancer 


Dr. Khalid Yusuf El-Zohry - Sohag Teaching Hospital (01118391123) 

Ref MRCPass OE OE 2012 PasTest 2009 PassMedicine 2009 PasTest Exam ReviseMRCP 



























El-zohry MRCP Questions Bank (Port 1) - 2013 


(For my personal use) 


[ Q: 50 ] MRCPass - Gastroenterology 

A 40 year old man has recurrent 
episodes of bloody diarrhea and lower 
abdominal cramping. Histologic evaluation of 
a biopsy from flexible sigmoidoscopy reveals 
acute inflammation without architectural 
distortion consistent with ulcerative colitis. 

Which of the following conditions is associated 
with the condition? 

1- Gallstones 

2- Sclerosing cholangitis 

3- Erythema nodosum 

4- Renal calculi 

5- Vitamin Bi 2 deficiency 



Answer & Comments 

Answer: 2- Sclerosing cholangitis 

75% of patients with primary sclerosing 
cholangitis have inflammatory bowel disease, 
especially ulcerative colitis. 

Primary sclerosing cholangitis (PSC) is a 
chronic cholestatic liver disease of unknown 
etiology. In the absence of underlying bile 
duct abnormalities, a generalized beading and 
stenosis of the intrahepatic and extrahepatic 
biliary tree characterize PSC. PSC is usually 
progressive, leading to cirrhosis, portal 
hypertension, and liver failure. 

Typically, a cholestatic jaundice picture is seen 
with the liver function tests. 



[ Q: 51 ] MRCPass - Gastroenterology 

A 40 year old woman presents with 
abdominal pain and watery diarrhoea. She has 
joint pains and also has been investigated for 
infertility. She was given a proton pump 
inhibitor by her GP and this helped to relieve 
some of her symptoms. 

Investigations show : 

Haemoglobin 13 g/dl 

Calcium 2.90 mmol/I 

Albumin 40 g/l 

Phosphate 0.8 mmol/l 

CRP 10 mg/I 

Endoscopy - multiple small duodenal ulcers 
H. pylori test - negative 
What is the likely diagnosis? 

1- NSAID induced duodenal ulceration 

2- Multiple endocrine neoplasia 

3- Small Bowel Lymphoma 

4- Coeliac disease 

5- Addison's disease 



Answer & Comments 

Answer: 2- Multiple endocrine neoplasia 

MEN 1 would fit the clinical diagnosis. The 
abdominal symptoms are likely to be due to 
gastrinoma and hypercalcaemia due to 
parathyroid adenoma. Infertility could be 
explained by a prolactinoma. 


[ Q: 52 ] MRCPass - Gastroenterology 

A 65 year man who is on warfarin for 
atrial fibrillation is complaining of worsening 
breathlessness over the last 2 months. On 
examination, he has an irregular heart beat, 
blood pressure of 110 / 60 mmHg and he 
looked pale. 

There was no evidence of organomegaly. 
Investigations show: 



Dr. Khalid Yusuf El-Zohry - Sohag Teaching Hospital (01118391123) 

Ref MRCPass OE OE 2012 PasTest 2009 PassMedicine 2009 PasTest Exam ReviseMRCP 



54 



























El-zohry MRCP Questions Bank (Port 1) - 2013 


(For my personal use) 


Haemoglobin 8.5 g/dL 
MCV 70 fL 
PLT 160 x 10 9 /L 

His upper gastrointestinal tract endoscopy and 
colonoscopy were normal. 


pain on swallowing and has regurgitation of 
food Swallowed several hours earlier. He has 
not lost weight. Barium Swallow demonstrates 
proximal dilatation of the oesophagus and 
failure of relaxation of the lower oesophageal 
sphincter. 


What is the next best investigation? 

1- Barium enema 

2- Small bowel enema 

3- Capsule endoscopy 

4- Mesenteric angiography 

5- CT abdomen 


Answer & Comments 

Answer: 3- Capsule endoscopy 

There is microcytic anaemia. In this case, there 
is still a possibility of angiodysplasia involving 
the small bowel which may not have been 
identified with OGD or colonoscopy. With 
Capsule Endoscopy, a patient Swallows a small 
pill which emits a radio frequency which is 
picked up by a sensor that the patient wears 
over several hours. Images are then 
downloaded as a continuous movie . At 
present, this technique is ideal for patients 
with suspected Gl bleeding within the small 
bowel. 



[ Q: 53 ] MRCPass - Gastroenterology 

A 50 year old man presents with a 
history of intermittent, but slowly progressive 
dysphagia for both solids and liquids. He has 



Which treatment option is best in the long 
term ? 

1- Sengstaken Blakemore tube 

2- Amyl nitrite 

3- Oesophageal myotomy 

4- Intrasphincteric botulinum toxin 

5- Terlipressin 


Answer & Comments 


Answer: 3- Oesophageal myotomy 


The diagnosis is achalasia of the cardia. This is 
a functional obstruction at the low er 
oesophageal sphincter caused by a failure of 
relaxation. Heller's oesophageal myotomy is 
the best treatment option, it can be done via 
an abdominal incision or laparascopically. 



[ Q: 54 ] MRCPass - Gastroenterology 

A 35 year old woman presented with 
a 5-month history of weight loss (half a stone), 
anorexia and generalized pruritus. On 
examination, she was jaundiced with 




Dr. Khalid Yusuf El-Zohry - Sohag Teaching Hospital (01118391123) 

Ref MRCPass OE OE 2012 PasTest 2009 PassMedicine 2009 PasTest Exam ReviseMRCP 






























El-zohry MRCP Questions Bank (Port 1) - 2013 


(For my personal use) 


numerous spider naevi, scratch marks, palmar 
erythema and hepatosplenomegaly. 

Investigations showed : 

Haemoglobin: 9.5 g/dl 

WCC: 7 x 10 9 /l 

erythrocyte sedimentation rate: 140mm/h 

serum albumin: 41 g/l 

serum bilirubin: 34 pmol/l 

alanine transaminase: 152 iu/l 

aspartate transaminase: 164 iu/l 

alkaline phosphatase: 83 iu/l 

The prothrombin time was prolonged but urea 
and electrolytes, calcium and phosphate 
concentrations were normal. 

In addition to the above findings, which one of 
the following would make the diagnosis of 
autoimmune hepatitis likely ? 

1- HLADR2 

2- HLADR6 

3- Anti neutrophil cytoplasmic antibodies 

4- Antimitochondrial antibodies 

5- High IgG 

Answer & Comments 

Answer: 5- High IgG 

Antinuclear antibodies ANA) of IgG class are 
frequently strongly positive (e.g. to a titre of 
1/10000) in autoimmune hepatitis. Antibodies 
to LKM, dsDNA and to smooth muscle are also 
good markers. 

HLA-DR3 and DR4 antigen are associated with 
autoimmune hepatitis. 


ALT 75 (5-35) U/l 
AST 70(1-31) U/l 
ALP 350 (20-120) U/l 
Bilirubin 70 (1-22) pmol/l 
Albumin 38 (37-49) g/l 
What is the likely diagnosis? 

1- Primary biliary cirrhosis 

2- Gallstones 

3- Cholangiocarcinoma 

4- Intrahepatic cholestasis of pregnancy 

5- Viral hepatitis 

Answer & Comments 

Answer: 4- Intrahepatic cholestasis of 
pregnancy 

Intrahepatic cholestasis of pregnancy usually 
presents during the third trimester, at a mean 
of 30 weeks of gestation. The characteristic 
symptom is itching (pruritus gravidarum), 
which involves the trunk, extremities, palms, 
and soles. The itching may be severe, and it is 
often worse at night. 

Jaundice develops in 20 to 60 percent of 
women one to four weeks after the onset of 
itching. The features of obstructive jaundice, 
including pale stools and dark urine, may be 
present, but patients do not have 
constitutional symptoms. Intrahepatic 
cholestasis is associated with an increased risk 
of prematurity and stillbirth. Women with 
intrahepatic cholestasis should be treated at 
centers capable of caring for premature 
infants. Cholestyramine, given in divided 
doses totaling 10 to 12 g per day, may help 
relieve pruritus. 


^ [ Q: 55 ] MRCPass - Gastroenterology 



A 

* A 78 vear nld laHv i<; 34 wppk^ 

|_ s 

[ Q: 56 ] MRCPass - Gastroenterology 

/ \ Z—m W V ^ Cl 1 1 1 Cl ^ V 1 O 1 V V ^ ^ lx o 

pregnant. This is her second pregnancy, the 

A 35 year old secretary has noticed 


first pregnancy was uneventful. She has 
pruritus and on examination, was mildly 
jaundiced. Liver function tests showed: 


jaundice over the last several months. Around 
this time, she has also noticed itchiness all 
over her body. On examination, she has 


Dr. Khalid Yusuf El-Zohry - Sohag Teaching Hospital (01118391123) 

Ref MRCPoss OE OE 2012 PosTest 2009 PassMedicine 2009 PosTest Exam ReviseMRCP 


























El-zohry MRCP Questions Bank (Part 1) - 2013 


(For my personal use) 


xanthelasma around her eyes and palpable 
hepatomegaly. 

Her blood tests show Bilirubin 110 pmol/l (1- 
22), AST of 240 U/l (1-30) and ALP of 650 U/l 
(5-35 U/l). AMA is positive. 

If she has primary biliary cirrhosis, what is the 
liver biopsy likely to show ? 

1- Granulomatous changes of hepatocytes 

2- Fatty changes of the liver parenchyma 

3- Piecemeal necrosis and fibrosis around 
portal veins 

4- Collagen layering around bile ducts 

5- White cell infiltrates causing biliary duct 
destruction 

Answer & Comments 

Answer: 5- White cell infiltrates causing biliary 
duct destruction 


^ [ Q: 57 ] MRCPass - Gastroenterology 

n - 

* A 45 year old man presents with an 
episode of severe haematemesis following a 
heavy meal. He also has had a day's history of 
malena. On examination, he has swelling 
around the gastric region and abdomen. 

What is the most likely diagnosis? 

1- Gastric ulcer 

2- Mallory weiss tear 

3- Oesophageal varices 

4- Oesophageal web 

5- Pancreatic carcinoma 

Answer & Comments 

Answer: 3- Oesophageal varices 

Post prandial swelling suggests oesophageal 
varices (portal hypertension). 


Inflammatory changes with biliary destruction 
are suggestive of primary biliary cirrhosis. 
Granulomatous changes would suggest 
sarcoidosis or Wegener's granulomatosis. 
Piecemeal necrosis and fibrosis suggests 
chronic hepatitis. 

Primary biliary cirrhosis can present with 
jaundice, pruritus, xanthelasma and signs of 
chronic liver disease. 

Antibodies (Anti Mitochondrial Antibody) and 
liver biopsy are helpful to distinguish between 
the PBC, autoimmune hepatitis and primary 
sclerosing cholangitis. 


Postprandial hyperemia leads to an increase in 
portal pressure that may contribute to 
esophageal variceal rupture. 


t 


[ Q: 58 ] MRCPass - Gastroenterology 

A 30 year old lady presents with 
bilateral swelling and discomfort around her 
face. This caused her pain when she is chew 
ing. It started 2 days ago and she also has mild 
fever and headache. She mentioned she had 
been in contact with someone with a similar 
illness 2 weeks ago. On examination, she has 
bilateral parotid enlargement which are 
tender. 


Which of the following is a likely cause? 

1- Myotonic dystrophy 

2- Marfan's syndrome 

3- Myeloma 

4- Carotid artery stenosis 

5- Mumps 


Histology showing destruction of bile ductules 
within the triads of the liver in PBC 



Dr. Khalid Yusuf El-Zohry - Sohag Teaching Hospital (01118391123) 

Ref MRCPass OE OE 2012 PasTest 2009 PassMedicine 2009 PasTest Exam ReviseMRCP 



























El-zohry MRCP Questions Bank (Port 1) - 2013 


(For my personal use) 


Answer & Comments 

Answer: 5- Mumps 

There are many causes of parotid swellings. 
Viruses include mumps, coxsackie A virus, 
parainfluenzae virus, CMV and varicella zoster 
virus. Hypothyroidism can do so, but not 
hypoparathyroidism. 

The incubation period for mumps is usually 18 
to 21 days. Mumps is caused by a 
paramyxovirus, and are spread from person to 
person by saliva droplets or direct contact 
with articles that have been contaminated 
with infected saliva. The parotid glands (the 
salivary glands between the ear and the jaw) 
are usually involved. Unvaccinated children 
between the ages of 2 and 12 are most 
commonly infected, but the infection can 
occur in other age groups. Orchitis (swelling of 
the testes) occurs in 10-20% of infected males, 
but sterility only rarely ensues; a viral 
meningitis occurs in about 5% of those 
infected. 



Parotid Swelling 


[ Q: 59 ] MRCPass - Gastroenterology 

A 50 year old man has a history of 
Crohn's colitis. He has relapses despite being 
on mesalazine. The last relapse, treated high 
doses of steroids, was complicated by 
gastrointestinal bleeding. 

Investigations show : 

Haemoglobin 11.8 g/L 



MCV 75 fL 
MCH 25 pg (28-32) 

White cell count 9 x 10 9 /L 

Platelets 350 x 10 9 /L 

Serum total protein 72 g/L (61-76) 

Serum albumin 28 g/L (37-49) 

Serum CRP 45 mg/L (<10) 

Abdo X ray is normal 

Which of the following is the most appropriate 
management? 

1- A trial of oral metronidazole 

2- Treatment with azathioprine 

3- Treatment withoral budesonide 

4- Ileostomy 

5- Total colectomy 


Answer & Comments 

Answer: 2- Treatment with azathioprine 

This patient has not responded to first line 
therapy. The next step is a trial of 
azathioprine, which used as a steroid sparing 
agent. This particularly so when theere are 
side effects from previous steroid treatment. 

Metronidazole is rarely effective in the 
treatment of active Crohn's colitis. Given that 
Crohn's disease can recur following surgery, 
surgery should not be undertaken without a 
trial of second line therapies such as 
azathioprine first. 

[ Q: 60 ] MRCPass - Gastroenterology 

A 35 year old lady has developed 
worsening pruritus and jaundice over the past 
year. Investigations revealed a positive 
antimitochondrial antibody and a liver biopsy 
confirmed the diagnosis of primary biliary 
cirrhosis. 

Which of the following is associated with the 
condition? 



Dr. Khalid Yusuf El-Zohry - Sohag Teaching Hospital (01118391123) 

Ref MRCPass OE OE 2012 PasTest 2009 PassMedicine 2009 PasTest Exam ReviseMRCP 





























El-zohry MRCP Questions Bank (Port 1) - 2013 


(For my personal use) 


1- Raised IgA 

2- Male 

3- Xanthelasma 

4- Anti gliadin antibody 

5- Diverticular disease 


Answer & Comments 

Answer: 3- Xanthelasma 

Primary biliary cirrhosis is more common in 
females and is usually associated with raised 
IgM levels. Xanthelasma are present due to 
cholestasis. Connective tissue disorders such 
as Sjogren's syndrome are associated. A 
positive antimitochondrial antibody and liver 
biopsy are best ways to confirm the diagnosis. 



Xanthelasma 


[ Q: 61 ] MRCPass - Gastroenterology 

A 50 year old man presents with 
lower back pains and lethargy. Investigations 
subsequently confirmed obstructive 
nephropathy and retroperitoneal fibrosis. His 
medication is reviewed on the ward round. 

Which medication is well known to cause 
retroperitoneal fibrosis ? 

1- Isoniazid 

2- Rifampicin 

3- Cyclosporin 

4- Aspirin 

5- Methysergide 



Answer & Comments 

Answer: 5- Methysergide 

Retroperitoneal fibrosis is a disease featuring 
the proliferation of fibrous tissue in the 
retroperitoneum, the compartment of the 
body containing the kidneys, aorta, renal tract 
and various other structures. It may present 
with low er back pain, renal failure, 
hypertension, deep vein thrombosis and other 
obstructive symptoms. 

Its may have an autoimmune etiology. One- 
third of the cases are secondary to 
malignancy, medication (methysergide, 
hydralazine, beta blockers), aortic aneurysm 
or certain infections. 

Methyldopa, methysergide, amphetamines, 
beta blockers, cocaine and LSD are drugs 
which cause retroperitoneal fibrosis. 
Azathioprine is used to treat the condition. 


I 


[ Q: 62 ] MRCPass - Gastroenterology 

A 35 year old man has HIV infection. 
Recently his CD4 count has dropped to 250 
from 600 and viral load has dropped from 
10,000 copies to 2000 copies /ml. He has now 
developed diarrhoea. 


Which one of the following is likely to cause 
diarrhoea in a HIV positive patient? 


1- Staph aureus 


2- Strep pyogenes 

3- Cryptosporidium 

4- Mycobacterium tuberculosis 

5- Neisseria meningitidis 


Answer & Comments 

Answer: 3- Cryptosporidium 

Common organisms such as entamoeba, 
salmonella, giarda, Campylobacter, 
Cryptosporidium, cyclospora, mycobacterium 
and also viral causes can cause diarrhoea in 
HIV positive patients. 


Dr. Khalid Yusuf El-Zohry - Sohag Teaching Hospital (01118391123) 

Ref MRCPass OE OE 2012 PasTest 2009 PassMedicine 2009 PasTest Exam 




ReviseMRCP 

59 




























El-zohry MRCP Questions Bank (Port 1) - 2013 


(For my personal use) 


[ Q: 63 ] MRCPass - Gastroenterology 

A 70 year old man is admitted with 
pruritus, lethargy and jaundice. He has not 
drunk any alcohol for five years. 

One month ago, he completed a course of 
CoAmoxiclav which was prescribed by his GP. 
He also takes diclofenac for osteoarthritis. 

Investigations reveal: 

Albumin 37 g/L 

Bilirubin 180 umol/L 

AST 220 iu/L 

Alkaline Phosphatase 210 iu/l 

Abdominal ultrasound reveals gallsones 
without biliary duct dilatation. 

Whot is the likely cause of his jaundice? 

1- Hepatitis C infection 

2- Diclofenac 

3- Cholangiocarcinoma 

4- Co-Amoxiclav 

5- Alcoholic cirrhosis 



Answer & Comments 


Answer: 4- Co-Amoxiclav 


5- Spironolactone 


Answer & Comments 

Answer: 4- Terlipressin 

Octreotide and terlipressin are used to 
prevent secondary variceal bleeding. 
Octreotide is a somatostatin analogue. 



[ Q: 65 ] MRCPass - Gastroenterology 

A 40 year old man presents with 
slurring of his speech and unsteadiness. 


He has a coarse tremor in his hands and 
ataxic, shuffling gait on examination. 


His blood tests show normal renal function. 


AST is 250 (1-30) U/l, ALP is 350 (1-250) U/l 
and Bilirubin is 45 (1-22). 

Which is the best investigation to confirm a 
diagnosis? 

1- Hepatitis screen 

2- Serum Cu and caeruloplasmin 

3- MRI of brain 

4- Serum ferritin 

5- IT 15 gene test for Huntington's 


Co amoxiclav (Augmentin) can cause 
hepatitic/cholestatic drug induced jaundice. A 
4 week delay in symptoms signs is not 
uncommon. 

^ [ Q: 64 ] MRCPass - Gastroenterology 

* A 50 year old alcoholic patient 
presents with upper Gl bleeding. Urgent 
endoscopy confirms oesophageal varices. 

Which of the following can be used to prevent 
secondary variceal haemorrhage? 

1- Thyroxine 

2- Cefotaxime 

3- Frusemide 

4- Terlipressin 


Answer & Comments 

Answer: 2- Serum Cu and caeruloplasmin 

Wilson's disease can present with early 
Parkinsons, psychiatric symptoms and 
dysarthria. It also causes abnormal liver 
function. It is an autosomal recessive 
condition. Serum copper and caeruloplasmin 
are low . Urinary copper excretion is raised. 


[ Q: 66 ] MRCPass - Gastroenterology 

A 30 year old Irish lady has a 15 year 
history of non-specific abdominal pains and 
diarrhoea. The symptoms are worse with 
certain foods. Her AXR is unremarkable. 

Which of the following tests is most likely to 
confirm the diagnosis? 



Dr. Khalid Yusuf El-Zohry - Sohag Teaching Hospital (01118391123) 

Ref MRCPass OE OE 2012 PasTest 2009 PassMedicine 2009 PasTest Exam ReviseMRCP 



60 

































El-zohry MRCP Questions Bank (Port 1) - 2013 


(For my personal use) 


1- Amylase 

2- Anti gliaden and anti endomysial antibodies 

3- Smooth muscle antibodies 

4- Ferritin 

5- B12 levels 


Answer & Comments 

Answer: 2- Anti gliaden and anti endomysial 
antibodies 

Coeliac disease (gluten sensitive enteropathy) 
is relatively common in Ireland. The patient's 
history suggests gluten sensitivity. Gluten is 
found in wheat, rye and barley. 


[ Q: 68 ] MRCPass - Gastroenterology 

A 50 year old man has returned from 
India recently and presents with diarrhoea. 

Which of the following is the commonest 
cause of traveller's diarrhoea? 

1- Giardia Lamblia 

2- Yersinia enterocolitica 

3- E. Coli 

4- Shigella Flexneri 

5- Entamoeba Histolytica 



Answer & Comments 


Answer: 3- E. Coli 


[ Q: 67 ] MRCPass - Gastroenterology 

A 35 year old man has who has a 
history of heavy alcohol intake presents with 
hematemesis and melena. On admission, he 
was in a preshock state with severe anemia 
and was resuscitated with blood and fresh 
frozen plasma. Endoscopic examination 
identified bleeding from the varices in the 
second portion of the duodenum. 

Which one of the following drugs can be used 
for treatment? 

1- Octreotide 

2- Vitamin C 

3- Simvastatin 

4- Carbimazole 



E. Coli is the commonest cause of travellers 
diarrhoea. It is usually a self limiting condition. 

[ Q: 69 ] MRCPass - Gastroenterology 

A 60 year old man presents with a 4 
month history of dysphagia and weight loss. A 
diagnosis of achalasia is made. 

Which of the following is the best way to 
provide symptomatic relief? 

1- Surgical cardiomyotomy 

2- Propanolol 

3- Isosorbide mononitrate 

4- Gaviscon 

5- Magnesium trisilicate 



5- Bromocriptine 

Answer & Comments 

Answer: 1- Octreotide 

Somatostatin and its derivative, octreotide, 
are often used for emergency treatment of 
bleeding oesophageal varices in patients with 
cirrhosis of the liver. It is given intravenously. 


Answer & Comments 

Answer: 1- Surgical cardiomyotomy 

Cardiomyotomy entails surgical division of the 
lower oesophageal sphincter. Mortality rates 
of less than 1% and success rates of over 85% 
have been recorded. There is However a high 
incidence (up to 10%) of oesophageal reflux 
post-operatively progressing in a number of 
cases to peptic stricture. For achalasia, 
nitrates and hydralazine can help in short 
term. 


Dr. Khalid Yusuf El-Zohry - Sohag Teaching Hospital (01118391123) 

Ref MRCPass OE OE 2012 PasTest 2009 PassMedicine 2009 PasTest Exam 




ReviseMRCP 

61 
































El-zohry MRCP Questions Bank (Port 1) - 2013 


(For my personal use) 


[ Q: 70 ] MRCPass - Gastroenterology 

A 35 year old man presents with 
jaundice. He has been on a drug which was 
prescribed by his GP for several weeks. His 
blood tests show a bilirubin of 34pmol/l, ALP 
of 450 U/land AST 50 U/l. 

Which one of the following drugs is likely to 
co use cholestatic jaundice? 

1- Chlorpromazine 

2- Paracetamol 

3- Ibuprofen 

4- Allopurinol 

5- Colchicine 



Answer & Comments 

Answer: 1- Chlorpromazine 

Chlorpromazine, tricyclic antidepressants, 
azathioprine, augmentin and erythromycin 
cause cholestatic jaundice and also associated 
hepatitis. 


the majority of binge-drinking alcoholics and is 
a common cause of anemia. Inadequate 
dietary intake, intestinal malabsorption, and 
impaired folate storage in the liver all 
contribute to folate deficiency. 

^ [ Q: 72 ] MRCPass - Gastroenterology 

fi - 

# A 20 year old patient has recently 
returned from travelling in South Africa, and is 
unwell with diarrhoea. She had a 2-week 
history of profuse loose brow n stools, 
lethargy, weakness, nausea and abdominal 
discomfort. The diarrhoea had initially settled 
with loperamide, but then recurred again. 

A faecal specimen was collected for 
microscopy and culture, and it subsequently 
grew Giardia lamblia. 

Whot medication should be used for 
treatment? 

1- Amoxycillin 

2- Albendazole 

3- Metronidazole 


[ Q: 71 ] MRCPass - Gastroenterology 

A 40 year old woman has drunk 50 
units a week of alcohol for 20 years. 

During investigations, she has : 

Haemoglobin 10.2 g/dL, MCV 110 fL (80-96), 
white cell count 2.1 x 10 9 /L (4-11), platelet 
count 75 x 10 9 /L (150-400). 

What might explain these results? 

1- Liver cirrhosis 

2- Folic acid deficiency 

3- Hepatitis C infection 

4- Iron deficiency 

5- Multiple myeloma 



Answer & Comments 

Answer: 2- Folic acid deficiency 

The haematological picture is that of B12 or 
folate deficiency. Folate deficiency occurs in 


4- Erythromycin 

5- Gentamicin 


Answer & Comments 

Answer: 3- Metronidazole 

Giardia lamblia infection can present with 
abdominal pains and diarrhoea or 
steatorrhoea. Metronidazole or tinidazole are 
first line treatments. It does not cause 
dysentery (cholera or amoebiasis do). 
Duodenal aspirate biopsy can confirm the 
diagnosis. Villous atrophy is associated. 


[ Q: 73 ] MRCPass - Gastroenterology 

A 60 year old woman presents with a 
12 month history of chest pain, dysphagia 
when consuming both solids and liquids. She 
smokes 15 cigarettes per day and drinks 12 
units of alcohol per week. Clinical examination 
was normal. 



Dr. Khalid Yusuf El-Zohry - Sohag Teaching Hospital (01118391123) 

Ref MRCPass OE OE 2012 PasTest 2009 PassMedicine 2009 PasTest Exam ReviseMRCP 



62 

































El-zohry MRCP Questions Bank (Port 1) - 2013 


(For my personal use) 


What is the likely diagnosis? 

1- Oesophagitis 

2- Oesophageal web 

3- Pharyngeal pouch 

4- Achalasia 

5- Oesophageal carcinoma 

Answer & Comments 

Answer: 4- Achalasia 

A longstanding history of dysphagia with both 
solids & liquids suggests a functional rather 
than mechanical cause. Achalasia, in which 
there failure of oesophageal peristalsis and 
relaxation of lower oesophageal sphincter fits 
the clinical picture best. 


^ [ Q: 75 ] MRCPass - Gastroenterology 

/ 

# A 50 year old man has 5 year history 
of bloody diarrhoea mixed with mucus. He 
had lost 2 stones in weight over this time. 
Flexible sigmoidoscopy showed loss of 
vascular appearance, erythema, and 
superficial ulceration consistent with 
ulcerative colitis. 

Which one of the following features is 
associated with the condition? 

1- Rheumatoid arthritis 

2- Acromegaly 

3- Osteomalacia 

4- Megaloblastic anaemia 

5- Episcleritis 


[ Q: 74 ] MRCPass - Gastroenterology 

A 55 year old man has a long history 
of alcohol use. On the average he drinks two 
bottles of wine a day. He presents with 
tremors, hallucinations and has signs of 
chronic liver disease as well as ascites. The 
senior house officer assesses the status of 
chronic liver disease with the Child Pugh 
Classification. 



Answer & Comments 

Answer: 5- Episcleritis 

Megaloblastic anaemia occurs in Crohn's as 
B12 is absorbed in the ileum. 

Ulcerative colitis is also associated with: 

aphthous ulcers of the mouth 

Iritis/Uveitis and episcleritis 


Which of the following is one of the criteria 
used to assess severity? 

1- Haemoglobin 

2- AST 

3- Sodium 

4- Creatinine 

5- Prothrombin time 

Answer & Comments 

Answer: 5- Prothrombin time 

Child Pugh classification includes bilirubin 
level, prothrombin tine, encephalopathy 
scores, ascites and albumin. AST is not used to 
assess severity of liver disease. 


seronegative arthritis, ankylosing spondylitis, 
sacroilitis 

erythema nodosum, pyoderma gangrenosum 

Primary sclerosing cholangitis, 

cholangiocarcinoma 



Episcleritis 


Dr. Khalid Yusuf El-Zohry - Sohag Teaching Hospital (01118391123) 

Ref MRCPass OE OE 2012 PasTest 2009 PassMedicine 2009 PasTest Exam ReviseMRCP 



























El-zohry MRCP Questions Bank (Port 1) - 2013 


(For my personal use) 


7 

[ Q: 76 ] MRCPass - Gastroenterology 

Answer & Comments 


A 60 year old man is admitted as an 

Answer: 4- Achlorhvdria 



emergency with severe abdominal pain. He 
smokes 30 cigarettes a day and takes 
approximately 30 units of alcohol per week 
but admits to exceeding this amount 
occasionally. He also complains of sudden 
onset poor vision. Ophthalmoscopy shows 
multiple micro infarcts (cotton wool spots). 

What investigation should be considered next? 

1- Carotid dopplers 

2- CT scan abdomen 

3- E.R.C.P. 

4- Upper Gl endoscopy 

5- Colonoscopy 

Answer & Comments 

Answer: 2- CT scan abdomen 

Ischaemic retinopathy, which causes retinal 
oedema and micro infarcts, is a complication 
of acute pancreatitis. CT scan of the abdomen 
will be useful in confirming diagnosis and 
assessing for the presence of a pancreatic 
abscess/pseudocyst. 


VIPomas in adults are usually neuroendocrine 
islet cell tumors of the pancreas that produce 
high amounts of VIP. Other secreted 
hormones may include secreted gastrin and 
pancreatic polypeptide. 

Achlorhydria (lack of acidity from the 
stomach) is classically associated with VIPoma. 
It can be diagnosed by measuring acid output 
from nasogastric contents (NG tube). Features 
of VIP syndrome include watery diarrhea, 
hypochlorhydria, hyperglycemia, 

hypercalcemia and flushing. Migratory 
erythema associated with glucagonoma. 

[ Q: 78 ] MRCPass - Gastroenterology 

A 45 year old woman presents with 
jaundice and several weeks before had 
pruritus all over her body. Her only past 
medical history is Hashimoto's thyroiditis. On 
examination, she has jaundice, xanthelasmata 
and hepatomegaly. 

Which of the following is the most likely cause 
of the jaundice? 



[ Q: 77 ] MRCPass - Gastroenterology 

A 43 year male presents with weight 
loss and watery diarrhoea for several weeks. 
Investigations reveal hypokalaemia. 

Which of the following would support a 
diagnosis of a VIPoma? 

1- Hypoglycaemia 

2- Raised Insulin levels 



1- Cholecystitis 

2- Primary biliary cirrhosis 

3- Systemic lupus erythematosus 

4- Hepatitis C 

5- Cholangiocarcinoma 


Answer & Comments 


Answer: 2- Primary biliary cirrhosis 


3- Pellagra 

4- Achlorhydria 

5- Erythema nodosum 


Primary biliary cirrhosis commonly affects 
women at the age of 45-60. Pruritus often 
precedes jaundice. 

Hashimoto's thyroiditis as well as other 
autoimmune conditions are associated. 
Xanthelasmata are often present in patients 
with primary biliary cirrhosis. 


Dr. Khalid Yusuf El-Zohry - Sohag Teaching Hospital (01118391123) 

Ref MRCPass OE OE 2012 PasTest 2009 PassMedicine 2009 PasTest Exam ReviseMRCP 



64 






























El-zohry MRCP Questions Bank (Port 1) - 2013 


(For my personal use) 



[ Q: 79 ] MRCPass - Gastroenterology 

A 35 year old woman has abdominal 
pain, jaundice and worsening ascites. She 
drinks 20 units of alcohol each week, and 
takes the oral contraceptive pill. 


Which of the following findings would make a 
diagnosis of Budd Chiari syndrome likely? 


1- Encephalopathy 


2- Tender hepatomegaly 

3- Ascites fluid protein of 42 g/L 

4- Alanine aminotransferase of 150 U/L 


5- Ankle oedema 


Answer & Comments 

Answer: 2- Tender hepatomegaly 

Budd-Chiari syndrome is an uncommon 
condition induced by thrombotic or 
nonthrombotic obstruction to hepatic venous 
outflow . The classic triad of abdominal pain, 
ascites, and hepatomegaly is observed in the 
vast majority of patients but is nonspecific. 
Tender hepatomegaly on examination is one 
of the hallmark signs. 


Proton pump inhibitors such as omeprazole, 
lansoprazole and pantoprazole are more 
effective than H2 receptor blockers such as 
ranitidine, cimetidine or nizatidine. Healing in 
oesophagitis is beter with a PPI although all of 
the options can reduce symptoms due to acid 
reflux. 


[ Q: 81 ] MRCPass - Gastroenterology 

/ - 

# A 65 year old man presents with 
dysphagia of solids for the past three months. 
There is a history of 10 kg weight loss in 4 
months following loss of appetite. He has had 
symptoms of indigestion and heartburn for 10 
years. He regularly takes Gaviscon and milk of 
magnesia. He is a heavy smoker. He 
undergoes endoscopy, which reveals a small 
tumour at the lower part of the oesophagus. 

What is the likely aetiological cause? 

1- Barrett's oesophagus 

2- Helicobacter pylori 

3- Oesophageal pouch 

4- Family history 

5- Colonic carcinoma metastases 


[ Q: 80 ] MRCPass - Gastroenterology 

A 50 year old lady complains of 
epigastric burning pains due to stress. She 
finally undergoes an OGD which shows gastro- 
oesophageal reflux. 

Which medication is the most effective for her 
condition? 

1- Nizatidine 

2- Bismuth 

3- Gaviscon 


Answer & Comments 

Answer: 1- Barrett's oesophagus 

The history is suggestive of gastro 
oesophageal reflux past 10 years. There may 
be dysplasia of the oesophageal mucosa 
known as Barrett's oesophagus. In Barrett's 
oesophagus the stratified squamous 
epithelium that normally lines the distal 
oesophagus is replaced by an abnormal 
columnar epithelium that has intestinal 
features. 


4- Magnesium trisilicate 

5- Pantoprazole 


Answer & Comments 


Answer: 5- Pantoprazole 


Modern data indicate that patients with 
Barrett's oesophagus develop oesophageal 
adenocarcinomas at the rate of 0.5% per year, 
a rate that is more than 30-fold higher than 
that of the general population. 


Dr. Khalid Yusuf El-Zohry - Sohag Teaching Hospital (01118391123) 

Ref MRCPass OE OE 2012 PasTest 2009 PassMedicine 2009 PasTest Exam ReviseMRCP 



65 




























El-zohry MRCP Questions Bank (Port 1) - 2013 


(For my personal use) 


[ Q: 82 ] MRCPass - Gastroenterology 

A 60 year old man has been referred 
by his GP for investigatio of hepatomegaly. His 
past medical history includes type II diabetes 
and longstanding arthralgia. 

Investigations show : 

Albumin 28 g/L 

Total bilirubin 45 umol/L 

Alkaline Phosphatase 170 U/L 

ALT 180 U/L 

gamma glutamyl transferase 160 U/L 
Ferritin 1200 microg/L (15-400) 

What is the diagnosis? 

1- Primary biliary cirrhosis 

2- Non alcoholic steatohepatitis 

3- Diabetes mellitus 

4- Wilson's disease 

5- Haemochromatosis 



Answer & Comments 

Answer: 5- Haemochromatosis 

The diagnosis of haemochromatosis is based 
on clinical features of the disease; these 
features include diffuse hyperpigmentation, 
non migratory polyarthritis, chondrocalcinosis, 
hepatomegaly, and diabetes mellitus. There is 
also deranged liver function tests and 
significantly high ferritin in this case, pointing 
tow ards the diagnosis. 


1- Propanolol 

2- Nasogastric tube 

3- Intravenous pantoprazole 

4- Terlipressin 

5- Lactulose 


Answer & Comments 

Answer: 4- Terlipressin 

The patient has clinical evidence of cirrhosis of 
the liver and likely oesophageal varices. 

Octreotide and terlipressin are widely used in 
acute variceal hemorrhage to reduce the 
bleeding rate. They act by causing mesenteric 
arterial vasoconstriction, thus reducing portal 
venous flow and portal pressure. 



[ Q: 84 ] MRCPass - Gastroenterology 

A 60 year man presents with 
malaise, weight loss, diarrhoea and pain in the 
joints. He is pigmented, has clubbing and 
lymphadenopathy. He has ascites and 
ophthalmoplegia. Investigations show that he 
is anaemic. ECG shows a right bundle branch 
block and paracentesis abdominis reveals 
chylous ascites. 

What is the likely diagnosis? 


1- Wilson's disease 


2- Tropical sprue 

3- Whipple's disease 

4- Coeliac disease 


[ Q: 83 ] MRCPass - Gastroenterology 


A 45 year old man presents with 
haematemesis and melaena. He gives a 
history of drinking 20 units of alcohol a day for 
more than 15 years. On examination he has 
spider naevi, Dupuytren's contracture, 
jaundice and ascites. 


Whilst awaiting endoscopy the initial 
management of this patient should be: 


5- Crohn's disease 


Answer & Comments 

Answer: 3- Whipple's disease 

The diagnosis is Whipple's disease. Histology 
of small bowel biopsy will show multiple 
macrophages in the lamina propria and the 
presence of rod shaped bacteria (Trophyrema 
Whippelli) both within and outside the 
abnormal macrophages. 


Dr. Khalid Yusuf El-Zohry - Sohag Teaching Hospital (01118391123) 

Ref MRCPass OE OE 2012 PasTest 2009 PassMedicine 2009 PasTest Exam ReviseMRCP 



66 




























El-zohry MRCP Questions Bank (Port 1) - 2013 


(For my personal use) 



PAS stained macrophages in duodenal biopsy 

(purple) 


[ Q: 85 ] MRCPass - Gastroenterology 

A 30 year old man presents with 
longstanding epigastric pains. An abdominal 
CT was organised and this showed dilated 
pancreatic ducts. 

The secretin test is positive. 

The patient has a HB of 13 g/dl, MCV of 105 fl 
and platelet count of 350 x 10 9 /l . 

The high MCV is most likely due to: 

1- Folic acid deficiency 

2- B12 deficiency 

3- Myelodysplastic syndrome 

4- Reticulocytosis 

5- Paroxysmal nocturnal haemoglobinuria 



Answer & Comments 

Answer: 2- B12 deficiency 

In chronic pancreatitis, trypsin secretion is 
reduced. Trypsin is required in the processing 
of dietary B12 which enables absorption and 
hence B12 deficiency is the most likely in this 
case. 



[ Q: 86 ] MRCPass - Gastroenterology 
A 40 year old heavy alcoholic 


presents unwell with difficulty walking and 
confusion. 

On examination, he has signs of chronic liver 
disease, hepatomegaly and ascitis. 

Blood tests confirm deranged liver function 
including AST and GGT. 

Which one of the following is the most 
important monifestotion of acute hepatic 
failure ? 

1- Prolonged prothrombin time 

2- Hepatic encephalopathy 

3- Elevated ALT and AST 

4- Jaundice 

5- Ascites 


Answer & Comments 

Answer: 2- Hepatic encephalopathy 

Hepatic encephalopathy is the cardinal 
manifestation of acute hepatic failure. 

It is characterized by various neurologic 
symptoms including changes in reflexes, 
changes in consciousness, and behavior 
changes that can range from mild to severe. 

The exact cause is unknown, but one 
substance believed to be particularly toxic to 
the central nervous system is ammonia which 
is normally detoxified by the liver. 


[ Q: 87 ] MRCPass - Gastroenterology 

fl 

m A 20 year girl is commenced on 
nasogastric feeding due severe anorexia 
nervosa. She becomes acutely confused after 
a week. 


Which one of the following investigations 
should be reguested? 

1- Troponins 

2- Serum Calcium 


3- Serum Bicarbonate 


4- Serum Magnesium 


Dr. Khalid Yusuf El-Zohry - Sohag Teaching Hospital (01118391123) 

Ref MRCPass OE OE 2012 PasTest 2009 PassMedicine 2009 PasTest Exam ReviseMRCP 



67 



























El-zohry MRCP Questions Bank (Port 1) - 2013 


(For my personal use) 


5- Serum Potassium 


* serum AST >250 IU/L 


Answer & Comments 

Answer: 4- Serum Magnesium 

Refeeding syndrome of hypophosphataemia, 
hypomagnesaemia, hypocalcaemia and fluid 
retention is seen in severely malnourished 
patients when they are started on enteral or 
parenteral nuitrition. In particular, 
hypomagnesaemia can cause cardiac rhythm 
problems and confusion. 


* serum LDH >350 IU/L 
At 48 hours 

* Calcium (serum calcium < 2.0 mmol/L (< 8.0 
mg/dL) 

* Hematocrit fall > 10% 

* Oxygen (hypoxemia P02 < 60 mmHg) 

* Urea increased by 1.8 or more mmol/L (5 or 
more mg/dL) after IV fluid hydration 


[ Q: 88 ] MRCPass - Gastroenterology 

A 45 year old man presents with 
epigastric pain and vomiting. He had pain 
located at the upper abdomen for the last 8 
hours prior to admission. The patient also 
complained of nausea and two episodes of 
vomiting. He has an amylase of 500 (60-180) 
U/l. Ultrasound of the abdomen showed a 
pancreas with enhanced echogenicity. 

Which of the following is o poor prognostic 
indicator? 

1- White cell count of 10 x 10 9 /L 

2- Urea of 7 mmol/l 

3- Glucose of 8 mmol/l 

4- ALT of 350 U/l 

5- Calcium of 2.8 mmol/l 



Answer & Comments 

Answer: 4- ALT of 350 U/l 

Poor prognostic indicators in acute 
pancreatitis can be assessed with the Ranson 
Criteria or APACHE scores. 

Ranson's criteria: 

At admission 

* age in years > 55 years 

* white blood cell count > 16000 cells/mm 3 

* blood glucose > 10 mmol/L (> 200 mg/dL) 


* Base deficit (negative base excess) > 4 
mEq/L 

* Sequestration of fluids > 6 L 


[ Q: 89 ] MRCPass - Gastroenterology 

A 55 year old chronic alcoholic is 
found on the street with a low conscious level. 
His GCS is 3/15 on arrival in casualty. On 
examination, his BP is 110/65 and he is 
jaundiced. There is marked ascites and he has 
7 spider naevi. 

ECG shows atrial fibrillation with a ventricular 
rate of 120. 

Which is the most practical test to elucidate 
why he is unconscious? 

1- EEG 

2- Ultrasound of abdomen 

3- BM stick 

4- Phenytoin and carbamazepine levels 

5- Paracetamol and salicylate levels 



Answer & Comments 

Answer: 3- BM stick 

In a patient with liver disease, it is important 
to exclude hypoglycaemia as a cause of 
unconsciousness. Following confirming that 
baseline observations and BM is normal, the 
next best step is to perform a CT of his head to 


Dr. Khalid Yusuf El-Zohry - Sohag Teaching Hospital (01118391123) 

Ref MRCPass OE OE 2012 PasTest 2009 PassMedicine 2009 PasTest Exam ReviseMRCP 



68 





























El-zohry MRCP Questions Bank (Port 1) - 2013 


(For my personal use) 


exclude a central neurological cause of 
decreased conscious level. 


[ Q: 90 ] MRCPass - Gastroenterology 

A 28 year old woman has symptoms 
of lethary, bloody diarrhoea and a previous 
history of DVT in the leg. On examination, the 
left eye was inflamed and multiple mouth 
ulcers were noted. Colonoscopy was 
organised and this confirmed colitis. She also 
has vulval ulcers. 

What is the likely diagnosis? 

1- Homocystinuria 

2- Erythema nodosum 

3- Ulcerative colitis 

4- Behcet's disease 

5- Crohn's disease 



Answer & Comments 

Answer: 4- Behcet's disease 

Recurrent oral ulceration is the commonest 
manifestation and the presenting feature in 
Behcet's disease. The commonest second 
systems involved are the genital mucosae and 
eyes. Vasculitis, pustules and subcutaneous 
nodules also occur. Vascular thrombosis 
occurs in about 10% of patients. 


[ Q: 91 ] MRCPass - Gastroenterology 

A 65 year man presents with a 6 day 
history of lower abdominal pains and 
diarrhoea. He has had several courses of 
antibiotics for recurrent chest infections over 
the past month. On examination he has a 
temperature of 38.2°C, a blood pressure of 
100/70 mmHg and a distended, tender 
abdomen. 



Which of the following is the most appropriate 
investigation? 

1- Plain abdominal and erect chest X ray 


2- Flexible sigmoidoscopy 

3- Stool PCR 

4- Sputum culture 

5- Ultrasound scan of abdomen 


Answer & Comments 

Answer: 1- Plain abdominal and erect chest X 
ray 

The diagnosis is pseudomembranous colitis 
due Clostridium Difficile infection. The patient 
was predisposed due to antibiotic usage. Plain 
AXR is useful in diagnosing toxic dilatation and 
erect CXR would exclude gas under the 
diaphragm. Stool testing for Clostridial toxin 
(CDT) would also be useful. 



[ Q: 92 ] MRCPass - Gastroenterology 

A 27 year old lady is referred for 
investigation of abnormal liver function tests. 
She has been taking several drugs to treat 
tuberculosis for a long period of time. 


Her albumin is 30 g/l, bilirubin 15 pmol/l, ALP 
is 250 U/l and ALT is 300 U/l. 


She has a liver biopsy which shows histological 
evidence of liver necrosis eroding the portal 
zone and extending into the liver lobule. 

What is the likely diagnosis? 

1- Hepatitis B infection 

2- Hepatitis C infection 

3- Primary biliary cirrhosis 

4- Chronic active hepatitis 

5- Autoimmune hepatitis 


Answer & Comments 

Answer: 4- Chronic active hepatitis 

Inflammatory conditions (e.g Wilson's disease) 
and drugs such as methyldopa and isoniazid 
can precipitate chronic active hepatitis. 

Autoimmune hepatitis is associated with anti¬ 
nuclear antibodies (70%), anti smooth muscle 


Dr. Khalid Yusuf El-Zohry - Sohag Teaching Hospital (01118391123) 

Ref MRCPass OE OE 2012 PasTest 2009 PassMedicine 2009 PasTest Exam ReviseMRCP 



69 





























El-zohry MRCP Questions Bank (Port 1) - 2013 


(For my personal use) 


(30%) and liver specific antibodies (ASGP-R 
asialoglycoprotein receptor; LKM liver kidney 
microsomal antibodies). 



[ Q: 93 ] MRCPass - Gastroenterology 

A 60 year old woman has recently 
been treated with amoxycillin for a tooth 
abscess. Since then, she has developed 
diarrhoea for up to 15 times a day. There is 
blood in the stools. 


Her Hb is 13 g/dl, WCC 13 x 10 9 /l, platelets 
500 x 10 9 /L. Her CRP is 80 mg/I. 

What is the likely infective organism? 

1- Clostridium difficile 


2- Cyclospora 

3- Cryptospridium 

4- Clostridium tetani 


5- Campylobacter 


Answer & Comments 

Answer: 1- Clostridium difficile 

In a patient who has recently had antibiotics, 
Clostridium difficile is the most likely 
infectdive organism. 


5- Somatostatin 


Answer & Comments 

Answer: 5- Somatostatin 

This patient has high grade liver cirrhosis 
according to Childs Pugh classification. 

Somatostatin acts to reduce portal pressures 
and is as effective as endoscopic procedures 
at controlling variceal bleeding in the acute 
setting. Betablockers can be used as oral 
prophylaxis oesophageal varices. IV 
omeprazole can also be given. 


^ [ Q: 95 ] MRCPass - Gastroenterology 

<i - 

# A 70 year old man presents with a six 
month history of diarrhoea and pale stools. He 
has lost 1 stone in weight over the past 6 
months. 

Investigations show : 

Calcium 1.7 mmol/I 
Phosphate 0.75 (0.8-8) pmol/l 
Alkaline phosphatase 360 U/L (45-105) 

What is the likely diagnosis? 

1- Amoebiasis 


Discontinuation of the offending antibiotic is 
important (not always easy if an infection such 
as a dental abscess persists). Metronidazole 
and vancomycin given orally are equally 
effective. 


2- Pancreatic carcinoma 

3- Whipple's disease 

4- Crohn's disease 

5- Small Intestinal bacterial overgrowth 


[ Q: 94 ] MRCPass - Gastroenterology 

A 48 year old man with Child's grade 
C cirrhosis presents with haematemesis. 



Which one of the following drugs , 
administered immediately intravenously , 
would be appropriate? 


1- Propranolol 


2- Sodium Nitroprusside 


3- Isosorbide dinitrate 


4- Pabrinex 


Answer & Comments 

Answer: 2- Pancreatic carcinoma 

Pancreatic carcinoma is the most probable 
diagnosis in view of his presenting symptoms 
and age. The patient has osteomalacia 
associated with malabsorption. Increased 
alkaline phosphatase would not be expected 
in Crohn's disease, Whipple's or bacterial 
overgrowth. 


Dr. Khalid Yusuf El-Zohry - Sohag Teaching Hospital (01118391123) 

Ref MRCPass OE OE 2012 PasTest 2009 PassMedicine 2009 PasTest Exam ReviseMRCP 






























El-zohry MRCP Questions Bank (Part 1) - 2013 


(For my personal use) 


^ [ Q: 96 ] MRCPass - Gastroenterology 

fj - 

# An 18 year old arts student presents 
with haematemesis. He has been out drinking 
with his friends and drank about 15 pints of 
beer. He vomited several times. The last time 
he vomited he brought up several cupfuls of 
blood. 

What is the most likely cause of his 
haematemesis? 

1- Mallory-Weiss tear 

2- Hyponatraemia 

3- Oesophageal varices 

4- Gastritis 

5- Peptic ulcer 


1- CT scan of abdomen 

2- ERCP 

3- Antimitochondrial antibody 

4- HCV DNA levels 

5- Liver biopsy 

Answer & Comments 

Answer: 5- Liver biopsy 

Liver biopsy is useful to determine the severity 
of the disease and for monitory progression 
when treatment with ribavirin and interferon 
is instigated. Cirrhosis develops in 20-25 
percent of patients with chronic hepatitis C 
after 10 years. 


Answer & Comments 

■ 7 

[ Q: 98 ] MRCPass - Gastroenterology 

Answer: 1- Mallorv-Weiss tear 

L4J 

A 30 year old woman presents in the 


A Mallory-Weiss tear occurs at the gastro- 
oesophageal junction when there is a sudden 
increase in intra-abdominal pressure, 
especially with recurrent vomiting. 


l 


second trimester of her first pregnancy with 
jaundice. There are no other symptoms, and 
her urine is not discoloured. 

Blood tests show ALT 32 (5-35) U/l, AST 28 (1- 
31) U/l, ALP 75 (20-120) U/l, GGT 31 (4-35) 
U/l, Bilirubin 65 (1-22) pmol/l, Albumin 37 (37- 
49) g/l. 

The likely diagnosis is: 

1- Dubin Johnson syndrome 

2- Cholestasis of pregnancy 

3- Gilbert's syndrome 

4- Alcoholic liver cirrhosis 

5- Primary biliary cirrhosis 


Answer & Comments 



[ Q: 97 ] MRCPass - Gastroenterology 

A 40 year old man has worsening 
jaundice and has deranged liver function tests 
which have worsened over the last 3 months. 
He has positive IgG to hepatitis C. 

Which of the following investigations is most 
useful for management of the condition? 


Answer: 3- Gilbert's syndrome 

This is unlikely to be intrahepatic cholestasis 
of pregnancy because there is no elevation of 
alkaline phosphatase and ALT/AST. The 
condition is usually associated with intense 
pruritus as well. 

The isolated raised bilirubin makes a diagnosis 
of Gilberts syndrome likely. 


Dr. Khalid Yusuf El-Zohry - Sohag Teaching Hospital (01118391123) 

Ref MRCPass OE OE 2012 PasTest 2009 PassMedicine 2009 PasTest Exam ReviseMRCP 



71 
































El-zohry MRCP Questions Bank (Port 1) - 2013 


(For my personal use) 


Gilbert's syndrome is an autosomal recessive 
condition characterized by intermittent 
jaundice in the absence of hemolysis or 
underlying liver disease. Unconjugated 
hyperbilirubinemia in Gilbert syndrome has 
long been recognized as due to underactivity 
of the conjugating enzyme system bilirubin- 
uridine diphosphate glucuronyl transferase 
(bilirubin-UGT). 

Jaundice in Gilbert syndrome may be 
precipitated by dehydration, fasting, 
menstrual periods, pregnancy, stress, such as 
an intercurrent illness or vigorous exercise. 

Dubin-Johnson syndrome is asymptomatic 
mild jaundice due to impaired excretion of 
bilirubin. In contrast to Gilbert's syndrome, 
the hyperbilirubinemia is conjugated and bile 
appears in the urine. 

[ Q: 99 ] MRCPass - Gastroenterology 

A 20 year old man is well but 
jaundiced. His urine colour is normal. His liver 
function tests show bilirubin 45 pmol/l, ALT 26 
U/l, AST 25 U/l, GGT 40 U/l and ALP 80 U/l. 

Ultrasound scan of abdomen shows normal 
parenchymal texture of the liver. 

Which diagnosis is most likely? 

1- Dubin Johnson syndrome 

2- Crigler Najjar syndrome 

3- Gilbert's syndrome 

4- Infectious mononucleosis 



Crigler Najjar is a more severe form in which 
the same enzyme is absent and there is 
neonatal jaundice due to very high levels of 
unconjugated hyperbilirubinaemia. In 
contrast, Dubin Johnson syndrome leads to 
high levels of conjugated bilirubin which will 
lead to dark urine. 



[Q: 100] MRCPass - 
Gastroenterology 


A 40 year old woman with epigastric pain 
undergoes upper Gl endoscopy. A biopsy was 
taken and it revealed a diagnosis of mucosal 
associated lymphoid tissue. 


Whot should be done initially? 


1- Bilroth's operation 


2- Proton pump inhibitor 

3- H pylori eradication 


4- Radiotherapy 

5- Chemotherapy 


Answer & Comments 

Answer: 3- H pylori eradication 

MALT lymphoma (or MALToma) is a relatively 
rare form of non-Hodgkin's lymphoma. 

70% of cases of MALT lymphoma affecting the 
stomach are H pylori positive. Eradication of H 
pylori is recommended. Chemotherapy 
(chlorambucil), surgery and radiotherapy can 
then be considered. 


5- Haemochromatosis 


Answer & Comments 

Answer: 3- Gilbert's syndrome 

Unconjugated hyperbilirubinemia in Gilbert 
syndrome is due to underactivity of the 
conjugating enzyme system bilirubin-uridine 
diphosphate glucuronyl transferase (bilirubin- 
UGT) which converts bilirubin to a conjugated, 
water soluble form. 


[Q: 101] MRCPass- 
Gastroenterology 

A 38 year old lady who is asymptomatic has an 
ultrasound of her abdomen done privately. It 
showed several gallstones and no biliary duct 
dilatation. She has no other past medical 
history. 

What should be done? 

1- Open cholecystectomy 

2- Laparascopic cholecystectomy 



Dr. Khalid Yusuf El-Zohry - Sohag Teaching Hospital (01118391123) 

Ref MRCPass OE OE 2012 PasTest 2009 PassMedicine 2009 PasTest Exam ReviseMRCP 



72 




























El-zohry MRCP Questions Bank (Port 1) - 2013 


(For my personal use) 


3- ERCP 

4- Lithotripsy 

5- Observation 


Cholera and giardial diarrhoea are not 
typically bloody. There is no history of 
immunocompromise to suggest Cryptosporidia 
infection. 


Answer & Comments 

Answer: 5- Observation 

The natural history of asymptomatic 
gallstones (GS) suggests that a large number 
of affected individuals will remain 
asymptomatic throughout life; only 1-4% per 
year will develop symptoms or complications 
of GS disease. Therefore, the natural history of 
asymptomatic GSs is so benign that surgery is 
generally not recommended and watchful 
waiting is the best course of management. 
Laparascopic cholecystectomy is 

recommended for asymptomatic patients with 
risks factors such as sickle cell disease and 
diabetes. 


A 23 year old man has recently travelled back 
from the Middle East. He has previously been 
well and there is no past medical history. He 
has a 5 day history of bloody diarrhoea and 
has acute abodminal pains. 

On examination, he has a temperature of 38°C 
and a tender lower abdomen. 

What is the likely causative organism? 

1- Cryptosporidium 

2- Shigellosis 

3- E coli 

4- Giardiasis 

5- Cholera 


[Q: 102] MRCPass - 
Gastroenterology 



Shigellosis is spread by means of fecal-oral 
transmission. Other modes of transmission 
include ingestion of contaminated food or 
water, contact with a contaminated inanimate 
object, and sexual contact. Sudden onset of 
severe abdominal cramping, high-grade fever, 
emesis, anorexia, and large-volume watery 
diarrhea are presenting features. Seizures, 
delirium and haemolytic uraemic syndrome 
are complications. 

Cotrimoxazole and ampicillin are 
recommended treatments. 




A 45 year woman is admitted a single episode 
of haematemesis after taking a tablet of 
ibuprofen 6 hours previously. 

On examination, she had a blood pressure of 
120/75 mmHg (lying) and 95/60 mmHg 
(standing). Her haemoglobin concentration is 
7.5 g/dL. 

What is the likely cause of the haemetemesis? 

1- Duodenal ulcer 


[Q: 103] MRCPass - 
Gastroenterology 


2- Oesophagitis 

3- Mallory Weiss Tear 

4- Gastric erosions 

5- Gastric ulcer 


Answer & Comments 


Answer: 4- Gastric erosions 


Answer & Comments 

Answer: 2- Shigellosis 

Bloody diarrhoea is more likely to be caused 
by shigella. 


The likely answ er is gastric erosions because 
the bleeding occurred after only one dose of 
Ibuprofen. There is no previous history of 
dyspepsia or upper Gl bleeding. Nonsteroidal 
anti-inflammatory drugs (NSAIDs), such as 
aspirin, ibuprofen, and naproxen, can be 
direct irritants and cause gastritis/erosions. 


Dr. Khalid Yusuf El-Zohry - Sohag Teaching Hospital (01118391123) 

Ref MRCPass OE OE 2012 PasTest 2009 PassMedicine 2009 PasTest Exam ReviseMRCP 




























El-zohry MRCP Questions Bank (Port 1) - 2013 


(For my personal use) 


Because of gravity, the irritants lie on the 
greater curvature of the stomach, and 
therefore, gastritis and ulcers are seen distally 
on or near the greater curvature of the 
stomach. 


[Q: 104] MRCPass - 
Gastroenterology 

A 60 year old man presents with a 2-year 
history of arthritis, fever, recurrent cough and 
chest pain. He has been feeling very lethargic. 
Recently he has developed diarrhoea 
(steatorrhoea), abdominal pain and weight 
loss. 

On examination he is pigmented, there is 
finger clubbing and lymphadenopathy. On 
auscultation, a pan-systolic murmur is heard in 
the cardiac apex. 

Which of the following investigations would 
confirm the clinical diagnosis? 

1- Serum autoimmune screen 

2- Abdominal X ray 

3- Small bowel biopsy 

4- Iron studies 

5- Amylase 





[Q: 105] MRCPass - 
Gastroenterology 


A 60 year old man has haematemesis and 
melaena. Examination reveals jaundice, 
confusion and a flapping tremor and ascites. 
He has a pulse rate of 110 bpm and blood 
pressure of 95/65 mmHg. An urgent 
endoscopy reveals small oesophageal varices, 
without evidence of bleeding but an oozing 
portal hypertensive gastropathy. 


Which of the following measures would be 
appropriate treatment? 


1- Endoscopic adrenaline injection 


2- Intravenous vitamin K 


3- Endoscopic banding 

4- Oral propranolol 

5- Endoscopic ethanolamine injection 


Answer & Comments 

Answer: 3- Small bowel biopsy 

The patient has Whipple's disease, which may 
be confirmed by small bowel biopsy. This will 
show large, foamy PAS positive macrophages 
in the lamina propria. Whipple's disease 
affects mainly men aged 30 to 60. It is caused 
by an infection with Trophyrema whippelii. 

Symptoms of Whipple's disease include 
diarrhoea, inflamed and painful joints, fever, 
and skin darkening. Severe malabsorption 
results in weight loss along with fatigue and 
weakness caused by anaemia. Antibiotics such 
as tetracycline, co-trimoxazole and penicillin 
can be used for treatment (6-12 months). 


Answer & Comments 

Answer: 1- Endoscopic adrenaline injection 

The endoscopy shows small varices with no 
evidence of bleeding but there is diffuse 
oozing of blood. Adrenaline injection (peri and 
intralesional injection of 1:10,000 adrenaline 
solution) will help to control the bleeding. 
Correction of any coagulopathy with vitamin K 
in this case will also be helpful. 

[ft 

I 

A 65 year old man is admitted with a history of 
sudden onset abdominal pain, followed by 
watery diarrhoea and subsequent profuse 
rectal bleeding. He has been a smoker for 40 
years and had a myocardial infarction 5 years 


[Q: 106] MRCPass 
Gastroenterology 


Dr. Khalid Yusuf El-Zohry - Sohag Teaching Hospital (01118391123) 

Ref MRCPass OE OE 2012 PasTest 2009 PassMedicine 2009 PasTest Exam ReviseMRCP 



74 




























El-zohry MRCP Questions Bank (Port 1) - 2013 


(For my personal use) 


ago. He has a family history of colon 
carcinoma. 

Whot is the likely diagnosis? 

1- Sigmoid volvulus 

2- Large bowel infarction 

3- Rectal carcinoma 

4- Ulcerative colitis 

5- Crohn's disease 


Answer & Comments 

Answer: 2- Large bowel infarction 

The history of pain, diarrhoea and bleeding 
per rectum in a patient with vascular risk 
factors suggests large bowel infarction. 
Ischaemic bowel disease may be acute or 
chronic. Most cases result from arterial 
occlusion, usually of the superior mesenteric 
artery. The small bowel is not commonly 
involved. 


A 60 year woman with a long history of 
alcohol abuse has been on Phenytoin for 
epilepsy. On examination, she has a palpable a 
liver edge. 

Her full blood count reveals: 

Haemoglobin 11.0 g/dL 
MCV116 fL 

white cell count 2.3 x 10 9 /L 

platelet count 95 x 10 9 /L 

Whot is the likely explanation for these 
results? 

1- Aplastic anaemia 

2- Vitamin C deficiency 

3- Myeloma 

4- Iron deficiency 

5- Folate deficiency 



[Q: 107] MRCPass - 
Gastroenterology 


Answer & Comments 

Answer: 5- Folate deficiency 

The patient has heavy alcohol intake and 
phenytoin, hence is high risk for folate 
deficiency causing the macrocytic anaemia. 



[Q: 108] MRCPass - 
Gastroenterology 


A GP refers a 35 year old lady for recurrent 
episodes of flushing. Over the past 10 years 
she had these symptoms. She describes each 
episode as starting with a freckled rash on her 
thorax and limbs, pounding in her heart, 
followed later by weakness, vomiting and 
diarrhoea. 


The patient had no other remarkable medical 
history. She specifically denied having 
pathologic fractures or peptic ulcer disease. 
She was not taking prescription or 
nonprescription medication; she had no 
known allergies to medication or food. 

Physical examination revealed a lady with 
hundreds of reddish-brow n macules on his 
arms, legs, and chest. The macules extended 
onto the non-sun-exposed areas of her inner 
forearms and urticated when stroked (positive 
Darier sign). 

Blood tests reveal eosinophilia, serum 
tryptase, 43.3 ng/mL (reference range, <11.5 
ng/mL); serum histamine, 1.2 ng/mL 
(reference range, <1.0 ng/mL). 

Which of the following is the likely diagnosis? 

1- Acromegaly 

2- Systemic mastocytosis 

3- Hypothyroidism 

4- Cushing's disease 

5- Asthma 


Answer & Comments 


Answer: 2- Systemic mastocytosis 


Dr. Khalid Yusuf El-Zohry - Sohag Teaching Hospital (01118391123) 

Ref MRCPass OE OE 2012 PasTest 2009 PassMedicine 2009 PasTest Exam 




ReviseMRCP 

75 

























El-zohry MRCP Questions Bank (Port 1) - 2013 


(For my personal use) 


Systemic mastocytosis is caused by mast cell 
release of histamine in the skin or connective 
tissue. It classically presents with episodes of 
flushing, vomiting, diarrhoea and abdominal 
pain. 

Diagnosis can be confirmed by raised levels of 
urinary N-methyl imidazole, blood 
eosinophilia and thrombocytopenia.Total 
serum tryptase levels of 20 ng/mL or higher in 
a baseline serum sample is also suggestive of 
systemic mast cell disease. 

When a cutaneous lesion is stroked, it typically 
urticates, becoming pruritic, edematous, and 
erythematous. This change is referred to as 
the Darier sign, which is explained by mast cell 
degranulation induced by physical stimulation. 


[Q: 109] MRCPass - 
Gastroenterology 

A 44 year old man presents with frequent 
diarrhoea and upper abdominal pains. He has 
had a partial gastrectomy 3 years ago for 
upper Gl bleeding. He is now on high dose 
omeprazole regularly. A repeat endoscopy 
now shows two oesopnageal ulcers. 

Whot is the appropriate investigation? 




[Q: 110] MRCPass - 
Gastroenterology 


A 48 year old man presents with lethargy and 
pruritus. He has lost one stone in weight over 
the past two years. 


He does not smoke. There is no family history 
of bowel malignancy. 


On examination he looks pale but is not 
jaundiced. There is no hepatomegaly and 
there are no signs of chronic liver disease. 
Liver ultrasound shows small gallstones. The 
gallbladder is not inflamed. There is no biliary 
duct dilatation. Bilirubin is 18, ALT is 180 U/l, 
ALP is 800 U/l. ANA and AMA are negative but 
pANCA is positive. 


What is the diagnosis? 

1- Primary biliary cirrhosis 

2- Cholangiocarcinoma 

3- Primary sclerosing cholangitis 


4- Wilson's disease 


5- Gallstones 


Answer & Comments 


Answer: 3- Primary sclerosing cholangitis 


1- Barium enema 

2- Insulin tolerance test 

3- H. pylori serology 

4- Colonoscopy 

5- Gastrin levels 


Primary sclerosing cholangitis presents at this 
age, there is cholestatic picture on the liver 
function tests, but not necessarily high 
bilirubin. pANCA is positive in about 90% of 
patients with PSC. A small proportion of 
patients (3%) will progress to 
cholangiocarcinoma at a later age. 


Answer & Comments 

Answer: 5- Gastrin levels 

Diarrhea and recurrent gastric ulceration is 
common with Zollinger Ellison syndrome 
(gastrinoma). There would be demonstrable 
high fasting plasma gastrin levels. Gastrinomas 
may occurs as part of a multiple endocrine 
neoplasia syndrome type 1. 


Dr. Khalid Yusuf El-Zohry - Sohag Teaching Hospital (01118391123) 

Ref MRCPass OE OE 2012 PasTest 2009 PassMedicine 2009 PasTest Exam ReviseMRCP 

























El-zohry MRCP Questions Bank (Part 1) - 2013 


(For my personal use) 



[Q: 112] MRCPass - 
Gastroenterology 

A 65 year old woman complains of fevers, 
weight loss, joint pains and diarrhoea. A 
jejunal biopsy reveals flattened mucosa 
containing periodic acid-Schiff (PAS) positive 
macrophages. 

What is the most likely diagnosis? 

1- Coeliac disease 

2- Campylobacter infection 

3- Tropical sprue 



MR cholangiogram showing stenoses and „ . , , ,. 

, r . . , ... 4- Whipple s disease 

irregularities of the hepatic ducts and bile 


ducts in Sclerosing Cholangitis 

5- Giardiasis 

^ [Q: 111] MRCPass- 

Answer & Comments 

S Gastroenterology 

Answer: 4- Whipple's disease 


A 22 year old man presents with a 10 month 
history weight loss, oral ulceration and 
diarrhoea. A flexible sigmoidoscopy is 
performed and a colonic biopsy was taken. 


PAS stained macrophages on the jejunal 
biopsy indicates a diagnosis of Whipple's 
disease. 


Which of the following histological features 
would favour a diagnosis of Crohn's disease 
over ulcerative colitis? 

1- Lymphocyte infiltrate of lamina propria 

2- Metaplastic polyps 

3- Caseating granulomata 

4- Crypt of lieberkuhn abscesses 

5- Goblet cell mucus depletion 


y 

A 40 year man with chronic hepatitis C has 
nonspecfic general deterioration. 

On examination he has a temperature of 37.8 
C, pressure of 100/70 mmHg and is jaundiced. 

His investigations reveal: 

Serum Sodium 129 mmol/l 



[Q: 113] MRCPass - 
Gastroenterology 


Answer & Comments 

Answer: 1- Lymphocyte infiltrate of lamina 
propria 

Crohn's disease is characterised by transmural 
inflammation, neutrophil infiltrates, lymphoid 
aggregates, fissures, preservation of crypt 
architecture and noncaseating granulomata. 


Serum potassium 4.2 mmol/l 

Serum Urea 20 mmol/l 

Serum Creatinine 350 micromol/l 

bilirubin 65 micromol/l 

AST 92 (10-40) U/l 

Aik Phosphatase 250 (50-120) U/l 

Albumin 30 (33-42) g/l 


Ulcerative colitis is typified by mucosal 
inflammation, general inflammatory cell 
infiltration, goblet cell mucus depletion and 
crypt abscesses. 


Urine sodium 15 mmol/l 
What is the likely diagnosis? 
1- Hepatitis B infection 


Dr. Khalid Yusuf El-Zohry - Sohag Teaching Hospital (01118391123) 

Ref MRCPass OE OE 2012 PasTest 2009 PassMedicine 2009 PasTest Exam ReviseMRCP 



























El-zohry MRCP Questions Bank (Port 1) - 2013 


(For my personal use) 


2- Spontaneous bacterial peritonitis 

3- Hepatorenal syndrome 

4- Hepatocellular carcinoma 

5- Discreet alcoholism 


2- CT scan of the abdomen 

3- X ray of the abdomen. 

4- Small bowel follow through 

5- Barium enema 


Answer & Comments 


Answer: 3- Hepatorenal syndrome 


Answer & Comments 


Answer: 2- CT scan of the abdomen 


Hepatorenal syndrome (HRS) is the 
development of renal dysfunction in patients 
with severe liver disease (acute or chronic) in 
the absence of any other identifiable causes of 
renal pathology. 

Tw o types of HRS have been described. Type 
1 is mainly associated with acute liver failure 
or alcoholic cirrhosis, but it can develop in any 
other form of liver failure. It is characterized 
by rapid deterioration of renal function, with 
marked increase in serum creatinine and urea. 
Hyponatremia and hyperkalemia are usual 
findings. Type 2 is a more stable form. 

Peritoneal dialysis, hemodialysis, and 
hemofiltration remain controversial because 
of their limited benefit. However, they are still 
employed by certain centers in patients 
awaiting liver transplantation. 



[Q: 114] MRCPass - 
Gastroenterology 


A 30 year old lady presents with a long history 
of abdominal pain and diarrhoea. She also 
gives a history of recurrent mouth ulcers. 
Recently the pain has become more intense, 
she has begun to lose weight and has 
developed fever with night sw eats and she 
also complains of a rash over her legs. 


On examination she is thin, she has clubbing 
and erythema nodosum. On examination of 
the abdomen a firm tender mass can be 
palpated in the right iliac fossa. 

In this patient , which of the following 
investigations is most appropriate? 


1- Colonoscopy 


The clinical features in this patient points tow 
ards a diagnosis of Crohn's disease. However, 
the features of increased pain, fever sw eats 
and a mass suggest the patient may have 
developed an abscess in the right iliac fossa. 
Hence, imaging this area with CT scan to 
detect and treat the collection is the 
investigation of choice. 


A 60 year old woman has a 15 year history of 
constipation. She undergoes a sigmoidoscopy 
and rectal biopsy. 

This shows epithelium with pigment laden 
macrophages in lamina propria. 

What is the likely cause of these findings? 

1- Coeliac disease 

2- Ulcerative colitis 

3- Crohn's diseae 

4- Mesenteric ischaemia 

5- Laxative abuse 



[Q: 115] MRCPass - 
Gastroenterology 


Answer & Comments 

Answer: 5- Laxative abuse 

Melanosis coli is usually associated with 
chronic laxative use (senna), dark pigment is 
deposited in the lamina propria of the colon. It 
does not require medical or surgical 
intervention and is considered to be a benign 
pigmentation disorder. 


Dr. Khalid Yusuf El-Zohry - Sohag Teaching Hospital (01118391123) 

Ref MRCPass OE OE 2012 PasTest 2009 PassMedicine 2009 PasTest Exam ReviseMRCP 






























El-zohry MRCP Questions Bank (Port 1) - 2013 


(For my personal use) 



Melanosis coli due to laxative abuse (dark 
appearance of colon) 



[Q: 116] MRCPass - 
Gastroenterology 


A 45 year old man has a 10 year history of 
type 2 diabetes mellitus. In the past he has 
had fertility problems and has long standing 
generalised joint pains. Recently, he has been 
noted to have abnormal liver function tests. 


The GP refers him to the clinic querying non¬ 
alcoholic fatty liver disease. On examination 
his skin is tanned, there is loss of body hair, 
gynaecomastia, hepatomegaly and testicular 
atrophy. 

How should this patient be treated? 

1- Oral hypoglycaemics 

2- Interferon 


3- Ribavirin 

4- Penicillamine 


5- Venesection 


Answer & Comments 

Answer: 5- Venesection 

The patient is likely to have 
haemochromatosis in view of the history of 
diabetes, arthropathy, deranged liver function 
tests and bronze skin pigmentation. Ferritin 
and liver biopsy will help to confirm the 
diagnosis. Venesection is the best method of 
depleting body iron stores. Treatment consists 


of bi-weekly venesection removing 
approximately 500 ml per week. 



[Q: 117] MRCPass - 
Gastroenterology 


A 37 year old presents with a 10-day history of 
loose stools and mild abdominal discomfort, 
and recent onset of blood in stool. Ulcerative 
colitis was suspected by the 
gastroenterologist. 


Which one of the following is the investigation 
of choice? 


1- Sigmoidoscopy 


2- Colonoscopy 


3- Barium enema 


4- Radionuclide scintigraphy 

5- Angiography 


Answer & Comments 

Answer: 2- Colonoscopy 

Colonoscopy is the investigation of choice to 
evaluate ulcerative colitis, features such as 
loss of vascular pattern, erythema, oedema, 
granular mucosa, blood, pseudopolyp, 
erosion, ulceration are seen. 



[Q: 118] MRCPass- 
Gastroenterology 


A 45 year old woman has been a heavy 
alcoholic for 20 years, but has cut dow n on 
drinking for the past year. 


She has abdominal pains, malaise and nausea. 
On examination she had moderate amounts of 
ascites and generalised abdominal tenderness. 


Investigations show : 
Haemoglobin 12 g/dL 
WCC 14 x 10 9 /L 


prothrombin time 22 s (<15s) 
serum albumin 25 g/L (37-49) 
serum total bilirubin 50 micromol/L (1-22) 


Dr. Khalid Yusuf El-Zohry - Sohag Teaching Hospital (01118391123) 

Ref MRCPass OE OE 2012 PasTest 2009 PassMedicine 2009 PasTest Exam ReviseMRCP 






























El-zohry MRCP Questions Bank (Port 1) - 2013 


(For my personal use) 


ascitic fluid protein 27 g/L 

ascitic fluid lactate 35 mg/dL 

ascitic fluid amylase normal 

ascitic fluid white cell count 650 cells/mL 

Whot is the likely reason for her deterioration ? 

1- Wilson's disease 

2- Spontaneous bacterial peritonitis 

3- Hepatoma 

4- Hepatic vein thrombosis 

5- Pancreatic abscess 


Answer & Comments 

Answer: 2- Spontaneous bacterial peritonitis 

Abdominal tenderness is found in more than 
50% of patients with Spontaneous Bacterial 
Peritonitis. Findings on the abdominal 
examination can range from mild tenderness 
to overt rebound and guarding. 

Traditionally, three fourths of SBP infections 
are caused by aerobic gram-negative 
organisms (50% of these being Escherichia 
coli), and one fourth of these infections are 
due to aerobic gram-positive organisms ( 
streptococcal species). 

An ascitic fluid neutrophil count of >250 
cells/mL and ascites lactate level of >25 mg/dL 
are the single best predictors of SBP. A 
combination of an aminoglycoside and 
ampicillin or cefotaxime can be used. 


[Q: 119] MRCPass - 
Gastroenterology 

A 33 year lady presents with intense pruritus 
in her third trimester of pregnancy. Clinically 
there are no abnormalities apart from scratch 
marks. 

Investigations reveal: ALT 150 U/L, Alkaline 
phosphatase 320 U/L, bilirubin 70 
micromoles/I. FBC is normal. 

What should be the next management step? 



1- Hepatitis test 

2- Liver biopsy 

3- Intrahepatic shunting 

4- Ursodeoxycholic acid 

5- Azathioprine 


Answer & Comments 

Answer: 4- Ursodeoxycholic acid 

The patient has intrahepatic cholestasis of 
pregnancy. Treatment is symptomatic with 
ursodeoxycholic acid (cholestyramine and high 
dose steroids can also be used). The most 
common symptom of intrahepatic cholestasis 
of pregnancy is itching which typically 
develops in the third trimester of pregnancy. 

The itching begins on the palms and soles, and 
then spreads to the rest of the body. The rash 
of ICP is caused by scratching the intensely 
itchy skin. Jaundice occurs in 10% to 15% of 
cases and typically develops 2 to 4 weeks after 
the itching starts. After delivery, both itching 
and jaundice resolve spontaneously. ICP does 
recur with subsequent pregnancies in 40% to 
50% of women. 


j 

A 35 year old man presents with epigastric 
pains which are burning in nature and worse 
at night. He undergoes endoscopy and the 
investigation confirms a diagnosis of gastro- 
oesophageal reflux disease (GORD). 

Which of the following is most likely to relieve 
his symptoms? 

1- Omeprazole 

2- Lactulose 

3- Brandy 

4- Ibuprofen 

5- Diclofenac 



[Q: 120] MRCPass - 
Gastroenterology 


Dr. Khalid Yusuf El-Zohry - Sohag Teaching Hospital (01118391123) 

Ref MRCPass OE OE 2012 PasTest 2009 PassMedicine 2009 PasTest Exam ReviseMRCP 


























El-zohry MRCP Questions Bank (Port 1) - 2013 


(For my personal use) 


Answer & Comments 

_ ^ 

[ Q: 122 ] MRCPass- 

Answer: 1- Omeprazole 

n 

-JF 

Gastroenterology 


The most effective drugs in Gastro- 
oesophageal reflux are proton pump inhibitors 
such as omeprazole. H2 receptor antagonists 
(ranitidine) and Cisapride have also been 
shown to be effective. Nitrates and caffeine 
can also relieve symptoms by relaxing lower 
oesophageal tone. 


A 55 year old woman presents with a 6 month 
history of bloody diarrhoea. She has not had 
any previous admissions or serious illnesses. 
On examination, her abdomen is distended 
and tender on the left side. 

Rectal examination reveals small amount of 
faeces. 


[Q: 121] MRCPass - 
Gastroenterology 

A 35 year old man is referred for lethargy to 
the gastroenterologist. His GP has recently 
diagnosed diabetes and he is on insulin 
injections. He drinks 12 units of alcohol per 
week. On examination he has a grey 
discolouration to his skin. There are no signs 
of chronic liver disease, but he has a smooth 
palpable liver edge of three finger breadths 
below the costal margin. 



Blood tests show a Hb 11.0 g/dl WCC 15.6 x 
10 9 /l. platelets 450 x 10 9 /l. Sodium is 139 
mmol/I, potassium is 4.6 mmol/l, urea 8.6 
pmol/l. creatinine 130 pmol/l. CRP is 35 mg/I. 

What is the next best further investigation? 

1- Colonoscopy 

2- X ray of abdomen 

3- Sigmoidoscopy 

4- CT of the abdomen 

5- MRI of abdomen 


His blood tests show a Hb 13.5 g/dl, WCC of 
12 x 10 9 /L, platelets 145 x 10 9 /L, sodium 133 
mmol/l, potassium 4.5 mmol/l, urea 9 pmol/l, 
creatinine pmol/l bilirubin 23 pmol/l, ALT 150 
U/l, ALP 110 U/l. 

Which further test is most likely to yield the 
diagnosis? 

1- Serum copper and caeruloplasmin 

2- Serum ferritin 

3- Ultrasound of abdomen 

4- Serum amylase 

5- Secretin test 

Answer & Comments 

Answer: 2- Serum ferritin 

Haemochromatosis is described. There is grey 
pigmentation of the skin and hepatomegaly. 
The condition can be made worse when there 
is additional alcohol intake. 


Answer & Comments 

Answer: 2- X ray of abdomen 

The imperative management is to exclude a 
toxic megacolon. This is a patient with bloody 
diarrhoea who could have ulcerative colitis, 
and a distended tender abdomen indicates 
possible acute abdomen. 




Toxic Megacolon 






Dr. Khalid Yusuf El-Zohry - Sohag Teaching Hospital (01118391123) 

Ref MRCPass OE OE 2012 PasTest 2009 PassMedicine 2009 PasTest Exam ReviseMRCP 





























El-zohry MRCP Questions Bank (Part 1) - 2013 


(For my personal use) 


12 

A 40 year old man is being investigated for a 3 
year history of arthralgia. On examination, his 
skin colour was normal and there was no 
evidence of jaundice or hepatomegaly. He has 
the following results: 

Alanine aminotransferase 32 U/L (5-35) 

Aspartate aminotransferase 28 U/L (1-31) 

Fasting plasma glucose 7.4 (3.0-6.0) 

Ferritin 550 ug/L (15-300) 

What is the next best investigation? 

1- Bone marrow biopsy 

2- Transferrin saturation 

3- Copper and caeruloplasmin 

4- Serum transferrin receptors 

5- Liver biopsy 

Answer & Comments 

Answer: 2- Transferrin saturation 

Serum ferritin has many false positives and 
the patient should have the transferrin 
saturation done. The diagnosis of iron 
overload and potentially hereditary 
hemochromatosis should be suspected in men 
whose transferrin saturation is greater than 
55% and in women whose transferrin 
saturation is greater than 50%. 


He has also noticed that he is developing 
itchiness and a generalised rash. 

Which one of the following skin conditions is it 
likely to be? 

1- Erythema marginatum 

2- Dermatitis herpetiformis 

3- Erythema chronicum migrans 

4- Pityriasis rosea 

5- Tinea versicolor 

Answer & Comments 

Answer: 2- Dermatitis herpetiformis 

In coeliac disease, dermatitis herpetiformis 
manifests as a pruritic rash. It is a chronic, 
extremely itchy rash consisting of papules and 
vesicles. Dermatitis herpetiformis is associated 
with sensitivity of the intestine to gluten in the 
diet (celiac sprue). The vesicles or papules 
appear on the elbow s, knees, back, and 
buttocks (pressure points). It may also present 
as a patch of red skin with little water blisters 
scattered about. 



' 

J k ki M 




. # P 94 

Y -> 

* 


k t 

Jfh 






[Q: 123] MRCPass 
Gastroenterology 


In patients with confirmed hemochromatosis, 
one of 2 gene defects described (CYS282 
tyrosine or H63D mutation) is found in >85% 
of patients of Northern European descent. 



[Q: 124] MRCPass - 
Gastroenterology 


A 35 year man has abdominal pains, weight 
loss and diarrhoea. He was subsequently 
found to be lactose intolerant and a small 
bowel biopsy showed changes consistent with 
in coeliac disease. 


Dermatitis herpetiformis 


[Q: 125] MRCPass- 
Gastroenterology 

A 40 year old man has had intermittent 
diarrhoea and joint pains for 6 months. On 
examination he has limited vertical eye 
movements. 

Which pathogen is the likely cause of his 
symptoms? 

1- Salmonella enteritidis 

2- Cryptosporidium 



Dr. Khalid Yusuf El-Zohry - Sohag Teaching Hospital (01118391123) 

Ref MRCPass OE OE 2012 PasTest 2009 PassMedicine 2009 PasTest Exam ReviseMRCP 



82 



























El-zohry MRCP Questions Bank (Port 1) - 2013 


(For my personal use) 


3- Trophyrema whippleii 

4- Prion protein 

5- Syphillis 


Answer & Comments 

Answer: 3- Trophyrema whippleii 

Whipple's disease characteristically occurs in 
middle-aged men, who present with weight 
loss, fever, abdominal pain, arthralgias and 
intestinal symptoms of diarrhea and 
malabsorption. 

Confusion, memory loss, or uncontrolled eye 
movements indicate that the infection has 
travelled to the CNS. 

Trimethoprim - sulfamethoxazole is 
recommended; treatment is continued for one 
year. 



[Q: 126] MRCPass - 
Gastroenterology 


A 40 year old man presented with a 5 day 
history of bloody diarrhoea. On examination, 
he was apyrexial, jaundiced and pale. There 
was no organomegaly but there was mild 
lower abdominal tenderness. 

Investigations reveal: 

Haemoglobin 8.1 g/dL 

White cell count 17.5 x 10 9 /l_ 

Platelets 70 x 10 9 /l_ 

urea 11 pmol/l 

creatinine 220 pmol/l 

aspartate aminotransferase 110 IU/L 

Prothrombin time 12s (11.5-15.5) 

Blood film shows fragmented red cells 

What is the likely diagnosis? 

1- Ulcerative colitis 

2- Escherichia coli 0157 colitis 

3- Salmonella enterocolitis 

4- Campylobecter colitis 


5- Yersinia colitis 


Answer & Comments 

Answer: 2- Escherichia coli 0157 colitis 

The combination of haemolytic anaemia and 
thrombocytopaenia is consistent with 
haemolytic uraemic syndrome (HUS - TTP). 
The most likely cause of diarrhoea is E coli 
0157. 

Hemolytic uremic syndrome (HUS) is 
characterized by the triad of microangiopathic 
hemolytic anemia, thrombocytopenia, and 
acute renal failure. Diarrhea and upper 
respiratory infection are the most common 
precipitating factors. Other bacterial agents 
include Shigella, Salmonella, Yersinia, and 
Campylobacter species. 


A 40 year old man visit his GP with symptoms 
of flushing and dizziness. He also has watery 
diarrhoea several times a month. On 
examination he has a systolic murmur in the 
tricuspid area and a parasternal heave over 
the left sternal edge. A 24 hour urine shows 
raised 5HT levels. 

What is the diagnosis? 

1- Tropical Sprue 

2- Crohn's disease 

3- Phaeochromocytoma 

4- Carcinoid syndrome 

5- Pernicious anaemia 



[Q: 127] MRCPass - 
Gastroenterology 


Answer & Comments 

Answer: 4- Carcinoid syndrome 

Carcinoid syndrome is diagnosed by raised 
urinary 5-HT levels. A precursor of 5HT, 
tryptophan is highly metabolised and 
consequently niacin deficiency (pellagra) 
occurs. The rest of the D's dementia, 


Dr. Khalid Yusuf El-Zohry - Sohag Teaching Hospital (01118391123) 

Ref MRCPass OE OE 2012 PasTest 2009 PassMedicine 2009 PasTest Exam ReviseMRCP 



83 





























El-zohry MRCP Questions Bank (Port 1) - 2013 


(For my personal use) 


dermatitis (a photosensitive rash) and 
diarrhoea occur in pellagra. 



Flushing seen in carcinoid syndrome 



[Q: 128] MRCPass - 
Gastroenterology 


A 65 year old man is investigated for lower 
back pain. He also complains of weight loss 
and fevers. On review ing his history, he has 
been told before that he has ankylosing 
spondylitis. 


On admission, his blood tests show a urea of 
25 pmol/l and a creatinine of 350 pmol/l. An 
ultrasound of the kidneys shows 
hydronephrosis bilaterally. CT scan shows 
fibrotic para-aortic masses. 


Which of the following is the diagnosis? 


1- Carcinoid syndrome 


2- Retroperitoneal fibrosis 

3- Amyloidosis 

4- Metastatic bladder cancer 


5- Lymphoma 


Answer & Comments 

Answer: 2- Retroperitoneal fibrosis 

The symptoms of fevers, weight loss and 
lower back pain are classical for 
retroperitoneal fibrosis. There is an 


association with inflammatory conditions such 
as SLE, rheumatoid arthritis, ankylosing 
spondylitis, Hashimoto's thyroiditis and 
glomerulonephrosis. CT or MRI shows fibrotic 
para-aortic masses causing ureteric 
obstruction. 


[Q: 129] MRCPass - 
Gastroenterology 

A 20 year old nurse has recently been 
diagnosed with hepatitis B and has had 
serology measured. 

Which of the following is the best marker of 
infectivity? 

1- DsDNA 

2- HbcAg 

3- HBeAg 

4- HBsAg 

5- HBs Ab 



Answer & Comments 

Answer: 3- HBeAg 

HBeAg (not HBeAg) is the best marker of 
infectivity, and is used as an important criteria 
for selection of patients who have chronic 
hepatitis B for interferon (a-2B) therapy. HBV 
DNA and HBeAg levels are measured in 
response to the therapy and undetectable 
levels are considered successful treatment. 






HB#Ag +- 

Anti-HBe 
HBsAg * 




+ ± 


+ + 
+ ± 



Dr. Khalid Yusuf El-Zohry - Sohag Teaching Hospital (01118391123) 

Ref MRCPass OE OE 2012 PasTest 2009 PassMedicine 2009 PasTest Exam ReviseMRCP 



































El-zohry MRCP Questions Bank (Port 1) - 2013 


(For my personal use) 



[Q: 130] MRCPass - 
Gastroenterology 

A 60 year old woman with arthritis was 
referred for investigation of iron deficiency 
anaemia. Endoscopy several superficial antral 
erosions. A small bowel biopsy showed mild 
villous blunting, apopotic bodies, occasional 
eosinophils, mild increase in chronic 
inflammatory cells. Colonoscopy was 
unremarkable. 

Whot is the likely cause of the anaemia? 

1- Whipple's disease 

2- Ulcerative colitis 

3- Nonsteroidal anti inflammatory drugs 

4- Myelodysplastic syndrome 

5- Coeliac disease 


Answer & Comments 

Answer: 3- Nonsteroidal anti inflammatory 
drugs 

Superficial ulceration on the OGD suggests 
that the anaemia is due to NSAID therapy. 
Small bowel biopsy is to exclude coeliac 
disease, in this case there is no villous atrophy. 

Colonoscopy is to exclude angiodysplasia and 
a tumour. 



[Q: 131] MRCPass - 
Gastroenterology 

A 65 year old lady has rheumatoid arthritis. In 
the last 6 months she has become more 
lethargic and undergoes evaluation. On 
examination, she has palpable splenomegaly. 

Her blood tests reveal Hb of 8.5 g/dl, WCC of 3 
x 10 9 /I with a neutrophil count of 0.9, and 
platelet count of 160 x 10 9 /L. 

What is the diagnosis? 

1- Aplastic anaemia 

2- Juvenile chronic arthritis 

3- Haemochromatosis 


4- Felty's syndrome 

5- Ankylosing spondylitis 


Answer & Comments 

Answer: 4- Felty's syndrome 

Felty's syndrome consists of a triad of 
neutropenia, hypersplenism and rheumatoid 
arthritis, which is the most likely unifying 
diagnosis Although the pathophysiology of 
Felty syndrome is not fully known, evidence 
points to splenic sequestration and 
subsequent granulocyte destruction. 

The extra-articular manifestations of RA (eg, 
rheumatoid nodules, pleuropericarditis, 
vasculitis, peripheral neuropathy, episcleritis, 
other forms of eye involvement, Sjogren 
syndrome, adenopathy, skin ulcers) are more 
common in patients who develop Felty 
syndrome. 


[Q: 132] MRCPass- 
Gastroenterology 

A 60 year old woman presents with a history 
of worsening dysphagia over many years. 
Recently there were episodes of atypical 
central chest discomfort and cough. She also 
mentioned occasional regurgitation of her 
food. 

An X ray shows narrow ing at the level of the 
gastroesophageal junction. 

What is the likely diagnosis? 

1- Oesophageal carcinoma 

2- Pharyngeal pouch 

3- Oesophagitis 

4- Barrett's oesophagus 

5- Achalasia 



Answer & Comments 

Answer: 5- Achalasia 

Achalasia presents often in 3rd to 5th decade. 
The classical triad of achalasia is dysphagia 


Dr. Khalid Yusuf El-Zohry - Sohag Teaching Hospital (01118391123) 

Ref MRCPass OE OE 2012 PasTest 2009 PassMedicine 2009 PasTest Exam ReviseMRCP 





























El-zohry MRCP Questions Bank (Port 1) - 2013 


(For my personal use) 


(difficulty swallowing) to fluids and later 
solids, regurgitation of undigested food, and 
chest pain. Other symptoms may include 
difficulty belching, frequent hiccups, cough. 
30% have a nocturnal cough due to aspiration 
of oesophageal contents. 

An X ray or barium Swallow shows narrow ing 
at the level of the gastroesophageal junction 
of the lower esophagus and various degrees of 
megaesophagus (esophageal dilation) as the 
esophagus is gradually stretched by retained 
food. Manometry is the key test for 
establishing the diagnosis. A probe measures 
the pressure waves in different parts of the 
esophagus and stomach during the act of 
swallowing. 



Achalasia 



[Q: 133] MRCPass - 
Gastroenterology 


A 35 year old patient has had a hepatitis test 
for investigation of jaundice. He has a positive 
Hep BsAg (HBsAg) and has IgM antibodies to 
Hep Bcore (anti-HBc). 


What is he likely to have? 


1- Hepatitis D infection 

2- Seroconversion illness 


3- Chronic hepatitis B infection 

4- Acute hepatitis B infection 

5- Recent vaccination 


Answer & Comments 

Answer: 4- Acute hepatitis B infection 

The positive anti-HBc (IgM) and HBsAg 
suggests acute infection. When the infection 
resolves, HBsAg becomes negative and anti- 
HBc (IgG) is positive. In patients who have 
been vaccinated, HBsAg is negative and anti- 
HBs is positive. 




[Q: 134] MRCPass - 
Gastroenterology 


A 25 year old woman has type 1 diabetes. She 
has weight loss of 1 stone over the past 3 
months, and irregular menstrual cycles. Her 
bowel habit has been unchanged. On 
examination her BMI was 24 kg/m 2 . 


Investigations show a haemoglobin of 8.1 g/dl 
with a MCV of 69 fl. 


Which is the likely diagnosis? 

1- Heavy Periods 

2- Bacterial overgrowth 

3- Crohn's disease 


4- Dietary iron deficiency 

5- Coeliac disease 


Answer & Comments 

Answer: 5- Coeliac disease 

Coeliac disease is most likely as this patient 
autoimmune phenomenon (diabetes), with an 
iron deficiency anaemia and no symptoms of 
diarrhoea. Patients with type 1 diabetes are at 
up to six times greater risk of having coeliac 



Dr. Khalid Yusuf El-Zohry - Sohag Teaching Hospital (01118391123) 

Ref MRCPass OE OE 2012 PasTest 2009 PassMedicine 2009 PasTest Exam ReviseMRCP 













































El-zohry MRCP Questions Bank (Port 1) - 2013 


(For my personal use) 


disease. Antibodies to gliadin should be sent 
and the diagnosis of coeliac disease can be 
confirmed by taking a biopsy from the 
duodenum (small bowel just beyond the 
stomach) which will show villous atrophy. 


2- Wilson's disease 

3- Budd Chiari syndrome 

4- Alpha 1 antitrypsin deficiency 

5- Alcoholic liver disease 



[Q: 135] MRCPass - 
Gastroenterology 


A 50 year old alcoholic patient presents with 
confusion and severe derangement of liver 
function. On examination, his MMSE score is 
22 / 30. He has signs of spider naevi, jaundice 
and gross ascites. 


Which one of the following features defines 
fulminant acute hepatic failure? 


1- Spider naevi 


2- Leukonychia 


3- Jaundice 


4- Ascites 

5- Encephalopathy 


Answer & Comments 

Answer: 5- Encephalopathy 

By definition, fulminant hepatic failure is 
associated with encephalopathy. 

Prognosis is poor irregardless of speed of 
disease progression. It typically presents 
acutely when previous hepatic disease is 
unknown. Commonest causes in the UK are 
paracetamol overdose and hepatitis. 


[Q: 136] MRCPass - 
Gastroenterology 

A 50 year old woman has symptoms of 
pruritis, steatorrhoea and easy bruising. On 
examination, she is jaundiced and had 
xanthelasmata. Her skin was pigmented and 
has 8 spider naevi. There is also 
hepatosplenomegaly. 

What is the likely overall diagnosis? 

1- Primary biliary cirrhosis 



Answer & Comments 

Answer: 1- Primary biliary cirrhosis 

She has chronic liver disease with portal 
hypertension. The 2 main conditions causing 
pigmentation along with signs of chronic liver 
disease are primary biliary cirrhosis (PBC) & 
haemochromatosis. Xanthelasmata are 
common in PBC but not in haemochromatosis. 
PBC is a chronic cholestatic inflammatory liver 
disease, aetiology of which probably 
autoimmune. It commonly affects middle 
aged women. 


Serum AntiMitochondrial Antibody is positive 
in 95 % of cases. 



Primary biliary cirrhosis 



[Q: 137] MRCPass - 
Gastroenterology 


A 45 year old lady has a two month history of 
nausea, jaundice and dark urine. She also has 
a history of arthralgia and autoimmune 
thyroid disease. 


On examination, her abdomen was non¬ 
tender with 4 cm hepatosplenomegaly. 

Investigations reveal AST 1675 U/L (5 - 40), 
ALT 3900 U/L (5 -40), GGT 179 U/L (10 - 60), 
bilirubin 65 (1-22) pmol/l, increase in total 
globulin and smooth muscle antibodies are 
positive. Serum immunoglobulins were 


Dr. Khalid Yusuf El-Zohry - Sohag Teaching Hospital (01118391123) 

Ref MRCPass OE OE 2012 PasTest 2009 PassMedicine 2009 PasTest Exam ReviseMRCP 



87 


























El-zohry MRCP Questions Bank (Port 1) - 2013 


(For my personal use) 


elevated with IgG of 2390 mg/dl (694 - 1618) 
and IgA of 306 mg/dl (68 - 263). Anti-nuclear 
antibodies (ANA) were positive at 1:320 titer 
in a homogenous speckled pattern. Anti¬ 
smooth muscle and liver-kidney microsomal 
antibodies were positive. 

Whot is the likely diagnosis? 

1- Wilson's disease 

2- Cholangiocarcinoma 

3- Primary biliary cirrhosis 

4- Autoimmune hepatitis 

5- Crohn's disease 

Answer & Comments 

Answer: 4- Autoimmune hepatitis 

In autoimmune hepatitis, response to steroids 
is excellent and it improves five-year survival, 
but does not prevent development of 
cirrhosis. Azathioprine is an useful adjunct to 
steroids and allows reduction of steroid doses, 
thus reduces the risk of osteoporosis. 

Chronic autoimmune hepatitis is a chronic 
hepatitis of unknown origin that 
predominantly affects young and middle aged 
women. It is associated with HLA types Al, B8, 
DR3 and Dw 3. Presentation is usually 
insidious. The patient may be generally unwell 
and jaundiced. Amenorrhoea is common. 
Examination may reveal signs of chronic liver 
disease, hepatomegaly and splenomegaly. 

Investigations show raised inflammatory 
markers and serum transaminases. ANA and 
smooth muscle antibodies are usually positive. 


Where is gastrin produced? 

1- Oesophagus 

2- Colonic mucosa 

3- G cells of gastric antrum 

4- Islet cells 

5- Small intestine 

Answer & Comments 

Answer: 3- G cells of gastric antrum 

Gastrin is produced in 2 forms by the G cells of 
gastric antrum. It stimulates parietal cells 
produce hydrochloric acid. Its production is 
stimulated by neural reflex pathways and also 
by direct effect of digested peptides on the G 
cells. It also stimulates the production of 
bicarbonate. 

3 

A 65 year old woman has a diagnosis of insulin 
dependent diabetes mellitus (poorly 
controlled) for 25 years. She complains of 
repeated episodes of abdominal pain 
following meals. These episodes have become 
more frequent over the past 3 months. There 
is no abdominal tenderness on palpation. 

Which of following finding is likely be present? 

1- Autonomic neuropathy 

2- Hepatitis 

3- Mesenteric artery occlusion 

4- Chronic pancreatitis 

5- Acute pancreatitis 


[Q: 139] MRCPass 
Gastroenterology 



[Q: 138] MRCPass - 
Gastroenterology 


A 50 year old man presents has a 6 month 
history of severe indigestion. 3 years ago he 
had a duodenal ulcer seen on endoscopy. 


Investigations reveal: 

Fasting gastrin 150 pmol/L (<55) 


Answer & Comments 

Answer: 3- Mesenteric artery occlusion 

Diabetes, especially Type 2 diabetes is 
associated with macrovascular disease. If the 
mesenteric artery is stenosed or occluded 
then lack of blood flow to the bowel will 
produce ischaemic pains. 


Dr. Khalid Yusuf El-Zohry - Sohag Teaching Hospital (01118391123) 

Ref MRCPass OE OE 2012 PasTest 2009 PassMedicine 2009 PasTest Exam ReviseMRCP 




























El-zohry MRCP Questions Bank (Port 1) - 2013 


(For my personal use) 


[Q: 140] MRCPass - 
Gastroenterology 

A 60 year old man is investigated for weight 
loss and dyspepsia. Endoscopic examination 
reveals an ulcerated lesion in stomach. Biopsy 
shows the presence of a low grade mucosa 
associated lymphoma and Helicobacter pylori. 
CT of chest & abdomen shows no metastases. 

Whot is the best treatment option for this 
patient? 

1- Radiotherapy 

2- Helicobacter pylori eradication 

3- Bilroth's gastrectomy 

4- Oral chlorambucil 

5- CHOP therapy 




A MALT tumour 


Answer & Comments 

Answer: 2- Helicobacter pylori eradication 

The diagnosis is a gastric MALT tumour. These 
are usually marginal zone B cell lymphomas 
associated an excellent prognosis. 

MALT lymphoma is the third most common 
type of non-Hodgkin lymphoma, although it 
only accounts for about 7-8% of these 
tumours. MALT lymphomas have been 
described at almost all extra-nodal sites, but 
are most commonly found in the 
gastrointestinal tract (stomach is the 
commonest). 

Low grade gastric MALT tumours which are 
associated with Helicobacter Pylori infection 
respond in over 80% of cases to helicobacter 
eradication. A proportion of patients will not 
respond to eradication therapy alone and will 
go on to more conventional anti-lymphoma 
therapies such as cyclophosphamide, 
chlorambucil, nucleoside analogues or 
radiotherapy. 


Dr. Khalid Yusuf El-Zohry - Sohag Teaching Hospital (01118391123) 

Ref MRCPass OE OE 2012 PasTest 2009 PassMedicine 2009 PasTest Exam 




ReviseMRCP 

89 






















El-zohry MRCP Questions Bank (Port 1) - 2013 


(For my personal use) 


[ Q: 141 ] MRCPass - Rheumatology 

#5 - 

A 35 year old man presents with a 6 
month history of arthralgia, mouth ulceration 
and eye irritation. On examination, he had 
some ulceration in the mouth, bilaterally 
swollen wrists and reduced range of 
movements of both knees. 

His investigations showed: white cell count 11 
x 10 9 /L, C reactive protein 100 mg/dl, 
Rheumatoid factor negative. 

What is the likely diagnosis? 

1- Reiter's syndrome 

2- Sjogren's syndrome 

3- Ankylosing spondylitis 

4- Behcet's syndrome 

5- Sarcoidosis 

Answer & Comments 

Answer: 4- Behcet's syndrome 

Behcet's syndrome is a multisystem disorder 
characterised by recurrent oral genital 
ulceration, eye lesions (anterior and posterior 
uveitis or retinal vasculitis), skin lesions, 
(erythema nodosum, papulopustular lesions 
folliculitis) and a positive pathergy test. 

The pathergy phenomenon is considered an 
outstanding feature of Behcet disease. 
Following a needle prick or intradermal 
injection with saline or dilute histamine, the 
puncture site becomes inflamed and develops 
a small sterile pustule due to hyperactivity of 
the skin to any intracutaneous insult. 



Oral Ulceration in Behcet's disease 


^ [ Q: 142 ] MRCPass - Rheumatology 

/ - 

# A 33 year old female with systemic 

lupus erythematosis has arthralgia involving 
her upper limbs. She also has a butterfly facial 
rash and a rash on the trunk. 

Urine dipstick shows no Proteinuria or 
haematuria. Her renal function is normal. 

Which one of the following medications is 
most appropriate? 

1- Methotrexate 

2- Prednisolone 

3- Azathioprine 

4- Hydroxychloroquine 

5- Cyclosporin 

Answer & Comments 

Answer: 4- Hydroxychloroquine 

NSAIDs and hydroxychloroquine are used for 
skin involvement and arthritis. 

NSAIDs are used for mild disease. 

Hydroxychloroquine is useful for disease not 
controlled by NSAIDS. Steroids are used in 
moderate to severe disease. 

Immunosuppressive treatments such as 
azathioprine and cyclophosphamide are used 
typically w hen there is renal or cerebal 

disease. 

^ [ Q: 143 ] MRCPass - Rheumatology 

/ - 

A 75 year old woman with long¬ 
standing Rheumatoid arthritis has great 

difficulty walking and comments that both her 
legs are stiff and 'jumpy'. 

What is the most likely cause of the 
presentation? 

1- Ankylosing spondylitis 

2- Syringomyelia 

3- Osteoporosis 

4- Atlanto-axial instability 

5- Disc compression 


Dr. Khalid Yusuf El-Zohry - Sohag Teaching Hospital (01118391123) 

Ref MRCPass OE OE 2012 PasTest 2009 PassMedicine 2009 PasTest Exam 




ReviseMRCP 

91 


























El-zohry MRCP Questions Bank (Port 1) - 2013 


(For my personal use) 


Answer & Comments 

Answer: 4- Atlanto-axial instability 

Cervical cord compression due to atlanto-axial 
instability is the most likely cause of UMN 
nerve weakness. 



[ Q: 144 ] MRCPass - Rheumatology 

A 50 year old woman has severe 
rheumatoid arthritis. She is admitted with 
worsening breathlessness. 


She is currently on ibuprofen, methotrexate, 
celecoxib, and paracetamol. On examination, 
she has features of rheumatoid changes in her 
hands and looks pale. There is no palpable 
lymphadenopathy and no abdominal masses. 


Investigations showed that her Hb level is 7.8 
g/dl with a MCV of 90, WCC 2.1 x 10 9 /l and 
platelets 55 x 10 9 /l. 

Reticulocyte count is 0.3%(0.5% to 1.5%). 

Whot is the likely couse of the onoemio? 

1- NSAID use and Gl bleed 


2- Treatment with celecoxib 

3- Treatment with methotrexate 


4- Anaemia of chronic disease 

5- Felty's syndrome 


Answer & Comments 

Answer: 3- Treatment with methotrexate 

The patient has an aplastic anaemia which can 
be caused by methotrexate or azathioprine, 
DMARDs (penicillamine or gold). 

Abnormally low reticulocyte count can be 
attributed to chemotherapy, aplastic anemia, 
pernicious anemia, bone marrow malignancies 
and lowerythropoietin levels. 


[ Q: 145 ] MRCPass - Rheumatology 

A 65 year old woman presents with 
dry eyes and a dry mouth. Her investigations 
show : ANA strongly positive (1:1600), 



antiRo/SSA antibodies strongly positive, 
rheumatoid factor positive, IgG markedly 
elevated at 42 g/l 

(normal <15 g/l), IgM and IgA levels are 
normal. 

Whot is the likely diagnosis? 

1- Monoclonal gammopathy of unknown 
significance 

2- Primary Sjogren's Syndrome 

3- Systemic Lupus Erythematosus 

4- Reiter's syndrome 

5- Polyarteritis Nodosa 


Answer & Comments 

Answer: 2- Primary Sjogren's Syndrome 

The clinical features and are consistent with 
primary Sjogren's Syndrome. 

Hypergammaglobulinaemia is present in 80% 
of individuals. ANA, AntiRo/SSA antibodies are 
present in approximately 90% of individuals 
and there can also be a weakly positive 
rheumatoid factor. 



Dry mouth seen in Sjogren's syndrome 


[ Q: 146 ] MRCPass - Rheumatology 

A 55 year old female has been on 
long-term steroids for chronic obstructive 
pulmonary disease. She complains of pain in 
her right groin radiating down the 
anteromedial thigh. She has an antalgic gait. 

On examination of the hip, there is decreased 
range of movement especially flexion, 
abduction and internal rotation. 



Dr. Kholid Yusuf El-Zohry - Sohog Teaching Hospital (01118391123) 

Ref MRCPass OE OE 2012 PasTest 2009 PassMedicine 2009 PasTest Exam ReviseMRCP 



92 





























El-zohry MRCP Questions Bank (Port 1) - 2013 


(For my personal use) 


What is the likely diagnosis? 

1- Osteoarthritis 

2- Rheumatoid arthritis 

3- Metastatic hip lesion 

4- Avascular necrosis of the femoral head 

5- Hairline fracture 


Answer & Comments 

Answer: 4- Avascular necrosis of the femoral 
head 

In a patient on long term steroids presenting 
with groin pains radiating to the thigh 
associated with an antalgic gait and decreased 
range of movement of the hip, the most likely 
diagnosis is avascular necrosis of the femoral 
head. 

In this condition, MRI is the most sensitive and 
specific technique and is useful for early 
diagnosis before collapse of bone occurs. CT 
scan and x-ray are useful to rule out advanced 
disease if duration is not clear. Bone scanning 
is more sensitive than x-ray but is non-specific. 



cough. The oxygen saturation was found be 
85% on air. 

The chest x ray shows a diffuse bilateral 
interstitial infiltrate. 

Blood cultures and sputum cultures are 
negative. 

Which drug is likely to have caused this 
adverse effect? 

1- Cyclosporin 

2- Cyclophosphamide 

3- Gold 

4- Methotrexate 

5- Sulphasalazine 


Answer & Comments 

Answer: 4- Methotrexate 

Pneumonitis is a serious and unpredictable 
side-effect of treatment with methotrexate 
(MTX) that may become life-threatening. 
Chest radiography reveals a diffuse interstitial 
or mixed interstitial and alveolar infiltrate, 
with a predilection for the lower lung fields. 

Pulmonary function tests show a restrictive 
pattern with diminished diffusion capacity. 
Lung biopsy reveals cellular interstitial 
infiltrates, granulomas or a diffuse alveolar 
damage pattern accompanied by perivascular 
inflammation. 



[ Q: 148 ] MRCPass - Rheumatology 

A 65 year old man presents has 
recently developed an acutely painful right 
knee. On examination, he had a temperature 
of 37.2°C a hot and swollen right knee. 


MRI showing avascular necrosis of the femoral 

head 


[ Q: 147 ] MRCPass - Rheumatology 

A 60 year old woman on treatment 
for longstanding rheumatoid arthritis presents 
with breathlessness. She complains of a dry 



His white cell count which was raised at 14 x 
10 9 /L. A knee X ray showed reduced joint 
space and chondrocalcinosis. 

Culture of aspirated fluid showed no growth. 

What is the likely diagnosis? 

1- Psoriatic arthropathy 


Dr. Khalid Yusuf El-Zohry - Sohag Teaching Hospital (01118391123) 

Ref MRCPass OE OE 2012 PasTest 2009 PassMedicine 2009 PasTest Exam ReviseMRCP 



93 



























El-zohry MRCP Questions Bank (Part 1) - 2013 


(For my personal use) 


2- Rheumatoid arthritis 

3- Pseudogout 

4- Septic arthritis 

5- Osteomalacia 


5- Ankylosing spondylitis 

Answer & Comments 


Answer: 2- Haemochromatosis 


Answer & Comments 

Answer: 3- Pseudogout 

The clinical picture is typical of pseudogout. 
Calcification of the articular cartilage would be 
consistent and the culture results excludes 
septic arthritis. The diagnosis can be 
confirmed by the presence of calcium 
pyrophosphate crystals in joint fluid 
demonstrating a positive birefringence under 
polarised light. 



The history of abdominal pain together with 
skin pigmentation and hepatomegaly suggest 
the patient has haemochromatosis. 
Haemochromatosis is associated with 
chondrocalcinosis, which commonly affects 
the 2nd and 3rd metacarpophalangeal joints. 
The acute arthropathy is likely to be 
pyrophosphate crystal arthritis. 

[ Q: 150 ] MRCPass - Rheumatology 

A 70 year old man presents with 
bony swellings of the DIP joints on both 
hands. They were painful a year ago but are 
now painless. 

The most likely diagnosis is : 

1- Heberden's nodes 

2- Bouchard's nodes 

3- Osier's nodes 


Calcium pyrophosphate crystals 


[ Q: 149 ] MRCPass - Rheumatology 

A 60 year old man presents with a 
painful swollen left knee as well as pain and 
stiffness of both his hands. 

On examination his skin is pigmented. There is 
tenderness and swelling of the 2nd and 3rd 
metacarpophalangeal joints of both hands, he 
has hepatomegaly of 8 cm below the costal 
margin. 

His knee is swollen and aspiration of the joint 
yields turbid, straw coloured fluid. 

Which one of the following diagnosis is likely? 

1- Wilson's disease 

2- Haemochromatosis 

3- Pseudohypoparathyroidism 

4- Pseudopseudohypoparathyroidism 



4- Gouty tophi 

5- Charcot's joints 


Answer & Comments 

Answer: 1- Heberden's nodes 

The most likely diagnosis is osteoarthritis. At 
the DIP joints, swelling are known as 

Heberden's nodes and at the PIP joints, they 
are known as Bouchard's nodes. 



Heberden's node 



94 


Dr. Khalid Yusuf El-Zohry - Sohag Teaching Hospital (01118391123) 

Ref MRCPass OE OE 2012 PasTest 2009 PassMedicine 2009 PasTest Exam ReviseMRCP 
































El-zohry MRCP Questions Bank (Part 1) - 2013 


(For my personal use) 


[ Q: 151 ] MRCPass - Rheumatology 

f - -— 

# A 60 year old man presents with 
gouty tophi. He has been commenced on 
allopurinol but develops severe joint pains 3 
days later. On examination, he has a 
temperature of 39°C and erythematous 
swelling of his wrists, knees and ankles. 

Investigations reveal: urate 0.6 (0.23-0.45), 
creactive protein 180 mg/L. 

Which of the following is likely to hove caused 
the presentation? 

1- Allopurinol 

2- Colchicine 

3- Prednisolone 

4- Pseudogout 

5- Septic arthritis 


What is the likely diagnosis? 

1- Compartment syndrome 

2- Deep vein thrombosis 

3- Reflex sympathetic dystrophy 

4- Raynaud's phenomenon affecting the leg 

5- Venous varicosities 

Answer & Comments 

Answer: 3- Reflex sympathetic dystrophy 

This clinical scenario is compatible with a 
diagnosis of reflex sympathetic dystrophy or 
complex regional pain syndrome following the 
previous injury. Reflex sympathetic dystrophy, 
also known as RSD, is a condition of burning 
pain, stiffness, swelling, and discoloration of 
the affected area. 


Answer & Comments 

Answer: 1- Allopurinol 

Allopurinol blocks uric acid production and is 
the drug most often used in long-term 
treatment for older patients and 
overproducers of uric acid. Allopurinol is taken 
orally once a day in doses of 100 mg to 600 
mg, depending on the patient's response to 
treatment. 

Between 3% to 5% of patients experience 
leukopenia, thrombocytopenia, diarrhea, 
headache, and fever. 


[ Q: 152 ] MRCPass - Rheumatology 

A 35 year old retired athlete 
presents with severe burning pain affecting 
the right leg and foot. 4 months earlier, he 
had several arthroscopic washouts for septic 
arthritis affecting his right knee, which 
followed an injury. 

Examination reveals a reduced range of 
movement of the right knee and ankle. There 
was diffuse swelling of the right leg and foot 
with overlying cool, scaly skin. 



The pain is often severe and disproportionate 
to the signs and follow s a non-anatomic 
distribution. The skin changes are due to the 
associated autonomic dysfunction. 



Reflex sympathetic dystrophy affecting the 

right foot 


•i 


[ Q: 153 ] MRCPass - Rheumatology 

A 75 year old has had increasing 
back and leg pains several years. X rays reveal 
bony sclerosis of sacroiliac, lower vertebral 
and upper tibial regions. He mentions greater 
difficulty hearing over the recent years. 


Dr. Khalid Yusuf El-Zohry - Sohag Teaching Hospital (01118391123) 

Ref MRCPass OE OE 2012 PasTest 2009 PassMedicine 2009 PasTest Exam 




ReviseMRCP 

95 
























El-zohry MRCP Questions Bank (Part 1) - 2013 


(For my personal use) 


Blood tests reveal an elevated serum alkaline 
phosphatase. 

What is the likely diagnosis? 

1- Paget's disease of bone 

2- Osteoarthritis 

3- Osteomalacia 

4- Monoclonal gammapothy of uncertain 
significance 

5- Multiple myeloma 

Answer & Comments 

Answer: 1- Paget's disease of bone 

In Paget's disease, onset of symptoms is 
usually insidious, with pain, stiffness, bone 
deformity, headaches, decreasing auditory 
acuity, and increasing skull size. 


^ [ Q: 154 ] MRCPass - Rheumatology 

sit - 

# A 55 year old patient presents with a 
two week history of pain and stiffness in her 
shoulders and wrists. The symptoms improve 
over the day. On examination, there was 
synovitis of both wrists and proximal muscle 
wasting. Her ESR was 40 mm/hr. 

What is the likely diagnosis? 

1- Dermatomyositis 

2- Systemic lupus erythematosus 

3- Rheumatoid arthritis 

4- Polymyalgia rheumatica 

5- Osteoarthritis 

Answer & Comments 

Answer: 3- Rheumatoid arthritis 


Signs may be bitemporal skull enlargement 
with frontal "bossing," dilated scalp veins, 
nerve deafness in one or both ears, angioid 
streaks in the fundus of the eye, and 
anterolateral bow ing of the thigh or leg with 
w armth and periosteal tenderness. 

Pagetic lesions are metabolically active and 
highly vascular and may lead to high-output 
heart failure. 

Deformities may develop from bow ing of the 
long bones or osteoarthritis of adjacent joints. 

Pathologic fractures may be the presenting 
finding. Characteristic x-ray findings include 
increased bone density, abnormal 
architecture, cortical thickening, bow ing, and 
overgrowth. 

Biochemistry includes elevated serum alkaline 
phosphatase (or bone-specific alkaline 
phosphatase) and increased urinary excretion 
of pyridinoline cross-links. Serum calcium and 
phosphorus levels usually are normal, but 
serum calcium may increase during bed rest. 


In a middle aged female, acute arthritis of 
shoulders and wrists along with synovitis are 
highly suggestive of acute Rhematoid Arthritis. 



Synovitis involving the wrist in rheumatoid 

arthritis 


[ Q: 155 ] MRCPass - Rheumatology 

A 45 year old woman presents with 
numbness and weakness of her upper and 
lower limbs. She developed asthma at the age 
of 30. 

On examination she looks unwell. She has 
palpable purpura over her face and over her 
elbows and knees. 

On neurological examination she has a right 
sided wrist drop and there is weakness of 



Dr. Khalid Yusuf El-Zohry - Sohag Teaching Hospital (01118391123) 

Ref MRCPass OE OE 2012 PasTest 2009 PassMedicine 2009 PasTest Exam ReviseMRCP 



96 



























El-zohry MRCP Questions Bank (Port 1) - 2013 


(For my personal use) 


dorsiflexion of her right foot. Sensation is also 
impaired over the dorsum of her right foot. 

Investigations: 

CXR is normal. 

Hb 10.9 g/dL 

MCV 90 fl 

WBC 23 x 10 9 /I 

Eosinophils 12% 

ANCA negative 

What is the likely diagnosis? 

1- Polyarteritis nodosa 

2- Churg Strauss syndrome 

3- SLE 

4- Allergic broncho pulmonary aspergillosis 

5- Takayasu's arteritis 

Answer & Comments 

Answer: 2- Churg Strauss syndrome 

The combination of mononeuritis multiplex, 
asthma, eosinophilia. 

The American College of Rheumatology (ACR) 
has proposed 6 criteria for diagnosis of Churg 
Strauss syndrome. The presence of 4 or more 
criteria yields a sensitivity of 85% and a 
specificity of 99.7%. 



Vasculitic rash on the skin of a patient with 
Churg Strauss syndrome 


^ [ Q: 156 ] MRCPass - Rheumatology 

/ -— 

# A 40 year old man has generalised 

joint pains and stiffness, particularly in the 

knees. He also has sore, dry eyes and difficulty 

tolerating contact lenses. On examination 

there are no joint swellings or effusions. His 

ESR is 80 mm/hour, Rheumatoid Factor 

positive with a titre of 1/1024. 

Which of the following is likely? 

1- Positive antibodies to Ro and La antigens 

2- Positive anti SCL-70 antibody 

3- Positive anti mitochondrial antibodies 

4- Positive anti-Sm antibodies 

5- Positive ANCA 


These criteria are 

(1) asthma (w heezing, expiratory rhonchi) 

(2) eosinophilia of more than 10% in 
peripheral blood 

(3) paranasal sinusitis 

(4) pulmonary infiltrates (may be transient) 

(5) histological proof of vasculitis with 
extravascular eosinophils 

(6) mononeuritis multiplex or polyneuropathy 


Answer & Comments 

Answer: 1- Positive antibodies to Ro and La 
antigens 

Ro is also known as anti ssA and La is known 
as anti ssB antibody, both are diagnostic tests 
for Sjogrens. The history of dry eyes 
(keratoconjunctivitis sicca) and joint pains 
with strongly positive RhF goes with Sjogrens. 




*1 J 


[ Q: 157 ] MRCPass - Rheumatology 


A 45 year old man has recurrent 


Dr. Khalid Yusuf El-Zohry - Sohag Teaching Hospital (01118391123) 

Ref MRCPass OE OE 2012 PasTest 2009 PassMedicine 2009 PasTest Exam ReviseMRCP 




























El-zohry MRCP Questions Bank (Port 1) - 2013 


(For my personal use) 


sinusitis and haemoptysis. He also has 
haematuria and mild renal impairment. 

Tests are sent for suspected Wegener's 
granulomatosis. 

Which one of the following tests has greatest 
specificity for Wegener's gronulomotosis? 

1- Anti glomerular basement antibody 

2- pANCA positive antibodies proteinase 3 

3- pANCA positive antibodies myeloperoxidase 

4- cANCA positive antibodies proteinase 3 

5- cANCA positive antibodies myeloperoxidase 


Answer & Comments 

Answer: 4- CANCA positive antibodies 
proteinase 3 

On immunofluoresecnce, if ANCA are present, 
the staining pattern may be cytoplasmic 
(cANCA) or perinuclear (pANCA). Typical 
antigen specificity includes c ANCA proteinase 
3 which is more common in Wegener's 
granulomatosis, p ANCA myeloperoxidase is 
more common in polyarteritis nodosa. 


^ [ Q: 158 ] MRCPass - Rheumatology 

- 

m A lady with w hiplash injury 5 years 
ago presents with pains in the neck and 
shoulder. They were not relieved by 12 co- 
codamols a day. 

What should be done next? 

1- Amitryptilline 

2- NSAIDs 

3- Physiotherapy 

4- Morphine 

5- Gabapentin 


Answer & Comments 

Answer: 3- Physiotherapy 

Whiplash injuries and radiculopathies causing 
back pains can be difficult to treat with 


medications. Physiotherapy has an important 
role for symptom relief in combination with 
analgesia. 

[ Q: 159 ] MRCPass - Rheumatology 

A 45 year old man presents with a 
week history of a painful right leg. He has had 
previous episodes of erythema nodosum, 
recurrent oral and scrotal ulceration. 
Examination reveals a diffusely swollen left 
leg. 

What is the likely cause of his swollen leg? 

1- Baker's cyst 

2- Gonococcal arthritis 

3- Reactive arthritis 

4- Deep vein thrombosis 

5- Cellulitis 



Answer & Comments 

Answer: 4- Deep vein thrombosis 

The overall diagnosis is Behcet's syndrome. 
There is a thrombotic tendency, hence the 
likely cause of a DVT. 


[ Q: 160 ] MRCPass - Rheumatology 

A 45 year old woman presents with 
pain in the wrist. The pain is centred over the 
radial styloid and is increased by abduction of 
the thumb against resistance. 

What is the most likely diagnosis? 

1- Carpal tunnel syndrome 

2- Rheumatoid arthritis 

3- De Quervain's tenosynovitis 

4- Osteoarthritis 

5- Adductor pollicis synovitis 



Answer & Comments 


Answer: 3- De Quervain's tenosynovitis 


Dr. Khalid Yusuf El-Zohry - Sohag Teaching Hospital (01118391123) 

Ref MRCPass OE OE 2012 PasTest 2009 PassMedicine 2009 PasTest Exam ReviseMRCP 































El-zohry MRCP Questions Bank (Port 1) - 2013 


(For my personal use) 


The pain in the thumb on resisted abduction is 
typical of De Quervain's. 


2- Rituximab 

3- Methotrexate 


De Quervain's disease occurs more frequently 
in w omen. The age group usually affected is 
30 to 50 year olds. 


4- Vincristine 

5- Hydroxychloroquine 


The history often is of unaccustomed or 
excessive activity such as rose pruning. The 
patient complains of pain on the radial side of 
the wrist. Abduction of the thumb against 
resistance is painful. Finkelstein's test is 
positive. This is performed with the thumb 
flexed across the palm of the hand, asking the 
patient to move the wrist into flexion and 
ulnar deviation. 

This stresses the tendons of abductor pollicis 
longus and extensor pollicis brevis and 
reproduces the pain of de Quervain's 
tenosynovitis. 



Finkelstein's test 


[ Q: 161 ] MRCPass - Rheumatology 

A 43 year rheumatoid arthritis was 
investigated routinely and had the following 
results: 

Haemoglobin 11.2 g/dL 
Platelets 385 x 10 9 /L 
White Cell Count 8.2 x 10 9 /L 
MCV110 fL 

Which drug is she likely to hove been on? 

1- Aspirin 



Answer & Comments 

Answer: 3- Methotrexate 

Methotrexate is associated with bone marrow 
suppression, and can lead to pancytopenia or 
a megaloblastic anaemia, especially if folate 
treatment is not given as well. 


[ Q: 162 ] MRCPass - Rheumatology 

A 75 year old woman has recently 
been commenced on alendronate for 
osteoporosis. 

Whot is the mechanism of action of 
alendronate? 

1- Increases osteoblast activity 

2- Inhibits osteoclast activity 

3- Increases vitamin D absorption 

4- Causes hypercalcaemia 

5- Increases the action of oestrogen on bone 



Answer & Comments 

Answer: 2- Inhibits osteoclast activity 

Alendronate is a bisphosphonate which can 
increase bone mineralisation by inhibiting 
osteoclastic activity. 



[ Q: 163 ] MRCPass - Rheumatology 

An 75 year old man presents with 
bilaterally painful knees. He has bilateral 
reduced knee movements and crepitus. X ray 
shows sclerosis, osteophytes and loss of joint 
space. 


Which one of the following is the most 
appropriate initial treatment? 


1- Ibuprofen 


Dr. Khalid Yusuf El-Zohry - Sohag Teaching Hospital (01118391123) 

Ref MRCPass OE OE 2012 PasTest 2009 PassMedicine 2009 PasTest Exam 




ReviseMRCP 

99 
























El-zohry MRCP Questions Bank (Port 1) - 2013 


(For my personal use) 


2- Ibuprofen and lansoprazole 

3- Paracetamol 

4- Codeine phosphate 

5- Celecoxib 


Answer & Comments 

Answer: 3- Paracetamol 

The principle goal of systemic therapy in 
osteoarthritis is to provide effective pain relief 
with least associated toxicity. Paracetamol is 
the recommended initial therapy, especially in 
the elderly due to possible gastrointestinal 
upset. 

^ [ Q: 164 ] MRCPass - Rheumatology 

# A 35 year old woman recently 
arrived in the UK from Ghana. She complains 
of neck pain with pins and needles affecting 
the right arm associated with a weak grip. 

Examination reveals tenderness over the 
cervical spine. X rays of the cervical spine 
shownarrow ing of the C3/4 and C4/5 joint 
space and partial collapse of C4. 

Investigations show : 

Hb 9.5 g/dl 

WCC 11.1 

platlets 520 

ESR 120 mm in the first hour 
CRP 250 g/l 

Calcium 2.21 micromol/l 
Albumin 32 g/l 

alkaline phosphatase 210 units/I 
phosphate 0.8 micromol/l. 

What is the most likely diagnosis? 

1- Potts disease 

2- Osteoporosis 

3- Multiple myeloma 

4- Ankylosing spondylitis 


5- Syringomyelia 

Answer & Comments 

Answer: 1- Potts disease 

Pott's disease is tuberculous infection of the 
spine with associated collapse of the vetebral 
body. The infection spreads from tw o 
adjacent vertebrae into the adjoining disc 
space. If only one vertebra is affected, the disc 
is normal, but if tw o are involved the 
intervertebral disc, which is avascular, 
collapses. 

Signs and symptoms include: Localised back 
pain, Paravertebral swelling, Neurological 
signs including paraplegia. 

Drug treatment (antituberculous drugs) is 
generally sufficient for Pott's disease, with 
spinal immobilisation if required. Surgery is 
required if there is spinal deformity or 
neurological signs of spinal cord compression. 



Potts disease on an MRI 



[ Q: 165 ] MRCPass - Rheumatology 
A 35 year old woman presents with 


Dr. Khalid Yusuf El-Zohry - Sohag Teaching Hospital (01118391123) 

Ref MRCPass OE OE 2012 PasTest 2009 PassMedicine 2009 PasTest Exam ReviseMRCP 

























El-zohry MRCP Questions Bank (Port 1) - 2013 


(For my personal use) 


tight skin over her hands with Raynaud's 
phenomenon. She has ulceration of the 
fingertips and associated small white deposits. 
She has noted increasing breathlessness over 
the past few years. Renal function is mildly 
impaired. 

What is the likely diagnosis? 

1- SLE 

2- Ataxia telangiectasia 

3- Polymyositis 

4- Limited cutaneous scleroderma 

5- Diffuse cutaneous scleroderma 


Answer & Comments 

Answer: 4- Limited cutaneous scleroderma 

Limited cutaneous scleroderma is also known 
as CREST syndrome (calcinosis, Raynaud's, 
(o)esophageal dysfunction, sclerodactyly, and 
telangiectasia). 'Limited' refers to the extent 
of skin involvement limited to the forearms 
and face. They generally develop pulmonary 
hypertension rather than pulmonary fibrosis, 
leading tow ards breathlessness. 



Tight thickened skin (sclerodactyly) seen in 

Scleroderma 


[ Q: 166 ] MRCPass - Rheumatology 

A 70 year old woman complains of 
pain at the base of her right thumb. There is 
tenderness and swelling of right first 
carpometacarpal joint. 

What is the likely diagnosis? 



1- De Quervain's tenosynovitis 

2- Rheumatoid nodule 

3- Osteoarthritis 

4- Psoriatic arthropathy 

5- Reiter's syndrome 


Answer & Comments 

Answer: 3- Osteoarthritis 

Osteoarthritis of the 1st carpometacarpal joint 
is common. Swelling is usually bony hard due 
osteophyte formation. PIP joint nodes are 
known as Bouchard's and DIP joint nodes are 
known as Heberden's nodes. 


v 


Arthritis of 
the thumb 

(carpal-metacarpal joi 



[ Q: 167 ] MRCPass - Rheumatology 

* A 70 year old patient has a set of 
investigations due to lethargy. She complains 
of polyuria and generalised back pains. Upon 
investigation, she has the following results: 

Hb 8 g/dl 

MCV 100 fl 

Platelets 190 x 10 9 /L 

total protein 90 g/l (60-76) gm% 

Albumin 35 (37-49) g/l 

calcium 2.9 (2.25-2.7) mmol/l 

phosphate 0.75 (0.8-8) pmol/l 

What is the appropriate next investigation? 

1- Complete liver function tests 

2- Urinary albumin 


Dr. Khalid Yusuf El-Zohry - Sohag Teaching Hospital (01118391123) 

Ref MRCPass OE OE 2012 PasTest 2009 PassMedicine 2009 PasTest Exam 




ReviseMRCP 

101 






















El-zohry MRCP Questions Bank (Port 1) - 2013 


(For my personal use) 


3- Plasma electrophoresis 

4- 24 hour urine protein 

5- Uric acid 


Answer & Comments 

Answer: 3- Plasma electrophoresis 

The patient has multiple myeloma as indicated 
by hypercalcaemia, polyuria and bone pains. 


[ Q: 168 ] MRCPass - Rheumatology 

A 72 year old lady has severe pain in 
the left knee and right hand. An X ray of the 
hand shows osteophytes and an X ray of the 
knee was normal. On examination, she has 
limited left hip flexion. 

Whot should be the next investigation for the 
knee? 

1- MRI of the knee 



echocardiography shows complete heart 
block. 


Which one of the following maternal 
autoantibodies is likely to be present? 


1- Ant 

2- Ant 

3- Ant 

4- Ant 


5- Ant 


Ro (SSA) 
dsDNA 
La (SSB) 
Jo 1 


centromere 


Answer & Comments 

Answer: 1- Anti Ro (SSA) 

AntiRo antibody is associated with congenital 
complete heart block. When congenital 
complete heart block occurs, SS-A antibodies 
are almost alw ays present in maternal and 
fetal serum (maternal anti-Ro(SS-A) antibody 
crosses the placenta). 


2- Bone scan 

3- Arthroscopy of the knee 

4- CT scan of the knee 

5- Ultrasound of the knee 


Answer & Comments 

Answer: 1- MRI of the knee 

The patient's presentation suggests 
osteoarthritis, but an MRI would be helpful to 
exclude other pathology as well as confirm 
osteoarthritis in the context of a normal knee 
X ray. The MRI is useful for assessing the state 
of the cruciate ligaments and the joint 
cartilage as well as avascular necrosis. 
Arthroscopy is helpful but invasive in this 
instance. 


^ [ Q: 169 ] MRCPass - Rheumatology 

/ - 

# A 28 year old patient with systemic 

lupus erythematosus attends the obstetric 
clinic at 25 weeks into her pregnancy. The 
foetal heart rate is 45 beats per minute. Foetal 


[ Q: 170 ] MRCPass - Rheumatology 

A 34 year old lady with systemic 
sclerosis complained of lethargy. Her blood 
pressure was 185/90 mmHg. 

Fundoscopy showed cotton wool spots. 

Investigations showed that her U+Es were: 

sodium 135 mmol/I 

potassium 4.5 mmol/I 

urea 12 mmol/l 

creatinine 225 pmol/l 

What is the treatment of choice for this 
patient? 

1- Oral Captopril 

2- IV Prostacyclin 

3- IV Labetalol 

4- IV Sodium nitroprusside 

5- Oral Atenolol 



Dr. Khalid Yusuf El-Zohry - Sohag Teaching Hospital (01118391123) 

Ref MRCPass OE OE 2012 PasTest 2009 PassMedicine 2009 PasTest Exam ReviseMRCP 



102 





























El-zohry MRCP Questions Bank (Port 1) - 2013 


(For my personal use) 


Answer & Comments 


Answer: 1- Oral Captopril 


Answer & Comments 


Answer: 2- Psoriasis 


A major complication of scleroderma is renal 
crisis. This is characterised by abrupt onset of 
severe hypertension, usually retinopathy, 
together with rapid deterioration of renal 
function and heart failure. 

Hypertension should be treated with an ACE 
inhibitor. This is because the underlying 
pathology causing hypertension is angiotensin 
ll-induced vasoconstriction, and trials have 
shown ACE -inhibitors to have the best 
antihypertensive efficacy and improved 
survival. 

The aim is to reduce pressure gradually, as an 
abrupt fall can lead to cerebral ischemia 
/infarctions (as in any accelerated 
hypertension), and may cause decreased renal 
perfusion as well as acute tubular necrosis. 
Calcium channel blockers may be added to 
ACE inhibitors. Parenteral antihypertensive 
agents (such as intravenous nitroprusside or 
labetalol) should be avoided as they cause 
abrupt blood pressure drops. 


[ Q: 171 ] MRCPass - Rheumatology 

A 45 year woman presents with an 8 
month history of joint pains and stiffness of 
the hands and feet. 

Examination reveals a synovitis of the distal 
interphalangeal joints, left index finger, right 
wrist and ankle joints. 

Nail pitting was noticed. Her ESR was 20 
mm/hr. 



The diagnosis is psoriatic arthritis. Psoriatic 
arthritis is subclassified according to different 
patterns of arthritis: asymmetrical 

oligoarthritis, symmetric polyarthritis, 
spondyloarthropathy and arthritis mutilans. In 
about 20% of patients there is a chronic, 
progressive, deforming arthropathy in an 
asymmetrical pattern, including distal 
interphalangeal joint involvement. 



Psoriatic Arthropathy - note onycholysis on 

the nails 


^ [ Q: 172 ] MRCPass - Rheumatology 

mi 

* A 16 year old girl presents with 
widespread palpable purpura over legs and 
buttocks. She has abdominal pains and is 
noted to have blood and protein on urine 
dipstick. 

What is the most likely diagnosis? 

1- Juvenile dermatomyositis 

2- Juvenile chronic arthritis 

3- Henoch Schonlein purpura 

4- Rheumatoid arthritis 


Which one of following conditions is 
associated with the pattern of joint 
involvement? 

1- SLE 

2- Psoriasis 

3- Rheumatoid arthritis 

4- Septic arthritis 

5- Reactive arthritis 


5- HUS-TTP 

Answer & Comments 

Answer: 3- Henoch Schonlein purpura 

Henoch-Schonlein purpura (HSP) is a systemic 
vasculitis mostly seen in children. It is a 
multisystem disorder involving the skin, joints, 
gastrointestinal and renal tracts. 


Dr. Khalid Yusuf El-Zohry - Sohag Teaching Hospital (01118391123) 

Ref MRCPass OE OE 2012 PasTest 2009 PassMedicine 2009 PasTest Exam 




ReviseMRCP 

103 




























El-zohry MRCP Questions Bank (Port 1) - 2013 


(For my personal use) 


Aetiology is unknown, but the syndrome is 
often preceded by infections such as Group A 
beta hemolytic streptococcal respiratory tract 
infection, Campylobacter jejuni, Mycoplasma 
pneumoniae and viruses such as varicella, 
hepatitis B, Epstein-Barr virus, and parvovirus 
B19. 

Pathology is due to intravascular deposition of 
IgA immune complexes with activation of 
complement and leucocyte infiltration. 
Patients often present with a purpuric rash 
usually involving the buttocks and lower limbs, 
arthralgia and joint swelling, severe colicky 
abdominal pain and tenderness caused by 
vasculitis-induced thrombosis in the gut. 

Renal involvement commonly presents as 
microscopic haematuria and proteinaemia. 
The most serious long-term 


nj 

m* 


complication from HSP is progressive renal 

failure. 


[ Q: 173 ] MRCPass - Rheumatology 


5- Ankylosing spondylitis 

Answer & Comments 

Answer: 5- Ankylosing spondylitis 

Ankylosing spondylitis is a seronegative 
arthropathy associated with HLAB27 
genotype. It leads to increased stiffness and 
fusion of the vertebrae causing a bamboo 
spine. Typically, the sacroiliac joints and hip 
joints are affected, but thoracic and cervical 
spines are eventually affected as well. 

There is a list of associations mostly starting 
with A. These are anterior uveitis/iritis, aortic 
regurgitation, aortic aneurysm, apical fibrosis, 
aspergillosis infection, amyloidosis and 
archilles tendinitis. 

In bamboo spine, there is fusion of the 
vertebral bodies and squaring of the 
vertebrae. One also expects syndesmophytes 
(thin vertical dense spicules bridging the 
vertebral bodies), Romano lesions (erosion at 
the corner of vertebral bodies) and 
enthesopathy (ligament or tendon 
calcification). 



fl 

# A 45 year old man has difficulty 
bending touching his toes w hen bending over. 
He has lower back pain, stiffness in the 
thoracic region and reduced chest expansion. 
His lower spine X-ray is shown above. He has 
negative rheumatoid factor, and a HLA-B27 
genotype. 

What is the diagnosis? 


Bamboo Spine 


1- Paget's disease 

2- Osteoarthritis 

3- Marble bone disease 

4- Marfan's syndrome 


[ Q: 174 ] MRCPass - Rheumatology 

A 35 year old woman presents with 
red scaly plaques on her cheeks and her 
forehead. On closer examination, there was 




Dr. Khalid Yusuf El-Zohry - Sohag Teaching Hospital (01118391123) 

Ref MRCPass OE OE 2012 PasTest 2009 PassMedicine 2009 PasTest Exam ReviseMRCP 

























El-zohry MRCP Questions Bank (Port 1) - 2013 


(For my personal use) 


plugging of some hair follicles and atrophy of 
the skin. 

Whot is the likely diagnosis? 

1- Drug induced lupus 

2- Psoriasis 

3- Discoid lupus 

4- Sarcoidosis 

5- Lupus pernio 


Answer & Comments 

Answer: 3- Discoid lupus 

The diagnosis is discoid lupus erythematosus. 
Lesions are discrete plaques, often 
erythematous, scaly, with extension into hair 
follicles. These lesions can occur on the face, 
scalp, in the pinnae, behind the ears or on the 
neck. There can also be active indurated 
erythema and central atrophic scarring. 



Discoid lupus 


Answer & Comments 

Answer: 3- Infliximab 

The Anti-TNF drugs are infliximab and 
adalimumab. The criteria for treatment with 
these drugs are that patients who have been 
treated with at least tw o DMARDs (disease¬ 
modifying drugs) continue to have active 
rheumatoid arthritis. 

Examples of the DMARDs are: gold injections, 
sulphasalazine, hydroxychloroquine, 

leflunomide, cyclosporin, azathioprine and 
methotrexate [One of the tw o must be 
methotrexate]. 



[ Q: 176 ] MRCPass - Rheumatology 

A 45 year old woman presents with a 
year's history of Raynaud's phenomenon, 
dyspepsia and joint pains. On examination, 
she has sclerodactyly and synovitis of small 
joints of her hands. Her ESR is 60 mm/hr (<10) 
but Rheumatoid factor and Antinuclear 
Antibody are both negative. 


What other clinical feature is likely in this 
patient? 

1- Splinter haemorrhages 

2- Erythema marginatum 


3- Butterfly rash 


4- Anterior uveitis 


[ Q: 175 ] MRCPass - Rheumatology 

A 50 year old patient with active 
rheumatoid arthritis has failed treatment with 
gold, methotrexate, hydroxychloroquine and 
sulphasalasine. She has ongoing joint pains 
and erosive damage. 

What is the most appropriate treatment ? 

1- High dose prednisolone 

2- COX2 inhibitors 

3- Infliximab 

4- Ciclosporin 

5- Azathioprine 



5- Small bowel hypomotility 


Answer & Comments 

Answer: 5- Small bowel hypomotility 

This woman features of a mixed connective 
tissue disorder such as CREST/systemic 
sclerosis, sclerodactyly, Raynaud's, dyspepsia 
and arthralgia. The other likely development 
would be malabsorption which is associated 
with hypomotility of the small bow el. 


Dr. Khalid Yusuf El-Zohry - Sohag Teaching Hospital (01118391123) 

Ref MRCPass OE OE 2012 PasTest 2009 PassMedicine 2009 PasTest Exam 




ReviseMRCP 

105 

























El-zohry MRCP Questions Bank (Port 1) - 2013 


(For my personal use) 


^ [ Q: 177 ] MRCPass - Rheumatology 

fi - 

# A 60 year old man is on treatment 
for chronic heart failure with diuretics. He has 
preivous history of athma. He presents with 
sudden onset of pain and swelling of the 
metatarso-phalangeal joint of his right big toe. 
Aspiration of the joint demonstrates crystals 
of monosodium urate. 

What is the recommended treatment? 

1- Aspirin 

2- Colchicine 

3- Non-steroidal anti-inflammatory drugs 

4- Cyclooxygenase 2 inhibitor 

5- Allopurinol 

Answer & Comments 

Answer: 2- Colchicine 


progress to other parts. It is due to autonomic 
nervous system dysfuction. Symptoms of 
extreme pain and burning can occur. 
Analgesics are often unhelpful. 



Flushing, shiny and atrophied skin on the left 
leg indicating reflex sympathetic dystrophy 


In this particular patient, colchicine is the best 
option. In acute gout, either colchicine or 
NSAIDs can be used. How ever, asthma 
contraindicates NSAIDs. 


[ Q: 178 ] MRCPass - Rheumatology 

A 35 year old man is renovating his 
apartment w hen he slams a door against his 
foot. He develops swelling, erythema and pain 
in all the digits of his foot. He has an ESR of 
20mm/hour and a temperature of 36°C. 

What is the likely diagnosis? 

1- Porphyria 

2- Cellulitis 

3- Gout 

4- Raynaud's phenomenon 

5- Reflex sympathetic dystrophy 



Answer & Comments 

Answer: 5- Reflex sympathetic dystrophy 

Reflex sympathetic dystrophy occurs following 
trauma to an injured part of the body, and can 


^ [ Q: 179 ] MRCPass - Rheumatology 

/ - 

m A 42 year old lady has tightening of 

the skin around her hands and mouth. She has 
several telangiectasia on her hands, and 
complains of severe cold hands in winter. She 
was noticed to be pale. 

Investigations show : 

Hb 4.5 g/dl 

MCV 105 fl 

WCC 6 x 10 9 /L 

platelets 230 x 10 9 /L 

Iron 22 (14-29) pmol/l 

Ferritin 155 (15-200) pmol/l 

total iron binding capacity 50 (45-72) umol/l 

Folate 11 (3-20) ?g/l 


Vitamin Bi 2 - 85 (120-700 pmol/l) 
What is the likely cause of anaemia? 

1- Pernicious anaemia 

2- Secondary folate deficiency 

3- Celiac disease 


Dr. Khalid Yusuf El-Zohry - Sohag Teaching Hospital (01118391123) 

Ref MRCPass OE OE 2012 PasTest 2009 PassMedicine 2009 PasTest Exam ReviseMRCP 



106 


























El-zohry MRCP Questions Bank (Port 1) - 2013 


(For my personal use) 


4- Methotrexate 

5- Bacterial overgrowth 


Answer & Comments 

Answer: 5- Bacterial overgrowth 

Scleroderma can cause folate deficiency due 
to malabsorption. However, in this case, the 
folate levels are normal and there is B12 
deficiency. 

B12 deficiency can occur in conditions where 
there is bacteria overgrowth in the small 
intestine (blind loop syndromes) such as 
jejunal diverticulosis, Crohns disease, fistulas 
and scleroderma. The anaerobic organisms 
metabolise vitamin Bn and impair absorption. 
When systemic sclerosis (SSc) involves the 
small intestine, normal peristaltic movements 
are lost and motility is impaired leading to 
stasis and dilatation. 



[ Q: 180 ] MRCPass - Rheumatology 

A 55 year old man develops sudden 
onset severe pain in his right big toe. 


On examination he has swelling of the 
metacarpophalangeal joint of his right hallux. 
The surround skin is erythematous. It is tender 
to touch. Initial investigations reveal a raised 
white cell, count and an elevated CRP. 


What should be done to confirm the 
diagnosis? 

1- MRI of the toe 


2- Serum uric acid 

3- Serum rheumatoid 

4- Joint fluid aspirate for microscopy 

5- Serum pyrophosphate levels 


Answer & Comments 

Answer: 4- Joint fluid aspirate for microscopy 

The likely diagnosis is acute gout. The serum 
inflammatory markers are raised, but uric acid 
levels may be normal. 


Joint fluid aspirate best test. Polarised light 
microscopy shows strongly birefringent 
(negative sign) needle-shaped crystals. 



Acute Gout involving the left foot 


[ Q: 181 ] MRCPass - Rheumatology 

A 80 year old man developed acute 
monoarthritis of his right ankle following an 
admission with congestive cardiac failure. He 
mentioned that he had ankle oedema and had 
recently been prescribed frusemide by the GP. 

What is the likely diagnosis? 

1- Rheumatoid arthritis 

2- Gout 

3- Pseudogout 

4- Osteoarthritis 

5- Septic arthritis 



Answer & Comments 

Answer: 2- Gout 

Gout can be precipitated by diuretics, e.g. 
frusemide. 


[ Q: 182 ] MRCPass - Rheumatology 

A 45 year old has a past history of 
systemic sclerosis. She now has headaches 
and blurred vision. On examination, she has a 
blood pressure of 220/100 mmHg and there is 
evidence of bilateral papilloedema. 

Which of the following medications should be 
prescribed? 



Dr. Khalid Yusuf El-Zohry - Sohag Teaching Hospital (01118391123) 

Ref MRCPass OE OE 2012 PasTest 2009 PassMedicine 2009 PasTest Exam 




ReviseMRCP 

107 





























El-zohry MRCP Questions Bank (Port 1) - 2013 


(For my personal use) 


1- Oral hydrochlorothiazide 

2- Oral Lisinopril 

3- Sublingual Nimodipine 

4- IV Sodium Nitroprusside 

5- IV Labetolol 


Answer & Comments 

Answer: 2- Oral Lisinopril 

A major complication of scleroderma is renal 
crisis which is characterised by abrupt onset of 
severe hypertension. The hypertension almost 
alw ays is severe with a diastolic BP over 100 
mmHg in 90% of patients. There is associated 
hypertensive retinopathy in about 85% of 
patients. ACE inhibitors are first line, with an 
aim to reduce the blood pressure gradually. 

[ Q: 183 ] MRCPass - Rheumatology 

A 50 year old man presents an acute 
monoarthritis of left knee. Gout is confirmed 
following joint aspiration and examination of 
fluid under polarised light microscopy. He had 
also underwent endoscopy 3 weeks earlier 
because of indigestion this confirmed a 
duodenal ulcer. 

Which one of the following is the best 
treatment for the patient? 

1- Allopurinol 

2- Intraarticular corticosteroid injection 

3- Indomethacin and Lansoprazole 

4- Celecoxib and Lansoprazole 

5- Indomethacin and Misoprostol 



Answer & Comments 

Answer: 2- Intraarticular corticosteroid 

injection 

All non-steroidals including Cox II selective 
inhibitors are relatively contraindicated in the 
presence of active ulceration. In a large joint 


such as the knee, the safest option would be 
inject corticosteroid into the joint. 

[ Q: 184 ] MRCPass - Rheumatology 

A 62 year old man has a 5 week 
history of pain and swelling affecting left knee, 
both ankles and his right wrist. 

He has lost 6 kg in weight. 

His investigations show : 

WCC 14.1 x 10 9 /L 

Hb 10.3 x 10 9 /L 

MCV 72 fl 

pit 510 x 10 9 /L 

ESR 63 mm in the first hour 

CRP 21 g/l 

CK 120 iu 

Rh F1/80 

ANA negative 

ENA negative 

XR of hands and feet normal. 

What is the most likely diagnosis? 

1- Paraneoplastic syndrome 

2- Osteoarthritis 

3- Polymyositis 

4- Scleroderma 

5- Behcet's syndrome 



Answer & Comments 


Answer: 1- Paraneoplastic syndrome 


The CK is not significantly raised. Weight loss 
and anaemia suggest underlying malignancy. 
Paraneoplastic syndrome can present with an 
assymetrical arthralgia which more commonly 
affects the lower limbs. False positive 
rheumatoid factor can occur but should be of 
low titre. 



[ Q: 185 ] MRCPass - Rheumatology 


Dr. Khalid Yusuf El-Zohry - Sohag Teaching Hospital (01118391123) 

Ref MRCPass OE OE 2012 PasTest 2009 PassMedicine 2009 PasTest Exam ReviseMRCP 






























El-zohry MRCP Questions Bank (Port 1) - 2013 


(For my personal use) 


A 45 year old lady has had long standing 
arthritis of her hands. Her has hand X-rays 
done. 

Which of the following X ray changes suggests 
rheumatoid arthritis instead of a seronegative 
arthropathy? 

1- Osteosclerosis 

2- Osteophytes 

3- Osteoporotic changes 

4- Periarticular erosions 

5- Loss of joint space 


Answer & Comments 

Answer: 4- Periarticular erosions 

Osteophytes and loss of joint space are 
commonly found in osteoarthritis, although 
they can also be found in rheumatoid arthritis. 
Periarticular erosions are most suggestive of 
rheumatoid arthritis. 



[ Q: 186 ] MRCPass - Rheumatology 

A 76 year old woman presents with 
weakness of his hand. On examination there 
was tenderness, crepitus and bony swellings 
over the base of the first metacarpal and 
wasting of the right thenar eminence. 

Investigations reveal an ESR of 25 mm/lst hr, 
a CRP of lOmg/L, a Urate concentration of 
0.42 (0.19-0.36). Her Rheumatoid factor was 



60 IU/L (<30). An xray of the right hand 
showed a loss of joint space, periarticular 
sclerosis and osteophytes of the first 
carpometacarpal joint. 

What is the likely diagnosis? 

1- Osteoarthritis 

2- DeQuervain's tenosynovitis 

3- Pseudogout 

4- Gout 

5- Rheumatoid arthritis 


Answer & Comments 

Answer: 1- Osteoarthritis 

The patient has clinical and radiological 
features consistent with osteoarthritis (OA) of 
the 1st right carpometacarpal (CMC) joint. The 
condition is characterised by joint pain, 
crepitus and stiffness after movement. Joint 
swellings are bony in nature (Bouchard's and 
Heberden's nodes), unlike boggy swellings 
which occurs in inflammatory arthritis. This 
patient's inflammatory markers are mildly 
raised only, making an inflammatory arthritis 
unlikely. 


[ Q: 187 ] MRCPass - Rheumatology 

A 70 year old man presents with 
severe back pains. An His total serum protein 
is 85 g/l with an albumin of 41 g/l. 

A chest X ray shows several lucencies in 
vertebral bodies. A sternal bone marrow 
aspirate obtains a dark red jellylike material in 
the syringe. 

The smear of aspirate is likely to show which 
prominent features ? 

1- Macrophages 

2- Osteoblasts 

3- Plasma cells 

4- Pneumocytes 

5- Osteosarcoma 



Dr. Khalid Yusuf El-Zohry - Sohag Teaching Hospital (01118391123) 

Ref MRCPass OE OE 2012 PasTest 2009 PassMedicine 2009 PasTest Exam ReviseMRCP 



109 


























El-zohry MRCP Questions Bank (Port 1) - 2013 


(For my personal use) 


Answer & Comments 

Answer: 3- Plasma cells 

The diagnosis is multiple myeloma. The bone 
marrow needle is likely to be in a lytic lesion 
filled with plasma cells. His serum globulins 
are high from the monoclonal gammopathy. 


^ [ Q: 189 ] MRCPass - Rheumatology 

fl - 

# A 25 year old Turkish man was noted 
by ophthalmologists to have a posterior 
uveitis requiring high dose 

immunosuppression. He has a history of 
recurrent mouth ulcers and painful ulcers on 
the scrotum. 



A plasma cell 


Whot is the likely diagnosis? 

1- Behcet's syndrome 

2- Reiter's syndrome 

3- Ankylosing spondylitis 

4- Takayasu's arteritis 

5- Giant cell arteritis 

Answer & Comments 

Answer: 1- Behcet's syndrome 


[ Q: 188 ] MRCPass - Rheumatology 


A 75 year old man presents with 
bilateral hip pains. Investigations reveal: 

Corrected calcium 2.5 (2.2 - 2.6) mmol/I 

ESR 22 mm/lst hr 


Alkaline phosphatase 800 iu/L (50 - 100) 

gammaGT 22 iu/L 

What is the likely diagnosis? 

1- Osteoporosis 

2- Osteomalacia 

3- Paget's disease of the pelvis 

4- Polymyalgia rheumatica 

5- Multiple myeloma 


Behcet's syndrome is classically characterized 
as a triad of symptoms that include recurring 
crops of mouth ulcers (called apthous ulcers), 
genital ulcers, and uveitis. The ulcers are 
usually painful. The disease is more frequent 
and severe in patients from the Eastern 
Mediterranean and Asia than those of 
European descent. 

Eye manifestations may result in blindness. In 
addition, iritis, retinal vessel occlusions and 
optic neuritis can be found. Hypopyon uveitis 
(pus in the anterior chamber of the eye), 
which is considered the hallmark of Behget's 
disease, is in fact a rare manifestation. The 
arthritis of Behget's disease is usually 
intermittent, self-limited, not deforming and 
localized to the knees and ankles. 


Answer & Comments 

Answer: 3- Paget's disease of the pelvis 

This elderly patient presenting with bone 
pains has significantly elevated alkaline 
phosphatase but normal calcium 
concentrations suggesting a diagnosis of 
Paget's. 




Dr. Khalid Yusuf El-Zohry - Sohag Teaching Hospital (01118391123) 

Ref MRCPass OE OE 2012 PasTest 2009 PassMedicine 2009 PasTest Exam ReviseMRCP 



























El-zohry MRCP Questions Bank (Port 1) - 2013 


(For my personal use) 


[ Q: 190 ] MRCPass - Rheumatology 

A 70 year old woman has polydipsia 
and polyuria for 3 months. She also complains 
of loin pains. She has not been on any 
medication. 

Investigations reveal: 

serum urea 6 (2.5-7.5) pmol/l 

serum creatinine 80 (60-110)pmol/l 

serum albumin 38 g/L (37-49) 

serum total calcium 3.1 (2.2-2.6) 

Whot is the likely couse of the 
hypercoicoemio? 

1- Multiple myeloma 

2- Sarcoidosis 

3- Paget's disease 

4- Primary hyperparathyroidism 

5- Vitamin D toxicity 



Answer & Comments 

Answer: 4- Primary hyperparathyroidism 

Primary hyperparathyroidism is caused by an 
overproduction of PTH. 

Excess PTH results in an increase in bone 
breakdown by means of osteoclastic 
resorption with subsequent fibrous 
replacement and reactive osteoblastic activity. 
Historically, in classic primary 
hyperparathyroidism, nephrolithiasis was 
noted in 50% of patients, and it was the most 
common clinical presentation of the disease. 
Additional manifestations of primary 
hyperparathyroidism include pancreatitis, 
peptic ulcer disease, and cardiovascular 
abnormalities. 

[ Q: 191 ] MRCPass - Rheumatology 

A 42 year old lady presents with 
backache. Her blood results are as follow s: 

Hb llg/dL 

Ca 1.9 mmol/L 



Phosphate 0.8 mmol/L 
Alkaline phosphatase 220 U/L 
Albumin 38 g/L 
Urea 7 mmol/L 
Sodium 142 mmol/L 
Potassium 3.9 mmol/L 
Whot is the diagnosis? 

1- Osteoporosis 

2- Paget's disease 

3- Osteomalacia 

4- X linked hypophosphataemic rickets 

5- Hypoparathyoroidism 

Answer & Comments 

Answer: 3- Osteomalacia 

The patient has osteomalacia with secondary 
hyperparathyroidism causing low phosphate 
levels. Osteomalacia is due to vitamin D 
deficiency which could be due to 
malabsorption or dietary defiency, or 
renal/liver disease. 



[ Q: 192 ] MRCPass - Rheumatology 

A 32 year old woman has a deep vein 
thrombosis. Her previous history included 
investigations for miscarriages. 


Investigations revealed: Haemoglobin 11.9 
g/dl, White cell count 4 x 10 9 /L, Platelet count 
30 x 10 9 /L. 


Which of these ore likely to be obnormol? 

1- Homocysteine level 


2- ANCA 


3- Protein C 

4- Antiphospholipid antibody 

5- Coomb's test 


Answer & Comments 


Answer: 4- Antiphospholipid antibody 


Dr. Kholid Yusuf El-Zohry - Sohog Teaching Hospital (01118391123) 

Ref MRCPass OE OE 2012 PasTest 2009 PassMedicine 2009 PasTest Exam 




ReviseMRCP 

in 



























El-zohry MRCP Questions Bank (Port 1) - 2013 


(For my personal use) 


Antiphospholipid syndrome leads to venous 
and arterial thrombosis, livedo reticularis, 
splinter hemorrhages, leg ulcer, multi-infarct 
dementia, chorea, Thrmobocytopenia (40% of 
patients), hemolytic anemia and late term 
miscarriages. 

[ Q: 193 ] MRCPass - Rheumatology 

A 35 year old man presents with 
acute stiffness swelling of his knees and 
ankles, a painful rash on his legs. The ESR was 
100 mm/hour. Chest Xray showed hilar 
lymphadenopathy bilaterally. 

What is the likely progression of the 
arthralgia? 

1- Chronic arthritis 

2- Septic arthritis 

3- Improvement only with steroids 

4- Spontaneous improvement 

5- Permanent joint destruction 



Answer & Comments 

Answer: 4- Spontaneous improvement 

The description typical of acute sarcoidosis - 
erythema nodosum, oligoarthropathy and 
hilar lymphadenopathy. This usually has a 
good prognosis, with resolution over 6-8 
weeks. 


[ Q: 194 ] MRCPass - Rheumatology 

A 75 year old lady presents with back 
pains. Lateral spine X-rays and pelvic x-rays 
show osteopenia. A serum corrected calcium 
is 1.8 mmol/l and phosphate is 0.6 mmol/I. 
Alkaline phosphatase is 360 U/l. 

Which diagnosis is most likely? 

1- Myeloma 

2- Osteoporosis 

3- Osteomalacia 

4- Paget's disease 



5- Ankylosing spondylitis 


Answer & Comments 

Answer: 3- Osteomalacia 

Osteomalacia is more likely than osteoporosis 
due to the low calcium, low phosphate and 
raised alkaline phosphatse. The condition is 
caused by low vitamin D levels. 

Alkaline phosphatase is raised w hen there is 
increased osteoblastic activity, conditions it 
may be raised in: 

Paget's disease 

osteomalacia and rickets 

renal osteodystrophy 

bone metastases 



[ Q: 195 ] MRCPass - Rheumatology 

A 22 year old lady has a sw inging 
fever, half a stone weight loss over 2 months, 
generalised myalgia, polyarthralgia affecting 
wrists, knees, ankles, elbows 
metacarpophalangeal joints and a sore throat. 


Investigations show : Hb 9.5g/l, MCV 85 fl, ESR 
92 mm in first hour, CRP 45 g/l, serum ferritin 
1600 mg/dl, RF negative, ANA negative, ENA 
negative, ASO titre <200iu. 


What is the likely diagnosis? 


1- Rheumatoid arthritis 


2- Ankylosing spondylitis 

3- Adult onset Still's disease 


4- Inclusion body myositis 

5- Polymyositis 


Answer & Comments 

Answer: 3- Adult onset Still's disease 

Adult Onset Still's Disease (AOSD) is an acute 
febrile illness in young adults. It usually affects 
multiple organs, but is a diagnosis of 
exclusion. 


Dr. Khalid Yusuf El-Zohry - Sohag Teaching Hospital (01118391123) 

Ref MRCPass OE OE 2012 PasTest 2009 PassMedicine 2009 PasTest Exam ReviseMRCP 





























El-zohry MRCP Questions Bank (Port 1) - 2013 


(For my personal use) 


Clinical features include a high fever, 
arthralgia and arthritis, phayngitis, typical rash 
(evanescent salmon-colored, macular or 
maculopapular eruption), lymphadenopathy, 
and serositis. Chronic arthritis and 
constitutional symptoms are common. 

The triad of fever, rash, and arthralgia are 
often absent during the first month of the 
illness. The usual joints affected are wrists, 
knees, and ankles in descending order. 

Tw o thirds of cases experience polyarticular 
arthritis and one third have monoarticular 
symptoms. 

Approximately 1/3 of patients have chronic 
persistent disease with progressive joint 
damage. 


[ Q: 196 ] MRCPass - Rheumatology 

A 65 year old woman has a swollen, 
erythematous knee joint. Aspiration of the 
joint was performed. Microscopy showed 
positively birefringent crystals. 



^ [ Q: 197 ] MRCPass - Rheumatology 

tl - 

* A 45 year old woman presents with 
claudication in her lower limbs. She is noted to 
have absent left arm pulses. 

The previous year she had a small hemispheric 
cerebrovascular infarct. 

What is the likely diagnosis? 

1- Giant cell arteritis 

2- Takayasu's arteritis 

3- Familial hypercholesterolaemia 

4- Coarctation of the aorta 

5- Anti phospholipid syndrome 


Answer & Comments 

Answer: 2- Takayasu's arteritis 

Takayasu's arteritis is a large vessel vasculitis 
of unknown origin. The vasculitic process 
involves structures such as the aorta, great 
vessels, the sclera and the cardiac conduction 
tissues. 


What are the crystals composed of? 

1- Calcium carbonate 

2- Magnesium sulphate 

3- Urate 

4- Calcium pyrophosphate 

5- Aminolaevulinic acid 


Women are affected more than men, usually 
in the second and third decades of life. 
Presentation is often with symptoms such as 
fever, weight loss, night sweats and 
arthralgias. Symptoms related to ischaemia 
may include ischaemic stroke, visual 
disturbances and claudication. 


Answer & Comments 

Answer: 4- Calcium pyrophosphate 

Pseudogout is caused by the deposition of 
calcium pyrophosphate deposition rather than 
the deposition of uric acid derivatives that 
cause gout. Also, in pseudogout, synovial fluid 
samples obtained with aspiration have 
positive birefringence. This finding is in direct 
contrast to the negative birefringence in gout. 



[ Q: 198 ] MRCPass - Rheumatology 

A 70 year old man develops 
weakness of the shoulders and around the 
hips over a 6 month period. He has also 
noticed weak finger flexors. He complained of 
difficulty sw allow ing liquids. There is no 
other significant past medical history. He 
smokes 15 cigarettes a day and drinks a bottle 
of w ine at the weekend. A creatinine kinase 
level comes back at 120 U/l. 


A muscle biopsy sample shows myopathic 
changes. There are also inflammatory 


Dr. Khalid Yusuf El-Zohry - Sohag Teaching Hospital (01118391123) 

Ref MRCPass OE OE 2012 PasTest 2009 PassMedicine 2009 PasTest Exam ReviseMRCP 



113 





























El-zohry MRCP Questions Bank (Port 1) - 2013 


(For my personal use) 


infiltrates and intracytoplasmic vacuoles 
present. 

What is the likely diagnosis? 

1- Polymyositis 

2- Fibromyalgia 

3- Polymyalgia rheumatica 

4- Dermatomyositis 

5- Inclusion body myositis 


Answer & Comments 

Answer: 5- Inclusion body myositis 

The diagnosis is inclusion body myositis (IBM). 
This is an inflammatory condition affecting 
patients over the age of 50. Proximal muscles 
in the upper and lower limbs, and finger 
flexors are predominantly involved. The onset 
of muscle weakness in IBM is generally 
gradual (over months or years).Dysphagia is 
common, occurring in 40-66% of patients. 

IBM occurs more frequently in men than w 
omen. CK may be normal. A muscle biopsy 
usually shows intracytoplasmic inclusions and 
also inflammatory infiltrates. 



Thigh atrophy seen in inclusion body myositis 



Inclusion bodies and inflammatory infiltrates 

(arrows) 


[ Q: 199 ] MRCPass - Rheumatology 

A 53 year old woman with 
rheumatoid arthritis was referred with iron 
deficiency anaemia. She had extensive 
investigations. Endoscopy showed gastritis 
and antral erosions, small bow el biopsy 
showing mild villous blunting, and apopotic 
bodies, Colonoscopy was normal. 

What is the cause of these findings? 

1- Methotrexate 

2- Azathioprine therapy 

3- Crohn's disease 

4- NSAID therapy 

5- Felty's syndrome 

Answer & Comments 

Answer: 4- NSAID therapy 

The endoscopy findings are suggestive of 
NSAID related gastritis, which is the most 
likely cause of iron deficiency anaemia. 



NSAID related gastritis 




Dr. Khalid Yusuf El-Zohry - Sohag Teaching Hospital (01118391123) 

Ref MRCPass OE OE 2012 PasTest 2009 PassMedicine 2009 PasTest Exam ReviseMRCP 


114 























El-zohry MRCP Questions Bank (Port 1) - 2013 


(For my personal use) 


[ Q: 200 ] MRCPass - Rheumatology 

A 45 year old woman complains of 
12 months of mid and lower back pain 
associated with stiffness in her shoulders, 
wrists, small joints of the hands, hips knees 
and feet. The joint stiffness is more 
pronounced in the morning and lasts 2-3 
hours and is associated with severe fatigue. 

She also experiences occasional pins and 
needles affecting all of the right hand and 
frequent frontal and temporal headache. 

Clinical examination demonstrates no 
synovitis but tenderness around the base of 
the cervical spine, across the shoulders, over 
the costochondral cartilages, greater 
trochanter and the knee. 

Investigations are as follow s : CRP <5 g/l, ESR 
20 mm in the first hour, CK 120 U/l, serum 
immunoglobulins and protein electrophoresis 
normal. 

Whot is the diagnosis? 

1- SLE 

2- Polymyalgia rheumatica 

3- Fibromyalgia 

4- Dermatomyositis 

5- Polymyositis 



Answer & Comments 


Answer: 3- Fibromyalgia 


1- Staphyloccocus aureus 

2- Neisseria gonorrhoeae 

3- (3-haemolytic streptococcus 

4- Pseudomonas aeruginosa 

5- Moraxella catarrhalis 


Answer & Comments 

Answer: 1- Staphyloccocus aureus 

Staph aureus is the commonest organism 
(over 50%). Neisseria gonorrhea can occur in 
patients with sexually transmitted disease. (3- 
haemolytic streptococci can cause impetigo, 
sore throat and rheumatic fever. Moraxella is 
a gram negative coccus which can cause 
pneumonia in COPD patients. 



[ Q: 202 ] MRCPass - Rheumatology 

A 66 year old lady had a fall and 
fractured femur. A DEXA scan shows reduced 
bone mineral density consistent with 
osteoporosis. 


Whot treatment should be given to reduce the 
likelihood of future fractures? 


1- Alendronate 


2- Alendronate and calcium 


3- Alendronate, calcium and vitamin D 

4- Calcium 


5- Vitamin D 


The above symptoms are all characteristic for 
fibromyalgia. The inflammatory markers and 
CK are not raised, making the other diagnoses 
unlikely 


Answer & Comments 

Answer: 3- Alendronate, calcium and vitamin 
D 


[ Q: 201 ] MRCPass - Rheumatology 

A 35 year old man presents with hot 
swollen elbow joint. He has a temperature of 
37°C and CRP of 350mg/l. A diagnosis of septic 
arthritis is suspected. 

Which of the following organisms is most 
commonly isolated from joints? 



Any patient above the age of 65 with 
osteoporosis is recommended to be on 
bisphosphonates. In addition, this patient has 
sustained a fracture, and should also be on 
calcium and vitamin D (calcichew D3) as well. 



[ Q: 203 ] MRCPass - Rheumatology 
A 82 year old woman presents with 


Dr. Khalid Yusuf El-Zohry - Sohag Teaching Hospital (01118391123) 

Ref MRCPass OE OE 2012 PasTest 2009 PassMedicine 2009 PasTest Exam ReviseMRCP 



115 





































El-zohry MRCP Questions Bank (Port 1) - 2013 


(For my personal use) 


confusion. Of note in her past history was 
Raynaud's phenomenon. 

Her investigations show : 

haemoglobin 9.7 g/dl 

white cell count 3.6 x 10 9 /L 

platelet count 100 x 10 9 /l_ 

serum total protein 120 g/l 

serum immunoglobulins : IgA 0.75 g/l (0.83), 
IgG 16 g/l (6-13), IgM 35 g/l (0.4-2.5) 

Which of the following complications is she 
likely develop? 

1- Urinary tract infection 

2- Hyperviscosity syndrome 

3- Pathological bone fracture 

4- Acute renal failure 

5- Erythema nodosum 

Answer & Comments 

Answer: 2- Hyperviscosity syndrome 

The likely diagnosis is Waldenstrom's 
Macroglobulinaemia (WM). 

Waldenstrom's macroglobulinaemia is a type 
of non-Hodgkin's lymphoma. It is a condition 
which typically presents in the seventh and 
eighth decade of life. It is characterized by the 
presence of a high level of a macroglobulin 
immunoglobulin M [IgM] and elevated serum 
viscosity in the presence of a 
lymphoplasmacytic infiltrate in the bone 
marrow . The treatment is chemotherapy 
(Chlorambucil or Fludarabine). 


2- De Quervain's tenosynovitis 

3- Osteoarthritis 

4- Psoriatic arthritis 

5- Rheumatoid arthritis 

Answer & Comments 

Answer: 3- Osteoarthritis 

The tenderness at the carpometacarpal joint is 
most likely due to osteoarthritis in a patient of 
this age. 

^ [ Q: 205 ] MRCPass - Rheumatology 

# A 45 year old man presents to the 
opthalmologist and was found to have 
anterior uveitis. Upon enquiry, he has mouth 
ulcers and a rash on his leg. 

Whot is the likely diagnosis? 

1- AL amyloidosis 

2- Familial mediterranean fever 

3- Behcet's disease 

4- Polymyalgia rheumatica 

5- SLE 

Answer & Comments 

Answer: 3- Behcet's disease 

Behget's disease may cause anterior or 
posterior uveitis, corneal ulceration, 
conjunctivitis, papillitis, SVC and IVC 
thrombosis, pulmonary embolism, erythema 
nodosum, pustular lesions, acneiform nodules, 
pathergy and oral ulceration. 


[ Q: 204 ] MRCPass - Rheumatology 

A 72 year old woman complained of 
pain at the base of her right thumb. There was 
tenderness and swelling of the right first 
carpo-metacarpal joint. Finkelstein's test is 
negative. 

What is the most likely diagnosis? 

1- Avascular necrosis of the scaphoid 





[ Q: 206 ] MRCPass - Rheumatology 


Dr. Khalid Yusuf El-Zohry - Sohag Teaching Hospital (01118391123) 

Ref MRCPass OE OE 2012 PasTest 2009 PassMedicine 2009 PasTest Exam ReviseMRCP 



116 






























El-zohry MRCP Questions Bank (Port 1) - 2013 


(For my personal use) 


In a patient with SLE, the risk of cardiovosculor 
abnormality is increased with presence of 
which one of the following? 

1- Anticardiolipin antibody 

2- Anti- Ro antibody 

3- Anti La antibody 

4- ANA antibody 

5- Anti Jo antibody 

Answer & Comments 

Answer: 2- Anti- Ro antibody 

Anti-Ro is important in pregnancy since it is 
associated with babies born with congenital 
heart block. 


appetite. Investigations show : ESR 95 mm/hr, 
normochromic normocytic anaemia and a 
positive rheumatoid factor. 

The likely diagnosis is: 

1- Fibromyalgia 

2- Polymyositis 

3- Dermatomyositis 

4- Polymyalgia Rheumatica 

5- Chronic myeloid leukaemia 

Answer & Comments 

Answer: 4- Polymyalgia Rheumatica 

Polymyalgia Rheumatica is associated with 
weight loss, anemia & malaise. 


[ Q: 207 ] MRCPass - Rheumatology 

A 50 year old secretary noticed 
tingling and numbness over the palmar 
surface of her thumb, index and middle 
fingers after several hours at her computer. 
Pain in the same areas often occur at night. 

What is the diagnosis? 

1- Rheumatoid arthritis 

2- Gout 

3- Dermatomyositis 

4- Carpal tunnel syndrome 

5- Psoriatic arthritis 



Answer & Comments 


Answer: 4- Carpal tunnel syndrome 


It can also be associated with a false positive 
rheumatoid factor, especially in the elderly. 

[ Q: 209 ] MRCPass - Rheumatology 

A 12 year old boy has fevers. He has 
had elbow , shoulder and hip and knee pains 
over the last few months. On examination, he 
has a temperature of 37.5 C and palpable 
hepatosplenomegaly. 

Investigations show a CRP of 30 and negative 
ANA. 

What is the likely diagnosis? 

1- Marfan's syndrome 

2- Ehler Danlos syndrome 

3- Perthe's disease 

4- Juvenile chronic arthritis 



The diagnosis is carpal tunnel syndrome, and 
the symptoms are due to tenosynovitis which 
is worsened by repetitive strain imposed by 
typing. 


5- Osteoarthritis 


Answer & Comments 


Answer: 4- Juvenile chronic arthritis 


[ Q: 208 ] MRCPass - Rheumatology 

A 75 year old man complains of pain 
and stiffness in both his shoulders. He has lost 
1/2 a stone in last 6 weeks due to loss of 



Juvenile chronic arthritis (JCA) is a form of 
seronegative arthritis in the young (adult form 
known as Still's disease) which is rheumatoid 
factor and ANA negative. Diagnostic criteria 
include high fever, hepatomegaly, 


Dr. Khalid Yusuf El-Zohry - Sohag Teaching Hospital (01118391123) 

Ref MRCPass OE OE 2012 PasTest 2009 PassMedicine 2009 PasTest Exam ReviseMRCP 



117 
































El-zohry MRCP Questions Bank (Port 1) - 2013 


(For my personal use) 


splenomegaly, lymphadenopathy, serositis 
(pleuritis, pericarditis), leucocytosis. Bone 
destruction and micrognathia occurs. 



[ Q: 210 ] MRCPass - Rheumatology 

A 25 year old lady presents to clinic 
with a painfully swollen hot right knee. She 
gives a 2 week history of migratory 
polyarthritis and urethritis. Clinical 
examination reveals a pustular skin rash, right 
Achilles tendinitis and left plantar fasciitis. She 
is HLA-B27 negative. 


Whot is the most likely diagnosis? 


1- Relapsing polychondritis 

2- Reiter's syndrome 


3- Gonoccocal arthritis 


4- Non specific urethritis 

5- Behcet's syndrome 


Answer & Comments 

Answer: 3- Gonoccocal arthritis 

Gonoccocal arthritis classically presents with a 
hot joint on a background of a migrating 
polyarthropathy. It affects w omen more 
frequently than men (4:1) and its highest 
incidence is among sexually active adolescent 
girls. There is also increased risk during 
menstruation and pregnancy. 

Tw o forms of arthritis exist -- one with skin 
rashes and multiple joint involvement, and a 
second, less common, form in which 
disseminated gonococcemia leads to infection 
of a single joint (monoarticular) and joint fluid 
cultures are positive. 


[ Q: 211 ] MRCPass - Rheumatology 

A 50 year old woman with 
longstanding rheumatoid arthritis, and a sicca 
syndrome. She presents with increasing 
malaise. Investigations showed: 

Hb 7.5 g/dl 



Neutrophil count 1.2 x 10 9 /L 

platelet count 90 x 10 9 /l_ 

Her ESR was 120 mm/hr and CRP 145 mg/dl 

On examination, she was pale and had 
palpable splenomegaly of 6 cm edge. 

The most likely cause of her haematologic 
abnormalities is: 

1- Anaemia chronic disease 

2- Upper Gl bleed 

3- Felty's syndrome 

4- Folate deficiency 

5- Iron deficiency 


Answer & Comments 

Answer: 3- Felty's syndrome 

Felty's syndrome is the triad of seropositive 
arthritis, splenomegaly and neutropenia. 

The cause of Felty's syndrome is not known, 
but is most commonly associated with 
rheumatoid arthritis. 


[ Q: 212 ] MRCPass - Rheumatology 

32 year old lady upper middle class 
white lady who used to be an athlete has been 
referred for investigation due to tiredness. 
She mentions fatigue which is chronic and 
occurs even after minor physical w ork. This 
has been going on for 3 years. Investigations 
including CK, ESR, EMGs and muscle biopsy 
have revealed no obvious medical cause for 
this. 

Which of the following is the best treatment? 

1- Erythropoietin injections 

2- Cognitive behavioural therapy 

3- Graded exercise programme 

4- Codeine 

5- Fluoxetine 



Dr. Khalid Yusuf El-Zohry - Sohag Teaching Hospital (01118391123) 

Ref MRCPass OE OE 2012 PasTest 2009 PassMedicine 2009 PasTest Exam ReviseMRCP 



























El-zohry MRCP Questions Bank (Port 1) - 2013 


(For my personal use) 


Answer & Comments 

Answer: 3- Graded exercise programme 

Chronic Fatigue syndrome is defined by 
symptoms and not signs. The clinical profile of 
an individual with CFS is of a high-achieving 
student or athlete who usually is female 
(80%), white, and middle-class to upper 
middle-class. Treatment is largely supportive 
and responsive to symptomatology. This 
includes physical therapy and modest aerobic 
or anaerobic exercise (if possible) to avoid 
cardiovascular deconditioning. Sleep may be 
addressed with medication; often, melatonin 
or night-time amitriptyline is helpful. If 
present and severe, pain often is addressed in 
a pain clinic. 


[ Q: 213 ] MRCPass - Rheumatology 

A 45 year old patient presents with 
proximal muscle weakness, particularly in the 
lower limbs. She has a heliotropic rash around 
the eyes and also Gottron's papules. 

Which one of the following antibodies is most 
strongly associated? 

1- La 




Gottron's papules 


[ Q: 214 ] MRCPass - Rheumatology 

A 9-year-old boy is brought to the 
paediatrician with complaints of fever for 1 
week. He also has cracked lips, which are 
painful. 

On examination, his eyes are red and he has a 
lymph node in the cervical region. Kawasaki 
syndrome is suspected. 

Which of the following is the best treatment 
choice? 

1- Intravenous immunoglobulim 

2- Hydrocortisone 

3- Abxicimab 



2- Ro 


3- Jo-1 


4- Amoxicillin 

5- Low molecular weight heparin 


4- SCL-70 

5- Anti DsDNA 

Answer & Comments 

Answer: 3- Jo-1 

The diagnosis is dermatomyositis. Anti Jo-1 
antibody is associated with acute onset 
myositis, particularly dermatomyositis. The 
limb girdle or proximal muscles are most 
severely affected in both polymyositis and 
dermatomyositis. 


Answer & Comments 

Answer: 1- Intravenous immunoglobulim 

This is a case of Kawasaki's syndrome, which is 
also known as mucocutaneous lymph node 
syndrome and occurs mainly in children under 
10 years of age. It is a form of vasculitis, which 
affects coronary arteries and is associated 
with the development of coronary aneurysms. 

Aspirin and IV immunoglobulin at high doses is 
the treatment of choice. Corticosteroids used 
to be contraindicated, but recently trials have 
shown that it reduced the risk of heart 
damage caused by coronary vessel vasculitis. 


Dr. Khalid Yusuf El-Zohry - Sohag Teaching Hospital (01118391123) 

Ref MRCPass OE OE 2012 PasTest 2009 PassMedicine 2009 PasTest Exam ReviseMRCP 



119 






























El-zohry MRCP Questions Bank (Port 1) - 2013 


(For my personal use) 


Infliximab is also being used in trials. Warfarin 
is used if coronary aneurysms develop as a 
complication. 

[ Q: 215 ] MRCPass - Rheumatology 

A 55 year old woman presents with a 
2 month history of pain affecting the cervical 
spine and both shoulders, this was preceded 
by pain in the lower back and both hips. Early 
morning stiffness lasts until lunchtime and she 
feels markedly tired and weak. Examination 
reveals bilateral knee effusions and a right 
carpal tunnel syndrome. 

Investigations demonstrate a normochromic 
nomocytic anaemia of Hb 9.8 g/dl, ESR 72 mm 
in the first hour, CRP 12 g/l, serum 
immunoglobulins and protein electrophoresis 
show a polyclonal increase in gamma globulins 
and elevated alpha 1 and alpha 2 globulins but 
no paraprotein band. 

What is the most likely diagnosis? 

1- Rheumatoid arthritis 

2- Paraneoplastic syndrome 

3- Mixed connective tissue disease 

4- Scleroderma 

5- Polymyalgia rheumatica 



Answer & Comments 

Answer: 5- Polymyalgia rheumatica 

Polymyalgia rheumatica (PMR) is a clinical 
diagnosis based on pain and stiffness of pelvic 
girdle and shoulder. It is more common after 
the age of 55. 

In contrast to polymyositis there is no 
muscular weakness. Early morning stiffness of 
the hip and shoulder girdles is common. A 
normochromic normocytic anaemia is 
associated. 


[ Q: 216 ] MRCPass - Rheumatology 

A 55 year old man presents with 
sudden onset of a right-sided foot drop with 
numbness over the dorsum of his right foot. 
He has weakness of dorsiflexion and eversion 
of his right foot. There is an area of diminished 
sensation over the dorsum of the right foot. 
He also has a left wrist drop with loss of 
sensation over the dorsal aspect of the first 
interosseus space. For the last tw o months, 
he has lost weight and he had generalized 
myalgia. 

Investigations reveal: 

Hb 11.5 g/dL 
WBC 13 x 10 9 /l_ 

Neutrophils 9.2 x 10 9 /L 
Lymphocytes 2.2 x 10 9 /L 
ESR 60 mm/lst hour 
Urinalysis: Protein ++, Blood ++ 

The most likely diagnosis is: 

1- Tuberculosis 

2- Polyarteritis nodosa 

3- Sarcoidosis 

4- Rheumatoid arthritis 

5- Systemic lupus erythematosus 



Answer & Comments 

Answer: 2- Polyarteritis nodosa 

The patient has a systemic illness with 
mononeuritis multiplex and renal 
involvement. Although all the listed conditions 
can cause mononeuritis, PAN is one of the few 
connective tissue disorders that usually occur 
in middle-aged men unlike RA and SLE, which 
are commoner in females. 


[ Q: 217 ] MRCPass - Rheumatology 

A 60 year old man presents with pain 
in his left foot. He mentions that he has had 
poor mobility. General examination reveals a 



Dr. Khalid Yusuf El-Zohry - Sohag Teaching Hospital (01118391123) 

Ref MRCPass OE OE 2012 PasTest 2009 PassMedicine 2009 PasTest Exam ReviseMRCP 




























El-zohry MRCP Questions Bank (Port 1) - 2013 


(For my personal use) 


swollen ankle. Neurological examination 
reveals absent ankle jerk and weak foot 
flexion/extension on the left. His ESR is 30 
mm/hr, CRP is <5mg/l. Joint x ray reveals 
subchondral fractures, soft tissue swelling and 
a narrowed joint space. 

The likely diagnosis is: 

1- Charcot's joint 

2- Osteoarthritis 

3- Gout 

4- Juvenile chronic arthritis 

5- Previous road traffic accident 


Answer & Comments 

Answer: 1- Charcot's joint 

Charcot joint or neuropathic joint disease can 
be caused by diabetic neuropathy, syphillis, 
syringomyelia or leprosy. The X ray changes 
are reduced joint space with periarticular 
destruction, subchondral bodies, loose bodies 
and occasionally joint dislocation. 



Charcot's joint 


[ Q: 218 ] MRCPass - Rheumatology 

A 55 year old man presents with 
lethargy, polyuria, polydipsia and stiffness of 
the hands. He has evidence of an arthopathy 
affecting the 2nd & 3rd metacarpophalangeal 
joints of both hands. Xray confirms evidence 
of degenerative disease at these sites. 

Which of the following the likely diagnosis? 

1- Behcet's syndrome 

2- Haemochromatosis 

3- Amyloidosis 



4- Reiter's syndrome 

5- Osteoarthritis 


Answer & Comments 

Answer: 2- Haemochromatosis 

The characteristic feature of 
haemochromatotic arthropathy is 
involvement of the second and third 
metacarpals. The arthropathy can lead to 
extensive joint destruction. 

These patients may have episodes of acute, 
inflammatory pseudogout from such 
deposition. 


^ [ Q: 219 ] MRCPass - Rheumatology 

* An 22 year old girl presents with a 6 
week history of polyarthralgia with early 
morning stiffness. Her symptoms responded 
well initially to Ibuprofen but she then they 
worsen again. She is otherw ise well apart 
from a history of acne which well controlled 
on Minocycline. Her mother severe 
rheumatoid arthritis. 

Investigations show : ESR 40 mm/hr, CRP 110 
mg/I, rheumatoid factor negative, ANA 
strongly positive (1:1600), antidsDNA 
antibodies negative. 

What is the likely cause of her symptoms? 

1- Drug induced lupus 

2- Systemic Lupus Erythematosus 

3- Psoriatic arthropathy 

4- Juvenile chronic arthritis 

5- Fibromyalgia 


Answer & Comments 

Answer: 1- Drug induced lupus 

The likely diagnosis is drug induced SLE. 
Minocycline is one of the causes well 
documented as a cause of drug induced SLE. 


Dr. Khalid Yusuf El-Zohry - Sohag Teaching Hospital (01118391123) 

Ref MRCPass OE OE 2012 PasTest 2009 PassMedicine 2009 PasTest Exam 




ReviseMRCP 

121 




























El-zohry MRCP Questions Bank (Port 1) - 2013 


(For my personal use) 


Characteristically, inflammatory markers such 
as ESR & CRP are both markedly elevated. 
ANA can be strongly positive but AntidsDNA 
antibodies are usually negative. Symptoms 
usually resolve following withdraw al of the 
drug after several months. 



[ Q: 220 ] MRCPass - Rheumatology 

A 50 year old woman presents to 
casualty with a 2 day history of pain and 
swelling of the left ankle. She denies any 
history of recent trauma. On examination, she 
was febrile, temperature 38.3 C. The left ankle 
was swollen and very tender with a reduced 
range of movement. 


Which of the following investigations would be 
most helpful? 

1- Urate level 


2- ESR 

3- Rheumatoid factor 

4- Aspiration of synovial fluid 

5- Knee X ray 


Answer & Comments 

Answer: 4- Aspiration of synovial fluid 

Joint fluid examination is important in 
excluding septic arthritis, and can also help to 
confim gout or pseudogout. 



[ Q: 221 ] MRCPass - Rheumatology 

A 35 year old woman with newly 
diagnosed rheumatoid arthritis enquires 
about risk factors. 


Which one of the following is a risk factor for 
rheumatoid arthritis? 


1- Pneumococcal infection 


Answer & Comments 

Answer: 2- HLA-DR4 

Risk factors for rheumatoid arthritis are : 

HLA-DR4 

EBV 

parvovirus B19 and rubella infections 

blood transfusion 

smoking (induces RF production) 

stress 

obesity 

Pregnancy and OCP are protective. 

[ Q: 222 ] MRCPass - Rheumatology 

A 26 year old man presents with a 6 
month history of low back pain. The pain 
radiates to his buttocks. There is associated 
stiffness which is w orse in the morning and 
after periods of inactivity. 

Which of the following signs is most likely to 
be present? 

1- Foot drop 

2- Positive femoral stretch test 

3- Positive Trendelenburg test 

4- Sacroiliac joint tenderness 

5- Kernig's sign 



Answer & Comments 

Answer: 4- Sacroiliac joint tenderness 

The diagnosis is ankylosing spondylitis as this 
is a young patient with lower back pains and 
morning stiffness. Sacroilitis is a common 
manifestation. 


2- HLA-DR4 

3- SLE 

4- HIV 

5- Renal failure 



[ Q: 223 ] MRCPass - Rheumatology 


A 62 year old man has complains of a 
pain in the temples, fevers, sweats and 
malaise for 4 weeks. The right side of the face 


Dr. Khalid Yusuf El-Zohry - Sohag Teaching Hospital (01118391123) 

Ref MRCPass OE OE 2012 PasTest 2009 PassMedicine 2009 PasTest Exam ReviseMRCP 

































El-zohry MRCP Questions Bank (Port 1) - 2013 


(For my personal use) 


was mildly swollen and he experienced visual 
loss in the right eye. 

On examination, he had prominent and tender 
temporal arteries on the right. 

Blood tests revealed: 

Hb 12.5 g/dl MCV 86 fl 

WCC 12 x 10 9 /L platelets 212 x 10 9 /L 

ESR of 90 mm/hr 

What is the likely diagnosis? 

1- Pituitary tumour 

2- Grave's disease 

3- Paget's disease 

4- Cataract 

5- Temporal arteritis 

Answer & Comments 

Answer: 5- Temporal arteritis 

The diagnosis is temporal arteritis (giant cell 
arteritis). 

The complications of Giant Cell Arthritis are 
related to arterial inflammation. These 
include: 

blindness 

absent pulses 

hypertension 

angina 

stroke 

claudication 



Prominent temporal arteries 

[ Q: 224 ] MRCPass - Rheumatology 

# - 

0 

A 60 year old woman presents with a 
history of progressive difficulty in climbing 
stairs and rising from chairs. 

She also complains of difficulty in swallowing. 
She complains that her hands discolour and 
become cold easily. 

On examination she has purple discolouration 
of her eyelids and periorbital oedema. There is 
weakness of the muscles of her limb girdles. 

What is the likely couse of the dysphagia? 

1- Oesophageal cancer 

2- Dermatomyositis 

3- Systemic Lupus Erythematosus 

4- Retrosternal goitre 

5- Scleroderma 

Answer & Comments 

Answer: 2- Dermatomyositis 

The diagnosis is dermatomyositis. Dysphagia 
in dermatomyositis is primarily due to 
weakness of the striated musculature of the 
posterior pharynx. Dysphagia may also result 
from cricopharyngeal obstruction secondary 
to inflammation or fibrosis of the 
cricopharyngeus muscles. 


Dr. Khalid Yusuf El-Zohry - Sohag Teaching Hospital (01118391123) 

Ref MRCPass OE OE 2012 PasTest 2009 PassMedicine 2009 PasTest Exam ReviseMRCP 
























El-zohry MRCP Questions Bank (Port 1) - 2013 


(For my personal use) 



[ Q: 225 ] MRCPass - Rheumatology 

A 60 year old man is on frusemide 
for leg oedema. He presents with a painful 
swollen knee. His temperature is 38 °C and he 
has a white cell count of 12 x 10 9 /L and CRP of 
120mg/l. Uric acid is 0.49 mmol/I (<0.42). 

A joint aspirate reveals pus cells and 
negatively birefringent crystals with polarized 
light. After treatment with NSAIDs for 48 
hours he has not improved and the swelling 
persists. There are no organisms cultured from 
the joint aspirate or blood cultures. 

What is the best course of action? 

1- Allopurinol 

2- MRI to look for osteomyelitis 

3- Intravenous flucloxacillin 

4- Arthroscopy and washout 

5- Further joint aspiration with depomedrone 
injection 



Answer & Comments 

Answer: 5- Further joint aspiration with 
depomedrone injection 

This is a case of acute gout which has not 
settled on medical therapy (NSAIDs or 
colchicine). The raised inflammatory markers 
can be due to acute gout. Uric acid levels are 
not necessarily high in acute gout. A 
depomedrone (steroid) injection with further 
aspiration of joint fluid is the next step. 


[ Q: 226 ] MRCPass - Rheumatology 

A 40 year old lady presents with 
polyuria and thirst. Her serum calcium of 2.85 
mmol/I and a parathyroid hormone of 12 (1-8) 
pmol/l. She has normal renal function. 

Which test provides the best assessment? 

1- CT of her spine 

2- Dual energy X-ray absorptiometry 

3- Vitamin D levels 

4- Isotope bone scan 

5- Urine bence jones protein 



Answer & Comments 

Answer: 2- Dual energy X-ray absorptiometry 

This patient is likely to have primary 
hyperparathyroidism (secondary to a 
parathyroid adenoma). The best assessment is 
to determine the severity of loss of bone 
density with a DEXA scan in order to consider 
parathyroid surgery is necessary. 


[ Q: 227 ] MRCPass - Rheumatology 

An 45 year old man has had 
worsening back pains, shoulder pains and 
right hip pain for 8 years. The pain is typically 
w orse at the end of day. He also has bony 
enlargement of the distal interphalangeal 
joints. An X ray of the shoulder reveals the 
presence of prominent osteophytes. There is 
sclerosis and narrow ing of the joint space at 
the hip joints on the pelvic X ray. 

What is the likely diagnosis? 

1- Osteomyelitis 

2- Osteoarthritis 

3- Osteomalacia 

4- Rheumatoid arthritis 

5- Paget's disease 



Answer & Comments 


Answer: 2- Osteoarthritis 



Dr. Khalid Yusuf El-Zohry - Sohag Teaching Hospital (01118391123) 

Ref MRCPass OE OE 2012 PasTest 2009 PassMedicine 2009 PasTest Exam 


ReviseMRCP 






























El-zohry MRCP Questions Bank (Port 1) - 2013 


(For my personal use) 


The clinical history suggests early onset 
osteoarthritis (which can be idiopathic or 
occur in athletes). The X ray features of 
osteophytes, sclerosis and narrow ing of joint 
space are suggestive of osteoarthritis. 





Osteophyte on a shoulder X ray 


[ Q: 228 ] MRCPass - Rheumatology 

A 35 year old woman has generalized 
joint pains and muscle aches. She also 
complains of gritty sensations in the eye in the 
mornings. Blood tests show Anti Nuclear 
Antibody++, Rheumatoid Factor++. 

Whot is the diagnosis? 

1- Polylmyositis 

2- Reiter's syndrome 

3- Primary Sjogren's syndrome 

4- Polyarteritis nodosa 

5- Rheumatoid arthritis 



Answer & Comments 

Answer: 3- Primary Sjogren's syndrome 

Dry mouth, dry eyes, fatigue, muscle aches 
and joint pains are typical of Sjogren's 
syndrome. 80% of patients are ANA positive 
and 75% are Rheumatoid factor positive. A 
useful diagnostic test is the Schirmer's test, w 
here a piece of filter paper is placed in the 
corner of the eye to measure the degree of w 
etting after five minutes. 


[ Q: 229 ] MRCPass - Rheumatology 

A 60 year old woman presents with a 
tw o week history of malaise and lower limb 
joint pains. On examination she had a 
vasculitic rash over her shins, thighs and 
buttocks. Investigations revealed: 

Hb 10.2 g/dL 

platelets 265 x 10 9 /L 

creatinine 380 pmol/L 

antinuclear antibodies - Negative 

antineutrophil cytoplasmic antibodies 
Negative 

antiglomerular basement membrane 
antibodies - Negative 

dipstix urinalysis - blood+++ 

protein + 

What is the likely diagnosis causing renal 
impairment? 

1- Psoriatic arthritis 

2- Henoch Schonlein purpura 

3- Polymyositis 

4- Membranous nephropathy 

5- Myeloma 



Answer & Comments 

Answer: 2- Henoch Schonlein purpura 

The distribution of the rash together with 
lower limb joint pains are suggestive of 
Henoch Schonlein purpura. This usually occurs 
in children aged 2-10 years but can occur in 
older age groups. The only w ay of 
differentiating this condition from other small 
vessel vasculitides is by biopsy. This would 
show IgA deposition in vessel w alls on direct 
immunofluorescence. 



[ Q: 230 ] MRCPass - Rheumatology 
A 28 year old woman presents with a 


Dr. Khalid Yusuf El-Zohry - Sohag Teaching Hospital (01118391123) 

Ref MRCPass OE OE 2012 PasTest 2009 PassMedicine 2009 PasTest Exam ReviseMRCP 



125 



























El-zohry MRCP Questions Bank (Port 1) - 2013 


(For my personal use) 


right knee joint pain and a 4 month history of 
weight loss. She thinks she has lost weight 
because of diarrhoea, which occurs several 
times a day. Examination reveals a swollen, 
tender right knee joint with a small effusion. 

The likely diagnosis is: 

1- Reiter's syndrome 

2- Inflammatory bowel disease 

3- Behcet's disease 

4- Campylobacter infection 

5- Rheumatoid arthritis 


Answer & Comments 

Answer: 2- Inflammatory bowel disease 

The description of weight loss, diarrhoea and 
a mono/oligoarthropathy suggests a diagnosis 
of inflammatory bow el disease. Peripheral 
arthritis, peripheral arthralgia without joint 
swelling or effusion, degenerative joint 
disease or seropositive arthritis can occur in 
inflammatory bow el disease. In patients with 
peripheral arthralgia and peripheral arthritis, 
there is a significantly greater prevalence of 
mucocutaneous manifestations of IBD i.e. oral 
ulceration, erythema nodosum, pyoderma 
gangrenosum, and uveitis. 



[ Q: 231 ] MRCPass - Rheumatology 

A professional tennis player presents 
with shoulder pains especially whilst serving 
the ball. He has limited passive and active 
shoulder abduction to less than 60°. His 
temperature is 36.5°C and he has a normal 
white cell count. There is tenderness around 
the anterior portion of the shoulder joint. 

Which diagnosis is likely? 

1- Glenohumeral joint osteoarthritis 

2- Bursitis 


3- Tennis elbow 

4- Supraspinatus tendonitis 

5- Septic arthritis 


Answer & Comments 

Answer: 4- Supraspinatus tendonitis 

Pain during abduction with limitation of 
movement is suggestive of supraspinatous 
tendonitis. Palpation or compression around 
the greater tubercle of the humerus is 
particularly tender. 


[ Q: 232 ] MRCPass - Rheumatology 

A 45 year woman has raynauds 
phenomenon. She also has difficulty in sw 
allow ing and dyspnoea. 

Echocardiography shows right heart strain. 
Blood tests reveal renal impairment. 

Which one of the following antibodies is 
specific to this lady’s condition? 

1- Anticentromere antibody 

2- Topoisomerase I 

3- Anti ds DNA antibody 

4- Anti SCL 70 antibody 

5- Anti mitochondrial antibody 



Answer & Comments 

Answer: 4- Anti SCL 70 antibody 

Anti SCL70 antibody (topoisomerase I) is 
typically found in progressive systemic 
sclerosis (not the limited cutaneous form, 
CREST). 

[ Q: 233 ] MRCPass - Rheumatology 

A 55 year old man presents with a 6 
week history of lethargy and diffuse purpuric 
rash. He is noted to have a right foot drop and 
a left ulnar nerve palsy. He complains of 
arthralgia but has no clinical evidence of 
inflammatory joint disease. Echocardiogram is 
unremarkable, blood cultures are negative, 
ESR 80 mm/hr, ANCA negative, ANA negative, 
rheumatoid factor strongly positive, C3 1.1 g/l 
(0.75 -1.6), C4 0.03 g/l (0.14 - 0.5). 

Dipstick urinalysis shows blood ++. 



Dr. Khalid Yusuf El-Zohry - Sohag Teaching Hospital (01118391123) 

Ref MRCPass OE OE 2012 PasTest 2009 PassMedicine 2009 PasTest Exam ReviseMRCP 



126 




























El-zohry MRCP Questions Bank (Port 1) - 2013 


(For my personal use) 


What is the likely diagnosis? 

1- Takayasu's arteritis 

2- ANA negative SLE 

3- Culture negative endocarditis 

4- Cryoglobulinaemia 

5- Rheumatic fever 


5- Stimulate parathyroid hormone secretion 


Answer & Comments 

Answer: 3- Inhibit osteoclasts 

Bisphosphonates inhibits osteoclasts and 
reduces progression tow ards bone 
destruction. 


Answer & Comments 

Answer: 4- Cryoglobulinaemia 

A low C4 together with a strongly positive 
rheumatoid factor suggests cryoglobulinaemia 
as a cause of mononeuritis multiplex and rash. 
Palpable purpura, arthralgia, 

hepatosplenomegaly, diffuse proliferative 
glomerulonephritis, Raynaud's phenomenon 
and thrombosis may occur. 

Type I cryoglobulinaemia may be associated 
with lymphoproliferative disorders, multiple 
myeloma, and monoclonal gammopathy of 
uncertain significance, and 

macroglobulinaemia. Plasmapheresis may 
reduce the levels of cryoglobulin. 



[ Q: 235 ] MRCPass - Rheumatology 

A 80 year old lady presents with a 5 
day history of severe left temporal headache 
radiating from her eye to the scalp. She had 
also experienced jaw discomfort during 
eating. 


Which of the following drugs should be given 
while awaiting results of diagnostic tests? 


1- Carbamazepine 

2- Prednisolone 


3- Azathioprine 

4- Infliximab 


5- Intravenous immunoglobulin 


Type II cryoglobulinaemia (mixed monoclonal) 
is usually composed of a monoclonal 
component (usually IgG, IgM or IgA) and a 
polyclonal component (mainly IgG). Causes 
are connective tissue diseases, Hepatitis B and 
C infection, infectious mononucleosis and 
lymphoma. 


[ Q: 234 ] MRCPass - Rheumatology 

A 70 year old woman with a history 
of multiple myeloma has a serum calcium of 
2.9 mmol/I. He is prescribed pamidronate 
infusion over 4 days. 

What is its mechanism of action? 

1- Promotes calcitonin 

2- Increases calcitriol levels 

3- Inhibit osteoclasts 

4- inhibit osteoblasts 



Answer & Comments 

Answer: 2- Prednisolone 

The history suggests temporal arteritis. In 
view of the vision threatening nature of 
disease, the patient should be commenced on 
steroids. Typically 60mg of prednisolone per 
day is recommended. 


[ Q: 236 ] MRCPass - Rheumatology 

A 30 year old woman has a 3 month 
history of arthralgia. There is swelling of the 
distal interphalangeal joints of the ring fingers 
of the hand. The wrist on the right and ankles 
are swollen as well. Onycholysis was noted on 
the nails. Her serum inflammatory markers are 
raised. 

Which of the following is the likely diagnosis? 
1- Polymyalgia rheumatica 



Dr. Khalid Yusuf El-Zohry - Sohag Teaching Hospital (01118391123) 

Ref MRCPass OE OE 2012 PasTest 2009 PassMedicine 2009 PasTest Exam 




ReviseMRCP 

127 





























El-zohry MRCP Questions Bank (Port 1) - 2013 


(For my personal use) 


2- Rheumatoid arthritis 

3- Gout 

4- Psoriatic arthropathy 

5- SLE 


2- CMV infection 

3- Tuberculosis 

4- Bronchial carcinoma 

5- Rheumatoid related pulmonary fibrosis 


Answer & Comments 


Answer: 4- Psoriatic arthropathy 


Answer & Comments 


Answer: 3- Tuberculosis 


Psoriatic arthritis effects distal interphalangeal 
joints tends be asymmetrical. 

There are 5 types of psoriatic arthritis. 

Asymmetrical oligoarticular arthritis is thought 
to be the most common type. Usually, the 
digits of the hands and feet are affected first, 
with inflammation of the flexor tendon and 
synovium occurring simultaneously, leading to 
the typical "sausage" appearance (dactylitis). 


The other types are symmetrical polyarthritis, 
DIP arthropathy, arthritis mutilans and 
spondylitis with or without sacroilitis. 



Psoriatic arthropathy 


[ Q: 237 ] MRCPass - Rheumatology 

m A 60 year old lady has severe 
rheumatoid arthritis. She is currently on 
Methotrexate 20 mg weekly for the past 5 
months and also has been receiving regular 
infusions of Infliximab. Her joint disease 
dramatically improved. She now presents with 
fevers, cough and there is evidence a large left 
sided pleural effusion on her CXR. 

What is the likely diagnosis? 

1- Methotrexate pneumonitis 


Serious opportunistic infections have been 
associated with the anti TNF alpha drug 
infliximab, but the frequency of TB exceeds 
that associated with other infections. 
Infliximab may increase the risk of lymphoma. 

[ Q: 238 ] MRCPass - Rheumatology 

A 75 year old man presents with an 
acute onset of severe pain and swelling of the 
left elbow . He mentions that he had a chest 
infection 3 weeks ago. On examination, he 
had a temperature of 38°C and the left elbow 
was erythematous, swollen and tender. 

What is the most appropriate investigation? 

1- C reactive protein 

2- Full blood count 

3- Joint aspiration 

4- Uric acid level 

5- X-ray of the joint 



Answer & Comments 

Answer: 3- Joint aspiration 

This patient is likely to have reactive arthritis. 
How ever, gram stain and culture are 
necessary to exclude septic arthritis. 


[ Q: 239 ] MRCPass - Rheumatology 

j 

A 75 year old man has significant 
bony pains which have been occuring for 3 
years. He presents to the clinic for assessment 
and the investigations results were obtained 
below: 

Corrected calcium 2.4 (2.2 - 2.6) 



Dr. Khalid Yusuf El-Zohry - Sohag Teaching Hospital (01118391123) 

Ref MRCPass OE OE 2012 PasTest 2009 PassMedicine 2009 PasTest Exam 




ReviseMRCP 

128 


































El-zohry MRCP Questions Bank (Port 1) - 2013 


(For my personal use) 


ESR 20 mm/lst hr 

Alkaline phosphatase 625 iu/L (50 - 100) 
gamma GT 42 iu/L (10-50) 

Prostate specific antigen 7.4 pg/L (0-6) 
What is the most likely diagnosis? 

1- Osteoporosis 

2- Osteomalacia 


2- Morphine 

3- Ibuprofen gel 

4- Higher dose of codeine 

5- Oral hydrocortisone 

Answer & Comments 

Answer: 3- Ibuprofen gel 


3- Metastatic prostatic carcinoma 

4- Paget's disease 

5- Multiple myeloma 


Answer & Comments 

Answer: 4- Paget's disease 

Paget's disease causes a high alkaline 
phosphatase and normal calcium levels. 

Paget's Disease represents an imbalance of 
bone formation and resorption. It typically 
begins with excessive bone resorption 
followed by excessive bone formation. The 
main disturbance is an exaggeration of 
osteoclastic bone resorption. 


NSAIDS tend to be better for pain control in 
osteoarthritis, but have significant side 
effects. Local NSAID application should be 
considered as well as intraarticular steroid 
injections. 


[ Q: 241 ] MRCPass - Rheumatology 

A 40 year old lady presents with a 
swollen right knee. This was aspirated. Under 
the polarised microscope, there were 
neutrophils +++ and some blue rhomboidal 
crystals under parallel polarised light. 

The most likely diagnosis is: 

1- Osteoarthritis 

2- Rheumatoid arthritis 



The most common sites of involvement 
include the spine, pelvis, skull, femur and 
tibia. Skull involvement may produce 
enlargement of the head characterized by 
more evident frontal bossing and dilated 
superficial cranial muscles. 

Conductive and/or sensorineural hearing loss 
may result from disease of the temporal bone 
or ossicles. 



[ Q: 240 ] MRCPass - Rheumatology 

A 75 year old patient with 
osteoarthritis of the knee has been taking 
codeine 30mg qds and also paracetamol 
regularly. He continues to have knee pains. 

What is the next best treatment? 

1- Diclofenac 


3- Gout 

4- Pseudogout 

5- Septic arthritis 


Answer & Comments 

Answer: 4- Pseudogout 

The blue colour and positively birefringent 
crystals under polarised light suggest 
pseudogout. Neutrophil levels are high in 
infection, gout and pseudogout. 



Dr. Khalid Yusuf El-Zohry - Sohag Teaching Hospital (01118391123) 

Ref MRCPass OE OE 2012 PasTest 2009 PassMedicine 2009 PasTest Exam ReviseMRCP 



129 































El-zohry MRCP Questions Bank (Port 1) - 2013 


(For my personal use) 



[ Q: 242 ] MRCPass - Rheumatology 

A 45 year old man has a renal 
transplant. He presents with a hot 


swollen ankle on the left and fevers. The 
symptoms have been present for 2 days. 

What should be done? 


1- Check serum uric acid level 



Illustration of supraspinatus muscle 


2- Inject intra-articular steroids 

3- Give colchicine 

4- Send joint fluid for microscopy and culture 

5- Intravenous cefuroxime 


Answer & Comments 

Answer: 4- Send joint fluid for microscopy and 
culture 

The differential diagnosis is gout (common in 
renal patients) but the patient will be in 
immunosupression and septic arthritis should 
be excluded with joint fluid culture. 

[ Q: 243 ] MRCPass - Rheumatology 

A 40 year old athlete has pain on 
abduction of her arm, particularly w hen 
resisted. 

Which tendon pathology is affected? 

1- Biceps tendonitis 

2- Supraspinatus tendonitis 

3- Teres minor tendonitis 

4- Infraspinatus tendinitis 

5- Subscapularis tendonitis 



Answer & Comments 

Answer: 2- Supraspinatus tendonitis 

Pain in abduction up to 90 degrees is due to 
supraspinatus tendonitis. 


[ Q: 244 ] MRCPass - Rheumatology 

A 60 year old man has worsening 
discomfort in both shoulders. He is 
haemodialysis dependent. Past medical 
history included bilateral carpal tunnel 
decompression. His Investigations reveal: 
haemoglobin 9.8 g/dl, ESR 35 mm/lst hr, C 
reactive protein 15 mg/L, Urate 0.58. 

What is the likely diagnosis? 

1- Reiter's syndrome 

2- Amyloidosis 

3- Polymyalgia rheumatica 

4- Gout 

5- Osteomalacia 



Answer & Comments 

Answer: 2- Amyloidosis 

b2 microglobulin deposition in joints may lead 
to amyloidosis. This can occur 10 years on 
from dialysis, and lead to carpal tunnel 
syndrome. 


^ [ Q: 245 ] MRCPass - Rheumatology 

a - 

# A 45 year old man presents with a 
painful swollen knee. He feels generally 
unwell and has fever. He has recently had a 
flulike illness, an erythematous rash on the 
trunk followed by a self-limiting episode of 
diarrhoea. A diagnosis of reactive arthritis is 
made by the rheumatologist. 

How should the patient be managed? 

1- High dose steroids 


Dr. Khalid Yusuf El-Zohry - Sohag Teaching Hospital (01118391123) 

Ref MRCPass OE OE 2012 PasTest 2009 PassMedicine 2009 PasTest Exam ReviseMRCP 
































El-zohry MRCP Questions Bank (Port 1) - 2013 


(For my personal use) 


2- Broad spectrum iv antibiotics 

3- Arthroscopy and washout of the joint 

4- If the symptoms becomes chronic, 
sulphasalazine may be useful 

5- Bone scan to look for a focus of infection 


Answer & Comments 

Answer: 4- If the symptoms becomes chronic, 
sulphasalazine may be useful 

The diagnosis is likely to be reactive arthritis 
following an infectious illness. Although joint 
aspiration may be useful, there is no need for 
arthroscopy at present. NSAIDs may be used 
for symptoms control now. If the symptoms 
persist, sulphasalazine or methotrexate may 
be useful. 


[ Q: 246 ] MRCPass - Rheumatology 

A 16 girl is investigated for swelling 
and pain in the right wrist, left knee and right 
ankle. 

Investigations show a positive ANA 1:160 with 
-ve Rheumatoid factor. 

What is she at risk of developing? 

1- Psoriasis 

2- Butterfly facial rash 

3- Erosive joint disease 

4- Uveitis 



juvenile chronic arthritis. Stiffness, 
amyloidosis, and osteoporosis also occur. 



Uveitis causing red eye (lenticular precipitates 

may be seen) 



[ Q: 247 ] MRCPass - Rheumatology 

A 22 year old man presents with a 4 
week history of a painful swollen left knee. He 
has a past medical history of a treated sexually 
transmitted disease 6 months ago. On 
examination there was a large effusion in the 
left knee. 


Synovial fluid analysis shows a white cell count 
of 15 x 10 9 /L but culture was negative. 

Which one of the following organisms is the 
most likely cause? 

1- Gardnerella 


2- Chlamydia 

3- Treponema pallidum 

4- Neisseria gonorrhoea 

5- Trichomonas vaginalis 


5- Bamboo spine 

Answer & Comments 

Answer: 4- Uveitis 

Juvenile chronic arthritis is a term used to 
describe arthritis occurring in someone who is 
less than 16 years old that lasts for more than 
three months. Large joints tend to be affected. 
Rheumatoid factor is often negative, and 
there is positive antinuclear antibody - 
especially in pauciarticular JCA. Acute anterior 
uveitis is most commonly in pauciarticular 


Answer & Comments 

Answer: 4- Neisseria gonorrhoea 

Gonococcal arthritis is caused by an infection 
with the gram-negative diplococcus N 
gonorrhoeae. Neisseria gonorrhoea occurs in 
young adults, often preceded by a migratory 
arthritis. 

The bacteremic phase is a classic triad of 
migratory polyarthritis, tenosynovitis, and 
dermatitis. This patient was treated previously 
for a sexually acquired infection hence may be 
the reason for the culture to be negative. 


Dr. Khalid Yusuf El-Zohry - Sohag Teaching Hospital (01118391123) 

Ref MRCPass OE OE 2012 PasTest 2009 PassMedicine 2009 PasTest Exam 




ReviseMRCP 

131 
























El-zohry MRCP Questions Bank (Port 1) - 2013 


(For my personal use) 


The initial treatment of choice for gonococcal 
arthritis is a third-generation beta-lactamase- 
resistant cephalosporin (eg, ceftriaxone, 
ceftizoxime, cefotaxime) or a penicillin, if the 
organism is sensitive. 



Multiple intracellular gram negative 

diplococcic 


A 50 year old man presents with chronic 
recurrent sinusitis and occasional 
haemoptysis. He has middle lobe patchy 
shadow ing on his chest X-ray. His creatinine is 
145 and urine dipstick shows ++ protein and 
blood. 

Which test result is most likely? 

1- Positive c ANCA 

2- Positive ANA 

3- Positive Kveim test 

4- Positive ASOT 

5- Sputum AFB positive 

Answer & Comments 

Answer: 1- Positive c ANCA 


[ Q: 248 ] MRCPass - Rheumatology 

A 35 year old lady has positive ANA, 
and has a butterfly shaped rash on her face. 
Her physician makes a diagnosis of SLE. She 
has flare ups of joint swellings and pains 
requiring several months treatment with 
prednisolone. 1 year later she presents with 
hip pain limiting her mobility. 

Which one of the following is o likely couse? 

1- Rheumatoid arthritis 

2- Septic arthritis 

3- Juvenile chronic arthritis 

4- Avascular necrosis 

5- Perthe's disease 



Answer & Comments 


Answer: 4- Avascular necrosis 


Nasal cavity, pulmonary and renal 
granulomatous involvement is classical in 
Wegener's granulomatosis. 70% of patients 
have a positive cANCA. In addition, w hen 
ANCA is positive, PR3 (Wegener's) and MPO 
(microscopic polyangitis) helps to distinguish 
between the two differential diagnoses. 



[ Q: 250 ] MRCPass - Rheumatology 

A 55 year old female presents 
complaining of bone pains and muscular 
weakness. Her gait is w addling in nature. 


X-ray shows pseudofractures of her pubic 
rami. 


Investigations show : Serum corrected calcium 
2.05 mmol/L, phosphate 0.43 mmol/L, alkaline 
phosphatase 230 U/L. 

Whot is the diagnosis? 

1- Pseudohypoparathyroidism 


15% of patients with SLE develop avascular 
necrosis of the bone. Nephritis, vasculitis and 
long term steroid use predispose to avascular 
necrosis. 



[ Q: 249 ] MRCPass - Rheumatology 


2- Secondary hyperparathyroidism 

3- Osteopetrosis 

4- Osteomalacia 

5- Paget's disease 


Dr. Kholid Yusuf El-Zohry - Sohog Teaching Hospital (01118391123) 

Ref MRCPass OE OE 2012 PasTest 2009 PassMedicine 2009 PasTest Exam ReviseMRCP 





























El-zohry MRCP Questions Bank (Port 1) - 2013 


(For my personal use) 


Answer & Comments 

Answer: 4- Osteomalacia 

The clinical radiological and biochemical 
features in this patient suggest she has 
osteomalacia. Osteomalacia is characterized 
by a low serum calcium and phosphate with 
elevated serum alkaline phosphatase. 

Osteomalacia may be caused by deficiency of 
vitamin D or phosphate deficiency. 
Malabsorption syndromes, renal failure and 
liver disease can result in vitamin D deficiency. 


[ Q: 251 ] MRCPass - Rheumatology 

A 22 year old Armenian man 
presents with pain and swelling of the left 
knee. He also gives a history of recurrent 
episodes of fever, pleuritic pains in the chest, 
and generalised rash. These episodes typically 
last for 2-3 days. Examination reveals 
splenomegaly, swollen knees and ankles. 
Urine dipstick reveals 2+ proteinuria. 

What is the likely diagnosis ? 

1- Ankylosing spondylitis 

2- Reactive arthritis 

3- Familial Mediterranean fever 

4- Tuberculosis 

5- Dermatomyositis 



Answer & Comments 


Answer: 3- Familial Mediterranean fever 


AA amyloidosis commonly involves the 
kidneys, spleen and Gl tract. Colchicine given 
prophylactically in FMF offers some protection 
against the development of amyloidosis in 
most patients. 

Familial Mediterranean fever primarily affects 
populations originating from the 
Mediterranean region, particularly people of 
Armenian, Arabic, Turkish, and North African 
Jew ish ancestry. Mutations in the MEFV gene 
cause familial Mediterranean fever. 



Rash seen in familial mediterranean fever 



[ Q: 252 ] MRCPass - Rheumatology 

A 35 year old man presents acutely 
with urethritis, conjunctivitis and arthritis. He 
has been having joint pains in the wrist and 
hips. 


On examination, he has a rash on the soles of 
his feet and also circinate balanitis. 


Rheumatoid factor is negative and he has 
raised inflammatory markers. 


Familial Mediterranean fever is an inherited 
condition characterized by recurrent episodes 
of painful inflammation in the abdomen, 
chest, or joints. These episodes are often 
accompanied by fever and sometimes a rash. 
The first episode usually occurs by the age of 
20 years, but in some cases, the initial attack 
occurs much later in life. Typically, episodes 
last 12 to 72 hours and can vary in severity 
and in the length of time Between attacks. 


Whot is the diagnosis? 

1- Behcet's syndrome 

2- Rheumatoid arthritis 

3- Reiter's syndrome 

4- Psoriatic arthropathy 

5- Ankylosing spondylitis 


Answer & Comments 


Answer: 3- Reiter's syndrome 


Dr. Khalid Yusuf El-Zohry - Sohag Teaching Hospital (01118391123) 

Ref MRCPass OE OE 2012 PasTest 2009 PassMedicine 2009 PasTest Exam ReviseMRCP 



133 






























El-zohry MRCP Questions Bank (Port 1) - 2013 


(For my personal use) 


Reiter's syndrome is urethritis, conjunctivitis, 
seronegative arthritis (cannot see, cannot pee, 
cannot climb a tree). The typical patient is a 
young man with recent urethritis or 
dysentery. The seronegative arthritis is usually 
a mono or oligoarthritis. 

Other features are anterior uveitis, 
keratoderma blenorrhagica (brow n abscesses 
on palms and soles), mouth ulcers, plantar 
fasciitis and archilles tendinitis 
(enthesopathy), circinate balanitis (painless 
rash) and aortic incompetence. Management 
is usually with rest and NSAIDs. 



Keratoderma Blenorrhagica in Reiter's 

syndrome 



[ Q: 253 ] MRCPass - Rheumatology 

A 50 year old patient with 
rheumatoid arthritis has the following full 
blood count results: Haemoglobin 10.5 g/dL, 
Platelets 450 x 10 9 /l_, White Cell Count 8.5 x 
10 9 /L, MCV 103 fL. 


Which drug is the likely cause of this? 

1- Infliximab 

2- Ciclosporin 

3- Leflunomide 


4- Prednisolone 


5- Methotrexate 


Answer & Comments 

Answer: 5- Methotrexate 

Methotrexate may lead to macrocytosis as a 
result of B12 or folate deficiency. It may also 
be associated with bone marrow suppression, 
causing leucopenia or thombocytopaenia. 
Methotrexate may also cause mouth ulcers, 
stomatitis, cough and dyspnoea. 


^ [ Q: 254 ] MRCPass - Rheumatology 

wit 

# A 32 year old man has had a year's 
history of bilateral hip pains and back pains. 
There is not past medical history of trauma to 
the back. Non steroidal anti-inflammatory 
drugs helped to relieve his symptoms. 

What is the likely diagnosis? 

1- Gluteus medius tendonitis 

2- Osteoarthritis 

3- Hip fracture 

4- Sacroilitis 

5- Osteoarthrosis 


Answer & Comments 

Answer: 4- Sacroilitis 

Pain and stiffness in the lower back or 
buttocks, especially in the morning is typical of 
sacroilitis. It is typically helped by NSAIDS or 
steroids. X rays will help to confirm the 
diagnosis. It is associated with various 
inflammatory diseases e.g. ankylosing 
spondylitis, psoriatic arthritis. 



[ Q: 255 ] MRCPass - Rheumatology 

A 35 year old lady presents with 
stiffness, pain and swelling of her hands and 
wrists. On examination she has firm 
subcutaneous nodules over her elbow s, 
swelling of her wrists, ulnar deviation at the 
metacarpophalangeal joints, dinner fork 
deformity and swelling of her proximal 
interphalangeal joints. 



Dr. Khalid Yusuf El-Zohry - Sohag Teaching Hospital (01118391123) 

Ref MRCPass OE OE 2012 PasTest 2009 PassMedicine 2009 PasTest Exam 


ReviseMRCP 





























El-zohry MRCP Questions Bank (Port 1) - 2013 


(For my personal use) 


Investigations reveal a normocytic, 
normochromic anaemia, elevated ESR and 
CRP. 

Which is the most commonly used test which 
could determine whether she is sero-positive? 

1- IgA antibody 

2- IgM antibody 

3- IgE antibody 

4- IgD antibody 

5- IgG antibody 

Answer & Comments 

Answer: 2- IgM antibody 

The patient has features of sero positive 
rheumatoid arthritis (presence of rheumatoid 
factor). Rheumatoid factor is a circulating 
antibody directed against the Fc fragment of 
immunoglobulin. The antibody may IgM, IgG, 
or IgA. The commonly employed test detects 
the IgM rheumatoid factor. 


The patient has Sjogren's syndrome. Sjogren 
syndrome is a chronic autoimmune disorder 
characterized by xerostomia (dry mouth), 
xerophthalmia (dry eyes), and lymphocytic 
infiltration of the exocrine glands. This triad is 
also known as the sicca complex. 

The Schirmer test is probably the only test 
available in the ED to strongly support or 
refute suspicion of Sjogren syndrome. A test 
strip of filter paper is placed near the lower 
conjunctival sac to measure tear formation. A 
positive test occurs when less than 5 mm of 
filter paper is wet after 5 minutes. 

Rheumatoid factor, ANA, Ro and La antigens 
are commonly present. 

As a result of the lymphocytic infiltration, 10% 
of patients may develop pseudolymphoma, a 
lymphoproliferative process. Approximately 
10% of these patients can develop non- 
Hodgkin lymphoma (1% of all patients with 
Sjogren syndrome). 


[ Q: 256 ] MRCPass - Rheumatology 

A 32 year old man has a 6 month 
history of dry eyes and mouth. On 
examination, there was evidence of 
keratoconjunctivitis, parotid gland 
enlargement and a Schirmer's test is positive. 

His blood tests reveal a positive ANA, Ro and 
La extra nuclear antigens are also positive. 

What is the diagnosis? 

1- SLE 

2- Rheumatoid arthritis 

3- Sjogren's syndrome 

4- Wegener's granulomatosis 

5- Pulmonary eosinophilia 



[ Q: 257 ] MRCPass - Rheumatology 

A 38 year old lady presents with 
myalgia and lethargy. Her blood tests show a 
positive ANA with a titre of 1:1024 and 
rheumatoid factor is negative. 

The CK is raised at 360 U/l. Extranuclear 
antigen tests show a negative Ro and negative 
La, negative Scl70 and positive ribonuclear 
protein antibody at 160 units. 

What is the diagnosis? 

1- Polymyalgia rheumatica 

2- Polymyositis 

3- Scleroderma 

4- Systemic lupus erythematosus 

5- Mixed connective tissue disease 



Answer & Comments 


Answer: 3- Sjogren's syndrome 


Answer & Comments 


Answer: 5- Mixed connective tissue disease 


Dr. Khalid Yusuf El-Zohry - Sohag Teaching Hospital (01118391123) 

Ref MRCPass OE OE 2012 PasTest 2009 PassMedicine 2009 PasTest Exam 




ReviseMRCP 

135 

























El-zohry MRCP Questions Bank (Port 1) - 2013 


(For my personal use) 


A positive ANA (speckled pattern), raised CK 
and positive anti RNP antibody suggests mixed 
connective tissue disease. 

[ Q: 258 ] MRCPass - Rheumatology 

A 50 year old man presents to the 
renal team with uraemic symptoms. He also 
has markedly reduced range of movement at 
the spine with a Schober's test of 10 mm. 

His CRP is 102 mg/dl and ESR 98 mm/hr. Urine 
dipstick shows proteinuria ++++. 

He has had treatment with penicillamine in 
the past for arthritis. He takes regular 
ibuprofen. 

What is the most likely couse of his renal 
symptoms? 

1- Nephritic syndrome 

2- NSAID nephropathy 

3- Scleroderma 

4- AL amyloid 

5- AA amyloid 



Answer & Comments 

Answer: 5- AA amyloid 

Nephrotic range proteinuria and renal failure 
in the context of a prolonged untreated 
inflammatory response suggests AA amyloid. 
Since circulating serum AA is the precursor of 
AA amyloid deposits, reduction of the 
precursor protein is the most rational 
approach at present for the management of 
amyloidosis. 

Prevention of amyloidosis is preferable to 
treatment of the established disease. Thus 
aggressive therapy of rheumatic diseases such 
as RA and JCA is desirable in this context. 


[ Q: 259 ] MRCPass - Rheumatology 


A 45 year old woman presents with 
confusion. On examination she was pyrexial, 



had livedo reticularis had a blood pressure of 
190/100 mmHg. 

Examination of the abdomen revealed left 
flank tenderness. 

Investigations revealed: 

Hb 13.9 g/dL 

white cell count 6.5 x 10 9 /l_ 

platelet count 110 x 10 9 /L 

serum creatinine 95 umol/L 

urine dipstick analysis: blood +++, protein + 

Which one of the following tests is likely to be 
positive? 

1- Anti Ro antibody 

2- Anti GBM antibody 

3- Anti cardiolipin antibody 

4- ASOT 

5- ANCA 


Answer & Comments 

Answer: 3- Anti cardiolipin antibody 

The diagnosis is SLE and antiphopholipid 
syndrome. The presentation would be 
consistent with renal vein thrombosis (flank 
pain and proteinuria). Antiphospholipid 
syndrome (APS) is a disorder characterized by 
recurrent venous or arterial thrombosis 
and/or fetal losses associated with typical 
laboratory abnormalities. These include 
persistently elevated levels of antibodies 
directed against membrane anionic 
phospholipids (ie, anticardiolipin [aCL] 
antibody, antiphosphatidylserine). 

Vascular thrombosis - DVT, Ml, CVA or 
miscarriages may occur. Other features are 
nonthrombotic neurologic symptoms, such as 
migraine headaches, chorea, seizures, 
transverse myelitis, Guillain-Barre syndrome, 
thrombocytopenia or hemolytic anemia, 
Livedo reticularis, Avascular necrosis of bone 
and Pulmonary hypertension. 


Dr. Kholid Yusuf El-Zohry - Sohog Teaching Hospital (01118391123) 

Ref MRCPass OE OE 2012 PasTest 2009 PassMedicine 2009 PasTest Exam ReviseMRCP 


























El-zohry MRCP Questions Bank (Port 1) - 2013 


(For my personal use) 


Aspirin or warfarin is recommended for 
patients with thrombotic syndromes. 


[ Q: 260 ] MRCPass - Rheumatology 

A 60 year old lady has 
polyarthropathy due to Rheumatoid Arthtitis. 

Which one of the following molecules ploys o 
central role in its pathogenesis? 

1- IFN gamma 

2- Interleukin 8 



The diagnosis is CREST syndrome. Renal 
hypertensive crisis is more common in diffuse 
systemic sclerosis and pulmonary 
hypertension is more common in limited 
cutaneous scleroderma. 



[ Q: 262 ] MRCPass - Rheumatology 

An 18 year old girl developed 
pulmonary haemorrhage and subsequent 
acute renal failure requiring dialysis. A renal 
biopsy shows crescentic glomerulonephritis. 


3- TNF alpha 

4- Endotoxin 

5- Nitric oxide 

Answer & Comments 

Answer: 3- TNF alpha 

In the context of rheumatoid arthritis, TNF a 
has involvement in cytokine regulation, cell 
recruitment, angiogenesis, and tissue 
destruction. Hence anti TNF alpha antibodies 
such as infliximab are used in therapy. 


^ [ Q: 261 ] MRCPass - Rheumatology 

fi - 

# A 50 year old woman presents with 
breathlessness. She has a 2 year history of 
Raynaud's. On examination, she had 
telangiectasia and tight skin around the 
mouth. Her investigations show an ESR of 70 
mm/hr and positive anti-centromere 
antibodies. 


Which one of the following antibodies is likely 
to be present? 

1- Anti mitochondrial 

2- Anti centromere 

3- Anti nuclear 

4- Anti phospholipid 

5- Anti myeloperoxidase 

Answer & Comments 

Answer: 5- Anti myeloperoxidase 

This patient manifests a pulmonary renal 
syndrome which is commonly due to an ANCA 
positive vasculitis. P ANCA which correlates 
with antimyeloperoxidase (MPO) antibodies, 
is highly sensitive and specific tow ards rapidly 
progressive glomerulonephritis and 

haemorrhagic alveolar capillaritis. Less 
commonly this could be due to Goodpasture's 
syndrome (anti GBM antibodies). 


Which of the following is a typical late 
complication of this disorder? 

1- Renal hypertensive crisis 

2- Lung malignancy 

3- Pulmonary hypertension 

4- Myositis 

5- Mitral regurgitation 


Answer & Comments 


Answer: 3- Pulmonary hypertension 


[ Q: 263 ] MRCPass - Rheumatology 

A 65 year old man with chronic 
leukaemia presents with symptoms of gout. 
He was given Allopurinol. 

How does Allopurinol prevent accumulation of 
uric acid? 

1- By increasing uric acid metabolism 

2- By enhancing its solubility 

3- By inhibiting purine synthesis 

4- By inhibiting pyrimidine synthesis 



Dr. Khalid Yusuf El-Zohry - Sohag Teaching Hospital (01118391123) 

Ref MRCPass OE OE 2012 PasTest 2009 PassMedicine 2009 PasTest Exam 




ReviseMRCP 

137 
































El-zohry MRCP Questions Bank (Port 1) - 2013 


(For my personal use) 


5- By inhibiting xanthine oxidase 

Answer & Comments 

Answer: 5- By inhibiting xanthine oxidase 


Transient osteoporosis of the hip is also a 
recognised cause of hip pain in pregnancy, but 
plain radiographs would usually show marked 
unilateral osteopenia of the femoral head and 
acetabulum. 


Allopurinol inhibits xanthine oxidase, the 
enzyme that catalyzes the conversion of 
hypoxanthine to xanthine and of xanthine to 
uric acid. Hypoxanthine and xanthine are 
breakdown products of purine. 


o 

I II >-H 

H 

Hypoxanthine 


Xanthine 

oxidase 


o 

ii 

j j >- h 

I > 

H H 

Xanthine 


OH H 

' l 

■ I II > 

H 

Allopurinol 


o 

ii 

a 


1 11 ; 


C-H 


Xanthine 


Xanthine 

oxidase 


Hv n^ C ^c^ n \ 

II c-o 

l H H 

Uric acid 


[ Q: 264 ] MRCPass - Rheumatology 

A 32 year old woman in the third 
trimester of her second pregnancy develops 
acute onset right groin pain. 

On examination, all right hip movements are 
painful and she is tender in the right groin and 
over the greater trochanter. Lumbar spine 
examination shows an exaggerated lordosis 
only and there are no neurological lower limb 
deficits. Initial plain films of the hip are 
normal. 

The most likely diagnosis is : 

1- Septic arthritis 

2- Hip dislocation 

3- Avascular necrosis of the hip 

4- Pregnancy exacerbating osteoarthritis 

5- Pseudogout 



Answer & Comments 

Answer: 3- Avascular necrosis of the hip 

This is a classical presentation of avascular 
necrosis of the femoral head in pregnancy. 


The arterial supply to the femoral head is 
easily damaged with any femoral neck 
fracture displacement. 

Atraumatic causes of this are : Alcohol abuse, 
Chemotherapy, Chronic liver disease, 
Corticosteroids, Gaucher disease, Gout, 
Hemoglobinopathy (eg, sickle cell disease), 
Metabolic bone disease, Pregnancy, Radiation, 
Systemic lupus erythematosus, Vasculitis. 


[ Q: 265 ] MRCPass - Rheumatology 

A 55 year old woman with psoriasis 
has significant joint pains. 

Which one of the following is effective in the 
treatment of psoriatic arthropathy? 

1- Codeine phosphate 

2- Methotrexate 

3- Beta interferon 

4- Capacitabine 

5- Buprenorphine 



Answer & Comments 

Answer: 2- Methotrexate 

NSAIDs, sulphasalazine, methotrexate and TNF 
alpha antagonists are useful in psoriatic 
arthropathy. 


[ Q: 266 ] MRCPass - Rheumatology 

A 65 year old woman attends has a 
12 week history of lethargy, neck pains and 
weakness in the lower limbs. 

She has a long history of lower back pains and 
generalised osteoarthritis. She takes 
diclofenac regularly. 

On examination there is wasting of her upper 
limbs. Tone mildly increased in the lower 



Dr. Khalid Yusuf El-Zohry - Sohag Teaching Hospital (01118391123) 

Ref MRCPass OE OE 2012 PasTest 2009 PassMedicine 2009 PasTest Exam ReviseMRCP 



138 


































El-zohry MRCP Questions Bank (Port 1) - 2013 


(For my personal use) 


limbs. There is inversion of right supinator 
reflex, triceps, knee and ankle jerks are brisk 
bilaterally. Right plantar is extensor and the 
left is flexor. 

Investigations show: 

Hb 11.6 g/l 

WCC 8 x 10 9 /L 

Plat 160 x 10 9 /L 

ESR 73 mm 

CRP 12 mg/I 

Na 138 mmol/I 

K 4.4 mmol/l 

Urea 5.8 mmol/l 

Creatinine 95 umol/l 

Protein 83 g/l 

Albumin 32 g/l 

Ca 2.33 mmol/l 

X ray cervical spine shows extensive 
osteophytes 

What is the likely diagnosis? 

1- Ankylosing spondylitis 

2- Polymyalgia rheumatica 

3- Cervical spondyloarthropathy 

4- Multiple sclerosis 

5- Syringomyelia 

Answer & Comments 

Answer: 3- Cervical spondyloarthropathy 


On examination, he appears unwell and has a 
temperature of 38°C. 

Investigations reveal: 

Hb 8.5 g/dL 

erythrocyte sedimentation rate 95 mm/hr 

serum creatinine 220 pmol/L 

urine analysis: blood ++ 

urine microscopy: white cells & red cell casts 
seen 

Which one of the following is the likely 
diagnosis? 

1- Multiple myeloma 

2- Antiphospholipid syndrome 

3- Takayasu's arteritis 

4- Polyarteritis nodosa 

5- Goodpasture's syndrome 

Answer & Comments 

Answer: 4- Polyarteritis nodosa 

This patient has a mononeuritis multiplex, 
fever and nephritic renal involvement 
suggesting a diagnosis of polyarteritis nodosa. 
PAN causes transmural necrotizing 
inflammation of small-sized or medium-sized 
muscular arteries. PAN is a rare condition. 

Although the causes are unknown in most 
cases, there is an association with: Hep B 
virus, Hep C virus, HIV, Cytomegalovirus, 
Parvovirus B19 and Human T-lymphotrophic 
virus. 


Cervical spondylosis may present with 
associated pains in the neck radiating down 
the arms and back. There may be upper motor 
neuron signs in the upper and lower limbs. 


[ Q: 267 ] MRCPass - Rheumatology 

A 70 year man presents with right 
foot drop, hand numbness, fevers, malaise, 
weight loss, polymyalgia and diffuse joint 
pains for 2 months. 



Approximately 20% of patients with classic 
PAN are positive for P-ANCA. Steroids 
(prednisolone) and immunosuppressive 
(cyclophosphamide) medications form the 
backbone of therapy. Plasma exchange is 
useful as a second-line treatment in PAN 
refractory to conventional therapy. 


Dr. Khalid Yusuf El-Zohry - Sohag Teaching Hospital (01118391123) 

Ref MRCPass OE OE 2012 PasTest 2009 PassMedicine 2009 PasTest Exam ReviseMRCP 



139 























El-zohry MRCP Questions Bank (Port 1) - 2013 


(For my personal use) 


[ Q: 268 ] MRCPass - Respiratory 

A man presents with symptoms 
suggestive of a pneumonia. The CXR confirms 
this. 

Which of the following features suggests poor 
prognosis? 

1- Respiratory rate of 20 

2- Temperature 38°C 

3- Age 60 

4- Urea of 10 

5- MTS score of 9 out of 10 



Answer & Comments 


Answer: 4- Urea of 10 


Answer & Comments 

Answer: 4- PH <7.2 

Although a high protein also points tow ards 
an exudates, a low pH is the best marker of 
infection in an effusion requiring chest drain 
insertion. 

[ Q: 270 ] MRCPass - Respiratory 

A 75 year old heavy smoker presents 
to the hospital with breathlessness, wheezing 
and a cough with yellow sputum. 

He has the following investigations: Hb 18 
g/dl, WCC 12 x 10 9 /L, ABGs show a pH of 7.38, 
p0 2 of 8.5 kPa, pC0 2 of 7 kPa. 

What is the diagnosis? 



The CURB-65 score for poor prognosis in 
pneumonia are : 

confusion (defined as an AMT of 8 or 
less) 

urea greater than 7 mmol/I 
respiratory rate of 30 breaths per 
minute or greater 

blood pressure less than 90 systolic or 
diastolic blood pressure 60 or less 
age 65 or older 


1- Bronchiectasis 

2- Chronic obstructive pulmonary disease 

3- Mesothelioma 

4- Tuberculosis 

5- Cryptogenic fibrosing alveolitis 

Answer & Comments 

Answer: 2- Chronic obstructive pulmonary 
disease 


[ Q: 269 ] MRCPass - Respiratory 

A 60 year old lady presents with 
shortness of breath. A chest X ray confirms 
that she has a right sided pleural effusion. An 
Aspirate was taken and the sample was sent 
for several tests. 

Which one of the following is an indication for 
a chest drain? 

1- High protein 

2- Low glucose 

3- Low LDH 

4- PH <7.2 

5- Blood stains 



This patient has type II respiratory failure, 
without an acidosis, suggesting chronic C02 
retention. He also is a heavy smoker and has 
polycythaemia, making COPD most likely. 



[ Q: 271 ] MRCPass - Respiratory 

A 45 year old man presents with a 
chronic cough productive of copious amounts 
of thick yellow sputum and occasional 
haemoptysis. He also has had multiple 
episodes of chest infection. On examination 
he has clubbing and on auscultation over the 
lung bases, coarse crepitations are heard. 


Which investigation will reveal the diagnosis? 


1- Chromosome testing 

2- Bronchoscopy 


Dr. Khalid Yusuf El-Zohry - Sohag Teaching Hospital (01118391123) 

Ref MRCPass OE OE 2012 PasTest 2009 PassMedicine 2009 PasTest Exam ReviseMRCP 



141 
































El-zohry MRCP Questions Bank (Port 1) - 2013 


(For my personal use) 


3- Sweat test 

4- Maxillary sinus x rays 

5- High resolution CT 

Answer & Comments 

Answer: 5- High resolution CT 


The diagnosis is bronchiectasis. HRCT is likely 
to show bronchial dilatation and wall 
thickening. 



HRCT showing bronchiectasis 


[ Q: 272 ] MRCPass - Respiratory 

A 40 year old man has emphysema. 
He is found to have the SS phenotype. 



What is his alpha 1 antitrypsin level likely to 
be? 


1 - 10 % 

2- 15% 

3- 25% 

4- 50% 

5- 75% 


Genotype 

Prevalence 

% 

Reduction AAT 
level (%) 

MM 

83 

0 

MS 

7 

20 

MZ 

4 

40 

SS 

1 

40 

SZ 

0.1 

70 

ZZ 

0.03 

90 


[ Q: 273 ] MRCPass - Respiratory 

A 70 year old man with known 
chronic obstructive pulmonary disease (COPD) 
is admitted with symptoms of worsening 
breathlessness and confusion. Following a 
dose of 200mg iv hydrocortisone and two 
doses of salbutamol and atrovent nebulisers, 
arterial blood gases were taken. 

His arterial blood gases are as follows: pH 
7.18, p0 2 6.6 kPa , pC0 2 12.0 kPa. 

What should be done next? 

1- Repeat dose of corticosteroids 

2- Mechanical ventilation 

3- 2 litres oxygen 

4- Monteleukast 

5- Magnesium infusion 2g 

Answer & Comments 

Answer: 2- Mechanical ventilation 

This patient has type II respiratory failure with 
acidosis, and hence should be considered for 
ventilation, either straight away or with a trial 
of NIPPV beforehand. 



Answer & Comments 

Answer: 4- 50% 

The normal genotype is MM and levels of 
enzyme is (100%). The relevant enzyme levels 
are MS (75%), MZ (55%), SS (50%) ZZ (15%.) 



[ Q: 274 ] MRCPass - Respiratory 

A 40 year old woman has been 
breathless and is undergoing investigations for 
the cause. She has a previous history of being 
a smoker and has a dry cough as well. Her 
ABGs show a p0 2 of 9 kPa and pC0 2 of 5 kPa. 
Lung function tests show reduced vital 
capacity, reduced TLCO but increased KCO. 


What is the likely diagnosis? 



Dr. Khalid Yusuf El-Zohry - Sohag Teaching Hospital (01118391123) 

Ref MRCPass OE OE 2012 PasTest 2009 PassMedicine 2009 PasTest Exam ReviseMRCP 









































El-zohry MRCP Questions Bank (Port 1) - 2013 


(For my personal use) 


1- COPD 

2- Bronchial obstruction from a tumour 

3- Kyphoscoliosis 

4- Obstructive sleep apnoea 

5- Fibrosing alveolitis 


Answer & Comments 

Answer: 3- Kyphoscoliosis 

In this scenario, the lung function indicates 
that the surface area for exchange are 
inadequate (low TLCO) hence suggesting that 
lung expansion is reduced. How ever, cardiac 
output is increased to compensate, and 
henced KCO is increased. Kyphoscoliosis 
would restrict lung expansion, or a 
neuromuscular disorder (eg spina bifida) could 
cause this. 



[ Q: 275 ] MRCPass - Respiratory 

A 50 year old woman is admitted 
with infective exacerbation of asthma. She 
responds to medical treatment but Aspergillus 
fumigatus is eventually cultured from her 
sputum. 


Further investigations show serum total IgE 
level was elevated at 350 ng/ml (normal 40- 
180 ng/ml), RAST to Aspergillus fumigatus was 
class III, Aspergillus fumigatus precipitins are 
negative. 

What is the most appropriate management? 


1- No change in medication 

2- High dose oral corticosteroids 

3- CT scan of the chest 


4- Chest X ray 

5- Itraconazole 


Answer & Comments 

Answer: 1- No change in medication 

With negative Aspergillus fumigatus 
precipitins and serum total IgE less than 1000 


ng/ml, this patient is unlikely to have Allergic 
Bronchopulmonary Aspergillosis (ABPA). 

Features which are found commonly in 
asthmatics without ABPA include: 

Positive immediate skin reactivity to 
Aspergillus fumigatus, which is present in 20- 
30% of asthmatics Positive serum precipitins 
to Aspergillus, which occur in 10% of 
asthmatics without ABPA Recurrent mucoid 
impaction and atelectasis Peripheral blood 
eosinophilia and elevation of serum total IgE. 


[ Q: 276 ] MRCPass - Respiratory 

A 60 year old man presents with 
small amounts of haemoptysis and 
breathlessness. He does not have pleuritic 
chest pains and has normal oxygen 
saturations. 

In considering the potential diagnosis, 
haemoptysis can occur with which of the 
following diagnosis? 

1- Pulmonary fibrosis 

2- Melanoma 

3- Goitre 

4- Thymoma 

5- Aspergilloma 



Answer & Comments 

Answer: 5- Aspergilloma 

Haemoptysis can be caused by pulmonary 
embolus, tuberculous infection, aspergilloma, 
bronchial carcinoma, Goodpasture's syndrome 
and Wegener's granulomatosis. 



[ Q: 277 ] MRCPass - Respiratory 

A 30 year old man gives a six-month 
history of worsening breathlessness and 
coughs up half a cupful of sputum daily. He 
was occasionally wheezy with viral illnesses as 
a young child, works in a factory and has 


Dr. Khalid Yusuf El-Zohry - Sohag Teaching Hospital (01118391123) 

Ref MRCPass OE OE 2012 PasTest 2009 PassMedicine 2009 PasTest Exam ReviseMRCP 



143 






























El-zohry MRCP Questions Bank (Port 1) - 2013 


(For my personal use) 


smoked 25 cigarettes per day for the last four 
years. 

The most likely diagnosis is: 

1- Lung carcinoma 

2- Bronchiectasis 

3- Asthma Asbestosis 

4- Chronic obstructive pulmonary disease 

5- Rheumatoid lung 


Answer & Comments 

Answer: 2- Bronchiectasis 

Bronchiectasis is most likely due to the 
extensive amounts of sputum production. In a 
young person, cystic fibrosis and 
hypogammaglobulinaemia should be 
considered. 


[ Q: 278 ] MRCPass - Respiratory 

A 60 year old man has smoked for 10 
years. He has a longstanding dry cough and 
expiratory wheeze. 

Examination reveals scattered rhonchi and 
reduced lung expansion. 

The most likely diagnosis is: 

1- Asthma 

2- Lung carcinoma 

3- Chronic bronchitis 

4- Bronchiectasis 

5- Extrinsic allergic alveolitis 



Answer & Comments 

Answer: 3- Chronic bronchitis 

The heavy smoking history and wheeze 
suggests chronic bronchitis. 

[ Q: 279 ] MRCPass - Respiratory 

A 14 year old boy develops red eye 
and rhinitis frequently during the start of the 
summer. 



What is the likely triggering agent? 

1- House dust mite 

2- Grass pollen 

3- Willow pollen 

4- Isocyanates 

5- Coal dust 


Answer & Comments 

Answer: 2- Grass pollen 

Grass pollen is released late May till August, 
and willow pollen is released from March till 
June. As the allergic rhinitis is triggered during 
the start of summer, this is most likely to be 
due to grass pollen. 


[ Q: 280 ] MRCPass - Respiratory 

A 65 year old man presents with 
breathlessness that has got gradually worse 
over three months. He has long-standing atrial 
fibrillation, for which he takes warfarin and 
amiodarone. 

On examination his pulse is 100/min in AF, 
oxygen saturation was 90% on air. His JVP was 
not raised and he has fine bibasal crackles. 
Arterial blood gas saturation showed mild 
hypoxia and a pulmonary function test 
revealed a moderate restrictive picture. 

The most likely diagnosis is: 

1- Bronchiectasis 

2- Pulmonary embolism 

3- Pulmonary haemorrhage 

4- Amiodarone induced interstitial lung 
disease 

5- Congestive cardiac failure 



Answer & Comments 

Answer: 4- Amiodarone induced interstitial 
lung disease 

The case scenario would fit amiodarone 
induced interstitial lung disease or pulmonary 


Dr. Khalid Yusuf El-Zohry - Sohag Teaching Hospital (01118391123) 

Ref MRCPass OE OE 2012 PasTest 2009 PassMedicine 2009 PasTest Exam ReviseMRCP 



144 






























El-zohry MRCP Questions Bank (Port 1) - 2013 


(For my personal use) 


fibrosis. This may take several months or years 
to develop. Lung function tests may show a 
restrictive picture with reduced transfer 
factor. A high resolution CT in this case is likely 
to show diffuse "ground glass" opacities or 
interlobular septal thickening. 


[ Q: 281 ] MRCPass - Respiratory 

A 65 year old man has been 
confirmed to have lung cancer. 

Which form of cancer has the worst 
prognosis? 

1- Small cell 

2- Squamous cell 

3- Adenocarcinoma 

4- Large cell 

5- Prostate metastasis 



Answer & Comments 

Answer: 1- Small cell 

Small cell lung cancer has the worst prognosis 
out of all lung cancers and is rarely suitable for 
surgical resection. Even when diagnosed early 
the 2-year survival is in the order of 20-25%. 
The main form of therapy is chemotherapy. 

Small cell cancer is associated with syndrome 
of inappropriate antidiuretic hormone (ADH) 
and squamous cell cancer is associated with 
paraneoplastic hypercalcaemia through 
increased parathyroid-related hormone. 75% 
of small cell cancers arise in the proximal 
airways. 


[ Q: 282 ] MRCPass - Respiratory 

A 22 year old female presents with a 
chest infection. She is unable to complete a 
sentence and her peak flow rate was 40% of 
her normal level. She is treated with high flow 
oxygen, nebulised bronchodilators and oral 
steroids but this is associated with little 
change in her condition. 



Which of the following treatments, given 
intravenously , would be the most appropriate 
for this patient? 

1- Aminophylline 

2- Augmentin 

3- Hydrocortisone 

4- Magnesium 

5- Salbutamol 


Answer & Comments 

Answer: 4- Magnesium 

IV magnesium is recommended by the British 
Thoracic Society in severe exacerbation of 
asthma. A dose of 2g (8 mmol) is given as a 
bolus iv dose in the acute presentation period. 

[ Q: 283 ] MRCPass - Respiratory 

A 60 year man has been a smoker of 
20 a day for 30 years. He has a cough and 
difficulty in breathing for 3 days which has 
worsened and he presents to hospital. He has 
home nebulisers but not home oxygen. 

On examination, his oxygen saturations are 
95%, blood pressure 110 / 60 mmHg and 
respiratory rate 32. He has diffuse wheezes 
and is using his respiratory muscles. Arterial 
blood gases done on 6 litres of oxygen show : 

pH 7.25 

p0 2 16 kPa 

pC0 2 8.2 kPa 

What should be the next management step? 

1- Non invasive ventilation 

2- Intravenous antibiotics 

3- Intubation and ventilation 

4- Reduce inspired oxygen concentration 

5- Increase oxygen concentration 



Dr. Khalid Yusuf El-Zohry - Sohag Teaching Hospital (01118391123) 

Ref MRCPass OE OE 2012 PasTest 2009 PassMedicine 2009 PasTest Exam 




ReviseMRCP 

145 


























El-zohry MRCP Questions Bank (Port 1) - 2013 


(For my personal use) 


Answer & Comments 

Answer: 4- Reduce inspired oxygen 

concentration 

This man with Chronic Obstructive Pulmonary 
Disease (COPD) has blood gases showing type 
II respiratory failure with acidosis. He has been 
placed on 6 litres of oxygen, which is too much 
for a patient with severe COPD as the history 
of home nebuliser use suggests. 

His respiratory drive is suppressed by too 
much oxygen inspired, and hence reduction to 
a lower concentration (e.g. 1 - 2 litres) to 
maintain a p02 above 8.5 kPa is 
recommended. 


reveals a left-sided pneumothorax with a lung 
edge measured 5 cm away from the chest 
wall. 

The most appropriate management of his 
condition is: 

1- Review with daily chest radiographs 

2- Intercostal tube drainage 

3- Refer to a respiratory outpatient clinic 

4- CT chest 

5- Simple aspiration 

Answer & Comments 

Answer: 2- Intercostal tube drainage 


[ Q: 284 ] MRCPass - Respiratory 

A 70 year woman has a history of dry 
cough for 2 months. She has lost 5 kg of 
weight over the 2 months. Her chest X ray 
shows a left apical shadowing. Blood tests 
reveal a raised white cell count of 16. She has 
not managed to cough up any sputum. 

Which test should be performed? 

1- CT scan of the chest 

2- Serum ANCA 

3- Ultrasound of the chest 

4- Kveim test 

5- Bronchoscopy 



Answer & Comments 

Answer: 5- Bronchoscopy 

This patient is likely to have TB due to the 
apical shadowing. In a patient who is unable 
to expectorate sputum, bronchoscopy with 
lavage (send for AFB) should be performed to 
confirm the diagnosis. 


[ Q: 285 ] MRCPass - Respiratory 

A 65 year old man with emphysema 
presented with increasing dyspnoea and left¬ 
sided pleuritic chest pain. A chest radiograph 



The lung edge measurement suggests a 
greater than 50% pneumothorax. A smaller 
pneumothorax may be amenable to 
aspiration. In the context of chronic 
underlying respiratory disease such as chronic 
obstructive pulmonary disorder and a large 
pneumothorax, patients are best managed by 
pleural drainage. 


[ Q: 286 ] MRCPass - Respiratory 

A 70 year old man attends the 
hospital with a history of proximal muscle 
weakness. He also gives a history of cough of 
12 weeks duration and complains of pain of 
the small joints of the hands. He has small 
haemorrhages in the nail folds, but is not 
clubbed. On examination of the chest he has 
bibasal crackles, and a chest radiograph 
reveals diffuse reticular infiltrates. Lung 
function tests confirm a restrictive pattern. 

What is the likely cause of his interstitial lung 
disease? 

1- SLE 

2- Dermatomyositis 

3- Ankylosing spondylitis 

4- Cryptogenic fibrosing alveolitis 

5- Amiodarone induced fibrosis 



Dr. Khalid Yusuf El-Zohry - Sohag Teaching Hospital (01118391123) 

Ref MRCPass OE OE 2012 PasTest 2009 PassMedicine 2009 PasTest Exam ReviseMRCP 



146 






























El-zohry MRCP Questions Bank (Port 1) - 2013 


(For my personal use) 


Answer & Comments 

Answer: 2- Dermatomyositis 

In polymyositis and dermatomyositis patients 
often develop proximal muscle weakness and 
of pain in the small joints of the fingers. They 
may have ragged cuticles and haemorrhages 
at the finger nail folds. Interstitial lung disease 
can occur. Underlying malignancy (lungs, 
ovaries, breasts and stomach) is present in 5% 
of cases. 



Bronchiectasis- Airway dilatation on the HRCT 


[ Q: 287 ] MRCPass - Respiratory 

A 40 year old man is referred for 
investigation of the cause of chronic cough. He 
describes long standing sputum production 
which is streaked with blood. There is no fever 
or night sweats, and he does not smoke. How 
ever there is a family history of such 
symptoms. Chest X ray shows mild bilateral 
lower zone shadowing. 



[ Q: 288 ] MRCPass - Respiratory 

A 40 year old man has a 3 month 
history of cough and dyspnoea. He smokes 20 
a day. There is no history of asbestos 
exposure. 

His WBC count is 24 x 10 9 /l_ with 70% 
neutrophils and 3.0 x 10 9 /L (5%) eosinophils. 
IgE level is elevated. 



Which is the most appropriate investigation? 

1- High resolution CT 

2- Exercise tolerance test 

3- Echocardiography 

4- Bronchoscopy and biopsy 

5- Serum precipitins 


He has decreased breath sounds 
corresponding to parenchymal infiltrates on 
the CXR. 

Which is the best test to confirm the 
diagnosis? 

1- Sputum for Acid Fast Bacilli 

2- HIV test 


Answer & Comments 

Answer: 1- High resolution CT 

The likely diagnosis is bronchiectasis, and a 
family history is suggestive of cystic fibrosis or 
Kartagener's syndrome. HRCT will help to 
confirm the diagnosis, if changes such as 
dilated central tubular bronchi and mosaic 
oligaemia were seen. 


3- Autoimmune screen 

4- Aspergillus RAST 

5- Stool for ova, cysts, parasites 

Answer & Comments 

Answer: 4- Aspergillus RAST 

The condition described is Allergic Broncho 
Pulmonary Aspergillosis, which is commoner 
among asthmatics and cystic fibrosis patients. 
Eosinophilia and high IgE levels are suggestive 
of this condition. RAST test for antibodies tow 
ards Aspergillus confirms the diagnosis. 


Dr. Khalid Yusuf El-Zohry - Sohag Teaching Hospital (01118391123) 

Ref MRCPass OE OE 2012 PasTest 2009 PassMedicine 2009 PasTest Exam ReviseMRCP 



147 





























El-zohry MRCP Questions Bank (Part 1) - 2013 


(For my personal use) 


^ [ Q: 289 ] MRCPass - Respiratory 

zi - 

# A 45 yearold woman is admitted 
with a 2-day history of fever, rigors and 
breathlessness. She looks extremely unwell 
and is confused, cyanosed, has a respiratory 
rate of 24/min and a systolic blood pressure of 
85 mmHg. 

There is bronchial breathing at her right base, 
where a chest radiograph reveals 
consolidation. 

Which would be the most appropriate 
antibiotic regimen? 

1- Intravenous ceftazidime and intravenous 
gentamicin 


2- Oral erythromycin 


3- 

Intravenous cefotaxime 

and 

oral 


erythromycin 



4- 

Intravenous amoxicillin 

and 

oral 


clarithromycin 




5- Oral amoxicillin and oral clarithromycin 


Answer & Comments 

Answer: 3- Intravenous cefotaxime and oral 
erythromycin 

Severe pneumonia as defined by the British 
Thoracic Society guidelines, is diagnosed when 
there are two of the following features: 
confusion, urea >7mmol/l, respiratory rate 
>30/min, and hypotension (SBP <90mmHg, 
DBP <60mmHg). 

Appropriate treatment is with intravenous 
antimicrobials: cefuroxime 1.5g three times 
daily or cefotaxime lg three times daily daily 
PLUS erythromycin 500 mg four times daily or 
clarithromycin 500mg twice daily. 


^ [ Q: 290 ] MRCPass - Respiratory 

- 

# A 72 year old man presents with a 
history of worsening breathlessness and 
cough. His arterial blood gases show the 
following recordings when taken at room air: 


pH 7.25 

p0 2 6.4kPa 

pC0 2 8.9 kPa 

Bicarbonate 31mmol/L 

What is the most likely diagnosis? 

1- Acute exacerbation of chronic obstructive 
pulmonary disease 

2- Obstructive sleep apnoea 

3- Pulmonary oedema 

4- Pulmonary embolus 

5- Pulmonary fibrosis 

Answer & Comments 

Answer: 1- Acute exacerbation of chronic 
obstructive pulmonary disease 

The blood gas result would be most 
compatible with a patient with severe COPD 
and chronic type 2 respiratory failure with an 
acute exacerbation (Hypoxia, respiratory 
acidodis with raised C02 and metabolic 
compensation). 


[ Q: 291 ] MRCPass - Respiratory 

A 70 year old man presents with a 
chronic cough. He is a heavy smoker of over 
40 cigarettes a day. CXR shows a peripheral 
right-sided lesion. A biopsy which was taken 
shows squamous cell carcinoma. No regional 
lymph nodes are involved. Lung function tests 
show a FEVi of less than 1.5 litres. 

The recommended treatment is: 

1- Chemotherapy 

2- Radiotherapy 

3- Lobectomy 

4- Pneumectomy 

5- Lung transplant 



Answer & Comments 


Answer: 2- Radiotherapy 


Dr. Khalid Yusuf El-Zohry - Sohag Teaching Hospital (01118391123) 

Ref MRCPass OE OE 2012 PasTest 2009 PassMedicine 2009 PasTest Exam ReviseMRCP 



148 




























El-zohry MRCP Questions Bank (Part 1) - 2013 


(For my personal use) 


Although surgical treatment is possible in non 
small cell lung carcinoma, a FEVi of less than 
1.5 litres contraindicates surgery. Hence, 
radiotherapy is recommended. 



[ Q: 292 ] MRCPass - Respiratory 

A 55 year old gentleman has pickw 
ickian syndrome. He has poor exercise 
tolerance of 50 yards and often feels lethargic 
at work. 


Which of the following is the best 
investigation? 

1- Echocardiography to assess cor pulmonale 


1- Normal 

2- Mixed defect 

3- Obstructive defect 

4- Restrictive defect 

5- Suggestive of haemorrhage 


Answer & Comments 

Answer: 4- Restrictive defect 

Reduced FEVi and FVC with normal FEVi ratio 
is compatible with restrictive defect. 

Causes of restrictive lung defect are : 


2- CT scan of the chest 

3- Blood gas 

4- Sleep study 

5- Exercise tolerance test 

Answer & Comments 

Answer: 4- Sleep study 


neurogenic or psychogenic causes 
abnormalities of the thoracic wall 
stiff parenchyma (pulmonary fibrosis) 
loss of lung tissue, e.g. 
pneumonectomy 
displacement 


The diagnosis of obstructive sleep apnoea can 
be made with a sleep study 
(polysomnography). In sleep apnoea, there is 
gross obesity and airways obstruction, 
occasionally leading to type II respiratory 
failure. During the sleep study, > 10 episodes 
of apneic episodes (pauses in breathing) 
satisfies the criteria for obstructive sleep 
apnoea. 

[ Q: 293 ] MRCPass - Respiratory 

A 60 year old man smoker of 35 pack 
years presents with a 6 month history of 
shortness of breath. His past medical history 
includes diabetes and cervical spondylosis. 

Spirometry shows FEVi of 1 litre - 65% 
predicted 

FVC 1.03 litres - 57% predicted 
FEVi/FVC ratio of 95. 

How would you interpret the spirometry 
results? 



[ Q: 294 ] MRCPass - Respiratory 

A 65 year old man has worsening 
breathlessness. He has a past medical history 
of arthritis and palpitations. 

On admission his oxygen saturations were 
90% on air. 

ABGs show p02 9 kPa. pC02 3.5 kPa, pH 7.36. 

His chest X ray shows patchy shadowing in 
both lung peripheries. 

Which one of the following drugs is likely to be 
responsible? 

1- Codeine 

2- Tramadol 

3- Simvastatin 

4- Amiodarone 

5- Prednisolone 



Dr. Khalid Yusuf El-Zohry - Sohag Teaching Hospital (01118391123) 

Ref MRCPass OE OE 2012 PasTest 2009 PassMedicine 2009 PasTest Exam ReviseMRCP 



149 



























El-zohry MRCP Questions Bank (Port 1) - 2013 


(For my personal use) 


Answer & Comments 

Answer: 4- Amiodarone 

The hypoxia and X ray changes suggest 
pulmonary fibrosis. Out of the list of 
medications, amiodarone is the most likely 
candidate. A lung function test with transfer 
factor and also high resolution CT will help to 
confirm the diagnosis of amiodarone related 
pulmonary fibrosis. 


1- Trial of inhaled corticosteroids 

2- Trial of anticholinergics 

3- Serial peak flow measurements 

4- Lung function tests 

5- Chest X ray 

Answer & Comments 

Answer: 3- Serial peak flow measurements 



[ Q: 295 ] MRCPass - Respiratory 

A 65 year old man has been 
diagnosed with chronic obstructive pulmonary 
disease (COPD). Spirometry confirms severe 
COPD with a FEVi of less than 25% predicted. 
In the last year he has been admitted to 
hospital on 6 occasions with COPD 
exacerbation. 


Which one of the following con help to reduce 
hospital admissions? 

1- Tiotropium 

2- Monteleukast 


3- Oral theophyllines 

4- Salmeterol 


5- Hydrocortisone 


Answer & Comments 


Answer: 1- Tiotropium 


The history of wheezing during pollen 
exposure suggests asthma. The best 
diagnostic test for asthma would be 
demonstration of variable airways obstruction 
with serial peak flow measurements. 
Asthmatic patients with exacerbation will 
demonstrate peak flows lower than their 
predicted peak flow, and also a morning dip. 
There is also reversibility if a bronchodilator 
(e.g. salbutamol) is administered. 

[ Q: 297 ] MRCPass - Respiratory 

A 27 year old man presents with 
sudden onset left sided pleuritic chest pain 
and breathlessness. 

On examination he is distressed, tachypnoeic 
and has tracheal displacement to the right 
together with a hyper-resonant percussion 
note on the left side. There were no breath 
sounds on the left. Chest x ray shows a < 2 cm 
left sided pneumothorax. 



Severe COPD is diagnosed if the FEVi is less or 
equal to 30% predicted. Studies have shown 
that patients treated with long acting 
anticholinergic (e.g. tiotropium) have few er 
exacerbations per year. 


[ Q: 296 ] MRCPass - Respiratory 

An 13 year old male has recent onset 
breathlessness. He has a history of wheezing 
particularly during the summer when the 
pollen count is high. 

Which of these tests would help to confirm the 
diagnosis? 



What is the best management procedure? 

1- High dose oxygen 

2- Intercostal drainage tube 

3- Chemical pleurodesis 

4- Aspiration of pneumothorax 

5- Repeat chest x ray in 6 hours 

Answer & Comments 

Answer: 4- Aspiration of pneumothorax 

Treatment of a pneumothorax of < 2 cm 
margin would be initial aspiration. 


Dr. Khalid Yusuf El-Zohry - Sohag Teaching Hospital (01118391123) 

Ref MRCPass OE OE 2012 PasTest 2009 PassMedicine 2009 PasTest Exam ReviseMRCP 



150 

































El-zohry MRCP Questions Bank (Port 1) - 2013 


(For my personal use) 


This is less painful, leads to a shorter duration 
of admission, reduces the need for 
pleurectomy. 


[ Q: 298 ] MRCPass - Respiratory 

A 65 year year old lady presents with 
weight loss and haemoptysis and is admitted 
to hospital. She has been a heavy smoker for 
25 years. A Chest X ray shows a mass in the 
right upper lobe. 

Which is the best test to investigate at 
present? 

1- Sputum cytology 

2- CT scan of the lung 

3- Transbronchial biopsy 

4- Lymph node biopsy 

5- Pleural biopsy 



Answer & Comments 

Answer: 2- Horner's syndrome 

An FEVi of < 1.1 L is a contraindication for 
most cardiothoracic surgical procedures. A 
malignant pleural effusion, distant 
metastases, contralateral mediastinal lymph 
node spread, vocal cord paralysis, phrenic 
nerve paralysis, Horner's syndrome, and SVC 
syndrome are contraindications to surgery in 
lung cancer. 



Left sided Horner's 


Answer & Comments 

Answer: 1- Sputum cytology 

The mass seen on the CXR is likely to be a 
malignancy in view of the history of smoking, 
haemoptysis and weight loss. The initial test 
should be sputum cytology, as the diagnosis 
can be confirmed most easily. Staging CT scans 
can then be done to guide further 
management after this. 


[ Q: 299 ] MRCPass - Respiratory 

A 70 year old man has a diagnosis of 
non small cell lung tumour, and has 
completed a set of investigations. 

Which one of the following is o 
contraindication to lung surgery? 



[ Q: 300 ] MRCPass - Respiratory 

A 20 year old male presents with 
breathlessness and wheezing. 

Which of the following is most likely to suggest 
asthma? 

1- Increased serum IgE 

2- Wheezing induced by smoking 

3- Obstructive picture in the lung function 
tests 

4- Response to prednisolone 

5- Diurnal PEFR variation > 20% 



Answer & Comments 


Answer: 5- Diurnal PEFR variation > 20% 


1- FEVi of 1.7 L (50% predicted) 

2- Horner's syndrome 

3- History of myocardial infarction 

4- Hypercalcaemia 

5- Neuropathy affecting lower limbs 


In asthma, diurnal PEFR variability is due to 
various degrees of bronchial hyperreactivity. 
This is the best indicator of likely asthma. A 
raised IgE indicates atopy but is not diagnostic 
of asthma. 


Dr. Khalid Yusuf El-Zohry - Sohag Teaching Hospital (01118391123) 

Ref MRCPass OE OE 2012 PasTest 2009 PassMedicine 2009 PasTest Exam 




ReviseMRCP 

151 




























El-zohry MRCP Questions Bank (Part 1) - 2013 


(For my personal use) 


^ [ Q: 301 ] MRCPass - Respiratory 

A - 

# A 70 year old patient with COPD 
presents with cough and breathlessness which 
has worsened over 2 months. 

On admission, he has the following arterial 
blood gas results. 

pH 7.32, p0 2 7 kPa, pC0 2 8 kPa, HC0 3 34 
mmol/I, Base Excess -1. 

Which one of the following biochemical states 
fits best? 

1- Chronic respiratory acidosis 

2- Chronic respiratory alkalosis 

3- Chronic metabolic acidosis 

4- Chronic metabolic alkalosis 

5- Acute metabolic acidosis 

Answer & Comments 

Answer: 1- Chronic respiratory acidosis 

There is a mild respiratory acidosis (pH < 3.5 
and pC02 > 6) and base excess is also not 
elevated (in this case only -1). There is 
metabolic compensation, as indicated by a 
high bicarbonate ( >30). The patient's history 
of COPD suggests that he has chronic hypoxia 
and chronic C02 retention, hence causing the 
picture of chronic respiratory acidosis. 

[ Q: 302 ] MRCPass - Respiratory 

«t - 

* A breathless 35 year old woman has 
the following lung function tests: 

FEVi 1.2 L (65%) 

FVC 1.4 L (60%) 

FEVi/FVC ratio = 82%predicted 

TLC = 65% predicted 

RV = 60% predicted 

TLCO = 57% predicted 

KCO = 105% predicted 

What is the most likely diagnosis? 

1- COPD 


2- Bronchiectasis 

3- Cystic fibrosis 

4- Scoliosis 

5- Pneumonia 

Answer & Comments 

Answer: 4- Scoliosis 

The lung function tests show a significant 
restrictive defect. Only kyphoscoliosis or a 
pneumonitis may fit this picture but given the 
normal/high KCO (i.e. after correcting for 
alveolar volumes), the most likely answer is 
kyphoscoliosis as the gas exchange after 
correcting for the alveolar volume is high. 

[ Q: 303 ] MRCPass - Respiratory 

fi - 

# A 70 year old man presented with 
shortness of breath. On examination, he had 
the signs of a large right-sided pleural 
effusion. 

Investigations revealed: Pleural fluid analysis - 
protein 65 g/L 

What is the most likely cause? 

1- Congestive cardiac failure 

2- Nephrotic syndrome 

3- Non specific pericarditis 

4- Liver cirrhosis 

5- Mesothelioma 

Answer & Comments 

Answer: 5- Mesothelioma 

It is a case of exudative pleural effusion 
(protein >30 g/l). Mesothelioma is the most 
likely cause in this case. Other causes are: 
malignancies, infection, autoimmune 
conditions. 


Dr. Khalid Yusuf El-Zohry - Sohag Teaching Hospital (01118391123) 

Ref MRCPass OE OE 2012 PasTest 2009 PassMedicine 2009 PasTest Exam ReviseMRCP 



152 




























El-zohry MRCP Questions Bank (Port 1) - 2013 


(For my personal use) 



[ Q: 304 ] MRCPass - Respiratory 

A 75 year old man with COPD is on 
long-term oxygen therapy (LTOT). He 
complains of persistent leg swelling during a 
routine review . 


His ABG on a supplemental oxygen flow rate 
of 2 l/min, shows: 

pH of 7.35, pC0 2 of 5.7kPa, p0 2 of 7.8kPa and 
HCO 3 of 28 mmHg 

What should be done? 


1- CXR 


5- Serum precipitins 


Answer & Comments 

Answer: 4- High resolution CT of the chest 

The diagnosis is likely to be pulmonary 
fibrosis. The signs clubbing and inspiratory 
crepitations suggest cryptogenic fibrosing 
alveolitis. High resolution CT would show 
reticular opacities or honeycomb changes if 
there is pulmonary fibrosis. HRCT of the chest 
shows basilar ground glass opacities as well as 
linear and reticular opacities 


2- Overnight oxygen saturation monitoring 

3- Echocardiogram 

4- Repeat ABG on air 

5- CT scan of the chest 


Answer & Comments 

Answer: 2- Overnight oxygen saturation 
monitoring 



The presence of persistent oedema or 
secondary polycythaemia suggests that the 
correction of overnight Sa02 may be 
inadequate. Nocturnal hypoxaemia may be 
evident during an overnight Sa02 monitoring. 


HRCT of the chest showing basilar ground 
glass opacities as well as linear and reticular 
opacities consistent with ^Pulmonary Fibrosis 



7 

•i j 

1_ 

[ Q: 306 ] MRCPass - Respiratory 

[ Q: 305 ] MRCPass - Respiratory 

A 40 year old man presented with 

A 50 year man presents with a six 

history 

of recurrent Haemoptysis. He has 


month history of cough and breathlessness. 
On examination he has clubbing and there are 
bilateral fine inspiratory crackles in the bases 
of his lungs. 


previously been treated for tuberculosis. 
Chest x ray showed a rounded soft tissue mass 
is seen within a cavity in the right upper lobe 
with an air crescent. 


A chest X ray reveals bilateral basal shadowing 
in the lung fields. His blood gases reveal 
hypoxia with a p0 2 of 9 kPa. 

What is the best investigation to confirm a 
diagnosis? 

1- Lung function tests 

2- Transbronchial biopsy 


What is the diagnosis? 

1- Chlamydia pneumonia 

2- Caplan's syndrome 

3- Extrinsic allergic alveolitis 

4- Aspergilloma 

5- Legionella infection 


3- Serum ACE level 


4- High resolution CT of the chest 


Dr. Khalid Yusuf El-Zohry - Sohag Teaching Hospital (01118391123) 

Ref MRCPass OE OE 2012 PasTest 2009 PassMedicine 2009 PasTest Exam ReviseMRCP 


































El-zohry MRCP Questions Bank (Port 1) - 2013 


(For my personal use) 


Answer & Comments 

Answer: 4- Aspergilloma 

Aspergillomas are masses of fungal mycelia 
that grow in preexisting lung cavities. 
Haemoptysis is a common symptom. They 
may require treatment with either anti- 
fungals or excision surgery if large. They are 
not associated with bronchiectasis (unlike 
allergic bronchopulmonary aspergillosis). 



Aspergilloma 


[ Q: 307 ] MRCPass - Respiratory 

A 55 year old man who had two 
episodes of hemorrhagic shock due to 
intestinal hemorrhage and post-operative 
secondary hemorrhage, was admitted to 
intensive care unit. 

During the following weeks this was followed 
by bronchopneumonia with symptoms of 
sepsis persisting over several weeks. Chest x 
ray showed progressive changes in the 
interstitial tissues and he became more 
significantly hypoxic. Pulmonary capillary 
wedge pressure was 13 mmHg. 

Whot is the diagnosis? 

1- Congestive cardiac failure 



2- Pneumocystis pneumonia 

3- Pulmonary embolism 

4- Adult respiratory distress syndrome 

5- Community acquired pneumonia 


Answer & Comments 

Answer: 4- Adult respiratory distress 

syndrome 

Respiratory distress syndrome is associated 
with profound hypoxia and increased vascular 
permeability (causing a V/Q mismatch). A 
normal PCWP differentiates the condition 
from pulmonary oedema. It does not respond 
to steroids. 



ARDS 


[ Q: 308 ] MRCPass - Respiratory 

A 40 year old man is HIV positive. He 
presents with breathlessness to the hospital. 
Chest XR shows bilateral interstitial lung 
markings. His oxygen levels desaturate upon 
mobilising. 

Whot should be commenced? 

1- Amoxycillin 

2- Amoxycillin and clarithromycin 

3- Iv co-trimoxazole 

4- Doxycycline 

5- Quadruple TB therapy 




Dr. Kholid Yusuf El-Zohry - Sohog Teaching Hospital (01118391123) 

Ref MRCPass OE OE 2012 PasTest 2009 PassMedicine 2009 PasTest Exam ReviseMRCP 


154 




























El-zohry MRCP Questions Bank (Port 1) - 2013 


(For my personal use) 


Answer & Comments 

Answer: 3- Iv co-trimoxazole 

The diagnosis is PCP. Silver staining rather 
than auramine should be done. Mortality is 
10%. Iv co-trimoxazole or pentamidine can be 
used. Desaturation is typical with exercise in 
PCP. Other bacterial infections such as TB are 
more common among Africans. 


[ Q: 309 ] MRCPass - Respiratory 

A 25 year old male with a 5 year long 
history of bronchial asthma, presented with 
worsening of asthma and fever of 20 days 
duration. The patient was well controlled on 
budesonide metered dose inhalations (400 
pg/day) and rarely ever required to take 
salbutamol inhalations. 

For the past 20 days, the patient was 
experiencing increasing breathlessness, fever, 
cough and production of mucoid 
expectoration for the past 20 days. 

Examination a BP of 130/80 mmHg, and 
temperature 38.2°C. 

Respiratory examination revealed bilateral 
widespread wheezes. 

Investigations revealed- Hemoglobin- 12.4 
g/dl, total white count - 13 x 10 9 /L, 
Neutrophils 6 (2-7.5) x 10 9 /L, Lymphocytes 3 
(1.3-3.5) x 10 9 /L, Eosinophils 4 (0.04-0.44) x 
10 9 /L. Serum IgE was 800 (0-380) klU/ml. 

The chest X-ray revealed pulmonary infiltrates 
in the right mid and lower zones. 

What is the likely diagnosis? 

1- Chronic obstructive pulmonary disease 

2- Allergic broncho pulmonary aspergillosis 

3- Pulmonary embolus 

4- Extrinsic allergic alveolitis 

5- Mesothelioma 



Answer & Comments 

Answer: 2- Allergic broncho pulmonary 
aspergillosis 

ABPA usually occurs in association with 
asthma, but can occur in cystic fibrosis. It is 
treated with either oral or inhaled steroids. 
There is some evidence supporting anti-fungal 
agents like itraconazole, but amphotericin is 
not used. IgG and IgE are raised. In addition 
skin-testing or examination of sputum with 
fungal stains can be used. 


Sputum culture is not often helpful. ABPA can 
lead to proximal bronchiectasis affecting the 
upper lobes if left untreated. 



CXR showing pulmonary infiltrates on the right 

in ABPA 


[ Q: 310 ] MRCPass - Respiratory 

A 16 year girl with know n cystic 
fibrosis presents with cough and fevers. Chest 
X ray shows right lower zone consolidation. 

What antibiotic should be commenced? 

1- Amoxycillin 

2- Metronidazole 

3- Ceftazidime 

4- Gentamicin 

5- Piperacillin 




Dr. Khalid Yusuf El-Zohry - Sohag Teaching Hospital (01118391123) 

Ref MRCPass OE OE 2012 PasTest 2009 PassMedicine 2009 PasTest Exam ReviseMRCP 



























El-zohry MRCP Questions Bank (Port 1) - 2013 


(For my personal use) 


Answer & Comments 

Answer: 3- Ceftazidime 

Patients with cystic fibrosis often have 
pseudomonas as a pathogenic infection. 
Ceftazidime and ciprofloxacin would cover the 
organism, and sometimes gentamicin or 
meropenem may be required due to 
resistance. 


[ Q: 311 ] MRCPass - Respiratory 

A 50 year old asthmatic treated with 
high dose of inhaled corticosteroids develops 
a skin rash. His routine blood tests shows mild 
renal dysfunction and blood eosinophilia. His 
chest radiograph is normal. 

Which blood test should be done next? 

1- Total Ig E level 

2- Antineutrophil cytoplasmic antibodies 

3- Anti GBM antibody 

4- Aspergillus fumigates precipitins 

5- Anti-nuclear antibody (ANA) 



Answer & Comments 

Answer: 2- Antineutrophil cytoplasmic 

antibodies 

Skin rash and renal dysfunction in an 
asthmatic suggests Churg- Strauss syndrome. 
Churg-Strauss syndrome, or allergic 
granulomatous angiitis, is a rare syndrome 
that affects small- to medium-sized arteries. 

The presence of 4 or more criteria indicates a 
high likelihood of Churg-Strauss syndrome. 

These criteria are: 

(1) asthma 

(2) eosinophilia of more than 10% in 
peripheral blood 

(3) paranasal sinusitis 

(4) pulmonary infiltrates (may be transient) 


(5) histological proof of vasculitis with 
extravascular eosinophils 

(6) mononeuritis multiplex or 
polyneuropathy. 

Hypergammaglobulinemia, increased 

immunoglobulin E (IgE) levels, rheumatoid 
factor, and positive ANCA are usually present. 



Pulmonary Infiltrates in Churg Strauss 

syndrome 



[ Q: 312 ] MRCPass - Respiratory 

A 70 year old man with chronic 
obstructive lung disease presents with a 
cough, fevers and green sputum. 

What is the antibiotic of choice? 

1- Teichoplanin 

2- Cefotaxime 

3- Erythromycin 

4- Amoxicillin 


5- Ciprofloxacin 


Answer & Comments 

Answer: 4- Amoxicillin 

In COPD, the 3 bacterial species account for 
most isolates are : Haemophilus influenzae, 
Streptococcus pneumoniae and Moraxella 
cattarhalis. First line treatment should be with 
amoxicillin, but if the patient is allergic, a 
tetracycline should be used. 


Dr. Khalid Yusuf El-Zohry - Sohag Teaching Hospital (01118391123) 

Ref MRCPass OE OE 2012 PasTest 2009 PassMedicine 2009 PasTest Exam ReviseMRCP 



156 



























El-zohry MRCP Questions Bank (Port 1) - 2013 


(For my personal use) 


^ [ Q: 313 ] MRCPass - Respiratory 

tl - 

# A 60 year old patient was involved in 
a car accident and sustained multiple 
fractures. He had been in ITU not long before 
there was a suspicion of severe respiratory 
distress syndrome. 

Which of these would help to confirm the 
diagnosis? 

1- CT scan of the chest 

2- Oesophageal manometry 

3- Pulmonary capillary wedge pressure 

4- Requirement for ventilatory support 

5- Arterial blood gases 


lesion. Out of the following causes , which is 
most likely to cause a cavitating lung lesion? 

1- Mesothelioma 

2- Small cell lung carcinoma 

3- Squamous cell lung carcinoma 

4- Adenocarcinoma of the lung 

5- Large cell carcinoma of the lung 

Answer & Comments 

Answer: 3- Squamous cell lung carcinoma 

Squamous cell carcinoma accounts for 
approximately one-third of all cases of 
bronchogenic carcinomas. 


Answer & Comments 

Answer: 3- Pulmonary capillary wedge 
pressure 

Respiratory distress syndrome can be caused 
by severe trauma, smoke inhalation, multiple 
blood transfusions, drow ning and aspiration, 
and drugs such as salicylates and narcotics. 
Chest XR would show bilateral infiltrates, and 
the presentation mimics heart failure. Hence 
the best test would be PCWP, which would be 
normal (<18mmHg). 



ARDS 


[ Q: 314 ] MRCPass - Respiratory 


A 68 year old man with weight loss 
has a chest x ray which shows a cavitating lung 



Squamous cell carcinomas tend to form firm, 
nonencapsulated, sharply circumscribed 
masses located in the main, lobar or 
segmental bronchi. Larger tumors often 
outgrow their vascular supply and may have 
central areas of hemorrhage, necrosis or 
cavitation. 



A cavitating carcinomatous lung lesion 



[ Q: 315 ] MRCPass - Respiratory 

A 45 year man has a long history of 
productive cough. He had complained of 
frequent chest infections. 


Examination reveals bilateral inspiratory 
crackles in the bases of the lungs and clubbing 
of the fingers. 



Dr. Khalid Yusuf El-Zohry - Sohag Teaching Hospital (01118391123) 

Ref MRCPass OE OE 2012 PasTest 2009 PassMedicine 2009 PasTest Exam ReviseMRCP 


























El-zohry MRCP Questions Bank (Port 1) - 2013 


(For my personal use) 


Which of following treatments is likely to 
reduce the frequency of exacerbations? 

1- Prophylactic antibiotics 

2- Inhaled corticosteroids 

3- Oral corticosteroids 

4- Postural drainage 

5- Lung transplant 


Answer & Comments 

Answer: 4- Postural drainage 

The patient has bronchiectasis, in which the 
common complication is difficulty 
expectorating, postural drainage of secretions 
is helpful. 

[ Q: 316 ] MRCPass - Respiratory 

A 60 year old man, ex-smoker (20 
pack-years), was admitted to the hospital 
because of a prolonged fever of up to 38.5°C 
over a period of 10 days, associated with a 
progressively worsening shortness of breath, a 
nonproductive cough, weakness, and fatigue. 

The chest radiograph, showed consolidations 
at the base of both lungs, prominent 
interstitial markings in the middle lung fields, 
and hazy infiltrates in the right upper lung 
field. 

A bronchoscopy was done and BAL fluid 
analysis showed alveolitis with macrophages 
of 65%, lymphocytes of 25%, and neutrophils 
of 10%. 

Which drug is most likely to be responsible? 

1- Sulphasalazine 

2- Penicillamine 

3- Gold 



The diagnosis is interstitial pneumonitis. 
Methotrexate is associated with interstitial 
pneumonitis. This is rare but a serious 
complication. Diagnosis is based on the clinical 
setting, clinical manifestations, radiographic 
abnormalities, bronchoalveolar lavage (BAL), 
and lung histology. 

[ Q: 317 ] MRCPass - Respiratory 

A 40 year old woman has a history of 
cough with copious phlegm with intermittent 
haemoptysis. She also gets recurrent chest 
infections and has a long history of joint pains. 
She is on non-steroidal anti-inflammatory 
drugs. 

Her chest radiograph shows linear 
radioluciences at both bases. A high- 
resolution computed tomography scan (HRCT) 
of the chest confirms bronchiectasis. 

What is the likely cause? 

1- Old tuberculosis 

2- Hypogammaglobulinaemia 

3- Rheumatoid arthritis 

4- Cystic fibrosis 

5- Kartagener's syndrome 



Answer & Comments 

Answer: 3- Rheumatoid arthritis 

The case scenario suggests rheumatoid 
arthritis. 4 % of patients with rheumatoid 
arthritis develop bronchiectasis. Other causes 
are chronic infection (TB, measles, whooping 
cough etc), foreign body aspiration, 
hypogammaglobulinaemia, Kartagener's 
syndrome, Young's syndrome, cystic fibrosis, 
allergic bronchopulmonary aspergillosis. 


4- Methotrexate 

5- Azathioprine 


Answer & Comments 


Answer: 4- Methotrexate 


[ Q: 318 ] MRCPass - Respiratory 

A 35 year old lady has a headache, 
cough and myalgia. 

On examination, she has a pyrexia and 
scattered crackles bilaterally on auscultation 



Dr. Khalid Yusuf El-Zohry - Sohag Teaching Hospital (01118391123) 

Ref MRCPass OE OE 2012 PasTest 2009 PassMedicine 2009 PasTest Exam ReviseMRCP 



158 






























El-zohry MRCP Questions Bank (Port 1) - 2013 


(For my personal use) 


of the chest. Rapid cold agglutinins was 
positive. 

Whot is the likely diagnosis? 

1- Tuberculosis 

2- Staphylococcal pneumonia 

3- Streptococcal pneumonia 

4- Mycoplasma pneumonia 

5- Legionella pneumonia 


Answer & Comments 

Answer: 4- Mycoplasma pneumonia 

Mycoplasma pneumonia is the commonest 
atypical pneumonia. 

Approximately 15% of pneumonias in adults 
are due to Mycoplasma pneumoniae. 
Transmission occurs from person to person by 
infected droplets. The incubation period is 9- 
21 days. The incidence is higher during the 
winter months. Fever, chills, cough and 
headache are early symptoms. Dyspnoea, 
chest pain and haemoptysis are rare. 

Small pleural effusions may occur but are rare. 
Cold agglutinins are usually present in a titre 
greater than 1:32. 


[ Q: 319 ] MRCPass - Respiratory 

A 32 year old man is admitted to 
hospital with a history of breathlessness of 4-6 
weeks duration. He initially had flu-like 
symptoms and was treated by his doctor with 
a 5-day course of amoxicillin. How ever, he 
then started coughing up blood, leading to 
urgent referral. 

On examination he was dyspnoeic at rest, with 
bilateral crackles on auscultation of the lungs. 

Investigation revealed anaemia and impaired 
renal function (creatinine 250 micromol/l). 
Pulmonary function tests were normal apart 
from an abnormally high diffusion factor. 
Urine dipstick testing showed the presence of 
red blood cells. 



Whot is the most likely diagnosis? 

1- Goodpasture's syndrome 

2- Farmer's lung 

3- Invasive Aspergillosis 

4- Extrinsic allergic alveolitis 

5- Chronic eosinophilic pneumonia 


Answer & Comments 

Answer: 1- Goodpasture's syndrome 

The clinical picture of pulmonary and renal 
involvement is typical of Goodpasture's 
syndrome. The condition is due to the 
presence of circulating anti-glomerular 
basement membrane antibodies (anti-GBM 
antibodies. Other causes of pulmonary 
haemorrhage and renal failure include 
Wegener's granulomatosis, microscopic 
polyangiitis and systemic lupus erythematous 
(SLE). 

[ Q: 320 ] MRCPass - Respiratory 

A 40 year old lady presents with 
breathlessness, hyperventilation and 
tachycardia. She has a temperature of 38°C. 
Over the last 5 days she had been 
deteriorating with a cough and fever despite a 
course of oral antibiotics, but her partner 
mentions that she has become much worse 
over the last day. 

WCC is 18 x 10 9 /I 

CRP is 220 mg/I 

p0 2 is 6.5 kPa despite high flow 0 2 

Pulmonary wedge pressure is 16mmHg 

The chest X-ray shows bilateral interstitial lung 
shadowing. 

Whot is the likely diagnosis? 

1- Pulmonary embolus 

2- Pneumothorax 

3- Severe pulmonary fibrosis 

4- Pulmonary oedema 



Dr. Kholid Yusuf El-Zohry - Sohog Teaching Hospital (01118391123) 

Ref MRCPass OE OE 2012 PasTest 2009 PassMedicine 2009 PasTest Exam ReviseMRCP 






























El-zohry MRCP Questions Bank (Port 1) - 2013 


(For my personal use) 


5- Adult respiratory distress syndrome 

Answer & Comments 

Answer: 5- Adult respiratory distress 

syndrome 

ARDS can be precipitated by severe infection, 
aspiration and illicit drug use. The main 
differential is pulmonary oedema when the X- 
ray shows bilateral interstitial lung field 
shadowing. Pulmonary capilary wedge 
pressure of < 19mmHg effectively excludes 
left ventricular failure. 



asthma was admitted to hospital for 
reassessment of his recurring dry cough, chest 
tightness and wheezing. 


Investigations show: 

Neutrophils 6 (2-7.5) x 10 9 /L 

Lymphocytes 3 (1.3-3.5) x 10 9 /L 

Eosinophils 9 (0.04-0.44) x 10 9 /L 

CXR shows reticulonodular shadowing 

Investigations to look for causes of the 
eosinophilia (e.g., parasitosis, 

immunodeficiency and malignant disease) 
included stool microscopy, tumour marker 
assays (for carcinoembryonic antigen and 
cancer antigen (CA) 125, CA 19-9, CA 15-3 and 
CA 72-4), mammography, 

gastroduodenoscopy, bronchoscopy, 


abdominopelvic ultrasonography, and vaginal 
and pelvic examination; all yielded normal 
findings. 

Pulmonary function tests revealed a mild 
obstructive and moderate restrictive pattern 
[FEVi] 46%,[FVC] 55%, ratio of FEVi to FVC 72, 
peak expiratory flow 53%, [DLCO] 46%. 

What is the likely diagnosis? 

1- Tuberculosis 

2- Loeffler's syndrome 

3- Sarcoidosis 

4- Pulmonary embolism 

5- Wegener's granulomatosis 

Answer & Comments 

Answer: 2- Loeffler's syndrome 

Hypereosinophilic syndrome is a rare 
condition where there is an idiopathic 
eosinophil count of > 15 x 10 A 9/dl. 

It is associated with Loeffler's syndrome which 
is a transient pulmonary reaction with nodular 
or reticular shadowing (diffuse, fanshaped 
shadowing) on chest radiology and 
eosinophilia. 

Hypereosinophilic syndrome generally affects 
young men ages 20-50. Thrombotic tendency, 
neurological involvement (loss of intellect, 
depressed mood and poor coordination) and 
restrictive cardiomyopathy occur. There is 
response to steroids. 

The lung involvement results in nocturnal 
cough, productive sputum, wheezing and 
dyspnea, which raises the 

suspicion of bronchial hyperreactivity. 
Patients may be misdiagnosed as having 
asthma. However, pulmonary function tests 
typically reveal no airflow limitation. 


Dr. Kholid Yusuf El-Zohry - Sohog Teaching Hospital (01118391123) 

Ref MRCPoss OE OE 2012 PosTest 2009 PassMedicine 2009 PosTest Exam ReviseMRCP 
























El-zohry MRCP Questions Bank (Port 1) - 2013 


(For my personal use) 


[ Q: 322 ] MRCPass - Respiratory 

A 40 year old man has a history of 
long standing cough with sputum production. 
He also has recurrent episodes of chest 
infections and sinusitis. He is married but has 
been unsuccessful in having children. He also 
has hearing difficulty. 

Which one of the following is likely? 

1- Hypogammaglobulinaemia 

2- Alpha 1 antitrypsin deficiency 

3- Defect in CFTR gene 

4- Kartaganer's syndrome 

5- Asthma variant 



Answer & Comments 

Answer: 4- Kartaganer's syndrome 

Kartagener's syndrome is hereditary. It 
comprises a triad of: situs inversus 
(transposition) of the viscera, abnormal 
frontal sinuses producing sinusitis and 
bronchiectasis, and immobility of the cilia. 

Symptoms and signs are dyspnoea, productive 
cough, recurrent respiratory infections, 
rheumatoid arthritis, renal abnormalities, 
malformations of renal vessels and anomalous 
subclavian artery. There is also otitis media, 
nasal speech, conductive hearing loss, 
anosmia or clubbing. 



Dextrocardia in Kartagener's syndrome 


[ Q: 323 ] MRCPass - Respiratory 

A 40 year old woman is referred with 
a history of red, painful legs of 3 weeks 
duration that have not responded to a course 
of flucloxacillin given for cellulitis. She is 
afebrile, does not have any other symptoms, 
and has never smoked. Examination reveals 
tender purple / red nodules on her shins. A 
chest radiograph shows prominent hilar 
regions. 

What is the appropriate management? 

1- Lung Biopsy 

2- Arrange CT scan of the lungs 

3- Arrange bronchoscopy and bronchoalveolar 
lavage to exclude malignancy 

4- Start prednisolone 

5- Arrange outpatients appointment for follow 
up 



Answer & Comments 

Answer: 5- Arrange outpatients appointment 
for follow up 

The combination of bilateral hilar 
lymphadenopathy and erythema nodosum is 
diagnostic of sarcoidosis. This is usually self- 
limiting. She should how ever be seen in 
outpatients with full lung function tests 
including transfer factor and lung volumes. 
Serum angiotensin-converting enzyme (ACE) 
level and lung functions can be used to 
monitor disease. Worsening disease can be 
treated with prednisolone. 


Dr. Khalid Yusuf El-Zohry - Sohag Teaching Hospital (01118391123) 

Ref MRCPass OE OE 2012 PasTest 2009 PassMedicine 2009 PasTest Exam 




ReviseMRCP 

161 























El-zohry MRCP Questions Bank (Port 1) - 2013 


(For my personal use) 



Sarcoidosis 


[ Q: 324 ] MRCPass - Respiratory 

A 50 year old patient has become 
more breathless over several days. He was 
assessed with a spectrum of tests. The chest X 
ray was normal. In the interpretation of his 
arterial blood gases on air the following 
results were obtained:- 

p0 2 : 8.0 kPa 

pC0 2 : 9.2 kPa 

pH: 7.40 

Base Excess +2 

What disease is likely to have caused this? 

1- Community acquired pneumonia 

2- Bronchiectasis 



She describes episodes of cutaneous flushing, 
which typically affects the head and neck. The 
episodes are often associated with an 
unpleasant warm feeling, itching feeling. They 
last for half an hour. 

She is slightly breathless but has had no 
evidence of leg swelling or chest pain. She 
smokes 10 cigarettes a day. 

She has lost 3 kg of weight in the past two 
months and has been experiencing night 
sweats. A Chest x ray shows sail sign in the left 
lower lobe, but no visible consolidation. 

Which one of the following is the most likely 
diagnosis? 

1- Wegener's granulomatosis 

2- Goodpasture's syndrome 

3- Carcinoid tumour 

4- Sarcoidosis 

5- Tuberculosis 


Answer & Comments 

Answer: 3- Carcinoid tumour 

A sail sign on the CXR indicates collapse of the 
left lower lobe. The likely diagnosis is a 
carcinoid tumour, which is associated with 
smoking. 


3- Tuberculosis 

4- Guillain barre syndrome 

5- Small cell carcinoma 


Answer & Comments 

Answer: 4- Guillain barre syndrome 

The raised C02 and hypoxia demonstrate type 
II respiratory failure. The causes of this could 
be obstructive lung disease, neurogenic, or 
musculoskeletal (kyphoscoliosis). 


[ Q: 325 ] MRCPass - Respiratory 

A 40 year lady presents with a 6 
month history of intermittent haemoptysis. 



An early and frequent (94%) symptom of 
carcinoid tumors is cutaneous flushing, which 
typically affects the head and neck. Episodes 
are often associated with an unpleasant warm 
feeling, itching, palpitation, upper-body 
erythema and edema, salivation, diaphoresis, 
lacrimation, and diarrhea. Exercise, stress, or 
certain foods (eg, cheese) may trigger an 
attack. Initial attacks are short, lasting only a 
few minutes. With time, the duration 
increases to hours. Flushes are reported to be 
longest in association with bronchial 
carcinoids. 

Carcinoid tumours are neuroendocrine 
tumours arising from Kultchitzsky cells. They 
can be central or peripheral, and are classified 


Dr. Khalid Yusuf El-Zohry - Sohag Teaching Hospital (01118391123) 

Ref MRCPass OE OE 2012 PasTest 2009 PassMedicine 2009 PasTest Exam ReviseMRCP 




























El-zohry MRCP Questions Bank (Port 1) - 2013 


(For my personal use) 


as typical or atypical depending on their 
histology. They are slow growing tumours of 
lung with a peak incidence around age 40. 
Bronchial obstruction is common. Diagnosis is 
by bronchoscopy and biopsy. 

Treatment is usually with lobectomy. 



Sail sign (straight line around the heart 

border) 


[ Q: 326 ] MRCPass - Respiratory 

A 68 year old man with emphysema 
is referred to a chest clinic for consideration of 
oxygen therapy. Despite maximal treatment 
with bronchodilators, his exercise tolerance is 
reduced to about 25 yards. 

Physical examination and pulmonary function 
test was consistent with emphysema. There 
was no evidence of heart failure. ECG was 
normal. CXR showed hyperinflated lung fields. 

Oxygen saturation was 90%. ABG showed ph 
of 7.36, pC0 2 of 3.7kPa, p0 2 of 7.6 kPa. 

What should be recommended? 

1- Overnight Sa0 2 monitoring 

2- Course of steroids 

3- Walking test with a trial of oxygen 

4- Repeated ABG in 6 weeks time 

5- Long-term oxygen therapy 



This patient does not meet the criteria for 
long term oxygen therapy, which is indicated 
when the pa02 is persistently below 7.3 kPa 
on air. Clinical stability is defined as the 
absence of exacerbation of chronic lung 
disease for the previous 5 weeks. 

Patients without chronic hypoxaemia and not 
on LTOT, should be considered for ambulatory 
oxygen therapy if they show evidence of 
exercise oxygen desaturation (a fall of Sa02 of 
at least 4% below 90%), improvement in 
exercise capacity with ambulatory oxygen 
therapy and motivation to use the ambulatory 
oxygen outside the house. 

Assessment should be performed on both air 
and supplemental oxygen with the patient 
blinded as to the content of the cylinder. 


[ Q: 327 ] MRCPass - Respiratory 

A 50 year old woman presents with 
breathlessness that has been getting gradually 
worse over a few weeks and she has difficulty 
mobilising. On physical examination she is 
found to have a large left sided pleural 
effusion. 

The presence of the effusion is confirmed by 
chest radiography. 

The most appropriate initial investigation 
would be: 

1- Diagnostic aspiration of pleural fluid 

2- Sputum cytology 

3- CT chest 

4- Pleural biopsy 

5- Mantoux test 



Answer & Comments 


Answer: 1- Diagnostic aspiration of pleural 
fluid 


Answer & Comments 


Answer: 3- Walking test with a trial of oxygen 


The first investigation should be diagnostic 
aspiration of pleural fluid. 


Dr. Khalid Yusuf El-Zohry - Sohag Teaching Hospital (01118391123) 

Ref MRCPass OE OE 2012 PasTest 2009 PassMedicine 2009 PasTest Exam 




ReviseMRCP 

163 

























El-zohry MRCP Questions Bank (Port 1) - 2013 


(For my personal use) 


Light's criteria can be used to distinguish 
transudates from exudates: in exudates at 
least one of the following three criteria are 
met: 

pleural fluid protein concentration greater 
than 50% of that in plasma; 

pleural fluid LDH greater than 60% of that in 
plasma; 

pleural fluid LDH more that two thirds the 
upper limit of normal in plasma 

In general, if the protein is < 30 g then it is 
considered to be a transudate, if > 30 g then it 
is an exudate. 

^ [ Q: 328 ] MRCPass - Respiratory 

# A 38 year old man is admitted to the 
hospital with a one-w eek history of dyspnoea. 
He is HIV positive. His chest radiograph shows 
bilateral alveolar infiltrates. The admitting 
doctor makes a diagnosis of Pneumocystis 
carinii pneumonia (PCP). 

What treatment should be started? 

1- AZT 

2- IV metronidazole 

3- IV co-trimoxazole 

4- IV fluconazole 

5- IV amphotericin 



Pneumocystis pneumonia 


[ Q: 329 ] MRCPass - Respiratory 

A 40 year old man has a 3 month 
history of cough, dyspnoea and sputum 
production. He smokes 20 cigarettes a day and 
has a history of asthma. There is no history of 
asbestos exposure. 

His WBC count is 24 x 10 9 /L with 70% 
neutrophils and 3.0 x 10 9 /L (5%) eosinophils. 
IgE level is elevated. He has decreased breath 
sounds corresponding to parenchymal 
infiltrates on the CXR. 

Which is the best test to confirm the 
diagnosis? 

1- HIV test 



Answer & Comments 

Answer: 3- IV co-trimoxazole 

PCP patients are usually hypoxic and chest 
radiograph characteristically shows bilateral 
alveolar infiltrates. 

The patient should receive intravenous co- 
trimoxazole in a dose of 120mg/kg for at least 
3 weeks as well as glucocorticoids (IV 
methylpred for 3 days). This, when used in 
patients with moderate to severe hypoxia, 
decreases the risk of respiratory failure and 
death by over 50%. 


2- Stool for ova,cysts,parasites 

3- Autoimmune screen 

4- Aspergillus precipitins 

5- Sputum for Acid Fast Bacilli 

Answer & Comments 

Answer: 4- Aspergillus precipitins 

The condition described is Allergic Broncho 
Pulmonary Aspergillosis, commoner among 
asthmatics and cystic fibrosis patients. 
Wheeze, shortness of breath and productive 
cough are symptoms. 



Dr. Khalid Yusuf El-Zohry - Sohag Teaching Hospital (01118391123) 

Ref MRCPass OE OE 2012 PasTest 2009 PassMedicine 2009 PasTest Exam ReviseMRCP 
























El-zohry MRCP Questions Bank (Port 1) - 2013 


(For my personal use) 


Allergic bronchopulmonary aspergillosis 
results from an allergic reaction to Aspergillus 
fumigatus which actually grow s in the walls of 
the bronchi. 

Eosinophilia and high IgE levels are suggestive 
of the condition. Aspergillus precipitins (lab 
test to detect antibodies) the diagnosis. The 
chest radiograph often shows evidence of 
proximal bronchiectasis. 



[ Q: 331 ] MRCPass - Respiratory 

A 50 year old lady has been 
complaining of worsening difficulty 
with breathing over the past year. She has 
previously been diagnosed with a goitre. On 
examination, she had marked stridor. 


Which of the following is helpful in 
investigating the extent of airways 
obstruction? 


[ Q: 330 ] MRCPass - Respiratory 

A 55 year old woman is admitted 
with a history of syncopal episodes and 
breathlessness. There is no prior history of 
recent travel. 

On examination she looks distressed. Her 
pulse rate is 120 beats per minute, blood 
pressure 85/60, JVP is elevated by 5 cm. 
Respiratory rate 26 per minute, there is a soft 
systolic murmur at the left sternal edge. 
Breath sounds are clear. 

Investigations show : HB 12.4 g/dL, Blood 
gases pH 7.42, PaC0 2 3.3 kPa, Pa0 2 8.5 kPa. 

She was put on high flow oxygen and given 
low molecular weight heparin. 

The next step in management of this patient 
should be: 

1- High dose aspirin 

2- Coronary angiography 

3- Urgent CTPA and consider thrombolysis 

4- Intravenous unfractionated heparin 

5- VQ scan 



Answer & Comments 

Answer: 3- Urgent CTPA and consider 
thrombolysis 

The diagnosis massive central pulmonary 
embolus as the patient is hypoxic and 
hypotensive. The best management is to 
obtain an urgent CTPA and then thrombolyse 
if there are no contraindications (e.g. high risk 
of bleeding). 


1- FEVi 

2- FVC 

3- Residual volume 

4- Flow volume loop 

5- Functional residual capacity 


Answer & Comments 

Answer: 4- Flow volume loop 

The flow volume loop is a graphic recording of 
airflow during maximal respiration and 
expiration at different lung volumes, and may 
be affected in a characteristic way by 
alterations in the airway resistance. Fixed 
lesions cause plateaus in both the inspiratory 
and expiratory limbs of the flow volume loop. 

Variable intrathoracic lesions are 
characterized by expiratory slowing and 
flattening of the expiratory limb. 

Hov 



Flow Volume Loop showing airways 
obstruction 


Dr. Khalid Yusuf El-Zohry - Sohag Teaching Hospital (01118391123) 

Ref MRCPass OE OE 2012 PasTest 2009 PassMedicine 2009 PasTest Exam 




ReviseMRCP 

165 




























El-zohry MRCP Questions Bank (Part 1) - 2013 


(For my personal use) 


^ [ Q: 332 ] MRCPass - Respiratory 

fi - 

# A 60 year old lady has arthritic 
changes on her hand joints of ulnar deviation 
and Butonniere's defomirty. She has chronic 
breathlessness. 

Which of the following is a likely association? 

1- Aspergillosis 

2- Pulmonary fibrosis 

3- Empyema 

4- Lung carcinoma 

5- Pulmonary emboli 


Answer & Comments 

Answer: 2- Pulmonary fibrosis 

The pulmonary complications of rheumatoid 
arthritis are : 

pulmonary fibrosis (interstitial lung disease) 

bronchiolitis obliterans with organizing 
pneumonia 

bronchiectasis 

interstitial pneumonitis secondary to drugs 


[ Q: 333 ] MRCPass - Respiratory 

A 35 year old man has significant 
wheezing and breathlessness. Recordings of 
peak flows shows diurnal variation. He was 
prescribed with salbutamol but continues to 
have frequent wheezy episodes. 

What is the next step in management? 

1- Phosphodiesterase inhibitors 

2- Leukotriene antagonists 

3- Oral antibiotics 

4- Inhaled corticosteroids 

5- Oral steroids 



Answer & Comments 


Answer: 4- Inhaled corticosteroids 


Diurnal PEFR variation points tow ards a 
diagnosis of asthma. First line treatment are 
short acting B agonists such as salbutamol and 
Patients who do not respond to B agonists 
should be treated with inhaled corticosteroids 
(becotide or flixotide) which help to reduce 
exacerbations in the long term. 


^ [ Q: 334 ] MRCPass - Respiratory 

/ - 

# A 55 year old lady presents with 

worsening breathlessness. She has a history of 
Raynaud's phenomenon, heartburn and reflux. 

On examination, the skin over her fingers is 
tight and shiny. She has multiple 
telangiectasia over her face and nail-fold 
capillary loops are seen. Her investigations 
show she has Anticentromere antibodies. 

What respiratory complication may occur? 

1- Churg Strauss syndrome 

2- Metastatic adenocarcinoma 

3- Pulmonary hypertension 

4- Allergic bronchopulmonary aspergillosis 

5- Mesothelioma 


Answer & Comments 

Answer: 3- Pulmonary hypertension 

The patient has the clinical features of limited 
cutaneous scleroderma (LcSScformerly CREST 
syndrome). Anticentromere antibodies occur 
in 70-80% of these patients. A significant 
respiratory complication is pulmonary 
hypertension. Treatment is with prostaglandin 
analogues e.g. iloprost (intravenous infusions) 
or bosentan (oral). 


[ Q: 335 ] MRCPass - Respiratory 

A 16 year old boy was brought to the 
A&E with wheeze and extensive rash whilst 
eating at a Chinese takeaway. 

On examination, he had extensive wheezes in 
his chest, stridor, as well as urticaria covering 



Dr. Khalid Yusuf El-Zohry - Sohag Teaching Hospital (01118391123) 

Ref MRCPass OE OE 2012 PasTest 2009 PassMedicine 2009 PasTest Exam ReviseMRCP 
































El-zohry MRCP Questions Bank (Port 1) - 2013 


(For my personal use) 


his upper and lower limbs. His BP is 82/50 
mmHg. 

What is the most likely diagnosis? 

1- Cl Esterase Deficiency 

2- Salmonella infection 

3- Idiopathic urticaria 

4- Asthma 

5- Allergy 

Answer & Comments 

Answer: 5- Allergy 


days of intravenous flucloxacillin and 
cefotaxime therapy. 

Total serum IgE titre was 1600 IU (normal 
range, 0-180 IU) and the skin prick test was 
positive for Aspergillus fumigatus. 

What should he be treated with? 

1- Praziquantel 

2- Aciclovir 

3- Itraconazole 

4- Rifampicin 

5- HAART 


The scenario is consistent with a food allergy, 
e.g. nuts, leading to an anaphylactic reaction. 
The patient should be treated with 
hydrocortisone (iv or im) as well as 
chlorpheniramine. 


[ Q: 336 ] MRCPass - Respiratory 

A 40 year old man with pneumonia is 
being examined. 

Which one of the following positive 
auscultatory signs is diagnostic of bronchial 
breathing? 

1- Rhonchi 

2- Increased vocal resonance 

3- Aegophony 

4- Whispering pectoriloquy 

5- Fine inspiratory crepitations 



Answer & Comments 

Answer: 4- Whispering pectoriloquy 

Whispering pectoriloquy is a diagnostic sign 
for bronchial breathing. 


[ Q: 337 ] MRCPass - Respiratory 

A 16 year old boy with previous 
tuberculosis had a 7-day history of progressive 
cough, wheeze and tachypnoea, despite 4 



Answer & Comments 

Answer: 3- Itraconazole 

Allergic bronchopulmonary aspergillosis 
(ABPA) is a diagnosis which can be confirmed 
by significantly elevated serum IgE titre, 
positive skinprick tests for aspergillus, positive 
IgG aspergillus precipitins. 

Treatment is with antifungals. Steroids may be 
required in patients with respiratory distress. 


[ Q: 338 ] MRCPass - Respiratory 

A 35 year old man whose condition 
has suddenly deteriorated is brought to A&E. 
He had arrived 30 minutes earlier with a 2- 
hour history of central pleuritic chest pain and 
breathlessness. He collapsed while aw aiting 
CXR. 

He is cyanosed and has a pulse 130/min and 
BP of 85/45 mmHg. Oxygen saturation is 
reading 81%, despite high flow oxygen via a 
re-breathe mask. 

Respiratory examination reveals reduced 
breath sounds in the right lung field with 
deviation of the trachea tow ards the left. 

What immediate course of action should be 
taken? 

1- Pleurodesis 



Dr. Khalid Yusuf El-Zohry - Sohag Teaching Hospital (01118391123) 

Ref MRCPass OE OE 2012 PasTest 2009 PassMedicine 2009 PasTest Exam 




ReviseMRCP 

167 






























El-zohry MRCP Questions Bank (Port 1) - 2013 


(For my personal use) 


2- Insert large bore needle into right 
hemithorax 

3- Arrange for urgent portable chest XR 

4- Check arterial blood gases and commence 
BIPAP 

5- Contact ITU to arrange for the patient to be 
ventilated 

Answer & Comments 

Answer: 2- Insert large bore needle into right 
hemithorax 

The diagnosis is a right sided spontaneous 
pneumothorax, which has now developed into 
a tension pneumothorax. 

As the patient is unstable, there is no time to 
arrange for portable chest XR, insertion of a 
large bore needle would reduce the pressure 
in the right hemithorax. 



Large right sided pneumothorax 


^ [ Q: 339 ] MRCPass - Respiratory 

/ -- 1 - 

* A 45 year old man develops 

breathlessness and a non productive cough. 

He has mild fevers and has lost half a stone in 

weight over the past six months. He has had 

previously treated tuberculosis. He works as a 

taxi driver. 


Over the past ten years he and his partner 
went fossil hunting in old quarries. Sputum 
samples are AFB negative. 

His chest X-ray shows nodular shadowing in 
the upper zone. 

What is the likely diagnosis? 

1- Silicosis 

2- Reactivation of TB 

3- Cadmium lung 

4- Histiocytosis X 

5- Asbestosis 

Answer & Comments 

Answer: 1- Silicosis 

Previous TB predisposes to silicosis, which can 
present as fever, breathlessness and weight 
loss. Coal miners, quarry workers and people 
whose hobbies predispose to exposure to 
silica are at risk. 



Silicosis 


^ [ Q: 340 ] MRCPass - Respiratory 

/I - 

# A 70 year retired sandblaster has 
worsening symptoms of cough, wheeze and 
breathlessness. He has been keeping parrots 
for 15 years as a hobby. 

Serum precipitins for M faeni are negative. 

Chest X ray shows hyperinflated lungs. 

His lung function tests show : 

FEVi 1.8L (predicted 2.6) 

FVC 3.0L (predicted 3.2) 



Dr. Kholid Yusuf El-Zohry - Sohog Teaching Hospital (01118391123) 

Ref MRCPass OE OE 2012 PasTest 2009 PassMedicine 2009 PasTest Exam ReviseMRCP 
























El-zohry MRCP Questions Bank (Port 1) - 2013 


(For my personal use) 


FEVi/FVC 60% 

Which of the following is the likely diagnosis? 

1- Silicosis 

2- Chronic obstructive airways disease 

3- Farmer's lung 

4- Allergic bronchopulmonary aspergillosis 

5- Pigeon fancier's lung 

Answer & Comments 

Answer: 2- Chronic obstructive airways 
disease 


The symptoms and findings clearly point out 
tow ards pulmonary embolism (PE). As the 
clinical probability of PE is high, measurement 
of D-dimer should not be performed, since the 
result would not alter the need for definitive 
investigation. Measurement of D-dimer 
should only be performed when the 
probability of PE is low , when a normal value 
would be taken as reassuring and further 
investigation would not be pursued. 

V/Q scan less likely to be unhelpful in view of 
her asthma; hence a CT pulmonary angiogram 
would be the imaging procedure of choice. 


The lung function tests show an obstructive 
picture (reduced FEVi/FVC ratio). Interestitial 
lung diseases are more likely to cause a 
restrictive picture on the lung function test. 



[ Q: 341 ] MRCPass - Respiratory 

A 60 year old asthmatic lady is 
admitted with sudden onset left sided 
pleuritic chest pain accompanied by shortness 
of breath. 


Arterial blood gases are as follows: pH of 7.30, 
p0 2 of 7.5 kPa, and pC0 2 of 3.8 kPa. 

Chest XR is normal. She is commenced on 
oxygen. 

What is the most appropriate immediate 
action ? 

1- Request a chest XR in expiration 

2- Request D-dimer 

3- Start low molecular weight heparin and 
request CT pulmonary angiography 

4- Start low molecular weight heparin and 
request a V/Q scan 

5- Broad spectrum antibiotics 


Answer & Comments 

Answer: 3- Start low molecular weight heparin 
and request CT pulmonary angiography 


[ Q: 342 ] MRCPass - Respiratory 

A 65 year old man has had 5 kg 
weight loss over 6 months. He is an ex smoker 
of 25 cigarettes a day for 40 years and used to 
work in a coal mine. 

A Chest X ray shows a large right sided pleural 
effusion and several pleural plaques in both 
lung peripheries. 

Pleural aspiration reveals an exudate with 42 g 
of protein. 

What investigation should be recommended? 

1- Bronchoscopy 

2- Lung function tests 

3- Spiral CT of the chest 

4- Thoracoscopy and biopsy 

5- Sputum for AFB 



Answer & Comments 

Answer: 4- Thoracoscopy and biopsy 

This patient probably has a malignant effusion 
as demonstrated by the exudate from the 
pleural effusion. As he is symptomatic, the 
best option would be to drain the fluid as well 
as confirm a diagnosis simultaneously. 

A video assisted thoracoscopy would help to 
do this. In this procedure, an illuminated tube 
is inserted through a small incision made betw 


Dr. Khalid Yusuf El-Zohry - Sohag Teaching Hospital (01118391123) 

Ref MRCPass OE OE 2012 PasTest 2009 PassMedicine 2009 PasTest Exam ReviseMRCP 



169 


























El-zohry MRCP Questions Bank (Port 1) - 2013 


(For my personal use) 


een the ribs. It allows the operator to visualize 
structures inside the chest and to perform 
simple procedures such as biopsy and nodule 
excision. 



[ Q: 343 ] MRCPass - Respiratory 

A 65 year old man with a smoking 
history of 50 a day for many years has been a 
chest radiograph showing a lung mass. He is 
presently waiting a bronchoscopy. 


Which one of the following supports o 
diagnosis of small cell lung cancer? 


1- Disseminated intravascular coagulation 


2- Hypertrophic pulmonary osteoarthropathy 
(HPOA) 


3- Syndrome of inappropriate antidiuretic 
hormone secretion (SIADH) 

4- Thrombocytosis 

5- Hypercalcaemia 


Answer & Comments 

Answer: 3- Syndrome of inappropriate 
antidiuretic hormone secretion (SIADH) 

SIADH is most comonly seen with small cell 
carcinoma rather than non-small cell 
carcinoma. HPOA, hypercalcaemia without 
bone metastasis is more common in 
squamous cell carcinoma. DIC and 
thrombocytosis are more common with 
adenocarcinoma. 


[ Q: 344 ] MRCPass - Respiratory 

A 60 year old woman presents with a 

6-month history of progressive shortness of 
breath. Her past medical history is 
unremarkable apart from Raynaud's syndrome 
for which she takes a calcium channel blocker. 

On examination she has telangiectasia. 

Her chest radiograph shows clear lung fields, 
prominent pulmonary arteries and mildly 
enlarged heart. Spirometry is normal, but gas 
transfer is reduced by 40% of predicted. 



What is the most likely diagnosis? 

1- Cor pulmonale 

2- Pulmonary arterial hypertension 

3- Sarcoidosis 

4- Pulmonary emboli 

5- Pulmonary oedema 


Answer & Comments 

Answer: 2- Pulmonary arterial hypertension 

The patient is likely to have scleroderma from 
the clinical history. 

There is also associated pulmonary 
hypertension which such patients are at risk 
of. 

[ Q: 345 ] MRCPass - Respiratory 

A 75 year old lady is admitted with 
an acute exacerbation of chronic obstructive 
pulmonary disease (COPD). One hour after 
admission she remains distressed with a 
respiratory rate of 35 per minute and is 
peripherally cyanosed. Repeated arterial 
blood gases show a severe respiratory acidosis 
with a pH of <7.2. 

Which of the following treatments is 
recommended? 

1- Give intravenous infusion of aminophylline 

2- Give intravenous hydrocortisone 

3- Repeat bronchodilator therapy and arrange 
repeat arterial gases 

4- Arrange for noninvasive positive pressure 
ventilation 

5- High flow oxygen 



Answer & Comments 

Answer: 4- Arrange for noninvasive positive 
pressure ventilation 

Non invasive positive pressure ventilation 
should be considered, especially in COPD 
patients when there is a need for ventilatory 


Dr. Khalid Yusuf El-Zohry - Sohag Teaching Hospital (01118391123) 

Ref MRCPass OE OE 2012 PasTest 2009 PassMedicine 2009 PasTest Exam ReviseMRCP 



170 





























El-zohry MRCP Questions Bank (Port 1) - 2013 


(For my personal use) 


assistance as indicated by worsening 
dyspnoea, acute respiratory acidosis and 
worsening oxygenation. If this does not work 
then intubation and ventilation may be 
necessary. 


[ Q: 346 ] MRCPass - Respiratory 

A 65 year old man presents with 
inspiratory stridor. A chest X-ray showed 
compression of the trachea by a retrosternal 
goitre. 

Which of the following investigations is the 
most useful to assess the severity of his 
airways obstruction ? 

1- Flow volume loop 

2- Forced expiratory volume 

3- Forced vital capacity 

4- Peak expiratory flow rate 

5- Residual volume 



Answer & Comments 

Answer: 1- Flow volume loop 

The flow volume loop is the best method of 
assessing extent of obstruction associated 
with a retrosternal mass. 

Hov 



Flow volume loop showing airways 
obstruction 


^ [ Q: 347 ] MRCPass - Respiratory 

n - 

* A 25 year old man who smokes, has 
progressive breathlessness. He has had a long 
history of recurrent chest infections. His lung 
function tests show a FEVi/FVC ratio of 65%, 
reduced FEVi, FVC and KCO of 45% predicted. 
He also has an uncle who had a similar 
presentation. 

Which is the likely diagnosis? 

1- Eosinophilic granuloma 

2- Asthma 

3- a 1 antitrypsin deficiency 

4- Fibrosing alveolitis 

5- Rheumatoid lung 


Answer & Comments 

Answer: 3- a 1 antitrypsin deficiency 

a 1 antitrypsin deficiency is autosomal 
recessive and causes an emphysematous 
(obstructive picture) on the lung function 
tests, with reduced transfer factor. The 
phenotypes are M S or Z. ZZ has the worse 
outcome. 


Smoking cessation is essential. 


Genotype 

Prevalence 

% 

Reduction AAT 
level (%) 

MM 

88 

0 

MS 

7 

20 

MZ 

4 

40 

SS 

1 

40 

SZ 

0.1 

70 

ZZ 

0.03 

90 


[ Q: 348 ] MRCPass - Respiratory 

«l - 

# A 55 year old woman with 
rheumatoid arthritis is referred with a history 
of recurrent chest infections, intermittent 
wheeze and production of half a teacupful of 
sputum daily, on occasions with blood stained. 

What is the most likely diagnosis? 

1- Pulmonary fibrosis 



Dr. Khalid Yusuf El-Zohry - Sohag Teaching Hospital (01118391123) 

Ref MRCPass OE OE 2012 PasTest 2009 PassMedicine 2009 PasTest Exam ReviseMRCP 

































El-zohry MRCP Questions Bank (Port 1) - 2013 


(For my personal use) 


2- Bronchiectasis 

3- Emphysema 

4- Tuberculosis 

5- Squamous lung carcinoma 


The cherry red skin colour occurs when COHb 
concentration exceeds 20% but it is rarely 
seen in life. Pulse oximetry gives falsely high 
oxygen saturation sand it is not 
recommended. 


Answer & Comments 

Answer: 2- Bronchiectasis 

Bronchiectasis is associated with rheumatoid 
arthritis, occurring in 3-4% of patients with 
this condition. As with all other causes of 
bronchiectasis, it presents with recurrent 
chest infections and excessive sputum. 
Recurrent haemoptysis is a common feature. 

[ Q: 349 ] MRCPass - Respiratory 

A 38 year old man with a history of 
depression is brought to the accident and 
emergency department after being found 
unconscious in a garage. A friend said that he 
complained of a headache, then had nausea 
and vomiting. 

He then became unrousable. 

Oxygen saturations are 95% on air and breath 
sounds are clear on auscultation. 

Whot is the likely diagnosis? 

1- Pulmonary eosinophilia 

2- Pulmonary embolus 

3- Adult respiratory distress syndrome 

4- Pneumocystis pneumonia 

5- Carbon monoxide poisoning 



Answer & Comments 

Answer: 5- Carbon monoxide poisoning 

Carbon monoxide poisoning is produced by 
the incomplete combustion of carbon 
containing fuels such as gas, coal, oil, wood 
and coke. 

Headache is the most common symptom 
(90%) followed by nausea & vomiting, vertigo, 
alteration in consciousness and weakness. 



[ Q: 350 ] MRCPass - Respiratory 

A 45 year man has breathlessness 
and a dry cough. On examination, there are 
bilateral basal crepitations in his lungs. Oxygen 
saturations are 95% on air. Circulating 
precipitins towards Micropolyspora faeni are 
positive. 

Whot is the likely diagnosis? 


1- ABPA 


2- Bagassosis 

3- Farmer's lung 

4- Bird fancier's lung 

5- Cheese worker's lung 


Answer & Comments 

Answer: 3- Farmer's lung 

This is a form of hypersensitivity pneumonitis. 
Farmer's lung is caused by the actinomycetes 
Micropolyspora faeni, and bagassosis is 
caused by Thermoactinomyces sacchari. 



[ Q: 351 ] MRCPass - Respiratory 

A 45 year old man develops 
breathlessness and a non productive cough. 
He has mild fevers and has lost half a stone in 
weight over the past six months. He has had 
previously treated tuberculosis. He works as a 
taxi driver. 


Over the past ten years he and his partner 
went fossil hunting in old quarries. Sputum 
samples are AFB negative. 

His chest X-ray shows nodular shadowing in 
the upper zone. 

Whot is the likely diagnosis? 

1- Silicosis 


Dr. Kholid Yusuf El-Zohry - Sohog Teaching Hospital (01118391123) 

Ref MRCPass OE OE 2012 PasTest 2009 PassMedicine 2009 PasTest Exam ReviseMRCP 



172 




























El-zohry MRCP Questions Bank (Port 1) - 2013 


(For my personal use) 


2- Cadmium lung 

3- Histiocytosis X 

4- Reactivation of TB 

5- Asbestosis 

Answer & Comments 

Answer: 1- Silicosis 

Previous TB predisposes to silicosis, which can 
present as fever, breathlessness and weight 
loss. Coal miners, quarry workers and people 
whose hobbies predispose to exposure to 
silica are at risk. 



Silicosis - Mid and Upper Zone linear and 
reticulonodular shadowing, 


Answer & Comments 

Answer: 2- Radiotherapy should be given to 
prevent seeding of tumour cells 

In mesothelioma no treatment has been 
shown to influence the universally fatal 
outcome. After obtaining a positive biopsy, 
radiotherapy should be given in an attempt to 
prevent seeding of tumour cells, around the 
area of the biopsy. 

^ [ Q: 353 ] MRCPass - Respiratory 

# A 50 year old iv drug user has been 
referred to the medical ward after being 
brought in to casualty. He has a chronic cough 
productive of sputum, loss of weight, and 
night sweats. On examination he is unkempt 
and emaciated. His trachea is deviated to the 
left and there are crepitations over the apex 
of the left lung. 

CXR shows fibrosis and cavitation in the left 
apex. 

The investigation most likely to confirm the 
diagnosis would be: 

1- CT chest 


[ Q: 352 ] MRCPass - Respiratory 

A 70 year old man who has 
previously worked in the building trade 
presents with a history of chest pain and 
dyspnoea. On examination he has evidence of 
a right-sided pleural effusion. Pleural 
aspiration is performed and a pleural biopsy 
taken. Histology from the biopsy shows 
mesothelioma. 

What should be done? 

1- The diagnosis should be queried 

2- Radiotherapy should be given to prevent 
seeding of tumour cells 

3- Surgery 

4- Chemotherapy should be given to prevent 
seeding of tumour 

5- Curative radiotherapy should be given 



2- Gastric lavage 

3- Sputum for acid and alcohol fast bacilli 

4- Mantoux test 

5- Fibreoptic bronchoscopy 


Answer & Comments 

Answer: 3- Sputum for acid and alcohol fast 
bacilli 

TB is likely. In a patient with a productive 
cough, AFBs should be positive in the sputum. 


Dr. Khalid Yusuf El-Zohry - Sohag Teaching Hospital (01118391123) 

Ref MRCPass OE OE 2012 PasTest 2009 PassMedicine 2009 PasTest Exam ReviseMRCP 



173 





























El-zohry MRCP Questions Bank (Port 1) - 2013 


(For my personal use) 



Red coloured AFB 


[ Q: 354 ] MRCPass - Respiratory 

A 55 year old smoker has a history of 
breathlessness and a dry cough. He has 
several nodules present in the perihilar region. 
His serum calcium is normal. A bronchoscopy 
and transbronchial biopsy is done. This shows 
non necrotic granulomas and multinucleated 
giant cells. 

Which of the following is likely? 

1- Histoplasmosis 

2- Aspergillosis 

3- Tuberculosis 

4- Small cell carcinoma 

5- Silicosis 



Answer & Comments 

Answer: 3- Tuberculosis 

Multinucleated giant cells are very large 
epidermis cells that have multiple nuclei. They 
can be present in viral infections (e.g. herpes), 
TB or lymphoma. Granulomatous disease 
suggests either sarcoid, TB or Wegener's 
granulomatosis. 



A multinucleated giant cell 


[ Q: 355 ] MRCPass - Respiratory 

A 30 year old woman with epilepsy 
has been diagnosed with pulmonary 
tuberculosis and is about to be started on 
quadruple therapy. 

Which drug may cause urine discolouration? 

1- Pyrazinamide 

2- Erythromycin 

3- Rifampicin 

4- Isoniazid 

5- Ethambutol 



Answer & Comments 


Answer: 3- Rifampicin 


Discoloration of urine is due to rifampicin. 
Rifampicin is a liver enzyme inducer but 
isoniazid is an enzyme inhibitor. 



Urine discolouration in a patient taking 

Rifampicin 


[ Q: 356 ] MRCPass - Respiratory 

A 30 year old male patient presents 
with worsening rhinitis, cough and wheezing, 
which has occurred for the past 2 years. On 
admission, his lung function tests show FEVi of 
55% predicted and FVC of 65% predicted and 
a chest X ray showed bilateral infiltrates. 

He had an eosinophil count of 5 x 10 9 /L (0.04- 
0.4) and sputum eosinophilia of 80% was also 
found. There was also elevated IGE levels of 
>1000 kU/L. Serum ANCA was positive at a 
dilution of 1:1024. 




Dr. Khalid Yusuf El-Zohry - Sohag Teaching Hospital (01118391123) 

Ref MRCPass OE OE 2012 PasTest 2009 PassMedicine 2009 PasTest Exam ReviseMRCP 




























El-zohry MRCP Questions Bank (Port 1) - 2013 


(For my personal use) 


A nasal biopsy showed chronic inflammation, 
with some areas suggestive of vasculitis, and 
eosinophilic infiltration. 

Which diagnosis is most likely? 

1- COPD 

2- Mesothelioma 

3- Churg Strauss syndrome 

4- Tuberculosis 

5- Extrinsic allergic alveolitis 

Answer & Comments 

Answer: 3- Churg Strauss syndrome 

There are many causes of pulmonary 
symptoms with eosinophilia including 
Loeffler's syndrome, Churg Strauss syndrome, 
pulmonary eosinophilic syndrome and ABPA. 
Extrinsic allergic alveolitis does not cause a 
wheeze and also does not cause eosinophilia. 

Churg-Strauss syndrome is an uncommon 
condition characterised by asthma and blood 
eosinophilia together with an eosinophilic 
vasculitis. The initial phase of the disorder is 
one of asthma and allergic rhinitis, often 
followed by peripheral blood eosinophilia with 
eosinophilic tissue disease. The vasculitic 
phase that follows is life-threatening; how 
ever, it can often be treated effectively with 
immunosuppression. It is associated with 
granuloma formation and vasculitis affecting 
several organs e.g. skin, pericardium, kidney 
and lung. 

Serum eosinophilia and elevated IgE levels are 
typical. Laboratory diagnosis is based on tissue 
biopsy and the antineutrophil cytoplasmic 
antibody (ANCA) test. About 25% of patients 
have cANCA and about 50% have pANCA. 



with haemoptysis. He had a chest X ray which 
shows masses with air halo around them in 
the upper zones. He has positive serum 
precipitins. 

What is the likely diagnosis? 

1- tuberculosis 

2- Aspergillosis 

3- Actinomycosis 

4- Extrinsic allergic alveolitis 

5- Coal worker's lung 

Answer & Comments 

Answer: 2- Aspergillosis 

The air halo sign is particularly associated with 
the fungal infection aspergillosis, This may be 
predisposed to by previous TB infection 
leading to cavitation. The positive serum 
precipitins are towards aspergillus. 


Dr. Khalid Yusuf El-Zohry - Sohag Teaching Hospital (01118391123) 

Ref MRCPoss OE OE 2012 PosTest 2009 PassMedicine 2009 PosTest Exom 




ReviseMRCP 

175 
























El-zohry MRCP Questions Bank (Port 1) - 2013 


(For my personal use) 



Aspergilloma 


[ Q: 358 ] MRCPass - Respiratory 

A 55 year lady has had a long history 
of productive cough and shortness of breath. 
She often coughs up whitish sputum and is 
prone to chest infections. On examination her 
temperature is 36° C, chest expansion is 
reduced and there are bilateral wet 
inspiratory crackles. 

Which one of the following treatments is most 
helpful? 

1- Intravenous antibiotics 

2- B agonist inhalers 

3- Postural drainage 

4- Morphine 

5- Prednisolone 



Answer & Comments 

Answer: 3- Postural drainage 

This lady has bronchiectasis. There is no 
suggestion of a chest infection during this 
episode of admission, hence postural drainage 
is the best treatment option. 


[ Q: 359 ] MRCPass - Respiratory 

A 55 year old farmer has had 
progressive breathlessness over the past 2 
years. He presents with a severe episode of 
dyspnea and productive cough which occurred 
6 hours after he started working. He mentions 
that these episodes are typical but today is 
more severe. On admission, his O 2 saturations 
are 94% on air. 

CXR shows bilateral interstitial shadowing in 
upper zones. 

Which of the following is the most useful test? 

1- Precipitins to aspergillus 

2- Sputum for AFB 

3- Kveim test 

4- Precipitins to Micropolyspora faeni 

5- Bronchoscopy 



Answer & Comments 

Answer: 4- Precipitins to Micropolyspora faeni 

This patient is likely to have extrinsic allergic 
alveolitis. The classic presentation of farmer's 
lung results from inhalational exposure to 
thermophilic Actinomyces species. 

Patients with extrinsic allergic alveolitis may 
present acutely with a flulike illness with 
cough. They can also present subacutely with 
recurrent pneumonia or chronically with 
exertional dyspnea, productive cough, and 
weight loss. 

The onset of symptoms after acute exposure 
is usually betw een 4 and 12 hours. Some 
antigens provoke symptoms after repeated 
exposure; these include bioaerosols of 
microbial or animal antigens and a few 
reactive chemicals. 

Thermophilic actinomycetes species which can 
cause EAA include Saccharopolyspora 
rectivirgula (formerly Micropolyspora faeni), 
Thermoactinomyces vulgaris, 



Dr. Kholid Yusuf El-Zohry - Sohog Teaching Hospital (01118391123) 

Ref MRCPass OE OE 2012 PasTest 2009 PassMedicine 2009 PasTest Exam ReviseMRCP 
























El-zohry MRCP Questions Bank (Port 1) - 2013 


(For my personal use) 


Thermoactinomyces viridis, and 

Thermoactinomyces sacchari. 



[ Q: 360 ] MRCPass - Respiratory 

A 60 year old woman had a CXR 
showing pulmonary fibrosis. Upon review , of 
her drug history, which of the following drugs 
might she hove been on in the post? 


1- Clarithromycin 

2- Amoxicillin 

3- Busulphan 


4- Omeprazole 

5- Ciprofloxacin 


Answer & Comments 

Answer: 3- Busulphan 

Busulphan, bleomycin, amiodarone and 
nitrofurantoin are drugs which commonly 
cause pulmonary fibrosis. 


[ Q: 361 ] MRCPass - Respiratory 

A 65 year old farmer has been 
getting worsening breathlessness for the past 
4 years. These symptoms occur during work 
and are often worse for approximately 10 
hours after. He is a non smoker. 

An X-ray of the chest revealed intensified 
interstitial lung markings and reticular changes 
in the lower parts of the lung. He also had 
raised serum precipitins to micropolyspora 
faeni. 

Whot is the diagnosis? 

1- Tuberculosis 

2- Wegener's granulomatosis 

3- Churg Strauss syndrome 

4- Extrinsic allergic alveolitis 

5- Pulmonary eosinophilia 



Answer & Comments 


Answer: 4- Extrinsic allergic alveolitis 


Extrinsic allergic alveolitis causes a 
neutrophilia due to cell mediation but 
eosinophil count is normal. Bronchoalveolar 
lavage shows lymphocytes and mast cells. 

EAA is a delayed hypersensitivity reaction 
which may be immune complex (III) mediated 
or cell mediated (type IV) in chronic disease. 

In Extrinsic Allergic Alveolitis, the most 
common antigens are thermophilic 
actinomycetes and avian proteins, and the 
most common diseases are farmer's lung and 
bird fancier's lung. Precipitins to 
micropolyspora faeni are seen in farmer's 
lung. Upper zone fibrosis causes audible 
crackles. Symptoms are typically of 
breathlessness but not wheeze. 



EAA causing upper zone fibrosis 



[ Q: 362 ] MRCPass - Respiratory 

A 22 year old woman has asthma for 
which she is using her salbutamol inhaler two 
or three times a day. 


Whot should be the next step in her 
monogementifshe worsens? 


1- Advise her to use the salbutamol inhaler 
regularly three times a day 


2- Add regular inhaled steroid, e.g. 
beclometasone 100 microg twice daily 


3- Add regular salmeterol twice daily 


Dr. Kholid Yusuf El-Zohry - Sohog Teaching Hospital (01118391123) 

Ref MRCPass OE OE 2012 PasTest 2009 PassMedicine 2009 PasTest Exam 




ReviseMRCP 

177 


























El-zohry MRCP Questions Bank (Part 1) - 2013 


(For my personal use) 


4- Add regular inhaled steroid, e.g. 

beclometasone 1000 microg twice daily 

5- Add regular inhaled steroid, e.g. 

beclometasone 1000 microg twice daily, 
plus regular long-acting inhaled beta 
agonist 


Answer & Comments 

Answer: 2- Add regular inhaled steroid, e.g. 
beclometasone 100 microg twice daily 

The British Thoracic Society Asthma guidelines 
are as follows: 

Step 1: PRN use of inhaled short-acting beta 
agonists 

Step 2: regular inhaled steroids 

Step 3: high-dose inhaled steroids, or low - 
dose inhaled steroids plus long-acting beta 
agonist 

Step 4: high-dose inhaled steroid and regular 
bronchodilators (sustained release 
theophylline, inhaled ipatropium, oral long- 
acting beta agonist, high-dose inhaled 
bronchodilators, cromoglycate / nedocromil) 
Step 5: addition of regular steroid tablets 


[ Q: 363 ] MRCPass - Respiratory 

A 60 year old woman with know n 
COPD is referred with a 4-month history of 
morning headaches. She describes a dull 
headache which is generalised, worst upon 
waking up. She has home nebuliser 
(salbutamol and atrovent) but is not on home 
oxygen. 

A routine arterial blood gas on admission 
showed a pH of 7.34 pC02 of 6.2 kPa, p02 of 
8.8 kPa and HCO3 30 mmHg. 

What should be done? 

1- Long-term oxygen therapy 

2- Ambulatory oxygen therapy 

3- CT chest 

4- Overnight Sa02 and CO 2 monitoring 



5- Repeat ABG 

Answer & Comments 

Answer: 4- Overnight Sa02 and C02 
monitoring 

Morning headaches are often ascribed to 
patients with nocturnal hypoxia or early 
morning hypercapnia. They are often 
associated with sleep apnea or a chronic 
respiratory disease such as COPD. 

The admission blood gases show mild hypoxia, 
and hypercapnia. There is respiratory acidosis 
with metabolic compensation. It suggests that 
there is a high likelihood of chronic hypoxia. 

If oxygen monitoring confirmed nocturnal 
hypoxia (defined as 02 sats of < 90% for more 
than 30% of sleep time), she may require 
nocturnal oxygen supplementation. 

^ [ Q: 364 ] MRCPass - Respiratory 

# A couple attend the GP surgery with 
their 2 year old daughter. She has a history of 
repeated chest infections, failure to thrive, 
and steatorrhoea (fatty stools). They also have 
an 8 year old child who is fit and well. 

What is the likely diagnosis? 

1- Pulmonary Eosinophilia 

2- Asthma 

3- Cystic fibrosis 

4- Congenital Tuberculosis 

5- Bochdalek Hernia 


Answer & Comments 

Answer: 3- Cystic fibrosis 

The gene defect in cystic fibrosis is in a 
mutation on chromosome 7. The inheritance 
is autosomal recessive so the other child 
without symptoms has not inherited two CF 
genes. Pulmonary disease develops over a few 
months after birth. Common infective 


Dr. Khalid Yusuf El-Zohry - Sohag Teaching Hospital (01118391123) 

Ref MRCPass OE OE 2012 PasTest 2009 PassMedicine 2009 PasTest Exam ReviseMRCP 



























El-zohry MRCP Questions Bank (Port 1) - 2013 


(For my personal use) 


organisms are pneumoccocus, Haemophilus 
influenzae and Pseudomonas aeruginosa. 



[ Q: 365 ] MRCPass - Respiratory 

A 45 year old man complains of 
wheeziness which is worse. He has a history of 
asthma. His current medication consists of a 
low dose of inhaled corticosteroids and 
inhaled short-acting beta 2 agonist taken 
three to four times a day. 

Whot should be done next? 


1- Add oral steroids 


2- Add an inhaled long-acting beta 2 agonist 

3- Add a long-acting anticholinergic 

4- Add a short acting anticholinergic 

5- Add a leucotriene receptor antagonist 


2- Streptococcus pneumoniae, Klebsiella 
pneumoniae, E coli 

3- Streptococcus pneumoniae, Haemophilus 
influenzae, Moxarella catarrhalis 

4- Streptococcus pneumoniae, Pseudomonas 
aeruginosa, Staphylococcus aureus 

5- Streptococcus pneumoniae, Haemophilus 
influenzae, Legionella pneumophilia 


Answer & Comments 

Answer: 3- Streptococcus pneumoniae, 
Haemophilus influenzae, Moxarella catarrhalis 

In infective exacerbations of COPD, 
Streptococcus pneumoniae and Haemophilus 
influenzae, as well as Moraxella are the 
commonest organisms. 


Answer & Comments 

Answer: 2- Add an inhaled long-acting beta 2 
agonist 

According to the British Thoracic Society 
guidelines the next step would be to add an 
inhaled long-acting beta 2 agonist (LABA) and 
then assess the situation. If there is a good 
response to LABA, this medication should be 
continued. If there is benefit from LABA but 
control is still inadequate, LABA should be 
continued, and the inhaled corticosteroids 
should be increased to a high dose. 

[ Q: 366 ] MRCPass - Respiratory 

A 65 year old heavy smoker is 
admitted with a history of increasing 
breathlessness. This has been precipitated by 
the development of a cough with yellow 
purulent sputum. 

The organisms that are likely to cause infective 
exacerbations of COPD are: 




[ Q: 367 ] MRCPass - Respiratory 

A 65 year old man has longstanding 
breathlessness on exertion. He has been 
smoking 20 cigarettes a day over a period of 
30 years. 


On examination, the patient is breathless with 
use of accessory muscles and resting 
activation of the abdominal muscles. The 
chest is barrel shaped. 


With regards to this case , which one of the 
following is known to be a predictor of 
mortality? 


1- Spirometry 

2- Fibrotic changes 

3- Body Mass Index 

4- Arterial blood gases 

5- How many cigarettes were smoked 


Answer & Comments 


Answer: 3- Body Mass Index 


1- Streptococcus pneumoniae, Legionella 
pneumophilia, Mycoplasma pneumoniae 


A low body mass index is a known predictor of 
mortality in patients with chronic obstructive 
pulmonary disease. 


Dr. Khalid Yusuf El-Zohry - Sohag Teaching Hospital (01118391123) 

Ref MRCPass OE OE 2012 PasTest 2009 PassMedicine 2009 PasTest Exam ReviseMRCP 






























El-zohry MRCP Questions Bank (Part 1) - 2013 


(For my personal use) 


^ [ Q: 368 ] MRCPass - Respiratory 

f | - 

* A 40 year man presents with 
shortness of breath. On admission he is unw 
ell and has a respiratory rate of 32 breaths per 
minute. His chest is wheezy on auscultation. 

Arterial blood gases show : 

p0 2 9.5 kPa, pC0 2 3.5 kPa, pH 7.48, HC0 3 25, 
BE+2. 

What is the likely clinical scenario? 

1- Lactic acidosis secondary to metformin 

2- Anxiety disorder 

3- Asthma attack 

4- Chronic bronchitis 

5- Pneumothorax 


Answer & Comments 

Answer: 3- Asthma attack 

The patient has a respiratory alkalosis (pH > 
7.45) which is acute since the bicarbonate 
levels are normal (22-28). There is hypoxia and 
the patient is hyperventilating in response and 
blowing off C02. 

This would be consistent with an asthma 
attack. 

In an anxiety attack, hypoxia would not be 
present. 


[ Q: 369 ] MRCPass - Respiratory 

A 30 year old man is admitted with a 
history of haemoptysis, cough and dyspnoea. 
He has been previously fit and has smoked 25 
cigarettes a day for the last 10 years. A chest 
XR shows diffuse alveolar infiltrates. He has a 
microcytic, hypochromic anaemia, urine 
dipstick confirms haematuria and proteinuria. 
His lung function tests shows a normal 
spirometry and a TLCO of 125% predicted. 



3- Goodpasture's syndrome 

4- Pulmonary tuberculosis 

5- Pneumonia 


Answer & Comments 

Answer: 3- Goodpasture's syndrome 

Goodpasture's syndrome is characterised by 
diffuse alveolar haemorrhage and 
glomerulitis. Men are commonly affected with 
most cases occurring betw een the ages of 20- 
30 years. It is more likely to occur in smokers. 
The anti glomerular basement antibody (Anti- 
GBM) is present in up to 90% of the patients. 
Renal histology usually shows a focal 
segmental necrotizing glomerulitis with 
crescent formation. The TLCO is increased 
during active bleeding and can be used to 
monitor disease activity. An increase above 
30% of baseline is suggestive of an intra- 
alveolar bleed. 


[ Q: 370 ] MRCPass - Respiratory 

f - 

# A 60 year old man presents with 
sever breathlessness following an upper 
respiratory tract infection. 

Which of the following would support a 
diagnosis of acute respiratory distress 
syndrome (ARDS)? 

1- High protein content of pulmonary oedema 
fluid 

2- Normal chest Xray 

3- High pulmonary capillary wedge pressure 

4- Increased lung compliance 

5- High C0 2 levels 


Answer & Comments 

Answer: 1- High protein content of pulmonary 
oedema fluid 


What is the most likely diagnosis? 

1- Pulmonary emboli 

2- Chest trauma 


Acute respiratory distress syndrome is 
characterised by hypoxaemia, reduced lung 
compliance, pulmonary hypertension and 


Dr. Khalid Yusuf El-Zohry - Sohag Teaching Hospital (01118391123) 

Ref MRCPass OE OE 2012 PasTest 2009 PassMedicine 2009 PasTest Exam ReviseMRCP 





























El-zohry MRCP Questions Bank (Port 1) - 2013 


(For my personal use) 


pulmonary infiltrates on chest X ray. There is 
damage to the capillary endothelial cell linings 
resulting in oedema leakage of proteins cells 
into interstitial alveolar spaces. 

A high pulmonary capillary wedge pressure 
suggests heart failure. High C02 reflect type II 
respiratory failure and are non specific. 


[ Q: 371 ] MRCPass - Respiratory 

A 60 year old man had a 
transbronchial biopsy confirming squamous 
cell carcinoma of the lung. 

Which of the following is a contraindication 
tow ards surgical resection? 

1- Hypercalcaemia 

2- Superior vena caval obstruction 

3- Previous radiotherapy 

4- Metastasis to local lymph nodes 

5- Pleural effusion 



Answer & Comments 

Answer: 2- Superior vena caval obstruction 

Extensive nodal spread, distal metastases, 
stage NIB or more, and superior vena caval 
obstruction are contraindications tow ards 
surgery for lung cancer. 


Answer & Comments 

Answer: 2- Chronic obstructive lung disease 

There is evidence for a small reduction in 
mortality in patients with COPD and resting 
hypoxia. Although indications for LTOT are 
largely based on mortality data, some studies 
have also suggested improvements in other 
outcome measures, including depression, 
cognitive function, quality of life, exercise 
capability, and frequency of hospitalisation. 



[ Q: 373 ] MRCPass - Respiratory 


A 50 year 
finding 


old woman has an 
of raised left 


incidental 

hemidiaphragm on the CXR. She has had no 
history of cardiothoracic surgery or trauma to 
the chest. 


What is the likely diagnosis? 

1- Vagus nerve palsy 

2- Horner's syndrome 

3- Hiatus hernia 


4- Phrenic nerve palsy 

5- Hepatomegaly 


Answer & Comments 


Answer: 4- Phrenic nerve palsy 


[ Q: 372 ] MRCPass - Respiratory 

In which of the following have 
randomised controlled trials shown that long¬ 
term oxygen therapy (LTOT) reduces 
mortality? 

1- Asthma 

2- Chronic obstructive lung disease 

3- Cryptogenic fibrosing alveolitis 

4- Cystic fibrosis 

5- Pulmonary sarcoidosis 



Causes of phrenic nerve palsy are : 

■ pneumonia 

■ pleurisy 

■ aortic aneurysm 

■ substernal goiter 

■ neoplasms 

■ thoracic surgery 

■ herpes zoster infection 

■ vasculitis 
diabetes 


Dr. Khalid Yusuf El-Zohry - Sohag Teaching Hospital (01118391123) 

Ref MRCPass OE OE 2012 PasTest 2009 PassMedicine 2009 PasTest Exam 




ReviseMRCP 

181 




























El-zohry MRCP Questions Bank (Port 1) - 2013 


(For my personal use) 



Raised Left hemidiaphragm 


When is LTOTindicated? 

1- p02 <7.2 kPa 

2- p0 2 <7.8 kPa 

3- p0 2 <8 kPa 

4- p0 2 <8.5 kPa 

5- p0 2 <9 kPa 


Answer & Comments 


Answer: 1- P02 <7.2 kPa 



[ Q: 374 ] MRCPass - Respiratory 

A 35 year old man has symptoms of 
wheezing and has been diagnosed as having 
late onset asthma. How ever, his GP measured 
routine blood tests and found that he had a 
creatinine of 250 umol/l. 


Whot antibody is likely to be helpful in 
confirming the diagnosis? 

1- Antinuclear antibody 

2- Anti phospholipids antibody 


When there is polycythaemia or pulmonary 
hypertension, Long Term Oxygen Therapy is 
indicated when p02 < 8kPa. In uncomplicated 
COPD, it is indicated when p02 < 7.2kPa 

[ Q: 376 ] MRCPass - Respiratory 

A 60 year old woman with asthma 
presents with a history of acute 
breathlessness and pleuritic chest pain. Her 
arterial blood gases show the following 
readings: 



3- Anti Ro antibody 

4- Anti nuclear cytoplasmic antibody 

5- Anti gliadin antibody 


pH 7.35, p0 2 6.8 kPa, pC0 2 4 kPa, bicarbonate 
25 mmol/L 

Which test is most specific to acute pulmonary 
embolism? 


Answer & Comments 

Answer: 4- Anti nuclear cytoplasmic antibody 

The clinical diagnosis is likely to be an ANCA 
positive small vessel vasculitis such as 
polyarteritis nodosa, as there is pulmonary 
and renal involvement. Churg Strauss 
syndrome should also be considered (only a 
small proportion of patients with Churg 
Strauss have a positive ANCA) 


1- MRI of the chest 

2- CT pulmonary angiogram 

3- Chest x-ray 

4- D-Dimers 

5- V/Q scan 

Answer & Comments 

Answer: 2- CT pulmonary angiogram 


[ Q: 375 ] MRCPass - Respiratory 

A 60 year old man has a long history 
of smoking and COPD. His resting p0 2 is 7.2 
kPa and his continues to be breathless despite 
being on home nebulisers. He is assessed for 
long term oxygen therapy (LTOT). 



Most of the tests are helpful but a CT 
pulmonary angiogram remains the gold 
standard diagnostic test for pulmonary 
embolism. 



[ Q: 377 ] MRCPass - Respiratory 
A 35 year old man presents with 



Dr. Khalid Yusuf El-Zohry - Sohag Teaching Hospital (01118391123) 

Ref MRCPass OE OE 2012 PasTest 2009 PassMedicine 2009 PasTest Exam ReviseMRCP 






























El-zohry MRCP Questions Bank (Port 1) - 2013 


(For my personal use) 


swelling of his lips and around the throat 
following consumption of prawns. His 

investigations show: 

serum IgE 150 kU/L (0-120) 

C3 level is 77 mg/dL (65-190) 

C4 level is 45 mg/dL (15-50) 

Which of the following diagnosis is likely? 

1- Cl esterase inhibitor deficiency 

2- Allergic reaction 

3- Systemic mastocytosis 

4- Moon face 

5- Cellulitis 

Answer & Comments 

Answer: 2- Allergic reaction 


Which one of the following diagnosis is likely? 

1- Aspergilloma 

2- Guillain-Barre syndrome 

3- Cryptogenic fibrosing alveolitis 

4- Asthma 

5- Congestive cardiac failure 

Answer & Comments 

Answer: 3- Cryptogenic fibrosing alveolitis 

A reduced FVC, with normal FEVi, FEVi% and 
PEF usually indicates restriction of lung 
volume . KCO (transfer factor) is also reduced 
in fibrotic lung disease, as in this case. The 
additional findings of inspirational crackles 
and clubbing suggests the diagnosis of 
cryptogenic fibrosing alveolitis. 


Mildly elevated IgE concentration suggests an 
allergic reaction to praw ns. In angioneurotic 
oedema due to Cl esterase inhibitor 
deficiency, a low C4 with normal C3 level is 
seen (C2 is also low but not commonly 
measured). 


[ Q: 378 ] MRCPass - Respiratory 

A 40 year old man has with a 2 year 
history of increasing shortness of breath. This 
is worse with exertion e.g. climbing the stairs 
at home and is even slightly so at rest. He is a 
non-smoker. 

On examination, his pulse is 95 beats/min and 
his blood pressure is 140/95 mmHg. He has 
finger clubbing and a crackling noise at the 
end of inspiration over the bases of the lungs. 
There was no ankle oedema Lung function 
tests show : 

FVC 2.5 I (predicted 3.2 ) 

FEVi 2.11 (predicted 2.4) 

FEVi % 76% (predicted 75%) 

Diffusing Capacity: Gas Transfer Factor for 
carbon monoxide: DLCO 17 ml/min/mmHg 
(predicted) 25 



[ Q: 379 ] MRCPass - Respiratory 

A 60 year old man has severe COPD 
requiring home nebulisers and home oxygen. 
He is admitted with an infective exacerbation. 

Which of the following results would be 
expected on the arterial blood gases? 

1- PH 7.25 paC0 2 7 pa0 2 7.5 HCOs 30 

2- PH 7.10 paC0 2 7 pa0 2 4 HCOs 24 

3- PH 7.30 paC0 2 4 pa0 2 8.5 HC0 3 30 

4- PH 7.40 paC0 2 4 pa0 2 4 HC0 3 22 

5- PH 7.45 paC0 2 3 pa0 2 12 HC0 3 24 



Answer & Comments 


Answer: 1- PH 7.25 paC02 7 pa02 7.5 HC03 
30 


A long standing COPD patient would be 
expected to have a high bicarbonate. However 
this patient is unwell with type II respiratory 
failure (high C02) and hence has 
uncompensated respiratory acidosis. 



[ Q: 380 ] MRCPass - Respiratory 


Dr. Khalid Yusuf El-Zohry - Sohag Teaching Hospital (01118391123) 

Ref MRCPass OE OE 2012 PasTest 2009 PassMedicine 2009 PasTest Exam ReviseMRCP 



183 

































El-zohry MRCP Questions Bank (Port 1) - 2013 


(For my personal use) 


A 60 year old miner has been in the 
occupation for 20 years. He presents with a 
cough and breathlessness. 

Chest XR shows diffuse interstitial shadowing. 
A sputum sample is positive for acid fast 
bacilli. 

Which of the following dusts is most likely to 
have predisposed the patient to tuberculosis? 

1- Beryllium 

2- Cadmium 

3- Coal 

4- Silica 

5- House dust 


Answer & Comments 


Answer: 4- Silica 


Slate workers, stonemasons and miners are 
exposed to silica dust. Silicosis impairs 
macrophage function, and in particular, 
predisposes to TB infection. 



[ Q: 381 ] MRCPass - Respiratory 

A 34-year-old woman presented 
with a dry cough, thorax constriction, and 
generalised weakness. During the preceding 5 
months, she had experienced these dry cough 
episodes tw ice a week. A lung function test 
showed a restriction of the vital capacity (71% 
of the adjusted reference value), and the 
diffusion capacity was also reduced (66 to 
68 %). 



Bronchoalveolar lavage specimens were 
obtained, the lymphocytes were increased up 
to 41% (norm, <10%), and neutrophils were 
increased up to 6% (norm, <2%) with a normal 
total cell count. 

What is the likely diagnosis? 

1- Asthma 

2- Pulmonary embolus 

3- Tuberculosis 

4- Extrinsic allergic alveolitis 

5- Alpha 1 antitrypsin deficiency 


Answer & Comments 

Answer: 4- Extrinsic allergic alveolitis 

Extrinsic Allergic Alveolitis is a type III or type 
IV response. There is no eosinophilia. IgG and 
lymphocytes are involved in immune 
response. Antigens of micropolyspora faeni 
and thermoactinomyces are 0.5-5 microns . 
These antigens which may be detected as 
serum precipitins. 

The acute form takes about 6 hours for 
sensitisation to the inhaled antigen. The 
chronic form may take weeks. 


[ Q: 382 ] MRCPass - Respiratory 

A 65 year old patient with COPD is 
on maximal treatment. 

Which one of the following is likely to prevent 
further disease progression? 

1- Steroids 

2- Beta agonist inhalers 

3- Stopping smoking 

4- Tiotropium 

5- Home oxygen 



Answer & Comments 


Answer: 3- Stopping smoking 


Dr. Khalid Yusuf El-Zohry - Sohag Teaching Hospital (01118391123) 

Ref MRCPass OE OE 2012 PasTest 2009 PassMedicine 2009 PasTest Exam ReviseMRCP 



184 



























El-zohry MRCP Questions Bank (Port 1) - 2013 


(For my personal use) 


In COPD, discontinuation of smoking is the 
only features which has been shown to reduce 
disease progression. 



[ Q: 383 ] MRCPass - Respiratory 

*tj 

A 60 year old male smoker has 


emphysema. 

His lung function tests show : 

FEVi is 0.5 (20% predicted) 

FVC is 2.2 (61% predicted) 

FEVi:FVC ratio of 26% 

His arterial blood gases show a p0 2 of 7.5 and 
7.2 on two separate occasions. He is mildly 
breathless at rest, but severely breathless on 
exertion. 

Whot is the best measure of his respiratory 
function? 

1- TLCO 

2 - FEVi 

3- FVC 

4- KCO 

5- TLC 


Answer & Comments 

Answer: 2- FEVi 

The breathlessness worsens considerably 
without much change in oxygen tension, 
suggesting that the cause of his dyspnoea is 
hyperinflation of his chest which worsens on 
exertion. 

Severity of emphysema is defined by the 
British Thoracic Society (BTS) in relation to 
FEVi, not FEVi:FVC ratio. Mild is 60-80% 
predicted; moderate 40-60% and severe 
<40%. 



[ Q: 384 ] MRCPass - Respiratory 
A 40 year old lady was admitted to 


hospital with fevers and cough productive of 
sputum. 

Chest X-ray shows diffuse patchy 
consolidation around the left lung. She has 
had a flu like illnes 4 weeks ago, and has a past 
medical history of asthma. She also smokes 10 
cigarettes a day. 

Which organism is likely to be responsible? 

1- Mycoplasma 

2- Pseudomonas 

3- Klebsiella 

4- Staphylococcus 

5- Tuberculosis 


Answer & Comments 

Answer: 4- Staphylococcus 

Following a viral infection, patients are 
predisposed to staphylococcal infection. The 
chest XR changes suggest staphlococcus 
rather than streptococcus (which would cause 
lobar consolidation). 


[ Q: 385 ] MRCPass - Respiratory 

• X 

A 50 year old man presents with a 3 
month history of cough and breathlessness. 
He was apyrexial on admission. 


His blood show ESR 60 mm/hr, urea 7 pmol/l, 
creatinine 100 pmol/l, sodium 137 mmol/I, 
potassium 4.1 mmol/l, corrected calcium 2.75 
(2.2-2.7) mmol/l, phosphate 0.82 (0.8- 

1.4)mmol/l. 


Chest X ray shows bilateral hilar 
lymphadenopathy and eggshell calcification. 

What is the likely diagnosis? 

1- Extrinsic allergic alveolitis 

2- Tuberculosis 


3- Sarcoidosis 


4- Allergic bronchopulmonary aspergillosis 

5- Leiomyoma 


Dr. Khalid Yusuf El-Zohry - Sohag Teaching Hospital (01118391123) 

Ref MRCPass OE OE 2012 PasTest 2009 PassMedicine 2009 PasTest Exam 




ReviseMRCP 

185 


























El-zohry MRCP Questions Bank (Port 1) - 2013 


(For my personal use) 


Answer & Comments 

Answer: 3- Sarcoidosis 

Sarcoidosis can cause many changes on the 
CXR. Among these are unilateral or bilateral 
hilar lymphadenopathy, diffuse parenchymal 
changes, eggshell calcification, pleural 
effusions and nodules. 



Bilateral hilar lymphadenopathy in sarcoidosis 


[ Q: 386 ] MRCPass - Respiratory 

A 40 year old woman presents with 
breathlessness, cough and fever. On 
examination, she has basal crackles in the lung 
fields. Circulating precipitans to 
Micropolyspora faeni are positive. 

Which of the following is the most likely 
diagnosis? 

1- Pigeon fanciers' lung 



The classic presentation of farmer's lung 
results from inhalational exposure to 
thermophilic Actinomyces species and 
occasionally from exposure to various 
Aspergillus species. 

Acute farmer's lung develops after large 
exposure to moldy hay or contaminated 
compost. Symptoms often spontaneously 
resolve within 12 hours to days if antigen 
exposure is eliminated or avoided. It manifests 
as new onset of fever, chills, nonproductive 
cough, chest tightness, dyspnea, headache, 
and malaise. 


[ Q: 387 ] MRCPass - Respiratory 

A 50 year old meat factory worker 
has been unw ell with fever, cough, sweats 
and lethargy. On examination, he had a 
purpuric rash and hepatosplenomegaly. 

What is the likely diagnosis? 

1- Pulmonary embolism 

2- Respiratory syncytial viral pneumonia 

3- Streptococcal pneumonia 

4- Q fever pneumonia 

5- Aspergilloma 



Answer & Comments 


Answer: 4- Q fever pneumonia 


2- Allergic Bronchopulmonary Aspergillosis 

3- Farmers' lung 

4- PCP infection 

5- Pulmonary fibrosis 


Answer & Comments 

Answer: 3- Farmers' lung 

Farmer's lung is the most common type of 
hypersensitivity pneumonitis. Hypersensitivity 
pneumonitis, also known as extrinsic allergic 
alveolitis, is associated with intense or 
repeated exposure to inhaled biologic dusts. 


Q fever is due to Coxiella burnetii and is 
acquired via animal contact. It can occur in 
outbreaks in farming communities and in 
abbatoirs. Treatment is with prolonged 
courses of tetracyclines. Rarely infection can 
be persistent leading to chronic symptoms 
including fatigue, malaise and sweats. 
Hepatitis, hepatosplenomegaly, 

maculopapular rash and endocarditis are 
associated. 



[ Q: 388 ] MRCPass - Respiratory 


A 50 year old woman, is admitted 
with malaise and fever. Four weeks previously 


Dr. Khalid Yusuf El-Zohry - Sohag Teaching Hospital (01118391123) 

Ref MRCPass OE OE 2012 PasTest 2009 PassMedicine 2009 PasTest Exam ReviseMRCP 



































El-zohry MRCP Questions Bank (Port 1) - 2013 


(For my personal use) 


she had suffered a chest infection for which 
she was given a course of oral amoxycillin. She 
felt better initially, but is now getting worse, 
complaining of intermittent fevers. Her chest 
radiograph shows a left sided pleural effusion. 

An aspirate shows fluid with a protein of 42 g 
and pH of 7.0. 

The most likely diagnosis is: 

1- Collapse of the left lung 

2- Empyema 

3- Serous pleural effusion 

4- Fibrotic lung disease 

5- Tuberculosis 


Answer & Comments 

Answer: 2- Empyema 

The likely cause is a pneumonia which has not 
resolved, leading to empyema. Examination of 
a pleural aspirate (microscopy for organisms, 
culture). A protein level >30g would be 
consistent with an exudate and a pH of < 7.2 is 
suggestive of infection. 



Empyema (air fluid level) 


Dr. Kholid Yusuf El-Zohry - Sohog Teaching Hospital (01118391123) 

Ref MRCPoss OE OE 2012 PosTest 2009 PassMedicine 2009 PosTest Exom 























El-zohry MRCP Questions Bank (Port 1) - 2013 


(For my personal use) 



[ Q: 389 ] MRCPass - Nephrology 

A 32 year old diabetic man has 
chronic renal failure and has on peritoneal 
dialysis for a few years. He is currently on 1- 
alfa-calcidol replacement. His calcium is 3 
mmol/l, phosphate 0.6 (0.8-1.4) mmol/l and 
PTH is 9 (0.8-9.0) pmol/l. 


Which of the following is the diagnosis? 


1- Hypercalcaemia secondary to vitamin D 
replacement 

2- Primary hyperparathyroidism 

3- Secondary hyperparathyroidism 


4- Tertiary hyperparathyroidism 

5- Normal finding in a dialysis patient 


Answer & Comments 

Answer: 4- Tertiary hyperparathyroidism 

Secondary hyperparathyroidism occurs in 
renal failure because of hypocalcaemia due to 
lack of 1 hydroxylation of 25(OH)Vit D. 
Tertiary hyperparathyroidism is due to 
longstanding secondary hyperparathyroidism. 
Despite Vit D replacement to treat secondary 
hyperparathyroidism, the parathyroid glands 
continue to secrete lots of PTH. Treatment 
should therefore be to remove the 
parathyroid glands surgically. 


[ Q: 390 ] MRCPass - Nephrology 

A 55 year old man with a history of 
hypertension has renal impairment. His 
creatinine is 250 umol/l. Urine dipstick shows 
blood ++ and protein +. 

Whot is the most useful investigation? 

1- Intravenous urogram 

2- Renal ultrasound 

3- Renal biopsy 

4- DTP A scan 

5- MRI of kidneys 



Answer & Comments 

Answer: 3- Renal biopsy 

Although there is a possibility of renovascular 
disease, this patient has urinalysis showing 
likely glomerulonephritis as a cause of renal 
impairment, hence renal biopsy is the best 
investigation. 



[ Q: 391 ] MRCPass - Nephrology 

A 55 year old man had a renal 
transplant 2 years ago. He is on ciclosporin, 
prednisolone and tacrolimus. He now has 
several symptoms. 


Which one of the following is a side effect of 
ciclosporin ? 


1- Hypokalaemia 

2- Alopecia 

3- Gum hypertrophy 


4- Hemiparesis 

5- Nephrotic syndrome 


Answer & Comments 

Answer: 3- Gum hypertrophy 

Hyperkalaemia, hirsutism, tremors, 
hypertension, nausea and vomiting, headache, 
gum hypertrophy, parasthesiae and 
hypomagnesaemia are side effects of 
ciclosporin. 



[ Q: 392 ] MRCPass - Nephrology 

A 25 year old man has loin pains on 
the left. His urine dipstick shows blood++ and 
no protein. MSU culture shows no growth. An 
abdominal XR shows 2 calculi in the left kidney 
with a ground glass appearance. 

Which of the following is the management? 


1- Treatment is with urine acidification 


2- D-penicillamine should be commenced 

3- He should be advised to drink less water 


Dr. Khalid Yusuf El-Zohry - Sohag Teaching Hospital (01118391123) 

Ref MRCPass OE OE 2012 PasTest 2009 PassMedicine 2009 PasTest Exam ReviseMRCP 



189 

































El-zohry MRCP Questions Bank (Port 1) - 2013 


(For my personal use) 


4- He should have the calculi removed 
surgically 

5- Bendrofluazide should be commenced 


Answer & Comments 

Answer: 2- D-penicillamine should be 
commenced 

The ground glass appearance of the stones 
suggests cysteine stones. Cystinuria, an 
autosomal recessive disease, results from 
excessive excretion of the four basic amino 
acids, cystine, ornithine, lysine, and arginine 
(COLA) into the urine. Cystine is relatively 
insoluble in acid urine. 


[ Q: 394 ] MRCPass - Nephrology 

A 40 year old woman has a history of 
Raynaud's phenomenon, stiffness in the 
hands, butterfly rash on her face and 
arthralgia. She has a blood pressure of 175/90 
mmHg. Urine dipstick shows protein ++ and 
blood +. 

Blood tests reveal a creatinine of 145 pmol/l, 
ESR 60 mm/hour and positive ANA 1/160 
titres. 

Which of the following is most likely to be the 
result of the renal biopsy? 

1- Mesangial IgA staining and proliferative 
glomerulonephritis 



Increased fluid intake is recommended to 
minimise calculi formation. D-penicillamine 
decreases the urinary excretion of cystine by 
binding cystine to form the more soluble 
cystine - S - penicillamine complex that is 50 
times more soluble than cystine. 


[ Q: 393 ] MRCPass - Nephrology 

A 65 year old man has a cystoscopy 
to investigate haematuria. He develops 
pyrexia a day after the procedure. 

Which one of the following organisms is most 
likely to be implicated? 

1- Proteus 

2- Enterococcus faecalis 

3- Pseudomonas 

4- Staph epidermidis 

5- Staph aureus 



Answer & Comments 

Answer: 4- Staph epidermidis 

The commonest organisms infecting causing 
UTI are E coli, and following that, klebsiella, 
proteus are next commonest and then 
enterococci. How ever, following cystoscopy / 
instrumentation, staph epidermidis (coagulase 
negative staph) infection is the commonest. 


2- Subendothelial deposits and focal 
glomerulonephritis 

3- Deposition of IgA, IgG, IgM and 
complements with proliferative 
glomerulonephritis 

4- Spikes on silver staining with crescentic 
glomerulonephritis 

5- Necrotising vasculitis 


Answer & Comments 

Answer: 3- Deposition of IgA, IgG, IgM and 
complements with proliferative 

glomerulonephritis 

There are various forms of lupus nephritis, but 
the pathognomonic feature is with a full 
house of immunostaining in the mesangium 
and capillary loops with all immunoglobulins 
and complements. 

Mesangial IgA staining is seen in IgA 
nephropathy. 

Spikes on silver staining is seen in 
membranous glomerulonephritis. 

Necrotising vasculitic changes are seen in 
small vessel vasculitis. 


Dr. Khalid Yusuf El-Zohry - Sohag Teaching Hospital (01118391123) 

Ref MRCPass OE OE 2012 PasTest 2009 PassMedicine 2009 PasTest Exam ReviseMRCP 



























El-zohry MRCP Questions Bank (Port 1) - 2013 


(For my personal use) 


[ Q: 395 ] MRCPass - Nephrology 

A diabetic patient is assessed for 
proteinuria in outpatients. 

Which of the following urine albumin 
concentrations over 24 hours signify 
microalbuminuria? 

1- 1 mg 

2- 10 mg 

3- 100 mg 

4- 500 mg 

5- lg 



HC0 3 40 mmol/l 

BE + 18 mmol/l 

What is the likely diagnosis? 

1- Bulimia 

2- Bartter's syndrome 

3- Diuretic abuse 

4- Laxative abuse 

5- Congenital adrenal hyperplasia 


Answer & Comments 


Answer: 2- Bartter's syndrome 


Answer & Comments 

Answer: 3-100 mg 

Microalbuminuria defined as a urine albumin 
excretion between 30-300 mg/day. A 
concentration above 300 mg/day signifies 
albuminuria. A concentration above 3g/day 
signifies overt proteinuria in the nephrotic 
range. 


[ Q: 396 ] MRCPass - Nephrology 

A 15 year old boy is being 
investigated for growth retardation. He has 
had no previous medical problems. He 
mentions that he had symptoms of worsening 
muscle weakness and fatigue. He also has 
polyuria. 

Investigations revealed: 
urea 8 mmol/l 

serum creatinine 118 pmol/l 
potassium 2.8 mmol/l 
sodium 133 mmol/l 
chloride 79 (95-107) mmol/l 
calcium 2.3(2.25-2.7) mmol/l 
phosphate 0.86 (0.8-8) mmol/l 
magnesium 0.9 (0.67-0.96) mmol/l 
A metabolic alkalosis was present: 
pH 7.58 



Bartter's syndrome is hypokalaemic alkalosis 
due to hyperaldosteronism. 

Classical Bartter's syndrome is characterized 
by early childhood onset and is due to 
defective chloride transport across the 
basolateral membrane in the distal nephron as 
a result of mutations in the chloride channel 
gene. 

Symptoms may include polyuria, polydipsia, 
vomiting, constipation, salt craving, and a 
tendency to dehydration. 

Hypokalaemia can lead to muscle weakness, 
spasms, tetany, or palpitations. Urine calcium 
excretion is high, leading to nephrocalcinosis, 
while serum magnesium levels are normal. 



[ Q: 397 ] MRCPass - Nephrology 

A 50 year old man presents with a 15 
year history of lithium carbonate therapy for 
bipolar affective disorder. He is polyuric and 
has 3 g/24 hours of proteinuria. Creatinine is 
145 micromol/l. 


What is a renal biopsy likely to show ? 

1- ANCA positive vasculitis 

2- Crescentic glomerulonephritis 

3- Minimal change glomerulonephritis 

4- Interstitial nephritis 

5- IgA glomerulonephritis 


Dr. Khalid Yusuf El-Zohry - Sohag Teaching Hospital (01118391123) 

Ref MRCPass OE OE 2012 PasTest 2009 PassMedicine 2009 PasTest Exam 




ReviseMRCP 

191 
























El-zohry MRCP Questions Bank (Port 1) - 2013 


(For my personal use) 


Answer & Comments 

Answer: 4- Interstitial nephritis 

With lithium toxicity, chronic interstitial 
nephritis occurs. A secondary FSGS lesion due 
to hyperfiltration of remnant nephrons can be 
seen in 30% of cases. 


[ Q: 398 ] MRCPass - Nephrology 

A 75 year old woman was brought to 
the hospital following a fall. Her neighbour 
thinks that she was on the floor for about 4 
days. 

Her blood results show: sodium 132 mmol/l, 
potassium 6.5 mmol/l, urea 38 pmol/l, 
creatinine 650 pmol/l, calcium 1.9 (25-2.7) 
mmol/l, phosphate 2.5 (0.8-1.4) mmol/l. 

Which of the following investigations is most 
likely to reveal the diagnosis? 

1- Ultrasound of the kidneys 

2- MRA of the renal arteries 

3- BM stick 

4- Urine dipstick and microscopy 

5- DMSA scan 



Answer & Comments 

Answer: 4- Urine dipstick and microscopy 

The most likely diagnosis is rhabdomyolysis 
and acute renal failure. 

The history is suggestive along with 
hyperkalaemia, hypocalcaemia and 
hyperphosphataemia. 

Urine dipstick will be positive for blood 
(myoglobinuria) but microscopy will not show 
red cells. A renal biopsy may also reveal the 
diagnosis. Treatment is with hydration 
(normal saline) and alkalinsation of urine with 
sodium bicarbonate infusion. 


[ Q: 399 ] MRCPass - Nephrology 

A man who is on antihypertensives 
has several symptoms and his medications 
require reviewing. 

Which one of the following is a complication of 
thiazide diuretic therapy? 

1- Neutrophilia 

2- Hyperkalaemia 

3- Gynaecomastia 

4- Hyperuricaemia 

5- Peptic ulceration 



Answer & Comments 

Answer: 4- Hyperuricaemia 

Thiazides can cause thrombocytopaenia, 
hypokalaemia (blocking NaCI channels), and 
hyperuricaemia. Gynaecomastia is caused by 
spironolactone. 


[ Q: 400 ] MRCPass - Nephrology 

A 50 year old man presents with 
shortness of breath and ankle oedema. 

Investigations show: 

urea 7 pmol/l 

creatinine 88 pmol/l 

albumin 18 g/L 

24 hour urinary protein excretion lOg (<0.2 g) 

Renal biopsy showed normal glomeruli and 
renal tubule appearances with 
immunofluorescence. 

What is the likely diagnosis? 

1- Minimal change disease 

2- Focal segmental glomerulosclerosis 

3- Renal vein thrombosis 

4- IgA nephropathy 

5- Membranous nephropathy 



Dr. Khalid Yusuf El-Zohry - Sohag Teaching Hospital (01118391123) 

Ref MRCPass OE OE 2012 PasTest 2009 PassMedicine 2009 PasTest Exam ReviseMRCP 






























El-zohry MRCP Questions Bank (Port 1) - 2013 


(For my personal use) 


Answer & Comments 

Answer: 3- Renal vein thrombosis 

Focal segmental glomerulosclerosis, 
membranous nephropathy, minimal change 
disease and IgA nephropathy would all show 
histological changes. 

Renal vein thrombosis can present with 
nephrotic syndrome, haematuria, loin pain 
and worsening renal failure. 

The most common cause is in fact, nephrotic 
syndrome. Other causes are renal cell cancer, 
renal transplantation, Behget syndrome, 
hypercoagulable states, and antiphospholipid 
antibody syndrome. Treatment is with 
heparin/w arfarin. 


[ Q: 401 ] MRCPass - Nephrology 

Which one of the following is the 
commonest cause of nephrotic syndrome in 
adults? 

1- Minimal change glomerulonephritis 

2- Post streptoccoccal glomerulonephritis 

3- IgA nephropathy 

4- Membrano proliferative glomerulonephritis 

5- Membranous nephropathy 



Answer & Comments 


Answer: 5- Membranous nephropathy 


transplant. Since the transplant, he has been 
on the following medications: 

Cyclosporine 125 mg BD 

Mycophenolate 1000 mg BD 

Prednisone 20 mg od 

He has, over the past week, been complaining 
of joint pains and upper abdominal pains. His 
temperature is currently 37.5°C and charts 
showed a fever for the last 3 days. 

Blood results showed: 

Hb 13.5 g/dl WCC 2.8 x 10 9 /L 

platelets 130 x 10 9 /L sodium 137 mmol/l 

potassium 4.8 mmol/l urea 10 mmol/l 

creatinine 127 pmol/l ALT 88 (5-35) U/l 

ALP 70 (20-120) U/l 

Bilirubin 18 (1-22) pmol/l 

Serology results showed: 

CMV IgM antibody - positive 

EBV IgG antibody - positive 

HIV test - positive. 

What is the likely cause of this presentation? 

1- CMV infection 

2- EBV infection 

3- Acute graft vs host disease 

4- Pneumocystic carinii 

5- Immunosuppressive drugs 


Membranous nephropathy is the commonest 
cause of the nephrotic syndrome in adults, 
whilst in children it is minimal change disease. 
The renal biopsy with membranous 
nephropathy shows a thickened glomerular 
basement membrane and granular IgG + C3 on 
immunostaining. 

^ [ Q: 402 ] MRCPass - Nephrology 

/ - 

# A 46 year old man had a right sided 

renal transplant 4 weeks ago. He has a 
creatinine which was 118 umol/l on day 1 post 


Answer & Comments 

Answer: 1- CMV infection 

The patient is at high risk of CMV infection 
due to immunosuppression and HIV infection. 

CMV infection post transplant can cause 
damage to the transplant graft and the 
presentation can be similar to graft rejection. 
Most cases of symptomatic CMV infection can 
be characterized by a self-limiting syndrome 
of episodic fever spikes for a period of 3 to 4 


Dr. Khalid Yusuf El-Zohry - Sohag Teaching Hospital (01118391123) 

Ref MRCPass OE OE 2012 PasTest 2009 PassMedicine 2009 PasTest Exam 




ReviseMRCP 

193 


























El-zohry MRCP Questions Bank (Port 1) - 2013 


(For my personal use) 


weeks, arthralgias, fatigue, anorexia, 
abdominal pain and diarrhea. 

Blood tests often show leukopenia (<3 x 10 9 /l), 
thrombocytopenia (150 x 10 9 /l) and liver 
enzyme elevation (ALT >50 U/l). 

Acute transplant rejection usually occurs 
within 5-10 days of the transplant. 

CMV is one of the pathogens that cause the 
most serious opportunistic viral infections in 
HIV-positive patients and is one of the most 
common causes of AIDS-related gastritis. 


[ Q: 404 ] MRCPass - Nephrology 

A 35 year old woman complains of 
renal colic. 

Which one of the following is the most 
common form of renal calculi? 

1- Calcium phosphate 

2- Cystine 

3- Calcium carbonate 

4- Calcium oxalate 

5- Urate 



^ [ Q: 403 ] MRCPass - Nephrology 

fm - 

# A 40 year old man with type 1 

diabetes attends a follow up appointment in 

the diabetic clinic. He has a blood pressure of 

145/86 mmHg. There is no evidence of 

diabetic retinopathy or neuropathy. He has an 

elevated albumin:creatinine ratio in his urine. 

Creatinine is 90 pmol/l. 

Which of the following should be done? 

1- Intravenous urogram 

2- Commence on an Angiotensin Receptor 
Blocker 

3- Better glycaemic control 

4- Consider a islet cell transplant 

5- Review blood pressure in 2 months 


Answer & Comments 

Answer: 2- Commence on an Angiotensin 
Receptor Blocker 

The patient has a raised albumin:creatinine 
ratio in the urine. He is likely to have 
microalbuminuria, which is a sign of risk to 
progressive diabetic nephropathy. 
Commencing an ARB to control hypertension 
has been show n to be of benefit in slowing 
the progression of nephropathy. 


Answer & Comments 

Answer: 4- Calcium oxalate 

Calcium oxalate are the commonest (75%). 
Others are calcium phosphate (20%), urate 
(5%) and cystine (1%). 


^ [ Q: 405 ] MRCPass - Nephrology 

# A 25 year old man has symptoms of 
lethargy. Blood results reveal a urea of 32 
pmol/l and creatinine of 350 pmol 


Which of the following indicates chronic 
(rather than acute) renal failure? 

1- Hyperkalaemia 

2- heavy proteinuria 

3- Urine osmolality of 300 m osmol/kg 

4- Hyponatraemia 

5- Anaemia 


Answer & Comments 

Answer: 5- Anaemia 

Anaemia (inadequate erythropoietin), renal 
osteodystrophy, small scarred kidneys suggest 
chronic rather than acute renal failure. 


Dr. Khalid Yusuf El-Zohry - Sohag Teaching Hospital (01118391123) 

Ref MRCPass OE OE 2012 PasTest 2009 PassMedicine 2009 PasTest Exam ReviseMRCP 



194 





























El-zohry MRCP Questions Bank (Port 1) - 2013 


(For my personal use) 


[ Q: 406 ] MRCPass - Nephrology 

A 50 year old woman who is 
morbidly obese presents with a plasma 
creatinine of 250 micromoles/l. 

Which form of disease is likely? 

1- Renal tubular acidosis 

2- Focal segmental glomerulosclerosis 

3- Nephritic syndrome 

4- Membranous glomerulonephritis 

5- Cholesterol emboli 



Answer & Comments 

Answer: 2- Focal segmental glomerulosclerosis 

There is an association between severe 
obesity and focal segmental 
glomerulosclerosis. 

[ Q: 407 ] MRCPass - Nephrology 

A 45 year old patient is admitted 
with decreased conscious level. She is oliguric 
and hyperventilating. Her investigations show 
: sodium 125, potassium 6.0, chloride 92 (95- 
107), Urea 15 mmol/I, creatinine 220 
micromol/L, arterial blood gas pH 7.2, 
bicarbonate 16 mmol/I. 

What is the anion gap? 

1- 14 

2- 16 

3- 18 

4- 23 

5- 25 



Answer & Comments 

Answer: 4- 23 

The anion gap is calculated with the formula 
(Na+K)-(CI+HC03). In this case it is 125 + 6-92 
-16 = 23. 


This example indicates high anion gap, 
possibly due to lactic acidosis or an overdose 
of a substance. 



[ Q: 408 ] MRCPass - Nephrology 

A 50 year old man with a history of 
hypertension was investigated for renal 
impairment. Renal angiography confirms renal 
artery stenosis. 


Which one of the following features is an 
indication for stenting in renal artery stenosis? 


1- Flash pulmonary oedema 


2- Abdominal bruit 


3- Heavy proteinuria 

4- Unilateral small kidney 

5- Hypotension 


Answer & Comments 

Answer: 1- Flash pulmonary oedema 

Revascularization is considered when the 
presence of hemodynamically significant renal 
artery stenosis is judged to be contributing to 
poorly controlled hypertension or progressive 
renal impairment. Other less common but 
equally important clinical indications for RAS 
revascularization include episodic pulmonary 
edema, congestive cardiac failure, and 
unstable angina. 


[ Q: 409 ] MRCPass - Nephrology 

A 20 year old male has taken a drug 
overdose of multiple drugs. 

Which one of the following can be effectively 
removed by haemodialysis? 

1- Amiodarone 

2- Digoxin 

3- Phenytoin 

4- Lithium 

5- Paraquat 



Dr. Khalid Yusuf El-Zohry - Sohag Teaching Hospital (01118391123) 

Ref MRCPass OE OE 2012 PasTest 2009 PassMedicine 2009 PasTest Exam 




ReviseMRCP 

195 































El-zohry MRCP Questions Bank (Port 1) - 2013 


(For my personal use) 


Answer & Comments 


Answer: 4- Lithium 


Answer & Comments 


Answer: 1- Renal amyloidosis 


Drugs with a large volume of distribution are 
poorly dialysed. Haemodialysis is effective in 
lithium poisoning. 



[ Q: 410 ] MRCPass - Nephrology 
A 25 year male diagnosed type I 


diabetes. 

Whot ore his chances of progressing tow ords 
End Stage Renal Disease (ESRD)? 


1 - 10 % 

2- 25% 

3- 50% 

4- 75% 

5- 100% 


Answer & Comments 

Answer: 3- 50% 

In type I diabetes, there is a 50% chance of 
progressing tow ards ESRD. In type II diabetics, 
there is a 15% chance of doing so. 


Renal vein thrombosis is most commonly 
associated with nephrotic syndrome, the 
commonest due to membranous 
glomerulonephritis. Other causes are 
nephrotic syndrome due to amyloidosis, 
vasculitis, dehydration and congestive cardiac 
failure. Treatment is with anticoagulation, and 
when response is poor, thrombolysis and 
thrombectomy can be considered. 


[ Q: 412 ] MRCPass - Nephrology 

An 8 year old boy has bowing of the 
weight-bearing long bones. 

Which one of the following is a feature of X- 
linked hypophosphataemic vitamin D - 
resistant rickets? 

1- High serum phosphate 

2- High urinary phosphate 

3- High parathyroid hormone (PTH) levels 

4- Hypercalcaemia 

5- Low bicarbonate 



^ [ Q: 411 ] MRCPass - Nephrology 

if - 

# A 55 year old man has a creatinine of 
280 pmol/l. He mentions a 10 day history of 
loin pain and pink coloured urine. An 
ultrasound of the kidney suggests renal vein 
thrombosis. 

Which of the following conditions is most likely 
to be associated? 

1- Renal amyloidosis 

2- Interstitial nephritis 

3- Systemic sclerosis 

4- Renal calculus 

5- Reflux nephropathy 


Answer & Comments 

Answer: 2- High urinary phosphate 

In X-linked hypophosphataemic Vit D resistant 
rickets, serum phosphate is low and urine 
phosphate is high due to inappropriate renal 
phosphate wasting. Serum parathyroid levels 
are usually normal or slightly elevated. 
Clinically, the most obvious of these aspects is 
the effect on bone formation and growth that 
causes very severe rickets, especially in 
affected males. Treatment is with oral 
phosphate (difficult to tolerate) and high dose 
activated Vitamin D. 



[ Q: 413 ] MRCPass - Nephrology 
A 55 year old woman has end stage 


Dr. Khalid Yusuf El-Zohry - Sohag Teaching Hospital (01118391123) 

Ref MRCPass OE OE 2012 PasTest 2009 PassMedicine 2009 PasTest Exam ReviseMRCP 


































El-zohry MRCP Questions Bank (Port 1) - 2013 


(For my personal use) 


renal failure and is being considered for 
dialysis. 

In which one of the following situations, might 
peritoneal dialysis be preferable to 
hemodialysis? 

1- In hypercatabolic patients 

2- Recent abdominal wound 

3- In patients bordering on respiratory failure 

4- In diabetic patients 

5- When there is a need to conserve plasma 
protein levels 


Answer & Comments 

Answer: 4- In diabetic patients 

Peritoneal dialysis can be useful for 
intraperitoneal insulin administration. 

Hemodialysis is preferable in hypercatabolic 
patients for rapid urea clearance. Stress is 
placed on the abdominal w ound healing by 
PD, and on the diaphragm in respiratory 
failure. There is a loss of protein by diffusion 
in PD, hence less useful when protein needs to 
be conserved. 


[ Q: 414 ] MRCPass - Nephrology 

* A 42 year old man is referred to the 
renal physician due to symptoms of 
haemoptysis and haematuria. These 
symptoms have been going on for 6 months. 
Investigations show: 

sodium 135 mmol/l 

potassium 4.6 mmol/l 

urea 18 mmol/l 

creatinine 260 pmol/l 

The likely disease is caused by an antibody 
against which of the following? 

1- Smooth muscle 

2- DsDNA 

3- Proteinase 3 


4- Macrophages 

5- Microfilaments 


Answer & Comments 

Answer: 3- Proteinase 3 

The diagnosis would fit Wegener's 
granulomatosis, which is commonly cANCA 
positive. In patients with vasculitis, 
approximately 90 percent of cytoplasmic 
ANCA are PR3-ANCA and approximately 90 
percent of perinuclear ANCA are MPO-ANCA. 


[ Q: 415 ] MRCPass - Nephrology 

A 45 year old woman presents with 
dipstick positive haematuria. Her blood tests 
reveal a urea of 25 pmol/l and creatinine of 
260 pmol/l. 

Which of the following most strongly suggests 
post streptococcal glomerulonephritis? 

1- History of previous attacks 

2- Papilloedema 

3- Sore throat 10 days ago 

4- Complete anuria 

5- Nephrotic syndrome 



Answer & Comments 

Answer: 3- Sore throat 10 days ago 

Post streptococcal glomerulonephritis is 
caused by Group A beta haemolytic 
streptococci. A second episode is unusual in 
those who have previously had the disease 
because there is good immunity to the cell 
wall proteins of Group A strep. Although 
features of hypertension with papilloedema, 
complete anuria and nephrotic syndrome can 
occur, the strongest predictor is the clinical 
history of sore throat suggesting infection 
with streptococci leading to nephritic 
syndrome. 


Dr. Khalid Yusuf El-Zohry - Sohag Teaching Hospital (01118391123) 

Ref MRCPass OE OE 2012 PasTest 2009 PassMedicine 2009 PasTest Exam 




ReviseMRCP 

197 

























El-zohry MRCP Questions Bank (Port 1) - 2013 


(For my personal use) 



This glomerulus is hypercellular and capillary 
loops are poorly defined in post streptococcal 

glomerulonephritis. 



[ Q: 416 ] MRCPass - Nephrology 

A 20 year old patient complains of 
ankle sw elling and breathlessness. Her CXR 
shows pleural effusions. 

Blood tests show : 

urea 7 pmol/l 

creatinine 80 pmol/l 


sodium 138 mmo 


potassium 4.2 mmol/l 
alanine transferase 20 U/L 
bilirubin 16 umol/L 
albumin 20 g/L (34-94) 
total cholesterol 8.8 mmo 


Whot is the next best investigation? 

1- Ultrasound of liver 


Proteinuria 

Hypoalbuminemia 

Oedema 

This is frequently accompanied by 
Hypercholesterolemia 

[ Q: 417 ] MRCPass - Nephrology 

A 45 year old man with no previous 
past medical history presents with a BP of 
180/100 mm Hg, frothy urine and peripheral 
oedema. There is + blood and +++ protein on 
urinalysis. 24-hour protein loss is 5 grams. 

Plasma albumin is 25 g/L. Plasma C3 is 0.10 
(low ). Plasma creatinine is 160 umol/l. A renal 
biopsy is performed. 

Which of the following is most likely to be 
found on the biopsy? 

1- Minimal change glomerulonephritis 

2- Mesangiocapillary glomerulonephritis 

3- IgA nephropathy 

4- Post streptococcal glomerulonephritis 

5- Focal segmental glomerulosclerosis 



Answer & Comments 

Answer: 2- Mesangiocapillary 

glomerulonephritis 


2- Antinuclear antibody 

3- Urinary protein estimation 

4- Renal biopsy 

5- 24 hour urine creatinine clearance 

Answer & Comments 

Answer: 3- Urinary protein estimation 

The high cholesterol and low albumin suggests 
nephrotic syndrome (>3g protein excretion in 
24 hour urine collection). 

The triad of features of nephrotic syndrome 
are: 


This man has nephrotic syndrome 

(hypoalbuminaemia, oedema, and 
proteinuria). MCGN presenting in a young 
man with nephrotic syndrome and 

hypertension and hypocomplementaemia 
would fit best. Post streptococcal 

glomerulonephritis is possible but the 
infection should be clinically apparent, or in 
the history. 

The two most common variants of MCGN are 
type I MCGN and type II MCGN (also called 
dense deposit disease). 

Type I is much more common than type II, 
which is a rare disease. They are characterized 


Dr. Kholid Yusuf El-Zohry - Sohog Teaching Hospital (01118391123) 

Ref MRCPass OE OE 2012 PasTest 2009 PassMedicine 2009 PasTest Exam 




ReviseMRCP 


198 


i 


























El-zohry MRCP Questions Bank (Port 1) - 2013 


(For my personal use) 


by capillary basement membrane thickening 
and mesangial cell proliferation. They are 
associated with low levels of C3. 
Mesangiocapillary glomerulonephritis is a 
significant cause of nephrotic syndrome in 
children (accounts for about 8% of cases) and 
adults (accounts for about 14%). 



The figure shows MCGN with coarse granular 
depostition of IgA, IgM and C3. 


[ Q: 418 ] MRCPass - Nephrology 

A 60 year old lady has been noted 
for the first time to have renal impairment. 

Which of the following features is most useful 
in determining whether the renal failure is 
acute or chronic? 

1- Renal size of 7.5 and 8 cm 

2- Hbof 12 g/dl 

3- Blood pressure of 160/90 

4- PTH level of 8 pmol/l 

5- Phosphate of 2 mmol/l 



Answer & Comments 

Answer: 1- Renal size of 7.5 and 8 cm 

Out of all the options small renal size is the 
best feature suggesting chronic renal failure. 


[ Q: 419 ] MRCPass - Nephrology 

A 15 year girl presents with a 
nonblanching rash over his shins and a sw 
ollen knee. He has noticed haematuria. 



Blood test show a urea of 12 pmol/l and 
creatinine 220 pmol/l. Urine dipstick show s: 
blood +++, protein +. 

What is the renal biopsy likely to show ? 

1- Tubular necrosis 

2- Podocyte fusion 

3- Mesangial deposits of IgA 

4- Thickening of basement membranes 

5- Focal segmental sclerosis 

Answer & Comments 

Answer: 3- Mesangial deposits of IgA 

The clinical presentation fits Henoch Schonlein 
purpura. This is an autoimmune disorder 
(cause unknow n but associated with 
mycoplasma), which is related to 
multisystemic IgA deposition. It is usually a 
self-limited disease. It is the most common 
form of childhood vasculitis and results in 
inflammatory changes in small vessels. 

The symptoms of Henoch-Schonlein Purpura 
usually begin suddenly and may include 
headache, fever, loss of appetite, cramping 
abdominal pain, and joint pain. Red or purple 
spots typically appear on the skin (petechial 
purpura). Inflammatory changes associated 
with Henoch-Schonlein Purpura can also 
develop in the joints, kidneys, digestive 
system, and, in rare cases, the brain and spinal 
cord. Mesangial Ig A deposits are typical 
features of HSP on the renal biopsy. 


/ i 

1 _ m 

i 

m 


Henoch Schonlein Purpura 


Dr. Khalid Yusuf El-Zohry - Sohag Teaching Hospital (01118391123) 

Ref MRCPass OE OE 2012 PasTest 2009 PassMedicine 2009 PasTest Exam ReviseMRCP 



199 
























El-zohry MRCP Questions Bank (Port 1) - 2013 


(For my personal use) 


^ [ Q: 420 ] MRCPass - Nephrology 

fi - 

# A 50 year old lady has been 
diagnosed with type 2 diabetes, a year ago. 
Since the diagnosis was made, she was found 
to have pre-proliferative diabetic retinopathy. 
She has a history of rheumatoid arthritis and 
has been taking ibuprofen regularly for 6 
years. She was referred for investigation of 
renal impairment. 

The HbAlC is 9 

urea 15 mmol/l 

creatinine 200 pmol/l 

Urine dipstick shows blood +, protein ++ 

What is the likely diagnosis? 

1- Diabetic nephropathy 

2- Amyloidosis 

3- Chronic interstitial nephritis 

4- Renal calculi 

5- Acute analgesic nephropathy 


Answer & Comments 

Answer: 1- Diabetic nephropathy 

Diabetic nephropathy, amyloidosis and 
chronic interstitial nephritis are all possible 
answers. The rationale is that the patient has 
had poorly controlled diabetes for a while in 
view of the high HbAlC and retinopathy, and 
has developed nephropathy. 

Diabetic Nephropathy progresses over 10 to 
25 years. Hyperfiltration without 
microalbuminuria appears initially and 
advances to a GFR 20 to 50% above normal 
and microalbuminuria > 300 mg/24 h. GFR 
normalizes with early renal injury and mild 
hypertension and progresses to frank 
hypertension, with proteinuria > 0.5 g/day. 
Heavy proteinuria and a progressive decline in 
renal function precedes end-stage renal 
disease. 


[ Q: 421 ] MRCPass - Nephrology 

A 30 year old man had progressive 
deafness and hematuria from childhood and 
had undergone renal transplant eight years 
ago for chronic renal failure. Examination 
revealed perceptive high-tone deafness and 
posterior lenticonus in both eyes. 

What is the likely diagnosis? 

1- Neurofibromatosis type II 

2- Romano Ward syndrome 

3- Alport's syndrome 

4- Adult polycystic kidney disease 

5- Wegener's granulomatosis 



Answer & Comments 

Answer: 3- Alport's syndrome 

Classic Alport's syndrome is an X linked 
disorder characterised by a triad of 
progressive hematuric nephritis, progressive 
perceptive high-tone hearing loss, and ocular 
signs (cataracts, spherophakia, and posterior 
lenticonus). 



[ Q: 422 ] MRCPass - Nephrology 

A 14 year old boy presents with 
generalised oedema. There was no relevant 
past medical history. 


Investigations revealed: Creatinine 75 pmol/l, 
Albumin 20 g/l, 24 hour urine protein 
excretion 3.2 g. Ultrasound scan shows normal 
renal size. 


Which of the following should the patient be 
treated with initially? 

1- Cyclophosphamide 

2- Peritoneal dialysis 

3- Thiazides 


4- Haemodialysis 

5- Prednisolone 


Dr. Khalid Yusuf El-Zohry - Sohag Teaching Hospital (01118391123) 

Ref MRCPass OE OE 2012 PasTest 2009 PassMedicine 2009 PasTest Exam ReviseMRCP 




























El-zohry MRCP Questions Bank (Port 1) - 2013 


(For my personal use) 


Answer & Comments 

Answer: 5- Prednisolone 

The diagnosis is minimal change 
glomerulonephritis presenting with nephrotic 
syndrome. 80% of patients achieve remission 
with prednisolone therapy. Cyclophosphamide 
treatment can be started for those who are 
steroid resistant. 


[ Q: 423 ] MRCPass - Nephrology 

A 16 year old girl has presented with 
joint pains, abdominal pains and has a rash on 
her lower limbs. 



patients with renal involvement, as many as 
10% may develop chronic renal failure and 
end-stage renal disease. Biopsy of affected 
renal tissue shows a spectrum of glomerular 
disease from minimal change to severe 
crescentic glomerulonephritis. IgA, C3, fibrin, 
properdin are seen as granular mesangial 
deposits on direct immunofluorescence. 


[ Q: 424 ] MRCPass - Nephrology 

A 25 year old woman has recurrent 
UTIs during childhood. She is now 28 weeks 
pregnant and has another episode of 
pyelonephritis. Her renal function is normal. 



Investigations show: 

Urea 14 mmol/I 

Creatinine 180 umol/l 

Urine dipstick: blood ++, protein + 

A renal biopsy was performed and results 
showed - IgA linear deposited in basement 
membrane. 


What is the likely predisposing problem? 

1- Toxic shock syndrome 

2- Renal calculi 

3- Reflux nephropathy 

4- Pre eclampsia 

5- Polycystic kidneys 


Whot is the diagnosis? 

1- Haemolytic uraemic syndrome 

2- Goodpasture's syndrome 

3- Wegener's granulomatosis 

4- Henoch Schonlein purpura 

5- IgA nephropathy 


Answer & Comments 


Answer: 4- Henoch Schonlein purpura 


In Henoch Schonlein purpura, a purpuric rash 
typically appear on the lower extremities and 
buttocks, but may also involve the upper 
extremities, face and trunk, and are 
accentuated in areas of pressure. HSP 
nephritis usually presents as macroscopic 
hematuria and proteinuria lasting days to 
weeks. 

These may be accompanied by increased 
plasma creatinine and/or hypertension, 
followed by microscopic hematuria. Of those 


Answer & Comments 


Answer: 3- Reflux nephropathy 

Pyelonephritis is not a common presentation 
in pregnancy and should be treated 
aggressively with antibiotics. In a patient with 
recurrent childhood UTIs, reflux nephropathy 
predisposes to pyelonephritis due to renal 
scarring. 



[ Q: 425 ] MRCPass - Nephrology 

A 25 year old woman is in the third 
semester of pregnancy. She has several 
investigations for proteinuria. 

Which one of the following results is 
significant? 

1- GFR 140 ml/min 


2- Urea 2 mmo 


3- Uric acid 1.5 mmol/I 

4- Creatinine 60 pmol/l 


Dr. Khalid Yusuf El-Zohry - Sohag Teaching Hospital (01118391123) 

Ref MRCPass OE OE 2012 PasTest 2009 PassMedicine 2009 PasTest Exam 




ReviseMRCP 

201 






























El-zohry MRCP Questions Bank (Port 1) - 2013 


(For my personal use) 


5- Sodium 135 mmol/l 


IgA nephropathy 


Answer & Comments 

Answer: 3- Uric acid 1.5 mmol/l 

Normal uric acid level is < 0.4 mmol/l. A high 
uric acid level may indicate pre-eclampsia. 


Henoch-Schonlein purpura 
systemic lupus erythematosus 
mixed cryoglobulinemia 
Goodpasture's syndrome 



[ Q: 426 ] MRCPass - Nephrology 

A 73 year old man presented with a 
two day history of a pruritic rash and oedema 
affecting both lower legs. 


Examination showed non-palpable, non¬ 
tender purpura affecting both legs, with 
pitting oedema, and urticarial lesions on the 
left knee and anterior chest. Testing of urine 
was positive for protein and blood. Urine 
microscopy showed > 100 erythrocytes per 

10 6 /l. 


What is the clinical scenario consistent with? 


1- Renal tubular acidosis 

2- Nephrotic syndrome 

3- Nephritic syndrome 

4- Renal artery stenosis 

5- Renal cell carcinoma 


Answer & Comments 

Answer: 3- Nephritic syndrome 

Acute nephritic syndrome most often results 
from infection by streptococcus. 

Infectious causes of acute nephritic syndrome 
are: 


Wegener's granulomatosis 


[ Q: 427 ] MRCPass - Nephrology 

A 45 year old man feels unwell 
following a sore throat and cough. He has a 
blood pressure of 170/90 mHg. 

Dipstick of his urine reveals blood ++ and 
protein ++. His blood tests show: 

Hb 10.5 g/dl, MCV 85 fl, WCC 7 x 10 9 /L, 
platelets 200 x 10 9 /L, sodium 135 mmol/l, 
potassium 4.2 mmol/l, urea 15 pmol/l, 
creatinine 260 pmol/l, ALT 25 (5-35) U/l, AST 
35 (1-35) U/l, ALP 75 (20-120) U/l, GGT 30 (4- 
35) U/l, Bilirubin 18 (1-22) pmol/l, Albumin 38 
(37-49) g/l, calcium 2.1 (2.25-2.7) mmol/l, 
phosphate 7.5 (0.8-8) pmol/l. C3 - 0.5 g/L (0.8 
- 1.6), C4 - 0.17 g/L (0.15 - 0.55). C3 nephritic 
factor is positive. 

Ultrasound scan of his kidneys reveal a 8.5 cm 
length kidney on the left and 9.3 cm on the 
right. 

What is the diagnosis? 

1- Post streptococcal glomerulonephritis 

2- Acute tubular necrosis 

3- Membrano-proliferative glomerulonephritis 

4- mesangiocapillary glomerulonephritis 



staphylococcus 


5- Adult polycystic kidney disease 


pneumococcus 

chickenpox 

malaria 

Non infectious causes: 
Membranoproliferative glomerulonephritis 


Answer & Comments 

Answer: 4- mesangiocapillary 

glomerulonephritis 

The patient has chronic renal failure likely 
secondary to mesangiocapillary 

glomerulonephritis. 


Dr. Khalid Yusuf El-Zohry - Sohag Teaching Hospital (01118391123) 

Ref MRCPass OE OE 2012 PasTest 2009 PassMedicine 2009 PasTest Exam ReviseMRCP 



























El-zohry MRCP Questions Bank (Port 1) - 2013 


(For my personal use) 


Membranoproliferative glomerulonephritis 
(MPGN) can present with the nephrotic 
syndrome, nephritic syndrome, or, most 
often, a mixture of the two. 

The two most common variants of MPGN are 
type I MPGN (also called mesangiocapillary 
glomerulonephritis) and type II MPGN (also 
called dense deposit disease). Type I is much 
more common than type II. Patients with 
MPGN often have hypocomplementemia and 
a circulating autoantibody called C3 nephritic 
factor, which binds to the C3 convertase of 
the alternative pathw ay. It causes inactivation 
of C3 in the alternate pathway by cleaving C3 
into two inactive fragments, C3c and C3d, 
instead of the normal C3b. 

Although the clinical scenario may fit post 
streptococcal glomerulonephritis, 

Mesangiocapillary GN is more likely because 
C3 nephritic factor is not usually associated 
with acute poststreptococcal 

glomerulonephritis. 


[ Q: 428 ] MRCPass - Nephrology 

An 18 year old male has had a 5 year 
history of renal failure. 

On examination, he had hilar adenopathy, but 
maculopapular rashes, erythema nodosum, 
arthritis, chronic lymphocytopenia, 
hepatomegaly, splenomegaly. 

A renal biopsy specimen revealed 
tubulointerstitial nephritis and non caseating 
granulomatous lesions. 

What is the diagnosis? 

1- Wegener's granulomatosis 



Sarcoidosis is a chronic, multi-system 
granulomatous disorder of unknown etiology 
characterized by non-caseating granulomas. 
The most commonly affected organ is the 
lung, other organs include the lymph nodes, 
skin, eyes, parotid glands, bones, joints, liver 
and kidney. Patients mostly present with 
respiratory symptoms, or joint and skin 
manifestations. 

Renal involvement and presentation with 
renal failure are relatively rare in sarcoidosis. 
Renal sacroidosis is reported to have three 
categories: 1) renal changes by abnormal 
calcium metabolism, 2) interstitial nephritis or 
granulomatous nephritis or granulomatous 
nephritis and 3) glomerulonephritis (mostly 
membranous). 



[ Q: 429 ] MRCPass - Nephrology 

A 35 year man has been referred for 
management of hypertension. On 
examination he had palpable kidneys. This 
was confirmed by abdominal ultrasound 
which showed multiple cysts. 


Which of the following is likely to be 
associated? 


1- Polycythaemia 

2- Nail pitting 

3- Hypermobility 

4- Mitral stenosis 


5- Hypogonadism 


Answer & Comments 


Answer: 1- Polycythaemia 


2- Lupus nephritis 

3- Renal tuberculosis 

4- Sarcoidosis 

5- Renal calculi 


Answer & Comments 


Answer: 4- Sarcoidosis 


Polycystic kidney disease is associated with 
increased levels of EPO which lead to 
polycythaemia. Aneurysms of cerebral arteries 
(berry aneurysms) have been reported in 10- 
50% of patients. A variety of cardiac and aortic 
abnormalities have been associated with 
APKD, including aortic root dilatation, aortic 
regurgitation, bicuspid aortic valves, 


Dr. Khalid Yusuf El-Zohry - Sohag Teaching Hospital (01118391123) 

Ref MRCPass OE OE 2012 PasTest 2009 PassMedicine 2009 PasTest Exam ReviseMRCP 



























El-zohry MRCP Questions Bank (Part 1) - 2013 


(For my personal use) 


coarctation of the aorta, mitral regurgitation 
and abdominal aortic aneurysm. 


^ [ Q: 430 ] MRCPass - Nephrology 

n - 

# A 30 year old lady has haematuria on 
the dipstick and was referred by the GP for 
further investigation. She eventually has a 
renal biopsy which shows mesangial 
hypercellularity with IgA deposition. 

What is the diagnosis? 

1- Mesangiocapillary glomerulonephritis 

2- IgA nephropathy 

3- HIV nephropathy 

4- Nephrotic syndrome 

5- Interstitial nephritis 


Answer & Comments 

Answer: 2- IgA nephropathy 

The diagnosis is IgA nephropathy. IgA 
nephropathy is the commonest 
glomerulonephritis but only 30% progress 
towards end stage renal failure. It rarely 
causes nephrotic syndrome, and more 
commonly presents with microscopic 
haematuria. HIV is associated with a focal 
segmental and immune complex 
glomerulonephritis. 


[ Q: 431 ] MRCPass - Nephrology 

A 27 year old lady who is 20 weeks 
pregnant is found to have 2+ proteinuria on 
dipstick. Her blood pressure is 155/80 mmHg. 
Creatinine is 100 pmol/l. A 24 hour urinary 
protein excretion was measured at 1.6 g. She 
is commenced on labetalol and her blood 
pressure is controlled. Ultrasound of the 
kidneys show normal size. 

Following the pregnancy she has an 
intravenous urogram which shows scarring in 
the upper pole of the left kidney. 

What is the likely diagnosis? 



1- Reflux nephropathy 

2- Minimal change glomerulonephritis 

3- IgA nephropathy 

4- Hypertensive nephropathy 

5- Membranous nephropathy 


Answer & Comments 

Answer: 1- Reflux nephropathy 

Reflux nephropathy can lead to proteinuria, 
renal impairment and hypertension. 
Pregnancy can precipitate the symptoms. 

The pathology may be due to reflux of 
infected urine, incompetent vesico-ureteric 
valves or abnormal renal papillae which result 
in intra-renal reflux. Grades I and II reflux are 
managed conservatively, with improved 
hygiene, high fluid intake and regular voiding. 
Grades III and IV reflux require surgical 
management - ie correction of the underlying 
abnormality. This is by tunneling the ureter 
through the bladder wall. 



[ Q: 432 ] MRCPass - Nephrology 

A 30 year old man has returned from 
a trip to India recently. He presents with 
bloody diarrhoea which started three weeks 
ago. He now has malaise and sw elling of his 
legs. He has poor urine output. Blood tests 
show: 


Hb 8 g/dL 

White cell count 14 x 10 9 /L 
Neutrophils 9 x 10 9 /L 
Platelets 20 x 10 9 /L 
PT 13s, APTT 33s 
Fibrinogen 4 g/dL 
sodium 136 mmol/L 
potassium 6.1 mmol/L 
urea 30 mmol/L 
creatinine 440 umol/L 


Dr. Khalid Yusuf El-Zohry - Sohag Teaching Hospital (01118391123) 

Ref MRCPass OE OE 2012 PasTest 2009 PassMedicine 2009 PasTest Exam ReviseMRCP 



























El-zohry MRCP Questions Bank (Port 1) - 2013 


(For my personal use) 


albumin 30 g/L. 

Urine dipstick: blood ++, protein +. 

Whot is the next best investigation to confirm 
the likely diagnosis? 

1- Renal biopsy 

2- Stool culture 

3- MRA of kidneys 

4- Ultrasound of kidneys 

5- 24 hour urine collection for creatinine 
clearance 

Answer & Comments 

Answer: 2- Stool culture 

The diagnosis is haemolytic uraemic syndrome 
(HUS). The typical presentation is with a triad 
of acute renal failure, Microangiopathic 
haemolytic anaemia and thrombocytopaenia. 
Most commonly the cause is E coli 0157 
(verotoxin producing E coli), hence stool 
cultures would be the best investigation to 
confirm the diagnosis in this case. Other 
causes are shigella, ciclosporin and connective 
tissue diseases. The thrombocytopenia is 
thought to be a consequence of platelet 
consumption at sites of endothelial injury. 
Despite this blood clotting times (prothrombin 
time, kaolin clotting time) are normal. 


Answer & Comments 

Answer: 5- Reflux nephropathy 

The term chronic pyelonephritis due to reflux 
nephropathy is reserved for a condition 
resulting from long-standing vesicoureteral 
reflux and infection leading to chronic 
interstitial nephritis and parenchymal scarring. 
This condition usually begins in childhood and 
occurs more frequently in females. 

^ [ Q: 434 ] MRCPass - Nephrology 

* A 44 year old renal transplant 
recipient has been on prednisolone and 
azathioprine for several months. He has 
suddenly developed a cough and fevers. On 
investigation, he was found to have a Hb of 9 
g/dl and a WCC of 3 x 10 9 /L. 

What is the most likely cause? 

1- Pneumonia 

2- Prednisolone 

3- Low EPO 

4- Folate deficiency 

5- Azathioprine 

Answer & Comments 

Answer: 5- Azathioprine 


[ Q: 433 ] MRCPass - Nephrology 

A 30-year-old female with chronic 
urinary tract infections presents for 
investigations. Ultrasound scan reveals 
scarring over the renal calyces. 

Which one of the following is most likely? 

1- Analgesic nephropathy 

2- Polycystic kidney disease 

3- Nephrotic syndrome 

4- Post streptoccocal glomerulonephritis 

5- Reflux nephropathy 



Immunosuppressives used in renal transplant 
can often cause a pancytopenia. In this case, 
azathioprine, which is well known to cause 
bone marrow suppression had led to the 
predisposition towards infection. 



[ Q: 435 ] MRCPass - Nephrology 

A 12 year old male was admitted to 
the emergency department with significant 
lethargy which had worsened over the past 
year. 


On examination he was conscious and 
orientated. His heart rate was 70 beats per 
minute, blood pressure was 120/70mmHg and 


Dr. Khalid Yusuf El-Zohry - Sohag Teaching Hospital (01118391123) 

Ref MRCPass OE OE 2012 PasTest 2009 PassMedicine 2009 PasTest Exam ReviseMRCP 

































El-zohry MRCP Questions Bank (Port 1) - 2013 


(For my personal use) 


he was afebrile. Clinical examination of the 
abdomen revealed minimal no organomegaly. 

His haemoglobin is 11.2 g/dL, white cell count 
of 8.2 x 10 9 /L and a platelet count of 365 x 
10 9 /l_. The plasma urea was 11.1 mmol/L, 
creatinine 117 umol/L, sodium 132 mmol/L , 
potassium 2.3 mmol/L, bicarbonate 8 mmol/L. 

The plasma liver function tests were within 
normal limits. His electrocardiogram 
demonstrated U waves. 

Which one of the following is the likely 
diagnosis? 

1- ACE inhibitor therapy 

2- Liddle's syndrome 

3- Pituitary tumour 

4- Renal artery stenosis 

5- Ciclosporin treatment 

Answer & Comments 

Answer: 2- Liddle's syndrome 


3-w eek history of malaise, fever and 
shivering. Urine dipstick showed microscopic 
haematuria. 

Over the next week, his blood urea rose 
steadily from 10 to 23mmol/l (NR 2.5-7.5) and 
the serum creatinine from 164 to 515pmol/l 
(NR 60-120). His haemoglobin was 9g/l,w hite 
cell count of 10.4 x 10 9 /L. 

His urine contained red cell casts and he 
rapidly became oliguric. Antinuclear 
antibodies, including anti-DNA antibodies, 
were not detected and serum C3 and C4 levels 
were normal. 

A renal biopsy specimen contained seven 
glomeruli: four showed focal necrotizing 
glomerulonephritis with epithelial crescents 
but the remaining three were normal. On 
immunofluorescence, linear staining with IgG 
was present along the glomerular capillary 
basement membrane. 

What is the diagnosis? 

1- Lupus nephritis 


Liddle's syndrome is caused by a mutation in 
the sodium channel (ENaC) in the distal 
nephron. 

There is increased sodium reabsorption and 
potassium loss. Hence the condition is 
characterised by hypokalaemic alkalosis, 
hypertension, and negligible aldosterone 
secretion. Treatment is with triamterene. The 
condition is also sometimes called 

pseudohyperaldosteronism. 

Hypokalemia with a metabolic alkalosis may 
be due to vomiting, excess nasogastric fluid 
loss, hyperaldosteronism, Bartter's syndrome, 
Gitelman's variant, Liddle's syndrome 

(pseudoaldosteronism), Cushings syndrome, 

ectopic corticotropin producing tumours, 

VIPoma, mineralocorticoid excess, excess 
liquorice intake and diuretics. 



[ Q: 436 ] MRCPass - Nephrology 
A 45 year old man presented with a 


2- Interstitial nephritis 

3- Wegener's granulomatosis 

4- Anti GBM disease 

5- IgA nephropathy 


Answer & Comments 

Answer: 4- Anti GBM disease 

Antiglomerular basement membrane (anti- 
GBM) antibody disease is a rare autoimmune 
disorder in which circulating antibodies are 
directed against an antigen normally present 
in the GBM and alveolar basement 
membrane. 

Presentation is with haemoptysis, haematuria, 
fever, malaise, weight loss, and fatigue. 

At immunofluorescent microscopy of renal 
biopsy specimens, the linear deposition of IgG 
along the glomerular capillaries and, 
occasionally, the tubules is nearly 
pathognomonic. 


Dr. Khalid Yusuf El-Zohry - Sohag Teaching Hospital (01118391123) 

Ref MRCPass OE OE 2012 PasTest 2009 PassMedicine 2009 PasTest Exam ReviseMRCP 
























El-zohry MRCP Questions Bank (Port 1) - 2013 


(For my personal use) 



Immunofluorescence staining for IgG reveals 
diffuse high-intensity linear staining of the 
glomerular basement membrane in a patient 
with anti-GBM disease. 

[ Q: 437 ] MRCPass - Nephrology 

A 20 year old man had type 1 
diabetes or 5 years. He now presents with leg 
oedema. On Investigations, he was found to 
have a urinary protein of 5 g/24 hours and 
serum cholesterol 8 mmol/l. 

Whot is the most likely diagnosis. 

1- Diabetic nephropathy 

2- IgA nephropathy 

3- Post streptococcal glomerulonephritis 

4- SLE 

5- Minimal change disease 



Answer & Comments 


Answer: 


5- Dialysis dependent renal failure never 
occurs 


Answer & Comments 

Answer: 4- Renal vein thrombosis 

Selective proteinuria is more of a feature in 
minimal change glomerulonephritis. 

Membranous glomerulonephritis accounts for 
1/3 of patients with nephritic syndrome. The 
renal biopsy shows IgG and C3 subepithelial 
deposition on the basement membrane. 
Streptococcal infection suggests nephritic 
syndrome, which would present with 
haematuria predominantly. End stage renal 
failure occurs in 1/3 of patients and if so, 
immunosuppresants such as 

cyclophosphamide, steroids and chlorambucil 
are recommended. Renal vein thrombosis 
occurs in about 10% of patients with nephrotic 
syndrome and membranous 

glomerulonephritis. 


[ Q: 439 ] MRCPass - Nephrology 

A 20 year old man presented with an 
18-month history of intermittent, painless 
haematuria, usually occurring after strenuous 
exercise, but without dysuria or increased 
frequency of micturition. He also had frequent 
colds and sore throats and believed that the 
haematuria increased at these times. 



[ Q: 438 ] MRCPass - Nephrology 

A 40 year old man has proteinuria 
with a dipstick demonstrating +++ of protein. 
He has a history of diabetes, and a albumin of 
26 g/l. A physician suspects membranous 
glomerulonephritis. 

Which of the following is likely to support this 
diagnosis? 

1- Selective proteinuria 

2- IgM deposits around the renal tubules 

3- Recent streptococcal throat infection 

4- Renal vein thrombosis 


On examination, he appeared fit and healthy; 
his blood pressure was 150/75. Urine analysis 
showed microscopic haematuria (3+) and a 
trace of protein. Intravenous urography, a 
micturating cystogram and cystoscopy were 
normal. His haemoglobin, white-cell count, 
blood urea and creatinine clearance were 
normal; the urinary protein excretion was 
0.95g/day. Immunoglobulin, CH50, C4 and C3 
levels were within normal limits. 

In view of the duration of haematuria, a renal 
biopsy was performed. The glomeruli showed 
a diffuse increase in mesangial cells with 
thickening of the matrix. Immunofluorescent 


Dr. Khalid Yusuf El-Zohry - Sohag Teaching Hospital (01118391123) 

Ref MRCPass OE OE 2012 PasTest 2009 PassMedicine 2009 PasTest Exam 




ReviseMRCP 

207 


























El-zohry MRCP Questions Bank (Port 1) - 2013 


(For my personal use) 


examination of the biopsy showed mesangial 
deposits of IgA and C3. 

What treatment should be commenced? 

1- Frusemide 

2- Ramipril 

3- Ciclosporin 

4- Tacrolimus 

5- Amoxycillin 

Answer & Comments 

Answer: 2- Ramipril 

IgA nephropathy (Berger's disease) is the most 
common glomerulonephritis worldw ide. 
There are mesangial IgA deposits in the 
kidney. Presentation may be with a rash, frank 
haematuria after an episode of pharyngitis, 
arthritis or symptoms of acute renal failure. 

Although it is a benign disease in most 
patients, chronic renal failure and ESRD occur 
in about 20-40% of patients. 


Investigations reveal: urea 7 mol/I, creatinine 
90 umol/L, anti dsDNA antibodies - strongly 
positive, 24 hour urinary protein excretion 2.5 

g. 

A renal biopsy shows membranous 
nephropathy. 

What is the most suitable treatment? 

1- Azathioprine 

2- Lisinopril 

3- Calcium phosphate 

4- Ibuprofen 

5- Insulin 

Answer & Comments 

Answer: 2- Lisinopril 

In lupus nephritis with membranous 
nephropathy, ACE inhibitors and steroids 
(prednisolone) have been show n to slow the 
progression tow ards worsening nephrotic 
syndrome. 


Currently, no cure exists for IgA nephropathy, 
but therapies that can delay the onset of need 
for dialysis and transplantation are available. 
ACE inhibitors are the preferred agents for low 
ering blood pressure. 



Light microscopy of a glomerulus from a 
patient with immunoglobulin A nephropathy 
showing increased mesangial matrix and 

cellularity 



[ Q: 441 ] MRCPass - Nephrology 

A 15 year old female presents with a 
short history of sudden onset of severely 
swollen ankles. She is otherwise well, and 
urinalysis shows +++ protein. A urine 
collection shows that she is excreting 5g 
protein per 24 hours, the plasma albumin is 23 
g/l, creatinine 90. 


What is the most likely underlying diagnosis? 


1- Minimal change nephrotic syndrome 

2- Focal Segmental glomerulosclerosis 


3- IgA nephropathy 

4- Membranous nephropathy 


5- Mesangiocapillary glomerulonephritis 


[ Q: 440 ] MRCPass - Nephrology 


A 25 year lady has a butterfly rash 
and arthralgia. Her blood pressure is 170/95. 



Answer & Comments 

Answer: 1- Minimal change nephrotic 
syndrome 


Dr. Khalid Yusuf El-Zohry - Sohag Teaching Hospital (01118391123) 

Ref MRCPass OE OE 2012 PasTest 2009 PassMedicine 2009 PasTest Exam ReviseMRCP 



208 





























El-zohry MRCP Questions Bank (Port 1) - 2013 


(For my personal use) 


The patient has nephrotic syndrome and at 
this age the commonest underlying diagnosis 
on renal biopsy is minimal change nephrotic 
syndrome. This would also be consistent with 
the lack of haematuria, the sudden onset and 
the normal excretory renal function. 

Minimal change nephropathy is responsible 
for 90% of the cases of nephrotic syndrome in 
children less than 5 years of age. It also occurs 
in adults - approx 20%. The name is derived 
from the fact that the only detectable 
abnormality histologically is fusion and 
deformity of the foot processes under the 
electron microscope. Minimal Change Disease 
is steroid-responsive and in general, does not 
lead to chronic renal failure. 



[ Q: 442 ] MRCPass - Nephrology 

A 57 year old man with diabetic 
nephropathy has a plasma creatinine of 
380pmol/l. 


He has the following blood results : potassium 
5.2 mmol/l, calcium 2.20 mmol/I, albumin 42 
g/l, phosphate 1.55 mmol/l, and PTH 1.6 
pmol/l (NR 1.1-6.8). 


Which of the following should be commenced? 

1- Alucaps 

2- Thyroxine 


3- Vitamin A 


4- Alfacalcidol 


5- Calcium acetate 


Answer & Comments 

Answer: 5- Calcium acetate 

Alfacalcidol could be considered for 
prophylaxis against renal bone disease and 
progressive hyperparathyroidism. How ever, 
the patient's phosphate level is already 
elevated, and vitamin D supplementation may 
increase this further. 


Aluminium-containing phosphate binders 
(alucaps) carry the risk of aluminium 
accumulation and CNS effects. 

Calcium acetate or calcium carbonate can be 
used. It should be taken with (or just before) 
meals and may offer advantages over calcium 
carbonate. 

[ Q: 443 ] MRCPass - Nephrology 

A 35 year old man has been given 
some painkillers by his GP for pleuritic chest 
pains following a chest infection. 

2 weeks later he presents again with ankle 
oedema. 

His blood pressure is 130/70 mmHg, and urine 
dipstick reveals protein +++ and no blood. 
Plasma creatinine is 145 pmol/l. 

What is the likely diagnosis? 

1- Renal amyloidosis 

2- Membranous nephropathy 

3- Interstitial nephritis 

4- Focal segmental glomerulosclerosis 

5- Minimal change nephropathy 



Answer & Comments 

Answer: 3- Interstitial nephritis 

The most frequent causes of acute interstitial 
nephritis can be found in one of three general 
categories: 

drug-induced (NSAID, anti TB drugs, 
sulphonamides), infection-associated 

(legionella, HIV, Hep C) cases associated with 
immune or neoplastic disorders (SLE, Sjogrens, 
Wegeners). 

The clinical presentation can range from 
asymptomatic elevation in creatinine or blood 
urea or abnormal urinary sediment, to 
generalized hypersensitivity syndrome with 
fever, rash, eosinophilia, and oliguric renal 
failure. 


Dr. Khalid Yusuf El-Zohry - Sohag Teaching Hospital (01118391123) 

Ref MRCPass OE OE 2012 PasTest 2009 PassMedicine 2009 PasTest Exam ReviseMRCP 



209 

























El-zohry MRCP Questions Bank (Port 1) - 2013 


(For my personal use) 


Proteinuria is much more common in NSAID- 
induced nephritis than other causes. 
Resolution of proteinuria should occur with 
discontinuation of NSAID drugs, although 
steroids can be used in cases of heavy 
proteinuria. 


4- IgA nephropathy 

5- Analgesic nephropathy 


Answer & Comments 


Answer: 5- Analgesic nephropathy 


[ Q: 444 ] MRCPass - Nephrology 

A 45 year old patient with chronic 
hepatitis C has now developed hypotension 
and oliguria. His liver and renal function tests 
suggest hepatorenal syndrome. 

Which of the following measures is likely to 
improve his prognosis? 

1- Renal transplantation 

2- Liver transplantation 

3- TIPSS 

4- Loop diuretics 

5- Haemodialysis 



Analgesic nephropathy is usually a result of 
prolonged or chronic ingestion of analgesics, 
especially over-the-counter (OTC) medications 
that contain phenacetin or acetaminophen 
and nonsteroidal antiinflammatory drugs 
(NSAIDs) including aspirin or ibuprofen. The 
ingestion may have been excessive over a 
period of years. This frequently occurs as a 
result of self-medication, often for some type 
of chronic pain. 

There is interstitial nephritis and renal 
papillary necrosis, eventually leading to acute 
renal failure or chronic renal failure. There 
may be haematuria but minimal or no 
proteinuria. 


Answer & Comments 

Answer: 2- Liver transplantation 

Hepatorenal syndrome is due to reduced renal 
cortical perfusion, caused by accumulation of 
toxins which would normally be cleared by the 
liver. Liver transplantation is the only option 
which would provide a resolution of this 
problem. 


[ Q: 445 ] MRCPass - Nephrology 

A 70 year old woman is referred for 
investigation of a creatinine of 250 
micromol/l. She mentioned several episodes 
of haematuria. She has a long history of low 
back pain treated with a combination of 
painkillers. An ultrasound shows two irregular 
shaped kidneys sized 8.5 and 9 cm. 

Which of the following is the most likely? 

1- Minimal change nephropathy 

2- Diabetic nephropathy 

3- Membranous glomerulonephritis 



[ Q: 446 ] MRCPass - Nephrology 

A 65 year old man is being 
considered for dialysis. 

Which of the following suggests that the 
patient should be considered for CAPD rather 
than haemodialysis? 

1- Hypertension 

2- Severe congestive cardiac failure 

3- Recent inguinal surgery 

4- COPD 

5- Glomerulonephritis 



Answer & Comments 

Answer: 2- Severe congestive cardiac failure 

Severe cardiac failure often leads to 
circulatory compromise (hypotension) during 
haemodialysis, hence such a patient would be 
better dialysed by CAPD. 


Dr. Khalid Yusuf El-Zohry - Sohag Teaching Hospital (01118391123) 

Ref MRCPass OE OE 2012 PasTest 2009 PassMedicine 2009 PasTest Exam ReviseMRCP 





























El-zohry MRCP Questions Bank (Port 1) - 2013 


(For my personal use) 


[ Q: 447 ] MRCPass - Nephrology 

A 30 year old intravenous drug user 
who is HIV positive, presents with 
oedematous ankles and breathlessness. 

There is no history of diabetes. He has a urea 
of 13 pmol/l, creatinine of 140 pmol/l. AST is 
25 U/l, ALP is 120 U/l, bilirubin 18 pmol/l and 
albumin is 25 g/l. A 24 hour urine protein is 
5.5g in total. 

Which of the following is the likely diagnosis? 

1- Focal segmental glomerulosclerosis 

2- Minimal change nephropathy 

3- Membranoproliferative glomerulonephritis 

4- Nephritic syndrome 

5- Post streptococcal glomerulonephritis 



Answer & Comments 


Answer: 1- Focal segmental glomerulosclerosis 


Focal segmental glomerulosclerosis is 
commonly associated with HIV positive 
patients and in view of the history this should 
be excluded. The other causes of nephrotic 
syndrome are minimal change, 
membranoproliferative, and membranous 
glomerulonephritis. 



Focal segmental glomerulosclerosis (FSGS). An 
area of collagenous sclerosis runs across the 
middle of this Glomerulus 


[ Q: 448 ] MRCPass - Nephrology 

A 15 year boy presents with Henoch 
Schonlein purpura related renal impairment 
with a creatinine of 200 pmol/l. 



What is the likely outcome in the long term? 

1- Intermittent haematuria 

2- Normal renal function 

3- Recurrent urinary tract infections 

4- Renal scarring 

5- Progression towards renal cell carcinoma 


Answer & Comments 

Answer: 2- Normal renal function 

Henoch Schonlein Purpura causes a nephritis 
which is related to IgA deposition and 
vasculitis. It is self limiting and the long term 
prognosis is good, with likely recovery of renal 
function. 


[ Q: 449 ] MRCPass - Nephrology 

A 60 year old man presents with 
severe pain in his left loin radiating to the left 
groin. Bedside testing of urine demonstrates 
haematuria. Abdominal ultrasound does not 
demonstrate any abnormality and X-ray KUB 
does not show any visible stones. 

Which form of calculus might the patient 
have? 

1- Calcium oxalate stone 

2- Cystine stone 

3- Uric acid stone 

4- Triple phosphate stone 

5- Calcium phosphate stone 



Answer & Comments 

Answer: 3- Uric acid stone 

Radiopaque stones are: Calcium oxalate, 
calcium phosphate, triple phosphate, cystine 
stones. 

Radiolucent stones are: Uric acid, xanthine 
stones. 


Dr. Khalid Yusuf El-Zohry - Sohag Teaching Hospital (01118391123) 

Ref MRCPass OE OE 2012 PasTest 2009 PassMedicine 2009 PasTest Exam ReviseMRCP 


























El-zohry MRCP Questions Bank (Port 1) - 2013 


(For my personal use) 


[ Q: 450 ] MRCPass - Nephrology 

A 45 year old man presents with 
recurrent sinusitis for over a year and 
occasionally noticed a rash on his chest. 



Routine blood tests show that he has a serum 
urea of 14 mmol/l and creatinine of 180 
micromoles/I. 


Which of the following is the most likely? 

1- Minimal change nephropathy 

2- Systemic vasculitis 

3- Membranous glomerulonephritis 

4- Goodpasture's syndrome 

5- Focal segmental glomerulosclerosis 


Answer & Comments 

Answer: 2- Systemic vasculitis 

This patient is likely to have vasculitis related 
to Wegener's granulomatosis. 

Sinusitis can arise from involvement of the 
nasal tract and sinuses in the Wegener's 
pattern of the disease. 

Often a patient with systemic vasculitis will 
have a long history of indolent disease, but 
will then present late with severe aggressive 
disease. Serum anti-neutrophil cytoplasmic 
antibodies (ANCA) and antibodies against the 
ANCA antigens myeloperoxidase and 
proteinase 3 may be positive. 


[ Q: 451 ] MRCPass - Nephrology 

A 57 year old man has presented 
with chest pain and pulmonary oedema. He is 
managed as acute coronary syndrome. He did 
not tolerate a GTN infusion as his blood 
pressure was 85/60 but improved with 
frusemide and his blood pressure stabilized. 
An ACE-inhibitor was held off due to renal 
impairment. He is coincidentally found to have 
a chest infection, his blood cultures grew 
streptococci and he was treated with 
augmentin. His blood tests on admission show 



a creatinine of 145 pmol/l rising up to 
190pmol/ and then 250pmol/l the day after. 

What is the likely cause of renal failure? 

1- Urinary tract infection 

2- Acute tubular necrosis 

3- Interstitial nephritis due to augmentin 

4- Post streptococcal glomerulonephritis 

5- Renal artery stenosis 


Answer & Comments 

Answer: 2- Acute tubular necrosis 

A rapid rise in creatinine following periods of 
hypotension is most commonly due to acute 
tubular necrosis. Acute tubular necrosis or 
(ATN) involves the death of tubular cells that 
form the tubule that transports urine to the 
ureters while reabsorbing 99% of the water. 
Tubular cells continually replace themselves 
and if the cause of ATN is removed then 
recovery is likely. 


^ [ Q: 452 ] MRCPass - Nephrology 

fi - 

# A 65 year old man has hypertension. 
On examination of his abdomen a bruit is 
audible in the epigastrium. 

Urinalysis shows : Proteins ++ Abdominal 
ultrasound reveals a difference in size 
between the two kidneys of 2.0 cms in length. 

What is the likely diagnosis? 

1- Glomerulonephritis 

2- Nephritic syndrome 

3- Renal artery stenosis 

4- Phaeochromocytoma 

5- Fibrous dysplasia 


Answer & Comments 


Answer: 3- Renal artery stenosis 


Dr. Khalid Yusuf El-Zohry - Sohag Teaching Hospital (01118391123) 

Ref MRCPass OE OE 2012 PasTest 2009 PassMedicine 2009 PasTest Exam ReviseMRCP 





























El-zohry MRCP Questions Bank (Port 1) - 2013 


(For my personal use) 


Hypertension, renal bruit and a difference in 
renal sizes are suggestive of renal artery 
stenosis. 

[ Q: 453 ] MRCPass - Nephrology 

A 45 year old man has a history of 
hypertension. He now presents with frank 
haematuria which has been present for 6 
months. His blood results show a urea of 16 
mmol/I and creatinine 180 micromoles/l. 

Which of the following is the most likely 
diagnosis? 

1- Focal segmental glomerulosclerosis 

2- IgA nephropathy 

3- Acute tubular necrosis 

4- Nephritic syndrome 

5- Wegener's granulomatosis 



Answer & Comments 

Answer: 2- IgA nephropathy 

IgA nephropathy is the commonest 
glomerulonephritis worldwide. IgA 
nephropathy or Berger's disease is a 
clinical/pathological entity defined by the 
presence of macroscopic or microscopic 
hematuria and mesangial IgA deposits. Signs 
and symptoms include proteinuria, nephrotic 
syndrome, acute nephritis, malignant 
hypertension, chronic renal failure, and acute 
renal failure. 

Proteinuria is typically within the mild range. 
There is a 2:1 male predominance. Findings on 
renal biopsy are characteristic. In all patients, 
IgA is present and is deposited mainly in the 
mesangium of the glomerular tuft. Other 
immunoglobulins and complement, especially 
C3, are often found in a mesangial pattern. 


[ Q: 454 ] MRCPass - Nephrology 

A 16 year old boy has had 
progressive high tone deafness since 
childhood. Investigations reveal: 



urea 27 pmol/l 

creatinine 310 pmol/l 

Urine dipstick: blood + and protein ++ 

Which one of the following is most likely? 

1- Wegener's granulomatosis 

2- Tuberous sclrerosis 

3- Alports syndrome 

4- Sarcoidosis 

5- Von Hippel Lindau disease 


Answer & Comments 

Answer: 3- Alports syndrome 

Classic Alport's syndrome is an X linked 
disorder characterised by a triad of 
progressive hematuric nephritis, progressive 
perceptive high-tone hearing loss, and ocular 
signs (lenticonus and cataracts). There is 
associated nephrotic syndrome in 30% of 
patients. Average progression to end stage 
renal failure is 30 years. 



[ Q: 455 ] MRCPass - Nephrology 

A 70 year old male with a three 
month history of constitutional symptoms 
including weight loss, night sweats, fatigue 
and malaise presents with rapidly progressive 
acute renal failure, hemoptysis and diffuse 
bilateral pulmonary infiltrates. 


Biopsy demonstrates a pauci-immune focal 
segmental proliferative, necrotizing and 
sclerosing glomerulonephritis with crescent 
formation and a superimposed necrotizing 
and granulomatous arteritis involving small 
intrarenal arteries. 


What is the likely diagnosis? 

1- Polyarteritis nodosa 

2- Anti GBM disease 


3- Post streptococcal glomerulonephritis 

4- Interstitial nephritis 

5- Wegener's granulomatosis 


Dr. Khalid Yusuf El-Zohry - Sohag Teaching Hospital (01118391123) 

Ref MRCPass OE OE 2012 PasTest 2009 PassMedicine 2009 PasTest Exam 




ReviseMRCP 

213 


























El-zohry MRCP Questions Bank (Port 1) - 2013 


(For my personal use) 


Answer & Comments 


Answer: 5- Wegener's granulomatosis 


secondary to increased erythropoietin 
production. 


The clinicopathologic differential diagnosis 
includes predominantly the microscopic form 
of polyarteritis nodosa (polyarteritis nodosa 
with glomerular involvement) and Wegener's 
granulomatosis. The presence of extrarenal 
(pulmonary) involvement and granulomatous 
vasculitis favors Wegener's granulomatosus. 


[ Q: 457 ] MRCPass - Nephrology 

A 52 year old man with chronic renal 
failure has been on EPO injections 3 times a 
week for several years. 

Which one of the following is o side effect of 
erythropoietin therapy? 



Crescentic glomerulonephritis or rapidly 
progressive glomerulonephritis (RPGN) is a 
term given to a diverse group of diseases 
which all have cresents present within the 
glomerular tuft. These include primary or 
renal limited (so-called idiopathic) crescentic 
glomerulonephritis, anti-glomerular basment 
membrane(anti-GBM) antibody diseases, and 
systemic disorders. Light microscopic 
examination typically shows at least 30% of 
the glomeruli involved by crescents. 

[ Q: 456 ] MRCPass - Nephrology 

A 35 year old man presents with 
hypertension. On examination he had 
palpable kidneys and abdominal ultrasound 
shows bilaterally enlarged cystic kidneys. 

Which of the following features is likely to be 
associated? 

1- Testicular atrophy 

2- Nail dystrophy 

3- Mitral stenosis 

4- Polycythaemia 

5- Short stature 



Answer & Comments 

Answer: 4- Polycythaemia 

In adult polycystic kidney disease, the most 
common presentation is a palpable mass, 
hypertension, abdominal pain, and hematuria. 
Hypertension often predates renal failure. 
Polycythemia is a rare but known association 


1- Hypokalaemia 

2- Anaemia 

3- Seizures 

4- Iron overload 

5- Osteoporosis 


Answer & Comments 

Answer: 3- Seizures 

EPO therapy may lead to hypertension and 
seizures. Other side effects of treatment with 
erythropoietin include hyperkalaemia in 
uraemic patients, increased PCV, 
thrombocythaemia, shunt thrombosis, iron 
deficiency and urticaria. 


[ Q: 458 ] MRCPass - Nephrology 

A 55 year old man presents with 
chest pain and is thrombolysed for an acute 
myocardial infarction. His blood pressure was 
85/40 for several hours on admission. Two 
days later his blood tests show a urea of 22 
mmol/I and creatinine of 300 pmol/l (U+Es 
were normal on admission). 

What would a renal biopsy show? 

1- Positive antibody to fibrinogen 

2- Thickened capillary loops 

3- Loss of tubular cells 

4- IgG deposition in the basement membrane 

5- IgA mesangial deposits 



Dr. Khalid Yusuf El-Zohry - Sohag Teaching Hospital (01118391123) 

Ref MRCPass OE OE 2012 PasTest 2009 PassMedicine 2009 PasTest Exam ReviseMRCP 



214 




























El-zohry MRCP Questions Bank (Port 1) - 2013 


(For my personal use) 


Answer & Comments 

Answer: 3- Loss of tubular cells 

Due to hypoperfusion, acute tubular necrosis 
may occur. This would show necrosis or loss of 
tubular cells. 



The epithelium of the tubules seen here is 
ragged from undergoing necrosis with acute 
tubular necrosis (ATN) from ischemia. 


[ Q: 459 ] MRCPass - Nephrology 

A 65 year old man has renal bruits, 
renal impairment and hypertension. 

Which one of the following tests is most 
appropriate? 

1- Intravenous urography 

2- Renal DTPA nuclear scan 

3- Renal DMSA nuclear scan 

4- Renal ultrasound scan 

5- Magnetic resonance angiography (MRA) of 
renal arteries 



Answer & Comments 

Answer: 5- Magnetic resonance angiography 
(MRA) of renal arteries 

For renal artery stenosis, MRA remains the 
best investigation with high sensitivities and 
specificities. 

[ Q: 460 ] MRCPass - Nephrology 

A 50 year woman who is on 
peritoneal dialysis has a plasma potassium of 
6.5 mmol/I. 

Which of the following should she avoid in the 
diet? 



1- Cereal 

2- Yoghurt 

3- Broccoli 

4- Carrots 

5- Tomato 


Answer & Comments 

Answer: 5- Tomato 

All meats, poultry and fish are high in 
potassium. Others are : Apricots (fresh more 
so than canned), Avocado, Banana, 
Cantaloupe, Honeydew , Kiwi, Lima beans, 
Milk, Oranges and orange juice, Potatoes, 
Prunes, Spinach, Tomatoes, Vegetable juice. 
Winter squash 



[ Q: 461 ] MRCPass - Nephrology 

A 27 year old man is admitted with 
pains radiating from his right loin to the groin. 
He also has haematuria. He gives a history of 
previous urinary tract infections. Clinical 
examination is unremarkable. Investigations 
are as follows: 


Serum Sodium 139 mmol/L 
Potassium 2.8 mmol/L 
Urea 6.6 mmol/L 
Creatinine 105 micromoles/L 
Chloride 116 mmol/l 
Bicarbonate 15 mol/L 

Which of the following would help treat the 
condition? 


1- Probenecid 

2- Ciprofloxacin 

3- Erythropoietin 

4- Cyclophosphamide 

5- Sodium bicarbonate 


Answer & Comments 


Answer: 5- Sodium bicarbonate 


Dr. Khalid Yusuf El-Zohry - Sohag Teaching Hospital (01118391123) 

Ref MRCPass OE OE 2012 PasTest 2009 PassMedicine 2009 PasTest Exam ReviseMRCP 
































El-zohry MRCP Questions Bank (Port 1) - 2013 


(For my personal use) 


The patient presented with a history of renal 
colic and previous urinary tract infections and 
investigations reveal a normal anion gap 
acidosis. These features suggest a diagnosis of 
distal renal tubular acidosis. Sodium 
bicarbonate is effective in decreasing the 
acidosis. 


[ Q: 462 ] MRCPass - Nephrology 

A 25 year old has a family history of 
adult polycystic kidney disease. 

Whot is the best test to exclude polycystic 
kidney disease in this woman? 

1- Genetic linkage analysis 

2- Renal ultrasound 

3- Intravenous urogram 

4- Urinalysis 

5- Isotope renography 



Answer & Comments 

Answer: 2- Renal ultrasound 

Ultrasonography with the criteria of more 
than 2 cysts in < 30 years age group is 
diagnostic of adult polycystic kidney disease. 
Genetic linkage analysis can be done as well 
(but less practical) to screen for PKD1 
mutation on chromosome 16 and a PKD2 
mutation on chromosome 4. 

[ Q: 463 ] MRCPass - Nephrology 

A 55 year old man who has been on 
haemodialysis for 20 years has progressive 
dementia. 

Which one of the following is most likely to 
have caused this? 

1- Urea disequilibrium 

2- Small vessel ischaemia 

3- Aluminium toxicity 

4- Amyloidosis 

5- Alzheimer's disease 



Answer & Comments 

Answer: 3- Aluminium toxicity 

Aluminium has been implicated in 'dialysis 
dementia'. Dialysis dementia may be part of a 
multisystem disorder which includes vitamin D 
resistant osteomalacia, proximal myopathy, 
and non-iron deficient, microcytic, 
hypochromic anaemia. The use of aluminium 
free dialysate may arrest, or even improve, 
the established case. Desferrioxamine 
infusions (aluminium chelator) are the 
mainstay of treatment of dialysis dementia, 
improving up to 70% of patients, sometimes 
to normal. 



[ Q: 464 ] MRCPass - Nephrology 

A 50 year old woman has 
investigations for recurrent renal calculi. 

A 24 hour urine collection revealed: 

calcium 18 mmol/24 hours (2-7) 

oxalate 180 mmol/24 hours (90-450) 

uric acid 2.8 mmol/24 hours (1.5-4.4) 

citrate 2 mmol/24hours (0.3-0.4) 

Which form of treatment is likely to be 
successful? 

1- Penicillamine 

2- Thiazide diuretics 


3- Insulin 

4- Aluminium phosphate 

5- Probenecid 


Answer & Comments 

Answer: 2- Thiazide diuretics 

Hypercalciuria is the most common 
identifiable cause of calcium kidney stone 
disease. The other significant causes are 
hyperoxaluria, hyperuricosuria, low urinary 
volume, and hypocitraturia. 

Hypercalciuria is defined as urinary excretion 
of more than 250 mg (6.2 mmol/24 h) or more 


Dr. Khalid Yusuf El-Zohry - Sohag Teaching Hospital (01118391123) 

Ref MRCPass OE OE 2012 PasTest 2009 PassMedicine 2009 PasTest Exam ReviseMRCP 





























El-zohry MRCP Questions Bank (Port 1) - 2013 


(For my personal use) 


than 275-300 mg (7.5 mmol/24 h) of calcium 
per day for men while on a regular, 
unrestricted diet. 

Thiazides currently are the mainstay of 
medical therapy for hypercalciuria. 
Orthophosphate therapy can also be used as a 
preventive treatment for kidney stones. 


3- Reduced d dimer levels 

4- Reduced antithrombin III activity 

5- Reduced fibrinogen concentration 

Answer & Comments 

Answer: 4- Reduced antithrombin III activity 


[ Q: 465 ] MRCPass - Nephrology 

A 40 year old man presents with 
microscopic haematuria and hypertension. 
Ultrasound scan of his kidneys shows several 
cysts. He has a family history of a brain 
tumour (father) and an aunt had renal failure 
and deafness. 

Which of the following conditions is likely? 

1- Neurofibromatosis 

2- Von Hippel Lindau 

3- Alport's syndrome 

4- Renal tubular acidosis type II 

5- Scleroderma 



Answer & Comments 

Answer: 2- Von Hippel Lindau 

von Hippel Lindau disease is transmitted as an 
autosomal dominant condition. 

Affected individuals may have renal cysts, 
Clear cell renal cell carcinoma (CCRCC), retinal 
angiomas, central nervous system 
haemangioblastoma, phaeochromocytoma. 

[ Q: 466 ] MRCPass - Nephrology 

A 60 year woman has bilateral sw 
ollen calves and ankles, with the left calf more 
painful and sw ollen. A 24 hr urine protein was 
8g/day. 

Which could explain these findings? 

1- Factor VIII deficiency 

2- Reduced concentration of Von Willebrand's 
factor 



This patient has nephrotic syndrome, and also 
a probable left DVT. The most likely cause is 
nephrotic syndrome related reduced 
antithrombin III activity. Acquired AT-III 
deficiency causes include chronic liver disease, 
and protein loss due to ascites or nephrotic 
syndrome. 

[ Q: 467 ] MRCPass - Nephrology 

An 70 year old man was found on 
the floor of his hallw ay by his neighbour. On 
assessment in casualty, his GCS was 11/15, 
and he had a dense right hemiparesis. 

His blood results are: 

Na 145 mmol/I 

K 6.2 mmol/I 

Bic 22 mmol/I 

Urea 20 umol/l 

Creat 235 umol/l 

Glu 7.5 mmol/I 

Calcium (total) 1.72 mmol/l 

Phosphate 1.6 mmol/l 

Bili 12 umol/l 

Aik phos 210 U/l 

Albumin 35 g/l 

WCC 12 x 10 9 /L 

Hb 12.5 g/dl 

PI 350 x 10 9 /L 

Urine Blood +++, protein negative 

Urine microscopy - No WC, No RBCs. No 
organisms seen 

The most likely cause is: 



Dr. Khalid Yusuf El-Zohry - Sohag Teaching Hospital (01118391123) 

Ref MRCPass OE OE 2012 PasTest 2009 PassMedicine 2009 PasTest Exam 




ReviseMRCP 

217 

























El-zohry MRCP Questions Bank (Port 1) - 2013 


(For my personal use) 


1- Amyloidosis 


Answer & Comments 


2- Porphyria cutanea tarda 

3- Pyelonephritis 

4- Rhabdomyolysis 

5- Renal tubular acidosis type II 


Answer & Comments 

Answer: 4- Rhabdomyolysis 

Raised CK (though not given) and renal failure 
makes rhabdomyolysis likely. Presence of 
urine myoglobin will cause a false positive 
urine dipstick to blood. 


[ Q: 468 ] MRCPass - Nephrology 

A 49 year old man with a history of 
heavy alcohol abuse and no other significant 
past medical problems was admitted for 
treatment of alcoholic hepatitis. He had no 
know n history of liver or kidney disease until 
1 month before admission, when jaundice 
developed. 

Initially, he received treatment and follow -up 
as an outpatient but was then admitted for 
inpatient care when his mental status 
deteriorated. He had jaundice, tense ascites, 
severe leg edema, and mild to moderate 
encephalopathy. 

His blood tests revealed: urea 32 pmol/l, 
creatinine 480 pmol/l, ALT 180 (5-35) U/l, AST 
140 (1-31) U/l, ALP 360 (20-120) U/l, Bilirubin 
45 (1-22) pmol/l, Albumin 30 (37-49) g/l. Urine 
sodium is 8 mmol/l. 

Which of the following should he be treated 
with ? 

1- Normal saline infusion 

2- Terlipressin 



Answer: 2- Terlipressin 

The diagnosis is hepatorenal syndrome. 
Treatment that combines peripheral 
vasoconstricting medications or splanchnic 
vasoconstricting medications (e.g. midodrine), 
with a volume expander (eg, albumin) has 
show n significant short-term efficacy. 
Promising results have been reported in 
studies and case reports with agonists of 
vasopressin VI receptors, such as terlipressin, 
which predominantly act on the splanchnic 
circulation. Such treatment is designed to 
decrease the effects of intravascular 
hypovolemia. 

^ [ Q: 469 ] MRCPass - Nephrology 

# A 60 year man has been on 

haemodialysis for 15 years due to diabetic 
nephropathy. He now has joint pains in the 
shoulders and knees. He has a past medical 
history of bilateral carpal tunnel 
decompression. 

His bloods reveakHb 11 g/dl, ESR 50 mm/hr, 
urea 18 pmol/l, creatinine 250 pmol/l, sodium 
142 mmol/l, potassium 4.4 mmol/l, Urate 0.6 
(< 0.45). 

What is the likely diagnosis? 

1- Gout 

2- Polymyalgia rheumatica 

3- Rheumatoid arthritis 

4- Calcium pyrophosphate deposition 

5- B2 microglobulin amyloidosis 


Answer & Comments 


Answer: 5- B2 microglobulin amyloidosis 


3- ACE inhibitor 

4- Thiazide diuretics 


The history of joint pains in a dialysis patient 
suggests amyloidosis. 


5- Renal transplantation 


Amyloid deposits composed of b2- 
microglobulin is laid down in periarticular 
surfaces of joints and can develop 


Dr. Khalid Yusuf El-Zohry - Sohag Teaching Hospital (01118391123) 

Ref MRCPass OE OE 2012 PasTest 2009 PassMedicine 2009 PasTest Exam ReviseMRCP 



218 


























El-zohry MRCP Questions Bank (Port 1) - 2013 


(For my personal use) 


approximately 10 years after the start of 
dialysis. 



[ Q: 470 ] MRCPass - Nephrology 

A 35 year old lady with SLE presents 
significant proteinuria and 
hypoalbuminaemia. A renal biopsy confirms 
diffuse proliferative glomeronephritis (WHO 
Class IV). 


with 


Which of the following treatment regimes is 
recommended? 


1- Prednisolone alone 


2- Azathioprine alone 

3- Azathioprine & Methotrexate 

4- Prednisolone & Methotrexate 

5- Prednisolone & intravenous 
Cyclophosphamide 


Answer & Comments 

Answer: 5- Prednisolone & intravenous 
Cyclophosphamide 

Diffuse proliferative glomerulonephritis is the 
commonest glomerulonephritis in SLE. There 
is mesangial and endothelial cell proliferation, 
polymorphonuclear cell infiltrate and granular 
subepithelial deposits of C3. It also carries the 
worst prognosis with progression towards 
renal failure. 

The currently recommended therapy is pulse 
intravenous corticosteroids in combination 
with pulse cyclophosphamide continued for at 
least 12 months after remission. Newer 
regimes include combinations of prednisolone 
and mycophenolate. 


[ Q: 471 ] MRCPass - Nephrology 

A 22 year old woman has taken an 
overdose of multiple drugs. 

Haemodialysis is useful in the removal of 
which one of the following drugs? 

1- Digoxin 



2- Amiodarone 

3- Paraquat 

4- Paracetamol 

5- Salicylates 


Answer & Comments 

Answer: 5- Salicylates 

Haemodialysis is useful for salicylates, NSAIDs, 
lithium, and in particular, Antifreeze 
poisioning. It is not useful for tricyclics, 
amiodarone and paraquat, paracetamol and 
digoxin, which is mostly tissue bound (use 
digoxin binding antibodies). 


[ Q: 472 ] MRCPass - Nephrology 

A 50 year diabetic patient is on the 
following medication: 

Metformin lg tds 

Gliclazide 80 mg bd 

Ramipril 5 mg daily 

atenolol 50 mg 

Her blood results show : 

urea 13 pmol/l 

creatinine 190 pmol/l 

sodium 138 mmol/I 

potassium 4.9 mmol/I 

Which of the following medications should be 
withdrawn? 

1- Gliclazide 

2- Metformin 

3- Atenolol 

4- Ramipril 

5- None of the above 



Answer & Comments 

Answer: 2- Metformin 

The half life of metformin is prolonged in renal 
impairment. There is a risk of lactic acidosis. 


Dr. Khalid Yusuf El-Zohry - Sohag Teaching Hospital (01118391123) 

Ref MRCPass OE OE 2012 PasTest 2009 PassMedicine 2009 PasTest Exam ReviseMRCP 



219 































El-zohry MRCP Questions Bank (Port 1) - 2013 


(For my personal use) 


Patients with a creatinine above 150 pmol/l 
should have metformin discontinued. 


Answer & Comments 


Answer: 1- Colostomy 



[ Q: 473 ] MRCPass - Nephrology 

A 60 year old man has chronic renal 
failure due to hypertension. His plasma 
creatinine reading is 300 pmol/L, potassium 
level is 5.2 mmol/L, urea 17.0 mmol/L, 
phosphate 1.7 mmol/L, calcium 2.15 mmol/L, 
haemoglobin 10.0 g/dL. He has symptoms of 
lethargy and breathlessness on exertion. 


Recent or prospective abdominal surgery is a 
contraindication to CAPD. 


[ Q: 475 ] MRCPass - Nephrology 

A 60 year old man has type II 
diabetes and hypertension. He is feeling 
lethargic and is seen in the general medical 
clinic. 



Which of the following treatments would be 
appropriate? 

1- Erythropoietin 

2- ACE inhibitor 

3- Alfacalcidol at 0.25 meg per day 

4- Haemodialysis 

5- Phosphate restriction 

Answer & Comments 

Answer: 1- Erythropoietin 

This patient would benefit from recombinant 
human EPO to correct his anaemia. Other 
causes of anaemia should be excluded first 
with haematinics. 


Investigations reveal: 

Na 140 mmol/I, 

K 5.5 mmol/I, 

Urea 18 mmol/l, 

Cr 220 pmol/1 

Ultrasound of kidneys show : Left kidney 8.2 
cm length, Right kidney 8.9 cm length. 

What is the likely cause of the renal failure? 

1- Acute tubular necrosis 

2- Wegener's granulomatosis 

3- Berger's disease 

4- Cholesterol emboli 

5- Renovascular disease 


[ Q: 474 ] MRCPass - Nephrology 

A 60 year old lady with multiple 
medical problems is being considered for renal 
dialysis. 

Which one of the following is an absolute 
contraindication to choosing continuous 
ambulatory peritoneal dialysis? 

1- Colostomy 

2- Polycystic kidneys 

3- Heart failure 

4- Previous ovarian surgery 

5- Diabetes 



Answer & Comments 

Answer: 5- Renovascular disease 

The vascular risk factors and asymmetrical 
atrophic kidneys suggest renovascular disease. 
A magnetic resonance angiogram or digital 
subtraction angiography should be performed. 



[ Q: 476 ] MRCPass - Nephrology 

A 40 year old man has been unwell 
for several weeks with lethargy. His BP is 
150/95 mmHg. Urinalysis shows no glucose, 
blood, ketones, nitrite but has protein +++. 


A 24 hour urine collection shows protein of 4 
g. His serum creatinine is 280 micromol/L, 


Dr. Khalid Yusuf El-Zohry - Sohag Teaching Hospital (01118391123) 

Ref MRCPass OE OE 2012 PasTest 2009 PassMedicine 2009 PasTest Exam ReviseMRCP 
































El-zohry MRCP Questions Bank (Port 1) - 2013 


(For my personal use) 


urea is 27 mmol/L. His hepatitis B surface 
antigen is positive. 

Which of the following diseases is likely? 

1- Minimal change glomerulonephritis 

2- Post streptococcal glomerulonephritis 

3- Membranous glomerulonephritis 

4- Acute nephritic syndrome 

5- Diabetic nephropathy 

Answer & Comments 

Answer: 3- Membranous glomerulonephritis 


Whot is the likely diagnosis? 

1- Ischaemic nephropathy 

2- Analgesic glomerulonephritis 

3- Analgesic interstitial nephritis 

4- Minimal change nephropathy 

5- IgA nephropathy 

Answer & Comments 

Answer: 1- Ischaemic nephropathy 

The likely diagnosis is renovascular disease 
due to the small kidney size. 


Membranous nephropathy is caused by 
thickening of the capillary wall of the 
glomerular basement membrane (the deepest 
membrane) by immune complexes. 

Causes of membranous glomerulonephritis 
are Hepatitis B, malaria, malignant tumors, 
non-Hodgkin's lymphoma, systemic lupus 
erythematosus, syphilis, gold, mercury, 
penicillamine. 

The goal of treatment is to minimize 
symptoms and slow the progression of the 
disease. Often, corticosteroids or 
immunosuppressive medications may be used 
to reduce symptoms and progression of the 
disorder. Medications to treat symptoms may 
include antihypertensive and diuretic and 
cholesterol low ering agents. 


[ Q: 477 ] MRCPass - Nephrology 

A 65 year old lady has investigations 
for renal impairment (creatinine 220 umol/l). 
She has a previous history of hypertension, 
peripheral vascular disease and osteoarthritis. 
She has been taking diclofenac for 6 years and 
penicillamine for the past 2 years (the drug 
was stopped 1 year ago). 

Investigations show: 

Urine : protein +, blood -ve 

Ultrasound of kidneys : right 7.8 cm, left 8 cm 
in length. 



There is only 1+ Proteinuria. The other clue is 
the history of hypertension. The normal length 
of kidneys is between 10-13 cm. 


[ Q: 478 ] MRCPass - Nephrology 

A 28 year old man was found to have 
acute renal failure. 

Which of the following is an indication for 
urgent dialysis in renal failure? 

1- Breathlessness 

2- Potassium of 6.5 mmol/I 

3- Glomerulonephritis 

4- Haematuria 

5- Pericardial effusion 



Answer & Comments 

Answer: 5- Pericardial effusion 

Indications for dialysis are: 

uraemia pericarditis with effusion 
pulmonary oedema 
significant hyperkalaemia (>7) with 
evidence of ECG changes 
rapidly rising urea and creatinine 


Dr. Khalid Yusuf El-Zohry - Sohag Teaching Hospital (01118391123) 

Ref MRCPass OE OE 2012 PasTest 2009 PassMedicine 2009 PasTest Exam 




ReviseMRCP 

221 


























El-zohry MRCP Questions Bank (Port 1) - 2013 


(For my personal use) 


^ [ Q: 479 ] MRCPass - Nephrology 

/ - 

# A 45 year woman presents with 

ankle oedema. Her blood pressure was 120/70 
mmHg. 

Investigations show: 

Creatinine 95 umol/L, Albumin 22 g/L, 
Urinalysis shows blood - protein ++++, Urinary 
protein excretion 8g /24hr. 


Answer & Comments 

Answer: 5- Calcium gluconate 

The important management step is to provide 
cardioprotection in the form of calcium 
gluconate. Following this the patient should 
have insulin and dextrose, as well as 
consideration for haemodialysis if the 
potassium does not improve. 


What is the best management step? 

1- ACE inhibitor 

2- Frusemide 

3- Insulin 

4- Steroids 

5- High protein diet 


[ Q: 481 ] MRCPass - Nephrology 

A 60 year old man has oliguria, 
peripheral oedema and breathlessness. He 
also gives a history of intermittent 
haemoptysis and dyspnoea over several 
months. There is no other significant past 
medical history. 



Answer & Comments 

Answer: 4- Steroids 

This patient has nephrotic syndrome as 
indicated by proteinuria, hypoalbuminuria, 
oedema (and hyperlipidaemia). High dose 
steroids should be the first line therapy e.g. 
prednisolone. 


On examination he is oedematous, cyanosed, 
tachypnoeic and there are widespread 
inspiratory crackles throughout the lung fields. 
Investigations are as follow s: Urine - Protein 
++, Blood ++, Immunology: Anti glomerular 
basement membrane antibody positive, 

The renal biopsy is likely to show : 

1- Minimal change glomerulonephritis 


[ Q: 480 ] MRCPass - Nephrology 

A 42 year old man has presented 
with palpitations and dizziness. Upon 
investigation he has the following results: 

sodium 132 mmol/l 

potassium 7.6mmol/l 

urea 22 pmol/l 

creatinine 360 pmol/l 

What should be the initial management? 

1- Frusemide 

2- Haemodialysis 

3- Sodium bicarbonate 

4- Insulin and dextrose 



2- Membranoproliferative glomerulonephritis 

3- Focal segmental glomerulosclerosis 

4- Crescentic glomerulonephritis 

5- Membranous glomerulonephritis 


Answer & Comments 

Answer: 4- Crescentic glomerulonephritis 

The diagnosis is Goodpasture's syndrome. 
Initially there is a focal proliferative 
glomerulonephritis and it later develops into a 
proliferative crescentic glomerulonephritis. 
The proliferative changes are frequently 
associated with epithelial crescent formation 
and necrosis. 


5- Calcium gluconate 


Dr. Khalid Yusuf El-Zohry - Sohag Teaching Hospital (01118391123) 

Ref MRCPass OE OE 2012 PasTest 2009 PassMedicine 2009 PasTest Exam ReviseMRCP 



222 






























El-zohry MRCP Questions Bank (Port 1) - 2013 


(For my personal use) 



This immunofluorescence pattern shows 
positivity with antibody to IgG. There is a 
smooth, diffuse, linear pattern that is 
characteristic for deposition of glomerular 
basement membrane antibody with 
Goodpasture syndrome. 


[ Q: 482 ] MRCPass - Nephrology 

A 55 year man has nephrotic 
syndrome diagnosed recently. His 
investigations show an albumin of 20 g/L, 
Total cholesterol of 9 mmol/l, 24 hour urine 
reveals 4.5g protein excretion. A renal biopsy 
shows focal segmental glomerulosclerosis. 

Which one of following is most likely to 
preserve renal function? 

1- Bendrofluazide 

2- Salt restriction 

3- Lisinopril 

4- Atorvastatin 

5- Aspirin 



Answer & Comments 

Answer: 3- Lisinopril 

In Focal Segmental Glomerulosclerosis (FSGS), 
ACE inhibitors usually reduce proteinuria and 
lipemia, and may slow the progression tow 
ards renal disease. 


[ Q: 483 ] MRCPass - Nephrology 

A 32 year old woman has loin pains 
and haematuria. She has a family history of 
polycystic kidney disease. 



Which one of the following tests should be 
performed? 

1- Intravenous urogram 

2- Renal ultrasound 

3- Genetic testing 

4- Isotope renography 

5- Urine dipstick 


Answer & Comments 

Answer: 2- Renal ultrasound 

Renal ultrasound is the simplest test. Genetic 
testing can also be done (PKD1 on 
chromosome 16, PKD2 on chromosome 4). 



[ Q: 484 ] MRCPass - Nephrology 

A 50 year old man has end stage 
renal failure due to diabetes. He receives 4 
hours of haemodialysis, 3x weekly. 


His pressure is 180/105 mmHg predialysis and 
160/90 mmHg postdialysis. Predialysis bloods 
show: 


Hb lOg/dL 

sodium 132 mmol/l 

potassium 6.5 mmol/L 

urea 45 mmol/l 

creatinine 1125 umol/L 

calcium (corrected) 2.15 mmol/L 

phosphate 1.2 mmol/l 

Whot is the best management step? 

1- Aluminium phosphate binders 

2- 1-alfacalcidol supplements 

3- Increase length of haemodialysis 

4- Erythropoietin supplements 

5- Calcium gluconate iv 


Answer & Comments 


Answer: 3- Increase length of haemodialysis 


Dr. Khalid Yusuf El-Zohry - Sohag Teaching Hospital (01118391123) 

Ref MRCPass OE OE 2012 PasTest 2009 PassMedicine 2009 PasTest Exam ReviseMRCP 



223 




























El-zohry MRCP Questions Bank (Port 1) - 2013 


(For my personal use) 


The high urea and creatinine, as well as 
hypertension suggests there is more room for 
dialysis. Although EPO or vit D supplements 
are probably beneficial, longer dialysis 
sessions would help clear the fluid overload 
and reno-toxins. 


striking vacuolar change in the renal tubular 
cells, with large nodular aggregates of foamy 
cells. Intracellular accumulation of several 
glycosphingolipids was suspected. 

Which one of the following is likely? 

1- Tuberculosis 


[ Q: 485 ] MRCPass - Nephrology 

A 40 year old patient with renal 
calculi has further investigation. 

High urinary levels of the following substances 
predispose to urinary tract stone formation, 
EXCEPT FOR: 

1- Cystine 

2- Citrate 

3- Calcium 

4- Urate 

5- Oxalate 



Answer & Comments 

Answer: 2- Citrate 

Calcium, oxalate, cystine and urate are all 
stone-forming substances, so high urinary 
levels promote stone formation. Low urinary 
citrate levels promote stone formation. 


[ Q: 486 ] MRCPass - Nephrology 

A 30-year-old woman presented 
with a history of nephrotic syndrome 
diagnosed when she was 4 years old. 

There was no family history of renal disease. 
An initial renal biopsy performed showed 
minimal-change disease. 

She was treated with steroids and was 
determined to be steroid-dependent. 
Subsequent trials of cyclosporine and 
cyclophosphamide failed to improve her 
condition and she was referred for additional 
evaluation. 

She has a second renal biopsy. Light 
microscopic examination showed a patchy but 



2- Diabetic nephropathy 

3- Anderson Fabry disease 

4- Glycogen storage disease 

5- Fibromuscular dysplasia 


Answer & Comments 

Answer: 3- Anderson Fabry disease 

Inheritance of Anderson Fabry disease is 
usually X linked recessive. A rare, inherited 
metabolic disease in which a glycolipid, 
ceramide trihexoside, accumulates in blood 
vessels, as well as in numerous tissues and 
organs. The excessive amounts present in the 
kidneys and other organs impairs their 
function. It is due to the absence of alpha 
galactosidase A. 

Patients present with the skin lesions 
(angiokeratoma corporis diffusum), small red 
spots seen on the lower abdomen, thighs and 
scrotum, corneal opacities, episodes of fever, 
primary aparasthesia of the extremities and 
peripheral oedema and renal failure. It is 
prevalent in males, who present with full-blow 
n syndrome. Females are more likely to 
present with a partial form. 



Angiokeratoma corporis diffusum 


Dr. Khalid Yusuf El-Zohry - Sohag Teaching Hospital (01118391123) 

Ref MRCPass OE OE 2012 PasTest 2009 PassMedicine 2009 PasTest Exam ReviseMRCP 


























El-zohry MRCP Questions Bank (Port 1) - 2013 


(For my personal use) 


^ [ Q: 487 ] MRCPass - Nephrology 

ti - 

# A 50 year old woman has ankle 
oedema. 

Urine dipstick analysis shows +++ Protein. 

Blood tests show: 

creatinine 90 micromol/l 

urea 8 mmol/l 

albumin 20g/l 

A renal biopsy shows foot process fusion on 
the electron microscopy. 

Which of the following agents should be 
started? 


Ferrtin concentration : decreased in three 
months from upper limit of normal to lower 
limit of normal 

What is the cause for the anemia? 

1- Inadequate erythropoietin 

2- Osteitis fibrosa cystica 

3- Inadequate dialysis 

4- Iron deficiency 

5- Hyperparathyroidism 

Answer & Comments 

Answer: 4- Iron deficiency 


1- Infliximab 

2- Prednisolone 

3- Clopidogrel 

4- Azathioprine 

5- Ciclosporin 


Answer & Comments 

Answer: 2- Prednisolone 

The diagnosis is minimal change 
glomerulonephritis. Initial therapy is with 
prednisolone rather than Azathioprine. 
Retraction of the epithelial foot processes is 
observed consistently in patients with MCD. 
This is, at times, described as foot-process 
fusion and results from disordered epithelial 
cell structure with withdraw al of the dendritic 
process. This leads to the loss of the normal 
charge barrier such that albumin selectively 
leaks out and proteinuria ensues. 


[ Q: 488 ] MRCPass - Nephrology 

A 51 year old man was on dialysis for 
renal failure. He was on erythropoietin. 
Investigations show: 

Hb 8 g/dl 

Parathyroid hormone 15 (0.8-8) pmol/l 



There are many causes of decreased 
responsiveness to erythropoietin in patients 
with chronic renal failure. The most common 
is insufficient iron to meet the demands of 
increased erythrocyte production. Other 
factors that can attenuate the response 
include aluminum toxicity, infection, 
inflammation, and folic acid deficiency. 

[ Q: 489 ] MRCPass - Nephrology 

A 18 year old male is admitted with a 
history of haematuria and facial sw elling. 
Since two weeks ago he complained of a 
sorethroat which was very bad and persistent. 
On examination there is facial and ankle 
oedema. 

His blood pressure is marginally elevated at 
150/90. Investigations show haematuria and 
red cell casts. Blood urea is elevated. C 3 and C 
4 levels are reduced. Abdominal ultrasound 
shows normal sized kidneys. 

What is the diagnosis? 

1- Ig A nephropathy 

2- Mesangiocapillary glomerulonephritis 

3- Post streptococcal acute nephritic 
syndrome 

4- Renal artery stenosis 

5- Focal segmental glomerulonephritis 



Dr. Khalid Yusuf El-Zohry - Sohag Teaching Hospital (01118391123) 

Ref MRCPass OE OE 2012 PasTest 2009 PassMedicine 2009 PasTest Exam 




ReviseMRCP 

225 
























El-zohry MRCP Questions Bank (Port 1) - 2013 


(For my personal use) 


Answer & Comments 

Answer: 3- Post streptococcal acute nephritic 
syndrome 

In post streptococcal acute nephritic 
syndrome, 

pontaneous improvement usually occurs and 
the aim of treatment is to prevent 
complications such as pulmonary oedema, 
uraemia or hypertensive encephalopathy 
whilst awaiting spontaneous improvement in 
renal function. 

Penicillin may help active bacterial infection of 
the throat. 


[ Q: 490 ] MRCPass - Nephrology 

An 60 year old woman was brought 
to the emergency department with 
hypertension and abdominal pain. 

Physical examination and initial laboratory 
work-up were unremarkable; except for 
severe abdominal tenderness and elevated 
lactate dehydrogenase. 

A CT scan showed a hypodense, triangle¬ 
shaped area in the right kidney suggestive of 
renal infarction. Renal duplex scanning 
revealed a focal increase of flow velocities 
followed by turbulence in multiple segments 
of the right renal artery. Arteriography 
showed multiple lesions in a "string of beads" 
pattern and a parenchymal filling defect. 

Whot is the diagnosis? 

1- Polyarteritis nodosa 

2- Wegener's granulomatosis 

3- Sarcoidosis 

4- Adult polycystic kidney disease 

5- Fibromuscular Dysplasia 

Answer & Comments 

Answer: 5- Fibromuscular Dysplasia 



Fibromuscular Dysplasia is an uncommon 

vascular disease, which results in arterial 

dilation and narrowing of vessel segments. 
Definitive diagnosis can be made by selective 
renal arteriographs and bilateral renal vein 
and systemic venous renin measurements. 

Fibromuscular dysplasia (FMD), is a 

developmental lesion of unknow n etiology 

which can affect multiple vessels. It consists of 
areas of heaped up intima, media, and 
adventitia alternating with areas of medial 
destruction, the latter resulting in small focal 
aneurysms. The disorder can be progressive 
and affects females greater than males in a 
ratio of 3:1. It more commonly affects middle 
aged patients. 



Image shows a diffusely beaded appearance 
of the right main renal artery to the level of 

the branch vessels. 


[ Q: 491 ] MRCPass - Nephrology 

A 35 year old woman with a 25-year 
history of diabetes treated with insulin is 
found to have lg proteinuria per 24 hour and 
a creatinine 200 pmol/l. He has significant 
peripheral oedema. Blood pressure is 145/85 
mm Hg. 

HbAlC is 8%. 

In order to preserve renal function, which of 
the following is the most important step? 

1- Commence frusemide 

2- Strict restriction of dietary phosphate 




Dr. Khalid Yusuf El-Zohry - Sohag Teaching Hospital (01118391123) 

Ref MRCPass OE OE 2012 PasTest 2009 PassMedicine 2009 PasTest Exam ReviseMRCP 

























El-zohry MRCP Questions Bank (Port 1) - 2013 


(For my personal use) 


3- Strict diabetic control aiming for HbAlc < 
7% 

4- Commence a beta blocker 

5- Commence an ACE inhibitor 


Answer & Comments 

Answer: 5- Commence an ACE inhibitor 

There is clear evidence that ACE inhibition 
(enalapril, captopril used in trials) in diabetics 
delays progression of renal failure even if they 
are normotensive. 


[ Q: 492 ] MRCPass - Nephrology 

A 19-year-old male is unwell. He has 
previously had a history of renal 
transplantation 5 years ago. Within three days 
of admission he developed acute renal failure 
with evidence of haemolytic anaemia and 
thrombocytopenia. 

A clinical diagnosis of haemolytic uraemic 
syndrome was made and he was treated with 
plasma exchange. 

Which one of the following drugs could have 
caused this? 

1- Ciclosporin 

2- Trimethoprim 

3- Prednisolone 

4- Amoxycillin 

5- Intravenous immunoglobulin 



Answer & Comments 

Answer: 1- Ciclosporin 

Ciclosporin can cause HUS in renal transplant 
recipients, especially in those also receiving 
rapamycin, which increases tissue 
concentrations of ciclosporin. 

[ Q: 493 ] MRCPass - Nephrology 
A 30 year old African woman 



presents with seizures, hypertension, a malar 
rash. 

Bloods show: 

Hb 11 g/dl, WCC 8 x 10 9 /L, platelets 180 x 
10 9 /L, urea 22 pmol/l, creatinine 290 pmol/l, 
sodium 140 mmol/l, potassium 5 mmol/I, ESR 
100 mm/hr, CRP 25 mg/I. 

What is the most likely diagnosis? 

1- Anti GBM disease 

2- Multiple myeloma 

3- SLE 

4- Wegener's granulomatosis 

5- Sickle cell disease 


Answer & Comments 

Answer: 3- SLE 

With the history of renal impairment, raised 
ESR, malar flush, SLE is the most likely 
diagnosis. 



[ Q: 494 ] MRCPass - Nephrology 

A 45 year old diabetic gentleman had 
a renal biopsy during investigation of 
worsening renal failure. 


Which of the following is typical of diabetic 
nephropathy? 


1- Hyaline thrombus formation 


2- Mesangial hypercellularity 


3- Congo red staining causing green 
birefringence 


4- Basement membrane spikes 

5- Basement membrane thickening and 
mesangial widening 


Answer & Comments 

Answer: 5- Basement membrane thickening 
and mesangial widening 

The earliest morphologic abnormality in 
diabetic nephropathy is the thickening of the 


Dr. Khalid Yusuf El-Zohry - Sohag Teaching Hospital (01118391123) 

Ref MRCPass OE OE 2012 PasTest 2009 PassMedicine 2009 PasTest Exam ReviseMRCP 



227 
































El-zohry MRCP Questions Bank (Port 1) - 2013 


(For my personal use) 


glomerular basement membrane (GBM) and 
expansion of the mesangium due to 
accumulation of extracellular matrix. Light 
microscopy findings show an increase in the 
solid spaces of the tuft, material (diffuse 
diabetic glomerulopathy). 


4- Ibuprofen 

5- Allopurinol 


Answer & Comments 


Answer: 4- Ibuprofen 


Large acellular accumulations also may be 
observed within these areas. These are 
circular on section and are known as the 
Kimmelstiel-Wilson lesions/nodules. 

Immunofluorescence microscopy may reveal 
deposition of immunoglobulin G along the 
GBM in a linear pattern, but this is not 
diagnostic. In advanced disease, the mesangial 
regions occupy a large proportion of the tuft, 
with prominent matrix content. Further, the 
basement membrane in the capillary walls (ie, 
the peripheral basement membrane) is thicker 
than normal. 



Diabetic glomerulopathy of Kimmelstiel- 

Wilson disease 


^ [ Q: 495 ] MRCPass - Nephrology 

# A 40 year old woman presents with 
sw ollen ankles. Her urine dipstick reveals 
protein +++, but no other abnormality. She 
has gout and diabetes. Plasma creatinine is 
120 micromoles/I and albumin 28g/l. 

Which of the following medications would be 
most likely to account the sw ollen ankles and 
proteinuria? 

1- Lisinopril 

2- Bendrofluazide 

3- Prednisolone 


Non-steroidal anti-inflammatory drugs can 
cause nephrotic syndrome and interstitial 
nephritis. 


[ Q: 496 ] MRCPass - Nephrology 

A 45 year old woman presents with 
high fever, purpura occurring on her arms and 
legs, haematuria and renal dysfunction. 

Investigation results were: 

sodium 135 mmol/I 

potassium 4.5 mmol/I 

urea 5 mmol/I 

creatinine 100 pmol/l 

A p-ANCA is positive 

Renal biopsy before steroid therapy confirmed 
the diagnosis of pauci-immune-type crescentic 
glomerulonephritis. 

What is the diagnosis? 

1- Post streptococcal glomerulonephritis 

2- Microscopic polyangitis 

3- Temporal arteritis 

4- Takayasu's arteritis 

5- Behcet's disease 



Answer & Comments 

Answer: 2- Microscopic polyangitis 

P-ANCA (perinuclear staining) is usually due to 
myeolperoxidase, and is found in : 

microscopic polyarteritis (80%) 

idiopathic crescentic GN(65%) 

Churg-Strauss Syndrome 

polyarteritis nodosa 


Dr. Khalid Yusuf El-Zohry - Sohag Teaching Hospital (01118391123) 

Ref MRCPass OE OE 2012 PasTest 2009 PassMedicine 2009 PasTest Exam ReviseMRCP 



228 

























El-zohry MRCP Questions Bank (Port 1) - 2013 


(For my personal use) 


Wegener's granulomatosus 


3- Iv dextrose (50% 50mls) insulin 15 units 


SLE 

Although p ANCA can be positive in many 
conditions, c ANCA is found more commonly 
in Wegener's Granulomatosis. 

Treatment of the condition is with high dose 
steroids. 

[ Q: 497 ] MRCPass - Nephrology 

A 40 year old man has a potassium 
of 7.5 mmol/I and requires treatment. ECG 
shows widened QRS complexes. 

Which one of the following agents is 
cardioprotective but does not lower 
potassium? 

1- NSAIDs 

2- Salbutamol 

3- ACE inhibitors 

4- Calcium gluconate 

5- Insulin and dextrose 



Answer & Comments 

Answer: 4- Calcium gluconate 

Calcium gluconate is cardioprotective but 
does not lower potassium. ECG changes of 
hyperkalaemia in progression of severity are 
tented T waves, decreased p wave amplitude, 
widened QRS complex, sinusoidal waves and 
asystole. 


[ Q: 498 ] MRCPass - Nephrology 

A 50 year old man who presents with 
dizziness has broad complexes on the ECG. His 
blood gas reveals that his potassium is 8 
mmol/l. 

Which of the following is the best medical 
therapy? 

1- Rectal calcium resonium 

2- Iv calcium gluconate lOmls 10% 



4- Nebulised salbutamol 

5- Iv sodium bicarbonate 


Answer & Comments 

Answer: 2- Iv calcium gluconate lOmls 10% 

The first line therapy for cardioprotection is 
calcium gluconate. 

Following this, insulin and dextrose should be 
given. 


[ Q: 499 ] MRCPass - Nephrology 

A 42 year old man has generalised 
oedema. Urine dipstick shows protein +++, 
blood -ve. 

Which one of the following findings is 
suggestive of amyloidosis? 

1- Abnormal liver function tests 

2- Cryoglobulinaemia 

3- Benign monoclonal gammopathy without 
myeloma 

4- Large numbers of granular casts, fat bodies 
and red cells in the urine 

5- Selective proteinuria 



Answer & Comments 

Answer: 3- Benign monoclonal gammopathy 
without myeloma 

Light chains in the urine suggest deposition 
causing amyloidosis (may be either myeloma 
or benign monoclonal gammopathy). 

Deposits occur in the tongue, nerves or heart. 
Cryoglobulinaemia is suggestive of lymphoma 
or myeloma. Granular casts, red cell casts and 
selective proteinuria suggest 

glomerulonephritis. 



[ Q: 500 ] MRCPass - Nephrology 
A 35 year old man has renal colic. 


Dr. Khalid Yusuf El-Zohry - Sohag Teaching Hospital (01118391123) 

Ref MRCPass OE OE 2012 PasTest 2009 PassMedicine 2009 PasTest Exam ReviseMRCP 



229 
































El-zohry MRCP Questions Bank (Port 1) - 2013 


(For my personal use) 


Ultrasound of the abdomen confirms renal 
calculi. Urinalysis showed typical hexagonal or 
benzene crystals. 

What is the diagnosis? 

1- Calcium oxalate stones 

2- Cystinuria 

3- Primary oxaluria 

4- Calcium carbonate stones 

5- Adult polycystic kidney disease 


Answer & Comments 

Answer: 2- Cystinuria 

Cystinuria is a disorder of proximal tubular 
cells. Amino aciduria (COAL - cystine, 
ornithine, arginine, lysine) - causes cystine 
stones (accounts for 1% of all stones). 
Urinalysis may show typical hexagonal or 
benzene crystals, which are essentially 
pathognomonic of cystinuria. Cystine stones 
are pale yellow. 

Renal calculi from a patient with cystinuria 
who had nephrectomy First-line therapy in 
most cases is a conservative approach, 
including large-volume fluid intake, regular 
urine pH monitoring (urine pH level of 7.5 and 
<8), dietary restrictions, and urinary 
alkalization with potassium citrate. 



[ Q: 501 ] MRCPass - Nephrology 

A 45 year woman has a renal 
transplantation. She has a history of diabetes. 
4 months after transplantation she presents 
acute pain in region of transplanted kidney. 



Urinalysis shows nitrites +, protein +, no blood 
or leucocytes. 

Which one of following is the likely diagnosis? 

1- Nephrolithiasis 

2- Pyelonephritis 

3- Graft rejection 

4- Diabetic nephropathy 

5- Native polycystic kidneys 


Answer & Comments 

Answer: 2- Pyelonephritis 

The patient has immunosuppression following 
transplantation. Graft rejection is possible, but 
the history would be most consistent with a 
pyelonephritis because of positive nitrites and 
loin pain. 


[ Q: 502 ] MRCPass - Nephrology 

A 50 year old woman has a corrected 
calcium of 2.90 mmol/I and phosphate 0.7 
mmol/l, parathyroid hormone (PTH) level is 
10.0 pmol/l (1.1 to 6.8), and the 24-hour 
urinary calcium excretion is 0.8 mmol/l. 

Plasma creatinine is 120 micromol/l and 
alkaline phosphatase 130U/I. There is a family 
history of hypercalcaemia. 

Which of the following is likely? 

1- The diagnosis is likely to be primary 
hyperparathyroidism 

2- Treatment with oral phosphate 
supplements should suppress her PTH level 

3- She should be screened for a mutation in 
the multiple endocrine neoplasia -1 gene. 

4- She has an abnormality in the calcium¬ 
sensing receptor 

5- Parathyroid surgery will be necessary in the 
future 




Dr. Khalid Yusuf El-Zohry - Sohag Teaching Hospital (01118391123) 

Ref MRCPass OE OE 2012 PasTest 2009 PassMedicine 2009 PasTest Exam 


ReviseMRCP 




























El-zohry MRCP Questions Bank (Port 1) - 2013 


(For my personal use) 


Answer & Comments 

Answer: 4- She has an abnormality in the 
calcium-sensing receptor 

The patient has an inappropriately high level 
of PTH, but the reduced urine calcium 
excretion suggests that she has familial 
hypocalcuric hypercalcaemia (FHH) due to a 
mutation in the calcium receptor. Familial 
hypocalciuric hypercalcemia (FHH) or familial 
benign hypercalcemia is an autosomal 
dominant inherited disorder of calcium 
metabolism. It is characterized by lifelong 
asymptomatic hypercalcemia associated with 
a relative hypocalciuria and a tendency to 
hypermagnesemia. The biochemical features 
of this disorder are difficult to distinguish from 
mild primary hyperparathyroidism. Several 
patients have had parathyroidectomy for 
hyperparathyroidism with no effect on 
calcium levels. 


stenosis or outflow obstruction, hence a renal 
biopsy is the best way to confirm a diagnosis. 



[ Q: 504 ] MRCPass - Nephrology 

A 45 year old woman presents with 
acute pain that radiates from his left loin to 
his left groin. The pain comes for a few 
seconds, and then goes. It is sharp in nature. A 
plain abdominal X Ray is unremarkable. 
Ultrasound examination demonstrates pelvi- 
calyceal dilatation and the presence of several 
masses. 


The most likely diagnosis is: 

1- Papillary necrosis 

2- Recurrent UTIs 


3- Cystine renal stones 

4- Uric acid renal stones 


5- Renal cell carcinoma 


\ 

[ Q: 503 ] MRCPass - Nephrology 

Answer & Comments 

r 

•i J 

A 60 year old man has a history of 

Answer: 4- Uric acid renal stones 


hypertension. His blood show : urea 20 pmo 
& creatinine 320 pmol/l. 

Urinalysis showed blood ++ protein ++. Renal 
ultrasound showed left kidney : 9 cm long, 
right be 8.5 cm long and no evidence of 
hydronephrosis. 

What is the next best investigation? 

1- Renal biopsy 

2- DMSA renography 

3- Intravenous urography 


Urinary stones made of calcium oxalate or of 
cystine are radio-opaque and would be visible 
on a plain X-ray. In contrast, uric acid stones 
are radiolucent, hence most likely in this case. 


[ Q: 505 ] MRCPass - Nephrology 

A 23-year-old man presents with a 
rash on his legs. He had a chest infection last 
week, and now has symptoms of joint pains. 
Stick testing of his urine reveals proteinuria 
and haematuria +++. 


+ 


4- Magnetic resonance angiography 

5- Retrograde pyelography 


Answer & Comments 

Answer: 1- Renal biopsy 

There is proteinuria and haematuria. Renal 
impairment is likely to be related to 
glomerular pathology rather than renal artery 


What is the most likely diagnosis? 

1- IgA nephropathy 

2- Mixed essential cryoglobulinaemia 

3- Membranous glomerulonephritis 

4- Acute nephritic syndrome 

5- Henoch Schonlein purpura 


Dr. Khalid Yusuf El-Zohry - Sohag Teaching Hospital (01118391123) 

Ref MRCPass OE OE 2012 PasTest 2009 PassMedicine 2009 PasTest Exam 




ReviseMRCP 

231 


























El-zohry MRCP Questions Bank (Port 1) - 2013 


(For my personal use) 


Answer & Comments 

Answer: 5- Henoch Schonlein purpura 

Henoch-Schonlein purpura is a disease that 
has the symptoms of purple spots on the skin, 
joint pain, gastrointestinal symptoms, and 
glomerulonephritis. The exact cause for this 
disorder is unknown. The syndrome is usually 
seen in children, but people of any age may be 
affected. It is more common in boys than in 
girls. 

Many people with Henoch-Schonlein purpura 
had an upper respiratory illness in the 
previous weeks. In this case, if a rash were not 
present, then IgA nephropathy would the 
most probable cause of his urinary findings. 

Mixed essential cryoglobulinaemia will often 
present with palpable purpura on the legs and 
nephritis, but is an uncommon disease of 
older patients. 



[ Q: 506 ] MRCPass - Nephrology 

A 60 year old man presents with pain 
in the lower back. The pain begins in the lower 
back and radiating to the lower abdomen. He 
has not been on any medications recently. 
Investigations reveal a normocytic 
normochromic anaemia, raised erythrocyte 
sedimentation rate and C-reactive protein. 
Renal function is impaired. Ultrasound scan 
shows bilateral hydronephrosis. 


Which of the following investigations should 
be done? 


1- CT abdomen 

2- 24 hour urine for creatinine clearance 


3- 24 hour urine protein 

4- Intravenous urogram 

5- MAG 3 scan 


The diagnosis is idiopathic retroperitoneal 
fibrosis. CT scanning will show a peri-aortic 
fibrotic, which can be confirmed by CT guided 
biopsy. 

[ Q: 507 ] MRCPass - Nephrology 

A 20-year-old man presented to a 
clinic with a tw o-day history of pain in his 
knees and shoulders. The pain had become 
severe enough that he was unable to walk. 
The patient also noted a low -grade fever and 
a purplish rash on his shins and feet on the 
morning of admission. 

Examination of the extremities revealed 
bilaterally tender, sw ollen wrists, knees and 
ankles without effusion or erythema. The 
patient had diffuse tenderness in several 
muscle groups, including the quadriceps, 
latissimus dorsi and trapezius. Skin 
examination revealed a palpable purpuric rash 
on both ankles that tapered up to the knees. 

Investigations showed renal impairment and 
also haematuria. 

Which one of the following is most likely? 

1- Henoch Schonlein purpura 

2- Infective endocarditis 

3- Cyclosporin toxicity 

4- Rhabdomyolysis 

5- Diabetic glomerulosclerosis 

Answer & Comments 

Answer: 1- Henoch Schonlein purpura 

Henoch Schonlein purpura (HSP) causes a rash 
around the trunk and polyarthritis. It is 
associated with renal failure caused by a 
glomerulonephritis which is similar to Ig A 
glomerulonephritis causing macroscopic 
haematuria. 



Answer & Comments 


Answer: 1- CT abdomen 


Apart from a rash, arthralgia and arthritis are 
the most common manifestations of Henoch- 
Schonlein purpura, occurring in 80 to 90 
percent of cases. Joint involvement is usually 


Dr. Kholid Yusuf El-Zohry - Sohog Teaching Hospital (01118391123) 

Ref MRCPass OE OE 2012 PasTest 2009 PassMedicine 2009 PasTest Exam ReviseMRCP 



232 



























El-zohry MRCP Questions Bank (Port 1) - 2013 


(For my personal use) 


oligoarticular and primarily affects the large 
joints of the shoulders, knees, wrists and 
elbows. The image below shows the rash of 
HSP. 



[ Q: 508 ] MRCPass - Nephrology 

An 25 year old man is referred to the 
outpatient clinic following the finding that he 
of microscopic haematuria and proteinuria on 
the urine dipstick. He also has bilateral 
sensorineural deafness. His sister also has 
deafness. 

What is the likely diagnosis? 

1- Liddle's syndrome 

2- Bartter's syndrome 

3- Alport's syndrome 

4- Von Hippel Lindau syndrome 

5- Holt Oram syndrome 



Answer & Comments 

Answer: 3- Alport's syndrome 

Inheritance can be X-linked dominant, 
autosomal dominant and, rarely, autosomal 
recessive in Alport's syndrome. It is a 
hereditary disease of basement membranes 
which is characterised by sensorineural 
deafness and renal failure. 

Patient's with Alport's syndrome often present 
with haematuria and proteinuria. Rarely 
cataracts also occur. 


Management include monitoring of renal 
function (until the need for dialysis) and 
management of hypertension. 


[ Q: 509 ] MRCPass - Nephrology 

A 50 year old lady has a diagnosis of 
scleroderma. She is admitted with dizziness. 
On examination, her pressure is 220/110 and 
there is bilateral papilloedema. Blood tests 
reveal a creatinine of 180 micromol/l. 

What is the best medication for treatment? 

1- Amlodipine 

2- Sodium nitroprusside 

3- Lisinopril 

4- Propafenone 

5- Labetalol 



Answer & Comments 

Answer: 3- Lisinopril 

The term "scleroderma renal crisis" has been 
used to characterize kidney involvement in 
scleroderma, because of the abrupt and 
potentially devastating consequences of 
kidney disease. 

When any persistent rise in blood pressure is 
detected, or if there is the appearance of 
proteinuria, treatment should promptly be 
instituted with angiotensin converting enzyme 
(ACE) inhibitors (enalapril, lisinopril etc). 

Angiotensin-converting enzyme (ACE) 
inhibition has significantly improved survival in 
such patients. 



[ Q: 510 ] MRCPass - Nephrology 

A 65 year old man was found on the 
floor in a flat and brought into hospital by 
ambulance. He has a temperature of 35°C, and 
a blood pressure of 110/70 mmHg. Dipstick 
urine analysis shows : blood +++. 

Blood test show : 


Creatinine, 280 micromol/L, LDH 900 U/L. 


Dr. Khalid Yusuf El-Zohry - Sohag Teaching Hospital (01118391123) 

Ref MRCPass OE OE 2012 PasTest 2009 PassMedicine 2009 PasTest Exam ReviseMRCP 






























El-zohry MRCP Questions Bank (Port 1) - 2013 


(For my personal use) 


What likely cause of renal failure? 

1- Diabetes 

2- Acute tubular necrosis 

3- Rhabdomyolysis 

4- Vasculitis 

5- Hypertension 


Answer & Comments 

Answer: 3- Rhabdomyolysis 

A patient who has been on the floor some 
time may have muscle injury and consequent 
rhabdomyolysis. There would be urine 
myoglobin and a CK measurement may be in 
the thousands. 



[ Q: 511 ] MRCPass - Nephrology 

A 75 year old gentleman referred for 
back and leg pains. He was unable to mobilise 
unaided and complains of urinary 
incontinence. He has had a TURP in the past 
for benign prostatic hypertrophy. 


On examination, he has 3/5 power in his left 
leg and 2/5 in his right, His plantars are 
upgoing. Rectal examination reveals an 
enlarged, irregular prostate and lax anal tone. 
There is sensory deficit over both the lower 
limbs upto the suprapubic level. 


There is a palpable bladder. 


Investigations show : 
Na 138 mmol/l 


K 4.5 mmol/l 
Urea 4.8 mmol/l 
Creatinine 100 mmol/l 


Glucose 13 mmol/l 


PSA 910 


He is commenced on high dose steroids. 
However, he appears to be deteriorating 
neurologically. 

What is the definitive treatment? 


1- Orchidectomy 

2- Further high dose dexamethasone 

3- Stilboestrol and aspirin 

4- Goserelin 

5- Bicatulamide 


Answer & Comments 

Answer: 1- Orchidectomy 

This man presents with spinal cord 
compression, which is potentially reversible. 
Out of the options, orchidectomy is the 
correct treatment, as this will eliminate all 
androgens immediately, which is the cause of 
the tumour growth. Bicalutamide, which is an 
androgen receptor antagonist, is slow acting 
and will not have an early effect. Goserelin is 
contraindicated in patients with acute spinal 
cord compression, as it will cause "tumour 
flare" and may worsen. It is usually prescribed 
5-7 days post bicalutamide loading in order to 
avoid the flare. 


[ Q: 512 ] MRCPass - Nephrology 

A 30 year old man has blood 
pressure of 200/90. His serum potassium is 
2.8 mmol/l, creatinine is 90 pmol/l. Urine 
dipstick shows protein trace. 

Which of the following diagnosis is likely? 

1- Conn's syndrome 

2- Phaeochromocytoma 

3- Diabetic nephropathy 

4- Hypertensive nephropathy 

5- Renal artery stenosis 



Answer & Comments 

Answer: 1- Conn's syndrome 

High levels of aldosterone in Conn's (primary 
aldosteronism) can also cause sodium 
retention, potassium loss and hypertension. 


Dr. Khalid Yusuf El-Zohry - Sohag Teaching Hospital (01118391123) 

Ref MRCPass OE OE 2012 PasTest 2009 PassMedicine 2009 PasTest Exam ReviseMRCP 



234 


























El-zohry MRCP Questions Bank (Port 1) - 2013 


(For my personal use) 


[ Q: 513 ] MRCPass - Nephrology 

A 15 year old girl has nephrotic 
syndrome due to minimal change 
glomerulonephritis. 

What is the likely long term outcome? 

1- Long term remission 

2- Recurrent relapses 

3- Chronic renal impairment 

4- Persistent proteinuria 

5- Persistent hypercholesterolaemia but not 
proteinuria 



Answer & Comments 

Answer: 1- Long term remission 

Minimal change disease usually responds well 
to medical treatment, with response to 
corticosteroids usually within the first month. 
90% of patients with minimal change disease 
in the younger age group achieve remission 
after 8 weeks of steroids. 


[ Q: 514 ] MRCPass - Nephrology 

A 45 year old man had recurrent 
flank pains and was found to have renal calculi 
on the ultrasound scan. His serum calcium 
measurements were normal. A 24 hour urine 
analysis revealed: 

calcium 12.2 mmol/24 hours (2.5 - 7.5) 

oxalate 320 mmol/24 hours (90 - 450) 

uric acid 3 mmol/24 hours (1.5 - 4.5) 

citrate 2.1 mmol/24hours(0.3-3.5) 

What is the recommended therapy to reduce 
stone formation? 

1- Prednisolone 



Answer & Comments 

Answer: 2- Calcium restriction 

The patient has hypercalciuria causing calculi 
formation. The first step should be to reduce 
calcium intake. The second could be to use 
thiazide diuretics. 


[ Q: 515 ] MRCPass - Nephrology 

A haemodialysis patient with 
hypertensive nephropathy, has the following 
blood results: calcium 2.85 (2.25-2.7) mmol/l, 
phosphate 2.3 (0.8-8)mmol/l. PTH is 1.2 (0.8-8 
pmol/l). The patient's medication includes 
alfacalcidol each day and calcium acetate as a 
phosphate binder before each meal. 

Which of the following should be 
recommended? 

1- The patient should have a low calcium diet 

2- Increased frequency of haemodialysis 

3- Dialysate calcium should be lowered 

4- Phosphate restriction in the diet 

5- Calcium acetate dose should be reduced 



Answer & Comments 

Answer: 4- Phosphate restriction in the diet 

Low ering the phosphate (rather than the 
calcium) is the most important measure, as 
the elevated phosphate stimulates 
parathyroid proliferation and PTH secretion. 
This is best done by reinforcing dietary 
restriction and ensuring that the patient is 
recieving adequate dialysis. The vitamin D 
analogue w ill be increasing absorption of 
calcium and phosphate so this should be 
reduced or stopped. 


2- Calcium restriction 

3- Penicillamine 

4- Increase alcohol consumption 

5- Bendrofluazide 



[ Q: 516 ] MRCPass - Nephrology 

In a patient with renal failure, use of 
which of the following medications would 
suggest a possible diagnosis of retroperitoneal 
fibrosis ? 


Dr. Khalid Yusuf El-Zohry - Sohag Teaching Hospital (01118391123) 

Ref MRCPass OE OE 2012 PasTest 2009 PassMedicine 2009 PasTest Exam 




ReviseMRCP 

235 






























El-zohry MRCP Questions Bank (Port 1) - 2013 


(For my personal use) 


1- Bendrofluazide 

2- Methysergide 

3- Diltiazem 

4- Aspirin 

5- Clopidogrel 

Answer & Comments 

Answer: 2- Methysergide 

Drugs associated with retroperitoneal fibrosis 
include: 

methysergide 


5- Metronidazole 

Answer & Comments 

Answer: 2- Vancomycin 

Peritonitis is characterized by abdominal pain 
and cloudy bags with greater than 100 WBC 
per ml of fluid. Typically when more than one 
organism is present, the diagnosis becomes 
more likely. Treatment is usually with a first 
generation cephalosporin, along with 
gentamicin (then adjusted according to 
culture sensitivities) or vancomycin with 
tobramicin. 


beta-adrenergic blockers 

lysergic acid diethylamide 

methyldopa 

amphetamines 

phenacetin 

pergolide 

cocaine 


[ Q: 517 ] MRCPass - Nephrology 

A 36 year old male patient was 
admitted with abdominal pain, diarrhea, 
nausea, vomiting and fever which had started 
one week before. 

The patient had been followed up with 
predialysis Chronic Renal Failure(CRF) 
diagnosis for 4 years and has been receiving 
continuous ambulatory peritoneal dialysis 
(CAPD) treatment for 4 months. 

In the peritoneal fluid, 1600/ mm 3 cells were 
detected and 70% of them were 
polymorphonuclear leukocytosis. 

Whot should the patient be treated with? 

1- Amoxycillin 

2- Vancomycin 

3- Ciprofloxacin 

4- Trimethoprim 



[ Q: 518 ] MRCPass - Nephrology 

A 45 year old man presents to to the 
hospital with severe lethargy. Initial 
investigations identify a significant anaemia 
with fragmentation on the blood film and low 
platelets. Clotting screen is normal. His U+Es 
show a creatinine of 260 umol/l. BP was 
recorded at 180/100. 

What is the most likely diagnosis? 

1- Thrombotic thrombocytopenic purpura 
(TTP) 

2- Disseminated intravascular coagulation 

3- Haemolytic uraemic syndrome 

4- Hypertensive nephropathy 

5- Polyarteritis nodosa 



Answer & Comments 

Answer: 3- Haemolytic uraemic syndrome 

HUS is a disorder characterised by 
thrombocytopenia, microangiopathic 

haemololytic anaemia (anaemia secondary to 
red blood cell fragmentation) and renal 
(kidney) failure. About 90 per cent of HUS 
cases are caused by a certain group of bacteria 
know n as verocytotoxin-producing E.coli 
(VTEC). The most commonly associated strain 
is E.coli 0157:H7. Sporadic cases of haemolytic 
uraemic syndrome (not associated with 


Dr. Khalid Yusuf El-Zohry - Sohag Teaching Hospital (01118391123) 

Ref MRCPass OE OE 2012 PasTest 2009 PassMedicine 2009 PasTest Exam ReviseMRCP 



236 



























El-zohry MRCP Questions Bank (Port 1) - 2013 


(For my personal use) 


diarrhoea) can be seen with HIV, malignancy, 
SLE. Treatment of sporadic HUS is with plasma 
exchange and fresh frozen plasma. 


[ Q: 519 ] MRCPass - Nephrology 

A haemodialysis patient has 
recurrent knee and ankle sw elling and pains. 
Aspiration of the knee synovial fluid reveals 
negatively birefringent crystals. 

Which one of the following treatments is the 
least effective either acutely or in the chronic 
situation? 

1- Probenecid 

2- Prednisolone 

3- Increased dialysis 

4- Allopurinol 

5- Colchicine 



Answer & Comments 

Answer: 1- Probenecid 

Negative birefringence with polarised light 
demonstrates urate crystals, making the 
diagnosis consistent with gout. Probenecid is a 
uricosuric agent (reduces the reabsorption of 
uric acid), hence likely to produce little benefit 
in a patient who has significant renal 
impairment. 

Allopurinol (xanthine oxidase inhibitor) will 
reduce urate production, prednisolone or 
colchicine have anti-inflammatory properties. 

Haemodialysis will also increase urate 
clearance. 



Intracellular monosodium urate crystal viewed 
under a polarized light microscope (right) and 
a conventional light microscope (left). 


[ Q: 520 ] MRCPass - Nephrology 

#1 - 

* A 45 year old patient with chronic 

hepatitis C has a creatinine of 140 pmol/l. His 
BP is 150/90. He has urine dipstick showing 
blood ++, protein +++. 


Which of the following might be demonstrated 
on the histology? 


1- Minimal change glomerulonephritis 


2- Membranous glomerulonephritis 

3- Focal segmental glomerulonephritis 


4- Membranoproliferative glomerulonephritis 

5- Glomerulosclerosis 


Answer & Comments 

Answer: 4- Membranoproliferative 

glomerulonephritis 

This is chronic hepatitis C infection. It is 
associated with cryoglobulinaemia (which may 
present as a vasculitic rash) and 
membranoproliferative glomerulonephritis. 

Also, it (less commonly) can cause 
membranous glomerulonephritis. 


Dr. Khalid Yusuf El-Zohry - Sohag Teaching Hospital (01118391123) 

Ref MRCPass OE OE 2012 PasTest 2009 PassMedicine 2009 PasTest Exam 




ReviseMRCP 

237 























El-zohry MRCP Questions Bank (Port 1) - 2013 


(For my personal use) 



The glomerulus has increased overall 
cellularity and increased mesangial cellularity 
in membranoproliferative glomerulonephritis. 


Mesangial proliferation may represent a 
subset of patients who respond poorly to 
steroids. 

^ [ Q: 522 ] MRCPass - Nephrology 

# A 22 year old lady has a 5 year 
history of loin pains. She noticed recently that 
she had passed a small stone when passing 
urine. A 24 hour urine collection showed 
normal calcium and phosphate levels. Urinary 
levels of arginine, cystine, lysine and ornithine 
were elevated. 


[ Q: 521 ] MRCPass - Nephrology 

A 34 year old patient has 4g of 
proteinuria over 24 hours. She is treated with 
corticosteroids. 

Which of the following predicts a good 
response to corticosteroid therapy in nephrotic 
syndrome? 

1- Hypertension 

2- Haematuria 

3- Proteinuria 

4- The onset is within the first month of life 



Which of the following is true? 

1- Renal failure does not occur 

2- The stone is radiolucent 

3- There is accumulation of cystine in 
epithelial cells 

4- There is tubular necrosis 

5- The condition is autosomal dominant 


Answer & Comments 

Answer: 3- There is accumulation of cystine in 
epithelial cells 


5- Proliferative changes are observed on renal 
biopsy 

Answer & Comments 

Answer: 3- Proteinuria 

Proteinuria is highly selective, occuring in 75% 
of patients with minimal change, which is 
responsive to steroids. 

There is poor response to steroids in 
membranous, membranoproliferative, focal 
segmental glomerulonephritis. 

Hypertension is aggravated by steroids. 
Haematuria indicates that more sinister 
causes such as streptococcal infection may 
have occurred. Early onset nephrotic 
syndrome is correlated with severity, peak 
onset age is 2-4 yrs. 


Cystinuria is an autosomal recessive defect in 
reabsorptive transport of cystine and the 
dibasic amino acids ornithine, arginine, and 
lysine from the luminal fluid of the renal 
proximal tubule and small intestine. The 
genetic defect impairs intestinal absorption 
and renal reabsorption of cystine, causing 
elevated urinary levels of cystine and 
subsequent crystallization and stone 
formation. The only phenotypic manifestation 
of cystinuria is cystine urolithiasis. Cystinuric 
patients usually present with renal colic. 

Uncommon presentations include hematuria, 
chronic backache, and urinary tract infection. 
Twenty-five percent of symptomatic patients 
report their first stone in the first decade of 
life. 


Dr. Kholid Yusuf El-Zohry - Sohog Teaching Hospital (01118391123) 

Ref MRCPass OE OE 2012 PasTest 2009 PassMedicine 2009 PasTest Exam ReviseMRCP 


























El-zohry MRCP Questions Bank (Port 1) - 2013 


(For my personal use) 


[ Q: 523 ] MRCPass - Nephrology 

A 50 year old man attends casualty 
concerned that on passing urine he also 
passed a small lump. He has a history of 
arthritis, diabetes and hypertension. His 
serum creatinine is 180 micromol/l and his 
albumin is 38 g/l. A frozen section is 
performed by a pathologist who reports that 
the lump is a renal papilla. 

What is the diagnosis related to? 

1- Diabetic nephropathy 

2- Membranous nephropathy 

3- Minimal change nephropathy 

4- NSAID nephropathy 

5- Hypertensive nephropathy 



Answer & Comments 

Answer: 4- NSAID nephropathy 

This is a case of papillary necrosis due to 
analgesic nephropathy. This is due to chronic 
vasoconstriction of blood vessels within the 
kidney (prostaglandin reduction by NSAIDs). 

Renal papillary necrosis (RPN) is the necrosis 
of the renal medullary pyramids and papillae 
brought on by a host of associated conditions 
and toxins.A useful mnemonic device for the 
conditions associated with RPN is POSTCARDS, 
which stands for the following: 


[ Q: 524 ] MRCPass - Nephrology 

A 35 year old man presents with a 
history of recurrent episodes of haematuria. 
This is worse during episodes of upper 
respiratory infections and comes on within 
approximately 12-24 hours of development of 
pharyngitis. He also describes haematuria. 
Clinically he looks well, there is no oedema, 
blood pressure is 120/70 and creatinine is 100 
pmol. 

The renal biopsy is likely to show: 

1- Proliferative glomerulonephritis with 
crescent formation 

2- Mesangial proliferation 

3- Minimal change 

4- Podocyte proliferation 

5- Basement membrane destruction 



Answer & Comments 

Answer: 2- Mesangial proliferation 

The description of recurrent haematuria 
related to pharyngitis in a young male with no 
physical signs points tow ards IgA 
nephropathy (Berger's disease). The light 
microscopic feature of this is mesangial 
proliferation. Immunofluorescence would 
demonstrate IgA deposition in the mesangium 
as confluent masses or discrete granules. 


Pyelonephritis 
Obstruction of urinary tract 
Sickle cell 
Tuberculosis 
Cirrhosis of the liver 
Analgesic abuse 
Renal transplant rejection 
Diabetes mellitus 
Systemic vasculitis 



[ Q: 525 ] MRCPass - Nephrology 

A 25 year old drug user, presented to 
hospital unwell. He has a past medical history 
of type 1 diabetes. He had injected cocaine 
into his leg a day ago. On examination, he was 
pale, BP was 70/40 mmHg, Temperature was 


36°C. He complained of a painful, sw ollen, 
right leg. 

Investigations showed: 
sodium 136 mmol/l 


potassium 5.9 mmol/l 
urea 15 mmol/l 


Dr. Khalid Yusuf El-Zohry - Sohag Teaching Hospital (01118391123) 

Ref MRCPass OE OE 2012 PasTest 2009 PassMedicine 2009 PasTest Exam ReviseMRCP 



239 




























El-zohry MRCP Questions Bank (Port 1) - 2013 


(For my personal use) 


creatinine 190 pmol/l 
Creatine Kinase 12,000 (24-170) U/l 
Urine dipstick showed myoglobinuria. 
What is the diagnosis? 

1- Rhabdomyolysis 

2- Hypocalcaemic hypocalciuria 

3- Rapidly progressive glomerulonephritis 

4- Diabetic nephropathy 

5- Endocarditis 


Answer & Comments 

Answer: 1- Rhabdomyolysis 

Rhabdomyolysis results from muscle injury 
and the release of myoglobin, which is toxic to 
the renal tubules. Myoglobinuria usually 
occurs. Muscle cells also release creatinine 
kinase and potassium when they are injured. 


[ Q: 526 ] MRCPass - Nephrology 

Which of the following options do 
you think would be the most appropriate in 
supporting a diagnosis of reflux nephropathy 
in a 35 year old woman with plasma creatinine 
330 micromole/litre and a history of repeated 
urinary tract infections? 

1- Intravenous urography 

2- CT of abdomen 



[ Q: 527 ] MRCPass - Nephrology 

A 75 year old diabetic man 
presented to his local hospital with unstable 
angina where he received conventional 
treatment with intravenous nitrates and 
heparin. His pain subsided but he 
subsequently developed recurrent episodes of 
"flash" pulmonary oedema and recalcitrant 
hypertension. These were accompanied by an 
decline in renal function over four weeks until 
eventually he became dependent on dialysis. 

Severe hypertension and signs of peripheral 
vascular disease were found on physical 
examination, but there w ere no audible renal 
bruits. Non-blanching purpuric lesions 
suggestive of embolisation were noted on his 
toes. 

Blood tests showed a marked peripheral blood 
eosinophilia of 0.59 x 10 9 /L (< 0.40) and a C3 
complement component just below the lower 
limit of normal, at 0.59 g/l (0.6-1.6). An 
angiogram performed using spiral computed 
tomography showed an atheromatous aorta 
but no evidence of renal artery stenosis. A 
percutaneous renal biopsy showed clefts in 
the lumen of intrarenal arterioles. 

What is the diagnosis? 

1- Diabetic nephropathy 

2- Minimal change glomerulonephritis 

3- Endocarditis 



3- MRI of abdomen 


4- Cholesterol embolisation 


4- Isotopic imaging with 99mTc-DTPA. 

5- Renal ultrasound 

Answer & Comments 

Answer: 5- Renal ultrasound 

Ultrasound is a good method of detecting 
renal scars. DMSA, which is taken up by 
tubular cells can be used to detect scars (as 
opposed to DTPA which is filtered by the 
glomerulus and not taken up by tubular cells, 
hence not good for detecting scarring). 


5- Lupus nephritis 

Answer & Comments 

Answer: 4- Cholesterol embolisation 

Cholesterol embolisation characterised by a 
classic triad of livedo reticularis acute renal 
failure, and eosinophilia. Occasionally the 
presentation is atypical, with fever, myalgia, 
and multiorgan involvement mimicking 
systemic vasculitis. CRP and ESR are typically 
elevated. Risk factors for cholesterol 


Dr. Khalid Yusuf El-Zohry - Sohag Teaching Hospital (01118391123) 

Ref MRCPass OE OE 2012 PasTest 2009 PassMedicine 2009 PasTest Exam ReviseMRCP 



240 


























El-zohry MRCP Questions Bank (Port 1) - 2013 


(For my personal use) 


embolism include hypertension, diabetes, and 
aortic aneurysm. 


[ Q: 528 ] MRCPass - Nephrology 

A 35 year old patient has proteinuria 
and suspected renal disease. 

In which one of the situations would 
corticosteroids be useful in treatment? 

1- Renal vein thrombosis 

2- Minimal change disease 

3- Scleroderma 

4- Membranous nephropathy 

5- Amyloidosis 



Answer & Comments 

Answer: 2- Minimal change disease 

Out of all the options, minimal change disease 
is the most well known to be steroid 
responsive, with a good prognosis. 


Dr. Khalid Yusuf El-Zohry - Sohag Teaching Hospital (01118391123) 

OE OE 2012 PosTest 2009 PassMedicine 2009 PosTest Exom 


ReviseMRCP 


Ref MRCPoss 


241 



















El-zohry MRCP Questions Bank (Port 1) - 2013 


(For my personal use) 


^ [ Q: 529 ] MRCPass - Neurology 

f | - 

# A 55 year old man patient presents 
with an 8 month history of progressive 
difficulty in swallowing, and dysarthria. He has 
lost 6 kg in weight. On examination he has a 
fasciculation on the tongue and a brisk jaw 
jerk. 

What is the likely diagnosis? 


Answer & Comments 

Answer: 4- Glioblastoma 

The rapid onset of symptoms make it likely 
that this is a malignant lesion, and the most 
common malignancy of the central nervous 
system is a glioma. Glioblastoma multiforme is 
by far the most common and most malignant 
of the glial tumors. 


1- Amyotrophic lateral sclerosis 

2- Subacute combined degeneration of the 
cord 

3- Senile dementia 

4- Multiple sclerosis 

5- Guillain Barre syndrome 


Answer & Comments 


Answer: 1- Amyotrophic lateral sclerosis 



A fifth of patients with amyotrophic lateral 
sclerosis have bulbar involvement. This is a 
classic presentation of a patient with this 
variant of motor neuron disease. 


[ Q: 530 ] MRCPass - Neurology 

A 60 year old man is brought has had 
6 seizures over the past week. The seizures 
are generalised tonic clonic. 

He has complained of a severe headache, and 
examination shows increased tone on the left 
with mild hemiparesis. There is papillodoema 
in both eyes. An emergency MRI scan shows a 
mass in the right cerebral hemisphere. 

What is the most likely diagnosis? 



Glioblastoma 


[ Q: 531 ] MRCPass - Neurology 

A 42 year old man has weakness in 
the extensors of the upper limb and flexors of 
the lower limb muscles. His speech has a nasal 
quality. There is tongue fasciculation. 

What is the likely diagnosis? 

1- Wilson's disease 

2- Creutzfeldt Jakob disease 

3- Motor neuron disease 

4- Myasthenia gravis 

5- Thyrotoxicosis 



1- Medulloblastoma 

2- Craniopharyngioma 


Answer & Comments 


Answer: 3- Motor neuron disease 


3- Meningioma 

4- Glioblastoma 

5- Astrocytoma 


In motor neuron disease, UMN signs include 
muscle spasticity, slowed recruitment of 
voluntary muscle strength, weakness 
especially in the extensors of the upper limb 
and flexors of the lower limb muscles, 
pseudobulbar palsy. 


Dr. Khalid Yusuf El-Zohry - Sohag Teaching Hospital (01118391123) 

Ref MRCPass OE OE 2012 PasTest 2009 PassMedicine 2009 PasTest Exam ReviseMRCP 






























El-zohry MRCP Questions Bank (Port 1) - 2013 


(For my personal use) 


LMN features include muscle wasting and 
fasciculation, depressed reflexes and bulbar 
palsy. Onset of the disease is usually in mid to 
late adult life with the incidence increasing 
with advancing age. 



[ Q: 532 ] MRCPass - Neurology 

Following a football injury, a man 
has developed foot drop and has lost 
sensation to the dorsal part of the foot. 

Which nerve is most likely to have been 
involved? 

1- Tibial 

2- Sciatic 

3- Common peroneal 

4- Femoral 

5- Gluteal 


Answer & Comments 

Answer: 3- Common peroneal 

The common peroneal nerve controls foot 
eversion and dorsiflexion. Sensation is 
supplied to the antero-lateral part of the leg 
and the dorsum of the foot. 


Coir iv on 
Peroneal 


Bi anch to 
Peroneal 

• ever ter) 
iruscles 


i 


i 

» 

i 



i 

i 


i 

l 


i 






i 




Knee 

Cap 


Bianch to 
oorsiflexors 




" [ Q: 533 ] MRCPass - Neurology 

t - 

A 70 year old woman with a history 
of hypertension and insulin-dependent 
diabetes mellitus presented with new onset of 
uncontrolled violent movements of the left 
extremities accompanied by headaches. 

Neurological examination was normal except 
for the constant, relentless, violent movement 
of her left arm and leg. Although she was fully 
awake, alert, and oriented, she could not stop 
the abnormal movements except for a short 
period of time. 

This presentation is likely to be due to a lesion 
in the: 

1- Substantia nigra 

2- Globus pallidus 

3- Putamen 

4- Hypothalamus 

5- Subthalamic nucleus 


Answer & Comments 

Answer: 5- Subthalamic nucleus 

Hemiballismus is a rare disorder characterised 
by involuntary wild flinging movements of the 
limbs. It is usually unilateral (hence 
hemiballismus) and is caused by lesions in the 
contralateral subthalamic nucleus. 

VL nucleus at 
thalamus 



Substantia 

nigra 




Globus 

paflidus 


Subthalamic 

nucleus 


Basal ganglia 
and associated 


structures 

Caudate 
nucleus 


Striatum 


Putamen 



[ Q: 534 ] MRCPass - Neurology 
A 20 year old lady presents with a 


Dr. Khalid Yusuf El-Zohry - Sohag Teaching Hospital (01118391123) 

Ref MRCPass OE OE 2012 PasTest 2009 PassMedicine 2009 PasTest Exam ReviseMRCP 



244 































El-zohry MRCP Questions Bank (Port 1) - 2013 


(For my personal use) 


history of a single tonic-clonic seizure. She had 
not been taking any illicit drugs. She also 
reports the occurrence of occasional absent 
spells and brief jerking of his upper limbs, 
when she has been out late partying. 

Which of the Following drugs is most 
appropriate? 

1- Lorazepam 

2- Carbamazepine 

3- Phenytoin 

4- Gabapentin 

5- Sodium valproate 


Answer & Comments 

Answer: 5- Sodium valproate 

This patient is likely to have juvenile myoclonic 
epilepsy (JME), and valproate is the treatment 
of choice. Juvenile myoclonic epilepsy (JME) is 
an idiopathic generalized epileptic syndrome 
characterized by myoclonic jerks, generalized 
tonic-clonic seizures (GTCSs), and sometimes 
absence seizures. 

These can be precipitated when the patient is 
sleep deprived. Apart from sodium valproate, 
lamotrigine and topirimate can also be used. 


[ Q: 535 ] MRCPass - Neurology 

A 30 year old patient has recurrent 
episodes of severe, unilateral, pain felt in the 
region of the eye and forehead associated 
with lacrimation. 

The most likely diagnosis is: 

1- Migraine 

2- Subarachnoid haemorrhage 

3- Retro-orbital tumour 

4- Cluster headache 

5- Migranous neuralgia 



Cluster headache causes severe, usually 
unilateral, pain felt in the region of the eye 
and forehead associated with lacrimation, 
conjunctival injection and occasionally 
transient Horner's syndrome. 

Oxygen and triptans may arrest attacks, 
prophylaxis with propranolol, pizotifen, and 
especially verapamil may prevent further 
episodes. 


[ Q: 536 ] MRCPass - Neurology 

A 70 year old man has a 2 month 
history of progressive weakness in his legs. He 
has weakness in the hip and knee distribution. 
Knee and ankle reflexes are brisk. Plantars are 
upgoing bilaterally. He has sensory loss from 
legs upwards towards the T5 level. 

Which one of the Following is a likely 
diagnosis? 

1- Subacute combined generation of cord 

2- Friedrich's ataxia 

3- Meningioma 

4- Hereditary spastic paraparesis 

5- Tropical spastic paraparesis 



Answer & Comments 

Answer: 3- Meningioma 

A meningioma involving high in the spinal cord 
(thoracic region) can cause a T5 level. The rest 
are all causes of spastic paraparesis but are 
unlikely to cause a sensory level on clinical 
examination. 


Answer & Comments 


Answer: 4- Cluster headache 


Dr. Khalid Yusuf El-Zohry - Sohag Teaching Hospital (01118391123) 

Ref MRCPass OE OE 2012 PasTest 2009 PassMedicine 2009 PasTest Exam 




ReviseMRCP 

245 



























El-zohry MRCP Questions Bank (Port 1) - 2013 


(For my personal use) 



Spinal Cord Meningioma 


[ Q: 538 ] MRCPass - Neurology 

A 45 year old man has presented 
with lower back pain, radiating to his buttocks, 
associated with lower limb parathesia over 
the last few days. He is unable to walk due to 
leg weakness. The ankle reflexes are absent. 

Whot is the diagnosis? 

1- Disc prolapse 

2- Transverse myelitis 

3- Guillain Barre syndrome 

4- Multiple sclerosis 

5- Friedrich's ataxia 



[ Q: 537 ] MRCPass - Neurology 

A 50 patient is assessed for gradually 
progressive dementia over several months. 
Whilst he was on the ward he was noticed to 
have myoclonic jerks of his hands. 

Whot is the likely couse? 

1- HIV encephalopathy 

2- Parkinson's disease 



Answer & Comments 

Answer: 3- Guillain Barre syndrome 

Guillain Barre syndrome is preceded by 
respiratory or gastrointestinal symptoms in 
two-thirds, but not all, cases. Lower back pain, 
often radiating to the buttocks occurs in a 
third of cases. Difficulty walking in this case is 
due to distal weakness and absent distal 
reflexes suggest GBS. 


3- Alzheimer's disease 

4- Pick's disease 

5- Creutzfeldt Jakob disease 


Answer & Comments 

Answer: 5- Creutzfeldt Jakob disease 

Creutzfeldt Jakob disease (CJD) usually occurs 
in the 45-75 year age group and most 
commonly presents as a rapidly evolving 
multifocal dementia with myoclonic jerks in 
the latter stages. New variant CJD, which has 
been strongly linked to infection from meat 
products of cows with BSE, tends to present 
with behavioural and psychiatric disturbances 
progressing to inco-ordination and dementia 
with myoclonic jerks. 



[ Q: 539 ] MRCPass - Neurology 

A 62 year old man presents with a 
resting tremor of his right arm. He was found 
to have cogwheeling and bradykinesia. His gait 
is shuffling in nature. 


Which one of the Following drugs is most likely 
to help her tremor? 


1- Amantadine 


2- Benzhexol 


3- Bromocriptine 

4- Co-Careldopa 

5- Selegiline 


Answer & Comments 


Answer: 2- Benzhexol 



Dr. Kholid Yusuf El-Zohry - Sohog Teaching Hospital (01118391123) 

Ref MRCPass OE OE 2012 PasTest 2009 PassMedicine 2009 PasTest Exam 


ReviseMRCP 






























El-zohry MRCP Questions Bank (Port 1) - 2013 


(For my personal use) 


Benzhexol is an anticholinergic drug (used to 
alleviate tremors in parkinson's disease). The 
first line treatment is with L-dopa which is the 
metabolic precursor of L-dopa. Benzhexol is 
not effective against bradykinesia. 


[ Q: 540 ] MRCPass - Neurology 

A 46 year old female had a traumatic 
left sided tooth extraction 6 years ago. Since 
then she is complaining of facial pain mainly 
over upper part of left face with intermittent 
exacerbation. Occasionally the pain radiates to 
right side of the face. 

What is the likely diagnosis? 

1- Atypical facial pain 

2- Trigeminal neuralgia 

3- Dry socket cyst 

4- Bell's palsy 

5- Periodontitis 



Answer & Comments 

Answer: 1- Atypical facial pain 

Complicated dental procedures or other forms 
of trauma can lead to a form of atypical facial 
pains which is also known as post traumatic 
facial pain/ neuralgia. The pain is usually self 
limiting after several years. 


[ Q: 541 ] MRCPass - Neurology 

A 25 year old secretary has had 
several episodes of brief jerking of the right 
arm over the past few weeks. 

There is no loss of consciousness. A CT scan of 
the head is unremarkable. 

Which is the best medication to commence? 

1- Carbamazepine 

2- Phenytoin 

3- Lorazepam 

4- Diazepam 

5- Levodopa 



Answer & Comments 

Answer: 1- Carbamazepine 

Brief episodes of jerking suggests simple 
partial seizures. Carbamazepine is first line 
therapy for this. 

[ Q: 542 ] MRCPass - Neurology 

A 63 year old man who has been 
diagnosed with a glioma is commenced on 
chemotherapy. 4 days later, he begins to 
behave strangely, and has suicidal ideation. 

Which one of the Following is most likely? 

1- Vincristine encephalitis 

2- Hyponatraemia 

3- Steroid psychosis 

4- Hypoglycaemia 

5- Hypocalcaemia 



Answer & Comments 

Answer: 3- Steroid psychosis 

Incidence of steroid associated cognitive 
changes including psychosis are high 
particularly when high dose steroids are used 
(e.g. dexamethaxone). 


t 


[ Q: 543 ] MRCPass - Neurology 

A 75 year old man was admitted to 
the hospital after being unable to cope. His 
neighbours say that he had been increasingly 
confused over the last month and two weeks 
ago was seen to have a generalised seizure. 
He had no neurological signs on examination 
apart from an upgoing plantar on the left. 


Which of the Following is most likely? 

1- Subarachnoid haemorrhage 


2- Meningitis 

3- Subdural haematoma 


4- Parkinson's syndrome 

5- Meningioma 


Dr. Khalid Yusuf El-Zohry - Sohag Teaching Hospital (01118391123) 

Ref MRCPass OE OE 2012 PasTest 2009 PassMedicine 2009 PasTest Exam 




ReviseMRCP 

247 
































El-zohry MRCP Questions Bank (Port 1) - 2013 


(For my personal use) 


Answer & Comments 

Answer: 3- Subdural haematoma 

A history of confusion along with a possible 
seizure/fall in the elderlly should alert towards 
subdural haematoma. 



Subdural Haematoma 



[ Q: 544 ] MRCPass - Neurology 

A 65 year old man has a history of 
hypertension. He presents with sudden onset 
dysarthria, vomiting hiccup and vertigo. 


On examination, he has a right sided Horner's 
syndrome, right-sided cerebellar ataxia, loss of 
pain and temperature sensation on the right 
hand side of the face, and loss of pain and 
temperature sensation in the left upper and 
lower limbs. 


What is the likely diagnosis? 

1- Cerebellar infarct 


2- Inferior parietal infarct 

3- Superior temporal infarct 

4- Subthalamic nucleus infarct 


5- Lateral medullary infarct 


Answer & Comments 

Answer: 5- Lateral medullary infarct 

In the lateral medullary syndrome (inferior 
cerebellar artery involvement), 9th and 10th 
nerve involvelement leads to dysphagia and 


dysarthria. There is also ipsilateral Horner's 
syndrome and facial sensory loss, and 
contralateral pain/ temperature sensory loss 
to upper and lower limbs. 



[ Q: 545 ] MRCPass - Neurology 

A 60 year old man presents with 
drowsiness and confusion. A friend says he 
has been confabulating about having his 
wallet stolen. On examination, he is unkempt, 
his pupil reflexes are normal and eye 
movements are normal. There were otherwise 
no focal neurological deficits. A CT scan of the 
brain showed mild cerebral atrophy. He has a 
MCV of 105 fl. 


Which diagnosis is most likely? 

1- Transient global amnesia 

2- Korsakoff's psychosis 

3- Wernicke's encephalopathy 

4- Alzheimer's 


5- Pick's disease 


Answer & Comments 

Answer: 2- Korsakoff's psychosis 

The high MCV suggests heavy alcohol drinking. 
He is delusional (having his wallet stolen) and 
also there are no signs of nystagmus or 
cerebellar signs to support Wernicke's 
encephalopathy. 

Short term memory loss, amnesia and 
personality changes are also seen in 
Korsakoff's psychosis. 



[ Q: 546 ] MRCPass - Neurology 

A 17 year old boy with mild learning 
disability presents for assessment. He was 
apparently well 5 years ago, when he 
developed jaundice which persisted for 4 
months. He was found to have difficulty 
speaking, walking up the stairs and he also 
had choreoathetoid movements of the arms. 


Dr. Khalid Yusuf El-Zohry - Sohag Teaching Hospital (01118391123) 

Ref MRCPass OE OE 2012 PasTest 2009 PassMedicine 2009 PasTest Exam ReviseMRCP 

































El-zohry MRCP Questions Bank (Port 1) - 2013 


(For my personal use) 


On examination, there was jaundice, but no 
clubbing, cyanosis or peripheral 
lymphadenopathy. A neurological examination 
showed weakness and wasting of muscles 
acting on the shoulder and hip joints and brisk 
deep reflexes. The plantar responses were 
flexor. There were no extrapyramidal signs. 
The patient had Kayser Fleisher rings 
confirmed by slit-lamp examination. 

What is the likely diagnosis? 

1- New variant CJD 

2- Alzheimer's disease 

3- Motor neuron disease 

4- Wilson's disease 

5- Pick's disease 


Answer & Comments 

Answer: 4- Wilson's disease 

Wilson disease is a rare autosomal recessive 
inherited disorder of copper metabolism. The 
condition is characterized by excessive 
deposition of copper in the liver, brain, and 
other tissues. 

Wilson's disease usually presents at ages less 
than 40. Hepatic dysfunction is the presenting 
feature in more than half of patients. Apart 
from hepatitis, neuropsychiatric presentation 
and movement disorders are common 
presentations. Kayser-Fleischer rings are 
observed in up to 90% of individuals with 
symptomatic Wilson disease. 

Manifestations include dystonia, 

choreoathetoid movements, spasticity, grand 
mal seizures, rigidity, and flexion contractures. 


[ Q: 547 ] MRCPass - Neurology 

A 60 year old man is on high dose 
prednisolone for a diagnosis of giant cell 
arteritis 4 months ago. He complains of a 
headache, which had worsened for the past 
week and double vision. 



On examination, there is neck stiffness, and 
right sided 7th nerve palsy. His temperature is 
38 degrees. A lumbar puncture reveals a 
protein of 0.8 g/l, glucose of 3.5, WCC of 30 
(predominant neutrophils). 

What is the most likely causative organism? 

1- Listeria monocytogenes 

2- Mycobacterium tuberculosis 

3- Mycoplasma pneumoniae 

4- Borrelia burgdoferi 

5- Herpes simplex 


Answer & Comments 

Answer: 1- Listeria monocytogenes 

The lumbar puncture findings are consistent 
with a bacterial meningitis but the glucose is 
not low enough to suggest TB. Listeria 
monocytogenes is an aerobic and facultatively 
anaerobic gram-positive bacillus. The risk of 
listeriosis is markedly increased in immuno¬ 
compromised patients, particularly among 
those undergoing renal transplantation, 
receiving high doses of corticosteroids, or 
suffering with AIDS or cancer. 

Ampicillin or penicillin has generally been 
recommended as the treatment of choice. 

[ Q: 548 ] MRCPass - Neurology 

A 45 year old lady presents with a 
week's history of spontaneous, deep, right 
shoulder pain radiating to the neck. 

There is also weakness and then wasting of 
the right deltoid, spinati and triceps muscles. 

Which is the likely diagnosis? 

1- Spinobulbar dystrophy 

2- C5 and C6 myelopathy 

3- Guillain barre syndrome 

4- Rotator cuff tendonitis 

5- Brachial neuritis 



Dr. Khalid Yusuf El-Zohry - Sohag Teaching Hospital (01118391123) 

Ref MRCPass OE OE 2012 PasTest 2009 PassMedicine 2009 PasTest Exam ReviseMRCP 



249 

























El-zohry MRCP Questions Bank (Port 1) - 2013 


(For my personal use) 


Answer & Comments 

Answer: 5- Brachial neuritis 

Idiopathic brachial neuritis is an immune- 
mediated disorder, often preceded by an 
upper respiratory tract infection or 
immunisation. The initial feature is the abrupt 
onset of unilateral arm pain or shoulder pain. 
There is also evolving weakness, which is 
worst 2-3 weeks after the onset of pain. 


take at least two paracetamol and two 
tramadol tablets per day. 

Which is the likely diagnosis? 

1- Cluster headaches 

2- Analgesic induced headaches 

3- Migrainous headaches 

4- Space occupying tumour 

5- Trigeminal neuralgia 


[ Q: 549 ] MRCPass - Neurology 

A 60 year old man presents with 
difficulty with his mobility. He has increased 
tone, bradykinesia, a pill rolling tremor and a 
shuffling gait. 

Which medication should be commenced first? 

1- Benztropine 

2- Amantadine 

3- Selegiline 

4- Apomorphine 

5- Co-careldopa 



Answer & Comments 

Answer: 2- Analgesic induced headaches 

Frequent use of some immediate-relief 
medications can result in recurring or 
persistent headache in those with pre-existing 
headache and an individual susceptibility. 
Three or more simple analgesics (aspirin and/ 
or acetaminophen) a day (more than 1000 
mg) more often than 5 days a week. 

Frequent use of short-acting NSAIDs such as 
ibuprofen and also opiate based drugs can 
also be a cause. 


Answer & Comments 


[ Q: 551 ] MRCPass - Neurology 

Answer: 5- Co-careldopa 

t\] 

1 -S 

A 60 year old patient has diplopia 


Co-careldopa is the first medication used to 
increase dopaminergic activity in the basal 
ganglia. Apomorphine is used for on-off 
fluctuations. Benztropine is used for 
anticholinergic side effects. Selegiline is an 
MAO inhibitor, and Amantadine is an antiviral 
drug. Both are used as a second line drugs. 
Apomorphine is used for on-off fluctuations. 


[ Q: 550 ] MRCPass - Neurology 

A 50 year old man presents with a 5 
year history of headaches. The pain has 
gradually worsened and is now present daily, 
particularly on waking. He describes the pain 
as dull, generalised and constant. It is 
exacerbated by bright light. Neurological 
examination is unremarkable. She needs to 



looking to the left. When the left eye is 
covered, the outer image disappears. 

Which nerve is involved? 

1- Left abducens 

2- Right abducens 

3- Left trochlear 

4- Right trochlear 

5- Left oculomotor 

Answer & Comments 

Answer: 1- Left abducens 

Covering the affected eye causes the false 
image (outer image) to disappear. In this case, 
the left eye is involved hence left lateral rectus 
(abducens nerve). 


Dr. Khalid Yusuf El-Zohry - Sohag Teaching Hospital (01118391123) 

Ref MRCPass OE OE 2012 PasTest 2009 PassMedicine 2009 PasTest Exam ReviseMRCP 
































El-zohry MRCP Questions Bank (Part 1) - 2013 


(For my personal use) 


[ Q: 552 ] MRCPass - Neurology 

f | - 

# A 22 year old man had an accident 3 
years ago. During that time he was 
documented to have a T12 lesion on the spinal 
cord. He now presents with numbness on his 
trunk. 

On examination, there is reduced sensation to 
pin prick from T6 to T10. 

What is the most likely cause of this? 

1- Brown sequard syndrome 

2- Subacute combined degeneration of the 
cord 

3- Anterior spinal artery thrombosis 

4- Post traumatic syrinx 

5- Arteriovenous malformation 

Answer & Comments 

Answer: 4- Post traumatic syrinx 


elevated CSF pressure but the constituents of 
CSF are normal. 

In this particular patient , which of the 
Following would suggest that a complication 
has arisen? 

1- Bitemporal hemianopia 

2- Loss of colour vision 

3- 4th nerve palsy 

4- Visual loss 

5- Torticollis 

Answer & Comments 

Answer: 4- Visual loss 

The diagnosis is idiopathic intracranial 
hypertension (benign intracranial 

hypertesion). 

In this condition papilloedema may result in 
an enlarged blind spot. 


Post traumatic syringomyelia (PTS) refers to 
the development and progression of a cyst 
filled with cerebrospinal fluid (CSF) within the 
spinal cord. Other symptoms include 
increased weakness, numbness, increased 
spasticity, and hyperhidrosis (increased sw 
eating). 

Ascending sensory level and sensory 
dissociation (selective loss of pain and 
temperature sensation) are very sensitive 
indicators for detecting progressive PTS. MRI 
is the preferred initial imaging study for the 
diagnosis of PTS. 

[ Q: 553 ] MRCPass - Neurology 

A 42 year old woman presents with a 
history of headaches. She also reports 
menstrual irregularities. On examination, she 
was noted to be obese. Fundoscopy revealed 
bilateral papilloedema. Tone, power and 
reflexes in the limbs were normal. 

Investigations show a normal CT scan and 
MRI. EEG normal. Lumbar puncture reveals 



Uniocular visual loss would suggest infarction 
of the optic nerve consequent on severe and 
long standing papilloedema. 



[ Q: 554 ] MRCPass - Neurology 

A 70 year old man presents with 
sudden onset dysphagia and dysarthria, 
vomiting hiccup and vertigo. 


On examination he has a right sided Horner's 
syndrome, right-sided cerebellar ataxia, loss of 
pain and temperature sensation on the right 
hand side of the face and loss of pain and 
temperature sensation in the left upper and 
lower limbs. 


Where is the lesion? 


1- Pons 

2- Lateral medulla 

3- Cerebellum 

4- Midbrain 


5- Tectum 


Dr. Khalid Yusuf El-Zohry - Sohag Teaching Hospital (01118391123) 

Ref MRCPass OE OE 2012 PasTest 2009 PassMedicine 2009 PasTest Exam 




ReviseMRCP 

251 
























El-zohry MRCP Questions Bank (Port 1) - 2013 


(For my personal use) 


Answer & Comments 

Answer: 2- Lateral medulla 

The patient has the lateral medullary 
syndrome. 


2- Neurocysticercosis 

3- Multiple sclerosis 

4- Cerebral toxoplasmosis 

5- Tuberculosis 


Multiple areas are involved : 9th and 10th 
nerve - dysphagia and dysarthria Vestibular 
nuclei - vertigo Inferior cerebellar peduncle - 
ipsilateral cerebellar ataxia Descending 
autonomic fibres - Horner's syndrome Fifth 
nerve nucleus - loss of pain and temperature 
sensation over the face (ipsilateral) Lateral 
leminiscus - loss of pain and temperature 
sensation in the contralateral limbs 


Answer & Comments 

Answer: 2- Neurocysticercosis 

Neurocysticercosis is caused by Taenia solium 
(pork tapew orm). There may be seizures due 
to localised inflammation that accompanies 
their degeneration in the cerebral cortex 
when calcified cysts occur. This disease is 
found in South America and Asia. 


[ Q: 555 ] MRCPass - Neurology 

A patient presents with weakness of 
knee extension and ankle inversion. 

Which of the Following nerve roots could be 
damaged? 



Neurocysticercosis typically is benign, and 
most lesions resolve spontaneously within 2-3 
months. An enzymelinked immunotransfer 
blot (EITB) assay of a patient's serum may 
confirm the diagnosis. Albendazole is the 
recommended treatment. 


1- L2 

2- L3 

3- L4 

4- L5 

5- SI 


Answer & Comments 

Answer: 3- L4 

L4 is involved in knee extension and ankle 
inversion. The tibial nerve carries L4 & L5 
roots. 


[ Q: 556 ] MRCPass - Neurology 

A 25 year old man has come from 
Mexico 5 years ago. Since a year ago, he has 
had two tonic clonic seizures a week. 

On examination, he appears well, with no 
focal neurological deficit. A CT scan shows 
multiple calcified cystic lesions in the brain. 

Which diagnosis is likely? 

1- Neurosarcoid 




Neurocysticercosis 



[ Q: 557 ] MRCPass - Neurology 

A 42 year old lady presents with 
weakness, diplopia and fatigue. She has a past 
medical history of rheumatoid arthritis. 


On examination there was bilateral partial 
ptosis and weakness of abduction of both 
eyes. 


What is the likely diagnosis? 


1- Myasthenia gravis 


2- Mononeuritis multiplex 

3- Guillain Barre syndrome 



Dr. Khalid Yusuf El-Zohry - Sohag Teaching Hospital (01118391123) 

Ref MRCPass OE OE 2012 PasTest 2009 PassMedicine 2009 PasTest Exam ReviseMRCP 




























El-zohry MRCP Questions Bank (Port 1) - 2013 


(For my personal use) 


4- Paraneoplastic syndrome 

5- Lambert Eaton myasthenic syndrome 

Answer & Comments 

Answer: 1- Myasthenia gravis 

The most likely diagnosis is myasthenia gravis. 
There is an association between myasthenia 
gravis, pernicious anaemia, systemic lupus 
erythematosis and rheumatoid arthritis. The 
condition is more common in women with a 
peak incidence around the age of 30. It 
characterised by fatiguability of the proximal 
limb muscles, ocular and bulbar muscles. 
Reflexes are initially preserved but may be 
fatiguable. 


may, for example, cause sudden outbursts of 
unexpected aggression or agitation, or it may 
be characterized by aura-like phenomena. 

Complex partial seizures are characterized by 
impaired aw areness. They lose awareness 
and tend to have a motionless stare 
accompanied by automatisms -- stereotyped, 
repetitive, involuntary movements such as lip 
smacking, chew ing, picking at objects, 
scratching, and gesturing. 

In some cases, a series of old memories 
resurfaces. Hallucinations of voices, music, 
people, smells, or tastes may occur. These 
features are called "auras" or "warnings." 
They may last for just a few seconds, or may 
continue as long as a minute or two. 


[ Q: 558 ] MRCPass - Neurology 

A 30 year old man presents with a 6 
month history of recurrent episodes altered 
beaviour. During these episodes, he develops 
a motionless stare with associated lip 
smacking, grimacing, chewing movements, 
scratching or gesturing. His partner describes 
him as having labile emotions, he may get 
sudden outbursts of aggression or agitation. 

Which of the Following is the likely diagnosis? 

1- Transient global amnesia 

2- Frontal lobe epilepsy 

3- Temporal lobe epilepsy 

4- Parietal lobe lesion 

5- Migraine 



Answer & Comments 

Answer: 3- Temporal lobe epilepsy 

The features of seizures beginning in the 
temporal lobe can be extremely varied, but 
certain patterns are common. In temporal 
lobe epilepsy, there may be a mixture of 
different feelings, emotions, thoughts, and 
experiences, which may be familiar or 
completely foreign. Temporal lobe epilepsy 


Carbamazepine and phenytoin are used to 
treat the condition. 


[ Q: 559 ] MRCPass - Neurology 

A 55 year old drug user who is 
homeless presents with lethargy. General 
examination reveals poor hygiene and a sacral 
sore. He has bilateral ptosis, and difficulty 
swallowing. There is also general weakness in 
all limbs. His lethargy is worse in the evenings. 

Which of the Following is the likely diagnosis? 

1- Myasthenia gravis 

2- Lambert eaton myasthenic syndrome 

3- Botulism 

4- Motor neuron disease 

5- HIV neuropathy 



Answer & Comments 

Answer: 3- Botulism 

The main differentials are myasthenia gravis, 
LEMS and botulism, but in this context 
botulism is more likely. Botulism is a paralytic 
disease caused by the neurotoxins of 
Clostridium botulinum. Wound botulism, 
caused by systemic spread of toxin produced 


Dr. Khalid Yusuf El-Zohry - Sohag Teaching Hospital (01118391123) 

Ref MRCPass OE OE 2012 PasTest 2009 PassMedicine 2009 PasTest Exam ReviseMRCP 



253 



























El-zohry MRCP Questions Bank (Port 1) - 2013 


(For my personal use) 


by organisms inhabiting wounds, trauma, 
surgery and subcutaneous heroin injection. 

The neurologic symptomatology often has 
been described as a progressive, descending 
weakness or paralysis that affects muscles 
innervated by the cranial nerves. Respiratory 
difficulty arises from airway obstruction and 
diaphragmatic weakness. Diplopia, dysarthria, 
dry mouth, and generalized weakness are 
among the most common presenting 
symptoms. 


hearing loss and vertigo. On examination, he 
has an absent corneal reflex on the left. There 
is also mild left sided facial weakness. 

Which one of the Following is most likely? 

1- Frontal lobe tumour 

2- Cavernous sinus thrombosis 

3- Ramsay Hunt syndrome 

4- Cerebellopontine angle tumour 

5- Syringomyelia 



[ Q: 560 ] MRCPass - Neurology 

A 50 year old teacher develops a 
sided facial weakness in association with 
hearing loss and pain in the right ear. 


On examination, there is a vesicular rash over 
the right ear, and right lower motor neuron 
7th nerve palsy. 


Whot is the likely couse? 


1- Lyme disease 


2- Herpes zoster 

3- Diabetes 


4- Polyarteritis nodosa 

5- Syphilis 


Answer & Comments 

Answer: 2- Herpes zoster 

This is Ramsay-Hunt syndrome. The primary 
pathophysiology of Ramsay Hunt syndrome is 
located in the geniculate ganglion of the 
seventh cranial nerve (CN VII). 

Classically, Ramsay Hunt syndrome has been 
associated with VZV. It is associated with 7th 
nerve palsy, vertigo, ipsilateral hearing loss 
and tinnitus. 


Answer & Comments 

Answer: 4- Cerebellopontine angle tumour 

Cerebellopontine angle (CPA) tumors can 
cause vertigo, unilateral hearing loss. Large 
tumors may cause subtle facial weakness, 
decreased corneal reflex, and facial 
dysesthesia. If there was opthalmoplegia or 
chemosis, then a cavernous sinus thrombosis 
would be more likely (it can also cause absent 
corneal reflexes). 


[ Q: 562 ] MRCPass - Neurology 

A 60 year old woman with headache 
and nausea is suspected of having posterior 
cerebral artery thrombosis. 

Which of the Following is o recognised feature 
of this occurrence? 

1- Cerebellar ataxia 

2- Hemiparesis 

3- Homonymous hemianopia 

4- Third nerve palsy 

5- Sixth nerve palsy 



Answer & Comments 


Answer: 3- Homonymous hemianopia 



[ Q: 561 ] MRCPass - Neurology 
A 55 year old man has left sided 


The posterior cerebral artery supplies the 
occipital lobe and occlusion causes damage to 
the visual cortex, resulting in homonymous 
hemianopia. 


Dr. Kholid Yusuf El-Zohry - Sohog Teaching Hospital (01118391123) 

Ref MRCPass OE OE 2012 PasTest 2009 PassMedicine 2009 PasTest Exam ReviseMRCP 































El-zohry MRCP Questions Bank (Port 1) - 2013 


(For my personal use) 



Posterior Cerebral Artery Infarct 



[ Q: 563 ] MRCPass - Neurology 

A 35 year old woman wakes up with 
a sudden onset severe sharp headache. She 
has no neurological signs. CT of her head is 
normal. 


Whot is the best next investigation? 

1- MRV 


2- Lumbar puncture 

3- Serum electrophoresis 

4- EEG 


5- Bone scan 


Answer & Comments 

Answer: 2- Lumbar puncture 

The diagnosis of subarachnoid haemorrhage 
needs to be exclude in a patient with acute 
sudden onset severe headache. The CSF 
sample should be sent for xanthochromia. 
Lumbar puncture is recommended 12 hours 
after the event to allow xanthochromia to 
develop. It is a yellow ish pigment (subtle and 
needs spectrophotometry) which indicates the 
presence of bilirubin in the CSF. 


[ Q: 564 ] MRCPass - Neurology 

A 30 year old man has a history of 
epilepsy. He was found on the street having a 
seizure and was brought to A+E. Rectal 



diazepam had been given by the ambulance 
crew. His seizure lasts more than 15 minutes. 

What is the best management step? 

1- Intravenous lorazepam 

2- Intravenous phenytoin 

3- Oral carbamazepine 

4- Intravenous phenobarbitone 

5- CT scan of the head 


Answer & Comments 

Answer: 2- Intravenous phenytoin 

This patient has status epilepticus. He has not 
responded to a benzodiazepine and hence the 
next step is to load with intravenous 
phenytoin at a dose of 15mg/kg. 

[ Q: 565 ] MRCPass - Neurology 

A 60 year old man is a vegetarian 
and presents with lethargy. He has frequent 
diarrhoea and mentions that he eats mostly 
maize. 

On examination, he has an erythematous rash 
across his face and chest. He is confused and 
disorientated. On examination, he has an 
MMSE score of 21 /30. Tone and reflexes are 
normal but he weak throughout the body. 

Which vitamin is his diet likely to be deficient? 

1- Thiamine 

2- B12 

3- Niacin 

4- Vitamin C 

5- Vitamin A 



Answer & Comments 

Answer: 3- Niacin 

The diagnosis is pellagra. There is a triad of 
dementia, diarrhoea and dermatitis. Niacin 
(nicotinamide or nicotinic acid) deficiency 
causes pellagra only if tryptophan, an amino 


Dr. Khalid Yusuf El-Zohry - Sohag Teaching Hospital (01118391123) 

Ref MRCPass OE OE 2012 PasTest 2009 PassMedicine 2009 PasTest Exam ReviseMRCP 



























El-zohry MRCP Questions Bank (Port 1) - 2013 


(For my personal use) 


acid, is also deficient. People who live in areas 
where maize (Indian corn) is the main food 
source are at risk of developing pellagra 
because maize is low in niacin and tryptophan. 

Pellagra affects the skin, digestive tract, and 
brain. A photosensitive rash may occur. Skin 
abnormalities are persistent, and the affected 
areas may become brown and scaly. 

The whole digestive tract is affected. Other 
symptoms include nausea, vomiting, 
constipation, and diarrhea. Later, fatigue, 
insomnia, and apathy develop. 
Encephalopathy usually follows. It is 
characterized by confusion, disorientation, 
hallucinations, and memory loss. 



Photosensitive rash seen in Pellagra 


[ Q: 566 ] MRCPass - Neurology 

A 45 year old woman has sensory 
loss and wasting of the small hand muscles. 
One examination, she also has a right sided 
Horner's syndrome. Her arms demonstrate 
thickening of the subcutaneous tissues. There 
is also evidence of Charcot's joints on the 
wrists. 

What is the diagnosis? 

1- Hereditary spinocerebellar ataxia 

2- Hereditary spastic paraparesis 

3- Motor neuron disease 

4- Syringomyelia 

5- Multiple sclerosis 



Answer & Comments 

Answer: 4- Syringomyelia 

Syringomyelia is chronic disorder 
characterised by the presence of glial-lined 
cavities situated in the central part of the 
spinal cord. 

Recognised causes include Chiari type I 
malformation, central cord tumours, basal 
arachnoiditis and trauma. 

There may be sensory loss, wasting of the 
small hand muscles, uni- or bilateral Horner's 
syndrome, abnormalities of sw eating, 
thickening of subcutaneous tissues, atrophy 
and decalcification of bones, development of 
Charcot's joints and Chiari I malformation (due 
to arachnoiditis). 


[ Q: 567 ] MRCPass - Neurology 

A 27 year old lady presents with a 
severe headache, which woke her up from 
sleep. She is not pregnant and was on no 
drugs in particular she was not on hormonal 
contraception. On examination she was 
afebrile and alert. On examination of the optic 
fundus the cup of the optic disc was filled and 
the medial margins of the disc were blurred. 
There was no other CNS abnormality, in 
particular no neck stiffness. A CT scan was as 
normal. 

What is the likely diagnosis? 

1- Meningioma 

2- Sagittal sinus thrombosis 

3- Benign intracranial hypertension 

4- Migraine 

5- Meningitis 



Answer & Comments 

Answer: 2- Sagittal sinus thrombosis 

This is the most likely cause, despite not being 
pregnant or on the OCP. In only 10% of cases 
is cerebral venous thrombosis due to damage 


Dr. Khalid Yusuf El-Zohry - Sohag Teaching Hospital (01118391123) 

Ref MRCPass OE OE 2012 PasTest 2009 PassMedicine 2009 PasTest Exam ReviseMRCP 



256 

























El-zohry MRCP Questions Bank (Port 1) - 2013 


(For my personal use) 


to the vessel wall by infection, tumour or 
trauma. 

Commonest causes are inherited disorders of 
coagulation of which factor V Leiden mutation 
is found in around 20% of cases. Often there is 
combination of causative factors: e.g. protein 
S deficiency and child birth, pregnancy and 
Behcet's disease, OCP and factor V Leiden 
mutation. 


[ Q: 568 ] MRCPass - Neurology 

A 25 year old female patient 
presents with a 5-day history of ascending 
muscle weakness in both hands and feet. 
EMGs confirm acute demyelinating sensory 
and motor neuropathy. 

Whot treatment should be started? 

1- Phenytoin 

2- Diazepam 

3- Amitriptyline 

4- Intravenous immunoglobulin 

5- Pyridostigmine 



Answer & Comments 

Answer: 4- Intravenous immunoglobulin 

The diagnosis is Guillain Barre syndrome. It is 
preceded by diarrhea (e.g. Campylobacter) 
and chest infections (e.g. mycoplasma) in two 
thirds of cases. Autonomic dysfunction and 
hyporeflexia are associated. Studies have 
shown that plasma exchange and IVIg are 
equally effective in treating people within two 
to four weeks of onset of GBS. 



[ Q: 569 ] MRCPass - Neurology 

A 32 year old man has difficulty with 
his vision. On examination he has impaired 
adduction of the right eye looking left. The left 
eye has jerky nystagmus. 


Which investigation is most likely to yield a 
diagnosis? 


1- Nerve conduction studies 

2- CT of the head 

3- Paired CSF and serum for oligoclonal bands 

4- Serum copper and caeruloplasmin 

5- Visual evoked potentials 


Answer & Comments 

Answer: 3- Paired CSF and serum for 
oligoclonal bands 

The clinical scenario is internuclear 
ophthalmoplegia. 

This is most commonly seen in multiple 
sclerosis. MRI of the brain and CSF are the 
best diagnostic tests. In this scenario the 
lesion is in the right medial longitudinal 
fasciculus. 



Internuclear opthalmoplegia - patient looking 

to the left 


^ [ Q: 570 ] MRCPass - Neurology 

f* - 

# A patient has, on examination, 
weakness in plantar flexion and foot inversion 
on the left. He also is unable to tiptoe on the 
same foot. Ankle jerk is absent. 

Which nerve lesion is most likely? 

1- Common peroneal nerve 

2- L4 nerve root 

3- Tibial nerve 

4- Sciatic nerve 

5- Femoral nerve 


Answer & Comments 


Answer: 3- Tibial nerve 


Dr. Khalid Yusuf El-Zohry - Sohag Teaching Hospital (01118391123) 

Ref MRCPass OE OE 2012 PasTest 2009 PassMedicine 2009 PasTest Exam 




ReviseMRCP 

2S7 



























El-zohry MRCP Questions Bank (Port 1) - 2013 


(For my personal use) 


The tibial nerve supplies the gastrocnemius 
muscle and leads to the above findings. The 
common peroneal nerve causes weakness of 
eversion and dorsiflexion. 

[ Q: 571 ] MRCPass - Neurology 

A 50 year old man has subjectively 
diminished light touch and pinprick sensation 
in the left hand extending to above the elbow. 
Joint position sense is intact. He has difficulty 
distinguishing a cigarette from a pen using the 
right hand with his eyes closed. His two-point 
discrimination is 11 mm. 

Where is the lesion? 

1- Anterior frontal 

2- Posterior frontal 

3- Anterior parietal 

4- Posterior parietal 

5- Cingulate gyrus 



Answer & Comments 

Answer: 3- Anterior parietal 

The sensory signs described here are 
indicative of a lesion of the anterior parietal 
cortex (mid postcentral gyrus). The cingulate 
gyrus is part of the limbic system (mood and 
emotions). 


[ Q: 572 ] MRCPass - Neurology 

A 55 year old man has been 
progressively getting more confused. His 
symptoms began about three years ago when 
he noticed leg stiffness. At present his entire 
body feels stiff and he has a resting tremor in 
the limbs. His w riting has become small. Over 
the last six months, he has developed 
hallucinations and reports seeing ghost 
figures, and is also becoming forgetful, on one 
occasion leaving the gas cooker fire on. 

On examination, his mini-mental score was 
10/30. He has increased tone throughout 
which is spastic and cogw heeling. There is a 



tremor in all limbs. Cranial nerve examination 
reveals mild restriction of conjugate upgaze 
eye movement. His gait was shuffling with a 
tendency to fall backwards. 

Whot is the most likely diagnosis? 

1- Parkinson's disease 

2- Pick's disease 

3- Lewy body dementia 

4- Huntington's disease 

5- Motor neuron disease 


Answer & Comments 

Answer: 3- Lewy body dementia 

The combination of progressive cognitive 
decline, fluctuating symptoms, visual 
hallucinations, extrapyramidal signs (rigidity 
and bradykinesia more prominent than 
tremor) suggest Lewy body dementia. It is 
progressive. Patients are at risk of falls and 
syncope. Symptoms and signs of lew y body 
dementia probably result in part from 
disruption of information flow from the 
striatum to the neocortex, especially the 
frontal 

lobe. 

The cause is multifactorial. Altered 
neuromodulator and/or neurotransmitter 
levels (eg, acetylcholine, dopamine) influence 
the function of many neuronal circuits. 

^ [ Q: 573 ] MRCPass - Neurology 

# A 35 year old patient presented to 
the hospital with generalised unsteadiness 
and limb weakness. There was a history of 
ascending weakness, beginning five days 
before admission. On examination, she had 
distal weakness with decreased reflexes in the 
lower limbs. 

She had a lumbar puncture. The results of the 
CSF showed 7 x 10 9 /L lymphocytes, no 
erythrocytes, and 1.2 g/l protein. EMGs 
showed reduction of mean conduction 


Dr. Kholid Yusuf El-Zohry - Sohog Teaching Hospital (01118391123) 

Ref MRCPass OE OE 2012 PasTest 2009 PassMedicine 2009 PasTest Exam ReviseMRCP 



258 


























El-zohry MRCP Questions Bank (Port 1) - 2013 


(For my personal use) 


velocity and prolonged distal latency (DL) 
were observed in the median, ulnar, and tibial 
nerves. 

Whot is the diagnosis? 

1- Multiple sclerosis 

2- Viral meningitis 

3- Syphilis 

4- Trigeminal neuralgia 

5- Guillain Barre syndrome 


Answer & Comments 

Answer: 5- Guillain Barre syndrome 

Conditions such as viral meningitis and 
multiple sclerosis cause mild protein elevation 
(above 0.5g). However, Guillain Barre 
syndrome causes marked protein elevation, 
often approaching or more than lg.M 

In Guillain Barre syndrome, there is acute 
demyelination of the nerves, leading to 
reduced conduction velocities on the EMGs. 


[ Q: 574 ] MRCPass - Neurology 

A 43 year old man presents with 
frequent headaches and loss of libido. He was 
found to have hypopituitarism on 
investigation. The CT scan shows a pituitary 
tumour with suprasellar extension. 

Which of the Following structures is likely be 
compressed? 

1- Abducens nerve 

2- Hypothalamus 

3- Trochlear nerve 

4- Optic chiasm 

5- 3rd Ventricle 



Answer & Comments 


Answer: 4- Optic chiasm 


Superior extension (suprasellar) of a pituitary 
tumour can lead to compression and invasion 
of the optic chiasm and nerve. 



[ Q: 575 ] MRCPass - Neurology 

A 32 year old woman has known 
migraine. She gets periodic episodes of 
headaches with associated visual symptoms. 


Which one of the Following drugs should be 
used first in a migraine attack? 


1- Ibuprofen 

2- Methysergide 

3- Subcutaneous sumatriptan 


4- Oral sumatriptan 

5- Morphine 


Answer & Comments 

Answer: 1- Ibuprofen 

In acute migraine attack, the first line 
treatments are simple analgesics such as 
aspirin, ibuprofen or paracetamol. 

Second line treatment in acute migraine are 
the triptans (e.g sumatriptan) which work by 
selectively stimulating 5-hydroxytriptamine 1 
(5HT1) receptors. 

[ Q: 576 ] MRCPass - Neurology 

A 30-year-old woman has an 18 
month history of unsteady gait, difficulty to 
speak and to perform fine movements with 
the fingers. She has a history of moderate 
alcohol intake. 

Neurological examination showed scanning 
dysarthria, horizontal nystagmus in the lateral 
gaze, severe bilateral dysmetria in the upper 
and lower limbs, bilateral dysdiadochokinesis, 
severe gait ataxia and inability to maintain the 
sitting. 

What is the diagnosis? 

1- Parkinson's disease 



Dr. Khalid Yusuf El-Zohry - Sohag Teaching Hospital (01118391123) 

Ref MRCPass OE OE 2012 PasTest 2009 PassMedicine 2009 PasTest Exam ReviseMRCP 



259 































El-zohry MRCP Questions Bank (Port 1) - 2013 


(For my personal use) 


2- Subacute combined degeneration 

3- Cerebellar syndrome 

4- Motor neuron disease 

5- Myasthenia gravis 


Answer & Comments 

Answer: 3- Cerebellar syndrome 

A mnemonic for cerebellar signs is VANISH'D - 
Vertigo, Ataxia, Nystagmus, Intention tremor, 
Scanning speech, Hypotonia and 
Dysdiadochokinesis. 

Cerebellar syndromes are commonly due to 
alcohol, cerebellar space occupying lesions, 
multiple sclerosis, and rarely, inherited 
sydromes such as Friedrich's Ataxia or 
Spinocerebellar ataxia. 

[ Q: 577 ] MRCPass - Neurology 

A 30 year old man presents with a 5 
day history of weakness in the arms and legs, 
accompanied by tingling. He had several 
episodes of bloody diarrhoea illness two 
weeks ago. 

On examination there was leg and arm 
weakness and flaccid deep tendon reflexes. 

Which test would best help confirm the 
diagnosis? 

1- EMG 

2- EEG 

3- MRI 

4- HIV serology 

5- Anti Ach antibody 



Answer & Comments 

Answer: 1- EMG 

The diagnosis is likely to be Guillain Barre 
syndrome. The two best tests are EMG (shows 
acute demyelination changes) and CSF (raised 
protein). Campylobacter serology should also 
be sent for this patient. 


[ Q: 578 ] MRCPass - Neurology 

A 70 year old man who had multiple 
episodes of unilateral amaurosis fugax. He has 
risk factors of being a smoker and 
hypertension. He is on atenolol and aspirin. 

An ECG shows atrial fibrillation. Carotid 
dopplers show 80% right carotid stenosis. 

What is the most appropriate management? 

1- Warfarin 

2- Carotidendarterectomy, then warfarinise 

3- Clopidogrel 

4- High dose aspirin 300mg with a proton 
pump inhibitor 

5- Tight hypertensive control 



Answer & Comments 

Answer: 2- Carotidendarterectomy, then 
warfarinise 

As there is > 70% carotid artery stenosis and 
symptoms suggestive of emboli, 
endarterectomy is recommended. The patient 
should also be anticoagulated afterwards in 
view of atrial fibrillation. 



[ Q: 579 ] MRCPass - Neurology 

A 50 year old man has a history of 
hypertension and is a smoker. He complains of 
visual loss. Assessment shows the presence of 
a right homonymous hemianopia. 


Which structure is damaged? 

1- Optic chiasm 


2- Optic radiation 

3- Left occipital lobe 


4- Right occipital lobe 

5- Temporal lobe 


Answer & Comments 


Answer: 3- Left occipital lobe 


Dr. Khalid Yusuf El-Zohry - Sohag Teaching Hospital (01118391123) 

Ref MRCPass OE OE 2012 PasTest 2009 PassMedicine 2009 PasTest Exam ReviseMRCP 

































El-zohry MRCP Questions Bank (Port 1) - 2013 


(For my personal use) 


In homonymous hemianopia, the contralateral 
occipital lobe is affected (usually infarct). 



Left occipital infarct on MRI 


[ Q: 580 ] MRCPass - Neurology 

A 66 year old man has had 
longstanding tremors in both his hands and 
forearms. Examination reveals normal tone, 
power and reflexes in his arms. The tremors 
improve when he drinks alcohol. 

Whot is the diagnosis? 

1- Parkinson's disease 

2- Motor neuron disease 

3- Benign essential tremor 

4- Prion infection 

5- Hemiballismus 



Answer & Comments 

Answer: 3- Benign essential tremor 

The Following features support a diagnosis of 
Essential Tremor: 

(1) bilateral action tremor of the hands and 
forearms 

(2) absence of other neurological signs, except 
the cogw heel phenomenon 


(3) may have isolated head tremor with no 
signs of dystonia 

Secondary criteria include a long disease 
duration (more than three years), a positive 
family history and beneficial response to 
alcohol (not anticholinergics). 


^ [ Q: 581 ] MRCPass - Neurology 

# A 45 year old man has difficulty 
getting out of the chair. 

On examination, he has proximal muscle 
weakness. 

Investigations show : 

Hb 12.5 g/dl 

MCV 79 fl 

WCC 7 x 10 9 /L 

platelets 220 x 10 9 /L 

urea 6 mmol/l 

creatinine 110 pmol/l 

Creatine Kinase 7,000 (24-170) U/l 

What investigation should be done next? 

1- Lumbar puncture 

2- CT brain 

3- MRI brain 

4- Muscle biopsy 

5- Tensilon test 


Answer & Comments 

Answer: 4- Muscle biopsy 

The clinical features are consistent with 
dermatomyositis or polymyositis. 

A suitable area should be identified by 
electromyography for muscle biopsy. Muscle 
biopsy shows muscle necrosis, phagocytosis of 
muscle fibres, and an inflammatory infiltrate. 


Dr. Khalid Yusuf El-Zohry - Sohag Teaching Hospital (01118391123) 

Ref MRCPass OE OE 2012 PasTest 2009 PassMedicine 2009 PasTest Exam 




ReviseMRCP 


261 


























El-zohry MRCP Questions Bank (Part 1) - 2013 


(For my personal use) 



Polymyositis - Inflammatory infiltrates in a 

muscle biopsy 


[ Q: 582 ] MRCPass - Neurology 

A 50 year old lady complains of 
gradual onset of blurred vision in her left eye. 
Examination reveals a left sided relative 
afferent pupillary defect. Fundoscopy reveals 
left sided optic atrophy. Visual fields show a 
left sided central scotoma and an upper 
quadrantic visual field defect in the right eye. 

Where is the lesion? 

1- Left optic nerve 

2- Left anterior optic chiasm 

3- Sphenoid wing 

4- Left occipital area 

5- Left optic radiation 



Answer & Comments 

Answer: 2- Left anterior optic chiasm 

Lesions of the anterior chiasm (junction of 
optic nerve and chiasm) will produce an 
ipsilateral central scotoma and a contralateral 
superior quadrantanopia. The contralateral 
defect is due to interruption of the crossing 
nasal fibres. 



Left v Sufll I ifrld 

\ 


Left eye 


Oplic 

chiasm 


Ekain 


/ 


R icjhi visual Field 


Right aye 


Optic 

rwrve 


Optic Tract 


[ Q: 583 ] MRCPass - Neurology 

A 40 year old patient has been 
having fevers, neck stiffness and confusion. 
There was a past medical history of HIV 
infection and diabetes. 

A CT scan was normal and lumbar puncture 
was performed. The opening pressure during 
LP was normal, and there were elevated CSF 
lymphocytes (88), and elevated CSF protein 
(5.5 g/l), and a low glucose. Gram staining and 
India ink preparation revealed 4-7?m, round 
budding yeasts with capsule and 8-10 
lymphocytes per high power field. 

What is the diagnosis? 

1- Mumps meningitis 

2- Mycobacterium tuberculosis 

3- Cryptococcal meningitis 

4- Carcinomatous meningitis 

5- Meningococcal meningitis 



Answer & Comments 

Answer: 3- Cryptococcal meningitis 

Cryptococcal meningitis is often seen in 
immunosuppressed patients. The organism is 



Dr. Khalid Yusuf El-Zohry - Sohag Teaching Hospital (01118391123) 

Ref MRCPass OE OE 2012 PasTest 2009 PassMedicine 2009 PasTest Exam ReviseMRCP 




























El-zohry MRCP Questions Bank (Port 1) - 2013 


(For my personal use) 


Cryptococcus Neoformans. Meningitis 
manifests with diffuse, nonfocal findings (eg, 
altered mental status, vomiting). 

A CT scan or MRI in patients with cryptococcal 
infection may reveal diffuse atrophy or 
cerebral edema with focal, homogenous, or 
contrast-enhanced areas. 

An India ink preparation is commonly used 
with CSF to identify the organism and to 
support a presumptive diagnosis. If performed 
correctly, 25-50% of patients with 
cryptococcal meningitis show cryptococci. 

In patients with AIDS, amphotericin B is given 
for 2 weeks, with or without 2 weeks of 
flucytosine, followed by fluconazole at 400 
mg/d for a minimum of 10 weeks. 


[ Q: 584 ] MRCPass - Neurology 

A young lady visits her neurologist 
complaining of episodes of generalised 
weakness after arguments with her partner. 
She also complains of seeing goblins upon 
waking up. At work as a secretary, she has 
difficulty staying awake and may have sleep 
attacks. 

Whot is the likely diagnosis? 

1- Narcolepsy 

2- Generalised epilepsy 

3- Petit mal seizures 

4- Obstructive sleep apnoea 

5- Jacksonian seizures 



Answer & Comments 

Answer: 1- Narcolepsy 

The condition described is narcolepsy. The 
episodes described are likely to be cataplexy. 

The HLA association is DQB1, Clomipramine is 
a tricyclic antidepressant which may help, 
there is early REM sleep and hypnagogic 
hallucinations occur. 



[ Q: 585 ] MRCPass - Neurology 

A 68 year old woman has neck pains 
and occipital headaches for 2 years. She is 
referred to the neurology outpatients for 
assessment. Investigations showed a normal 
CT of the brain. Cervical X ray showed 
degenerative changes of narrow ed disc 
spaces and loss of cervical lordosis. 


Whot is the likely diagnosis? 


1- Cerebellar haemorrhage 

2- Temporal arteritis 

3- Occipital neuralgia 


4- Epidural haemorrhage 

5- Cervical spondylosis 


Answer & Comments 

Answer: 5- Cervical spondylosis 

In cervical spondylosis, several overlapping 
syndromes are seen: neck and shoulder pain, 
suboccipital pain and headache, radicular 
symptoms, and cervical spondylotic 
myelopathy. 

Examination findings include neck pain, 
radicular signs, and myelopathic signs. Cervical 
spine films can demonstrate disk space 
narrow ing, osteophytosis, loss of cervical 
lordosis, uncovertebral joint hypertrophy, 
apophyseal joint osteoarthritis, and vertebral 
canal diameter. 


f 


[ Q: 586 ] MRCPass - Neurology 

r - 

* A 45 year old woman is referred to 
you for investigation of headache. She has had 
headaches for 10 years. 


Initially it responded to proprietary painkillers, 
but she is currently using the maximum dose 
of paracetamol, tramadol and diclofenac. The 
headaches are frequent throughout the day 
and last for hours. There are no associated 
visual symptoms. 


Which is the next best management step? 


Dr. Khalid Yusuf El-Zohry - Sohag Teaching Hospital (01118391123) 

Ref MRCPass OE OE 2012 PasTest 2009 PassMedicine 2009 PasTest Exam 




ReviseMRCP 

263 



























El-zohry MRCP Questions Bank (Port 1) - 2013 


(For my personal use) 


1- Iv aspirin 

2- Caffeine 

3- Withdrawal of analgesics 

4- Pizotifen 

5- Sumatriptan 


Answer & Comments 

Answer: 3- Withdrawal of analgesics 

The history of chronic use of analgesics and 
nature of headaches suggests analgesic 
induced headache. In some patients the 
headaches will improve. 


[ Q: 587 ] MRCPass - Neurology 

A patient is undergoing examination 
of the eye. The patient has a direct response 
to light shone in the right eye, but no 
consensual response. Light shone in the left 
eye elicits a consensual response, but not a 
direct response. 

During pursuit eye movements, the left eye is 
fixed in an inferior and lateral position. 

Where is the lesion? 

1- Right trochlear nerve 

2- Left optic nerve 

3- Left oculomotor nerve 

4- Right abducents nerve 

5- Left trochlear nerve 



Answer & Comments 

Answer: 3- Left oculomotor nerve 

A left third nerve palsy will cause a dilated left 
pupil, with the eye in a 'down and out' 
position. The afferent pathway is controlled by 
the optic nerve and the efferent pathway by 
the oculomotor nerve - hence a dilated poorly 
reacting pupil. 



Left Third Nerve Palsy 


[ Q: 588 ] MRCPass - Neurology 

A 18 year old male is wheelchair 
bound and has difficulty with respiration. He 
also has upper limb weakness. 



When he was younger he developed marked 
hypertrophy of his muscles. Blood tests reveal 
a raised creatine kinase. 


What is a muscle biopsy likely to show? 

1- Necrotic muscle fibres 

2- Absence of dystrophin 

3- Excessive lipid storage 

4- Macrophage infiltration 

5- Vasculitic changes 


Answer & Comments 

Answer: 2- Absence of dystrophin 

Mutation in the dystrophin gene causes 
deficiency of dystrophin in Duchenne's 
muscular dystrophy. Patients develop 
progressive upper and lower limb weakness 
with pseudohypertrophy of calves and 
quadriceps. 


[ Q: 589 ] MRCPass - Neurology 

A 30 year old lady is found to have a 
left sided posterior communicating artery 
aneurysm on cerebral angiography. 

Which of the Following would you expect to 
find? 

1- Facial nerve palsy 

2- Left pupillary constriction 

3- Sensory loss to the left side of the face 



Dr. Kholid Yusuf El-Zohry - Sohog Teaching Hospital (01118391123) 

Ref MRCPass OE OE 2012 PasTest 2009 PassMedicine 2009 PasTest Exam ReviseMRCP 



264 





























El-zohry MRCP Questions Bank (Part 1) - 2013 


(For my personal use) 


4- Downgaze palsy 

5- Third nerve palsy 

Answer & Comments 

Answer: 5- Third nerve palsy 

A posterior communicating artery aneurysm 
will cause compression of the third nerve, and 
therefore pupillary involvement from 
compression of the parasympathetic fibres 
that run on the outside of the third nerve. This 
leads to a dilated pupil. Other features of a 
third nerve palsy include ptosis, and a 'down 
and out' eye. Upgaze and adduction is 
affected. 


on MRI in association with white matter 
changes, the most likely cause would be a 
demyelinating lesion. 

Large lesions such as these can cause 
weakness or cranial nerve defects. 



[ Q: 590 ] MRCPass - Neurology 

A 35 year old teacher has a right 
sided headache and blurring of her vision in 
the right eye. She has previously had an 
episode of optic neuritis 3 years beforehand, 
in the right eye. On examination, there was a 
right afferent pupillary defect and pale optic 
disc. There was weakness of the facial muscles 
on the right. Tone and reflexes were brisk on 
the right with power of 3/5 in the arm and the 
leg. She was apyrexial. 

MRI of the brain reveals a 4 cm left temporo¬ 
parietal mass, which was incompletely ring 
enhancing. There were also two small white 
matter lesions also visible in the frontal area. 

What is the most likely diagnosis? 

1- Cerebral lymphoma 

2- Multiple sclerosis 

3- Acute demyelinating encephalomyelitis 

4- Lyme disease 

5- Sarcoidosis 



White matter lesions on MRI seen in multiple 

sclerosis 



[ Q: 591 ] MRCPass - Neurology 

A 75 year old man has dysphasia and 
left sided arm weakness. He is known to have 
hypertension, asthma and rheumatoid 
arthritis. In addition he suffered from cluster 
headaches. He smokes 20 cigarettes a day. 


On examination, he has some weakness of the 
left hand. Visual fields, speech and sensation 
are normal. Blood pressure is 190/90 mmHg. 


Which of the Following is most likely? 

1- Partial posterior circulation infarct 


2- Right hemisphere lacunar infarct 

3- Right pontine haemorrhage 

4- Total anterior circulation infarct 


5- Vertebrobasilar insufficiency 


Answer & Comments 


Answer: 2- Right hemisphere lacunar infarct 


Answer & Comments 

Answer: 2- Multiple sclerosis 

The episodes of optic neuritis are suggestive 
of MS. In view of the appearance of the lesion 


The history suggests the dysarthria-clumsy 
hand syndrome, one of the classic lacunar 
syndromes that are strokes in the subcortical 
regions (or brain stem) secondary to small 
vessel disease. The usual site of damage in the 


Dr. Khalid Yusuf El-Zohry - Sohag Teaching Hospital (01118391123) 

Ref MRCPass OE OE 2012 PasTest 2009 PassMedicine 2009 PasTest Exam ReviseMRCP 



265 


























El-zohry MRCP Questions Bank (Port 1) - 2013 


(For my personal use) 


dysarthria-clumsy hand syndrome is the 
internal capsule or pons. 


Answer & Comments 


Answer: 5- IV lorazepam 


^ [ Q: 592 ] MRCPass - Neurology 

n - 

# A 65 year old lady presents to the 
hospital with an ataxic gait. On examination, 
the patient had difficulty standing without 
assistance and minor movements of her head 
and body caused vertigo. There was mild 
postural tremor and marked hypotonia of the 
right limbs, mostly the arm. 

Finger to nose testing elicited marked 
intention tremor and disdiadochokinesis in the 
right arm. Speech was dysarthric but 
comprehension was good. 

Where is the lesion likely to be? 

1- Right pons 

2- Left medulla 

3- Right cerebellar hemisphere 

4- Corpus callosum 

5- Right basal ganglia 


Current consensus is that a benzodiazepine, 
notably lorazepam (Ativan), is the initial class 
of drug for the treatment of status epilepticus. 
A phenytoin, phenytoin sodium or 
fosphenytoin is the next drug to be 
administered. 

^ [ Q: 594 ] MRCPass - Neurology 

# A 28 year old woman is 30 weeks 
pregnant. She complains of a sudden onset 
generalised headache. 

On examination, she has pupils are which 
reactive bilaterally and there is a right sided 
third nerve palsy. 

Which test is most appropriate? 

1- MRA 

2- MRV 

3- MRI 

4- CT head 


Answer & Comments 

Answer: 3- Right cerebellar hemisphere 

Lesions of the cerebellum (intention tremor, 
disdiadochokinesis) lead to motor signs 
ipsilateral to the lesion. 



[ Q: 593 ] MRCPass - Neurology 

A 25 year old man is known to have 
epilepsy. He had a generalized tonic clonic 
seizures for 15 minutes. 

What drug should be given? 

1- IV phenytoin 

2- IV sodium valproate 

3- IV gabapentin 

4- IV lamotrigine 

5- IV lorazepam 


5- Lumbar puncture 


Answer & Comments 

Answer: 2- MRV 

A headache in a pregnant patient, with 
associated cranial nerve palsy suggests 
cerebral venous sinus thrombosis. Treatment 
is with intravenous or low molecular weight 
heparin. 


[ Q: 595 ] MRCPass - Neurology 

A 45 year old man has severe 
episodes of dizziness with associated vomiting 
and pain in the right ear. This occurs once or 
twice a week. During these attacks he feels 
the surrounding environment spinning 
around. He also mentions a high pitched 
sound frequently being present. 



Dr. Khalid Yusuf El-Zohry - Sohag Teaching Hospital (01118391123) 

Ref MRCPass OE OE 2012 PasTest 2009 PassMedicine 2009 PasTest Exam ReviseMRCP 



266 
































El-zohry MRCP Questions Bank (Port 1) - 2013 


(For my personal use) 


On examination, during an attack, he has right 
horizontal nystagmus. Audiological testing 
reveals right-sided sensorineural deafness. 

Whot is the diagnosis? 

1- Vestibular nystagmus 

2- Meniere's disease 

3- Benign paroxysmal positional vertigo 

4- Acoustic neuroma 

5- Cerebellopontine angle tumour 


Answer & Comments 

Answer: 2- Meniere's disease 

Meniere's disease is caused by distension of 
the endolymphatic compartment of the inner 
ear. The symptoms of Meniere's disease 
include vertigo, hearing loss and tinnitus. The 
dizziness is described as a spinning or whirling 
feeling and may cause problems with balance. 
Some people feel nauseated and vomit during 
an attack. Tinnitus refers to a ringing or 
roaring sound in the ear. Others may notice 
some hearing loss, especially with sounds that 
have a low frequency. 

Horizontal nystagmus is more commonly seen 
on examination, but vertical nystagmus may 
also occur. 

[ Q: 596 ] MRCPass - Neurology 

A 30 year old gynmast has sudden 
onset vertigo and dizziness. On examination, 
there is horizontal nystagmus, with a full 
range of eye movements. Her speech is 
slurred. There is intention tremor and 
disdiadochokinesis which is asymmetrical. She 
has an ataxic gait. 

Which of the Following investigations would 
be most appropriate? 

1- CT of the head 

2- MRI and MRA of head and neck 

3- MRI with enhancement 

4- Lumbar puncture 



5- Otological testing 


Answer & Comments 

Answer: 2- MRI and MRA of head and neck 

The clinical picture is of an acute onset 
cerebellar syndrome, which suggest a vascular 
cause. This would involve the posterior 
(vertebrobasilar) circulation. If headache or 
neck pain were associated, a vertebral artery 
dissection w ould be most important to 
exclude. A Magnetic Resonance Angiography 
(MRA) will help to diagnose dissection, 
stenosis or thrombosis. 

[ Q: 597 ] MRCPass - Neurology 

A 30 year old gym instructor 
complained of shoulder pain and weakness 
which has been progressive over the past 5 
years. He is upset about having difficulty lifting 
weights which he had previously been able to 
do so without difficulty. 

On examination, there was winging of the 
scapula. Power was reduced in the muscles 
around the shoulder, with bilateral wasting. 
He also has some facial difficulty raising his 
eyebrow s. His serum CK is 400. 

What is the likely diagnosis? 

1- Duchenne's muscular dystrophy 

2- Becker's muscular dystrophy 

3- Myotonic dystrophy 

4- Fascioscapulo humeral dystrophy 

5- Polymyalgia rheumatica 



Answer & Comments 

Answer: 4- Fascioscapulo humeral dystrophy 

Facioscapulohumeral dystrophy (FSHD) is one 
of the most common types of muscular 
dystrophy. It is of autosomal dominant 
inheritance. Onset is usually age 20 years. 


Dr. Khalid Yusuf El-Zohry - Sohag Teaching Hospital (01118391123) 

Ref MRCPass OE OE 2012 PasTest 2009 PassMedicine 2009 PasTest Exam 




ReviseMRCP 

267 

























El-zohry MRCP Questions Bank (Port 1) - 2013 


(For my personal use) 


Initial weakness is seen in facial muscles, 
starting in the orbicularis oculi, orbicularis 
oris, and zygomaticus. 

Shoulder weakness is the presenting symptom 
in more than 82% of patients. Winging of the 
scapula is the most characteristic sign. 
Creatine kinase levels are raised. The drug 
Albuterol which relaxes bronchial smooth 
muscle has been shown to increase lean 
muscle mass when used over a period of 
months. 



Winging of the scapula in FSHD 



[ Q: 598 ] MRCPass - Neurology 

A 40 year old man presented with 
double vision and was found to have normal 
vertical eye movements. 


On left lateral gaze, there was absence of 
adduction of the right eye, and nystagmus in 
the abducting left eye. 


This eye movement disorder con be explained 
by o lesion in the: 


1- Left cerebellopontine angle 

2- Right parietal area 

3- Right medial longitudinal fasciculus 


4- Left medial longitudinal fasciculus 

5- Left lateral medulla 


Answer & Comments 


Answer: 3- Right medial longitudinal fasciculus 


The diagnosis is right internuclear 
ophthalmoplegia due to a lesion in the right 
medial longitudinal fasciculus. The likely 
underlying pathology is multiple sclerosis, 
other causes of INO include a glioma or 
vascular lesion. 


[ Q: 599 ] MRCPass - Neurology 

A 70 year old man presents with a 
history of falls. He has difficulty reading and 
walking down stairs. He has dysarthria, 
akinesia and rigidity. Power of the muscles is 
normal, reflexes are brisk. 

Whot physical sign will help to confirm the 
diagnosis? 

1- Gait 

2- Eye movements 

3- Romberg's sign 

4- Abdominal reflexes 

5- Plantar reflexes 

Answer & Comments 

Answer: 2- Eye movements 

The patient has progressive supranuclear 
palsy - parkinsonian features and gaze palsy. 
Demonstration of impairment of voluntary 
gaze will help confirm the diagnosis. 

[ Q: 600 ] MRCPass - Neurology 

A 55 year old man has slowly 
progressive weakness of his upper limbs. On 
examination of the patient the physical signs 
are wasting and weakness of the small 
muscles of the hand, flattening of the muscles 
of the ulnar border of the forearm. 

The upper limb reflexes are absent. Pain and 
temperature sensation are reduced over the 
upper limbs and upper chest whereas light 
touch and proprioception remain intact. 
Lower limb reflexes are exaggerated and 
plantars are extensor. 

What is the likely diagnosis? 





Dr. Khalid Yusuf El-Zohry - Sohag Teaching Hospital (01118391123) 

Ref MRCPass OE OE 2012 PasTest 2009 PassMedicine 2009 PasTest Exam ReviseMRCP 




























El-zohry MRCP Questions Bank (Port 1) - 2013 


(For my personal use) 


1- Normal pressure hydrocephalus 

2- Multiple sclerosis 

3- Arnold Chiari malformation 

4- Severe kyphoscoliosis 

5- Kennedy's syndrome 

Answer & Comments 

Answer: 3- Arnold Chiari malformation 


kidney disease. There is a family history of the 
condition. 

What is the likely diagnosis? 

1- Phaeochromocytoma 

2- Von Hippel Lindau syndrome 

3- Hereditary haemorrhagic telangiectasia 

4- Multiple sclerosis 

5- Friedriech's ataxia 


Arnold-Chiari Malformation is a condition in 
which the cerebellum portion of the brain 
protrudes into the spinal canal. It may or may 
not be apparent at birth. 

Arnold-Chiari I type malformation usually 
causes symptoms in young adults and is often 
associated with syringomyelia, in which a 
tubular cavity develops within the spinal cord. 

Arnold-Chiari II type malformation is 
associated with myelomeningocele (a defect 
of the spine) and hydrocephalus (increased 
cerebrospinal fluid and pressure within the 
brain), which usually are apparent at birth. 
The patient described has the features of an 
intramedullary lesion of the spinal cord. 



Chiari Malformation 


— 

[ Q: 601 ] MRCPass - Neurology 

m 

• l 

1 J 

A 30 year old woman has been 


diagnosed with a cerebellar cyst with MRI 
scanning. She is also known to have polycystic 


Answer & Comments 

Answer: 2- Von Hippel Lindau syndrome 

The diagnosis is likely to be von Hippel Lindau 
disease. There may be cerebellar 
haemangioblastomas, retinal angiomas and 
polycystic liver or kidneys. Ectopic 
erythropoietin secretion by the 
haemangioblastomas cause polycythaemia. 



Retinal Angioma seen in von Hippel Lindau 

syndrome 



[ Q: 602 ] MRCPass - Neurology 

An 80 year old man is admitted with 
confusion over the last 3 days. He has a past 
medical history of hypertension and has had 
frequent falls in the past. Clinical examination 
is unremarkable. 


Which of these conditions needs to be 
excluded? 


1- Intracranial haemorrhage 

2- Subdural haematoma 


3- Meningitis 



Dr. Khalid Yusuf El-Zohry - Sohag Teaching Hospital (01118391123) 

Ref MRCPass OE OE 2012 PasTest 2009 PassMedicine 2009 PasTest Exam ReviseMRCP 


























El-zohry MRCP Questions Bank (Port 1) - 2013 


(For my personal use) 


4- Vertebrobasilar stroke 

5- Encephalitis 


Answer & Comments 

Answer: 2- Subdural haematoma 

A patient who has had a fall may have hit his 
head and developed a subdural haematoma. 
This can be excluded by a CT head scan. 



Right sided subdural haematoma 


t 


[ Q: 603 ] MRCPass - Neurology 

A 45 year old man presents with a 
sudden onset of headache in the posterior 
region, associated with vomiting. 


Neurological examination, including 
fundoscopy is unremarkable apart from 
slightly brisk reflexes. There is no neck 
stiffness or photophobia. 


Which of the Following management options 
would be the most appropriate? 


1- CT of the head and lumbar puncture 


2- MRI of the head 


3- CT of the head 


4- Skull X ray 

5- Discharge from hospital 


Answer & Comments 

Answer: 1- CT of the head and lumbar 
puncture 


A subarachnoid haemorrhage (SAH) needs to 
be excluded. CT brain scan is normal in a third 
of patients with SAH. A lumbar puncture to 
look for xanthochromia in the CSF should then 
be performed. 



[ Q: 604 ] MRCPass - Neurology 

A 40 year old patient has presented 
with a generalised tonic clonic seizure for the 
first time. This lasted for 10 minutes. 

What advice should be given regarding driving 
a car? 

1- No driving for 1 month 

2- No driving for 6 months 

3- No driving for 1 year 

4- Driving is allowed if EEG is normal 

5- Driving is allowed if CT scan is normal 


Answer & Comments 

Answer: 3- No driving for 1 year 

For a single seizure, driving is not permitted 
for 1 year. Also, a medical review is required 
before one is to do so and it is a requirement 
for the patient to inform the Driver and 
Vehicle Licensing Authority. 


[ Q: 605 ] MRCPass - Neurology 

The sister of a patient who died from 
a subarachnoid hemorrhage due to a cerebral 
aneurysm is worried and is asking about her 
chances of having the same problem. She 
mentions that there family history of other 
deaths from subarachnoid haemorrhage. She 
is how ever, asymptomatic. 

What should be done? 

1- Reassure and nothing else 

2- CT scan of head 

3- MRI head scan 

4- Cerebral angiography 

5- Lumbar puncture 




Dr. Khalid Yusuf El-Zohry - Sohag Teaching Hospital (01118391123) 

Ref MRCPass OE OE 2012 PasTest 2009 PassMedicine 2009 PasTest Exam 


ReviseMRCP 






























El-zohry MRCP Questions Bank (Port 1) - 2013 


(For my personal use) 


Answer & Comments 

Answer: 3- MRI head scan 

This may be a case of familial subarachnoid 
haemorrhage. In those who have a first 
degree relative who genuinely suffered a SAH, 
their risk of also suffering one is 3-7 times that 
of the general population. MRI is better than 
CT scan for screening. Angiography is 
diagnostic but too invasive for screening. 


[ Q: 606 ] MRCPass - Neurology 

A 45 year old lady has a 4 week 
history of pain and difficulty seeing out of her 
right eye. She has a visual acuity of 6/18 in the 
right and 6/6 in the left. There is also a right 
afferent pupillary defect. 

Which is the most likely cause from the list 
below? 

1- Thyroid eye disease 

2- Diabetic retinopathy 

3- Astrocytoma 

4- Multiple sclerosis 

5- Retinitis pigmentosa 



[ Q: 607 ] MRCPass - Neurology 

A 40 year old lady presents with 
drooping of her eye lids and double vision. She 
does not have proptosis. There is no muscle 
wasting around the face. She has diplopia on 
downgaze during examination and also 
proximal muscle weakness of her upper limbs. 
Myasthenia gravis is diagnosed. 

Which drug is most likely to improve her 
symptoms? 

1- Beta interferon 

2- Intravenous immunoglobulin 

3- Benztropine 

4- Pyridostigmine 

5- Bromocriptine 



Answer & Comments 

Answer: 4- Pyridostigmine 

The condition described is Mysthenia Gravis 
rather than Grave's eye disease or Myotonic 
dystrophy (frontal balding). Pyridostigmine is 
an anticholinesterase which reduces 
acetylcholine breakdown and hence improve 
symptoms of fatiguability in myasthenia 
gravis. 


Answer & Comments 


Answer: 4- Multiple sclerosis 

Optic neuritis secondary to multiple sclerosis 
can present in this manner. 



Disc pallor (optic neuritis) 


^ [ Q: 608 ] MRCPass - Neurology 

-——- 

# A 40 year old bank clerk presents 
with a headache, nausea and ptosis of the left 
eye with blurred vision. She does not have 
fatiguability of her eye movements. 
Examination revealed sw elling on the left side 
of face, proptosis and chemosis of the left eye, 
left mastoid sw elling and left 
ophthalmoplegia involving cranial nerves. 

The pupil sizes were equal. A CT of her head is 
normal. 

What is the likely diagnosis? 

1- Third nerve palsy 

2- Myasthenia gravis 

3- Pituitary tumour 


Dr. Khalid Yusuf El-Zohry - Sohag Teaching Hospital (01118391123) 

Ref MRCPass OE OE 2012 PasTest 2009 PassMedicine 2009 PasTest Exam 




ReviseMRCP 

271 





























El-zohry MRCP Questions Bank (Port 1) - 2013 


(For my personal use) 


4- Cavernous sinus thrombosis 

5- Horner's syndrome 


Answer & Comments 

Answer: 4- Cavernous sinus thrombosis 

A history of headache and no other obvious 
cause of ptosis is suggestive of cavernous 
sinus thrombosis. Third nerve palsy is 
associated with dilated pupil and Horner's 
syndrome is associated with miosis. A CT can 
be normal, and diagnosis is confirmed with 
MRI. 


[ Q: 609 ] MRCPass - Neurology 

A 65 year old man presents with an 
episode of amnesia for the second time. 2 
days ago he had an episode of confusion, 
according to his wife. He was, how ever, able 
to have a normal conversation despite having 
been found wandering. After 2 hours, he 
abruptly returned to normal and could not 
remember what happened. 

Whot is the most likely diagnosis? 

1- Alcoholic encephalopathy 

2- Subarachnoid haemorrhage 

3- Complex partial seizure 

4- Transient ischaemic attack 

5- Transient global amnesia 



Answer & Comments 

Answer: 5- Transient global amnesia 

Transient global amnesia (TGA) is a temporary 
and isolated disorder of memory which may 
last several hours. Heavy exercise and the cold 
are known precipitants. 


[ Q: 610 ] MRCPass - Neurology 

A 40 year old man presents with 
finger weakness which was diagnosed as an 
ulnar nerve lesion. 



Which of the Following muscles is supplied by 
the ulnar nerve? 

1- Interossei 

2- Lateral two lumbricals 

3- Opponens pollicis 

4- Abductor pollicis brevis 

5- Flexor pollicis brevis 


Answer & Comments 

Answer: 1- Interossei 

The interossei muscles and medial two 
lumbricals are supplied by the ulnar nerve. 
The lateral two lumbricals (anatomical 
position), opponens pollicis, abductor pollicis 
brevis and flexor pollicis brevis are supplied by 
the median nerve. 



[ Q: 611 ] MRCPass - Neurology 

A 35 year old alcoholic presents with 
unsteadiness whilst walking. On examination 
he has increased tone and brisk reflexes in the 
right leg. Proprioception is abnormal in the 
right leg. There is loss of vibration sense in the 
right leg. There is decrease in pain and 
temperature sensation in the left leg. 


Which one of the following conditions is most 
likely to be responsible for his weakness? 


1- Syringomyelia 


2- Subacute combined degeneration of cord 

3- Friedrich's ataxia 


4- Guillain Barre syndrome 

5- Brown Sequard syndrome 


Answer & Comments 

Answer: 5- Brown Sequard syndrome 

Brown Sequard syndrome which describes 
hemisection of the spinal cord, causes 
ipsilateral UMN signs and proprioception loss 
(corticospinal tract and dorsal column 
decussate at the medulla), and contralateral 


Dr. Khalid Yusuf El-Zohry - Sohag Teaching Hospital (01118391123) 

Ref MRCPass OE OE 2012 PasTest 2009 PassMedicine 2009 PasTest Exam ReviseMRCP 



272 

































El-zohry MRCP Questions Bank (Part 1) - 2013 


(For my personal use) 


sensory loss in pain and temprature (the 
spinothalamic tracts decussate at the same 
level). The rest of the conditions 
(syringomyelia, subacute degeneration of 
cord, Friedrich's ataxia) can cause cerebellar 
signs or patchy sensory loss but should be 
bilateral. 


Which is the most appropriate treatment? 

1- Glucose 

2- Lorazepam 

3- IV thiamine 

4- IV Vitamin B n 

5- IV vitamin K 


7 


[ Q: 612 ] MRCPass - Neurology 

* A 45 year old man has bilateral 
ptosis. He mentions a past history of cataracts, 
frontal balding and weakness of the facial 
muscles. On examination, he has a firm grip 
with difficulty relaxing. 

What is the diagnosis? 

1- Multiple sclerosis 

2- Motor neuron disease 

3- Parkinson's disease 


4- Dermatomyositis 

5- Myotonic dystrophy 


Answer & Comments 

Answer: 5- Myotonic dystrophy 

Myotonic dystrophy is autosomal dominant. It 
is a trinucleotide repeat disorder which 
exhibits anticipation (worse with successive 
generations). Associated features are 
cataracts, diabetes, testicular atrophy and 
cardiac conduction abnormalities. 



Myotonic Dystrophy 


[ Q: 613 ] MRCPass - Neurology 


A 50 year old alcoholic is admitted to 
A+E with unsteadiness and confusion. BM is 7. 



Answer & Comments 

Answer: 3- IV thiamine 

This patient is likely to have Wernicke's 
encephalopathy causing confusion. IV 
thiamine should given to reduce the 
progression. This is contained in Pabrinex. 


[ Q: 614 ] MRCPass - Neurology 

A man presents with generalised 
weakness. On examination, fatiguability was 
demonstrated. A diagnosis of Eaton Lambert 
syndrome was made. 

What form of antibody is found in this 
condition? 

1- Anti Purkinje 

2- Anticholinesterase 

3- Neuromuscular junction 

4- Potassium channels 

5- Voltage gated calcium channels 



Answer & Comments 

Answer: 5- Voltage gated calcium channels 

Eaton Lambert syndrome is frequently 
associated with a malignancy e.g. bronchial. 
The disorder is associated with antibodies 
against voltage gated calcium channels. 


[ Q: 615 ] MRCPass - Neurology 

A 20 year old man injured himself 
whilst snow boarding. On examination, he has 
weakness of elbow flexion and loss of 



Dr. Khalid Yusuf El-Zohry - Sohag Teaching Hospital (01118391123) 

Ref MRCPass OE OE 2012 PasTest 2009 PassMedicine 2009 PasTest Exam 




ReviseMRCP 

273 






























El-zohry MRCP Questions Bank (Port 1) - 2013 


(For my personal use) 


sensation over the radial aspect of her 
forearm. 

Which of the Following nerves is damaged? 

1- Musculocutaneous nerve 

2- Brachioradialis nerve 

3- Radial nerve 


Which one of the Following needs to be 
excluded? 

1- Vaso vasagal syncope 

2- Pseudoseizure 

3- Anxiety disorder 

4- Visual hallucinations 


4- Median nerve 

5- Ulnar nerve 


Answer & Comments 

Answer: 1- Musculocutaneous nerve 

The fibers of the musculocutaneous nerve 
originate in the lower cervical spinal cord 
(usually C5 to C7), travel via the lateral cord of 
the brachial plexus, and supply sensory and 
motor innervation to the upper arm, elbow , 
and forearm. It supplies the biceps which 
controls elbow flexion. Sensation is to the 
lateral area (lateral cutaneous nerve) of the 
forearm. 


Musculocutaneous Nerve 



formed from lateral 
cord of brachial plexus 



pierces coracobracliialis 
then lies between biceps 
and brachialis 

becomes lateral cutaneous 
nerve of forearm at elbow 


^ [ Q: 616 ] MRCPass - Neurology 

# An 18 year old woman is referred to 
the GP by her teacher. The teacher was 
concerned about frequent episodes of day 
dreaming during class and poor examination 
results over the past year. 


5- Absence seizures 


Answer & Comments 

Answer: 5- Absence seizures 

Daydreaming in children can be easily 
confused with absence or complex partial 
seizures, in which staring is a prominent and 
common feature. However, lip smacking, eye 
blinking, or stiffening of muscle groups is 
common during seizures but not during 
daydreaming. 


[ Q: 617 ] MRCPass - Neurology 

A 20 year old man presents with a 6 
month history of depression and painful 
sensory disturbance in both legs. He has also 
become very confused. There are myoclonic 
jerks observed in his legs. His MRI scan reveals 
thalamic hyperintensity and EEG is normal. 

The most likely diagnosis is: 

1- New variant CJD 

2- Huntington's disease 

3- Wilson's disease 

4- Progressive multifocal leucoencephalopathy 

5- Paraneoplastic syndrome 



Answer & Comments 

Answer: 1- New variant CJD 

New variant CJD commonly presents in young 
adults painful sensory symptoms in the lower 
limbs and also psychiatric symptoms. 
Cognitive impairment, pyramidal signs, 
myoclonus and primitive reflexes then 
develop. 



Dr. Khalid Yusuf El-Zohry - Sohag Teaching Hospital (01118391123) 

Ref MRCPass OE OE 2012 PasTest 2009 PassMedicine 2009 PasTest Exam ReviseMRCP 



























El-zohry MRCP Questions Bank (Port 1) - 2013 


(For my personal use) 


MRI commonly shows high signal on T2-w 
eighted images in the pulvinar (posterior 
aspect of thalamus). EEG is often normal, 
unlike sporadic CJD, in which triphasic waves 
are observed. 

[ Q: 618 ] MRCPass - Neurology 

A 60 year old woman is admitted 
with a severe headache. CT scan conforms a 
subarachnoid haemorrhage. She initially 
makes satisfactory progress but 7 days later 
her level of consciousness begins to 
deteriorate. 

The most likely couse of the deterioration is: 

1- Cerebral oedema 

2- Coning of the medulla 

3- Meningitis 

4- Encephalitis 

5- Acute hydrocephalus 



Answer & Comments 

Answer: 5- Acute hydrocephalus 

Organised blood in the subarachnoid space 
may cause obstruction to the flow of 
cerebrospinal fluid (impaired absorption in the 
arachnoid villi). 10% of patients will require 
CSF diversion or shunting. 

[ Q: 619 ] MRCPass - Neurology 

A 62 year old woman has several 
episodes of dizziness particularly when she 
turns her head. 2 months ago, she had an 
attack of vertigo, without deafness or tinnitus, 
lasting for a few minutes. Over the last month, 
she had five further attacks of vertigo, 
accompanied by moderate headache and a 
left homonymous hemianopia, lasting for 
about a quarter of an hour. After the last 
episode she developed persistent 
unsteadiness of gait, and was admitted to 
hospital. 



On examination there was normal visual 
fields, were full. There was rhythmic 
horizontal nystagmus, slight weakness of the 
right external rectus muscle without diplopia, 
and ataxia of gait, provoked by turning. The 
bloodpressure was 160/80 mmHg. 

Whot is the diagnosis? 

1- Parietal lobe CVA 

2- Frontal lobe CVA 

3- Vertebrobasilar insufficiency 

4- Syringomyelia 

5- Brown sequard syndrome 


Answer & Comments 

Answer: 3- Vertebrobasilar insufficiency 

Vertebrobasilar (posterior) circulation 
constitutes the arterial supply to the brain 
stem, cerebellum, and occipital cortex. 
Bilateral visual loss, dizziness, speech 
disturbances, drop attacks and transient 
global amnesia are features of vertebrobasilar 
insufficiency. MRI / MRA are good 
investigations to investigate for vertebral or 
basilar arterial disease. 



[ Q: 620 ] MRCPass - Neurology 

A 55 year old man has developed 
weakness over the past 3 weeks which has 
affected his walking. He has no significant past 
medical history. On examination, he had 
decreased sensation peripherally in the legs 
and also flaccid reflexes in the ankles. A CT of 
the head was normal and lumbar puncture 
was done. 


Results were: 
protein 0.75( <0.43 g/l) 
glucose 4 (3.3 to 4.4 mmol/l) 
lymphocytes 7 (< 5/mm 3 ) 
Whot is the likely diagnosis? 

1- Multiple sclerosis 

2- Guillain Barre syndrome 


Dr. Kholid Yusuf El-Zohry - Sohog Teaching Hospital (01118391123) 

Ref MRCPass OE OE 2012 PasTest 2009 PassMedicine 2009 PasTest Exam 




ReviseMRCP 

275 

























El-zohry MRCP Questions Bank (Port 1) - 2013 


(For my personal use) 


3- Lymphocytic meningitis 

4- Tuberculous meningitis 

5- Syringomyelia 


Answer & Comments 

Answer: 2- Guillain Barre syndrome 

In Guillain Barre syndrome, CSF protein is 
elevated in most patients after the second or 
third week of illness. The gamma globulin 
fraction is usually raised. Cells, usually 
monocytic, are found in 20% of cases. 


[ Q: 621 ] MRCPass - Neurology 

A 65 year old patient has progressive 
dementia. His wife mentions that he has 
urinary incontinence and an ataxic gait. 

What is his CT scon likely to show ? 

1- Parasagittal mass 

2- Multiple infarcts 

3- Large ventricles 

4- Cerebellar tumour 

5- Berry aneurysm 



Answer & Comments 

Answer: 3- Large ventricles 

The diagnosis is normal pressure 
hydrocephalus. Dementia, urinary 
incontinence and unsteady gait are seen. 
Typically there is no papilloedema. There are 
large ventricles caused by communicating 
hydrocephalus. Ventricular shunting may help 
improve the symptoms. 



Normal Pressure Hydrocephalus 


[ Q: 622 ] MRCPass - Neurology 

A 60 year old lady presents with 
acute onset unsteadiness and dizziness. 
Neurological examination shows a right-sided 
Horner's syndrome and nystagmus. There is 
also loss of pain and temperature sensation 
on the left side of the trunk and in the left arm 
and leg. Her gait is ataxic. 

Which is the correct diagnosis? 

1- Posterior inferior cerebellar artery 
occlusion 

2- Medullary infarct 

3- Posterior cerebral artery occlusion 

4- Middle cerebral artery occlusion 

5- Posterior communicating artery 
haemorrhage 



Answer & Comments 

Answer: 1- Posterior inferior cerebellar artery 
occlusion 

There are a complex of symptoms caused by 
occlusion of the posterior inferior cerebellar 
artery or one of its branches supplying the 
lower portion of the brain stem, resulting in 
sensory and sympathetic disturbances, 
cerebellar and pyramidal tract signs, and 
evidence of partial involvement of the fifth, 
ninth, tenth, and eleventh cranial nerves. 


Dr. Kholid Yusuf El-Zohry - Sohog Teaching Hospital (01118391123) 

Ref MRCPass OE OE 2012 PasTest 2009 PassMedicine 2009 PasTest Exam ReviseMRCP 




























El-zohry MRCP Questions Bank (Port 1) - 2013 


(For my personal use) 


Onset is usually acute with severe vertigo. 
Nausea, vomiting, ipsilateral ataxia, muscular 
hypertonicity, pastpointing and other 
cerebellar signs are often present. Horner's 
syndrome is usually present. Sensory 
disturbances include ipsilateral loss of pain 
and temperature perception of the face and 
contralateral hypoesthaesia for pain and 
temperature of the trunk and extremities. 

The affected persons have difficulty in 
swallowing. Persons well over 40 years of age 
are most often affected. 


[ Q: 623 ] MRCPass - Neurology 

A 60 year old man has new onset 
receptive and expressive dysphasia. His past 
medical history includes diabetes and 
hypertension. 

On examination, he has increased tone and 
extensor plantar reflex on the right. He also 
has weakness of the right leg with sensory loss 
over the same side. 



walking is even greater than expected from 
the weakness present. 

Dysphasia could occur with the occlusion of a 
branch of the left ACA. Branch occlusion of the 
ACA can cause only parts of the total 
syndrome, producing a spastic weakness or 
cortical sensory loss in the opposite foot and 
leg. 



Anterior Cerebral Artery Infarct 


Which vascular lesion is likely? 

1- Anterior cerebral artery 

2- Superior middle cerebral artery 

3- Inferior middle cerebral artery 

4- Posterior cerebral artery 

5- Posterior inferior cerebellar artery 


Answer & Comments 

Answer: 1- Anterior cerebral artery 

A middle cerebral artery occlusion is likely to 
cause total hemiplegia. In view of the partial 
weakness, an anterior cerebral artery (ACA) 
lesion is more likely. 

Complete infarction due to occlusion of one 
ACA distal to the anterior communicating 
artery results in a sensory-motor deficit of the 
opposite foot and leg and a lesser degree of 
paresis of the arm with sparing of the face. 
Foot drop is a common finding and difficulty in 


[ Q: 624 ] MRCPass - Neurology 

A 35 year old man is admitted with 
acute right sided weakness and slurred 
speech. He does not have a history of 
hypertension, diabetes or high cholesterol. 
There is no family history of CVA. He does not 
smoke. He complains of headaches and 
generalised limb weakness infrequently. 

On examination, he has hypotonia and 
weakness in the proximal muscles more than 
the distal muscles. His laboratory tests show a 
high lactate to pyruvate ratio. 

What is the likely diagnosis? 

1- Polymyositis 

2- Inclusion body myositis 

3- ME LAS 

4- Polymorphonuclear leukoencephalopathy 

5- Neuroacanthocytosis 



Dr. Khalid Yusuf El-Zohry - Sohag Teaching Hospital (01118391123) 

Ref MRCPass OE OE 2012 PasTest 2009 PassMedicine 2009 PasTest Exam 




ReviseMRCP 


in 

























El-zohry MRCP Questions Bank (Port 1) - 2013 


(For my personal use) 


Answer & Comments 

Answer: 3- MELAS 

This patient has MELAS (myopathy, 
encephalopathy, lactic acidosis and stroke like 
episodes). 

Lactic acidosis is a very important feature of 
this disorder, as measured by a high lactate to 
pyruvate ratio. 

How ever, in general, lactic acidosis does not 
lead to systemic metabolic acidosis, and it 
may be absent in patients with impressive 
involvement of the central nervous system. 

Patients have a myopathy causing proximal 
muscle weakness and hypotonia, seizures and 
strokelike episodes. It is a mitochondrial 
inherited disorder. 


^ [ Q: 625 ] MRCPass - Neurology 

f L -—- 

m A 75 year old lady complains of a 
headache for 2 days in the right side of the 
head. The pain is worse when she is chewing 
or talking. She has also had mild fevers and sw 
eats. Her ESR is 80 mm/hr. The vision in the 
right eye is 6/18 and her left eye is 6/6. 

What is the best course of action? 

1- Start iv methylprednisolone 

2- Organise and await temporal artery biopsy 

3- CT of the head to exclude space occupying 
lesion 

4- MRI of the brain 

5- Refer to an ophthalmologist 


Answer & Comments 

Answer: 1- Start iv methylprednisolone 

There are early signs of visual loss so high 
dose steroids should be commenced with the 
suspicion of temporal arteritis. Although all 
the other options are reasonable, they may 
take time and there should not be delay in 
commencing steroids. 


[ Q: 626 ] MRCPass - Neurology 

A 35 year old woman presents with 
double vision that is worst when trying to read 
a book or walk down stairs. 

The most likely diagnosis is: 

1- Progressive supranuclear palsy 

2- 4th nerve palsy 

3- 3rd nerve palsy 

4- 6th nerve palsy 

5- Internuclear ophthalmoplegia. 



Answer & Comments 

Answer: 2- 4th nerve palsy 

The superior oblique muscle is innervated by 
the fourth nerve. The action is to depress the 
eye and is maximally effective when the eye is 
looking medially, hence diplopia on reading or 
going downstairs is typical of fourth nerve 
paralysis. 



[ Q: 627 ] MRCPass - Neurology 

A 65 year old man has muscle 
weakness, particularly around the thighs and 
shoulders. His CK is 2,200 U/l. EMG shows 
reduced amplitude and duration of motor 
units. 

What is the likely diagnosis? 

1- Dermatomyositis 

2- Myasthenia gravis 

3- Myotonic dystrophy 




Dr. Khalid Yusuf El-Zohry - Sohag Teaching Hospital (01118391123) 

Ref MRCPass OE OE 2012 PasTest 2009 PassMedicine 2009 PasTest Exam ReviseMRCP 




























El-zohry MRCP Questions Bank (Port 1) - 2013 


(For my personal use) 


4- Peripheral neuropathy 

5- Multiple sclerosis 

Answer & Comments 

Answer: 1- Dermatomyositis 

These EMG changes are consistent with a 
myositis. In motor neuron disease, fibrillation 
is seen. In myasthenia, there is diminished 
response to repetitive stimulation. 


only. The episode lasted a few seconds and he 
has been relatively well. Examination reveals a 
mild hemiparesis of the left arm and leg. 

Which is the likely diagnosis? 

1- Pontine haemorrhage 

2- Primary epilepsy 

3- Medullary haemorrhage 

4- Right internal capsule infarct 

5- Left internal capsule infarct 


[ Q: 628 ] MRCPass - Neurology 

A 42 year old man presents to A+E 
complaining of severe lower back pain 
Following carpentry work. The pain radiates to 
his left buttock and thigh. 

On examination, he was able to straight leg 
raise to 45 degrees only on the left side. The 
sciatic stretch test is positive. He has difficulty 
plantar flexing his left ankle and has abnormal 
sensation on the plantar aspect of the foot. 

What is the diagnosis? 

1- Cauda equina syndrome 

2- L2/L3 disc prolapse 

3- L4/L5 disc prolapse 



Answer & Comments 

Answer: 4- Right internal capsule infarct 

This patient is likely to have a lacunar infarct 
involving the internal capsule, causing 
transient contralateral hemiparesis. 


[ Q: 630 ] MRCPass - Neurology 

A 25 year old man has had 
behavioural disturbance recently. His parents 
mentioned that his brother has been 
investigated for liver problems recently. On 
examination, he has a MMSE score of 28/30. 
He has a mask like face and was noticed to 
have hypersalivation. 



4- L5/S1 disc prolapse 

5- Common peroneal nerve injury 

Answer & Comments 

Answer: 4- L5/S1 disc prolapse 

Ankle dorsiflexion is generally supplied by 
L4/L5 and plantar flexion supplied by S1/S2. 


When investigations are complete, which drug 
is most likely to be used for treatment? 

1- Desferrioxamine 

2- Co careldopa 

3- Penicillamine 

4- Interferon alpha 

5- Chlorpromazine 


this case is likely to be due to sciatic nerve 
palsy. 


[ Q: 629 ] MRCPass - Neurology 


A 60 year old man is brought to 
hospital having collapsed to the ground 
suddenly and was unable to move his left leg 
or arm. There was no loss of consciousness. 
He has a past medical history of hypertension 


Answer & Comments 

Answer: 3- Penicillamine 

The likely diagnosis is Wilson's disease. Most 
patients who present with neuropsychiatric 
manifestations have cirrhosis. The most 
common presenting neurologic feature is 
asymmetric tremor, occurring in 
approximately half of individuals with Wilson 


Dr. Khalid Yusuf El-Zohry - Sohag Teaching Hospital (01118391123) 

Ref MRCPass OE OE 2012 PasTest 2009 PassMedicine 2009 PasTest Exam ReviseMRCP 



279 































El-zohry MRCP Questions Bank (Port 1) - 2013 


(For my personal use) 


disease. Frequent early symptoms include 
difficulty speaking, excessive salivation, ataxia, 
masklike facies, clumsiness with the hands, 
and personality changes. The disease is 
autosomal recessive. Penicillamine is used as a 
copper chelator. 


[ Q: 631 ] MRCPass - Neurology 

A 55 year old man presents with a 
history of slowly progressive, abnormal 
movements of his body. On examination there 
are jerky, semi-purposive movements 
involving the entire body and abnormal 
tongue movements. 

What is the likely diagnosis? 

1- Parkinson's disease 

2- Motor neuron disease 

3- Frontal lobe tumour 

4- Cervical spondylosis 

5- Huntington's disease 



described as "explosive" and were graded 9 to 
10 in severity on a visual analog scale of 1 to 
10. 

The quality of the head pain was reported to 
be throbbing, sharp, shooting, as well as 
aching. The pain was localized around the 
right eye, behind the right ear, and in the 
occipital region. There are no associated visual 
symptoms, but occasionally flushing is 
associated. He gets these headaches 
frequently during the winter. 

What is the likely diagnosis? 

1- Migraine 

2- Trigeminal neuralgia 

3- Absence seizures 

4- Sagittal sinus thrombosis 

5- Cluster headache 


Answer & Comments 


Answer: 5- Cluster headache 


Answer & Comments 

Answer: 5- Huntington's disease 

There are many causes of chorea. 

Inherited- Ataxia-telangiectasia, Huntington 
disease, Wilson disease. 


In cluster headaches, there is an association 
with autonomic features, particularly miosis 
and ptosis. Cluster headaches occur during the 
same months in the year typically and are 
almost five times more common in males. A 
opthalmic division of trigeminal nerve 
distribution involvement is also common. 


Drugs - Anticonvulsants (eg, phenytoin, 
carbamazepine, phenobarbital), 

Antidopaminergic agents (eg, phenothiazines, 
haloperidol, metoclopramide) 

Behget disease, antiphospholipid antibody 
syndrome, Bacterial endocarditis, Herpes 
simplex encephalitis, Lyme disease. 

[ Q: 632 ] MRCPass - Neurology 

A 40 year old man has a 5-year 
history of right-sided cluster headaches with 
recurrent right sided headaches which last for 
2 hours. There is a pattern of daily occurrence 
for 2 to 3 weeks, followed by a month or so 
without headaches. The headaches were 



[ Q: 633 ] MRCPass - Neurology 

A 30 year old man complains of 
spasms in his neck. He has, over the past three 
years, noticed a crampy sensation in his neck 
associated with contraction of the muscle on 
the left. This has now got frequent and 
uncontrollable. Neurological examination is 
normal, but on tapping the left side of the 
neck muscle, it contracts and spasms occur. 

Which of the Following medication may help 
this? 

1- Methysergide 

2- Levodopa 



Dr. Khalid Yusuf El-Zohry - Sohag Teaching Hospital (01118391123) 

Ref MRCPass OE OE 2012 PasTest 2009 PassMedicine 2009 PasTest Exam ReviseMRCP 



280 


























El-zohry MRCP Questions Bank (Port 1) - 2013 


(For my personal use) 


3- Phenytoin 

4- Lamotrigine 

5- Methyldopa 


Answer & Comments 

Answer: 2- Levodopa 

This patient has a dystonia in the neck. Other 
examples are blepharospasm and torticollis. 

This could be helped by levodopa or 
diazepam. In severe cases, botulinum 
injections may also help. 


[ Q: 635 ] MRCPass - Neurology 

A 30 year old man has a painful right 
eye. On examination, there is decreased visual 
acuity and a relative afferent pupillary defect 
of the right eye. 

What is the diagnosis? 

1- Optic atrophy 

2- Optic neuritis 

3- Glaucoma 

4- Retinitis pigmentosa 

5- Internuclear opthalmoplegia 



\7 

[ Q: 634 ] MRCPass - Neurology 

Answer & Comments 

•U 

A 45 year old man was referred for 

Answer: 2- Optic neuritis 


assessment of unsteady gait, which has been 
present for 6 months. He has lost a stone in 
weight over the several months. An MRI of the 
brain shows multiple high signal areas. 


A painful eye with loss of vision, and also 
RAPD suggests optic neuritis. The most likely 
underlying cause is multiple sclerosis. 


A biopsy was taken of one of the lesions. The 
report shows perivascular infiltrates of 
lymphocytes affecting white and gray matter. 
There was minimal myelin loss. CSF 
examination shows raised white cell count, 
and oligoclonal bands were not raised. 

What is the most likely diagnosis? 

1- Creutzfeldt Jakob disease 

2- Progressive multifocal leucoencephalopathy 

3- CNS lymphoma 

4- Glioma 

5- HSV encephalitis 



[ Q: 636 ] MRCPass - Neurology 

A 50 year old man has developed 
unsteadiness. He has hypertension and is a 
smoker of 20/day. On examination he has a 
cerebellar ataxia and pass pointing. His CXR 
shows a right hilar mass. 


Which of the Following is most likely to reveal 
the diagnosis? 


1- Anti GM1 antibodies 


2- Anti Yo antibodies 


3- Phytanic acid levels 

4- Serum copper 


Answer & Comments 

Answer: 3- CNS lymphoma 

The white matter lesions suggest either 
multiple sclerosis or lymphoma. Infiltration 
with lymphocytes in this type of presentation 
makes a primary lymphoma of the central 
nervous system likely. 


5- Serum ferritin 

Answer & Comments 

Answer: 2- Anti Yo antibodies 

The most likely diagnosis is a paraneoplastic 
syndrome. Anti Yo antibodies are found in 
around half of all patients with paraneoplastic 
cerebellar degeneration. Associated with small 
cell carcinoma, ovarian tumours and Hodgkin's 


Dr. Khalid Yusuf El-Zohry - Sohag Teaching Hospital (01118391123) 

Ref MRCPass OE OE 2012 PasTest 2009 PassMedicine 2009 PasTest Exam ReviseMRCP 



281 




































El-zohry MRCP Questions Bank (Port 1) - 2013 


(For my personal use) 


lymphoma. Anti Hu antibodies are associated 
with small cell carcinoma of the lung. It is 
usually associated with sensory neuropathy or 
with encephalomyelitis. 

[ Q: 637 ] MRCPass - Neurology 

A 65 year old man has early signs of 
cognitive impairment. His wife describes 
urinary incontinence and unsteadiness on 
walking in the past six months. Examination 
reveals ataxia and he has an MTS score of 
7/10. 



[ Q: 638 ] MRCPass - Neurology 

A 40 year old man has a 4 week 
history of dizziness, and vomitting. The onset 
was acute. He feels that the world was 
spinning and his balance is poor. His hearing is 
normal. There is no familiy history. On 
examination, he has no cerebellar signs. 

Whot is the most likely diagnosis? 

1- Benign positional vertigo 

2- Friedrich's ataxia 

3- Acoustic neuroma 



What is the most likely diagnosis? 


4- Vestibular neuronitis 


1- Lewy body disease 

2- Alzheimer's dementia 

3- Creutzfeldt jakob disease 

4- Normal pressure hydrocephalus 


5- Vertebro basilar circulation insufficiency 


Answer & Comments 


Answer: 4- Vestibular neuronitis 


5- Shy drager syndrome 


Answer & Comments 

Answer: 4- Normal pressure hydrocephalus 

Normal pressure hydrocephalus (NPH) is a 
clinical symptom complex characterized by 
abnormal gait, urinary incontinence, and 
dementia. Ventricular enlargement occurs out 
of proportion on the CT scan. Surgical CSF 
shunting remains the main treatment 
modality. 



Vestibular neuronitis often presents with 
acute vestibular disturbance that gradually 
resolves over a few weeks. It is associated 
with viral infections. Treatment is conservative 
and supportive. 


[ Q: 639 ] MRCPass - Neurology 

A 62 year old man complains of 
headache and on examination of his visual 
fields you detect a right upper quadrantopia. 

Where is the lesion? 

1- Optic chiasm 

2- Right temporal lobe 

3- Right parietal lobe 

4- Left parietal lobe 

5- Left temporal lobe 



Answer & Comments 

Answer: 5- Left temporal lobe 

Upper quadrantopia is due to temporal lobe 
lesions and lower quadrantopia to parietal 
lobe lesions. The visual field is caused by the 
contralateral lesion. 


Dr. Khalid Yusuf El-Zohry - Sohag Teaching Hospital (01118391123) 

Ref MRCPass OE OE 2012 PasTest 2009 PassMedicine 2009 PasTest Exam ReviseMRCP 



282 





























El-zohry MRCP Questions Bank (Port 1) - 2013 


(For my personal use) 


^ [ Q: 640 ] MRCPass - Neurology 

tl - 

# A 65 year old man presents with 
muscle weakness and difficulty swallowing. On 
examination, he has proximal and distal upper 
and lower limb weakness. There is wasting of 
the intrinsic muscles of the fingers and of his 
thigh muscles. CK is elevated and EMG 
findings are consistent with a myopathic 
process. 

Which condition is likely? 

1- Polymyositis 

2- Dermatomyositis 

3- Inclusion body myositis 

4- Motor neuron disease 

5- Duchenne muscular dystrophy 


associated with nauusea. She mentions a 
blackout a few weeks prior to admission. 

On examination of the patient's fundus, the 
optic cups were filled and the medial margins 
of the discs were blurred. 

Which is the most likely cause of this patient's 
condition? 

1- Cerebellar tumour 

2- Frontal lobe tumour 

3- Temporal lobe epilepsy 

4- Benign intracranial hypertension 

5- Migraine 

Answer & Comments 

Answer: 2- Frontal lobe tumour 


Answer & Comments 


Answer: 3- Inclusion body myositis 


The distribution of the muscles involved are 
typical of inclusion body myositis, although 
the distribution is usually asymmetric. In this 
condition, dysphagia and respiratory 
involvement can also occur. Muscle biopsy 
shows intracellular inclusions (amyloid 
precursor protein, ubiquitins) and 
inflammatory infiltrates. 



[ Q: 641 ] MRCPass - Neurology 

A 65 year old female is admitted 
history of severe headaches. The 
headaches are worse in the morning and are 



with a 


The history of headaches which are worse in 
the morning and accompanied by nausea 
suggests increased intracranial pressure. This 
is confirmed by the fact the optic discs have 
blurred margins. The history of a probable 
seizure with collapse, in combination with all 
the other factors suggests a likely frontal lobe 
tumour. 



Frontal Lobe Tumour 


[ Q: 642 ] MRCPass - Neurology 

# A 40 year old woman presents with 
gradual onset severe headache and visual 
blurring. On examination, she has bilateral 
papilloedema. 



Dr. Khalid Yusuf El-Zohry - Sohag Teaching Hospital (01118391123) 

Ref MRCPass OE OE 2012 PasTest 2009 PassMedicine 2009 PasTest Exam ReviseMRCP 






























El-zohry MRCP Questions Bank (Port 1) - 2013 


(For my personal use) 


Which of the Following may help improve the 
condition? 

1- Vitamin A 

2- Prednisolone 

3- Minocycline 

4- Cyclosporin 

5- Acetazolamide 


Answer & Comments 

Answer: 5- Acetazolamide 

Vitamin A, prednisolone, minocycline and 
cyclosporin make idiopathic intracranial 
hypertension (used to be known as benign 
intracranial hypertension worse. 

Acetazolamide is used to treat IIH. IIH is likely 
to be due to high pressure caused by the 
buildup or poor absorption of cerebrospinal 
fluid in the subarachnoid space surrounding 
the brain. The disorder is most common in 
women between the ages of 20 and 50. 
Symptoms include headache, nausea, 
vomiting, and pulsating intracranial noises, 
closely mimicking symptoms of brain tumors. 



[ Q: 643 ] MRCPass - Neurology 

A 35 year old woman has been 
admitted to hospital for investigation of 
progressive weakness in her legs. For the past 
5 years. The patient's mother has similar 
difficulties with weakness and sensory 
problems. 


Examination revealed power of 3/5 distally in 
the upper and lower limbs with a glove and 
stocking pattern sensory loss to pain and 
touch. 


What is the likely diagnosis? 

1- Hereditary neuropathy with liability to 
pressure palsies 

2- Friedrich's ataxia 

3- Chronic inflammatory demyelinating 
polyneuropathy 


4- Multiple sclerosis 

5- Hereditary sensori motor neuropathy 


Answer & Comments 

Answer: 5- Hereditary sensori motor 

neuropathy 

In view of the family history, this patient is 
most likely to have hereditary sensori motor 
neuropathy type I (Charcot Marie Tooth 
disease). 

HMSN 1 is the most common form of 
hereditary neuropathy. Severely and 
uniformly slowed nerve conduction velocities 
(NCVs) and primary hypertrophic myelin 
pathology with prominent onion bulbs and 
secondary axonal changes are the hallmarks of 
the disease. Motor symptoms predominate 
over sensory symptoms. Often, patients 
report loss of balance, muscle weakness, and 
foot deformities. Onset in the first decade of 
life is typical, but disease develops in some 
patients in young or mid adulthood. 

HMSN 2, on the other hand, represents the 
nondemyelinating neuronal type with 
relatively normal NCVs and primary axonal 
pathology. Although nerves are not enlarged 
in the neuronal form, weakness often is less 
marked and onset of this neuropathy is 
delayed. Peripheral nerves are not enlarged 
clinically, and weakness of feet and leg 
muscles predominates; hands are less severely 
affected than the legs. Patients experience 
sensory loss in the distal extremities, and foot 
deformities (ie, pes cavus) tend to be less 
marked than those of HMSN 1. 


[ Q: 644 ] MRCPass - Neurology 

An 80 year old lady is admitted 
complaining of acute onset weakness in both 
legs and low back pain for the previous 4 
weeks. She has lost 5 kg of weight recently, 
and has a Hb of 8.5 g/dl. Her cranial nerves 
and upper limbs were normal but she has 
reduced power in both lower limbs of 3/5. She 



Dr. Kholid Yusuf El-Zohry - Sohog Teaching Hospital (01118391123) 

Ref MRCPass OE OE 2012 PasTest 2009 PassMedicine 2009 PasTest Exam ReviseMRCP 

























El-zohry MRCP Questions Bank (Port 1) - 2013 


(For my personal use) 


has a sensory level at T12 and plantar reflexes 
are upgoing. Rectal tone was normal. 

Which is the best next investigation? 

1- Bone scan 

2- CT scan of the abdomen 

3- MRI scan of the spine 

4- CT scan of the brain 

5- MRI of the brain 


Answer & Comments 

Answer: 3- MRI scan of the spine 

This patient is likely to have cord compression 
from a metastases to the spine. 

Urgent MRI scan is required to confirm the 
diagnosis and a referral for either surgery or 
radiotherapy can then be made. 


choreiform movements and cognitive 
impairment. Cerebellar signs are more typical 
in Friedrich's ataxia. 

[ Q: 646 ] MRCPass - Neurology 

A 60 year old man develops 
swallowing difficulties and complains of 
difficulty walking up stairs. 

On examination, there is weakness and 
wasting of the distal muscles of the arms. 
There are visible fasciculations. Reflexes are 
brisk and the plantars are upgoing. There are 
no sensory abnormalities. Speech and 
language assessment suggests the presence of 
pharyngeal muscle weakness. 

What is the most likely diagnosis? 

1- Guillain Barre 

2- Motor neuron disease 



[ Q: 645 ] MRCPass - Neurology 

A 30 year old man had a progressive 
four month history of stiffness, tremors and 
unsteadiness. There is no family history of a 
similar problem. 

On examination, there is globally increased 
tone. Power is normal and there are brisk 
reflexes throughout. A resting tremor and 
some abnormal writhing movements in the 
arms were observed. He has an MMSE score 
of 25/30. 

What is the most likely diagnosis? 

1- Motor neuron disease 

2- Polymyalgia rheumatica 

3- Friedrich's ataxia 

4- Wilson's disease 

5- Temporal arteritis 



Answer & Comments 

Answer: 4- Wilson's disease 

Neuroacanthocytosis, Wilson's, Huntington's, 
paraneoplastic syndrome can present with 


3- Myasthenia gravis 

4- Chronic inflammatory demyelinating 
polyneuropathy 

5- Paraneoplastic syndrome 


Answer & Comments 

Answer: 2- Motor neuron disease 

This is a classical presentation of motor 
neuron disease. There is a pseudobulbar palsy 
in association with fasciculations of the 
muscles and a mixture of upper and lower 
motor neuron signs. 



[ Q: 647 ] MRCPass - Neurology 

A 35-year-old female was admitted 
to the hospital after experiencing a sudden 
and severe episode of headache. 


No precipitating factor was identified. Except 
for mild neck stiffness, general physical and 
neurological examinations were 

unremarkable. 


On admission, computed tomography (CT) 
scan showed a thin collection of blood on the 


Dr. Khalid Yusuf El-Zohry - Sohag Teaching Hospital (01118391123) 

Ref MRCPass OE OE 2012 PasTest 2009 PassMedicine 2009 PasTest Exam 




ReviseMRCP 

285 


























El-zohry MRCP Questions Bank (Port 1) - 2013 


(For my personal use) 


right sylvian cistern, establishing the diagnosis 
of subarachnoid haemorrhage. 

The admitting doctor is considering further 
management. 

Which of the Following medications have been 
shown to decrease the incidence of cerebral 
infarction in patients with subarachnoid 
haemorrhage? 

1- Amlodipine 

2- Prednisolone 

3- Nimodipine 

4- Acetazolamide 

5- Bendrofluazide 


Answer & Comments 

Answer: 3- Nimodipine 

Nimodipine (calcium channel blocker) has 
been shown to improve outcome in patients 
with subarachnoid haemorrhage. It is given 
60mg orally, 4-hourly. 



[ Q: 648 ] MRCPass - Neurology 

A 28 year old woman is assessed for 
easy fatigue. She complains of weakness of 
her hands causing poor coordination and 
double vision. This is worst in the evenings. 


What is the diagnosis? 


1- Guillain barre syndrome 

2- Multiple sclerosis 

3- Myasthenia gravis 


4- Paraneoplastic syndrome 

5- Central pontine myelinolysis 


Answer & Comments 

Answer: 3- Myasthenia gravis 

Myasthenia gravis is an acquired autoimmune 
disorder characterised by weakness, typically 
of the periocular, facial, bulbar, and girdle 
muscles. Diplopia, ptosis and slurring of the 


speech are common symptoms. There is 
typically fatiguabulity on examination, and 
symptoms are worse during the evenings. It is 
associated with serum IgG antibodies to 
acetylcholine receptors in the postsynaptic 
membrane of the neuromuscular junction. 


[ Q: 649 ] MRCPass - Neurology 

A 35 year old lady has developed 
progressive weakness in the hands and feet 
over a week. There is some associated 
numbness and tingling in the hands and feet. 
She complains of breathing difficulty. She had 
several episodes of diarrhoea two weeks ago. 

On examination, there is slurring of speech. 
Her reflexes are all absent and there is sensory 
loss in a glove and stocking pattern. 

Which is the next appropriate step? 

1- MRI 

2- Lumbar puncture 

3- Vital capacity 

4- EMG 

5- Anti Ach antibody 



Answer & Comments 

Answer: 3- Vital capacity 

Guillain Barre syndrome is the likely diagnosis. 
As she is complaining of breathlessness, vital 
capacity measurement would be essential 
because of potential rapid deterioration in 
respiratory function. 



[ Q: 650 ] MRCPass - Neurology 

A 40 year old patient has been 
diagnosed with migraine recently. The 
migraines seem to have increased in 
frequency since being on a triptan. 


What is the next medication to use? 


1- Propanolol 

2- Tramadol 

3- Carbamazepine 


Dr. Khalid Yusuf El-Zohry - Sohag Teaching Hospital (01118391123) 

Ref MRCPass OE OE 2012 PasTest 2009 PassMedicine 2009 PasTest Exam ReviseMRCP 



286 





























El-zohry MRCP Questions Bank (Port 1) - 2013 


(For my personal use) 


4- Neurofen 

5- Aspirin 


Answer & Comments 

Answer: 1- Propanolol 

In migraines, beta blockers, calcium channel 
blockers and antidepressants may be helpful 
in prophylaxis. 



Broca 


Wernicke 


Left Side View 


[ Q: 651 ] MRCPass - Neurology 

A 50 year old man presents with 
speech difficulty. He has difficulty with word 
identification and repetition. He is able to pick 
out correct objects when instructed to do so 
and is not confused. 

Where is the lesion? 



[ Q: 652 ] MRCPass - Neurology 

A 60 year old man has presented 
with an episode of transient visual loss lasting 
for 5 minutes. He maintained consciousness 
throughout. A CT scan did not show any 
changes. He mentions that he is driving 
currently. 



1- Superior frontal lobe 

2- Parietal lobe 

3- Medial superior temporal lobe 

4- Posterior, superior temporal lobe 
(Wernicke's area) 

5- Inferior frontal lobe (Broca's area) 

Answer & Comments 

Answer: 5- Inferior frontal lobe (Broca's area) 

Broca's Area (Inferior frontal lobe) damage 
causes impaired fluency, intact 
comprehension, impaired repetition 
[expressive dysphasia]. 

Wernicke's Area (posterior, superior temporal 
lobe) damage causes normal fluency, impaired 
comprehension, impaired repetition 
[receptive dysphasia]. 


How long is a patient required to avoid driving 
a car Following recovery from a single 
transient ischaemic attack? 

1- 6 weeks 

2- 1 month 

3- 3 months 

4- 6 months 

5- 1 year 

Answer & Comments 

Answer: 2-1 month 

Following a TIA, a patient should not drive for 
1 month. If there were recurrent TIAs, then 
the patient should be symptom free for 3 
months before driving. 

Following a seizure with altered conscious 
level, a patient should not drive for 1 year. All 
cases should be reported to the Driver Vehicle 
Licensing Agency (DVLA). 



[ Q: 653 ] MRCPass - Neurology 

A 40 year old man complained of 
severe headache. Following this, he then 
developed nausea and vomiting and came to 


Dr. Khalid Yusuf El-Zohry - Sohag Teaching Hospital (01118391123) 

Ref MRCPass OE OE 2012 PasTest 2009 PassMedicine 2009 PasTest Exam ReviseMRCP 



287 






























El-zohry MRCP Questions Bank (Port 1) - 2013 


(For my personal use) 


A+E to be assessed. A CT scan demonstrated 
blood in the left sylvian fissure. 


Abductors (DAB) and Palmar: Adductors 
(PAD). 


Which of the Following is the most likely 
couse? 

1- Migraine 

2- Subdural haematoma 

3- Sagittal sinus thrombosis 

4- Carotid dissection 

5- Berry aneurysm rupture 


Answer & Comments 

Answer: 5- Berry aneurysm rupture 

The history is consistent with a subarachnoid 
haemorrhage, caused by rupture of a berry 
(saccular) aneurysm. 


[ Q: 654 ] MRCPass - Neurology 

A 46 year old man had suffered from 
classical seropositive rheumatoid arthritis for 
10 years. He has never received treatment 
with phenylbutazone, gold, or antimalarials. 
intense pain in the right hand. Over a course 
of 3 weeks, he noticed pain, numbness, and 
tingling on the lateral aspect of the palm and 
in the fourth and fifth fingers of the right 
hand. On examination, he has weakness in the 
abductors and adductors of the fingers, and 
his hand is held in a clawed position. 

Whot is the diagnosis? 

1- Common peroneal nerve lesion 

2- Lateral cutaneous nerve lesion 

3- Radial nerve lesion 

4- Median nerve lesion 

5- Ulnar nerve lesion 



A lesion in the anterior interosseous nerve 
results in weakness of the terminal phalanges 
of the thumb and index fingers. 



Claw Hand 


[ Q: 655 ] MRCPass - Neurology 

A 65 year old man has recently been 
involved in an accident. On examination, there 
was increased tone in the right leg, he had 
right sided leg weakness and loss of 
proprioception on the right side of the lower 
limb. There was also left sided loss of 
sensation to pain on the left leg. 

Whot is the diagnosis? 

1- Myalgic encephalitis 

2- Motor neuron disease 

3- Brown Sequard syndrome 

4- Creutzfeldt Jakob disease 

5- Subacute combined degeneration 
syndrome 



Answer & Comments 


Answer: 3- Brown Sequard syndrome 


Answer & Comments 

Answer: 5- Ulnar nerve lesion 

The ulnar nerve supplies the lumbricals (claw 
hand) and also the Interossei- Dorsal: 


Brown Sequard or hemisection of the spinal 
cord causes in loss of sensation to pain and 
temperature contralateral to the lesion, with 
ipsilateral loss of sensation for position and 
vibration, and upper motor neuron paralysis 
ipsilateral to the lesion. 


Dr. Kholid Yusuf El-Zohry - Sohog Teaching Hospital (01118391123) 

Ref MRCPass OE OE 2012 PasTest 2009 PassMedicine 2009 PasTest Exam ReviseMRCP 



288 





























El-zohry MRCP Questions Bank (Port 1) - 2013 


(For my personal use) 


[ Q: 656 ] MRCPass - Neurology 

A 38 year old lady has difficulty 
walking. On examination, she has an ataxic 
gait, increased tone in the legs with 
hyperreflexia and motor weakness. 

What diagnostic investigation should be 
arranged? 

1- CT scan of the brain and spine 

2- MRI of brain and spine 

3- Lumbar spine X ray 

4- EMG 

5- EEG 



Answer & Comments 

Answer: 2- MRI of brain and spine 

The diagnosis is likely to be multiple sclerosis. 
MRI would be useful to assess for 
demyelinating lesions. Following this lumbar 
puncture is also helpful to confirm the 
presence of oligoclonal bands (when 
compared to serum). 


^ [ Q: 657 ] MRCPass - Neurology 

/i - 

A 18 year old man has difficulty in 
walking. He has had problems with vision 
which were difficult to correct with glasses. 

On examination, there is nystagmus and pallor 
of the optic discs. He has a dysarthria and 
intention tremor. There are absent reflexes in 
the knees and ankles. Plantars are equivocal. 
Pes cavus was noted. 

What is the most likely diagnosis? 

1- Spinocerebellar ataxia 

2- Friedrich's ataxia 

3- Alcoholic cerebellar degeneration 

4- Wilson's disease 

5- Hereditary sensorimotor neuropathy 


Answer & Comments 

Answer: 2- Friedrich's ataxia 

Friedrich's Ataxia is an autosomal recessively 
inherited. Patients may have cardiomyopathy 
and diabetes. Neurological abnormalities 
include optic atrophy and retinitis pigmentosa, 
nystagmus, cerebellar disease and signs, loss 
of dorsal column sensation and weakness. Pes 
cavus (high arched feet) is usually present. 



Pes Cavus 


[ Q: 658 ] MRCPass - Neurology 

A 65 year old man has been 
admitted Following an episode of confusion. 
He was on two different diuretic drugs on 
admission which were discontinued due to 
hyponatraemia. On day 1, he had a sodium of 
112 mmol/l and on day 3 he had a sodium of 
145 mmol/. He had developed by day 3, 
spastic quadriparesis and pseudobulbar palsy. 

What is the likely diagnosis? 

1- Multiple sclerosis 

2- Tuberculous meningitis 

3- Motor neuron disease 

4- Central pontine myelinolysis 

5- Glioma 



Answer & Comments 

Answer: 4- Central pontine myelinolysis 

The upper motor neuron features and rapid 
correction of hyponatraemia suggests central 


Dr. Khalid Yusuf El-Zohry - Sohag Teaching Hospital (01118391123) 

Ref MRCPass OE OE 2012 PasTest 2009 PassMedicine 2009 PasTest Exam ReviseMRCP 



289 




























El-zohry MRCP Questions Bank (Port 1) - 2013 


(For my personal use) 


pontine myelinolysis. The neurological 
features may improve gradually but certain 
patients have permanent neurological deficits 
from rapid changes in serum sodium. 



[ Q: 659 ] MRCPass - Neurology 

A 40 year old man complains of pain 
in the right eye, which had worsened over the 
last few days. 


On examination, he had a ptosis of right eye. 
He also had weakness of superior upgaze of 
the right eye and loss of accommodation 
reflex. 


Which of the Following is likely? 

1- Myasthenia gravis 

2- Neurosyphilis 

3- Multiple sclerosis 

4- Posterior communicating artery aneurysm 

5- Myotonic dystrophy 


Answer & Comments 

Answer: 4- Posterior communicating artery 
aneurysm 

Patients with painful, isolated third nerve 
palsy with pupillary involvement is most likely 
due to a posterior communicating artery 
aneurysm. Other causes of mononeuritis 
multiplex (diabetes, vasculitis, syphilis), can 
also cause a third nerve palsy, but seldom are 
painful. 



Arteriogram - Lateral view showing a posterior 
communicating artery aneurysm. 


^ [ Q: 660 ] MRCPass - Neurology 

- 

# A 55 year old man who was a heavy 
smoker in the past complains of arm 
weakness. On examination, there was postural 
hypotension. He has proximal muscle 
weakness with fatiguability and loss of tendon 
reflexes. 

Whot is the likely diagnosis? 

1- Motor neuron disease 

2- Myasthenia gravis 

3- Transverse myelitis 

4- Guillain Barre syndrome 

5- Lambert Eaton myasthenic syndrome 


Answer & Comments 

Answer: 5- Lambert Eaton myasthenic 
syndrome 

In Lambert Eaton myasthenic syndrome, 60% 
of cases are paraneoplastic (small cell lung ca 
is most associated). The clinical features are 
proximal weakness, loss of tendon reflexes 
and autonomic dysfunction. 


[ Q: 661 ] MRCPass - Neurology 

A 60 year old man has a right wrist 
drop. On examination he has an absent triceps 
jerk on the right as well as an area of sensory 
loss over the dorsum of the middle finger on 
the right hand. 

Where is the lesion? 

1- Median nerve 

2- Radial nerve 

3- Brachial nerve 

4- Ulnar nerve 

5- Musculocutaneous nerve 



Answer & Comments 


Answer: 2- Radial nerve 



Dr. Kholid Yusuf El-Zohry - Sohog Teaching Hospital (01118391123) 

Ref MRCPass OE OE 2012 PasTest 2009 PassMedicine 2009 PasTest Exam ReviseMRCP 





























El-zohry MRCP Questions Bank (Port 1) - 2013 


(For my personal use) 


Radial nerve dysfunction involves impaired 
movement or sensation of the back of the arm 
(triceps), the forearm, or the hand caused by 
damage to the radial nerve. There is weakness 
of wrist and elbow extension, and absent 
triceps jerk. 



Wrist Drop 



[ Q: 662 ] MRCPass - Neurology 

A 24 year old tennis player find that 
he is unable to extend his right wrist. On 
examination, there is weakness of the 
extensors of the wrist and digits. 

Which of the Following structures is damaged? 


1- Radial nerve 



Radial Nerve Palsy 


[ Q: 663 ] MRCPass - Neurology 

A 50 year old lady has a painful 
cheek (maxillary area) when chewing. The 
neurologist diagnoses trigeminal neuralgia. 



Which is the best medication to try? 


1- Benztropine 

2- Tramadol 


3- Diclofenac 

4- Propanolol 

5- Carbamazepine 


Answer & Comments 


Answer: 5- Carbamazepine 


2- Musculocutaneous nerve 

3- Median nerve 

4- Medial cord of the brachial plexus 

5- Lateral cord of the brachial plexus 

Answer & Comments 

Answer: 1- Radial nerve 


For trigeminal neuralgia, carbamazepine is the 
first treatment of choice, but baclofen can also 
be tried. 

[ Q: 664 ] MRCPass - Neurology 

A 46 year old woman complains of 
diplopia. On examination, she has double 
vision on looking to the left only. 



The radial nerve may be damaged anyw here 
in its course. It is most commonly affected in 
the upper arm where it winds round the 
humerus and in the extensor muscle 
compartment of the forearm affecting the 
posterior interosseous branch. Radial nerve 
palsy causes weakness of the wrist and elbow 
extension, and weakness of forearm 
supination. Sensory loss on the dorsum of 
hand and forearm may also be present. 


There is failure of adduction in the right eye, 
and nystagmus in the left eye whilst looking to 
the left. 

Which one of the Following is likely? 

1- Superior rectus palsy 

2- Inferior rectus palsy 

3- Lateral rectus palsy 

4- Medial longitudinal fasciculus defect 

5- Inferior cerebellar peduncle lesion 



Dr. Khalid Yusuf El-Zohry - Sohag Teaching Hospital (01118391123) 

Ref MRCPass OE OE 2012 PasTest 2009 PassMedicine 2009 PasTest Exam ReviseMRCP 





























El-zohry MRCP Questions Bank (Port 1) - 2013 


(For my personal use) 


Answer & Comments 

Answer: 4- Medial longitudinal fasciculus 
defect 

Diplopia on eye abduction is likely to be due to 
either a lateral rectus palsy or medial 
longitudinal fasciculus (MLF) defect. This 
scenario fits one of internuclear 
opthalmoplegia (which is due to an MLF 
defect). In this scenario it is a right sided INO 
same side as the eye which fails to adduct. 



Internuclear opthalmoplegia 


[ Q: 665 ] MRCPass - Neurology 

A 35 year old patient with HIV is 
assessed for new onset right arm weakness. 
An MRI scan of his head shows a temporo¬ 
parietal ring enhancing lesion. 

Which is the most likely couse for this? 

1- Lymphoma 

2- Toxoplasmosis 

3- Behcet's disease 

4- CMV 

5- HSV 




Cerebral Toxoplasmosis 


[ Q: 666 ] MRCPass - Neurology 

A 45 year old lady is with a sudden 
onset of severe headache. It came on with a 
'bang'. A CT scan of the brain is reported as 
normal. There are no neurological signs on 
examination. 

The next best management step is to: 

1- Start ergotamine 

2- MRI of the brain 

3- Perform a lumbar puncture immediately 

4- Perform a lumbar puncture 24 hours after 
the onset of headache 

5- Repeat CT scan 24 hours later 



Answer & Comments 

Answer: 2- Toxoplasmosis 

Cerebral toxoplasmosis is the most common 
cause of a ring enhancing lesion causing 
neurological defect in a HIV patient. 


Answer & Comments 

Answer: 4- Perform a lumbar puncture 24 
hours after the onset of headache 

Lumbar puncture should be performed in 
suspected SAH (ideally after 24 hours from 
onset of headache) if the CT scan is not 
diagnostic. The CSF specimen should be 
examined by spectrophotometry for the 
presence of xanthochromia due to the 
presence of oxyhaemoglobin and bilirubin. 



Dr. Khalid Yusuf El-Zohry - Sohag Teaching Hospital (01118391123) 

Ref MRCPass OE OE 2012 PasTest 2009 PassMedicine 2009 PasTest Exam ReviseMRCP 


292 


























El-zohry MRCP Questions Bank (Port 1) - 2013 


(For my personal use) 


[ Q: 667 ] MRCPass - Neurology 

A 40 year old man was injured in a 
road traffic accident. On examination, he is 
unable to extend his right hand at the wrist. 
The triceps jerk is diminished, and there is 
weakness of wrist flexion. A small area of over 
the right middle finger has sensory loss to pain 
and touch. 


Answer & Comments 

Answer: 3- 3rd nerve palsy 

Pilocarpine is a miotic, and all the rest are 
causes of small pupils as well. Although there 
is no mention of ptosis or oculomotor nerve 
palsy, this is the only answ er which fits with a 
fixed dilated pupil 



Where is the lesion? 

1- Radial nerve 

2- Ulnar nerve 

3- C5 root 

4- C7 root 

5- T1 root 


Answer & Comments 

Answer: 4- C7 root 

A C5 root lesion causes weakness in abduction 
of the shoulder and biceps, as well as sensory 
loss in the upper arm. A C7 root lesion causes 
weak wrist extensors and flexors, weak finger 
extensors and sensory loss to middle finger. A 
radial nerve lesion would not involve finger 
flexors. A T1 lesion would cause w eak intrinsic 
hand muscles. 


[ Q: 668 ] MRCPass - Neurology 

A 62 year old woman presents with 
diplopia. On examination, she has a fixed 
dilated pupil on the right side. Direct light 
reflex and accommodation reflex are totally 
absent. 



[ Q: 669 ] MRCPass - Neurology 

A 60 year old man has previously 
diagnosed with pernicious anaemia. He was 
non compliant with medication for several 
years. He now has unsteadiness and difficulty 
walking. 

Which of the Following lesions is likely? 

1- Corpus callosum 

2- Spinothalamic tract 

3- Dorsal columns 

4- Thalamus 

5- Basal ganglia 



Answer & Comments 

Answer: 3- Dorsal columns 

The diagnosis is Subacute Combined 
Degeneration of the cord. The posterior 
columns and corticospinal tracts are 
specifically damaged, but the clinical picture is 
complicated by the early development of 
coexistent peripheral nerve damage. 


Which one of the Following is most likely? 


1- Argyll Robertson pupil 


2- Horner's syndrome 


3- 3rd nerve palsy 

4- Pilocarpine 


5- Uveitis 


Dr. Kholid Yusuf El-Zohry - Sohog Teaching Hospital (01118391123) 

Ref MRCPass OE OE 2012 PasTest 2009 PassMedicine 2009 PasTest Exam 




ReviseMRCP 

293 
































El-zohry MRCP Questions Bank (Part 1) - 2013 


(For my personal use) 


Lateral 

sp*nothaiamfC tract 
(pam. temperature) 



Posterior column 
(touch, pressure, 
vibration. |omt position) 


Gracite Cuneate 
fasciculus fasciculus 


Anterior spmotfiatamic tract 
(touch, pressure) 


1- L2 

2- L3 

3- L4 

4- L5 

5- SI 


Answer & Comments 

Answer: 3- L4 

In contrast, L5 lesion can cause pain radiating 
through the buttock, down the postero lateral 
aspect of the thigh, lateral aspect of calf and 
across the dorsum of the foot. 


[ Q: 670 ] MRCPass - Neurology 

A 25 year old lady presents with two 
month's history of episodic visual loss 
affecting the right eye. Over the last two 
years, she had gained a considerable amount 
of weight. Examination showed bilateral optic 
disc swelling and small retinal haemorrhages 
in the right eye. 

Which diagnosis is likely? 

1- Multiple sclerosis 

2- Grave's opthalmopathy 

3- Retinitis pigmentosa 

4- Benign intracranial hypertension 

5- Kearn Sayre's disease 



Answer & Comments 


Answer: 4- Benign intracranial hypertension 



[ Q: 672 ] MRCPass - Neurology 

A 21 year old man presents with a 
fever and headache. His temperature persists 
and he now complains of a bad headache. He 
is disorientated in casualty where he is 
assessed. During examination, he has a 
generalized seizure. A CT scan is performed 
and it is normal. CSF examination shows a 
protein of 0.3g and 10 white cells, 
predominantly lymphocytes. An EEG showed 
periodic sharp wave activity. 


What is the most likely diagnosis? 

1- Meningococcal meningitis 


2- TB meningitis 

3- HSV encephalitis 

4- Progressive multifocal leucoencephalopathy 


5- Listeria meningitis 


A high BMI is associated with BIH. The 
condition can be worsened by tetracycline and 
oral contraceptive pills. 


Answer & Comments 


Answer: 3- HSV encephalitis 


[ Q: 671 ] MRCPass - Neurology 

A 60 year old lady presents with back 
pain that radiates to the knee and down the 
medial side of the calf towards the medial 
malleolus. 

Which nerve root is affected? 


HSV encephalitis is the most likely - there can 
be disorientation or dysphasia and seizures. 
There is also mild lymphocytosis on the CSF. In 
HSV encephalitis, MRI may show frontal or 
temporal lobe involvement. EEG may show 
periodic sharp wave activity temporally and 
background of focal or diffuse slow ing. 


Dr. Khalid Yusuf El-Zohry - Sohag Teaching Hospital (01118391123) 

Ref MRCPass OE OE 2012 PasTest 2009 PassMedicine 2009 PasTest Exam ReviseMRCP 



294 

































El-zohry MRCP Questions Bank (Port 1) - 2013 


(For my personal use) 



Right temporal lobe involvement in HSV 

encephalitis 


[ Q: 673 ] MRCPass - Neurology 

A 65 year old man is assessed on the 
ward for weakness in his legs. He is an ex 
smoker and drinks 15 units of alcohol in a 
week. His wife mentions that he is confused. 
On examination, his MMSE score is 20/30. He 
has an ataxic gait. There is bilateral pyramidal 
weakness and coordination is impaired. 
Routine blood tests are normal. An MRI scan 
of the head shows diffuse white matter 
changes, more in the cerebellar region than 
the cerebrum. 

Which of these tests would help most in 
confirming the diagnosis? 

1- CSF for oligoclonal bands 

2- CSF for anti Hu and anti Yo antibodies 

3- CSF for TB culture 



of late presentation and is not commonly 
associated with dementia. 


[ Q: 674 ] MRCPass - Neurology 


A 45 year old lady has numbness in 
the right little finger and aspect of the palm 
shown in the picture. The small muscles of the 
hand are weak. 



Which nerve is affected? 

1- Median nerve 

2- Radial nerve 

3- Ulnar nerve 

4- Anterior interosseous nerve 

5- Posterior interosseous nerve 


Answer & Comments 

Answer: 3- Ulnar nerve 

The ulnar nerve innervates the third and 
fourth lumbricals, the interossei and adductor 
pollicis. Sensation is supplied to the fifth finger 
and the ulnar part of the fourth finger. 


4- EEG 

5- EMG 


Answer & Comments 

Answer: 2- CSF for anti Hu and anti Yo 
antibodies 

Anti Hu and anti Yo antibodies would help 
confirm a diagnosis of paraneoplastic 
syndrome. Multiple sclerosis is unlikely in view 


[ Q: 675 ] MRCPass - Neurology 

A 35 year old man has bradykinesia 
and rigidity of his limb movements. He was 
also noticed to have choreiform movements 
when observed. He has a history of poor 
development during the latter part of school 
education. 

What is the most likely diagnosis? 

1- New variant CJD 



Dr. Khalid Yusuf El-Zohry - Sohag Teaching Hospital (01118391123) 

Ref MRCPass OE OE 2012 PasTest 2009 PassMedicine 2009 PasTest Exam 




ReviseMRCP 

295 




























El-zohry MRCP Questions Bank (Port 1) - 2013 


(For my personal use) 


2- Parkinson's disease 

3- Alzhheimer's disease 

4- Normal pressure hydrocephalus 

5- Huntington's disease 


Answer & Comments 

Answer: 5- Huntington's disease 

In Huntington's disease, as the disease 
progresses, chorea coexists with and gradually 
is replaced by dystonia and parkinsonian 
features, such as bradykinesia, rigidity, and 
postural instability. It is also associated with 
cognitive impairment as well as psychiatric 
manifestations. 


^ [ Q: 676 ] MRCPass - Neurology 

- 

# A 65 year old man has had a 6 month 
history of progressive worsening of confusion. 
He had left the cooker on several times and 
accused the neighbours of spying on him. 

On examination, his face is expressionless and 
he has a monotonous speech. Cranial nerve 
are normal. 

Increased tone is present in all 4 limbs. 
Reflexes, power and sensation are all normal. 

What is the most likely diagnosis? 

1- Parkinson's disease 

2- Lewy body dementia 

3- Alzheimer's dementia 

4- Huntington's chorea 

5- Creutzfeldt Jakob disease 


Answer & Comments 


Answer: 2- Lewy body dementia 


[ Q: 677 ] MRCPass - Neurology 

A 74 year old lady lives alone and is 
self caring. She has home help twice a week, 
two days ago, she was found wandering the 
street and appears confused. The ambulance 
crew who brought her to hospital has noticed 
that she has been incontinent of urine. 

On examination, she walks with a wide based 
gait and has an MMSE score of 17 / 30. Her 
temperature is 36.9°C and she has a CRP of 30 
mg/l. 

What is the most likely diagnosis? 

1- Urinary tract infection 

2- Subdural haematoma 

3- Drug overdose 

4- Normal pressure hydrocephalus 

5- Alzheimer's disease 



Answer & Comments 

Answer: 4- Normal pressure hydrocephalus 

Normal pressure hydrocephalus (NPH) is a 
clinical symptom complex characterized by 
abnormal gait, urinary incontinence, and 
dementia. The CRP is not specific and there is 
insufficient evidence for a UTI in this case. 


[ Q: 678 ] MRCPass - Neurology 

A 50 year old man presents with 
weakness in his arms and legs which is worse 
at the end of the day. On examination he has 
bilateral ptosis and decreased reflexes. 

What test will confirm that his muscle 
weakness is due to Myasthenia Gravis rather 
than Lambert Eaton myasthenic syndrome? 

1- Tensilon test 



This patient has parkinsonism, with 
bradykinesia and rigidity. In view of the 
deterioration in mental function and delusion, 
Lew y body dementia is more likely than 
Parkinson's disease. 


2- Trial of pyridostigmine 

3- Trial of prednisolone 

4- EMG 

5- Muscle biopsy 


Dr. Khalid Yusuf El-Zohry - Sohag Teaching Hospital (01118391123) 

Ref MRCPass OE OE 2012 PasTest 2009 PassMedicine 2009 PasTest Exam ReviseMRCP 





























El-zohry MRCP Questions Bank (Port 1) - 2013 


(For my personal use) 


Answer & Comments 

Answer: 4- EMG 

The main differential is myasthenia gravis and 
LEMS. Myasthenia gravis can be differentiated 
from Eaton Lambert myasthenic syndrome by 
electromyography. Repetitive stimulation in 
myasthenia gravis leads to a decrement of 
evoked muscle action potentials, whilst in 
myasthenic syndrome the condition improves 
by repetitive stimulation. 


[ Q: 679 ] MRCPass - Neurology 

A 60 year old patient has been 
confused for 1 week. His wife also mentions 
he had two episodes of tonic clonic seizures. 
He has a recent diagnosis of small-cell lung 
cancer which is treated with radiotherapy. 

What is the diagnosis? 

1- Myasthenia gravis 

2- Meningitis 

3- Paraneoplastic syndrome 

4- Cerebellar stroke 

5- Wilson's disease 



Answer & Comments 

Answer: 3- Paraneoplastic syndrome 

A wide variety of paraneoplastic neurological 
manifestations have been described, but 
parkinsonism is uncommon. Cerebellar 
degeneration, tremor and movement 
disorders, and Lambert Eaton Myasthenic 
syndromes are the commonest. 


^ [ Q: 680 ] MRCPass - Neurology 

A - 

# A patient has marked dizziness and 
unsteadiness during walking. On examination, 
he has a left sided Horner's syndrome and left 
sided weakness. There is loss of sensation to 
pinprick on the right side. 

What is the likely diagnosis? 

1- Left internal capsule infarct 


2- Posterior inferior cerebellar artery 
occlusion 

3- Medullary infarct 

4- Multiple sclerosis 

5- Vertebral artery dissection 


Answer & Comments 

Answer: 2- Posterior inferior cerebellar artery 
occlusion 

Also known as Wallenberg's syndrome, the 
signs are 

vertigo, ipsilateral cerebellar signs and 
weakness, and contralateral sensory loss. 
There is also cranial nerve 

involvement causing dysphagia and dysarthria. 



[ Q: 681 ] MRCPass - Neurology 

A 55 year old man has type 2 
diabetes. He complains of difficulty walking. 
On examination, he is unable to evert his right 
foot. He has intact knee reflexes. There is also 
sensory loss in the lateral aspect of the foot to 
pin prick. 


Which nerve root is involved? 


1- L2 

2- L3 

3- L5 

4- SI 

5- S2 


Answer & Comments 

Answer: 3- L5 

The common peroneal nerve is involved. 
Nerve roots L5 and SI supply the nerve for 
foot eversion. 

The cutaenous sensory supply corresponds to 
the lateral part of the foot. 


Dr. Khalid Yusuf El-Zohry - Sohag Teaching Hospital (01118391123) 

Ref MRCPass OE OE 2012 PasTest 2009 PassMedicine 2009 PasTest Exam 




ReviseMRCP 

297 































El-zohry MRCP Questions Bank (Port 1) - 2013 


(For my personal use) 



[ Q: 682 ] MRCPass - Infectious 
disease 

A 50 year old woman returned from Sri Lanka 
several days ago. She presents to hospital with 
headaches, myalgia and a fever of 38.5°C. 

Examination reveals multiple petechiae. 

Her bloods show Hb 13 g/dl, WCC 11 x 10 9 /L, 
platelet count 70 x 10 9 /L 

Thick and thin films for malaria are negative. 

Which one of the following infections is most 
likely? 

1- Plasmodium malariae 

2- Brucella 

3- Rickettsia 

4- Plasmodium ovale 

5- Dengue fever 


Answer & Comments 

Answer: 5- Dengue fever 

Dengue fever is a condition caused by an RNA 
arbovirus which is common in tropical and 
subtropical areas, particularly India, South 
East Asia and the Pacific. Fevers, joint pains, 
myalgia, rash and retro-orbital pain are 
common. A petechial rash suggests the 
possibility of impending serious sequelae 
(dengue haemorrhagic fever/shock syndrome) 
and indicates that she has been previously 
infected with another serotype. 




[ Q: 683 ] MRCPass - Infectious 
disease 


A 25 year old man presents with middle lobe 
pneumonia. He is allergic to penicillin, so 
cefuroxime is considered for treatment. 


What is the risk of allergy to a cephalosporin 
antibiotic in someone with a penicillin allergy? 

1- 1 in 10000 

2- 1 in 1000 


3- 1 in 100 


4- 1 in 10 

5- All are allergic 


Answer & Comments 

Answer: 4-1 in 10 

The risk of allergy is usually 5-10%. 


j 

A 25 year old man presented to an emergency 
department with a 1-day history of fever, 
headache and myalgia. 

Two weeks before his presentation, he had 
returned from a 10-day trip to Costa Rica, 
where he had injured the sole of his foot on 
coral. After injuring his foot, he had swum in 
freshwater rivers. Thick and thin blood films 
examined at the time for malaria parasites 
were negative. 

What is the likely diagnosis? 

1- Amoebiasis 

2- Leishmaniasis 

3- Schistosomiasis 

4- Leptospirosis 

5- Brucella abortius 



[ Q: 684 ] MRCPass - Infectious 
disease 


Answer & Comments 

Answer: 4- Leptospirosis 

Leptospira species are endemic in feral and 
domestic mammals, reptiles and amphibians. 
Rats and other rodents are the most 
important sources for human infection. This 
usually occurs through contact with urine- 
contaminated soil or water, contact with 
infected animal tissue, or through rat bites. 

Pathogenic leptospiras rapidly invade the 
bloodstream after penetrating skin or mucous 
membranes, and multiply in small blood vessel 
endothelium, resulting in damage and 
vasculitis in major organs. The mortality rate 
ranges from 4% to 10%, and adverse 


Dr. Khalid Yusuf El-Zohry - Sohag Teaching Hospital (01118391123) 

Ref MRCPass OE OE 2012 PasTest 2009 PassMedicine 2009 PasTest Exam ReviseMRCP 






























El-zohry MRCP Questions Bank (Part 1) - 2013 


(For my personal use) 


indicators are dyspnoea, oliguria, raised white 
cell count, abnormalities on ECG, and alveolar 
infiltrates on chest x-ray. 

Oral doxycycline is highly efficacious. 


A 2 year old, was brought to casualty with a 
running nose, sneezing, and slight irritability. 
Her mother said that child was healthy and 
was running around until several days ago. 

No specific treatment was initiated for this 
"mild cold". Two weeks later, her mother 
returns because the child had a cyanotic 
(turned blue) episode during a coughing spell. 
She indicates that the cough appears to be 
worsening, particularly at night. It often 
comes in spasms and she hears "gasping" 
sound after a coughing spell, and he 
frequently vomits after coughing. 

On examination, he appears mildly 
dehydrated but not distressed. His body 
temperature is 37.2°C. His chest is clear and 
his abdominal examination is normal. A full 
blood count reveals leucocytosis and a marked 
lymphocytosis. 

What is the likely diagnosis? 

1- Whooping cough 

2- Tuberculosis 

3- Epiglottitis 

4- Pneumococcal pneumonia 

5- Varicella pneumonia 



[ Q: 685 ] MRCPass - Infectious 
disease 


Answer & Comments 

Answer: 1- Whooping cough 

Whooping cough (pertussis) is caused by the 
gram negative bacterium Bordetella pertussis. 
Infection is characterised by paroxysms of 
coughing (with a 'w hoop'). Lymphocytosis is 
commonly seen. 


3 

A 45 year old HIV positive man presents with 
breathlessness. He has a temperature of 38°C. 
The blood gases show a pH of 7.30, p0 2 of 
8kPa and pC0 2 of 3 kPa. CXR shows bilateral 
interstitial and alveolar consolidation. 

Which of the following medications should be 
used? 

1- Quadruple anti TB therapy 

2- Amphotericin 

3- Co-trimoxazole 

4- Gentamicin 

5- Teicoplanin 


[ Q: 686 ] MRCPass - Infectious 
disease 


Answer & Comments 

Answer: 3- Co-trimoxazole 

The patient has pneumocystis carinii 
pneumonia and is hypoxic on the blood gases. 
IV co-trimoxazole, clindamycin or pentamidine 
can be used to treat this. 


[ Q: 687 ] MRCPass - Infectious 
disease 

A 42 year old woman is referred to A&E with 
bilateral leg weakness 6 weeks after returning 
from a holiday in Eastern Europe. She also 
complains of general malaise. 

On examination she appeared unwell and was 
had a temperature of 38.8°C. She has cervical 
lymphadenopathy. 

Her pharynx was inflamed with areas of 
exudate on the pharyngeal wall. 

What is the most likely diagnosis? 

1- Campylobacter infection 

2- Staphylococcal sore throat 

3- Infectious mononucleosis 

4- Hodgkin's disease 

5- Diphtheria 



Dr. Khalid Yusuf El-Zohry - Sohag Teaching Hospital (01118391123) 

Ref MRCPass OE OE 2012 PasTest 2009 PassMedicine 2009 PasTest Exam ReviseMRCP 





























El-zohry MRCP Questions Bank (Port 1) - 2013 


(For my personal use) 


Answer & Comments 

Answer: 5- Diphtheria 

Diphtheria is an acute disease caused by 
Corynebacterium diphtheriae, still more 
common in Eastern Europe. The diagnosis is 
clinical but can also be confirmed by culture of 
the organism from a throat swab. 

Diphtheria anti-toxin should be given 
intramuscularly if there is any clinical 
suspicion of diphtheria. Penicillin or 
erythromycin should be given for a week. 



3- Bartonella 

4- Streptococcus viridans 

5- Coxsackie virus 

Answer & Comments 

Answer: 5- Coxsackie virus 

The clinical presentation is consistent with 
myocarditis. Myocarditis can present with 
pleuritic chest pains as well as troponin rise 
and T wave inversion on the ECG. The 
commonest organism is coxsackie virus 
(serology can measure this). Other causes are 
influenza, rubella, diphtheria, typhoid and 
tuberculosis. 


A 25 year old student returns from Ghana with 
fevers and nocturnal sweats. She has a blood 
film showing 5.5% of red blood cells infected 
with plasmodium falciparum. She was 
commenced on intravenous quinine due to 
the high parasite count. 

Which one of the following is a well known 
side effect of quinine therapy? 



[ Q: 689 ] MRCPass - Infectious 
disease 


Diphtheria 


1- Diarrhoea 


[ Q: 688 ] MRCPass - Infectious 
disease 

A 40 year old lady presents with fevers and 
chest pains. The chest pains are worse when 
she takes a deep breath. Her temperature is 
38.9°C, BP 105/70 mmHg and pulse 110. An 
ECG shows diffuse T wave inversion. An 
echocardiogram shows mild tricuspid 
regurgitation, no vegatations on any valves 
and no pericardial effusion. Her troponin T is 
0.3 ng/mL (normal <0.05). 

Which of the following organisms is most 
likely? 

1- Respiratory syncytial virus 

2- Legionella 


2- Hypoglycaemia 

3- Psychosis 

4- Lipodystrophy 

5- Deranged liver function 

Answer & Comments 

Answer: 2- Hypoglycaemia 

Quinine can be given orally to treat falciparum 
malaria. Intravenous infusion of quinine is 
usually indicated only in severe cases (e.g. 
cerebral malaria). 

Hypoglycaemia is an important side effect of 
quinine therapy. Glucose should be monitored 
in those having intravenous quinine. 



Dr. Khalid Yusuf El-Zohry - Sohag Teaching Hospital (01118391123) 

Ref MRCPass OE OE 2012 PasTest 2009 PassMedicine 2009 PasTest Exam ReviseMRCP 




























El-zohry MRCP Questions Bank (Port 1) - 2013 


(For my personal use) 


[ Q: 690 ] MRCPass - Infectious 
disease 

A 25 year old man developed progressive 
pneumonia which is responding poorly to 
amoxycillin. Methenamine silver staining of 
his sputum showed small circular cysts. 
Giemsa staining demonstrated small, punctate 
nuclei of trophozoites and intracystic 
sporozoite. 

Which is the likely organism? 

1- Mycobacterium tuberculosis 

2- Leishmania donovani 

3- Pneumocystis carinii 

4- Toxoplasma gondii 

5- Trypanosoma cruzi 



Answer & Comments 

Answer: 3- Pneumocystis carinii 

Pneumocystis carinii may be identified on 
microscopy after 

(a) methenamine silver staining which shows a 
cystic phase of the organism 

(b) Giemsa staining which demonstrates 
sporozoites and trophozoites with small, 
punctate nuclei. 

[ Q: 691 ] MRCPass - Infectious 
disease 

A 75 year old woman presents with a Two day 
history of confusion. She had complained to 
her husband of a headache and neck stiffness. 
A lumbar puncture was performed. CSF 
microscopy revealed: 

WBC 650 cells/mL (< 5) 

90% neutrophils 

A few Gram positive diplococci on staining 
Whot is the likely cause of her meningitis? 

1- Listeria monocytogenes 

2- Streptococcus pneumoniae 



3- Neisseria meningitidis 

4- HIV 

5- Herpes simplex 


Answer & Comments 

Answer: 2- Streptococcus pneumoniae 

Streptococci are gram positive cocci. 
Pneumococcal meningitis is commoner in 
older patients. Neisseria meningitidis are gram 
negative diplococci, whilst listeria 
monocytogenes is a small gram positive 
bacillus. 



[ Q: 692 ] MRCPass - Infectious 
disease 

A 42 year old lady presented with cough, 
shortness of breath and confusion. She was 
admitted to hospital with a diagnosis of 
pneumonia. Pulse oximetry showed an oxygen 
saturation of 86 percent on air. Her chest x- 
ray showed increased markings at the base of 
the right lung. 

A sputum culture grew normal flora, and Two 
blood cultures showed no growth. Paired sera 
taken on showed raised antibody titres to 
Chlamydophila antigens (from <1:10 to 1:320). 

What is the diagnosis? 

1- Mycoplasma pneumoniae 

2- Legionaire's disease 

3- Byssinosis 

4- Psittacosis 

5- Pulmonary eosinophilia 


Answer & Comments 

Answer: 4- Psittacosis 

Psittacosis is caused by chlamydia psittaci. It is 
spread by all sorts of birds, not just pigeons or 
parrots. Children are less predisposed than 
adults to the disease. The disease does spread 
from person to person. Treatment is with 
tetracycline. 


Dr. Khalid Yusuf El-Zohry - Sohag Teaching Hospital (01118391123) 

Ref MRCPass OE OE 2012 PasTest 2009 PassMedicine 2009 PasTest Exam ReviseMRCP 































El-zohry MRCP Questions Bank (Port 1) - 2013 


(For my personal use) 


^ Q: 693 ] MRCPass - Infectious 
I disease 

A 45 year old lady is being vaccinated 
following a recent relative being diagnosed 
with meningitis. 

Vaccines exist in the UK for which major 
groups of meningococcus? 

1- B 

2- A and C 

3- B and C 

4- A and B 

5- A, B and C 


Answer & Comments 

Answer: 2- A and C 

The vaccine for A has been used for travellers 
for some years, and the vaccine for C has been 
introduced more recently for high risk groups. 



[ Q: 694 ] MRCPass - Infectious 
disease 


A 18 year old student is admitted with a 
history of headaches, lethargy, anorexia and 
vomiting of a few weeks duration. She had 2 
witnessed seizures. Over the last few months 
she has been treated for oral thrush. 


On examination she is febrile, drowsy and she 
has neck stiffness. CT scan of her head is 
unremarkable. 


CSF shows: 


4- Herpes simplex virus 

5- Mycobacterium tuberculosis 


Answer & Comments 

Answer: 3- Cryptococcus neoformans 

The patient has chronic meningitis with raised 
CSF lymphocytosis. 

Immunosuppression is suggested by oral 
candidiasis. The likely cause is cryptococcal 
meningitis. 


[ Q: 695 ] MRCPass - Infectious 
disease 

A 40 year old man has symptoms of diplopia, 
dysphagia and dysarthria. After 12 hours he 
develops weakness of his arms but remains 
afebrile and is alert. 

Following this, in the next 12 hours, he 
deteriorates to the extent of requiring 
artificial ventilation. 

What is the likely diagnosis? 

1- Diphtheria 

2- Polio 

3- Strychnine poisoning 

4- Tetanus 

5- Botulism 



Answer & Comments 


Answer: 5- Botulism 


White cells 80 /ml (65% lymphocyte, 35 % 
polymorphs) 

Protein 1.2g/L 

Glucose 2.5 mmol/L 

Which organism is the likely cause of the 
condition? 

1- Meningococcus 

2- Streptococcus 

3- Cryptococcus neoformans 


Botulism typically produces a descending 
paralysis which starts with diplopia or blurred 
vision due to difficulty with accommodation 
and progresses to weakness of the neck, arms 
and respiratory muscles. Botulism is caused by 
the neurotoxins of Clostridium botulinum and 
in rare cases, Clostridium butyricum and 
Clostridium baratii. These gram-positive 
spore-forming anaerobes can be found in soil 
samples and marine sediments throughout 
the world. Therapy consists of approximately 
10,000 IU of antibodies against toxin types A, 


Dr. Khalid Yusuf El-Zohry - Sohag Teaching Hospital (01118391123) 

Ref MRCPass OE OE 2012 PasTest 2009 PassMedicine 2009 PasTest Exam ReviseMRCP 





























El-zohry MRCP Questions Bank (Port 1) - 2013 


(For my personal use) 


B, and E to neutralize serum toxin 
concentrations, and also supportive care (e.g. 
ventilation). 

Tetanus and strychnine poisoning both 
produce muscle spasm which may lead to 
respiratory failure but not muscle weakness. 


A 35 year old secretary returns from Mumbai 
in India following a 2 week holiday. He had 
headaches, arthralgia, myalgia, 

diarrhoea/contipation and was mildly 
confused. Malarial film was negative. 

What is the likely diagnosis? 

1- HIV 

2- Syphilis 

3- Schistosomiasis 

4- Plasmodium ovale malaria 

5- Typhoid fever 

Answer & Comments 

Answer: 5- Typhoid fever 

Headaches, arthralgia, myalgia, 

diarrhoea/contipation and confusion or 
delirium can occur in typhoid fever (caused by 
S typhi) which should be suspected in 
travellers with pyrexia. Jaundice does not 
occur. 



[ Q: 696 ] MRCPass - Infectious 
disease 



Salmonella typhi 


In 

•i 

i j 

A 22 year old man who is asymptomatic has 
been referred by the GP for investigation of 
lymphadenopathy. 

On examination, he has palpable cervical 
lymph nodes. The Chest X ray is normal. 

Which one of the following tests should be 
done? 

1- Fine needle aspiration and cytology 

2- Excision biopsy 

3- CT scan of the head and neck 

4- Bone marrow biopsy 

5- Genetic karyotyping 


[ Q: 697 ] MRCPass - Infectious 
disease 


Answer & Comments 

Answer: 1- Fine needle aspiration and cytology 

As the patient is asymptomatic, there is little 
clue towards a diagnosis. A needle aspiration 
and cytology is essential to exclude lymphoma 
/ carcinoma /TB. Other possible causes are 
infection e.g. HIV or infectious mononucleosis. 



[ Q: 698 ] MRCPass - Infectious 
disease 


A 40 year old man presented to hospital with 
fever, intermittent rigors and lethargy. He had 
returned from a holiday to West Africa six 
months previously. 


What is the likely diagnosis? 

1- Plasmodium falciparum malaria 


2- Plasmodium ovale malaria 


3- Typhoid fever 

4- Brucellosis 


5- Leishmaniasis 


Answer & Comments 


Answer: 2- Plasmodium ovale malaria 


Dr. Khalid Yusuf El-Zohry - Sohag Teaching Hospital (01118391123) 

Ref MRCPass OE OE 2012 PasTest 2009 PassMedicine 2009 PasTest Exam ReviseMRCP 

































El-zohry MRCP Questions Bank (Port 1) - 2013 


(For my personal use) 


Plasmodium ovale malaria and its cyclic 
paroxysms occur every 48 hours (tertian 
fever). Plasmodium ovale is the rarest of the 
four species and is apparently more restricted 
in distribution. 

However, it is common in the West African 
countries of Ghana, Liberia, and Nigeria. 



[ Q: 699 ] MRCPass - Infectious 
disease 

A 25 year old veterinarian presents with a 2 
week history of high fevers, night sw eats, dry 
cough, and myalgia. 

He had no medical history and was taking no 
regular drug treatment. A travel and 
occupational history showed that he had lived 
and worked as a vet in northern India until 6 
months ago, when he moved to London. 

On examination, the patient had a fever of 
38.5°C, a solitary cervical lymph node 
measuring 1 cm x 0.5 cm in size, and a 
palpable splenic tip. 

Initial investigations showed a CRP of 84 U/l 
and white cell count of 4.8 x 10 9 /L. Three 
blood films for malaria parasites were 
negative. Standard liver function tests showed 
a raised serum alkaline phosphatase 
concentration of 520 U/l, a raised (gamma)- 
glutamyltransferase concentration of 450 U/l, 
and a raised serum aspartate 
aminotransferase concentration of 248 U/l); 
the serum bilirubin concentration was at the 
upper end of the normal range, at 18 pmol/l. 
The serum angiotensin converting enzyme 
concentration was also raised, at 113 U/l. 

Which one of the following tests is most likely 
to yield the diagnosis? 

1- Chlamydia serology 

2- Mycoplasma serology 

3- Brucella serology 

4- Legionella serology 

5- Paul bunnell test 


Answer & Comments 

Answer: 3- Brucella serology 

The occupation suggests that the patient had 
worked with animals e.g. cattle. Brucella is 
transmitted through milk and meat, especially 
in abbatoirs. The commonest cause is Brucella 
melitensis. Detection of brucella may require 
extended culture of 6 weeks and blood agar 
plates. Detection of Brucella agglutinins (with 
the Coomb's test) also helps confirm the 
diagnosis. 

Fever and rigors, followed by possible 
osteomyelitis, polyarthritis, endocarditis, 
pneumonia, hepatitis/jaundice, splenic 
abscess, meningitis/encephalitis, skin changes, 
orchitis/cervicitis and retinitis. 

Past infection causes positive serology and 
does not necessarily indicate active infection. 
Tetracycline and gentamicin are treatments of 
choice (better intracellular penetration). 


Brucella melitensis - Gram-negative, aerobic, 

coccobacillus 


j 

A 31-year-old white male presented to a 
general practitioner with a tw o-day history of 
fever, headaches, generalised aches and 
pains, lethargy and loss of appetite. He had 
travelled to Papua New Guinea 2 months ago. 

A full blood count showed: lymphocytes, 0.71 
x 10 9 /L; nneutrophils, 1.95 x 10 9 /L; and 
platelets, 33 x 10 9 /L; normal, 150-450 x 10 9 
/L). Haemoglobin concentration was in the 
normal range. Blood film shows schizonts. 

Which one of the following is the most likely 
organism? 

1- Schistosoma 



[ Q: 700 ] MRCPass - Infectious 
disease 


Dr. Khalid Yusuf El-Zohry - Sohag Teaching Hospital (01118391123) 

Ref MRCPass OE OE 2012 PasTest 2009 PassMedicine 2009 PasTest Exam 




ReviseMRCP 

305 























El-zohry MRCP Questions Bank (Port 1) - 2013 


(For my personal use) 


2- Mycobacterium tuberculosis 

3- Pasteurella 


Alkaline phosphatase: 120 U/l 
HbsAg positive 


4- Plasmodium vivax 


IgG anti-HBc positive 


5- Leptospira 


Answer & Comments 


Answer: 4- Plasmodium vivax 

The history and FBC count is most consistent 
with malaria. In this case, vivax malaria. 



Plasmodium vivax trophozoites 



[ Q: 701 ] MRCPass - Infectious 
disease 


During routine medical evaluation, a 30 year 
old man was found to have abnormal liver 
tests. A history failed to identify a clinically 
recognized, past episode of acute hepatitis. 
The patient had never received a blood 
transfusion. He had not used injection drugs. 
He had had unprotected sexual encounters 
with multiple partners over several years. 


On physical examination, he had no signs of 
chronic liver disease. On receiving the 
following results from the laboratory, the GP 
referred him to a liver specialist. 

Laboratory Results: 


ALT: 180 
AST: 140 IU 


Bilirubin: 15 umol/l 


HbeAg positive 

HBV DNA 1.5 x 10 7 copies/ml 

A liver biopsy showed moderately active 
hepatitis with a necroinflammatory histologic 
activity index score of 10/18 and a fibrosis 
score of 3/6. 

What should be done? 

1- Commence inferferon alpha 

2- Commence prednisolone 

3- Commence ribavirin 

4- Observation for several months 

5- Refer for liver transplant 


Answer & Comments 

Answer: 4- Observation for several months 

A case of hepatitis B with mildly deranged liver 
function tests. There are signs of hepatitis 
which are mild, and the liver function tests are 
not significantly deranged, hence observation 
of inflammatory activity is recommended. 


A 35 year old man is very unw ell. He has had 
a headache for 2 weeks and this is now 
associated with neck stiffness and 
photophobia. On examination, he has 
temperature of 38.5°C, white lesions on the 
tongue and axillary lymphadenopathy. 
Fundoscopy is normal. CXR was normal. An 
enhanced CT head scan is also normal. 

Blood tests show lymphopenia. 

What is the most likely diagnosis? 

1- Secondary syphillis 

2- Herpes simplex encephalitis 

3- Cerebral toxoplasmosis 


[ Q: 702 ] MRCPass - Infectious 
disease 



Dr. Khalid Yusuf El-Zohry - Sohag Teaching Hospital (01118391123) 

Ref MRCPass OE OE 2012 PasTest 2009 PassMedicine 2009 PasTest Exam ReviseMRCP 



306 





























El-zohry MRCP Questions Bank (Port 1) - 2013 


(For my personal use) 


4- Disseminated candidal infection 

5- Cryptococcal meningitis 


Answer & Comments 


Answer: 5- Cryptococcal meningitis 


The major clinical syndromes caused by the 
genus Leishmania are cutaneous, mucosal and 
visceral leishmaniasis. Leishmaniasis is 
transmitted by Phlebotomus or Lutzomyia 
sandflies and infects dogs and foxes as well as 
humans. 


Lymphopenia suggests HIV infection and this 
patient also has candidal infection in the oral 
cavity suggesting immunodeficiency. Hence 
cryptococcal meningitis (which usually 
presents in an indolent manner) is the most 
appropriate answer. 



[ Q: 703 ] MRCPass - Infectious 
disease 

A 22 year old man had been to a Greek island 
on holiday 2 months ago. He presented with a 

2-w eek history of dry cough, diarrhoea, 
lethargy, anorexia and fever. He had a past 
history of cutaneous vasculitis of uncertain 
aetiology. He was not taking any regular 
medication. 

On examination, the patient's temperature 
was 37.5°C, but no other abnormalities were 
evident. Investigations showed that he had 
pancytopenia, raised erythrocyte 

sedimentation rate and 

hypergammaglobulinaemia. 

A second bone marrow biopsy at this time 
revealed occasional macrophages containing 
amastigotes. Further history revealed that he 
had been bitten by sandflies. 

Whot is the diagnosis? 

1- Schistosomiasis 

2- Cutaneous leishmaniasis 

3- Visceral leishmaniasis 

4- Malaria 

5- Dengue fever 


Answer & Comments 


Answer: 3- Visceral leishmaniasis 


Visceral leishmaniasis is caused by Leishmania 
donovani, L. infantum or L. chagasi. 

The incubation period of visceral leishmaniasis 
is usually 2-8 months. Many infections are 
subclinical, but the classic presentation is with 
fever, weight loss, hepatosplenomegaly, 
pancytopenia and hypergammaglobulinaemia. 

The definitive diagnosis depends on 
demonstrating either amastigotes in tissue or 
promastigotes in culture. Splenic puncture is 
the most sensitive means of obtaining a 
diagnosis, but biopsy of the bone marrow and 
liver is almost as good. 

Pentavalent antimony compounds have been 
used to treat leishmaniasis for decades, but 
often have severe side effects, and resistance 
is developing.Amphotericin can achieve 98% 
long-term cure in both 

antimonialunresponsive and previously 
untreated patients. 


A 33 year man chronic hepatitis C admitted 
general deterioration. He missed many of his 
previous outpatient appointments currently 
receiving any treatment. On examination he 
had a temperature of 38°C, and was 
jaundiced. 

His blood tests were: 

Hb 12.0 g/dl 
MCV 90 fl 
WCC 9 x 10 9 /L 
platelets 180 x 10 9 /L 
urea 25 pmol/l 
creatinine 340pmol/l 



[ Q: 704 ] MRCPass - Infectious 
disease 


Dr. Kholid Yusuf El-Zohry - Sohog Teaching Hospital (01118391123) 

Ref MRCPass OE OE 2012 PasTest 2009 PassMedicine 2009 PasTest Exam 




ReviseMRCP 

307 

























El-zohry MRCP Questions Bank (Port 1) - 2013 


(For my personal use) 


sodium 138 mmol/l 
potassium 3.5 mmol/l 
bilirubin 60 pmol/l 
AST 85 U/l 
ALP 350 U/l 
albumin 32 g/l 
CRP 110 mg/I 

Which of the following is likely? 

1- Hepatocellular carcinoma 

2- Acute liver decompensation 

3- Hepatorenal syndrome 

4- Superimposed HIV infection 

5- Hepatitis delta infection 


The diagnosis is non gonococcal urethritis. 
This is commonly due to Chlamydia. 
Treatment of choice is doxycycline. 


A 30 year old man presents to your office after 
passing something he thought was a 
rubberband in his stool. He was worried when 
he saw the object moving in the toilet. He is 
otherw ise healthy and is taking no 
medications. He has had no recent changes in 
bowel habits or stool appearance. He denies 
fever, abdominal pain, cough, or rash. 

Whot is the likely diagnosis? 

1- Hookworm 


[ Q: 706 ] MRCPass - Infectious 
disease 



Answer & Comments 

Answer: 3- Hepatorenal syndrome 

The deranged LFTs and also impaired renal 
function suggests hepatorenal syndrome. 
Hepatitis delta (HepD) infection is 
superimposed with Hepatitis B infection. 


/*■' 

t 



[ Q: 705 ] MRCPass - Infectious 
disease 


A 19 year old female has a several day history 
of urethral discharge. Mid stream urine is 
negative. A swab was sent but no organisms 
were grown. 


Which one of the following should be 
prescribed? 


1- Penicillin V 


2- Trimethoprim 

3- Metronidazole 


4- Doxycycline 

5- Ciprofloxacin 


2- Schistosomiasis 

3- Ascariasis 

4- Leishmaniasis 

5- Echinococcus 


Answer & Comments 

Answer: 3- Ascariasis 

Ascaria lumbricoides is roundw orm which 
infects the ileum and may cause Gl symptoms. 
It also causes pneumonitis and bronchospasm 
when the larvae migrate via the bloodstream 
to the alveoli. Once mature they crawl back up 
the bronchi into the gut. Mebendazole, 100 
mg orally tw ice daily for 3 days or 500 mg 
orally once, is the treatment of choice. 



Answer & Comments 


Answer: 4- Doxycycline 


Ascaris lumbricoides 


Dr. Kholid Yusuf El-Zohry - Sohog Teaching Hospital (01118391123) 

Ref MRCPass OE OE 2012 PasTest 2009 PassMedicine 2009 PasTest Exam 




ReviseMRCP 

308 


























El-zohry MRCP Questions Bank (Port 1) - 2013 


(For my personal use) 


[ Q: 707 ] MRCPass - Infectious 
disease 

A 40 year old single man returned 
from holiday in Europe with bloody diarrhoea 
which had lasted for Two weeks. 



He had lost 1 kg in weight, and has occasional 
low er abdominal cramping discomfort. He 
also has a painful sw elling in his right elbow. 

Which is the likely couse? 

1- Bacillus cereus 


The patient follow ed Two but not three-step 
commands. 

A provisional diagnosis of an aphasic temporal 
lobe lesion was made. A CT scan showed a low 
attenuation lesion involving the medial and 
posterior aspect of the left temporal lobe and 
inferior basal ganglia. 

Which of the following is likely? 

1- Polymorpho leukoencephalopathy 

2- Cerebral toxoplasmosis 


2- E coli diarrhoea 

3- Gonococcal septicaemia 

4- Amoebiasis 


3- Herpes simplex encephalitis 

4- Multiple sclerosis 

5- Meningococcal meningitis 


5- Campylobacter infection 


Answer & Comments 

Answer: 5- Campylobacter infection 

Reactive arthritis, which is the likely cause of 
elbow sw elling in this case, can develop 
following infection with Shigella, Salmonella, 
Campylobacter and Yersinia. All these 
organisms are common in travel to Europe or 
North Africa 

Campylobacter infection is one of the 
commomest causes of inflammatory 
diarrhoea. Abdominal pain is often a 
prominent feature of the illness, frequently at 
the right iliac fossa. The diarrhoea is often 
bloody. Symptoms last for a w eek typically. 

E coli diarrhoea is less likely to be associated 
with a reactive arthritis. 



[ Q: 708 ] MRCPass - Infectious 
disease 


A 62 year old lady presents with fever and 
persistent difficulty in speaking. Her signs 
show a termperature of 39°C. The patient was 
alert and oriented with respect to time but 
unable to name objects properly. Dysarthria 
and occasional word substitution were noted. 


Answer & Comments 

Answer: 3- Herpes simplex encephalitis 

Herpes simplex viruses (HSV-1 and HSV-2) 
produce a variety of infections involving 
mucocutaneous surfaces, the CNS, and 
occasionally visceral organs. HSV encephalitis 
is the most common identified cause of acute, 
sporadic viral encephalitis. Clinically, HSV 
encephalitis presents with acute onset of 
fever and focal neurology, especially temporal 
lobe signs. 


A 18 year old man was admitted to the 
emergency room due to fever, headache, 
vomiting, irritability, and myalgia that had 
begun 24 hours ago. There was no evidence of 
previous infection in the upper airw ays. On 
examination, the patient was lethargic, 
disoriented and hypotensive, with petechiae 
in the legs and upper limbs, and he had 
conjunctival suffusion. 

Cerebrospinal fluid (CSF) was turbid, with 
5300 cells/ mm 3 (97% neutrophils and 3% 
monocytes)and protein was 0.9 (<0.5). 

Whot is the treatment of choice? 

1- Erythromycin 


[ Q: 709 ] MRCPass - Infectious 
disease 



Dr. Kholid Yusuf El-Zohry - Sohog Teaching Hospital (01118391123) 

Ref MRCPass OE OE 2012 PasTest 2009 PassMedicine 2009 PasTest Exam ReviseMRCP 































El-zohry MRCP Questions Bank (Port 1) - 2013 


(For my personal use) 


2- Gentamicin 

3- Ceftriaxone 

4- Metronidazole 

5- Hydrocortisone 


Answer & Comments 

Answer: 3- Ceftriaxone 

The patient has meningococcal meningitis 
(suggested by the purpuric rash). Until the 
organism (Neisseria meningitidis) is isolated 
and sensitivities tested, the patient should be 
on a cephalosporin or benzylpenicillin. 



Meningococcal Rash 



[ Q: 710 ] MRCPass - Infectious 
disease 


A 25 year old lady presents to A&E with a 3 
day history of headache and fever. On 
examination, she has neck stiffness and a 
fever but no rash or focal neurology. She is 
given IV ceftriaxone and a lumbar puncture is 
performed. 


CSF analysis shows: 

Protein 0.35 g/L 
Glucose 2/3 of plasma level 


Microscopy shows 250 white cells, 
predominantly lymphocytes 

The most likely diagnosis is: 


1- Listeria meningitis 

2- Herpes simplex meningitis 

3- Tuberculous meningitis 


4- Meningococcal meningitis 


5- Enteroviral meningitis 


Answer & Comments 

Answer: 5- Enteroviral meningitis 

The CSF is suggestive of a viral as opposed to a 
bacterial meningitis. 

Enteroviruses are the commonest cause of 
viral meningitis. The next best option would 
be herpes simplex meningitis. 



[ Q: 711 ] MRCPass - Infectious 
disease 


A 30 year old Turkish woman arrived in the UK 
with a 6 month history of weight loss and 
intermittent fevers. On examination, the 
patient was febrile, an enlarged liver and 
spleen were palpable. 


Investigations revealed: 
Haemoglobin 7.7 g/dL 
White cell count 2.7 x 10 9 /L 
Platelet count 95 x 10 9 /l_ 


Thick and thin films showed no malarial 
parasites 

Whot is the likely diagnosis? 

1- Kawasaki's disease 


2- Falciparum malaria 

3- Visceral leishmaniasis 


4- HIV infection 

5- Tuberculosis 


Answer & Comments 

Answer: 3- Visceral leishmaniasis 

The causative agent of visceral leishmaniasis is 
Leishmania donovani. 

Fever, malaise, anaemia, weakness and weight 
loss are common presenting symptoms and 
signs. 


Dr. Khalid Yusuf El-Zohry - Sohag Teaching Hospital (01118391123) 

Ref MRCPass OE OE 2012 PasTest 2009 PassMedicine 2009 PasTest Exam ReviseMRCP 






























El-zohry MRCP Questions Bank (Port 1) - 2013 


(For my personal use) 


Hepatosplenomegaly develops gradually and 
also the skin develops a grey colour, giving rise 
to the Indian name of the disease -'kala azar' - 
meaning black fever. 



[ Q: 712 ] MRCPass - Infectious 
disease 


A 35 year old man is admitted with a 5 day 
history of fever, cough and lethargy. 


On examination, he was pyrexial and had signs 
consistent with a left low er lobe pneumonia. 


In considering antibiotics for pneumonias , 
which one of the following microorganisms is 
sensitive to Benzylpenicillin? 


1- Klebsiella 


2- Mycoplasma pneumoniae 

3- Aspergillus fumigatus 

4- Streptococcus Pneumoniae 

5- Legionella pneumophila 


increased protein of 1.2 mg/dL, cell count of 5 
(all lymphocytes). She gradually recovers over 
the next month. 

Which of the following conditions is likely to 
have preceded the onset of her illness? 

1- Staphylococcus sepsis 

2- CLL 

3- Campylobacter infection 

4- Folate deficiency 

5- Behcet's syndrome 


Answer & Comments 

Answer: 3- Campylobacter infection 

The overall diagnosis is Guillain Barre 
syndrome, which causes peripheral sensori 
motor neuropathy which is ascending. In 
about half of all cases the onset of the 
syndrome follows a viral or bacterial infection, 
such as the following: 


Answer & Comments 

Answer: 4- Streptococcus Pneumoniae 

Penicillin is mainly useful for Group A and 
Group B Streptococci, meningococcal and 
pneumococcal infections. Strep pneumonia is 
one of the commonest organisms causing 
community acquired bacterial pneunomia. 


flu 

gastrointestinal viral infection 
infectious mononucleosis 
viral hepatitis 
Campylobacter 
porphyria 



[ Q: 713 ] MRCPass - Infectious 
disease 


A 45 year old woman develops a progressive, 
ascending motor weakness over several days. 
1 week ago she had been non specifically 
unwell with fevers, diarrhoea and a dry cough. 
Neurological examination revealed peripheral 
low er motor neuron weakness and also 
sensory loss to pain and touch. 


She required intubation and ventilation 
following rapid deterioration from admission. 
A CT scan of her brain is normal. A lumbar 
puncture is performed. The opening pressure 
was normal. There was clear, colorless CSF, 


An elevated level of protein in the CSF is 
characteristic of GBS. Nerve conduction 
studies may show changes consistent with 
demyelination. 


[ Q: 714 ] MRCPass - Infectious 
disease 

An 78 year old presents with confusion 
associated with a chest infection. 4 days later 
she developed green, then bloody diarrhoea. 

Which of following organisms is likely to cause 
the diarrhoea? 

1- Methicillin resistant Staphylococcus aureus 



Dr. Khalid Yusuf El-Zohry - Sohag Teaching Hospital (01118391123) 

Ref MRCPass OE OE 2012 PasTest 2009 PassMedicine 2009 PasTest Exam ReviseMRCP 
































El-zohry MRCP Questions Bank (Port 1) - 2013 


(For my personal use) 


2- Salmonella 

3- Clostridium difficile 

4- Escherichia coli 0157 

5- Campylobacter jejuni 


Answer & Comments 

Answer: 3- Clostridium difficile 

This is a clinical scenario of Clostridium 
infection causing pseudomembranous colitis 
predisposed to by prior treatment broad 
spectrum antibiotics such as cefuroxime or 
augmentin. 

[ Q: 715 ] MRCPass - Infectious 
disease 

A 25 year old man presented to the hospital 
10 days after returning from a six month visit 
to Pakistan. He had complained of fever, 
rigors and headache. On examination he was 
febrile (38°C) and his abdomen was tender in 
the right upper quadrant. 

Investigations showed: 

Hb 10.0 g/dL 

WBC 14.5 x 10 9 /L 

Neutrophils 12.5 x 10 9 /l 

Platelets 370 x 10 9 /L 

Blood film - No malaria parasites seen 

Aik Phos 480 iU/L 

AST 60 iU/L 

CRP 110 mg/L 

Stool culture Negative 

Chest xray: Small right pleural effusion 

Which of the following investigations would be 
of diagnostic value? 

1- OGD 



5- Stool for ova, cysts and parasites 

Answer & Comments 

Answer: 3- Ultrasound scan of the liver 

The presenting features of RUQ pain and 
pyrexia suggests a possible amoebic liver 
abscess, hence an ultrasound of the liver to 
look for it (this may be amenable to aspiration 
and would confirm the diagnosis). Amoebic 
cysts may also be found in the stool, 
microscopy would be the next test to be done. 



[ Q: 716 ] MRCPass - Infectious 
disease 

A 17 year old Sudanese boy was evaluated for 
generalized pruritus and an eruption on his 
legs. He complained of episodic decreased 
visual acuity and eye pain every few months. 
He denied fever, weight loss, or lethargy, and 
his medical history was otherw ise 
unremarkable. 

A skin examination revealed diffuse xerosis 
and lichenified, asteatotic patches distributed 
over the knees and pretibia and on the dorsal 
aspect of the feet. The affected skin was shiny 
and atrophic in areas. There were also subtle 
hypopigmented, xerotic patches over the 
buttocks. 

The peripheral blood cell count was normal 
except for 30% eosinophilia (normal, <5.5%). 
The eye examination (slitlamp and indirect 
ophthalmoscopy) showed bilateral corneal 
infiltrates. 

What is the likely diagnosis? 

1- Babesiosis 

2- Trypanosomiasis 

3- Onchocerciasis 

4- Yellow fever 


2- Typhoid serology 

3- Ultrasound scan of the liver 

4- Blood cultures 


5- Herpes simplex infection 


Answer & Comments 


Answer: 3- Onchocerciasis 


Dr. Khalid Yusuf El-Zohry - Sohag Teaching Hospital (01118391123) 

Ref MRCPass OE OE 2012 PasTest 2009 PassMedicine 2009 PasTest Exam ReviseMRCP 



312 





























El-zohry MRCP Questions Bank (Port 1) - 2013 


(For my personal use) 


Onchocerciasis (river blindness) is a common, 
chronic, multisystemic disease caused by the 
nematode Onchocerca volvulus. The disease 
characteristically includes dermatologic, 
lymphatic, ophthalmologic, and systemic 
manifestations. Human transmission of the 
disease is caused by a bite from the 
intermediate host, the black fly (genus 
Simulium). Treatment is with ivermectin. 



Leopard-spot pattern of depigmentation on 
the shins in onchocerciasis 


[ Q: 717 ] MRCPass - Infectious 
disease 

A 38 year old woman has just returned from a 
2 month holiday in East Africa. She mentions 
that she had been swimming in fresh water. 
She now has fevers, breathlessness and is 
wheezy. 

On examination, she has a generalized rash 
and palpable hepatosplenomegaly. 

Her investigations show : Hb 12.5 g/dl, WCC 
14.5 x 10 9 /L, platelet count 190 x 10 9 /L. Thick 
and thin malarial films are negative. 

What is the likely diagnosis? 

1- Schistosomiasis 

2- Ankylostoma infection 

3- Leishmaniasis 

4- Amoebiasis 

5- Infectious mononucleosis 



Answer & Comments 

Answer: 1- Schistosomiasis 

Acute schistosomiasis can cause fevers, an 
urticarial rash, diarrhoea, 

hepatosplenomegaly (Katayama fever). 

The clue here is sw imming. The larvae of 
Schistosoma (known as cercariae) breed in 
snails and are present in rivers and lakes. They 
can penetrate human skin, and from there on 
can migrate to organs such as lung or liver. 

A rash known as 'swimmer's itch' or cercarial 
dermatitis can occur. Pulmonary involvement 
can lead to wheeze and breathlessness. 

Leishmaniasis is spread by sandflies - there 
may be hepatosplenomegaly, but there is 
usually no pulmonary involvement. 



Schistosomal egg from a tissue biopsy 



[ Q: 718 ] MRCPass - Infectious 
disease 


A 22 year old man presents with pus 
discharging from the urethra. Microscopy of a 
sample of the pus showed neutrophils. There 
were no organisms seen on gram stain. 


Which of the following organisms is most 
likely? 

1- Ureaplasma urealyticum 


2- Chlamydia trachomatis 

3- Trichomonas vaginalis 


4- Candidiasis 


5- Neisseria gonorrhoeae 


Dr. Khalid Yusuf El-Zohry - Sohag Teaching Hospital (01118391123) 

Ref MRCPass OE OE 2012 PasTest 2009 PassMedicine 2009 PasTest Exam ReviseMRCP 


























El-zohry MRCP Questions Bank (Port 1) - 2013 


(For my personal use) 


Answer & Comments 

Answer: 2- Chlamydia trachomatis 

A patient who presents with urethral 
discharge has either gonococcal or non¬ 
gonococcal urethritis (NGU). Since there were 
no gram negative diplococci seen on 
microscopy, it is NGU. 


What is the most likely diagnosis? 

1- Primary Syphilis 

2- Infectious mononucleosis 

3- Mycoplasma pneumonia 

4- Wegener's granulomatosis 

5- Parvovirus B19 infection 


The organism are as follows: 

■ Chlamydia trachomatis 40% 
Ureaplasma urealyticum 20-40% 
Trichomonas vaginalis (rare) < 2% 
Candidasis (rare) < 2% 

■ Herpes simplex (rare) < 2% 

The most common organism is Chlamydia and 
hence the best answer. 



Chlamydia inclusions (intracellular parasites) 


A 32 year old woman presents with a fever, 
headache, cough, malaise and arthralgia. On 
examination she has a w idespread maculo- 
papular rash but no other findings. CXR is 
normal. 

Investigations show : Hb 13.6 g/dl, WCC 11 x 
10 9 /L, platelets 220 x 10 9 /L, sodium 138 
mmol/I, potassium 4.5 mmol/l, urea 5 pmol/l, 
creatinine 110 pmol/l, CRP 115. 

Cold agglutinin IgM antibody 1:64 

FT-Antibodies negative 

VDRL serology positive 


[ Q: 719 ] MRCPass - Infectious 
disease 


Answer & Comments 

Answer: 3- Mycoplasma pneumonia 

Parvovirus B19 infection is unlikely with the 
high CRP and syphilis is unlikely due to the 
negative FTA-Antibodies. Mycoplasma 
infection is a recognised cause of false positive 
VDRL. Cold agglutinins also suggest M. 
pneumonia IgM antibodies. 


a 

I J 

A group of holiday makers have been to the 
river Nile on a cruise holiday in Egypt. Many of 
the tourists develop bloody diarrhoea a few 
days later. 

What is the likely organism? 

1- Cryptosporidium 

2- Entamoeba Histolytica 

3- Shigella sonnei 

4- Mycobacterium bovis 

5- Giardia Lamblia 


[ Q: 720 ] MRCPass - Infectious 
disease 


Answer & Comments 

Answer: 3- Shigella sonnei 

This is a smallprint fact but there is a past year 
question based on travel to the Nile. 

Salmonella, Campylobacter and shigella are 
common causes of bloody diarrhoea among 
people travelling in the region of Nile River 
Delta of Egypt. E coli is also a cause of 
diarrhoea, but not typically bloody. 


Dr. Khalid Yusuf El-Zohry - Sohag Teaching Hospital (01118391123) 

Ref MRCPass OE OE 2012 PasTest 2009 PassMedicine 2009 PasTest Exam ReviseMRCP 




























El-zohry MRCP Questions Bank (Port 1) - 2013 


(For my personal use) 


Shigella are non-motile gram-negative bacilli 
that can cause diarrheal illness in humans. 
Symptoms of a Shigella infection include 
diarrhoea, fever, and abdominal cramps that 
begin 24 to 48 hours after exposure. The 
diarrhoea is often bloody. 


[ Q: 721 ] MRCPass - Infectious 
disease 

A 65 year old man had been on holiday to 
Arizona in the united states 6 weeks ago. He 
was brought to hospital w ith high fever, 
rigors, malaise, and mild confusion. He had a 
generalised, non-pruritic maculopapular rash, 
predominantly on the trunk but also on the 
extremities, including the palms and soles. 
There was no history of animal or arthropod 
exposure, but his house was on the edge of 
forest. 

Whot is the most likely diagnosis? 

1- Falciparum malaria 

2- Rickettsial spotted fever 

3- Tuberculosis 

4- Allergic bronchopulmonary aspergillosis 

5- Schistosomiasis 



Answer & Comments 

Answer: 2- Rickettsial spotted fever 

Rocky Mountain or Rickettsial spotted fever 
(RMSF), classically characterized by fever, 
myalgias, headache, and a petechial rash, is 
the most common tick-borne disease in the 
United States. It is also found in Mexico and 
South America, southern Africa, and Asia. 

Recommended treatment is doxycycline. 



Maculopapular rash due to Rickettsial disease 


3L 

A 35 year old man presented to hospital with 
a 7 day history of progressive severe thoracic 
back pain and a 5 day history of left knee pain. 
The back pain had occurred suddenly after 
lifting weights, and radiated to the right chest 
wall. His family history in relation to 
rheumatological or inflammatory disorders 
was unremarkable, and he had never used 
intravenous drugs. He had no symptoms of 
fever, conjunctivitis, urethritis, rash, early 
morning joint stiffness or neurological 
dysfunction. 

On initial examination he looked well but had 
a slightly raised temperature (37.8°C). There 
was tenderness over the fifth to the seventh 
thoracic vertebrae, and his left tibiofibular 
joint was erythematous and warm. 

Initial investigations revealed a neutrophil 
leukocytosis of 9.5 x 10 9 /L (1.7-7.0 x 10 9 /L) 
and raised C-reactive protein, 73 mg/L (< 3 
mg/L) and erythrocyte sedimentation rate, 47 
mm/h (0-10 mm/h). An MRI scan showed 
epidural abscess. 

What is the most likely organism? 

1- Streptococcus pyogenes 

2- Staphylococcus aureus 

3- Pasteurella multicoda 

4- Enterococcus faecalis 

5- E coli 


[ Q: 722 ] MRCPass - Infectious 
disease 


Dr. Khalid Yusuf El-Zohry - Sohag Teaching Hospital (01118391123) 

Ref MRCPass OE OE 2012 PasTest 2009 PassMedicine 2009 PasTest Exam 




ReviseMRCP 

315 

























El-zohry MRCP Questions Bank (Part 1) - 2013 


(For my personal use) 


Answer & Comments 

Answer: 2- Staphylococcus aureus 

Epidural abscesses are most commonly caused 
by staph aureus. Patients typically present 
with either a triad of tenosynovitis, dermatitis 
and polyarthralgia without purulent arthritis, 
or purulent arthritis without skin lesions. For 
the Investigation of symptoms of localised 
back pain and fever, magnetic resonance 
imaging is the investigation of choice. 


Ai 

A 30 year old woman presents with a Two 
week history of severe headache, myalgia and 
a red maculopapular rash. She had returned 
from Malaysia a week ago, where she 
mentions she had been bitten by mosquitoes. 

On examination his blood pressure was 80/45 
mmHg. A diagnosis of dengue fever was likely. 

Which of following is the best treatment 
option? 

1- Intravenous hydrocortisone 

2- Intravenous fluids 

3- Intravenous artemesinin 

4- Ribavirin 

5- Intravenous aciclovir 


[ Q: 723 ] MRCPass - Infectious 
disease 


Answer & Comments 

Answer: 2- Intravenous fluids 

The treatment for Dengue fever is supportive 
treatment with intravenous fluids (either 
crystalloids or colloids). Steroids and antiviral 
drugs have not been proven effective. 
Artemisinin (Chinese wormwood) is used to 
treat malaria. 



[ Q: 724 ] MRCPass - Infectious 
disease 

An 18 year old boy has recently been to 
Pakistan. He now presents to hospital with a 


7-day history of a high fever and vomiting. He 
had a soft palpable spleen. His total leukocyte 
count was 3 x 10 9 /L Malarial parasites were 
not seen on examination of thin and thick 
smears of peripheral blood. 

The results of a routine urinalysis and chest x 
rays were normal. A blood Widal test showed 
a titer of 320 against "O" (somatic) antigen of 
Salmonella enterica serovar Typhi. Blood 
culture yielded the growth of Salmonella 
Typhi. 

What should he be treated with? 

1- Amoxycillin 

2- Metronidazole 

3- Ciprofloxacin 

4- Tetracycline 

5- Gentamicin 


Answer & Comments 

Answer: 3- Ciprofloxacin 

This a case of typhoid fever. The non 
blanching macular rash is known as 'Rose 
Spots'. 

Fluoroquinolones such as ciprofloxacin are the 
most effective antimicrobial agents for 
treating enteric fevers. 



Rose Spots 

^ [ Q: 725 ] MRCPass - Infectious 

jJ[ | disease 

A previously healthy 50 year old man has 
recently been to the Amazon on a jungle 
trekking trip. He returns with a 4-day history 
of crampy abdominal pain and a 1-day history 


Dr. Khalid Yusuf El-Zohry - Sohag Teaching Hospital (01118391123) 

Ref MRCPass OE OE 2012 PasTest 2009 PassMedicine 2009 PasTest Exam ReviseMRCP 






























El-zohry MRCP Questions Bank (Port 1) - 2013 


(For my personal use) 


of fever of 39.3°C and severe headache. He 
mentions he was bitten by mosquitoes 
frequently during the trip. 

On admission, physical examination revealed 
an ill-appearing man. Laboratory tests 
documented leukopenia, anemia (hemoglobin 
10.5 g/dL), thrombocytopenia, abnormal 
coagulation (INR 6.5), renal failure and 
deranged liver function (AST: 13,700 U/L 
[normal: 15-37 U/L]. 

Malarial film and rickettsial serology were 
negative. 

What diagnosis should be considered? 

1- Chagas disease 

2- Malaria 

3- Yellow fever 

4- Visceral leishmaniasis 

5- Schistosomiasis 


Answer & Comments 

Answer: 3- Yellow fever 

Yellow fever (YF) is a mosquitoborne viral 
disease. Although no specific treatment exists 
for YF and the case-fatality rate for severe YF 
is approximately 20%, an effective vaccine is 
available. Symptoms are of sudden onset of 
fever, bradycardia and headache. Severe cases 
progress to intense albuminuria , jaundice, 
and hemorrhage. 


A 65 year old woman has leg cellulits which is 
not responding to antibiotics. Would swab 
now grows MRSA. 

Which would be the best antibiotic regime to 
use in a patient who has spreading methicillin- 
resistant Staphylococcus aureus infection of a 
skin wound? 

1- Gentamicin 

2- Doxycycline 


[ Q: 726 ] MRCPass - Infectious 
disease 



3- Flucloxacillin 

4- Rifampicin 

5- Vancomycin 


Answer & Comments 

Answer: 5- Vancomycin 

Vancomycin should be used first line for MRSA 
with rifampicin added for synergistic action. 
More recently, oral linezolid has also been 
show n to be effective. 


j 

A 30 year old Mexican male presented with 3 
weeks history of fever, malaise, nausea, 
vomiting and right upper quadrant pain. Bow 
els were regular with normal stools. 

On abdominal examination, there was right 
upper quadrant tenderness without rigidity or 
guarding. There was no organomegaly, 
masses, or ascites and bow el sounds were 
normal. 

Liver functions showed elevated alkaline 
phosphatase (152 IU/L) and a low albumin. 

Abdominal CT scan showed a well- 
demarcated abscess in the right lobe of liver. 
Aspirate from the abscess is reddish brow n in 
colour. 

What is the likely diagnosis? 

1- Tuberculous abscess 

2- Malaria 

3- Sarcoidosis 

4- Schistosomiasis 

5- Amoebic abscess 



[ Q: 727 ] MRCPass - Infectious 
disease 


Answer & Comments 

Answer: 5- Amoebic abscess 

Amoebic liver abscess is caused by Entameba 
histolytica.The trophozoites reach the liver via 
portal circulation and block the small portal 


Dr. Khalid Yusuf El-Zohry - Sohag Teaching Hospital (01118391123) 

Ref MRCPass OE OE 2012 PasTest 2009 PassMedicine 2009 PasTest Exam ReviseMRCP 






























El-zohry MRCP Questions Bank (Port 1) - 2013 


(For my personal use) 


radicles leading to inflammation and 
subsequent necrosis. 

The abscess is commonly found in right lobe 
of liver. Aspirate from the abscess is reddish 
brow n "anchovy sauce" and mainly consists 
of degenerate liver cells and blood. 


The most common treatment of amebic liver 
abscess is - Metronidazole 750 mg t.i.d. for 5- 
10 days. 



Amoebic liver abscess in the right lobe of the 
liver 



[ Q: 728 ] MRCPass - Infectious 
disease 


A 70 year old man with chronic renal failure 
for which he receives regular haemodialysis 
treatment, is admitted with fever and 
lethargy. Blood cultures grow a vancomycin- 
resistant enterococcus (VRE). 


Which of the following drugs would be most 
suitable? 


1- Septrin 

2- Meropenem 

3- Enterocid 

4- Linezolid 


5- Vancomycin 


Answer & Comments 

Answer: 4- Linezolid 

Vancomycin-resistant enterococci are an 
emerging problem in hospitals, where 


extensive glycopeptide use may select for 
resistant strains. In noncomparative trials of 
treatment of VRE infections, 
quinupristin/dalfopristin and linezolid are 
effective. Some are teicoplanin sensitive. 



[ Q: 729 ] MRCPass - Infectious 
disease 


A 33 year old male presents with a history of 
dysuria, and penile discharge. He had 
unprotected sexual intercourse with a casual 
partner 3 days prior to the development of 
symptoms. An urethral swab is taken and on 
gram stain this shows numerous polymorphs 
with many intracellular gram-negative 
diplococci. 


The causative organism is: 

1- Treponema pallidum 

2- Ureaplasma urealyticum 


3- Neisseria gonorrhoeae 

4- Chlamydia trachomatis 


5- Trichomonas vaginalis 


Answer & Comments 

Answer: 3- Neisseria gonorrhoeae 

The presence of gram-negative intracellular 
diplococci suggests the diagnosis is gonococcal 
urethritis. Non-gonococcal urethritis (NGU) 
may be due to Chlamydia trachomatis, 
Trichomonas vaginalis, Ureoplasma 
urealyticum, Mycoplasma genitilium and 
Bactroides spp. 



[ Q: 730 ] MRCPass - Infectious 
disease 


A 35 year old man has lymphopenia on his 
white cell differential count. He has been 
complaining of headaches over the last 6 
weeks. 


On examination, he has a temperature of 38.5 
C and no focal neurological signs. A CT scan of 


Dr. Khalid Yusuf El-Zohry - Sohag Teaching Hospital (01118391123) 

Ref MRCPass OE OE 2012 PasTest 2009 PassMedicine 2009 PasTest Exam ReviseMRCP 































El-zohry MRCP Questions Bank (Port 1) - 2013 


(For my personal use) 


his brain showed a 5 cm ring enhancing lesion 
in the frontal lobe. 

Which of the following organisms is likely? 

1- Cryptosporidia 

2- Toxoplasma gondii 

3- Aspergillus 

4- Cryptococcus neoformans 

5- Mycobacterium avium intracellulare 


Answer & Comments 

Answer: 2- Toxoplasma gondii 

The lymphopenia suggests HIV infection. Ring 
enhancing lesions in the brain suggest either 
cerebral toxoplasmosis or lymphoma. 



Ring enhancing lesion in the Basal ganglia 

[ Q: 731 ] MRCPass - Infectious 
disease 

A 28 year old lady present with a solitary, 
crusted, thickened lesion on her leg 6 weeks 
after returning from a holiday in a suburban 
area in Sao Paulo, Brazil. She had no new 
sexual contact whilst there. She thinks she 
might have been bitten by some flies. 

Whot is the likely diagnosis? 

1- Cutaneous anthrax 

2- Syphillis 

3- Erysipelas 

4- Onchocerciasis 

5- Cutaneous leishmaniasis 



Answer & Comments 

Answer: 5- Cutaneous leishmaniasis 

Cutaneous leishmaniasis can be caused by 
several Leishmania species, including L. 
braziliensis, L. mexicana and L. panamensis. It 
is spread by sandflies. The incubation period is 
variable, ranging from 2 weeks to several 
months. 

Lesions usually occur on sunexposed areas. 
Diagnosis can be confirmed with biopsy 
demonstrating amastigotes. 

Treatment is usually with pentavalent 
antimonial drugs (sodium stibogluconate). 

Cutaneous anthrax is unlikely because it 
usually occurs in patients who have contact 
with meat, wool or hides e.g. veterinarians or 
farm workers. Cutaneous onchocerciasis is 
rare. 



Cutaneous leishmaniasis: indolent, slow ly 
healing lesion 



Sandfly 


A 38 year old woman has had multiple 
resections of the bow el on account of 



[ Q: 732 ] MRCPass - Infectious 
disease 


Dr. Khalid Yusuf El-Zohry - Sohag Teaching Hospital (01118391123) 

Ref MRCPass OE OE 2012 PasTest 2009 PassMedicine 2009 PasTest Exam ReviseMRCP 



319 





























El-zohry MRCP Questions Bank (Port 1) - 2013 


(For my personal use) 


recurrent Crohn's disease. This has resulted in 
intestinal failure and she is dependent on 
home parenteral nutrition. She has a central 
line inserted. 

She presents with fever accompanied by chills 
and rigors. No physical signs are 
demonstrable. Cultures taken both centrally 
and peripherally demonstrate the presence of 
methicillin-sensitive Staphylococcus aureus. 

Whot should be done next? 

1- Treatment with intravenous vancomycin for 
14 days 

2- Treatment with intravenous gentamicin for 
14 days 

3- Treatment with intravenous teicoplanin for 
14 days 

4- Remove the intravenous line 


3- Pasteurella multicoda 

4- E coli 

5- Pseudomonas aeruginosa 

Answer & Comments 

Answer: 3- Pasteurella multicoda 

Pasteurella multicoda is an oral commensal of 
domestic pets known to be an opportunistic 
human pathogen after traumatic animal 
contact. The most common infections in 
humans are skin and pulmonary infections. 

Pasteurella meningitis occurs at extremes of 
age (infants), in the immunocompromised 
(associated with liver cirrhosis, renal disease 
and haematological malignancies) and after 
traumatic head injury. 


5- 6 weeks of intravenous flucloxacillin and 
gentamicin 


Answer & Comments 

Answer: 4- Remove the intravenous line 

Line infection with Staphylococcus aureus and 
Candida are absolute indications for line 
removal. 



[ Q: 733 ] MRCPass - Infectious 
disease 


A 2 year old girl presented with a 12-hour 
history of fever and poor feeding. The family 
ow ned Two cats. Her temperature was 
39.5°C, and she was irritable, with no 
localising signs or skin lesions. A full septic 
screen was performed. Cerebrospinal fluid 
(CSF) showed a neutrophilic pleocytosis and 
gram-negative coccobacilli. She was treated 
with intravenous cefotaxime and gentamicin. 
Within 24 hours both CSF and blood cultures 
showed growth of gram-negative bacilli. 


What is the likely infective organism? 


1- Legionella pneumophilia 


2- Haemophilus influenzae 


[ Q: 734 ] MRCPass - Infectious 
disease 

A 34 year old Turkish lady was admitted with 
fever for the past few weeks. On examination 
she was emaciated, with massive 
splenomegaly and hepatomegaly. 

What is the treatment of choice? 

1- Ciprofloxacin 

2- Symptomatic treatment 

3- Pentavalent antimonial 

4- Quinine 

5- Praziquantel 



Answer & Comments 

Answer: 3- Pentavalent antimonial 
The features of leishmaniasis are: 
fever 

hepatosplenomegaly 

pancytopenia 

Leishmaniases are widespread in most 
countries in the Mediterranean basin, 


Dr. Khalid Yusuf El-Zohry - Sohag Teaching Hospital (01118391123) 

Ref MRCPass OE OE 2012 PasTest 2009 PassMedicine 2009 PasTest Exam ReviseMRCP 




























El-zohry MRCP Questions Bank (Port 1) - 2013 


(For my personal use) 


including Turkey. Treatment is with the 
pentavalent antimonial - Sodium 
stibogluconate. 



[ Q: 735 ] MRCPass - Infectious 
disease 

A 40 year old man who is HIV positive has 
been non compliant with antiretroviral 
medication. He presents with progressive 
(over 3 weeks) right sided upper and lower 
limb weakness. MRI scan of the brain shows 
multiple white matter lesions. CSF 
examination showed normal cell count and no 
organisms were seen. These lesions are non¬ 
enhancing. 

Which one of the following is most likely? 

1- Tuberculous infection 


2- Progressive multifocal 

leukoencephalopathy 

3- Cerebral toxoplasmosis 

4- Haemorrhagic stroke 

5- Cryptococcal infection 


Answer & Comments 

Answer: 2- Progressive multifocal 

leukoencephalopathy 

Although all the diagnoses are possible the 
presence of non-enhancing white matter 
lesions suggest JC virus infection (PML). CSF 
PCR reveals JC virus in >90% of cases. If the 
lesions had been enhancing, toxoplasmosis 
would have been the most likely answer and a 
trial of sulphadiazine/pyrimethamine would 
have been indicated. 




[ Q: 736 ] MRCPass - Infectious 
disease 


A 55 year old man presents with confusion. 
Along the way to the hospital, he had a 
witnessed generalised tonic clonic seizure. 


On examination, he had a temperature of 38 C 
and MMSE score of 17/30. There was no focal 
neurolgical signs and he did not have a cardiac 
murmur. The admitting doctor organises a 
lumbar puncture and MRI scan of the brain to 
investigate the cause. 


Which one of the following features would be 
most suggestive of Herpes simplex 
encephalitis? 


1- Occipital change on MRI 


2- Temporal lobe change on MRI 

3- Negative CSF for bacteria and enteroviruses 

4- Normal temperature 


5- Lymphocytosis in CSF 


Answer & Comments 

Answer: 2- Temporal lobe change on MRI 

HSV encephalitis is a rare disease but can 
affect patients who are otherw ise fit and well. 
Neurological signs occur in around a third of 
cases. Fever is alw ays present. Typically the 
temporal lobes are involved on MRI. 

Subtype 1 virus accounts for 95% of cases. 
Brain biopsy is the "gold standard" for 
diagnosis but in practice a +ve PCR for HSV in 
CSF confirms the diagnosis. Treatment is with 
high-dose intravenous acyclovir. 


Large PML lesion in the left frontal lobe 


Dr. Khalid Yusuf El-Zohry - Sohag Teaching Hospital (01118391123) 

Ref MRCPass OE OE 2012 PasTest 2009 PassMedicine 2009 PasTest Exam 




ReviseMRCP 

321 
























El-zohry MRCP Questions Bank (Part 1) - 2013 


(For my personal use) 



Medial Temporal Lobe changes on MRI in HSV 


encephalitis 


A 

A 40 year old African man, has been back to 
Africa for a holiday for several weeks. 6 
months after coming back to the UK, he 
presented with fever and intermittent rigors. 

What is the likely diagnosis? 

1- Plasmodium malaria 

2- Plasmodium falciparum 

3- Plasmodium ovale 

4- Dengue 

5- Trypanosomiasis 


[ Q: 737 ] MRCPass - Infectious 
disease 


Answer & Comments 

Answer: 3- Plasmodium ovale 

Plasmodium ovale produces hypnozoites 
which are latent in the liver. Relapses can 
occur up to 5 years after infection. 
Plasmodium vivax infection may present 
similarly. 



Plasmodium ovale parasite 


[ Q: 738 ] MRCPass - Infectious 
disease 

A 35 year old man with known HIV infection 
presents with a headache and neck stiffness. A 
lumbar puncture w as performed. 

CSF analysis showed: 

Protein 0.55 g 

Glucose 3.3 (serum glucose 6.6) 

White cells : 30 with lymphocytosis 
What is the likely diagnosis? 

1- Listeria meningitis 

2- Tuberculous meningitis 

3- HSV meningitis 

4- Cryptococcal meningitis 

5- Subarachnoid haemorrhage 



Answer & Comments 

Answer: 2- Tuberculous meningitis 

The history of immunosuppression, low 
glucose and lymphocytosis suggests 
tuberculous meningitis. 



[ Q: 739 ] MRCPass - Infectious 
disease 


A 45 year old man complains of general 
malaise. He has just returned from a 2 week 
holiday in Slovakia. On examination he 
appeared was pyrexial and had large palpable 
cervical lymph nodes bilaterally. His pharynx 


Dr. Khalid Yusuf El-Zohry - Sohag Teaching Hospital (01118391123) 

Ref MRCPass OE OE 2012 PasTest 2009 PassMedicine 2009 PasTest Exam 




ReviseMRCP 

322 





























El-zohry MRCP Questions Bank (Port 1) - 2013 


(For my personal use) 


had areas of exudate with a pale grey 
membrane. 

What is the best treatment option? 

1- Penicillin 

2- Tetanus injection 

3- Anti TB therapy 

4- Amphotericin 

5- Teicoplanin 


Answer & Comments 

Answer: 1- Penicillin 

The diagnosis is diphtheria. Diphtheria can 
cause sorethroat with exudate, fever, cervical 
lymphadenopathy and occasionally 
neurological signs of flaccid paralysis. The 
illness is still present in Eastern Europe and 
Russia. Treatment is with diphtheria antitoxin, 
penicillin or erythromycin. 


Southeast Asia, eastern Europe, and South 
America. Cerebral cysticercosis may present as 
epilepsy, meningoencephalitis or focal 
neurological deficit. Subcutaneous 
cysticercosis presents as hard, small, pea-sized 
nodules in the subcutaneous tissues. 
Praziquantel or albendazole with steroid cover 
is the treatment of choice. 


( 

Ja 

A 40 year old man is admitted for 
investigation of fevers having recently come 
back from Peru. The fever is intermittent and 
comes on every 72 hours. He has 
splenomegaly. A Giemsa stained thin blood 
film shows malarial parasites. 

Which species is it likely to be? 

1- Plasmodium vivax 

2- Plasmodium ovale 


[ Q: 741 ] MRCPass - Infectious 
disease 


[ Q: 740 ] MRCPass - Infectious 
disease 

A 42 year old man emigrated to the UK from 
South America 3 years ago. He has long 
standing left hemiparesis and also visual loss 
in the right eye. Several hard subcutaneous 
nodules are noted on several areas on the 
body. 

Which one of the following is the likely 
infection ? 

1- Filariasis 

2- Cysticercosis 

3- Schistosomiasis 

4- Syphilis 

5- Cutaneous tuberculosis 



Answer & Comments 

Answer: 2- Cysticercosis 

Cysticercosis is due to infection with Taenia 
solium and human tissue is invaded by the 
larval form. It commonly is found in Africa, 


3- Plasmodium falciparum 

4- Plasmodium malariae 

5- Plasmodium knowlesi 


Answer & Comments 

Answer: 4- Plasmodium malariae 

Plasmodium malariae causes quartan fever 
(every 72 hours or fourth day) whereas the 
other species cause tertian fever (every 48 
hours or third day). It is found particulary in 
temperate climates. 


A 22 year old woman presented with a 1-day 
history of fever, chills and severe back pain, 
with no other focal symptoms. On 
examination, she was febrile with a blood 
pressure of 75/40 mmHg, and had begun 
vomiting. 

She was treated empirically with intravenous 
ceftriaxone and flucloxacillin and resuscitated 



[ Q: 742 ] MRCPass - Infectious 
disease 


Dr. Khalid Yusuf El-Zohry - Sohag Teaching Hospital (01118391123) 

Ref MRCPass OE OE 2012 PasTest 2009 PassMedicine 2009 PasTest Exam 




ReviseMRCP 

323 
































El-zohry MRCP Questions Bank (Port 1) - 2013 


(For my personal use) 


with intravenous fluids. Over several hours, 
the back pain resolved, and a widespread 
erythrodermic rash developed, centred mainly 
on the trunk. Further questioning revealed 
that the patient had removed a tampon 
shortly before presentation, as she had just 
ceased menstruating. 

What is the diagnosis? 

1- Haemolytic uraemic syndrome 

2- E coli sepsis 

3- Fungal infection 

4- Toxic shock syndrome 

5- Meningococcal septicaemia 

Answer & Comments 

Answer: 4- Toxic shock syndrome 

Toxic shock syndrome is due to toxin-1 (TSST- 
1), a protein secreted by S. 

aureus or streptococci, was the first of many 
toxins associated with the syndrome to be 
identified. Treatment is with penicillin and 
ceftriaxone. 


A 55 year old man who has a known history of 
chronic alcohol abuse presents with a week's 
history of fevers, night sweats and a cough 
productive of purulent sputum. 

On examination he was pyrexial with a temp 
of 39.1°C. Percussion note was dull over the 
left apex and there was bronchial breathing 
over this area on auscultation. The chest xray 
showed left upper lobar consolidation. 

Other investigations revealed: 

WBC 19 x 10 9 /l_ 

Neutrophils 18.3 x 10 9 /L 

What is the most likely diagnosis? 

1- PCP 

2- Tuberculosis 


[ Q: 743 ] MRCPass - Infectious 
disease 



3- Mycoplasma pneumonia 

4- Klebsiella pneumonia 

5- Legionnella pneumonia 


Answer & Comments 

Answer: 4- Klebsiella pneumonia 

Community acquired Klebsiella pneumonia is a 
disease of commonly affecting middle aged to 
older men with alcoholism. Klebsiella 
pneumonia characteristically affects one of 
upper lobes of the lung. There is an increased 
tendency toward abscess formation. 
Mycoplasma pneumoniae infections have a 
more insidious onset. 


[ Q: 744 ] MRCPass - Infectious 
disease 

A 23-year-old man presented to a hospital 
with a 3 day history of fever, rigors, confusion 
and malaise. A chest x-ray showed left low er- 
lobe pneumonia. He works in a factory close 
to a water tank which acts as a heat exchange 
for the welding cooling system. 

What is the most likely infective organism? 

1- Pneumococcus 

2- Legionella 

3- Mycoplasma 

4- Tuberculosis 

5- Klebsiella 



Answer & Comments 

Answer: 2- Legionella 

Legionnaire's disease is a severe form of 
pneumonia caused by the gram negative 
bacterium Legionella pneumophila. It is an 
airborne infection with the bacterium 
sometimes living in air-conditioning systems, 
whirlpool spas and domestic hot water 
systems in large buildings. 


Dr. Khalid Yusuf El-Zohry - Sohag Teaching Hospital (01118391123) 

Ref MRCPass OE OE 2012 PasTest 2009 PassMedicine 2009 PasTest Exam ReviseMRCP 




























El-zohry MRCP Questions Bank (Port 1) - 2013 


(For my personal use) 



[ Q: 745 ] MRCPass - Infectious 
disease 

A 34 year old man with HIV infection presents 
with several episodes of seizures. His CD4 
count is 150 cells/mm 3 . He had an MRI scan 
which shows multiple ring enhancing lesions. 

Whot should be given for treatment? 

1- Prednisolone 

2- Co-trimoxazole 

3- Pyrimethamine 

4- Foscarnet 

5- Rifampicin 


Answer & Comments 

Answer: 3- Pyrimethamine 

The likely diagnosis is cerebral toxoplasmosis. 
Tuberculosis usually causes single enhancing 
lesions and PML is less frequently ring 
enhancing. Lymphoma is also possible but not 
applicable in this scenario. Treatment for 
cerebral toxoplasmosis is pyrimethamine and 
sulphadiazine. 



Ring enhancing lesion Overall, this question 

has been 



[ Q: 746 ] MRCPass - Infectious 
disease 


A 38 year old man who is an intravenous drug 
abuser complains of chest pain and dyspnoea. 


Chest X ray shows bilateral cavities at apex. 


Whot is the most likely diagnosis? 

1- Tuberculosis 

2- Tricuspid endocarditis 

3- Pulmonary embolism 

4- Sarcoidosis 

5- Extrinsic allergic alveolitis 


Answer & Comments 

Answer: 2- Tricuspid endocarditis 

Right-sided endocarditis accounts for 5% of all 
cases of infective endocarditis and it is most 
often seen in intravenous drug abusers, the 
immunosuppressed and patients with central 
lines. The cavities on the chest x ray may 
represent abscesses due to embolic 
vegetations from endocarditis. 


A 28 year old banker goes on holiday to Brazil. 
He visits various holiday spots including a 
cattle ranch. He has headaches and a 
temperature of 38 °C. Examination reveals 
hepatosplenomegaly and spinal tenderness. 
His blood cultures do not grow any organisms. 

Which is the likely infective organism? 

1- Brucella melitensis 

2- Listeria 

3- Cryptosporidium 

4- Shigella 

5- Schistosomiasis 


[ Q: 747 ] MRCPass - Infectious 
disease 



Answer & Comments 

Answer: 1- Brucella melitensis 

Brucella is spread by contact with cattle, 
drinking unpasteurized/raw milk. It causes 
sacroilitis and discitis, and hepatomegaly. 
Should be treated by doxycycline or rifampicin 


Dr. Khalid Yusuf El-Zohry - Sohag Teaching Hospital (01118391123) 

Ref MRCPass OE OE 2012 PasTest 2009 PassMedicine 2009 PasTest Exam 




ReviseMRCP 

325 




























El-zohry MRCP Questions Bank (Port 1) - 2013 


(For my personal use) 


^ [ Q: 748 ] MRCPass - Infectious 

0 disease 

A pregnant 18 year old woman came to the 
clinic with a low -grade fever,malaise, and 
headache. She was sent home with a 
diagnosis of influenza. She again sought 
treatment 7 days later with a macular rash on 
her trunk, arms, hands, and feet. Further 
questioning of the patient when serology 
results were known revealed that I month 
previously, she had a painless ulcer on her 
vagina that healed spontaneously. 

Which of the following is the most likely 
diagnosis? 

1- Lyme disease 

2- Lymphogranumoma venereum 

3- Behcet's disease 

4- Endocarditis 

5- Syphilis 


Answer & Comments 

Answer: 5- Syphilis 

The initial lesion of primary syphilis develops 
at the site of transmission after an incubation 
period of 10-90 days, with a mean of about 
21-28 days, and then heals spontaneously in 

3-7 weeks. 

T pallidum is sensitive to the penicillins and is 
easily treatable in the early stages 



Maculopapular rash in secondary syphilis 



[ Q: 749 ] MRCPass - Infectious 
disease 


A 40 year old man has HIV infection. He is 
admitted with a 4 month history of weight 
loss, lethargy and diarrhoea. 


He presented 6 months previously with PCP 
pneumonia. He is on AZT, lamivudine and 
efavirenz therapy. 


Investigations reveal: 

Hb 9.8 g/dl, WCC 6.5 x 10 9 /L, platelets 75 x 
10 9 /L, MCV 95 fl, CD4 count 280 X 10 6 /ml, Aik 
Phos 270 U/l, Bilirubin 


8 U/l, ALT 20 U/l, GGT 30U/I. 


CXR: normal. 


What is the cause of the anaemia? 

1- Upper Gl bleed 

2- Haemolytic anaemia 

3- Vitamin B i2 deficiency 

4- Folate deficicency 

5- Zidovudine therapy 


Answer & Comments 

Answer: 5- Zidovudine therapy 

AZT has common side effects such as nausea 
and headaches, anaemia, bone marrow 
suppression and proximal myopathy. Some of 
these side effects may be caused by the 
sensitivity of ?-DNA polymerase in cell 
mitochondria to AZT. 


On the medical wards, there has been several 
new cases of Clostridium difficile diarrhoea. 

What is the best way to prevent spread of 
Clostridium difficile? 

1- Hand washing by staff and patients 

2- Isolation of patients affected 

3- Isolation of any staff affected 


[ Q: 750 ] MRCPass - Infectious 
disease 



Dr. Khalid Yusuf El-Zohry - Sohag Teaching Hospital (01118391123) 

Ref MRCPass OE OE 2012 PasTest 2009 PassMedicine 2009 PasTest Exam ReviseMRCP 



326 




























El-zohry MRCP Questions Bank (Port 1) - 2013 


(For my personal use) 


4- Treatment of asymptomatic carriers with 
oral vancomycin 

5- Ward closure and sterilisation 


Answer & Comments 

Answer: 1- Hand washing by staff and patients 

Infection control with hand washing has been 
a topical issue and there is great emphasis on 
the effectiveness of handw ashing to prevent 
spread of many infections. 


A 20 year old woman has fevers and central 
chest pains which are pleuritic in nature. She 
also gives a history of arthralgia. On 
examination, she has a systolic murmur 
loudest in the mitral region. She also has 
serpigionous erythematous rash on the 
abdomen and has subcutaneous nodules in 
the right elbow. 

Blood tests show Hb 9.8 g/dl, WCC 14 x 10 9 /L, 
platelets 280 x 10 9 /L, CRP 120 mg/I. 
Echocardiogram shows moderate mitral 
regurgitation. 

What is the likely diagnosis? 

1- Coxsackie virus infection 

2- Libman sacks endocarditis 

3- Rheumatoid arthritis 

4- Rheumatic fever 

5- Lyme disease 



[ Q: 751 ] MRCPass - Infectious 
disease 


Answer & Comments 

Answer: 4- Rheumatic fever 

The patient has symptoms which would satisfy 
the Duckett Jones criteria for rheumatic fever. 
Either 2 major, or 1 major/ 2 minor criteria 
would be required with evidence of 
Streptocoocal infection e.g. ASOT. 


Ji 

46 year old man has had a history of HIV 
infection known for 6 years. He presented 
with confusion and has had a lumbar 
puncture. 

Results of the CSF showed: 

White cells 150 /mm 3 
Glucose 4 mmol/I 
Protein 0.55 g/l 
What is the likely diagnosis? 

1- AIDS dementia complex 

2- CNS lymphoma 

3- HSV encephalitis 

4- Disseminated tuberculosis 

5- Cerebral toxoplasmosis 


[ Q: 752 ] MRCPass - Infectious 
disease 


Answer & Comments 

Answer: 3- HSV encephalitis 

The glucose is not sufficiently low to suggest 
tuberculosis, and the scenario fits with HSV 
encephalitis. This is not an AIDS defining 
illness but does occur in patients who are 
immunosuppressed. 


A 25 year old lady has recently returned from 
Namibia in Africa. On examination his 
temperature was 38°C, and inspiratory 
respiratory crackles were present. He had a 
maculopapular rash and a few some blue-grey 
spots in the buccal area. 

Which of these diagnoses is likely? 

1- Subacute sclerosing panencephalitis 

2- Dengue fever 

3- Measles 

4- Behcet's disease 

5- Rubella 



[ Q: 753 ] MRCPass - Infectious 
disease 


Dr. Khalid Yusuf El-Zohry - Sohag Teaching Hospital (01118391123) 

Ref MRCPass OE OE 2012 PasTest 2009 PassMedicine 2009 PasTest Exam 




ReviseMRCP 

327 
































El-zohry MRCP Questions Bank (Port 1) - 2013 


(For my personal use) 


Answer & Comments 

Answer: 3- Measles 

Measles is still common in Africa, atlhough 
immunisation has greatly reduced incidences 
in many areas. The presentation is usually 
with fever, dry cough, conjunctivitis, a rash, 
lymphadenopathy or hepatosplenomegaly. 
The blue-grey spots on the buccal mucosa are 
known as Koplik spots. In young children, 
latent infection can involve the central 
nervous system - known as subacute 
sclerosing panencephalitis. Treatment is with 
supportive care. 



Koplik's spots 



[ Q: 754 ] MRCPass - Infectious 
disease 

A 25 year old male man has urethral 
discharge. Gram stain of the discharge sample 
shows gram negative intracellular diplococci. 
The patient is treated with cefotaxime as a 
500mg single intramuscular dose. A week 
later, the patient still has urethral discharge. 

Which of the following organisms is likely to be 
responsible? 

1- Neisseria gonorrhoeae 

2- Ureaplasma 

3- Chlamydia trachomatis 

4- Staphyloccocus aureus 


5- Escherichia coli 

Answer & Comments 

Answer: 3- Chlamydia trachomatis 

Although the diplococci are likely to be 
gonorrhoea, this patient has been treated and 
it makes this less likely. Recommended 
treatment options for gonococcal infection are 
ceftriaxone 250mg single im dose OR 
cefotaxime (Claforan) 500mg single im dose 
for gonorrhoea. Persistent discharge suggests 
another organism, in this case likely 
Chlamydia. Doxycycline would is therefore 
recommended. 


A 22 year old girl presents with headache and 
neck stiffness. She was pyrexial but had no 
signs of raised intrcranial pressure on 
examination. Cerebrospinal fluid analysis 
showed: Cell count 130 /mL (predominantly 
lymphocytes), Protein 0.6 g/L, Glucose 3.3 
mmol/L, Gram stain showed no organisms. 

Which diagnosis is likely? 

1- Meningococcocal meningitis 

2- Tuberculous meningitis 

3- Herpes encephalitis 

4- Viral meningitis 

5- Aseptic meningitis 



[ Q: 755 ] MRCPass - Infectious 
disease 


Answer & Comments 

Answer: 4- Viral meningitis 

The combination of lymphocytosis, raised 
protein and normal glucose suggests viral 
meningitis. Common causes are enterovirus 
and mumps 



Dr. Khalid Yusuf El-Zohry - Sohag Teaching Hospital (01118391123) 

Ref MRCPass OE OE 2012 PasTest 2009 PassMedicine 2009 PasTest Exam ReviseMRCP 


























El-zohry MRCP Questions Bank (Port 1) - 2013 


(For my personal use) 



[ Q: 756 ] MRCPass - Infectious 
disease 


A 30 year old white woman was referred by 
her general practitioner to her local hospital 
with a painful vesicular genital rash. On 
presentation she was complaining of 
headache, neck stiffness, unsteadiness when 
walking, double vision, and photophobia. 


On admission, she was alert and oriented, 
pyrexial (38°C), and hypertensive (blood 
pressure 180/80 mm Hg). An extensive 
vesicular rash was noted on the posterior 
aspect of the right thigh, and bilaterally in the 
sacral and perianal areas. There were no 
vesicles elsew here. Neurological examination 
revealed an intention tremor, marked upper 
limb and truncal ataxia and variable 
ophthalmoplegia with diplopia and 
nystagmus. 


Cerebrospinal fluid analysis showed a 
predominantly lymphocytic pleocytosis. 


What is the likely diagnosis? 


1- Pneumococcal meningitis 

2- Progressive multifocal 

leukoencephalopathy 

3- Tuberculous meningitis 


4- Meningococcal meningitis 

5- HSV encephalitis 


Answer & Comments 

Answer: 5- HSV encephalitis 

In adults HSV-2 usually causes uncomplicated 
genital herpes, but occasionally this may 
predispose to HSV-2 encephalitis. Neurological 
signs vary. MRI can confirm the diagnosis, 
demonstrating hyperintensity. Treatment is 
with intravenous aciclovir. 



HSV encephallitis causing hyperintensity in the 

medulla (arrow) 



[ Q: 757 ] MRCPass - Infectious 
disease 


A 30 year old lady presents to renal transplant 
clinic with fever and diarrhoea. She is 7 weeks 
post live-related-donor transplantation. Her 
initial course had been complicated by an 
episode of acute rejection which responded to 
treatment with anti-thymocyte globulin (ATG). 
She is currently maintained on prednisolone, 
cyclosporin and azathioprine and has just 
finished a course of oral ciprofloxacin for an E. 
coli urinary tract infection. Blood tests reveal a 
mild hepatitis and leucopenia, and her 
creatinine has been rising. 


The most likely diagnosis is: 

1- Recurrent urinary tract infection 


2- Cytomegaloviral infection 

3- Graft-versus-host disease 


4- Ciprofloxacin toxicity 

5- Cyclosporin toxicity 


Answer & Comments 

Answer: 2- Cytomegaloviral infection 

Several weeks post transplantation would be 
right for a presentation of cytomegaloviral 
(CMV) infection. It is commonest betw een 1-3 
months post transplantation and the risk of 
severe infection is increased by the use of 


Dr. Khalid Yusuf El-Zohry - Sohag Teaching Hospital (01118391123) 

Ref MRCPass OE OE 2012 PasTest 2009 PassMedicine 2009 PasTest Exam ReviseMRCP 





























El-zohry MRCP Questions Bank (Port 1) - 2013 


(For my personal use) 


immunosuppressants such as Anti Thymocyte 
Globulin. 

A number of complications can occur with 
CMV infection : pneumonitis, colitis, hepatitis, 
retinitis, leucopenia and thrombocytopenia 
can occur. Diagnosis can be made by detection 
of CMV antigen or with polymerase chain 
reaction (PCR) from blood. Tissue biopsy may 
show inclusion bodies. 



[ Q: 758 ] MRCPass - Infectious 
disease 

A 44 year old man has arrived from Saudi 
Arabia to the UK 5 days ago. He now presents 
with painful episodes of trismus. He mentions 
he had a cut to his foot while walking barefoot 
in his garden 5 days ago. He suddenly has an 
episode of generalised tetanus with 
respiratory arrest. 

Whot should be given to him? 

1- IM tetanus antitoxin 

2- IV gentamicin 

3- PR paracetamol 

4- Amputate the foot 

5- lonotropes 


[ Q: 759 ] MRCPass - Infectious 
disease 

A 30 year old lady has a mild fever, sore throat 
and a erythematous, macular rash develops 
on the face giving the appearance of a slapped 
cheek. Her child had a similar illness a week 
ago. Her blood results are normal. 

Which one of the following features is 
associated? 

1- Renal failure 

2- Jaundice 

3- Aplastic anaemia 

4- Polyarthritis 

5- Meningitis 



Answer & Comments 

Answer: 4- Polyarthritis 

Parvovirus infection has an incubation period 
of 6 - 14 days. Presentation is usually with a 
prodrome of mild fever with a sore throat and 
gastrointestinal disturbance which lasts for up 
to 4 days. An erythematous rash giving a 
slapped cheek appearance is classical. 
Polyarthritis and lymphadenopathy are also 
associated. 


Answer & Comments 

Answer: 1- IM tetanus antitoxin 

The antibiotic of choice is IV metronidazole. If 
he is having generalized spasms he may 
require ventilation and muscle relaxants with 
or without neuromuscular blockade. IM 
antitoxin should be given immediately to 
absorb up any unbound tetanus toxin. After 
recovery from infection he will still have no 
immunity to tetanus and the first dose of 
tetanus toxoid is given, with Two further 
doses to follow. 


[ Q: 760 ] MRCPass - Infectious 
disease 

A 35 year old woman presented w ith a non¬ 
healing genital ulcer. She had travelled 
through Africa 3 years previously. 

On examination, An indurated 1.5 cm ulcer 
was present on the inner aspect of the left 
labia majoris. 

Skin biopsy of the lesion revealed a granuloma 
surrounding a schistosome egg. Schistosoma 
haematobium eggs were detected in terminal 
urine collected betw een midday and 2 pm. 

What should she be treated with? 

1- Quinine 

2- Benzylpenicillin 



Dr. Khalid Yusuf El-Zohry - Sohag Teaching Hospital (01118391123) 

Ref MRCPass OE OE 2012 PasTest 2009 PassMedicine 2009 PasTest Exam ReviseMRCP 




























El-zohry MRCP Questions Bank (Port 1) - 2013 


(For my personal use) 


3- Tetracycline 

4- Flucloxacillin 

5- Praziquantel 

Answer & Comments 

Answer: 5- Praziquantel 

Schistosomal infestation may persist and 
present long after leaving an endemic area. 

Serological tests are the most sensitive 
method of screening, but are not species- 
specific. Indirect hemagglutination titres ? 64 
suggest infection. 

Treatment is with praziquantel. 



disease that may infect the baby and cause 
abnormalities. They may develop congenital 
defects, mainly heart defects, deafness and 
cataracts (eye lens opacities). The risk drops as 
pregnancy advances beyond 20 weeks. 

Oral acyclovir is safe in pregnancy. At a later 
stage the main danger is that a new born child 
would become infected with no transfer of 
antibody from the mother. In that case VZig 
should be given to the child, but it has no role 
in therapy of the mother. 



Chickenpox 


Schistosomal Egg in the urine 

[ Q: 761 ] MRCPass - Infectious 
disease 

A woman who is 32 weeks pregnant presents 
with blistering vesicles present all over her 
body. Varicella zoster virus infection is 
diagnosed. 

How should the patient be treated? 

1- Antiretroviral therapy 

2- Oral acyclovir 

3- Varicella Zoster immune globulin 

4- Methylprednisolone 

5- Caesarean section 



Answer & Comments 

Answer: 2- Oral acyclovir 

This patient has chickenpox (varicella zoster 
infection). In the non-immune pregnant 
woman, chickenpox is a potentially dangerous 



[ Q: 762 ] MRCPass - Infectious 
disease 


A 32 year old man has returned from Thailand 
4 months ago. He presents with vomiting, 
lethargy, myalgia and right upper quadrant 
pains. He mentions that whilst he was in 
Thailand on holiday, he was treated for severe 
falciparum malaria. Investigations show : 


ALT 2,100(5-35) U/l 
AST 1,700 (1-31) U/l 
ALP 350 (20-120) U/l 
GGT 70 (4-35) U/l 


Bilirubin 320 (1-22) pmol/l 
Albumin 30 (37-49) g/l 
Glucose 3 mmol/I 


What is the likely diagnosis? 

1- Falciparum malaria 

2- Leishmaniasis 


3- Hepatitis B 


Dr. Khalid Yusuf El-Zohry - Sohag Teaching Hospital (01118391123) 

Ref MRCPass OE OE 2012 PasTest 2009 PassMedicine 2009 PasTest Exam 




ReviseMRCP 

331 

























El-zohry MRCP Questions Bank (Port 1) - 2013 


(For my personal use) 


4- Dengue 

5- Schistosomiasis 

Answer & Comments 

Answer: 3- Hepatitis B 

The incubation period for hepatitis B is 6 
weeks to 6 months. The symptoms and 
deranged LFTs along with hypoglycaemia 
would suggest severe hepatitis and also 
seroconversion illness associated with 
hepatitis B. 


[ Q: 764 ] MRCPass - Infectious disease 

An 75 year old man presents to hospital 
unwell with diarrhea. He has a BP of 100/70, 
heart rate 110 and Temp 38°C. A diastolic 
murmur is heard in aortic area. 

His bloods show Hb 9.0 g/dl, MCV 85 fl, WCC 
13 x 10 9 /L, platelets 270 x 10 9 /l_, urea 6 
pmol/l, creatinine 80 pmol/l, sodium 140 
mmol/I, potassium 3.8 mmol/I, ESR 80 mm/hr, 
CRP 220 mg/I. 

Which organism is likely to grow in the blood 
cultures ? 


[ Q: 763 ] MRCPass - Infectious 
disease 

A middle-aged, obese man had previously 
been treated for chronic venous insufficiency- 
related sw elling and cellulitis. He had 
hyperpigmentation and hemosiderin 
deposition. A further wound swab grew 
Pseudomonas aeruginosa. 

Which one of the following antibiotics is 
recommended? 

1- Ciprofloxacin 

2- Flucloxacillin 

3- Benzylpenicillin 

4- Cephalexin 

5- Metronidazole 



Answer & Comments 

Answer: 1- Ciprofloxacin 

Pseudomonas can also cause otitis media, 
pneumonia (in cystic fibrosis), urinary tract 
infection and rarely, endocarditis. Gentamicin, 
ciprofloxacin and meropenem are antibiotics 
which are effective against pseudomonas 
infection. 



1- Streptococcus mitis 

2- Staphylococcus aureus 

3- Streptococcus bovis 

4- Escherichia coli 

5- Brucella melitensis 

Answer & Comments 

Answer: 3- Streptococcus bovis 

Streptococcus bovis usually enters the 
bloodstream via the gastrointestinal tract. 
Nearly all patients with S bovis endocarditis 
are older than 50 years, and there is also an 
association with malignancy of the Gl tract. 
Treatment is with penicillin and vancomycin. 



[ Q: 765 ] MRCPass - Infectious 
disease 


A 75 year old woman presented with a non 
healing ulcer on her foot. This has been 
managed by the podiatrist for the past 8 
weeks. Blood cultures and a wound swab both 
grew MRSA. 


What antibiotics would you consider in 
addition to vancomycin? 


1- Flucloxacillin 


2- Metronidazole 


3- Rifampicin 

4- Ciprofloxacin 


Dr. Khalid Yusuf El-Zohry - Sohag Teaching Hospital (01118391123) 

Ref MRCPass OE OE 2012 PasTest 2009 PassMedicine 2009 PasTest Exam ReviseMRCP 



332 






























El-zohry MRCP Questions Bank (Port 1) - 2013 


(For my personal use) 


5- Azithromycin 

Answer & Comments 

Answer: 3- Rifampicin 

For serious infections caused by MRSA strains 
that are susceptible to rifampicin, adding this 
agent to vancomycin or a fluoroquinolone 
may contribute to improved outcomes. 
Vancomycin continues to be the drug of 
choice for treating most MRSA infections 
caused by multidrug resistant strains. 


[ Q: 766 ] MRCPass - Infectious 
disease 

A 50 year old health care worker has been to 
India for several months. He was fully 
vaccinated prior to travel. 

Following return to the UK, a relative notices 
that he is yellow. He has a bilirubin of 46 
umol/l and ALT of 2500 U/l. 



A 

A 30 year old male was readmitted to surgical 
ward with history of pain and sw elling in the 
left shoulder, following a lipoma removal. 
Small vesicles appeared over the left shoulder 
and arm. 

On examination, patient looked ill, the pulse 
rate was 100 beats per minute and blood 
pressure was 118/76 mm Hg. X-ray of the left 
should and arm revealed diffuse gas bubbles 
in the intermuscular tissue planes. 

Which one of the following is the most likely 
infective organism? 

1- Streptococcus pyogenes 

2- Neisseria gonorrhoeae 

3- Staphylococcus aureus 

4- Clostridium perfringens 

5- Tuberculosis 


[ Q: 767 ] MRCPass - Infectious 
disease 


Whot is the most likely diagnosis? 

1- Hepatitis B 

2- Infectious mononucleosis 

3- Hepatitis E 

4- Yellow fever 

5- Leptospirosis 

Answer & Comments 

Answer: 3- Hepatitis E 

Both hepatitis A and E are transmitted by the 
faeco-oral route. He is likely to have been 
vaccinated against A prior to travel and B in 
the context of his occupation. This would lead 
to a transient marked transaminitis and 
jaundice. The condition should resolve with 
conservative management but is associated 
with a 1-2% mortality due to the risk of 
fulminant hepatitis in patients with underlying 
liver disease and pregnant women in the last 
trimester in whom the mortality is around 
20 %. 


Answer & Comments 


Answer: 4- Clostridium perfringens 

Cases of gas gangrene are known to occur 
following trauma and surgical procedures. C. 
perfringens is the causative agent of gas 
gangrene. Both the enterotoxin producing 
strains of Clostridium perfringens and 
Clostridium difficile can cause diarrhoea. C. 
tetany and C. botulinum form neurotoxins 
(Botulism is associated with a flaccid 
paralysis). 


a 


[ Q: 768 ] MRCPass - Infectious 
disease 


A 30 year old woman presents with bloody 
diarrhoea. The diarrhoea started Two weeks 
ago, and was associated with increasing 
malaise. There was mild sw elling of the low er 
limbs. She has had difficulty passing urine. She 
had eaten rare steak during a party recently. 
On examination she was pale, and there was 


Dr. Khalid Yusuf El-Zohry - Sohag Teaching Hospital (01118391123) 

Ref MRCPass OE OE 2012 PasTest 2009 PassMedicine 2009 PasTest Exam 


over her legs. 

Blood 





ReviseMRCP 

333 


































El-zohry MRCP Questions Bank (Port 1) - 2013 


(For my personal use) 


pressure was 150/95 on admission. On 
examination she was apyrexial, but had a 
resting tachycardia. There were also crackles 
on inspiration at both lung bases. 

Investigations show : 

Haemoglobin 9 g/dL 

White cell count 12.2 x 10 9 /L 

Neutrophils 8.7 x 10 9 /L 

Platelets 42 x 10/L 


blood film may show haemolysis and evidence 
of fragmented erythrocytes. 


13 

A 45 year old man went on a boat cruise in the 
Carribean. Several days later he developed 
abdominal cramps and bloody diarrhoea. On 
examination, he has a pyrexia Temp 38 C and 
generalised tenderness in the lower abdomen. 


[ Q: 769 ] MRCPass - Infectious 
disease 


PT 13 sec 


What is the likely cause? 


APTT 36 sec 


1- Rotavirus 


Fibrinogen 5 g/dL 

Serum sodium 138 mmol/L 

Serum potassium 6.3 mmol/L 


2- Sodium monoglutamate excess 

3- Entamoeba Histolytica 

4- Giardia Lamblia 


Serum urea 30 mmol/L 
Serum creatinine 426 umol/L 
Serum albumin 28 g/L 
Dipstick urine Blood ++ Protein + 


5- Salmonella species 

Answer & Comments 

Answer: 5- Salmonella species 


What is the most important next investigation 
which might yield a diagnosis? 

1- Renal tract ultrasound 

2- Stool culture 

3- Urine microscopy 

4- CT scan of the abdomen 

5- Echocardiogram 


The likely organism from this history causing 
bloody diarrhea is Salmonella. Entamoeba is 
uncommon around the carribean. Shigella, 
salmonella and Campylobacter are all 
possibilities which may cause bloody 
diarrhoea. Giardia does not usually cause 
fevers and rotavirus does not usually cause 
bloody diarrhoea. 



4 * 1 


[ Q: 770 ] MRCPass - Infectious 

Answer & Comments 




disease 


Answer: 2- Stool culture 

This patient has haemolytic uraemic syndrome 
(HUS). It typically presents with a triad of 

acute renal failure, Microangiopathic 
haemolytic anaemia, thrombocytopaenia and 
deranged clotting. Haemolytic uraemic 
syndrome is most commonly a complication of 
infection with verocytotoxin producing E.coli 
usually of serotype 0157. Stool cultures would 
best confirm the diagnosis. In addition, a 


A 75 year old man has fevers, neck stiffness 
and headaches. He has a past medical history 
of polymyalgia rheumatica and has been on 
steroids long term. Meningitis is suspected 
and a lumbar puncture is performed. 

CSF shows protein of 1.2 g/l, glucose 3.4 
mmol/l, white cell count 95 (75% 

lymphocytes, 20% polymorphs). A few gram 
positive rods are seen on microscopy. 

What is the likely infective organism? 


1- Meningococcus 


Dr. Khalid Yusuf El-Zohry - Sohag Teaching Hospital (01118391123) 

Ref MRCPass OE OE 2012 PasTest 2009 PassMedicine 2009 PasTest Exam ReviseMRCP 


























El-zohry MRCP Questions Bank (Port 1) - 2013 


(For my personal use) 


2- Mycobacterium tuberculosis 

3- Cryptococcus 

4- Listeria monocytogenes 

5- Staphylococcus aureus 

Answer & Comments 

Answer: 4- Listeria monocytogenes 

The elderly, especially those with 
immunocompromise, are prone to Listeria 
meningitis. Treatment is with high dose IV 
ampicillin or amoxycillin (2g qds). 


mortality when inhaled. Ciprofloxacin [400 mg 
intravenously (i.v.) 12 hrly] the drug of choice. 



Cutaneous Anthrax 



[ Q: 771 ] MRCPass - Infectious 
disease 

A 18 month old child in India has painless 
ulcerated lesion over the right buttock of 10 
days duration. The infant's mother initially 
noticed a papule in that area that increased in 
size and ulcerated spon-taneously to form a 
blackish eschar. The only other significant 
history was the death of a cattle in the 
neighborhood. 

Cutaneous examination revealed a dark 
hemorrhagic eschar surrounded by a zone of 
edema and erythema, studded with several 
small vesicles that have coalesced. 

What is the likely diagnosis? 

1- Chickenpox 

2- Impetigo 

3- Anthrax 

4- Tetanus 

5- Rabies 


Answer & Comments 

Answer: 3- Anthrax 

This is a case of cutaneous anthrax. Bacillus 
anthracis, a spore-forming bacterium, is the 
etiologic agent of anthrax. B. anthracis spores 
can be aerosolized, are relatively easy to 
produce, and are capable of producing high 


SJ 

An asymptomatic 25 year HIV positive male 
patient is follow ed up at the clinic. 
Investigations reveal a viral load of 270,000 
copies/ml and a CD4 count of 190 x 10 6 /ml. 

What is the appropriate treatment strategy in 
this patient? 

1- Start antiretroviral therapy 

2- Start antiretrovirals when count < 100 

3- Start prednisolone 

4- MRI brain 

5- Lumbar puncture 


[ Q: 772 ] MRCPass - Infectious 
disease 


Answer & Comments 

Answer: 1- Start antiretroviral therapy 

Generally, antiretrovrial therapy should be 
initiated in asymptomatic patients if a CD4 
count betw een 200 to 350 (or less). A HIV 
RNA level (viral load) of >30,000 copies/ml 
also meets the criteria for starting therapy. 


A 30 year old lady has fevers, muscular pains 
all over, vomiting and diarrhoea. Her 
temperature is 39°C and blood pressure is 
85/40 mmHg. There is an erythematous 



[ Q: 773 ] MRCPass - Infectious 
disease 


Dr. Khalid Yusuf El-Zohry - Sohag Teaching Hospital (01118391123) 

Ref MRCPass OE OE 2012 PasTest 2009 PassMedicine 2009 PasTest Exam ReviseMRCP 



335 

































El-zohry MRCP Questions Bank (Port 1) - 2013 


(For my personal use) 


desquamating rash in both her hands and legs. 
Her tongue is also red. 

Which is the likely diagnosis? 

1- Toxic shock syndrome 

2- Haemolytic uraemic syndrome 

3- Disseminated intravascular coagulation 

4- Meningococcal sepsis 

5- JC virus infection 


Answer & Comments 

Answer: 1- Toxic shock syndrome 

The presentation of pyrexia, shock, diarrnoea 
and vomiting, myalgia, desquamating rash and 
mucous membrane involvement would be 
consistent with toxic shock syndrome. It can 
also present with abnormal liver and renal 
function, as well as thrombocytopenia. Toxic 
shock syndrome can be caused by both 
staphylococcus (tampon related) and 
streptococcus (skin infection). 



[ Q: 774 ] MRCPass - Infectious 
disease 


A 35 year old man was in South East Asia on 
holiday backpacking alone. He has returned 5 
days ago, having been there for a month. His 
temperature is 38°C and he has a sw ollen 
ankle and elbow joint. He also complains of 
purulent penile discharge. 


Which of the following conditions/infections is 
likely? 


1- Reiter's syndrome 


2- Chlamydia trachomatis 


3- Neisseria gonorrhoeae 

4- Staphyloccocus aureus 


5- Treponema pallidum 


This is a typical presentation for gonorrhoeae. 
There is penile discharge and knee effusions. 
The discharge and knee aspirate may grow 
gram negative diplococci. Current 
recommended treatment is ceftriaxone 125 
mg IM single dose. Concurrent treatment for 
chlamydia should be given for 3-6 weeks, to 
include oral tetracycline 500 mg 4 times a day 
or oral doxycycline 100 mg tw ice a day. 



% . * * • 


Gram negative diplococci - N. Gonorrhoeae 



[ Q: 775 ] MRCPass - Infectious 
disease 


A 38 year old man with previously treated 
early syphilis and hepatitis C infection 
presented to a hospital complaining of 3 
months of tender right inguinal 
lymphadenopathy. An excisional biopsy 
showed the formation of necrotising 
granuloma indicative of Lymphogranuloma 
venereum. 


Whot should the patient be treated with? 

1- Benzylpenicillin 

2- Erythromycin 

3- Clindamycin 

4- Gentamicin 


5- Doxycycline 


Answer & Comments 


Answer: 5- Doxycycline 


Answer & Comments 


Answer: 3- Neisseria gonorrhoeae 


Confirmation of a diagnosis of LGV requires 
serological tests or PCR on genitourinary 
specimens. Prolongedtreatment with 


Dr. Khalid Yusuf El-Zohry - Sohag Teaching Hospital (01118391123) 

Ref MRCPass OE OE 2012 PasTest 2009 PassMedicine 2009 PasTest Exam ReviseMRCP 



























El-zohry MRCP Questions Bank (Port 1) - 2013 


(For my personal use) 


doxycycline or roxithromycin for 3 weeks is 
required for affected patients. 



Lymphogranuloma venereum 


^ Q: 776 ] MRCPass - Infectious 
S disease 

A 30 year old woman has just given birth to a 
baby 2 weeks premature. The baby has a 
hypoplastic right eye. 

She mentions that early on in the pregnancy 
she has a flu-like illness associated with a rash. 

What is the probably cause of the congenital 
defect? 

1- Rheumatic fever 

2- Varicella zoster 

3- Mycoplasma 

4- Toxic shock syndrome 

5- Parvovirus B19 infection 


Answer & Comments 

Answer: 2- Varicella zoster 

The patient is likely to have had chickenpox 
infection during the second trimester of 
pregnancy (risk at this stage = 2% due to limb 
development, risk in 1st trimester = 1% unlike 
other congenital infections). Rubella infection 
of the mother may cause spontaneous 
abortion or causes serious damage to the 
surviving foetus - characterised by deafness, 
blindness and heart defects (risk >90% in 1st 
trimester). Parvovirus causes severe anaemia 
in the foetus and may result in hydrops 
foetalis as a result of heart failure. 



[ Q: 777 ] MRCPass - Infectious 
disease 

J I- 

A 42 year old man presented 10 days after 
returning from a 8-w eek holiday in South-East 
Asia. He had an eight-day history of malaise, 
chills, headache, sore throat and generalised 
rash. He had reported many mosquito bites. 


He had fever, a macular rash and generalised 
lymphadenopathy with mild splenomegaly, 
but no meningism and no eschar present. 


Full blood examination revealed 
lymphocytosis with numerous atypical 
lymphocytes and thrombocytopenia. Blood 
cultures and malaria films were negative. Liver 
function tests revealed marginally elevated 
serum transaminase levels. Serological testing 
revealed past infection with Epstein-Barr virus 
and cytomegalovirus and was negative for Q 
fever, dengue, rubella, measles and rickettsial 
infection. 


What is the most likely diagnosis? 

1- HIV 


2- Dengue fever 

3- Syphilis 

4- Lyme disease 

5- Malaria 


Answer & Comments 

Answer: 1- HIV 

Acute HIV seroconversion may mimic several 
tropical diseases, including dengue and 
typhus, as well as infectious mononucleosis. 


^ [ Q: 778 ] MRCPass - Infectious 

| disease 

A 25 year old patient was prescribe amoxicillin 
for tonsillitis. She goes to see her GP 
complaining that she has developed a 
maculopapular rash on her trunk and arms. 

Which infection is she likely to have? 

1- Cytomegalovirus 


Dr. Khalid Yusuf El-Zohry - Sohag Teaching Hospital (01118391123) 

Ref MRCPass OE OE 2012 PasTest 2009 PassMedicine 2009 PasTest Exam 




ReviseMRCP 

337 

























El-zohry MRCP Questions Bank (Port 1) - 2013 


(For my personal use) 


2- Tuberculosis 

3- Ebstein Barr virus 

4- Echovirus 

5- Coxsackie virus 


Answer & Comments 

Answer: 3- Ebstein Barr virus 

In glandular fever (EBV infection or infectious 
mononucleosis), there is an increased risk of 
developing a rash with amoxycillin or 
ampicillin. Hence, these antibiotics are 
contraindicated. 



[ Q: 779 ] MRCPass - Infectious 
disease 


A 45 year man with alcoholic cirrhosis was 
admitted unw ell. He had marked ascites and 
a temp of 38°C. An ascitic tap showed a white 
cell count of 350 cells per mm3. 


[ Q: 780 ] MRCPass - Infectious disease 

A 18-year-old man who was previously well 
developed an upper respiratory tract infection 
follow ed by fever and cough, excessive 
sweating and progressive dyspnea. He works 
in a factory with humidifiers. One day prior to 
admission the patient developed hemoptysis, 
left-sided pleuritic chest pain and dizziness. 

On examination, the patient was drow sy and 
tachypneic. Blood pressure was 130/70 
mm/Hg, pulse rate 92/min, respiratory rate 
28/min, and temperature 40 C. Chest 
auscultation revealed bilateral crepitations in 
the low er lung zones. The patient was 
commenced on intravenous amoxycillin. 

Legionella titer available on the third day was 
1:256. 

Whot antibiotic should be added? 

1- Erythromycin 

2- Tazocin 


Which of following is the appropriate 
antibiotic? 

1- Iv cefotaxime 

2- Iv metronidazole 

3- Iv ciprofloxacin 

4- Iv amoxycillin 

5- Iv vancomycin 


Answer & Comments 

Answer: 1- Iv cefotaxime 

This lady has spontaneous bacterial peritonitis 
(cells >300). The organisms are usually E. 

Coli, Pseudomonas, Klebsiella, S Pneumoniae 
and Enterococci. Initial treatment should be 
broad spectrum such as cefotaxime. 
Gentamicin & ampicillin should be considered. 



3- Linezolid 

4- Rifampicin 

5- Gentamicin 


Answer & Comments 

Answer: 1- Erythromycin 

Legionella species can survive for a long 
period of time in water and has been found in 
moist soil. This may explain the association of 
infection with recent excavation, air 
conditioner cooling towers, respiratory 
devices and humidifiers. A macrolide e.g. 
erythromycin is recommmended for 
treatment. 


[ Q: 781 ] MRCPass - Infectious 
disease 

A 35 year old patient has a stiff neck, myalgia 
and joint pains in the knees, shoulders and 
elbows. He had felt that his heart beat was 
irregular. He also developed a rash that came 
and went on the back over several weeks. 



Dr. Khalid Yusuf El-Zohry - Sohag Teaching Hospital (01118391123) 

Ref MRCPass OE OE 2012 PasTest 2009 PassMedicine 2009 PasTest Exam ReviseMRCP 































El-zohry MRCP Questions Bank (Port 1) - 2013 


(For my personal use) 


Which is the best diagnostic test? 

1- Monospot test 

2- Herpes virus serology 

3- Immunofluorescent antibodies to Borrelia 
burgdorferi 

4- Coxsackie virus serology 

5- Serum Anti streptolysin 0 titres 


Answer & Comments 

Answer: 3- Immunofluorescent antibodies to 
Borrelia burgdorferi 

This is Lyme disease. It is caused by tick bites 
spreading Borrelia burgdorgferi. The rash is 
erythema chronicum migrans. Joint pains and 
irregular heart beats are common symptoms. 
Heart block can occur. 



Erythema Chronicum Migrans 


^ Q: 782 ] MRCPass - Infectious 

ft 

S disease 

Which one of the following cytokines is 
commonly secreted by the T helper 2 cell? 

1- IL-1 

2- IL-4 

3- IL-2 

4- TNF alpha 

5- IFN gamma 


Answer & Comments 


Answer: 2- IL-4 


TH1 cells commonly secrete IFN gamma and 
IL2, leading to B cell, natural killer and 
macrophage activation. TH2 cells secrete IL3, 
4,5 and 6, leading to mast cells and eosinophil 
activation. 


A 23 year old student has just returned from 
India having been on a holiday. He was bitten 
by flies whilst he was there. He has been 
lethargic for Two months and has a fever. 
Clinical examination reveals 

hepatosplenomegaly. 

Ultrasound of the abdomen reveals 
lymphadenopathy. One of the lymph nodes 
are biopsied (a smear shows amastigotes 
within a macrophage). 

What is the diagnosis? 

1- Leishmaniasis 

2- Babesiosis 

3- Schistosomiasis 

4- Malaria 

5- Amoebiasis 



[ Q: 783 ] MRCPass - Infectious 
disease 


Answer & Comments 

Answer: 1- Leishmaniasis 

Leishmaniasis (Kala Azar) is spread by bites 
from sandflies. Cutaneous lesions can occur at 
the site of the bite. Visceral leishmaniasis can 
occur, causing hepatosplenomegaly. The 
smears can show Donovan bodies 
(amastigotes of Leishmania donovani). 



Dr. Khalid Yusuf El-Zohry - Sohag Teaching Hospital (01118391123) 

Ref MRCPass OE OE 2012 PasTest 2009 PassMedicine 2009 PasTest Exam ReviseMRCP 



339 





























El-zohry MRCP Questions Bank (Port 1) - 2013 


(For my personal use) 


Three Leishmania amastigotes, each with a 
clearly visible nucleus 


| *1 

A 45 year old man had roast beef lunch on 
Sunday. Later that night (about 8 hours later), 
he developed abdominal pains and diarrhoea. 
On examination, he had a temperature of 37.8 
C and generalised abdominal tenderness. 


[ Q: 784 ] MRCPass - Infectious 
disease 


3 

A 55 year old man has a Two week history of 
fever, dry cough, central pleuritic chest pain 
and breathlessness. 

On examination, he has a blood pressure of 
110/ 60 mmHg, heart rate of 115 bpm and the 
JVP is elevated by 8 cm. Chest X ray shows 
pulmonary shadowing. The ECG shows global 
ST elevation and T wave inversion. 


[ Q: 785 ] MRCPass - Infectious 
disease 


Stool examination reveals no evidence of 
blood. 

Which of the following infective organisms is 
likely? 

1- E coli 

2- Shigella 

3- Enterovirus 


What is the most likely diagnosis? 

1- Coxiella pneumonia 

2- Coxsackie virus infection 

3- Mycoplasma pneumonia 

4- Pneumocystis pneumonia 

5- Pulmonary tuberculosis 


4- Clostridium perfingens 

5- Rotavirus 


Answer & Comments 


Answer: 2- Coxsackie virus infection 


Answer & Comments 

Answer: 4- Clostridium perfingens 

The incubation time of 12-24 hours suggests 
that Clostridium perfingens is the most likely 
organism. Although the organism is well 
known for complications following trauma / 
wounds leading to gas gangrene, it can also 
cause diarrhoea. 

Clostridium perfingens multiplies within the 
gut with release of endotoxin during 
sporulation. It accounts for about 20% of 
bacterial diarrhoea. 

Clinically, there is abdominal pain and 
diarrhoea, rarely, vomiting, with onset of 
symptoms betw een 12 and 18 hours after 
incubation, and usually lasting for one day. 

Spores are ubiquitous - in animal and human 
gut, and the soil. Treatment is conservative for 
diarrhoea, but in the case of gas gangrene, 
penicillin is the antibiotic of choice. 


The pleuritic pains and ECG changes suggests 
myocarditis. One of the commonest cause of 
this is coxsackie virus. 

This may have led to pulmonary oedema, or 
even a pericardial effusion causing a raised 
JVP. 



[ Q: 786 ] MRCPass - Infectious 
disease 


A 37 year old male who was known to be HIV 
positive presents with malaise, and confusion. 
His CD4 count measured 1 month ago was 150 
x 10 6 /l. There was a witnessed generalized 
seizure 12 hours ago. MRI shows multiple ring 
enhancing mass lesions in the brain. 


What is the treatment of choice? 


1- Fluconazole 


2- Sulphadiazine and pyrimethamine 

3- Rifampicin and pyrazinimide 

4- Ceftazidime 


5- Prednisolone 


Dr. Khalid Yusuf El-Zohry - Sohag Teaching Hospital (01118391123) 

Ref MRCPass OE OE 2012 PasTest 2009 PassMedicine 2009 PasTest Exam ReviseMRCP 



























El-zohry MRCP Questions Bank (Port 1) - 2013 


(For my personal use) 


Answer & Comments 


Answer: 2- Sulphadiazine and pyrimethamine 


Answer & Comments 


Answer: 2- Toxoplasmosis 


The diagnosis is likely to be cerebral 
toxoplasmosis due to the multiple ring 
enhancing lesions on CT /MRI scans. 

There is an acute onset of focal neurological 
deficit over a few days e.g. hemiparesis, 
apraxia, visual field defects or cerebellar signs. 
Seizures may occur. 

There is commonly clouding of consciousness 
with fever and constitutional symptoms. 
Treatment of choice is with sulphadiazine and 
pyrimethamine. 



Cerebral Toxoplasmosis causing Ring 
Enhancing lesions 


[ Q: 787 ] MRCPass - Infectious 
disease 

A 6 year old child presented for a 
comprehensive eye exam. Her mother said 
she was born with some type of eye disease 
that left her blind in the left eye since birth 
and with very poor vision in the right eye. 
Dilated fundus exam demonstrated 
chorioretinal scars in both eyes. 

What is the likely infective diagnosis? 

1- CMV 

2- Toxoplasmosis 

3- Varicella zoster 

4- Herpes zoster 

5- Meningococcus 



Ocular toxoplasmosis results from infection of 
the retina by Toxoplasma gondii, an 
intracellular parasite that resides in cats' 
intestines, undercooked meats or other foods 
that contain the tissue cysts, or from mother 
to child during pregnancy.The ocular findings 
are the most common features of congenital 
toxoplasmosis. Affected infants usually are 
born with bilaterally healed chorioretinal scars 
in the posterior pole. 


- 

• 9 4 

# y 


Fundus showing choroidoretinal scars - Ocular 

Toxoplasmosis 


[ Q: 788 ] MRCPass - Infectious 
disease 

A 35 year old lecturer is taken ill on returning 
from a 2 week walking holiday around Eastern 
Europe. He presents w ith headache, neck 
stiffness, photophobia, and right sided Bell's 
palsy. He complains of polyarthralgia affecting 
shoulders, hips and knees associated with 
fever and fatigue for the previous 2 weeks 
associated with an urticarial type rash 
affecting the right thigh. 

What is the diagnosis? 

1- Infectious Mononucleosis 

2- Rheumatic fever 

3- Leishmaniasis 

4- Schistosomiasis 

5- Lyme disease 



Dr. Khalid Yusuf El-Zohry - Sohag Teaching Hospital (01118391123) 

Ref MRCPass OE OE 2012 PasTest 2009 PassMedicine 2009 PasTest Exam 




ReviseMRCP 

341 



























El-zohry MRCP Questions Bank (Port 1) - 2013 


(For my personal use) 


Answer & Comments 

Answer: 5- Lyme disease 

This patient is presenting in the second stage 
of Lyme disease as characterised by the 
neurological involvement of his Bell's palsy. 
Lyme disease due to Borellia burgdorferi is the 
commonest vector bourne disease in the USA 
and occurs widely throughout Europe and the 
former Soviet Union. 


Erythema Chronicum Migrans due to Lyme 

disease 


Temporal lobe changes on the MRI suggest 
that HSV encephalitis is likely. Treatment is 
with high dose IV acyclovir 10-15 mg/kg tds. 


[ Q: 790 ] MRCPass - Infectious 
disease 

A 30 year university lecturer develops fevers, 
myalgia, lethargy and joint pains over 5 days, 
after contact with a colleague with a similar 
illness. She has a temperature of 39 °C. On 
examination, she has a cheek rash, synovitis of 
the hand and knee joints, and palpable lymph 
nodes in the cervical area. 

What is the likely diagnosis? 

1- Infectious mononucleosis 

2- Lyme disease 

3- Listeriosis 

4- Leptospirosis 

5- Parvovirus B19 




^ Q: 789 ] MRCPass - Infectious 

ft 

S disease 

A 65 year old woman presents with a 2 day 
history of fever, generalized headaches and 
confusion. An MRI scan shows increased signal 
in the right temporo-parietal area. CSF shows 
100 white cells (85% lymphocytes), protein 
0.65 g/l and an opening pressure of 21 cm. 
Glucose is normal. No organisms are seen on 
microscopy. 

What is the most likely organism? 

1- Neisseria meningitidis 

2- Herpes Simplex Virus type 1 

3- Mycobacterium avium intracellulare 

4- Mycobacterium tuberculosis 

5- Streptococcus viridans 


Answer & Comments 

Answer: 2- Herpes Simplex Virus type 1 

The fever and confusion suggest encephalitis 
and are often accompanied by fitting. 


Answer & Comments 

Answer: 5- Parvovirus B19 

Parvovirus infection or fifth's disease can 
cause the 'slapped cheek syndrome'. 

There is a cheek rash with sw ollen in the 
wrist, hands and knees. Diagnosis can be 
confirmed with an IgM antibody to parvovirus 
B19. 



[ Q: 791 ] MRCPass - Infectious 
disease 

A 34 year old patient has a CD4+ count of 
80/mm. He has had a generalised seizure 
recently. An MRI scan is performed, it shows 
multiple 1 cm white matter lesions. 


Which of these diagnoses is likely? 

1- Progressive multifocal 

leukoencephalopathy 

2- Demyelination 


3- Calcified tubers 


Dr. Khalid Yusuf El-Zohry - Sohag Teaching Hospital (01118391123) 

Ref MRCPass OE OE 2012 PasTest 2009 PassMedicine 2009 PasTest Exam ReviseMRCP 






























El-zohry MRCP Questions Bank (Port 1) - 2013 


(For my personal use) 


4- Behcet's disease 

5- Systemic lupus erythematosus 


Answer & Comments 

Answer: 1- Progressive multifocal 

leukoencephalopathy 

Progressive multifocal leukoencephalopathy 
(PML) is caused by the JC virus, and causes 
white matter lesions in the brain. A CD4+ of 
90/mm (<400) is low and suggests HIV 
infection. Anti-retroviral therapy is the main 
treatment for PML. 



Multifocal areas of demyelination in PML 



[ Q: 792 ] MRCPass - Infectious 
disease 


A 20 year old student returns from a 
backpacking trip in Nepal. He had a 3 week 
history of diarrhoea with associated weight 
loss. There was no blood in his stools. 


Whot is the likely infective organism? 

1- Giardia lamblia 


2- Shigella flexneri 

3- Yersinia enterocolitica 

4- Escherichia coli 


5- Salmonella typhi 


Answer & Comments 

Answer: 1- Giardia lamblia 

Giardiasis is most likely as it presents as 
chronic diarrhoeal illness (without blood) due 
to duodenal infestation. The rest of the 
organisms cause more acute diarrhoea, 


salmonella and shigella diarrhea are 
associated with blood. 



Giardia 



[ Q: 793 ] MRCPass - Infectious 
disease 

A 30 year old man presents with fevers, 
malaise and a cough. There was associated 
myalgia. He is a type 1 diabetic. He works in a 
water purifier factory, and legionella infection 
is supected. 

Which of the following tests is most practical 
for confirming the diagnosis? 

1- Serum Immuno Fluorescent Antibody 

2- Sputum Immuno Fluorescent Antibody 

3- Sputum microscopy and culture 

4- Urinary antigen 

5- PCR for legionella DNA 


Answer & Comments 

Answer: 4- Urinary antigen 

The urine antigen test is a rapid, relatively 
inexpensive, and practical test for the 
detection of L pneumophila antigen excreted 
in the urine or present in pleural fluid. Direct 
fluorescent antibody (DFA) staining is a rapid 
test that can be performed on respiratory 
samples and tissue and requires only 2-4 
hours for results. It is very specific but not 
sensitive, hence a negative result does not 
rule out legionella infection. PCR is not widely 
available. 


Dr. Khalid Yusuf El-Zohry - Sohag Teaching Hospital (01118391123) 

Ref MRCPass OE OE 2012 PasTest 2009 PassMedicine 2009 PasTest Exam ReviseMRCP 



343 




























El-zohry MRCP Questions Bank (Port 1) - 2013 


(For my personal use) 


^ [ Q: 794 ] MRCPass - Infectious 

0 disease 

A 75 year old man presents with sudden onset 
of weakness of his right arm on a background 
of a 6 week history of lumbar back pain, 
weight loss, fever and night sweats. 

Blood tests show : 

Hb 9.9 g/dL 

white cell count 13.5 

platelets 470 

erythrocyte sedimentation rate (ESR) 100 
mm/hr 

creatinine 195 micromol/L 
calcium 2.45 (2.25-2.7) mmol/l 
phosphate 0.22 (0.8-8) pmol/l 
IgA 1.3(0.5-4.0) g/l 
IgG 14 (5.0-13.0) g/l 
IgM 2.7 (0.3-2.2) g/l 

Urine dispstick - microscopic haematuria 
What is the most likely diagnosis? 

1- Endocarditis 

2- Secondary syphilis 

3- Paget's disease 

4- Tuberculosis 

5- Myeloma 

Answer & Comments 

Answer: 1- Endocarditis 


disease 

A 17-year-old female student from an inner- 
city high school presented to her general 
practitioner requesting a first prescription for 
the oral contraceptive pill. The patient began 
sexual activity 6 months previously, had had 
five sexual partners and never used condoms. 
She had no genital symptoms. 

What is a urethral swab most likely to grow ? 

1- Syphilis 

2- Chlamydia trachomatis 

3- Ureaplasma 

4- Klebsiella 

5- Trichomonas vaginalis 

Answer & Comments 

Answer: 2- Chlamydia trachomatis 

Chlamydia is the commonest organism causing 
non gonococcal urethritis. 

Urethritis can be caused by a number of 
organisms, including Neisseria gonorrheae, 
Chlamydia trachomatis, Ureaplasma 
urealyticum, Mycoplasma genitalium, 
Trichomonas vaginalis, Herpes simplex virus, 
and Candida albicans. About half of all men 
and three-quarters of all women who have 
chlamydia have no symptoms and do not 
know that they are infected. Urethritis can 
cause dysuria eg. a burning sensaton. 
Doxycycline or Azithromycin are the 
treatment of choice. 


Infective endocarditis with associated 
osteomyelitis would explain the clinical 
picture of back pains, raised inflammatory 
markers and dipstick positive for blood. 

The mildly raised immunoglobulins would go 
with infection rather than myeloma. 



[ Q: 795 ] MRCPass - Infectious 



[ Q: 796 ] MRCPass - Infectious 
disease 


A 25 year old secretary comes to the clinic 
complaining of fevers, crampy abdominal 
pains and diarrhoea. She has returned from 
Turkey on a holiday. Whilst there, she visited 
Two spas and spent a long time in jacuzzis. 


Which of the following organisms might be 
isolated from stool culture? 


Dr. Khalid Yusuf El-Zohry - Sohag Teaching Hospital (01118391123) 

Ref MRCPass OE OE 2012 PasTest 2009 PassMedicine 2009 PasTest Exam ReviseMRCP 




























El-zohry MRCP Questions Bank (Port 1) - 2013 


(For my personal use) 


1- Vibrio cholerae 

2- Cryptosporidium 

3- Salmonella 

4- Shigella 

5- Actinomyces 

Answer & Comments 

Answer: 2- Cryptosporidium 

Cryptosporidium is a protozoan which is 
commonly water borne spread. Swimming in 
hot tubs and pools, lakes, ponds are risk 
factors. It can also be spread via uncooked 
food. Treatment is conservative in adults, 
whilst the drug nitazoxanide is licensed for 
treatment in children age < 12. Symptoms 
typically last for 1-2 weeks. 


African tick typhus (a form of rickettsial 
infection) is characterised by fevers, rash 
(which may be maculopapular or petechial) 
and an eschar at the site of tick bite (the bite 
is often not noticed by the patient). Rickettsial 
infection is confirmed serologically with acute 
and convalescent titres. Other examples of 
rickettsial infection include Rocky Mountain 
spotted fever and louse borne typhus. The 
treatment of choice is doxycycline. 



Eschar of tick typhus 


[ Q: 797 ] MRCPass - Infectious 
disease 

An 18 year old man has recently returned 
from Kruger National Park in South Africa. He 
has a fever, headaches and arthralgia. He has 
a small black ulcer on the inner arm and a 
widespread maculopapular rash. 

Investigations show : 

Hb 11.5 g/l 

WBC 6.6 x 10 9 /l_ 

LDH 680 iu/l 

Pits 115 x 10 9 /L 

What is the likely diagnosis? 

1- Rickettsial infection 

2- Malaria 

3- Leishmaniasis 

4- Schistosomiasis 

5- Brucella 



Answer & Comments 


Answer: 1- Rickettsial infection 


j 

A 29 year old banker presents with fever and 
loss of weight. He has spent several months 
travelling across the countries around Africa 
and Asia. He admits that he did not take 
malarial prophylaxis. On examination he has 
lymphadenopathy and hepatosplenomegaly. 
The full blood count shows pancytopaenia. 
Aspirates are taken from bone marrow and 
Giemsa stained smears of these aspirates 
show amastigotes. 

Which is the most likely infective organism? 

1- Falciparum malaria 

2- Leishmania donovani 

3- Leishmania major 

4- Trypasonoma cruzi 

5- Trypasonoma brucei 



[ Q: 798 ] MRCPass - Infectious 
disease 


Answer & Comments 

Answer: 2- Leishmania donovani 

This patient has clinical features of visceral 
leishmaniasis and aspirates have 


Dr. Khalid Yusuf El-Zohry - Sohag Teaching Hospital (01118391123) 

Ref MRCPass OE OE 2012 PasTest 2009 PassMedicine 2009 PasTest Exam ReviseMRCP 





























El-zohry MRCP Questions Bank (Port 1) - 2013 


(For my personal use) 


demonstrated amastigotes confirming the 
diagnosis. Visceral leishmaniasis is caused by 
parasites of the Leishmania donovani. 



L donovani amastigotes 



[ Q: 799 ] MRCPass - Infectious 
disease 


A 45 year old man has become progressively 
more unw ell since return from a business trip 
to Ghana 2 days ago. 


He had a high fever on admission, but was not 
confused. A malarial film shows 12% 
parasitaemia with Plasmodium falciparum. 


What is the most appropriate treatment 
option? 


1- Atovaquone 

2- Chloroquine 

3- Proguanil 

4- Quinine 


5- Co-trimoxazole 


Answer & Comments 

Answer: 4- Quinine 

In severe falciparum malaria either 
intravenous quinine or artesunate should be 
administered. Atovaquone is a component of 
Malarone (also proguanil) which is liscenced in 
uncomplicated falciparum malaria. This 
patient may also be considered for exchange 
transfusion (considered in parasitaemia >20% 
or >10% with organ failure). 






[ Q: 800 ] MRCPass - Infectious 



disease 

A 65 year old man has been admitted to the 
ward following a myocardial infarction. He is a 
mild diabetic and is hypertensive but both 
these conditions are well controlled. 

Seven days after admission the patient 
develops fever, tachycardia and tachypnoea. 
On auscultation of his chest crepitations are 
heard over both lung bases. Chest X-ray 
demonstrates bilateral basal pulmonary 
infiltrates. 

Empirical antibiotic treatment for this 
condition will be based on the assumption 
that the most likely causative organisms are: 

1- Gram-negative organisms 

2- Staphylococcus aureus 

3- Pneumococcus 

4- Mycoplasma 

5- Neisseria meningitidis 


Answer & Comments 

Answer: 1- Gram-negative organisms 

Gram-negative organisms are the most likely 
cause of hospital-acquired pneumonia. 
Examples are Klebsiella, Pseudomonas, 
Enterobacter, Serratia. Powerful antibiotics 
used against these organisms include the 
fourth-generation cephalosporins, 

carbapenems, ciprofloxacin alone or in 
combination with an aminoglycides (entamicin 
or tobramycin). 



[ Q: 801 ] MRCPass - Infectious 
disease 


Two days after returning from a 1-week trip 
around Thailand, a 30 year old woman 
presents with sudden onset of fever, 
headache and myalgia. Three days after her 
symptoms started she develops a generalized 
erythematous rash. Her investigations show : 


Hb is 12 g/ dl 
WCC 2.2 x 10 9 / I 


Dr. Khalid Yusuf El-Zohry - Sohag Teaching Hospital (01118391123) 

Ref MRCPass OE OE 2012 PasTest 2009 PassMedicine 2009 PasTest Exam ReviseMRCP 






























El-zohry MRCP Questions Bank (Port 1) - 2013 


(For my personal use) 


platelets 75 x 10 9 / I 

What is the most likely diagnosis? 

1- Hepatitis C 

2- Dengue fever 


Mycobacterium bovis infection can be 
transmitted from contact with farm animals. 
Complications include granulomatous 
prostatitis,hepatitis, skin abscess, ureteral 
obstruction and epididymoorchitis. 


3- Tick-borne encephalitis 

4- Syphilis 

5- Malaria 


Answer & Comments 

Answer: 2- Dengue fever 

This presentation is typical presentation of 
someone with dengue fever which has an 
incubation period of 5 to 8 days. Dengue fever 
is a condition caused by an RNA virus 
(arbovirus), which is common in India, South 
East Asia and the Pacific. Spread is by 
mosquitos. 


A 70 year old white man has spent several 
years in Greece where he was exposed to 
farm animals and also had a history of 
household tuberculosis (TB) contact. 

He was referred for pain, sw elling and 
decreased function of his left knee prosthesis 
which was implanted 5 years ago. The knee 
was aspirated and a tissue biopsy was 
performed. Sections of soft tissue showed 
chronic synovitis with a histiocytic reaction 
and a rare focus of epithelioid granulomas. 

What organism is likely to be cultured? 

1- Staphylococcus aureus 

2- Streptococcus faecalis 

3- Chlamydia 

4- Mycobacterium bovis 



[ Q: 802 ] MRCPass - Infectious 
disease 



Epitheloid granulomas 



[ Q: 803 ] MRCPass - Infectious 
disease 


A 45 year old man has a history of multiple 
episodes of sudden, abdominal pain and back 
pains. 


A haemoglobin electrophoresis shows 95% Hb 
S, 4% Hb F, 1% Hb A2. He also has increasing 
pain in his right groin radiating to the anterior 
aspect of the thigh. His temperature was 38°C. 
An X ray reveals irregular bony destruction of 
the femoral head. 


Which is the most likely organism to be 
responsible? 

1- Mycoplasma 

2- Yersinia pestis 

3- Salmonella 

4- Candida 


5- Moraxella 


Answer & Comments 


Answer: 3- Salmonella 


5- Neisseria gonorrhoeae 


Answer & Comments 


Answer: 4- Mycobacterium bovis 


Salmonella osteomyelitis is frequently seen in 
patients with sickle cell anemia. Patients with 
sickle cell anemia also exhibit increased 
susceptibility to other common infectious 


Dr. Khalid Yusuf El-Zohry - Sohag Teaching Hospital (01118391123) 

Ref MRCPass OE OE 2012 PasTest 2009 PassMedicine 2009 PasTest Exam 




ReviseMRCP 

347 




























El-zohry MRCP Questions Bank (Part 1) - 2013 


(For my personal use) 


agents, including Mycoplasma pneumoniae. 
Staphylococcus aureus, and Escherichia coli. 


vh 

A 12 year old girl with sickle cell presents with 
abdominal pains and generalised lethargy. On 
examination, she w as moderately unw ell and 
was noted to be pale. Her temperature was 
37°C. Her blood tests show Hb 4.0 g/dl, WCC 4 
x 10 9 /L, platelets 50 x 10 9 /L, urea 6 pmol/l, 
creatinine 90 pmol/l, sodium 140 mmol/l, 
potassium 4.2 mmol/l, bilirubin 28 pmol/l. 

Which one of these infections is likely? 

1- Hepatitis B 

2- CMV 

3- EBV 

4- Parvovirus B19 

5- Polio virus 


[ Q: 804 ] MRCPass - Infectious 
disease 


Answer & Comments 

Answer: 4- Parvovirus B19 

In sickle cell disease, infection with parvovirus 
can cause aplastic anaemia, as seen in this 
case, where there is a pancytopenic picture in 
the full blood count profile. The condition is 
self limiting and the patient may recover 
within one to Two weeks. 


y 

A 30 year old man presented with bloody 
diarrheoa. This started 2 days ago. He 
returned from a business trip to Egypt recently 
1 week ago. 

What is the most likely causative organism? 

1- Cholera 

2- E coli 

3- Giardiasis 

4- Shigella 


[ Q: 805 ] MRCPass - Infectious 
disease 



5- Crytosporidiosis 

Answer & Comments 

Answer: 4- Shigella 

The common causes of bloody diarrhoea 
include Salmonella , Shigella , Campylobacter 
and amoebiasis. E. coli type E 0157 can also 
cause bloody diarrhoea with Haemolytic 
uraemic syndrome. 


A 70 year old man presents to casualty with a 
fever and headache. On examination he is 
confused, has neck stiffness and a right 6th 
cranial nerve palsy. He has no visible rash. 

Investigations show : 

CSF Protein 3 g/L 

CSF Glucose 1.1 (3.3 to 4.4 mmol/l), plasma 
glucose 5 mmol/l 

CSF Microscopy 350 white cells, 
predominantly lymphocytes 

Serum VDRL positive and TPHA is negative 

The most likely diagnosis is: 

1- Meningococcal meningitis 

2- Herpes simplex encephalitis 

3- Listeria meningitis 

4- Lymphocytic meningitis 

5- Tuberculous meningitis 

Answer & Comments 

Answer: 5- Tuberculous meningitis 

In TB meningitis, the prodrome is nonspecific, 
including headache, vomiting, photophobia 
and fever. Cranial nerve palsies can occur. The 
duration of presenting symptoms may vary 
from 1 day to 9 months. CSF typically shows 
elevated protein level, markedly low glucose, 
and a pleocytosis, initially polymorphs then 
lymphocytes. 


[ Q: 806 ] MRCPass - Infectious 
disease 



Dr. Khalid Yusuf El-Zohry - Sohag Teaching Hospital (01118391123) 

Ref MRCPass OE OE 2012 PasTest 2009 PassMedicine 2009 PasTest Exam ReviseMRCP 































El-zohry MRCP Questions Bank (Port 1) - 2013 


(For my personal use) 


In this case, VDRL may be a sign of previous 
syphilis or may be false positive. TPHA is much 
more specific, hence since it is negative in this 
case, syphilis infection is unlikely. 

The best antimicrobial agents in the treatment 
of TBM include isoniazid (INH), rifampicin 
(RIF), pyrazinamide (PZA), and streptomycin 
(SM), all of which enter CSF readily in the 
presence of meningeal inflammation. 
Ethambutol (EMB) is less effective in 
meningeal disease unless used in high doses. 


[ Q: 808 ] MRCPass - Infectious disease 

A 40 year old lady presents with a cellulitis in 
the left leg. She had a laceration to the area 3 
days ago. Blood cultures grow a gram-negative 
rod subsequently identified as Pasteurella 
multocida. 

What was the most likely cause of the 
penetrating injury? 

1- Snake bite 

2- Cat bite 



[ Q: 807 ] MRCPass - Infectious 
disease 


A 45 year old patient presents with 
meningism. There is no past medical history. 
CT scan was normal and he had the following 
CSF results: 


3- Spider bite 

4- Bee sting 

5- Scorpion bite 


Answer & Comments 


Answer: 2- Cat bite 


150 X 10 6 /ml white cells (90% lymphocytes) 
protein was 6g/l 
glucose 2.2 mmol/I 

Microscopy revealed no gram positive 
organisms and no Acid fast bacilli were seen. 

Which is the next best test? 

1- TB PCR of the cerebrospinal fluid 

2- CSF cytology 

3- Heaf test 

4- Herpes viral serology 

5- Blood cultures 


Answer & Comments 

Answer: 1- TB PCR of the cerebrospinal fluid 

A CSF lymphocytosis and low glucose points 
tow ards TB meningitis. 

Despite no AFBs being seen, the TB PCR is a 
rapid way to confirm his diagnosis definitively. 


Pasteurella multocida is found in the snouts of 
both dogs and cats. Soft tissue infection 
results following bites and may progress to 
tenosynovitis, osteomyelitis or lymphangitis 
depending on the site of the bite. Drug 
therapy is with penicillin. 



[ Q: 809 ] MRCPass - Infectious 
disease 


An 18 year old man has been working on a 
farm during the summer holidays from 
university. He is now unwell. 


On examination his BP is 115/65 mmHg and 
temperature is 38 C. He has jaundice and mild 
hepatosplenomegaly. 


Blood tests reveal: Hb 13.5 g/dl, WCC 11.2 x 
10 9 /L, platelets 225 x 10 9 /L, sodium 137 
mmol/I, potassium 4.2 mmol/I, urea 14 
pmol/l, creatinine 180 pmol/l. 


Which test is most likely to reveal the 
underlying diagnosis? 



1- Brucella antibodies 

2- Paul Bunnell test 

3- Leptospira antibodies 


Dr. Khalid Yusuf El-Zohry - Sohag Teaching Hospital (01118391123) 

Ref MRCPass OE OE 2012 PasTest 2009 PassMedicine 2009 PasTest Exam ReviseMRCP 




























El-zohry MRCP Questions Bank (Port 1) - 2013 


(For my personal use) 


4- MRI to look for cystic lesions 

5- HIV test 

Answer & Comments 

Answer: 3- Leptospira antibodies 

Leptospirosis is spread by rodents, those at 
risk are farmw orkers and those working at 
sewers. It can lead to fevers, jaundice, 
haemoptysis and renal impairment. The 
diagnosis is confirmed by dark field 
microscopy of urine and serologically. 


5- Pneumocystis pneumonia 


Answer & Comments 


Answer: 5- Pneumocystis pneumonia 

The diagnosis is pneumocystis pneumonia. HIV 
infection is suggested by the low white cell 
count and oral candidiasis. Dry cough, fever 
and tachypnoea are typical in PCP. The 
organism lies in the alveolar space (foam), 
causing hypoxia and a low transfer factor. 
Typically there are no crackles, although it 
may occasionally be present. 



Spiral shaped Leptospira interrogans 



Pneumocystis pneumonia 


^ [ Q: 810 ] MRCPass - Infectious 



fjt disease 

\%\ 

• i j 

i_? 

[ Q: 811 ] MRCPass - Infectious 

A 45 year old African American male presents 

disease 


with sudden-onset chest pain which he 
associated with a high fever, dry cough, and 
shortness of breath. He was found to have low 
oxygen saturation betw een 80-90%. Oral 
examination revealed moist mucosa but mild 
thrush was noted on tongue. Respiratory 
exam showed fine crackles in low er one-third 
of bilateral bases. 

A chest X-ray revealed prominent interstitial 
markings. Blood tests reveal a white cell count 
of 3 x 10 9 /L. 

Whot is the likely diagnosis? 

1- Asthma 

2- Allergic Bronchopulmonary Aspergillosis 

3- Legionella Pneumonia 

4- Histoplasmosis 


A 30 year old lady presents with headache and 
neck stiffness. Her temperature is 38.5°C, BP 
100/65 and she has a petechial rash in the 
thigh. CSF examination reveals gram negative 
diplococci. 

Which is the best antibiotic therapy? 

1- Gentamicin 

2- Flucloxacillin 

3- Cefuroxime 

4- Ciprofloxacin 

5- Benzylpenicillin 

Answer & Comments 

Answer: 5- Benzylpenicillin 


Dr. Khalid Yusuf El-Zohry - Sohag Teaching Hospital (01118391123) 

Ref MRCPass OE OE 2012 PasTest 2009 PossMedicine 2009 PasTest Exam ReviseMRCP 




























El-zohry MRCP Questions Bank (Port 1) - 2013 


(For my personal use) 


The diagnosis is meningococcal meningitis (it 
would be pneumococcal meningitis if gram 
positive diplococci were seen). Intravenous 
ceftriaxone or benzylpenicillin are treatment 
of choice. 



Meningococcal Rash 



[ Q: 812 ] MRCPass - Infectious 
disease 


A 32 year old woman was admitted to hospital 
with cough and breathlessness. She had been 
well until two w eeks previously, when she 
developed headache and nausea followed by 
a cough. Her general practitioner prescribed 
amoxycillin with clavulanic acid on the day of 
admission. She had no history of foreign travel 
and kept no pets. 


On admission she was unw ell and pyrexial. 
The main abnormal signs were a raised heart 
rate (140 beats/min), 

and respiratory rate (32 breaths/min). There 
were left basal crackles. Chest x ray showed 
mild shadowing. 


Which one of the following tests should be 
sent for making a diagnosis of legionella 
infection ? 


1- Serum immunofluorescent antibody 

2- Sputum immunofluorescent antibody 


3- Sputum culture 


4- Urinary antigen 

5- Blood cultures 


Answer & Comments 

Answer: 4- Urinary antigen 

The urine antigen test (a radioimmunoassay) 
is a rapid, relatively inexpensive, and practical 
test for the detection of L pneumophila 
antigen excreted in the urine or present in 
pleural fluid. 



[ Q: 813 ] MRCPass - Infectious 
disease 


A 60 year old man presented with fatigue. He 
gave a history of Two similar episodes of 
extreme fatigue in the past five years. During 
one of these episodes, elevated liver enzymes 
were found. An examination showed that he 
was otherw ise healthy. He was not on 
medications, and he denied drinking. He had 
no known family history of liver disease. 


Laboratory Results: 
AST: 349 U/l 
ALT: 452 U/l 


Total bilirubin: 70 umol/l 


HBsAg: positive 
Anti-HCV: negative 

After several months, he was follow ed up by 
a gastroenterologist, his symptoms had 
resolved. Repeat testing at this time showed 
the following: 

Laboratory Results: 

AST: 55U/I 

ALT: 68 U/l 

Total bilirubin: 25 umol/l 
HBeAg: negative 
Anti-HBe: positive 
HBV DNA: 125,000 copies/mL 
What is the diagnosis? 

1- Superimposed hepatitis E 

2- Superimposed hepatitis D 

3- Acute antigen negative hepatitis B 


Dr. Khalid Yusuf El-Zohry - Sohag Teaching Hospital (01118391123) 

Ref MRCPass OE OE 2012 PasTest 2009 PassMedicine 2009 PasTest Exam 




ReviseMRCP 

351 

























El-zohry MRCP Questions Bank (Port 1) - 2013 


(For my personal use) 


4- Chronic antigen negative hepatitis B 

5- Hepatitis B in remission 


Answer & Comments 

Answer: 4- Chronic antigen negative hepatitis 
B 

This is a case of patients with E-antigen 
negative chronic Hepatitis. Despite E-antigen 
being negative, these patients can continue to 
have active HBV DNA level and active liver 
disease. 


^ Q: 814 ] MRCPass - Infectious 
0 disease 

A 52 year man enquired about whether it was 
advisable to have vaccination prior a holiday 
abroad. He had asthma treated with long term 
steroids. Frequently, courses of Prednisolone 
in excess of 30mg daily were given. 

Which one of the following vaccinations is 
contraindicated in the patient? 

1- Hepatitis B 

2- Diphtheria toxoid 

3- Yellow fever 

4- Meningococcus 

5- Tetanus toxoid 


Answer & Comments 

Answer: 3- Yellow fever 
The live vaccines are: 

■ BCG 

■ Mumps 

■ Measles 

■ Rubella 

■ Yellow fever 
Smallpox 






[ Q: 815 ] MRCPass - Infectious 



disease 

An 60 year lady has a 3 month history of dry 
cough. She feels sw eaty at night. Her 
temperature is 39 C and chest X ray shows a 
cavitating lesion in the right upper lobe. 
Induced sputum was attempted but was 
unsuccessful. 

Which of the following investigations would be 
useful in establishing the cause of this lesion? 

1- CT of the chest 

2- Ultrasound of the lesion 

3- Bronchoscopy 

4- Aspergillus serology 

5- Percutaneous biopsy 


Answer & Comments 

Answer: 3- Bronchoscopy 

This patient is likely to have tuberculosis, but 
induced sputum has been unsuccessful to 
send for AFBs. The best way of obtaining a 
diagnosis is to get a bronchoscopy with 
bronchial washings to send for TB culture. 


A 35 year old female is referred due to 
positive serology for syphilis. She gives a 
history of treatment for syphilis 5 years ago. 
Tests show a positive venereal disease 
reference laboratory (VDRL) titre of 1:128 and 
a positive Treponema pallidum 
haemagglutination assay (TPHA) titre of 
1:1024. 

Which of the following explanations is most 
consistent with these data? 

1- Tertiary syphilis 

2- Active syphilis reinfection 

3- Inadequate previous treatment 

4- Pregnancy 

5- Superimposed gonorrhoea treatment 



[ Q: 816 ] MRCPass - Infectious 
disease 


Dr. Khalid Yusuf El-Zohry - Sohag Teaching Hospital (01118391123) 

Ref MRCPass OE OE 2012 PasTest 2009 PassMedicine 2009 PasTest Exam ReviseMRCP 































El-zohry MRCP Questions Bank (Port 1) - 2013 


(For my personal use) 


Answer & Comments 

Answer: 2- Active syphilis reinfection 

Successful therapy for syphilis leads to a 
steady fall in the VDRL or rapid plasma reagin 
(RPR) titre. Following primary disease the 
VDRL is generally negative within 1 year, and 
within 2 years for secondary syphilis. A small 
number of treated patients have a persistently 
low detectable VDRL. 

This patient's high VDRL titre of 1:128 most 
likely represents recent acquisition of 
infection - within the past 12 months. 


n 

A 40 year old man presented 14 days after 
return from a 6-w eek field trip to Papua New 
Guinea. He had a six day history of high fevers 
and rigors. On the day of presentation, he had 
become vague and confused. He had taken 
antimalarials as prophylaxis, but ceased when 
he found that local people did not take them. 

His temperature was 40oC, pulse rate 140 
bpm, respiratory rate 28 per minute, and 
blood pressure 100/60 mmHg. 

He had dry mucous membranes, mild 
jaundice, pallor, splenomegaly and 
generalised crackles in both lungs. 

Full blood examination revealed: 

6.5 g/dL 

WCC 2.5 x 10 9 /L 

Platelet 10 x 10 9 /L 

bilirubin 60 pmol/L (3-20 pmol/L) 

lactate dehydrogenase 489 U/L (100-225 U/L) 
creatinine 250 umol/l 

What is the likely diagnosis? 

1- Leishmaniasis 


[ Q: 817 ] MRCPass - Infectious 
disease 


2- Tick bite fever 

3- Endocarditis 


4- Falciparum malaria 


5- Viral haemorrhagic fever 

Answer & Comments 

Answer: 4- Falciparum malaria 

This patient has severe malaria suggested by 
altered consciousness, focal neurological 
signs, jaundice, oliguria, severe anaemia, 
hypoglycaemia, hypotension and acidosis. 


Severe malaria requires treatment with 
intravenous quinine. 



P. falciparum rings in erythrocytes 


j 

A 62 year old woman was admitted to the 
emergency department with deterioration in 
her level of consciousness. On examination, 
her pupils were equal and reactive, Kernigs 
and Brudzinski's signs were positive. 

Investigastions showed: 

CT scan of the head - cerebrovascular disease. 

Lumbar puncture was performed and this 
showed an opening pressure > 25 cmH 2 0. 

CSF showed a white cell count in 
cerebrospinal of 133 (all polymorphs), red 
blood cell count of 25 (no xanthochromia), 
and protein level of 0.75 g/l. Glucose 2.5 
mmol/l. 



[ Q: 818 ] MRCPass - Infectious 
disease 


Dr. Khalid Yusuf El-Zohry - Sohag Teaching Hospital (01118391123) 

Ref MRCPass OE OE 2012 PasTest 2009 PassMedicine 2009 PasTest Exam 




ReviseMRCP 

353 






























El-zohry MRCP Questions Bank (Part 1) - 2013 


(For my personal use) 


Which is the likely causal organism ? 

1- Listeria monocytogenes 

2- Streptococcus pneumoniae 

3- E coli 

4- Klebsiella 

5- Pseudomonas aeruginosa 


Answer & Comments 

Answer: 2- Streptococcus pneumoniae 

The commonest cause of bacterial meningitis 
in the elderly is Strep pneumoniae. The CSF 
glucose is only slightly low and hence is not 
consistent with TB meningitis. 



[ Q: 819 ] MRCPass - Infectious 
disease 

A 35 year old man develops a fever 7 days 
post bone marrow transplantation. He is 
placed empirically on Cefuroxime but remains 
febrile. After a few days, he develops a few 
painless, red, papular lesions on his trunk and 
lower limbs. 


Investigation results are: 

Hb 10.5 g/dl 
MCV 87 fl 

WCC 18 x 10 9 /L (50% lymphocytes) 

platelets 130 x 10 9 /L 

ALT 130 (5-35) U/l 

AST 95 (1-31) U/l 

ALP 140(20-120) U/l, 

Bilirubin 35 (1-22) pmol/l 

Albumin 36 (37-49) g/l 

What is the likely cause of these lesions? 

1- Stevens Johnsons syndrome 

2- Staphylococcal infection 


3- Candidal infection 


4- Graft versus host disease 

5- Aspergillus infection 


Answer & Comments 

Answer: 4- Graft versus host disease 

The presentation of acute graft versus host 
disease is often a triad of dermatitis, hepatitis, 
and gastroenteritis, although symptoms may 
occur alone or in different combinations. 


Maculopapular rash may present with the 
onset occurring within 5-47 days after 
transplantation. Pruritus involving the palms 
and soles may precede the rash. 

Anaemia and thrombocytopenia are common. 
The liver is the second most common organ 
involved. GVHD also manifests as elevated 
liver transaminases levels. Cholestatic 
jaundice is common. 


Successful therapeutic intervention of life- 
threatening GVHD is possible, although the 
consequence can be the development of fatal 
opportunistic infections. Therefore, the best 
approach to manage GVHD should be its 
prevention. 



Maculopapular rash seen in Graft versus Host 

Disease. 


Dr. Kholid Yusuf El-Zohry - Sohog Teaching Hospital (01118391123) 

Ref MRCPass OE OE 2012 PasTest 2009 PassMedicine 2009 PasTest Exam ReviseMRCP 
























El-zohry MRCP Questions Bank (Port 1) - 2013 


(For my personal use) 



[ Q: 820 ] MRCPass - Infectious 
disease 

A 31 year old patient who is HIV positive has a 

CD4 count of 200 cells /mm * 1 2 3 4 5 . He has a viral 

load of 220,000 cells/ml. He is feeling well at 

present. 

Whot should be the next management step? 

1- Start antiretroviral therapy now 

2- Start antiretroviral therapy when viral load 
is > 300,000 cells/ml 

3- Start antiretroviral therapy when CD4 count 
is < 100 

4- Start antiretroviral therapy when CD4 count 
is < 150 

5- Start antiretroviral therapy when patient is 
symptomatic 


Answer & Comments 

Answer: 2- Trophyrema whippelli 

Whipple's disease typically presents as a 
gastrointestinal illness caused by the organism 
Tropheryma whippelli. The illness is 
characterized by diarrhea, abdominal cramps, 
and sometimes frank malabsorption. If 
gastrointestinal disease is prominent, 
duodenal biopsy is performed, it often yields 
evidence of Trophyrema whippelli by light 
microscopy, electron microscopy, or PCR, 
allowing the diagnosis to be substantiated. 

Histopathologically, one sees macrophages 
containing periodic acid-Schiff (PAS)-positive 
material. The characteristic rod-shaped 
intracellular organism is seen by electron 
microscopy. 


Answer & Comments 

Answer: 1- Start antiretroviral therapy now 

The general recommendation for considering 
HAART and best prognosis is to they start 
when patients' CD4 count is <200 cells/mm 3 or 
viral load is >10,000 cells/mL. 



[ Q: 821 ] MRCPass - Infectious 
disease 

A 45 year old man presents with malaise, 
weight loss and diarrhoea. On examination his 
skin is pigmented, his fingers are clubbed. 
There is cervical and axillary 
lymphadenopathy. Upper gastrointestinal 
endoscopy is performed and distal duodenal 
biopsies demonstrate stunted villi. The lamina 
propria is distended with multiple periodic 
acid-Schiff (PAS) positive macrophages. 

The aetiological organism is: 

1- Mycobacterium tuberculosis 

2- Trophyrema whippelli 

3- Candida albicans 

4- Giardia lamblia 

5- Amoeba 


Current recommendations are for a tw o-w 
eek course of intravenous ceftriaxone, to be 
followed by one to two years of double¬ 
strength oral trimethoprim-sulfamethoxazole. 



Periodic acid-Schiff (PAS) stain reveals 
intensely PAS positive rod shaped and 
granular inclusions in macrophages 



[ Q: 822 ] MRCPass - Infectious 
disease 

A 42 year old patient with hepatitis B infection 
is being considered for therapy in outpatients. 

Which antiretroviral is also used for the 
therapy of hepatitis B virus (HBV) infection? 

1- Lamivudine 

2- Zidovudine 

3- Nevirapine 


Dr. Khalid Yusuf El-Zohry - Sohag Teaching Hospital (01118391123) 

Ref MRCPass OE OE 2012 PasTest 2009 PassMedicine 2009 PasTest Exam 




ReviseMRCP 

355 


























El-zohry MRCP Questions Bank (Port 1) - 2013 


(For my personal use) 


4- Indinavir 

5- Saquinavir 


Answer & Comments 

Answer: 1- Lamivudine 

Lamivudine (3TC) is a nucleoside analogue 
that inhibits viral DNA synthesis. Those who 
benefit the most from treatment are chronic 
hepatitis B infected patients with persistently 
elevated transaminases who are actively 
replicating virus (with viral DNA testing 
positive) and have evidence of chronic 
hepatitis on liver biopsy. Lamivudine may also 
be used to suppress hepatitis B infection in 
patients undergoing liver transplantation or 
others where immunosuppression is required. 
Alpha Interferon is the other drug which is 
currently used to treat hepatitis B. 


A 32 year old man is referred following a 6 
month history of confusion. He is depressed 
and has frequent mood swings. He also has 
recurrent, asymetrical, jerks in all 4 limbs. 

Which of the following investigations is most 
likely to be useful in reaching a diagnosis? 

1- CT head 

2- EMG 

3- Chest X Ray 

4- Inflammatory markers 

5- EEG 



[ Q: 823 ] MRCPass - Infectious 
disease 


Answer & Comments 


Answer: 5- EEG 


sporadic CJD may show significant 
abnormalities involving deep brain areas 
(thalamus) with diffuse, nonspecific changes, 
developing into stereotyped high voltage 
components on a slow backgound. 


A 35 year old man has returned from a field 
trip in Malaysia. He now has a fever of 39°C, 
headache, muscular aches and pains. 
Examination reveals an erythematous rash on 
his abdomen and thighs. There was cervical 
and inguinal lymphadenopathy. 

Blood tests show : 

Hb 14.0 g/dI 

MCV 80 fl 

WCC 7 x 10 9 /L 

platelets 120 x 10 9 /L 

urea 7 pmol/l 

creatinine 100 pmol/l 

sodium 142 mmol/l 

potassium 4.2 mmol/l 

bilirubin 16 pmol/l 

AST 120 U/l 

ALP 1500 U/l 

albumin 32 g/l 

ESR 60 mm/hr 

CRP 180 mg/I 

Which of the following infections is likely? 

1- Syphillis 

2- Chlamydia 

3- Dengue fever 



[ Q: 824 ] MRCPass - Infectious 
disease 


The presentation of rapid cognitive decline in 
a young person with myoclonus is suggestive 
of Creutzfeld Jakob disease. This may be new 
variant (in which the EEG is often normal) or 
sporadic (in which case characteristic EEG 
abnormalities may be expected). The EEG in 


4- Yellow fever 

5- Typhoid fever 


Answer & Comments 


Answer: 3- Dengue fever 


Dr. Khalid Yusuf El-Zohry - Sohag Teaching Hospital (01118391123) 

Ref MRCPass OE OE 2012 PasTest 2009 PassMedicine 2009 PasTest Exam 




ReviseMRCP 

356 

























El-zohry MRCP Questions Bank (Port 1) - 2013 


(For my personal use) 


Dengue fever caused by an arthropod borne 
flavivirus (typically the Aedes mosquito 
insect). It is present in South East Asia, Africa, 
Middle East and India. The disease has an 
incubation period of 7 days. Headaches, retro- 
orbital pain, musculoskeletal pains and a 
maculopapular rash can occur. Treatment is 
conservative with antipyretics and bedrest. 



[ Q: 825 ] MRCPass - Infectious 
disease 


An 18 year old girl was studying for 
examinations together with a friend who was 
hospitalised 2 days ago with meningitis. The 
blood cultures in her friend grew 
meningococcus group A. 



Which of the following actions should be token 
towards the girl who was in contact with the 
patient? 

1- Immunisation with meningococcus A 
vaccine 


2- Immunisation with meningococcus A and C 
vaccine 


3- Immunisation with meningococcus A and C 
vaccine, and rifampicin 

4- Rifampicin only 

5- Full treatment for meningitis A 


Answer & Comments 

Answer: 3- Immunisation with meningococcus 
A and C vaccine, and rifampicin 

Immunisation is available against strains A and 
C of this bacteria, however strain B is the most 
often implicated in meningococcal meningitis. 
Due to close contact, this girl should be given 
both available vaccines and also rifampicin. 
Apart from rifampicin, minocycline and 
ceftriaxone can also be used for prophylaxis. 


7 

n 

A 45 year old pig farmer is admitted to A+E 
following Two generalised seizures. He has no 
significant neurological history. On 
examination, he had no focal neurological 
signs. A CT scan of the head shows 
periventricular cystic lesions. There is 
eosinophilia of 10%. 

Which of the following infective organisms is 
likely? 

1- Toxocara canii 

2- Ascaris lumbricoides 

3- Schistosoma mansoni 

4- Yersinia enterolitica 

5- Taenia solium 


[ Q: 826 ] MRCPass - Infectious 
disease 


Answer & Comments 

Answer: 5- Taenia solium 

Toxocariasis is commonly passed on from dog 
and cat faeces. Ascariasis is roundw orm 
infection which commonly causes abdominal 
symptoms. Yersninia is a bacterial infection 
spread from half cooked meat and 
unpasteurised milk, causing abdominal 
symptoms and diarrhoea. Taenia solium is the 
pork tapeworm which causes the condition 
cysticercosis described above. Cysts are 
commonly found in the brain and seizures are 
common. 





Taenia Solium 


Dr. Khalid Yusuf El-Zohry - Sohag Teaching Hospital (01118391123) 

Ref MRCPass OE OE 2012 PasTest 2009 PassMedicine 2009 PasTest Exam 




ReviseMRCP 

357 























El-zohry MRCP Questions Bank (Port 1) - 2013 


(For my personal use) 


| *1 

A 55 year old man has lived in Saudi Arabia for 
several years but moved to the UK. He 
presents with severe right hypochondrial pain, 
nausea, vomiting, fever, cough and chest pain 
of one day duration. 

Physical examination revealed tachypnoea, 
tachycardia and a temperature of 39 C. Chest 
examination showed dullness, diminished air 
entry and bronchial breathing in the right low 
er zone of the chest posteriorly. There were 
tenderness and guarding in the right upper 
quadrant of the abdomen and the right low er 
intercostal spaces. 

A chest x ray showed consolidation of the 
right low er lobe of the lung with pleural 
effusion. Abdominal ultrasound several cystic 
lesions in the liver. 

What is the likely diagnosis? 

1- Polycystic liver diseaes 

2- Hydatid disease 

3- Tuberculosis 

4- Sarcoidosis 

5- HIV infection 

Answer & Comments 

Answer: 2- Hydatid disease 

Hydatid disease due to Echinococcus 
granulosusis (found in in cattle-and 
sheepraising regions) of the world such as 
Central Europe, the Mediterranean countries, 
the Middle East, South America, and South 
Africa. Hydatid cysts are known commonly to 
affect the liver and lung. 

The treatment of hydatid cysts is surgical. Pre- 
and post-operative 1-month courses of 
Albendazole and 2 weeks of Praziquantel 
should be given. 


[ Q: 827 ] MRCPass - Infectious 
disease 



Hydatid cysts in the liver 



[ Q: 828 ] MRCPass - Infectious 
disease 


A 28 year old man has recently been 
diagnosed as HIV-positive. In view of a high 
viral load and low CD4 count he w as 
commenced on septrin (960 mg alternate 
days), Zidovudine 250 mg b.d., Lamivudine 
150mg b.d. and Abacavir 300mg b.d. 8 weeks 
into this regimen he becomes progressively 
unw ell over 3 days - he develops a pyrexia of 
40°C, an erythematous macular rash affecting 
the limbs and trunk, nausea, vomiting and 
abdominal pains. He has also become 
significantly more breathless this evening. 


What immediate course of action is advisable? 


1- Stop zidovudine 


2- Initiate steroids 


3- Stop septrin 

4- Stop abacavir 

5- Stop lamivudine 


Answer & Comments 

Answer: 4- Stop abacavir 

Abacavir hypersensitivity occurs in 5% of 
individuals. It may present without a rash and 
should be considered in the differential 
diagnosis of any febrile illness after 
commencing abacavir. Abacavir should be 
stopped and an alternative anti-retroviral 
agent commenced to maintain triple 
combination therapy. 


Dr. Khalid Yusuf El-Zohry - Sohag Teaching Hospital (01118391123) 

Ref MRCPass OE OE 2012 PasTest 2009 PassMedicine 2009 PasTest Exam ReviseMRCP 


























El-zohry MRCP Questions Bank (Port 1) - 2013 


(For my personal use) 



[ Q: 829 ] MRCPass - Infectious 
disease 

A 40 year old man has spent a year in South 
America working in the computer industry. He 
develops fevers, night sw eats, vomiting and 
pain in the right upper quadrant. Blood tests 
reveal a raised white cell count but not 
eosinophil count. An CT of his abdomen shows 
a large cyst. 

Which is the best treatment? 

1- Hepatectomy of hepatoma 

2- Surgical removal of aspergilloma 

3- Albendazole for hydatid cyst 

4- Metronidazole for amoebic liver abscess 

5- Quadruple therapy for TB 


Answer & Comments 

Answer: 4- Metronidazole for amoebic liver 
abscess 

Amoebiasis is caused by Entamoeba 
histolytica is spread by faeco oral route. It can 
present months or a year after infection. RUQ 
and referred pain to the shoulders as well as 
with systemic symptoms are common 
presentations. 

[ Q: 830 ] MRCPass - Infectious 
disease 

A 78 year old woman who is resident in a 
nursing home has been treated by her GP for 
pneumonia 2 weeks ago, and now presents 
with diarrhea and abdominal pains. The 
diarrhoea is occuring 8 times a day and has a 
greenish colour. 

What is the best treatment for her? 

1- Metronidazole 

2- Ciprofloxacin 

3- Gentamicin 

4- Erythromycin 

5- Tazocin 



Answer & Comments 

Answer: 1- Metronidazole 

The diagnosis is likely to be 
pseudomembranous colitis. C. difficile toxin 
should be sent off. As the index of suspicion is 
high, she should be given metronidazole. 


[ Q: 831 ] MRCPass - Infectious 
disease 

A 40 year old woman in otherw ise good 
health, was cleaning debris on her land and 
was exposed to animal feces. The patient was 
removing rocks when she accidentally grasped 
a piece of barbed wire concealed by the 
murky water. This led to four cuts to the 
palmer surface of four fingers on her right 
hand. She cleaned the injury w ith an 
antibacterial soap and immediately continued 
working. 

Seven days later, she presented to the 
emergency room (ER). left side of her jaw 
became painful and badly sw ollen. She had 
inability to open her mouth, and difficulty 
breathing. 

What is the most likely infective organism? 

1- Clostridium tetani 

2- Rabies virus 

3- Ross River virus 

4- Staphylococcus aureus 

5- Streptococcus pyogenes 



Answer & Comments 

Answer: 1- Clostridium tetani 

Tetanus, sometimes known as lockjaw is a 
disease manifested by uncontrolled spasms, 
due to the introduction of Clostridium tetani 
toxin into tissues. Skin punctures, 
contaminated wounds with soil, dust, burns 
have a role in the development of the disease. 

The spores produce a neurotoxin 
(tetanospasmin) which causes severe spasm 


Dr. Khalid Yusuf El-Zohry - Sohag Teaching Hospital (01118391123) 

Ref MRCPass OE OE 2012 PasTest 2009 PassMedicine 2009 PasTest Exam ReviseMRCP 






























El-zohry MRCP Questions Bank (Port 1) - 2013 


(For my personal use) 


all over the body leading to painful muscle 
contraction and laryngeal spasm which 
interfere with breathing and muscle tears. 


syndrome. Clindamycin has effects of reducing 
protein synthesis and exotoxin production by 
the bacteria. 


The incubation period is typically between 1-2 
weeks. Vaccination with tetanus toxoid has 
been proved to be effective. Booster 
immunization to whose who have been 
injured is advisable, especially for those whose 
last immunization received was 10 years or 
more. The conventional treatment of severe 
tetanus which is supportive along with 
penicillin, is still the most effective treatment. 

Risus sardonicus is an abnormal, sustained 
spasm of the facial muscles that is most often 
observed as a symptom of tetanus. Trismus is 
a pathological, sustained spasm of the neck 
and masseter (jaw) muscles that can make it 
difficult or impossible to open the mouth, also 
most often associated with tetanus. 


A 40 year old diabetic lady presents with a hot 
swollen left leg. On examination has a 
temperature of 39°C and her leg is tender to 
compression. She was treated with 
intravenous flucloxacillin and benzylpenicillin. 
However, the erythema has spread even 
further after 3 days, she is persistently 
hypotensive with a systolic BP of < 90 mmHg. 

Which of the following antibiotics should be 
added? 

1- Gentamicin 

2- Tazocin 

3- Chloramphenicol 

4- Clindamycin 

5- Amoxycillin 



[ Q: 832 ] MRCPass - Infectious 
disease 


Answer & Comments 


Answer: 4- Clindamycin 


[ Q: 833 ] MRCPass - Infectious 
disease 

A 35 year old man has travelled to South East 
Asia 2 weeks ago. He presents to the hospital 
with fevers, diffuse rash and lethargy. 

The rash was a widespread maculopapular 
rash affecting the palms of his hands and soles 
of feet. There was also a rash on the face, 
mouth ulcers and exudative pharyngitis. 

He also had several raw, red, mouth ulcers. 

What is the most likely infection? 

1- Measles 

2- Lyme disease 

3- HIV 

4- Syphilis 

5- Tuberculosis 

Answer & Comments 

Answer: 3- HIV 

Both seroconversion of HIV and secondary 
syphilis are possibilities. How ever, 
seroconversion can occur in 2 - 4 weeks of 
infection, but secondary syphilis usually occurs 
2-4 months after. Primary syphilis w ould 
present within 2 weeks with a chancre. 




Rash in seroconversion illness 



There is suspicion that this lady may have 
Streptococcus A infection with toxic shock 



[ Q: 834 ] MRCPass - Infectious 



Dr. Khalid Yusuf El-Zohry - Sohag Teaching Hospital (01118391123) 

Ref MRCPass OE OE 2012 PasTest 2009 PassMedicine 2009 PasTest Exam ReviseMRCP 




























El-zohry MRCP Questions Bank (Port 1) - 2013 


(For my personal use) 


disease 

A 45 year old man presented with a cough and 
night sw eats. He had CXR changes showing 
upper zone fibrosis. 

Chemotherapy was commenced (rifampicin, 
isoniazid and pyrazinamide ). Two weeks later 
he develops stridor. 

Repeat CXR shows enlarged hilar lymph nodes 
compressing on the bronchi. 

Which is the best management strategy? 

1- Add ethambutol 

2- Mediastinoscopy 

3- Surgical decompression 

4- Dapsone 

5- Prednisolone 

Answer & Comments 

Answer: 5- Prednisolone 

The patient has mediastinal lymphadenitis 
with evidence of bronchial compression. 


What is the most likely diagnosis? 

1- Smallpox 

2- Chickenpox 

3- Influenzae 

4- Icthyosis 

5- Shingles 

Answer & Comments 

Answer: 1- Smallpox 

Chickenpox, which is the more common 
disease, causes a rash which is typically 
central, and does not affect the hands and 
feet. 

The smallpox rash is more distal and causes 
deep lesions which leave scars. The criteria for 
smallpox include: 

(1) a centrifugal distribution of lesions, with 
the first lesions on the oral mucosa or palate, 
face, or forearms 

(2) a toxic or moribund appearance 


Steroids are highly effective in reducing 
lymphadenopathy in this situation and should 
be the first option. 


(3) the slow evolution of lesions of 1-2 days 
per stage 

(4) lesions that appear on the palms and soles 




[ Q: 835 ] MRCPass - Infectious 
disease 


A 35 year old man is unw ell and comes to the 
hospital. He complains of a headache and 
backache that started suddenly the previous 
afternoon with fever and chills. 


His temperature is 39 C, heart rate is 100/min, 
respiratory rate is 20/min, and blood pressure 
is 110/70 mm Hg. 

On examination, he is unw ell. There are 
several small erythematous macular lesions on 
the oral mucosa. There are also small vesicles 
with the majority on his face, forearms, palms, 
and legs consistent with a centrifugal rash. 

They appear to be deeply embedded and firm. 
A few lesions are on his trunk. 


Smallpox is caused by the variola virus (genus 
Orthopoxvirus). Patients exposed to the most 
common form of smallpox, variola major, will 
have a symptom-free incubation period of 7- 
17 days, with an average of 12 days. 



Smallpox rash 


Dr. Khalid Yusuf El-Zohry - Sohag Teaching Hospital (01118391123) 

Ref MRCPass OE OE 2012 PasTest 2009 PassMedicine 2009 PasTest Exam 




ReviseMRCP 

361 






















El-zohry MRCP Questions Bank (Port 1) - 2013 


(For my personal use) 



[ Q: 836 ] MRCPass - Infectious 
disease 

A 55 year old woman has recently travelled to 
Spain and developed watery diarrhoea. 

Which of the following is the commonest 
world wide couse of traveller's diarrhoea? 

1- E coli 

2- Giardia 

3- Shigella 

4- Salmonella 

5- Campylobacter 


Listeria can cause disease in the 
immunosuppressed (including pregnant 
women). Ampicillin is the drug of choice to 
cover listeria in addition to ceftriaxone (which 
covers strep pneumoniae and meningococci). 

[ Q: 838 ] MRCPass - Infectious 
disease 

A 60 year old woman who had a long history 
of alcohol abuse presents with diarrhoea and 
back pain. She had recently had an iv cannula 
for a drip following a drunken episode. On 
examination she had a fever of 39°C. 



Answer & Comments 

Answer: 1- E coli 

Traveller's diarrhoea is an extremely common 
occurrence, affecting up to half of travellers to 
high risk areas such as Africa, Asia and South 
America. The commonest infective cause 
world-w ide is Escherichia coli. Other bacterial 
causes include Shigella, Salmonella and 
Campylobacter, all of which can cause 
dysentery (diarrhoea with blood). 



[ Q: 837 ] MRCPass - Infectious 
disease 


A lady who is 25 years of age, presents with a 
headache, neck stiffness and photobophia. 
She was been treated with ceftriaxone, and 
also with ampicillin added. 


What organism is ampicillin intended to cover 
in this case? 


1- Klebsiella 


2- Meningococcus group B 

3- Meningococcus group C 

4- Listeria 


5- Enterococci 


Answer & Comments 


Answer: 4- Listeria 


What is the likely diagnosis? 

1- Endocarditis 

2- Pancreatitis 

3- Staphylococcal discitis 

4- Diverticulitis 

5- Subacute bacterial peritonitis 


Answer & Comments 

Answer: 3- Staphylococcal discitis 

Staphylococci are skin organisms which can be 
introduced insertion of lines (cannulas or 
central lines). This may lead to discitis which 
can cause back pain or endocarditis (which 
would present differently). 


A 45 year old man presents 2 weeks after 
returning from a holiday in Tanzania. He has a 
serpiginous rash on the finger and a low - 
grade fever. 

Which of the following is the most likely 
diagnosis? 

1- Group-A Streptococcal infection 

2- Cutaneous myiasis 

3- Cutaneous larval migrans 

4- Rickettsial chancre 



[ Q: 839 ] MRCPass - Infectious 
disease 


Dr. Khalid Yusuf El-Zohry - Sohag Teaching Hospital (01118391123) 

Ref MRCPass OE OE 2012 PasTest 2009 PassMedicine 2009 PasTest Exam ReviseMRCP 



362 































El-zohry MRCP Questions Bank (Port 1) - 2013 


(For my personal use) 


5- Trypanosomal chancres 

Answer & Comments 

Answer: 3- Cutaneous larval migrans 

Cutaneous larva migrans, caused by various 
Ankylostoma (hookw orm) species is 
characterised by a slow ly lengthening, 
serpiginous, intensely itchy rash. 



Cutaneous Larval Migrans 


Dr. Kholid Yusuf El-Zohry - Sohog Teaching Hospital (01118391123) 

Ref MRCPoss OE OE 2012 PosTest 2009 PassMedicine 2009 PasTest Exom ReviseMRCP 



363 



















El-zohry MRCP Questions Bank (Part 1) - 2013 


(For my personal use) 


[ Q: 840 ] MRCPass - Haematology 

Si - 

# An 8 year old boy with sickle cell 
disease presents with breathlessness. His Hb is 
4.5 g/dl, WCC is 3 x 10 9 /L and platelet count is 
35 x 10 9 /l_. 

Which organism is likely to be responsible? 

1- Coronavirus 

2- HIV 

3- HSV 


Answer & Comments 

Answer: 3- Chronic myeloid leukaemia 

A high neutrophil count, platelet count points 
towards myeloid leukaemia. 

Acute leukaemia is defined as blast cells 
comprising 30% (in this case only 10% of 
myelocytes) of the cell type. 

Hence it makes CML more likely than AML. 


4- Parvovirus 

5- Epstein barr virus 


Answer & Comments 

Answer: 4- Parvovirus 

Parvovirus B19 is the commonest cause of 
aplastic crisis in sickle cell anaemia. Recovery 
should occur within 10 days with conservative 
treatment. 


[ Q: 841 ] MRCPass - Haematology 

/ - 

# A 45 year old man is being 
investigated for easy bruising and malaise. 
Investigations reveal: 

Haemoglobin 9.5 g/dL 

White cell count 90 x 10 9 /L 

Neutrophils 45 x 10 9 /L (1.5-7) 

Lymphocytes 3.5 x 10 9 /L (1.5-4) 

Myelocytes 30 x 10 9 /L 

Myeloblasts 3 x 10 9 /L 

Platelet count 750 x 10 9 /L 

Which of the following diagnosis is likely? 

1- Acute myeloid leukaemia 

2- Acute lymphocytic leukaemia 

3- Chronic myeloid leukaemia 

4- Chronic lymphocytic leukaemia 

5- Polycythaemia rubra vera 


[ Q: 842 ] MRCPass - Haematology 

A 30 year old lady attends A&E with 
severe nosebleeds. Her investigations show : 



Hb 10.5 g/dl 
MCV 80 fl 
WCC 7 x 10 9 /L 
platelets 3 x 10 9 /L 

Blood film report: No platelet clumps seen. 
Normal rbc and w be 

Clotting screen normal 

What is the most likely diagnosis? 

1- Thrombotic thrombocytopenic purpura 

2- Haemolytic uraemic syndrome 

3- Acute lymphoblastic leukaemia 

4- Disseminated intravascular coagulation 

5- Immune thrombocytopenia 


Answer & Comments 

Answer: 5- Immune thrombocytopenia 

As there is no abnormality in the red and 
white blood cells on the blood film, this is 
most likely to be immune thrombocytopenia. 
Features consistent with a diagnosis of 
immune thrombocytopenic purpura (ITP) are 
thrombocytopenia with platelets being normal 
in size or may appear larger than normal, but 
uniformly giant platelets (approaching the size 
of red cells) should be absent. The 
morphology of red blood cells and white blood 
cells should be normal. 


Dr. Khalid Yusuf El-Zohry - Sohag Teaching Hospital (01118391123) 

Ref MRCPass OE OE 2012 PasTest 2009 PassMedicine 2009 PasTest Exam ReviseMRCP 



365 





























El-zohry MRCP Questions Bank (Port 1) - 2013 


(For my personal use) 


[ Q: 843 ] MRCPass - Haematology 

A 72 year old lady has recently been 
found to be anaemic. Further blood tests 
show : 



Hb of 9.2 g/dl 

WCC of 8.0 x 10 9 /L 

platelet count of 200 x 10 9 /L 

MCV is 104 fl (80-96) 

Ferritin is 120 pg/l (15-200) 

red cell folate is 350 ?g/l (150-650) 

B12 is 400 pmol/l (120-700) 

Blood film shows anisocytosis and 
poikilocytosis 

Which of the following diagnosis is likely? 

1- Chronic lymphocytic leukaemia 

2- Autoimmune haemolytic anaemia 

3- Sideroblastic anaemia 

4- Iron deficiency 

5- Lymphoma 


Answer & Comments 


Answer: 3- Sideroblastic anaemia 


3- Serum IgE level 

4- Venom toxin level 

5- Complement C3 level 


Answer & Comments 

Answer: 3- Serum IgE level 

Type I hypersensitivity is occurring in this case 
of anaphylaxis. It takes 30 minutes from time 
of exposure antigen. The reaction involves 
production of IgE which is released from mast 
cells. 


[ Q: 845 ] MRCPass - Haematology 

A 25 year old woman presents with 
diffuse lymphadenopathy, fever and malaise. 
Her blood film shows atypical lymphocytes 
and red cell agglutination. 

What is the most likely diagnosis? 

1- Legionella 

2- Infectious mononucleosis 

3- Meningococcal meningitis 

4- Non-Hodgkin's lymphoma 

5- Autoimmune haemolytic anaemia 



A high MCV with normal folate and B12 levels, 
normal iron and a blood film showing 
anisocytosis and poikilocytosis suggests 
sideroblastic anaemia. 


^ [ Q: 844 ] MRCPass - Haematology 

/ - 

# A 20 year old man presents with 

acute severe dyspnoea. He had been stung by 
a wasp several hours ago. On examination, he 
was hypotensive and had signs of 
bronchospasm. 

Which one of the following investigations 
would confirm the type of hypersensitivity 
reaction? 

1- Plasma tryptase level 

2- ESR 


Answer & Comments 

Answer: 2- Infectious mononucleosis 

Infectious mononucleosis is caused by Epstein 
Barr virus. It is one of the common causes of 
atypical lymphocytes, along with 
cytomegalovirus, HIV and Toxoplasma. The 
features of lymphadenopathy and atypical 
lymphocytes suggest infectious 

mononucleosis. 



[ Q: 846 ] MRCPass - Haematology 

A 62 year old man who was 
asymptomatic, was referred for investigation 
of a high white cell count routinely found by 
the GP. On examination, he had palpable 
splenomegaly and looked pale. 


Dr. Khalid Yusuf El-Zohry - Sohag Teaching Hospital (01118391123) 

Ref MRCPass OE OE 2012 PasTest 2009 PassMedicine 2009 PasTest Exam ReviseMRCP 



366 






























El-zohry MRCP Questions Bank (Port 1) - 2013 


(For my personal use) 


Results reveal: 

Haemoglobin 10.5 g/dl (11.5-16.5) 

Platelet count 175 x 10 9 /L (150-400) 

White cell count 32 x 10 9 /I (4-11) 

Neutrophil count 4 x 10 9 /L (1.5-7) 

Lymphocyte count 27 x 10 9 /L (1.5-4) 

His Blood film shows many mature 
lymphocytes 

What is the best initial management? 

1- Prednisolone 

2- Period of observation 

3- Radiotherapy 

4- Splenectomy 

5- Chlorambucil 


Answer & Comments 

Answer: 2- Period of observation 

In chronic lymphocytic leukaemia, Indications 
for therapy include fatigue, lymphadenopathy, 
anaemia or thrombocytopenia. 

All of the others are treatment options which 
can be used. 


[ Q: 847 ] MRCPass - Haematology 

A 20 year old man presented with a 
cough and fevers. He was diagnosed as having 
a chest infection and was prescribed two 
different antibiotics. He felt worse two days 
later and mentioned that he had dark urine. 

Investigations showed: 

Hb 8.5 g/dl 

MCV 75 fl 

WCC 12 x 10 9 /L 

platelets 155 x 10 9 /L 



2- Autoimmune haemolytic anemia 

3- Aplastic anemia 

4- Immune thrombocytopenic purpura 

5- Acute myeloid leukaemia 


Answer & Comments 

Answer: 1- G6PD deficiency 

There is evidence of haemolysis (bite cells are 
schistocytes), in this case most likely due to 
G6PD deficiency. Drugs normally causing 
haemolysis in G6PD deficiency are sulphur 
containing-dapsone, anti-malarials, 

bactrim/septrim, sulphonamides, 

primaquinine. 




[ Q: 848 ] MRCPass - Haematology 

A 65 year old man has recently been 
diagnosed as having a deep vein thrombosis. 
He also has symptoms of headaches and 
lethargy. On examination he was flushed. 
Investigations reveal: 


haemoglobin 19.5 g/dL 


haematocrit 0.6 (0.4-0.52) 

white cell count 10.5 x 10 9 /L (4-11) 

platelet count 450 x 10 9 /L (150-400) 


Which one of following is the most 
appropriate investigation? 


1- Serum EPO level 


2- Bone marrowaspirate 

3- Neutrophil alkaline phosphatase 

4- Red cell mass 

5- Serum Vitamin Bn levels 


Answer & Comments 


Answer: 4- Red cell mass 


Blood film showed: anisopokilocytosis and 
bite cells 

What is the diagnosis? 

1- G6PD deficiency 


The most appropriate initial investigation will 
be red cell mass studies which would will 
distinguish between true relative 
polycythaemia from secondary polycythaemia. 


Dr. Khalid Yusuf El-Zohry - Sohag Teaching Hospital (01118391123) 

Ref MRCPass OE OE 2012 PasTest 2009 PassMedicine 2009 PasTest Exam ReviseMRCP 





























El-zohry MRCP Questions Bank (Port 1) - 2013 


(For my personal use) 


In Polycythaemia Rubra Vera, the serum EPO 
is low (an elevated EPO level suggests 
secondary polycythaemia). 

Haematocrit is high as is the Hb concentration. 
Thrombocytosis and leukocytosis can occur. 
The NAP score and B12 levels are frequently 
increased. 


[ Q: 849 ] MRCPass - Haematology 

A 55 year old woman who had a 
cerebrovascular accident ten month ago, was 
referred for investigation of recurrent 
episodes of proximal deep venous thrombosis 
(DVT) of lower limbs in the last seven months. 

Investigations show : 

hemoglobin 7.9 g/dl 

hematocrit 25% 

mean corpuscular volume 99 fl 

mean corpuscular hemoglobin 32 pg 

white blood cells 4 x 10 9 .l 

platelets 93 x 10 9 /L 

reticulocytes 5.4% 

lactate dehydrogenase 944 Ul/I 

total bilirubin 50umol/l. 

A bone marrow biopsy showed a slight 
hyperplasia of erythrocytic bone marrow cell 
line. 

Urine Dipstick - blood +++ 

What is the likely diagnosis? 

1- Haemolytic uraemic syndrome 

2- Antithrombin III deficiency 

3- Paroxysmal nocturnal haemoglobinuria 

4- Protein C deficiency 

5- Protein S deficiency 



Answer & Comments 

Answer: 3- Paroxysmal nocturnal 

haemoglobinuria 


Paroxysmal nocturnal haemoglobinuria (PNH) 
is an aplastic anaemia like syndrome which 
red cells are predisposed to complement lysis 
and resultant haemolytic anaemia. There is a 
pancytopenia as well as a tendency towards 
Budd Chiari thrombosis. 

The diagnostic test is the HAM test. Serum 
(which contains complements) is acidified 
(activates the complement pathway) and 
mixed with red cells which undergo lysis. 
Haemosiderin is a by product of haem 
breakdown containing iron. Excess amounts 
leads to renal damage, and is also lost in the 
urine. 

In PNH, there is a loss of anchor protein (GPI 
glycosylphosphatidyl inositol) which hold 
different antigens e.g. CD59, CD14. These are 
regulatory proteins for the complement 
pathway. 



[ Q: 850 ] MRCPass - Haematology 

A 17 year old patient has sickle cell 
disease. He presents unwell with abdominal 
pain. He also has right sided facial weakness. 
Investigations: 


Hb 7.5 g/dl 
platelets 140 x 10 9 /L 
Urea 8 mmol/l 


creatinine 100 pmol/l 
sodium 141 mmol/l 


potassium 3.8 mmol/l 
bilirubin 45 pmol/l 


AST 35 U/l 
ALP 105 U/l 
Alb 42 g/l 
LDH 1250 U/l 


Blood film shows sickle cells 

Which of the following is the most important 
management? 

1- Iv fluids 


Dr. Khalid Yusuf El-Zohry - Sohag Teaching Hospital (01118391123) 

Ref MRCPass OE OE 2012 PasTest 2009 PassMedicine 2009 PasTest Exam ReviseMRCP 























El-zohry MRCP Questions Bank (Port 1) - 2013 


(For my personal use) 


2- Iv antibiotics 

3- Diamorphine injections 

4- Blood transfusion 

5- Exchange transfusion 


Answer & Comments 

Answer: 5- Exchange transfusion 

When there is neurological damage or visceral 
sequestration crisis in sickle cell crisis, 
exchange transfusion is indicated. Exchange 
transfusion involves drawing out the patient's 
blood while exchanging it for donor red blood 
cells. It can be done manually or automatically 
with erythrocytapheresis. It prevents stroke 
and also may be used in patients with severe 
acute chest syndrome and to reduce the risk 
of iron overload in patients who require 
chronic transfusion therapy. Studies suggest 
that it may improve oxygenation and reduce 
hemoglobin S levels. 

[ Q: 851 ] MRCPass - Haematology 

A 35 year old man presents with 
pallor and breathlessness. Blood tests show 
anaemia with a Hb of 7.5 g/dl. A blood film 
shows Heinz bodies. 

Which one of the following diagnoses is most 
likely? 

1- Autoimmune haemolytic anaemia 

2- Sideroblastic anaemia 

3- G6PD deficiency 

4- Post splenectomy 

5- Sickle cell disease 



Answer & Comments 


Answer: 3- G6PD deficiency 


[ Q: 852 ] MRCPass - Haematology 

Which one of these patient's results 
is most likely to have a diagnosis of 
chronic lymphatic leukaemia? 



1- A white cell count of 35 x 10 9 /l_ and 
immature lymphocytes with prominent 
nucleoli in the peripheral blood 


2- A white cell count of 15 x 10 9 /L and mature 
lymphocytes with cleaved nuclei in the 
peripheral blood film 


3- A white cell count of 65 x 10 9 /l_ with 
neutophils, myelocytes and promyelocytes 
on the blood film 


4- A white cell count of 25 x 10 9 /l_ and smear 
cells on the peripheral blood film 

5- A white cell count of 6 x 10 9 /l_, and mature 
lymphocytes with polar villi on the blood 
film 


Answer & Comments 

Answer: 4- A white cell count of 25 x 10 A 9/I 
and smear cells on the peripheral blood film 

Chronic lymphatic leukaemia is characterised 
by a lymphocytosis. The blood film shows 
mature lymphocytes with smear or smudge 
cells (they are squashed cells). 



[ Q: 853 ] MRCPass - Haematology 

A 65 year old woman has a diagnosis 
of chronic lymphocytic leukaemia (CLL). 
During one follow up appointment she 
mentions that she has got progressively more 
lethargic. 


Her investigations show : 

Hb 7.5 g/dl, MCV 118 fl, platelets 180 x 10 9 /L, 
lymphocytes 43 x 10 9 /L, reticulocyte count 
10 %. 


Heinz bodies are precipitated, denatured Hb 
within red cells. They are present in G6PD 
deficiency. (Fava beans cause haemolysis in 
G6PD - 'Beans means Heinz' mnemonic). 


Which test is most likely to give the correct 
diagnosis? 

1- Folate level 

2- Marrow trephine 


Dr. Khalid Yusuf El-Zohry - Sohag Teaching Hospital (01118391123) 

Ref MRCPass OE OE 2012 PasTest 2009 PassMedicine 2009 PasTest Exam ReviseMRCP 



369 






























El-zohry MRCP Questions Bank (Port 1) - 2013 


(For my personal use) 


3- Serum electrophoresis 

4- Ferritin 

5- Coomb's test 


Answer & Comments 

Answer: 5- Coomb's test 

A raised reticulocyte count could have led to 
the high MCV. The clinical picture is one of 
haemolysis which is occasionally seen in CLL. 
The Coomb's test will help to confirm this. 


[ Q: 854 ] MRCPass - Haematology 

A 28 year old man with glucose-6- 
phosphate dehydrogenase deficiency presents 
with fatigue and jaundice. The features 
developed following a pneumonia a week ago. 

Which of the following is likely to be found? 

1- Low mean cell volume 

2- Positive direct antiglobulin test 



[ Q: 855 ] MRCPass - Haematology 

A 60 year old Afro-Carribean man is referred 
with abdominal discomfort. On examination, 
he has massive splenomegaly. The FBC shows: 

Hb 8.2 g/dl 

WBC 15 x 10 9 /L 

Platelets 110 x 10 9 /L 

Blood smear: erythroblastic picture 

Which of the following diagnoses is the most 
likely? 

1- Myelofibrosis 

2- Polycythaemia rubra vera 

3- Non-Hodgkin's lymphoma (NHL) 

4- Aplastic anaemia 

5- Chronic myeloid leukaemia 


Answer & Comments 


Answer: 1- Myelofibrosis 


3- Spherocytes present on blood film 

4- Haemoglobinuria 

5- Reduced reticulocyte count 


Answer & Comments 

Answer: 4- Haemoglobinuria 

The clinical scenario describes haemolytic 
anaemia. Haemoglobinuria is seen in 
haemolytic anaemia. Patient may present with 
fatigue and tiredness. Low mean cell volume 
would mean lack of reticulocytosis. This is 
unlikely, there is usually increased reticulocyte 
count in all haemolytic anaemias including 
G6PD deficiency. Spherocytes are seen in 
hereditary spherocytosis and the antiglobin 
test is positive in autoimmune haemolytic 
anaemia (not just G6PD deficiency). 



Myelofibrosis existis likely to be the case 
described above. Median age at diagnosis is 
about 60 years, and median life expectancy 
from onset of symptoms is 10 years. In 
contrast, acute MF in adulthood is a rapidly 
fatal disorder in which splenomegaly is not 
usually observed; bone marrow examination 
typically reveals numerous bizarre 
megakaryocytes and blasts. 


[ Q: 856 ] MRCPass - Haematology 

A 32 year old woman presents to the 
casualty with worsening dyspnoea over 3 
weeks. She has no history of jaundice of 
anaemia. 

On examination, she had a blood pressure of 
125/65 mmHg. Her conjunctivae were pale. 
Abdominal examination was unremarkable 
and there was no splenomegaly. 
Investigations show : 

Hb 6.5 g/dl 

WBC 13.5 x 10 9 /L 



Dr. Kholid Yusuf El-Zohry - Sohog Teaching Hospital (01118391123) 

Ref MRCPass OE OE 2012 PasTest 2009 PassMedicine 2009 PasTest Exam ReviseMRCP 



370 






























El-zohry MRCP Questions Bank (Port 1) - 2013 


(For my personal use) 


Pits 255 x 10 9 /L 
MCV 105 fl 
LDH 680 lU/dl 

Direct Coomb's test positive 

Film: Spherocytes ++, reticulocytes ++ 

What should be the treatment for her 
condition? 

1- Iron replacement 

2- Bone marrow examination 

3- Vitamin K 

4- Immunosupressants 

5- B12 and folate 


Answer & Comments 

Answer: 4- Immunosupressants 

This woman is most likely to have 
autoimmune haemolytic anaemia (anaemia, 
high LDH, spherocytes on the blood film), and 
positive Direct Coomb's test. Steroids, 
intravenous immunoglobulin may be used as 
first line treatment, and blood transfusion 
may be necessary. Autoimmune haemolytic 
anaemia can be due to immune disorders 
(SLE), toxic chemicals and drugs, (methyldopa, 
penicillin), antiviral agents (eg, ribavirin), 
physical damage, and infections (infectious 
mononucleosis). 


^ [ Q: 857 ] MRCPass - Haematology 

0 A 35 year old man presents with 
fevers and lymphadenopathy. A bone marrow 
biopsy was done and confirms Hodgkin's 
lymphoma. 

Which one of the following form has the best 
prognosis? 

1- Nodular sclerosing 

2- Lymphocyte predominant 

3- Lymphocyte depleted 

4- Mixed cellularity 

5- Promyelocytic 


Answer & Comments 

Answer: 2- Lymphocyte predominant 

Hodgkin's lymphoma is rare in children. 
Nodular sclerosing is the commonest and 
lymphocyte depleted is the rarest form. The 
lymphocyte predominant form has the best 
prognosis, whilst the lymphocyte depleted 
form has the worst. 


[ Q: 858 ] MRCPass - Haematology 

A 30 year old man presents with 
malaise and is found to be anaemic clinically. 
His blood tests reveal : 

Hb of 10.5 g/dl 

WCC 8 x 10 9 /L 

platelet count 180 x 10 9 /dl 

reticulcyte count 160 x 10 9 /L (50-100) 

Bilirubin is 80 mmol/l 

AST 30 U/l 

ALP 110 U/l 

LDH us 380 U/l (10-250) 

Blood film shows spherocytosis 

Which of the following tests is most 
appropriate? 

1- Direct antiglobulin test 

2- G6PD activity 

3- Hb electrophoresis 

4- Urinary haemosiderin 

5- Methaemoglobin levels 



Answer & Comments 

Answer: 1- Direct antiglobulin test 

The blood tests with high bilirubin, 
reticulocyte count and high LDH suggests 
haemolysis. Spherocytes on blood film 
suggests hereditary spherocytosis (HS). In HS 
the red cells are smaller, rounder, and more 
fragile than normal. The condition is 
commoner among Northern Europeans. The 


Dr. Khalid Yusuf El-Zohry - Sohag Teaching Hospital (01118391123) 

Ref MRCPass OE OE 2012 PasTest 2009 PassMedicine 2009 PasTest Exam 




ReviseMRCP 

371 

























El-zohry MRCP Questions Bank (Port 1) - 2013 


(For my personal use) 


direct antiglobulin test will help to confirm 
this. 

[ Q: 859 ] MRCPass - Haematology 

A 40 year old man has presented 
with seizures and has a confirmed cerebral 
infarct on head scan. He is commenced on 
phenytoin. 3 weeks later he presents with 
lethargy. 

His bloods show Hb 8.0 g/dl, MCV 95 fl, WCC 
3.2 x 10 9 /l_, platelets 65 x 10 9 /l_, Reticulocyte 
count 1%. Ham's test was negative. Bone 
marrowaspirate and trephine biopsy showed 
marked hypocellularity of the marrow with 
some lymphoid aggregates. 

Whot is the likely diagnosis? 

1- Folate deficiency 

2- Myelofibrosis 

3- Aplastic anaemia 

4- Multiple myeloma 

5- Bony metastasis 



Answer & Comments 

Answer: 3- Aplastic anaemia 

The diagnosis is likely to be phenytoin related 
aplastic anaemia. MCV is normal and there is a 
low reticulocyte count as well as hypocellular 
bone marrow . Side effects of phenytoin are 
cerebellar syndrome, phenytoin 

encephalopathy, psychosis, locomotor 
dysfunction, hyperkinesia, megaloblastic 
anemia, decreased serum folate level, 
decreased bone mineral content, liver disease, 
IgA deficiency and gingival hyperplasia. 


Haemoglobin 6.6 g/dL 
MCV 108 fL 
MCH 32.0 pg 

White cell count 3.0 x 10 9 /L 
Platelets 75xl0 9 /L 
Serum Vitamin Bi 2 normal 
Folate 2 (3-20) ?g/l 
Whot should be done? 

1- Treat congestive cardiac failure then 
transfuse 

2- Immediate blood transfusion 

3- Serum electrophoresis 

4- Iron replacement 

5- B12 and folate replacement 


Answer & Comments 

Answer: 5- B12 and folate replacement 

Blood transfusion may worsen cardiac failure 
in this case. In patients who are folate 
deficient erythropoiesis rapidly resolves when 
supplements are given, and transfusion is 
rarely needed in the elderly (unless the 
anaemia is very severe). 


[ Q: 861 ] MRCPass - Haematology 

A 35 year old man has recently been 
diagnosed with Hodgkin's lymphoma. 

In reviewing his symptoms, which one of the 
following indicates the poorest prognosis in 
Hodgkin's lymphoma? 

1- Mediastinal, inguinal lymphadenopathy and 
fever 



^ [ Q: 860 ] MRCPass - Haematology 

/ - 

# A 70 year old woman was admitted 
to hospital with severe breathlessness. On 
examination her blood pressure was 100/55 
mmHg and she had a raised JVP by 4 cm. 
Chest x ray showed mild pulmonary oedema. 

Investigations revealed: 


2- Mediastinal lymphadenopathy and night 
sweats 

3- Abdominal and inguinal lymphadenopathy, 
and night sweats 

4- Cervical and mediastinal lymphadenopathy 

5- Mediastinal and inguinal lymphadenopathy. 


Dr. Khalid Yusuf El-Zohry - Sohag Teaching Hospital (01118391123) 

Ref MRCPass OE OE 2012 PasTest 2009 PassMedicine 2009 PasTest Exam ReviseMRCP 



372 


























El-zohry MRCP Questions Bank (Port 1) - 2013 


(For my personal use) 


Answer & Comments 

Answer: 1- Mediastinal, inguinal 

lymphadenopathy and fever 

Stage III disease occurs when lymph nodes are 
present across both sides of diaphragm, hence 
worse prognosis than when lymph nodes are 
localised to the same side of the diaphragm. 
Presence of B symptoms - night sweats and 
fevers also worsen prognosis. 


[ Q: 862 ] MRCPass - Haematology 

A 25 year old man was admitted with 
a 2 month history of rash, fatigue, intermittent 
hemoptysis, and purpura, culminating in a 
seizure. On examination, widespread 
petechiae and purpura with scleral icterus 
were noted. 

There was no lymphadenopathy or 
splenomegaly. 



Answer & Comments 

Answer: 3- Plasma exchange 

Thrombotic thrombocytopenic purpura (TTP) 
is characterised by microangiopathic 
haemolysis and thrombocytopenia. There is a 
spectrum of presentations with TTP-HUS. 
Neurological features are present in 60% of 
patients of TTP and renal failure is often 
associated in HUS (haemolytic uraemic 
syndrome). 



[ Q: 863 ] MRCPass - Haematology 
A 8 year old boy presents to his GP 


with lethargy and pallor. 


His investigations show : 
Hb 5.5 g/dl 
WBC 2.7 x 10 9 /L 
Pits 42 x 10 9 /L 


Investigations show : 
platelet count 3 x 10 9 /L 
hemoglobin 5.5 g/dL 
mean corpuscular volume 90 fL 
white cell count 19.6 x 10 9 /L 


Neutrophils 0.9 x 10 9 /L 

What is the next best investigation? 

1- Peripheral blood immunophenotyping 

2- Bone marrow cytogenetics 

3- Haematinics 


urea 16 mmol/L 
creatinine 270 pmol/L 
lactate dehydrogenase 2200 U/L 
total bilirubin 79 pmol/L 
haptoglobin 6 g/L 

Blood film shows anisocytosis, moderate to 
marked polychromasia, and slight to moderate 
poikilocytosis, predominantly schistocytes. 

What is the best treatment option? 

1- Haemodialysis 

2- Azathioprine 

3- Plasma exchange 

4- Bone marrow transplant 

5- Intravenous immunoglobulins 


4- Bone marrow aspirate and trephine 

5- ANA and Rheumatoid factor 

Answer & Comments 

Answer: 4- Bone marrow aspirate and 
trephine 

Pancytopenia may be due to bone marrow 
failure (aplastic anaemia) or to bone marrow 
infiltration (leukaemia, lymphoma or non- 
haemopoietic malignancy). Aplastic anaemia 
may be idiopathic or secondary to drugs, 
paroxysmal nocturnal haemoglobinuria or 
Fanconi's anaemia. In a child of this age, 
leukaemia (ALL, AML) or aplastic anaemia 
would be the most likely causes of 
pancytopenia. 


Dr. Khalid Yusuf El-Zohry - Sohag Teaching Hospital (01118391123) 

Ref MRCPass OE OE 2012 PasTest 2009 PassMedicine 2009 PasTest Exam ReviseMRCP 



373 































El-zohry MRCP Questions Bank (Port 1) - 2013 


(For my personal use) 



[ Q: 864 ] MRCPass - Haematology 

A 38 year old woman presents to the 
haematologist for reviewas she has 


lethargy. 

She is on iron tablets. Her blood results show: 


Hb 9.5 g/dl 
MCV 105 fl 
WCC 7 x 10 9 /L 
platelets 218 x 10 9 /L 

Blood film shows anisopoikilocytosis and 
poikilocytosis 

What should be done next? 

1- Intramuscular iron therapy 

2- Blood transfusion 

3- Erythropoietin 

4- Investigation for folate eficiency 

5- No immediate action 


Answer & Comments 

Answer: 5- No immediate action 

The blood film and poor response to iron 
therapy suggests sideroblastic anaemia. 

Sideroblastic anaemia is managed by 
removing the precipitating factors e.g. alcohol 
or myelodysplasia. 


5- Hepatitis C infection 


Answer & Comments 

Answer: 2- Haemolysis due to G6PD deficiency 

G6PD deficiency is common in the 
Mediterranean and African populations. 
Inheritance is X-linked. Intravascular 
haemolysis is usually precipitated by oxidative 
stress, such as infections and drugs. The most 
common drugs implicated are anti-malarials, 
dapsone and sulphonamides. 


[ Q: 866 ] MRCPass - Haematology 

A 20 year old man complains of 
intermittent dark urine and abdominal pains. 
He is found to have a haemoglobin of 9.7 g/dl, 
but the rest of the full blood count is normal. 

What is the most likely diagnosis? 

1- Autoimmune haemolytic anaemia 

2- G6PD deficiency 

3- Paroxysmal nocturnal haemoglobinuria 

4- Paroxysmal cold hemoglobinuria 

5- Hereditary spherocytosis 

Answer & Comments 

Answer: 3- Paroxysmal nocturnal 

haemoglobinuria 



[ Q: 865 ] MRCPass - Haematology 

A 23 year old man presents with 
jaundice during a planned holiday to Africa. 
He has been taking malarial prophylaxis. 

He is apyrexial and apart from lethargy, feels 
well. He reports passing dark urine for the 
past two days. 

What is the likely cause? 

1- Beta thalassemia 

2- Haemolysis due to G6PD deficiency 

3- Sickle cell crisis 

4- Falciparum malaria 



Paroxysmal nocturnal haemoglobinuria is 
caused by a defect in the formation of a red 
cell surface protein anchor, called GP1. As a 
result of the lack of this surface protein 
anchor, the red blood cells are more sensitive 
to complement lysis. Patients have 
intravascular haemolysis, leading to 
haemoglobinuria, and increased risk of 
thrombosis, often occurring in the mesenteric 
vessels and the portal vein. Treatment is 
supportive or with bone marrow 
transplantation. 



[ Q: 867 ] MRCPass - Haematology 


Dr. Khalid Yusuf El-Zohry - Sohag Teaching Hospital (01118391123) 

Ref MRCPass OE OE 2012 PasTest 2009 PassMedicine 2009 PasTest Exam ReviseMRCP 



374 































El-zohry MRCP Questions Bank (Port 1) - 2013 


(For my personal use) 


A 35 year old man has had a 4 day history of 
dark urine. He has recently been on an 
antibiotic for a presumed urinary tract 
infection. 

His blood tests show: 

Hb 5.0 g/dl MCV 103 fl 

MCHC 34 g/dl (32-35) WCC 8 x 10 9 /L 
reticulocytes 160 x 10 9 /L (50-100) 
platelets 130 x 10 9 /L 

PT 13s (11.5-15.5) APTT 38 s (30-40) 

urea 6 mmol/l creatinine 90 mmol/I 

sodium 140 mmol/l potassium 4 mmol/l 

bilirubin 48 (1-22) mmol/l 

AST 18 (1-31) U/l ALP 150 (20-120) U/l 

albumin 32 g/l 

LDH 1550 U/l (10-250) 

Blood film shows blister cells. 

Whot is the diagnosis? 

1- Autoimmune haemolytic anaemia 

2- Hereditary spherocytosis 

3- Paroxysmal nocturnal haemoglobinuria 

4- G6PD deficiency 

5- Porphyria 


Answer & Comments 

Answer: 4- G6PD deficiency 

The patient's clinical and laboratory findings 
(eg, markedly decreased hemoglobin and 
hematocrit levels with a markedly increased 
serum LDH activity), are characteristic of acute 
oxidant damage to the red blood cells and 
hemolysis due to glucose-6-phosphate 
dehydrogenase (G6PD) deficiency. 

The blood film in G6PD deficiency shows 
blister cells (membrane protrusion) (Heinz 
bodies may also be seen when there is no 
haemolysis). Treatment is with blood 
transfusion, or in severe cases, exchange 
transfusion. 


Hemolytic crisis occurs only after exposure to 
certain offending agents, including drugs, 
infections, exposure to fava beans, and 
diabetic acidosis. Drugs associated with 
hemolysis in G6PD deficiency include 
antimalarials (Primaquine, pamaquine), 
sulphonamides (Sulphamethoxazole), 

nitrofurantoin, analgesics (acetaminophen, 
aspirin, phenacetin), isoniazid (I NH), 
methylene blue, and nalidixic acid. 


[ Q: 868 ] MRCPass - Haematology 

A 32 year old Cypriot patient is being 
investigated for anaemia. He has a Hb of 7.5 
g/dl and MCV is 70 fl. His brother and sisters 
are also anaemic. 

Which one of the following is most likely? 

1- Increased IgM band on serum 
electrophoresis 

2- Red cells show marked hypochromia 

3- Severe iron deficiency due to Gl bleeding 

4- Severe B12 deficiency due to pernicious 
anaemia 

5- Severe folate deficiency due to celiac 
disease 



Answer & Comments 

Answer: 2- Red cells show marked 
hypochromia 


This patient is likely to have thalassaemia 
(probably major). Hb electrophoresis may 
show increased HbA2 in thalassaemia minor. 
The severe imbalance of globin chain synthesis 
(alpha » beta) results in ineffective 
erythropoiesis and severe microcytic 
hypochromic anemia, there may also be 
precipitates within damaged red cells. 



[ Q: 869 ] MRCPass - Haematology 


A 35 year old man has known type 1 
Von Willebrand's disease. 


Dr. Kholid Yusuf El-Zohry - Sohog Teaching Hospital (01118391123) 

Ref MRCPass OE OE 2012 PasTest 2009 PassMedicine 2009 PasTest Exam 




ReviseMRCP 

375 

























El-zohry MRCP Questions Bank (Port 1) - 2013 


(For my personal use) 


Prior to surgery , which is the best test to 
assess bleeding tendency? 

1- Prothrombin time 


However, after adolescence, the disease tends 
to run a chronic relapsing course and 
therefore requires therapy. 


2- Factor VIII antigen 

3- Factor VIII levels 

4- Bleeding time 

5- Thrombin time 


Answer & Comments 

Answer: 2- Factor VIII antigen 

Bleeding time is usually prolonged, and does 
not provide quantification of bleeding 
tendency. Factor VIII antigen measures the 
presence of vWF and gives a good estimate of 
tendency to bleed. 


^ [ Q: 870 ] MRCPass - Haematology 

m An 19 year old man presents to the 
A&E with a petechial rash and platelet count 
of 5 x 10 9 /l_. He is otherwise well. A diagnosis 
of idiopathic thrombocytopenic purpura is 
made. 


Which of the following statements is true? 


First line therapy is oral steroids. Patients who 
are refractory to, or are intolerant of steroids 
may respond to intravenous immunoglobulins 
(IVIg) or anti-D. 



[ Q: 871 ] MRCPass - Haematology 

A 35 year old man has a faint 
maculopapular rash on his chest and a few 
shotty lymph nodes. His bloods show Hb 13.5 
g/dl, WCC 14.0 x 10 9 /L, pits 300 x 10 9 /L. Blood 
film shows reactive lymphocytes. 

Which of the following diagnosis is likely? 

1- Tuberculosis 

2- Non hodgkin's lymphoma 

3- Hepatitis B 

4- Infectious mononucleosis 


5- Pneumonia 


Answer & Comments 


Answer: 4- Infectious mononucleosis 


1- The patient should be given a platelet 
transfusion 

2- The patient should be observed 

3- The patient should be treated with Anti-D 

4- The patient should be treated with 
intravenous immunoglobulin 

5- The patient should be commenced on 
steroids 


Answer & Comments 

Answer: 5- The patient should be commenced 
on steroids 


There are several reactive lymph nodes as well 
as reactive lymphocytes suggestive of Ebstein 
Barr virus infection / infectious 
mononucleosis. Other causes of reactive 
lymphocytes are CMV infection, toxoplasmosis 
and HIV. 


[ Q: 872 ] MRCPass - Haematology 

A 65 year old woman has symptoms 
of easy bruising. She was referred by the GP 
for investigation. On examination, she had 
splenomegaly. 

Results show : 



In younger patients with ITP, the disease 
usually remits spontaneously within several 
weeks and no treatment is usually required 
unless there is significant bleeding. 


Haemoglobin 6.5 g/dL (11.5-16.5) 
White cell count 17 x 10 9 /I (4-11) 
Platelet count 32 x 10 9 /I (150-400) 


Dr. Khalid Yusuf El-Zohry - Sohag Teaching Hospital (01118391123) 

Ref MRCPass OE OE 2012 PasTest 2009 PassMedicine 2009 PasTest Exam ReviseMRCP 



376 





























El-zohry MRCP Questions Bank (Port 1) - 2013 


(For my personal use) 


Blood film shows lymphocytosis, myeloblasts 
and promyelocytes. 

Which one of following investigations is of 
prognostic value in this situation? 

1- Blood film 

2- Bone marrowaspirate 

3- Cytogenetic karyotyping 

4- Immunophenotyping 

5- Serum electrophoresis 


Answer & Comments 

Answer: 3- Cytogenetic karyotyping 

Cytogenetic monitoring of the clinical course 
of acute myeloid leukaemia (suggested by 
blasts) is often associated with a specific 
chromosomal change, ie, t(8;21) in M2. 
Establishment of the change at diagnosis 
allows recognition of the leukemic cells in the 
marrow when relapse or residual disease is to 
be evaluated. It also provides a prognostic 
determinant. 


[ Q: 873 ] MRCPass - Haematology 

A 52 year old female presents with 
acute chest pain and breathlessness. The 
chest pains were pleuritic and started to 
develop a week ago. Examination reveals 
prominent P2 and clear breath sounds. She 
had bilateral ankle oedema. A urine dipstick 
showed protein +++. 



AT III deficiency is associated with venous 
thrombosis. In this case, the history is 
consistent with a clinical diagnosis of 
pulmonary emboli and renal vein thrombosis. 


^ [ Q: 874 ] MRCPass - Haematology 

/% - 

# A 35 year old man has had allogeneic 

bone marrow transplantation which is HLA 
matched. 2 weeks later he develops a diffuse 
rash all over his body, feels sick and vomits 
several times. His temperature is 38°C. Blood 
tests show : 

Hb 11.0 g/dl 

WCC 3 x 10 9 /L 

Neutrophils 1.5 x 10 9 /L 

platelets 18 x 10 9 /L 

PT 18s (11.5-15.5) 

urea 7 pmol/l 

creatinine 70 pmol/l 

sodium 135 mmol/I 

potassium 4 mmol/l 

bilirubin 28 pmol/l 

AST 48 U/l 

ALP 155 U/l 

albumin 32 g/l 

LDH 550 U/l 

Which of the following is most likely? 

1- Bone marrow failure 


Which is the most likely explanation for these 
findings? 

1- Factor V Leiden 

2- Reduced antithrombin III activity 

3- Reduced levels of Von Willebrand's factor 

4- Reduced d dimer concentration 

5- Reduced factor VIII 


2- Parvovirus infection 

3- Leukaemic spread 

4- Aplastic anaemia 

5- Graft versus host disease 


Answer & Comments 


Answer: 5- Graft versus host disease 


Answer & Comments 


Answer: 2- Reduced antithrombin III activity 


The rash, systemic symptoms, deranged liver 
enzymes point towards GVHD. T cells from the 
donor are attacking the recipient. Treatment 


Dr. Khalid Yusuf El-Zohry - Sohag Teaching Hospital (01118391123) 

Ref MRCPass OE OE 2012 PasTest 2009 PassMedicine 2009 PasTest Exam 




ReviseMRCP 

377 

























El-zohry MRCP Questions Bank (Port 1) - 2013 


(For my personal use) 


is with immunosuppression: ciclosporin, 
methylprednisolone, methotrexate or 
antithymocyte globulin (ATG). 


month history of lower back pain and 
weakness. 

His blood tests show : 


[ Q: 875 ] MRCPass - Haematology 

A 25 year old woman presented 
unwell with diarrhoea occurring 5 times a day 
for 4 days. She had not passed urine for a day. 

Investigations: 

Haemoglobin 8.2 g/dL 

White cell count 14.2 x 10 9 /L 

Neutrophils 10.5 x 10 9 /L 

Platelets 32 x 10/L 

Fibrinogen 5 g/dL 

Serum sodium 138 mmol/L 

Serum potassium 6.3 mmol/L 

Serum urea 38 mmol/L 

Serum creatinine 450 umol/L 

Serum albumin 29 g/L 

Dipstick urine Blood + Protein + 

What is the diagnosis? 

1- Idiopathic thrombocytopenic purpura 

2- Myelodysplastic syndrome 

3- Disseminated intravascular coagulation 

4- Haemolytic uraemic syndrome 

5- Aplastic anaemia 



Answer & Comments 


Answer: 4- Haemolytic uraemic syndrome 


The most likely diagnosis is Haemolytic 
uraemic syndrome due to diarrhoea 
associated with E coli infection. A stool sample 
for culture and blood film are important 
investigations to be performed. 


r 



[ Q: 876 ] MRCPass - Haematology 
A 60 year old man has a several 


Hb 11.0 g/dl 
MCV 95 fl 
WCC 2.5 x 10 9 /L 
platelets 130 x 10 9 /L 
PT 13 s (11.5-15.5) 

APTT 28s (30-40) 

urea 26 pmol/l 

creatinine 280 pmol/l 

sodium 138 mmol/l 

potassium 4 mmol/l 

bilirubin 38 pmol/l 

AST 26 U/l 

ALP 150 U/l 

albumin 33 g/l 

total protein 95 g/l 

What is the most likely diagnosis? 

1- Multiple myeloma 

2- Metastatic bladder carcinoma 

3- Lymphoma 

4- Paraglanglioma 

5- Chronic myeloid leukaemia 


Answer & Comments 


Answer: 1- Multiple myeloma 


Multiple myeloma is most likely. There is 
raised protein (60-80 normal range) indicated 
probably paraproteinaemia. There is also low 
white cell count due to bone marrow 
infiltration, and renal failure. The symptoms of 
bone pain also suggests infiltration. 



[ Q: 877 ] MRCPass - Haematology 
A 20 year old girl is being 


Dr. Khalid Yusuf El-Zohry - Sohag Teaching Hospital (01118391123) 

Ref MRCPass OE OE 2012 PasTest 2009 PassMedicine 2009 PasTest Exam ReviseMRCP 

































El-zohry MRCP Questions Bank (Port 1) - 2013 


(For my personal use) 


investigated for anaemia. Her father has 
previously had a splenectomy. Her blood film 
shows spherocytes and anaemia. 

In view of the likely diagnosis , which is the 
most useful investigation? 

1- Reticulocyte count 

2- Mott cell 

3- Haemosidnerinuria 

4- Haptoglobin 

5- IgG and C3 complement 


Answer & Comments 

Answer: 5- IgG and C3 complement 

This patient is likely to have hereditary 
spherocytosis in view of the family history and 
that the father has had splenectomy as 
treatment. How ever, raised reticulocyte 
count and decreased haptoglobins and 
increased haemosidurinuria will be all be 
present as a single test, unhelpful. The Direct 
Antiglobulin Test is used to detect IgG or C3 
bound to the surface of the red cell. In this 
scenario, it is helpful to exclude autoimmune 
haemolytic anaemia, since spherocytes would 
also be present on the blood film in AIHA. 

[ Q: 878 ] MRCPass - Haematology 

A 34 year old lady has a past history 
of an episode of deep vein thombosis and two 
miscarriages. She presents now with further 
episoder of DVT. She had a thrombophilia 
screen and was found to have a positive anti 
cardiolipin antibody. 

What is the best treatment? 

1- Clopidogrel 

2- Warfarin 3 months 

3- Long term low molecular weight heparin 

4- Aspirin and Warfarin 

5- Lifelong warfarin 



Answer & Comments 

Answer: 5- Lifelong warfarin 

This patient with recurrent DVTs has the 
presence of lupus anticoagulant. She requires 
lifelong warfarin treatment. 

[ Q: 879 ] MRCPass - Haematology 

A 40 year old male who has a 
rheumatoid arthritis is admitted with a urinary 
tract infection. 

Results show : 

haemoglobin 7.5 g/dL (11-16) 

white cell count 1.5 x 10 9 /L (4-11) 

platelets 70 x 10 9 /L (150-400) 

Which one of the following drugs is the most 
likely cause of pancytopenia? 

1- Azathioprine 

2- Cyclophosphamide 

3- Prednisolone 

4- Cyclosporin 

5- Chloroquine 



Answer & Comments 

Answer: 1- Azathioprine 

Azathioprine is a thiopurine analogue drug 
which is metabolised in the liver to 
mercaptopurine. The main side effects are 
bone marrow suppression (may lead to a 
pancytopenia) and also drug induced 
hepatitis. 


[ Q: 880 ] MRCPass - Haematology 

A 50 year old man presents with 
multiple bruises in the arms after working in 
the garden. 

Investigations showed: Hb 13.2 g/dl, WCC 5 x 
10 9 /L, platelet count 5 x 10 9 /L. 

A bone marrow examination showed normal 
numbers of megakaryocytes and a diagnosis 



Dr. Khalid Yusuf El-Zohry - Sohag Teaching Hospital (01118391123) 

Ref MRCPass OE OE 2012 PasTest 2009 PassMedicine 2009 PasTest Exam ReviseMRCP 



379 

































El-zohry MRCP Questions Bank (Port 1) - 2013 


(For my personal use) 


of idiopathic thrombocytopenic purpura was 
made. 

Whot is the most appropriate treatment? 

1- Tranexemic acid 

2- Oral prednisolone 

3- Blood transfusion 

4- Platelet transfusions 

5- Intravenous immunoglobulin 


Answer & Comments 

Answer: 2- Oral prednisolone 

The most appropriate treatment for this 
patient who is symptomatic from ITP is oral 
prednisolone. If the bleeding becomes severe, 
then IV immunoglobulin should be considered 
in addition to the steroids. 


[ Q: 881 ] MRCPass - Haematology 
An 12 year old girl has recurrent 

epistaxis. 

Her investigations show: 

Hb 11 g/dl, Pits 300 x 10 9 /L, PT 16 sec (13-16 
sec), aPTT 95 sec (28-38 sec). 

Which of the following deficiencies is most 
likely? 

1- Factor V deficiency 

2- Factor VII deficiency 

3- Von Willebrand's factor 

4- Anticardiolipin antibody 

5- Factor X deficiency 



Answer & Comments 

Answer: 3- Von Willebrand's factor 

von Willebrand's disease would be most likely 
due to the prolonged APTT, the rest of the 
factors (same ones as those which warfarin 
act on) prolong PT. 


[ Q: 882 ] MRCPass - Haematology 

A 70 year old man is on lifelong oral 
anticoagulation for recurrent DVT. 

He presents with minor bleeding from his 
gums for 1 day. 

His INR is 9.0. 

All other investigations are normal and he is 
otherwise well. 

What is the most appropriate course of 
action ? 

1- Stop warfarin, monitor INR and restart 
when INR <5.0 

2- Stop warfarin and administer Vitamin K 2 
mg 

3- Stop warfarin and institute either LMW 
heparin 

4- Stop warfarin and give FFP 

5- Reduce dose of warfarin to 0.5 mg until INR 
normalises 



Answer & Comments 

Answer: 1- Stop warfarin, monitor INR and 
restart when INR <5.0 

If there is only minor bleeding, then cessation 
of warfarin is all that is required. If there are 
other risk factors or if there is major bleeding 
then the use of vitamin K or fresh frozen 
plasma should be considered. 


[ Q: 883 ] MRCPass - Haematology 

A 65 year old lady has a diagnosis of 
Non Hodgkin's lymphoma and has recently 
commenced chemotherapy. She now 
complains of feeling breathless and unwell. 

On examination, she is pale and slightly 
jaundiced. She has splenomegaly. 

Investigations show : 

Hb 3.5 g/dl 

MCV 106 fl 

WCC 8 x 10 9 /L 



Dr. Khalid Yusuf El-Zohry - Sohag Teaching Hospital (01118391123) 

Ref MRCPass OE OE 2012 PasTest 2009 PassMedicine 2009 PasTest Exam ReviseMRCP 



380 






























El-zohry MRCP Questions Bank (Port 1) - 2013 


(For my personal use) 


platelets 250 x 10 9 /L 
Reticulocytes 125 x 10 9 /L (N 20-90) 

Her FBC pre-chemotherapy was normal. 
Whot is the most likely explanation for this? 

1- Paroxysmal cold haemoglobinuria 

2- Bone marrow suppression 

3- Megaloblastic anaemia 

4- Autoimmune haemolysis 

5- Paroxysmal nocturnal haemoglobinuria 


Answer & Comments 


Answer: 4- Autoimmune haemolysis 


[ Q: 885 ] MRCPass - Haematology 

A 13 year old child has had recurrent 
episodes of bone pain. He has been admitted 
to hospital several times due to severe pains 
in the last 5 years. 

He has X rays which show necrosis of the hip. 
Which of the following diagnosis is likely? 

1- Multiple myeloma 

2- Paget's disease 

3- Osteopetrosis 

4- Sickle cell disease 

5- Thalassemia 



Anaemia, raised MCV and high reticulocyte 
count suggests haemolysis. This may occur 
secondary to NHL. In addition, there is a 
strong association with fludarabine. 


[ Q: 884 ] MRCPass - Haematology 

A patient has had a splenectomy 
because of hereditary spherocytosis. 

How long should penicillin prophylaxis be 
used? 

1- During acute infections 

2- 1 year 

3- 10 years 

4- 15 years 

5- Life long 



Answer & Comments 


Answer: 5- Life long 


Answer & Comments 

Answer: 4- Sickle cell disease 

Aseptic necrosis of the hip, cholecystitis, renal 
papillary necrosis and proliferative 
retinopathy are clinical features of sickle cell 
disease. 



[ Q: 886 ] MRCPass - Haematology 

A 20 year old woman with sickle cell 
anemia presents with acute shortness of 
breath. A chest x ray obtained with a portable 
unit initially showed no abnormalities except 
for bibasilar hazy opacities. 


Five and a half hours after admission, her 
oxygen saturation decreased to 76 percent 
with a respiratory rate of 24 breaths per 
minute. A repeat chest radiograph revealed 
increased interstitial markings. 

How should she be treated? 


Following splenectomy, patients should 
receive lifelong penicillin prophylaxis. The 
major complication of splenectomy is overw 
helming sepsis with encapsulated bacteria (eg, 
S pneumoniae, H influenzae, N meningitidis). 
The overall risk of sepsis in asplenic patients is 
approximately 2% but varies depending on the 
age and underlying diseases. 


1- Antibiotics and fluids 

2- Intubation and ventilation 

3- Noninvasive ventilation and plasma 
exchange 

4- Splenectomy 

5- High flow oxygen 


Dr. Khalid Yusuf El-Zohry - Sohag Teaching Hospital (01118391123) 

Ref MRCPass OE OE 2012 PasTest 2009 PassMedicine 2009 PasTest Exam 




ReviseMRCP 

381 





























El-zohry MRCP Questions Bank (Port 1) - 2013 


(For my personal use) 


Answer & Comments 

Answer: 3- Noninvasive ventilation and plasma 
exchange 


Haemolytic disease of the new born was 
suspected. 

Which of these statements is likely? 


This is a case of acute chest syndrome related 
to sickle cell anaemia. Non invasivve 
ventilation (CPAP) and plasma exchange is the 
best option, often along with antibiotics 
because the chest syndrome can be 
precipitated by infection. 

[ Q: 887 ] MRCPass - Haematology 

A 25 year old female has had her 
first DVT when she started taking the oral 
contraceptive pill. 

She reveals that her mother has also had DVT 
before. 

Which of the following is she likely to hove? 

1- Factor V leiden deficiency 

2- Protein C deficiency 

3- Protein S deficiency 

4- Antithrombin III deficiency 


1- Father is 0 Rh -ve 

2- Father is AB Rh +ve 

3- Mother is AB Rh +ve 

4- Father is AB Rh -ve 

5- Mother is 0 Rh +ve 

Answer & Comments 

Answer: 2- Father is AB Rh +ve 

To Answer this question, ABO group is less 
relevant and Rh status is relevant. 

Rh grouping of foetus is decided by the Rh 
status of the father. The first child would have 
been Rh +ve and led to sensitisation 
(antibodies developed by the mother). The 
mother is Rh -ve and the father is Rh +ve (who 
could also be homozygous or heterozygous for 
Rh). 



5- Lupus anticoagulant 


Answer & Comments 

Answer: 1- Factor V leiden deficiency 

Although they are all possibilities, the family 
history suggests factor V leiden or 
antithrombin III deficiency. A female who has 
DVT precipitated by the OCP suggests factor V 
leiden more so than antithrombin III (male 
would suggest this). 


[ Q: 889 ] MRCPass - Haematology 

A 35 year old man complains of leg 
cramps and is given quinine for the first time 
by his GP. He then presents unwell and 
complains of dark urine, his Hb is 7.4 g/dl. 
Direct antiglobulin test (DAT) test is negative. 

Which of the following is likely? 

1- G6PD deficiency 

2- Autoimmune haemolytic anaemia 

3- Paroxysmal cold haemoglobinuria 



[ Q: 888 ] MRCPass - Haematology 

A 32 year old lady had a new born 
baby with marked jaundice. 

Serum bilirubin was 359 mmol/L and 
haemoglobin low . 



4- Phosphokinase deficiency 

5- Sickle cell disease 


Answer & Comments 


Answer: 1- G6PD deficiency 


The mother has had one previous normal 
delivery. 


The direct antiglobulin test (Coomb's) is 
negative and suggests that this is not 
autoimmune since there is no antibody 


Dr. Kholid Yusuf El-Zohry - Sohog Teaching Hospital (01118391123) 

Ref MRCPass OE OE 2012 PasTest 2009 PassMedicine 2009 PasTest Exam ReviseMRCP 

































El-zohry MRCP Questions Bank (Port 1) - 2013 


(For my personal use) 


opsonisation on red cells. Quinine can 
precipitate haemolysis in G6PD deficiency as 
can aspirin, sulphonamides, fava beans and 
antimalarial agents. 



[ Q: 890 ] MRCPass - Haematology 

A 40 year old lady with presents with 
chronic discomfort in left upper quadrant of 
the abdomen. Investigations show : 


Hb 16.9 g/dl 
MCh 55 (28-32) pg 
MCV 69 fl 


White cell count 11 x 10 9 /L 

Platelets 490 x 10 9 /L 

Whot is the underlying couse ? 


Laboratory investigations revealed 
hemoglobin of 4.5 gm/dL, and platelet count 
342 x 10 9 /L. 

Peripheral smear showed anisocytosis(+), 
poikilocytosis(+), spherocytosis(-), heinz body 
(+), sickling(-) and reticulocyte count 6.2%. 

Other results: aspartate aminotransferase 420 
iu/L, alanine aminotransferase 104 iu/L, 
indirect bilirubin 5.2 mg/dL, lactate 
dehydrogenase 721 iu/L. Direct Coombs test 
was negative. 

Urine analysis: red colored, protein (+), 
billirubin (+++). 

Whot is the most likely diagnosis? 

1- Haemolytic uraemic syndrome 

2- Sickle cell disease 


1- Essential thrombocythaemia 

2- Primary polycythaemia 

3- Renal cell carcinoma 

4- Myelodysplasia 

5- Chronic myeloid leukaemia 

Answer & Comments 

Answer: 2- Primary polycythaemia 

The raised Hb, white cell and platelet count 
are consistent with polycythaemia rubra vera. 
Splenomegaly is common, and occasionally 
splenic infarction as well which may lead to 
left upper quadrant pains. Essential 
thrombocythaemia is associated with 
anaemia. 


[ Q: 891 ] MRCPass - Haematology 

A 30 year old man presents with 
reddish urine and paleness. He mentions 
recently travelling to Asia and being on 
malarial prophylaxis. On physical examination 
he had an axillary temperature of 36.5oC, 
pulse of 120/min, respiratory rate of 28/min 
and blood pressure of 80/60 mmHg. Other 
physical findings were normal. 



3- Malaria 

4- G6PD deficiency 

5- Hereditary spherocytosis 

Answer & Comments 

Answer: 4- G6PD deficiency 

Patients with G6PD deficiency are often 
asymptomatic. Chronic haemolysis does not 
occur, but haemolysis rather is precipitated by 
drugs such as chloroquine, primaquine, 
vitamin K, sulphonamides. G6PDH catalyzes 
the synthesis of NADPH from the hexose 
monophosphate pathway. Deficiency leads to 
oxidative damange and red cell haemolysis. 


) 

) 




o 


U| 

lo 


°oo < 

o'- O' 

eO A • C 

oo -O 

oo 0o 

° O /* o. 


o. 


© 


Acute hemolysis in G6PD deficiency, with two 


Dr. Kholid Yusuf El-Zohry - Sohog Teaching Hospital (01118391123) 

Ref MRCPass OE OE 2012 PasTest 2009 PassMedicine 2009 PasTest Exam 


" (arrows). 






ReviseMRCP 

383 
























El-zohry MRCP Questions Bank (Part 1) - 2013 


(For my personal use) 


^ [ Q: 892 ] MRCPass - Haematology 

ft - 

# An 12 year old boy bleeds 
excessively after a laceration. Investigations 
show: 

Hb 13.5 g/dl 
WBC 5.8 x 10 9 /l_ 

Pits 270 x 10 9 /L 
PT 15 sec (13-16 sec) 

APTT 85 sec (28-38 sec) 

Factor VIII and Factor IX levels : normal 


APTT 50:50 mix with normal plasma : 37 sec 

Which of the following is the most likely 
diagnosis? 

1- Haemophilia A 

2- Factor XI deficiency 

3- Factor X deficiency 

4- Factor XII deficiency 

5- Factor VII deficiency 


Blood cultures did not grow any organisms. 
Despite intravenous antibiotics, the patient 
remained pyrexial. 

What is the likely cause? 

1- CMV 

2- Candidiasis 

3- Metastatic disease 

4- Tuberculosis 

5- Rubella 


Answer & Comments 

Answer: 1- CMV 

The likely cause of persisting pyrexia plus 
hepatitis in this immunocompromised patient 
after treated with appropriate antibiotics 
would be a CMV infection. CMV infection can 
also cause a pneumonitis and colitis. 
Treatment with an antiviral agent such as 
ganciclovir could be considered. 


Answer & Comments 

Answer: 2- Factor XI deficiency 

An isolated prolonged APTT will be caused by 
deficiencies in factors VIII, IX, XI and XII and by 
von Willebrand's disease. Factor XII deficiency 
is not associated with increased bleeding. 

With normal Factor VIII and IX levels, this 
patient is most likely to have Factor XI 
deficiency. Factor XI deficiency is a mild 
bleeding disorder with autosomal co¬ 
dominant inheritance. Diagnosis is by specific 
Factor XI level estimation. 

[ Q: 893 ] MRCPass - Haematology 

A 48 year old man has a diagnosis of 
acute myeloid leukaemia. He was given 
chemotherapy. A few weeks into induction 
chemotherapy, he develops jaundice and 
fevers. 



[ Q: 894 ] MRCPass - Haematology 

A 60 year old woman presents with 
cervical lymphadenopathy and 

hepatosplenomegaly. Investigations reveal: 

Hb 10.5 g/dl 

WBC 4.6 x 10 9 /L 

Plats 125 x 10 9 /L 

serum electrophoresis : IgM paraprotein 
detected, IgA and IgG levels are normal 

The most likely diagnosis is: 

1- Monoclonal gammopathy of uncertain 
significance (MGUS) 

2- Chronic lymphocytic leukaemia 

3- Hodgkin's lymphoma 

4- Waldenstrom's macroglobulinaemia 

5- Multiple myeloma 



Dr. Khalid Yusuf El-Zohry - Sohag Teaching Hospital (01118391123) 

Ref MRCPass OE OE 2012 PasTest 2009 PassMedicine 2009 PasTest Exam ReviseMRCP 



384 



























El-zohry MRCP Questions Bank (Port 1) - 2013 


(For my personal use) 


Answer & Comments 

Answer: 4- Waldenstrom's 

macroglobulinaemia 

The fact that there is IgM paraprotein 
suggests Waldenstrom's macroglobulinaemia. 
Waldenstrom's macroglobulinemia is a 
lymphoplasmacytic lymphoma (invasion of 
bone marrow ) that leads to secretion of IgM. 
Increased blood viscosity may result in 
thrombotic phenomenon, weakness, 
cryoglobulinemia, neurologic disorders, and 
fatigue. 



Multiple plasmacytoid cells in Waldenstrom's 

macroglobulinaemia 


[ Q: 895 ] MRCPass - Haematology 

history of night sweats. On 
examination, he had lymphadenopathy 
palpable in the cervical region and 
hepatosplenomegaly. 

Investigations: 

hemoglobin 8 g/dL 

leukocyte count of 6.6 x 10 9 /L 

A lymph node biopsy of right neck was 
performed. It showed effaced normal 
architecture and clusters and sheets of Reed- 
Sternberg cells 

What is the most likely diagnosis? 

1- Essential thrombocythaemia 

2- Acute myeloid leukaemia 

3- Chronic myeloid leukaemia 

4- Hodgkin's disease 



5- Infectious mononucleosis 

Answer & Comments 

Answer: 4- Hodgkin's disease 

Common presenting features for Hodgkin's 
diseaese are Pel Ebstein fever, weight loss, 
alcohol induced pain and lymphadenopathy. 
Cold agglutinins can occur, leading to possible 
haemolytic anaemia. 



Reed-Sternberg cell in Hodgkin's disease with 
large, prominent nucleoli. 


[ Q: 896 ] MRCPass - Haematology 

A 45 year old woman has a history of 
recurrent anaemia was noted have target cells 
and Howell Jolly bodies on a blood film 
examination. Investigations show : 

Haemoglobin 7.8 g/dL 

MCV 75 fl MCH 27 pg (28-32) 

Serum B12 132 ug/L (160-760) 

Red cell folate 90 ug/L (160-640) 

Serum ferritin 9 ug/L (15-300) 

Which antibody is likely to be present? 

1- Anti mitochondrial antibody 

2- Intrinsic factor antibody 

3- Anti endomysial antibody 

4- Anti thyroid antibody 

5- Anti gastric parietal cell antibody 



Answer & Comments 


Answer: 3- Anti endomysial antibody 


Dr. Khalid Yusuf El-Zohry - Sohag Teaching Hospital (01118391123) 

Ref MRCPass OE OE 2012 PasTest 2009 PassMedicine 2009 PasTest Exam ReviseMRCP 





























El-zohry MRCP Questions Bank (Port 1) - 2013 


(For my personal use) 


This patient has iron, folate and B12 
deficiency. This is most likely due to coeliac 
disease. 

FBC shows anaemia in 50% of coeliac disease 
patients; iron and folate deficiency are both 
common (microcytes and macrocytes), 
hypersegmented leucocytes and Howell-Jolly 
bodies (hyposplenism). 

The preferred investigations are IgA Anti¬ 
tissue transglutaminase (tTG) or IgA 
Endomysial (EMA) antibodies. Antigliadin 
antibodies are less specific, they can be 
positive in Crohn's as well. 



Howell-Jolly bodies are spherical blue-black 
inclusions within red blood cells (iron 
deficiency, thalassemia, post splenectomy) 


[ Q: 897 ] MRCPass - Haematology 

A 35 year old woman presents with 
jaundice and lethargy. Investigations reveal: 

Haemoglobin 8.5 g/dL (11-16) 

White cells 7 x 10 9 /L 

Platelets 190 x 10 9 /L 

reticulocyte count 130 x 10 9 /L (25-85) 

serum bilirubin 55 umol/L (1-20) 

Blood film shows spherocytes 

Which of the following should be done? 

1- Direct antiglobulin test 

2- Ultrasound of the abdomen 

3- Bone marrow biopsy 

4- Bone marrow aspirate 

5- G6PD enzyme level 



Answer & Comments 

Answer: 1- Direct antiglobulin test 

One of the first tests to consider in a patient 
with haemolytic anaemia is the direct 
antiglobulin test (Coomb's). This to exclude 
autoimmune haemolytic anaemia. 



[ Q: 898 ] MRCPass - Haematology 

A 42 year old woman has a history of 
positive lupus anticoagulant. She had a 
pulmonary embolus diagnosed 8 years ago, 
and two presentations which were consistent 
with deep vein thrombosis 6 and 12 months 
go. 


Whot is the best management? 


1- High dose aspirin 

2- Lifelong warfarin 

3- Antenatal advice 


4- Avoidance of oral contraceptive pill 

5- 6 months of warfarin then reassess 


Answer & Comments 

Answer: 2- Lifelong warfarin 

More than one thrombotic event with the 
presence of lupus anticoagulant suggests that 
the patient requires lifelong warfarin. 


[ Q: 899 ] MRCPass - Haematology 

A 65 year old woman has a 
haemoglobin of 5.5 g/dl. She has lethargy but 
no other symptoms. 

Her blood film shows oval macrocytes and 
hypersegmented neutrophils. She has a 
history of hypothyroidism and is on thyroid 
replacement. She also has vitiligo. 

What is the most likely diagnosis? 

1- Multiple myeloma 

2- Myelodysplasia 

3- Pernicious anaemia 



Dr. Khalid Yusuf El-Zohry - Sohag Teaching Hospital (01118391123) 

Ref MRCPass OE OE 2012 PasTest 2009 PassMedicine 2009 PasTest Exam ReviseMRCP 



386 




























El-zohry MRCP Questions Bank (Part 1) - 2013 


(For my personal use) 


4- Iron deficiency anaemia 

5- Haemolytic anaemia 


adrenal vein thrombosis and can cause 
hypoadrenalism as in this case. 


Answer & Comments 

Answer: 3- Pernicious anaemia 

Pernicious anaemia (PA) is a disease of the 
stomach that is characterised by megaloblastic 
anaemia due to vitamin Bn deficiency. It is 
secondary to intrinsic factor deficiency and 
gastric atrophy. It usually has an autoimmune 
basis. Pernicious Anaemia primarily affects the 
elderly - most patients are over 60 years of 
age. Women are affected more often than 
men, in a ratio of 3:2. It may be associated 
with autoimmune diseases, such as Addison's 
disease, hypothyroidism and also an increased 
risk of gastric carcinoma. 


[ Q: 900 ] MRCPass - Haematology 

A 40 year old lady has been on 
warfarin for previous DVT. She is now 
breathless and a CTPA confirms pulmonary 
embolus despite her INRs being in therapeutic 
range of 2-3. She is also hyponatraemic with a 
sodium of 129 mmol/l. 

With the short synacthen test, she has a low 
cortisol of 80nmol at 0 min going up to 200 
nmol at 30 min. 

Which of the following diagnosis is likely? 

1- Autoimmune polyendocrine syndrome 

2- Adrenal tumour 

3- Protein C deficiency 

4- Factor V leiden deficiency 

5- Presence of lupus anticoagulant 



Answer & Comments 


Answer: 5- Presence of lupus anticoagulant 



[ Q: 901 ] MRCPass - Haematology 

A 20 year old girl receives a blood 
transfusion. 5 minutes after the transfusion is 
commenced, she develops a tachycardia and 
abdominal pains. 


Which of the following is the correct 
management of an acute haemolytic 
transfusion reaction due to ABO blood group 
incompatibility? 


1- Stop transfusion and assess 


2- Repeat cross match and re-transfuse 


3- Intravenous dextrose 


4- Hydrocortisone lOOmg intravenously 

5- Continue transfusion slowly 


Answer & Comments 

Answer: 1- Stop transfusion and assess 

The immediate treatment of an acute 
haemolytic transfusion reaction due to a 
major blood group incompatibility is to 
discontinue the blood transfusion 
immediately. This should be followed by 
assessment for possible shock and 
resuscitation with fluids e.g. colloids. 



[ Q: 902 ] MRCPass - Haematology 

A 35 year old man has recently been 
commenced on low molecular weight heparin 
and then warfarin following a diagnosis of 
DVT. 


Which of the following is well known long term 
side effect of heparin? 

1- Polycythaemia 

2- Visual loss 


Antiphospholipid syndrome is most likely due 
to the recurrent thrombotic tendency, lupus 
anticoagulant or anticardiolipin antibodies 
may be present. It has a propensity towards 


3- Renal impairment 

4- Osteoporosis 

5- Hirsutism 


Dr. Khalid Yusuf El-Zohry - Sohag Teaching Hospital (01118391123) 

Ref MRCPass OE OE 2012 PasTest 2009 PassMedicine 2009 PasTest Exam 




ReviseMRCP 

387 





























El-zohry MRCP Questions Bank (Port 1) - 2013 


(For my personal use) 


Answer & Comments 

Answer: 4- Osteoporosis 

Heparin induced thrombocytopenia, 
osteoporosis and thrombosis can occur. 
Warfarin can cause skin necrosis. 


Answer & Comments 

Answer: 4- Antiphospholipid syndrome 

Apart from a thrombotic tendency, 
antiphospholipid syndrome is associated with 
a low white cell count and thrombocytopenia. 


[ Q: 903 ] MRCPass - Haematology 

A patient who is known to have 
hereditary spherocytosis and has mild 
jaundice and gallstones is awaiting 
splenectomy. 

How long prior to splenectomy should 
pneumococcal voccinotion be administered? 

1- 1 day 

2- 5 days 

3- 3 weeks 

4- 3 months 

5- 6 months 



Answer & Comments 

Answer: 3- 3 weeks 

Pneumococcal immunisation should be 
administered to the patient 2-4 weeks before 
splenectomy. 


-1 

[ Q: 904 ] MRCPass - Haematology 

\n 

A 28 year old woman has had a 


diagnosis of pulmonary embolus. She has the 
following investigations: 

Haemoglobin 11.3 g/dl, white cell count 4.0 x 
10 9 /l_, platelet count 45 x 10 9 /L. 

Which of the following diagnoses is more 
likely? 

1- Homocystinuria 

2- Protein C deficiency 

3- Factor V leiden deficiency 

4- Antiphospholipid syndrome 

5- Protein S deficiency 



[ Q: 905 ] MRCPass - Haematology 

A 28 year old lady is 30 weeks 
pregnant when she presents with a left sided 
DVT. She has had a previous miscarriage 
before. Her investigations show : 


Hb 10.2 g/dl 
MCV 68 fl 
WBC 8.0 x 10 9 /L 
Pits 250 x 10 9 /L 


Direct Coomb's Test: positive 
Reticulocyte count 90 x 10 9 /L (25-125) 
APTT 51 sec (normal 28-38 sec) 

PT 16 sec (normal 13-16 sec) 

Protein C activity 0.75 iu/ml (0.67-1.38) 
Total protein S 100% (64-154) 

Which of the following diagnosis is likely? 

1- Heparin induced thrombocytopenia 

2- Protein C deficiency 

3- Protein S deficiency 

4- Factor V leiden 

5- Antiphospholipid syndrome 


Answer & Comments 

Answer: 5- Antiphospholipid syndrome 

Of the following choices, antiphospholipid 
syndrome is most likely because of the 
recurrent thrombotic tendency. Raised APTT 
and positive Direct Coomb's test (measures 
presence of antibodies on red cells) can be 
caused by lupus anticoagulant. 




[Iaj 


[ Q: 906 ] MRCPass - Haematology 


Dr. Khalid Yusuf El-Zohry - Sohag Teaching Hospital (01118391123) 

Ref MRCPass OE OE 2012 PasTest 2009 PassMedicine 2009 PasTest Exam ReviseMRCP 



388 

































El-zohry MRCP Questions Bank (Port 1) - 2013 


(For my personal use) 


A 32 year old woman presents with bleeding 
gums and easy bruising. Her medications are 
lansoprazole and cispride for reflux 
oesophagitis. 

Investigations show: 

Haemoglobin 12.5 g/dL (13.0-16.5) 

MCV 90 fl (83-95) 

Platelets 35 x 10 9 /L (150-400) 

Blood film : occasional giant platelets 

Prothrombin time 12 s (11.5-15.5) 

What is the likely diagnosis? 

1- Disseminated intravascular coagulation 

2- Immune thrombocytopenia 

3- Thrombotic thrombocytopenic purpura 

4- Megakaryocytic thrombocytopenia 

5- Drug-induced thrombocytopenia 


Answer & Comments 

Answer: 2- Immune thrombocytopenia 

This is a case of immune thrombocytopenia in 
which low platelets with other counts being 
normal apart from slight anaemia (due to 
bleeding). 


[ Q: 907 ] MRCPass - Haematology 

A child has severe anaemia and been 
diagnosed as having thalassemia major. 

Which of the following is the major form of 
haemoglobin present when the condition 
exists? 

1- Haemoglobin A2 

2- Haemoglobin C 

3- Haemoglobin F 

4- Haemoglobin H 

5- Haemoglobin A 



Answer & Comments 


Answer: 3- Haemoglobin F 


In Beta-thalassaemia major there is a 
complete defect in production of beta globin 
chains, which leads to impaired formation of 
HbA (which is made up of 2 alpha and 2 beta 
chains). Haemoglobin F is the major 
haemoglobin as this haemoglobin is made up 
of alpha and gamma chains. 

Patients with thalassemia major require 
lifelong blood transfusions (hypertransfusions) 
with iron chelation therapy (desferrioxamine). 


[ Q: 908 ] MRCPass - Haematology 

A 20-year-old man was referred for 
investigation of lifelong hemolytic anemia. 
Jaundice accompanied by anemia and 
splenomegaly had been apparent since early 
life. Episodes of jaundice were more marked 
during infections or after fasting and less 
pronounced following exposure to sunlight, 
was conspicuous. 

On investigation the following results were 
obtained: 

hemoglobin, 11.8 g/dL 

mean corpuscular volume [MCV] 85.5 fL 

mean corpuscular hemoglobin [MCH] 29.1 pg 

mean corpuscular hemoglobin concentration 
[MCHC] 34 g/dL 

reticulocyte count 5.44 

Blood film shows spherical red cells with lack 
of pallor in the central area. 

What is the diagnosis? 

1- Anaemia of chronic disease 

2- Sideroblastic anaemia 

3- Megaloblastic anaemia 

4- Hereditary spherocytosis 

5- Iron deficiency anaemia 



Answer & Comments 


Answer: 4- Hereditary spherocytosis 


Dr. Khalid Yusuf El-Zohry - Sohag Teaching Hospital (01118391123) 

Ref MRCPass OE OE 2012 PasTest 2009 PassMedicine 2009 PasTest Exam ReviseMRCP 



389 




























El-zohry MRCP Questions Bank (Port 1) - 2013 


(For my personal use) 


Hereditary spherocytosis gene for ankyrin (cell 
membrane protein) has been mapped to 
chromosome 8 and is autosomal dominant. It 
presents in childhood with jaundice and 
splenomegaly. 

Treatment is with splenectomy. 



Red cells are more spherical in hereditary 
spherocytosis and lack the central area of 

pallor. 


[ Q: 909 ] MRCPass - Haematology 

A 60 year old man has been on 
warfarin for a DVT. He had an INR of 9 during 
a follow up appointment when he was noticed 
to have haematuria and epistaxis. 

Whot is the appropriate management? 

1- 2 units blood transfusion 

2- 2 units of platelets 

3- Fresh frozen plasma 1 unit 

4- Stop warfarin and observe 

5- 0.5 mg of vitamin K iv 



Answer & Comments 


Answer: 5- 0.5 mg of vitamin K iv 


The patient has minor bleeding. According to 
The British Society of Haematology guidelines, 
when INR is > 8 with minor bleeding, warfarin 
should be discontinued until the INR is < 5. IV 
vitamin K 0.5 mg or oral vitamin K 5 mg should 
then be given. 



[ Q: 910 ] MRCPass - Haematology 


A 55 year old man has had varicose 
vein repair. He develops a swollen leg five 


days following surgery and an ultrasound of 
the leg confirms a DVT. This is despite him 
having prophylactic low molecular weight 
heparin. 

His Hb is llg/dl, WCC 13 x 10 9 /l_, platelet 
count is 45 x 10 9 /L 

Which of the following could have caused the 
DVT? 

1- Behcet's disease 

2- Factor V leiden deficiency 

3- Protein C deficiency 

4- Immune thrombocytopenic purpura 

5- Heparin induced thrombocytopenia 


Answer & Comments 

Answer: 5- Heparin induced 

thrombocytopenia 

Type I heparin induced thrombocytopenia 
(HIT) occurs within a few days of heparin and 
is usually mild. 

In this case, type II HIT is more likely, and this 
occurs slightly later (5-15 days). It is associated 
with thrombosis and a low platelet count. 
Alternative anticoagulation should be used 
(hirudin, danaparoid sodium). 

[ Q: 911 ] MRCPass - Haematology 

A 55 year old man has G6PD 
deficiency. He presents with a haemolytic 
crisis after a drug was started. 

Which one of the following is probable? 

1- Carbamazepine 

2- Gentamicin 

3- Paracetamol 

4- Chloramphenicol 

5- Phenytoin 



Answer & Comments 


Answer: 4- Chloramphenicol 


Dr. Khalid Yusuf El-Zohry - Sohag Teaching Hospital (01118391123) 

Ref MRCPass OE OE 2012 PasTest 2009 PassMedicine 2009 PasTest Exam ReviseMRCP 



390 































El-zohry MRCP Questions Bank (Port 1) - 2013 


(For my personal use) 


Haemolysis in G6PD deficiency is due to 
oxidative damage (decreased generation of 
NADPH due to enzyme deficiency). 

The common categories of drugs are: 

Sulphonamides 


failure and also neurological features. 
Treament of choice is plasma exchange with 
fresh frozen plasma infusion. High dose 
steroids may also be beneficial. Plasma 
exchange removes antibodies which is the 
main pathogenic problem in the disease. 


Antimalarials 

Antipyretics (aspirin + paracetamol) 

Others : Chloramphenicol, nitrofurantoin, 
Dapsone,Probenecid, Vit K 

[ Q: 912 ] MRCPass - Haematology 

A 32 year old woman who is 20 
weeks pregnant presented with lethargy, 
confusion and drowsiness. 

On examination she has bilateral leg weakness 
and a purpuric rash was noticed on both legs. 

Investigations showed: 

Hb 8.2 g/dl 

WCC 7.2 x 10 9 /L 

platelets 25 x 10 9 /l_ 

reticulocytes 3% 

Urea 28 mmol/I 

Creatinine 360 umol/l 

Blood film showed: fragmented cells and 
polychromasia 

What treatment should be administered? 

1- Dexamethasone 



[ Q: 913 ] MRCPass - Haematology 

A 23 year old woman presented very 
unwell with a miscarriage. On examination, 
she was very pale and breathless. She had a 
blood pressure of 90/60 mmHg. 

Investigations revealed: 

Haemoglobin 9 g/dL 

Platelets 52 x 10 9 /L 

Prothrombin time 20 sec (11-15) 

APTT 55 sec (30-40) 

Fibrinogen 0.3 g/L (-4) 

Blood film : Fragmented cells 

What should be administered? 

1- Intravenous hydrocortisone 

2- Vitamin C 

3- Tranexemic acid 

4- DDAVP 

5- Cryoprecipitate 



Answer & Comments 


Answer: 5- Cryoprecipitate 


2- Plasma exchange 

3- Platelet transfusion 

4- Cyclophosphamide 

5- Blood transfusion 

Answer & Comments 

Answer: 2- Plasma exchange 

This patient has thrombotic thrombocytopenic 
purpura as suggested by haemolysis on the 
blood film, anaemia, thrombocytopenia, renal 


This is a case of disseminated intravascular 
coagulation (DIC) in which fibrinolytic system 
becomes activated, leading to thrombin 
formation. Unregulated fibrinolysis and 
systemic fibrinogenolysis occurs with release 
of plasmin into the circulation. 

Typically, the blood film shows fragmented 
red blood cells. Treatment aims to correct the 
coagulopathy with blood products e.g. 
cryoprecipitate to replace fibrinogen, or fresh 
frozen plasma. Vitamin K can also be given in 
event of significant bleeding. 


Dr. Khalid Yusuf El-Zohry - Sohag Teaching Hospital (01118391123) 

Ref MRCPass OE OE 2012 PasTest 2009 PassMedicine 2009 PasTest Exam 




ReviseMRCP 

391 























El-zohry MRCP Questions Bank (Port 1) - 2013 


(For my personal use) 



[ Q: 914 ] MRCPass - Haematology 

A 55 year old patient is known to 
have alcoholic liver disease. He 
drinks 15 pints of beer a day and has 
oesophageal varices when he had endoscopy 
3 months ago. He now has melaena with the 
following blood results : 


Hb 7.5 g/dl 
MCV 103 fl 
WCC 11 x 10 9 /L 


platelets 100 x 10 9 /L 


PT 20 s (11.5-15.5) 

APTT 40s (30-40) 

Fibrinogen 0.8.0g/L (1.8-5.4) 
urea 17 pmol/l 


creatinine 105 pmol/l 
sodium 130 mmol/I 


potassium 4 mmol/I 
bilirubin 62 pmol/l 
AST 328 U/l 
ALP 200 U/l 
albumin 32 g/l 

Apart from blood transfusion, which of the 
following would be useful? 

1- Factor VIII 


2- Cryoprecipitate 

3- Exchange transfusion 

4- Haemodialysis 

5- Albumin 


Answer & Comments 

Answer: 2- Cryoprecipitate 

With alcoholic liver disease, there is a 
prolonged PT and low platelet count. How 
ever, in severe alcoholic liver disease 
fibrinogen can also be lowas in this case, thus 
cryoprecipitate would be useful. 



[ Q: 915 ] MRCPass - Haematology 

A 50 year old man presents with 
hypertension. Further blood tests 
reveal the following: Hb 18.6g/dl, WCC 16 x 
10 9 /L, pits 600 x 10 9 /L. The erythropoietin 
level is normal. 


What is the most likely diagnosis? 

1- Secondary polycythaemia 

2- Polycythaemia rubra vera 

3- Myelofibrosis 

4- Gaucher's disease 


5- Recombinant EPO use 


Answer & Comments 

Answer: 2- Polycythaemia rubra vera 

In polycythaemia rubra vera, the Hb, WCC and 
platelet counts are high along with a normal 
EPO level. EPO is raised in secondary 
polycythaemia (e.g. hypoxia). 


[ Q: 916 ] MRCPass - Haematology 

An 80 year old lady complains of 
mild breathlessness and lethargy. She 
mentions that she is a vegetarian. There is no 
history of haemetemesis or melaena. She has 
a past medical history of congestive cardiac 
failure. 

On examination, she is pale, and has vitiligo on 
her hands. She has a JVP of +4 cm and fine 
inspiratory crepitations. 

Her investigations show : 

Hb 4.5 g/dl MCV 105 

WBC 3.3 x 10 9 /L Pits 120 x 10 9 /L 

What is the most important initial 
management? 

1- Blood transfusion 

2- Start vit B12 and folic acid 

3- Red cell mass studies 

4- Bone marrow aspirate 



Dr. Khalid Yusuf El-Zohry - Sohag Teaching Hospital (01118391123) 

Ref MRCPass OE OE 2012 PasTest 2009 PassMedicine 2009 PasTest Exam ReviseMRCP 




























El-zohry MRCP Questions Bank (Port 1) - 2013 


(For my personal use) 


5- Schilling test 

Answer & Comments 

Answer: 2- Start vit B12 and folic acid 

Pancytopenia and raised MCV suggests severe 
B12 or folate deficiency. Vitiligo is also a clue 
as to autoimmune phenomenon, and 
pernicious anaemia may be associated. In this 
lady, blood transfusion may exacerbate 
cardiac failure, and she is not actively 
bleeding, hence replacement of B12 and 
folate is a better option (symptoms will 
improve within 1-2 weeks). 

B12 deficiency can occur as a result of 
pernicious anaemia (intrinsic factor 
deficiency), dietary e.g. vegetarian, Crohns 
disease, Serum folate level less than 5 ng/ml 
or serum vitamin Bi 2 level less than 100 pg/ml 
is diagnostic. 

Folate deficiency is treated by giving folic acid 
orally at 1 to 5 mg daily. B12 deficiency is 
usually treated by parenteral administration 
of B12. The treatment schedule consists of 
giving 1000?g cobalamin intramuscularly daily 
for 10 - 14 days follw ed by 1000 ?g once a 
week till hematocrit becomes normal followed 
by 1000 ?g once a month for life in patients 
with pernicious anemia or those with 
malabsorption. Therapeutic doses of folate 
will correct the hematologic abnormalities due 
to cobalamin deficiency also but the 
neurologic abnormallities can w orsen, it is 
best to give B12 first or both B12 and folate 
but never folate alone. 


[ Q: 917 ] MRCPass - Haematology 

A 50 year old woman presented with 
a five year history of pain in the middle of 
both feet. She also had a history of back pain, 
pain in both sides of her hip, and pain in both 
metacarpals. 

She had a serum ferritin concentration of 
1087 pg/l, with normal results in liver function 
tests and a normal glucose concentration, full 



blood count, and erythrocyte sedimentation 
rate. 

Whot is the diagnosis? 

1- Wilson's disease 

2- Ochronosis 

3- Marble bone disease 

4- Haemochromatosis 

5- Thalassemia 


Answer & Comments 

Answer: 4- Haemochromatosis 

Haemochromatosis has an autosomal 
recessive pattern of inheritance and affects 1 
in 250 of the northern European population, 
with up to 10% of people carrying the gene. 
Inheritance of the disease has long been 
associated with the tissue type HLA A3. A 
specific mutation of the gene, C282Y is 
common. 

The clinical presentation of 

haemochromatosis is variable and not 
confined to the classic triad of cirrhosis, 
diabetes, and skin pigmentation. In this case 
the presentation is with arthropathy. 


[ Q: 918 ] MRCPass - Haematology 

A 56 year old man was diagnosed 
with myelofibrosis. 

Which of the following is the most common 
presentation of the disease? 

1- Bleeding 

2- Respiratory pain 

3- Hyperuricaemia 

4- Fatigue 

5- Bone pain 



Answer & Comments 

Answer: 4- Fatigue 

Clinical features of myelofibrosis include: 


Dr. Khalid Yusuf El-Zohry - Sohag Teaching Hospital (01118391123) 

Ref MRCPass OE OE 2012 PasTest 2009 PassMedicine 2009 PasTest Exam 




ReviseMRCP 

393 



























El-zohry MRCP Questions Bank (Port 1) - 2013 


(For my personal use) 


usually develops in adults over age 50 patients 
commonly present with fatigue and weakness 
spleen is often massively enlarged 
hepatomegaly occurs in over half of cases 


[ Q: 919 ] MRCPass - Haematology 

A 60 year old woman was admitted 
with a 10-month history of rash, fatigue, 
intermittent hemoptysis, and purpura. On 
admission, widespread petechiae and purpura 
with scleral icterus were noted. No 
lymphadenopathy or splenomegaly was 
present. 

Investigations showed: 
platelet count of 3 x 10 9 /L 
hemoglobin level 5.5 g/dL 
mean corpuscular volume 103 fL 
white cell count 3.6 x 10 9 /L 
neutrophils at 0.67 without myeloid blasts 
reticulocyte count was 0.20 
serum urea 15.4 mmol/L (43 mg/dL) 
creatinine, 239 pmol/L 
lactate dehydrogenase (LDH) 2505 U/L 
total bilirubin 19 pmol/L (4.6 mg/dL) 
haptoglobin, 6 g/L 

Review of the peripheral smear revealed 
notable red cell morphology and 24 nucleated 
red blood cells per 100 white blood cells, 
many of which were dysplastic. 

What is the most likely diagnosis? 

1- Idiopathic thrombocytopenic purpura 

2- Thalassemia 

3- Sickle cell disease 

4- Myelodysplasia 

5- Chronic myeloid leukaemia 



Answer & Comments 


Answer: 4- Myelodysplasia 


'"'There is a gradual history of progression and 
the patient has a pancytopenia, she is also in 
the right age group for myelodysplasia. 
Myelodysplasia can be classified into five 
subtypes - 

Refractory anaemia 

Refractory anaemia with ring sideroblasts 

Refractory anaemia with excess blasts 

Refractory anaemia with excess blasts in 
transformation (near AML) 

CML. 

Few patients require aggressive therapy such 
as chemotherapy, it is reserved for younger 
patients to prevent progression to AML. 
Supportive therapy includes blood 
transfusions, platelet transfusions or G-CSF to 
improve blood counts. How ever median 
survival is only 2 years. 


[ Q: 920 ] MRCPass - Haematology 

An 18 year old girl presents with 
epistaxis. She is found to have a prolonged 
APTT. Her Mother has had previous bleeding 
episodes with similar coagulation test results. 

What is the most likely diagnosis? 

1- Factor VII deficiency 

2- Factor V deficiency 

3- Protein C deficiency 

4- Von Willebrand's disease 

5- Haemophilia B 



Answer & Comments 

Answer: 4- Von Willebrand's disease 

The APTT is a general clotting screen which 
detects defects in the intrinsic clotting 
pathway (factors XII, XI, IX, and VIII, to which 
Von Willebrand factor is linked). Von 
Willebrand's disease is a predominantly 
autosomal dominant condition which is 


Dr. Khalid Yusuf El-Zohry - Sohag Teaching Hospital (01118391123) 

Ref MRCPass OE OE 2012 PasTest 2009 PassMedicine 2009 PasTest Exam ReviseMRCP 



394 

























El-zohry MRCP Questions Bank (Port 1) - 2013 


(For my personal use) 


associated with a bleeding tendency, which is 
usually mild and with a prolonged APTT. 

The tests to diagnose vWD include: 


1- Fresh frozen plasma 

2- Factor X 

3- Factor IX 


bleeding time (prolonged) 

factor VIII level test (measures the level of 
factor VIII and its ability to function) 

von Willebrand factor antigen test (the 
disorder is considered mild if a person has 
20% to 40% of the normal amount, severe if 
the amount is less than 10% of normal) 

ristocetin cofactor activity test (measures how 
well the von Willebrand factor is working) 


4- Aspirin 

5- DDAVP 


Answer & Comments 


Answer: 5- DDAVP 

Treatment may include Desmopressin 
(DDAVP), Factor VIII and tranexemic acid in 
von Willebrand's disease. 



1 W J 

[ Q: 923 ] MRCPass - Haematology 

[ Q: 921 ] MRCPass - Haematology 

A 15 year-old boy presented to clinic 
6 month history of anorexia and 

A patient presents with acute 

with a 


promyelocytic leukaemia. 

What is the likely mechanism underlying 
leukaemogenesis? 

1- Aberrant fusion of 2 genes 

2- Posttranslational modification 


malaise. On examination, he had palpable 
inguinal lymphadenopathy and splenomegaly. 

Investigations show : Hb 13.1 g/dl 

White cell count 20 x 10 9 /L 

erythrocyte sedimentation rate 15 mm/h 


3- Over expression of cellular oncogene 

4- Impaired degradation of protein 

5- Short telomere 

Answer & Comments 

Answer: 1- Aberrant fusion of 2 genes 

Acute promyelocytic leukaemia is frequently 
due to chromosomal translocation t (15; 17). 


Peripheral blood film showed: atypical 
lymphocytes, blast cells and neutropenia 

What is the most likely diagnosis? 

1- Aplastic anaemia 

2- Acute myeloid leukaemia 

3- Chronic myeloid leukaemia 

4- Paroxysmal nocturnal haemoglobinuria 

5- Acute lymphoblastic leukaemia 


[ Q: 922 ] MRCPass - Haematology 

A 15 year old male comes to the 
hematology clinic for his specialty care for von 
Willebrand Disease. The past medical history 
reveals that he was diagnosed with Type 1 von 
Willebrand Disease as a toddler after 
abnormal bruising and prolonged bleeding 
was noted by his family. 

What treatment is recommended if he were to 
have a significant episode of bleeding? 


Answer & Comments 

Answer: 5- Acute lymphoblastic leukaemia 

Blast cells on the blood film and a 
lymphocytosis would suggest acute 
lymphoblastic leukaemia. The malignant cells 
are immature lymphoid blast cells. The patient 
with acute lymphoblastic leukaemia is usually 
a child. 


Dr. Khalid Yusuf El-Zohry - Sohag Teaching Hospital (01118391123) 

Ref MRCPass OE OE 2012 PasTest 2009 PassMedicine 2009 PasTest Exam 




ReviseMRCP 

395 



























El-zohry MRCP Questions Bank (Port 1) - 2013 


(For my personal use) 



Bone Marrow biopsy showing predominant 

lymphoblasts 


[ Q: 924 ] MRCPass - Haematology 

A 25 year old woman has the 
following investigations at the antenatal clinic: 

Hb 10.3 g/dl 

WBC 5.6 x 10 9 /L 

Pits 290 x 10 9 /L 

MCV 69 fl 

MCH 17.2 

Iron 20 (14-29) pmol/l 

Ferritin 150 (15-200) ?mol/l 

What is the most useful investigation? 

1- Myeloma screen 

2- Haemoglobin electrophoresis 

3- Folate levels 

4- HbF level 

5- HbA2 level 



Answer & Comments 

Answer: 5- HbA2 level 

Iron deficiency anaemia and thalassaemia trait 
are the most likely diagnoses of microcytic 
anaemia. Iron deficiency is unlikely in this case 
in view of the iron studies being normal. Beta 
thalassaemia trait is diagnosed by the 
presence of a raised HbA2. If both conditions 
are excluded, then alpha thalassaemia is the 
most likely diagnosis. 


[ Q: 925 ] MRCPass - Haematology 

A 30 year old lady with von 
Willebrand's disease is due to have plastic 
surgery to her face and seeks advice from the 
haematologist. She mentions that she has a 
history of epistaxis and bleeding gums. 

Which of the following is the most useful 
assessment of her coagulation status? 

1- Prothrombin time 

2- Activated partial thromboplastin time 

3- Thrombin time 

4- Bleeding time 

5- Factor VIII activity assay 



Answer & Comments 

Answer: 5- Factor VIII activity assay 

Although bleeding time is prolonged in von 
Willebrand's disease, the factor VIII activity 
assay will give a measurement of the severity 
of her disease. The other useful tests would be 
the ristocetin cofactor assay and vWF antigen 
assays for von Willebrand's disease. 


[ Q: 926 ] MRCPass - Haematology 



A 20 year old patient has been found 
to have a mediastinal mass on the 
chest X ray during investigation for a 


pneumonia. 


He has a Hb of 12 g/dl, WCC 180 x 10 9 /L, 
platelets 45 x 10 9 /L. 


Blood film shows blast cells with have 
prominent nucleoli. 


There is little cytoplasm within the white cells, 
and the nucleoli are convoluted. 


Which is the likely diagnosis? 

1- Acute myeloid leukaemia 

2- Acute lymphoblastic leukaemia 

3- Multiple myeloma 

4- Hodgkin's lymphoma 


Dr. Khalid Yusuf El-Zohry - Sohag Teaching Hospital (01118391123) 

Ref MRCPass OE OE 2012 PasTest 2009 PassMedicine 2009 PasTest Exam ReviseMRCP 



396 



























El-zohry MRCP Questions Bank (Port 1) - 2013 


(For my personal use) 


5- Non Hodgkin's lymphoma 

Answer & Comments 

Answer: 2- Acute lymphoblastic leukaemia 

There is a very high white cell count which 
should make leukaemia suspicious. ALL occurs 
in childhood and young adulthood. The blood 
film described above distinguishes ALL from 
AML (contains elongated inclusions called 
Auer rods). 



Multiple lymphoblasts in ALL 


[ Q: 927 ] MRCPass - Haematology 

A 25 year old lady who was pregnant 
was treated for a deep vein thrombosis with 
intravenous heparin. A recent test shows: 

Haemoglobin 10.2 g/dL 

White Cell Count 8 x 10 9 /L 

Platelets 32 x 10 9 /L 

Whot is the best course of action this woman? 

1- Change to clexane 

2- Commence warfarin 

3- Change to aspirin 

4- Change to danaparinoid 

5- Continue iv heparin 



Answer & Comments 

Answer: 4- Change to danaparinoid 

This patient has Heparin Induced 
Thrombocytopaemia. When HIT is suspected, 
heparin treatment should be converted to 
danaparoid , which is a low molecular weight 


heparinoid. It is usually given as an 
intravenous infusion 


[ Q: 928 ] MRCPass - Haematology 

A 65 year old man complains of 
breathlessness and tiredness. He has 
confirmed pulmonary emboli. His Hb is 18 
g/dl, WCC is 15 x 10 9 /L and platelet count is 
700 x 10 9 /l. 

Which of the following can be helpful in 
confirming the diagnosis? 

1- Blood film 

2- Bone marrow biopsy 

3- Red cell mass 

4- NAP score 

5- Kleihauer test 



Answer & Comments 

Answer: 3- Red cell mass 

The diagnosis is polycythaemia rubra vera and 
this can be confirmed by a raised red cell 
mass. NAP score is decreased in CML. A 
Kleihauer test is used to confirm 
transplacental blood loss from fetus to 
mother. 


[ Q: 929 ] MRCPass - Haematology 

A 50 year old man with non 
Hodgkin's lymphoma is on Rituximab. 

Which of the following antigens does 
Rituximab have an action on? 

1- CD8 

2- CD8 

3- CD19 

4- CD20 

5- CD154 



Answer & Comments 


Answer: 4- CD20 


Dr. Khalid Yusuf El-Zohry - Sohag Teaching Hospital (01118391123) 

Ref MRCPass OE OE 2012 PasTest 2009 PassMedicine 2009 PasTest Exam 




ReviseMRCP 

397 




























El-zohry MRCP Questions Bank (Port 1) - 2013 


(For my personal use) 


Rituximab is an antibody to CD20 expressed 
on B cells and is used in B cell lymphomas (to 
try to cause cell lysis). The receptor is present 
in more than 90% of B-cell non-Hodgkin's 
lymphomas. Molecules that attach to CD20 
can affect the growth and development of the 
tumor cells. Rituximab is an antibody that was 
developed using cloning and recombinant 
DNA technology from human and murine 
genes. 

[ Q: 930 ] MRCPass - Haematology 

A 32 year old who is known to have 
ITP presents with bleeding of her gums. Her 
platelet count normally runs at 87 x 10 9 /l_ but 
now has dropped to 42 x 10 9 /L. 

What is the best management plan? 

1- Observation 

2- Steroids 

3- Platelet transfusion 

4- FFP 

5- Whole blood transfusion 


Halfway through the first unit of blood, she 
experiences generalised discomfort. 

What is the most appropriate course of 
action ? 

1- Chest X ray and abdominal x ray 

2- Blood cultures 

3- Give analgesia and continue 

4- IV steroids 

5- Stop the blood transfusion and give IV fluids 

Answer & Comments 

Answer: 5- Stop the blood transfusion and give 
IV fluids 

Acute transfusion reactions can cause 
generalized discomfort, loin pain and pain at 
the cannula site may all precede 
haemoglobinuria and renal failure. If a 
transfusion reaction is suspected, the 
transfusion should be stopped immediately 
and IV fluids should be administered, to 
prevent shock. 


Answer & Comments 

Answer: 2- Steroids 

Chronic ITP rarely resolves spontaneously. 
First line treatment is with prednisolone. 
Patients with chronic ITP who require surgery 
may be given intravenous immunoglobulins 
which produce a transient rise in platelet 
count by blocking Fc receptors on splenic 
macrophages. 

Platelet transfusion should be given only in 
life-threatening haemorrhage to enhance 
haemostasis. 



[ Q: 931 ] MRCPass - Haematology 

A 45 year old woman presents with 
an upper Gl bleed and requires a blood 
transfusion. 



[ Q: 932 ] MRCPass - Haematology 

A 55 year old man enquires about 
risks of blood transfusion. 

Which of the following infections is blood 
screened for? 

1- Varicella zoster 

2- Hepatitis B 


3- Cytomegalovirus 

4- Malaria 


5- Salmonella 


Answer & Comments 

Answer: 2- Hepatitis B 

CMV, malaria and salmonella can all be 
transmitted by blood products. 

In the UK, routine testing for donor blood is 
for: 


Dr. Khalid Yusuf El-Zohry - Sohag Teaching Hospital (01118391123) 

Ref MRCPass OE OE 2012 PasTest 2009 PassMedicine 2009 PasTest Exam ReviseMRCP 



398 




























El-zohry MRCP Questions Bank (Port 1) - 2013 


(For my personal use) 


HIV 

Hep B&C 
Syphilis 
ABO + RhD 

[ Q: 933 ] MRCPass - Haematology 

A 60 year old man presents with 
extensive bruising. He has a history of fatigue 
and dizziness for the past few months. On 
examination, he has a purpuric rash on his 
trunk and limbs. Investigations show : 

Hb 7.5 g/dl 

MCV 105 fl 

WCC 7 x 10 9 /L 

platelets 100 x 10 9 /L 

Prothrombin time 20 (12-17)s 

Fibrinogen 90 (150-460) mg/dL 

Blood film: 50% blast cells. 

What is the clinical picture consistent with? 

1- Erythroleukaemic reaction 

2- Aplastic anaemia 

3- Disseminated intravascular coagulation 

4- Haemolytic anaemia 

5- Immune thrombocytopenic purpura 



associated with DIC. In leucoerythroblastic 
picture, nucleated red cells and white cell 
precursors are found in the peripheral blood. 



[ Q: 934 ] MRCPass - Haematology 

A 60 year old woman has had a 
prolonged ITU stay due to severe pneumonia 
and sepsis requiring mechanical ventilation. 
She was noted to have worsening anaemia 
following discharge from ITU at 4 weeks. Her 
Hb is 6 g/dl, MCV 109 fl, WCC 2.2 x 10 9 /L, 
platelets 110 x 10 9 /L. 


What is the likely cause of anaemia? 


1- Upper Gl bleed 

2- Aplastic anaemia 


3- Acute myeloid Leukaemia 

4- Immune thrombocytopenic purpura 


5- Acute folate deficiency 


Answer & Comments 

Answer: 5- Acute folate deficiency 

A patient who has been in intensive care for a 
significant period may not be getting enough 
folate, especially with increased needs for 
recovery. An acute deficiency state may thus 
develop. This would precipitate a 
pancytopenia and macrocytic anaemia. 


Answer & Comments 

5 

[ Q: 935 ] MRCPass - Haematology 

Answer: 3- Disseminated intravascular 

/[ 

^ J 

A 20 year old female presents with 


coagulation 

The clinical diagnosis is likely to be acute 
myeloblastic leukaemia (AML). AML subtypes 
are distinguished from other related blood 
disorders by the presence of more than 30% 
blasts in the blood, bone marrow , or both. 
One of the common complications is DIC, 
which results in an elevated prothrombin 
time, decreased fibrinogen level and increased 
fibrin degradation products. Acute 
promyelocytic leukemia (APL), also known as 
M3, is the most common subtype of AML 


severe colicky abdominal pain, vomiting and 
constipation of 3 days duration. 

She had a previous history of admission to 
hospital with similar features. 

Her abdominal x-ray and ultrasound scan were 
normal. She was treated with antibiotics, 
analgesics and antiemetics. Her urine was 
discoloured and she had a tonic- clonic seizure 
whilst on the ward. 

What is the likely diagnosis? 

1- Variegate porphyria 


Dr. Khalid Yusuf El-Zohry - Sohag Teaching Hospital (01118391123) 

Ref MRCPass OE OE 2012 PasTest 2009 PassMedicine 2009 PasTest Exam ReviseMRCP 































El-zohry MRCP Questions Bank (Part 1) - 2013 


(For my personal use) 


2- Acute intermittent porphyria 

3- Fabry's disease 

4- Gaucher's disease 

5- Mature onset diabetes of the young 


Answer & Comments 

Answer: 2- Acute intermittent porphyria 

Acute intermittent porphyria is autosomal 
dominant disorder caused by a defect in 
porphobilinogen deaminase activity. If 
peripheral neuropathy, such as pain in the 
back and legs or parathesias occurs it is almost 
always preceded by abdominal pain. 

Other autonomic neuropathies that may be 
seen are sweating, vascular spasm, labile 
hypertension, and sinus tachycardia. Central 
nervous dysfunction can be seen as well with 
seizures, coma, bulbar paralysis, or cerebellar 
involvement. 

The defect in porphobilinogen deaminase 
causes a build up of ALA and porphobilinogen 
(PBG) which causes their increased secretion 
in the urine. 


[ Q: 936 ] MRCPass - Haematology 

A 35 year old lady has had frequent 
menorrhagia over the last few months. She 
feels well but looks pale on examination. 

Investigations reveal: 

Hb 8.6 g/dl 

MCV 70 fl 

MCHC27 (32-35) g/dl 

WCC 7 x 10 9 /L 

platelets 225 x 10 9 /L 

Iron 9 (14-29) pmol/l 

Ferritin 12 (15-200) ?mol/l 

total iron binding capacity 95 (45-72) umol/l 

What feature is likely to be found on the blood 
film? 



1- Fragmented cell 

2- Helmet cell 

3- Polychromasia 

4- Spherocytes 

5- Target cells 


Answer & Comments 

Answer: 5- Target cells 

This lady has iron deficiency anemia. Common 
blood film features are pencil cells, target cells 
and pokilocytosis. 

Causes of target cells are liver disease, post 
splenectomy, iron deficiency and thalassemia. 



Target Cells 



[ Q: 937 ] MRCPass - Haematology 

A 50 year old man has a history of 
epistaxis. He is also generally very tired. On 
examination, he has no lymphoadenopathy or 
splenomegaly. 


His blood tests reveal: 


Hb 7.6 g/dl 

MCHC 32 g/dl (32-35) 
Neutrophils 1.5 x 10 9 /L 
urea 8 pmol/l 
sodium 143 mmol/l 
bilirubin 25 pmol/l 
ALP 150 U/l 
LDH 120 U/l (10-250) 


MCV 90 fl 
WCC 3 x 10 9 /L 
platelets 29 x 10 9 /L 
creatinine 125 pmol/l 
potassium 3.6 mmol/l 
AST 18 U/l 
albumin 32 g/l 


Which of the following investigations would be 
most helpful? 

1- CT of abdomen 


Dr. Khalid Yusuf El-Zohry - Sohag Teaching Hospital (01118391123) 

Ref MRCPass OE OE 2012 PasTest 2009 PassMedicine 2009 PasTest Exam 



i 

ReviseMRCP 


400 


l 



























El-zohry MRCP Questions Bank (Port 1) - 2013 


(For my personal use) 


2- Genetic studies 

3- PET scan 

4- Bone marrow aspiration and cytology 

5- Bone marrow biopsy 


Answer & Comments 

Answer: 5- Bone marrow biopsy 

On the blood tests, there is aplastic anaemia 
which chould be due to infiltration by a 
tumour, autoimmune or drug induced. 
Although both bone marrow aspiration is 
useful, a biopsy is required to assess cellularity 
and to exclude abnormal infiltration due to a 
mlaignant process. In aplastic anaemia, 
erythropoietic cells, megakaryocytes and 
granulocytic cells are reduced. 

Bone marrow aspiration involves aspirating 
fluid contents of marrow w hilst bone marrow 
biopsy involves more force to obtaine bone 
morrow core. See pictures: 



Aplastic Anaemia - Bone Marrow Biopsy 


5- Iron infusion 


Answer & Comments 

Answer: 4- Packed red cells 

The patient who is anaemic and bleeding 
needs a blood transfusion with packed red 
cells (blood). This also contains some white 
cells, platelets & a small amount of plasma 
plus 60 -100 ml of additive. 



[ Q: 939 ] MRCPass - Haematology 

A 75 year old woman with chronic 
myeloid leukemia (CML), treated with 
hydroxyurea and interferon for 12 years 
suffered from gradual disease progression for 
one year. 


Investigations show : 

Haemoglobin 11.6 g /dL 

white cell count 47 x 10 9 /L (neutrophils, 80%; 
lymphocytes, 13%; metamyelocytes, 6%; 
blasts, 1%) 

platelet count 1220 x 10 9 /L 

Whot should the patient be treated with? 


1- Cyclophosphamide 

2- Prednisolone 


3- Radiotherapy 

4- Desferrioxamine 


5- Imatinib 


^ [ Q: 938 ] MRCPass - Haematology 

* A 45 year old woman presents with 
haemetemesis. She has a haemoglobin (Hb) of 
4.5 g/dL and platelet count of 350 x 10 9 /L. 

Which of the following is the most appropriate 
product or drug to use? 


Answer & Comments 

Answer: 5- Imatinib 

Gleevec (imatinib mesylate, Novartis), is an 
oral drug which interferes with the action of 
the abnormal Bcr-Abl tyrosine kinase in CML 
white blood cells. 


1- Fresh frozen plasma (FFP) 

2- Iv methylprednisolone 

3- Plasma protein fraction 


Before Gleevec, the most common drugs used 
to treat CML were the oral treatments 
hydroxyurea and busulphan. 


4- Packed red cells 


Dr. Khalid Yusuf El-Zohry - Sohag Teaching Hospital (01118391123) 

Ref MRCPass OE OE 2012 PasTest 2009 PassMedicine 2009 PasTest Exam 




ReviseMRCP 

401 


























El-zohry MRCP Questions Bank (Port 1) - 2013 


(For my personal use) 


An intravenous treatment, cytarabine, is 
sometimes used in combination with immune 
therapy (interferon). Bone marrow or stem 
cell transplantation tends to be limited to 
younger patients. 


[ Q: 940 ] MRCPass - Haematology 

A 30 year old woman is being 
investigated for right upper quadrant pains. 
She reports occasional episodes of jaundice, 
especially associated with infections. Her 
mother, sister and aunt has previously had 
gallstones. On examination, she has 
splenomegaly. Her FBC is normal, but her 
bilirubin is mildly elevated. 

What is the likely cause for her probable 
gallstones? 

1- Sickle cell disease 

2- Hereditary spherocytosis 

3- Beta thalassemia 

4- Gilbert's syndrome 

5- Paroxysmal nocturnal haemoglobinuria 



Answer & Comments 

Answer: 2- Hereditary spherocytosis 

Hereditary spherocytosis is inherited in an 
autosomal dominant fashion. It is 
characterised by increased red cell fragility. 
There is increased haemolysis during 
infections. Gallstones are commonly 
associated. Management is usually supportive, 
although some cases require splenectomy to 
reduce transfusion requirements. 



Spherical red cells in hereditary spherocytosis 


[ Q: 941 ] MRCPass - Haematology 

A 30 year old patient has significant 
Gl bleeding, but is concerned about the risks 
of blood transfusion. 

Which of the following is screened for in 
donated blood? 

1- JC virus 

2- Human T cell leukaemia virus 

3- HIV-1 

4- New variant CJD 

5- Toxoplasmosis 



Answer & Comments 

Answer: 3- HIV-1 

In the UK every blood donation is tested for 
evidence of hepatitis B, hepatitis C, HIV-1, HIV- 
2 and syphilis. How ever, although there are 
recent concerns regarding tranmission of new 
variant CJD, there are no reliable screening 
methods yet. 


^ [ Q: 942 ] MRCPass - Haematology 

# A 35 year old woman with a 

haematological condition has been transfused 
with group specific platelets on several 
occasions. Her platelet count drops quickly 5 
days following platelet transfusion. 

What should she be treated with? 

1- Fresh frozen plasma 


Dr. Khalid Yusuf El-Zohry - Sohag Teaching Hospital (01118391123) 

Ref MRCPass OE OE 2012 PasTest 2009 PassMedicine 2009 PasTest Exam 




ReviseMRCP 


402 


l 



























El-zohry MRCP Questions Bank (Port 1) - 2013 


(For my personal use) 


2- Cryoprecipitate 

3- Packed red cells 

4- Intravenous immunoglobulin 

5- Factor VIII 


Answer & Comments 

Answer: 4- Intravenous immunoglobulin 

Post transfusion purpura is a transfusion 
reaction occurring occurs 5 - 14 days after 
transfusion of platelets or fresh frozen plasma. 
This occurs when individuals lacking the PLA-1 
antigen are transfused with blood containing 
PLA-1 positive platelets. It is uncommon as 
only 2-3% of the population are PLA-1 
negative. 

Treatment of choice is intravenous 
immunoglobulin or plasma exchange. Further 
platelet transfusions should be w ashed or be 
HPA-1A negative. 


[ Q: 943 ] MRCPass - Haematology 

A 70 year old female presents with 
left upper quadrant pain and multiple 
ecchymoses (bruising). 

Investigations: 

hemoglobin 9.5 g/dL 

platelet count 30 x 10 9 /L 

white cell count of 8.2 x 10 9 /L 

The bone marrow biopsy is hypercellular 
(75%) and shows diffuse infiltration by large 
atypical cells with abundant cytoplasm. 

What is the diagnosis? 

1- Acute myeloid leukaemia 

2- Chronic lymphocytic leukaemia 

3- Hodgkin's lymphoma 

4- Waldenstrom's macroglobulinaemia 

5- Polycythaemia rubra vera 



Answer & Comments 

Answer: 2- Chronic lymphocytic leukaemia 

Chronic lymphocytic leukaemia arises from 
the neoplastic proliferation of relatively 
mature lymphocytes which infiltrate the 
blood, bone marrow or lymphoreticular 
structures. Most are clonal malignancies of B 
lymphocytes rather than T lymphocytes. Bone 
marrow aspiration / biopsy - typically there is 
infiltration by lymphocytes. 



Diffuse infiltration by large atypical cells in CLL 

[ Q: 944 ] MRCPass - Haematology 

A 35 year old man who works in a 
factory has accidentally drunk a large amount 
of dye. He is brought to the hospital looking a 
blue colour. 

His Hb is 16 g/dl. Blood gases show a pH of 
7.37, p0 2 of 13 kPa, pC0 2 of 4.5 kPa and 0 2 
sats of 80% on the sats monitor. 

Which is the diagnosis? 

1- Methaemoglobinaemia 

2- Down's syndrome 

3- Fallot's tetralogy 

4- Carbon monoxide poisoning 

5- Polycythaemia rubra vera 



Answer & Comments 

Answer: 1- Methaemoglobinaemia 

Methaemoglobinaemia is the most likely 
diagnosis due to ingestion of aniline dye in this 
case. The p02 is often normal but the oxygen 
saturations are reduced in 

methaemoglobinaemia. 


Dr. Khalid Yusuf El-Zohry - Sohag Teaching Hospital (01118391123) 

Ref MRCPass OE OE 2012 PasTest 2009 PassMedicine 2009 PasTest Exam 




ReviseMRCP 

403 



























El-zohry MRCP Questions Bank (Part 1) - 2013 


(For my personal use) 


^ [ Q: 945 ] MRCPass - Haematology 

fh - 

* A 40 year old woman has a long 
history of anaemia, and is not compliant with 
medications. 

Investigations show : 

Haemoglobin 7.8 g/dL (11.3-16.5) 

MCV 85 fl (80-96) 

MCH 26 pg (28-32) 

WCC 7 x 10 9 /L 

Platelets 160 x 10 9 /L 

Serum B12 130 ug/L (160-760) 

Red cell folate 95 ug/L (160-640) 

Serum ferritin 11 ug/L (15-300) 

Which of the following antibodies is likely to 
be present with the condition? 

1- Antigliadin antibody 

2- Anti intrinsic factor antibody 


Hct 0.612 

Platelet count 468 x 10 9 /L 
ESR 1 mm/lst hour 
coagulation screen normal 
What is the diagnosis? 

1- Essential thrombocythaemia 

2- Chronic myeloid leukaemia 

3- Polycythaemia rubra vera 

4- Idiopathic thrombocytopenic purpura 

5- Thalassemia 

Answer & Comments 

Answer: 3- Polycythaemia rubra vera 
The criteria for PRV are: 

1) increased red cell mass 

2) splenomegaly 


3- Anti parietal cell antibody 

4- Anti nuclear antibody 

5- Anti phospholipid antibody 

Answer & Comments 

Answer: 1- Antigliadin antibody 

There are mixed iron and folate deficiency due 
to coeliac disease. This causes a normal MCV 
(dimorphic picture because of both micro and 
macrocytic features). Antibodies which are 
present in coeliac disease are : antiendomysial 
and antigliadin. vitamin Bn concentrations 
normalize on a gluten-free diet alone, but 
symptomatic patients may require 
supplementation. 


3) increased platelets, leucocytes, INCREASED 
NAP score and B12 (increased B12 binding 
protein release). 

Gout occurs due to increased cell turnover, 
cerebral and myocardial ischaemia occurs due 
to fall in perfusion and raised blood viscosity. 

The NAP score is a semiquantitative 
cytochemical assessment of alkaline 
phosphatase in neutrophils. The NAP score is 
based on staining intensity, with a possible 
score of 0-400. It differentiates chronic 
myeloid leukaemia (low) from reactive 
leucocytosis (high), eg bacterial infection. It 
may assist in the differentiation of 
polycythaemia rubra vera (high) from other 
causes of erythrocytosis (normal). 


[ Q: 946 ] MRCPass - Haematology 

A 60 year old lady is investigated for 
recurrent episodes of gout. On examination, 
she looked plethoric and has a 6 cm 
splenomegaly. She has the following results: 

Hb 18.9 gm/dl 



[ Q: 947 ] MRCPass - Haematology 

A 25 year old woman presented to 
the hematology clinic with a 5 year history of 
pallor and anemia. On physical examination, 
the patient was found to have mild 
splenomegaly. 



Dr. Khalid Yusuf El-Zohry - Sohag Teaching Hospital (01118391123) 

Ref MRCPass OE OE 2012 PasTest 2009 PassMedicine 2009 PasTest Exam ReviseMRCP 



























El-zohry MRCP Questions Bank (Port 1) - 2013 


(For my personal use) 


Results show: 

Hemoglobin 7.9 g/dL 
platelet count of 226 x 10 9 /L 
MCV 68 fl 

MCH was 20 pg (24.0-31.0 pg) 

MCHC 30.1 g/dL (32.0-36.0 g/dL) 

serum iron 31.6 ?mol/L (9.0-26.9) 

ferritin 380 ng/mL (22-400 ng/mL) 

transferrin 161 mg/dL (185-370 mg/dL) 

Bone marrow aspirate demonstrated 
significant hypercellularity associated with 
marked erythroid hyperplasia. 

Stainable iron stores were increased. A 
striking feature was the presence of numerous 
blasts in which the perinuclear iron granules 
encircled more than one third of the nuclear 
circumference. 

What is the diagnosis? 

1- Haemochromatosis 

2- Sideroblastic anaemia 

3- Multiple myeloma 

4- Waldenstrom's macroglobulinaemia 

5- Thalassemia 


Answer & Comments 

Answer: 2- Sideroblastic anaemia 

Sideroblasts are abnormal red cell precursors 
with iron loaded mitochondria, forming a ring 
around the nucleus. Sideroblastic anaemia is 
associated with alcohol, lead, drugs and 
myelodysplasia. There is a defect in haem 
synthesis, thus excess loading of iron to 
compensate in red cell precursors and into 
iron stores, sometimes causing 
haemosiderosis in the liver, and 
desferrioxamine therapy may help. 

Ringed sideroblasts are precursors and hence 
are found in the bone marrow . Anti 
tuberculous drugs interfere with haem 
metabolism by interefering with pyridoxine 


availability. Some cases respond to pyridoxine 
therapy (not panthotenic acid). 



r 


[ Q: 948 ] MRCPass - Haematology 

fft 

m A 15 year old Filipino girl is noted to 
have a hemoglobin of 10.6 g/dl with an MCV 
of 65 fl on routine testing. 


She reports regular menses lasting 4-5 days 
each cycle. She has no specific complaints. She 
is unaw are of a family history of anemia. On 
examination, there is no hepatosplenomegaly, 
jaundice, or scleral jaundice. 


What is the likely diagnosis? 

1- Iron deficiency 


2- Lead poisoning 

3- Thalassemia 


4- Sickle cell anaemia 


5- Acute lymphoblastic leukaemia 


Answer & Comments 

Answer: 3- Thalassemia 

This patient is likely to have thalassemia trait 
(probably alpha thalassemia). Those with 
alpha thalassemia trait are clinically normal, 
but their hemoglobin is slightly low and their 
hemogram demonstrates microcytic indices. 


[ Q: 949 ] MRCPass - Haematology 

A 50 year old woman has a life-long 
history of anemia. Her mother has a similar 
history. Laboratory values show 

Haemoglobin 10.5 g/dl 

Haematocrit 33 



Dr. Khalid Yusuf El-Zohry - Sohag Teaching Hospital (01118391123) 

Ref MRCPass OE OE 2012 PasTest 2009 PassMedicine 2009 PasTest Exam 




ReviseMRCP 

405 
























El-zohry MRCP Questions Bank (Port 1) - 2013 


(For my personal use) 


MCV 66 fl 

Blood film: microcytic, normochromic red 
cells. A fewelliptocytes and target cells are 
noted. 

Which of the following is the most likely 
condition? 

1- Iron deficiency anaemia 

2- Hereditary spherocytosis 

3- Thalassemia 

4- G6PD deficiency 

5- Autoimmune haemolytic anaemia 


Answer & Comments 


Answer: 3- Thalassemia 


This patient has either thalassemia trait or 
beta thalassemia minor. Mutations in globin 
genes cause thalassemias. Alpha thalassemia 
affects the alpha-globin gene(s). Beta 
thalassemia affects one or both of the beta- 
globin genes. In beta thalassemia major (ie, 
homozygous beta thalassemia), the 
production of beta-globin chains is severely 
impaired, because both beta-globin genes are 
mutated. In beta thalassemia minor, one of 
the beta-globin chains is impaired. 


The severe anemia resulting from this disease, 
if untreated, can result in high-output cardiac 
failure, which causes the highest mortality. 



Blood film showing target cells (low er left and 
upper right Conner as well as elliptocyte in the 

lower left) 



[ Q: 950 ] MRCPass - Haematology 
A 48 year old male was referred for 


investigation of anemia. 

Reults show : 

hemoglobin of 7.2 g/dL 

a white blood cell count of 3 x 10 9 /L 

platelet count of 60 x 10 9 /l_ 

On physical exam, his spleen was enlarged. 

Peripheral blood showed teardrops and a 
leukoerythroblastic smear. Bone marrow 
biopsy showed increased numbers of 
megakaryocytes and grade III fibrosis. 

What is the diagnosis? 

1- Folate deficiency anaemia 

2- Coomb's positive haemolytic anaemia 

3- Myelofibrosis 

4- Multiple myeloma 

5- Chronic myeloid leukaemia 


Answer & Comments 

Answer: 3- Myelofibrosis 

In myelofibrosis, splenomegaly occurs with a 
fibrotic process. Haemolytic anaemia is not 
typically steroid responsive. 

Leucoerythroblastic anaemia (red cell and 
white cell precursors) are seen on the blood 
film. Bone pain, bleeding (platelet 
dysfunction) may occur but are not 
characteristic. Fibrous tissue infiltration of the 
bone makes it difficult to aspirate bone 
marrow . 



Marrow biopsy demonstrating sigificant 

fibrosis 


Dr. Kholid Yusuf El-Zohry - Sohog Teaching Hospital (01118391123) 

Ref MRCPass OE OE 2012 PasTest 2009 PassMedicine 2009 PasTest Exam 




ReviseMRCP 


406 


l 
























El-zohry MRCP Questions Bank (Port 1) - 2013 


(For my personal use) 


[ Q: 951 ] MRCPass - Haematology 

A 16 yearold boy with sickle cell 
disease complains of acute breathlessness. He 
has a respiratory rate of 35 breaths per 
minute. O 2 saturations are 75% on room air 
and 85% on 100% oxygen. There is reduced air 
entry bilaterally, but no added sounds. 
Investigations show : 

p0 2 6.2 kPa 

CXR: bilateral basal infiltrates 



[ Q: 952 ] MRCPass - Haematology 

A 35 year old patient has 
schizophrenia. He is on clozapine, temazepam 
and amoxycillin for a recent chest infection. 

He is admitted unwell the following blood 
results: 

Hb 2.0 g/dl, WCC 2 x 10 9 /L, Neutrophils 0.3 x 
10 9 /L, platelets 180 x 10 9 /L, urea 6 mmol/l, 
creatinine 80 mmol/l, sodium 140 mmol/l, 
potassium 3.8 mmol/l. 



Hb 7.5 g/dl 

WCC 14 x 10 9 /L 

platelets 200 x 10 9 /L 

The most appropriate management is: 

1- IV antibiotics 

2- Non invasive ventilation 

3- IV fluids 

4- Blood transfusion 

5- Urgent exchange transfusion 


Which of these is likely to be the cause of 
neutropenia? 

1- Clozapine 

2- Amoxycillin 

3- Myelofibrosis 

4- Myeloma 

5- Sepsis 


Answer & Comments 


Answer: 1- Clozapine 


Answer & Comments 

Answer: 5- Urgent exchange transfusion 

The diagnosis is acute chest syndrome. The 
acute chest syndrome (ACS) in sickle cell 
disease (SCD) can be defined as: 

1. a new infiltrate on chest x-ray 

2. associated with one or more NEW 
symptoms: fever, cough, sputum production, 
dyspnea, or hypoxia. 

Exchange blood transfusions are indicated in 
cases of cerebrovascular accidents and acute 
chest syndrome. 

They are performed occasionally in patients 
with acute sequestration crisis or in cases of 
priapism that do not resolve after adequate 
hydration and analgesia . Exchange 
transfusion consists of replacing the patient's 
RBCs by normal donor RBCs, decreasing HbS 
to less than 30%. 


Clozapine is associated with neutropenia and 
agranulocytosis are main haematological 
complications. Severe neutropenia can be 
treated with G-CSF. 


[ Q: 953 ] MRCPass - Haematology 

A 30 year old male presents with 
painless cervical and axillary 
lymphadenopathy. He also complains of fever 
and pruritus. Lymph node biopsy 
demonstrates Reed-Sternberg cells. 

Which feature would indicate the worst 
prognosis? 

1- Sweating 

2- Inguinal lymphadenopathy 

3- Bone marrow involvement 

4- Pruritus 

5- Fever 



Dr. Khalid Yusuf El-Zohry - Sohag Teaching Hospital (01118391123) 

Ref MRCPass OE OE 2012 PasTest 2009 PassMedicine 2009 PasTest Exam 




ReviseMRCP 

407 

























El-zohry MRCP Questions Bank (Port 1) - 2013 


(For my personal use) 


Answer & Comments 

Answer: 3- Bone marrow involvement 

The patient has Hodgkin's disease. 
Involvement of the bone marrow would 
classify the patient as stage IV (modified Ann 
Arbor classification) indicating poor prognosis. 

* Stage I is involvement of a single lymph node 
region (I) or single extralymphatic site (le) 

* Stage II is involvement of two or more lymph 
node regions on the same side of the 
diaphragm (II) or of one lymph node region 
and a contiguous extralymphatic site (lie) 

* Stage III is involvement of lymph node 
regions on both sides of the diaphragm, which 
may include the spleen (Ills) and/or limited 
contiguous extralymphatic organ or site (llle, 
llles) 

* Stage IV is disseminated involvement of one 
or more extralymphatic organs 



Reed Sternberg cell showing prominent 

nucleoli 


[ Q: 954 ] MRCPass - Haematology 

A 25 year old lady has a mother who 
has had a splenectomy for anaemia. She 
presents unwell with abdominal pain and 
vomiting. Examination reveals a tender right 
upper quadrant and jaundice, as well as 4 cm 
splenomegaly. 

Her bloods show : 

Hb 9.0 g/dl MCV 95 fl 

MCHC 33 g/dl (32-35) WCC 11 x 10 9 /L 

platelets 200 x 10 9 /L 



10 9 /L (50-100) 


reticulocytes 180 x 
urea 5.5 pmol/l 
sodium 137 mmol/l 
bilirubin 48 pmol/l 
ALP 450 U/l 
LDH 650 U/l (10-250) 


creatinine 65 pmol/l 
potassium 4.2 mmol/l 
AST 60 U/l 
albumin 38 g/l 


Which one of the following is the likely 
diagnosis? 

1- Autoimmune haemolytic anaemia 

2- Pancreatitis 

3- Hereditary spherocytosis 

4- Sickle cell disease 

5- Acute myeloid leukaemia 


Answer & Comments 

Answer: 3- Hereditary spherocytosis 

Hereditary spherocytosis is most likely due to 
the family history and presentation. The blood 
tests show likely haemolysis. The history also 
suggests cholecystitis due to gallstones. 


Gallstones occur in patients with recurrent 
haemolysis (pigmented stones). 



Spherocytes are small, round erythrocytes 
that lack central pallor (arrow s) 


[ Q: 955 ] MRCPass - Haematology 

A 22 year old woman presents with 
an acute pulmonary embolism in the 8th week 
of pregnancy. 



Dr. Kholid Yusuf El-Zohry - Sohog Teaching Hospital (01118391123) 

Ref MRCPass OE OE 2012 PasTest 2009 PassMedicine 2009 PasTest Exam ReviseMRCP 



408 


























El-zohry MRCP Questions Bank (Port 1) - 2013 


(For my personal use) 


What is the most appropriate treatment for 
this patient throughout her pregnancy? 

1- Clopidogrel 

2- Intravenous heparin 

3- Subcutaneous low molecular weight 
heparin 

4- Dipyridamole 

5- Warfarin 


Answer & Comments 

Answer: 3- Subcutaneous low molecular 
weight heparin 



[ Q: 957 ] MRCPass - Haematology 

A 80 year old woman presents with 
generalised abdominal pains. 


investigations show : 


Hb 10.3g/dl WBC 17 x 10 9 /L 

pits 80 x 10 9 /L MCV 85 fl 


Blood film shows nucleated red cells, small 
numbers of promyelocytes, myelocytes and 
metamyelocytes 


What is the most likely cause of these 
haematology results? 


1- Folate deficiency 


Warfarin should only be used in the third 
trimester. It is teratogenic and use in the first 
2 trimesters are not recommended. At 
present, the patient should have LMWH. 

^ [ Q: 956 ] MRCPass - Haematology 

#1 - 

* A 45 year old woman has von 

Willebrand's disease. She is going to undergo 
tooth extraction. 

Which one of the following is the best 
management option in mild von Willebrand's 
disease prior to surgery? 

1- DDAVP 

2- Factor VIII concentrate 


2- Sickle cell disease 

3- Osteoporosis 

4- Thalassaemia minor 

5- Myelofibrosis 

Answer & Comments 

Answer: 5- Myelofibrosis 

Blood film findings describe a 
leucoerythroblastic blood picture which is 
characterized by granulocyte and erythroid 
precursors in the peripheral blood. Common 
causes include : 

myelofibrosis 


3- Fresh frozen plasma 

4- Factor IX concentrate 

5- Blood transfusion 


bone marrow infiltration with leukaemia 
severe megaloblastic anaemia 
sickle cell crisis 


Answer & Comments 


Answer: 1- DDAVP 


thalassaemia major 
osteopetrosis 


Out of all the choices, DDAVP is the most 
pragmatic option. Fresh frozen plasma or von 
Willebrand factor can be used in cases of 
severe bleeding but should not be used in mild 
cases. 


Dr. Khalid Yusuf El-Zohry - Sohag Teaching Hospital (01118391123) 

Ref MRCPass OE OE 2012 PasTest 2009 PassMedicine 2009 PasTest Exam ReviseMRCP 




























El-zohry MRCP Questions Bank (Port 1) - 2013 


(For my personal use) 


°*> 

ft© o 

0 

0 

O 

<4©< 

lOnOOO 

O rt <?0 

* <sy 

ioe- 

ko°'° 

O °o°i 



o Oo 

3 O ( 

o#in 

9 o 



A leucoerythroblastic picture - presence of 
immature myeloid and nucleated red cells in 
the peripheral blood 


[ Q: 958 ] MRCPass - Haematology 

A 9 year old boy is unwell having 
ingested a bottle of dye. On examination, he is 
afebrile but has tachypnea, cyanosis, and 
drow siness. He is given 100% oxygen but does 
not improve. 

Whot is the most likely diagnosis? 

1- Methaemoglobinaemia 

2- Sickle cell anaemia 

3- Thalassemia 

4- Congenital cyanotic heart disease 

5- Henoch Schonlein purpura 



Answer & Comments 

Answer: 1- Methaemoglobinaemia 

Cyanosis that is unresponsive to oxygen 
therapy is most likely due to 
methemoglobinemia. 

Methaemoglobinaemia is a cause of cyanosis 
because it causes the formation of reduced Hb 
>1.5 g/dl. It is due to oxidised iron from Fe2+ 
to Fe3+ in Hb and may cause precipitation as 
Heinz bodies. 

Chemicals which are oxidising agents may 
cause this e.g. aniline dyes, chlorates, nitrates, 
nitrophenols, primaquine and sulphonamides. 
Treatment is with methylene blue if 
methaemoglobin >3.0g/dL. 


[ Q: 959 ] MRCPass - Haematology 

An 75 year old woman is admitted 
for routine surgery. She is found to have a 
haemoglobin of 7.8 g/dL with hypochromic, 
microcytic indices and the blood film shows 
pencil cells. 

Whot is the most appropriate management? 

1- Transfuse blood 

2- Continue with surgery 

3- Send haematinics and treat with ferrous 
sulphate 

4- Send haematinics and treat with Vitamin Bn 

5- Request a bone marrow examination to 
exclude myelodysplasia 

Answer & Comments 

Answer: 3- Send haematinics and treat with 
ferrous sulphate 

Iron deficiency is a common cause of anaemia, 
especially in the elderly. In this age group, it is 
often due to poor dietary intake, although 
OGD or colonoscopy may need to be done to 
exclude Gl bleeding as a cause of blood loss. 

^ [ Q: 960 ] MRCPass - Haematology 

# A 12 year old boy presents with 
breathlessness and cough. On examination he 
is pale and jaundiced. His Hb is 5.5 g/dl and 
peripheral smear shows 50% sickled cells with 
<1% reticulocytes. 

Which of the following is responsible for his 
condition? 

1- Salmonella Infection 

2- Pneumococcal infection 

3- H. Influenzae infection 

4- Cytomegalovirus infection 

5- Parvovirus infection 



Answer & Comments 


Answer: 5- Parvovirus infection 


Dr. Khalid Yusuf El-Zohry - Sohag Teaching Hospital (01118391123) 

Ref MRCPass OE OE 2012 PasTest 2009 PassMedicine 2009 PasTest Exam ReviseMRCP 



410 




























El-zohry MRCP Questions Bank (Port 1) - 2013 


(For my personal use) 


The child has sickle cell anaemia with an 
aplastic crisis. This is most commonly 
precipitated by parvovirus B19. Salmonella 
can cause osteomyelitis and H influenzae can 
cause pneumonia in patients with sickle cell 
disease. 


[ Q: 961 ] MRCPass - Haematology 

A 48 year old man being has 
polyuria, polydipsia and impotence. 

On examination, he has a palpable enlarged 
liver. 



white cell count 3.5 x 10 9 /L (4-11) 
platelet count 275 x 10 9 /L (150 - 400) 
total protein 85 g/L (61 - 76) 
albumin 32 g/L (37 - 49) 
urea 18 mmol/l (3-7) 
creatinine 350 micromol/L (60 -110) 
calcium 2.85 mmol/l (2.25-2.7) 
plasma glucose 5.5 (3.0 - 6.0) 
urine dipstick analysis protein + blood + 
renal ultrasound normal sized kidneys 


Investigations show: 

Alanine aminotransferase 80 U/L (5-35) 

Aspartate aminotransferase 92 U/L (1-31) 

Albumin 36 g/l Fasting plasma glucose 7.4 
(3.0-6.0) 

Ferritin 800 ug/L (15-300) 


Which one of the following investigations is 
appropriate? 

1- Liver ultrasound 

2- Thrombophilia screen 

3- Serum electrophoresis 

4- HAM test 


Which one of following is the next best 
investigation? 

1- Oral glucose tolerance test 

2- Serum transferrin receptors 

3- Liver biopsy 

4- Transferrin saturation 

5- Bone marrow biopsy 


Answer & Comments 

Answer: 4- Transferrin saturation 

The diagnosis is haemochromatosis suggested 
by high ferritin. The best test now is 
transferrin saturation. If this is high, HFE gene 
analysis should also be performed. 


[ Q: 962 ] MRCPass - Haematology 

A 65 year old woman presents with a 
6 month history of back pain. 

She also mentions polyuria and lethargy. 

Investigations reveal: 

haemoglobin 9.2 g/dL (11.5 -16.5) 



5- Thrombin time 


Answer & Comments 

Answer: 3- Serum electrophoresis 

Anaemia, bone pain and hypercalcaemia are 
typical presenting features of multiple 
myeloma. Serum electrophoresis will show a 
monoclonal band of either IgG, IgM or IgA 
variety. Bence Jones protein may also be 
found in the urine. About 15% of patients 
have BJ protein in their urine without a 
paraproteinaemia. 



Plasma cells in multiple myeloma 


Dr. Khalid Yusuf El-Zohry - Sohag Teaching Hospital (01118391123) 

Ref MRCPass OE OE 2012 PasTest 2009 PassMedicine 2009 PasTest Exam 




ReviseMRCP 

411 






















El-zohry MRCP Questions Bank (Port 1) - 2013 


(For my personal use) 


[ Q: 963 ] MRCPass - Haematology 

A 13 year old female with known 
sickle cell anaemia has been unwell with 
fevers, anorexia and severe pains in the 
muscles and joints of her upper and low er 
limbs. 

Her blood tests show : 

Hb 4 g/dl 
WCC 3.9 x 10 9 /L 
Pit 75 x 10 9 /L 

Absolute reticulocyte count 0.3(0.5-1.5) 

LDH 280 (85-285) IU/L 

Bilirubin 15 (1-22) pmol/l 

Blood film reveals sickle cells with absence of 
polychromasia. 

What is the likely scenario? 

1- Aplastic sickle crisis due to parvovirus 

2- Acute haemolysis 

3- Splenic sequestration 

4- Sickle chest syndrome 

5- Folate deficiency 



Answer & Comments 

Answer: 1- Aplastic sickle crisis due to 
parvovirus 

The most likely cause is aplastic crisis, the 
commonest cause being parvovirus. The 
reticulocyte count is low , bilirubin is normal 
and LDH is normal. 

Polychromasia is the presence of grey 
coloured red cells on film, indicating presence 
of increased reticulocytes. 

The lack of polychromasia on the blood film 
suggests aplastic anaemia. 





Sickle Cell anaemia Aplastic Crisis. Numerous 
sickled RBC's are present (small arrows)and A 
single nucleated RBC is noted (large arrow ). 
Note the absence of polychromasia. 



[ Q: 964 ] MRCPass - Haematology 

A 70 year old man is admitted for 
investigation of jaundice and anorexia for 
several weeks. 6 weeks ago, he had been 
prescribed a two week course of Augmentin 
by his GP for a severe upper respiratory tract 
infection and was also taking Ibuprofen for 
gout. He lives alone and has not had recent 
travel. He drinks 2 units of alcohol a week. 


Investigations reveal: 

Albumin 40 g/L 
Bilirubin 255 umol/L (1-22) 

AST 260 iu/L (5-35) 

Alkaline Phosphatase 220 iu/l (50-110) 

Abdominal ultrasound reveals gallsones but 
no evidence of cholecystitis. 

What is the most likely cause of the jaundice? 

1- Allopurinol 

2- Augmentin 

3- Alcoholic liver disease 


4- Viral hepatitis 

5- Ibuprofen 


Answer & Comments 


Answer: 2- Augmentin 


Dr. Khalid Yusuf El-Zohry - Sohag Teaching Hospital (01118391123) 

Ref MRCPass OE OE 2012 PasTest 2009 PassMedicine 2009 PasTest Exam ReviseMRCP 



412 























El-zohry MRCP Questions Bank (Port 1) - 2013 


(For my personal use) 


The blood results are consistent with 
cholestatic jaundice. Co-amoxiclav 
(augmentin) is a well known cause of this. 
There is often a latency time betw een the use 
of drug and onset of cholestatic jaundice. 
Studies of case reports showed that the onset 
of jaundice is typically from 2 weeks to 6 
weeks for penicillins. 

Some drugs cause a reaction even later - 
several months. How ever, one should 
scrutinise the data in your exam question 
because the structure may be different. 

[ Q: 965 ] MRCPass - Haematology 

A 27 year old woman has routine 
blood tests during her pregnancy. 

Hb 9.8 g/dl 

WBC 5.4 x 10 9 /L 

Plts 260 x 10 9 /L 

MCV 69 fl 

MCH 17.2 

What is the most likely diagnosis? 

1- Folate deficiency 

2- Beta thalassemia trait 

3- B12 deficiency 

4- Sideroblastic anaemia 

5- Anaemia of chronic disease 



Answer & Comments 

Answer: 2- Beta thalassemia trait 

Iron deficiency anaemia and thalassemia trait 
are the two most likely diagnoses of 
microcytic anaemia. Iron deficiency is best 
diagnosed by a low ferritin level. Beta 
thalassemia trait is diagnosed by the presence 
of a raised HbA2 (with Hb electrophoresis). 


^ [ Q: 966 ] MRCPass - Haematology 

A 42 year old woman with a long 
history of drinking alcohol has epilepsy. She 


has been on phenytoin and carbamazepine 
since the diagnosis was made 5 years ago. 
Investigations reveal: 

Haemoglobin 9.5 g/dL (13-16) 

MCV 118 fL (80-96) 

white cell count 2.5 x 10 9 /l_ (4-11) 

platelet count 72 x 10 9 /l_ (150-400) 

What is the most likely explanation for these 
results? 

1- Myelodysplasia 

2- Aplastic anaemia 

3- Folic acid deficiency 

4- Side effect of carbamazepine 

5- Chronic lymphocytic leukaemia 


Answer & Comments 

Answer: 3- Folic acid deficiency 

Folic acid deficiency would fit the clinical 
description and is a known adverse effect of 
long term phenytoin therapy. 



[ Q: 967 ] MRCPass - Haematology 

A 40 year old lady who has been on 
w arfarin for deep venous thrombosis 
presents with upper Gl bleeding. Her INR was 
9.1. 


What is the appropriate treatment? 

1- Protamine concentrate 


2- Platelet transfusion 


3- Cryoprecipitate 

4- Factor VIII transfusion 

5- Tranexemic acid 


Answer & Comments 

Answer: 3- Cryoprecipitate 

Fresh frozen plasma and cryoprecipitate are 
employed in treating patients with 
coagulopathies due to deficiency of one or 


Dr. Khalid Yusuf El-Zohry - Sohag Teaching Hospital (01118391123) 

Ref MRCPass OE OE 2012 PasTest 2009 PassMedicine 2009 PasTest Exam 




ReviseMRCP 

413 




























El-zohry MRCP Questions Bank (Port 1) - 2013 


(For my personal use) 


more coagulation factors. These conditions 
may occur due to accelerated consumption of 
coagulation factors, (e.g. bleeding, DIC), 
impaired factor production states (vitamin K 
deficiency, w arfarin effect, liver disease, 
congenital factor deficiencies). 

Cryoprecipitate contains fibrinogen, Factor 
VIII, von Willebrand's Factor, Factor XIII, and 
fibronectin. 


[ Q: 968 ] MRCPass - Haematology 
A 65 year old woman presents with 

lethargy. 

She has a Hb of 9.0 g/dl, WCC of 12 x 10 9 /l_, 
platelets of 100 x 10 9 /dl, blood film shows 
spherocytes, polychromasia and smear cells. 
Direct Coomb's test is positive. 

Which is the likely couse of the onoemio? 

1- Thrombotic thrombocytopenic purpura 

2- Autoimmune haemolytic anaemia 

3- Idiopathic thrombocytopenic purpura 

4- Hereditary spherocytosis 

5- B12 deficiency 



Answer & Comments 

Answer: 2- Autoimmune haemolytic anaemia 

The blood film and positive Coomb's test 
points tow ards autoimmune haemolytic 
anaemia. This may be related to a leukaemic 
process in the patient with also raised WCC 
and thrombocytopenia 



[ Q: 969 ] MRCPass - Haematology 

An 18 year old girl is being 
investigated for w orsening menorrhagia and 
gum bleeding. She undergoes a series of blood 
tests which are shown below : 


Hemoglobin 12.3 g/dl (10.5-13.5) 
WBC 7.6 x 10 9 /L (6.0-17.5) 
Platelets 328 x 10 9 /L (156-369) 


APTT 52.6 s (28.0-38.0) 

Bleeding Time 7 1/2 minutes (<5 minutes) 
Prothombin Time 11.6 s (10.0-12.8) 

Thrombin Time 17.3 s (16.0-22.0) 

Factor VIII 0.18 U/ml (0.60-1.50) 

Factor IX 0.92 U/ml (0.60-1.50) 
vWFAg 0.16 s (0.78-1.53) 

VWF ristocetin cofactor <0.10 U/ml(0.50-1.50) 
Which of the following is the likely diagnosis? 

1- Von Willebrand's disease 

2- Factor V leiden 

3- Carrier for Haemophilia A 

4- Acute myeloid leukaemia 

5- Idiopathic thrombocytopenia 


Answer & Comments 

Answer: 1- Von Willebrand's disease 

She is most likely to have type 1 Von 
Willebrand disease (vWD), w here the 
prolongation of the APTT is due a low factor 
VIII level which occurs secondary to the low 
VWF level. 

Von Willebrand disease (VWD) is a group of 
genetically heterogenous disorders resulting 
in abnormal function of the Von Willebrand 
factor (VWF). More than 100 mutations have 
been described. Symptoms include 
mucocutaneous bleeding (epistaxis, easy 
bruising, prolonged bleeding after minor 
trauma, menorrhagia and gastrointestinal 
bleeding) of varying severity. Hemarthrosis is 
relatively uncommon. Unlike hemophilia, the 
mode of inheritance is predominantly 
autosomal dominant (some autosomal 
recessive variants have been described). 

Type 1 vWD is characterized by a partial 
quantitative decrease of qualitatively normal 
vWF and FVIII. 


Dr. Kholid Yusuf El-Zohry - Sohog Teaching Hospital (01118391123) 

Ref MRCPass OE OE 2012 PasTest 2009 PassMedicine 2009 PasTest Exam ReviseMRCP 



414 
























El-zohry MRCP Questions Bank (Port 1) - 2013 


(For my personal use) 


Type 2A vWD is inherited is characterized by 
normal-to-reduced plasma levels of factor 
Vlllc (FVIIIc) and vWF. 

Type 2B vWD is characterized by a reduction 
in the proportion of high molecular weight 
vWF multimers, while the proportion of low 
molecular weight fragments are increased. 


[ Q: 970 ] MRCPass - Haematology 

A 5 year old boy was brought to the 
emergency department by his mother for 
oozing blood from his mouth following a fall. 
His mother said that he tended to bleed for 
prolonged periods from his immunization 
sites, but there was no history of bruising or 
hematomas. The patient was on antibiotics for 
a recent ear infection. There was a family 
history of similar bleeding - his sister and 
mother being affected. 

Blood tests show : 

Hemoglobin 13.3 g/dl (10.5-13.5) 

Hematocrit 35.4% (33.0-39.0) 

WBC 6.9 x10 9 /L (6.0-17.5) 

Platelets 350 x 10 9 /L (156-369) 

PT 12.3 s (10.0-12.8) 

APTT 38.2 s (24.4-33.2) 

Bleeding time 12 minutes (2-9) 

Whot is the diagnosis? 

1- Haemophilia A 

2- Haemophilia B 

3- Von willebrand's disease 

4- Acute lymphoblastic leukaemia 

5- Acute myeloid leukaemia 



Symptoms of von willebrand's disease include 
mucocutaneous bleeding (epistaxis, easy 
bruising, prolonged bleeding after minor 
trauma, menorrhagia and gastrointestinal 
bleeding) of varying severity. The quantitative 
assay (VWF AG) and functional assay (VWF 
ristocetin cofactor/ collagen binding capacity) 
are recommended for diagnostic purposes. 
Approximately 25% of patients with type 1 
VWD have aPTT results outside of the 
reference range. 

DDAVP can raise the levels of vWF in the 
blood. 

Factor VIII concentrates and plasma products 
can be used. 

Using the ear lobe method, a normal bleeding 
time is betw een 1 and 4 minutes. Using the 
forearm method, a normal bleeding time is 
betw een 2 and 9 minutes. 

[ Q: 971 ] MRCPass - Haematology 

A 75 year old lady has presented 
with symptoms consistent with a UTI. Her 
blood tests show Hb of 10.0 g/dl 

WCC of 45 x 10 9 /L 

platelets 160 x 10 9 /L 

neutrophil count is 12 x 10 9 /L (1.5-7) 

lymphocyte count is 27 x 10 9 (1.5-4) 

Which of the following tests is the best to 
elucidate a diagnosis? 

1- Bone marrow trephine 

2- White cell immunophenotyping 

3- Hb electrophoresis 

4- Ultrasound of abdomen 



Answer & Comments 

Answer: 3- Von willebrand's disease 

In this case, the family history and also 
prolonged bleeding time suggests von 
Willebrand's disease. Von Willebrand's disease 
has mostly autosomal dominant inheritance. 


5- Splenic biopsy 

Answer & Comments 

Answer: 2- White cell immunophenotyping 

A high white cell count with predominant 
lymphocytosis and anaemia 


Dr. Khalid Yusuf El-Zohry - Sohag Teaching Hospital (01118391123) 

Ref MRCPass OE OE 2012 PasTest 2009 PassMedicine 2009 PasTest Exam 




ReviseMRCP 

415 























El-zohry MRCP Questions Bank (Port 1) - 2013 


(For my personal use) 


suggests a possible leukaemia such as Chronic 
lymphocytic leukaemia. Immunophenotyping 
can be used for classification of 
undifferentiated leukemia as lymphoid or 
myeloid and subclassification of leukemias. 


[ Q: 972 ] MRCPass - Haematology 

A 28 year old woman attends A+E 
with a history of decreased consciousness. Her 
investigations show : 

Hb 6.7 g/dl 

WBC 7.8 x 10 9 /L 

pits 15 x 10 9 /L 

APTT 34 secs 

PT 16 secs 

Fibrinogen 1.6 g/dl 

Creatinine 180 pmol/l 

Blood film: red cell fragmentation, 
polychromasia and Burr cells. 

What is the most likely diagnosis? 

1- Multiple myeloma 

2- Thrombotic thrombocytopenia purpura 

3- Severe iron deficiency 

4- Disseminated intravascular coagulation 

5- Acute lymphoblastic leukaemia 



Answer & Comments 

Answer: 2- Thrombotic thrombocytopenia 
purpura 

Thrombotic thrombocytopenic purpura is a 
syndrome characterized by microangiopathic 
hemolytic anemia, thrombocytopenia, 
neurologic abnormalities, fever, and renal 
dysfunction. A spectrum of presentations 
related to thrombotic events can occur, 
altered consciousness, seizures, fever, myalgia 
and arthralgia occur. With the introduction of 
plasma exchange (recommended treatment), 
the survival rate has improved from 
approximately 3% prior to the 1960s to 82%. 



The blood film in microangiopathic haemolytic 
anaemia demonstrating 
schistocytes(fragmented cells) and Burr cells. 



[ Q: 973 ] MRCPass - Haematology 

A 30 year old man presents with 
painless lumps in the neck. This has been 
present for the last 7 weeks. He has lost about 
10 kgs in weight over the last six months and 
complains of fever with night sw eats. He 
smokes 20 cigarettes a day. 


On examination there are several enlarged 
lymph nodes in the left supraclavicular fossa. 
Investigations are as follows: 

Hb 10.3 g/dL 

MCV 85 fl 

WBC 16.0 x 10 9 /L 


Neutrophils 55% 
Lymphocytes 34 % 
ESR 57 mm/hour 


Which of the following tests would be most 
appropriate to confirm the diagnosis? 

1- Chest x ray 

2- Kveim test 


3- Sputum for AFB 

4- Lymph node biopsy 

5- Ultrasound scan of abdomen 


Answer & Comments 


Answer: 4- Lymph node biopsy 


Dr. Khalid Yusuf El-Zohry - Sohag Teaching Hospital (01118391123) 

Ref MRCPass OE OE 2012 PasTest 2009 PassMedicine 2009 PasTest Exam 



t 

ReviseMRCP 


416 


l 
























El-zohry MRCP Questions Bank (Port 1) - 2013 


(For my personal use) 


The most likely diagnosis in a patient who has 
cervical lymphadenopathy and B symptoms is 
lymphoma. 

[ Q: 974 ] MRCPass - Haematology 

A 31 year old white woman has 
recurrent episodes of epistaxis. Physical 
examination revealed telangiectasias on her 
forehead and buccal surface of the oral 
mucosa. 

Whot is the likely diagnosis? 

1- Wegener's granulomatosis 

2- Goodpasture's syndrome 

3- Osier Rendu Weber syndrome 

4- Haemophilia A 

5- Von Willebrand's disease 



Answer & Comments 

Answer: 3- Osier Rendu Weber syndrome 

The diagnosis is hereditary haemorrhagic 
telangiectasia (Osier Rendu Weber syndrome). 
Multiple telangiectasia are usually seen on the 
hands and around the mouth. Arteriovenous 
malformations are associated (pulmonary or 
cranial). 


[ Q: 975 ] MRCPass - Haematology 

A 50 year old man has bronze 
pigmentation. He has a family history of liver 
problems. Clinical examination reveals 
hepatomegaly. His investigations show : 

Hb 14.0 g/dl 

MCV 90 fl 

MCHC 30 g/dl (32-35) 

WCC 8 x 10 9 /L 
platelets 180 x 10 9 /L 
PT 17s (11.5-15.5) 

APTT 35s (24-38) 
urea 5 prnol/l 



creatinine 80 pmol/l 
sodium 140 mmol/l 
potassium 3.6 mmol/l 
bilirubin 26 pmol/l 
AST 70 U/l 
ALP 140 U/l 
albumin 32 g/l 
iron 50 ?mol/l (14-29) 
ferritin 650 pg/l (15-200) 
transferrin saturation 80% 

Which of the following is the likely diagnosis? 

1- Alcoholic liver disease 

2- Haemochromatosis 

3- Addison's disease 

4- Chronic hepatitis C 

5- Porphyria 


Answer & Comments 

Answer: 2- Haemochromatosis 

There is high iron and ferritin. Transferrin 
saturation of >50% is high. This is likely to be 
haemochromatosis, which is autosomal 
recessive. Venesection can help reduce ferritin 
levels (aiming for 50 ?g/l). 


[ Q: 976 ] MRCPass - Haematology 

A 30 year old man has had 2 
episodes of haemetemesis following 
consumption of several pints of beer. 

Investigations reveal: Hb 6 g/dl, WCC 1.7 x 
10 9 /L, platelets 4 x 10 9 /L, PT 19s, APTT 52s, 
fibrinogen 0.3 g/l (2-5), fibrin degradation 
products 120ug/ml (<10). 

Blood film reveals predominantly blast cells 
containing Auer Rods. 

Which of the following is the most likely 
diagnosis? 

1- Aplastic anaemia 



Dr. Khalid Yusuf El-Zohry - Sohag Teaching Hospital (01118391123) 

Ref MRCPass OE OE 2012 PasTest 2009 PassMedicine 2009 PasTest Exam 




ReviseMRCP 

417 




























El-zohry MRCP Questions Bank (Port 1) - 2013 


(For my personal use) 


2- Disseminated intravascular coagulation 

3- Immune thromobocytopenic purpura 

4- Acute promyelocytic leukaemia 

5- Acute lymphoblastic leukaemia 


Answer & Comments 

Answer: 4- Acute promyelocytic leukaemia 

Acute myeloid leukaemia is defined as 20% or 
more myeloblasts in the bone marrow . Acute 
promyelocytic leukaemia (M3) is characterized 
by presence of promyelocytes. There is an 
association with the cytogenetic translocation 
t(15:17). 



Auer rods are elongated, bluish-red rods 
composed of fused lysosomal granules, seen 
in the cytoplasm of myeloblasts. 


[ Q: 977 ] MRCPass - Haematology 

A 62 year old woman had successful 
knee surgery. A week following surgery, she 
had a DVT despite prophylactic doses of low 
molecular weight heparin. 

Her blood tests showed: INR 1.1, APPT 37 s, 
Fibrinogen 4.6 g/l, Hb 12.8 g/dl, platelets 18 x 
10 9 /l_, WCC 22 x 10 9 /L Blood film showed 
anisocytosis. 

What is the likely diagnosis? 

1- Idiopathic thrombocytopenic purpura 

2- Disseminated intravascular coagulation 

3- Thrombotic thrombocytopenic purpura 

4- Heparin-induced thrombocytopenia and 
thrombosis 



5- Haemolytic uraemic syndrome 

Answer & Comments 

Answer: 4- Heparin-induced 

thrombocytopenia and thrombosis 

The timing of events, about 7 days after 
commencing heparin suggests an immune- 
mediated phenomenon. Despite 

thrombocytopenia the patient is predisposed 
to thrombosis. Platelet aggregation results in 
thromboembolic events The normal 
fibrinogen suggests DIC is unlikely. 


[ Q: 978 ] MRCPass - Haematology 

A 35 year old lady has had treatment 
with penicillamine for rheumatoid arthritis. 
She presents with weakness and pallor 
claiming that it may be a side effect of the 
drug. 

Her Hb is 5 g/dl, WCC is 2 x 10 9 /I, platelet 
count is 15 x 10 9 /L, INR is 1.0 and APTT is 27 s. 

Which of the following is the most likely 
diagnosis? 

1- Acute myeloid leukaemia 

2- Myelodysplasia 

3- Aplastic anaemia 

4- Folate deficiency 

5- B12 deficiency 



Answer & Comments 

Answer: 3- Aplastic anaemia 

There is pancytopenia due to bone marrow 
failure. Aplastic anaemia can be congenital 
(Fanconi's anaemia) or acquired due to drugs 
(benzene compounds, insecticides, gold or 
penicillamine). 

Treatment is with antilymphocyte globulin, 
cyclosporin or methylprednisolone. 

J 



[ Q: 979 ] MRCPass - Haematology 


Dr. Khalid Yusuf El-Zohry - Sohag Teaching Hospital (01118391123) 

Ref MRCPass OE OE 2012 PasTest 2009 PassMedicine 2009 PasTest Exam 




ReviseMRCP 

418 




























El-zohry MRCP Questions Bank (Port 1) - 2013 


(For my personal use) 


An 8 year old boy is being investigated for 
short stature. 

Examination revealed 2 cafe au lait spots. 
Blood tests showed: Hb 9 g/dl, WCC 2.5 x 
10 9 /l_, platelets 28 x 10 9 /L. 

What is the likely cause of anaemia? 

1- Iron deficiency 

2- Acute lymphoblastic leukaemia 

3- Fanconi's anaemia 

4- Folate deficiency 

5- Multiple myeloma 

Answer & Comments 

Answer: 3- Fanconi's anaemia 

Fanconi's anaemia often presents at age < 10 
with growth retardation, renal defects and 
cafe au lait spots. Inheritance is autosomal 
recessive. 10% of patients may develop acute 
myeloid leukaemia with time. 


Answer & Comments 

Answer: 3- Direct antiglobulin test 

The direct antiglobulin test (DAT) is used to 
detect IgG or C3 bound to the surface of the 
red cell. In patients with hemolysis, the DAT is 
useful in determining whether there is an 
immune etiology. 

Non-immune causes of hemolysis such as DIC, 
thrombotic thrombocytopenic purpura, 
mechanical hemolysis such as those due to 
artificial valves or burns, hemoglobinopathies 
(sickle cell, thalassemia), red cell enzyme 
deficiencies (G6PDP, pyruvate kinase), and red 
cell membrane defects (hereditary 
spherocytosis, PNH) will have a negative DAT. 

Immune causes of hemolysis including 
autoimmune hemolytic anemias, drug induced 
hemolysis, and delayed or acute hemolytic 
transfusion reactions are characterized by a 
positive DAT. 


[ Q: 980 ] MRCPass - Haematology 

A 35 year old woman presents with 
jaundice and lethargy. 

Her investigations reveal: 

Haemoglobin 8.0 g/dL 

reticulocyte count 150 x 10 9 /L (25-85) 

serum bilirubin 75 umol/L 

Her blood film reveals presence of 
spherocytes 

Which of the following is the next useful 
investigation? 

1- Endoscopy 

2- Glucose 6phosphate dehydrogenase activity 

3- Direct antiglobulin test 

4- Red cell osmotic fragility 

5- Haemoglobin electrophoresis 




[ Q: 981 ] MRCPass - Haematology 

A 40 year old man has had a bow el 
operation. 48 hours later he becomes febrile, 
hypotensive and unwell. His investigations 
show : 


Hb 12.6g/dl 
WBC 17.4 x 10 9 /L 
pits 45 x 10 9 /L 

D-dimer 16,000 (<500) ng/dl 
Fibrinogen 82 (180-363) mg/dl 
Haptoglobin 6 (16-200) mg/dl 
INR 2.4(1) 

APTT 50 (<34) 

What is the most likely cause of the 
thrombocytopenia ? 

1- Immune thrombocytopenia 

2- Disseminated intravascular coagulation 

3- Heparin induced thrombocytopenia 

4- Thrombotic thrombocytopenic purpura 


Dr. Khalid Yusuf El-Zohry - Sohag Teaching Hospital (01118391123) 

Ref MRCPass OE OE 2012 PasTest 2009 PassMedicine 2009 PasTest Exam ReviseMRCP 



























El-zohry MRCP Questions Bank (Port 1) - 2013 


(For my personal use) 


5- Aplastic anaemia 


Answer & Comments 

Answer: 2- Disseminated intravascular 

coagulation 

Disseminated intravascular coagulation is 
caused by inappropriate and excessive 
activation of the haemostatic systems. 60% 
are caused by Gram negative sepsis. 

Other causes include viral infections, 
metastatic carcinoma, leukaemia, obstetric 
causes, extensive trauma and burns. 

APTT, PT (INR) and TT are all prolonged, 
platelets and fibrinogen are low , D- 
dimers/FDPs are elevated. Other presenting 
laboratory abnormalities include uremia, 
elevated creatinine, elevated lactate 
dehydrogenase, decreased haptoglobin, 
bilirubinemia and lactic acidosis. Schistocytes 
usually are evident on peripheral smear. 
Treatment is of underlying causes and by 
control of the haemorrhagic state. Platelets, 
blood, cryoprecipitate and fresh frozen plasma 
may all be required. 


[ Q: 982 ] MRCPass - Haematology 

A 30 year old man presents with 
painless enlargement of his cervical lymph 
nodes. He also complains of fever and night 
sw eats. He has lost 1 stone in weight over the 
past 3 months. Chest x ray shows mediastinal 
widening. 

A lymph node biopsy is performed and this 
reveals a background of lymphocytes, plasma 
cells, histiocytes, eosinophils, neutrophils and 
fibroblasts. Scattered within this background 
infiltrate are a number of large cells with two 
large nuclei with prominent nucleoli. 

The diagnosis is: 

1- Tuberculosis 

2- ALL 

3- Hodgkin's lymphoma 



4- Non Hodgkin's lymphoma 

5- CLL 


Answer & Comments 

Answer: 3- Hodgkin's lymphoma 

The clinical features are suggestive in 
Hodgkin's disease and histology demonstrates 
Reed-Sternberg cells, which are 
pathognomonic. Reed-Sternberg cells are 
characteristic bi-nucleate or multinucleate 
cells found in Hodgkin's disease (ow Is eye 
nuclei or church plate nuclei). 

Staging is via the Modified Ann Arbor 
classification: 

I - Involvement of a single lymph node region 
or a single extralymphatic site or organ. 

II - Involvement of two or more lymph node 
regions on the same side of the diaphragm (II) 
or one or more lymph node regions plus an 
extralymphatic site (ME). 

III - Involvement of lymph nodes on both sides 
of the diaphragm. 

IV - Involvement of one or more 
extralymphatic organs (Lung, liver, bone 
marrow , with or without lymph node 
involvement). 



Chest x ray showing mediastinal widening due 
to lymphadenopathy in Hodgkin's lymphoma 



[ Q: 983 ] MRCPass - Haematology 


Dr. Kholid Yusuf El-Zohry - Sohog Teaching Hospital (01118391123) 

Ref MRCPass OE OE 2012 PasTest 2009 PassMedicine 2009 PasTest Exam 




ReviseMRCP 

420 
























El-zohry MRCP Questions Bank (Port 1) - 2013 


(For my personal use) 


A 12 year old boy has recently been found to 
be anaemic and is undergoing investigations. 

He is short and has an abnormal facies with 
frontal and parietal bossing, enlargement of 
the malar (maxillary) bones and protruding 
teeth. 

On examination of the abdomen he has 
hepatosplenomegaly. Investigations are as 
follows: Hb 7.5 g/dl MCV 65 fl 

Pit 160 x 10 9 /L 

Which of the following is likely to treat the 
anaemia? 

1- Ascorbic acid 

2- Ferrous sulphate 

3- Vitamin B u 

4- Folic acid 

5- Blood transfusion 


Answer & Comments 

Answer: 4- Folic acid 

The history of chronic anaemia and the 
examination features suggest a chronic 
haemolytic anaemia with extramedullary 
erythropoiesis. This suggests the diagnosis of 
thalassaemia. In thalassaemia, folic acid 
supplementation is useful in treatment of 
anaemia as there is increased metabolic 
demand for folic acid. 

[ Q: 984 ] MRCPass - Haematology 

A 30 year old woman is bleeding a 
lot after a thyroidectomy. 

Investigations show: 

Hb 11.3 g/dl 

WBC 5.2 x 10 9 /L 

Pits 230 x 10 9 /L 

PT 15 sec (13-16 sec) 

APTT 86 sec (28-38 sec) 

APTT 50:50 mix with normal plasma 37 sec 



Which of the following is the most likely 
diagnosis? 

1- Factor V deficiency 

2- Anti-phospholipid syndrome 

3- Factor VII deficiency 

4- Factor XII deficiency 

5- Von Willebrand's disease 


Answer & Comments 

Answer: 5- Von Willebrand's disease 

An isolated prolonged APTT will be caused by 
deficiencies in factors VIII, IX, XI and XII and by 
von Willebrand's disease. 

Anti-phospholipid syndrome can cause a 
prolonged APTT but is not associated with 
bleeding and the APTT is not corrected with 
normal plasma. Factor X and V deficiency are 
associated with both a prolonged PT and 
APTT. 

Factor VII deficiency is associated with a 
prolonged PT. 



[ Q: 985 ] MRCPass - Haematology 

A 60 year old man presents with 
back pains, abdominal pains and polyuria. 

He has a Hb of 12 g/dl, WCC of 8 x 10 9 /L, pit of 
300 x 10 9 /L. 


Serum calcium is 2.9 (2.25-2.7) mmol/l and 
phosphate 1.2 (0.8-8) mmol/l. 

What might the bone marrow examination 
show? 

1- Increased blast cells 

2- Increased proportion of plasma cells 

3- Increased promyelocytes 

4- Increased infiltration of lymphatic cells 

5- Increased fibrosis 


Dr. Khalid Yusuf El-Zohry - Sohag Teaching Hospital (01118391123) 

Ref MRCPass OE OE 2012 PasTest 2009 PassMedicine 2009 PasTest Exam 




ReviseMRCP 

421 























El-zohry MRCP Questions Bank (Port 1) - 2013 


(For my personal use) 


Answer & Comments 

Answer: 2- Increased proportion of plasma 
cells 

The diagnosis is multiple myeloma, and the 
bone marrow shows increased amounts of 
plasma cells (>30%) as seen in the image 
below. 



[ Q: 986 ] MRCPass - Haematology 

A 55 year old man has acute onset 
confusion, headache, nausea and vomiting 
and visual disturbance. He had prescribed 
been prescribed NSAIDS and antibiotics for 
knee arthritis. 

On examination he was overweight, he looked 
plethoric, and cyanosed. There was 3 
fingerbreadths hepatomegaly and the spleen 
was enlarged about 5 fingerbreadths. 

Investigations show : 

Hb 21.2 g/dl MCV 71 fl 

WBC 18 x 10 9 /L Na 135 mmol/I 

K 3.8 mmol/l Urea 6.2 mmol/l 

Creatinine 88 micromoles/l 

Chloride 105 mmol/l 

Bicarbonate 32 mmol/l 

Calcium 2.5 

Albumin 36 g/l 

Phosphate 0.9 mmol/l 

ESR 15 mm/lst hour 

Whot is the best management? 

1- Hyperbaric oxygen 

2- Splenectomy 



3- Prednisolone 

4- Broad spectrum antibiotics 

5- Venesection 


Answer & Comments 

Answer: 5- Venesection 

The diagnosis is polycythaemia rubra vera. 
Increased serum viscosity may arise from 
hyperglobulinaemia or from an increased red 
cell mass, polycythaemia. As a guideline, 
erythrocytosis should be suspected in men 
with a haemoglobin concentration greater 
than 18.0 g /L or in w omen with values 
greater than 17.0 g /L. Treatment of 
hyperviscosity syndrome should be with fluid 
replacement and venesection. 

The cause of cyanosis is due to small vessel 
insufficiency and thrombosis, which will 
improve with venesection. 

Hydroxyurea and anagrelide are 
chemotherapeutic agents which can also be 
considered. 


[ Q: 987 ] MRCPass - Haematology 

A 35 year old man has diabetes. On 
examination he also has a slate grey 
discolouration around his forearm. He has a 
Hb of 13.5 g/dl, platelet count 350 x 10 9 /L, AST 
of 35 U/l, ALP is 120 U/l, Albumin 35 g/l, 
ferritin is 500 pg/l. 

Which of the following tests is most helpful? 

1- Copper and caeruloplasmin 

2- Transferrin saturation 

3- Fasting glucose 

4- 72 hour fast 

5- Short synacthen test 



Answer & Comments 


Answer: 2- Transferrin saturation 


Dr. Khalid Yusuf El-Zohry - Sohag Teaching Hospital (01118391123) 

Ref MRCPass OE OE 2012 PasTest 2009 PassMedicine 2009 PasTest Exam ReviseMRCP 



422 




























El-zohry MRCP Questions Bank (Port 1) - 2013 


(For my personal use) 


The likely diagnosis is haemochromatosis due 
to the diabetes, pigmentation, and raised 
ferritin. Transferrin saturation would be raised 
in haemochromatosis. 


[ Q: 988 ] MRCPass - Haematology 

A 72 year old woman presents with 
malaise, headaches and weakness in her arms 
and legs. Clinical examination reveals 
lymphadenopathy and hepatosplenomegaly. 
Nerve conduction tests show a sensory 
neuropathy. 

Her blood tests reveal Hb of 7.9g/dl, MCV 95 
fl, WCC 9 x 10 9 /L. Her ESR is 80 and IgM 
paraprotein of 18 g/l (0-5). 

Which is the most likely diagnosis? 

1- Multiple myeloma 

2- CML 

3- CLL 

4- Waldenstrom's macroglobulinaemia 

5- AML 



Answer & Comments 

Answer: 4- Waldenstrom's 

macroglobulinaemia 

In Waldenstrom's macroglobulinaemia, 
increased serum proteins leads to a variety of 
symptoms: 

Neuropathy 

Headache and focal nervous system 
impairment 

congestive cardiac failure. 

Diagnosis is confirmed by high Ig M 
paraprotein levels (also known as a spike). 



A plasma cell on the blood film in 
Waldenstrom's 


^ [ Q: 989 ] MRCPass - Haematology 

si - 

# A 60 year old woman undergoes a 
colostomy, which is complicated by a post¬ 
operative haemorrhage. 

Following transfusion of 4 units of blood, her 
haemoglobin is 12. g/dl. 

A week later, she has the following results: 

Hb 7.7 g/dl WBC 6.6 x 10 9 /L 

Pits 377 x 10 9 /L MCV 96 fl 

Bilirubin 66 umol/l 

Direct Coombs test positive 

Whot is the most likely diagnosis? 

1- Further post-operative bleed 

2- Subacute endocarditis 

3- Delayed haemolytic transfusion reaction 

4- Acute haemolytic transfusion reaction 

5- Autoimmune haemolytic anaemia 


Answer & Comments 

Answer: 3- Delayed haemolytic transfusion 
reaction 

The most likely diagnosis is a delayed 
haemolytic transfusion reaction. These are 
due to incompatibilities in red cell antigens 
other than the ABO groups. The antibodies are 
acquired rather than naturally occurring so 
they occur in patients who have been 
pregnant in the past or who have had blood 


Dr. Khalid Yusuf El-Zohry - Sohag Teaching Hospital (01118391123) 

Ref MRCPass OE OE 2012 PasTest 2009 PassMedicine 2009 PasTest Exam 




ReviseMRCP 

423 
























El-zohry MRCP Questions Bank (Port 1) - 2013 


(For my personal use) 


transfusions. When stimulated by transfusion, 
antibody levels increase over 7-10 days to 
cause a delayed haemolytic transfusion 
reaction. 


[ Q: 990 ] MRCPass - Haematology 

A 65 year old lady presents with 
malaise and weight loss, having been referred 
by the GP for investigation of anaemia. His 
investigations show : 

Hb 8.0 g/dl 

MCHC 33 g/dl (32-35) 
platelets 130 x 10 9 /L 
creatinine 90 pmol/l 
potassium 4 mmol/I 
AST 28 U/l 

iron 50 ?mol/l (14-29) 

ferritin 550 pg/l (15-200) 

Bone marrow aspirate shows increased 
haemosiderin, normoblastic hyperplasia and 
ringed red blood cells 

Which of the following is likely? 

1- Acute myeloid leukaemia 

2- Plasmacytoma 

3- Chronic myeloid leukaemia 

4- Sideroblastic anaemia 

5- Chronic lymphoblastic leukaemia 


MCV 105 fl 
WCC 11 x 10 9 /L 
urea 6 pmol/l 
sodium 140 mmol/l 
bilirubin 18 pmol/l 
ALP 180 U/l 




Ringed sideroblasts in a case of 
myelodysplasia 


[ Q: 991 ] MRCPass - Haematology 

A 30 year old woman was sent to the 
hospital for investigation following a upper 
respiratory tract infection which was slow to 
resolve. The following results were seen: Hb 
11.5 g/dl, MCV 79 fl, platelets 650 x 10 9 /L, 
WCC 10 x 10 9 /L, normal PT and APTT. 

What is the likely couse of the 
thrombocytosis? 

1- Polycythaemia rubra vera 

2- Sideroblastic anaemia 

3- Reactive thrombocytosis 

4- Myelodysplasia 

5- Idiopathic thrombocytopenic purpura 



Answer & Comments 

Answer: 4- Sideroblastic anaemia 

In sideroblastic anaemia, there is increased 
bone marrow iron. This is reflected in the 
increased iron stores in ferritin and also 
haemosiderin and ringed premature red blood 
cells (sideroblasts) due to excess iron. 
Sideroblastic anaemia occurs due to an 
enzyme deficiency (?ALA synthase 2 
deficiency), alcohol, drugs (anti TB), 
myelodysplasia. 


Answer & Comments 

Answer: 3- Reactive thrombocytosis 

Reactive thrombocytosis may be due to 
haemorrhage, iron deficiency, malignancy, 
infection and connective tissue diseases. 


[ Q: 992 ] MRCPass - Haematology 

A 75 year old patient has been 
unwell. Her blood tests show : 

Hb 7.0 g/dl 




Dr. Kholid Yusuf El-Zohry - Sohog Teoching Hospitol (01118391123) 

Ref MRCPass OE OE 2012 PasTest 2009 PassMedicine 2009 PasTest Exam 


ReviseMRCP 





























El-zohry MRCP Questions Bank (Port 1) - 2013 


(For my personal use) 


MCV110 fl 
WCC 4 x 10 9 /L 
platelets 70 x 10 9 /L 
urea 5 pmol/l 
creatinine 110 pmol/l 
sodium 140 mmol/I 
potassium 4 mmol/l 

The blood film shows ring sideroblasts with 
15% blast cells. 

Bone marrow show hypercellularity 
Which is the most likely diagnosis? 

1- Acute myeloid leukaemia 

2- Chronic myeloid leukaemia 

3- Myelodysplastic syndrome 

4- Chronic lymphatic leukaemia 

5- Non hodgkin's lymphoma 


Answer & Comments 

Answer: 3- Myelodysplastic syndrome 

Myelodysplastic syndromes are associated 
with pancytopenias along with 
dyserythropoietic ringed sideroblasts and 
blast cells in the peripheral circulation. 
Sideroblasts would be unlikely in AML and 
CML, although blast cells could be seen. If the 
bone marrow was hypocellular, then aplastic 
anaemia would be possible in this question. 



Bone Marrow biopsy in myelodysplasia 
showing hypercellularity and abnormal 
megakaryocytes 



[ Q: 993 ] MRCPass - Haematology 

A 45 year old intravenous drug user 
is treated with unfractionated heparin for a 
DVT as he is being assessed for endocarditis. 
Two months previously, he had received 
heparin. After 3 days of treatment, his platelet 
count has fallen from a baseline of 180 x 10 9 /L 
to 120 x 10 9 /L. Upon enquiry to the GP, he had 
previously had blood tests which showed a 
normal platelet count and he had no history of 
bleeding problems. 

Which of these statements is true? 


1- Deep vein thrombosis 

2- Type I HIT 


3- Essential thrombocytopenia 

4- Idiopathic thrombocytopenic purpura 


5- Henoch Schonlein Purpura 


Answer & Comments 

Answer: 2- Type I HIT 

The most likely diagnosis is heparin induced 
thrombocytopenia, in view of the previous 
normal platelet counts and recent use of 
heparin. Although the patient becomes mildly 
thrombocytopaenic, therapy can continue as it 
is likely to remain mild. In Type II HIT, 
antibodies would be more readily detectable 
by ELISA and the thrombocytopenia would be 
more severe. 


[ Q: 994 ] MRCPass - Haematology 

A 30 year old woman attends A&E 
with marked breathlessness. Subsequently, a 
pulmonary embolism is confirmed by V/Q 
scan. 

Blood tests reveal the following results: 

PT 11/11s 

APTT 67/31 s (50:50 mix test:normal plasma 
55s) 

TT 19/18 s 
Hb 10.2 g/dl 



Dr. Khalid Yusuf El-Zohry - Sohag Teaching Hospital (01118391123) 

Ref MRCPass OE OE 2012 PasTest 2009 PassMedicine 2009 PasTest Exam 




ReviseMRCP 


425 

i 

























El-zohry MRCP Questions Bank (Port 1) - 2013 


(For my personal use) 


WCC 1.8 x 10 9 /L 
Plats 90 x 10 9 /L 

Which one of the following investigations is 
most appropriate? 

1- Antiphospholipid antibody 

2- Bone marrow examination 


Methaemoglobinaemia occurs when 
haemoglobin is oxidised and is unable to carry 
oxygen. 

Drugs which can cause this condition include: 

antibiotics (dapsone, sulphonamides and 
trimethoprim) 


3- Clotting factor levels 


nitrites and nitrates 


4- Testing for lupus anticoagulant 

5- Thrombophilia screening 


Answer & Comments 

Answer: 1- Antiphospholipid antibody 

The lupus anticoagulant is a form of 
antiphospholipid antibody, and this is likely to 
be present as there is prolonged APTT. The 
name was given to the antibody because it 
was first found in patients with lupus. The 
presense of this is likely to have predisposed 
the patient to a pulmonary embolus. Although 
thrombophilia testing is indicated it is best left 
until after the initial period of anticoagulation. 


[ Q: 995 ] MRCPass - Haematology 

A 35 year old woman was admitted 
with breathlessness. On admission she looked 
cyanotic and was given high flow oxygen 
immediately. Arterial blood gas analysis 
revealed a pH of 7.40, p0 2 of 11 kPa, pC0 2 of 4 
and oxygen saturation of 50% by co-oximeter. 

What treatment should be given? 

1- N acetyl cysteine 

2- Haemocdialysis 

3- Methylene blue 

4- Oral activated charcoal 

5- Ascorbic acid 



Answer & Comments 


Answer: 3- Methylene blue 


local anaesthetics (lignocaine and prilocaine). 

Pulse oximeters measure both oxyHb and 
metHb, therefore giving false reassurance in 
patients with high levels of metHb. Methylene 
blue is used in those with severe poisoning or 
MetHb levels greater than 30%; excessive 
doses of methylene blue can themselves 
cause methaemoglobinaemia. Dapsone 
poisoning can be treated by activated 
charcoal, which adsorbs it. 


[ Q: 996 ] MRCPass - Haematology 

A 38 year old woman presents to the 
hospital with a history of headaches, 
decreased consciousness and fevers. Her 
blood results show : 

Hb 10.5 g/dl 

WBC 14 x 10 9 /L 

Pits 14 x 10 9 /L 

Clotting screen normal 

urea 15 pmol/l 

creatinine 210 pmol/l 

What is the most likely diagnosis? 

1- Acute lymphoblastic leukaemia 

2- Thrombotic thrombocytopenic purpura 

3- Disseminated intravascular coagulation 

4- Chronic myeloid leukaemia 

5- Acute myeloid leukaemia 



Answer & Comments 

Answer: 2- Thrombotic thrombocytopenic 
purpura 


Dr. Khalid Yusuf El-Zohry - Sohag Teaching Hospital (01118391123) 

Ref MRCPass OE OE 2012 PasTest 2009 PassMedicine 2009 PasTest Exam ReviseMRCP 




























El-zohry MRCP Questions Bank (Port 1) - 2013 


(For my personal use) 


The classic 5 features associated with TTP are: 
fever 

thrombocytopenia 

microangiopathic haemolytic anaemia 
renal failure 
neurological symptoms 


[ Q: 997 ] MRCPass - Haematology 

A 35 year old man presents with a 10 
day history of lethargy. He has a maculo- 
papular rash and a 1 cm sized palpable 
cervical lymph node. Investigations show : 

Hb 13.6 g/dl 

WBC 13.2 x 10 9 /L 

Pits 280 x 10 9 /L 

Blood film shows reactive lymphocytes 
What is the most likely diagnosis? 

1- Tuberculosis 

2- SLE 

3- Infectious mononucleosis 

4- Acute myeloblastic leukaemia 

5- Non-Hodgkin's disease 



Answer & Comments 

Answer: 3- Infectious mononucleosis 

Common causes of a reactive lymphocytosis 
include infectious mononucleosis (EBV 
infection), CMV infection and toxoplasma 
infection. 



Reactive Lymphocytes 



[ Q: 998 ] MRCPass - Haematology 

A 40 year old man presents with 
fatigue, weakness, and shortness of breath 
with exertion over the past few days. 


On examination, he is pale and jaundiced. His 
heart rate is 110 and blood pressure is 110/65 
mmHg. His breath sounds were clear, there is 
a soft flow murmur audible in the aortic area. 


Blood results show : 

Hb 6.5 g/dl 

MCV 105 fl 

WCC 6.2 x 10 9 /L 

platelets 250 x 10 9 /L 

Reticulocyte count 14% (0.5% to 1.5%) 

AST 27 (1-31) U/l 

ALP 78 (20-120) U/l 

Bilirubin 65 (1-22) pmol/l 

lactate dehydrogenase 410 (105-333) U/L 

The blood film shows spherocytes. 

Which test should be performed next? 

1- Direct Coomb's test 

2- Osmotic fragility 

3- HAM's test 


4- Bone marrow aspirate 

5- Haemoglobin electrophoresis 


Answer & Comments 


Answer: 1- Direct Coomb's test 


Dr. Kholid Yusuf El-Zohry - Sohog Teaching Hospital (01118391123) 

Ref MRCPass OE OE 2012 PasTest 2009 PassMedicine 2009 PasTest Exam 



".1 

ReviseMRCP 


427 

i 

i 



























El-zohry MRCP Questions Bank (Port 1) - 2013 


(For my personal use) 


The diagnosis is autoimmune haemolytic 
anaemia. In haemolytic anaemia, the bilirubin 
and LDH are raised, there is a reticulocytosis, 
and blood film shows spherocytes. 
Autoimmune haemolytic anaemia is 
associated with viral infections, drugs, 
lymphoproliferative diseases and autoimmune 
diseases. 

The direct Coomb's test (DAT) is used to 
detect IgG or C3 bound to the surface of the 
red cell. In patients with hemolysis, the DAT is 
useful in determining w hether there is an 
immune etiology. Immune causes of 
hemolysis, including autoimmune hemolytic 
anemias, drug induced hemolysis, and delayed 
or acute hemolytic transfusion reactions, are 
characterized by a positive DAT. If the DAT 
test is negative, then hereditary spherocytosis 
should be considered. 

Autoimmune haemolytic anaemia can be due 
to warm or cold reacting antibodies, 
depending on the cause of the autoimmune 
reaction. Warm antibodies tend to be IgG and 
lead to splenic destruction of red blood cells, 
forming spherocytes. Cold antibodies are 
usually IgM antibodies. They cause 
intravascular haemolysis. 



OtibOly- ulUJOtLii ITIeaIi vulMa 

roabti.rlw'j in a. tji- ri* 1 ?iiiL |'C o d mk 1 kffUJtiulBQ, 

iTurunj bn] 


Coomb's test 


Dr. Khalid Yusuf El-Zohry - Sohag Teaching Hospital (01118391123) 

Ref MRCPass OE OE 2012 PasTest 2009 PassMedicine 2009 PasTest Exam ReviseMRCP 



428 






















El-zohry MRCP Questions Bank (Port 1) - 2013 


(For my personal use) 


^ [ Q: 999 ] MRCPass - Endocrinology 

tl - 

# A 22 year old student is diagnosed 
with Grave's disease. She enquires about the 
long term complications of radioactive iodine, 
which is being considered. 

What side effect is most likely? 

1- Hyperthyroidism 

2- Hypoparathyroidism 

3- Hypothyroidism 

4- Thyroid malignancy 

5- Recurrent laryngeal nerve damage 


4- Doxazosin 

5- Bendrofluazide 

Answer & Comments 

Answer: 3- Spironolactone 

The diagnosis is primary hyperaldosteronism 
(Conn's syndrome). Spironolactone 200-400 
mg is recommended as first line, but ACE- 
inhibitors can also be used. 

Surgical treatment (removal of an adenoma) is 
definitive. 


Answer & Comments 

Answer: 3- Hypothyroidism 

Radioactive iodine generally has few side 
effects, is permanent, and very effective. It 
cannot be used during pregnancy due to risks 
of teratogenicity. It can often be performed as 
an outpatient or with a short hospital stay. 
The long-term risk is hypothyroidism, because 
thyroid function is reduced so effectively that 
thyroid replacement may be required. 


[ Q: 1000 ] MRCPass - Endocrinology 

A 40 year old man has a blood 
pressure of 170/110 mmHg. 

Laboratory findings include: 

sodium 147 mmol/L (135-145) 

potassium 2.6 mmol/L (3.5-5.0) 

chloride 101 mmol/L (95-105) 

glucose 5 mmol/L (3.5-5.5) 

creatinine 90 umol/L (70-110) 

His plasma renin activity is 0.15 ng/mL/hr and 
his serum aldosterone 800 pmol/L (100-500). 

Which drug is recommended for hypertension? 

1- Ramipril 

2- Atenolol 

3- Spironolactone 




[ Q: 1001 ] MRCPass - Endocrinology 

A 22 year old man who has been 
taking a drug has now developed hirsutism. 

Which one of the following drugs is most likely 
to be the couse? 

1- Prednisolone 

2- Minoxidil 


3- Amphetamine 

4- Propanolol 

5- Acetylsalicylic acid 


Answer & Comments 

Answer: 2- Minoxidil 

Causes of hirsutism include: 

cyclosporin A 

Risperidone 

Minoxidil 

Phenytoin 

Ovarian tumours 

Polycystic Ovary syndrome 

Congenital adrenal hyperplasia 



[ Q: 1002 ] MRCPass - Endocrinology 
A 60 year old woman presents with 


Dr. Kholid Yusuf El-Zohry - Sohog Teaching Hospitol (01118391123) 

Ref MRCPass OE OE 2012 PasTest 2009 PassMedicine 2009 PasTest Exam ReviseMRCP 



429 

































El-zohry MRCP Questions Bank (Port 1) - 2013 


(For my personal use) 


anorexia, nausea vomiting and weight loss. On 
examination she was dehydrated and looked 
unwell. 

Investigations reveal: 

Hb 15 g/dl 
WBC 5.7 x 10 9 /L 
ESR40 

Sodium 138 mmol/l 
Potassium 3.7 mmol/l 
Chloride 112 mmol/l 
Bicarbonate 25 mmol/l 
Urea 19.3 mmol/l 
Creatinine 260 micromoles/l 
Albumin 42 g/l 
Calcium 3.3 mmol/l 
Phosphate 0.60 mmol/l 

Which of the following is the most likely 
co use? 

1- Vitamin D toxicity 

2- Pseudopseudohypoparathyroidism 

3- Congenital adrenal hyperplasia 

4- Osteoporosis 

5- Paget's disease 


Which is the likely cytologic finding? 

1- Squamous cell carcinoma 

2- Small cell (oat cell) carcinoma 

3- Large cell carcinoma 

4- Adenocarcinoma 

5- Teratoma 


Answer & Comments 

Answer: 2- Small cell (oat cell) carcinoma 

The pigmentation and Cushingoid features are 
typical features of ectopic ACTH secretion. 
This is most commonly due to small cell 
carcinoma, but may be associated with a 
carcinoid tumour. 


[ Q: 1004 ] MRCPass - Endocrinology 

A 38 year old lady has fasting blood 
glucose of 6.8 mmol/L. 

Whot is the next investigation of choice? 

1- Random glucose 

2- C peptide level 

3- Insulin level 

4- Oral glucose tolerance test 

5- Insulin tolerance test 



Answer & Comments 

Answer: 1- Vitamin D toxicity 

This patient has probably been overtreated 
with vitamin D. Severe hypercalcaemia (Serum 
calcium > 3 mmol/l) may be associated with 
malignant disease, hyperparathyroidism, renal 
failure and vitamin D therapy. 


^ [ Q: 1003 ] MRCPass - Endocrinology 

# A 50 year old woman presents with 
Cushingoid facies and hyperpigmentation of 
the skin on her face. She smoked 20 cigarettes 
per year for 20 years. Her chest X ray reveals a 
3 cm mass in the right upper lobe. A CT guided 
needle biopsy of the lung lesion is performed. 


Answer & Comments 

Answer: 4- Oral glucose tolerance test 

In the oral glucose tolerance test the patient, 
after fasting for 15 hours, drinks 75 g of 
glucose in 300 ml of water, over 5 minutes. 
Blood glucose is measured before the drink 
and after 30, 60, 90 and 120 minutes. There is 
a normal glucose tolerance if the venous 
plasma value is less than 7 mmol/l after the 2 
hour period. If after 2 hours after the glucose 
load the value is betw een 7 and 11 mmol/l, 
then there is impaired glucose tolerance. If 
glucose is greater than or equal to 11.1 
mmol/l, then there is diabetes mellitus. 


Dr. Kholid Yusuf El-Zohry - Sohog Teaching Hospital (01118391123) 

Ref MRCPass OE OE 2012 PasTest 2009 PassMedicine 2009 PasTest Exam ReviseMRCP 




























El-zohry MRCP Questions Bank (Port 1) - 2013 


(For my personal use) 


[ Q: 1005 ] MRCPass - Endocrinology 

A 30 year old lady has recurrent 
attacks of dizziness and blackouts. A 72 hour 
fast reveals periods where her plasma glucose 
is 2.5 mmol/I with elevated insulin and C 
peptide levels. 

Which is the next best investigation? 

1- Repeat 72 hour fast 

2- Glucagon stimulation test 

3- Glucose tolerance test 

4- MRI of abdomen 

5- Insulin antibodies 



Answer & Comments 

Answer: 4- MRI of abdomen 

The tests so far suggest an insulinoma. 
Localisation of the insulinoma can be done 
with MRI, CT, superior mesenteric 
angiography or pancreatic venous 
catheterisation. 



Insulinoma 


[ Q: 1006 ] MRCPass - Endocrinology 

A 25 year old lady has mature onset 
diabetes of the young (MODY). Her blood 
sugars have been well controlled with 
gliclazide 80mg bd. Her obstetrician refers her 
to the physician for shared management of 
diabetes. 

Which is the best management step? 

1- Increase gliclazide doses to 160 mg bd 



2- Convert gliclazide to actrapid and insulatard 
insulin injections 

3- Convert gliclazide to metformin 

4- Leave gliclazide doses unchanged 

5- Stop gliclazide and monitor blood glucose 


Answer & Comments 

Answer: 2- Convert gliclazide to actrapid and 
insulatard insulin injections 

Gliclazide can cross the placenta and cause 
fetal hypoglycaemia. In order to avoid 
macrosomia and congenital disorders, 
glycaemic control is best achieved with insulin 
therapy during pregnancy. A tds actrapid 
(short acting) regime with night insulatard 
(long acting) will achieve best glycaemic 
control. 


[ Q: 1007 ] MRCPass - Endocrinology 

A 45 year old man with type 2 
diabetes attends the clinic. His HbAlc is 
10.5%. 

What average plasma glucose concentration is 
this HbAlc value eguivalent to? 

1- 7 mmol/I 

2- 10 mmol/l 

3- 12.5 mmol/l 

4- 16 mmol/l 

5- 20 mmol/l 



Answer & Comments 

Answer: 4-16 mmol/l 

A HbAlc of 7% would translate into an 
average glucose concentration of 9.5 mmol/l, 
and a HbAlc of 10% into 15.5 mmol/l. 


^ [ Q: 1008 ] MRCPass - Endocrinology 

fL --" 

# A 46 year old lady has been gaining 
weight over the past year. She also has 
features of hirsutism and abdominal striae. 


Dr. Khalid Yusuf El-Zohry - Sohag Teaching Hospital (01118391123) 

Ref MRCPass OE OE 2012 PasTest 2009 PassMedicine 2009 PasTest Exam ReviseMRCP 





























El-zohry MRCP Questions Bank (Port 1) - 2013 


(For my personal use) 


On examination, she was found to have 
proximal myopathy. 

Whot test should be done? 

1- Short synacthen test 

2- Random cortisol level 

3- Low dose dexamethasone suppression test 

4- Insulin tolerance test 

5- Oral glucose tolerance test 


Answer & Comments 

Answer: 3- Low dose dexamethasone 
suppression test 

This lady is likely to have Cushing's syndrome. 
A random cortisol is not that helpful - a 24 
hour urine free cortisol collection should be 
done, and also a low dose dexamethasone 
suppression test (over 2 days) will confirm the 
diagnosis if there is failure of cortisol 
suppression. 



[ Q: 1009 ] MRCPass - Endocrinology 

A 55 year old woman with Cushing's 
syndrome presents with left sided hip pain 
acutely. 


Which one of the following is most likely to 
hove occurred? 


1- Hip dislocation 

2- Neuropathy 

3- Necrosis of femoral head 


4- Metaststic lesion 


5- Iliac crest fracture 


Answer & Comments 

Answer: 3- Necrosis of femoral head 

Cushing's Syndrome can present with necrosis 
of femoral head due to osteoporosis. Necrosis 
of the femoral head can also be caused by 
avascular causes such as sickle cell disease, 
diabetes, SLE and scleroderma. 



Necrosis of the femoral head 


[ Q: 1010 ] MRCPass - Endocrinology 

A 22 year old man presents with 
worsening lethargy over the past several 
years. He also mentions long standing polyuria 
and nausea. 

Examination reveals that he has a BMI of 22 
kg/m 2 and a blood pressure of 110/60 mmHg. 

Examination of the cardiovascular and 
abdominal systems are unremarkable. 
Investigations reveal: 

sodium 139 mmol/I 

potassium 2.8 mmol/I 

urea 6.6 mmol/I 

creatinine 97 pmol/l. 

ABGs showed pH of 7.5, normal pC> 2 , 
bicarbonate of 32 mmol/l (21 - 28). 

Further investigation showed : 

aldosterone 1195 pmol/l (111 - 863) - 
ambulatory 

renin 92 ng/ml/hr (7 - 76) 

Whot is the likely diagnosis? 

1- Type IV renal tubular acidosis 

2- Conn's syndrome 

3- Bartter's syndrome 

4- Cushing's syndrome 

5- Hypokalaemic periodic paralysis 



Dr. Kholid Yusuf El-Zohry - Sohog Teaching Hospital (01118391123) 

Ref MRCPass OE OE 2012 PasTest 2009 PassMedicine 2009 PasTest Exam ReviseMRCP 



432 























El-zohry MRCP Questions Bank (Port 1) - 2013 


(For my personal use) 


Answer & Comments 

Answer: 3- Bartter's syndrome 

Bartter's syndrome is an autosomal recessive 
renal disorder. There is a Na-K-2CI 
cotransporter gene defect. 

Presentation is often in childhood with 
gastrointestinal upset and polyuria. There is 
associated hypokalaemic metabolic alkalosis, 
elevated renin and aldosterone levels. High 
urine chloride levels can be detected. 
Vomiting, constipation, polyuria and 
polydipsia are common symptoms. 

Possible alternative diagnoses are diuretic 
(thiazide) abuse or Gitelman's syndrome (the 
difference is that Gitelman's syndrome causes 
hypocalciuria and Bartters causes 
hypercalciuria). Type 4 renal tubular acidosis 
occurs in diseases associated with a failure of 
aldosterone action or hypoaldosteronism. 


[ Q: 1011 ] MRCPass - Endocrinology 

A 55 year old man has 
gynaecomastia. 

Which one of the following drugs is most likely 
to be responsible? 

1- Ketoconazole 

2- Morphine 

3- Frusemide 

4- Metoprolol 



[ Q: 1012 ] MRCPass - Endocrinology 

f - 

# An 17 year lady presents with a 6 
month history of secondary amenorrhoea. She 
had previously been prescribed Temazepam 
for anxiety. On examination, she had 
galactorrhoea expression. Her prolactin 
concentration was 4500 mu/I (50-450). 
Pregnancy test was negative. 

What is the likely diagnosis? 

1- Acromegaly 

2- Pituitary microadenoma 

3- Polycystic ovarian syndrome 

4- Drug induced hyperprolactinaemia 

5- Turner's syndrome 


Answer & Comments 

Answer: 2- Pituitary microadenoma 

The history examination findings together 
grossly elevated prolactin concentration are 
suggestive of a microprolactinoma. The most 
frequent symptoms at onset are 
oligoamenorrhoea (60%) and galactorrhoea 
(50%), and headaches. Treatment is with 
bromocriptine (e.g. 5 mg od). 


[ Q: 1013 ] MRCPass - Endocrinology 

A 30 year old lady has Polycystic 
syndrome. She mentions difficulty 
conceiving. 



ovarian 


5- Paracetamol 


Answer & Comments 


Answer: 1- Ketoconazole 


Which one of the following drugs can help 
improve fertility? 

1- Metformin 

2- Testosterone 


Testosterone replacement therapy, Testicular 
tumours, ACE inhibitors, calcium antagonists, 
digoxin, cimetidine, ketoconazole, 
metronidazole, tricyclic anti-depressants and 
benzodiazepines may cause gynaecomastia. 


3- Ethinyl oestradiol 

4- Spironolactone 

5- Cyproterone acetate 


Answer & Comments 


Answer: 1- Metformin 


Dr. Khalid Yusuf El-Zohry - Sohag Teaching Hospital (01118391123) 

Ref MRCPass OE OE 2012 PasTest 2009 PassMedicine 2009 PasTest Exam 




ReviseMRCP 

433 































El-zohry MRCP Questions Bank (Port 1) - 2013 


(For my personal use) 


Polycystic Ovarian Syndrome (PCOS) is 
characterized by irregular ovulation and 
menses, obesity, insulin resistance, acne, and 
hirsutism (excessive hair growth). Impaired 
fertility is a prominent feature of PCOS. This is 
believed to result from elevated insulin levels 
that stimulate excess androgen production by 
the ovaries. The androgens cause premature 
follicular wasting which causes inconsistent or 
absent ovulation, which is associated with 
infertility. 


Metformin has been show n to increase rate 
of conception in PCOS through improved 
insulin sensitivity. 



A polycystic ovary 



[ Q: 1014 ] MRCPass - Endocrinology 

A 25 year old lady has 
oligomenorrhoea. As part of the hormone 
screen, she has a prolactin level of 700 mU/l 
(90-520). 


Which one of the following is a couse of this 
biochemicol picture? 

1- Thyrotoxicosis 

2- Chlorpromazine 

3- Primary ovarian failure 

4- Endometrial tumour 


5- Post transsphenoidal surgery 


Hyperprolactinaemia can be caused by drugs 
(OCP or phenothiazines), hypothyroidism, liver 
or renal failure, pituitary adenoma 

/acromegaly. 

[ Q: 1015 ] MRCPass - Endocrinology 

A 53 year old man with insulin 
dependent diabetes has routine follow up. On 
examination, neovascularization w as found 
on fundoscopy. Blood pressure was 

146/92mm Hg. 

Whot is the treatment of choice? 

1- Better glycaemic control 

2- Follow up after 3 months 

3- Photocoagulation 

4- Better blood pressure control 

5- Statin 



Answer & Comments 

Answer: 3- Photocoagulation 

Treatment is directed at regressing 
neovascularisation using Argon laser pan- 
retinal photocoagulation. The new vessels 
themselves are not targeted but 
photocoagulation is spread over a wide area in 
order to destroy ischaemic retina and remove 
the vasoproliferative stimulus. There is 
variable visual loss and night blindness. 






Photocoagulation scars following laser 

treatment 


Answer & Comments 


[ Q: 1016 ] MRCPass - Endocrinology 

Answer: 2- Chlorpromazine 

A 60 year old man has had a 



Dr. Kholid Yusuf El-Zohry - Sohog Teaching Hospital (01118391123) 

Ref MRCPass OE OE 2012 PasTest 2009 PassMedicine 2009 PasTest Exam ReviseMRCP 






























El-zohry MRCP Questions Bank (Port 1) - 2013 


(For my personal use) 


thyroidectomy following a diagnosis of 
follicular carcinoma. 2 days later, he develops 
tingling sensations in his limbs and 
neuromuscular irritability. 

Which of the following is important? 

1- Free T4 

2- TSH 

3- Triidothyronine 

4- Corrected calcium 


[ Q: 1018 ] MRCPass - Endocrinology 

A 18 year old female with Addison's 
has hydrocortisone treatment with a 
dose of 20 mg in the morning and 10 mg in 
the evening. 

What dose of prednisolone would provide on 
equivalent doily dose os her hydrocortisone? 

1- 7.5 mg 

2- 10 mg 



disease 


5- Magnesium 


Answer & Comments 

Answer: 4- Corrected calcium 

The diagnosis is likely to be 
hypoparathyroidism related hypocalcaemia. 
Treatment is with intravenous calcium and 
calcitriol supplementation. Tetany and carpo 
pedal spasm may occur. 


[ Q: 1017 ] MRCPass - Endocrinology 

An 22 year old engineering student 
presents with polyuria and polydipsia. 

Which one of the following features will help 
to confirm o diagnosis of diabetes mellitus? 

1- 3+ ketonuria 

2- A fasting plasma glucose of 7.5 mmol/L 

3- A plasma glucose of 9.2 mmol/l 2 hours 
after 75 grams of oral glucose. 

4- An HbAlc of 6.5% 

5- A random plasma glucose of >9 mmol/L 



Answer & Comments 

Answer: 2- A fasting plasma glucose of 7.5 
mmol/L 


3- 12.5 mg 

4- 15 mg 

5- 20 mg 


Answer & Comments 

Answer: 1- 7.5 mg 

Equivalent dose strength of prednisolone: 
hydrocortisone is 4 : 1. The total dose of 
hydrocortisone is 30mg and prednisolone is 
7.5mg. 

[ Q: 1019 ] MRCPass - Endocrinology 

A 75 year old man has a decreased 
conscious level and is brought into hospital. 
He has a blood glucose of 45 mmol/L and 
sodium of 150 mmol/l. Urinalysis reveals no 
ketones. 

How should the patient be managed? 

1- DIGAMI regime 

2- Half normal saline and normal sliding scale 
insulin 

3- Half normal saline and reduced insulin 
sliding scale 

4- 10% dextrose infusion 

5- 5% dextrose with insulin sliding scale 



A random glucose of >11.1 and a fasting 
glucose of >7.0 mmol/L would be regarded as 
confirmatory. A raised glycosolated 
haemoglobin (HbAlc) is also highly suggestive 
but diagnostic. 


Answer & Comments 

Answer: 3- Half normal saline and reduced 
insulin sliding scale 


Dr. Khalid Yusuf El-Zohry - Sohag Teaching Hospital (01118391123) 

Ref MRCPass OE OE 2012 PasTest 2009 PassMedicine 2009 PasTest Exam 




ReviseMRCP 

435 




























El-zohry MRCP Questions Bank (Port 1) - 2013 


(For my personal use) 


The patient has hyperosmolar non ketotic 
coma. When sodium is greater than 145 
mmol/l, the patient should be given half 
normal saline, and a sliding scale with low er 
insulin doses (there is increased insulin 
sensitivity). 


[ Q: 1020 ] MRCPass - Endocrinology 

A 70 year old man has who 
presented with palpitations has runs of non 
sustained ventricular tachycardia during 
telemetry. Investigations show a serum 
magnesium of 0.3 mmol/l (0.7-1.5). 

Which one of the following is likely to have 
caused this biochemical abnormality? 

1- Elevated PTH level 

2- Diuretics 

3- Hyperphosphataemia 

4- Chronic renal failure 

5- Antacids 



Answer & Comments 


Answer: 2- Diuretics 


3- Phaeochromocytoma 

4- Hepatocellular carcinoma 

5- Addison's disease 


Answer & Comments 

Answer: 5- Addison's disease 

Addison's disease is described. Many 
autoimmune diseases are associated e.g. 
vitiligo, diabetes, primary ovarian failure and 
pernicious anaemia. 

[ Q: 1022 ] MRCPass - Endocrinology 

A 45 year old man presents to his GP 
complaining a disseminated, asymptomatic 
papular eruption. Physical examination 
revealed obesity with hundreds of pink to 
yellow ish, soft papules located over the 
buttocks, knees, elbows and neck. 

He is found to have a triglyceride level of 5.2 
(<1.69) mmol/l cholesterol 6.5 (<5.2) mmol/l. 

Which one of the following is most likely to be 
associated? 

1- Nephrotic syndrome 



Hypomagnesaemia may lead to life 
threatening ventricular arrhythmias especially 
in conjunction with hypokalaemia. 

Posssible causes of hypomagnesaemia are: 
Alcoholism, Drugs (Loop Diuretics, 
Gentamicin, Cisplatin), Gastrointestinal 
disorders (Vomiting, Diarrhoea, 

Malabsorption), Renal loss, Hypercalcaemia, 
Renal tubular acidosis. 

[ Q: 1021 ] MRCPass - Endocrinology 

fi - 

# A 35 year old lady has skin 
pigmentation, hypotension, hyponatraemia. A 
short synacthen test shows a rise in cortisol 
from 100 to 140 pg/ml. 

What is the diagnosis? 

1- Papillary thyroid carcinoma 

2- Ovarian fibroids 


2- Loop diuretic 

3- Amyloidosis 

4- Kartagener's syndrome 

5- Hyperthyroidism 

Answer & Comments 

Answer: 1- Nephrotic syndrome 

List of causes of raised triglycerides are 

nephrotic syndrome 

hypothyroidism 

steroids 

diabetes mellitus 

renal failure 

oral contraceptive pill 


Dr. Khalid Yusuf El-Zohry - Sohag Teaching Hospital (01118391123) 

Ref MRCPass OE OE 2012 PasTest 2009 PassMedicine 2009 PasTest Exam ReviseMRCP 



436 




























El-zohry MRCP Questions Bank (Port 1) - 2013 


(For my personal use) 


glycogen storage disease (Von Gierke's) 

This patient has eruptive xanthomas, have 
also been reported in patients with acquired 
forms of hypertriglyceridemia secondary to 
uncontrolled diabetes mellitus, nephrotic 
syndrome, alcohol and drug abuse. 

In combination with elevated cholesterol, 
nephrotic syndrome is a likely diagnosis. 



Eruptive Xanthoma 


[ Q: 1023 ] MRCPass - Endocrinology 

An 70 year old lady is admitted with 
loss of consciousness. She has a temperature 
of 35 C, has a heart rate of 55, evidence of 
cardiac failure, hypoventilation, 

hypoglycaemia and hyponatraemia. 

Which medication doses should be 
administered? 

1- Free T3 40 micrograms daily 

2- Dexamethasone 8 mg orally daily 

3- Hydrocortisone 100 mgs iv 8 hourly 

4- Oral thyroxine 125 micrograms daily 

5- Levothyroxine 2000 micrograms by slow 
intravenous infusion every 8 hours 



Answer & Comments 

Answer: 3- Hydrocortisone 100 mgs iv 8 hourly 

The patient has myxoedema coma. 
Hydrocortisone 100 mgs iv 8 hourly should be 
used to protect against the possibility of 
associated adrenocortical deficiency. An initial 
levothyroxine (T4) dose of 100 to 500 ?g 
administered intravenously should be 


followed by 75 to 100 ?g administered 
intravenously daily until the patient is able to 
take oral replacement. 

[ Q: 1024 ] MRCPass - Endocrinology 

A 52 year old man has been to her 
GP a week ago for pharyngitis. He presents to 
casualty feeling unwell with fevers and he has 
diffuse, widespread areas of purpuric rash all 
over his body. 

Investigations show : 

Sodium 125 mmol/L (135-145) 

Potassium 5.2 mmol/L (3.5-5.0) 

Chloride 91 mmol/L (95-105) 

Glucose 2.6 mmol/L (3.0-5.5) 

The most likely diagnosis is: 

1- MEN 2 

2- Waterhouse-Friderichson syndrome 

3- Addison's disease 

4- Sheehan's syndrome 

5- Alkaptonuria 



Answer & Comments 

Answer: 2- Waterhouse-Friderichson 

syndrome 

Waterhouse-Friderichson syndrome is due to 
meningococcal septicaemia resulting in 
adrenal haemorrhagic insufficiency. 

Symptoms and signs include hypotension, 
abdominal pain, hyponatremia. The 
progression is rapid. There may also be 
disseminated intravascular coagulation in 
patients with this syndrome. Mortality may 
approach 100%. 

The treatment is as that for meningococcal 
infection, but with the addition of adrenal 
support with hydrocortisone, given 
intravenously in a dose of 200 mg per four 
hours. 


Dr. Khalid Yusuf El-Zohry - Sohag Teaching Hospital (01118391123) 

Ref MRCPass OE OE 2012 PasTest 2009 PassMedicine 2009 PasTest Exam 




ReviseMRCP 

437 
























El-zohry MRCP Questions Bank (Port 1) - 2013 


(For my personal use) 



Diffuse purpuric rash in Waterhouse- 
Friderichson syndrome 


[ Q: 1025 ] MRCPass - Endocrinology 

A previously fit 36 year old man 
presents with a 3 month history of weight 
loss, pallor and lethargy.There is no 

relevant past medical history and he does not 
take regular medication. His blood pressure 
was 103/56 mmHg with 

a 20 mm Hg postural change measured. He 
had unremarkable neurological and systemic 
examination with a 

normal computerised tomography and 
magnetic resonance imaging of the brain. 
Investigations show : 

Haemoglobin 10.5 g/dL 

serum sodium 128 mmol/L 

serum potassium 6.2 mmol/L 

serum urea 4.0 mmol/L 

serum creatinine 85 umol/L 

serum Total T4 60 nmol/L (50 - 150) 

serum TSH 9 mU/L (0.2 -5.5) 

Which of the following is the most useful 
diagnostic investigation? 

1- Anti-thyroid peroxidase antibody titre 

2- Short synacthen test 

3- Free T3 concentration 

4- Glucose tolerance test 

5- Growth hormone level 



Answer & Comments 

Answer: 2- Short synacthen test 

Hyponatraemia and mild hypothyroidism 
suggests that this patient might have 
Addison's disease. The clinical history of 
lethargy, weight loss and postural hypotension 
also fits the diagnosis. 


[ Q: 1026 ] MRCPass - Endocrinology 

A 35 year old secretary has a blood 
pressure of about 200/100 mmHg on several 
occasions. She complains of polyuria, and 
muscle weakness. 

Investigations reveal: 

sodium 146 mmol/L 

potassium 2.4 mmol/L 

Chloride 105 mmol/L 

Bicarbonate 31 mmol/L 

Urea 8.4 mmol/l 

Which one of the following conditions is most 
likely? 

1- Medullary thyroid carcinoma 

2- Pancreatic tumour 

3- Adrenal adenoma 

4- Adrenal hyperplasia 

5- Hypoaldosteronism 



Answer & Comments 

Answer: 3- Adrenal adenoma 

The features of high sodium and low 
poatassium along with hypertension are 
suggestive of primary hyperaldosteronism. 
The most common cause of primary 
hyperaldosteronism is unilateral adenoma 
(Conn's syndrome ). 


Dr. Khalid Yusuf El-Zohry - Sohag Teaching Hospital (01118391123) 

Ref MRCPass OE OE 2012 PasTest 2009 PassMedicine 2009 PasTest Exam ReviseMRCP 



438 




























El-zohry MRCP Questions Bank (Port 1) - 2013 


(For my personal use) 



An adrenal adenoma 


[ Q: 1027 ] MRCPass - Endocrinology 

A 22 year old girl is admitted with a 

3-day history of rigors due to a urinary tract 
infection. On examination, she has a 
temperature of 39°C, looks unwell, has a body 
mass index of 30 kg/m * 1 2 3 4 5 . 

Initial investigations reveal: 

Potassium 4.2 mmol/L (3.5-5) 

Urea 6 mmol/L (2.5-7) 

Glucose 30 mmol/L (3.0-6.0) 

pH 7.4 (7.36-7.44) 

Standard bicarbonate 12 mmol/l 

Base deficit -12 

Which one of the following is the best initial 
treatment? 

1- Metformin 

2- Gliclazide 

3- Acarbose 

4- Sliding scale IV insulin infusion 

5- Glargine insulin 



breast-feed following delivery of a baby. Her 
pregnancy was complicated by severe 
intrapartum bleeding due to an undiagnosed 
placenta praevia. She remains amenorrhoeic 3 
months after delivery. She complains of 
extreme lethargy. 

Investigations show : sodium 129 mmol/l, 
potassium 3.4 mmol/l, urea 5 pmol/l, 
creatinine 90 pmol/l. 

The diagnosis is likely to be: 

1- Bilateral adrenal infarction 

2- Normal pregnancy physiology 

3- Prolactinoma 

4- Multiple endocrine neoplasia 

5- Pituitary apoplexy 

Answer & Comments 

Answer: 5- Pituitary apoplexy 

The word apoplexy is defined as a sudden 
neurologic impairment, usually due to a 
vascular process. Pituitary apoplexy is 
characterized by sudden onset of headache, 
visual symptoms, altered mental status, and 
hormonal dysfunction due to acute 
hemorrhage or infarction of a pituitary gland. 
The pituitary gland is susceptible to infarction 
during pregnancy due to its increased 
size/blood flow . Hyponatraemia, 
hyperkalaemia and hypoglycaemia are as a 
result of secondary adrenal failure due to lack 
of ACTH. 


Answer & Comments 

Answer: 4- Sliding scale IV insulin infusion 

The patient has mild acidodis and 
hyperglycaemia, suggestive of diabetic 
ketoacidosis, and hence should be on a sliding 
scale insulin regime. 



[ Q: 1028 ] MRCPass - Endocrinology 
A 25 year old lady is unable to 


Dr. Khalid Yusuf El-Zohry - Sohag Teaching Hospital (01118391123) 

Ref MRCPass OE OE 2012 PasTest 2009 PassMedicine 2009 PasTest Exam ReviseMRCP 


























El-zohry MRCP Questions Bank (Port 1) - 2013 


(For my personal use) 



MRI showing a heterogenous pituitary mass in 
the sella, which is non enhancing - Pituitary 

Apoplexy 



[ Q: 1029 ] MRCPass - Endocrinology 

A 60 year old woman with a past 
medical history of hypertension is assessed. 
She has a 45 pack year smoking history. She 
comes to the urgent care clinic today 
complaining of a cough and shortness of 
breath for the past w eek. Her physical exam is 
notable for both mild wheezing and rhonchi, 
more pronounced on the right side than the 
left. 


Lab results include the following: 
Na 126 mmol/I 
K 4.4 mmol/l 
Creatinine 120 umol/l 
Glucose 6 mmol/l 
Urine osmolality is 400 mosm/kg 
Plasma osmolality 285 mosm/kg 
What is the diagnosis? 

1- Renal salt wasting 

2- Cranial diabetes insipidus 

3- Nephrogenic diabetes insipidus 

4- SIADH 


5- Psychogenic polydipsia 


Answer & Comments 


Answer: 4- SIADH 

SIADH is confirmed by inappropriately 
elevated urine osmolality (often above 300 
mOsm/kg) and urine sodium concentration 
(usually above 20 mEq/liter). This case 
scenario is consistent with a lung carcinoma. 

* Hyponatremia with hypo-osmolality 

* Elevated renal excretion of sodium (> 20 mEq/L) 

* Normal volume status 

* Inappropriately elevated urine osmolality for the plasma 

osmolality 


SIADH 


[ Q: 1030 ] MRCPass - Endocrinology 

A 40 year old patient has 
investigation for Cushingoid features of 
buffalo hump and central obesity. Serum 
potassium is low at 2.5 mmol. Her ACTH value 
is 200 pg/ml (5-50) and 24 hour urine cortisol 
is grossly elevated. 

Which one of the following is the most likely 
diagnosis? 

1- Adrenal carcinoma 

2- Adrenal adenoma 

3- Pituitary dependent Cushing's 

4- Ectopic ACTH syndrome 

5- ACTH overdose 



Answer & Comments 

Answer: 4- Ectopic ACTH syndrome 

In ectopic ACTH syndrome, hypokalaemic 
alkalosis is typical. Ectopic ACTH is not 
suppressed by high doses of steroids such as 8 
mg dexamethasome. Hypocalcaemia is not 
caused. 

Ectopic ACTH is typically caused by small cell 
carcinoma of the lung. Thymoma, carcinoid 
tumour, medullary carcinoma of the thyroid, 
pancreatic carcinoma and 

phaeochromocytoma are associated with 
ectopic ACTH secretion. Squamous cell 



Dr. Khalid Yusuf El-Zohry - Sohag Teaching Hospital (01118391123) 

Ref MRCPass OE OE 2012 PasTest 2009 PassMedicine 2009 PasTest Exam ReviseMRCP 
























El-zohry MRCP Questions Bank (Port 1) - 2013 


(For my personal use) 


carcinoma is frequently associated with 
hypercalcaemia. 


[ Q: 1031 ] MRCPass - Endocrinology 

/ - 

# A 50 year old woman has weight 

loss, palpitations and diarrhoea. 

Examination reveals a single nodule on the left 
lobe of the thyroid, measuring 3 cm in size. 
Thyroid scintigraphy with Tc99 shows 
increased uptake within this nodule. 

Thyroid function tests showed a free thyroxine 
of 29 pmol/L (9-25 pmol/L) and TSH <0.04 
mU/L (0.5-5). 

What is the best management? 

1- Carbimazole 

2- Propylthiouracil 

3- Total thyroidectomy 

4- Partial thyroidectomy 

5- Radioactive iodine 


Answer & Comments 

Answer: 5- Radioactive iodine 

The technetium scan suggests that the 
thyrotoxicosis is due to a solitary toxic nodule. 
Toxic thyroid nodules are best treated with 
radioactive iodine. 


[ Q: 1032 ] MRCPass - Endocrinology 

A 44 year old man is investigated for 
hypertension. His blood pressure is 
consistently above 180/90 mmHg. 

Blood results show : 

pH 7.5 

p0 2 -13 kPa 

pC0 2 - 4 kPa 

bicarbonate 32 (20-28) mmol/I 
sodium 138 mmol/I 
potassium 2.7 mmol/l 
urea 6 pmol/l 



creatinine 100 pmol/l 
What is the likely diagnosis? 

1- Essential hypertension 

2- Conn's syndrome 

3- Secondary hyperaldosteronism 

4- Phaeochromocytoma 

5- Congenital adrenal hyperplasia 


Answer & Comments 

Answer: 2- Conn's syndrome 

Hypokalaemic alkalosis with refractory 
hypertension suggests primary 

hyperaldosteronism (Conn's syndrome). 

Secondary hyperaldosteronism would also be 
possible but tends to cause hypertension 
which is easier to control. 

Liddle's syndrome and Bartter's syndrome also 
cause hypokalaemic alkalosis. If this case 
scenario was a child with precocious puberty 
then consider congenital adrenal hyperplasia. 

[ Q: 1033 ] MRCPass - Endocrinology 

A 35 year old lady has had no 
periods for tw o years. She also notices 
increased hair growth. Examination reveals 
male pattern balding and hair grow th, and 
clitoromegaly. 

What is the likely diagnosis? 

1- Cushing's disease 

2- Adrenal tumour 

3- MEN 

4- Congenital adrenal hyperplasia 

5- Ovarian tumour 



Answer & Comments 

Answer: 2- Adrenal tumour 

An adrenal tumour is most likely, with 
androgen secretion. Virilisation occurs. These 


Dr. Khalid Yusuf El-Zohry - Sohag Teaching Hospital (01118391123) 

Ref MRCPass OE OE 2012 PasTest 2009 PassMedicine 2009 PasTest Exam 




ReviseMRCP 

441 


























El-zohry MRCP Questions Bank (Port 1) - 2013 


(For my personal use) 


are aggressive tumours and the treatment 
option is surgery or radiotherapy. 



MRI of a Large Right adrenal tumour 


[ Q: 1034 ] MRCPass - Endocrinology 

A 55 year old man presents with 
generalised lethary, dizziness, headache, 
visual disturbance, impotence and decreased 
facial hair. 

On examination he has a blood pressure of 
110/70 with a postural drop. 

On examination of his visual fields he is found 
to have a bitemporal hemianopia. The 
prolactin level is 4500 mlU/L. 

An insulin tolerance test is performed and the 
blood glucose levels falls to 1.3 mmol/L and 
the patient becomes sweaty and tachycardic. 

What is the diagnosis? 

1- Medullary thyroid tumour 

2- Addison's disease 

3- Adrenal tumour 

4- Phaeochromocytoma 

5- Pituitary macroadenoma 



This patient is likely to have an impaired 
response to the insulin tolerance test as 
prolactinoma may cause hyposecrestion of 
other hormones. It is likely that the patient 
has ACTH and cortisol deficiency as he has 
features that are suggestive (tiredness, 
dizziness, postural BP drop). 



MRI showing a macroadenoma 


[ Q: 1035 ] MRCPass - Endocrinology 

A 60 year man has headaches and 
visual impairment. He is suspected of having 
acromegaly. 

Which one of following is the best 
investigation to confirm the diagnosis? 

1- Insulin like growth factor 1 (IGF1) 

2- Growth hormone releasing hormone test 

3- 9 am growth hormone concentrations 

4- Glucose tolerance test with growth 
hormone concentration 

5- Insulin tolerance test with growth hormone 
concentrations 



Answer & Comments 

Answer: 5- Pituitary macroadenoma 

The combination of headache, bitemporal 
hemianopia and the prolactin level of > 3,600 
mlU/L suggests the patient has a 
macroadenoma. 


Answer & Comments 

Answer: 4- Glucose tolerance test with growth 
hormone concentration 

The diagnosis of acromegaly is confirmed by 
inadequate suppression of Growth Hormone 
concentrations below 2 mll/l in an oral 
glucose tolerance test. 



Dr. Khalid Yusuf El-Zohry - Sohag Teaching Hospital (01118391123) 

Ref MRCPass OE OE 2012 PasTest 2009 PassMedicine 2009 PasTest Exam ReviseMRCP 



























El-zohry MRCP Questions Bank (Port 1) - 2013 


(For my personal use) 


[ Q: 1036 ] MRCPass - Endocrinology 

A 45 year old woman has 
palpitations, tremors, and episodes of anxiety 
with associated sweating and confusion. There 
is no history of diabetes. Blood insulin levels 
are increased with increased C peptide levels 
when measured during one of these episodes. 

It is most likely due to: 

1- Alcohol intoxication 

2- Exogenous insulin 

3- End stage renal disease 

4- Insulinoma 

5- Sepsis 



Answer & Comments 

Answer: 4- Insulinoma 

Insulinoma caused high endogenous insulin 
secretion and increased C peptide levels. This 
would cause hypoglycaemia and symptoms 
such as those listed above. 


A raised aldosterone level and suppressed 
renin levels suggest primary rather than 
secondary hyperaldosteronism. 


[ Q: 1038 ] MRCPass - Endocrinology 

A 42 year woman presents with 
episodic sw eats and tremors which are 
relieved by sugary drinks. She gained 
approximately 5 kg in weight in the past 3 
months. 

Her investigations show normal full blood 
count, normal urea and electrolytes, and a 
fasting plasma glucose concetration of 3.5 
mmol/I. 

Whot is the most appropriate investigation for 
this patient? 

1- Water deprevation test 

2- 72 hour fast 

3- Oral glucose tolerance test 

4- CT scan of the head 

5- Random Insulin C peptide concentration 



[ Q: 1037 ] MRCPass - Endocrinology 

A 35 year old lady was referred from 
her GP for hypertension. Her plasma K is 2.8 
mmol/L. Her BP was 210/130. 

Fundoscopy revealed a bilateral papilloedema. 
Investigations show : Supine Plasma 
aldosterone 650 (100-450) Erect 30mins 620 
pmol/L Supine Renin activity <0.2 (1.1-2.7), 
Erect <0.2 (2.8-4.5). 

The underlying diagnosis is: 

1- Liddle's syndrome 

2- Bartter's syndrome 

3- Addison's disease 



Answer & Comments 

Answer: 2- 72 hour fast 

The diagnosis is insulinoma. The standard 
method of clinching the diagnosis during a 72 
hour fast is by the demonstration of 
inappropriately high insulin C peptide during 
spontaneous hypoglycaemia. 

Measurement of C peptide is also useful in 
excluding factitious hypoglycaemia from self 
injection of insulin. Insulin preparations do not 
contain C peptide, hence if insulin levels were 
high and C peptide levels were undetectable 
then exogenous insulin abuse is likely. 


4- Secondary hyperaldosteronism 

5- Primary hyperaldosteronism 


Answer & Comments 


Answer: 5- Primary hyperaldosteronism 



[ Q: 1039 ] MRCPass - Endocrinology 

A 35 year old woman with 
thyrotoxicosis is commenced on carbimazole. 


Which one of the following is the first blood 
test to improve? 


Dr. Khalid Yusuf El-Zohry - Sohag Teaching Hospital (01118391123) 

Ref MRCPass OE OE 2012 PasTest 2009 PassMedicine 2009 PasTest Exam ReviseMRCP 

































El-zohry MRCP Questions Bank (Port 1) - 2013 


(For my personal use) 


1- Thyroid stimulating hormone 

2- Free T3 

3- Total T4 

4- Thyroglobulin 

5- Total T3 

Answer & Comments 

Answer: 3- Total T4 

Thyroxine (T4) is formed by coupling of 
iodinated tyrosine residues within 
thyroglobulin (TG). 

Carbimazole acts by blocking the iodination of 
tyrosine residues, hence reducing levels of T4. 


[ Q: 1041 ] MRCPass - Endocrinology 

* A 45 year old woman has undergone 
investigations following routine blood tests 
showing hypercalcaemia. 

She has no symptoms. She has a calcium of 3.0 
mmol/I (range 2.4-2.6 mmol/l), and 24-hour 
urinary calcium excretion 25 mg (100-300). 

What is the likely diagnosis? 

1- Primary hyperparathyroidism 

2- Secondary hyperparathyroidism 

3- Multiple myeloma 

4- Familial hypocalciuric hypercalcaemia 

5- Widespread bony metastases 


[ Q: 1040 ] MRCPass - Endocrinology 

A 35 year old woman has had a 
blood pressure of 180/105 mmHg for the past 
month. She has also had headaches, 
palpitations and sweaty episodes several 
times a day. Urine adrenaline is 350 (<80) 
nmol/l and noradrenaline is 2300 (<780) 
nmol/l. 

Which one of the following is the likely 
diagnosis? 

1- Renal artery stenosis 

2- Conn's syndrome 

3- Essential hypertension 

4- Phaeochromocytoma 

5- White coat syndrome 



Answer & Comments 

Answer: 4- Phaeochromocytoma 

The diagnosis is phaeochromocytoma. 
Although paraganglioma (10% of 
phaeochromocytomas are extra-adrenal) 
should be considered, it is associated with a 
raised noradrenaline. Both raised adrenaline 
and noradrenaline should make one suspect 
adrenal phaeochromocytoma. 


Answer & Comments 

Answer: 4- Familial hypocalciuric 

hypercalcaemia 

The diagnosis here is familial hypocalciuric 
hypercalcaemia. 


Familial hypocalciuric hypercalcaemia is a rare 
autosomal dominant disorder which has only 
recently been recognised in which there is 
reduced urinary calcium excretion in the 
presence of hypercalcaemia. 


Diagnosis is made on family history and 
determination of low urinary calcium 
clearance (this does not happen in 
hyperparathyroidism). 


Familial Hypocalciuric Hypercalcemia 

Hypercalcemia with normal or low urinary 
calcium 

iPTH levels mav be elevated, but 
hvpocalciuria is not dependent on PTH 

Do not form calcium stones 

Urinarv Ca/Creatinine clearance ratio of 01 
* 

or below helps distinguish from 
liyperparathyroidism 



[ Q: 1042 ] MRCPass - Endocrinology 


Dr. Khalid Yusuf El-Zohry - Sohag Teaching Hospital (01118391123) 

Ref MRCPass OE OE 2012 PasTest 2009 PassMedicine 2009 PasTest Exam ReviseMRCP 



444 




























El-zohry MRCP Questions Bank (Port 1) - 2013 


(For my personal use) 


A 20 year old lady presents to casualty with 
dizziness, lethargy, nausea and vomiting. On 
examination, she has a BP of 95/50 mmHg and 
generalised hyperpigmentation. Blood tests 
showed: Na 129 mmol/l K 5.8 mmol/I, Urea 
6.5 umol/l, Creat 110 umol/l. 

The underlying diagnosis is likely to be: 

1- Acute intermittent porphyria 

2- Hypoparathyroidism 

3- Hypothyroidism 

4- Secondary adrenal insufficiency 

5- Primary adrenal insufficiency 


Answer & Comments 

Answer: 5- Primary adrenal insufficiency 

The clinical picture is consistent with an 
addisonian picture. ACTH is produced in 
primary adrenal insufficiency, hence causing 
hyperpigmentation. This would not be 
expected in secondary adrenal insufficiency. 


pituitary. There may also be a gradual loss of 
sexual function or libido. 


[ Q: 1044 ] MRCPass - Endocrinology 

A 60 year old man is found to have a 
blood pressure of 185/110 mmHg. Serum 
biochemistry showed normal electrolytes, and 
normal renal function. An abdominal 
ultrasound scan show s a 5-cm right adrenal 
mass. Adrenal surgery is scheduled. 

Which one of the following tests would hove 
been most helpful prior to surgery? 

1- Short synacthen test 

2- 24 hour urinary cortisol 

3- MRI scan of the abdomen 

4- Renin and aldosterone levels 

5- Urinary free catecholamines 



Answer & Comments 


Answer: 5- Urinary free catecholamines 


[ Q: 1043 ] MRCPass - Endocrinology 

Which one of the following is the 
commonest presentation of o proloctinomo in 
moles? 

1- Galactorrhoea 

2- Impotence 

3- Gynaecomastia 

4- Obesity 

5- Depression 



Answer & Comments 

Answer: 2- Impotence 

In men with prolactinoma the commonest of 
the features mentioned above is impotence. 

Because men have no reliable indicator such 
as menstruation, many men delay seeking 
medical advice until they have headaches or 
visual problems caused by the enlarged 


Urinary catecholamines would confirm a 
diagnosis of a secretory phaeochromocytoma. 
If so, the patient needs alpha blockade prior to 
beta blockade, then surgery. MRI of the 
abdomen or MIBG scan is also helpful 
following biochemical confirmation of the 
diagnosis. 



[ Q: 1045 ] MRCPass - Endocrinology 

A 60 year old woman who complains 
tiredness. Her relatives mentioned depression 
and has been on medication. 


She has also been treated for cardiac failure 
with diuretics. Her blood tests reveal a 
corrected calcium 2.90 (2.2-2.7) mmol/l. 

Which one of the following is most likely to 
hove caused the raised calcium? 


1- Frusemide 


2- Paroxetine 

3- Lisinopril 


Dr. Khalid Yusuf El-Zohry - Sohag Teaching Hospital (01118391123) 

Ref MRCPass OE OE 2012 PasTest 2009 PassMedicine 2009 PasTest Exam 




ReviseMRCP 

445 




























El-zohry MRCP Questions Bank (Port 1) - 2013 


(For my personal use) 


4- Vitamin D deficiency 

5- Lithium 


4- Carbimazole 

5- Radioiodine therapy 


Answer & Comments 


Answer: 5- Lithium 


Answer & Comments 


Answer: 4- Carbimazole 


Drugs producing hypercalcemia include 
lithium, alkaline antacids, DES, diuretics 
(chronic administration of thiazides), 
estrogens (incl. oral contraceptives), and 
progesterone. 


[ Q: 1046 ] MRCPass - Endocrinology 

What dose of prednisolone is 
equivalent in its glucocorticoid potency to 100 
mg of hydrocortisone? 

1- 5 mg 

2- 25 mg 

3- 50 mg 

4- 75 mg 

5- 100 mg 



Answer & Comments 

Answer: 2- 25 mg 

Prednisolone is 4 x more potent than 
hydrocortisone, and dexamethasone is 
between 40 x more potent than 
hydrocortisone. 


[ Q: 1047 ] MRCPass - Endocrinology 

A 60 year old man has been taking 
amiodarone for paroxysmal atrial fbrillation. 
He presents with lethargy and weight loss of 
half a stone over 2 months. Investigations 
reveal: Free T 4 35 pmol/L (10-22), TSH <0.01 
mU/L (0.5-4.5). 

How should this patient be treated? 

1- Thyroxine 

2- Thyroidectomy 

3- Hydrocortisone 



The appropriate initial treatment of 
amiodarone induced hyperthyroidism would 
be carbimazole. 

Two types of amiodarone induced 
hyperthyroidism are recognised. The first is a 
consequence of iodine overload contained 
within amiodarone of which the above is a 
typical example. The second type is due to an 
acute thyroiditis. This is best treated by 
prednisolone. 



[ Q: 1048 ] MRCPass - Endocrinology 

A 42 year old lady presents with 
malaise, thirst and increasing nocturia. The 
symptoms have worsened over the last 2 
months. She had an episode of renal colic 
previously. Her GP had noted an eruptive, 
painful, erythematous rash on her anterior 
shins as well previously. 


What is the likely metabolic abnormality? 


1- Hypomagnesaemia 

2- Hyperglycaemia 

3- Hypercalcaemia 

4- Hyponatraemia 

5- Hyperoxaluria 


Answer & Comments 

Answer: 3- Hypercalcaemia 

This lady has sarcoidosis. Hypercalcemia in 
sarcoidosis is due to the uncontrolled 
synthesis of 1,25-dihydroxyvitamin D3 by 
macrophages. There is associated renal calculi 
due to hypercalcaemia and also erythema 
nodosum. 


Dr. Khalid Yusuf El-Zohry - Sohag Teaching Hospital (01118391123) 

Ref MRCPass OE OE 2012 PasTest 2009 PassMedicine 2009 PasTest Exam ReviseMRCP 



446 































El-zohry MRCP Questions Bank (Port 1) - 2013 


(For my personal use) 


[ Q: 1049 ] MRCPass - Endocrinology 

A 55 year old patient with type 2 
diabetes is reviewed in the diabetic clinic. 

Which of the following is o feature of diobetic 
neuropathy to watch out for? 

1- Cervical myelopathy 

2- Brisk reflexes 

3- Muscle hypertrophy 

4- Loss of vibration sense 

5- Myotonia 



Answer & Comments 

Answer: 4- Loss of vibration sense 

Autonomic neuropathy to the gut, bladder 
and sexual organs (impotence) can occur. A 
3rd nerve mononeuropathy can occur. Motor 
neuropathy can cause muscle wasting, and 
sensory neuropathy causes vibration sensory 
loss. With myotonia, prolonged contraction of 
muscle fibres associated with muscle 
dystrophy is due to genetic causes of muscle 
protein abnormality and is not neurologically 
dependent. 


[ Q: 1050 ] MRCPass - Endocrinology 

A 40 year old patient who is diabetic 
is concerned about having kidney problems in 
the future. He has 1+ proteinuria on urine 
dipstick. 

Which of the following is most important in 
order to preserve renal function? 

1- Tight glycaemic control 

2- Tight hypertension control 

3- Going on to insulin 

4- Regular renal ultrasound and 24 hour urine 
check 

5- Calcium replacement 



Aggressive hypertension control is the best 
way of preventing progression from 
microalbuminuria to macroalbuminuria. 
Although glycaemic control is important, it is 
not as important as hypertensive control in 
preventing progression tow ards nephropathy. 


[ Q: 1051 ] MRCPass - Endocrinology 

A 27-year-old female has been 
diagnosed as having thyrotoxicosis and has 
been started on carbimazole. 

Which one of the following is a major side 
effect of carbimazole? 

1- Agranulocytosis 

2- Thrombocytosis 

3- Cholangiocarcinoma 

4- Pityriasis rosea 

5- Vascular thrombosis 



Answer & Comments 

Answer: 1- Agranulocytosis 

Major side effects of carbimazole are: 
agranulocytosis, thrombocytopaenia, acute 
hepatic necrosis, cholestatic hepatitis, lupus¬ 
like syndrome and vasculitis. 


\7 

[ Q: 1052 ] MRCPass - Endocrinology 


A 40 year old man with hypertension 


was also found to be significantly 
hypokalemic. 

Which of the following investigation is most 
appropriate? 

1- Renal arteriography 

2- Ultrasound of the abdomen 

3- Renin: Aldosterone Ratio 

4- Plasma ACTH 

5- Plasma cortisol level 


Answer & Comments 


Answer: 2- Tight hypertension control 


Answer & Comments 


Answer: 3- Renin: Aldosterone Ratio 


Dr. Khalid Yusuf El-Zohry - Sohag Teaching Hospital (01118391123) 

Ref MRCPass OE OE 2012 PasTest 2009 PassMedicine 2009 PasTest Exam ReviseMRCP 



447 




































El-zohry MRCP Questions Bank (Port 1) - 2013 


(For my personal use) 


Conn's syndrome should be considered in a 
patient who is not on diuretics who has the 
following features: 

hypertension, hypokalaemia and alkalosis. 
Investigation of choice is plasma renin and 
aldosterone - low renin and high aldosterone 
(raised aldosterone: renin ratio) suggests 
primary hyperaldosteronism. 


2- Cyclical oestrogen and progestogen 

3- Calcium and vitamin D supplements 

4- Etidronate 

5- Calcium supplements 

Answer & Comments 

Answer: 2- Cyclical oestrogen and progestogen 



[ Q: 1053 ] MRCPass - Endocrinology 

A 50 year old lady has a history of 
breast cancer 5 years ago. She now has the 
following investigation results: 


calcium 2.9 (2.25-2.7) mmo 


phosphate 0.75 (0.8-8) pmol/l 
Parathyroid hormone 5.1 (0.8-8) pmo 


Skeletal survey - generalised osteopenia 
What is the diagnosis? 

1- Primary hyperparathyroidism 

2- Breast cancer metastasis 

3- Multiple myeloma 

4- Vitamin D overdose 


5- Paget's disease 


Answer & Comments 

Answer: 2- Breast cancer metastasis 

The high calcium and slightly low phosphate 
would be consistent with high PTH, but the 
PTH level is normal. Bone metastases can 
cause PTH related peptide which would least 
to hypercalcaemia. 


As she is just recently post menopausal, 
combined HRT treatment is the most 
appropriate. At a later stage, calcium and 
vitamin D supplements would be beneficial. 
Etidronate is licensed for the prevention of 
further osteoporotic fractures and as 
prophylaxis against corticosteroid-induced 
osteoporosis. 



[ Q: 1055 ] MRCPass - Endocrinology 

A 65 year old woman has had 
increasing cold intolerance and weight gain. 
Observations include sinus bradycardia, 
depressed reflexes and a diffuse goitre. 


Her serum TSH is 9 mll/L (0.5-4) with a free 
thyroxine of 7.2 pmol/L (9-25 pmol/L). Anti¬ 
thyroid peroxidase antibodies and anti- 
microsomal autoantibodies were detected at 
high titres. 


What is the diagnosis? 


1- Hashimoto's thyroiditis 

2- Previous carbimazole therapy 


3- Pituitary tumour 

4- Follicular thyroid carcinoma 

5- De Quervain's thyroiditis 


[ Q: 1054 ] MRCPass - Endocrinology 

A 60 year old woman is assessed in 
the rheumatology clinic for osteoporosis. She 
is 1 and a half years post menopausal and has 
a family history of osteoporosis. 

What therapy should be commenced? 

1- Vitamin D supplements 


Answer & Comments 

Answer: 1- Hashimoto's thyroiditis 

The clinical picture is hypothyroidism. This is 
likely to be due to primary autoimmune 
hypothyroidism, also know n as Hashimoto's 
thyroiditis. This disorder occurs most 
commonly in middle-aged women and is 


Dr. Khalid Yusuf El-Zohry - Sohag Teaching Hospital (01118391123) 

Ref MRCPass OE OE 2012 PasTest 2009 PassMedicine 2009 PasTest Exam ReviseMRCP 





























El-zohry MRCP Questions Bank (Port 1) - 2013 


(For my personal use) 


caused by the reaction of the immune system 
against the thyroid gland. It may occur in 
people with a family history of thyroid 
diseases or with other autoimmune diseases, 
especially type 1 diabetes or adrenal 
insufficiency. 

De Quervain's thyroiditis (less likely diagnosis 
in this case) is a subacute painful thyroiditis 
and may result in hypothyroidism in the long 
term. 

[ Q: 1056 ] MRCPass - Endocrinology 

A 60 year old woman has received 
radioactive iodine over five years ago. She 
now comes for her annual thyroid function 
assessment. Her results reveal: Free Thyroxine 
11 pmol/l (9.8-23), TSH 14 mU/l (0.5-4.5 
mU/l), Total cholesterol 6.5 mmol/I (<5 
mmol/l), Plasma triglycerides 2.1 mmol/I (<2 
mmol/I). 

Whot is the most appropriate treatment for 
this patient's dyslipidaemia? 

1- Hormone replacement therapy 

2- Simvastatin 

3- Thyroxine 

4- Carbimazole 

5- Fibrate 



Answer & Comments 

Answer: 3- Thyroxine 

The patient has subclinical hypothyroidism as 
reflected by a normal T4 but elevated TSH. A 
hypercholesterolaemia or 

hypertriglyceridaemia is frequently associated 
due impaired lipoprotein lipase function. 

Treatment should be with thyroxine 
replacement first as the 

hypercholesterolaemia should resolve. 


[ Q: 1057 ] MRCPass - Endocrinology 

/ - 

* A 22 year old lady complains of 

feeling lethargic for several months. She also 
has generalized abdominal discomfort. 

Investigations: 

Hb 12.7 g/l 

WBC 5.7 x 1071- 

Platelet 290 x 107L 

ESR 42 mm/hr 

Na 133 (135-144) mmol/l 

K 2.9 (3.4-4.5) mmol/l 

Urea 6.0 (3-7) pmol/l 

Creat 88 (50-100) pmol/l 

Bicarbonate 36 (20-28) mmol/l 

Alkaline phosphotase 95 (50-110) iu/l 

bilirubin 15 (0-17)pmol/l 

AST 35 (5-40) iu/l 

Albumin 38 (33-44) g/l 

What is the likely underlying diagnosis? 

1- Pituitary tumour 

2- Phaeochromocytoma 

3- Acromegaly 

4- Anorexia nervosa 

5- Addisons disease 


Answer & Comments 

Answer: 4- Anorexia nervosa 

Persistent vomiting with anorexia nervosa 
would lead to dehydration, rebound 
peripheral edema, low sodium, low 
potassium, low chloride, metabolic alkalosis, 
low magnesium, and low phosphate. 


[ Q: 1058 ] MRCPass - Endocrinology 

A 40 year old lady with a history of 
end stage renal failure (on peritoneal dialysis) 
and hypertension presented to the hospital 



Dr. Khalid Yusuf El-Zohry - Sohag Teaching Hospital (01118391123) 

Ref MRCPass OE OE 2012 PasTest 2009 PassMedicine 2009 PasTest Exam ReviseMRCP 































El-zohry MRCP Questions Bank (Port 1) - 2013 


(For my personal use) 


with abdominal pain. She has been on 
treatment with a vitamin D analog calcitriol. 

calcium 2.88 (2.25-2.7) mmol/l 

phosphate 0.9 (0.8-1.4) mmol/l 

PTH level 314 pg/ml (10-60 pg/ml). 

Which one of the following is the most likely 
diagnosis? 

1- Digeorge syndrome 

2- Osteoporosis 

3- Tertiary hyperparathyroidism 

4- Waldenstrom's macroglobulinaemia 

5- Osteomalacia 

Answer & Comments 

Answer: 3- Tertiary hyperparathyroidism 

Secondary hyperparathyroidism occurs when 
the parathyroid glands secrete increased 
levels of PTH in response to low calcium; renal 
failure is the most common cause of 
secondary hyperparathyroidism. 

If the causes of secondary 
hyperparathyroidism persist, one parathyroid 
gland may become autonomous; this gland 
produces excess PTH even when calcium is 
normal or elevated. This is known as tertiary 
hyperparathyroidism. 


Answer & Comments 

Answer: 2- Hypertensive left ventricular failure 

In acromegaly, cardiovascular disease is the 
most important cause of mortality. 
Hypertensive cardiomyopathy is a common 
cause. 

^ [ Q: 1060 ] MRCPass - Endocrinology 

fi - 

# A 36 year old woman with type I 
diabetes complains of unsteadiness during 
walking. Visual acuity is normal. 

Fundoscopy shows preproliferative diabetic 
retinopathy. There is loss of proprioception in 
the toes and ankles bilaterally and a small 
painless ulcer is noted under the right 1st 
metatarsal head. 

Which one of the following complications is 
likely? 

1- Diabetic retinopathy 

2- Autonomic neuropathy 

3- Peripheral neuropathy 

4- Atherosclerosis 

5- Diabetic ketoacidosis 

Answer & Comments 

Answer: 3- Peripheral neuropathy 


^ [ Q: 1059 ] MRCPass - Endocrinology 

# A 60 year old man with acromegaly 
is enquiring about prognosis. 

What is the most likely cause of mortality if 
the condition is untreated? 

1- Trauma from visual loss 

2- Hypertensive left ventricular failure 

3- Nephropathy 

4- Colon carcinoma 

5- Thyroid carcinoma 


Peripheral sensory neuropathy in a glove and 
stocking distribution is a frequent 
complication of long-standing diabetes 
mellitus. This causes loss of vibration, pain and 
temperature sensation early on and later 
proprioception may also be affected. 


[ Q: 1061 ] MRCPass - Endocrinology 

A 55 year old woman with Grave's 
disease is being considered for radioiodine 
treatment. 



What is the most common effect of 
radioiodine treatment? 


1- Thyrotoxicosis 


Dr. Khalid Yusuf El-Zohry - Sohag Teaching Hospital (01118391123) 

Ref MRCPass OE OE 2012 PasTest 2009 PassMedicine 2009 PasTest Exam ReviseMRCP 






























El-zohry MRCP Questions Bank (Port 1) - 2013 


(For my personal use) 


2- Hypercalcaemia 

3- Hypoglycaemia 

4- Thyroid carcinoma 

5- Hypothyroidism 


Answer & Comments 

Answer: 5- Hypothyroidism 

Hypothyroidism is the commonest of these, 
and may be transient in the early period, post 
radioiodine. Hypothyroidism is the mosst 
common complication. TSH should be 
monitored 6 monthly after radioiodine. 
Incidence of thyroid carcinoma is not 
increased. 


[ Q: 1062 ] MRCPass - Endocrinology 

A 55 year old woman has 
hypertension due to phaeochromocytoma. 
She has attacks of hot flushes and 
palpitations. Her 24-hour urinary 
catecholamines show a markedly raised 
noradrenaline. 

How should she be treated? 

1- Phenoxybenzamine only 

2- Atenolol 

3- Amlodipine and Propanolol 

4- Phenoxybenzamine, Propanolol and then 
surgery 

5- Surgery without delay 



Answer & Comments 

Answer: 4- Phenoxybenzamine, Propanolol 
and then surgery 

Hypertension should be managed with 
phenoxybenzamine initially, increasing up to 
80 mg per day, with addition of propanolol 
after 3-4 days of alpha blockade. Surgery 
without adequate alpha and beta blockade 
can result in hypertensive crisis (leading to 
high CVA, Ml complications). 



[ Q: 1063 ] MRCPass - Endocrinology 

A 20 year old girl with an unusual 
facial appearance is found to have coarctation 
of the aorta. On examination, she had short 
stature, a webbed neck and lymphedema. 

Which is the likely diagnosis? 

1- Patau's syndrome 

2- Noonan's syndrome 

3- Turner's syndrome 

4- Down's Syndrome 

5- William's syndrome 


Answer & Comments 

Answer: 3- Turner's syndrome 

Turner's syndrome is typically associated with 
coarctation of the aorta. They also have 
features of webbed neck, cubitus valgus, short 
fourth metacarpal, lymphoedema, low set 
ears and hypertension. 

Noonan's syndrome has a phenotype similar 
to Turner's, and is associated with pulmonary 
stenosis rather than coarctation. 


[ Q: 1064 ] MRCPass - Endocrinology 

A 50 year woman complains of 
weight gain and menstrual irregularities. Her 
BMI is 34 kg/m 2 , blood pressure is 165/90 
mmHg. Urinalysis show s 2+ glucose. 

Which investigation is likely to reveal the 
diagnosis? 

1- Renin and aldosterone 

2- Fasting glucose 

3- LH and FSH levels 

4- 24 hour urine cortisol 

5- Prolactin 



Answer & Comments 


Answer: 4- 24 hour urine cortisol 


Dr. Khalid Yusuf El-Zohry - Sohag Teaching Hospital (01118391123) 

Ref MRCPass OE OE 2012 PasTest 2009 PassMedicine 2009 PasTest Exam 




ReviseMRCP 

451 





























El-zohry MRCP Questions Bank (Port 1) - 2013 


(For my personal use) 


Weight gain, high BMI, hypertension, 
menstrual irregularities and glycosuria 
suggests a diagnosis of Cushing's syndrome. 
Investigations for this include the 24 hour 
urine cortisol, dexamethasone suppression 
test, ACTH levels and intra petrosal sinus 
sampling. 


[ Q: 1065 ] MRCPass - Endocrinology 

A 25 year old man presents with 
episodes of sw eating and tachycardia. He has 
an enlarged adrenal gland on the left on the 
CT of the abdomen. 

In phoeochromocytomo, which of the 
hormones is predominantly secreted by the 
adrenal medulla? 



[ Q: 1066 ] MRCPass - Endocrinology 

An 18 yr old man has lethargy and 
nausea. His plasma calcium is 2.72 mmol/L. 
There is a family history of asymptomatic 
hypercalcaemia in the siblings. 

Investigations: PTH 70 pg/ml (<60), F E Ca 
0.8% (fractional excretion of calcium). 

The likely cause for this gentleman's 
hypercalcaemia is: 

1- Multiple myeloma 

2- Primary hyperparathyroidism 

3- Secondary hyperparathyroidism 

4- Familial hypocalciuric hypercalcaemia 

5- Vitamin D toxicity 



1- Natriuretic peptide 

2- Antidiuretic hormone 

3- Cortisol 

4- Adrenaline 

5- Aldosterone 

Answer & Comments 

Answer: 4- Adrenaline 

The diagnosis is phaeochromocytoma, which 
is a tumour of the adrenal medulla. The 
adrenal cortex produces aldosterone, cortisol 
(glucocorticoid) and adrenal androgens. The 
medulla produces adrenaline and 
noradrenaline. 



Phaeochromocytoma 


Answer & Comments 

Answer: 4- Familial hypocalciuric 

hypercalcaemia 

Familial hypocalciuric hypercalcaemia is an 
autosomal dominant disease. The 
pathophysiology is due to a defective calcium 
receptor on the membranes of the 
parathyroid and renal tubular cells. This 
results in a decreased renal clearance of 
calcium, PTH is usually normal or increased, 
P04 is usually decreased. Typically, the 
Fractional Excretion of Ca is <1%. 

Familial Hypocalciuric Hypercalcemia 

Hypercalcemia with normal or low urinary 
calcium 

iPTH levels mav be elevated, but 

«# 

hypocalciuria is not dependent on PTH 

Do not form calcium stones 

Urinary Ca Creatimne clearance ratio of .01 
or below helps distinguish from 
liyperparathyroidism 


4 % [ Q: 1067 ] MRCPass - Endocrinology 

/ 

A diagnosis of diabetes mellitus 
being considered in 30 year old woman who is 


Dr. Khalid Yusuf El-Zohry - Sohag Teaching Hospital (01118391123) 

Ref MRCPass OE OE 2012 PasTest 2009 PassMedicine 2009 PasTest Exam ReviseMRCP 



452 




























El-zohry MRCP Questions Bank (Port 1) - 2013 


(For my personal use) 


12 weeks pregnant. Her body mass index 
(BMI) was 20 kg/m * 1 2 3 4 5 . A 75g oral glucose 
tolerance test show s the following results: 

Time Plasma glucose concentration 

0 hour 5.5 mmol/l 

2 hour 12.8 mmol/l 

Which of the following is the appropriate next 
step in the patient's management? 

1- Gliclazide therapy 

2- Subcutaneous insulin 

3- Diet control 

4- Metformin therapy 

5- Repeat OGTT in four weeks 


Answer & Comments 

Answer: 3- Androgen insensitivity syndrome 

Androgen insensitivity syndrome (AIS), 
formerly known as testicular feminization, is 
an X-linked recessive condition resulting in a 
failure of normal masculinization of the 
external genitalia in chromosomally male 
individuals. 

Most patients with complete androgen 
insensitivity have a female gender. Some 
patients are first seen in the teenage years for 
evaluation of primary amenorrhea, but most 
are identified in the new born period by the 
presence of inguinal masses, which later are 
identified as testes during surgery. 


Answer & Comments 

Answer: 2- Subcutaneous insulin 

A pregnant lady with either gestational 
diabetes or undetected diabetes should go on 
to subcutaneous insulin to achieve good 
glycaemic control in view of the potential 
complications otherw ise. 


[ Q: 1068 ] MRCPass - Endocrinology 

A 16 year old female is evaluated in 
the pediatric endocrinology clinic for primary 
amenorrhea. Her family reported that she had 
breast development at 11 years of age, but no 
other pubertal changes. A testosterone level 
was elevated at 3.08 ng/ml, and both pelvic 
ultrasound and CT scan showed absence of 
reproductive organs. 

What is the likely diagnosis? 

1- Anorexia nervosa 

2- Early menopause 

3- Androgen insensitivity syndrome 

4- Klinefelters's syndrome 

5- Turner's syndrome 



[ Q: 1069 ] MRCPass - Endocrinology 

A 45 year old woman presents with a 
history of increasing tiredness. On 
examination there is pigmentation of her skin 
creases and buccal mucosa. Her blood 
pressure is 85/50. 

Investigations are as follows: Blood urea 8.2 
mmol/l, Na 128 mmol/l, Potassium 6.2 
mmol/l, Chloride 98 mmol/l, Bicarbonate 26 
mmol/l. 

What is the likely diagnosis? 

1- Addison's disease 

2- Porphyria 

3- Hypoparathyroidism 

4- Cushing's syndrome 

5- Conn's syndrome 



Answer & Comments 

Answer: 1- Addison's disease 

This patient has Addison's disease as 
suggested by hyperpigmentation, 

hyponatraemia and hypotension. A high 9 am 
plasma ACTH level with low or normal cortisol 
will confirm the diagnosis of primary 
hypoadrenalism. A low cortisol response with 


Dr. Khalid Yusuf El-Zohry - Sohag Teaching Hospital (01118391123) 

Ref MRCPass OE OE 2012 PasTest 2009 PassMedicine 2009 PasTest Exam ReviseMRCP 



453 




























El-zohry MRCP Questions Bank (Port 1) - 2013 


(For my personal use) 


the short ACTH (synacthen) test would also 
show that the adrenal gland is not responding 
to ACTH. 


4- Primary hypothyroidisim 

5- Secondary hypothyroidism 


[ Q: 1070 ] MRCPass - Endocrinology 

A 35 year old lady has recently been 
commenced on a thiazide diuretic. She has 
routine blood tests which reveal a sodium of 
110 mmol/I, potassium 4.0 mmol/l, urea 6 
pmol/l and creatinine 60 pmol/l. 

Which of the following features is most likely 
to occur? 

1- Hypertension 

2- Tachycardia 

3- Decrease in conscious level 

4- Elevated moods 

5- Miosis 



Answer & Comments 

Answer: 3- Decrease in conscious level 

Severe hyponatraemia can cause seizures, 
obtundation/decreased conscious level, 
headaches, upper motor neuron signs (central 
pontine myelinolysis), bradycardia (not 
tachycardia), hypotension and mydriasis. 


[ Q: 1071 ] MRCPass - Endocrinology 

A 35 year old woman has symptoms 
of constipation, dry skin and low moods. 
These symptoms have developed gradually 
over the past several months. 

Her TFTs show : 

TSH 0.01 (0.3-4.0)mU/l 

Free T 4 8.2 (10-24)pmol/l 

low T3 of 2.1 pmol/L 

What is the likely explanation? 

1- Sick euthyroid syndrome 

2- Grave's thyroiditis 

3- Iodine deficiency 



Answer & Comments 

Answer: 5- Secondary hypothyroidism 

Primary hypothyroidism is associated with 
increased TSH and low T4 and T3. Secondary 
hypothyroidism is due to pituitary failure, 
resulting in low TSH, low T4 and T3. 

Sick euthyroid syndrome can cause low T3 and 
T4, but in this case unlikely because there is 
no association with a critical illness. 


[ Q: 1072 ] MRCPass - Endocrinology 

A 50 year old man has recently had 
an MRI scan to investigate for bitemporal 
visual field loss. His pituitary gland is 15 mm in 
diameter. A glucose load of lOOg orally fails to 
suppress human growth hormone levels. 

Which one of the following is the best 
treatment? 

1- Transphenoidal adenectomy 

2- Pituitary radiotherapy 

3- Bromocriptine 

4- Octreotide 

5- Pegvisomant 



Answer & Comments 

Answer: 1- Transphenoidal adenectomy 

The best treatment option for a large pituitary 
tumour in acromegaly is transphenoidal 
removal of the tumour. Octreotide 
(somatostatin analogue) and pegvisomant 
(Growth hormone receptor blocker) are 
effective forms of treatment. Bromocriptine 
(dopamine agonist) is less effective. 


Dr. Khalid Yusuf El-Zohry - Sohag Teaching Hospital (01118391123) 

Ref MRCPass OE OE 2012 PasTest 2009 PassMedicine 2009 PasTest Exam ReviseMRCP 





























El-zohry MRCP Questions Bank (Port 1) - 2013 


(For my personal use) 



Pituitary tumour in acromegaly 


[ Q: 1073 ] MRCPass - Endocrinology 

A 55 year old woman has weight 
loss, palpitations and diarrhoea. Examination 
reveals a goitre with a single nodule on the 
right lobe of the thyroid, about 3 cm in size. 
Thyroid scintigraphy with Tc 99 shows 
increased uptake within this nodule. 

Thyroid function tests showed a free thyroxine 
of 30 pmol/L (9-25 pmol/L) and TSH <0.08 
mU/L (0.5-5). 

What treatment should be recommended? 

1- Carbimazole 

2- Propylthiouracil 

3- Thyroidectomy 

4- Prednisolone 

5- Radioactive iodine 



[ Q: 1074 ] MRCPass - Endocrinology 

— 

A 50 year old man has progressive 
deafness. He also complains of aching in the 
legs and fatigue. 

On examination, he has bowed legs and the 
legs feel warm. 

What is the most likely diagnosis? 

1- Osteosarcoma 

2- Osteoarthritis 


3- Bony metastases 

4- Paget's disease 

5- Primary hyperparathyroidism 


Answer & Comments 

Answer: 4- Paget's disease 

Paget's disease is characterized by excessive 
breakdow n of bone tissue. The new bone is 
structurally enlarged, but weakened and filled 
with new blood vessels. 

Frequently, bones of the pelvis, leg, spine, 
arm, or the collar bone are involved. The 
effect on the skull may enlarge head size and 
cause hearing loss, if the cranial nerves are 
damaged by the bone growth. 

Fractures can occur. Other symptoms include 
bone pain, bow ing of the legs, neck pains, 
headaches and deafness. 


Answer & Comments 

Answer: 5- Radioactive iodine 

The thyroid uptake scan confirms that this 
lady's thyrotoxicosis is due to a solitary toxic 
nodule. Toxic thyroid nodules are best treated 
with radioactive iodine as this concentrates on 
the overactive adenoma cells. Radioiodine 
treatment is contraindicated in young 
children, pregnant and lactating w omen. 


Deafness may occur not just due to nerve 
compression, but also secondary to pagetic 
involvement of the bony ossicles. 


Dr. Khalid Yusuf El-Zohry - Sohag Teaching Hospital (01118391123) 

Ref MRCPass OE OE 2012 PasTest 2009 PassMedicine 2009 PasTest Exam 




ReviseMRCP 

455 























El-zohry MRCP Questions Bank (Port 1) - 2013 


(For my personal use) 



Bowed Tibia in Paget's disease 


[ Q: 1075 ] MRCPass - Endocrinology 

A 40 year old man presents with 
chronic diarrhoea. He is suspected of having 
the VIPOMA syndrome. 

Which one of the following is a recognised 
feature? 

1- Weight gain 

2- Metabolic alkalosis 

3- Hypokalaemia 

4- Hypoglycaemia 

5- Increased gastric acid secretion 




CT scan showing a VIPOMA 


t 


[ Q: 1076 ] MRCPass - Endocrinology 

A 42 year old lady presents with 
excessive weight gain, hirsutism and back 
pain. Examination shows blood pressure 
180/105, kyphosis and proximal myopathy. Of 
note, her blood glucose is 9 mmol/l. A Dexa 
scan shows a T-score -3.5 and a Z-score -2.5. 


What is the diagnosis? 

1- Multiple endocrine neoplasia 

2- Multiple myeloma 

3- Adrenal adenoma 


4- Pheochromocytoma 

5- Medullary thyroid carcinoma 


Answer & Comments 


Answer: 3- Hypokalaemia 


Answer & Comments 


Answer: 3- Adrenal adenoma 


VIPOMAs secrete vasoactive intestinal peptide 
(VIP) from a ganglioneuroma. 

Features are: 


The diagnosis is Cushing's syndrome due to 
the clinical features of insulin resistance and 
osteoporosis. Out of all the options, the best 
fit as a cause is adrenal adenoma. 


watery diarrhea 

hypochlorhydria 

hyperglycemia 

hypercalcemia 

flushing 


[ Q: 1077 ] MRCPass - Endocrinology 

A 45 year old man has type II 
diabetes and is on oral medication. He 
presents with vomiting and feels generally 
unwell. 



weight loss 
metabolic acidosis 


On examination, his BP is 110/70 mmHg. He 
has a BM of 17. 



Dr. Khalid Yusuf El-Zohry - Sohag Teaching Hospital (01118391123) 

Ref MRCPass OE OE 2012 PasTest 2009 PassMedicine 2009 PasTest Exam ReviseMRCP 





























El-zohry MRCP Questions Bank (Port 1) - 2013 


(For my personal use) 


Investigations reveal: urea 21 mmol/I 

creatinine 190 umol/l 

HCO3- 14 mmol/l 

lactate 6 (0.93-1.65) mmol/L 

pH 7.22 

Urine dipstick - ketone -ve 

Whot is the likely couse of these findings? 

1- Chronic renal failure 

2- Metformin 

3- Gastroparesis induced vomiting 

4- Renal tubular acidosis 

5- Diabetic ketoacidosis 


Answer & Comments 


Answer: 2- Metformin 


3- NSAID related stomach ulcers 

4- Multiple endocrine neoplasia 

5- Tropical sprue 


Answer & Comments 

Answer: 4- Multiple endocrine neoplasia 

The likely diagnosis is MEN la. Gastrinoma 
may lead to duodenal ulceration and diarrhea. 
Parathyroid adenomas may cause 
hypercalcaemia. Infertility may be due to a 
prolactinoma. 

[ Q: 1079 ] MRCPass - Endocrinology 

A 40 year old has been on intensive 
care for 3 weeks following a difficult post 
operative period, but now is rehabilitating 
well. 



The patient has features of metabolic acidosis 
which is most likely to be due to lactic acidosis 
caused by metformin. 


[ Q: 1078 ] MRCPass - Endocrinology 

A 41 year old woman presents with 
significant abdominal pains and frequent 
diarrhoea. She has been previously 
investigated for infertility. Lansoprazole which 
was prescribed by her doctor helped to relieve 
her symptoms. 

Investigations: 

Haemoglobin 11.8 g/dl 

Calcium 2.78 mmol/l 

Albumin 41 g/l 

Phosphate 0.75 mmol/l 

CRP 11 mg/I 

Endoscopy multiple duodenal ulcers 

H. pylori negative 

Whot is the likely diagnosis? 

1- Ulcerative colitis 

2- Coeliac disease 



Her TFT's show : TSH 3.0 (0.3-4) mU/l, Free T 4 
7.3 (8-24) pmol/l, T3 of 2.7 (3.3- 5.5) pmol/l. 

Which diagnosis is the most likely 
explanation? 

1- Iodine deficiency 

2- Sick euthyroid syndrome 

3- Primary hypothyrodisim 

4- Secondary hypothyroidis 

5- Thyrotoxicosis 


Answer & Comments 

Answer: 2- Sick euthyroid syndrome 

Sick euthyroid is seen in unwell patients who 
are clinically euthyroid but have low levels of 
T3 and T4. The syndrome is very common and, 
in fact, may be found in up to 70% of 
hospitalized patients. This is often why TFT's 
are not accurate on patients in ITU. The 
thyroid function tests should be repeated in a 
few weeks' time. 



[ Q: 1080 ] MRCPass - Endocrinology 
A 70 year old man is found with a 


Dr. Khalid Yusuf El-Zohry - Sohag Teaching Hospital (01118391123) 

Ref MRCPass OE OE 2012 PasTest 2009 PassMedicine 2009 PasTest Exam ReviseMRCP 



457 

































El-zohry MRCP Questions Bank (Part 1) - 2013 


(For my personal use) 


decreased conscious level. He has a blood 
glucose of 40 mmol/L (3.5-5.0). 

Urinalysis reveals no ketosis or proteinuria, 
but there is 4+ glycosuria. 

What is the diagnosis? 

1- Insulinoma 

2- Normal anion gap metabolic acidosis 

3- Addison's disease 

4- Hyperosmolar non ketotic coma 

5- Diabetic ketoacidosis 


Answer & Comments 

Answer: 4- Hyperosmolar non ketotic coma 

Hyperosmolar non ketotic coma is 
characterised by markedly raised blood sugar, 
often >50 mmol/L. There is no significant 
ketosis and acidosis. 

It occurs in patients with Type 2 diabetes of 
middle age or older. Treatment should be with 
isotonic saline , low dose insulin and 
potassium replacement. 


[ Q: 1081 ] MRCPass - Endocrinology 

A 50 year old patient is being seen in 
the ophthalmology clinic for pre-proliferative 
retinopathy. He is a type II diabetic. 

Which one of the following is most likely to 
delay disease progression when treated? 

1- Soft exudates 

2- Hypertension 

3- Glycaemic control 

4- Hypercholesterolaemia 

5- Stop smoking 



Answer & Comments 

Answer: 2- Hypertension 

Diabetic retinopathy occurs in both type 1 and 
type 2 diabetes. Progression may be slowed 
by improving glycaemic and hypertensive 


control, but hypertensive control has been 
shown be more effective at reducing 
progression (UKPDS). There are no data at 
present suggest Statin therapy reduces 
disease progression. Soft exudates are a 
feature of preproliferative diabetic 
retinopathy. 

[ Q: 1082 ] MRCPass - Endocrinology 

A 70 year woman with low bone 
densitometry readings is currently taking 
Raloxifene. 

What form of drug is Raloxifene? 

1- Selective estrogen receptor modulator 
(SERM) 

2- Anti interferon antibody 

3- Corticosteroid 

4- Hormone replacement therapy 

5- Bisphosophonate 



Answer & Comments 

Answer 1- Selective estrogen receptor 
modulator (SERM) 

Selective estrogen receptor modulators 
(SERMs) exhibit a pharmacologic profile 
characterized by estrogen agonist activity in 
some tissues with estrogen antagonist activity 
in other tissues. The first widely used SERM, 
tamoxifen, has estrogen antagonist activity in 
breast tissue but shows estrogen-like activity 
in other tissues. Raloxifene is another SERM in 
clinical use, and it was developed to avoid 
some of the undesirable estrogen agonist 
actions of other SERMs to improve the drug 
safety profile. 

Raloxifene has been introduced for clinical use 
in treatment and prevention of 
postmenopausal osteoporosis. 


17 

[ Q: 1083 ] MRCPass - Endocrinology 

L*lJ 

A 65 year old woman has been 


Dr. Khalid Yusuf El-Zohry - Sohag Teaching Hospital (01118391123) 

Ref MRCPass OE OE 2012 PasTest 2009 PassMedicine 2009 PasTest Exam ReviseMRCP 



458 































El-zohry MRCP Questions Bank (Port 1) - 2013 


(For my personal use) 


prescribed thyroxine 150 ?g daily following a 
diagnosis of hypothyroidism. 

Having had replacement for several weeks, 
her investigations revealed: 

serum total T4 concentration 65 nmol/L (55 - 
145) 

serum total T3 concentration 0.7 nmol/L (0.9 - 
2.5) 

serum TSH concentration 15 mll/L (0.5 - 4) 
Whot should be done? 

1- Thyroid uptake scan 

2- No action as it will resolve 

3- Measurement of free T4 levels 

4- Check anti thyroid antibodies 

5- Enquire about compliance 


Answer & Comments 

Answer: 5- Enquire about compliance 

Non compliance would explain the thyroid 
function tests. The patient may have taken 
thyroxine prior to coming for investigation, 
hence a normal T4 but lowT3 concentration. 


[ Q: 1084 ] MRCPass - Endocrinology 

A 55 year old man presents with 
lethargy and dizziness upon standing. He gives 
a history of having had treatment for 
tuberculosis when he was a child. 

On examination he looks thin, his skin is 
pigmented and there is pigmentation of his 
buccal mucosa and pigmentation of the 
palmar creases. There is no vitiligo. His blood 
pressure is 120/80 lying and 85/60 on 
standing. 

His blood urea 8.4 mmol/L, Na 122 mmol/L, K 
5.7 mmol/L. 

Which of the following investigations is most 
likely to identify the condition? 

1- Short synacthen test 

2- 0900 ACTH level 



3- MRI pituitary 

4- 24 hour urine catecholamines 

5- Serum aldosterone /plasma renin activity 


Answer & Comments 

Answer: 1- Short synacthen test 

The patient has Addison's disease, 
predisposed to by previous adrenal 
tuberculosis. The synacthen test is used to test 
adrenal reserve. Synacthen is tetracosactrin, 
the first 24 amino acids of ACTH. 

The short synacthen test is done by: 

1) take a basal sample for cortisol at time 0 
min. 

2) give 250 microgramme Synacthen i.v. or 
i.m. 

3) sample for cortisol are taken at 30 mins. 
There should be a significant response unless 
the patient is addisonian. 



[ Q: 1085 ] MRCPass - Endocrinology 

A 32 year old woman has been 
treated for depression. She now complains of 
thirst and drinking excessive amounts of 
water. At the end of am 8 hour water 
deprivation test, she has a serum osmolality of 
290 mosmol/kg and urine osmolality of 100 
mosmol/kg. 


DDAVP (20ug intra-nasally) was given once 
these results were seen, and her repeat urine 
osmolality was 95 molsmol/kg. 


Whot is the likely diagnosis? 


1- Cranial diabetes insipidus 


2- Nephrogenic diabetes insipidus 

3- SIADH 


4- Ectopic ACTH seretion 

5- Psychogenic polydipsia 


Dr. Kholid Yusuf El-Zohry - Sohog Teaching Hospital (01118391123) 

Ref MRCPass OE OE 2012 PasTest 2009 PassMedicine 2009 PasTest Exam ReviseMRCP 



459 

























El-zohry MRCP Questions Bank (Port 1) - 2013 


(For my personal use) 


Answer & Comments 

Answer: 2- Nephrogenic diabetes insipidus 

The clinical picture is possibly drug induced 
(lithium) nephrogenic diabetes insipidus since 
there is a history of depression. The plasma 
osmolality normal range is 278-300 
mosmol/kg and urine osmolality normal range 
is 350-1000 mosmol/kg. Hence she has 
inappropriately dilute urine despite water 
deprivation. 

At the end of the water deprivation, it is not 
possible to tell whether it is cranial or 
nephrogenic unless she is given a test dose of 
DDAVP. If she starts concentrating the urine 
(response to the synthetic ADH) then it is 
likely to be cranial Dl. If she still does not 
respond, as in this case, then it is likely to be 
nephrogenic Dl. 


^ [ Q: 1087 ] MRCPass - Endocrinology 

f --— 

# A 45 year old man has newly 
diagnosed Conn's syndrome biochemically. 

What is the most common cause of Conn's 
syndrome in the UK? 

1- Adrenocortical carcinoma 

2- Adrenocortical adenoma 

3- Phaeochromocytoma 

4- MEN 1 

5- MEN 2 

Answer & Comments 

Answer: 2- Adrenocortical adenoma 

Conn's syndrome is most often caused by an 
adrenocortical adenoma. 


[ Q: 1086 ] MRCPass - Endocrinology 

A 45 year old lady has recently been 
diagnosed as a diabetic. Despite strict diet 
control, her blood sugars are running at 12 
mmol/l. She weighs 80kg. 

Which of the following is the best medication 
to start with? 




1- Insulin 

2- Glibenclamide 

3- Metformin 

4- Troglitazone 

5- Gliclazide 


Answer & Comments 


Answer: 3- Metformin 



[ Q: 1088 ] MRCPass - Endocrinology 

A 32 year old patient with type 1 
diabetes has now pregnant and is being 
assessed in the diabetic clinic. 


Which one of the following is the most 
important? 

1- Tight glycaemic control 

2- Tight hypertensive control 


Metformin is a biguanide. It improves insulin 
sensitivity and is helpful especially in patients 
who are overweight as it does not stimulate 
appetites in the way that sulphonylureas do. 


3- Manage only on oral hypoglycaemics 

4- Anticoagulation with low molecular weight 
heparin 

5- Dipstick urine for ketonuria 


Dr. Khalid Yusuf El-Zohry - Sohag Teaching Hospital (01118391123) 

Ref MRCPass OE OE 2012 PasTest 2009 PassMedicine 2009 PasTest Exam ReviseMRCP 






























El-zohry MRCP Questions Bank (Port 1) - 2013 


(For my personal use) 


Answer & Comments 

Answer: 1- Tight glycaemic control 

Tight glycaemic control and close monitoring 
with HbAlc is essential in view of 
complications, including worsening diabetic 
retinopathy. Many patients go on to insulin 
rather than stay on oral hypoglycaemics. 

^ [ Q: 1089 ] MRCPass - Endocrinology 

# A 50 year old lady complains of 
fevers, headache, tremor and palpitations. On 
examination she has a BP of 220/110 mmHg, a 
pulse rate of 120 and glycosuria. 24 hour 
urinary vanillyl mandelic acid is measured at 
85 micromoles/ 24h (normal 5-35 
micromoles/24 h). 

The hypertension should be treated with: 

1- Clonidine 


the past 3 months. Her BM was 23. Her ABG 
showed a pH 7.28, HCO 16 mmol/I. Plasma 
ketones are negative. 

What is the likely diagnosis? 

1- Diabetic ketoacidosis 

2- Addison's disease 

3- Hypothyroidism 

4- Hyperparathyroidism 

5- Hyper osmolar ketotic coma 

Answer & Comments 

Answer: 1- Diabetic ketoacidosis 

Ketonuria may not be present at the early 
stage of diabetic ketoacidosis. Some 
laboratories can only detect a specific type of 
ketone (acetoacetate) and the major ketone 
present in DKA is betahydroxybutyrate. 


2- Methyldopa 

3- Propranolol before phenoxybenzamine 

4- Phenoxybenzamine before propranolol 

5- Bisoprolol 


Answer & Comments 

Answer: 4- Phenoxybenzamine before 

propranolol 

The diagnosis is phaeochromocytoma. The 
treatment of hypertension in 
phaeochromocytoma is with alpha blockade 
prior to beta blockade. Alpha blockade 
reverses the peripheral vasoconstriction 
whereas beta blockade prevents tachycardia. 
The preferred a-blocker phenoxybenzamine, 
as is not a selective alpha 1 blocker but an 
irreversible a-blocker whose effects cannot be 
overcome by an increase of catecholamines. 


[ Q: 1090 ] MRCPass - Endocrinology 

A 15 year old girl presents to the 
casualty with lethargy, malaise, polyuria and 
polydipsia. She has lost 1 stone in weight over 




[ Q: 1091 ] MRCPass - Endocrinology 

A 25 year old patient has a father 
who has previously been diagnosed with 
medullary thyroid carcinoma, 

hyperparathyroidism and 

phaeochromocytoma. The patient has been 
screened genetically and has the same gene 
predisposing to the condition. 

Which of the following is the most appropriate 
management strategy? 

1- Bilateral adrenalectomy and lifelong steroid 
replacement 

2- MRI of the brain 

3- Thyroidectomy and regular 24 hour urine 
for catecholamines 


4- No further action unless symptomatic 

5- Parathyroidectomy and bilateral 
adrenalectomy 


Answer & Comments 

Answer: 3- Thyroidectomy and regular 24 hour 
urine for catecholamines 


Dr. Khalid Yusuf El-Zohry - Sohag Teaching Hospital (01118391123) 

Ref MRCPass OE OE 2012 PasTest 2009 PassMedicine 2009 PasTest Exam ReviseMRCP 



































El-zohry MRCP Questions Bank (Port 1) - 2013 


(For my personal use) 


The condition is MEN 2. Since medullary 
thyroid carcinoma is incurable, prophylactic 
thyroidectomy is advised before disease 
progression. How ever, bilateral 
adrenalectomy is not necessary until there is 
evidence for phaeochromocytoma 
development. 



[ Q: 1092 ] MRCPass - Endocrinology 

A 45 year old woman has been 
referred for investigation of abnormal liver 
function tests. She drinks 40 units of alcohol a 
week. On examination she is obese with 
mainly truncal obesity, with a moon face and a 
buffalo hump shaped deposit of fat across her 
shoulders. Her face is plethoric and there are 
numerous telangiectasia. 


The abdomen is protuberant and there are 
striae. 


What is the likely diagnosis? 

1- Adrenal adenoma 


2- Pituitary dependent Cushings 

3- Alcoholic pseudocushings 

4- Ectopic ACTH secretion 

5- Steroid use 


Answer & Comments 

Answer: 3- Alcoholic pseudocushings 

The diagnosis is alcoholic pseudocushings, in 
which ACTH, cortisol levels and dynamic 
testing should not be abnormal. 

[ Q: 1093 ] MRCPass - Endocrinology 

A 18 year old man has poorly 
developed secondary sexual characteristics. 
He complains of a poor sense of smell. On 
examination, he has little axillary or pubertal 
hair, a small penis and testicular volumes of 
approximately 4 ml bilaterally. 

Investigations reveal: 

Testosterone 3 nmol/L (10-30) 



Prolactin 360 mll/L (<450) 

FSH 2.5 iu/L (1-7) 

LH 1.9 iu/L (1-10) 

What is the most likely diagnosis? 

1- 21 hydroxylase deficiency 

2- 17 hydroxylase deficiency 

3- Testicular feminisation syndrome 

4- Kallman's syndrome 

5- Adrenal adenoma 


Answer & Comments 

Answer: 4- Kallman's syndrome 

Kallman's syndrome describes the occurrence 
of hypothalamic gonadotrophin releasing 
hormone deficiency and deficient olfactory 
sense - hyposmia or anosmia. It is usually 
inherited as an X-linked or autosomal 
recessive disorder with greater penetrance in 
the male. Nerve deafness, colour blindness, 
mid-line cranio-facial deformities, and 
cryptorchidism also occur. 


[ Q: 1094 ] MRCPass - Endocrinology 

# A 17 year old boy was seen at the 
clinic for investigation of gynecomastia. On 
examination, he was unusually tall, mildly 
overw eight, he had gynecomastia and 
hypoplastic testes. 

What is the likely genetic karyotype? 


1- 46 XO 

2- 46 XY 

3- 46 XX 

4- 47 XYY 

5- 47 XXY 


Answer & Comments 

Answer: 5- 47 XXY 

This is the karyotype of Klinefelter's 
syndrome. 


Dr. Khalid Yusuf El-Zohry - Sohag Teaching Hospital (01118391123) 

Ref MRCPass OE OE 2012 PasTest 2009 PassMedicine 2009 PasTest Exam ReviseMRCP 



462 




























El-zohry MRCP Questions Bank (Port 1) - 2013 


(For my personal use) 


Boys with Klinefelter's syndrome often show 
very discrete clinical features, including tall 
stature, obesity, gynecomastia and eunuchoid 
habitus, therefore the syndrome is often not 
diagnosed. In addition, they have small testes, 
a normal to low testosterone level and are 
infertile. 


mr 

1 


[ Q: 1095 ] MRCPass - Endocrinology 

A 20 year old lady with polycystic 
ovary syndrome was prescribed Metformin. 

How does metformin work in the condition? 

1- Increasing oestradiol levels 

2- Increasing follicle stimulating hormone 
levels 

3- Increasing gluconeogenesis 

4- Increasing insulin levels 



[ Q: 1096 ] MRCPass - Endocrinology 

A 35 year old lady has complained of 
syncopal episodes. She has had one previous 
documented BM of 2.6. 


Which of the following is the most appropriate 
investigation? 

1- 24 hour tape recording 

2- Oral glucose tolerance test 

3- Insulin tolerance test 


4- 72 hour fast with insulin, C peptide and 
plasma glucose sent when BM < 4 

5- MRI of brain 


Answer & Comments 

Answer: 4- 72 hour fast with insulin, C peptide 
and plasma glucose sent when BM < 4 


5- Increasing peripheral glucose uptake 

Answer & Comments 


Answer: 5- Increasing peripheral glucose 
uptake 

Within the context of PCOS, metformin 
decreases hyperinsulinism and increases 
peripheral glucose uptake, reduces plasma 
levels of luteinizing hormone (LH), and 
reduces ovarian androgen production. 



The diagnosis is likely to be an insulinoma. 
Hypoglycaemic attacks are likely to be 
witnessed during a 72 hour fast. Insulin and C 
peptide levels are high, whilst glucose is low . 
Sulphonylurea levels are also sent in cases 
where oral hypoglycaemic drug misuse is 
suspected. 

^ [ Q: 1097 ] MRCPass - Endocrinology 

n - 

A 35 year old woman had no 
menstrual periods for the past 6 months. She 
is not pregnant and has not been on any 
medication. Within the past week, she has 
noted milk production from her breasts. She 
has also had headaches for several months. 

Which of the following laboratory test findings 
is most likely to be present? 

1- High prolactin level 

2- Hypocalcaemia 

3- High serum cortisol 

4- Low serum growth hormone 

5- High free thyroxine 



Dr. Khalid Yusuf El-Zohry - Sohag Teaching Hospital (01118391123) 

Ref MRCPass OE OE 2012 PasTest 2009 PassMedicine 2009 PasTest Exam ReviseMRCP 















































El-zohry MRCP Questions Bank (Port 1) - 2013 


(For my personal use) 


Answer & Comments 

Answer: 1- High prolactin level 

The headache and visual disturbance suggests 
a macroadenoma. This could be prolactinoma 
or acromegaly. The presence of galactorrhoea 
and menstrual disturbance suggests high 
prolactin levels. Levels of prolactin >3000 
rnLI/L would be expected in 
macroprolactinoma, and levels above 6000 
rnLI/L are diagnostic. Prolactinomas are the 
commonest functioning pituitary tumours. 


[ Q: 1098 ] MRCPass - Endocrinology 

A 25 year old man is investigated for 
cryptorchidism. He has a cleft palate and 
colour blindness. He is of normal stature. LH is 
0.2 mlU/ml(2-18) and FSH is 1.2 mlU/ml(2-18). 

Which one of the following is the likely 
diagnosis? 

1- Testicular feminisation syndrome 

2- Kallmann's syndrome 

3- Marfan's syndrome 

4- Klinefelter's syndrome 

5- Congenital adrenal hyperplasia 17 
progesterone form 



normal or above average stature. Females 
may present with primary amenorrhoea; 
males with cryptorchidism. LH and FSH levels 
are typically low. 



[ Q: 1099 ] MRCPass - Endocrinology 

A 45 year old woman has a 2 year 
history of treated hypothyroidism. There was 
a short history of weight loss. 

On examination she had a temperature of 
37.7 C, a blood pressure of 85/35 mmHg. She 
also had vitiligo. 

Which one of the following should be given 
intravenously initially? 

1- Cefuroxime 

2- 10% dextrose infusion and insulin 



Answer & Comments 

Answer: 2- Kallmann's syndrome 

Kallman's syndrome describes the occurrence 
of hypothalamic gonadotrophin releasing 
hormone deficiency and deficient olfactory 
sense - anosmia. It is usually inherited as an X- 
linked or autosomal recessive disorder with 
greater penetrance in the male. 
Gonadotrophin deficiency arises from a failure 
of embryonic migration of GnRH secreting 
neurons from their site of origin in the nose. 

More than half of patients have associated 
nerve deafness, colour blindness, mid-line 
cranio-facial deformities such as cleft palate or 
harelip, and renal abnormalities. Most are of 


3- T3 

4- Fludrocortisone 

5- Hydrocortisone 

Answer & Comments 

Answer: 5- Hydrocortisone 

The diagnosis is Addison's disease. Many of 
the presenting signs and symptoms are 
nonspecific. Patients with mineralocorticoid 
insufficiency may show signs of sodium and 
volume depletion (eg, orthostatic 
hypotension, tachycardia). Hydrocortisone 
100 mg or 200 mg IV should be given, and 
continued as a QDS dose. Fluid replacement 
with dextrose is also recommended. Following 
that, fludrocortisone acetate 


Dr. Khalid Yusuf El-Zohry - Sohag Teaching Hospital (01118391123) 

Ref MRCPass OE OE 2012 PasTest 2009 PassMedicine 2009 PasTest Exam ReviseMRCP 
































El-zohry MRCP Questions Bank (Port 1) - 2013 


(For my personal use) 


(mineralocorticoid) 0.1 mg qds should also be 
given. 

[ Q: 1100 ] MRCPass - Endocrinology 

f - 

# A 70 year old man presents with 
significant back pains and lethargy, and 
associated polydipsia. 

His investigations show : 

Haemoglobin 10.7 g/dl 

Urea 18.5 mmol/l 

Creatinine 320 micromol/l 

Calcium 3.46 mmol/l 


Blood tests show : 

Glucose 7.5 mmol/L (3.5 -6.5) 

LH 2.8 U/L (0.7-6) 

FSH 5.3 U/L (<6) 

Prolactin 320 mU/L (<425) 

Testosterone 13.2 nmol/L (10-35) 

GH 7 mU/L (<10) 

Cortisol random 523 nmol/L (250-700) 

TSH 2.8 mU/L (0.3-6) 

FT4 17.5 pmol/L (9.4 -25) 

What is the most likely underlying diagnosis? 


Albumin 32 g/l 
Total protein 98 g/l 

Thoracic spine X rays show collapse of T7-T8 
vertebrae. 

Which investigation is most likely to confirm 
diagnosis? 

1- Blood film 

2- Ultrasound of the liver 

3- Parathyroid hormone levels 

4- Serum electrophoresis 

5- Creatinine Clearance 


Answer & Comments 

Answer: 4- Serum electrophoresis 

The diagnosis is multiple myeloma, which is 
suggested by the hypercalcaemia (polydipsia 
and polyuria), renal failure and pathological 
thoracic vertebral fractures. 



[ Q: 1101 ] MRCPass - Endocrinology 

A 50 year old man presents with a 6 
month history of severe headaches. He has a 
history of hypertension and osteoarthrits. On 
examination, he has coarse facial features 
with a prominent jaw . Visual fields 
demonstrate bi-temporal hemianopia. BP 
145/105 mmHg and glycosuria. 


1- Macroprolactinoma 

2- Conn's syndrome 

3- ACTH secreting tumour 

4- Sheehan's syndrome 

5- Acromegaly 


Answer & Comments 

Answer: 5- Acromegaly 

The clinical picture is consistent with 
acromegaly, there is impaired glucose 
tolerance, hypertension and enlarged pituitary 
causing bitemporal hemianopia. 



Enlarged hands and coarse facial features in 

Acromegaly 


[ Q: 1102 ] MRCPass - Endocrinology 

A 40 year old patient with Addison's 
disease is intolerant of her hydrocortisone 
treatment which she takes at a dose of 20mg 
in the morning and lOmg in the evening. 



Dr. Khalid Yusuf El-Zohry - Sohag Teaching Hospital (01118391123) 

Ref MRCPass OE OE 2012 PasTest 2009 PassMedicine 2009 PasTest Exam 




ReviseMRCP 

465 


























El-zohry MRCP Questions Bank (Port 1) - 2013 


(For my personal use) 


Which one of the following closes of 
prednisolone would provide on equivalent 
doily dose ? 

1- 2 mg 

2- 5 mg 

3- 7.5 mg 

4- 10 mg 

5- 20 mg 


Answer & Comments 

Answer: 3- 7.5 mg 

Total dose of hydrocortisone is 30mg. The 
equivalent ratio of prednisolone: 
hydrocortisone is 1:4. Hence 30/4 = 7.5 mg. 
The ratio for dexamethasone: hydrocortisone 
is 1:40. 


[ Q: 1103 ] MRCPass - Endocrinology 

A 45 year old woman complains of 
parasthesiae in her hands following a 
thyroidectomy. She has spasms in her hands. 

Whot blood test is most useful in this 
situation? 

1- Thyroid function test 

2- Magnesium 

3- Calcium 



lethargy and weight loss. This has gradually 
occured over the last 6 months. 

On examination, a small goitre is palpable. 
There is no evidence of eye signs or pretibial 
myxedema. 

Investigations reveal : 

Free T 4 of 21.1 (9.8-23) pmol/l 

T3 of 5.3 (3.3-5.5) pmol/l 

a TSH of 0.05 (0.1-5) mU/l 

Thyroid autoantibody titres are all negative 

A thyroid uptake scan show s patchy uptake 

Whot is the diagnosis? 

1- Sick euthyroid syndrome 

2- Toxic nodule 

3- Grave's disease 

4- Hashimoto's thyroiditis 

5- De Quervain's thyroiditis 


Answer & Comments 

Answer: 2- Toxic nodule 

This patient has subclinical hyperthyroidism. 
Grave's disease is unlikely due to negative 
antithyroid antibodies. This is because the T4 
and T3 levels are normal. How ever, the TSH is 
reduced, so the hyperthyroidism is subclinical. 


4- Potassium 

5- Sodium 


Answer & Comments 

Answer: 3- Calcium 

This patient has hypoparathyroidism post 
thyroidectomy, leading to hypocalcaemia. 
Treatment is with calcium and calcitriol 
supplementation. Signs are tetany, carpal 
spasm and Chvostek's sign. 



[ Q: 1104 ] MRCPass - Endocrinology 
A 65 year old man presents with 


Toxic multinodular goiter or toxic nodule is 
more common in the elderly. Very recently, 
activating point mutations in the TSH 
receptor, which result in continuous thyroid 
stimulation, have been described in the 
solitary nodule. It can cause hyperthyroidism 
or subclinical hyperthyroidism. 

In patients with multinodular goitre, the 
thyroid uptake scan (with radioactive iodine) 
usually reveal patchy uptake, with areas of 
both increased and decreased uptake. 
Patients with Graves disease usually have 
homogeneous diffuse uptake. Glands with 
thyroiditis have low uptake. 


Dr. Kholid Yusuf El-Zohry - Sohog Teaching Hospital (01118391123) 

Ref MRCPass OE OE 2012 PasTest 2009 PassMedicine 2009 PasTest Exam ReviseMRCP 



























El-zohry MRCP Questions Bank (Port 1) - 2013 


(For my personal use) 


Sick Euthyroid syndrome is usually associated 
with critically ill patients. Hashimoto's and De 
Quervain's thyroiditis are usually associated 
with hypothyroidism. 


[ Q: 1105 ] MRCPass - Endocrinology 

A 35 year old type 1 Diabetic patient 
presents to the clinic for review . Her urine 
dipstick show s blood +, protein ++, leucocytes 
-. HbAlc is 8.2. 

Which drug, when commencedwill hove on 
impact on mortality? 

1- Insulin 

2- Metformin 

3- Lisinopril 

4- Gliclazide 



Answer & Comments 

Answer: 1- Primary hyperparathyroidism 

The disorder is relatively common disorder 
amongst elderly females. In primary 
hyperparathyroidism, typically there is 
hypercalcaemia, low phosphate and raised 
alkaline phosphatase. PTH acts directly on 
kidney (glomerular Ca++ resorption) and bone 
Ca++ resorption (rate of dissolution of bone 
mineral) and indirectly on intestine via control 
of vit D derivative l-alpha-25-(OH)2D 
(calcitriol) synthesis in the kidney. Levels of 
PTH are regulated by a classic feedback loop 

Incidence of primary hyperparathyroidism is 
1:800. 

Causes include : 


5- Bendrofluazide 


Adenomas (single 80%) 


Answer & Comments 


Answer: 3- Lisinopril 


Parathyroid hyperplasia 
Parathyroid carcinomas (2-3% cases) 


ACE inhibitors have been show n to be of 
benefit in d nephropathy - with an improved 
mortality outcome in the long term. 


[ Q: 1106 ] MRCPass - Endocrinology 

A 70 year lady has lethargy and is 
investigated at the hospital. Her investigations 
reveal: 



[ Q: 1107 ] MRCPass - Endocrinology 

A 55 year old man has large spade 
like hands and finds that he is having to buy 
shoes of increasing size. On examination of his 
visual fields, bitemporal hemianopia was 
found. 

Which one of the following tests should be 
done? 



Corrected calcium 2.86 (2.2-2.6) mmol/I 
Phosphate 0.80 (0.81-1.4) mmol/l 
Alkaline phosphatase 120 U/L (20-95) 

PTH concentration 5.8 pmol/L (0.9-5.4) 

Whot is the likely diagnosis? 

1- Primary hyperparathyroidism 

2- Multiple myeloma 

3- Osteoporosis 

4- Paget's disease 

5- Ectopic PTH related peptide (PTHrp) 
secretion 


1- Insulin tolerance test 

2- Hydrocortisone curve 

3- CT scan of the chest 

4- Dexamethasone suppression test 

5- Oral glucose tolerance test with growth 
hormone 

Answer & Comments 

Answer: 5- Oral glucose tolerance test with 
growth hormone 


Dr. Kholid Yusuf El-Zohry - Sohog Teaching Hospital (01118391123) 

Ref MRCPass OE OE 2012 PasTest 2009 PassMedicine 2009 PasTest Exam 




ReviseMRCP 

467 






























El-zohry MRCP Questions Bank (Port 1) - 2013 


(For my personal use) 


Growth hormone suppression during oral 
glucose tolerance and elevated IGF-I levels 
form the main diagnostic criteria in 
acromegaly. An MRI scan (to look for 
macroadenoma) is also important. 


[ Q: 1108 ] MRCPass - Endocrinology 

A 8 year old boy is investigated for 
short stature. He has short limbs, trunk and 
and saddle shaped nose. X rays reveal 
epiphyseal dysplasia. 

Whot is the diagnosis? 

1- Achondroplasia 

2- Paget's disease 

3- X linked hypophosphataemic rickets 

4- Congenital adrenal hyperplasia 

5- Congenital osteoporosis 



Answer & Comments 

Answer: 1- Achondroplasia 

Achondroplasia is an autosomal dominant 
condition which is one of commonest forms of 
inherited dwarfism. 

Epiphyseal dysplasia occurs and there is a 
diminished columnar arrangement short thick 
bones, spinal length almost alw ays normal. 
Features include short limbs, trunk, large 
head, saddle nose and exaggerated lumbar 
lordosis. 



Achondroplasia 


[ Q: 1109 ] MRCPass - Endocrinology 

A 45 year old man has a blood 
pressure of 180/105 mmHg found by the GP 
and was referred for further investigation. 

Blood tests show : 

serum sodium 144 mmol/L (135-145) 

potassium 2.5 mmol/L (3.5-5.0) 

chloride 102 mmol/L (95-105) 

glucose 5.3 mmol/L (3.5-5.5) 

creatinine 100 umol/L (70-110) 

His plasma renin activity is 0.1 ng/mL/hr and 
serum aldosterone is 680 pmol/L (100-500). 

Which is the best drug to prescribe? 

1- Atenolol 

2- Lisinopril 

3- Bendrofluazide 

4- Spironolactone 

5- Losartan 



Answer & Comments 

Answer: 4- Spironolactone 

Primary hyperaldosteronism (Conn's 
syndrome due to a single adenoma), responds 


Dr. Kholid Yusuf El-Zohry - Sohog Teaching Hospital (01118391123) 

Ref MRCPass OE OE 2012 PasTest 2009 PassMedicine 2009 PasTest Exam ReviseMRCP 


























El-zohry MRCP Questions Bank (Port 1) - 2013 


(For my personal use) 


w ell to Spironolactone 200-400 mg. ACE 
inhibitor and angiotensin II antagonists can 
also be used. The definitive treatment how 
ever is surgery. 



[ Q: 1110 ] MRCPass - Endocrinology 

A 22 year old girl complains of 
feeling tired for the last 6 months. She also 
has generalized abdominal discomfort and 
poor bow el movement. Examination show s a 
pale and thin young woman. Her blood 
pressure is 110/60 mmHg. 


Investigations reveal: 
Hb 13.6 g/l 
WBC 3.2 x 10 9 /L 
Platelet 230 x 10 9 /L 
ESR 25 mm/hr 
Na 132 mmol/I 


K 2.6 mmol/I 


Urea 4 mmol/l 


Creat 80 umol/l 


Bicarbonate 35 mmol/l 
alkaline phosphotase 85 iu/l (50-110) 
bilirubin 14 (0=17) umol/l 
AST 35 iu/l (5-40) 

Albumin 32g/l 

Which one of the following is the likely 
underlying diagnosis? 

1- Phaechromocytoma 

2- Conns syndrome 

3- Diabetes type 1 


4- Anorexia nervosa 


5- Addisons disease 


Answer & Comments 

Answer: 4- Anorexia nervosa 

A low sodium, potassium and metabolic 
alkalosis can be due to self induced vomiting. 


Hence the clinical picture is most consistent 
with anorexia nervosa. 




[ Q: 1111 ] MRCPass - Endocrinology 


A 65 year old man presents with 
chest pain. His ECG show s anterior ST 
elevation and he is thrombolysed with 
tenecteplase. He has a history of type 2 
diabetes and has a BM of 15 on admission. His 
HbAlc is 10%. 


Which one of the following is the most 
appropriate therapy? 

1- Maximise gliclazide dose 

2- Metformin 


3- Pioglitazone 

4- Sliding scale insulin 

5- 5% dextrose 


Answer & Comments 

Answer: 4- Sliding scale insulin 

In diabetics who have had an Ml, the DIGAMI 
study showed that intravenous insulin for 24 
hours and subcutaneous insulin for 3 months 
improved mortality rates for up to 3 years 
after. 

[ Q: 1112 ] MRCPass - Endocrinology 

A 25 year old woman has polydipsia 
and polyuria. She has had no history of 
diabetes, but had a history of head injury 
several years ago. 

Investigations reveal: 

sodium 155 (135-145)mmol/l 

potassium 4.5 (3.5-4.9)mmol/l 

calcium 2.35 (2.2-2.6)mmol/l 

glucose 4.6 (3.0-6.0)mmol/l 

Which one of the following is most likely to 
confirm the diagnosis? 

1- Oral glucose tolerance test 

2- Water deprivation test 



Dr. Khalid Yusuf El-Zohry - Sohag Teaching Hospital (01118391123) 

Ref MRCPass OE OE 2012 PasTest 2009 PassMedicine 2009 PasTest Exam ReviseMRCP 



469 




























El-zohry MRCP Questions Bank (Port 1) - 2013 


(For my personal use) 


3- Calcium levels 

4- ADH levels 

5- Trial of low dose DDAVP 


Answer & Comments 

Answer: 2- Water deprivation test 

With the history of head injury, she may have 
had pituitary damage causing cranial diabetes 
insipidus, which may have manifested late. A 
high sodium level is consistent with this. A 
water deprivation test would help confirm 
this, and challenge with DDAVP at the end of 
the test may help to distinguish betw een 
cranial and nephrogenic diabetes insipidus. 


[ Q: 1113 ] MRCPass - Endocrinology 

A 50 year old man presents with 
episodes of sw eating and tremors, which are 
relieved by glucose. He has gained 1 stone of 
weight in the past 2 months and drinks 
approximately 10 units of alcohol weekly. 

His investigations show normal full blood 
count, normal urea and electrolytes and a 
fasting plasma glucose concentration of 3.8 
mmol/l (3-6). 

Whot is the next most appropriate 
investigation ? 

1- Insulin tolerance test 

2- CT scan of pancreas 

3- Insulin and C-peptide concentration 

4- 24 hour urine catecholamines 

5- Glucose tolerance test 



Answer & Comments 

Answer: 3- Insulin and C-peptide 

concentration 

If insulin and C-peptide levels both are 
increased during a hypoglycaemic episode, 
then insulinoma would be the cause. If the C- 
peptide level was not raised but insulin level 


is, then exogenous insulin use may be the 
cause. 



Insulin is derived from proinsulin (pictured) by 
cleavage of the C-peptide structure (in blank 

circles) 


[ Q: 1114 ] MRCPass - Endocrinology 

An 23 year old woman presents with 
a history of 10 kg weight loss in the previous 6 
months. She had menstruated only once 
during this time. On examination she had fine 
lanugo hair. She has a blood pressure of 
110/60 mmHg. 

Which one of the following would support the 
likely clinical diagnosis? 

1- Suppressed thyroid stimulating hormone 
concentration 

2- High plasma follicle stimulating hormone 
concentration 

3- Normal plasma cortisol concentration 

4- Low plasma testosterone concentration 

5- High SHBG concentration 



Answer & Comments 

Answer: 3- Normal plasma cortisol 

concentration 

The likely diagnosis is anorexia nervosa. 
Weight loss and oligomenorrhoea are 
associated features. FSH and LH 
concentrations are usually low . TSH and 
testosterone levels can be high. A normal 
cortisol level would be consistent. 


Dr. Khalid Yusuf El-Zohry - Sohag Teaching Hospital (01118391123) 

Ref MRCPass OE OE 2012 PasTest 2009 PassMedicine 2009 PasTest Exam ReviseMRCP 



470 


























El-zohry MRCP Questions Bank (Port 1) - 2013 


(For my personal use) 


[ Q: 1115 ] MRCPass - Endocrinology 

A 45 year old lady presents with 
lethargy, constipation and headaches. He has 
a previous medical history of gastrinoma 
diagnosed. Her calcium is 2.85 mmol/l, 
phosphate is 0.6 mmol/l and PTH is 10.3 
pmol/l (0.8-8.0). 

Which of the following is the likely diagnosis? 

1- Lung cancer 

2- Secondary hyperparathyroidism 

3- MEN type I 

4- Adrenal tumour 

5- Medullary thyroid carcinoma 



Answer & Comments 

Answer: 3- MEN type I 

She has a high calcium, low phosphate and 
raised PTH. The headaches could be caused by 
a pituitary tumour. In association with a 
pancreatic tumour (gastrinoma) and 
parathyroid gland tumour, MEN I is the most 
likely diagnosis. Diagnosis of gastrinoma is 
made on the basis of a high fasting plasma 
gastrin, high gastric acid secretion or a 
demonstrable pancreatic or gastrointestinal 
tumour - by CT or venous sampling for gastrin. 


MULTIPLE 

ENDOCRINE NEOPLASIA 
TYPE I 



prrufTARV 


PARATHYROID 


PANCREATIC 

ISLET 


[ Q: 1116 ] MRCPass - Endocrinology 

A 30 year old man has a recent 
history of head injury. Having recovered for 
several weeks, he is no increasingly lethargic. 

His investigations show : 

Sodium 158 (133-145) mmol/l 

Potassium 4.2 (3.5 - 5)mmol/l 

Chloride 118 (100 - 112)mmol/l 

Urea 6.0 (3 -7)pmol/l 

Creatinine 85 (50 - 100)pmol/l 

Urine Osmolality 80 mosmol/kg (50-1200) 
mosmol/kg 

Whot is the diagnosis? 

1- Addison's disease 

2- Syndrome of inappropriate antidiuretic 
hormone secretion (SIADH) 

3- Diabetes insipidus 

4- Diabetes mellitus 

5- Hyperchloraemic metabolic acidosis 



Answer & Comments 


Answer: 3- Diabetes insipidus 



Dr. Khalid Yusuf El-Zohry - Sohag Teaching Hospital (01118391123) 

Ref MRCPass OE OE 2012 PasTest 2009 PassMedicine 2009 PasTest Exam ReviseMRCP 






















El-zohry MRCP Questions Bank (Port 1) - 2013 


(For my personal use) 


The likely cause of this presentation is 
Diabetes Insipidus. Urine osmolality is low and 
there is hypernatraemia. Diabetes insipidus 
can occur in patients with pituitary damage (in 
this example from head injury) because of 
insufficient anti diuretic hormone secretion. 

[ Q: 1117 ] MRCPass - Endocrinology 

A 55 year old woman was found by 
the opthalmologist to have bitemporal 
hemianopia and headaches. MRI scan shows a 
pituitary tumour. 

Which one of the following is the best 
investigation to confirm the diagnosis of 
acromegaly in this patient? 

1- Insulin-like growth factor-1 (IGF-1) 

2- Glucose tolerance test with growth 
hormone concentrations 

3- An insulin tolerance test with growth 
hormone concentrations 

4- 9am growth hormone concentrations 

5- Growth hormone releasing hormone test 



Answer & Comments 

Answer: 2- Glucose tolerance test with growth 
hormone concentrations 

The diagnosis of acromegaly is confirmed by 
inadequate suppression of growth hormone 
concentrations below 2 mU/l in an oral 
glucose tolerance test. 


[ Q: 1118 ] MRCPass - Endocrinology 

A 35 year old woman has a thyroid 
goitre and complains of polyuria. Thyroid 
function tests were normal, but 
hypercalcaemia is noted. Her blood pressure is 
155/105 mmHg. A chest radiograph is normal. 

A thyroidectomy is performed, frozen sections 
of several thyroid masses show a malignant 
neoplasm composed of polygonal cells in 
nests. Immunoperoxidase staining of the 
sections is positive for calcitonin. 



The diagnosis is likely to be: 

1- Papillary thyroid carcinoma 

2- Parathyroid hyperplasia 

3- Multiple endocrine neoplasia type 2 

4- Follicular thyroid carcinoma 

5- Anaplastic thyroid carcinoma 


Answer & Comments 

Answer: 3- Multiple endocrine neoplasia type 
2 

Medullary thyroid cancer, 

hyperparathyroidism, and 

phaeochromocytoma form part of the MEN 2 
syndrome. 

MEN 2a is associated with medullary thyroid 
carcinoma (MTC), parathyroid tumours (10- 
20%) and pheochromocytoma (20-50%). 

MEN 2b is associated with presentation of 
medullary thyroid carcinoma, parathyroid 
tumours and pheochromocytoma + 
ganglioneuromatosis (pathognomonic), and 
marfanoid habitus. 

^ [ Q: 1119 ] MRCPass - Endocrinology 

# A 25 year old patient has 

palpitations. She has lost 1 stone of weight 
over the past 3 months. On 

examination, she has tremors in her hands 
and palmar erythema. Knee and ankle reflexes 
are brisk. 

What is the likely diagnosis? 

1- Acromegaly 

2- Congenital adrenal hyperplasia 

3- Phaeochromocytoma 

4- Benign essential tremor 

5- Thyrotoxicosis 


Answer & Comments 


Answer: 5- Thyrotoxicosis 


Dr. Khalid Yusuf El-Zohry - Sohag Teaching Hospital (01118391123) 

Ref MRCPass OE OE 2012 PasTest 2009 PassMedicine 2009 PasTest Exam ReviseMRCP 



472 





























El-zohry MRCP Questions Bank (Port 1) - 2013 


(For my personal use) 


Recognised features of thyrotoxicosis are: 
Weight loss, Palpitations, Dyspnoea, 
Irritability, Psychosis, Tremor, Pruritus, 
Diarrhoea, Palmar erythema, Hypercalcaemia, 
Hyper-reflexia, Bone mineral loss and 
Alopecia. 


[ Q: 1120 ] MRCPass - Endocrinology 

A 30 year old lady has lethargy, 
fevers and palpitations. Her GP sends thyroid 
function which show a Free T 4 of 30 pmol/l 
and TSH of <0.01 mU/l. Her symptoms seem 
to spontaneously resolve 6 weeks later. 

Which one of the following is the most likely 
diagnosis? 

1- Multinodular goitre 

2- Thyroid carcinoma 

3- Grave's disease 

4- Subacute thyroiditis 

5- Iodine deficiency 

Answer & Comments 

Answer: 4- Subacute thyroiditis 

Subacute thyroiditis is a transient thyroditis 
which is thought to be of viral aetiology. There 
is a sw ollen painful thyroid gland with 
hyperthyroidism. This is usually followed by a 
period of hypothyroidism with raised TSH 
weeks later. Antithyroid medication is not 
effective. Steroids can be used in severe cases. 


Subacute Painless Thyroiditis 

(Silent) 

• . i- n 1 

r 


lit 





[ Q: 1121 ] MRCPass - Endocrinology 

A 40 year old woman has developed 
secondary amenorrhoea, headaches and 
bilateral galactorrhoea. She undergoes 
investigations for a suspected pituitary 
tumour. An MRI show s a 15 mm pituitary 
tumour. 


Which one of the following is the next most 
appropriate step? 

1- Skull X ray 

2- Formal visual field testing 

3- Transsphenoidal hypophysectomy 

4- An oral glucose tolerance test 

5- Octreotide treatment 


Answer & Comments 

Answer: 4- An oral glucose tolerance test 

A macroadenoma (> 10mm) suggests a large 
tumour. Often this may turn out to be 
acromegaly or a macroprolactinoma and 
hence oral glucose tolerance test is useful. 

Octreotide should be used only if acromegaly 
is confirmed, and Cabergoline for a 
macroprolactinoma. 


[ Q: 1122 ] MRCPass - Endocrinology 

A 33 year old lady has a painless 
thyroid nodule. Her mother had a similar 
condition after which she had surgery and 
thyroid replacement therapy. 

Thyroid function tests show : 

Free T 4 8 (10-24) pmol/l 

TSH 7 (0.3-4) mU/l 

What is the likely diagnosis? 

1- Single nodular goitre 

2- Grave's disease 

3- Dyshormonogenetic goitre 

4- Hashimoto's thyroiditis 

5- Papillary thyroid carcinoma 




Dr. Khalid Yusuf El-Zohry - Sohag Teaching Hospital (01118391123) 

Ref MRCPass OE OE 2012 PasTest 2009 PassMedicine 2009 PasTest Exam ReviseMRCP 
























El-zohry MRCP Questions Bank (Port 1) - 2013 


(For my personal use) 


Answer & Comments 

Answer: 4- Hashimoto's thyroiditis 

Hashimo's thyroiditis can be familial. Long 
standing thyroiditis leads to fibrotic changes 
which can lead to a palpable lump. 
Hypothyroidism is frequently associated. If 
there is lymphadenopathy, then papillary 
thyroid carcinoma should be considered). 
Dyshormonogenetic goitre is very rare and 
congenital. It tends to present in childhood / 
teenage years with hypothyroidism. 


^ [ Q: 1124 ] MRCPass - Endocrinology 

f --— 

# A 65 year old man has had type II 
diabetes for 5 years. He has been lethargic 
and undergone a series of tests which reveal 
that he has type IV renal tubular acidosis. 

Which one of the following should be used for 
treatment? 

1- Metyrapone 

2- Lithium 

3- Aminophylline 


[ Q: 1123 ] MRCPass - Endocrinology 

A 55 year old man is under 
investigation for weakness of his legs, 
pigmentation of the skin, hypertension and 
glycosuria. 

Results of serum cortisol estimations are: 

Time Cortisol 

0900 1000 nmol/l 

2400 1050 nmol/l 

After 48 hours of dexamethasone 8 mgs daily: 

Time Cortisol 

0900 1075 nmol/l 

What is the likely diagnosis? 



4- Fludrocortisone 

5- Acetic Acid 


Answer & Comments 

Answer: 4- Fludrocortisone 

Type 4 RTA is caused by a defect in the distal 
tubule, but it is different from classic distal 
RTA and proximal RTA because it results in 
hyperkalaemia rather than hypokalaemia. RTA 
type 4 is in effect hyporeninaemic 
hypoaldosteronism. 

It occurs when blood levels of aldosterone are 
low or when the kidneys do not respond to it. 
Fludrocortisone is usually effective as a form 
of treatment. 


1- Renal tubular acidosis 

2- Conn's syndrome 

3- Acromegaly 


Type IV RTA may result from may result from 
sickle cell disease, urinary tract obstruction, 
lupus, amyloidosis, or transplantation. 


4- Ectopic ACTH secretion 

5- Panhypopituitarism 

Answer & Comments 

Answer: 4- Ectopic ACTH secretion 

The patient has Cushing's syndrome with 
failure to suppress cortisol levels during the 
dexamethasone suppression test. This will 
indicate either an adrenal tumour or an 
ectopic source of ACTH. 


[ Q: 1125 ] MRCPass - Endocrinology 

* A 45 year old man presents with a 
blood pressure of 180/95. He has a sodium of 
149 mmol/l and potassium of 3.1 mmol/I. 

Which one of the following diagnosis is most 
likely? 

1- Phaeochromocytoma 

2- SIADH 

3- 21-hyroxylase deficiency 

4- Conn's syndrome 


Dr. Khalid Yusuf El-Zohry - Sohag Teaching Hospital (01118391123) 

Ref MRCPass OE OE 2012 PasTest 2009 PassMedicine 2009 PasTest Exam ReviseMRCP 






























El-zohry MRCP Questions Bank (Port 1) - 2013 


(For my personal use) 


5- Addison's disease 

Answer & Comments 

Answer: 4- Conn's syndrome 

Conn's syndrome is the result of 
mineralocorticoid excess due to an adrenal 
adenoma (60%) or bilateral hyperplasia (40%). 
A low potassium (< 3.5 mmol/L) accompanied 
by metabolic alkalosis is characteristic. Renin 
and aldosterone measurements may help to 
determine the diagnosis. There is autonomous 
aldosterone secretion in the presence of low 
or suppressed renin activity. 



Conn's syndrome tumour 


[ Q: 1126 ] MRCPass - Endocrinology 

fi - 

* A 65 year old man with diet 
controlled type 2 diabetes mellitus and a 
creatinine of 350 umol/l. 


utilization. Thus, some insulin must be 
produced for it to have an effect. It is 
associated hypoglycaemia although this side 
effect is unusual. It is contraindicated in 
subjects with renal failure, hepatic failure and 
heart failure due to an association with lactic 
acidosis. 


[ Q: 1127 ] MRCPass - Endocrinology 

A 52 year old female presents with a 
history of weight loss, tremor and increased 
sweating. Investigations reveal: 

Free thyroxine 35 pmol/L (10-24) 

TSH <0.1 mU 

This patient is treated with radioactive iodine. 

Which one of the following is on adverse effect 
of rodiooctive iodine therapy? 

1- Hyperthyroidism 

2- Hypothyroidism 

3- Goitre 

4- Medullary thyroid carcinoma 

5- Hypercalcaemia 



Answer & Comments 


Answer: 2- Hypothyroidism 


Which of the following drugs should be 
avoided if possible? 

1- Isophane insulin 

2- Actrapid insulin 

3- Glimepiride 

4- Gliclazide 

5- Metformin 


Answer & Comments 

Answer: 5- Metformin 

Metformin is a biguanide which acts by 
improving insulin sensitivity through 
mechanisms involinge hepatic 

gluconeogenesis and improved muscle glucose 


The major complication of treatment with 
radioactive iodine is the progressive incidence 
of hypothyroidism and thyroid replacement 
may be necessary in the future. 


[ Q: 1128 ] MRCPass - Endocrinology 

A 45 year old Type 1 diabetic has an 
annual checkup. 

Which one of the following fundoscopy 
findings warrants an urgent referral to the 
opthalmologist? 

1- Photocoagulation scars 

2- Vitreous haemorrhage 

3- Peripheral microaneurysms 

4- Hard exudates close to the macula 



Dr. Khalid Yusuf El-Zohry - Sohag Teaching Hospital (01118391123) 

Ref MRCPass OE OE 2012 PasTest 2009 PassMedicine 2009 PasTest Exam 




ReviseMRCP 

475 



























El-zohry MRCP Questions Bank (Port 1) - 2013 


(For my personal use) 


5- Blot haemorrhages 

Answer & Comments 

Answer: 2- Vitreous haemorrhage 

Vitreous haemorrhage is a sign of proliferative 
diabetic retinopathy but may also occur with 
central retinal vein occlusion and age-related 
macular degeneration with breakthrough 
bleeding. Retinal detachment may occur and 
vision may be impaired. 



Vitreous Haemorrhage in diabetes 


[ Q: 1129 ] MRCPass - Endocrinology 

A 35 year old woman presented with 
vomiting. Blood tests reveal high parathyroid 
levels and hypercalcaemia. 

Which one of the following is most likely to 
have caused primary hyperparathyroidism? 

1- Medullary thyroid carcinoma 

2- Parathyroid adenoma 

3- Parathyroid carcinoma 

4- Papillary thyroid carcinoma 

5- Follicular thyroid carcinoma 



Answer & Comments 

Answer: 2- Parathyroid adenoma 

Primary hyperparathyroidism can be caused 
by hyperplasia, carcinoma. 


Adenomas are the most common, 
representing 80-85% of cases. 



A Tc-MIBI exam demonstrating a parathyroid 

adenoma 


[ Q: 1130 ] MRCPass - Endocrinology 

A 68 year old man presents with 
seizures. Past medical history includes a 
smoking history of 35 cigarettes a day for 
many years. His family report a 2 stone weight 
loss over the past 8 months. 

Serum electrolytes show : 

Sodium 109 mmol/L (135-145) 

Potassium 4.2 mmol/L (3.5-5.0) 

Glucose 7 mmol/L 

Urea 3.0 mmol/L (3.2-8.1) 

These findings are most likely to be due to: 

1- Epilepsy 

2- Syndrome of inappropriate ADH secretion 

3- Diuretics 

4- Diabetes insipidus 

5- Pituitary adenoma 



Answer & Comments 

Answer: 2- Syndrome of inappropriate ADH 
secretion 

The biochemistry points to a syndrome of 
inappropriate ADH secretion, with a low 
serum osmolality [2(Na + K) + urea + glucose]. 
The clinical picture is consistent with small cell 
carcinoma of the lung, which is often 
associated. 



Dr. Khalid Yusuf El-Zohry - Sohag Teaching Hospital (01118391123) 

Ref MRCPass OE OE 2012 PasTest 2009 PassMedicine 2009 PasTest Exam ReviseMRCP 



























El-zohry MRCP Questions Bank (Part 1) - 2013 


(For my personal use) 


Hyponatremia with hypo-osmolality 

Elevated renal excretion of sodium (>20 mEq/L) 

Normal volume status 

Inappropriately elevated urine osmolality for the plasma 
osmolality 


SIADH 


[ Q: 1131 ] MRCPass - Endocrinology 

A 65 year old man presents with a 
history of increased sw eating and daily 
headaches. On examination the patient has 
large spade like hands and the facial features 
are exaggerated with large nose, prominent 
jaw and thick lips. 

Which screening test will yield the diagnosis? 

1- Alpha feto protein 

2- Prolactin level 

3- Serum glucose 

4- Plasma Insulin-like Growth Factor levels 

5- Serum phosphate 



testosterone 10 (11-36) nmol/L 
LH 6 (0.5-9) IU/L 
FSH 7 (1-8 ) IU/L 

Early morning cortisol 450 (130-690) nmol/L 

Growth hormone 2.5 (< 5.5) mlU/L 

Which one of the following would support a 
diagnosis of pituitary tumor? 

1- Raised FSH and LH 

2- High prolactin 

3- Low GH 

4- Low testosterone 

5- Low cortisol 


Answer & Comments 

Answer: 2- High prolactin 

A high prolactin level indicates likely 
prolactinoma in this case causing the 
macroadenoma seen on the CT scan. 


Answer & Comments 

\7 

[ Q: 1133 ] MRCPass - Endocrinology 

Answer: 4- Plasma Insulin-like Growth Factor 

h_] 

A 35 year old female presents with 


levels 


Elevated GH levels increase IGF-1 blood levels. 
Because IGF-1 levels are much more stable 
over the course of the day, they are often a 
more practical and reliable measure than GH 
levels. Elevated IGF-1 levels almost alw ays 
indicate acromegaly. 


sw eatiness, tremors and palpitations. 
Examination reveals a exopthalmos and a 
goitre. Her GP requests TFT's which show a 
TSH of 0.01mU/l, FT4 35 pmol/l, FT3 3.1 
nmol/l. She has positive antithyroid 
antibodies. 

What is the likely diagnosis? 


The oral glucose tolerance test is also used to 
diagnose acromegaly, because ingestion of 75 
g of the sugar glucose low ers blood GH levels 
less than 2 ng/ml in healthy people. In 
patients with acromegaly, this reduction does 
not occur 


1- Hashimoto's thyroiditis 

2- Grave's disease 

3- Iodine deficiency 

4- Post radioiodine treatment 

5- Papillary thyroid carcinoma 


^ [ Q: 1132 ] MRCPass - Endocrinology 

- 

# A 20 year old man with headaches 
had a CT scan which show s a 1.5 cm diameter 
pituitary mass. His investigations show: Graves disease is the diagnosis - thyroid 

, # . „ autoantibodies are increased. Almost 80% of 

prolactin concentration 1,520 (50-450)mu/l 


Answer & Comments 


Answer: 2- Grave's disease 


Dr. Khalid Yusuf El-Zohry - Sohag Teaching Hospital (01118391123) 

Ref MRCPass OE OE 2012 PasTest 2009 PassMedicine 2009 PasTest Exam ReviseMRCP 



ATT 

































El-zohry MRCP Questions Bank (Port 1) - 2013 


(For my personal use) 


patients have exopthalmos. Medical 
treatment such as carbimazole or radioiodine 
treatment are recommended rather than 
surgery. There will be increased uptake on the 
thyroid radioisotope scan. 



Exopthalmos in Grave's disease 


[ Q: 1134 ] MRCPass - Endocrinology 

A 45-year-old female is under 
investigation for excessive weight gain. She 
has central obesity with abdominal striae. She 
is hirsute. 

A dexamethasone suppression test has been 
performed with 8 mg dexamethasone a day 
and the results are as follows: 

Day 9 am Cortisol 

0 970 nmol/L 

2 335 nmol/l 

3 110 nmol/l 
What is the diagnosis? 

1- Adrenal adenoma 

2- Ectopic ACTH 

3- Cushing's disease 

4- Phaeochromocytoma 

5- Conn's syndrome 



Answer & Comments 

Answer: 3- Cushing's disease 

Cushing's disease is pituitary dependent 
Cushing's (excessive ACTH secretion. 


Cushing's syndrome refers to all forms of 
Cushing's including adrenal adenoma. 

In the dexamethasone suppression test 
normal individuals suppress cortisol levels to < 
50 nmol/L. 


90 % of patients with pituitary dependent 
disease suppress production of cortisol to < 50 
% of the baseline cortisol level on day 2. 



A Cushingoid patient 



[ Q: 1135 ] MRCPass - Endocrinology 

An 8 year old boy with bilateral 
gynaecomastia is being investigated. 


Which one of the features points towards 
hypogonadotrophic hypogonadism being a 
likely cause? 


1- Small testes 


2- Microphallus 

3- Hypospadia 

4- Hirsutism 


5- Anosmia 


Answer & Comments 

Answer: 5- Anosmia 

Hypospadias, micropenis, cryptorchidism can 
all point tow ards genital developmental 
disorder. Although hypogonadotrophic 

hypogonadism can be caused by pituitary 
disorders, a common presentation in 

childhood is with anosmia (Kallman's 

syndrome). 



Dr. Khalid Yusuf El-Zohry - Sohag Teaching Hospital (01118391123) 

Ref MRCPass OE OE 2012 PasTest 2009 PassMedicine 2009 PasTest Exam ReviseMRCP 

























El-zohry MRCP Questions Bank (Port 1) - 2013 


(For my personal use) 


^ [ Q: 1136 ] MRCPass - Endocrinology 

- 

# A 42 year old man has episodes of 
dizziness, sw eating and tremors, which are 
relieved by glucose. He has gained some 
weight recently and drinks approximately 10 
units of alcohol weekly. 

His investigations show normal full blood 
count, normal urea and electrolytes and a 
fasting plasma glucose concentration of 3.5 
mmol/I. 

Whot is the best investigation to do? 

1- 24 hour urine catecholamines 

2- Glucose tolerance test 

3- Water deprivation test 

4- 72 hour fast 


Answer & Comments 

Answer: 3- Intravenous sodium nitroprusside 

In the management of acute hypertensive 
crisis of phaeochromocytoma as in this case, 
IV administration of sodium nitroprusside, 
nitroglycerine, or phentolamine can be used. 

Preoperatively, the recommendation is 
preoperative adrenergic-blockade of al and 
a2 receptors with phenoxybenzamine (10-30 
mg tw ice daily), or al receptors with prazosin 
(starting with 1 to 2 mg three times daily. 

Beta blockers can be useful for arrhythmias, 
but should not be commenced before alpha 
blockers because b-blockade alone can cause 
marked hypertension. 


5- MRI scan of the pituitary 


Answer & Comments 

Answer: 4- 72 hour fast 

An insulinoma should be excluded (high 
insulin levels, low C peptide levels during 
hypoglycaemic episode) with a 72 hour fast. 



[ Q: 1137 ] MRCPass - Endocrinology 

A 35 year old woman is admitted 
with a blood pressure of 230/120. She has a 
sinus tachycardia of HR 160 with intermittent 
runs of non sustained ventricular tachycardia. 
24 hour urine show s increased Adrenaline of 
720 (<80 nmol/24 hours) and Noradrenaline 
2300 (<780 nmol/24 hours). 

Which one of the following medications would 
be most useful? 

1- Intravenous labetalol 

2- Intravenous amiodarone 

3- Intravenous sodium nitroprusside 

4- Intravenous diltiazem 

5- Oral flecainide 



Phaeochromocytoma 


[ Q: 1138 ] MRCPass - Endocrinology 

# A 40 year old lady has increasing 
hirsutism. She is embarassed about having to 
shave her chin and also her chest. Her voice is 
becoming deeper. 


Which one of the following is the most likely 
diagnosis? 

1- Drug induced hirsutism 


2- Polycystic ovarian syndrome 

3- Adrenal tumour 


4- Congenital adrenal hyperplasia 

5- Ovarian carcinoma 


Dr. Khalid Yusuf El-Zohry - Sohag Teaching Hospital (01118391123) 

Ref MRCPass OE OE 2012 PasTest 2009 PassMedicine 2009 PasTest Exam ReviseMRCP 





























El-zohry MRCP Questions Bank (Port 1) - 2013 


(For my personal use) 


Answer & Comments 


Answer: 3- Adrenal tumour 


urine osmolality high plasma osmolality. This 
is most likely to be caused by lithium. 


Rapid development of hirsutism is usually 
caused by an adrenal tumour. There are high 
testosterone or DHEA levels in the plasma. 


MRI showing an adrenal tumour 


[ Q: 1140 ] MRCPass - Endocrinology 

A 45 year old woman has a blood 
pressure of 210/100 mmHg and is assessed in 
the hypertension clinic. She is complaining of 
headaches and dizziness. Her fundi show silver 
wiring and tortuosity of vessels. 

Her blood tests are Hb 12 g/dl, WCC 7 x 10 9 /L, 
Platelets 250 x 10 9 /L, urea 7 pmol/l, creatinine 
75 pmol/l, sodium 146 mmol/I, potassium 2.7 
mmol/I, cholesterol 4.5 mmol/l, triglyceride 
1.8 mmol/l, bicarbonate 32 mmol/l. 

Which one of the following investigations is 
most important? 

1- 24 hour urine for catecholamines 




[ Q: 1139 ] MRCPass - Endocrinology 

A 46 year male presents to the A&E 
with complaints of polyuria. He mentions that 
he has been passing about 4 litres of urine per 
day. He has recently been prescribed a new 
drug. Investigations show : 

Serum sodium 141 mmol/l 

Serum potassium 4.2 mmol/l 

Plasma osmolality 290 mosmol/l (275-290) 

Urine osmolality 330 mosmol/l (350-1000) 

What drug was likely to have been prescribed? 

1- Carbamazepine 

2- Chlorpropamide 

3- Fluoxetine 



2- 24 hour tape recording 

3- Renin-aldosterone levels 

4- Glucose measurement 

5- Urine electrolytes 


Answer & Comments 

Answer: 3- Renin-aldosterone levels 

The combination of high sodium, low 
potassium, high bicarbonate and hypertension 
in this patient makes one suspicious of Conn's 
syndrome. A high aldosterone and low renin 
level makes Conn's a possible diagnosis. An 
ambulant aldosterone/renin ratio of 25 
ng/mU has been used as a cut off as being 
high. 


4- Furosemide 

5- Lithium 


Answer & Comments 


Answer: 5- Lithium 


[ Q: 1141 ] MRCPass - Endocrinology 

A 52 year old lady presents with 
palpitations. She has a Thyroid-stimulating 
hormone of 0.01 mU/l (0.3-4.0) and free 
thyroxine of 33 (10-24) pmol/l. 



The patient has drug induced Diabetes 
Insipidus based upon high urine output, low 


Which one of the following conditions is 
consistent with these results? 

1- Radioiodine treatment 



Dr. Khalid Yusuf El-Zohry - Sohag Teaching Hospital (01118391123) 

Ref MRCPass OE OE 2012 PasTest 2009 PassMedicine 2009 PasTest Exam 


ReviseMRCP 






























El-zohry MRCP Questions Bank (Port 1) - 2013 


(For my personal use) 


2- Secondary hyperthyroidism 

3- Secondary hypothyroidism 

4- Iodine deficiency 

5- Graves disease 


Answer & Comments 

Answer: 5- Graves disease 

Graves disease is the most likely from the 
picture of thyrotoxicosis. Secondary 
hyperthyroidism results in increased secretion 
of TSH (Thyroid Stimulating Hormone) by the 
pituitary, therfore there is an increase in both 
TSH and serum thyroxine. 


[ Q: 1142 ] MRCPass - Endocrinology 

A 18 year old man is referred to the 
endocrine clinic by his GP because he had 
poorly developed secondary sexual 
characteristics. He describes anosmia. On 
examination, he has lack of axillary or pubertal 
hair, small sexual organs (testicular volumes of 
approximately 5 ml). 

Investigations reveal: 

Testosterone 3.5 nmol/L (10-30) 

Prolactin 350 mll/L (<450) 

FSH 2.2 iu/L (1-7) 

LH 1.8 iu/L (1-10) 

What is the most likely diagnosis? 

1- Testicular feminisation syndrome 

2- Kallman's syndrome 

3- Congenital adrenal hyperplasia 

4- Marfan's syndrome 

5- Klinefelter's syndrome 



Answer & Comments 

Answer: 2- Kallman's syndrome 

X-linked Kallmann's syndrome (KS) is a genetic 
disease characterized by anosmia and 
hypogonadism due to impairment in the 


development of olfactory axons and in the 
migration of gonadotropin-releasing hormone 
(GnRH)-producing neurons. 50% of the 
patients also have unilateral renal agenesis. 
Abnormalities of the sexual organs, mainly 
cryptorchidism and testicular atrophy are 
common. 



[ Q: 1143 ] MRCPass - Endocrinology 

A 55 year old patient with known 
acromegaly is reviewed during a 6 monthly 
outpatient appointment. 


Which one of the following clinical features is 
an indicator of disease activity? 


1- Increase in the loss of visual field 


2- Palpitations 

3- Depression 

4- Worsening nystagmus 

5- Low oxygen saturations 


Answer & Comments 

Answer: 1- Increase in the loss of visual field 

In acromegaly, the features of disease activity 
are headache, increase in ring size, increased 
denture size, increased sweating, skin tags, 
glycosuria, hypertension and increased loss of 
visual field. 

[ Q: 1144 ] MRCPass - Endocrinology 

A 66 year old man has no previous 
medical history but presents with tendon 
xanthomas. He is not on regular medication. 
He drinks 5 pints of beer a day. A GP has 
investigated has the following blood results: 
cholesterol 6.5 (<5.2) mmol/I, triglycerides 1.9 
(0.45-1.69) mmol/l. Thyroid function and 
albumin are normal. 

Which one of the following is most likely to be 
the cause? 

1- Oestrogen therapy 

2- Familial hypercholesterolemia 



Dr. Khalid Yusuf El-Zohry - Sohag Teaching Hospital (01118391123) 

Ref MRCPass OE OE 2012 PasTest 2009 PassMedicine 2009 PasTest Exam 




ReviseMRCP 

481 




























El-zohry MRCP Questions Bank (Port 1) - 2013 


(For my personal use) 


3- Alcoholism 

4- SLE 

5- Thyrotoxicosis 


This is most likely due to inaccurate sampling 
from a drip arm. In the other causes of 
hyponatraemia, the plasma osmolality should 
be low as well. 


Answer & Comments 

Answer: 3- Alcoholism 

Causes of secondary hyperlipidaemia include: 

Type 2 diabetes mellitus 

■ chronic renal failure 

■ alcoholism 

■ drugs including: thiazides, 

glucocorticoids, oestrogens, 

cyclosporine, protease inhibitors 

■ hypothyroidism 
nephrotic syndrome. 


[ Q: 1146 ] MRCPass - Endocrinology 

* A 40 year old man attends a fertility 
clinic. On examination, he is tall, slim and has 
bilateral gynaecomastia. 

Investigations show high levels of plasma 
gonadotrophins. 

What is the likely diagnosis? 

1- Testicular feminisation syndrome 

2- Klinefelter's syndrome 

3- Marfan's syndrome 

4- Noonan's syndrome 

5- Homocystinuria 


[ Q: 1145 ] MRCPass - Endocrinology 

A 30 year old man has been 
admitted diarrhoea. His blood pressure is 
130/80 mmHg, and temperature 38°C. 

Blood results show Na 118 mmol/I, K 2.8 
mmol/I urea 2 pmol/l, creatinine 50 pmol/l, 
chloride 70 mmol/I, bicarbonate 20 mmol/I. 
Serum osmolality is 287 mosm/l and urine 
osmolality is 700 mosm/l. 

Which is the most likely diagnosis for the 
hyponatraemia? 

1- Addisonian crisis 

2- Diarrhoea and salt loss 

3- SIADH 

4- Blood taken from a drip arm 

5- Drugs 



Answer & Comments 

Answer: 4- Blood taken from a drip arm 

There is hyponatraemia, hypokalaemia and 
low urea with normal osmolality. 


Answer & Comments 

Answer: 2- Klinefelter's syndrome 

Klinefelter's syndrome is the most common 
cause of male hypogonadism with an 
incidence of 1 in 1000 male births. Individuals 
have an extra X chromosome. Usually, the 
karyotype is 47, XXY. Accelerated atrophy of 
germ cells before puberty results in sterility 
with small, firm testes. Many patients are tall 
with relatively long legs. 

Behavioural disorders and delayed speech 
development are common. Testosterone 
therapy may be used to improve the 
development of secondary sexual 
characteristics. 


^ [ Q: 1147 ] MRCPass - Endocrinology 

mlt 

# An 22 year old law yer has noted 
have persistent polyuria in excess of 4 litres 
per day. She mentions recently being involved 
in an accident and sustained head injury which 
she thinks she has recovered from. 

Investigations reveal: 


Dr. Khalid Yusuf El-Zohry - Sohag Teaching Hospital (01118391123) 

Ref MRCPass OE OE 2012 PasTest 2009 PassMedicine 2009 PasTest Exam ReviseMRCP 



482 




























El-zohry MRCP Questions Bank (Port 1) - 2013 


(For my personal use) 


potassium 4.2 mmol/l 
calcium 2.3 (2.2-2.6) mmol/l 
glucose 5.2 (3.0-6.0) 

Which one of the following is the best way of 
confirming the diagnosis? 

1- Cortisol levels 

2- Water deprivation test 

3- ADH concentration 

4- Measure autoantibodies to vasopressin 

5- Therapeutic trial of low dose DDAVP 


Answer & Comments 

Answer: 2- Water deprivation test 

The diagnosis is likely to be diabetes insipidus. 
This can be confirmed a w ater deprivation 
test where failure of urine concentration 
would be expected. During this test a patient 
is not allowed to drink and plasma ADH, 
plasma osmolality and urine osmolality are 
measured. 

A therapeutic trial of DDAVP is only 
appropriate if a diagnosis of Dl is confirmed 
(on the water deprivation test) and helps to 
differentiate betw een cranial or nephrogenic 
diabetes insipidus. 


[ Q: 1148 ] MRCPass - Endocrinology 

A 30 year old woman with a 10-year 
history of chronic renal insufficiency 
secondary to bilateral obstructive 
hydronephrosis presented for evaluation of 
pain in the right hip. She had undergone 
bilateral ureterostomies, but her creatinine 
level remained chronically elevated. 

An X ray revealed a fracture of the right 
femoral neck necessitating internal fixation. 
The patient also reported that the tips of her 
fingers had increased in size over the past six 
months. 



On examination, she had clubbing of her 
fingers with no skin lesions, joint sw elling or 
tenderness. 

What is the diagnosis? 

1- Rheumatoid arthritis 

2- Heberden's nodes 

3- Osteopetrosis 

4- Pseudohypoparathyroidism 

5- Hyperparathyroidism 


Answer & Comments 

Answer: 5- Hyperparathyroidism 

In this patient, hyperparathyroidism is 
secondary to renal osteodystrophy. The hip 
fracture and clubbing are consistent with 
hyperparathyroidism. 


[ Q: 1149 ] MRCPass - Endocrinology 

A 72 year old woman complains of 
muscle aches and pains and difficulty standing 
up. Investigations show : 

Serum calcium 1.9 mmol/L (2.2-2.6) 

Phosphate 0.9 mmol/L (0.8-1.6) 

Albumin 36 mg/L 

Alkaline phosphatase 280 IU/L (30-230) 

The most likely diagnosis is: 

1- Primary hyperparathyroidism 

2- Paget's disease 

3- Chronic renal failure 

4- Osteoporosis 

5- Osteomalacia 



Answer & Comments 

Answer: 5- Osteomalacia 

A low calcium, normal phosphate and mildly 
raised alkaline phosphatase fits best with 
osteomalacia (vitamin D deficiency). Features 
of osteomalacia include bony pain and 


Dr. Khalid Yusuf El-Zohry - Sohag Teaching Hospital (01118391123) 

Ref MRCPass OE OE 2012 PasTest 2009 PassMedicine 2009 PasTest Exam ReviseMRCP 



483 




























El-zohry MRCP Questions Bank (Port 1) - 2013 


(For my personal use) 


deformity, increased tendency to fracture, 
proximal myopathy and hypocalcaemia. In 
adults, treatment is with a daily dose of 
calciferol (20-25 micrograms). 


2- Commence a beta blocker 

3- Add insulin 

4- No action and repeat routine checks in 3 
months 



[ Q: 1150 ] MRCPass - Endocrinology 

A 30 year old of average height and 
weight presents with polyuria and thirst. He 
has a blood glucose of 15 mmol/l. There is no 
ketonuria and pH on the blood gas is 7.35. 

How should he be treated? 


1- Start metformin 


2- A fasting blood glucose should be sent 
before treatment 

3- Subcutaneous insulin should be started 

4- Commence on gliclazide and reassessment 
with BM monitoring at home 

5- Dietary advice, review in a month with 
repeat glucose without any treatment 


Answer & Comments 

Answer: 4- Commence on gliclazide and 
reassessment with BM monitoring at home 

He is a type 1 diabetic but there are no 
features of ketonuria or acidosis. He may have 
some residual (3 islet cell function and hence 
sulphonylureas may help to stimulate insulin 
production. 


[ Q: 1151 ] MRCPass - Endocrinology 

A 45 year old type II diabetic patient 
is screened at the clinic. His HBAlc is 6.7 % 
and his BMs are varying betw een 5 to 10. He 
has blood pressure of 135 / 70. There is no 
signs of dot and blot haemorrhages on 
fundoscopy. 

He has urine dipstick showing protein ++ and 
blood +. Creatinine is 80pmol/l. 

Which one of the following is the best 
management step? 

1- Refer to a renal physician 



5- Commence ACE inhibitor 


Answer & Comments 

Answer: 5- Commence ACE inhibitor 

In diabetic patients with microalbuminaemia 
demonstrated on urine dipstick, ACE inhibitors 
have been show n to reduce progression tow 
ards diabetic nephropathy. 


[ Q: 1152 ] MRCPass - Endocrinology 

A 62 year old type II diabetic patient 
with a medial malleolus ulcer is being 
examined. 

Which one of the following indicates a bad 
prognosis? 

1- Flat foot 

2- Absent pain sensation 

3- Absent pulses 

4- Absent vibration sense 

5- Previous foot ulceration 



Answer & Comments 

Answer: 4- Absent vibration sense 

All are associated with foot ulceration. Strong 
predictors of foot ulceration are altered foot 
sensation, foot deformities, and previous foot 
ulcer or amputation, with altered foot 
sensation being one of the strongest 
predictors. 


[ Q: 1153 ] MRCPass - Endocrinology 

A 42 year old woman presents with a 
6 month history of galactorrhoea. She also 
complains of a 5 year history of dyspepsia. 

Examination reveals a BMI of 23.5 kg/m 2 a 
small amount of galactorrhoea was noticed. 



Dr. Khalid Yusuf El-Zohry - Sohag Teaching Hospital (01118391123) 

Ref MRCPass OE OE 2012 PasTest 2009 PassMedicine 2009 PasTest Exam ReviseMRCP 



484 
































El-zohry MRCP Questions Bank (Port 1) - 2013 


(For my personal use) 


Slight neck enlargement was noticed during 
examination. 

Investigations show: 

a prolactin concentration of 980 mU/l (50-500 
mU/l) 

an oestradiol of 85 pmol/l (130-500) 
a LH of 3.1 mU/l (3.5-8) 
a FSH of 2.8 mU/l (3-8) 

Ultrasonography (revealed a parathyroid 
enlargement and a 99mTc04/MIBI scan 
showed tw o hyperplasic lesions. 

Whot is the likely diagnosis? 

1- Sheehan's syndrome 

2- Acromegaly 

3- Addison's disease 

4- MEN type 1 

5- Drug induced hyperprolactinaemia 

Answer & Comments 

Answer: 4- MEN type 1 

The biochemical picture suggests a diagnosis 
of a microprolactinoma. The patient is also 
likely to have a parathyroid tumour. In the 
presence of dyspepsia, a gastrinoma should be 
considered to form an overall diagnosis of 
MEN 1 [pituitary tumour, parathyroid tumour, 
pancreatic tumour]. 

Hyperparathyroidism is the most common 
manifestation of MEN 1, caused by 
hyperplasia of multiple parathyroid glands. 
Pancreatic islet cell tumors represent the 
second most common manifestation of MEN 1 
and occur in 80% of patients. Gastrinomas, 
insulinomas or glucagonomas are all 
pancreatic tumours which may occur. 

[ Q: 1154 ] MRCPass - Endocrinology 

# A 50 year old woman with 
Cushingoid features has been shown to have 
elevated early morning and midnight plasma 


cortisol levels. 

Which one of the following techniques is most 
specific in differentiating between ectopic 
Cushing's syndrome from pituitary dependent 
Cushing's disease? 

1- Low dose Dexamethasone suppression test 

2- High dose Dexamethasone suppression test 

3- Inferior petrosal sinus sampling 

4- ACTH concentrations 

5- Urine free cortisol 

Answer & Comments 

Answer: 3- Inferior petrosal sinus sampling 

The inferior petrosal sinus sampling test, an 
elevated central ACTH concentration 
compared to a peripheral value (from arm 
veins) indicates pituitary dependent Cushing's 
disease. The test involves a microcatheter 
being advanced through initially the femoral 
vein and eventually into the inferior petrosal 
sinuses which lie along the internal aspect of 
the skull base which drain blood from the 
pituitary gland. 

Both High Dose Dex Suppression Test (HDDST) 
and Inferior Petrosal Sinus Sampling are for 
differentiating pituitary tumour and ectopic 
cortisol production. Because the tumours are 
sometimes very small an impossible to image 
these tests are useful. They are based on 
different principles - IPSS measuring ACTH 
levels close to pituitary and peripherally. 
HDDST measuring ACTH and cortisol levels 
before and after giving Dex. 

The most specific and most trouble is the IPSS 
(this is nearly 100% specific), so if there is a 
genuine question then its the best test. Thats 
because of how the test is done - the IPSS 
actually samples small amounts of blood from 
close to the pituitary and then further 
peripherally - so if there is a pit tumour it is 
very likely to show high levels of ACTH close to 
it and low er levels peripherally e.g. from the 
arm. 


Dr. Khalid Yusuf El-Zohry - Sohag Teaching Hospital (01118391123) 

Ref MRCPass OE OE 2012 PasTest 2009 PassMedicine 2009 PasTest Exam ReviseMRCP 



485 






















El-zohry MRCP Questions Bank (Part 1) - 2013 


(For my personal use) 


High Dose Dex Sup Test is much easier to do - 
all you need is 2 days and keep taking blood 
tests and give them dex tablets . How ever, if 
you think about the High Dose test - its not 
100% specific because a small number of 
pituitary tumours will be so aggressive that 
even High Dose Dex does not suppress its 
ACTH production. Suppression is quoted at 
75% for pituitary tumours with the HDDST. 

Inferior petrosal sinus sampling 


Dr. Khalid Yusuf El-Zohry - Sohag Teaching Hospital (01118391123) 

Ref MRCPass OE OE 2012 PasTest 2009 PassMedicine 2009 PasTest Exam ReviseMRCP 


486 














El-zohry MRCP Questions Bank (Port 1) - 2013 


(For my personal use) 



[Q: 1155] MRCPass- Clinical 
pharmacology 

A 20 year old woman has been taking oral 
medication for acne. She develops 
polyarthritis, pleuritic chest pains and raised 
liver enzyme tests. 

Investigations show: 

AST 90 

ALT 180 

bilirubin 19 

antinuclear antibodies strongly positive at 
1/640 

Which drug is likely to have caused this? 

1- Isotretinoin 


5- Chlorpropramide 

Answer & Comments 

Answer: 5- Chlorpropramide 

Oral contraceptive pills have an oestrogenic 
effect. 

Phenothiazines such as chlorpropramide and 
thioridazine can have dopamine antagonistic 
action) as does metoclopramide. 

Bromocriptine is a dopamine agonist and this 
inhibits prolactin release, [one way to 
remember is this is that some of the drugs 
which can make Parkinson's worse can also 
cause galactorrhoea] 


2- Oxytetracycline 

3- Amoxycillin 

4- Minocycline 

5- Erythromycin 

Answer & Comments 

Answer: 4- Minocycline 

Minocycline can cause drug induced SLE. 
Procainamide, hydralazine and quinidine are 
the commonest drugs causing drug induced 
lupus. Muscle and joint pain and swelling, flu¬ 
like symptoms of fatigue and fever and 
serositis are common symptoms. 

[Q: 1156] MRCPass-Clinical 
pharmacology 

A 50 year old woman who has recently been 
commenced on a new medication has 
galactorrhoea. 

Which one of the following medications is 
associated with galactorrheoa? 

1- Testosterone 

2- Erythromycin 

3- Rifampicin 

4- Insulin 


[Q: 1157] MRCPass-Clinical 
pharmacology 

A 45 year old man has drunk 10 pints of lager 
beers a day for 20 years. He presents with 
haemetemesis and has an OGD which shows 
bleeding oesophageal varices. 

Which one of the following medications is of 
prognostic and mortality benefit in this clinical 
situation? 

1- Octreotide 

2- Terlipressin 

3- Amoxycillin and metronidazaole 

4- Propanolol 

5- Omeprazole 

Answer & Comments 

Answer: 2- Terlipressin 

Octreotide and propanolol reduce rebleeding. 
Trials have shown that vasopressin analogues 
such as glypressin and terlipressin (34% 
relative risk) provide mortality benefit. 

Reference: 

Cochrane Database Syst Rev. 2003;(1):CD002147. 




Dr. Khalid Yusuf El-Zohry - Sohag Teaching Hospital (01118391123) 

Ref MRCPass OE OE 2012 PasTest 2009 PassMedicine 2009 PasTest Exam ReviseMRCP 



487 






























El-zohry MRCP Questions Bank (Part 1) - 2013 


(For my personal use) 


3 

A 35 year old man has chest pains. He 
mentions heavy cocaine use several hours 
ago. 

Which one of the following is a well known 
side effect of cocaine? 

1- Myocarditis 

2- Pulmonary embolus 

3- complete heart block 

4- Cardiac ischaemia 

5- Pericardial effusion 


[Q: 1158] MRCPass-Clinical 
pharmacology 


Answer & Comments 


Answer: 4- Cardiac ischaemia 


Plasma concentration of sulphonylureas 
(glibenclamide) is increased by fluconazole 
and miconazole. This has led to the 
hypoglycaemia in this case. 



[Q: 1160] MRCPass-Clinical 
pharmacology 


A 33 year old lady who who has a past history 
of treated hypertension is in her 3rd trimester 
of pregnancy and requires on-going anti¬ 
hypertensive treatment. 


Which medication should be avoided? 


1- Nifedipine 

2- Enalapril 

3- Labetalol 


4- Methyldopa 


Cocaine is a stimulant and can cause agitation, 
hyperthermia and hypertension in overdose. A 
significant side effect in significant cocaine 
overdose is coronary arterial vasoconstriction 
leading to cardiac ischaemia. 



[Q: 1159] MRCPass-Clinical 
pharmacology 


A 54 year old diabetic complained of 
lightheadedness and a near collapse episode. 
When his wife measured his blood sugar it 
was 2.3. He was taking glibenclamide for the 
diabetes. 


Which one of the following drugs potentiates 
hypoglycemia caused by glibenclamide? 

1- Steroid 



5- Hydralazine 


Answer & Comments 

Answer: 2- Enalapril 

In pregnancy, angiotensin-converting enzyme 
(ACE)-inhibitors should be avoided because 
they may cause oligohydramnios, renal failure 
and intra-uterine death. 


[Q: 1161] MRCPass-Clinical 
pharmacology 

Which one of the following drugs affects the 
opioid receptors present on the circular and 
longitudinal muscles of the gut? 

1- Ondansetron 



2- Ranitidine 

3- Phenytoin 

4- Fluconazole 

5- Aspirin 


2- Omeprazole 

3- Loperamide 

4- Metoclopromide 

5- Ranitidine 


Answer & Comments 


Answer: 4- Fluconazole 


Answer & Comments 


Answer: 3- Loperamide 


Dr. Khalid Yusuf El-Zohry - Sohag Teaching Hospital (01118391123) 

Ref MRCPass OE OE 2012 PasTest 2009 PassMedicine 2009 PasTest Exam ReviseMRCP 




































El-zohry MRCP Questions Bank (Port 1) - 2013 


(For my personal use) 


Loperamide acts on the opioid receptors along 
the small intestine to decrease circular and 
longitudinal muscle activity. Loperamide 
exerts its antidiarrheal action by slowing 
intestinal transit and increasing contact time, 
and also by directly inhibiting fluid and 
electrolyte secretion via stimulating salt and 
water absorption. 

^ [ Q: 1162 ] MRCPass - Clinical 

ft 

* pharmacology 

Antinuclear antibodies will develop in the 
plasma of patients taking procainamide if the 
patients are: 

1- Fast acetylators 

2- Slow acetylators 

3- Thiopurine methyl transferase deficient 

4- Slow oxidisers 

5- Fast oxidisers 


4- Adrenaline 

5- Prochlorperazine 

Answer & Comments 

Answer: 2- Procyclidine 

Dystonic reactions usually subsides within 24 
hours following cessation of treatment and 
can be treated with procyclidine 5-10 mg i.m. 

They are well-recognized with dopamine 
receptor antagonists (neuroleptics). 
Phenothiazines, prochloperazine, haloperidol 
and metclopramide are examples of drugs 
which can cause dystonic reactions. 

The newer antipsychotic drugs (e.g. 
risperidone, onlanzapine) are more selective 
for dopamine D2 receptors and therefore do 
not usually cause these adverse effects. They 
occur shortly after starting therapy, 
particularly in girls and young women. 


Answer & Comments 

Answer: 2- Slow acetylators 

Hydralazine and procainamide cause 
antinuclear antibodies to appear in the plasma 
of slow acetylators (drug induced lupus may 
occur). Drugs that possess an amide group (- 
NH2) are metabolised by acetylation. Slow 
acetylators are likely to develop adverse 
effects whereas rapid acetylators need higher 
doses of drugs. 



[Q: 1163] MRCPass-Clinical 
pharmacology 


A 35 year old woman is transferred from the 
psychiatry ward with acute dystonia and 
oculogyric crisis after being treated with 
metoclopramide. 


Which treatment should be started? 


1- Phenytoin 

2- Procyclidine 

3- Dopamine 


[Q: 1164] MRCPass-Clinical 
pharmacology 

A 45 year old man who has had a renal 
transplant is concerned about the side effects 
of ciclosporin. 

Which of the following is a side effect? 

1- Hypotension 

2- Skin pigmentation 

3- Alopecia 

4- Gum hypertrophy 

5- Cough 



Answer & Comments 

Answer: 4- Gum hypertrophy 

Ciclosporin causes hirsutism and gum 
hypertrophy (sodium valproate causes 
alopecia). Concomitant use of erythromycin 
can cause toxicity. It is a cyclic 11 aa 
polypeptide. 


Dr. Khalid Yusuf El-Zohry - Sohag Teaching Hospital (01118391123) 

Ref MRCPass OE OE 2012 PasTest 2009 PassMedicine 2009 PasTest Exam ReviseMRCP 
































El-zohry MRCP Questions Bank (Port 1) - 2013 


(For my personal use) 



Gum hypertrophy 


Whot drug is likely to have caused this? 

1- Amiodarone 

2- Hydroxychloroquine 

3- Amocycillin 

4- Minocycline 

5- Tetracycline 

Answer & Comments 

Answer: 4- Minocycline 



[Q: 1165] MRCPass-Clinical 
pharmacology 


A 70 year old man who is known to have 
ischaemic heart disease is admitted with left 
ventricular failure. He has been on NSAIDS for 
arthritis. 


What is likely to occur due to the fact that he 
has been on NSAIDs? 


1- Hyperkalaemia 


This patient developed skin pigmentation in 
her venous eczema due treatment of her acne 
rosacea with minocycline. 


I *1 

A 62 year old man with a history of angina is 
on atorvastatin. 

Which one of the following is the correct 
mechanism of action of statins? 


[Q: 1167] MRCPass-Clinical 
pharmacology 


2- Renal tubular acidosis 


1- HMGcoA reductase promoter 


3- Reduced frusemide induced diuresis 

4- Metabolic acidosis 


2- LDL Receptor down regulation 

3- Increase chylomicron levels 


5- Increased bicarbonate excretion 


Answer & Comments 

Answer: 3- Reduced frusemide induced 
diuresis 

NSAIDS especially indomethacin reduce 
frusemide induced diuresis, probably by 
inhibiting the formation of prostaglandins in 
the renal tubules. 



[Q: 1166] MRCPass-Clinical 
pharmacology 

An 82 year lady had a history of a red facial 
rash suffered venous eczema of legs. She was 
treated for acne rosacea by her GP. 

On examination, she had bluish pigmentation 
on both the legs. 


4- Decrease endogenous liver cholesterol 
synthesis 

5- Reduce triglyceride levels 


Answer & Comments 

Answer: 4- Decrease endogenous liver 
cholesterol synthesis 

Statins upregulate LDL receptors. They are 
HMGcoA reductase inhibitor. This leads to 
decreased liver cholesterol synthesis. 


[Q: 1168] MRCPass-Clinical 
pharmacology 

A 28 year old woman was commenced on 
Carbamazepine for epilepsy. She had admitted 
to drinking 60 units a week and was also 
advised to discontinue alcohol consumption. 



Dr. Khalid Yusuf El-Zohry - Sohag Teaching Hospital (01118391123) 

Ref MRCPass OE OE 2012 PasTest 2009 PassMedicine 2009 PasTest Exam ReviseMRCP 


































El-zohry MRCP Questions Bank (Port 1) - 2013 


(For my personal use) 


Therapeutic concentrations of Carbamazepine 
were achieved within 5 days with a dose of 
200mg daily but the dose needed to be 
increased to 400 mg daily within two weeks to 
achieve a therapeutic plasma concentration. 

Which one of the following is likely to account 
for this observation? 

1- Auto-induction of liver enzymes 

2- Auto-inhibition of liver enymes 

3- Discontinuation of alcohol intake 

4- Drug interaction with an oral contraceptive 

5- Reduced bioavailablity of Carbamazepine 


Answer & Comments 

Answer: 1- Auto-induction of liver enzymes 

In a patient who has discontinued alcohol, 
hepatic enzymes are auto-induced. The 
elimination of carbamazepine increases over 
the first fewweeks because of the enzyme 
induction. 


Drugs which are acetylated include isoniazid, 
hydralazine, dapsone, procainamide and 
sulphasalazine. Slow acetylators have 
increased risk of isoniazid induced peripheral 
neuropathy, and procainamide induced SLE. 
Fast acetylators are were considered more risk 
of isoniazid induced hepatitis. 



[Q: 1170] MRCPass- Clinical 
pharmacology 


A 65 year old man presents with chest pain. 
His ECG shows ST eleveation consistent with 
myocardial infarction and he is thrombolysed 
with tenecteplase. 


Which one drug has not been proven to reduce 
future cardiovascular events and mortality? 


1- Ramipil 

2- Amlodipine 


3- Atenolol 


4- Aspirin 

5- Simvastatin 



[Q: 1169] MRCPass-Clinical 
pharmacology 


A 40 year old woman who had commenced on 
a new drug has developed fevers and myalgia. 


Which one of the following drugs is likely to 
cause a systemic lupus like syndrome? 


1- Amoxycillin 

2- Pyrazinamide 

3- Procainamide 

4- Mesalazine 


5- Methotrexate 


Answer & Comments 


Answer: 3- Procainamide 


Answer & Comments 

Answer: 2- Amlodipine 

The four drugs which are shown to be of 
benefit are aspirin (ISIS-2), atenolol (ISIS-1), 
Ramipril (captopril studies in ISI-4), 
simvastatin in 4S study. Amlodipine does not 
provide mortality benefit. 



[Q: 1171] MRCPass-Clinical 
pharmacology 


A 55 year old lady has atrial fibrillation. The 
rate control of the condition is poor. The 
medical student asked why a digoxin level was 
measured 6 hours after a dose. 


This is due to its: 


A gene which is responsible for activity of 
hepatic N acetyl transferase resulting in slow 
and fast determines the likelihood of drug 
induced lupus. 45% of UK population are slow 
acetylators. 


1- Rate of clearance 

2- Rate of absorption 

3- First pass metabolism 

4- Rate of distribution 


Dr. Khalid Yusuf El-Zohry - Sohag Teaching Hospital (01118391123) 

Ref MRCPass OE OE 2012 PasTest 2009 PassMedicine 2009 PasTest Exam 




ReviseMRCP 

491 




























El-zohry MRCP Questions Bank (Port 1) - 2013 


(For my personal use) 


5- Enzyme breakdown 

Answer & Comments 

Answer: 4- Rate of distribution 

Following digoxin administration, a 6 to 8 hour 
tissue distribution phase is observed. 

This is followed by a much more gradual 
decline in the serum concentration of the 
drug, which is dependent on the elimination 
of digoxin from the body. 


Which one of the following is the most likely 
drug? 

1- Glue sniffing 

2- Tricyclic antidepressant 

3- Selective serotonin reuptake inhibitor 

4- Methadone 

5- Ethanol 

Answer & Comments 

Answer: 2- Tricyclic antidepressant 


[Q: 1172] MRCPass- Clinical 
pharmacology 

A 55 year old with ischaemic heart disease has 
a cholesterol of 6.5 mmol/I. 

What drug should he be commenced on? 

1- 5HT3 antagonist 

2- Beta agonist 

3- Anticholinesterase 

4- HMGCoA inhibitors 

5- Immunoglobulin 



Answer & Comments 

Answer: 4- HMGCoA inhibitors 

Statins e.g. simvastatin, are HMGCoA 
inhibitors. They increase the uptake of LDL 
(not HDL) by the liver. HMGCoA inhibitors 
should be offered to all patients with LDL 
above 3 mmol/l with IHD to bring it down 
below this level. 

Myopathy is a serious side effect. 



[Q: 1173] MRCPass-Clinical 
pharmacology 


An 18 year old girl presents with a drug 
overdose. She is vomiting and generally 
unwell. She does not have a history of drug 
abuse, but has been depressed for the past 2 
years. ECG shows a QT interval of 575ms 
(normal < 470). 


Out of these choices the most likely candidate 
causing prolonged QT is a tricyclic 
antidepressant. Amphetamine is also a 
common cause (not listed here). Methadone 
may cause QT prolongation at high doses, but 
because of the case scenario (no history of 
drug abuse) tricyclic antidepressant is the 
better answer. 

A 50 year old patient is hyperventilating. An 
ABG shows pH 7.30, pC> 2 12 kPa, pCC >2 4.5 kPa, 
BE -10 mmol/l. 

What does base excess mean? 

1- There is an excess amount of bicarbonate in 
the system 

2- Serum bicarbonate concentration is 10 
below the normal range 

3- The amount of base that would have to be 
added or removed to obtain the pH 
actually measured 

4- Shows the amount of excess alkalosis which 
might occur in a controlled environment 

5- A high base excess is expected in a 
hyperventilating patient 


[Q: 1174] MRCPass-Clinical 
pharmacology 


Answer & Comments 

Answer: 3- The amount of base that would 
have to be added or removed to obtain the pH 
actually measured 


Dr. Khalid Yusuf El-Zohry - Sohag Teaching Hospital (01118391123) 

Ref MRCPass OE OE 2012 PasTest 2009 PassMedicine 2009 PasTest Exam ReviseMRCP 

































El-zohry MRCP Questions Bank (Port 1) - 2013 


(For my personal use) 


The base excess is a figure calculated by many 
blood gas machines to aid interpretation of 
data. 

The principles of the calculation are as follows: 
predict the pH that would arise in normal 
blood in the presence of the pC02 actually 
measured; then calculate the amount of acid 
or base that would have to be added to the 
blood to change the calculated pH into the pH 
as actually measured. This value is the base 
deficit or excess, in mmol/l, which quantifies 
the metabolic component (rather than the 
respiratory) of acid-base disturbance. 



[Q: 1175] MRCPass-Clinical 
pharmacology 


A 50 year old homeless man is brought to the 
accident and emergency department by 
ambulance. He is unconscious (GCS 5) with 
pin-point pupils and a slow respiratory rate. 


Immediate specific treatment should be: 


1- Naloxone (0.4 mg) intravenously 

2- N-acetyl cysteine (150 mg/kg over 15 min) 
intravenously 


recently been considered by the diabetologist 
for insulin glargine. 

Why might this be the case? 

1- Poor control due to high blood sugars 

2- He is working night shifts 

3- Hypoglycaemic episodes 

4- He is being considered for islet cell 
transplantation 

5- He is non compliant with insulin 


Answer & Comments 

Answer: 3- Hypoglycaemic episodes 

Insulin glargine is a long-acting insulin 
analogue, there is a smooth, prolonged 
absorption profile with no peaks. As such, it is 
a long-acting agent, suitable for providing a 
basal level of insulin which mimics the normal 
physiological state. 

Its smooth profile reduces the risk of 
hypoglycaemia, and when given at night, 
provides good control of the fasting blood 
glucose. 


3- Gelofusin 1 litre immediately 

4- Check paracetamol and salicylate levels 
urgently 

5- Conservative management and cardiac 
monitoring 


Answer & Comments 


Answer: 1- Naloxone (0.4 mg) intravenously 


[Q: 1177] MRCPass-Clinical 
pharmacology 

A 35 year old woman has overdosed on iron 
tablets which were prescribed for her 
mother's anaemia. She has had 2 generalised 
seizures and has dark stools. Plasma iron 
concentration was 80 mmol/l. 

What drug should be used for treatment? 



The clinical picture is consistent with an opioid 
overdose, the treatment for which is 
intravenous naloxone (0.4 mg), repeated up to 
a total dose of 2 mg depending on the 
response. 


1- Desferrioxamine 

2- Methylene blue 

3- Flumazenil 

4- N acetylcysteine 


5- Naloxone 


[a 

\ _ -J* 


[Q: 1176] MRCPass-Clinical 



pharmacology 


Answer & Comments 


A 25 year old Type I diabetic is currently on a Answer: 1- Desferrioxamine 

human mixtard 30 twice a day. He has 


Dr. Khalid Yusuf El-Zohry - Sohag Teaching Hospital (01118391123) 

Ref MRCPass OE OE 2012 PasTest 2009 PassMedicine 2009 PasTest Exam 




ReviseMRCP 

493 





























El-zohry MRCP Questions Bank (Port 1) - 2013 


(For my personal use) 


Iron overdose can lead to convulsions, Gl 
haemorrhage, hepatic and renal failure, 
pulmonary oedema and DIC. Gl haemorrhage 
as early as a few hours after the overdose. 

Desferrioxamine (maximum dose per day is 80 
mg/kg) is appropriate for: 

I. Patients with a serum iron level of 55-90 
mmol/L with Gl haemorrhage 

II. Patients with a serum iron > 90 mmol/L 



[Q: 1178] MRCPass-Clinical 
pharmacology 


A 40 year old man works in a factory with 
industrial chemicals. He has become confused 
and depressed over the last 4 months. 


On examination he has gait ataxia, tremor in 
the limbs and reduced pain and temperature 
sensation in his feet. 


Which is the most likely chemical causing 
these symptoms? 

1- Lead 

2- Bismuth 


3- Manganese 

4- Mercury 

5- Carbon monoxide 


Answer & Comments 


calomel) are used in some products to inhibit 
the growth of fungi and bacteria. 



[Q: 1179] MRCPass-Clinical 
pharmacology 


A 35 year old lady was prescribed haloperidol 
for radiation sickness. 


What is the mechanism of action of 
Haloperidol as an anti emetic? 

1- HI receptor antagonism 

2- D2 receptor antagonism at the 
chemoreceptor trigger zone 

3- D2 receptor antagonism at the brain centre 

4- D2 receptor antagonism at the periphery 

5- Reduced Gl motility 


Answer & Comments 

Answer: 2- D2 receptor antagonism at the 
chemoreceptor trigger zone 

The butyrophenones (haloperidol) are 
dopamine antagonists and act centrally by 
blocking the chemoreceptor trigger zone. 
They are of considerable value for the 
prophylaxis and treatment of nausea and 
vomiting associated with diffuse neoplastic 
disease, radiation sickness, and the emesis 
caused by drugs such as opioids, general 
anaesthetics, and cytotoxics. 


Answer: 4- Mercury 

The primary symptoms of mercury poisoning 
are vague psychiatric ones. Short-time 
memory can deteriorate. Organic mercury can 
cross the blood-brain barrier and cause 
irreversible nervous system and brain damage, 
e.g., loss of motor control, numbness in limbs, 
blindness, and inability to speak. 

Elemental mercury (the silver liquid familiar 
from thermometers) is the most common 
occupational source. Exposure typically comes 
from inhaling mercury vapors. Inorganic salts 
of mercury (e.g., mercurous chloride, or 



[Q: 1180] MRCPass-Clinical 
pharmacology 


A heroin addict who is on methadone program 
was involved in a motor vehicle accident and 
sustained multiple pelvic fractures. 


How would you manage his analgesia? 


1- Change analgesia to paracetamol with PRN 
oramorph 


2- Continue on methadone and add in 
diclofenac 


3- Continue methadone and titrate to pain 
requirement 


Dr. Khalid Yusuf El-Zohry - Sohag Teaching Hospital (01118391123) 

Ref MRCPass OE OE 2012 PasTest 2009 PassMedicine 2009 PasTest Exam ReviseMRCP 



494 



























El-zohry MRCP Questions Bank (Port 1) - 2013 


(For my personal use) 


4- Continue on methadone and add in 
morphine as required 

5- Discontinue methadone and start on 
morphine 


Answer & Comments 

Answer: 5- Discontinue methadone and start 
on morphine 

Although methadone is an effective analgesic, 
many clinicians prefer to select an alternative 
opioid such as morphine, hydromorphone or 
oxycodone to provide analgesia in 
methadone-maintained patients to allow clear 
distinction between treatment of addiction 
and treatment of pain. 


n 

• i 

j 

A 50 year old man of African origin presents 
with blood pressure 220/110mmHg. Urinalysis 
is negative. 

Fundoscopy shows AV nipping but no 
haemorrhages or papilloedema. 

Which of the following treatment options is 
most appropriate? 

1- Oral atenolol 50mg od and outpatient 
review 

2- Oral lisinopril 20mg and outpatient review 

3- Oral amlodipine 10 mg od and outpatient 
review 

4- Urgent 24 hour catecholamines 

5- Urgent admission for control of 
hypertension 


[Q: 1181] MRCPass- Clinical 
pharmacology 


Answer & Comments 

Answer: 3- Oral amlodipine 10 mg od and 
outpatient review 

The diagnosis of accelerated hypertension 
requires the finding of fundal haemorrhages 
and exudates, with or without papilloedema. 
The BTS guidelines recommends that older 


(>55) and afro-carribean patients should have 
C (calcium channel blockers) or D (diuretics). 
Hence amlodipine is the best choice. 



[Q: 1182] MRCPass-Clinical 
pharmacology 

A 75 year old woman has general lethargy. 
Her drug list includes aspirin, atenolol, 
bendrofluazide and chlorpropramide. 

Investigations show: 

Sodium 110 (137-144) 

Potassium 3.1 (3.5-4.9) 

Urea 6.2 (2.5-7.5) 

Creatinine 95 micromol/L (60-110) 

Glucose 12.2 (3.0-6.0) 

ACTH levels : normal 

chest Xray: normal 

What is the likely cause of the hyponatraemia? 

1- Cyclophosphamide 

2- Bendrofluazide 

3- Hypoadrenalism 

4- Syndrome of inappropriate secretion of 
antidiuretic hormone 

5- Diabetes 


Answer & Comments 

Answer: 2- Bendrofluazide 

Diuretics are most likely to cause combined 
hyponatraemia and hypokalaemia, as in this 
case. Combined hypokalaemia with 
hyponatraemia occurs in very few conditions 
such as increased antidiuretic hormone with 
raised ACTH, vomiting, and diuretics. 



[Q: 1183] MRCPass-Clinical 
pharmacology 


A 35 year old man attends the alcoholic 
addictions clinic. He is keen to try to stop 
drinking and is given some disulfram. 


Dr. Khalid Yusuf El-Zohry - Sohag Teaching Hospital (01118391123) 

Ref MRCPass OE OE 2012 PasTest 2009 PassMedicine 2009 PasTest Exam 




ReviseMRCP 

495 




























El-zohry MRCP Questions Bank (Port 1) - 2013 


(For my personal use) 


What is the mode of action of the drug? 

1- Reduces appetite for alcohol 

2- Inhibits acetaldehyde dehydrogenase 
activity 

3- Reduces the likelihood of hangover 

4- Inhibits alcohol dehydrogenase activity 

5- Protects the liver 


Answer & Comments 

Answer: 2- Inhibits acetaldehyde 

dehydrogenase activity 

Alcohol is metabolized in the liver to form 
acetaldehyde by alcohol dehydrogenase. 
Acetaldehyde is then oxidized to acetate 
(acetic acid) by acetaldehyde dehydrogenase 
(AcDH). Disulfram produces irreversible 
inhibition of this enzyme, resulting in 
accumulation of acetaldehyde which may be 
responsible for most of the signs and 
symptoms occurring after ethanol ingestion in 
disulfiram-treated patients. 


is 

What does this mean? 

1- It is more easily bioavailable 

2- It has a higher maximal response 

3- It has a lower maximal response 

4- Less drug is required to achieve the same 
effect 

5- There are more side effects 


[Q: 1184] MRCPass-Clinical 
pharmacology 

more efficacious than drug B. 



Drug A 


Answer & Comments 

Answer: 2- It has a higher maximal response 

Efficacy relates to the maximal response that 
can be produced by the drug when taken to 
high levels. For example, the diuretic 
furosemide eliminates much more salt and 
water through urine than does the diuretic 


chlorothiazide. Thus, furosemide has greater 
efficacy than chlorothiazide. 


Drug A Drug B 



Log [Dose] 


Drug A and Drug B have the same efficacy. 

Drug A has greater potency than B or C 
because the dose of B or C must be larger to 
produce the same effect as A. Drug C has the 
lowest efficacy and potency. 



[Q: 1185] MRCPass-Clinical 
pharmacology 


A 65 year old woman treated for several years 
for atrial fibrillation with digoxin, presents 
with nausea and vomiting. ECG revealed 
ventricular tachycardia and electrolytes 
revealed hypokalemia. 


Which one of the following signs indicates 
digoxin toxicity? 

1- Diarrhoea 


2- Miosis 


3- Tremor 

4- Jaundice 

5- Xanthopsia 


Answer & Comments 

Answer: 5- Xanthopsia 

Hypokalaemia and hypomagnesaemia 
increase the risk of digoxin toxicity. 
Hemodialysis has no role in digitalis toxicity. 
Administration of Fab antibody fragments is 
the most effective treatment. Xanthopsia 


Dr. Khalid Yusuf El-Zohry - Sohag Teaching Hospital (01118391123) 

Ref MRCPass OE OE 2012 PasTest 2009 PassMedicine 2009 PasTest Exam ReviseMRCP 






























El-zohry MRCP Questions Bank (Port 1) - 2013 


(For my personal use) 


(yellow vision), nausea and vomiting are 
common side effects. 


[Q: 1186] MRCPass-Clinical 
pharmacology 

A 35 year old lady has been started on 
azathioprine for rheumatoid arthritis. She 
complains of lethargy and has a blood test 
which shows: Hb 7.5 g/dl, WCC 3 x 10 9 /L, 
platelets 45 x 10 9 /L. 

The reaction is more likely if she : 

1- Is drinking alcohol 

2- Has thiopurine methyl transferase 
deficiency 



5- Corneal microdeposits 


Answer & Comments 

Answer: 4- Myopathy 

The combination of amiodarone and a statin 
increases the risk of developing myopathy. 



[Q: 1188] MRCPass-Clinical 
pharmacology 


Which of the following drugs could couse o 
raised prolactin level? 


1- Metoclopramide 

2- Ramipril 


3- Has increased liver enzyme activity 


3- Lansoprazole 


4- Is on warfarin 


4- Thiazides 


5- Is a fast acetylator 


5- Propanolol 


Answer & Comments 

Answer: 2- Has thiopurine methyl transferase 
deficiency 

The patient has developed a pancytopaenia 
due to azathioprine toxicity. Approximately 1 
in 300 Caucasians have thiopurine methyl 
transferase (TPMT) deficiency. 

TPMT is the enzyme that metabolises 6- 
mercaptopurine and its deficiency results in 
high risk of azathioprine toxicity. 


A 55 year old lady has atrial fibrillation which 
is being treated with amiodarone. She has also 
been recently started on simvastatin. 

This combination of drugs puts her at 
increased the risk of developing: 

1- Osteoporosis 

2- Pulmonary fibrosis 

3- Optic neuritis 

4- Myopathy 



[Q: 1187] MRCPass-Clinical 
pharmacology 


Answer & Comments 

Answer: 1- Metoclopramide 

The drugs that may increase prolactin levels 
are: Phenothiazines, haloperidol, 

metoclopramide, methyldopa and oestrogens. 



[Q: 1189] MRCPass-Clinical 
pharmacology 


A 38 year old man with several medical 
conditions was investigated for infertility. 


Which one of the following drugs is most likely 
to cause this? 


1- Mesalazine 

2- Sulfasalazine 


3- Aspirin 

4- Azathioprine 

5- Amoxycillin 


Answer & Comments 


Answer: 2- Sulfasalazine 


Dr. Khalid Yusuf El-Zohry - Sohag Teaching Hospital (01118391123) 

Ref MRCPass OE OE 2012 PasTest 2009 PassMedicine 2009 PasTest Exam 




ReviseMRCP 

497 

































El-zohry MRCP Questions Bank (Part 1) - 2013 


(For my personal use) 


Sulfasalazine, anabolic steroids, 

cyclophosphamide, chlorambucil, busulfan 
and cisplatin are drugs which cause 
azoospermia. 


4- Diltiazem 

5- Metoprolol 


Answer & Comments 


^ [Q: 1190] MRCPass-Clinical 

S pharmacology 

A 45 year old man is admitted with an upper 
lobe pneumonia. Investigations show 
hyponatraemia and mildly deranged LFTs. 

His CXR shows shadowing in the right mid and 
upper zones. Therapy is started and 5 days 
later he becomes acutely jaundiced with red 
discolouration of the urine. 

Which one of the following drugs is the likely 
co use? 

1- Rifampicin 

2- Amoxycillin 

3- Tetracycline 

4- Erythromycin 

5- Cefuroxime 


Answer & Comments 

Answer: 1- Rifampicin 

The patient could have been commenced on 
rifampicin for suspected Legionella 
pneumonia or tuberculosis. Rifampicin is a 
hepatic enzyme inducer and can lead to acute 
jaundice, and patient should be told that urine 
will turn a red discolouration. 


Answer: 3- Anti digoxin antibodies 


Brady arrhythmias and ventricular arrhythmias 
can occur in digoxin overdose. Most often 
seen are heart block with or without 
supraventricular arrhythmias. Other effects 
include vomiting, bradycardia, and 
hyperkalemia. In toxicity, Digoxin Immune Fab 
fragments (derived from specific antidigoxin 
antibodies produced in sheep)can be given. 


KMt04l 



VW'—r'Vp*— f “7^ 



CAr-C-l 

M - V « 


ECG showing Digoxin effect causing reversed 

Tick ST segments 


[Q: 1192] MRCPass-Clinical 
pharmacology 

A 39 year old man presents with cocaine 
overdose. 

Which of the following should be avoided? 

1- Nitrates 

2- Beta blocker 

3- Haloperidol 

4- Diazepam 

5- Amlodipine 



\h 

1 


[Q: 1191] MRCPass-Clinical 



pharmacology 


Answer & Comments 


A 60 year old woman presents with a digoxin 
overdose. Her ECG shows reversed tick ST 
segments and she has a heart rate of 30 bpm. 

Which of the following agents should be used? 

1- Atropine 

2- Charcoal 

3- Anti digoxin antibodies 


Answer: 2- Beta blocker 

Treatment of acute cocaine intoxication can 
be difficult. Although (3-blockers have been 
used successfully to manage supraventricular 
tachyarrhythmias, they also produce 
unopposed alpha-stimulation in coronary 
vasospasm, which can exacerbate cocaine- 
induced hypertension and, at the same time, 
cause reductions in coronary blood flow . 


Dr. Khalid Yusuf El-Zohry - Sohag Teaching Hospital (01118391123) 

Ref MRCPass OE OE 2012 PasTest 2009 PassMedicine 2009 PasTest Exam ReviseMRCP 





































El-zohry MRCP Questions Bank (Port 1) - 2013 


(For my personal use) 



[Q: 1193] MRCPass-Clinical 
pharmacology 

A 28 year old woman is 20 weeks 
pregnant.She complains of fever and dysuria. 
An MSU has showed a significant growth of 
Gram negative bacilli. 

Whot is the best choice antibiotic in this 
situation? 

1- Teicoplanin 

2- Tazosin 

3- Cefaclor 

4- Trimethoprim 

5- Ciprofloxacin 


Answer & Comments 

Answer: 3- Cefaclor 

Trimethoprim is a folate antagonist and can 
increases the risk of neural tube defects. 
There is relative contraindication for 
ciprofloxacin in pregnancy due to the possible 
teratogenic effect. Augmentin, cefaclor, 
nitrofurantoin and metronidazole are safe in 
pregnancy. 


[Q: 1194] MRCPass-Clinical 
pharmacology 

A 30 year old female presented with a 12-hr 
history of progressive bluish discolouration of 
lips and limbs. She denied ingesting or inhaling 
any drug or substance. A high pa0 2 in the 
presence of 'cyanosis' and 'dark blood' led to 
suspicion of methaemoglobinaemia. Co¬ 
oximetry revealed the methaemoglobin level 
to be 47%. 

Which of the following is most likely to have 
caused the condition? 

1- Ascorbic acid 

2- Charcoal 

3- Methylene blue 

4- Paracetamol 

5- Chloroquine 



Answer & Comments 

Answer: 5- Chloroquine 

Methaemoglobinaemia can be caused either 
by a genetic defect in red cell metabolism or 
haemoglobin structure, or acquired by a 
variety of drugs and toxins. About forty 
substances have been implicated in causing 
this condition, the most prominent being 
dapsone, nitrates, prilocaine, antimalarials, 
sulphonamides and dyes. 

Domestic causes of acquired 
methaemoglobinaemia include ingestion of 
food and water high in nitrites and nitrates, 
exposure to aniline dyes in dyed blankets, 
laundry markings, freshly dyed shoes, and 
cleaning solution. 

Standard pulse oximeters give spuriously low 
readings in the presence of excess 
methaemoglobin. Methylene blue is indicated 
in any patient with symptoms and/or signs of 
hypoxia (mental changes, tachycardia, 
dyspnoea, chest pain). It is contraindicated in 
G6PD deficiency. High flow oxygen should be 
administered. 



Methaemoglobinaemia causing cyanosis 
(hand on the right of the picture) 



[Q: 1195] MRCPass-Clinical 
pharmacology 

Which of the following describes the mode of 
action of alendronate? 

1- Osteoclast inhibition 


Dr. Khalid Yusuf El-Zohry - Sohag Teaching Hospital (01118391123) 

Ref MRCPass OE OE 2012 PasTest 2009 PassMedicine 2009 PasTest Exam ReviseMRCP 






























El-zohry MRCP Questions Bank (Port 1) - 2013 


(For my personal use) 


2- Osteoblast stimulation 


3- Proximal convoluted tubule 


3- Promotes collagen synthesis 


4- Distal convoluted tubule 


4- Causes hypocalcaemia 

5- Promotes bone calcification 


Answer & Comments 

Answer: 1- Osteoclast inhibition 

Bisphosphonates inhibit bone resorption 
through inhibition of osteoclastic activity. 



[Q: 1196] MRCPass- Clinical 
pharmacology 


An epileptic patient is on carbamazepine. He is 
commenced on a new drug and noticed that 
the frequency of his seizures has increased. 


Which one of these drugs is likely to be 
responsible? 


1- Amoxycillin 


2- Phenobarbitone 


3- Tetracycline 

4- Paracetamol 


5- Digoxin 


Answer & Comments 

Answer: 2- Phenobarbitone 

Liver enzyme inducers can interact and 
increase breakdown of antiepileptic drugs. 


5- Collecting duct 


Answer & Comments 

Answer: 4- Distal convoluted tubule 

Thiazides block Na+ and Cl- reabsorption in 
the distal tubule. There is usually passive Na+ 
and Cl- co transport. With this blocked, 
natriuresis occur. The distal convoluted tubule 
accounts for 5% of total sodium chloride 
reabsorption. 

[Q: 1198] MRCPass-Clinical 
pharmacology 

A 65 year old man has been taking 
amiodarone 200 mg daily for troublesome 
atrial fibrillation. He seems to be euthyroid 
with no palpable goitre. 

Investigations revealed: Serum Free T 4 - 21 
pmol/L (9-26); Serum total T3 - 0.7 nmol/L 
(0.9-2.8); Serum TSH - 6.3 mU/L (<5). 

Which of the following explains these results? 

1- Amiodarone-induced hypothyroidism 

2- Amiodarone drug interaction with digoxin 

3- 'Sick euthyroid' syndrome 

4- TSH secreting tumour 

5- Carbimazole ingestion 



The enzyme inducers are PC BRAS - phenytoin, 
carbamazepine, barbiturates 

(phenobarbitone), rifampicin, alcohol and 
sulphonamides. 



[Q: 1197] MRCPass-Clinical 
pharmacology 


Which port of the renal system do thiazides 
act on ? 


1- Ascending loop of Henle 


2- Descending loop of Henle 


Answer & Comments 

Answer: 1- Amiodarone-induced 

hypothyroidism 

This patient is likely to have amiodarone- 
induced hypothyroidism. 

There is normal T4 and a low T3 with elevated 
TSH. This is because one of the effects of 
amiodarone is to inhibit the peripheral 
conversion of T4 to T3. 


Dr. Khalid Yusuf El-Zohry - Sohag Teaching Hospital (01118391123) 

Ref MRCPass OE OE 2012 PasTest 2009 PassMedicine 2009 PasTest Exam ReviseMRCP 



500 































El-zohry MRCP Questions Bank (Port 1) - 2013 


(For my personal use) 



[Q: 1199] MRCPass-Clinical 
pharmacology 

Drug A is more potent than drug 8. There is 
more maximal effect 

1- There is less maximal effect 

2- What does this mean? 

3- There is greater bioavailability 

4- Less drug is required for the same effect 

5- More side effects may occur 

Answer & Comments 

Answer: 4- Less drug is required for the same 
effect 


Which of the following drugs is appropriate for 
prophylaxis against migraine? 

1- Paracetamol 

2- Lamotrigine 

3- Clomiphene 

4- Propanolol 

5- Thyroxine 

Answer & Comments 

Answer: 4- Propanolol 

Propranolol, sodium valproate, pizotifen and 
amitriptyline can be used for prophylaxis in 
migraine. 


The potency of a drug relates to the amount 
of drug needed to produce a given effect. Less 
drug is hence required for the same effect for 
a drug with higher potency. For example, if 5 
milligrams of drug A relieves pain as 
effectively as 10 milligrams of drug B, drug A is 
twice as potent as drug B. 


Drug A Drug B 



Log [Dose] 


Drug A and Drug B have the same efficacy. 

Drug A has greater potency than B or C 
because the dose of B or C must be larger to 
produce the same effect as A. Drug C has the 
lowest efficacy and potency. 


[ Q: 1200 ] MRCPass - Clinical 
Xf | pharmacology 

A 65 year old woman has increasing frequency 
of headaches. She has been diagnosed as 
having migraines by the neurologist a year 
ago. 


| »1 

A 50 year old patient has a history of COPD 
and needs to be considered for 
chemotherapy. 

Which of the following drugs may cause lung 
fibrosis and should be avoided? 

1- 5-fluorouracil 

2- Tamoxifen 

3- Vincristine 

4- Bleomycin 

5- Cytarabine 


[Q: 1201] MRCPass-Clinical 
pharmacology 


Answer & Comments 

Answer: 4- Bleomycin 

Busulphan, bleomycin and methotrexate are 
cytotoxic agents which can cause lung fibrosis. 
The changes are usually in the lower zones of 
the lung. 



[ Q: 1202 ] MRCPass - Clinical 
pharmacology 


A 58 year old man has a history of obesity, 
gastro oesophageal reflux disease, lower back 
pain and coronary disease. He presents with 


Dr. Khalid Yusuf El-Zohry - Sohag Teaching Hospital (01118391123) 

Ref MRCPass OE OE 2012 PasTest 2009 PassMedicine 2009 PasTest Exam 




ReviseMRCP 

501 































El-zohry MRCP Questions Bank (Port 1) - 2013 


(For my personal use) 


large, itchy wheals over the trunk and a 
sensation of tightness in throat. 

Which one of the following drugs is likely to 
have triggered this skin eruption? 

1- Nitrates 

2- Atorvastatin 

3- Lansoprazole 

4- Aspirin 

5- Paracetamol 

Answer & Comments 

Answer: 4- Aspirin 

Among this list of drugs, aspirin is most likely 
to cause drug related urticarial reaction 
(NSAIDs as well can cause this). 


[Q: 1204] MRCPass- Clinical 
pharmacology 

A 75 year old man presents to the hospital 
with drowsiness and confusion. He is 
tachycardic and tachypnoeic. 

His pulse oximeter reading is 88% on room air 
and he is not cyanosed. He has another family 
member admitted with similar symptoms a 
month ago. 

Which one of the following is most likely? 

1- Antifreeze poisoning 

2- Carbon monoxide poisoning 

3- Tuberculosis 

4- Amitriptyline overdose 

5- Atenolol overdose 




[ Q: 1203 ] MRCPass - Clinical 
pharmacology 

A 65 year old woman has fast atrial fibrillation. 
Her serum creatinine concentration is 330 
umol/L. 

What is the main factor that would aid in 
choosing the loading dose ofdigoxin? 

1- Lipid solubility 

2- A. Absorption Volume of distribution 

3- Renal clearance 

4- Plasma half-life 

5- First pass metabolism 


Answer & Comments 

Answer: 3- Renal clearance 

The loading dose should take into account the 
volume of distribution of a drug and also 
clearance. In the case of digoxin, renal 
clearance is the more important factor as 
toxicity is much more likely when there is 
moderate or severe renal impairment. 


Answer & Comments 

Answer: 2- Carbon monoxide poisoning 

In carbon monoxide poisoning, it is essential 
to measure CO levels, since pulse oximeters 
cannot distinguish between COHb and Hb02. 



[Q: 1205] MRCPass-Clinical 
pharmacology 

An 45 year old man who has hypertension. He 
has recently become depressed from losing 
his job and took an overdose of atenolol. He 
has a heart rate of 35 bpm. 

What should be given? 

1- Sliding scale insulin 

2- Metformin 

3- Diltiazem 

4- Atropine 

5- Eprex 


Answer & Comments 

Answer: 4- Atropine 

Bradycardia is a common feature of significant 
B blocker overdose and should be treated by 


Dr. Khalid Yusuf El-Zohry - Sohag Teaching Hospital (01118391123) 

Ref MRCPass OE OE 2012 PasTest 2009 PassMedicine 2009 PasTest Exam ReviseMRCP 
































El-zohry MRCP Questions Bank (Port 1) - 2013 


(For my personal use) 


the administration of atropine. Intravenous 
glucagon may also be given, particularly in 
patients with haemodynamic compromise. 
Temporary cardiac pacing may be necessary in 
patients unresponsive to drug therapy. 



[ Q: 1206 ] MRCPass - Clinical 
pharmacology 

A 25 year old man presents after ingesting a 
drug at a disco. Investigations reveals a serum 
creatine kinase of 15,000 IU/L (24-195) 

Which one of the following drugs is most likely 
to be responsible? 

1- Lorazepam 

2- Gamma hydroxy butyrate (GHB) 

3- Ecstasy (MDMA) 

4- Antifreeze 

5- Diamorphine 


Answer & Comments 

Answer: 3- Hyperthermia 

Cocaine stimulates the central nervous 
system, causing agitation, dilated pupils, 
tachycardia, hypertension, hallucinations, 
hyperthermia, hypertonia, and hyperreflexia; 
cardiac effects include chest pain(due to 
vasospasm), myocardial infarction, and 
arrhythmias. 



[ Q: 1208 ] MRCPass - Clinical 
pharmacology 


A 75 year old man with atrial fibrillation has 
been stable on warfarin for many years. He is 
admitted with a epistaxis and his INR is found 
to be 10. 


Recent prescription of which of the following 
drugs might explain this? 

1- Barbiturates 


Answer & Comments 

Answer: 3- Ecstasy (MDMA) 

The elevated Creatinine Kinase levels suggest 
rhabdomyolysis. Features of acute MDMA 
(ecstasy) toxicity include agitation, 
tachycardia, hypertension, dilated pupils, 
sweating, hyperthermia, disseminated 
intravascular coagulation (DIC), 

rhabdomyolysis and acute renal failure. 



[Q: 1207] MRCPass-Clinical 
pharmacology 


A 43 year old man was brought to the A&E 
confused and agitated after cocaine use. 


Which of the following finding would be 
consistent with cocaine abuse? 


1- Hypothermia 

2- Hyperkalemia 

3- Hyperthermia 

4- Hypokalemia 

5- Hypernatremia 


2- Ciprofloxacin 

3- Griseofulvin 

4- Phenytoin 

5- Carbamazepine 


Answer & Comments 

Answer: 2- Ciprofloxacin 

Ciprofloxacin is a liver enzyme inhibitor 
(CYP450) and hence potentiates the action of 
warfarin, leading to a high INR. 



[ Q: 1209 ] MRCPass - Clinical 
pharmacology 


A 60 year old man has been started on Viagra 
by his GP. The patient has symptoms of 
angina. 


Which one of the following drugs is 
contraindicated? 


1- Atenolol 


2- Clopidogrel 

3- Glyceryl trinitrate 


Dr. Khalid Yusuf El-Zohry - Sohag Teaching Hospital (01118391123) 

Ref MRCPass OE OE 2012 PasTest 2009 PassMedicine 2009 PasTest Exam 




ReviseMRCP 

503 



































El-zohry MRCP Questions Bank (Port 1) - 2013 


(For my personal use) 


4- Simvasatin 

5- Ramipril 


Answer & Comments 

Answer: 3- Glyceryl trinitrate 

Both sildenafil (viagra) and nitrates have a 
vasodilatory effect, hence potentially 


2- HRT 

3- Calcium gluconate 

4- Ondansetron 

5- Azathioprine 

Answer & Comments 

Answer: 1- Amitriptyline 


cumulative hypotensive effect. Caution should 
be taken in patients with ischaemic heart 
disease and sildenafil. 


[Q: 1210] MRCPass- Clinical 
pharmacology 

A 70 year old woman with ascites secondary 
to carcinoma of the ovary is complaining of 
abdominal distension and intermittent 
vomiting. 

The antiemetic of choice is: 

1- Dexamethasone 



Causes or triggering factors in oculogyric crisis 
include: neuroleptics, benzodiazepines, 

carbamazepine, chloroquine, cisplatin, 

influenza vaccine, levodopa, lithium, 
metoclopramide, nifedipine, reserpine, 

tricyclics. 

They occur shortly after starting therapy, 
particularly in girls and young women as well 
as the elderly. The problem usually subsides 
within 24 hours following cessation of 
treatment and can be treated with 
procyclidine 5-10 mg i.m. 


2- Cyclizine 

3- Metoclopramide 

4- Ondanestron 

5- Haloperidol 


Answer & Comments 

Answer: 3- Metoclopramide 

Metoclopramide as a prokinetic agent will 
increase the rate of transit of food through 
the gastrointestinal tract and alleviate her 
symptoms. 


SJ 

A 25 year old lady presents with acute 
dystonia and oculogyric crisis after being 
treated with a drug. 

Which one of the following drugs is most 
likely? 

1- Amitriptyline 


[Q: 1211] MRCPass-Clinical 
pharmacology 



[Q: 1212] MRCPass-Clinical 
pharmacology 


A 25 year-old woman presents with acute 
abdominal pain. She is diagnosed with an 
acute exacerbation of acute intermittent 
porphyria (AIP). 


What is the most likely predisposing drug? 
1- Barbiturates 


2- Amoxycillin 

3- Oral contraceptive pill 

4- Chlorpromazine 

5- Paracetamol 


Answer & Comments 

Answer: 3- Oral contraceptive pill 

benzodiazepines, oral contraceptive pills, 
sulphonamides (co-trimoxazole), phenytoin 
and rifampicin. 


Dr. Khalid Yusuf El-Zohry - Sohag Teaching Hospital (01118391123) 

Ref MRCPass OE OE 2012 PasTest 2009 PassMedicine 2009 PasTest Exam ReviseMRCP 

































El-zohry MRCP Questions Bank (Port 1) - 2013 


(For my personal use) 



[Q: 1213] MRCPass-Clinical 
pharmacology 

A 70 year old man is assessed in the psychiatry 
clinic. The SHO is concerned about lithium 
toxicity. 

Which of the following is o sign that his lithium 
level is toxic? 

1- Weight loss 

2- Poor appetite 

3- Xanthopsia 

4- Tremors 

5- Skin pigmentation 


Answer & Comments 

Answer: 4- Tremors 

tremors, oliguria, blurred vision, diarrhoea, 
vomiting, hyperreflexia, convulsions and 
decreased consciousness. 



[Q: 1214] MRCPass-Clinical 
pharmacology 


A 70 year old man is referred to the 
rheumatology clinic because of inadequate 
pain relief. He has osteoarthritis affecting both 
hips. His GP has prescribed paracetamol and 
codeine 30mg four times daily but he has 
found little improvement in his pains. 


U 


[Q: 1215] MRCPass-Clinical 
pharmacology 

A 44 year old is on several 


immunosuppresants. 


Which one of the following is o colcineurin 
inhibitor? 


1- Sulphasalazine 

2- Cyclophosphamide 

3- Methotrexate 


4- Cyclosporin 

5- Azathioprine 


Answer & Comments 

Answer: 4- Cyclosporin 

Examples of calcineurin inhibitors (CNIs) such 
as ciclosporin and tacrolimus. 



[Q: 1216] MRCPass-Clinical 
pharmacology 

A 40 year old patient who has depression is 
being assessed. She was found to have a heart 
rate of 140. Upon enquiry she takes diazepam 
and dothiepin. 

What should be done next? 

1- Iv flumazenil 

2- Iv naloxone 


What is the likely cause? 

1- Fast acetylator status 

2- Slow acetylator status 

3- Non compliance 

4- Inadequate dose of Codeine 

5- Interaction of codeine with paracetamol 

Answer & Comments 

Answer: 4- Inadequate dose of Codeine 

Studies have shown that paracetamol lg 
combined with codeine at dose of 60mg have 
the best analgesic outcomes. 


3- Echocardiogram 

4- Electrocardiogram 

5- Iv magnesium 

Answer & Comments 

Answer: 4- Electrocardiogram 

Although neither drug on its own is 
particularly implicated for QT prolongation, 
the likelihood is increased in combination. An 
ECG is the best initial assessment for a patient 
with tachycardia to exclude ventricular 
arrhythmia. 


Dr. Khalid Yusuf El-Zohry - Sohag Teaching Hospital (01118391123) 

Ref MRCPass OE OE 2012 PasTest 2009 PassMedicine 2009 PasTest Exam 




ReviseMRCP 

505 
































El-zohry MRCP Questions Bank (Port 1) - 2013 


(For my personal use) 


is 

i gj 

A 50 year old lady has a diagnosis of 
scleroderma. She complains of lethargy, 
anorexia, heartburn and weight loss. Her 
blood pressure is 150/100 on more than one 
occasion. Urine dipstick shows protein ++. Her 
investigations are as follows: urea is 9.5 
mmol/l, Creatinine 125 umol/l, Na 138 
mmol/l, K 4.2mmol/l. 

Which medication should be used to treat 
hypertension ? 

1- Alpha blocker 

2- Calcium antagonist 

3- Thiazide 

4- ACE inhibitor 

5- Beta-blocker 


[Q: 1217] MRCPass-Clinical 
pharmacology 


Answer & Comments 

Answer: 4- ACE inhibitor 

Renal involvement in systemic sclerosis 
requires tight control of hypertension (which 
may worsen the renal impairment) and ACE 
inhibitors are the drugs of first choice. 



[Q: 1218] MRCPass-Clinical 
pharmacology 

A 25 year old epileptic woman is now 
pregnant and in the second trimester. She has 
been taking sodium valproate for several 
years with good control of epilepsy. 

Which of the following is a significant risk with 
sodium valproate? 

1- Anaemia 

2- Fetal neural tube defect 

3- Hypoglycaemia 

4- Weight gain 

5- Abdominal striae 


Answer & Comments 

Answer: 2- Fetal neural tube defect 

Sodium valproate is associated with a 1.5% 
risk of neural tube defects. This may be 
attributed in part to its effect in reducing 
serum folate. Other abnormalities with 
sodium valproate include: hypospadias, heart 
defects, craniofacial and skeletal anomalies, 
and developmental delay. 



[Q: 1219] MRCPass-Clinical 
pharmacology 

A 35 year old man is brought to casualty from 
a nightclub where he had collapsed following 
ingestion of several tablets of ecstasy. 

Which of the following is a recognised side 
effect? 

1- Urinary incontinence 

2- Constipation 

3- Bradycardia 

4- Hyperthermia 

5- Crying 


Answer & Comments 

Answer: 4- Hyperthermia 

The main effects of ecstasy intoxication are 
hypertension, tachycardia, increased 
respiratory rate, hyperthermia and increased 
sweating. More serious complications are 
malignant hyperthermia, liver failure and 
cerebral oedema. 



[ Q: 1220 ] MRCPass - Clinical 
pharmacology 

A 25 year old man attends casualty 12 hours 
after taking an overdose of 30 g of 
paracetamol and 5 tablets of 3mg codeine. On 
examination, he is drowsy with a Glasgow 
Come Scale of 13. His pulse is 110 beats per 
minute, blood pressure is 110/70 mmHg and 


Dr. Khalid Yusuf El-Zohry - Sohag Teaching Hospital (01118391123) 

Ref MRCPass OE OE 2012 PasTest 2009 PassMedicine 2009 PasTest Exam ReviseMRCP 



































El-zohry MRCP Questions Bank (Port 1) - 2013 


(For my personal use) 


he has pinpoint pupils, with saturations of 
98% on air. 

What is the best treatment option? 

1- Naloxone 

2- N-acetylcysteine 

3- Haemodialysis 

4- Flumazenil 

5- Activated charcoal 


Answer & Comments 

Answer: 2- N-acetylcysteine 

Although the patient has pinpoint pupils, he 
had ingested 30g of paracetamol which is a 
toxic dose (lethal if > 30g in people with 
normal liver function). Hence N-acetylcysteine 
is the best management option. His GCS and 
respiratory rate (oxygen saturations) are not 
significantly compromised, otherwise 
naloxone would be the best option. 


[Q: 1221] MRCPass-Clinical 
pharmacology 

A 30 year old man is on warfarin and is given 
advice regarding foods. 

Which one of these foods is a liver enzyme 
inhibitor? 

1- Rice 

2- Potatoes 

3- Grapefruit juice 

4- Apple juice 

5- Spinach 



Answer & Comments 

Answer: 3- Grapefruit juice 

Grapefruit juice is an inhibitor of the enzyme 
cytochrome P450 3YA. 



[ Q: 1222 ] MRCPass - Clinical 
pharmacology 


A 30 year old lady has been on treatment for 
depression. She complains of constipation. On 
examination, she has a palpable goitre. 


Her blood results are as follows: 


Serum calcium 2.75 mmol/l 
phosphate 0.7 mmol/l 
TSH 18 mU/l 


Free T 4 8 pmol/l 

Which drug is likely to be responsible? 

1- Paroxetine 

2- Amitriptyline 

3- Citalopram 

4- Lithium 

5- Venlafaxine 


Answer & Comments 

Answer: 4- Lithium 

Lithium can cause hypercalcaemia and 
hypothyroidism along with a goitre. The other 
side effects are fine tremor, weight gain, 
diabetes insipidus and cardiac arrhythmias. 



[ Q: 1223 ] MRCPass - Clinical 
pharmacology 


A 50 year old man presents with painless 
bilateral arm weakness and abdominal pain. 
On examination he has bilateral wrist drop. 


Which one of the following forms of poisoning 
is likely? 


1- Mercury 

2- Lead 


3- Arsenic 

4- Carbon monoxide 

5- Organophosphates 


Dr. Khalid Yusuf El-Zohry - Sohag Teaching Hospital (01118391123) 

Ref MRCPass OE OE 2012 PasTest 2009 PassMedicine 2009 PasTest Exam 




ReviseMRCP 

507 




























El-zohry MRCP Questions Bank (Port 1) - 2013 


(For my personal use) 


Answer & Comments 

Answer: 2- Lead 

Blood lead levels over 10 micrograms/dL can 
cause neurological damage. Peripheral 
neuropathy and bilateral wrist drop is classic. 
Severe lead poisoning can cause persistent 
vomiting, seizures, coma, and death. 

Ingestion of lead-based paint is the most 
common source of lead poisoning in children. 


3- Amyloid deposition 

4- Interstitial nephritis 

5- Mesangial thinning 

Answer & Comments 

Answer: 4- Interstitial nephritis 

NSAIDS may cause interstitial nephritis, 
glomerulonephritis and renal papillary 
necrosis. 



[Q: 1224] MRCPass-Clinical 
pharmacology 


A 60 year old woman who is on several drugs 
has a potassium of 2.7 mmol/I. 


Which one of the following drugs is likely to 
couse hypokoloemio? 

1- Ramipril 

2- Hydrochlorothiazide 


3- Amiloride 


4- Losartan 

5- Spironolactone 


Answer & Comments 

Answer: 2- Hydrochlorothiazide 

ACE inhibitor (ramipril), angiotensin receptor 
blocker (losartan), and potassium sparing 
diuretics (amiloride, spironolactone) cause 
hyperkalaemia. Thiazide diuretics do not 
contribute to hyperkalaemia. 


is 

• t 

A 40 year old woman who has been taking 
NSAIDs has sudden deterioration of renal 
function. 

Which one of the following is the most likely 
effect of NSAIDS on the kidney? 

1- IgA nephropathy 

2- Increased sensitivity to ADH 


[Q: 1225] MRCPass-Clinical 
pharmacology 


[ Q: 1226 ] MRCPass - Clinical 

i pharmacology 

____--' s 

An 18 year old female attends casualty 6 
hours after ingesting approximately 30g of 
Paracetamol and 360mg of Dihydrocodeine. 

On examination, she is drowsy with a Glasgow 
Coma Scale of 12. Her pulse is 100 beats per 
minute, blood pressure is 110/66 mmHg and 
she has pinpoint pupils, with saturations of 
96% on air. 

Whot is the most appropriate treatment for 
this patient? 

1- 10% Dextrose infusion 

2- Activated charcoal by mouth 

3- Gastric lavage 

4- N-Acetylcysteine intravenously 

5- Naloxone intravenously 


Answer & Comments 

Answer: 4- N-Acetylcysteine intravenously 

The patient has decreased GCS but not severe 
enough for naloxone. 

In this case NAC treatment early will reduce 
the likelihood of significant liver damage due 
to paracetamol poisoning. 


[Q: 1227] MRCPass-Clinical 
pharmacology 

A 20 year old university student has been 



Dr. Khalid Yusuf El-Zohry - Sohag Teaching Hospital (01118391123) 

Ref MRCPass OE OE 2012 PasTest 2009 PassMedicine 2009 PasTest Exam ReviseMRCP 



508 


































El-zohry MRCP Questions Bank (Port 1) - 2013 


(For my personal use) 


hearing voices for a week. He thinks that his 
girlfriend is trying to kill him. A week ago he 
was at a disco and admits to having taken a 
substance. 

Which of these drugs could couse psychosis? 

1- Cocaine 

2- LSD 

3- Marijuana 

4- Amphetamines 

5- Antifreeze 


Answer & Comments 


Answer: 4- Amphetamines 


balance (replacement if she becomes 
hypernatraemic or dehydrated from polyuria). 


A 

A 80 year old lady has end stage ovarian 
carcinoma and has worsening significant 
abdominal pains. Her symptoms had been 
previously well controlled on oxycodone SR 
40mg bd. 

Whot should the onolgesio be altered to? 

1- Im morphine 

2- Oral tramadol 

3- Oral codeine 


[ Q: 1229 ] MRCPass - Clinical 
pharmacology 


Amphetamines (speed) is most likely to 4- Prn oxycodone 

produce a schizophreniform type psychosis. _ _ . . . . . .. . . 

5- Syringe driver with diamorphine 



[ Q: 1228 ] MRCPass - Clinical 
pharmacology 


A 28 year old patient with poorly controlled 
bipolar disorder is on 800 mg/day dose of 
lithium. She complains of going to the toilet 
frequently and also thirst. 


Whot should be done? 


1- Check thyroid function 

2- Check calcium levels 


3- Monitor electrolytes and fluid balance 

4- Increase lithium doses 


5- MRI of pituitary 


Answer & Comments 

Answer: 3- Monitor electrolytes and fluid 
balance 

Side effects of lithium are - tremor, muscle 
weakness, hypothyroidism, hypereflexia, 
ataxia, weight gain, leucocytosis, nephrogenic 
diabetes insipidus. This patient has 
nephrogenic diabetes insipidus, which can 
persist even if lithium is discontinued, for 
months or years. Hence the best management 
will be medium term management of fluid 


Answer & Comments 

Answer: 5- Syringe driver with diamorphine 

A syringe driver with the correct dose of 
diamorphine according to the amount of 
opiates the patient has had previously, with 
further stat doses as necessary of s/c 
diamorphine or oramorph can be used. Often 
antiemetics (cyclizine) are added as well. 



[ Q: 1230 ] MRCPass - Clinical 
pharmacology 


A 25 year old patient has the following results. 

Sodium 122 mmol/I 

Potassium 2.9 mmol/I 

Urea 13 mmol/l 

Creatinine 160 umol/l 

Bicarbonate 12 mmol/l 

Which drug is likely to be responsible? 

1- Frusemide 

2- Thiazide 

3- Acetazolamide 

4- Amiloride 


Dr. Kholid Yusuf El-Zohry - Sohog Teaching Hospital (01118391123) 

Ref MRCPass OE OE 2012 PasTest 2009 PassMedicine 2009 PasTest Exam ReviseMRCP 



































El-zohry MRCP Questions Bank (Port 1) - 2013 


(For my personal use) 


5- Metolazone 


Answer & Comments 


Answer: 3- Acetazolamide 


1- Tiotropium 

2- Aminophylline 

3- Prednisolone 

4- Salbutamol 


Acetazolamide (carbonic anhydrase inhibitor) 
can cause hyponatraemia, hypokalaemia, 
worsen renal impairment and also a metabolic 
acidosis. Thiazides tend to cause a metabolic 
alkalosis. 


5- Atrovent 


Answer & Comments 


Answer: 3- Prednisolone 


In 

• i 

i j 

A 60 year old lady presents to A&E with 
confusion, headache and tinnitus. Her GP has 
recently started her on an analgesic and there 
is concern she may have taken an overdose. 

Which of the following would most likely 
explain her symptoms? 

1- Dihydrocodeine 

2- Diclofenac 

3- Aspirin 

4- Morphine sulphate tablets 

5- Paracetamol 


[Q: 1231] MRCPass-Clinical 
pharmacology 


Answer & Comments 


Answer: 3- Aspirin 


The patient has signs which would go with 
iatrogenic Cushing's syndrome caused by long 
term steroid therapy. 



[ Q: 1233 ] MRCPass - Clinical 
pharmacology 


A 30 year old man with G6PD deficiency has to 
be cautious when he need to take any 
medication. 


Which one of the following is associated with 
high risk of haemolysis in a patient with G6PD 
deficiency? 

1- Amoxycillin 

2- Trimethoprim 

3- Nitrofurantoin 


4- Metronidazole 

5- Cefuroxime 


Aspirin in excess can cause symptoms of 
nausea, vomiting, headache, confusion and 
tinnitus or hearing difficulties. Whilst the 
dihydrocodeine and MST could cause 
confusion, they would not cause the tinnitus. 


Answer & Comments 

Answer: 3- Nitrofurantoin 

Drugs with high risk of haemolysis in Glucose- 
e-phosphate dehydrogenase are: 



[ Q: 1232 ] MRCPass - Clinical 
pharmacology 


A 45 year old woman with COPD has severe 
bruising around her abdomen and upper 
limbs. She also has centripetal obesity and is 
hypertensive. 


Which of the following of her medication is the 
likely cause for her presentation? 


dapsone 

methylene blue 

nitrofurantoin 

primaquine 

quinolones 

sulphonamides 


Dr. Khalid Yusuf El-Zohry - Sohag Teaching Hospital (01118391123) 

Ref MRCPass OE OE 2012 PasTest 2009 PassMedicine 2009 PasTest Exam ReviseMRCP 
































El-zohry MRCP Questions Bank (Port 1) - 2013 


(For my personal use) 



Haemolysis in G6PD deficiency 



[Q: 1234] MRCPass-Clinical 
pharmacology 


A 45 year old gentleman with type 2 diabetes 
mellitus is brought in following an overdose of 
oral hypoglycaemic agents. There is concern 
that he may have taken an overdose of 
metformin. 


markers suggest a chest infection. She is also 
in atrial fibrillation at a rate of 160/min. Her 
electrolytes are normal (K 4.2 mmol/l). 

As well os treating her pneumonia, she could 
be digitalised with: 

1- Two doses of Digoxin 0.5 mg orally with six 
hours in between 

2- Digoxin 0.25 mg orally once daily 

3- Digoxin 1.0 mg intravenously stat 

4- Digoxin 0.125 mg orally once daily 

5- Digoxin 0.25 mg orally three times daily 


Answer & Comments 

Answer: 1- Two doses of Digoxin 0.5 mg orally 
with six hours in between 


Which of the following is the patient at risk of 
developing? 

1- Hyperglycaemia 

2- Anaemia 


The appropriate loading dose of Digoxin is two 
doses of 0.5 mg (or 500 meg) with six hours in 
between, then a maintenance dose of 0.125 
mg or 0.25 mg a day thereafter. 


3- Respiratory depression 

4- Lactic acidosis 

5- Methaemoglobinaemia 


Answer & Comments 

Answer: 4- Lactic acidosis 

Metformin causes a type-B lactic acidosis in 
overdose (high anion gap), especially in 
patients who have co-ingested alcohol or who 
have underlying renal or hepatic dysfunction. 
Main symptoms of toxicity include 
gastrointestinal upset and a severe lactic 
acidosis. Hypoglycaemia is not often seen in 
metformin overdose. If lactic acidosis occurs 
following overdose, mortality is usually 
greater than 50%. 



[ Q: 1236 ] MRCPass - Clinical 
pharmacology 


A 62 year old woman has hypertension, 
congestive cardiac failure, osteoarthritis, 
urinary tract infections and depression. She 
now feels non specifically unwell. Blood tests 
show that she has acute renal failure, with 
serum creatinine 850 micromol/l. Renal biopsy 
shows acute interstitial nephritis. 


Which medication is most likely to be 
responsible for this condition? 


1- Atenolol 


2- Lisinopril 

3- Ibuprofen 

4- Amitriptyline 

5- Paracetamol 


7 

• LJ 

[Q: 1235] MRCPass 
pharmacology 

- Clinical 

Answer & Comments 

, 66 

year old woman 

presents with 

Answer: 3- Ibuprofen 


breathlessness. Chest X ray and inflammatory 


Dr. Khalid Yusuf El-Zohry - Sohag Teaching Hospital (01118391123) 

Ref MRCPass OE OE 2012 PasTest 2009 PassMedicine 2009 PasTest Exam 




ReviseMRCP 

511 



























El-zohry MRCP Questions Bank (Part 1) - 2013 


(For my personal use) 


The drugs that most commonly cause acute 
interstitial nephritis are penicillins, non¬ 
steroidal anti-inflammatory drugs and thiazide 
diuretics. 


3 

A 55 year old man had suffered trauma to the 
lumbar spine from a car accident. He had 
spasticity and urinary symptoms. A drug was 
prescribed, several weeks later he developed 
jaundice. 

What is the drug which was used? 

1- Oxybutinin 

2- Tizanidine 


[Q: 1237] MRCPass-Clinical 
pharmacology 


Answer & Comments 

Answer: 4- Amitriptyline 

Patients who have taken an amitriptyline 
overdose may present with dilated pupils, dry 
mouth, drowsiness, sinus tachycardia, urinary 
retention (palpable bladder), increased 
tendon reflexes, and extensor plantar 
responses. Hepatitis can also occur with 
amitriptyline. 



[ Q: 1239 ] MRCPass - Clinical 
pharmacology 


A 44 year old patient with previous tonic 
clonic seizures is on sodium valproate 400mg 
bd. 


3- Baclofen 

4- Diazepam 

5- Ibuprofen 


Answer & Comments 


Answer: 2- Tizanidine 


Which of the following is a common side-effect 
of sodium valproate? 

1- Gum hypertrophy 

2- Weight loss 

3- Hirsutism 

4- Tremor 


Tizanidine is an alpha 2 agonist drug. It is used 
to treat muscle spasms. Its side effects are 
nausea, anorexia and jaundice. 


A 35 year old lady with depression is unwell 
and brought to hospital by her friend. On 
examination, she had jaundice, dry mouth and 
was drowsy. She had a palpable bladder. Her 
friend mentioned that she may have taken an 
overdose. 

Which drug may have been taken? 

1- Paroxetine 

2- Paracetamol 

3- Ibuprofen 

4- Amitriptyline 

5- Codeine 



[ Q: 1238 ] MRCPass - Clinical 
pharmacology 


5- Thrombocytosis 


Answer & Comments 

Answer: 4- Tremor 

Side effects of sodium valproate are tremor, 
weight gain, transient hair loss and 
thrombocytopenia. 



[Q: 1240] MRCPass-Clinical 
pharmacology 


Which of the following medications is an ADP 
receptor antagonist? 

1- Infliximab 

2- Abxicimab 

3- Tirofibran 

4- Clopidogrel 

5- Ezetimide 


Dr. Khalid Yusuf El-Zohry - Sohag Teaching Hospital (01118391123) 

Ref MRCPass OE OE 2012 PasTest 2009 PassMedicine 2009 PasTest Exam 




ReviseMRCP 

512 

































El-zohry MRCP Questions Bank (Port 1) - 2013 


(For my personal use) 


Answer & Comments 

Answer: 4- Clopidogrel 

Clopidogrel is an ADP receptor antagonist. 
Abxicimab and tirofibran are G2B3A 
antagonists which are also used in unstable 
coronary syndromes. 


3- Treat with folinic Acid 

4- Assess respiratory function 

5- U + E measurement 

Answer & Comments 

Answer: 4- Assess respiratory function 



[Q: 1241] MRCPass-Clinical 
pharmacology 


A 50 year old man was found collapsed on the 
street. When in casualty, he was alert initially, 
but then develops a grand mal convulsion. 


Which is the most appropriate drug therapy? 

1- Carbamazepine 200mg bd orally 

2- Phenobarbitone 10 mg/kg body weight, 
intravenously 


3- Lorazepam 2 mg intravenously 

4- Phenytoin intravenously at a rate of 100- 
150 mg 

5- Diazepam 10 mg intravenously 


Answer & Comments 

Answer: 3- Lorazepam 2 mg intravenously 

First-line treatment should be with 
intravenous benzodiazepine, with lorazepam 
preferred to diazepam because of its longer 
duration of action. 


A 18 year old student was admitted eight 
hours after taking an overdose of Diazepam 
40mg, Methotrexate 400mg, which were her 
mother's medications. 

On examination, her Glasgow Coma Score was 
10/15. 

Which of the following is the most appropriate 
immediate action? 

1- Gastric lavage 

2- Venous bicarbonate 



[Q: 1242] MRCPass-Clinical 
pharmacology 


Her depressed GCS is likely to be due to 
diazepam. The most appropriate inital 
treatment would be to assess her respiratory 
function. Following this, she should be given 
folinic acid. 

Methotrexate overdose is rare but potentially 
fatal hepatotoxicity and renal toxicity are 
reported. The appropriate treatment is folinic 
acid (Leucovorin). Leucovorin can reverse 
many effects of a methotrexate overdose but 
must be taken soon after the methotrexate 
was taken. 



[Q: 1243] MRCPass-Clinical 
pharmacology 


A 45 year old man has right sided hemiparesis 
from a CVA sustained a month ago. He now 
has constant burning pains in the right arm 
and leg. 


Which of the following is likely to be effective 
in relieving his symptoms? 


1- Ibuprofen 

2- Gabapentin 


3- Tramadol 


4- Oxycodone 

5- Paracetamol 


Answer & Comments 

Answer: 2- Gabapentin 

Although all the drugs can help from an 
analgesic point of view , the best drug to treat 
neuropathic pains is gabapentin. It resembles 
gaba aminobutyric acid, a pain inhibitor in the 
central nervous system, although its mode of 
action is unclear. 


Dr. Khalid Yusuf El-Zohry - Sohag Teaching Hospital (01118391123) 

Ref MRCPass OE OE 2012 PasTest 2009 PassMedicine 2009 PasTest Exam 




ReviseMRCP 

513 
































El-zohry MRCP Questions Bank (Port 1) - 2013 


(For my personal use) 



[Q: 1244] MRCPass-Clinical 
pharmacology 

A 40 year old patient presents with blurring of 
the vision. He is on various medications. 

Which one of the following drugs con couse 
visual disturbance, despite being within 
therapeutic range? 

1- Diltiazem 

2- Digoxin 

3- Lisinopril 

4- Carbamazepine 

5- Amiodarone 


Answer & Comments 

Answer: 5- Amiodarone 

Digoxin which is the other possible option 
usually causes visual disturbance only in toxic 
doses. Amiodarone causes reversible corneal 
microdeposits. 



[Q: 1245] MRCPass-Clinical 
pharmacology 

A 72 year old woman is assessed for visual 
disturbance. She complains of visual haloes 
and mild photophobia, which have been 
present for a few weeks. 

Which one of the following drugs is the most 
likely cause of her symptoms? 

1- Aspirin 

2- Amlodipine 

3- Amiodarone 

4- Atenolol 

5- Thiazide 


Answer & Comments 

Answer: 3- Amiodarone 

Amiodarone may cause corneal 
microdeposits, and symptoms include visual 
haloes and photophobia. 




Corneal Microdeposits seen with a slit lamp 


[Q: 1246] MRCPass-Clinical 
pharmacology 

A 60 year old woman complains of nausea, 
vomiting, headache, confusion and tinnitus. 

What is she likely to have overdosed on? 

1- Codeine 

2- Tramadol 

3- Oramorph 

4- Aspirin 

5- Metoclopramide 



Answer & Comments 

Answer: 4- Aspirin 

Aspirin in excess causes symptoms of nausea, 
vomiting, headache, confusion and tinnitus or 
hearing difficulties. Tinnitus is unusual with 
the other medications. 



[Q: 1247] MRCPass-Clinical 
pharmacology 


A 50 year old man complains of pain and 
swelling in the chest wall. 


Which of the following drugs is likely to cause 
gynaecomastia? 


1- Atenolol 


2- Frusemide 

3- Cimetidine 


4- Lansoprazole 

5- Ramipril 


Dr. Khalid Yusuf El-Zohry - Sohag Teaching Hospital (01118391123) 

Ref MRCPass OE OE 2012 PasTest 2009 PassMedicine 2009 PasTest Exam ReviseMRCP 

































El-zohry MRCP Questions Bank (Part 1) - 2013 


(For my personal use) 


Answer & Comments 

Answer: 3- Cimetidine 

Drugs which can cause gynaecomastia are : 

■ digoxin 
oestrogens 

■ spironolactone 
cimetidine 

■ verapamil 
nifedipine 



stimulating the osteoblasts to produce an 
inhibitor of osteoclast formation. Osteoclast 
inactivation is associated with bisphosphonate 
uptake from the bone surface. Furthermore, 
the bisphosphonates may act by shortening 
the life span of the osteoclasts, possibly 
through apoptosis. 

1. Direct inhibition of osteoclasts when they take 
up bisphosphonate present on bone. 



^ [ Q: 1249 ] MRCPass - Clinical 

jf | pharmacology 

A 48 year man has impotence. He also has a 
history of angina and hypertension. 

Which one of the following drugs which he 
takes would present a contraindication 
towards Sildenafil? 

1- Aspirin 

2- Bendrofluazide 


[Q: 1248] MRCPass-Clinical 
pharmacology 

A 60 year old lady is on alendronate once a 
week. 

Which of the following best describes the 
mode of action of alendronate? 



3- Isosorbide Mononitrate 

4- Atenolol 

5- Lisinopril 


Answer & Comments 


Answer: 3- Isosorbide Mononitrate 


1- Binds vitamin D 

2- Promotes collagen synthesis 

3- Promotes bone matrix calcification 

4- Inhibits osteoclastic activity 


Nitrate use with Sildenafil (Viagra) are 
contraindicated due to precipitant drops in 
blood pressure. Viagra is also associated with 
increases in intraocular pressure and hence 
should be avoided in glaucoma. 


5- Inhibits osteoblastic activity 



* *J 


[Q: 1250] MRCPass-Clinical 

Answer & Comments 



pharmacology 


Answer: 4- Inhibits osteoclastic activity 

Bisphosphonates acts at the cellular level. 
They act directly or indirectly on the 
osteoclasts. The effect can be on the 
formation of osteoclasts and/or on their 
activity. A decrease in osteoclast number can 
occur either through direct action on 
osteoclast precursors, or indirectly by 


A man in his twenties is brought to the 
accident and emergency department by 
ambulance from a night club. He is has a GCS 5 
with pin-point pupils and a slow respiratory 
rate. 

Immediate specific treatment should be: 

1- N-acetyl cysteine (150 mg/kg over 15 min) 
intravenously 


Dr. Khalid Yusuf El-Zohry - Sohag Teaching Hospital (01118391123) 

Ref MRCPass OE OE 2012 PasTest 2009 PassMedicine 2009 PasTest Exam 




ReviseMRCP 

515 































El-zohry MRCP Questions Bank (Port 1) - 2013 


(For my personal use) 


2- Insert NG tube and give activated charcoal 

3- Dextrose (50 ml of 50% solution) 
intravenously 

4- Naloxone (0.4 mg) intravenously, repeated 
if no effect 

5- Flumazenil O.lmg/kg intravenously, 
repeated if no effect 


Answer & Comments 

Answer: 4- Naloxone (0.4 mg) intravenously, 
repeated if no effect 

The likely diagnosis is (in view of the pinpoint 
pupils) opioid overdose, the treatment for 
which is intravenous naloxone (0.4 mg), 
repeated up to a total dose of 2 mg depending 
on clinical response. 

7 

• 1 J 

A 22 year old man has taken a cocaine 
overdose. He presents unwell to A&E. 

Which one of the following is o major side 
effect that should be observed for? 

1- Hypothermia 

2- Convulsions 

3- Jaundice 

4- Hypotension 

5- Hypotonia 


[Q: 1251] MRCPass- Clinical 
pharmacology 


Answer & Comments 

Answer: 2- Convulsions 

Convulsions, pyrexia, cardiorespiratory 
depression, hypertension, agitation and 
hypertonia are major side effects in cocaine 
toxicity. 



[Q: 1252] MRCPass-Clinical 
pharmacology 


A 65 year old man with atrial fibrillation and 
mitral valve disease has been stable on 


warfarin for many years. He is admitted with a 
severe epistaxis and his INR is found to be 
grossly elevated at 9. 

Recent prescription of which of the following 
drugs could explain this? 

1- Barbiturates 

2- Phenytoin 

3- Rifampicin 

4- Ciprofloxacin 

5- Griseofulvin 


Answer & Comments 

Answer: 4- Ciprofloxacin 

Warfarin is metabolized by the cytochrome 
P450 enzyme system. Ciprofloxacin inhibits 
CYP450 (liver enzyme inhibitor) and hence 
potentiates the action of warfarin, which 
could explain this presentation with bleeding 
and high INR. 


A 28 year old Type I diabetic is currently on a 
basal-bolus regime, comprising twice a day 
basal Isophane insulin, complemented by 
short-acting insulin at meal times. He is 
exploring new insulin regimes as he works 
night shifts. 

Which form of insulin is recommended? 

1- Lispro 

2- Mixtard 50 

3- Mixtard 30 

4- Actrapid 

5- Insulin glargine 


[Q: 1253] MRCPass-Clinical 
pharmacology 



Answer & Comments 

Answer: 5- Insulin glargine 

Insulin glargine is a long-acting insulin 
analogue, produced by modifying the 
chemical structure of insulin. This gives it a 


Dr. Khalid Yusuf El-Zohry - Sohag Teaching Hospital (01118391123) 

Ref MRCPass OE OE 2012 PasTest 2009 PassMedicine 2009 PasTest Exam ReviseMRCP 































El-zohry MRCP Questions Bank (Port 1) - 2013 


(For my personal use) 


prolonged absorption profile with no peaks. 
When given at night, it provides good control 
of the fasting blood glucose and reduces the 
risk if hypoglycaemia 


3- 160 mg with 10 mg prn 

4- 80 mg with 15 mg prn 

5- 80 mg with 1 mg prn 



[Q: 1254] MRCPass- Clinical 
pharmacology 


A 60 year old man is unwell having ingested a 
bottle of dye. On examination, he is afebrile 
but has tachypnea, cyanosis, and drowsiness. 
He is given 100% oxygen but does not 
improve. A lab test confirms methaemoglobin 
levels >70%. 


Whot should be given? 

1- Hyperbaric oxygen 

2- Ibuprofen 

3- Haemodialysis 

4- 10% glucose infusion 

5- Methylene blue 


Answer & Comments 

Answer: 5- Methylene blue 

Methaemoglobinaemia is a cause of cyanosis 
because it causes the formation of reduced Hb 
>1.5 g/dl. Chemicals which are oxidising 
agents may cause this e.g. aniline dyes, 
chlorates, nitrates, nitrophenols, primaquine 
and sulphonamides. Treatment is with 
methylene blue if methaemoglobin >3.0g/dL. 


A 60 year old patient with carcinoma of the 
colon has good pain control on MST 120 mg 
bd. She is admitted with increasing weakness 
and has difficulty with swallowing tablets. It is 
therefore decided to convert her to a 24-hour 
diamorphine syringe driver. 

The correct dose of diamorphine is: 

1- 160 mg with 30 mg prn 

2- 160 mg with 20 mg prn 



[Q: 1255] MRCPass-Clinical 
pharmacology 


Answer & Comments 

Answer: 4- 80 mg with 15 mg prn 

The total morphine dose is 240mg. The 24 
hour diamorphine dose should be 1/3 of the 
24 hour morphine dose, which is 80mg. The 
PRN dose should be 1/6 of the total 24 hour 
diamorphine dose, which is 13.3mg, rounded 
up to 15 mg. 



[Q: 1256] MRCPass-Clinical 
pharmacology 

Which one of the following drugs is associated 
with both a raised anion and osmolar gap? 

1- Aspirin 

2- Ethanol 

3- Phenytoin 

4- Cyanide 

5- Metformin 


Answer & Comments 

Answer: 2- Ethanol 

The anion gap is calculated as (Na+ + K+) - (Cl- 
+ HC03-). It is normally between 10 to 16. 

Drug causes of a raised anion gap following 
overdose include ethanol, salicylates, 
paracetamol, metformin, cyanide and 
isoniazid. 

Calculation of plasma osmolality is: 2(Na+ + 
K+) + Urea + Glucose. Drug causes of a raised 
osmolar gap include ethanol, ethylene glycol 
and methanol. 


Dr. Khalid Yusuf El-Zohry - Sohag Teaching Hospital (01118391123) 

Ref MRCPass OE OE 2012 PasTest 2009 PassMedicine 2009 PasTest Exam 




ReviseMRCP 

517 




























El-zohry MRCP Questions Bank (Port 1) - 2013 


(For my personal use) 


#§ 

• J 

A 55 year old man who has rheumatoid 
arthritis is referred by the GP for lethargy. He 
has been on weekly injections. 

A full blood count showed : 

Hb 6.6 g/dl 

MCV 82 fl 

WCC 0.8 x 10 9 /L 

Pit 35 x 10 9 /L 

He has noticed a sore throat for 2 days and 
also observed a purpuric rash over his 
abdomen. 

Which is the following is the most likely drug 
to have caused this? 

1- Penicillamine 

2- Ibuprofen 

3- Gold 

4- Etanercept 

5- Infliximab 


[Q: 1257] MRCPass-Clinical 
pharmacology 


Answer & Comments 

Answer: 3- Gold 

The patient describes features of bone 
marrow suppression (anaemia, leukopenia 
and thrombocytopenia) induced by one of the 
disease modifying anti-rheumatic drugs 
(DMARDs). In this case, gold is most likely to 
be given as a weekly injection. 



[Q: 1258] MRCPass-Clinical 
pharmacology 

A 70 year old Caucasian patient was referred 
by the GP due to recordings of blood pressure 
of 180/100 mmHg initially and recordings of 
170/95 and 170/90 when repeated 3 months 
later. Urine dipstick shows no protein and no 
blood. 

Which agent should be commenced? 

1- Atenolol 


2- Bendrofluazide 

3- Ramipril 

4- Losartan 

5- Moxonidine 


Answer & Comments 

Answer: 2- Bendrofluazide 

In a patient of this age group a diuretic e.g. 
bendrofluazide should be commenced first. 
Followihg BHS criteria - Older and Black 
patients could have either a C (calcium 
channel blocker) or D (diuretic). 



[Q: 1259] MRCPass-Clinical 
pharmacology 

A 22 year old lady who is on oral 
contraceptives is seeking advice regarding her 
medication. 

Which of the following may make the 
contraceptive LESS effective? 

1- Erythromycin 

2- Ketoconazole 

3- Isoniazid 

4- Gliclazide 

5- Valproate 


Answer & Comments 

Answer: 4- Gliclazide 

Gliclazide is a sulphonylurea which is a liver 
enzyme inducer. The rest of the options are 
liver enzyme inhibitors. 

LIVER ENZYME INDUCERS (PCBRAS): 

Phenytoin 

■ Carbamazepine 

■ Barbiturates 

■ Rifampicin 
Alcohol 

Sulphonylureas 


Dr. Khalid Yusuf El-Zohry - Sohag Teaching Hospital (01118391123) 

Ref MRCPass OE OE 2012 PasTest 2009 PassMedicine 2009 PasTest Exam ReviseMRCP 































El-zohry MRCP Questions Bank (Port 1) - 2013 


(For my personal use) 



[ Q: 1260 ] MRCPass - Clinical 
pharmacology 

A 30 year old lady has taken overdoses of 
several drugs, and a casualty officer is 
considering administration of charcoal. 

Which of the following drugs would not be 
adsorbed by activated charcoal? 

1- Paracetamol 

2- Salicylates 

3- Theophylline 

4- Digoxin 

5- Lithium 


Answer & Comments 

Answer: 5- Lithium 

Ethanol, ethylene glycol, iron, gold, mercury, 
lithium, many acids and alkalis are not 
adsorbed by activated charcoal. Activated 
charcoal binds most other drugs that are 
commonly used in poisonings. 



[Q: 1261] MRCPass-Clinical 
pharmacology 

A 58 year old woman has been on Nifedipine 
for hypertension. She mentions several 
possible side effects which she blames the 
medication for. 

Which one of the following is a side effect with 
Nifedipine? 

1- Gum hypertrophy 

2- Hirsutism 

3- Bone marrow suppression 

4- Dry cough 

5- Weight loss 


Side effects of nifedipine are gum hyperplasia, 
headache, myalgia, tremors, cholestatic 
jaundice and visual disturbance. 


A 25 year old woman with no significant past 
medical history, presents 20 weeks into 
pregnancy with a painful swollen calf. 
Ultrasound examination confirms that she has 
a deep venous thrombosis. 

How should this be managed up to the time of 
delivery? 

1- Initiate and then continue treatment with 
warfarin until delivery 

2- Initiate and then continue treatment with 
heparin until delivery 

3- No anticoagulation 

4- Initiate treatment with both heparin and 
warfarin and then continue until delivery 

5- Initiate treatment with heparin, and 
convert to warfarin, continued until after 
delivery 


[ Q: 1262 ] MRCPass - Clinical 
pharmacology 



Answer & Comments 

Answer: 2- Initiate and then continue 
treatment with heparin until delivery 

Warfarin in the first trimester can cause fetal 
hypoplasia of the nose and limbs. After this 
period warfarin is associated with neurological 
damage - mental retardation, microcephaly, 
optic atrophy and blindness. There is an 
option to convert from heparin to warfarin in 
the third trimester, but the patient will have 
to be re-converted back to heparin before 
delivery. 


Answer & Comments 


Answer: 1- Gum hypertrophy 



[ Q: 1263 ] MRCPass - Clinical 
pharmacology 

A 60 year old man with adenocarcinoma of 
the colon has been taking morphine sulphate 
continus (MST) 100 mg B.d. for 3 months. 


Dr. Khalid Yusuf El-Zohry - Sohag Teaching Hospital (01118391123) 

Ref MRCPass OE OE 2012 PasTest 2009 PassMedicine 2009 PasTest Exam ReviseMRCP 






































El-zohry MRCP Questions Bank (Port 1) - 2013 


(For my personal use) 


Following a course of chemotherapy including 
oxaliplatin and 5 Fluorouracil, he developed a 
2 day history of acute frequent diarrhoea. 

The diarrhoea discontinued but he presents 
with a 5 day history of drowsiness, lethargy 
and ankle oedema. 

Examination reveals pin point pupils. 

Whot is the most likely cause of the 
symptoms? 

1- Metastatic disease 

2- Interaction between chemotherapy and 
morphine 

3- Excessive antiemetic use 

4- Renal failure leading to accumulation of 
morphine 


5- Rifampicin 

Answer & Comments 

Answer: 2- Ciprofloxacin 

Liver enzyme inhibitors are omeprazole, 
disulfram, erythromycin, valproate, isoniazid, 
cimetidine, ciprofloxacin, ethanol and 
sulphonamides. 

^ [ Q: 1265 ] MRCPass - Clinical 

S pharmacology 

/ I- -- - - , | . ■„ . ........ .... -~ 

— 

receptor blockers are used in hypertension 
and prostate hypertrophy. 

Which one of the following occurs with 
stimulation of alpha receptors? 


5- Liver failure due to opiates 


Answer & Comments 

Answer: 4- Renal failure leading to 
accumulation of morphine 

Renal failure can accentuate the effects of 
opiates as in this instance, leading to 
drowsiness and pinpoint pupils. In this case 
the diarrhoea may have been caused by 
chemotherapy. 


^ [Q: 1264] MRCPass-Clinical 

" pharmacology 

A 60 year old woman with atrial fibrillation 
and previous transient ischaemic attacks has 
been on warfarin for several years. She 
complains of excessive bleeding and presents 
for a review . The INR result was 9. Upon 
further enquiry, she mentions having been on 
a new drug recently. 

Which of the following is the most likely 
co use? 

1- Carbamazepine 

2- Ciprofloxacin 

3- Theophylline 

4- Sulphonylurea 


1- Bronchoconstriction 

2- Increase in gut motility 

3- Vasoconstriction 

4- Uterus relaxation 

5- Increase in pancreatic exocrine secretion 


Answer & Comments 

Answer: 3- Vasoconstriction 

&^Dk|haEbuEic Nd-i. 41 * AdrvikHl GLand 

\ / 

Epdto&pirihiC 

Nun* pi rap hr In? 

/ \ 

Alplij-rucupGiKr Etda.iniTfpUH- 

I i 

SlniMilh Mnw.'k> Cunlrat I h»ii S-miHdb IV [lurk * 1 2 3 4 Hdii u I mhi 
UK-TITUS- tnl Kim* CtLTxlJtM SEllllllLkCL'jn, 

Hum-iiM-iE Xcin pi iiyjilu iiu- SHLjEl'l] Aim aji s 

lllLTl'HM.ll 



[ Q: 1266 ] MRCPass - Clinical 
pharmacology 


A 60 year old lady who has significant 
respiratory disease is on long term steroids. 
Her bone mineral density is low . She has been 
commenced on vitamin D and calcium 
supplements. 


Dr. Khalid Yusuf El-Zohry - Sohag Teaching Hospital (01118391123) 

Ref MRCPass OE OE 2012 PasTest 2009 PassMedicine 2009 PasTest Exam ReviseMRCP 
































El-zohry MRCP Questions Bank (Port 1) - 2013 


(For my personal use) 


Which of the following would most likely be 
recommended os treatment for corticosteroid- 
induced osteoporosis? 

1- Vitamin A 

2- Vitamin C 

3- Infliximab 

4- Hormone replacement therapy 

5- Erythropoietin 


Answer & Comments 

Answer: 4- Hormone replacement therapy 

Hormone replacement therapy should be 
recommended for postmenopausal women, 
unless contraindicated. 

Although calcium does not completely 
suppress bone loss, patients given high-dose 
inhaled or systemic corticosteroids should 
have a calcium intake of at least 1,000 
mg/day. Patients receiving limited sun 
exposure, especially during the winter, may 
benefit from supplements containing 400 
lU/day of vitamin D. Although supplemental 
calcium and vitamin D may counteract the 
effects of corticosteroids on calcium transport, 
studies have shown that they do not 
completely eliminate corticosteroid-induced 
bone loss. 


[Q: 1267] MRCPass-Clinical 
pharmacology 

A 60 year old woman has increasing frequency 
of migraine attacks despite NSAIDs, and has 
been referred for further treatment. 

Which one of the following drugs would be 
appropriate for acute treatment against 
migraine? 

1- Sodium valproate 

2- Propranolol 

3- Amitriptyline 

4- Pizotifen 

5- Sumatriptan 



Answer & Comments 

Answer: 5- Sumatriptan 

Sumatriptan is a 5HT1 agonist and may be 
useful in the treatment of acute migraine 
attacks, is available in injectable, intranasal, 
and oral formulations. Ergotamine tartrate is 
also effective in acute migraine. Propanolol, 
valproate, NSAIDs, amitriptyline, pizotifen and 
gabapentin are effective as prophylactic drugs 
in migraine. 



[ Q: 1268 ] MRCPass - Clinical 
pharmacology 


A 70 year old man has a creatinine of 350 
umol/l. 


Which one of the following drugs is likely to 
worsen the renal failure? 

1- Erythromycin 

2- Sulfasalazine 


3- Paracetamol 


4- Olanzepine 

5- Omeprazole 


Answer & Comments 

Answer: 2- Sulfasalazine 

Sulfasalazine, NSAIDS, allopurinol, and ACE 
inhibitors are examples of nephrotoxic drugs. 


A 35 year old man presents following an 
overdose with amitryptiline and 
anticholinergic syndrome is suspected. 

Which one of the following is likely? 

1- Urinary incontinence 

2- Mydriasis 

3- Pale skin 

4- Bradycardia 

5- The patient is calm 


[ Q: 1269 ] MRCPass - Clinical 
pharmacology 



Dr. Khalid Yusuf El-Zohry - Sohag Teaching Hospital (01118391123) 

Ref MRCPass OE OE 2012 PasTest 2009 PassMedicine 2009 PasTest Exam 




ReviseMRCP 

521 




























El-zohry MRCP Questions Bank (Port 1) - 2013 


(For my personal use) 


Answer & Comments 


Answer: 2- Mydriasis 


5- Oral nifedipine capsules 20mg tds if 
hypertension confirmed over 1-2 weeks 


Anticholinergic syndrome occurs following 
overdose with drugs that have anticholinergic 
activity. 

Examples of these are tricyclic 
antidepressants, antihistamines and atropine. 

Features include flushed skin, urinary 
retention, tachycardia, mydriasis (dilated 
pupils) and agitation. Although physostigmine, 
a reversible inhibitor of acteylcholinesterase, 
is effective in treating symptoms, there is a 
significant risk of cardiac toxicity (bradycardia, 
AV conduction defects and asystole). 



Mydriasis 



[Q: 1270] MRCPass-Clinical 
pharmacology 


A 55 year old man of African origin presents 
with blood pressure 210/100mmHg. Urinalysis 
is negative and fundoscopy shows AV nipping. 


Which of the following treatment options is 
most appropriate? 


1- Urgent admission for control of malignant 
hypertension 


2- Oral enalapril lOmg bd if hypertension 
confirmed over 1-2 weeks 


3- Oral atenolol 50mg od if hypertension 
confirmed over 1-2 weeks 


4- Oral nifedipine LA 30mg od if hypertension 
confirmed over 1-2 weeks 


Answer & Comments 

Answer: 4- Oral nifedipine LA 30mg od if 
hypertension confirmed over 1-2 weeks 

The British Hypertension Society guidelines 
suggest that the finding of blood pressure 
200-219/110-119mmHg should be confirmed 
with repeat measurements after 1-2 weeks, 
then treated if still elevated. 

Black patients have low renin hypertension, 
therefore ACE inhibitors and (3-blockers as 
single agents are less effective in this group. 
Calcium channel blockers and diuretics are 
effective agents. Nifedipine should be used as 
a long-acting preparation, not a short acting 
one. 



[Q: 1271] MRCPass-Clinical 
pharmacology 


Clozapine is an atypical antipsychotic drug 
that has fewer side effects than older 
antipsychotics. 

This is because of: 

1- Decreased neurotransmitter release 

2- Low affinity for 5HT receptors 

3- Low affinity for dopamine D2 receptors 

4- It does not cause agranulocytosis 

5- There is better renal clearance 


Answer & Comments 

Answer: 3- Low affinity for dopamine D2 
receptors 

Clozapine has fewer extrapyramidal adverse 
effects than older antipsychotics. This has 
been attributed to its relatively low affinity for 
D2 dopamine receptors. Unlike older 
antipsychotics, clozapine has relatively high 
affinity for 5HT receptors. Agranulocytosis is a 
recognized complication of clozapine. 



Dr. Khalid Yusuf El-Zohry - Sohag Teaching Hospital (01118391123) 

Ref MRCPass OE OE 2012 PasTest 2009 PassMedicine 2009 PasTest Exam 


ReviseMRCP 



























El-zohry MRCP Questions Bank (Port 1) - 2013 


(For my personal use) 



[Q: 1272] MRCPass-Clinical 
pharmacology 

A 60 year old patient has recently been 
commenced on digoxin. 

Which of the following features is likely to 
suggest digoxin toxicity? 

1- Vomiting 

2- Reverse tick in the lateral leads on ECG 

3- Blindness 

4- Chest pain 

5- Supraventricular ectopics 


Answer & Comments 

Answer: 1- Vomiting 

Digoxin toxicity can cause symptoms of 
nausea and vomiting, xanthopsia (yellowness 
in vision), dizziness, bradycardia and lethargy. 
Reversed tick in the ST segments can be 
associated with digoxin use and is not 
necessarily a sign of toxicity. 



[Q: 1273] MRCPass-Clinical 
pharmacology 


A 65 year old lady presents with transient loss 
of vision which lasts half an hour. She has a 
past medical history of hypertension. Her ECG 
confirms atrial fibrillation. 


Which one of the following therapy is most 
appropriate? 

1- Aspirin 

2- Aspirin and dipyridamole 

3- Aspirin and clopidogrel 

4- Clopidogrel 

5- Warfarin 


Answer & Comments 

Answer: 5- Warfarin 

In a patient with previous TIA or CVA and 
atrial fibrillation, warfarinisation must be 


considered unless there are contraindications. 
Using the CHADS-2 score, the patient scores 1 
for a TIA, and 1 for Hypertension - a score of 2 
and above suggests warfarin is appropriate. 


A 35 year old lady has an aspirin overdose. She 
is hyperventilating and complains of tinnitus. 

What should be given? 

1- N acetyl cysteine 

2- Oil of wintergreen 

3- Activated charcoal 

4- Naloxone 

5- Methylene blue 


[Q: 1274] MRCPass-Clinical 
pharmacology 



Answer & Comments 

Answer: 3- Activated charcoal 

Initially after a salicylate overdose, 
hyperventilation due to respiratory drive 
stimulation causes a respiratory alkalosis. This 
is followed by metabolic acidosis. Oil of 
wintergreen contains salicylates. Management 
includes repeated doses of activated charcoal 
until plasma salicylate concentrations have 
peaked. 



[Q: 1275] MRCPass-Clinical 
pharmacology 


A 60 year woman has rheumatoid arthritis. 
She states that she is allergic to Co- 
trimoxazole. 


Which of the following DMARDs should not be 
used? 

1- Gold 

2- Penicillamine 


3- Methotrexate 

4- Sulfasalazine 

5- Hydroxychloroquine 


Dr. Khalid Yusuf El-Zohry - Sohag Teaching Hospital (01118391123) 

Ref MRCPass OE OE 2012 PasTest 2009 PassMedicine 2009 PasTest Exam ReviseMRCP 



523 

































El-zohry MRCP Questions Bank (Port 1) - 2013 


(For my personal use) 


Answer & Comments 


Answer: 4- Sulfasalazine 


Answer & Comments 


Answer: 3- Ramipril 


Co-trimoxazole is trimethoprim and 
sulfamethoxazole. The patient is allergic to 
sulphonamides. 



[Q: 1276] MRCPass-Clinical 
pharmacology 


A 15 year old asthmatic patient who has been 
given theophylline is now tachycardic with a 
heart rate of 130 beats per minutes. 


What is the mechanism of action of the drug? 


1- Adenosine receptor antagonism 


2- B1 receptor stimulation 


In patients with reduced ejection fraction, ACE 
- inhibitors have been shown to reduce 
cardiovascular events (CVA and Ml) and 
mortality. There are many trials with ACE- 
inhibitors including SOLVD (enalapril), ISIS-4 
(captopril) and HOPE (ramipril) trials. 


[Q: 1278] MRCPass-Clinical 
pharmacology 

An 18 year old student is admitted with severe 
crushing central chest pain with associated 
nausea and profuse sweating. He admits to 
smoking cocaine. 



3- Alpha receptor agonist 

4- Leukotriene inhibitor 

5- Anticholinergic 

Answer & Comments 

Answer: 1- Adenosine receptor antagonism 

Theophylline is a methylxanthine which is a 
phosphodiesterase enzyme inhibitor. This 
action increases cyclic AMP levels. It is also an 
antagonist of adenosine receptors. 


What is the mechanism of myocardial 
ischaemia? 

1- Coronary thrombosis 

2- Tachycardia 

3- Alpha receptor blockade 

4- Coronary vasospasm 

5- Parasympathetic activity 

Answer & Comments 

Answer: 4- Coronary vasospasm 


[Q: 1277] MRCPass-Clinical 
pharmacology 

A 65 year old man has cardiac risk factors of 
hypertension and is on aspirin. 
Echocardiography shows reduced ejection 
fraction at 50%. 

Which one medication may reduce the risk of 
future cardiovascular events? 

1- Furosemide 

2- Isosorbide mononitrate 

3- Ramipril 

4- Verapamil 

5- Clopidogrel 



Cocaine causes coronary vasospasm sufficient 
to present as severe ischaemia. 

Cocaine causes an increase in circulating 
catecholamines. Therefore alpha-adrenergic 
mediated focal or generalized coronary artery 
spasm has been presumed to be the likely 
mechanism to induce ischemia. 


A 43 year old lady with known liver cirrhosis 
has now been found to have a post prandial 
glucose of 16 and had been high on two 
previous occasions. 

What drug should be commenced? 



[Q: 1279] MRCPass-Clinical 
pharmacology 


Dr. Khalid Yusuf El-Zohry - Sohag Teaching Hospital (01118391123) 

Ref MRCPass OE OE 2012 PasTest 2009 PassMedicine 2009 PasTest Exam ReviseMRCP 



































El-zohry MRCP Questions Bank (Port 1) - 2013 


(For my personal use) 


1- Preprandial insulin 

2- Metformin 

3- Rosiglitazone 

4- Gliclazide 

5- Acarbose 


Answer & Comments 

Answer: 4- Gliclazide 

Type 2 diabetes in liver cirrhotic patients 
respond well to sulfonylureas which enhance 
peripheral tissue sensitivity to insulin. This 
should be considered first and insulin 
subsequently added when glycaemic control is 
not achieved. 


colestyramine). Ezetimibe is currently licensed 
for use in combination with a statin in patients 
who fail to reach desired lipid profiles or as 
monotherapy in patients intolerant to a statin. 



[Q: 1281] MRCPass-Clinical 
pharmacology 


Frusemide (furosemide) acts on which part of 
the kidney? 

1- Bowman's capsule 

2- Ascending loop of Henle 

3- Descending loop of Henle 

4- Distal convoluted tubule 

5- Proximal convoluted tubule 


[ Q: 1280 ] MRCPass - Clinical 
% pharmacology 

A 60 year old man with elevated cholesterol 
has failed to reach a desired cholesterol level 
on statin treatment. 


The endocrinologist suggests ezetimibe. 

Which of the following is on effect of 
ezetimibe? 

1- Elevation in plasma triglyceride 

2- Decreased absorption of fat soluble 
vitamins 

3- Reduction in high density lipoprotein (HDL) 

4- Reduction in low-density lipoprotein (LDL) 

5- Prevents cholesterol synthesis 


Answer & Comments 

Answer: 4- Reduction in low-density 

lipoprotein (LDL) 

Ezetimibe acts by prevent cholesterol 
absorption from the small intestine. Typically 
it reduces LDL-cholesterol by approximately 
20%, triglycerides by up to 5% and raises HDL- 
cholesterol by approximately 5%. It does not 
inhibit the absorption of fat-soluble vitamins 
unlike the anionexchange resins (e.g. 


Answer & Comments 

Answer: 2- Ascending loop of Henle 

Frusemide acts on the thick portion of the 
ascending loop of Henle.lt inhibits Na+ and Cl- 
reabsorption there via Na+, K+, -ATPase- 
dependent pump. Owing to the large NaCI 
absorptive capacity of the loop of Henle, 
agents that act at this site produce a diuretic 
effect much greater than that seen with other 
diuretic groups. 


A 25 year old lady has with multiple sclerosis is 
considered for (3-interferon by her neurologist. 
She has relapsing episodes of decreasing 
mobility and bladder dysfunction. 

When is beta interferon recommended? 

1- First onset of the disease 

2- When there is urinary incontinence 

3- Chronic progressive multiple sclerosis 

4- Relapsing remitting multiple sclerosis 

5- During a relapse 


[ Q: 1282 ] MRCPass - Clinical 
pharmacology 



Dr. Kholid Yusuf El-Zohry - Sohog Teaching Hospital (01118391123) 

Ref MRCPass OE OE 2012 PasTest 2009 PassMedicine 2009 PasTest Exam 




ReviseMRCP 

525 





























El-zohry MRCP Questions Bank (Port 1) - 2013 


(For my personal use) 


Answer & Comments 

Answer: 4- Relapsing remitting multiple 
sclerosis 


sodium 137 mmol/l 
potassium 4.3 mmol/l 
urea 25 mmol/l 


(3-interferon is not commenced during the first 
event seen. It is a long term treatment (as 
opposed to steroids for acute relapses) which 
is of benefit only in the relapsing remitting 
form (about 40% of MS patients have this 
form), and slows progression of disability and 
reduces demyelinating lesions. 


^ Q: 1283 ] MRCPass - Clinical 

ft 

Z pharmacology 

I KaB99S9 _ ss _ 9 ^ 

A 50 year old lady is given methotrexate for 
severe rheumatoid disease causing joint pains 
and functional limitation. She also has to be 
given folinic rescue therapy. 

What is the mechanism by which 
methotrexate acts? 

1- Increasing folic acid excretion 

2- DNA binding 

3- Binding to dihydrofolate reductase 

4- Increasing nucleotide synthesis 

5- Inhibiting DNA gyrase 


Answer & Comments 


Answer: 3- Binding to dihydrofolate reductase 


creatinine 360 pmol/l 
Creatine Kinase 11,000 U/l 
What is the diagnosis? 

1- Alcohol intoxication 

2- Tricyclic antidepressant overdose 

3- Methaemoglobinaemia 

4- Rhabdomyolysis 

5- Cocaine overdose 

Answer & Comments 

Answer: 4- Rhabdomyolysis 

The likely diagnosis is ethylene glycol 
poisoning, the clue being the alcohol smell. 
There are many causes of rhabdomyolysis, 
such as trauma and burns, sepsis and drug 
oversose. 



[Q: 1285] MRCPass-Clinical 
pharmacology 


A 40 year old lady is being considered for 
treatment for rheumatoid arthritis. 


Which of the following is a disease modifying 
anti rheumatic drug? 


Binding of methotrexate to dihydrofolate 
reductase reduces nucleotide synthesis as well 
as amino acids serine and methionine. Folinic 
acid rescue is usually given after methotrexate 
therapy (e.g. 24 hours) to reduce 

myelosuppression side effects. 


1- Mercury 

2- Phenylephrine 

3- Silver 

4- Sulfasalazine 

5- Benzoyl peroxide 



[Q: 1284] MRCPass-Clinical 
pharmacology 


A 46 year old patient was found unconscious 
by a friend and brought to hospital. On 
examination he smelled of alcohol and had a 
GCS of 5/15. 


Investigations showed: 


Answer & Comments 

Answer: 4- Sulfasalazine 

DMARDS include gold, choloroquine, 
sulfasalazine, penicillimine, methotrexate, 
azathioprine and leflunomide. 


Dr. Khalid Yusuf El-Zohry - Sohag Teaching Hospital (01118391123) 

Ref MRCPass OE OE 2012 PasTest 2009 PassMedicine 2009 PasTest Exam ReviseMRCP 































El-zohry MRCP Questions Bank (Port 1) - 2013 


(For my personal use) 



[Q: 1286] MRCPass-Clinical 
pharmacology 

Which one of following drugs works by 
inhibiting tumour necrosis factor? 

1- Cyclosporin 

2- Methotrexate 

3- Montelukast 

4- Infliximab 

5- Mesalazine 


Answer & Comments 

Answer: 4- Infliximab 

Etanercept and infliximab inhibit TNF. They 
are licensed in the treatment of rheumatoid 
arthritis. 


Dr. Kholid Yusuf El-Zohry - Sohog Teaching Hospital (01118391123) 

Ref MRCPass OE OE 2012 PasTest 2009 PassMedicine 2009 PasTest Exam ReviseMRCP 























El-zohry MRCP Questions Bank (Port 1) - 2013 


(For my personal use) 


[ Q: 1287 ] MRCPass - Cardiology 

A 60 year man who is type 2 diabetic 
is admitted with chest pain. His ECGs show 
inferior ST elevation of 1 mm in 2 leads. He is 
thrombolysed with steptokinase. His BM 
measurement is 15, and has missed his 
gliclazide dose today. 

What is the best management? 

1- Continue with gliclazide 



1- Restrictive cardiomyopathy 

2- Dilated cardiomyopathy 

3- Constrictive pericarditis 

4- Ischaemic cardiomyopathy 

5- Pulmonary embolus 


Answer & Comments 


Answer: 1- Restrictive cardiomyopathy 


2- PRN actrapid to keep the BMs controlled 

3- Iv sliding scale insulin 

4- Commence metformin 

5- Ignore the BMs and focus on his cardiac 
side 


Answer & Comments 

Answer: 3- Iv sliding scale insulin 

The DIGAMI study compared "conventional" 
anti-diabetic therapy to intensive insulin 
therapy consisting of acute insulin infusion 
during the early hours of Ml and thrice-daily 
subcutaneous insulin injection for the 
remainder of the hospital stay and a minimum 
of 3 months thereafter. For patients with 
insulin and better glycaemic control, mortality 
of patients were decreased at one year. 


[ Q: 1288 ] MRCPass - Cardiology 

A 55 year old patient presented with 
breathlessness and ankle oedema. The blood 
pressure is 135/80 mmHg. On examination, 
her JVP rises with inspiration. She has a soft 
systolic murmur and a third heart sound. 

Blood tests reveal a Hb 10.5 g/dl, WCC 7.5 x 
10 9 /L, Platelets 150 x 10 9 /L, sodium 136 
mmol/I, potassium 3.5 mmol/I, creatinine 140 
pmol/l, urea 6 pmol/l. 

ECG shows poor R wave progression. An 
echocardiogram shows no pericardial effusion, 
the ventricles are stiff and systolic function is 
mildly impaired. 

Which of the following is the likely diagnosis? 



In this scenario, the symptoms can be caused 
by any form of cardiomyopathy. The rise in 
JVP with inspiration suggests either 
constrictive or restrictive cardiomyopathy. 

Echocardiography showing no pericardial 
effusion and stiffness suggests restrictive 
rather than constrictive cardiomyopathy. The 
transmitral dopplers on the echo may show 
E/A wave reversal and high velocities which 
may suggest restrictive picture. 

This may be due to infiltration due to 
haemochromatosis, endomyocardial fibrosis, 
sarcoidosis, myeloma, lymphoma or 
connective tissue disease. 



[ Q: 1289 ] MRCPass - Cardiology 

A 40 year old patient had a mitral 
valve replacement for mitral stenosis 1 month 
ago. She presents with fevers, lethargy and 
rigor. 


Her blood tests reveal Hb 9.5 g/dl, WCC 13 x 
10 9 /L, platelets 500 x 10 9 /L, sodium 136 
mmol/I, potassium 4.2 mmol/l, ESR 90 mm/hr, 
CRP 180 mg/I. 


She also has 2 splinter haemorrhages and 2+ 
of blood on urine dipstick. 3 sets of blood 
cultures are taken. 


Which of the following organisms is most likely 
to be grown in the blood cultures? 

1- E coli 


2- Listeria monocytogenes 

3- (3 haemolytic Group A streptococcus 


Dr. Khalid Yusuf El-Zohry - Sohag Teaching Hospital (01118391123) 

Ref MRCPass OE OE 2012 PasTest 2009 PassMedicine 2009 PasTest Exam ReviseMRCP 



























El-zohry MRCP Questions Bank (Port 1) - 2013 


(For my personal use) 


4- Klebsiella 

5- Staphylococcus epidermidis 


Answer & Comments 

Answer: 5- Staphylococcus epidermidis 

In a patient with prosthetic valve, causes can 
be divided into early (within 60 days of 
surgery) and late stages. 

Early prosthetic valve endocarditis is usually 
the result of perioperative contamination. 
Causative organisms include Staphylococcus 
epidermidis (30%), Staphylococcus aureus 
(20%), and gram-negative aerobes (20%). 


The two best options are carvedilol and 
spironolactone. Both B blocker trials (CIBIS II, 
Merit HF, Copernicus) and spironolactone 
trials (RALES) have shown symptomatic 
improvement and decreased mortality. 

In this patient with little signs of fluid 
overload, a beta blocker can be started first, 
and then spironolactone added as well. 

[ Q: 1291 ] MRCPass - Cardiology 

A 55 year old man has progressive 
shortness of breath and ankle oedema. He has 
come from Africa originally and gives a history 
of probably tuberculosis many years ago. 



Late prosthetic valve endocarditis is usually 
the result of bacteremia from dental or 
genitourinary sources, Gl surgery, or 
intravenous drug abuse. The causative 
organisms are similar to those causing native 
valve endocarditis. These include 
Streptococcus viridans (30%), S epidermidis 
(30%) and S aureus (12%). 

^ [ Q: 1290 ] MRCPass - Cardiology 

/ ---- “ 

# A 65 year old lady has ischaemic 
cardiomyopathy and symptoms of 
breathlessness walking up one flight of steps. 
Her breath sounds are clear. Chest X ray 
shows cardiomegaly and clear lung fields. She 
is currently on frusemide 40mg bd and 
perindopril 4 mg at night. 

Whot medication should be added? 

1- Spironolactone 

2- Carvedilol 

3- Digoxin 

4- Amiodarone 


On examination, he has a blood pressure of 
105/55 mmHg, raised JVP +6 cm with a rapid y 
descent and significant ankle oedema. 

What is the likely diagnosis? 

1- Cardiac tamponade 

2- Aortic regurgitation 

3- Restrictive cardiomyopathy 

4- Pericardial constriction 

5- Left ventricular impairment causing heart 
failure 

Answer & Comments 

Answer: 4- Pericardial constriction 

The signs of pericardial constriction / 
constrictive pericarditis are rapid y descent, 
raised JVP and Kussmaul's sign. 

The y descent is often blunted in cardiac 
tamponade. A further clue is the history of 
tuberculosis which predisposes to constrictive 
pericarditis. 


5- Diltiazem 


Answer & Comments 


Answer: 2- Carvedilol 


[ Q: 1292 ] MRCPass - Cardiology 

A 75 year old man has become 
progressively more breathless. On 
examination he has a displaced cardiac apex 
and a third heart sound. Chest XR confirms 
cardiomegaly. An echocardiogram shows Left 



Dr. Khalid Yusuf El-Zohry - Sohag Teaching Hospital (01118391123) 

Ref MRCPass OE OE 2012 PasTest 2009 PassMedicine 2009 PasTest Exam ReviseMRCP 



530 




























El-zohry MRCP Questions Bank (Port 1) - 2013 


(For my personal use) 


ventricle size of 6.5 cm (<6cm) and LV ejection 
fraction of 25%.He is on Frusemide, 
Perindopril and spironolactone. 

Which of the following drugs should be added 
to his current therapy? 

1- Diltiazem 

2- Verapamil 

3- Atorvastatin 

4- Ezetemide 

5- Bisoprolol 


Answer & Comments 

Answer: 5- Bisoprolol 

The CIBIS trial showed that bisoprolol showed 
improvement in functional status and reduced 
hospitalisation. Mortality however, was not 
decreased on bisoprolol in this trial. There are 
other trials which have shown a small 
mortality benefit for beta blockers in heart 
failure. Other B blockers which can be used 
are metoprolol (MERIT HF study) and 
carvedilol. 



[ Q: 1293 ] MRCPass - Cardiology 

A 50 year old man has suffered an 
uncomplicated myocardial infarct 3 days ago 
and at the moment he has no problems. He is 
a tourist and wishes to travel back home as 
soon as possible. 

How soon after an uncomplicated myocardial 
infarct may a patient travel by flight safely? 

1- 3 days 

2- 14 days 

3- 4 weeks 

4- 2 months 

5- 3 months 


Answer & Comments 


Answer: 2-14 days 


A patient should be fit to fly 14 days after an 
uncomplicated myocardial infarct, after 
normal activities have been resumed. For 
complicated Mis, the patient has to w ait until 
symptoms stabilise. Coronary artery bypass 
graft and other chest surgery - recommended 
wait about 2 weeks so that any air introduced 
into chest will have become absorbed. 


[ Q: 1294 ] MRCPass - Cardiology 

A 65 year old man has an angioplasty 
to the left anterior descending artery. 



In considering medical therapy , which of the 
following reduces restenosis rates following 
angioplasty? 


1- Angiotensin converting enzyme inhibitors 


2- Abciximab 


3- Beta blockers 


4- Low-molecular weight heparin 

5- Pravastatin 


Answer & Comments 

Answer: 2- Abciximab 

There is some data to suggest that the use of 
the anti-platelet glycoprotein llb/llla receptor 
blocker (abciximab) may reduce the risk or re¬ 
stenosis in both diabetic and non-diabetic 
patients. 


[ Q: 1295 ] MRCPass - Cardiology 

A 50 year old man presents with 
palpitations. His ECG shows AV dissociation. 

Which of the following conditions is most likely 
to cause AV dissociation on an ECG? 

1- 1st degree AV block 

2- Atrial flutter 

3- Mobitz type II block 

4- Wenkebach 

5- Complete heart block 



Dr. Khalid Yusuf El-Zohry - Sohag Teaching Hospital (01118391123) 

Ref MRCPass OE OE 2012 PasTest 2009 PassMedicine 2009 PasTest Exam 




ReviseMRCP 

531 




























El-zohry MRCP Questions Bank (Port 1) - 2013 


(For my personal use) 


Answer & Comments 


Answer: 5- Complete heart block 


Answer & Comments 


Answer: 2- Troponin T 


Complete (3rd degree) heart block and 
ventricular tachycardia are most likely to 
cause AV dissociation on an ECG. 



[ Q: 1296 ] MRCPass - Cardiology 

A 60 year old man has palpitations. 
An admission ECG shows QTc prolongation of 
0.48 s. 

Which of the following conditions might be 
associated? 

1- Digitalis 

2- Hyponatraemia 

3- Hyperthermia 

4- Hypocalcaemia 

5- Hyperkalaemia 


Answer & Comments 

Answer: 4- Hypocalcaemia 

Hypokalaemia, hypocalcaemia , 

hypomagnesaemia and hypothermia can 
cause prolonged QT interval on the ECG. 
Prolonged QT is classified as a QTc interval 
(corrected) of more than 0.44 seconds. 


[ Q: 1297 ] MRCPass - Cardiology 

A 45 year old man has chest pain 
walking up hills. 

Which of the following investigations is the 
best for risk stratification of angina? 

1- ECG 

2- Troponin T 

3- Echocardiogram 

4- Coronary angiogram 

5- Thallium myocardial perfusion scan 



The high risk factors are positive troponin and 
dynamic ST depression on ECG with angina. 

Those in the high risk group should be 
referred for urgent coronary angiography. 


[ Q: 1298 ] MRCPass - Cardiology 

A 65 year old man has palpitations. 
He has a broad complex tachycardia on the 
ECG. In a broad complex tachycardia. 

Which of the following would be the strongest 
indication towards a diagnosis of VT? 

1- Discordant QRS complexes in the chest 
leads 

2- Extreme right axis deviation 

3- Hemodynamic instability 

4- Trifascicular block on ECG 

5- Cannon a waves 



Answer & Comments 

Answer: 5- Cannon a waves 

Any evidence of AV dissociation such as 
cannon a waves effectively rules out a 
supraventricular tachycardia. Features 
suggestive of VT, rather than SVT with bundle 
branch block are: 

■ cannon a waves on JVP 

■ fusion and/or capture beats 

■ Left axis deviation 

QRS duration > 140 msec 
concordance of the QRS complexes in 
the chest leads 

history of ischaemic heart disease 


Dr. Khalid Yusuf El-Zohry - Sohag Teaching Hospital (01118391123) 

Ref MRCPass OE OE 2012 PasTest 2009 PassMedicine 2009 PasTest Exam ReviseMRCP 



532 


































El-zohry MRCP Questions Bank (Port 1) - 2013 


(For my personal use) 


[ Q: 1299 ] MRCPass - Cardiology 

A 20 year old university student 
complains of palpitations which last 10 
minutes and are irregular. He has had three 
episodes over the past year and has come to 
outpatients to be assessed. He is aware of the 
palpitations but does not feel unwell. His ECG 
shows delta waves. 

What should be the next course of action? 

1- Discharge and no further investigation 

2- Routine referral to a cardiologist 

3- He is high risk and needs admission into 
hospital 

4- Routine DC cardioversion 

5- Anticoagulation 



Answer & Comments 

Answer: 2- Routine referral to a cardiologist 

He is in sinus rhythm at the moment. The 
delta waves and short PR interval suggests 
Wolff Parkinson White syndrome. He needs a 
referral to a cardiologist to discuss ablation of 
accessory pathway, and should have a 24 hour 
tape and echocardiogram. 





2- Aspirin 

3- Warfarin 

4- DC cardioversion 

5- Atenolol 


Answer & Comments 

Answer: 3- Warfarin 

The patient's heart rate is not fast. She scores 
2 on the CHADS -65 score (she is above age 65 
and she has hypertension) there is a 
significant benefit in formal anticoagulation 
with w arfarin in view of the risk of 
thromboembolism 

^ [ Q: 1301 ] MRCPass - Cardiology 

0 A 30 year old patient complains of 
dizzy spells. Upon admission, he has an ECG 
which shows a PR interval of 0.6s and QT 
interval of 0.5s. 

Which one of the following medications could 
be responsible? 

1- Aspirin 

2- Amoxycillin 

3- Terfenadine 

4- Digoxin 

5- Acetazolamide 


Answer & Comments 


Answer: 3- Terfenadine 


Short PR interval and delta waves 


[ Q: 1300 ] MRCPass - Cardiology 

A 75 lady was found to have Atrial 
Fibrillation on her ECG. She has a history of 
hypertension. The rate was 85 min and BP was 
140/85 mmHg. 

What management should be considered 
next? 

1- Digoxin 



A QT interval of >0.45 is prolonged. Common 
drugs are tricyclic antidepressants (eg. 
amitryptiline), quinidine, erythromycin, 
amiodarone, phenothiazines 

(chlorpropramide), antihistamines 

(terfenadine) and grapefruit juice. 



[ Q: 1302 ] MRCPass - Cardiology 

A 65 year old man with diabetes 
mellitus has a blood pressure of 190/90 
mmHg. Clinical examination was normal. 


Dr. Khalid Yusuf El-Zohry - Sohag Teaching Hospital (01118391123) 

Ref MRCPass OE OE 2012 PasTest 2009 PassMedicine 2009 PasTest Exam ReviseMRCP 










































El-zohry MRCP Questions Bank (Port 1) - 2013 


(For my personal use) 


An ECG reveals evidence of left ventricular 
hypertrophy. 

Which one of the following drugs appropriate 
treatment for hypertension? 

1- Sodium nitroprusside 

2- Labetalol 

3- Atenolol 

4- Doxazosin 

5- Bendrofluazide 


Answer & Comments 

Answer: 5- Bendrofluazide 

The British Hypertensive Society guidelines 
would suggest either a Diuretic or Calcium 
channel blocker [C or D for older or black 
patient]. Although ACE inhibitor is 
recommended for diabetic patients, this is not 
among the options, hence the best choice is 
the diuretic (D). 

[ Q: 1303 ] MRCPass - Cardiology 

A 40 year old lady noticed transient 
loss of vision and presented for assessment. 
She has had fevers for 3 months, 
breathlessness and orthopnoea. Her CRP and 
ESR are elevated. On examination, she is 
clubbed. She has a loud first heart sound and a 
mid diastolic murmur. 

What is the diagnosis? 

1- Mitral stenosis 

2- ASD 

3- Left ventricular aneurysm 

4- Left atrial appendage thrombus 

5- Atrial myxoma 



Answer & Comments 

Answer: 5- Atrial myxoma 

An atrial myxoma can present with fevers, 
raised inflammatory markers and 
breathlessness worst lying down. There may 


be embolic causes of CVA or TIA. The murmur 
could be a diastolic 'plop 1 or there may be 
clinical signs similar to mitral stenosis. 



Atrial Myxoma 


[ Q: 1304 ] MRCPass - Cardiology 

A 35 year old woman presented with 
a history of intermittent light-headedness. 
Clinical examination and 12-lead ECG were 
normal. 

Which of the following, if present on a 24 hour 
Ho Iter ECG tracing , would be the most 
clinically important? 

1- Atrial premature beats 

2- Profound sleep-associated bradycardia 

3- Supraventricular tachycardia 

4- Transient Mobitz type 1 atrioventricular 
block 

5- Ventricular premature beats 



Answer & Comments 

Answer: 3- Supraventricular tachycardia 

Mobitz type 1 is Wenkebach and does not 
usually cause symptoms. The most likely 
arrhythmia which needs treating e.g. with a 
beta blocker, is SVT. 


Dr. Khalid Yusuf El-Zohry - Sohag Teaching Hospital (01118391123) 

Ref MRCPass OE OE 2012 PasTest 2009 PassMedicine 2009 PasTest Exam ReviseMRCP 



534 



























El-zohry MRCP Questions Bank (Part 1) - 2013 


(For my personal use) 


1 


; j/ J; b |/J;- / 

wfff 1^44444 



iu 


r'rTTTTt^^rrrrr| 






^TTTTTTTTTTTTTTT^ 

1 


SVT 


Which of the following is the most likely 
diagnosis? 

1- Myocardial infarction 

2- Pericarditis 

3- Myocarditis 

4- Pulmonary emboli 

5- Pericardial effusion 


\fi 

[ Q: 1305 ] MRCPass - Cardiology 

Answer & Comments 

• ll 

A 50 year old woman with previous 

Answer: 3- Mvocarditis 


rheumatic heart disease has been lost to 
follow up for some time. She now has 
symptoms of breathlessness. On examination, 
she has a tapping apex beat, diastolic murmur 
and a large V wave in the JVP. 

What is the likely diagnosis? 

1- Isolated mitral stenosis 

2- Mixed mitral valve disease 

3- Mitral stenosis and tricuspid regurgitation 

4- Cardiac tamponade 

5- Aortic regurgitation 

Answer & Comments 

Answer: 3- Mitral stenosis and tricuspid 
regurgitation 

Rheumatic fever has predisposed the patient 
tow ards mitral stenosis, which would 
manifest as signs of a low rumbling diastolic 
murmur and a tapping apex beat. The large V 
wave in the JVP suggests tricuspid 
regurgitation which is secondary to pulmonary 
hypertension (also may have a loud second 
heart sound and a right ventricular heave). 


( 


[ Q: 1306 ] MRCPass - Cardiology 

A 33 year old man presents with 
chest pains and fatigue for several days. 

His observations show a temperature of 38°C, 
pulse 100 bpm, BP 100/70 mmHg. ECG shows 
T wave inversion in the anterior and inferior 
leads. The troponin I is 3 ng/mL (<0.04). 



Pyrexia, chest pain (pleuritic), raised troponin, 
T wave changes on the ECG would be 
suggestive of myocarditis. 

Pericarditis usually causes saddle shaped ST 
elevation on ECG. Coxsackie B is the 
commonest cause. Other causes are HIV, 
diphtheria, Chagas disease, Lyme disease, SLE 
and arsenic poisoning. 

[ Q: 1307 ] MRCPass - Cardiology 

A 20 year old lady is short in stature 
for her age. She has a webbed neck and 
oligomenorrhoea. On physical examination, 
she a continuous murmur heard over both 
front of the chest as well as her back. A chest 
radiograph reveals a prominent left heart 
border and rib notching. 

Which of the following lesions best explains 
these findings? 

1- Thoracic aortic aneurysm 

2- Single large atrioventricular valve 

3- Supravalvular narrowing in aortic root 

4- Coarctation of the aorta 

5- Shortening and thickening of chordae 
tendineae of mitral valve 

Answer & Comments 

Answer : 4- Coarctation of the aorta 

Coarctation of the aorta is the most likely 
diagnosis in this patient with Turner's 


Dr. Khalid Yusuf El-Zohry - Sohag Teaching Hospital (01118391123) 

Ref MRCPass OE OE 2012 PasTest 2009 PassMedicine 2009 PasTest Exam 




ReviseMRCP 

535 

































El-zohry MRCP Questions Bank (Port 1) - 2013 


(For my personal use) 


syndrome. Rib notching on the chest X ray is a 
big clue. The underlying abnormality is a 
deformity of the aortic media that causes 
eccentric narrow ing of the lumen. 



Rib Notching in Coarctation of the aorta (ribs 
4-8 bilaterally). This occurs due to collateral 

vessel formation. 


[ Q: 1308 ] MRCPass - Cardiology 

A 40 year old man presented 
because he is concerned about his family 
history of hypertrophic obstructive 
cardiomyopathy. His brother has recently 
deceased from due to the condition. 

What investigation should be offered? 

1- Transthoracic echocardiogram 

2- Transoesophageal echocardiogram 

3- Electrophysiological study 

4- Coronary angiogram 

5- Cardiac thallium scan 



echocardiogram, and beyond that genetic 
testing may be helpful. 



[ Q: 1309 ] MRCPass - Cardiology 

A 42 year woman who has received 
radioactive iodine many years ago has routine 
blood tests. She does not have any symptoms 
suggestive of angina. 


Her results reveal: 


Free Thyroxine 10.2 pmol/l (9.8-23) 
TSH 13 mU/l (0.5-4.5 mU/l) 


Total cholesterol 7.8 mmol/l (<5 mmol/I) 

Plasma triglycerides 2.5 mmol/l (<2 mmol/l) 

What is the appropriate treatment for the 
elevated lipid status? 

1- Diet control 

2- Simvastatin 

3- Thyroxine replacement 

4- Cholestyramine 

5- Clofibrate 


Answer & Comments 

Answer: 3- Thyroxine replacement 

Medical conditions associated with 
hypothyroidism include anemia, dilutional 
hyponatremia, and hyperlipidemia. Subclinical 
hypothyroidism (as demonstrated in this case) 
is usually indicated by a serum TSH level betw 
een the upper limit of normal (about 5 mll/L) 
and 15 mLI/L; occasionally the level may be 
higher. The serum free T4 level is, by 
definition, within the normal range. 
Treatment of choice is T4 replacement with 
levothyroxine sodium. 


Answer & Comments 

Answer: 1- Transthoracic echocardiogram 

Relatives of patients with HOCM should be 
offered screening especially in the context of a 
fatality. The best form is transthoracic 


[ Q: 1310 ] MRCPass - Cardiology 

A 65 year old man has symptoms of 
breathlessness and chest pains. He has cardiac 
risk factors of smoking and elevated 
cholesterol. On examination, he has a harsh 
systolic murmur and quiet second heart sound 




Dr. Khalid Yusuf El-Zohry - Sohag Teaching Hospital (01118391123) 

Ref MRCPass OE OE 2012 PasTest 2009 PassMedicine 2009 PasTest Exam 


ReviseMRCP 



























El-zohry MRCP Questions Bank (Port 1) - 2013 


(For my personal use) 


consistent with aortic stenosis. 
Echocardiography reveals a gradient of 75 
mmHg across the aortic valve. 

Which of the following is the most useful 
investigation? 

1- Transoesophageal echocardiography 

2- Coronary angiography 

3- Right heart cardiac catheter 

4- Cardiac electrophysiological study 

5- CT pulmonary angiogram 


Answer & Comments 

Answer: 2- Coronary angiography 

The diagnosis is severe aortic stenosis with 
significant clinical symptoms. The patient is 
likely to require surgical aortic valve 
replacement. Along with the risk factors, there 
is high risk of coronary artery disease. 
Coronary angiography should be the next 
investigation to investigate w hether coronary 
bypass grafts are necessary as well. 


[ Q: 1311 ] MRCPass - Cardiology 

A 65 year old man had a myocardial 
infarction 2 months ago. He has had a stent 
placed in the LAD artery following angioplasy. 
During a routine follow up Exercise Test he 
has a 15 beat run of nonsustained VT. The 
nonsustained VT occurred halfway through 
Stage 2. 

Whot is the next definitive investigation? 

1- Thallium scan 

2- Cardiac electrophysiological study 

3- Echocardiogram 



Post Ml VT is most commonly due ventricular 
scar tissue. This patient has had coronary 
revascularisation and still has significant runs 
of VT. Hence an electrophysiological study to 
attempt to stimulate VT is necessary. If the 
study demonstrated VT, then the patient 
should have an defibrillator (ICD) inserted 
[MADIT trial, AmJCardiol 1997;79:167]. 


[ Q: 1312 ] MRCPass - Cardiology 

A 55 year old man has palpitations 
and broad complex tachycardia on the ECG. 

In a broad complex tachycardia, which of the 
following suggests a diagnosis of supra¬ 
ventricular tachycardia (SVT)? 

1- Cannon a waves 

2- Termination with adenosine 

3- QRS duration >140msec 

4- Fusion beats 

5- Concordance of V leads 



Answer & Comments 

Answer: 2- Termination with adenosine 

Features which favour of VT include: 

evidence of AV dissociation (cannon a waves 
on JVP) 

fusion and/or capture beats 

left axis deviation (betw een 90 and 180 
degree) 

QRS duration > 140 msec 
concordance of V leads (monophasic QRS) 
Features which favor of SVT: 


4- 24 hour tape recording 

5- Repeat coronary angiography 

Answer & Comments 

Answer: 2- Cardiac electrophysiological study 


Long-short cycle sequence 

Slow ing or termination by increasing vagal 
tone or with adenosine 

Trileafed QRS in VI 

RP interval < 100 msec 


Dr. Khalid Yusuf El-Zohry - Sohag Teaching Hospital (01118391123) 

Ref MRCPass OE OE 2012 PasTest 2009 PassMedicine 2009 PasTest Exam 




ReviseMRCP 

537 



























El-zohry MRCP Questions Bank (Port 1) - 2013 


(For my personal use) 



[ Q: 1313 ] MRCPass - Cardiology 

A 70 year old man has a history of 
progressive tiredness and breathlessness. On 
examination there is pallor, his pulse rate is 65 
beats per minute, and is slow rising in nature. 
The JVP is not elevated. 


His apex beat is at the 5th left intercostal 
space mid-clavicular line and it is heaving in 
nature. There is a systolic thrill in the right 
sternal edge and there is a systolic murmur in 
the aortic area radiating to the neck. 

Investigations reveal: 

Hb 8.0 g/dl 

MCV 70 fL. 

Upon further investigation of this patient's 
anaemia the most useful test would be: 

1- Capsule endoscopy 

2- Barium swallow 

3- Colonoscopy 

4- CT pneumocolon 

5- Flexible sigmoidoscopy 


Answer & Comments 

Answer: 3- Colonoscopy 

The patient has features of aortic stenosis and 
iron deficiency anaemia. Aortic stenosis may 
be associated with angiodysplasia of the colon 
and the best test to elucidate this is 
colonoscopy. 



[ Q: 1314 ] MRCPass - Cardiology 

A 25 year man presents with 
episodes lightheadedness. He has no 
significant past medical history. Cardiac 
examination reveals no heart murmurs, chest 
X-ray and ECG are normal. A 24 hour tape is 
requested. 

Which arrhythmia might cause his symptoms? 

1- Atrial extrasystole 

2- Supraventricular tachycardia 


3- Wenkebach 

4- Ventricular extrasystole 

5- First degree heart block 


Answer & Comments 


Answer: 2- Supraventricular tachycardia 


Out of the following options, the most likely 
rhythm which may cause symptoms are 
supraventricular tachycardia. The other 
rhythms may cause palpitations but would be 
unusual to cause light headedness / 
presyncope. 






wJHjJI 


r 




^rrrT M rrf^rt^rrrr j/; - 






Supraventricular tachycardia 


[ Q: 1315 ] MRCPass - Cardiology 

A 50 year old man presents with 
broad complex tachycardia. He has a BP of 
100/70 and a pulse rate of 170. The duty 
medical registrar is considering administration 
of antiarrhythmics. 

Which one of the following medications is 
contraindicated in this patient? 

1- Magnesium 

2- Amiodarone 

3- Verapamil 

4- Procainamide 

5- Lignocaine 

Answer & Comments 

Answer: 3- Verapamil 

The patient is likely to have ventricular 
tachycardia in view of the very fast rate and 
broad complexes. All are useful agents which 



Dr. Khalid Yusuf El-Zohry - Sohag Teaching Hospital (01118391123) 

Ref MRCPass OE OE 2012 PasTest 2009 PassMedicine 2009 PasTest Exam ReviseMRCP 



538 
































El-zohry MRCP Questions Bank (Part 1) - 2013 


(For my personal use) 


can be used in conjunction cautiously for VT 
except for Verapamil. 

Verapamil is contraindicated in VT because it 
can cause the blood pressure to drop 
drastically due to negative ionotropic action. 

[ Q: 1316 ] MRCPass - Cardiology 

# A 65 year old man has a systolic 
murmur in the aortic area. 

In assessing him , which of the following is a 
clinical feature of severe aortic stenosis? 

1- Loud murmur 

2- Slow rising pulse 

3- Enlarged left atrium 

4- Palpable apex beat 

5- Early diastolic murmur 


2- ACE inhibitor, (3 blockers, aldosterone 
antagonist 

3- ACE inhibitor, loop diuretic 

4- (3 blockers, angiotensin II blocker 

5- ACE inhibitor, (3 blockers, statin 

Answer & Comments 

Answer: 2- ACE inhibitor, (3 blockers, 
aldosterone antagonist 

Of all the combinations, ACE inhibitor / 
angiotensin II blocker with (3 blockers and 
aldosterone antagonist (spironolactone, 
eplerenone) is the most appropriate given the 
fact that this patient has symptomatic heart 
failure. Trials have shown that these 
medications have a mortality and also 
symptomatic benefit. 


Answer & Comments 

[ Q: 1318 ] MRCPass - Cardiology 

mm 

Answer: 2- Slow rising pulse 

A 45 year old Caucasian man has 


The clinical signs consistent with severe aortic 
stenosis are: 

Slow rising pulse character 


hypertension. He has a blood pressure 
consistently above 160/90 mmHg. He does 
not have signs of malignant hypertension or 
renal failure. 


4th heart sound 

Soft aortic 2nd heart sound 


Which of the following is the most appropriate 
antihypertensive? 

1- Diltiazem 


Systolic thrill 


2- Methyldopa 


Extended length of murmur in systole 


3- Sodium nitroprusside 


[ Q: 1317 ] MRCPass - Cardiology 

A 60 year old man has had previous 
myocardial infarction. He has symptoms of 
breathlessness consistent with NYHA class III 
heart failure. Echocardiography shows an LV 
ejection fraction of 35%. 

Which of the following combinations of 
medication is most appropriate? 

1- ACE inhibitor, (3 blockers, angiotensin II 
blocker 



4- Lisinopril 

5- Bendrofluazide 


Answer & Comments 

Answer: 4- Lisinopril 

According to the British Hypertension Society 
guidelines, a patient who is non-black and 
under 55 years of age should be considered 
for an ACE inhibitor or Beta blocker. In this 
case the ACE-I lisinopril is the best choice. 


Dr. Khalid Yusuf El-Zohry - Sohag Teaching Hospital (01118391123) 

Ref MRCPass OE OE 2012 PasTest 2009 PassMedicine 2009 PasTest Exam ReviseMRCP 



539 





























El-zohry MRCP Questions Bank (Port 1) - 2013 


(For my personal use) 


[ Q: 1319 ] MRCPass - Cardiology 

A 32 year old lady complains of 
periodic episodes of palpitations and light¬ 
headedness for about 20-30 minutes. It 
occurred once a week for the last few weeks. 
An ECG showed normal sinus rhythm. 

What is the investigation of choice? 

1- 24 hr holter monitoring 

2- Echo 

3- Exercise tests 

4- Patient controlled Loop recorder 

5- Electrophysiological study 



Answer & Comments 

Answer: 4- Patient controlled Loop recorder 

The frequency of the symptoms is low , hence 
a 24 hour tape may not capture an 
arrhythmia. A patient controlled loop recorder 
allows recording of rhythms around the time 
of the event, hence a much higher likelihood 
of recording an abnormal rhythm. 



[ Q: 1320 ] MRCPass - Cardiology 

A 61 year old man with a previous 
history of myocardial infarction, presents with 
recurrent loss of consciousness. On 
examination there were signs of left 
ventricular aneurysm. BP was 110/70 mmHg 
and pulse rate was 90/min. CT head was 
normal. 


What is the cause of loss of consciousness? 

1- Vasovagal syncope 

2- Pulmonary embolism 

3- Stroke 


4- Ventricular tachycardia 

5- Hypersensitive carotid syndrome 


Answer & Comments 


Answer: 4- Ventricular tachycardia 


A patient with ischaemic heart disease and 
ventricular aneurysm is at high risk of 
developing ventricular arrhythmias. This can 
certainly lead to the loss of consciousness. 



[ Q: 1321 ] MRCPass - Cardiology 

A 50 year old man has reversed 
splitting of the second heart sound on 
examination. 


Which of the following conditions is this seen 
in? 


1- Atrial septal defect 

2- Pulmonary regurgitatoin 

3- Mild aortic stenosis 

4- Left bundle branch block 

5- Ventricular septal defect 


Answer & Comments 

Answer: 4- Left bundle branch block 

Reversed splitting occurs with reversal of the 
normal A2 - P2 pattern. Thus A2 may be 
delayed as with severe aortic stenosis, and left 
bundle branch block (LBBB). P2 may also be 
early with Wolff-Parkinson-White type B and 
Persistent ductus arteriosus. Atrial septal 
defects show wide fixed splitting, and right 
bundle branch block (RBBB) has wide splitting. 


[ Q: 1322 ] MRCPass - Cardiology 

With regard to the conducting 
system of the heart , where are the Purkinje 
fibres situated? 

1- At the superior portion of the myocardium 

2- In the middle of the myocardium 

3- Subendocardial 

4- Transmural 

5- Subepicardial 



Answer & Comments 


Answer: 3- Subendocardial 


Dr. Khalid Yusuf El-Zohry - Sohag Teaching Hospital (01118391123) 

Ref MRCPass OE OE 2012 PasTest 2009 PassMedicine 2009 PasTest Exam ReviseMRCP 



540 
































El-zohry MRCP Questions Bank (Port 1) - 2013 


(For my personal use) 


The Purkinje fibres run in a subendocardial 
position. This results in depolarisation of the 
heart from the endocardium to the 
epicardium. 

[ Q: 1323 ] MRCPass - Cardiology 

A 40 year old lady has a pulmonary 
systolic murmur which is louder on 
inspiration. She also has a right ventricular 
heave. The second heart sound is wide and 
fixed with splitting. 

What is the most likely diagnosis? 

1- Tricuspid atresia 

2- Patent ductus arteriosus 

3- Atrial septal defect 

4- Aortic stenosis 

5- Mitral stenosis 




[ Q: 1324 ] MRCPass - Cardiology 

A 25 year old man presents with a 
collapse w hilst running for the bus. He has no 
previous known past medical history. An ECG 
done on admission to the hospital fulfils the 
criteria for LVH. On examination, he has a 
jerky pulse and prominent apex beat. There is 
a systolic murmur heard in the aortic area. 


What is the likely diagnosis? 


1- Dilated cardiomyopathy 

2- Hypertrophic cardiomyopathy 

3- Ischaemic cardiomyopathy 


4- Restrictive cardiomyopathy 

5- Haemochromatosis 


Answer & Comments 


Answer: 2- Hypertrophic cardiomyopathy 


Answer & Comments 

Answer: 3- Atrial septal defect 

With atrial septal defect (ASD), a Primum 
defect causes RBBB and LAD, w hilst 
Secundum causes RBBB and RAD on the ECG. 
Secundum ASD is associated with Holt-Oram 
syndrome (tri-phalangeal thumb and radial 
abnormalities). A systolic murmur is heard in 
the pulmonary area because of increased 
pulmonary valve flow due to pulmonary 
hypertension. Similarly, a left parasternal 
heave is present due to RVH. 



i 

F ntargad Right Ventode 
SECUNDUM ATRIAL SEPTAJL DEFECT 


Hypertrophic cardiomyopathy which is 
familial, can present with presyncopal 
episodes as described above. Over a hundred 
different mutations in at least nine different 
genes, all encoding sarcomeric polypeptides, 
have been associated with hypertrophic 
cardiomyopathy (HCM). LVH changes with 
strain pattern and prominent q waves are 
common on the ECG. A prominent, jerky pulse 
is a clue. 


1 






■ 


























1 












































i A 








































I 




1 













_ 



if . 
































1 




1 






1 





| 



♦ 


1 



































_J 












1 



! 1 
L 



I - 






L % 




— i 


































j r 






r 






, i 





















( / 







71 

































F I f 










—A/ 











1 

























‘ i 



J 












i • 

















T J 









A 





















V—— 




























































1 









| | 


t 

1 






JU 






1 




j 



i 

\\ 

; 

| 

\ f 

1 




rv 


























T/i 





































































1 i 













-- t, 1 






LVH in Hypertrophic cardiomyopathy 


[ Q: 1325 ] MRCPass - Cardiology 

A 50 year old man who had an 
angioplasty asks about Clopidogrel. 



What is Clopidogrel's mode of action? 


Dr. Khalid Yusuf El-Zohry - Sohag Teaching Hospital (01118391123) 

Ref MRCPass OE OE 2012 PasTest 2009 PassMedicine 2009 PasTest Exam 




ReviseMRCP 

541 






































El-zohry MRCP Questions Bank (Port 1) - 2013 


(For my personal use) 


1- ADP receptor antagonist 

2- Affects APTT 

3- Inhibits cyclooxygenase 

4- Hydroxymethyl Coenzyme A inhibitor 

5- Glycoprotein 11 b/Ilia inhibitor 


Answer & Comments 

Answer: 1- ADP receptor antagonist 

Clopidogrel prevents platelet aggregation 
through antagonism of the ADP receptor on 
platelets. 


[ Q: 1326 ] MRCPass - Cardiology 

A 55 year old man with 
breathlessness presents to A&E. On 
examination, he has an inspiratory systolic fall 
in arterial pressure of lOmmHg. 

Which one of the following is most likely to be 
ossocioted? 

1- Myocardial infarction 

2- Pulmonary stenosis 

3- Cardiac tamponade 

4- Pneumonia 

5- Myocarditis 



Answer & Comments 

Answer: 3- Cardiac tamponade 

Pulsus paradoxus is most commonly related to 
cardiac tamponade. The y descent (diastolic) 
phase is absent in tamponade. Pulsus 
paradoxus is defined as an inspiratory systolic 
fall in arterial pressure of lOmmHg. It not only 
occurs in cardiac tamponade, but also in 
massive PE, severe COPD and 
hypotension/shock. 


What is the ECG likely to show now ? 

1- Prominent p waves 

2- Delta waves 

3- ST depression 

4- Prolonged PR interval 

5- LBBB 


Answer & Comments 

Answer: 5- LBBB 

Permanent pacing can be single chamber 
atrial (e.g. AAI), single chamber ventricular 
(e.g. VVI) or dual chamber (e.g. DDD). A VVI 
pacemaker would mean that the pacing lead is 
placed in the right ventricle, causing a LBBB 
pattern. 


[ Q: 1328 ] MRCPass - Cardiology 

A 27 year old man presents with 
lethargy, poor exercise tolerance and weight 
loss. On examination he looks very slim and 
has a pulse rate of 120 beats per minute 
regular. The pulse volume decreases during 
inspiration. 

His JVP is elevated 5 cms and the level 
increases during inspiration. On auscultation 
of the heart sounds an early diastolic sound is 
audible at the left sternal edge. On 
examination of the abdomen there is 4 
fingerbreadths of hepatomegaly and ascites. 

What is the diagnosis? 

1- Mitral valve prolapse 

2- Mitral stenosis 

3- Constrictive pericarditis 

4- Myocardial infarction 

5- Aortic regurgitation 



[ Q: 1327 ] MRCPass - Cardiology 

A 70 year old man with complete 
heart block has had a VVI permanent 
pacemaker implanted a week ago. 



Answer & Comments 

Answer: 3- Constrictive pericarditis 

The patient has the clinical features of 
constrictive pericarditis. In constrictive 


Dr. Khalid Yusuf El-Zohry - Sohag Teaching Hospital (01118391123) 

Ref MRCPass OE OE 2012 PasTest 2009 PassMedicine 2009 PasTest Exam ReviseMRCP 

































El-zohry MRCP Questions Bank (Part 1) - 2013 


(For my personal use) 


pericarditis, an early diastolic sound is heard 
(pericardial knock). This sound is due to 
turbulence caused by the arrest of rapid 
ventricular filling into a non-distensible 
pericardial sac. 


[ Q: 1329 ] MRCPass - Cardiology 

A 70 year old man has a nine month 
history of worsening breathlessness. His blood 
pressure is 120/90 mmHg, O 2 sats 89% on air, 
temperature is 36.5°C, there is a systolic 
murmur heard throughout the precordium. 

Blood tests show 

Hb 14 g/dl 

WCC 5 x 1071- 

Platelets 280 x 1071- 

urea 7 pmol/l 

creatinine 75 pmol/l 

sodium 142 mmol/l 

potassium 4.2 mmol/l 

cholesterol 6.2 mmol/l 

triglyceride 2.1 mmol/l 

Which of the following is most likely? 

1- Dilated cardiomyopathy 

2- Aortic regurgitation 

3- Aortic stenosis 

4- Pericardial effusion 

5- Coarctation of the aorta 



Answer & Comments 

Answer: 3- Aortic stenosis 

There is evidence of left sided heart failure 
clinically. The murmur throughout the 
praecordium and narrow pulse pressure 
suggest aortic stenosis. The patient is also 
predisposed to this by a high cholesterol. 

^ [ Q: 1330 ] MRCPass - Cardiology 

- 

A 50 year old man had a ventricular 


fibrillation cardiac arrest whilst at a railway 
station. He was successfully resuscitated by a 
passerby. Coronary angiography showed no 
stenotic lesions in the coronary arteries. 

Which is the next management step? 

1- Lifelong amiodarone 

2- Implantable cardiac defibrillator 

3- Procainamide 

4- Permanent pacemaker 

5- No further action necessary 

Answer & Comments 

Answer: 2- Implantable cardiac defibrillator 

A cardiac electrophysiological study would be 
necessary to help confirm w hether this 
patient has inducible ventricular tachycardia, 
and ablation of the pathways if so. However in 
a young patient with no coronary artery 
disease demonstrable to revascularise, ICD is 
necessary to treat a possible further cardiac 
arrest. 



iinptaftiitbic 

CaidioviHter 

DHibiillator 


Pacing LimiIs 


tdl Atrium 


Right 

Right 


Lfclt Vfntride 


^ [ Q: 1331 ] MRCPass - Cardiology 

fl - 

* A 25 year old biology student 
presents with palpitations and has the ECG 
(above) recorded. Adenosine is given at 6mg 
and 12mg intravenously. He felt nauseous for 
a few seconds and there is a rhythm change to 
sinus rhythm. 

How does adenosine work? 


Dr. Khalid Yusuf El-Zohry - Sohag Teaching Hospital (01118391123) 

Ref MRCPass OE OE 2012 PasTest 2009 PassMedicine 2009 PasTest Exam ReviseMRCP 































El-zohry MRCP Questions Bank (Port 1) - 2013 


(For my personal use) 


1- Decrease sensitivity of conduction bundle 
to electrolytes 

2- Altering Purkinje fiber response 

3- Blocking an accessory pathway 

4- Blocking AV node conduction 

5- QT prolongation 

Answer & Comments 

Answer: 4- Blocking AV node conduction 

Adenosine w orks by slow ing atrioventricular 
(AV) conduction, hence breaking the re-entry 
circuit which can be a AVNRT or AVRT in supra 
ventricular tachycardia. 


and systolic BP is 120 mmHg in the left arm, 
80 mmHg systolic in the right arm. 

Which one of the following is o feature of 
severity requiring immediate (surgical) action? 

1- Persistent vomiting 

2- Headache 

3- Pulmonary oedema on the CXR 

4- Hypertension 

5- Loud diastolic murmur in the aortic area 

Answer & Comments 

Answer: 5- Loud diastolic murmur in the aortic 
area 


[ Q: 1332 ] MRCPass - Cardiology 

A 10 year old patient has Down's 
syndrome. He has over the past year become 
progressively more breathless, is losing weight 
and feeling lethargic. He is also becoming blue 
when running with the dog during walks. 

Which of the following cardiac lesions might 
be responsible? 

1- Atrial septal defect 

2- Aortic regurgitation 

3- Tetralogy of fallot's 

4- Mitral stenosis 

5- Ventricular septal defect 



Answer & Comments 

Answer: 5- Ventricular septal defect 

An endocardial cushion defect (betw een the 
atrial and ventricular septum) can lead to ASD 
or VSD. With rapid deterioration a VSD is most 
likely to cause breathlessness and cyanosis. 


[ Q: 1333 ] MRCPass - Cardiology 

A 60 year old man presents with 
central crushing chest pain radiating to the 
back. He has a w eak pulse in the right arm 



In aortic dissection, hypertension should be 
treated with an infusion such as labetalol. A 
loud diastolic murmur in the aortic area 
suggests aortic regurgitation. Aortic 
regurgitation and pericardial effusion 
(haemopericardium) suggest dissection down 
to the aortic root. These features, or persisting 
chest pain suggest that the patient requires 
prompt surgery. 


[ Q: 1334 ] MRCPass - Cardiology 

A 66 year old man presents with 
tearing chest pain radiating to the back. The 
pain started 5 hours ago and is continuous. He 
has a past medical history of previous stroke, 
hypertension and osteoarthritis. His blood 
pressure is 180/95 mmHg. There is an early 
diastolic murmur in the aortic area and the 
chest X ray showed a widened mediastinum. 
An urgent CT of his chest with contrast shows 
a dissection flap. 

What is the best treatment at present? 

1- Iv amlodipine 

2- Iv labetalol 

3- Iv tirofibran 

4- Oral sodium nitroprusside 

5- Clopidogrel 



Dr. Khalid Yusuf El-Zohry - Sohag Teaching Hospital (01118391123) 

Ref MRCPass OE OE 2012 PasTest 2009 PassMedicine 2009 PasTest Exam ReviseMRCP 



544 





























El-zohry MRCP Questions Bank (Port 1) - 2013 


(For my personal use) 


Answer & Comments 


Answer: 2- Iv labetalol 


Answer & Comments 


Answer: 2- Anticoagulation with warfarin 


The history and CT scan findings confirm aortic 
dissection in this case. Blood pressure control 
is vital in the early management of aortic 
dissection, prior to considering cardiac 
surgery. The best initial treatments are iv 
labetalol or iv sodium nitroprusside. 



Aortic dissection - arrow points to the flap 


[ Q: 1335 ] MRCPass - Cardiology 

A 70 year old woman has a history of 
palpitations for 4 months. An ECG at that time 
showed atrial fibrillation. She was given 
digoxin, diuretics and aspirin. She now 
presents with two short-lived episodes of 
altered sensation in the left face, left arm and 
leg. She also had intermittent dysphasia. 
There is poor coordination of the left hand. 

An echocardiogram was normal as was a CT 
head scan. 



The patient has thrombotic events related to 
atrial fibrillation, hence formal anticoagulation 
is recommended. 

The Age and stroke gives her a CHADS-65 
score of 3. A CHADS-65 score of 2 or above 
suggests the need for warfarinisation. 


[ Q: 1336 ] MRCPass - Cardiology 

A 13 year boy had a syncopal 
episode followed by generalised jerking of all 
his limbs, whilst having dental filling. 

He was particularly nervous about the 
procedure. When he was assessed in casualty, 
his BP was 145/80, 0 2 sats were 99% and ECG 
showed normal sinus rhythm. 

Whot is the likely diagnosis? 

1- Generalised tonic clonic seizures 

2- Vasovagal syncope 

3- Carotid sinus hypersensitivity 

4- VF arrest 

5- Meningitis 



Answer & Comments 

Answer: 2- Vasovagal syncope 

Vasovagal syncope is not uncommon during a 
procedure which a patient is frightened of. 
There may be transient brain hypoxia which 
may caused generalised seizure like activity. 


Whot is the most appropriate next step in 
management? 

1- Carotid endarterectomy 

2- Anticoagulation with warfarin 

3- Clopidogrel 

4- Corticosteroid treatment 

5- No action 



[ Q: 1337 ] MRCPass - Cardiology 

A 60 year male diabetic presents to 
the clinic. He takes metformin 850 mg tds, 
bendrofluazide 2.5 mg and asprin 75 mg daily. 
He has had a previous myocardial infarction 
but is currentl asymptomatic. His body mass 
index was 33.5 kg/m * 1 2 3 4 5 , he has a pulse of 90 
beats per minute and a blood pressure of 


Dr. Khalid Yusuf El-Zohry - Sohag Teaching Hospital (01118391123) 

Ref MRCPass OE OE 2012 PasTest 2009 PassMedicine 2009 PasTest Exam ReviseMRCP 



545 
































El-zohry MRCP Questions Bank (Port 1) - 2013 


(For my personal use) 


160/9 mmHg. His cholesterol concentration is 
3.8 mmol/l (< 5.5). 

Whot is the most appropriate therapy for this 
patient? 

1- Ramipril 

2- Clopidogrel 

3- Nicorandil 

4- Orlistat 

5- Simvastatin 



ECG in WPW showing short PR interval and 
rapid upstroke in WPW 


Answer & Comments 

Answer: 1- Ramipril 

This patient has vascular disease and several 
vascular risk factors. An ACE inhibitor would 
reduce cardiovascular risk as suggested by the 
HOPE study. 


[ Q: 1338 ] MRCPass - Cardiology 

A 21 year-old woman has a history of 
palpitations and light headedness. ECG shows 
short PR interval and inferior Q waves. Her 
symptoms improve with atenolol 25 mg/day 
but she has had two short episodes of similar 
symptoms in the previous 24 hours. 

What is the long-term management of choice? 



[ Q: 1339 ] MRCPass - Cardiology 

A 55 year old man presented with 
chest pain, facial flushing and elevated jugular 
venous pressure but no leg oedema. He had 
been exposed to tuberculosis in childhood, 
had a strongly positive tuberculin test and had 
been followed up in the tuberculosis 
surveillance program with regular chest x-rays, 
but had never been diagnosed with 
tuberculosis. Chest x-ray on presentation 
showed calcified plaques and masses in the 
pericardium and mediastinum. 

Coronary angiography showed 70% stenosis in 
the left anterior descending artery. Cardiac 
catheterization showed equalisation of 
diastolic pressures in all four chambers, with a 
positive square root sign. 



1- Anticoagulation 


What is the likely diagnosis? 


2- Oral amiodarone 


1- Pericardial effusion 


3- Oral digoxin 

4- Increase the dose of atenolol 

5- Radiofrequency ablation 

Answer & Comments 

Answer: 5- Radiofrequency ablation 

The diagnosis is w olff Parkinson white 
syndrome and the patient has atrial 
tachycardias which are not controlled by 
medical therapy. She should be referred to a 
cardiac physiologist for radiofrequency 
ablation of the accessory pathway. 


2- Cardiac tamponade 

3- Constrictive pericarditis 

4- Sarcoidosis 

5- Angina 

Answer & Comments 

Answer: 3- Constrictive pericarditis 

The cardiac catheter data shows a pattern of 
ventricular diastolic pressure characteristic of 
constrictive pericarditis. Tuberculous 
pericarditis is seen in l%-2% of all cases of 
pulmonary tuberculosis and can lead to 


Dr. Khalid Yusuf El-Zohry - Sohag Teaching Hospital (01118391123) 

Ref MRCPass OE OE 2012 PasTest 2009 PassMedicine 2009 PasTest Exam ReviseMRCP 



546 




























El-zohry MRCP Questions Bank (Port 1) - 2013 


(For my personal use) 


constriction. Treatment is with corticosteroids 
and anti TB therapy. 

The atrial waveform manifests: 

-augmented "a" wave, reflecting enhanced 
atrial contraction into a stiff ventricle 

-a rapid "x" descent attributable to 
subsequent accelerated atrial relaxation 


systolic murmur heard at the base of the 
heart. 

The inheritance of this condition is most likely: 

1- Mitochondrial inheritance 

2- X-linked dominant 

3- X-linked recessive 

4- Autosomal recessive 


-a steep "y" descent reflecting rapid, 
resistance-free early diastolic filling. 

Right and left heart chamber filling pressures 
are typically increased and equalized (LVEDP = 
RVEDP), reflecting the common constraining 
effects of the pericardium. 



Traces showing equalisation of diastolic 
pressures in constrictive pericarditis 


[ Q: 1340 ] MRCPass - Cardiology 

An 30 year old man presents with a 
history of syncope related to exercise. There is 
a family history of sudden cardiac death. On 
examination, the carotid pulse is jerky, there is 
a double apical pulsation. On auscultation 
there is a fourth heart sound, an ejection 



5- Autosomal dominant 

Answer & Comments 

Answer: 5- Autosomal dominant 

Hypertrophic obstructive cardiomyopathy is 
most commonly of Autosomal dominant 
inheritance. It occurs in at least on in 1,000 to 
one in 500 of the general population. 

[ Q: 1341 ] MRCPass - Cardiology 

A 30 year old man is seen annually in 
the cardiac clinic. During auscultation of his 
heart there is wide fixed splitting of the 
second heart sound. 

In which of following conditions does this 
occur? 

1- Atrial septal defect 

2- Aortic regurgitation 

3- Constrictive pericarditis 

4- Ebstein's anomaly 

5- Right Bundle Branch Block 



Answer & Comments 

Answer: 1- Atrial septal defect 

Wide fixed splitting is seen in ASD. In RBBB 
there is wide splitting of S2 but it not fixed. 



[ Q: 1342 ] MRCPass - Cardiology 

A 65 year old lady has recently had a 
cholecystectomy 2 days ago. She is now very 
breathless, has central pleuritic chest pain and 
feels dizzy. She is only able to say a few words 



Dr. Kholid Yusuf El-Zohry - Sohog Teaching Hospital (01118391123) 

Ref MRCPass OE OE 2012 PasTest 2009 PassMedicine 2009 PasTest Exam ReviseMRCP 































































El-zohry MRCP Questions Bank (Port 1) - 2013 


(For my personal use) 


and looks pale. Examination reveals a sinus 
tachycardia and flow murmur across the aortic 
area. Her blood pressure is is 85 / 50 mmHg, 
0 2 saturations are 85% on 6 litres of 0 2 . ECG 
shows non specific T wave abnormalities. 

What should be the next management step? 

1- CT pulmonary angiogram 

2- Intravenous heparin 

3- Coronary angiogram 

4- Thrombolysis with tenecteplase 

5- Transthoracic echocardiography 


Answer & Comments 

Answer: 4- Thrombolysis with tenecteplase 

This patient has recently had surgery, and her 
presentation would be consistent with life 
threatening massive pulmonary embolus. She 
is medically too unstable to wait for a 
diagnostic test in radiology, thus thrombolysis 
should be done immediately. If this does not 
work then the patient may need ventilation, 
CT to confirm the diagnosis and surgical 
embolectomy. 


[ Q: 1343 ] MRCPass - Cardiology 

A 75 year old man is admitted with 
sudden onset shortness of breath, poor 
exercise tolerance and ankle oedema. 

Which of the following laboratory test would 
identify the reason for his presentation? 

1- Alpha 1 antitrypsin levels 

2- Serum electrophoresis 

3- Serum triglycerides 

4- Brain natriuretic peptide 

5- Urea and electrolytes 



Answer & Comments 

Answer: 4- Brain natriuretic peptide 

The clinical scenario fits with acute pulmonary 
oedema. Brain or B type natriuretic peptide is 


increasingly used as a marker of congestive 
heart failure. It has high specificity for the 
condition. It is released by ventricular 
cardiomyocytes due to stretch as part of the 
pathophysiology of cardiac failure. 



[ Q: 1344 ] MRCPass - Cardiology 

A 55 year old man complains of 
dyspnoea on exertion. He recently returned 
from Africa on a long holiday. He has distant 
heart sounds on auscultation of the chest. A 
chest radiograph reveals a thin rim of 
calcification surrounding the cardiac outline. 


Which of the following is likely? 


1- Myocarditis 

2- Silent myocardial infarction 


3- Subacute bacterial endocarditis 


4- Small pneumothorax 

5- Constrictive pericarditis 


Answer & Comments 

Answer: 5- Constrictive pericarditis 

The likely diagnosis is constrictive pericarditis. 
The is probably related to previous 
tuberculous infection because of the 
calcification. Acute pericarditis is less likely to 
cause calcification. 



Rim of calcification seen on the CXR 


Dr. Khalid Yusuf El-Zohry - Sohag Teaching Hospital (01118391123) 

Ref MRCPass OE OE 2012 PasTest 2009 PassMedicine 2009 PasTest Exam ReviseMRCP 



548 



























El-zohry MRCP Questions Bank (Port 1) - 2013 


(For my personal use) 


[ Q: 1345 ] MRCPass - Cardiology 

A 45 year old lady has previous 
history of rheumatic fever. She has a loud first 
heart sound and a diastolic murmur 
suggestive of mitral stenosis. 

Which of these is the best indicator of severe 
mitral stenosis? 

1- Increased pulse pressure 

2- Clubbing 

3- Loud second heart sound and right 
parasternal heave 

4- Tapping apex beat 

5- Rumbling mid diastolic murmur 



Answer & Comments 

Answer: 3- Loud second heart sound and right 
parasternal heave 


This patient isolated systolic hypertension 
(systolic BP >160 mmHg). Based upon studies 
such as the Systolic Hypertension in the 
Elderly Program (SHEP) the BHS guidelines 
suggest treatment with either Calcium 
antagonists or Diuretics (C or D). 


[ Q: 1347 ] MRCPass - Cardiology 

A 65 year old man experiences 
substernal chest pain upon exertion for 6 
months. An electrocardiogram shows T wave 
inversion in the anterolateral leads at rest. He 
has a total serum cholesterol of 8 mmol/l. On 
angiography, he has a 95% narrow ing of the 
left anterior descending artery. 

Which of following events is likely occur in this 
patient? 

1- Pulmonary embolism from a left ventricular 
mural thrombus 



A loud P2 and features of right heart strain is 
suggestive of pulmonary hypertension, hence 
severe mitral stenosis. 


[ Q: 1346 ] MRCPass - Cardiology 

A 80 year old woman is referred by 
her GP for high blood pressure. Over the last 
three months her pressure has been recorded 
at around 175/80 mmHg for 3 occasions. She 
has a body mass index of 26 kg/m * 1 2 3 4 5 and is a 
nonsmoker. 

Which of the following is the most appropriate 
treatment for her blood pressure? 

1- Calcium channel blocker 

2- Beta blocker 

3- Alpha Blocker 

4- Angiotensin Receptor Blocker 

5- Angiotensin Converting Enzyme (ACE) 
Inhibitor 



2- Pulmonary embolism from thrombosis in a 
internal jugular vein 

3- A systemic artery embolus from thrombosis 
in a the hepatic vein 

4- A systemic artery embolus from a left 
ventricular mural thrombus 

5- A systemic artery embolus from a left atrial 
appendage thrombus 


Answer & Comments 

Answer: 4- A systemic artery embolus from a 
left ventricular mural thrombus 

A significant stenosis of the left anterior 
descending artery may lead to infarction of 
anterior region of the left ventricle. Thrombus 
may form on an area of the dyskinetic 
ventricle (mural thrombus). Therefore he at 
risk of arterial embolus of thrombus from the 
LV. 


Answer & Comments 


Answer: 1- Calcium channel blocker 


[ Q: 1348 ] MRCPass - Cardiology 

A 60 year old man presents with 
sharp chest pain radiating to the epigastrium. 



Dr. Khalid Yusuf El-Zohry - Sohag Teaching Hospital (01118391123) 

Ref MRCPass OE OE 2012 PasTest 2009 PassMedicine 2009 PasTest Exam ReviseMRCP 



549 
































El-zohry MRCP Questions Bank (Port 1) - 2013 


(For my personal use) 


He has ST elevation in the anterior leads on his 
ECG and is thrombolysed. The chest pain 
persists despite thrombolysis. He develops a 
raised JVP up to the ear lobes and BP drops to 
95/60. There are signs of pulsus paradoxus. 

What is the likely diagnosis? 

1- Aortic dissection 

2- Pericarditis 

3- Pulmonary embolism 

4- Anterior Ml with myocardial rupture 

5- Patent ductus arteriosus 

Answer & Comments 

Answer: 1- Aortic dissection 

Aortic dissection can present with pain 
radiating to the epigastric region or back. If 
the dissection flap involves close to the aortic 
root, then the coronary flow can be affected, 
causing either inferior Ml in right coronary 
artery or anterior Ml in left anterior 
descending artery involvement. Cardiac 
tamponade can also be caused by dissection 
(blood flow ing into pericardial space) and 
leading to a raised JVP and pulsus paradoxus. 


In this patient the blood pressure is too high 
to safely administer thrombolysis. GTN should 
be given to lower the blood pressure in the 
first instance and to vasodilate the coronary 
arteries. 

^ [ Q: 1350 ] MRCPass - Cardiology 

# A 72 year old man is reviewed at 
medical outpatient clinic complaining of 
tiredness. He takes amiodarone, aspirin, 
atenolol and atorvastatin. His heart rate on 
examination is 85 bpm. Recent 24-hr ECG 
shows sinus rhythm throughout with 
occasional ventricular ectopics. 

Investigations reveal: 

Free T 4 32 nmol/I (9-22) 

TSH <0.02 mU/l (0.4-4) 

What is the best management strategy for this 
patient? 

1- Stop amiodarone only 

2- Stop amiodarone and start carbimazole 

3- Stop amiodarone and start prednisolone 

4- Check antithyroid antibodies 

5- Prednisolone only 


[ Q: 1349 ] MRCPass - Cardiology 

A 60 year old man was admitted with 
severe chest pain. On examination his Blood 
Pressure was 205/115mm Hg. 

ECG showed >2mm ST elevation in Leads V2-4. 
He was given morphine and aspirin. 

What is the next appropriate management? 

1- IvtPA 

2- Iv streptokinase 

3- Iv GTN 

4- Clopidogrel 

5- Tirofiban 



Answer & Comments 


Answer: 3- Iv GTN 


Answer & Comments 

Answer: 2- Stop amiodarone and start 
carbimazole 


This patient has probable amiodarone- 
induced hyperthyroidism. 


The amiodarone was used to maintain sinus 
rhythm in this patient who was admitted with 
ischaemic heart disease and atrial 
fibrillation/flutter that spontaneously settled. 
The amiodarone should be discontinued, and 
carbimazole started. When the patient is 
euthyroid, the dose of carbimazole may be 
reduced (amiodarone can stay in the system 
for several weeks). 



[ Q: 1351 ] MRCPass - Cardiology 


Dr. Khalid Yusuf El-Zohry - Sohag Teaching Hospital (01118391123) 

Ref MRCPass OE OE 2012 PasTest 2009 PassMedicine 2009 PasTest Exam ReviseMRCP 



550 
































El-zohry MRCP Questions Bank (Port 1) - 2013 


(For my personal use) 


A 60 year old man who was previously 
asymptomatic suddenly develops severe 
anterior chest pain radiating to the back. 
Within minutes, he becomes unconscious. He 
has a history of hypertension. 


1- Ramipril 

2- Frusemide 

3- Digoxin 

4- Diltiazem 


On examination, he has a blood pressure of 
120 / 60 mmHg in the right arm and 105 / 55 
mmHg in the left arm. An early diastolic 
murmur was audible in the lower left sternal 
edge. 


5- Spironolactone 

Answer & Comments 

Answer: 3- Digoxin 


Which of the following is the most likely 
diagnosis? 

1- Malignant hypertension 

2- Internal carotid artery dissection 

3- Acute myocardial infarction 

4- Aortic dissection 

5- Pulmonary embolus 


Answer & Comments 


Answer: 4- Aortic dissection 


Digoxin toxicity can occur especially with renal 
impairment. It typically causes nausea & 
vomiting. ST depression occurs along with 
bradycardia on the ECG. The patient may also 
get xanthopsia (yellow vision). 



Digoxin Effect 


The acute history of sharp pain radiating to 
the back is suggestive of aortic dissection. In 
addition, there is predisposing risk factor of 
hypertension and he collapses (suggesting 
that the dissection flap may have involved 
large vessels leading tow ards vascular 
insufficiency) and hence the neurological 
symptoms. 

When the flap dissects down to the aortic 
root, aortic regurgitation may also be found 
(early diastolic murmur). 


[ Q: 1352 ] MRCPass - Cardiology 

A 62 year old man has atrial 
fibrillation and was found to be in pulmonary 
oedema clinically. He improves clinically in 3 
days, but then begins to feel nauseous. His 
creatinine has risen from 120 to 210 pmol/l. 
He has ST changes on his ECG and complains 
of altered vision. 

Which of these drugs is the most likely cause 
of the effects? 



[ Q: 1353 ] MRCPass - Cardiology 

A 60 year old man has had a 
myocardial infarction. His pulse rate is 45 and 
he is feeling lightheaded. Blood pressure is 
90/65. 

In which one of the following conditions is 
temporary pacing indicated when symptoms 
are present? 

1- First degree heart block 

2- Wenkebach 

3- 2:1 Mobitz type II heart block 

4- Left bundle branch block 

5- Bifascicular block 



Answer & Comments 

Answer: 3- 2:1 Mobitz type II heart block 

The first form of second degree heart block, 
Mobitz type I (Wenkebach) is due to 
progressive prolongation of PR interval and 
then missing a beat. Mobitz type II second 


Dr. Khalid Yusuf El-Zohry - Sohag Teaching Hospital (01118391123) 

Ref MRCPass OE OE 2012 PasTest 2009 PassMedicine 2009 PasTest Exam ReviseMRCP 



551 

































El-zohry MRCP Questions Bank (Port 1) - 2013 


(For my personal use) 


degree heart block can occur with 2:1 (only 1 
QRS is conducted for 2 p waves) or 3:1. In a 
patient who is compromised with symptoms 
and hypotension, temporary pacing is 
indicated. 

[ Q: 1354 ] MRCPass - Cardiology 

A 60 year old woman has atrial 
fibrillation controlled with digoxin and beta 
blocker. She has a previous history of 
hypothyroidism but her last thyroid function 
tests were normal. She feels lightheaded and 
is brought to hospital. 

Her renal function is abnormal and she has a 
pulse rate of 35. 

What is the likely diagnosis? 

1- Myxedema crisis 

2- Addisonian crisis 

3- Hypercalcaemia 

4- Sick sinus syndrome 

5- Digoxin toxicity 



Answer & Comments 

Answer: 5- Digoxin toxicity 

Factors which predispose to digoxin toxicity 
are renal impairment, hypokalaemia, 
hypomagnesaemia and hypercalcaemia. It can 
lead to bradyarrhythmias - first degree heart 
block and Wenkebach (Mobitz type II second 
degree heart block is rare), and complete 
heart block. Other features of digoxin toxicity 
are xanthopsia (yellow vision), nausea & 
vomiting and dyspnoea. The reversed tick sign 
on the ECG is not a feature of toxicity but is 
associated with digoxin use. 


currently taking Ramipril 10 mg od and 
Frusemide 80 mg OD. 

What is the best treatment option? 

1- Losartan 

2- Amiodarone 

3- Bisoprolol 

4- Digoxin 

5- Spironolactone 


Answer & Comments 

Answer: 3- Bisoprolol 

Beta blockers improve mortality quality of life 
in chronic heart failure (COPERNICUS, 
MERITHF, CIBIS trials). They should be 
initiated once patients are stable (rather than 
in the acute setting) and can be used in all 
classes of heart failure. 

[ Q: 1356 ] MRCPass - Cardiology 

A 50 year old lady has palpitations. 
Her ECG shows a broad complex tachycardia. 

Which of these features suggests that the 
tachycardia is more likely to be of ventricular 
origin ? 

1- QRS of 180 ms 

2- Left bundle branch block and left axis 
deviation 

3- P wave for every QRS complex 

4- History of atrial fibrillation 

5- Heart rate of 150 



Answer & Comments 


Answer: 1- QRS of 180 ms 


[ Q: 1355 ] MRCPass - Cardiology 

A 75 year old man with dilated 
cardiomyopathy remains symptomatic in 
NYHA class 2 group. He has chronic heart 
failure. On examination his BP 150/95. He is 



Features that favour VT : 

QRS of > 140ms 

dissociated p waves 

history of ischaemic heart disease 


Dr. Khalid Yusuf El-Zohry - Sohag Teaching Hospital (01118391123) 

Ref MRCPass OE OE 2012 PasTest 2009 PassMedicine 2009 PasTest Exam ReviseMRCP 

































El-zohry MRCP Questions Bank (Port 1) - 2013 


(For my personal use) 


right bundle branch block with left axis 
deviation 

HR > 170 beats per minute 



[ Q: 1357 ] MRCPass - Cardiology 

A 50 year old woman has new 
symptoms of palpitations. She has no previous 
cardiac history. An ECG reveals atrial 
fibrillation. 


Which one of the following drugs is most likely 
to restore sinus rhythm? 

1- Diltiazem 


Answer & Comments 

Answer: 4- Posterior Ml 

The changes of posterior myocardial infarction 
are seen indirectly in the anterior precordial 
leads. Leads VI to V3 face the endocardial 
surface of the posterior wall of the left 
ventricle. As these leads record from the 
opposite side of the heart instead of directly 
over the infarct, the changes of posterior 
infarction are reversed in these leads. The R 
waves increase in size, becoming broader and 
dominant, and are associated with ST 
depression and upright T waves. 


2- Digoxin 

3- Atenolol 

4- Flecainide 

5- Labetalol 


Answer & Comments 

Answer: 4- Flecainide 

Flecainide is a class Ic antiarrhythmic drug. It 
helps restoration of sinus rhythm and prevent 
recurrence of AF. Flecainide may prolong QT 
interval. The drug was used in the Cardiac 
Arrhythmia Suppression Trial (CAST) trial, and 
was associated with increased mortality in 
patients with ischaemic heart disease. 



[ Q: 1359 ] MRCPass - Cardiology 

A 29 year old lady complained of 
transient blurred vision following a long flight. 
She has no other past medical history and is 
on no regular medications. On examination, 
there were no audible heart murmurs. CT scan 
of her head revealed no abnormality. 


What is the most likely underlying cause of her 
presentation? 


1- Antithrombin III deficiency 

2- Atrial septal defect 


3- Brain tumour 


4- Patent foramen ovale 


5- Vestibulitis 


7 

[ Q: 1358 ] MRCPass - Cardiology 


L-l 

A 53 year old man presents with 

Answer & Comments 

severe 

crushing chest pains. His ECG showed 

Answer: 4- Patent foramen ovale 


dominant R waves in VI and V2 leads. The T 
waves were also tall. 

What is the most likely cause? 

1- Lateral Ml 


The clinical scenario is one in which a 
thrombus has developed probably in the leg, 
and embolised via a patent foramen ovale, 
causing a transient ischaemic attack. 


2- Anterior Ml 


3- Inferior Ml 


4- Posterior Ml 

5- Pulmonary embolism 


Dr. Khalid Yusuf El-Zohry - Sohag Teaching Hospital (01118391123) 

Ref MRCPass OE OE 2012 PasTest 2009 PassMedicine 2009 PasTest Exam 




ReviseMRCP 

553 



























El-zohry MRCP Questions Bank (Port 1) - 2013 


(For my personal use) 



Patent Foramen Ovale 



[ Q: 1360 ] MRCPass - Cardiology 

A 60 year old man is acutely short of 
breath. ECG shows atrial fibrillation. His HR is 
170 and BP is 110/70. There are no cardiac 
murmurs but he has a raised JVP and few 
basal crackles in the lungs. 


Which of these is the best monogement 
strategy? 


1- Atenolol, frusemide, low molecular weight 
heparin 


2- Digoxin and low molecular weight heparin 

3- Iv amiodarone, iv frusemide 


4- Digoxin, frusemide, low molecular weight 
heparin 

5- Frusemide and DC cardioversion 


Answer & Comments 

Answer: 4- Digoxin, frusemide, low molecular 
weight heparin 

The patient has evidence of pulmonary 
oedema related to tachycardia and AF. She 
needs anticoagulation, rate control and 
diuresis to relieve pulmonary oedema. In 
addition, if her blood pressure allows, a 
nitrate (GTN) infusion would be helpful. DC 
cardioversion should be considered if rate 
control is poor but is associated with high risk 
of thromboembolism if the AF is not new. 


[ Q: 1361 ] MRCPass - Cardiology 

A 23 year old lady has a 6 month 
history of fever and pains in her elbows, wrist 
and knee joints. There is a soft systolic 
murmur and a pericardial rub on auscultation. 

Her bloods reveal: 

Hb 12.0 g/dl, WCC 10 x 10 9 /L, platelets 280 x 
10 9 /L, urea 5 pmol/l, creatinine 70 pmol/l, 
sodium 138 mmol/I, potassium 3.8 mmol/I, 
bilirubin 18 pmol/l, AST 18 U/l, ALP 180 U/l, 
albumin 35 g/l, ESR 100 mm/hr, CRP 140 mg/I. 

Which of the following is the likely diagnosis? 

1- Viral pericarditis 

2- Rheumatic fever 

3- Subacute bacterial endocarditis 

4- Atrial myxoma 

5- Polyarteritis nodosa 



Answer & Comments 

Answer: 2- Rheumatic fever 

This patient has polyarthritis, carditis (2 major 
criteria), fever and raised inflammatory 
markers (2 minor criteria). The history is 
consistent with rheumatic fever ((3 haemolytic 
strep Group A) infection. 

Rh eu m atic Fever/ 

Rheumatic Heart Disease 

Major criteria : Carditis, 
polyarthritis, chorea, 
subcutaneous nodules, 
and erythema marginatum 



[ Q: 1362 ] MRCPass - Cardiology 

A 55 year old Afro-Caribbean man in 
the outpatient clinic has uncomplicated 
essential hypertension. His blood pressure 


Dr. Khalid Yusuf El-Zohry - Sohag Teaching Hospital (01118391123) 

Ref MRCPass OE OE 2012 PasTest 2009 PassMedicine 2009 PasTest Exam 



i 

ReviseMRCP 


554 


i 
































El-zohry MRCP Questions Bank (Port 1) - 2013 


(For my personal use) 


today is 160/100 mmHg despite optimization 
of non-pharmacological therapy. 

Which one of the following drugs should be 
used? 

1- Atenolol 50mg od 

2- Amlodipine 5mg od 

3- Enalapril 5mg bd 

4- Nifedipine lOmg tds 

5- Lisinopril 2.5mg od 


Answer & Comments 

Answer: 2- Amlodipine 5mg od 

Studies indicate that drugs such as ACE 
(angiotensin-converting enzyme) inhibitors 
and Beta-receptor antagonists are less 
effective in Afro-Caribbeans. The reason 
appears to be related to the finding that the 
renin-angiotensin-aldosterone (RAA) system is 
commonly suppressed in the majority of Afro- 
Caribbeans. As such, drugs that suppress the 
RAA system are less likely to be effective. 

Calcium-channel blockers (CCBs) and diuretics 
appear to be more effective in this subgroup. 
Short-acting CCBs do not provide prolonged 
BP control, can cause reflex tachycardia and 
may be associated with higher mortality. 

Therefore, long-acting CCB should be the first- 
line drug of choice. Ideally, a once-daily agent 
with that provides a smooth 24-hour BP 
control (e.g. Nifedipine LA 30mg od or 
Amlodipine 5 mg od). 


3- LDH 

4- CK-MB 

5- CK 


Answer & Comments 

Answer: 4- CK-MB 

Troponins tend to be elevated for up to 14 
days. CK-MB comes down to normal level 
within 48-72 hours, and is the most specific of 
the CK enzymes. Within 4 days of the first Ml, 
the CK-MB fraction should not be raised, and 
if so, indicates a possible second Ml. 


^ [ Q: 1364 ] MRCPass - Cardiology 

f - 

A 65 year old patient has had a 
previous myocardial infarction 5 years ago 
followed by coronary artery bypass grafting. 
She is on aspirin, ramipril and furosemide. 
During review , she is well. Blood pressure is 
140/70 mmHg, pulse is 80 and regular, JVP is 
not raised. Her breath sounds are clear. 

Which drug should be considered ? 

1- Digoxin 

2- Lisinopril 

3- Carvedilol 

4- Clopidogrel 

5- Tirofibran 


Answer & Comments 


Answer: 3- Carvedilol 


[ Q: 1363 ] MRCPass - Cardiology 

A 65 year old patient diagnosed as 
having myocardial infarction on admission to 
hospital. 4 days later he suffered an attack of 
prolonged chest pain while still in the hospital. 

Which of the enzymes would best help in 
diagnosing a possible second Ml? 

1- Troponin I 

2- Troponin T 



The Carvedilol Prospective Randomized 
Cumulative Survival Study in patient with 
severe chronic heart failure was stopped early 
because of a significant beneficial effect of 
carvedilol on survival. 



[ Q: 1365 ] MRCPass - Cardiology 
A 70 year old man presents with an 


Dr. Kholid Yusuf El-Zohry - Sohog Teaching Hospital (01118391123) 

Ref MRCPass OE OE 2012 PasTest 2009 PassMedicine 2009 PasTest Exam 




ReviseMRCP 

555 




























El-zohry MRCP Questions Bank (Port 1) - 2013 


(For my personal use) 


episode of collapse. He has had 3 similar 
episodes recently. 2 years ago he suffered an 
anterior myocardial infarction. 

On examination he was orientated and 
symptom-free with a regular pulse rate of 100 
bpm, BP 140/80 mmHg and the apex beat was 
displaced to the left. There was an apical 
systolic murmur. There were no focal 
neurological signs. 

ECG showed sinus rhythm, Q waves and ST 
segment elevation anteriorly without 
reciprocal depression. CXR suggests left 
ventricular aneurysm. 

Whot is the likely couse of the col lapses? 

1- Acute anterior myocardial infarction 

2- Cerebrovascular accident 

3- Epileptic seizure 

4- Left ventricular aneurysm 

5- Ventricular tachycardia 


have spontaneously reverted to sinus rhythm. 
An echocardiogram is normal. 

Whot is the appropriate management? 

1- Advice to stop drinking 

2- Amiodarone 

3- Sotalol 

4- Aspirin 3 months 

5- Warfarin for one month 

Answer & Comments 

Answer: 1- Advice to stop drinking 

Excessive Alcohol is a well recognized cause of 
atrial fibirillation, likely to be the cause in this 
case. Ischaemic heart disease is unlikely, as is 
structural heart disease in view of a normal 
echocardiogram. Therefore, advice to stop 
drinking is most likely to help maintain sinus 
rhythm. 


Answer & Comments 

Answer: 5- Ventricular tachycardia 

A patient with anterior Ml, likely ventricular 
scarring with a collapse is likely to have 
ventricular arrhythmias. The fact that there 
are no focal neurological signs makes 
thromboembolism or stroke unlikely. 



Ventricular Tachycardia 


[ Q: 1367 ] MRCPass - Cardiology 

A 60 year old security guard presents 
with sudden onset of severe crushing central 
chest pain. The E.C.G shows sinus bradycardia 
with ST segment elevation in leads II, III, aVF. 

Where is the likely lesion? 

1- Diagonal artery 

2- Left anterior descending 

3- Right coronary artery 

4- Circumflex artery 

5- Obtuse marginal artery 



Answer & Comments 


Answer: 3- Right coronary artery 


[ Q: 1366 ] MRCPass - Cardiology 


A 25 year old man is brought into 
hospital with symptoms of alcohol 
intoxication. An initial ECG reveals atrial 
fibirillation. The next morning he was found to 


The patient has an inferior myocardial 
infarction. This area of the myocardium is 
supplied by the right coronary artery. The sino 
atrial node is also supplied by an artery which 
arises from the right coronary artery and 
hence there may be bradycardia or heart 
block. 


Dr. Khalid Yusuf El-Zohry - Sohag Teaching Hospital (01118391123) 

Ref MRCPass OE OE 2012 PasTest 2009 PassMedicine 2009 PasTest Exam ReviseMRCP 



556 










































El-zohry MRCP Questions Bank (Part 1) - 2013 


(For my personal use) 


[ Q: 1368 ] MRCPass - Cardiology 

f | - 

# A 65 year old man has presented 
with chest pain and has anterior ST elevation 
on the ECG which is thrombolysed with 
tenecteplase. He becomes very breathless. On 
examination, he has a loud pansystolic 
murmur in the lower left sternal edge and 
bilateral coarse crepitations in the lung bases. 

Which of the following investigations is most 
important to confirm the diagnosis? 

1- Chest X ray 

2- Transthoracic echocardiogram 

3- CT pulmonary angiogram 

4- Repeat ECG 

5- Arterial blood gas 

Answer & Comments 

Answer: 2- Transthoracic echocardiogram 


2- Regular outpatient follow up with 
echocardiography 

3- Anticoagulation with warfarin 

4- Transoesphageal echocardiography 

5- Refer for cardiac surgery 

Answer & Comments 

Answer: 2- Regular outpatient follow up with 
echocardiography 

The normal left ventricular function and 
dimensions, as well as moderate mitral 
regurgitation suggests that cardiac surgery can 
be held off. She is in sinus rhythm and does 
not require anticoagulation (left atrium is only 
mildly enlarged - < 4cm is normal). Antibiotic 
prophylaxis is necessary. Transoesophageal 
echocardiography is not necessary unless she 
is being considered for surgery, or there is 
suspicion of endocarditis. 


Although he is likely to have pulmonary 
oedema, a CXR will not confirm the diagnosis. 
With a large antero-septal Ml, he could have 
either have mitral regurgitation (function or 
papillary muscle damage) or a VSD causing the 
systolic murmur. As this is in the lower left 
sternal edge, it would be important to get an 
urgent echocardiogram to confirm and refer 
for cardiothoracic surgery. 


[ Q: 1369 ] MRCPass - Cardiology 

A 55 year old woman has a systolic 
murmur which is discovered by the GP who 
referred the patient for a cardiac opinion. She 
has a transthoracic echocardiogram which 
shows normal left ventricular function. There 
is no significant cardiac enlargement and she 
has a mitral valve prolapse with moderate 
mitral regurgitation. The left atrium diameter 
is 4.3 cm. 

What is the most appropriate course of 
action ? 

1- No endocarditis prophylaxis necessary 




[ Q: 1370 ] MRCPass - Cardiology 

A 40 year old patient has a 
transthoracic echocardiogram as a follow up. 
He has a diagnosis of hypertrophic obstructive 
cardiomyopathy. Clinically he has a systolic 
murmur heard loudest in the right upper 
sternal edge. His ECG shows grossly large QRS 
complexes with LVH strain pattern. 

Which of the following suggests highest risk 
for sudden death? 

1- A left ventricular outflow tract gradient of 
20 mmHg 

2- Tricuspid regurgitation 


3- Systolic anterior motion of mitral valve 

4- ECG showing ventricular ectopics 

5- Interventricular septal thickness of 4 cm 


Answer & Comments 

Answer: 5- Interventricular septal thickness of 
4 cm 


Dr. Khalid Yusuf El-Zohry - Sohag Teaching Hospital (01118391123) 

Ref MRCPass OE OE 2012 PasTest 2009 PassMedicine 2009 PasTest Exam ReviseMRCP 



557 





























El-zohry MRCP Questions Bank (Port 1) - 2013 


(For my personal use) 


The LV outflow tract gradient is not 
significantly high in this patient, however, this 
may be variable with exercise. The large 
interventricular septal thickness of 4 cm 
(normal <1.3 cm) suggests very hypertrophic 
myocardium and high risk of outflow tract 
obstruction with exertion. 



[ Q: 1371 ] MRCPass - Cardiology 

A 60 year old man presents with 
breathlessness, visual blurring and headaches 
to A&E. During examination, he has a 
generalised tonic clonic seizure. He has a 
blood pressure of 240/120. Fundoscopy 
reveals retinal haemorrhages and exudates. 
The optic disc margin is not visualised. 


Which of the following medications is first 
line? 


1- Sodium nitroprusside 

2- Labetalol 


mitral stenosis has atrial fibrillation. She 
presents unwell with fast AF and was DC 
cardioverted successfully. 

Which of the following medications is most 
effective for maintenance of sinus rhythm? 

1- Digoxin 

2- Amiodarone 

3- Atenolol 

4- Diltiazem 

5- Labetalol 


Answer & Comments 

Answer: 2- Amiodarone 

Amiodarone is most effective for maintenance 
of sinus rhythm. However, because of its side 
effect profile, in a young patient, it is usually 
worth trying a beta blocker or calcium blocker 
before treating with amiodarone. 


3- Diltiazem 

4- Atenolol 

5- Bendrofluazide 

Answer & Comments 

Answer: 1- Sodium nitroprusside 

Malignant hypertension can present with 
pulmonary oedema, chest pain, headache, 
visual disturbance and also seizures. A 
fundoscopic examination may reveal flame¬ 
shaped retinal hemorrhages, soft exudates, or 
papilledema. 


[ Q: 1373 ] MRCPass - Cardiology 

# A 25 year old man is being 
investigated for hypertension. The pressure 
data from cardiac catheterisation are as follow 
s: RA 2 mm Hg, RV 22/2 mm Hg, PA 25/14 mm 
Hg, LA 7 mm Hg, LV 210/0-8 mm Hg, Aorta 
180/100 mm Hg, Femoral artery 95/60 mm 
Hg. 

What is the likely diagnosis? 

1- Ascending aortic aneurysm 

2- Aortic stenosis 

3- Coarctation of the aorta 


Sodium Nitroprusside has an immediate onset 
of action and short half-life. It acts by causing 
relaxation of vascular smooth muscle, 
resulting in vasodilation and inotropy. The 
dose is 0.25-10 mcg/kg/min IV. IV hydralazine, 
beta blockers and calcium channel blockers 
can also be used. 



[ Q: 1372 ] MRCPass - Cardiology 
A 35 year old patient with mild 


4- Aortic dissection 

5- Aortic regurgitation 


Answer & Comments 

Answer: 3- Coarctation of the aorta 

The history of hypertension and the pressure 
data suggest a diagnosis of coarctation of the 
aorta. 


Dr. Khalid Yusuf El-Zohry - Sohag Teaching Hospital (01118391123) 

Ref MRCPass OE OE 2012 PasTest 2009 PassMedicine 2009 PasTest Exam ReviseMRCP 






























El-zohry MRCP Questions Bank (Port 1) - 2013 


(For my personal use) 



Coarctation of the aorta 


[ Q: 1374 ] MRCPass - Cardiology 

A 60 year old lady is known to have 
severe mitral stenosis. She presents with 
breathlessness. 

On examination , which one of the following 
features is expected? 

1- Clubbing 

2- Loud first heart sound 

3- Wide pulse pressure 

4- Ejection systolic murmur 

5- Small left atrium 



Answer & Comments 

Answer: 2- Loud first heart sound 

A loud first heart sound and tapping apex beat 
are common in significant mitral stenosis. A 
narrow pulse pressure is expected in severe 
mitral stenosis. Left atrium is inevitably dilated 
on CXR and echocardiography. 



[ Q: 1375 ] MRCPass - Cardiology 

A 45 year old patient has been 
commenced on nifedipine for hypertension. 


Which of the following effect describes the 
action of nifedipine? 

1- Reduced sympathetic activity 


2- Decreased activation of renin angiotensin 
aldosterone axis 

3- Reduced peripheral capillary pressure 

4- Reduced peripheral vascular resistance 

5- Decrease vascular release of nitric oxide 


Answer & Comments 

Answer: 4- Reduced peripheral vascular 
resistance 

Nifedipine is a dihydropyridine. The 
mechanism of action is smooth muscle 
relaxation and reduction in peripheral vascular 
resistance. However, peripheral capillary 
pressure is increased (this causes oedema in 
the ankles), there is increased sympathetic 
activity and increased activation of the renin 
angiotensin aldosterone axis. There is also 
increased vascular release of nitric oxide. 



[ Q: 1376 ] MRCPass - Cardiology 

A 70 year man has been short of 
breath for 1 year. An electrocardiogram shows 
T wave inversion and q waves in the 
anterolateral leads at rest. He has 
cardiomegaly on the chest X ray. Clinical 
examination shows a third heart sound, a soft 
systolic murmur in the mitral area and also 
bilateral inspiratory crepitations in his lungs. 


Which of the following is he at risk of? 


1- Deep vein thrombosis 

2- Pulmonary embolus 

3- Systemic arterial embolus from mural 
thrombus 


4- Venous thrombosis due to mural thrombus 


5- Coronary artery thrombus due to mural 
thrombus 


Answer & Comments 

Answer: 3- Systemic arterial embolus from 
mural thrombus 


Dr. Khalid Yusuf El-Zohry - Sohag Teaching Hospital (01118391123) 

Ref MRCPass OE OE 2012 PasTest 2009 PassMedicine 2009 PasTest Exam ReviseMRCP 



559 


































El-zohry MRCP Questions Bank (Port 1) - 2013 


(For my personal use) 


This man has features indicating that he has 
an enlarged left ventricle from previous 
anterior myocardial infarction in the LAD 
artery territory. Poor LV function also causes 
his symptoms of heart failure. He is at risk of 
developing mural thrombus with embolus to 
the arterial circulation. 



[ Q: 1377 ] MRCPass - Cardiology 

A 45 year old man was referred by 
the GP for evaluation of a murmur. On 
examination, he was found to have a slow 
rising pulse and ejection systolic murmur in 
the aortic area. Aortic stenosis was suspected. 


In a patient presenting with aortic stenosis, 
which of following features would be helpful in 
establishing a diagnosis of congenital bicuspid 
valve? 


1- Calcified leaflets 

2- Old age 

3- A systolic ejection click 

4- Commissural fusion on the echocardiogram 

5- History of rheumatic fever 


Answer & Comments 


Answer: 3- A systolic ejection click 


4- Pericarditis 

5- Myocardial infarction 


Answer & Comments 

Answer: 3- Constrictive pericarditis 

Constrictive pericarditis leads to signs of right 
sided heart failure which are listed above. 
There is also a prominent and rapid X and Y 
descent of the waveform. 

[ Q: 1379 ] MRCPass - Cardiology 

A 57 year old lady who had a past 
history of Myocardial Infarction and 
subsequent CABG was known to have left 
ventricular dysfunction on the 
echocardiogram. She has no symptoms at 
present. She was on aspirin, ramipril and 
diuretic. On examination there were no signs 
of heart failure. 

What drug should be added next? 

1- Amlodipine 

2- Bisoprolol 

3- Clopidogrel 

4- Atorvastatin 

5- Digoxin 



An aortic ejection sound (ejection click) soon 
after SI is especially common if a bicuspid 
aortic valve is present. 


Answer & Comments 


Answer: 2- Bisoprolol 


[ Q: 1378 ] MRCPass - Cardiology 

A 40 year old lady has presented 
with breathlessness. Her JVP is noted to be 
raised. Manometry reveals a rapid Y descent. 

On examination, she has hepatomegaly, 
ascites and ankle oedema. 

What is the diagnosis? 

1- Dilated cardiomyopathy 

2- Left ventricular failure 

3- Constrictive pericarditis 



In a patient with left ventricular dysfunction 
and known history of ischaemic heart disease, 
a beta blocker should be added (CIBIS II 
Study). 


[ Q: 1380 ] MRCPass - Cardiology 

A 35 year old lady has progressive 
shortness of breath. Following an 
echocardiogram, she is found to have likely 
rheumatic mitral stenosis is now being 
considered for percutaneous mitral 
valvuloplasty. 



Dr. Khalid Yusuf El-Zohry - Sohag Teaching Hospital (01118391123) 

Ref MRCPass OE OE 2012 PasTest 2009 PassMedicine 2009 PasTest Exam ReviseMRCP 
































El-zohry MRCP Questions Bank (Port 1) - 2013 


(For my personal use) 


Which of the following would contraindicate 
this procedure? 

1- Dilated left atrium 

2- Atrial fibrillation 

3- Aortic regurgitation 

4- Heavy calcification of the mitral valve 

5- Long history of mitral stenosis 


Answer & Comments 

Answer: 4- Heavy calcification of the mitral 
valve 

The contraindications tow ards valvuloplasty 
are heavy MV calcification, thrombus in the 
left atrial appendage on transoesophageal 
echocardiography and severe mitral 
regurgitation. These patients are indicated for 
mitral valve surgery instead. 

[ Q: 1381 ] MRCPass - Cardiology 

A 25 year old man whose brother 
had hypertrophic cardiomyopathy was 
referred for a cardiological assessment. His 
echocardiogram confirmed the condition. 

Which one of following echocardiographic 
features is the most important risk factor for 
sudden cardiac death? 

1- Thickness of septal wall 

2- Systolic anterior motion of the mitral valve 

3- Severity of mitral regurgitation 

4- Gradient across left ventricular outflow 
tract 

5- An enlarged left atrium 



Answer & Comments 


Answer: 1- Thickness of septal wall 



[ Q: 1382 ] MRCPass - Cardiology 

A 55 year old patient had myocardial 
infarction 6 days ago. He suddenly develops 
dyspnoea, cough and frothy sputum. For the 
first time a harsh systolic murmur is heard 
over the praecordium. 


This sequence of events might be caused by: 

1- Pulmonary embolism 


2- Aortic dissection 


3- Tricuspid regurgitation 

4- Ruptured papillary muscle 

5- Ruptured aortic cusp 


Answer & Comments 

Answer: 4- Ruptured papillary muscle 

Following an Ml, ruptured papillary muscle or 
interventricular septum is most likely to cause 
the combination of pulmonary oedema and 
new murmur (either mitral regurgitation or 
due to VSD). 


[ Q: 1383 ] MRCPass - Cardiology 

A 30 year old man is brought to A & 
E with severe acute chest pain radiating to the 
back. He is pale and anxious. 

On examination the blood pressure is 150/90 
mmHg and he was noted to have blue sclerae. 

Which of the following conditions has 
predisposed to this complication? 

1- Coarctation of the aorta 

2- Ischaemic heart disease 

3- Rheumatic fever 

4- Hypertrophic cardiomyopathy 

5- Marfan's syndrome 



The greater thickness of septum, the more 
likelihood of left ventricular outflow tract 
obstruction. The next most important factor 
would be the gradient across the outflow 
tract. 


Answer & Comments 

Answer: 5- Marfan's syndrome 

Marfan's syndrome is one of the causes of 
blue sclerae. The clinical diagnosis is aortic 


Dr. Khalid Yusuf El-Zohry - Sohag Teaching Hospital (01118391123) 

Ref MRCPass OE OE 2012 PasTest 2009 PassMedicine 2009 PasTest Exam ReviseMRCP 



561 




































El-zohry MRCP Questions Bank (Port 1) - 2013 


(For my personal use) 


dissection. Other predisposing disorders to 
aortic dissection are: essential hypertension, 
atherosclerosis , bicuspid aortic valve, 
connective tissue disorders, Marfan's 
syndrome, Ehlers-Danlos syndrome, adult 
polycystic kidney disease. 


[ Q: 1384 ] MRCPass - Cardiology 

A 68 year old man with severe aortic 
stenosis presented with melaena. Upper Gl 
endoscopy was normal. 

Whot is the likely couse of Gl bleeding? 

1- Haemorrhoids 

2- Caecal carcinoma 

3- Duodenal ulcer 

4- Angiodysplasia 

5- Ulcerative colitis 



Answer & Comments 

Answer: 4- Angiodysplasia 

Angiodysplasia is the most common vascular 
abnormality of the Gl tract. After 
diverticulosis, it is the second leading cause of 
lower Gl bleeding in patients older than 60 
years. Angiodysplasia has been reported to be 
associated with aortic stenosis. 



Angiodysplasia 



[ Q: 1385 ] MRCPass - Cardiology 
A 35 year old man has a dominant R 


in lead VI on the ECG during routine 
examination. 

Which one of the following is o recognised 
couse? 

1- Pericarditis 

2- Turner's syndrome 

3- Dextrocardia 

4- Aortic stenosis 

5- Tricuspid regurgitation 


Answer & Comments 

Answer: 3- Dextrocardia 

A dominant R in lead VI on the ECG is 
associated with : 

-primary pulmonary HT 

-Right bundle branch block (RBBB) (including 
Ebstein's anomaly) 

-Wolf-Parkinson-White syndrome Type A 

-Dextrocardia 

-Posterior Ml 

-Duchene muscular dystrophy 



Dextrocardia ECG 


[ Q: 1386 ] MRCPass - Cardiology 

A 62 year old lady with known 
congestive heart failure and renal impairment. 
She presents with features of worsening heart 
failure, and also mentioned about recent 
decreased urine output. 

Whot is the most useful investigation for the 
current condition? 




Dr. Kholid Yusuf El-Zohry - Sohog Teaching Hospital (01118391123) 

Ref MRCPass OE OE 2012 PasTest 2009 PassMedicine 2009 PasTest Exam 


ReviseMRCP 


































El-zohry MRCP Questions Bank (Port 1) - 2013 


(For my personal use) 


1- Electrolytes 

2- Urea 

3- Creatinine 

4- Echocardiogram 

5- Chest x ray 

Answer & Comments 

Answer: 4- Echocardiogram 

A patient with symptoms of congestive cardiac 
failure and renal impairment may have 
deteriorating left ventricular function. This 
may subsequently lead to cardiogenic shock, 
the severity of LV function should be 
documented to guide further management. 


echocardiogram and refers the patient to you 
with a report. 

Which one of the following measures is 
essential for the calculation of ejection 
fraction ? 

1- Ventricular wall thickness 

2- Aortic valve diameter 

3- Left ventricular end diastolic diameter 

4- Stroke volume 

5- Aortic valve velocity 

Answer & Comments 

Answer: 3- Left ventricular end diastolic 
diameter 


[ Q: 1387 ] MRCPass - Cardiology 

A 28 year woman who is 20 weeks 
pregnant is seen in the outpatient clinic and 
noted to have a sustained blood pressure of 
180/95 mmHg. 

What is the most appropriate antihypertensive 
therapy for this patient? 

1- Methyldopa 

2- Hydralazine 

3- Nifedipine 

4- Lisinopril 

5- Atenolol 



Answer & Comments 

Answer: 1- Methyldopa 

Methyldopa is the safest agent to use in the 
first and second trimester of pregnancy. 

Beta blockers may cause intrauterine growth 
retardation. ACE inhibitors have also been 
suggested to have teratogenic effects. 


Ejection fraction measurement requires 
measurements during both the end systolic 
phase or end diastolic phases on the 
transthoracic echocardiogram. This could be in 
the form of ventricular diameter 
measurement or area of the ventricle (in both 
cases volumes are estimated by the echo 
machine). The end systolic and end diastolic 
volumes are then subtracted to obtain an 
ejection fraction. Stroke volume itself is not 
sufficient to give the ejection 'fraction' hence 
needs comparison either systolic or diastolic 
measurements. 


[ Q: 1389 ] MRCPass - Cardiology 

A 55 year old man with angina has 
familial hypercholesterolaemia. His father and 
uncle both have coronary heart disease. 

Which one of the following is a typical feature 
for the condition? 

1- Plantar xanthomas 

2- Autosomal recessive 

3- Increased LDL concentrations 




[ Q: 1388 ] MRCPass - Cardiology 
A cardiac technician has done an 


4- Hypertriglyceridaemia 

5- Increased expression of LDL receptors 


Dr. Khalid Yusuf El-Zohry - Sohag Teaching Hospital (01118391123) 

Ref MRCPass OE OE 2012 PasTest 2009 PassMedicine 2009 PasTest Exam 




ReviseMRCP 

563 




























El-zohry MRCP Questions Bank (Port 1) - 2013 


(For my personal use) 


Answer & Comments 

Answer: 3- Increased LDL concentrations 

Familial hypercholesterolaemia is an 
autosomal dominant condition. There are 
increased LDL concentrations due to reduced 
numbers of LDL receptors. 

Hypertriglyceridaemia does not usually occur 
and HDL concentrations are usually 
decreased. Tendon xanthomas occur, not 
plantar xanthomas. 


^ [ Q: 1390 ] MRCPass - Cardiology 

A - 

# A 48 year old male is referred with 
impotence. He has a history of angina, 
hypertension and type 2 diabetes. 

Which one of the following drugs that he takes 
would present a contraindication towards him 
receiving Sildenafil? 

1- Aspirin 

2- Bendrofluazide 

3- Isosorbide Mononitrate 

4- Lisinopril 

5- Metformin 


Answer & Comments 

Answer: 3- Isosorbide Mononitrate 

There is a significant risk of hypotension when 
sildenafil is used with nitrates. Hence it is 
contraindicated in patients with a history of 
angina or ischaemic heart disease. 


2- Hypothyroidism 

3- Ramipril use 

4- Hyperkalaemia 

5- Hypokalaemia 


Answer & Comments 

Answer: 5- Hypokalaemia 

Hypomagnesaemia, hypokalaemia, and 
hypercalcaemia are common metabolic 
disturbances which may w orsen digoxin 
toxicity. 

Bradycardia, prolonged PR interval, shortened 
QT interval and various forms of heart block 
can occur in digoxin toxicity. 


[ Q: 1392 ] MRCPass - Cardiology 

A 70 year old man has a history of 
syncope. He is found to have runs of 
nonsustained ventricular tachycardia on ECGs 
done in casualty. He has a past medical history 
of hypertension. Investigations show a serum 
magnesium of 0.6 mmol/l (0.75-1). 

Which one of the following is the most likely 
cause of hypomagnesaemia? 

1- Calcium channel blockers 

2- Diuretic therapy 

3- Hyperphosphataemia 

4- Diarrhoea and vomiting 

5- Hypercalcaemia 




[ Q: 1391 ] MRCPass - Cardiology 

A 65 year old woman who is on 
digoxin has symptoms of nausea and 
dizziness. On examination, her heart rate is 35 
bpm and an ECG shows prolonged PR interval 
with first degree heart block. 


Digoxin toxicity is more likely with which of the 
following conditions? 


Answer & Comments 

Answer: 2- Diuretic therapy 

Magnesium is present in greatest 
concentration within the cell and is the second 
most abundant intracellular cation after 
potassium. Most renal reabsorption of 
magnesium occurs in the proximal tubule and 
the thick ascending limb of the loop of Henle. 


1- Hypocalcaemia 


Significant losses of magnesium that result in 
hypomagnesemia may result from chronic 


Dr. Khalid Yusuf El-Zohry - Sohag Teaching Hospital (01118391123) 

Ref MRCPass OE OE 2012 PasTest 2009 PassMedicine 2009 PasTest Exam ReviseMRCP 
































El-zohry MRCP Questions Bank (Port 1) - 2013 


(For my personal use) 


diarrhea, laxative abuse, inflammatory bow el 
disease, or neoplasm, diuretics (thiazide, loop 
diuretics). 

[ Q: 1393 ] MRCPass - Cardiology 

A 30 year old lady was diagnosed 
with long QT syndrome on routine 
investigation for her insurance scheme. 

Which one of the following drugs should be 
started? 



Answer & Comments 

Answer: 3- Atrial septal defect 

The second heart sound typically occurs with 
A2 (aortic) and P2 (pulmonary). 

Wide splitting can occur with delayed 
pulmonary closure (P2) as in right bundle 
branch block (RBBB), pulmonary stenosis and 
ventricular septal defect (VSD). However, 
FIXED wide splitting only occurs in ASD. 


1- Atenolol 

2- Digoxin 

3- Amiodarone 

4- Verapamil 

5- Lignocaine 


Answer & Comments 

Answer: 1- Atenolol 

In the management of a case of congenital 
LQT syndrome, beta-blockade is usually 
effective in preventing ventricular 
tachyarrhythmias in the patient. If 
symptomatic or if there are ventricular 
arrhythmias documented, an intracardiac 
cardioverter defibrillator should be 
considered. 


^ [ Q: 1395 ] MRCPass - Cardiology 

fj - 

# A 65 year old man with a history of 
previous myocardial infarction presents with 
palpitations. The ECG shows a broad complex 
tachycardia at a rate of 150 beats/min and a 
blood pressure of 90/55 mmHg. 

The first line treatment should be: 

1- Sotalol 

2- Flecainide 

3- Verapamil 

4- Amiodarone 

5- Lignocaine 


Answer & Comments 


Answer: 4- Amiodarone 


[ Q: 1394 ] MRCPass - Cardiology 

A 23-year-old male presented with a 

2-year history of breathlessness on exertion. 

On examination, there was a systolic murmur 
in the pulmonary area and wide fixed splitting 
of the second heart sound. 



The likely diagnosis is ventricular tachycardia. 
There are early signs of haemodynamic 
instability (blood pressure is low). In this case, 
amiodarone is the best agent as first line. 
Lignocaine and flecainide can also be used but 
are not considered first line when patients are 
unstable. 


In which of these conditions is fixed and wide 
splitting of the second heart sound seen? 

1- Fallot's tetralogy 

2- Ventricular septal defect 

3- Atrial septal defect 

4- Pulmonary stenosis 

5- Aortic regurgitation 


[ Q: 1396 ] MRCPass - Cardiology 

A 50 year old taxi driver is followed 
up 8 weeks after an anterior myocardial 
infarction. He underwent rescue angioplasty 
with stenting to the LAD vessel following 
failed thrombolysis. Since then he has had no 
further symptoms of angina. 



Dr. Khalid Yusuf El-Zohry - Sohag Teaching Hospital (01118391123) 

Ref MRCPass OE OE 2012 PasTest 2009 PassMedicine 2009 PasTest Exam 




ReviseMRCP 

565 






























El-zohry MRCP Questions Bank (Port 1) - 2013 


(For my personal use) 


Which should be the next investigation? 

1- Bruce protocol exercise test 

2- Modified Bruce protocol exercise test 

3- Repeat coronary angiography to check stent 
patency 

4- Dobutamine stress echocardiography 

5- Cardiac thallium scan 


Answer & Comments 

Answer: 1- Bruce protocol exercise test 

For public vehicle drivers and heavy goods 
vehicles drivers which are classed under 
Group 2 entitlement by the DVLA, driving is 
disqualified for 6 weeks for Ml, CABG and 
angioplasty. 

Following this, they have to undergo a Bruce 
protocol exercise test to stage III without 
significant ST changes or anginal symptoms. 



[ Q: 1397 ] MRCPass - Cardiology 

A 45 year old man has worsening 
breathlessness. On examination, he has a 
systolic blood pressure of 115 mmHg which 
drops to 90mmHg during inspiration. 


Which of the following conditions is most likely 
to be present? 


1- Congestive cardiac failure 


2- Cardiac tamponade 

3- Atrial fibrillation 


4- Ventricular tachycardia 

5- ICD implantation 


Answer & Comments 

Answer: 2- Cardiac tamponade 

The clinical feature is pulsus paradoxus. This is 
defined as a drop in blood pressure by more 
than 20mmHg during inspiration. It is a 
definite sign of cardiac tamponade, but may 
occasionally occur with acute constrictive 
pericarditis and COPD. 


[ Q: 1398 ] MRCPass - Cardiology 

A 50 year old lady is currently 
asymptomatic but undergoes a 
medical examination. Her ECG shows left 
bundle branch block. 



During clinical examination, which one of 
these signs is likely to be found? 

1- Tricuspid regurgitation 

2- Systolic murmur in the mitral area 

3- Third heart sound 


4- Reversed splitting of the second heart 
sound 


5- Diastolic murmur in the pulmonary area 


Answer & Comments 

Answer: 4- Reversed splitting of the second 
heart sound 

The second heart sound comprises of aortic 
(A2) and pulmonary (P2) component. In LBBB, 
the aortic closure is delayed because the left 
ventricle contracts later. This then causes 
reversed splitting (A2P2 ? P2A2) of the second 
heart sound. 


^ [ Q: 1399 ] MRCPass - Cardiology 

# A 50 year man presents with 
lightheadness. He has frequent nonsustained 
ventricular tachycardia on the ECG and cardiac 
monitor. His bloods show a Hb 13.0 g/dl, WCC 
7 x 10 9 /l_, platelets 230 x 10 9 /L, urea llmol/l, 
creatinine 80pmol/l, sodium 134 mmol/I, 
potassium 3.2 mmol/I, serum magnesium of 
0.6 mmol/I (0.75). 

Which one of the following is likely to be 
responsible for his arrhythmias? 

1- Poor diet 

2- Alcoholism 

3- Frusemide 

4- Diarrhoea 

5- Hyperphosphataemia 


Dr. Khalid Yusuf El-Zohry - Sohag Teaching Hospital (01118391123) 

Ref MRCPass OE OE 2012 PasTest 2009 PassMedicine 2009 PasTest Exam ReviseMRCP 





























El-zohry MRCP Questions Bank (Port 1) - 2013 


(For my personal use) 


Answer & Comments 

Answer: 3- Frusemide 

The likely cause of the arrhythmias is 
hypomagnesaemia and hypokalaemia, which 
is most commonly associated with diuretic 
use. 

[ Q: 1400 ] MRCPass - Cardiology 

A 35 year old man presents with 
chest pain to casualty following attendance at 
a party. His friend reports that he has using 
large doses of crack cocaine. His ECG shows ST 
elevation of 4 mm in the anterior leads, and 
he is managed as having acute myocardial 
infarction. There is no previous cardiac 
history. 

Which of the following should be avoided? 

1- GTN 



valve replacement 8 months ago is admitted 
with fevers, positive blood cultures and 
suspected prosthetic valve endocarditis. 

Which one of the following features suggests 
worsening of the condition? 

1- Paresthesiae 

2- Vitiligo 

3- Prosthetic valve click 

4- Systolic flow murmur 

5- Prolonged PR interval on the ECG 


Answer & Comments 

Answer: 5- Prolonged PR interval on the ECG 

Infections of the prosthetic valve beyond 6 
months after surgery are most often due to 
Streptococcus viridans. Early infections are 
usually due to Staphylococcus epidermidis 


2- Aspirin 

3- Diltiazem 

4- Diamorphine 

5- Atenolol 


Answer & Comments 

Answer: 5- Atenolol 

The main effect of cocaine is inhibition of 
noradrenaline and dopamine re-uptake in the 
synaptic terminals. It is a potent 
sympathomimetic agent, and causes a rise in 
heart rate and blood pressure. It also causes 
significant coronary artery spasm, and can 
precipitate myocardial infarction which is 
potentiated by the increase in myocardial 
oxygen demand. Beta blockers such as 
atenolol can precipitate worsening of the 
coronary vasospasm and should be avoided. 
Treatment is with nitrates or calcium channel 
blockers (which are vasodilators). 


(coagulase negative). 

One of the major dangers with aortic valve 
endocarditis is an aortic root abscess. This can 
lead to prolonging of the PR interval by 
erosion into the adjacent AV node, hence daily 
ECGs are useful for monitoring. 


[ Q: 1402 ] MRCPass - Cardiology 

A 60 year old man has had an 
anterior infarct. He hates to take medications 
and agrees to have only one drug a day. 

Which of the following drug classes has been 
shown to have the maximal benefit in the peri- 
infarct period? 

1- Alpha blockers 

2- Nitrates 

3- Calcium Channel blockers 

4- Beta blockers 

5- Statin 




[ Q: 1401 ] MRCPass - Cardiology 
A 50 year old man who had a aortic 


Answer & Comments 


Answer: 4- Beta blockers 


Dr. Khalid Yusuf El-Zohry - Sohag Teaching Hospital (01118391123) 

Ref MRCPass OE OE 2012 PasTest 2009 PassMedicine 2009 PasTest Exam ReviseMRCP 



567 




































El-zohry MRCP Questions Bank (Port 1) - 2013 


(For my personal use) 


Beta blockade has been shown to be 
beneficial when started as soon as possible. 
This was demonstrated in the ISIS-1 trial 
where atenolol reduced mortality compared 
to the control group at 1 year. 


[ Q: 1403 ] MRCPass - Cardiology 

A 20 year old man attends A+E 
because of palpitations. Before they could do 
an ECG his palpitations self terminated. The 
ECG which was done showed sinus rhythm 
with a PR interval of 70 ms. 

What is the mechanism of the patient's 
tachycardia? 

1- Atrial flutter 

2- Atrioventricular reentry tachycardia 

3- Atrioventricular nodal reentry tachycardia 

4- Ventricular tachycardia 

5- Atrial tachycardia 



Answer & Comments 

Answer: 2- Atrioventricular reentry 

tachycardia 

The PR interval is short (<3 small squares or 
<120 ms) suggesting wolff parkinson white 
syndrome. There may be an accessory 
pathway which predisposes to AVRT rather 
than AVNRT. 

[ Q: 1404 ] MRCPass - Cardiology 

A 22 year old engineering student 
presents with a history of breathlessness on 
exertion and orthopnoea, fatigue and 
anorexia. 

On examination he has peripheral oedema, a 
low volume pulse, elevated JVP with a rapid y 
descent, no inspiratory increase in JVP, a quiet 
praecordium and characteristic auscultatory 
features of his condition. He also has 
hepatomegaly and ascites. 

ECG shows a widened QRS complex with 
diffuse non-specific repolarisation changes. 



What is the likely diagnosis? 

1- Constrictive pericarditis 

2- Cardiomyopathy 

3- Congestive cardiac failure 

4- Restrictive cardiomyopathy 

5- Myocarditis 


Answer & Comments 

Answer: 4- Restrictive cardiomyopathy 

The clinical radiological and ECG features 
suggest the patient has a restrictive 
cardiomyopathy. The characteristic 
auscultatory feature of this condition is a 
fourth heart sound reflecting increased atrial 
contraction in an effort to overcome the 
reduced compliance of the ventricle. A 
pericardial knock would be a feature of 
constrictive pericarditis, which has been ruled 
out by the absence of Kussmaul's sign 
(inspiratory increase in JVP) and absence of 
pericardial calcification. 

[ Q: 1405 ] MRCPass - Cardiology 

A 42 year old man presents with a 
regular narrow complex tachycardia of 160 
bpm. Adenosine is considered by the duty 
medical registrar. 

Which of the following is a contraindication to 
using iv adenosine? 

1- Wolff parkinson white syndrome 

2- Ventricular tachycardia 

3- Asthma 

4- Ischaemic heart disease 

5- Gastric ulcer 



Answer & Comments 

Answer: 3- Asthma 

Adenosine can produce profound 
bronchospasm especially in asthmatics, and it 
is contraindicated. 


Dr. Khalid Yusuf El-Zohry - Sohag Teaching Hospital (01118391123) 

Ref MRCPass OE OE 2012 PasTest 2009 PassMedicine 2009 PasTest Exam ReviseMRCP 






























El-zohry MRCP Questions Bank (Port 1) - 2013 


(For my personal use) 


[ Q: 1406 ] MRCPass - Cardiology 

A 40 year old man is being assessed 
for endocarditis according to Duke's criteria. 

Which of the following is a major criteria for 
diagnosing infective endocarditis? 

1- Roth's spots 

2- Vegetation seen on Echocardiogram 

3- Splinter haemorrhages 

4- Fever 


Echocardiography shows asymmetric left 
ventricular hypertrophy and systolic anterior 
motion of the mitral valve. 

The patient's symptoms may be treated with: 

1- Ramipril 

2- Frusemide 

3- Aspririn 

4- Atenolol 

5- Digoxin 


5- Glomerulonephritis 

Answer & Comments 

Answer: 2- Vegetation seen on 

Echocardiogram 

A definite diagnosis of endocarditis (Duke's 
criteria) is achieved when 2 major criteria are 
present, or 1 major and 3 minor criterias. 

Major criteria: 

blood culture positive for typical organisms 


Answer & Comments 

Answer: 4- Atenolol 

The diagnosis is hypertrophic obstructive 
cardiomyopathy as suggested by the signs of 
jerky pulse, ejection systolic murmur and 
echodardiographic findings. In these patients, 
symptoms such as breathlessness is best 
treated with (3-blockers and verapamil either 
alone or in combination. Disopyramide is also 
used second line. 


persistent bacteremia 
positive ECHO for vegetations 
abscess or valve dehiscence 
Minor criteria: 

valvular heart disease or IV drug user 

fever greater than 38°C 

vasculitis 


[ Q: 1408 ] MRCPass - Cardiology 

A 50 year old man has a history of 
myocardial infarction 6 months ago. He has 
had 2 episodes of collapses and a 24 hour tape 
shows episodes of non sustained VT. Coronary 
angiography shows a 99% LAD artery stenosis 
and angioplasty was unsuccessful due to the 
tortuosity of the vessel. Echocardiography 
shows a dilated LV at 5.9cm with an ejection 
fraction of 30%. 



skin lesions 


Which is the most appropriate therapy? 


suggestive ECHO (but not definite) 
positive blood culture 

[ Q: 1407 ] MRCPass - Cardiology 

A 40 year old man presents with a 
history of dyspnoea. On examination he has a 
jerky carotid pulse, an ejection systolic 
murmur is audible at the base of the heart and 
a pan systolic murmur at the apex. 



1- Amiodarone 

2- Atenolol 

3- Permanent pacemaker 

4- Implantable cardiac defibrillator 

5- Stem cell therapy 


Answer & Comments 


Answer: 4- Implantable cardiac defibrillator 


Dr. Khalid Yusuf El-Zohry - Sohag Teaching Hospital (01118391123) 

Ref MRCPass OE OE 2012 PasTest 2009 PassMedicine 2009 PasTest Exam ReviseMRCP 



569 




























El-zohry MRCP Questions Bank (Port 1) - 2013 


(For my personal use) 


This patient satisfies two NICE guidelines for 
ICD insertion. The first is an ejection fraction 
of <30% with syncopal VT. The second is 
syncopal VT following myocardial infarction. 

The MADIT II trial showed that in patients with 
a previous Ml and reduced left ventricular 
ejection fraction (<30%), the prophylactic use 
of an ICD, in addition to medications, 
significantly reduced the risk of death. 


The attending doctor decides to start digoxin. 

Which one of these factors is most important 
in taking into account the appropriate loading 
dose of the drug? 

1- Volume of distribution 

2- Absorption 

3- First pass metabolism 

4- Creatinine clearance 



[ Q: 1409 ] MRCPass - Cardiology 

A 50 year old patient has tearing 
interscapular chest pain. His ECG shows no ST 
changes, but a CT scan with contrast showed 
aortic dissection. His blood pressure is 140/70 
mmHg. 

Which is the most important management? 


1- Intravenous labetalol 


2- Ramipril 

3- Refer for urgent cardiothoracic surgery 

4- Echocardiography 

5- Coronary angiogram 


Answer & Comments 

Answer: 1- Intravenous labetalol 

Immediate management of aortic dissection is 
aggressive blood pressure control. 

Intravenous labetalol or sodium nitroprusside 
can be used. Once this is done, the patient can 
be further assessed with coronary 
angiography or be referred for urgent surgery. 


5- Patient weight 


Answer & Comments 

Answer: 4- Creatinine clearance 

Digoxin is renally excreted. The plasma level of 
the drug and its half life is dependent on the 
volume of distribution and renal clearance. In 
this example, the more important factor is 
renal clearance. A lower loading dose should 
be chosen in patients with renal impairment 
and 6 hour post dose digoxin levels should be 
measured. 


[ Q: 1411 ] MRCPass - Cardiology 

A 55 year old lady has dental phobia 
but has finally gone to the dentist following 
severe toothaches and is told she has dental 
abscesses. She is brought into A+E with high 
fevers by a relative. She is found to have a 
diastolic murmur in the aortic area. 

Which one of these is a sign of endocarditis? 

1- Roslyn's Spot 

2- Hepatomegaly 



[ Q: 1410 ] MRCPass - Cardiology 

A 40 year old man with previous 
history of ischaemic heart disease and Type I 
diabetes presents with heart rate of 150. His 
ECG shows atrial fibrillation. 

His blood tests show Hb 15 g/dl, WCC 10 x 
10 9 /L, Platelets 280 x 10 9 /L, urea 13 pmol/l, 
creatinine 160 pmol/l, sodium 138 mmol/l, 
potassium 3.8 mmol/l. 



3- Loss of peripheral pulses 

4- Pulmonary fibrosis 

5- Blood on urine dipstick 


Answer & Comments 

Answer: 5- Blood on urine dipstick 

Blood on the urine dipstick indicates renal 
vasculitis due to microemboli from 


Dr. Khalid Yusuf El-Zohry - Sohag Teaching Hospital (01118391123) 

Ref MRCPass OE OE 2012 PasTest 2009 PassMedicine 2009 PasTest Exam ReviseMRCP 



570 




























El-zohry MRCP Questions Bank (Port 1) - 2013 


(For my personal use) 


vegetations in endocarditis. Osier's nodes are 
small reddish tender areas on the pulp of 
fingers or toes. Janew ay lesions are reddish 
spots on fingers or toes, but are painless. 
Splenomegaly occurs. Roth's spots are 
vasculitic lesions ont he retina. Bouchard's 
nodes occur in osteoarthritis. 


Dr. Kholid Yusuf El-Zohry - Sohog Teaching Hospital (01118391123) 

Ref MRCPoss OE OE 2012 PosTest 2009 PassMedicine 2009 PosTest Exom 




ReviseMRCP 

571 


L 















El-zohry MRCP Questions Bank (Part 1) - 2013 


(For my personal use) 


^ [ Q: 1412 ] MRCPass - Basic Science 

/ - - -CL--- 

# A 30 year old patient develops 
haemolytic anaemia, which is thought to be 
due to penicillin treatment. 

Which is the correct type of hypersensitivity 
reaction to describe this? 


Answer & Comments 

Answer: 3- Duffy 

Duffy negative patients are resistant to 
developing plasmodium vivax infection. The 
Duffy antigen receptor facilitates entry of 
Plasmodium Vivax into red blood cells. 


1- Type I 

2- Type II 

3- Type III 

4- Type IV 

5- Type V 


[ Q: 1414 ] MRCPass - Basic Science 

A 20 year old man has been new ly 
diagnosed with Gaucher's Disease. This is 
associated with a deficiency of which one of 
the following enzymes? 



Answer & Comments 

Answer: 2- Type II 

Drug induced haemolytic anaemia is due to a 
type II hypersensitivity reaction. The types 
are: 


1- Glucocerebrosidase 

2- Sphingomyelinase 

3- Iduronidase 

4- Hexosaminidase A 

5- Arylsulphatase A 


Type I - Anaphylaxis due to IgE from mast 
cells and involving basophils 

Type II - Cytotoxic due to free antibodies in 
the circulation (usually IgG, IgM and IgA) 
leading to cell lysis due to antigen-antibody 
cross-linking and complement fixation 

Type III - Immune complex deposition 
associated with circulating IgG 


Answer & Comments 

Answer: 1- Glucocerebrosidase 

Gaucher's disease is associated with the 
enzyme glucocerebrosidase. As a result, 
glucocerebroside accumulates, principally in 
the phagocytic cells of the body but also 
sometimes in the central nervous system 
neurones. 


Type IV - Cell-mediated due to interaction 
between T cells and membrane-bound 
antigens 



[ Q: 1413 ] MRCPass - Basic Science 

Which red blood cell antigen is 
involved in entry of Plasmodium vivax into red 
cells ? 

1- Rhesus S 

2- Rhesus D 


3- Duffy 

4- GP24 


5- Kell 


Three types of Gaucher disease are described. 
Common to all three types are the presence 
of hepatosplenomegaly and of large 
glucocerebroside containing 

reticuloendothelial histiocytes, or Gaucher 
cells, in the bone marrow . 

The other associations are: 

Tay Sachs disease - Hexosaminidase A 
deficiency 

Niemann Pick disease - Sphingomyelinase 
deficiency 

Metachromatic leukodystrophy 
Arylsulphatase A deficiency 


Dr. Khalid Yusuf El-Zohry - Sohag Teaching Hospital (01118391123) . 

Ref MRCPass OE OE 2012 PasTest 2009 PassMedicine 2009 PasTest Exam ReviseMRCP 573 






























El-zohry MRCP Questions Bank (Part 1) - 2013 


(For my personal use) 


Hurler's syndrome - Iduronidase deficiency 


[ Q: 1415 ] MRCPass - Basic Science 

Which of the following diseases is X- 
linked inherited? 

1- Alpha 1 antitrypsin deficiency 

2- Haemochromatosis 

3- Marfan's syndrome 

4- G6PD deficiency 

5- Wilson's disease 



Answer & Comments 


Answer: 4- G6PD deficiency 


cystathionine synthase results in 
accumulation of homocysteine methionine. 
Osteoporosis and osteopetrosis are also seen 
in homocystinuria. 


ATP Pi ♦ PPi 


X 



Vitamin 


Cysteine 

Metabolism of Homocysteine 


G6PD deficiency's inheritance is X linked 
recessive. Duchenne muscular dystrophy is 
another example of a disease which has X 
linked recessive inheritance. X linked 
hypophosphataemic rickets is X linked 
dominant. 



[ Q: 1416 ] MRCPass - Basic Science 

A 22 year old male who is tall and 
thin, is found to have a high arched palate, 
downw ard dislocation of lens, chest wall 
deformities and livedo reticularis. 


Which one of the following is also associated 
with this syndrome? 

1- Fibrillin gene defect 

2- Positive Guthrie test 


3- Autosomal Dominant inheritance 


4- Osteopetrosis 

5- Methionine accumulation 


Answer & Comments 

Answer: 5- Methionine accumulation 

The diagnosis is homocystinuria (Marfan's 
causes upw ards lens dislocation and fibrillin 
gene defect). Homocystinuria is an autosomal 
recessive disorder. Reduced activity of 



[ Q: 1417 ] MRCPass - Basic Science 
A postgraduate student is studying 


HIV replication. 


Which of the following is important in the 
replication or transmission of HIV-1 ? 


1- Trypsin 


2- Bax 


3- GP 120 

4- P53 

5- MHC 


Answer & Comments 

Answer: 3- GP 120 

HIV reverse transcriptase, integrase and 
protease are key enzymes essential for HIV 
replication. The HIV genome contains the 
genes: tat and rev along with nef, env, gag 
and pol. The GP 120 is the major protein on 
the surface of HIV that interacts with host 
cells. HIV binds to cell surface CD4 but enters 
cells through chemokine receptors including 
CXCR4 and CCR5. Thymidine kinase is 
produced by the herpes simplex virus. 


Dr. Khalid Yusuf El-Zohry - Sohag Teaching Hospital (01118391123) 

Ref MRCPass OE OE 2012 PasTest 2009 PassMedicine 2009 PasTest Exam ReviseMRCP 



574 













































El-zohry MRCP Questions Bank (Port 1) - 2013 


(For my personal use) 



C134+ lymphocyte 
or macrophage 


[ Q: 1418 ] MRCPass - Basic Science 

/ - 

Which of the following anatomical 
structures , if damaged , leads to choreiform 
movement abnormalities? 

1- Caudate nucleus 

2- Subthalamic nucleus 

3- Substantia nigra 

4- Hippocampus 

5- Corpus callosum 


Answer & Comments 

Answer: 1- Caudate nucleus 

Caudate nucleus, putamen and globus 
pallidus are areas within the basal ganglia 
which, when impaired, can lead to choreiform 
movements. Subthalamic nucleus lesion 
causes hemiballismus. Hippocampus 
involvement can cause memory loss. 




[ Q: 1419 ] MRCPass - Basic Science 

A 35 year old man presents with 
bleeding oesophageal varices. 


On examination he has Kayser- Fleischer rings 
in the cornea. The inheritance of this disorder 
is: 


1- Polygenic inheritance 

2- X-linked dominant 

3- X-linked recessive 

4- Autosomal recessive 

5- Autosomal dominant 


Answer & Comments 

Answer: 4- Autosomal recessive 

The diagnosis is Wilson's disease, which has 
autosomal recessive inheritance. 



Kayser Flescher Ring in Wilson's disease 



[ Q: 1420 ] MRCPass - Basic Science 



Dr. Khalid Yusuf El-Zohry - Sohag Teaching Hospital (01118391123) 

Ref MRCPass OE OE 2012 PasTest 2009 PassMedicine 2009 PasTest Exam ReviseMRCP 




























El-zohry MRCP Questions Bank (Part 1) - 2013 


(For my personal use) 


Which of these is the correct description of 

peptide bonds? 

1- Bonds between 2 carboxylic acid groups of 
amino acids 

2- Hydrogen bonds between 2 side chains of 
amino acids 

3- Bonds between the carboxylic acid group 
of one amino acid and the amino group of 
the next 

4- Bonds between alternating purine and 
pyrimidine molecules 

5- Covalent bonds between two amino acids 


4- 10% 

5- 5% 

Answer & Comments 

Answer: 4-10% 

The virus can be found in other cell types 
apart from hepatocytes, such as renal tubular 
cells. 10% of patients with hepatitis B develop 
chronic infection as compared to hepatitis C 
where 80% develop chronic infection. The 
likelihood of virus clearance increases with 
better cell mediated immune responses. 


Answer & Comments 

Answer: 3- Bonds between the carboxylic acid 
group of one amino acid and the amino group 
of the next 


^ [ Q: 1422 ] MRCPass - Basic Science 

# A 25 year old man with cystic 
fibrosis is referred for investigation of poor 
fertility. 


Peptide bonds are linkages between the 
carboxylic acid (COOH) group of one amino 
acid and the amino (NH2) group of the next 
amino acid. 


H 


H 


\ 


H 


/ 


N-C-C 




O H 


H 


\ 


N-C-C 




H 


\ / 

OH H 


\ 


H 


O 

OH 


H 


H O H H 


\ 


H 


/ 


N-C-C-N-C—C 




\ 


H 


H 


O 

OH 


Peptide Bond 


What is likely to have caused this? 

1- Malabsorption and vitamin deficiency 

2- Abnormal oestrogen metabolism 

3- Acidification of seminal fluid 

4- Hypopituitarism 

5- Failure of development of vas deferens 


Answer & Comments 

Answer: 5- Failure of development of vas 
deferens 

Infertility in cystic fibrosis is due to 
maldevelopment of vas deferens. 


[ Q: 1421 ] MRCPass - Basic Science 

A 45 year old man with a new 
diagnosis of hepatitis B is keen to know more 
about the prognosis of the disease. 

What percentage of patients is likely to 
develop chronic infection? 

1 - 100 % 

2- 80% 

3- 50% 




[ Q: 1423 ] MRCPass - Basic Science 

A child has an endocrine condition 
which has caused short stature. 

Which of the following conditions may be 
detectable by growth monitoring? 

1- Thyrotoxicosis 

2- Pseudohypoparathyroidism 

3- Hypothyroidism 

4- Insulin dependent diabetes mellitus 


Dr. Khalid Yusuf El-Zohry - Sohag Teaching Hospital (01118391123) 

Ref MRCPass OE OE 2012 PasTest 2009 PassMedicine 2009 PasTest Exam ReviseMRCP 



576 


































El-zohry MRCP Questions Bank (Part 1) - 2013 


(For my personal use) 


5- XYY Syndrome 

Answer & Comments 

Answer: 3- Hypothyroidism 

Growth monitoring can aid detection of the 
following conditions: 


1- None of the sons of an affected woman 

2- Half of the daughters of an affected 
woman 

3- All of the sons of an affected woman 

4- All children of an affected woman 

5- All children of an affected man 


hypothyroidism 

growth hormone insufficiency 

Turners & Noonan's syndrome, skeletal 
dysplasias 

coeliac disease, inflammatory bow el disease 
intracranial tumours 



[ Q: 1424 ] MRCPass - Basic Science 

A 35 year old lady has psychogenic 
diabetes insipidus. Her blood results show - 
sodium 124 mmol/I; potassium 4.0 mmol/l; 
Urea 5 mmol/l; Creatinine 30 mmol/l; Glucose 
8 mmol/l. 


What is the serum osmolality [mosmol/Kg]? 


1- 255 

2- 261 

3- 264 

4- 278 

5- 284 


Answer & Comments 

Answer: 2- 261 

Serum osmolality is 2 X sodium + Urea + 
Glucose = 248 + 5 + 8 = 261 mOsmol/Kg. 
Normal is 280 to 305 hence she has low 
serum osmolality. An alternative formula is 
1.9 x (Na + K) + Ur + Glucose. 



[ Q: 1425 ] MRCPass - Basic Science 


Which one of the following set of 


offsprings would a X-linked dominant 
condition be transmitted to? 


Answer & Comments 

Answer: 2- Half of the daughters of an 
affected woman 

The classical example of an X-linked dominant 
inherited condition is familial 
hypophosphataemic rickets. Remember to 
differentiate between X linked recessive and 
X linked dominant. 

An affected woman has the chromosomes 
XxX and will transmit this to half of her 
daughters who will either have 

XX or XxX chromosomes. Similarly, half of the 
sons will have the disease XxY or XY 
chromosomes. 



Unaffected Affected Affected Unaffected 


son daughter son daughter 

X linked dominant inheritance with an 
affected mother 


Dr. Khalid Yusuf El-Zohry - Sohag Teaching Hospital (01118391123) . 

Ref MRCPass OE OE 2012 PasTest 2009 PassMedicine 2009 PasTest Exam ReviseMRCP 577 





































El-zohry MRCP Questions Bank (Part 1) - 2013 


(For my personal use) 



[ Q: 1426 ] MRCPass - Basic Science 

Which one of these conditions is an 
Autosomal Dominant disorder? 

1- Ataxia Telangiectasia 

2- Achondroplasia 

3- Alkaptonuria 

4- Cystic Fibtosis 

5- Phenylketonuria 


7 


[ Q.: 1427 ] MRCPass - Basic Science 
What is the effect of leptin on body 


metabolism? 

1- Decreases energy usage 

2- Stimulates neuropeptide Y 

3- Increases appetite 

4- Stimulates pancreatic proteases 

5- Reduces adipose tissue mass 


Answer & Comments 


Answer: 2- Achondroplasia 


Answer & Comments 


Answer 5- Reduces adipose tissue mass 


The list of autosomal dominant disorders are: 

■ achondroplasia 

■ antithrombin III deficiency 

■ Ehlers-Danlos syndrome 

■ Familial hypercholesterolaemia 
Gilbert's disease 

■ hereditary haemorrhagic 

telangiectasia 

hereditary elliptocysis, hereditary 

spherocytosis 

Huntington's disease 

■ idiopathic hypoparathyroidism 

■ intestinal polyposis 

■ marble bone disease 

■ Marfan's syndrome 

■ neurofibromatosis 

■ polycystic kidney disease (adult) 

■ protein C deficiency 

■ osteogenesis imperfecta 
Treacher Collins syndrome 

■ tuberous sclerosis 

Von Willebrand's disease 


Leptin is expressed by adipocytes and 
mediate effects via the hypothalamic centre 
which controls hunger and energy 
expenditure. It inhibits neuropeptide Y (which 
is a potent appetite stimulator), increases 
oxygen consumption, body temperature and 
reduces adipose tissue mass. 


Reproduction 

Growth 


-■PL 


Hypothalamus 
LRb 





\ 


--E 


Corticosteroids 

4 

y 

Leptin 
resistance 


Metabolic rate. 

th i TOld ' immune 

sympathetic tone 

I k 


B 


Leptin | 

t 


g Feeding 


k Insulin 
sensitivity 


<3 

Adipocyte^v 
Fat storage . 1 


4 f 


[ Q: 1428 ] MRCPass - Basic Science 

A 65 year old man has a parietal 
lobe infarct. 

Which one of the following is a likely 
manifestation? 

1- Finger agnosia 

2- Homonymous hemianopia 

3- Bitemporal hemianopia 

4- Expressive dysphasia 

5- Dysdiadochokinesis 



Dr. Khalid Yusuf El-Zohry - Sohag Teaching Hospital (01118391123) 

Ref MRCPass OE OE 2012 PasTest 2009 PassMedicine 2009 PasTest Exam ReviseMRCP 

































El-zohry MRCP Questions Bank (Part 1) - 2013 


(For my personal use) 


Answer & Comments 

Answer: 1- Finger agnosia 
Parietal lobe signs are: 

■ loss of two point discrimination 
agraphia 

■ finger agnosia 
astereognosis 

■ dyslexia 
Gerstmann syndrome 
receptive dysphasia 

dressing and constructional dyspraxia 


3- Metabolic acidosis with respiratory 
compensation 

4- Metabolic alkalosis with respiratory 
compensation 

5- Normal anion gap metabolic acidosis 

Answer & Comments 

Answer: 4- Metabolic alkalosis with 

respiratory compensation 

This patient is bulimic and has lost HCI from 
excessive vomiting. She has metabolic 
alkalosis which is compensated by 
hypoventilation (hence low p02 and high 
C02). 


Gerstmann syndrome includes four features 
(acalculia, agraphia, finger agnosia, left right 
disorientation), and is due to a lesion in the 
dominant hemisphere. 



discriminate different fingers) 


[ Q: 1429 ] MRCPass - Basic Science 

An 18 year old ballet dancer 
presents to A&E unwell. She has poor 
dentition. 

Her arterial pH is 7.44 and she has a pCCb of 6 
kPa and p02 of lOkPa. Her chloride is 85 (95- 
107) mmol/l, HCO3 is 30 (20-28) mmol/I. 

Which of the following describes her acid base 
balance? 


1- Respiratory 
compensation 

acidosis 

with 

metabolic 

2- Respiratory 
compensation 

alkalosis 

with 

metabolic 



[ Q: 1430 ] MRCPass - Basic Science 

Which one of following biochemical 
abnormalities would be most commonly seen 
in a diagnosis of Bartter's syndrome? 

1- Hypochloraemia 

2- Hyperkalemia 

3- Hyponatraemia 

4- Hypokalemia 

5- Acidosis 



Answer & Comments 

Answer: 4- Hypokalemia 

Bartter's syndrome is chracterised by: 

hypokalaemic alkalosis, elevated renin and 
aldosterone levels 


[ Q: 1431 ] MRCPass - Basic Science 

A man who has common variable 
immunodeficiency comes to see the doctor in 
the genetics clinic with his girlfriend. They 
would like to start a family. 

What is the probability that his children will 
inherit his disease? 

1 - < 10 % 



Dr. Khalid Yusuf El-Zohry - Sohag Teaching Hospital (01118391123) 

Ref MRCPass OE OE 2012 PasTest 2009 PassMedicine 2009 PasTest Exam ReviseMRCP 





































El-zohry MRCP Questions Bank (Part 1) - 2013 


(For my personal use) 


2- 25% 

3- 50% 

4- 75% 

5- 100% 


Answer & Comments 

Answer: 1- <10% 

Common Variable Immunodeficiency (CVID) is 
a disorder characterized by low levels of 
serum immunoglobulins and an increased 
susceptibility to infections. A clear mode of 
inheritance is not defined (there are multiple 
modes) and hence there is a <10 % chance of 
passing on the disease. 


[ Q: 1432 ] MRCPass - Basic Science 

fy - 

m Transcription RNA (tRNA) has three 

bases specific to a particular amino acid, 
which it binds to messenger RNA (mRNA). 

This specific area of tRNA known as: 

1- Codon 

2- Intron 

3- Anticodon 

4- Transposon 

5- Exon 


Answer & Comments 

Answer: 3- Anticodon 

Transfer RNA (tRNA) is a small RNA chain (74- 
93 nucleotides) that transfers a specific amino 
acid to a grow ing polypeptide chain at the 
ribosomal site of protein synthesis during 
translation. It has sites for amino-acid 
attachment and codon (a particular sequence 
of 3 bases) recognition. The codon 
recognition is different for each tRNA and is 
determined by the anticodon region, which 
contains the complementary bases to the 
ones encountered on the mRNA. Each tRNA 
molecule binds only one type of amino acid, 
but because the genetic code is degenerate, 


more than one codon exists for each amino 
acid. 


Amino Acid — — 

n n n n 3 

tRNA 


I 1 1 I 1 s 1 "1 

- - H 5 n —I 


Anticodon 

Codon 


5 ' 


3 1 


mRNA 


Anticodon and Codon 


[ Q: 1433 ] MRCPass - Basic Science 

Which one of these organelles have 
DNA which can self replicate? 

1- Golgi apparatus 

2- Nuclear membrane 

3- Mitochondria 

4- Peroxisomes 

5- Transcription factors 



Answer & Comments 

Answer: 3- Mitochondria 

Only mitochondria have self replicating DNA 


[ Q: 1434 ] MRCPass - Basic Science 

A 25 year old woman was referred 
for investigation of iron deficiency anaemia. 
Her mother died aged 35 years from colonic 
carcinoma and had Peutz Jegher syndrome. 

Which is the likely mode of inheritance of 
Peutz J eg hers syndrome? 

1- Autosomal dominant 

2- Autosomal recessive 

3- X linked dominant 

4- X linked recessive 

5- Mitochondrial 



Answer & Comments 


Answer: 1- Autosomal dominant 


Dr. Khalid Yusuf El-Zohry - Sohag Teaching Hospital (01118391123) 

Ref MRCPass OE OE 2012 PasTest 2009 PassMedicine 2009 PasTest Exam ReviseMRCP 



580 












































El-zohry MRCP Questions Bank (Part 1) - 2013 


(For my personal use) 


Peutz Jegher syndrome is an autosomal 
dominant condition associated with 
pigmentation, mainly, of the lips, buccal 
mucosa, genitalia, hands and feet. 

In addition, there are multiple 
hamartogenous polyps of the gastrointestinal 
tract - most often in the small bow el but may 
occur affect any portion of the Gl tract.The 
polyps themselves have a very low malignant 
potential. About 10-20% of patients develop 
gastrointestinal carcinoma, but there is also 
increased risk of pancreatic, lung and breast 
carcinoma. 



Mucosal Pigmentation in Peutz Jegher 

Syndrome 


[ Q: 1435 ] MRCPass - Basic Science 

The main histological abnormality in 
Huntington's disease is seen in the: 

1- Caudate lobe 

2- Midbrain 

3- Red nucleus 

4- Inferior colliculus 

5- Hippocampus 



Answer & Comments 

Answer: 1- Caudate lobe 

The main histological abnormality in 
Huntington's chorea is seen in the caudate 
lobe and putamen where there is extensive 
neuronal loss. 



Caudate nucleus / lobe 


[ Q: 1436 ] MRCPass - Basic Science 

a 

,_ At sympathetic nerve endings what 

happens to the majority of amount of 
noradrenaline following depolarisation ? 

1- Reuptake by the nerve terminal ending 

2- Breakdown by monoamine oxidase 

3- Conversion to adrenaline 

4- Stays at the nerve junction 


5- Recycled by catechol-0 methyl transferase 


Answer & Comments 

Answer: 1- Reuptake by the nerve terminal 
ending 

Most noradrenaline is taken up back into 
neurosecretory granules. MAO and COMT 
metabolise NA in small amounts. 


% 


[ Q: 1437 ] MRCPass - Basic Science 
A 62 year old man with epigastric 


pains and weight loss is suspected of having 
pancreatic carcinoma. 

Which one of the following markers is useful? 


1- Alpha feto protein 


2- CA 19.9 

3- CA 125 


4- Prostate specific antigen 

5- Carcinoembryonic antigen 


Dr. Khalid Yusuf El-Zohry - Sohag Teaching Hospital (01118391123) 

Ref MRCPass OE OE 2012 PasTest 2009 PassMedicine 2009 PasTest Exam ReviseMRCP 




























El-zohry MRCP Questions Bank (Part 1) - 2013 


(For my personal use) 


Answer & Comments 

Answer: 2- CA 19.9 

CA 19.9 is found in 70-90% of patients with 
pancreatic carcinoma. CA125 and CEA may 
also be positive but are less frequently so. 

[ Q: 1438 ] MRCPass - Basic Science 

The RET Proto-oncogene is 
associated with which one of the following? 

1- Insulinoma 

2- Anaplastic thyroid carcinoma 

3- Bronchial carcinoma 

4- Medullary thyroid carcinoma 

5- Pituitary tumour 


^ [ Q: 1440 ] MRCPass - Basic Science 

/ --—_ —L- 1 — 

# A 62 year male presents with 
bilateral neuropathic leg pains. There was 
relevant past history of excess alcohol use. He 
is currently on oral thiamine and omeprazole. 
On examination, both knee reflexes are 
reduced and there is reduced sensation to 
fine touch in both feet. 

What is the next best investigation to confirm 
the diagnosis? 

1- EMG 

2- Chest X ray 

3- CT head 

4- CT spine 

5- Blood sugar 


Answer & Comments 

Answer: 4- Medullary thyroid carcinoma 

The RET proto-oncogene is associated with 
multiple endocrine neoplasia MEN2A, MEN2B 
and medullary thyroid carcinoma 1, 
Hirschsprung disease. 


Answer & Comments 

Answer: 5- Blood sugar 

The clinical features are consistent with a 
diagnosis of peripheral neuropathy and 
although he has another possible cause due 
to alcohol, diabetes needs to be excluded. 



[ Q: 1439 ] MRCPass - Basic Science 

A young man presents with fevers 
and has several blood tests. 

Which one of the following, if increasedl 
suggests infection? 

1- Albumin 

2- Transferrin 


3- Ferritin 


4- Alpha 2-macroglobulin 

5- Leptin 


Answer & Comments 


Answer: 3- Ferritin 


[ Q: 1441 ] MRCPass - Basic Science 

A 65 year old Mediterranean man 
who has leg cramps is started on quinine. He 
presents 10 days later, with a history of 
darkened urine, increasing breathlessness, 
back pains and fatigue. Investigations show a 
haemoglobin of 6.5 g/dl and raised 
reticulocyte count. 

Which of the following best explains this drug 
reaction ? 

1- Hereditary spherocytosis 

2- Hereditary elliptocytosis 

3- Autoimmune haemolytic anaemia 

4- Pyruvate kinase deficiency 



Ferritin is well known as an acute phase 
protein which is increased in sepsis. 


5- Glucose 6 phosphate dehydrogenase 
deficiency 


Dr. Khalid Yusuf El-Zohry - Sohag Teaching Hospital (01118391123) 

Ref MRCPass OE OE 2012 PasTest 2009 PassMedicine 2009 PasTest Exam ReviseMRCP 



582 

































El-zohry MRCP Questions Bank (Part 1) - 2013 


(For my personal use) 


Answer & Comments 

Answer: 5- Glucose 6 phosphate 

dehydrogenase deficiency 

Glucose 6 phosphate dehydrogenase 
deficiency (X linked recessive) is seen in 
African, Mediterranean, Iraqi, Jew and South 
East Asian Chinese people. 

It predisposes to a haemolytic anaemia 
reaction to drugs and infection. Implicated 
drugs include aspirin, sulphonamides, 
antimalarials and quinidine. 



Blood film showing haemolysis in G6PD 

deficiency 


[ Q: 1443 ] MRCPass - Basic Science 

# A 60 year old man has numbness 
and tingling of the upper outer part of the left 
thigh. On examination, there is sensory 
impairment over the anterolateral aspect of 
the thigh. 

Where is the lesion? 


1- Sacral nerve 


2- Lateral cutaneous nerve of thigh 

3- Pudendal nerve 

4- Femoral nerve 


5- Sciatic nerve 


Answer & Comments 

Answer: 2- Lateral cutaneous nerve of thigh 

The diagnosis is meralgia paraesthetica, an 
entrapment neuropathy of the lateral 
cutaneous nerve of the thigh as it passes 
under the inguinal ligament. Obesity is a risk 
factor for the condition. 



[ Q: 1442 ] MRCPass - Basic Science 

A 40 year old female presents with a 
movement disorder. There is a family history 
of Huntington's chorea and the family is 
worried that she may be developing this 
condition. 

What is its inheritance? 


1- Autosomal recessive 


2- Autosomal dominant 

3- Mitochondrial inheritance 


4- X linked recessive 

5- X linked dominant 


Answer & Comments 

Answer: 2- Autosomal dominant 

Huntington's chorea is inherited as an 
autosomal dominant trait. 




[ Q: 1444 ] MRCPass - Basic Science 


Dr. Khalid Yusuf El-Zohry - Sohag Teaching Hospital (01118391123) 

Ref MRCPass OE OE 2012 PasTest 2009 PassMedicine 2009 PasTest Exam ReviseMRCP 































El-zohry MRCP Questions Bank (Part 1) - 2013 


(For my personal use) 


Which of the following haematological 
disorders is inherited as an autosomal 
recessive condition? 

1- Acute intermittent porphyria 

2- Antithrombin III deficiency 

3- Pyruvate kinase deficiency 

4- Glucose 6 phosphate dehydrogenase 
deficiency 

5- Protein C deficiency 


Answer & Comments 

Answer: 3- Pyruvate kinase deficiency 

Pyruvate kinase deficiency is a rare congenital 
haemolytic anaemia inherited as an 
autosomal recessive manner. 

The other condition's inheritance are: 

■ Acute intermittent porphyria 
autosomal dominant 

■ Antithrombin 3 (AT3) - autosomal 
dominant 

Glucose 6 phosphate dehydrogenase 
deficiency - X linked recessive 

■ Protein C deficiency - autosomal 
dominant 


[ Q: 1445 ] MRCPass - Basic Science 

A 22 year old lady has had several 
episodes w heezing with associated flushing 
of the face, lips and hand swelling over the 
past few years. 

What investigation should be done? 

1- Cl esterase inhibitor level 

2- Skin patch test with latex 



Answer & Comments 

Answer: 1- Cl esterase inhibitor level 

Hereditary angioneurotic oedema is an 
autosomal dominantly inherited condition 
caused by a deficiency of Cl esterase 
inhibitor. The main clinical feature is the 
intermittent oedema in the skin around the 
face, hands, feet, larynx and gastrointestinal 
tract. Laryngeal oedema may cause wheezing. 
The C2 and C4 level are low in between 
attacks and C3 is normal. There are 2 varieties 
of the disorder. Type I is most common and 
results from an abnormally low level of 
normal Cl-INH. Type II results from normal or 
abnormally elevated levels of a dysfunctional 
Cl-INH. In both types of the disease, initial 
proteolytic components of the complement 
cascade (eg, Clr, Cls) go relatively 
unopposed and lead to the characteristic 
presentation and laboratory abnormalities 
(eg, low levels of C2 and C4). 




Swollen lip in hereditary angioedema 

[ Q: 1446 ] MRCPass - Basic Science 

A 32 year old lady is admitted with a 
history of epistaxis. Investigations reveal iron 
deficiency anaemia. On examination, multiple 
telangiectasia are noted around her lips and 
in her mouth. 




3- Skin prick test with latex 

4- Serum ANCA 

5- Rheumatoid factor 


What is the mode of inheritance for the 
condition? 

1- Polygenic inheritance 

2- Autosomal recessive 


Dr. Khalid Yusuf El-Zohry - Sohag Teaching Hospital (01118391123) 

Ref MRCPass OE OE 2012 PasTest 2009 PassMedicine 2009 PasTest Exam ReviseMRCP 



584 






























El-zohry MRCP Questions Bank (Part 1) - 2013 


(For my personal use) 


3- Autosomal dominant 

4- Autosomal dominant X-linked recessive 

5- X-linked dominant 

Answer & Comments 

Answer: 3- Autosomal dominant 

The patient has the features of hereditary 
haemorrhagic telangiectasia (Osler-Rendu- 
Weber syndrome) which has autosomal 
dominant inheritance. 




Hereditary haemorrhagic telangiectasia 


The karyotype 46 XX, t (4;8)(q26;p21.3) 
describes a female with a normal number of 
chromosomes but a translocation between 
the long arm of chromosome 4 (q) and the 
short arm of chromosome 8 (p). 

[ Q: 1448 ] MRCPass - Basic Science 

A 28 year lady presents multiple 
cafe au lait spots. A diagnosis of 
neurofibromatosis type 1 made. 

Which of the following is true regarding the 
NF1 gene's inheritance and location? 

1- Inherited in an autosomal recessive fashion 

2- Inherited in an X linked fashion 

3- Found on chromosome 17 

4- Found in the mitochondrial genome 

5- Is identical to the NF2 gene 


7 \ 

[ Q: 1447 ] MRCPass - Basic Science 


\fi 

Which of the following statements 

Answer & Comments 


describes this karyotype 46 XX, t 
(4;8)(q26;p21.3)? 

1- Transversion between the long arm of 
chromosome 4 (q) and the short arm of 
chromosome 8 (p) 

2- Transversion between the short arm of 
chromosome 4 (q) and the short arm of 
chromosome 8 (p) 

3- Transversion between the long arm of 
chromosome 4 (p) and the short arm of 
chromosome 8 (q) 

4- Translocation between the short arm of 
chromosome 4 (q) and the long arm of 
chromosome 8 (p) 

5- Translocation between the long arm of 
chromosome 4 (q) and the short arm of 
chromosome 8 (p) 

Answer & Comments 

Answer: 5- Translocation between the long 
arm of chromosome 4 (q) and the short arm 
of chromosome 8 (p) 



Answer: 3- Found on chromosome 17 

Neurofibromatosis can be due to a defect of 
either the NF1 or NF2 gene. 

The NF1 gene is found on chromosome 17 
inherited in an autosomal dominant fashion. 

The NF2 gene is found on Chromosome 22. 

[ Q: 1449 ] MRCPass - Basic Science 

Regarding two loci A and B, which 
are in linkage disequilibrium, which one of the 
following statements is true? 

1- The inheritance of an allele at A will almost 
certainly exclude the inheritance of one of 
the alleles at B 

2- The degree of linkage disequilibrium can be 
highly variable 

3- The four alleles at A and B are inherited 
independently provided that the 
population is of sufficient size 

4- The loci A and B are not linked 


Dr. Khalid Yusuf El-Zohry - Sohag Teaching Hospital (01118391123) . 

Ref MRCPass OE OE 2012 PasTest 2009 PassMedicine 2009 PasTest Exam ReviseMRCP 585 






























El-zohry MRCP Questions Bank (Part 1) - 2013 


(For my personal use) 


5- It is a random association of alleles in a 
breeding population 


Answer & Comments 

Answer: 2- The degree of linkage 

disequilibrium can be highly variable 

Linkage disequilibrium is a non-random 
association of alleles in a breeding 
population. The loci A and B are likely to be 
linked. Hence inheritance of an allele A 
usually occurs with the inheritance of allele B 
(rather than excluded). The inheritance is still 
dependent despite population size. Linkage 
disequilibrium almost alw ays occurs between 
alleles at genetic loci that are closely linked in 
the genome. The degree of linkage 
equilibrium can how ever, be highly variable. 


^ [ Q: 1450 ] MRCPass - Basic Science 

A ---- 

0 McArdle's disease is a genetic defect 
in the phosphorylase enzyme, which affects 
the breakdown of glycogen. 

What is glycogen made up of? 

1- Chains of glucose residues 

2- Chains of fructose residues 

3- Chains of sucrose residues 

4- Chains of galactose residues 

5- Chains of alternating galactose and glucose 
residues 


Answer & Comments 

Answer: 1- Chains of glucose residues 

The structure of glycogen consists of long 
polymer chains of glucose units connected by 
an alpha acetal linkage. Chains of glucose 
residues are linked in glycogen by alpha 1,4- 
glycosidic bonds (i.e. between the first carbon 
atom Cl of one glucose and the fourth carbon 
atom C4 of the next). 



Glycogen - polymer of glucose molecules 



[ Q: 1451 ] MRCPass - Basic Science 

A patient with Angelman's 
syndrome expresses genomic imprinting. 


What does this mean? 


1- That two genes are inherited together 

2- That a gene is mitochondrially inherited 

3- The severity of a disease worsens from 
generation to generation 

4- That one allele of a gene is not expressed 

5- The differential expression of alleles is 
dependent on their parental origin 


Answer & Comments 

Answer: 5- The differential expression of 
alleles is dependent on their parental origin 

Genomic imprinting is the term used to refer 
to the differential expression of alleles 
dependent on their parental origin. 

An example is when the same gene having 
different phenotypic expression is due to 
either maternal inheritance (e.g. Prader-Willi 
syndrome) or due to paternal inheritance 
(e.g. Angelman's syndrome). 



Genomic Imprinting - the same gene has 
different phenotypic expression depending 
on w hether it is maternally or paternally 

inherited 


Dr. Khalid Yusuf El-Zohry - Sohag Teaching Hospital (01118391123) 

Ref MRCPass OE OE 2012 PasTest 2009 PassMedicine 2009 PasTest Exam 



' i 

ReviseMRCP 


586 


_ s 
















































El-zohry MRCP Questions Bank (Port 1) - 2013 


(For my personal use) 


^ [ Q: 1452 ] MRCPass - Basic Science 

A ■--- 

# A 25 year old patient with acquired 
Factor VIII deficiency was given a monoclonal 
antibody drug following an episode of severe 
bleeding. 

Which one of these is the likely drug? 

1- Prednisolone 

2- Cyclosporin 

3- Enoxaparin 

4- Rituximab 

5- Cyclophosphamide 


Answer & Comments 

Answer: 4- Rituximab 

Rituximab is a chimeric, human IgGl 
monoclonal antibody specific for the CD20 
antigen expressed on the surface of B 
lymphocytes. The antibody is known to 
induce rapid in vivo depletion of both normal 
B lymphocytes and lymphoma B cells. The 
drug's limited toxicity has led to the recent 
use of rituximab for the treatment of 
autoimmune disorders, anticipating a 
decrease in antibody production by CD20+ B 
cells. Examples include ITP, autoimmune 
hemolytic anaemia, and acquired hemophilia 
A (factor VIII deficiency). 

Monoclonal antibodies are made by fusing a 
mouse B cell with myeloma cell line. 
Antibodies can be purified and cell lines are 
grown in vitro. The antibodies can be used to 
measure hormone levels with immunoassays. 




Tumor cells ere injected into a mouse to 
stimluate production of B cells , which produce 
different types of antitumor antibodies. 


2. Immortalized 
myeloma cells 
are collected. 



Myeloma cell 


Hybrid 


4. The hybrid cell that 
produces the needed 
antibody is selected 
and cloned to produce 
unlimited quantities of 
a monoclonal antibody. 



3. The B cells are fused 
with the myeloma cells 
to produce immortalized, 
antibody-producing 
hybrid cells. 


Producing Monoclonal Antibodies 


[ Q: 1453 ] MRCPass - Basic Science 

A 55 year old carpenter has pain in 
his shoulder. On examination, there was pain 
during resistance of abduction. 

Which muscle is likely to be involved? 

1- Infraspinatus 

2- Supraspinatus 

3- Pectoralis major 

4- Teres minor 

5- Latissimus dorsi 



Answer & Comments 

Answer: 2- Supraspinatus 

Supraspinatus tendonitis is also known as 
rotator cuff syndrome. Supraspinatus 
tendinitis typically affects patients between 
40 and 60 years of age following prolonged or 
excessive use of the shoulder. Pain is usually 
more severe but of shorter duration in 
younger patients due to a more vigorous 
repair process. Pain is felt in the shoulder and 
over the deltoid muscle but there is no 
obvious outw ard sign of inflammation or 
swelling. There is pain on active or resisted 
abduction. 



[ Q: 1454 ] MRCPass - Basic Science 
A 25 year old man has the condition 


ME LAS. 


Dr. Kholid Yusuf El-Zohry - Sohog Teaching Hospital (01118391123) 

Ref MRCPass OE OE 2012 PasTest 2009 PassMedicine 2009 PasTest Exam ReviseMRCP 






























El-zohry MRCP Questions Bank (Part 1) - 2013 


(For my personal use) 


Which one of the following is a feature of the 
disease? 

1- Cardiac arrhythmia 

2- Colour blindness 

3- Hirsutism 

4- Lactic acidosis 


Answer & Comments 

Answer: 5- Musculocutaneous nerve 

The musculocutaneous nerve suppleis the 
biceps, coracobrachialis and brachialis 
muscles. It also supplies sensation over the 
lateral aspect of the forearm. 


5- Ketoacidosis 

Answer & Comments 

Answer: 4- Lactic acidosis 

Mitochondrial disorders such as MELAS 
(Mitochondrial myopathy, encephalopathy, 
lactic acidosis, stroke) and MERRF have 
muscle, brain, nerve and pancreatic 
involvement. With pancreatic and muscle 
involvement, diabetes and lactic acidosis can 
occur, but ketoacidosis is infrequent. 



Scattered abnormal, vacuolated fibers with 
clear rim in MELAS: H & E stain 



[ Q: 1455 ] MRCPass - Basic Science 

Which nerve lesion causes weakness 
of biceps, coraco-brachialis and brachialis and 
sensory loss over the lateral aspect of the 
forearm ? 

1- Radial nerve 


2- Brachial nerve root 


3- Axillary nerve 

4- C5 and C6 root 

5- Musculocutaneous nerve 


Musculocutaneous Nerve 



formed from lateral 
cord of brachial plexus 


pierces coracobrachialis 
then lies between biceps 
and brachialis 

becomes lateral cutaneous 
nerve of forearm at elbow 


[ Q: 1456 ] MRCPass - Basic Science 

A 35 year old male presents with 
oral and genital mucocutaneous ulcerations. 
He also has associated polyarthritis affecting 
the lower limbs. He is currently on an recent 
episode of pulmonary embolism. 

Which of the genetic association is common 
with such a presentation? 

1- HLA A3 

2- HLADR2 

3- HLADR3 

4- HLA B27 

5- HLA B5 



Answer & Comments 

Answer: 5- HLA B5 

The features of genital and oral ulceration, as 
well as prothrombotic states are consistent 
with Behcet's disease. Familial occurrence has 
been reported and it seems to occur in 



Dr. Khalid Yusuf El-Zohry - Sohag Teaching Hospital (01118391123) 

Ref MRCPass OE OE 2012 PasTest 2009 PassMedicine 2009 PasTest Exam ReviseMRCP 































El-zohry MRCP Questions Bank (Part 1) - 2013 


(For my personal use) 


patients from eastern Mediterranean 
countries and Japan. The disease appears to 
be linked to HLA-B5, HLA -B51 and HLA-DR5 
alleles. 





[ Q: 1458 ] MRCPass - Basic Science 

Which of the following anatomical 
structures is likely to cause chorea , if 
damaged? 


1- Hippocampus 

2- Subthalamic nucleus 


3- Thalamus 


4- Caudate nucleus 


5- Substantia nigra 


Answer & Comments 

Answer: 4- Caudate nucleus 

Damage to the caudate nucleus is most likely 
to cause chorea. This is involved in 
Huntington's chorea. 


Oral Ulceration in Behcet's Disease 


[ Q: 1457 ] MRCPass - Basic Science 


A 65 year old man was admitted 
with an exacerbation of chronic obstructive 
pulmonary disease. 


His arterial blood gases on air showed pH 
7.29, paC0 2 8.5 kPa, pa0 2 8.0 kPa, and 
standard bicarbonate 30.5 mmol/l. 


What is the acid-base disturbance? 


VL nucleus of 
thalamus 



Basal ganglia 
and associated 
structures 


Caudate 

nucleus 


Putamen 


Substantia 

nigra 


Globus 

patlidus 


Subthalamic 

nucleus 


Striatum 


1- Metabolic alkalosis 

2- High anion gap metabolic acidosis 

3- Normal anion gap metabolic acidosis 

4- Respiratory alkalosis 

5- Respiratory acidosis 


Answer & Comments 

Answer: 5- Respiratory acidosis 

This patient had an acidosis with a high 
PaC02 and normal standard bicarbonate- 
respiratory acidosis. This is a common finding 
in acute exacerbations of chronic obstructive 
pulmonary disease, especially with type II 
respiratory failure. 


[ Q: 1459 ] MRCPass - Basic Science 


A 60 year old woman has 
hyponatraemia. Following investigation, she 
was diagnosed as having SIADH. 


Which of the following is the most likely 
cause? 


1- Ramipril 

2- Ibuprofen 

3- Chlorpromazine 

4- Alcohol 


5- Atenolol 


Dr. Khalid Yusuf El-Zohry - Sohag Teaching Hospital (01118391123) . 

Ref MRCPass OE OE 2012 PasTest 2009 PassMedicine 2009 PasTest Exam ReviseMRCP 589 

























El-zohry MRCP Questions Bank (Part 1) - 2013 


(For my personal use) 


Answer & Comments 

Answer: 3- Chlorpromazine 

SIADH can be caused by many drugs 
(mnemonic starting with C) - carbamazepine, 
chlorpropramide, cyclophosphamide, 

chlorpromazine and clomipramine (i.e. 
neuroleptics and antidepressants including 
SSRIs). Alcohol decreases ADH release. 



[ Q: 1460 ] MRCPass - Basic Science 

A 40 year old diabetic patient 
overdosed on 30 tablets of an unknown 
medication. 


She has blood gases showing a pH of 7.32, 
p0 2 of 16 kPa and pC0 2 of 3 kPa. Her HC0 3 is 
8 mmol/I and base excess is -5. 

What is the likely scenario? 


ankle jerk. She also had sensory loss over the 
lateral aspect of her ankle. 

Which of these nerve lesions is likely? 

1- Deep peroneal nerve 

2- Femoral nerve 

3- Sciatic nerve 

4- Lumbosacral plexus 

5- Inferior gluteal nerve 


Answer & Comments 

Answer: 3- Sciatic nerve 

The sciatic nerve branches into the tibial 
nerve and common peroneal nerve. Damage 
to these branches will cause almost total 
weakness around the ankle and absent ankle 
jerk. 


1- Metabolic acidosis with respiratory 

compensation 

2- Metabolic alkalosis with respiratory 

compensation 

3- Respiratory acidosis with metabolic 

compensation 

4- Respiratory alkalosis with metabolic 

compensation 

5- Normal anion gap metabolic acidosis 


Answer & Comments 

Answer: 1- Metabolic acidosis with 

respiratory compensation 

The patient is likely to have overdosed on 
metformin and has metabolic acidosis. The 
pH is acidotic and she is hyperventilating for 
respiratory compensation of acid base 
balance, hence the low C02 and high 02 



[ Q: 1461 ] MRCPass - Basic Science 

A 50 year old lady has pain in her 
right leg. On examination, there was 
weakness of her right ankle and absent right 




[ Q: 1462 ] MRCPass - Basic Science 

Which form of nerve damage leads 
to complete inability to raise the arm at the 
shoulder with sensory loss over the deltoid ? 

1- Lateral cutaneous nerve 

2- Axillary nerve 

3- Radial nerve in the axilla 

4- Suprascapular nerve 

5- C5 and C6 of the brachial nerve plexus 


Dr. Khalid Yusuf El-Zohry - Sohag Teaching Hospital (01118391123) 

Ref MRCPass OE OE 2012 PasTest 2009 PassMedicine 2009 PasTest Exam ReviseMRCP 
































El-zohry MRCP Questions Bank (Part 1) - 2013 


(For my personal use) 


Answer & Comments 


Typical examples are : 


Answer: 2- Axillary nerve 

The axillary nerve supplies the deltoid and 
teres minor as well as the skin over the 
deltoid. 



AXILLARY 


Oeaoti. prxxje titers 


Deltoid trrlonof fitwft 


Anterior brands 


posterior fibers 


Posaenor oraocr 


rmnor 



[ Q: 1463 ] MRCPass - Basic Science 

A 40 year old man presented 2 years 
ago with an inherited neurological disorder 
causing weakness. His father developed the 
disease in his 70s and his daughter was born 
5 years ago with a severe form of the 
condition. His mother, sister, wife and son are 
unaffected. 


What is the likely mode of inheritance? 

1- Autosomal dominant 


2- Autosomal recessive 


Huntington's disease, myotonic dystrohpy, 
fragile X syndrome & Friedriech's ataxia. 


[ Q: 1464 ] MRCPass - Basic Science 

A 20 year old patient has been 
kicked in the shin during an aggressive 
football match. He is unable to evert his foot 
and dorsiflex his ankle. 

Which nerve is injured? 

1- Femoral nerve 

2- Saphenous nerve 

3- Gluteal nerve 

4- Common peroneal nerve 

5- Anterior tibial nerve 



Answer & Comments 

Answer: 4- Common peroneal nerve 

The peroneal muscles around the lateral part 
of the shin help to dorsiflex the ankle, extend 
the toes and evert the foot. They are supplied 
by the common peroneal nerve. The common 
peroneal nerve is a branch of the sciatic 
nerve. The manner in which the common 
peroneal nerve snakes around the fibular 
head exposes it to injury. 


3- X linked inheritance 

4- Trinucleotide repeat disease 

5- Mitochondrial inheritance 


Answer & Comments 


Answer: 4- Trinucleotide repeat disease 


[ Q: 1465 ] MRCPass - Basic Science 

i 

A 10 year old boy with blue sclerae 
and recurrent fractures has been diagnosed 
with Osteogenesis imperfecta. 

What abnormality predisposes to bone 
fragility? 



The inheritance showing increasing disease 
severity with earlier onset of disease in 
subsequent generations is called anticipation. 
This is typical of trinucleotide repeat disease 
where there is expansion of repetitive 
sequence of three nucleotides with each 
generation. 


1- Metalloproteinase 

2- Type 1 collagen 

3- Fibronectin 

4- Laminin 

5- Elastin 


Dr. Khalid Yusuf El-Zohry-Sohag Teaching Hospital (01118391123) . 

Ref MRCPass OE OE 2012 PasTest 2009 PassMedicine 2009 PasTest Exam ReviseMRCP 591 


































El-zohry MRCP Questions Bank (Part 1) - 2013 


(For my personal use) 


Answer & Comments 

Answer: 2- Type 1 collagen 

Osteogenesis imperfecta (01) is a condition 
resulting from abnormality in the type I 
collagen, which most commonly manifests as 
fragility of bones. 

[ Q: 1466 ] MRCPass - Basic Science 

A 65 year old man has familial 
hypercholesterolaemia. 

Which one of the following is a characteristic 
feature of the condition? 

1- Palmar xanthomas 

2- Autosomal recessive inheritance 

3- Reduced expression of LDL receptors 

4- Hypertriglyceridaemia 

5- Elevated chylomicrons 



Answer & Comments 

Answer: 3- Reduced expression of LDL 
receptors 

The characteristics of familial 
hypercholesterolaemia are: 

■ autosomal dominant condition 

■ increased LDL concentrations 

■ reduced HDL concentrations 

■ reduced numbers of LDL receptor 

■ cardiovascular disease 
tendon xanthomatas 



[ Q: 1467 ] MRCPass - Basic Science 

A 65 year old man has a right sided 
homonymous hemianopia and right sided 
upper and lower limb weakness. His reflexes 
are brisk on the right side. There is no sensory 
abnormalities. 


Which of the following area could be 
infarcted? 


1- Left temporal 

2- Left parietal 

3- Left cingulate gyrus 

4- Right medial thalamus 

5- Left frontal lobe 


Answer & Comments 

Answer: 2- Left parietal 

The cingulate gyrus forms part of the limbic 
system, which is associated with mood and 
emotions. Frontal lobe lesions are not usually 
associated with homonymous hemianopia 


[ Q: 1468 ] MRCPass - Basic Science 

A 50 year old woman has right sided 
weakness, headache and vomiting. On 
examination she has a hemiplegia affecting 
the right face, arm and leg. She also has 
unilateral internuclear ophthalmoplegia with 
failure of adduction to the left and nystagmus 
to the left. Fundoscopy reveals papilloedema. 

In this patient , the papilloedema is due to 
obstruction at: 

1- The foramen of Monro 

2- The foramen of Morgagni 

3- The foramen of Magendie 

4- The aqueduct of Sylvius 

5- The foramen of Luschka 



Answer & Comments 

Answer: 4- The aqueduct of Sylvius 

The aqueduct of the midbrain (the aqueduct 
of Sylvius) runs in the tegmentum of the 
midbrain and joins the third and fourth 
ventricles. Compression of the aqueduct can 
result in obstructive hydrocephalus and 
papilloedema. 

Headache and vomiting can occur because of 
raised intracranial pressure. Malignant or 
benign intracranial tumors, colloidal cysts, 


Dr. Khalid Yusuf El-Zohry - Sohag Teaching Hospital (01118391123) 

Ref MRCPass OE OE 2012 PasTest 2009 PassMedicine 2009 PasTest Exam ReviseMRCP 



































El-zohry MRCP Questions Bank (Part 1) - 2013 


(For my personal use) 


arachnoid cysts, and neurocysticercosis can 
also cause compression and need to be ruled 
out. 


Lateral Ventricles 


Choroid Plexus 


* , ' \Aqueduct of Sylvius 

(. 



3^//— Fourth Ventricle 


Foramen of Luschka 


CSF drainage 


[ Q: 1469 ] MRCPass - Basic Science 

A 75 year old man has a posterior 
cerebral artery territory infarct. 

Which one of the following is likey to occur? 




> = posterior 
communicating A 


Posterior Cerebral Artery 


Foramen of Magendie 


[ Q: 1470 ] MRCPass - Basic Science 


H 

fm 

K 

A 30 year old lady has palpitations 


and is subsequently confirmed to have 
thyrotoxicosis due to Grave's disease. 

Which of the following statements is true? 

1- Grave's disease is associated with muscular 
dystrophy 


1- Bitemporal hemianopia 

2- Expressive dysphasia 

3- Receptive dysphasia 


2- Propylthiouracil is preferred over 
carbimazole in pregnancy 

3- Radioactive iodine always improves Grave's 
disease 


4- Colour blindness 

5- Cortical blindness 


Answer & Comments 


Answer: 5- Cortical blindness 


4- Smoking history is irrelevant 

5- Steroid eye drops are typically used to 
treat Grave's eye disease 

Answer & Comments 


The posterior cerebral artery supplies the 
occipital lobe and the inferior portion of 
temporal lobe. Homonymous hemianopia, 
cortical blindness, verbal dyslexia and 
hemivisual neglect can occur. 


Answer: 2- Propylthiouracil is preferred over 
carbimazole in pregnancy 

Grave's disease is associated with other 
autoimmune conditions e.g. myasthenia 
gravis. 


Propylthiouracil is preferred to Carbimazole 
in pregnancy because carbimazole crosses the 
placenta and can cause nail/finger 
abnormalities (aplasia cutis) in the baby. 


Radioactive iodine can worsen Grave's 
disease. Smoking is a risk factor for Grave's 
disease. 


Dr. Khalid Yusuf El-Zohry - Sohag Teaching Hospital (01118391123) . 

Ref MRCPass OE OE 2012 PasTest 2009 PassMedicine 2009 PasTest Exam ReviseMRCP 593 



























El-zohry MRCP Questions Bank (Part 1) - 2013 


(For my personal use) 


High dose oral or iv steroids are required in 
Grave's eye disease. 



[ Q: 1471 ] MRCPass - Basic Science 

Which one of the following 
conditions has autosomal dominant 


inheritance? 


1- Oculocutaneous albinism 

2- Betathalassaemia 

3- Marfan's syndrome 

4- Wilson's disease 

5- Xeroderma Pigmentosa 


1- DAF 

2- CR1 

3- C3b 

4- Factor I 

5- MCP 


Answer & Comments 

Answer: 3- C3b 

C3b is an active fragment of C3, and can 
activate the alternative pathw ay. DAF, CR1, 
Factor I and MCP are complement pathway 
inhibitors. 


Answer & Comments 

7 ) 

[ Q: 1473 ] MRCPass - Basic Science 

Answer: 3- Marfan's svndrome 

fi 

The thymus gland contains 3 major 


Marfan's syndrome inheritance is autosomal 
dominant. The rest of the conditions are 
autosomal recessive. 

The list of autosomal recessive conditions are: 

oculocutaneous albinism 
alkaptonuria 
Bartter's syndrome 
cystic fibrosis 

endemic goitrous cretinism 
galactosaemia 
Gaucher's disease 


cell populations-epithelial, hemopoietic, and 
accessory cells. 

Which of the following cells develop in the 
thymus? 

1- Macrophages 

2- T cells 

3- Erythrocytes 

4- B cells 

5- Hairy cells 

Answer & Comments 

Answer: 2- T cells 


glycogen storage disease 
phenylketonuria 
Wilson's disease 
xeroderma pigmentosa 


[ Q: 1472 ] MRCPass - Basic Science 

Activation of the complement 
components is associated with potent 
biological functions to counteract infections. 



B cells are not only produced in the bone 
marrow but also mature there. How ever, the 
precursors of T cells leave the bone marrow 
and mature in the thymus. 


[ Q: 1474 ] MRCPass - Basic Science 

A 55 year old male has been a heavy 
smoker and has had previous exposure to 
silica dust. He presents to A&E with 
worsening longstanding breathlessness. 



Which of the following activates rather than 
inhibits the complement pathw ay? 


Dr. Khalid Yusuf El-Zohry - Sohag Teaching Hospital (01118391123) 

Ref MRCPass OE OE 2012 PasTest 2009 PassMedicine 2009 PasTest Exam ReviseMRCP 



594 

































El-zohry MRCP Questions Bank (Port 1) - 2013 


(For my personal use) 


His arterial pH is 7.36. pC0 2 of 7.7 kPa and 
p0 2 of 7.7kPa. His HCO3 is 32 (20-28) mmol/l 
and Base excess is 2. 

Which is the accurate description of his acid 
base balance? 

1- Metabolic acidosis with respiratory 

compensation 

2- Metabolic alkalosis with respiratory 

compensation 

3- Respiratory acidosis with metabolic 

compensation 

4- Respiratory alkalosis with metabolic 

compensation 

5- Normal anion gap metabolic acidosis 


Answer & Comments 

Answer: 3- Respiratory acidosis with 
metabolic compensation 

There is chronic type 2 respiratory failure 
causing respiratory acidosis because of the 
hypoxia and hypercapnia. This is 
compensated metabolically by HC03 
retention and pH is restored to within a 
normal range. 



[ Q: 1475 ] MRCPass - Basic Science 

Which is the structure formed by the 
roots of the lumbar and sacral nerves? 

1- Falx cerebri 


2- Amygdala 


Cord 


Conus 
medularis - - 

Cauda 

equina 



[ Q: 1476 ] MRCPass - Basic Science 

A 40 year old woman has MELAS. 

Which of the following is correct regarding 
the risk of the grandsons and granddaughters 
having the disease? 

1- Son's children 100%, Daughter's children 
100 % 

2- Son's children 0%, Daughter's children 
100 % 

3- Son's children 50%, Daughter's children 
100 % 

4- Son's daughters 100%, Son's sons 100% 

5- Daughter's daughters 100%, Daughter's 
sons 0% 



Answer & Comments 

Answer: 2- Son's children 0%, Daughter's 
children 100% 


3- Cisterns 

4- Medulla oblongata 

5- Cauda equina 


Answer & Comments 

Answer: 5- Cauda equina 

The cord is tapered at the lower end to form 
the conus medullaris. 

The roots of the lumbar and sacral nerves are 
long and they form the cauda equina. 


Kearn Sayre's, MELAS, MERRF, progressive 
external opthalmoplegia, Leber's optic 
atrophy are mitochondrially inherited 
diseases. The mitochondrial DNA is passed on 
only from the mother to all children. 



[ Q: 1477 ] MRCPass - Basic Science 

A 35 year old lady presents with w 
asting of the quadriceps and weakness of 
knee extension. There is loss of the knee jerk 
and sensory impairment over the front of the 


Dr. Khalid Yusuf El-Zohry - Sohag Teaching Hospital (01118391123) 

Ref MRCPass OE OE 2012 PasTest 2009 PassMedicine 2009 PasTest Exam ReviseMRCP 




























El-zohry MRCP Questions Bank (Part 1) - 2013 


(For my personal use) 


thigh and over the subcutaneous surface of 
the tibia. 

The likely lesion is: 

1- Peroneal nerve 

2- Brachial nerve 

3- Obturator nerve 

4- Femoral nerve 

5- Sacral nerve 



Lisch Nodules 


Answer & Comments 

Answer: 4- Femoral nerve 

The femoral nerve supplies the iliacus and 
pectineus, and the muscles on the anterior 
thigh. The nerve also provides cutaneous 
filaments to the front and inner side of the 
thigh and to the leg and foot (via saphenous 
nerve). 


^ [ Q: 1479 ] MRCPass - Basic Science 

fi - 

A 17 year old lady has small, raised 
lesions on her trunk and also has axillary 
freckles. 

What is the likely mode of inheritance of this 
condition? 

1- Autosomal dominant 

2- Autosomal recessive 


[ Q: 1478 ] MRCPass - Basic Science 

Which one of the following features 
is found in Neurofibromatosis Type 1 (NF1)? 

1- Webbed neck 

2- Calcinosis 

3- Lens dislocation 

4- Lisch Nodules 

5- Roths spots 

Answer & Comments 

Answer: 4- Lisch Nodules 

Lisch nodules (pigmented spots) of the iris are 
present in more than 90% of patients with 
neurofibromatosis type 1. Bilateral acoustic 
neuromas are a hallmark feature of 
neurofibromatosis type 2. 

The diagnosis is suggested by six or more cafe 
au lait spots. Although the condition is 
autosomal dominant, almost half of all cases 
are new mutations. 



3- Trinucleotide repeats 

4- X linked recessive 

5- X linked dominant 


Answer & Comments 

Answer: 1- Autosomal dominant 

The condition described is neurofibromatosis. 
Inheritance is autosomal dominant. 


The gene defect for NF-1 is on chromosome 
17, and for NF-2 is on chromosome 22. 



Neurofibromas 



[ Q: 1480 ] MRCPass - Basic Science 



Dr. Khalid Yusuf El-Zohry - Sohag Teaching Hospital (01118391123) 

Ref MRCPass OE OE 2012 PasTest 2009 PassMedicine 2009 PasTest Exam ReviseMRCP 






























El-zohry MRCP Questions Bank (Port 1) - 2013 


(For my personal use) 


A 35 year old diabetic man has loss of 
sensation in the anterior and lateral part of 
the thigh. 

Which nerve is likely to be affected? 

1- Sciatic nerve 

2- Lateral cutaneous nerve 


1- Phosphofructokinase 

2- Glucokinase 

3- Hexokinase 

4- Fructokinase 

5- Glucose 6 phosphatase 


3- Gluteal nerve 

4- Pudendal nerve 

5- Femoral nerve 


Answer & Comments 

Answer: 2- Lateral cutaneous nerve 

Trauma around the inguinal ligament can lead 
to damage in the lateral cutaneous nerve 
supplying the anterolateral portion of the 
thigh. It is a purely sensory nerve which 
travels lateral to the psoas muscle. 



Lateral Cutaenous Nerve (marked NCL) 


[ Q: 1481 ] MRCPass - Basic Science 

Which of the following enzymes 
converts glucose to glucose-6-phosphote? 


Answer & Comments 

Answer: 3- Hexokinase 

Hexokinase catalyses the conversion of 
glucose to glucose-6-phosphate, using a 
phosphate group donated from ATP. 



[ Q: 1482 ] MRCPass - Basic Science 

Which of the following is true 
regarding the role of restriction enzymes? 

1- Anneal DNA together 

2- Synthesize DNA 

3- Are involved in the cell cycle arrest 

4- Cut DNA 


5- Degrade DNA 


Answer & Comments 

Answer: 4- Cut DNA 

Restriction enzymes cut DNA at nucleotide 
sequences specific to each restriction 
enzyme. 

Hindlll and EcoRI are examples of restriction 
enzymes. DNA ligase and polymerase are 
involved in joining and linking DNA together. 

EcoRI as an example of a restriction enzyme 


[ Q: 1483 ] MRCPass - Basic Science 

Respiratory distress syndrome can 
be associated with reduction of lung 
surfactant. Surfactant is produced in which 
cell in the lung? 

1- Alveoli white cells 

2- Vessel endothelium 



Dr. Khalid Yusuf El-Zohry - Sohag Teaching Hospital (01118391123) 

Ref MRCPass OE OE 2012 PasTest 2009 PassMedicine 2009 PasTest Exam ReviseMRCP 

































El-zohry MRCP Questions Bank (Part 1) - 2013 


(For my personal use) 


3- Type II pneumocyte 

4- Small cell 

5- Keratinocytes 


Sensory loss occurs over the lateral three and 
a half digits of the hand and the lateral aspect 
of the palm. There may also be vasomotor 
and trophic changes. 


Answer & Comments 

Answer: 3- Type II pneumocyte 

Lung surfactant is produced by type II 
pneumocytes. 


[ Q: 1484 ] MRCPass - Basic Science 

A 22 year old cricket player presents 
with an injury to his right upper limb. On 
examination of the right upper limb there is 
incomplete and defective pronation. The 
wrist flexors are paralysed when examined 
against resistance. 

When this is tested the tendon of flexor carpi 
ulnaris stands out and the hand becomes 
ulnar deviated. Flexion of the ulnar Two 
fingers is possible although it is w eaker than 
normal. Abduction and opposition of the 
thumb is defective. There is sensory loss over 
the lateral three and a half digits of the hand 
and the lateral aspect of the palm. 

The injury is located at: 

1- Ulnar nerve at the wrist 

2- Ulnar nerve at the elbow 

3- Brachial nerve 

4- Musculocutaneous nerve 

5- Median nerve at the elbow 

Answer & Comments 

Answer: 5- Median nerve at the elbow 

An injury to the median nerve at the elbow 
causes weakness of pronator teres, radial 
flexors of the wrist, the long finger flexors 
except the ulnar half of the deep flexors, 
most of the muscles of the thenar eminence 
and the Two radial lumbricals. 



median 

nerve 



1 brrnthe to pronator to :tf , 
pdmm$ longus ,flox. c up .rod., 
flex.dig profundnu 

2 p (i riaiti niaro * s eoui 


3. median nm r t lying 1) rtuwi 
flex dig. nip. and profundnu. 


4. palmar cuianeoiu brrnch 

5. medimnavelidew 
flexor refoiandum 


€ .branch, to flimar aninaue - 
flex.pol.bredid., ifad.pol.farwu. 
opponau pollicu 

7 farm the* to medial himJmc ifa 
phu cutaneous to three and one 
half digits 


[ Q: 1485 ] MRCPass - Basic Science 

Which of the following is true 
regarding autosomal recessive inheritance 
involving parents and children? 

1- For parents with one affected child, the risk 
of having another affected child is 1 in 2 

2- For parents with one affected child, the risk 
of having another affected child is 1 in 8 

3- Unaffected siblings of an affected child 
have a 1 in 3 chance of being carriers 

4- Unaffected siblings of an affected child 
have a 2 in 3 chance of being carriers 

5- Unaffected siblings of an affected child are 
definite carriers 

Answer & Comments 

Answer: 4- Unaffected siblings of an affected 
child have a 2 in 3 chance of being carriers 

The best way to understand is to draw a 
family tree with the parents both being 
carriers of the recessive gene [AAx and BBx] 
and four possible inherited combinations [ 
AB, AxB, AxB and AxBx]. 




Dr. Khalid Yusuf El-Zohry - Sohag Teaching Hospital (01118391123) 

Ref MRCPass OE OE 2012 PasTest 2009 PassMedicine 2009 PasTest Exam ReviseMRCP 



























El-zohry MRCP Questions Bank (Port 1) - 2013 


(For my personal use) 


For parents with one affected child, the risk 
of having another affected child is 1 in 4. 

Unaffected siblings have a 2 in 3 chance of 
being carriers because the last possibility of 
both recessive genes is eliminated (AxBx). 


Bacterium 




[ Q: 1486 ] MRCPass - Basic Science 

Which ONE of the following 
organelles have self replicating DNA ? 

1- Endoplasmic Reticulum 

2- Golgi apparatus 

3- Peroxisome 

4- Mitochondria 


5- Lysosomes 


Answer & Comments 

Answer: 4- Mitochondria 

Mitochondria have DNA, which can pass on 
inherited mitochondrial diseases (e.g. MELAS 
/ MERRF ). 



[ Q: 1487 ] MRCPass - Basic Science 
A Plasmid best described as 


1- A recombinant section of DNA 

2- Bacterial DNA separate from chromosome 

3- Multiple origins of replication 

4- Viral RNA 


5- Consist of multiple copies of a single gene 


Answer & Comments 

Answer: 2- Bacterial DNA separate from 
chromosome 

Plasmids are circular molecules of bacterial 
DNA separate from the bacterial 
chromosome. They are usually small, 
consisting of a few thousand base pairs. They 
carry one of a few genes and have a single 
origin of replication. 



[ Q: 1488 ] MRCPass - Basic Science 

A 30 year old man has Lipoprotein 
lipase deficiency. 

Which one of the following features is most 
likely? 

1- Marked hypercholesterolaemia 

2- Reduced chylomicrons 

3- Marked hypertriglyceridaemia 

4- Familial Hypercholesterolaemia 

5- Combined hyperlipidaemia 


Answer & Comments 

Answer: 3- Marked hypertriglyceridaemia 

Mutations in the LPL gene cause familial 
lipoprotein lipase deficiency. 

The breakdown of chylomicrons releases fat 
molecules for storage in fat (adipose) cells or 
for energy use. 

Inheritance is autosomal recessive. The 
breakdown of chylomicrons releases 
triclyceride molecules for storage in adipose 
cells or for energy use. Mutations in the LPL 
gene prevent lipoprotein lipase from breaking 
down chylomicrons effectively, leading to 
high triglyceride levels in the plasma. 



[ Q: 1489 ] MRCPass - Basic Science 

Which one of the following is true 
regarding mitochondrial DNA diseases? 


1- Mitochondrial DNA is inherited from the 
father 


2- Mitochondrial DNA is composed of a 
circular loop of doublestranded DNA 


Dr. Khalid Yusuf El-Zohry - Sohag Teaching Hospital (01118391123) 

Ref MRCPass OE OE 2012 PasTest 2009 PassMedicine 2009 PasTest Exam ReviseMRCP 

































El-zohry MRCP Questions Bank (Part 1) - 2013 


(For my personal use) 


3- Mitochondrial genome encodes for nuclear 
proteins 

4- Mutations of mitochondrial DNA occurs in 
multiple sclerosis related optic atrophy 

5- Simvastatin depletes muscle mitochondrial 
DNA 


Answer & Comments 

Answer: 2- Mitochondrial DNA is composed of 
a circular loop of doublestranded DNA 

Mitochondrial DNA is inherited from the 
mother. Mitochondrial DNA codes for 
proteins in the oxidative phosphorylation / 
electron transport chain. Leber's optic 
atrophy is a form of mitochondrial disease. 
AZT (zidovudine) is an example of a drug 
which does deplete muscle mitochondrial 
DNA. 


Mitochondrial DNA 



control region 


rtf NA 


cytochrome b 


vj&urwts of HA0H 
dchydroonas* 


MitHjMts of cytochrome c 
/ otklif* \. 


MfftRF 

#344 


V 

\ 

\ 

KARP 

8993 


LHON 

3460 




14484 


. m s 


14459 


Mf.AS 


3?4 3 


. HON 


11771 


ADPO 
4 336 


w' 

Trafttfer RNA < — 


Mitochondrial DNA 


[ Q: 1490 ] MRCPass - Basic Science 

Which of the following is degraded 
to uric acid? 

1- Uracil 

2- Thymine 

3- Cytosine 

4- Guanine 

5- Orotic acid 



Answer & Comments 

Answer: 4- Guanine 

The purine bases adenine and guanine are 
degraded to uric acid. Adenosine and 
Xanthine are also metabolised to uric acid. 
Uracil, thymine, cytosine and orotic acid are 
pyrimidine bases. 


"do novo" synthesis 


PRPP 

I 

I 

* © 

SAICAR -► SAICAnbosxJe 


© ^ 


Furr 


AICAH 


© 

S AMP — S Ado 

\®/ oV um 


I 



Xanthine 


Unc ac»d 



Purine Metabolism 


[ Q: 1491 ] MRCPass - Basic Science 

Which one of the following features 
does trinucleotide repeat disorders exhibit? 

1- Anticipation 

2- Linkage 

3- Methylation 

4- Reduction 

5- Genomic imprinting 



Answer & Comments 

Answer: 1- Anticipation 

Trinucleotide repeat disorders typically 
worsen if there are expansion in the numbers 
of repeats. This is labeled anticipation. The 
repeats may be involved in coding sequences 
of proteins e.g. in Huntington's. Fragile X 
syndrome causes cognitive impairment. 



[ Q: 1492 ] MRCPass - Basic Science 
A 35 year old patient has a high 


Dr. Khalid Yusuf El-Zohry - Sohag Teaching Hospital (01118391123) 

Ref MRCPass OE OE 2012 PasTest 2009 PassMedicine 2009 PasTest Exam ReviseMRCP 





































El-zohry MRCP Questions Bank (Port 1) - 2013 


(For my personal use) 


arched palate and aortic regurgitation. He has 
a tall stature, and upw ards lens dislocation. 

Which gene abnormality does he have? 

1- Myosin 

2- Actin 

3- Fibrillin 

4- Spectrin 

5- Ankyrin 


Answer & Comments 

Answer: 3- Fibrillin 

In Marfan's syndrome, a mutation in a gene 
causes a defect in the body's production of 
fibrillin, an important building block of 
connective tissue. In many families with 
inherited Marfan's syndrome, the mutation 
affects the FBN1 gene on chromosome 15. 


/ 

/ 



[ Q: 1493 ] MRCPass - Basic Science 

A 30 year old woman has recently 
delivered a baby. She complains of groin 
pains. On examination, she has weakness of 
adduction and internal rotation of the hip. 
There is sensory impairment over the medial 
aspect of the thigh. 

Which nerve is affected? 


1- Femoral nerve 


2- Sciatic nerve 

3- Sacral nerve 

4- Obturator nerve 

5- Lateral cutaneous nerve of the thigh 


Answer & Comments 

Answer: 4- Obturator nerve 

The obturator nerve supplies gracilis, the 
adductor (longus, brevis, magnus) and the 
skin over the medial aspect of the thigh. 



[ Q: 1494 ] MRCPass - Basic Science 

A study reveals an immediate rise in 
blood pressure following infusion of a 
hormone in a group of volunteers. 

Which of the following hormones is likely to 
have been used? 

1- Angiotensin I 

2- Angiotensin II 

3- Growth hormone 

4- Atrial natriuretic peptide 

5- Brain natriuretic peptide 

Answer & Comments 

Answer: 2- Angiotensin II 




Dr. Khalid Yusuf El-Zohry - Sohag Teaching Hospital (01118391123) 

Ref MRCPass OE OE 2012 PasTest 2009 PassMedicine 2009 PasTest Exam ReviseMRCP 


























El-zohry MRCP Questions Bank (Part 1) - 2013 


(For my personal use) 


The final active messenger of the renin- 
angiotensin pathway is angiotensin II. 

Angiotensin II binds to ATI receptors to cause 
vasoconstriction and fluid retention, both of 
which lead to an increase in blood pressure. 
Angiotensin II receptor blockers lower blood 
pressure by blocking the ATI receptors. 


[ Q: 1495 ] MRCPass - Basic Science 

A 36 year old man has 
gynaecomastia secondary to cirrhotic liver 
disease. 

What is likely to have caused the 
gynaecomastia? 

1- Reduced testosterone production 

2- Increased testoterone metabolism 

3- Increased oestrogen production 

4- Reduced oestrogen metabolism 

5- Increased LH levels 



Answer & Comments 

Answer: 4- Reduced oestrogen metabolism 

Gynaecomastia in liver disease is due to an 
imbalance in androgen and oestrogen levels, 
the main contributor being reduced 
oestrogen metabolism. 


[ Q: 1496 ] MRCPass - Basic Science 

A 30 year old male presents has 
intermittent jaundice and anaemia. He is 
diagnosed with glucose 6 phosphate 
dehydrogenase (G6PD) deficiency. His wife 
has normal G6PD activity. 

What is the likelihood of their children 
developing the condition phenotypically? 

1- All their children will be affected 

2- All their sons will be affected 

3- All their daughters will be affected 

4- 50% of their daughters will be affected 



Answer & Comments 

Answer: 5- None of their children will be 
affected 

Glucose 6 phosphate dehydrogenase (G6PD) 
deficiency has X linked inheritance. The 
affected patient has a chromosome XxY and 
wife is XX. Therefore all daughers will be 
carriers XxX and all sons normal XY. None of 
these patients will have phenotypical G6PD 
deficiency. 


X chromosome w3h 
mutation 


Normal 
ft male 


A 

r 

l X f 

1 


V 

UF «| 

- 

0 


nn-al t 


$ 


0* 


Alt daughters 
art carriers 



* 

o' 




ffc 


V 

nil sum are 

normal 


[ Q: 1497 ] MRCPass - Basic Science 

A 40 year old man has G6PD 
deficiency. 

Which one of the following substances is likely 
to lead to red cell haemolysis? 

1- Chloroquine 

2- Paracetamol 



5- None of their children will be affected 


3- Trimethoprim 



Dr. Khalid Yusuf El-Zohry - Sohag Teaching Hospital (01118391123) 

Ref MRCPass OE OE 2012 PasTest 2009 PassMedicine 2009 PasTest Exam ReviseMRCP 





















































































El-zohry MRCP Questions Bank (Part 1) - 2013 


(For my personal use) 


4- Erythromycin 


Answer & Comments 


5- Baked beans 


Answer: 3- Anticipation 


Answer & Comments 

Answer: 1- Chloroquine 

Oxidative stress can be caused by drugs such 
as chloroquine, quinine, primaquine and 
sulphonamide (sulfomethoxazole), 

nitrofurantoin, NSAIDs, dapsone and aspirin. 
Fava beans can also cause haemolysis. 



[ Q: 1498 ] MRCPass - Basic Science 
Which one of the following cells 


secretes intrinsic factor? 

1- Pancreatic islet cells 

2- Paneth cells 

3- Gastric parietal cells 

4- Hepatocytes 


5- Myocytes 


Answer & Comments 

Answer: 3- Gastric parietal cells 

Intrinsic factor is secreted by the gastric 
parietal cells. It is a glycoprotein which binds 
to vitamin Bi 2 best in an acid environment. It 
then releases vitamin Bi 2 in the terminal 
ileum where proteases digest the binding 
proteins and vitamin B i2 is absorbed 



[ Q: 1499 ] MRCPass - Basic Science 


A 33 year old man has myotonic 


dystrophy. 


Which one of the following features is the 
disease likely to exhibit? 


Anticipation refers to increasing severity with 
subsequent generations. It is common in 
trinucleotide repeat disorders like 
Huntington's disease, myotonic dystrophy 
and fragile X syndrome, where triplet repeat 
mutations in DNA are implicated. 

The mechanism behind the expansion of the 
triplet repeats is little understood. One 
theory is that the increasing number of 
repeats influence the overall shape of the 
DNA, which can have an effect on its 
interaction with DNA polymerase and thus 
the expression of the gene. 



[ Q: 1500 ] MRCPass - Basic Science 

A 46 year old woman complains of 
numbness in her left hand. On examination, 
there was loss of sensation over the thumb 
and first 2 fingers with paralysis of abductor 
pollicis and opponens pollicis. 


Which nerve is injured? 


1- Median 

2- Ulnar 

3- Radial 


4- Posterior interosseous 


5- Anterior interosseous 


Answer & Comments 

Answer: 1- Median 

The median nerve supplies the following 
small muscles in the hands: 

o Abductor pollicis brevis 


1- Decreasing incidence with generations 


o Opponens pollicis 


2- Skips generations 

3- Anticipation 

4- Apoptosis 

5- Mutation 


o ± Flexor pollicis brevis 
o 1st & 2nd lumbricals 

The sensory supply is to palmar surface of 
thumb, 2nd, 3rd & lateral 1/2 of 4th finger 


Dr. Khalid Yusuf El-Zohry - Sohag Teaching Hospital (01118391123) . 

Ref MRCPoss OE OE 2012 PasTest 2009 PassMedicine 2009 PasTest Exam ReviseMRCP 603 
































El-zohry MRCP Questions Bank (Part 1) - 2013 


(For my personal use) 


[ Q: 1501 ] MRCPass - Basic Science 

Which of the following is a tumour 
suppressor gene? 

1- Ras 

2- C-myc 

3- N-myc 

4- P53 

5- Src 





1- Wasp toxin levels 

2- Serum complement C3 level 

3- Serum complement C4 level 

4- Serum total IgE level 

5- Plasma tryptase activity 


Answer & Comments 


Answer: 4- Serum total IgE level 


Answer & Comments 

Answer: 4- P53 

Mutated proto-oncogenes that cause cancer 
are called oncogenes. All of the above are 
oncogenes except for p53. Ras oncogene is 
involved in sporadic tumours (colon and lung) 
and rhabdomyosarcomas. c-myc 

translocation occurs in Burkitt's lymphoma. 
N-myc proto-oncogene is seen in 
neuroblastoma. SRC oncogene is associated 
with sarcoma. p53 is a tumour suppressor 
gene. 



Examples of Oncogenes 



[ Q: 1502 ] MRCPass - Basic Science 

A 18 year old man presents to 
casualty complaining of difficulty breathing. 
He had collapsed shortly after being stung on 
the leg by a wasp. On examination, his 
pressure was 80/40 mmHg, and there was 
significant facial swelling. 


Which one of the following investigations is 
likely to confirm the nature of reaction? 


This is a form of Type I hypersensitivity, also 
known as immediate anaphylactic 
hypersensitivity. It usually takes 15 to 30 
minutes from the time of exposure to the 
antigen. The reaction involves production of 
IgE, in response certain antigens, which in 
turn initiates a sequence of events. 



[ Q: 1503 ] MRCPass - Basic Science 

Which one of the following 
conditions exhibit genomic imprinting? 


1- Abetalipoproteinaemia 

2- Patau's syndrome 


3- Prader Willi syndrome 

4- Down's syndrome 


5- Motor Neuron disease 


Answer & Comments 

Answer: 3- Prader Willi syndrome 

Genomic imprinting is exhibited by the 
following conditions: 

Prader Willi syndrome 

Angelman syndrome 

Beckw ith-Wiedemann syndrome 


Dr. Khalid Yusuf El-Zohry - Sohag Teaching Hospital (01118391123) 

Ref MRCPass OE OE 2012 PasTest 2009 PassMedicine 2009 PasTest Exam ReviseMRCP 



604 








































El-zohry MRCP Questions Bank (Port 1) - 2013 


(For my personal use) 


Father 


Mol her 


imprittieti 





\ j 


T'hc ■mLirk". 


■ The ’impnnlT 


GAMbTbS 




/fi 

m 


== 


[Sperm 

j i 


It 

I | hr J *1 

f 

T 

\l 

1 w \l 

V 

i) 


The next 
generation 



Genomic imprinting - Disease severity 
depends on w hether it is paternally or 
maternally inherited. 


[ Q: 1504 ] MRCPass - Basic Science 

Which of these receptors helps os o 
receptor to ollow HIV to enter octivoted T 
cells ? 

1- CD4 

2- CD8 

3- CXCR4 

4- CCR4 

5- CCR3 



Answer & Comments 

Answer: 3- CXCR4 

CCR5 and CXCR4 are co-receptors which help 
HIV binding (gpl20 to CD4 receptor) to 
activated T cells. 




034+ lymphocyte 
or macrophage 


CD4 


molecules are known to play a central role, 
but CXCR4 (fusin) and CCR5 are also involved, 
possibly by removing gpl20 from the HIV 
particle, thus exposing gp41, which is 
necessary for the membrane attachment of 

HIV. 



[ Q: 1505 ] MRCPass - Basic Science 

A 40 year old man with pleurisy for 
five days was assessed. A moderately sized 
pneumothorax was seen in a chest 
radiograph. 


His arterial blood gases on air showed pH 
7.44, paC0 2 3.0 kPa, pa0 2 30.5 kPa, standard 
bicarbonate 16 mmol/l. 


How con the clinicol picture be explained? 

1- Respiratory acidosis 

2- Compensated metabolic alkalosis 

3- Compensated metabolic acidosis 

4- Compensated respiratory alkalosis 

5- Compensated respiratory acidosis 


Answer & Comments 

Answer: 4- Compensated respiratory alkalosis 

This patient had a normal pH but had both a 
low PaC02 and a low standard bicarbonate. 
The history indicates five days of 



Dr. Kholid Yusuf El-Zohry - Sohog Teaching Hospital (01118391123) 

Ref MRCPass OE OE 2012 PasTest 2009 PassMedicine 2009 PasTest Exam ReviseMRCP 


1 





























































El-zohry MRCP Questions Bank (Part 1) - 2013 


(For my personal use) 


hyperventilation, so this is likely to be a 
compensated respiratory alkalosis. 



[ Q: 1506 ] MRCPass - Basic Science 

An 18 year old male with 
meningococcal meningitis has further 
investigations. It was found that he had low 
properdin levels measured by the ELISA test. 

How is this likely to have been inherited? 

1- Autosomal dominant 

2- Autosomal recessive 


3- X linked recessive 


CLASSICAL 


LECTIN 


ALTERNATIVE 

PATHWAY 


PATHWAY 


PATHWAY 


IMMUNE CARBOHYDRATES ACTIVATING 

COMPLEXES COLLECTINS SURFFACES 


Clq 

Clr 

Cls 

C4 

C2 L 


MBP 

MASP 

C4 

C2 


Clr 

Cls 

C4 

c2 


C3b 

P 

D 

B 


▼ 

C3 

i 

C3b 

t 

C5b678(9)n (MAC) 

Complement Pathways 


4- X linked dominant 

5- Mitochondrial inheritance 


Answer & Comments 

Answer: 3- X linked recessive 

The pathw ays include the classic pathway 
(Clqrs, C2, C4) and the alternative pathway 
(C3, factor B, properdin). Properdin is a 
protein encoded on the X chromosome. 
Properdin stabilizes the C3 convertase 
(C3bBb) of the alternative pathw ay, involved 
in opsonisation. Meningococcal disease is a 
prominent manifestation in a significant 
fraction of reported cases in all clinical 
patterns of complement deficiency, 
particularly 


[ Q: 1507 ] MRCPass - Basic Science 

Northern blotting is a process which 
involves detection of which of the following? 

1- Immunoglobulins 

2- Proteins 

3- Viruses 

4- RNA 

5- DNA 



Answer & Comments 

Answer: 4- RNA 

Northern blotting detects RNA, whilst 
Southern blotting detects DNA. 


those where opsonisation is defective. 
Properdin deficiency states are X-linked 
recessive, while other genetic defects within 
the complement system appear to be 
transmitted as autosomal recessive traits. 


[ Q: 1508 ] MRCPass - Basic Science 

A medical student reviews the 
physiology of the oxygen dissociation curve. 

Which one of the following is associated with 
increased affinity of Hb for oxygen? 

1- Chronic hypoxia 

2- Anaemia 

3- Increased 2,3 DPG 

4- Cold temperature 

5- Diabetic ketoacidosis 



Dr. Khalid Yusuf El-Zohry - Sohag Teaching Hospital (01118391123) 

Ref MRCPass OE OE 2012 PasTest 2009 PassMedicine 2009 PasTest Exam ReviseMRCP 



606 






































El-zohry MRCP Questions Bank (Part 1) - 2013 


(For my personal use) 


Answer & Comments 

Answer: 4- Cold temperature 

Acidosis, raised 2,3 DPG, raised temperature, 
hypoxia and anaemia all shift the 02-Hb 
dissociation curve to the right, leading to 
reduced affinity to 02. 



[ Q: 1509 ] MRCPass - Basic Science 

A patient with Crohn's disease and 
an ileostomy has the following results: 

Na 134 (135-145 mmol/l) 

K 3.1 (3.5-5 mmol/I) 

Cl 112 (92-107 mmol/l) 

Bicarbonate 12 (20-30 mmol/l) 

Urea 13 (3.2-8.1 mmol/l) 

Creatinine 120 (70-110 mmol/l) 

pH 7.25 (7.35-7.45 kPa) 

paC0 2 3.1 (3.5-5 kPa) 

The diagnosis is likely to be: 

1- Normal anion gap metabolic acidosis 

2- Metabolic alkalosis 

3- Respiratory alkalosis 

4- Respiratory acidosis 

5- Hypochloraemic metabolic acidosis 



Answer & Comments 

Answer: 1- Normal anion gap metabolic 
acidosis 

Anion gap = (Na + K) - (CI+HC03), [normal 
range 10-18 mmol/L]. In this case (134 + 3.2) - 
(112 + 12) = 13.1. 

Low bicarbonate in the presence of acidosis 
suggests a metabolic cause, the probable 
cause in this case is gastrointestinal 
bicarbonate loss. 


[ Q: 1510 ] MRCPass - Basic Science 

A 40 year old patient has had EMGs 
done to investigate a cause of peripheral leg 
weakness. 



Which of the following features suggests 
axonal neuropathy? 

1- Latency 

2- Reduced conduction velocity 

3- Reduced muscle action potential amplitude 

4- Decreased frequency of conduction signals 

5- Conduction block 


Answer & Comments 

Answer: 3- Reduced muscle action potential 
amplitude 

Reduced amplitude of action potential is seen 
in axonal neuropathy. Reduced conduction 
velocity or conduction block is seen in 
demyelination. 


[ Q: 1511 ] MRCPass - Basic Science 

A i - 1 - 

Which of the following are found in 
both eukaryotic AND prokaryotic cells? 

1- Linear DNA 

2- Ribosomes 

3- Chromosomes 

4- Nuclear membrane 

5- Introns 


Dr. Khalid Yusuf El-Zohry - Sohag Teaching Hospital (01118391123) 

Ref MRCPass OE OE 2012 PasTest 2009 PassMedicine 2009 PasTest Exam ReviseMRCP 

































El-zohry MRCP Questions Bank (Part 1) - 2013 


(For my personal use) 


Answer & Comments 

Answer: 2- Ribosomes 

Eukaryotes (higher organisms) have muliple 
chromosomes in a genome which is 
separated from the rest of the cell by a 
nuclear membranes. Prokaryotes lack a 
membrane bound nucleus, their DNA occurs 
in a circular form. Transcription of eukaryotic 
genes requires noncoding sequences (introns) 
in the mRNA which is spliced out before 
translation at the ribosome. Both eukaryotes 
and prokaryotes have ribosomes. 


[ Q: 1512 ] MRCPass - Basic Science 

i ^ ) Which of the following is 
characteristically inherited in an autosomal 
recessive manner? 


1- Adult polycystic kidney disease 

2- Cl esterase inhibitor deficiency 

3- Sickle cell disease 


4- Achondroplasia 

5- Familial hypercholesterolaemia 


Answer & Comments 

Answer: 3- Sickle cell disease 

Sickle cell disease is inherited in an autosomal 
recessive manner (sickle cell trait is inherited 
in an autosomal dominant manner). 

Achondroplasia, adult polycystic kidney 
disease, Cl esterase inhibitor deficiency 
(hereditary angiooedema) and familial 
hypercholesterolaemia are usually inherited 
in an autosomal dominant manner. 



[ Q: 1513 ] MRCPass - Basic Science 

Which one of the following 
statements describes genomic imprinting? 


1- Expansion of repeats with time 

2- Genotypic variability from maternal and 
paternal chromosomes 


3- Phenotypic presentation depends on either 
maternal or paternal chromosome 

4- Imprinting of mutations on genomes 

5- Mendelian inheritance 


Answer & Comments 

Answer: 3- Phenotypic presentation depends 
on either maternal or paternal chromosome 

Genomic imprinting refers to the difference in 
phenotypic presentation depending on the 
origin of the disease chromosome from either 
maternal or paternal. 


Father Mother 



Genomic imprinting - The imprinting "mark" is 
represented by a stippled box, and the 
imprinted state is indicated with an X. 



[ Q: 1514 ] MRCPass - Basic Science 

Phosphorylation of protein tyrosine 
residues is associated with which of the 
following? 

1- Protein synthesis 

2- DNA replication 

3- Proteosomal degradation 

4- Cell signaling pathways 

5- Protein degradation 



Dr. Khalid Yusuf El-Zohry - Sohag Teaching Hospital (01118391123) 

Ref MRCPass OE OE 2012 PasTest 2009 PassMedicine 2009 PasTest Exam ReviseMRCP 





































































El-zohry MRCP Questions Bank (Part 1) - 2013 


(For my personal use) 


Answer & Comments 

Answer: 4- Cell signaling pathways 

Protein tyrosine kinases (PTKs) are enzymes 
which catalyze the phosphorylation of 
tyrosine residues. These enzymes are 
involved in cellular signalling pathw ays and 
regulate key cell functions such as 
proliferation, differentiation, anti-apoptotic 
signalling and neurite outgrow th. 
Unregulated activation of these enzymes, 
through mechanisms such as point mutations 
or over-expression, can lead to various forms 
of cancer as well as benign proliferative 
conditions. Indeed, more than 70% of the 
known oncogenes and proto-oncogenes 
involved in cancer code for PTKs. 



Activated tyrosine-kinase 
receptor (phosphoryiated 
dimer) 

Tyrosine Kinase Receptor 


Cellular 

response 

Cellular 

response 


Signal molecules 


Activated 

proteins 


[ Q: 1515 ] MRCPass - Basic Science 

A man with mild bleeding disorder is 
being considered for aspirin for acute 
coronary syndrome. 



Which one of the effects of aspirin is 
beneficial in coronary artery disease? 


1- Reduction in thromboxane A2 synthesis 

2- Increase in the prostaglandins 

3- Glycoprotein MB MIA receptor inhibition 


4- ADP receptor antagonism 

5- Increase in COX enzymes 


Answer & Comments 

Answer: 1- Reduction in thromboxane A2 
synthesis 

Aspirin blocks the synthesis of COX1 and 
COX2 enzymes. 


This leads to a reduction in the prostaglandin 
PGG2, PGH2 —» thromboxane TXA2 synthesis 
i . This leads to i platelet aggregation. 


Phospholipids 


Phospho- -> 

Ilpm 

AA 


Cyelo- 

r 



Soiicyfpfts 

Iftiiprofott 


Thromboxane 

Vasoconstrictor 

Prothrombin 


pgi 2 

PGEn 

* 

Vasodilation 

Antithrombotic 


Aspirin action (salicylates) 


[ Q: 1516 ] MRCPass - Basic Science 

A 12 year old child is investigated 
for multiple skin abscesses. Swabs grow 
staphylococcus aureus. His cousin had died 
from a serious septic illness a few years ago. 

What form of immune deficiency is the child 
likely to have? 

1- Eosinophil 

2- Neutrophil 

3- Complement 

4- Lymphocyte 

5- Immunoglobulin A 



Answer & Comments 


Answer: 2- Neutrophil 


Dr. Khalid Yusuf El-Zohry - Sohag Teaching Hospital (01118391123) 

Ref MRCPass OE OE 2012 PasTest 2009 PassMedicine 2009 PasTest Exam ReviseMRCP 


































El-zohry MRCP Questions Bank (Part 1) - 2013 


(For my personal use) 


Primary neutrophil deficiencies are rare and 
are due to an abnormality, usually inherited, 
of the neutrophil itself. The problem can 
affect phagocytosis (e.g. deficiency of an 
adhesion molecule, CD18/LFA deficiency, on 
the neutrophil surface). Patients with 
neutrophil defects suffer from recurrent chest 
infections with bacteria or fungi, recurrent 
skin abscesses often caused by 
Staphylococcus aureus and poor wound 
healing. 


[ Q: 1517 ] MRCPass - Basic Science 

A 40 year old man attends a fertility 
clinic. Examination shows that he is tall, thin 
and has bilateral gynaecomastia. 



Investigation show high levels of urinary 
gonadotrophins. 


What is the likely diagnosis? 


1- Homocystinuria 

2- Marfan syndrome 

3- Testicular feminisation syndrome 

4- Noonan's syndrome 

5- Klinefelter's syndrome 


Answer & Comments 

Answer: 5- Klinefelter's syndrome 

Klinefelter's syndrome is the most common 
chromosomal disorder associated with male 
hypogonadism and infertility. It is defined 
classically by a 47, XXY karyotype with 
variants demonstrating additional X and Y 
chromosomes. 

The syndrome is characterized by 
hypogonadism (small testes, 

azoospermia/oligospermia), gynecomastia at 
late puberty, psychosocial problems, 
hyalinization and fibrosis of the seminiferous 
tubules, and elevated urinary gonadotropins. 



[ Q: 1518 ] MRCPass - Basic Science 

Which one of the following amino 
thyroxine derived from? 

1- Leucine 

2- Glycine 

3- Cystathione 

4- Tryptophan 

5- Tyrosine 


acids is 


Answer & Comments 

Answer: 5- Tyrosine 

Each molecule of thyroxine is derived from 
Two tyrosine molecules and three (T3) or four 
(T4) iodine molecules. 

[ Q: 1519 ] MRCPass - Basic Science 

A 30 year man has acute onset of 
pain around his left eye. On examination, he 
has a left ptosis and a small left pupil but both 
react normally to light. Visual acuity, fields 
and eye movements are normal. 

The site of injury is to which of the following? 

1- Midbrain 

2- Superior cervical ganglion 

3- Stellate ganglion 

4- Pons 




Dr. Khalid Yusuf El-Zohry - Sohag Teaching Hospital (01118391123) 

Ref MRCPass OE OE 2012 PasTest 2009 PassMedicine 2009 PasTest Exam ReviseMRCP 






































El-zohry MRCP Questions Bank (Part 1) - 2013 


(For my personal use) 


5- Geniculate ganglion 

Answer & Comments 

Answer: 2- Superior cervical ganglion 

The diagnosis is Horner's syndrome. The 
sympathetic nerve fibres from the 
hypothalamus travel through brainstem and 
cervical cord to T1/T2. These synapse on 
preganglionic sympathetic fibres, travel up 
sympathetic chain to superior cervical 
ganglion, and then synapse onto 
postganglionic fibres which travel with 
common and internal carotid arteries. 



*'©IA* 

"favr 


CAVrUNOUB 


riR»T //f 
ecRvicAtO' / 


sruvt 


CAROTID nUUK 


uracil ccftviCAi 
CAftCUD* 


• URCRIOft CARDIAC 
NCftVC 


M'OOIC CCRVICAL 

CAROltOR 

MIDDLE CARDIAC 
NCMVC 


AUlVAMT *I»R 




f i m T 


ItCRVC 


IIUNTM 

CERVICAL 
Nf AVC 


[ Q: 1520 ] MRCPass - Basic Science 

Two strains of Staph aureus are 
isolated and both are resistant to ampicillin. 
Strain 1 retains its resistance to amplicillin 
when grown from multiple generations in the 
absence of ampicillin. How ever strain 2 loses 
its resistance when grow n in the absence of 
ampicillin. 



Which of the following best explains the loss 
of antibiotic resistance in strain 2? 

1- Ampicillin has eliminated resistant bacteria 

2- Variability with generations 

3- Transposition of another sequence into the 
resistance gene 

4- Changes in bacterial DNA ligase 

5- Loss of a plasmid containing the resistance 
gene 


Answer & Comments 

Answer: 5- Loss of a plasmid containing the 
resistance gene 

Bacteria develop resistance to antibiotics by 
gaining genes which encode particular 
proteins which offer protection organism. 
Sometimes this occurs by mutation but at 
other times gene may be acquired from 
another bacterial species. The genes are 
contained in plasmids (circular segments of 
DNA) separate from bacterial chromosome. 
Plasmids can easily spread from one bacteria 
or equally lost. Transfer, loss and gain of 
plasmids are relatively common compared to 
single mutations. 



[ Q: 1521 ] MRCPass - Basic Science 
A patient has the following blood 


results. 

sodium 131 mmol/l 

potassium 4.5 mmol/l 

urea 5 mmol/l 

creatinine 100 pmol/l 

glucose 12 mmol/l 

What is her plasma osmolality? 


1- 262 

2- 267 

3- 279 

4- 280 

5- 290 


Dr. Khalid Yusuf El-Zohry - Sohag Teaching Hospital (01118391123) 

Ref MRCPass OE OE 2012 PasTest 2009 PassMedicine 2009 PasTest Exam ReviseMRCP 



























El-zohry MRCP Questions Bank (Part 1) - 2013 


(For my personal use) 


Answer & Comments 

Answer: 3- 279 

Plasma osmolality is estimated by the 
following formula 2x Na + Urea + glucose. 262 
+ 5 + 12 = 279 (mmol/kg). An example: high 
serum osmolality could be caused by HONK. 



[ Q: 1522 ] MRCPass - Basic Science 

The level of cellular telomerase 
activity will affect which of the following? 

1- Cell death 


2- The number of potential cell divisions 

3- RNA synthesis 

4- The rate of cell growth 

5- Cell survival 


Answer & Comments 

Answer: 2- The number of potential cell 
divisions 

The telomere is a DNA sequence at end of 
each chromosome which becomes 
progressively shorter with each division the 
cell undergoes. The enzyme telomerase is 
able the lengthen telomere thus preventing 
the limitation tow ards cell division. 


Telomerase 
RNA ^ 



Telomerase Activity 


2- Stellate ganglion 

3- Trochlear ganglion 

4- Vestibular nucleus 

5- Ciliary ganglion 


Answer & Comments 

Answer: 5- Ciliary ganglion 

The pathway of the pupillary light reflex 
consists of: retinal receptor cells, bipolar cells, 
ganglion cells, optic nerve and tract, lateral 
geniculate bodies, superior colliculus and 
pretectal nucleus of the high midbrain, 
Edinger-Westphal nucleus, efferent Two 
neurone pathway via the oculomotor nerve 
(lllrd nerve), ciliary ganglion, constrictor 
muscle of the iris. 



[ Q: 1524 ] MRCPass - Basic Science 

Which one of the following is the 
best indicator of osteoblastic activity? 

1- Aspartate aminotransferase 

2- Calcium 

3- Albumin 

4- Alkaline phosphatase 

5- Leucocytes 



Answer & Comments 


Answer: 4- Alkaline phosphatase 


Bone alkaline phosphatase (B-ALP) is 
produced by the osteoblast It is an index of 
early osteoblast differentiation and activity. 

the pupillary reflex? 

1- Occipital cortex 



[ Q: 1523 ] MRCPass - Basic Science 
Which of these areas is involved in 


Dr. Khalid Yusuf El-Zohry - Sohag Teaching Hospital (01118391123) 

Ref MRCPass OE OE 2012 PasTest 2009 PassMedicine 2009 PasTest Exam ReviseMRCP 



612 





































El-zohry MRCP Questions Bank (Part 1) - 2013 


(For my personal use) 


^ [ Q: 1525 ] MRCPass - Basic Science 

t ---- 

0 Which one of the following organs is 

in direct contact anterior surface of left 

kidney , without being separated from it by 

peritoneum? 

1- Spleen 

2- Descending colon 

3- Stomach 

4- Pancreas 

5- Liver 


4- All the sons are affected 

5- Half of the sons are affected 

Answer & Comments 

Answer: 4- All the sons are affected 

G6PD deficiency is X linked recessive. 
Assuming the husband is not a gene carrier - 
her genotype is XXXX and her husband is XY. 
Both daughters will have the genotype XXX 
and will be carriers. Both sons will have the 
genotype XXY and are affected. 


Answer & Comments 

Answer: 4- Pancreas 

Primarily retroperitoneal organs are those 
that develop and remain behind (outside) the 
peritoneum (kidneys, aorta, pancreas). 

Secondarily retroperitoneal organs are those 
that develop within the peritoneal sac but are 
pushed behind it during growth (e.g. 
ascending colon, most of duodenum). 



Normal structures of the Retroperitoneum 


[ Q: 1526 ] MRCPass - Basic Science 

A 45 year old woman has G6PD 
deficiency. 

Which one of the following is true regarding 
the children? 

1- Half the daughters will be affected 

2- All the daughters will be affected 

3- Half the daughters will be carriers 



[ Q: 1527 ] MRCPass - Basic Science 

A 53 year old lady with previous 
peptic ulcer disease was admitted with 
persistent vomiting. She looked dehydrated. 

Her blood results were sodium 140 mmol/l, 
potassium 2.5 mmol/l, chloride 86 mmol/l, pH 
7.5, paC0 2 6.0 kPa, pa0 2 14 kPa, standard 
bicarbonate 40 mmol/l. 

What is the acid-base disturbance? 

1- Hyperchloraemic metabolic alkalosis 

2- Hypochloraemic metabolic alkalosis 

3- Respiratory acidosis due to type II 

respiratory failure 

4- Respiratory acidosis due to type I 

respiratory failure 

5- High anion gap metabolic acidosis 



Answer & Comments 

Answer: 2- Hypochloraemic metabolic 

alkalosis 

This patient had alkalosis due to a high 
standard bicarbonatemetabolic alkalosis. The 
PaC02 was appropriately low in 
compensation. This was a hypokalaemic 
hypochloraemic state because of potassium 
and chloride loss from vomiting. Treatment 
was of the underlying cause (pyloric stenosis) 
and intravenous sodium chloride with 
potassium. 


Dr. Khalid Yusuf El-Zohry - Sohag Teaching Hospital (01118391123) . 

Ref MRCPass OE OE 2012 PasTest 2009 PassMedicine 2009 PasTest Exam ReviseMRCP 613 
































El-zohry MRCP Questions Bank (Part 1) - 2013 


(For my personal use) 


^ [ Q: 1528 ] MRCPass - Basic Science 

f --—-- - 

Glycolysis converts glucose into 
which of the following? 

1- Glycerol 

2- Acetyl co A 

3- Fructose 

4- Pyruvate 

5- Citrate 


5- Klinefelter's syndrome 

Answer & Comments 

Answer: 3- Fragile X syndrome 

Chromosomal analysis is useful for Turner's 
(XO), Down's (trisomy 21) and Klinefelter's 
(XXY). In Fragile X DNA analysis is useful to 
determine trinucleotide repeats (CGG 
repeats) 


Answer & Comments 


Answer: 4- Pyruvate 

Glucose, a six-carbon sugar, is converted to 
Two molecules of a three-carbon unit, 
pyruvate in glycolysis. 


G lycolysis/ G luconeogenesis Overview 


ATP Glucose 



ADP 

Fructose _ ADP ATP G-6-P 


(Muscle, kidney, 
adipose—UK) 


r\ 

1 UDPGal UDPG ATP ADP 
Gal-l-P 



Galactose 


Pi + NAD+: 

NADH + H+" 

adp: 

atp: 


ATP-^ F ' 6 - p 

► p i 

ADP’*' y | NADH NAD+ 

F-l fa- top 4 4 Glycerol-3-P 

A V 

* DHAP ■*- F-lFructose 

(major route in liver-GK) 



Glycerol 
ATP ADP 


GA-3-P •+ -► ^- y 

ADP^ ^ 

I G1 ^ ceK 



Glyceraldehyde 


1 toPGA 


A! 


ATP 


3-PGA 


GDP +Pi GTP 


NAD+ NADH+H+ 

t 


CO 2 +NADH +H + : 



* Heavy arrows indicate 
biologicall y i rreversi ble 
reactions 

*Number of arrows 
indicates relative 
flux of reactions. 


Oxaloacetate 


Lactate ~z v v ZTPyruvate 
CoASH + NAD+ — -I. 


Acetyl CoA 



[ Q: 1529 ] MRCPass - Basic Science 

In which of the following conditions 
would DNA analysis be useful? 

1- Turner's syndrome 

2- Down's syndrome 

3- Fragile X syndrome 

4- Creutzfeld Jakob syndrome 


[ Q: 1530 ] MRCPass - Basic Science 

Which of the following describe a 

codon correctly? 

1- A 3-base pair unit of DNA that codes for an 
amino acid 

2- A 3-base pair unit of RNA that codes for an 
amino acid 

3- A 2-base pair unit of DNA that codes for a 4 
base pair unit of RNA 

4- A 5-base pair unit of RNA that codes for an 
amino acid 

5- A 1-base pair unit of DNA that codes for an 
amino acid 



Answer & Comments 

Answer: 2- A 3-base pair unit of RNA that 
codes for an amino acid 

During translation of mRNA, the bases are 
'read' in a 3 base pair or triplet code, each 3- 
base pair unit being referred to as a codon. 


C 

5 


Amino Acid 


tRNA 


n n m m 


Hlo. 


Anticodon 

Codon 


mRNA 



[ Q: 1531 ] MRCPass - Basic Science 
An 18 year old woman has an 



Dr. Khalid Yusuf El-Zohry - Sohag Teaching Hospital (01118391123) 

Ref MRCPass OE OE 2012 PasTest 2009 PassMedicine 2009 PasTest Exam ReviseMRCP 


















































El-zohry MRCP Questions Bank (Port 1) - 2013 


(For my personal use) 


inherited vitamin D metabolic disorder. She 
has Two brothers who are unaffected. 

She has three sisters who are all affected. Her 
father is affected but not her mother. 

What is the mode of inheritance? 

1- Autosomal Dominant 

2- Autosomal Recessive 

3- Mitochondrial inheritance 

4- X linked Recessive 

5- X linked Dominant 


Answer & Comments 

Answer: 5- X linked Dominant 

X linked dominant disorders are rare (e.g. 
Vitamin D resistant rickets). The condition 
affects both sexes but females more than 
males. All children of a homozygous mother 
are affected (XxXx). Half of the sons and half 
of the daughters inherit the disorder from an 
affected mother with the trait (XxX). An 
affected father (XxY) passes the disease to all 
his daughters (XxX) but none of his sons (XY). 


A systemic anaphylactoid reaction can occur 
with iv N acetylcysteine. 

Features of this are bronchospasm, 
hypotension, tachycardia, flushing, 
angioedema and rash. 

[ Q: 1533 ] MRCPass - Basic Science 

A patient has multiple skin lesions 
that consist of sessile and pedunculated 
papules and nodules over the entire surface 
of his body. In addition, there are multiple 
pigmented macules on his trunk and axillary 
freckling. There are Lisch nodules on the iris. 

The inheritance of this condition is: 

1- Polygenic inheritance 

2- Autosomal recessive 

3- Autosomal dominant 

4- X-linked dominant 

5- X-linked recessive 



Answer & Comments 


Answer: 3- Autosomal dominant 


[ Q: 1532 ] MRCPass - Basic Science 

An 18 year old girl took an overdose 
of 20 g of paracetamol with 3 pints of beer. 
Upon presentation to hospital 6 hours later, 
she was commenced on intravenous N - 
acetylcystine immediately. 1 hour later she 
developed tachycardia, flushing and w 
heezing. 

What is likely to have caused this? 

1- IgE hypersensitivity reaction 

2- Disulfiram type reaction 

3- Interaction with alcohol 

4- Interaction with paracetamol 

5- Late effects of paracetamol overdose 



Answer & Comments 


Answer: 1- IgE hypersensitivity reaction 


The diagnosis is neurofibromatosis, which has 
autosomal dominant inheritance. 


r 


Skin neurofibromas 


[ Q: 1534 ] MRCPass - Basic Science 

• A 55 year old woman has visual 
problems. On examination, she has 
a right sided third nerve palsy. 


Dr. Khalid Yusuf El-Zohry - Sohag Teaching Hospital (01118391123) . 

Ref MRCPass OE OE 2012 PasTest 2009 PassMedicine 2009 PasTest Exam ReviseMRCP 615 


i 































El-zohry MRCP Questions Bank (Part 1) - 2013 


(For my personal use) 


Which one of the following occurs typically in 
a third nerve palsy? 

1- Small pupil 

2- Reactive pupil 

3- Exopthalmos 

4- Ptosis 


3- Autosomal dominant 

4- X linked recessive 

5- Multifactorial 


Answer & Comments 


Answer: 2- Autosomal recessive 


5- Eye looks upward 


Answer & Comments 

Answer: 4- Ptosis 

3rd nerve palsy leads to ptosis, dilated 
unreactive pupil and eye looking down and 
out (due to unopposed superior oblique and 
abducent nerves. Exopthalmos can be 
associated (e.g. graves disease) but is not a 
feature of 3rd nerve palsy. 



Third Nerve Palsy on the Right 



[ Q: 1535 ] MRCPass - Basic Science 

An 23 year old male has a chronic 
cough and recurrent bronchopulmonary 
infections. On examination he is clubbed and 
there are coarse late inspiratory crepitations 
(crackles) at both lung bases. His sweat 
sodium concentration is 80 mmol/L (normal 
60 mmol/L). 


The mode of inheritance of the condition is: 

1- X linked dominant 


2- Autosomal recessive 


Cystic fibrosis is caused by a gene mutation 
(CFTR gene) on the long arm of chromosome 
7. It is autosomal recessive. 


[ Q: 1536 ] MRCPass - Basic Science 

A 35 year old man had an injury 
whilst doing DIY work. He has numbness 
around the half of his fourth finger and last 
digit. 

Which of these motor functions is likely to be 
impaired? 

1- Flexion of the fourth finger 

2- Extension of the fourth finger 

3- Extension of the little finger 

4- Thumb abduction 

5- Thumb adduction 



Answer & Comments 

Answer: 5- Thumb adduction 

The distribution of sensory loss implies an 
ulnar nerve lesion. Flexion of the fingers and 
thumb abduction is supplied by the median 
nerve. Extension of the fingers are supplied 
by radial nerve. 

[ Q: 1537 ] MRCPass - Basic Science 

A 25 year old male presents with 
wasting and weakness of the muscles of the 
pelvic girdle. There is evidence of generalised 
muscular hypertrophy. His maternal 
grandfather had a similar disorder. 

The mode of inheritance is: 

1- Mitochondrial inheritance 



Dr. Khalid Yusuf El-Zohry - Sohag Teaching Hospital (01118391123) 

Ref MRCPass OE OE 2012 PasTest 2009 PassMedicine 2009 PasTest Exam ReviseMRCP 































El-zohry MRCP Questions Bank (Port 1) - 2013 


(For my personal use) 


2- X linked dominant 

3- Autosomal dominant 

4- Autosomal recessive 

5- X-linked recessive 


Answer & Comments 

Answer: 5- X-linked recessive 

The diagnosis is Becker's muscular dystrophy. 
This is X linked recessive. 



[ Q: 1538 ] MRCPass - Basic Science 

Which one of the following describes 
formation of proteins along the mRNA in a 5' 
to 3' direction ? 


1- Reverse transcription 

2- Transcription 

3- Translation 

4- Duplication 

5- Splicing 


Answer & Comments 


Answer: 3- Translation 


Translation alw ays begins with a methionine 
residue. The mRNA is translated in the 5' to 3' 
direction and is read in groups of 3 bases, 
which are known as codons. New amino acids 
are added to the carboxyl terminus of the 
grow ing peptide chain. 


Gly Leu 


Tyr 


Growing polypeptide chi 


Set 


sin 




lieu) 


Asn 


i Gly ) 






C G T C A A 


I 'll” 


. 


CUCUUGG6UCC6CAGUUAAUUUCUAUC 

I 1 I I I ' l \ I i I I I I I I I I I I I M I i I I 


mRNA 


Ribosome 


Translation and Protein synthesis 


[ Q: 1539 ] MRCPass - Basic Science 

A 35 year old woman presents with 
a right sided Horner's syndrome 
with anhydrosis of the medial side of the right 
forehead. 

The cause of the Horner’s is most likely due to: 

1- Cervical rib 

2- Lateral medulla infarct 

3- Injury around internal carotid artery 

4- Compression from pancoast tumour 

5- Hypothalamus insult 



Answer & Comments 

Answer: 3- Injury around internal carotid 
artery 

All are potential causes of Horner's syndrome. 
How ever, the sympathetic plexus around the 
internal carotid artery supplies the medial 
side of the forehead, hence anhydrosis to 
that region specifically points tow ards injury 
to the internal carotid artery at that site. 



Right sided Horner's (ptosis, miosis, 

anhydrosis) 


[ Q: 1540 ] MRCPass - Basic Science 

Which of the following is the product 
of the beta-oxidation of fatty acids? 

1- Glucose 6 phosphate 

2- Pyruvate 

3- Oxaloacetate 

4- Hydroxymethylglutaryl Coenzyme A 

5- Acetyl Coenzyme A 



Dr. Khalid Yusuf El-Zohry - Sohag Teaching Hospital (01118391123) . 

Ref MRCPass OE OE 2012 PasTest 2009 PassMedicine 2009 PasTest Exam ReviseMRCP 617 































El-zohry MRCP Questions Bank (Part 1) - 2013 


(For my personal use) 


Answer & Comments 


Answer: 5- Acetyl Coenzyme A 

Beta oxidation is the process by which fatty 
acids are broken down to smaller units such 
that substances such as Acetyl Coenzyme A 
can enter the citric acid cycle. 

p a 

H s Hj Hi 9 
?—r— r — r —L 


Acetyl Co-A is a two-carbon unit which is 
formed from pyruvate. Pyruvate is the end 
product of glycolysis. This reaction is 
catalysed by pyruvate dehydrogenase. Acetyl 
coA is the substrate for the citric acid cycle. 
With each turn of the cycle, Two carbon 
atoms enter as acetyl-CoA and Two carbon 
atoms are lost as C02. 


16 C R—C C—C —“—SC Oft pal mitoyl-CoA 

FAO 


1. 


acyl-CoA I 

hydrriasa I T* FADI-lj 

1 P 

H 0 


CoA pyruvate 

NAD' 'N. I pyruvate 

)\ dehydrogenase 

NADH-* - ' \ 

CO, 

acetyl-CoA 


-sa- 


t 


■SCoA iran*-A-anoyl-CoA 


enoyl-CoA 

hydrolase 


1 / 


H 2 0 



H,0 


citrate 


aconitase c/s-aconitate H,0 


repeat mee 


14 O'HH 0 


R-C—ll—SCoA l> 


A |!| 


p'hjdroK y-aeyl ■ CoA 


NADH 

NAD' 




malate 
dehydrogenase 


isocitrate 


NAD' 


3* 


pkhydroxyacyl-CoA 1 

1 - NAD 

malate 

isocitrate A 

dehydrogenase j 

NADH+ H' 

f 

dehydrogenase | 

P 


A fumarase 

J 




o 


M o 


A 


iP"k^^SCyl- CoA 


a-ketoglutarate 


fumarate 


^ acyl-CcAacetyltransferase II CoASH 


Hj 

14 c R-C—i-SCoA 4 - HjC-i—SCoA^ 

fio^-CoA ae^CoA 

myri stoyl-CoA 


0 



a-ketoglutarate 

dehydrogenase 


succinyl-CoA 



NADH 

CO, 


repeal 6 more times 


I 1 

0 


7 HjC—U-SC(rt2C 
age*yl-CoA 

Fatty Acid Oxidation 


[ Q: 1541 ] MRCPass - Basic Science 





Which of the following is the end 
product of glycolysis which feeds into the 
citric acid cycle? 

1- Glucose 

2- Fructose 

3- Citrate 

4- Pyruvate 

5- Acetyl coA 

Answer & Comments 

Answer: 4- Pyruvate 


[ Q: 1542 ] MRCPass - Basic Science 

A 60 year old man with type 1 
diabetes and diabetic nephropathy was 
recovering on a surgical ward after a total 
colectomy and ileostomy. He had persistent 
metabolic acidosis and the surgeons 
concerned about his high potassium 
concentration and that there may have been 
some ischaemia in the abdomen causing the 
acidosis. 

However, the patient appeared well perfused 
and had normal vital signs. He had normal 
fluid balance and his results showed: 

sodium 130 mmol/l 

potassium 8.5 mmol/l 

creatinine 200 pmol/l (2.16 mg/dl) 

chloride 109 ?mol/l 

8 am cortisol 500 nmol/I (18 pg/dl) 

pH 7.29 


Dr. Khalid Yusuf El-Zohry - Sohag Teaching Hospital (01118391123) 

Ref MRCPass OE OE 2012 PasTest 2009 PassMedicine 2009 PasTest Exam ReviseMRCP 



618 














































El-zohry MRCP Questions Bank (Port 1) - 2013 


(For my personal use) 


paC0 2 3.5 kPa 
pa0 2 14 kPa 

standard bicarbonate 12 mmol/l 
Whot is the metabolic disturbance? 

1- Hyporeninaemic hyperaldosteronism 

2- Hyporeninaemic hypoaldosteronism 

3- Hyperreninaemic hyperaldosteronism 

4- Hypokalaemic hyperaldosteronism 

5- Hypochloraemic hypoaldosteronism 


Anticipation refers to the increased severity 
of disease at earlier age of onset with 
successive generations. 

It is a feature of trinucleotide repeat 
disorders, which are: 

■ spinocerebellar ataxia 

■ fragile X 

■ myotonic dystrophy 
Huntington's disease 


Answer & Comments 

Answer: 2- Hyporeninaemic 

hypoaldosteronism 

This man had diabetic nephropathy which 
predisposes to renal tubular acidosis. Type 4 
(hyporeninaemic hypoaldosteronism) RTA is 
typically associated with high potassium and 
is found in diabetic and hypertensive renal 
disease. This patient also had acidosis due to 
low bicarbonate. The PaC02 was 
appropriately low in compensation. The anion 
gap was normal (13.5 mmol/l). This makes 
intra-abdominal ischaemia (which causes 
lactic acidosis) unlikely. 


[ Q: 1543 ] MRCPass - Basic Science 

Genetic anticipation occurs 
characteristically in which one of the 
following conditions? 

1- Wilson's disease 

2- Spinocerebellar ataxia type 1 

3- Haemochromatosis 

4- Neurofibromatosis 




[ Q: 1544 ] MRCPass - Basic Science 

A 20 year old man with a tall stature 
has a cardiac murmur characterized by a 
midsystolic click. An echocardiogram reveals 
mitral valve insufficiency with upw ard 
displacement of one leaflet. There is also 
aortic root dilatation of 4 cm. 


A mutation involving which of the following 
genes is likely to be present in this patient? 

1- Fibrillin 

2- Ankyrin 

3- Spectrin 

4- VEGF 

5- CFTR 


Answer & Comments 

Answer: 1- Fibrillin 

Marfan's syndrome is described. This is a 
connective tissue disorder with the fibrillin 
gene abnormality, leading to associated 
floppy mitral valve and also cystic medial 
necrosis which predisposes aortic dissection. 


5- Tuberous Sclerosis 

Answer & Comments 

Answer: 2- Spinocerebellar ataxia type 1 


^ [ Q: 1545 ] MRCPass - Basic Science 

f§ --'- 

# A 45 year old woman presents with 
weakness of her left upper limb, which 
developed overnight. She says she usually 
sleeps on an armchair. 


Dr. Khalid Yusuf El-Zohry - Sohag Teaching Hospital (01118391123) . 

Ref MRCPass OE OE 2012 PasTest 2009 PassMedicine 2009 PasTest Exam ReviseMRCP 619 





























El-zohry MRCP Questions Bank (Part 1) - 2013 


(For my personal use) 


On examination there is weakness of 
extension of her left elbow , a wrist drop and 
absent sensation over the first interosseus 
space of her left hand on the dorsal aspect. 

Where is the lesion? 

1- Brachial nerve 


Answer & Comments 

Answer: 3- 50% 

Hereditary spherocytosis is inherited in an 
autosomal dominant manner and hence the 
chance of the child being affected is 50%. 


2- Radial nerve 

3- Ulnar nerve 

4- Musculocutaneous nerve 

5- C5 C6 root at the brachial plexus 


Answer & Comments 

Answer: 2- Radial nerve 

The weakness of extension of the elbow 
indicates weakness of the triceps, hence the 
lesion should be in the radial nerve, probably 
higher up at the axilla. Lesions of the radial 
nerve in the spiral groove spare the triceps. 



Wrist Drop in Radial Nerve injury 


[ Q: 1546 ] MRCPass - Basic Science 

A 27 year old man has hereditary 
spherocytosis (heterozygous) is married to an 
unaffected female. They seek advice 
regarding inheritance of the condition. 


What is the chance of their child being 
affected? 


1 - 0 % 


[ Q: 1547 ] MRCPass - Basic Science 

A 49 year old man was admitted 
with chest pain. The ECG showed an anterior 
wall myocardial infarction. He initially settled, 
but after 48 hours, he complained of further 
chest pain with ECG changes. 

Which enzyme is the earliest to rise in 
myocardial infarction? 

1- LDH 

2- CK-MB 

3- Myoglobin 

4- Troponin T 

5- AST 



Answer & Comments 

Answer: 3- Myoglobin 

Myoglobin rises rapidly in myocardial 
infarction. A doubling of the enzyme within 2 
hours is very suggestive of an Ml. Troponin 
and CKMB start to rise after 3 hours, and LDH 
after 12 hours. 


A 20 year old man has metabolic 
acidosis with a pH of 7.2. 

He has the following results: sodium 135 
mmol/I, potassium 4.8 mmol/I, urea 8 pmol/l, 
creatinine 110 pmol/l, chloride 105 (95-107) 
mmol/l, bicarbonate 18 (20-28) mmol/l. 



[ Q: 1548 ] MRCPass - Basic Science 


2- 25% 

3- 50% 

4- 75% 

5- 100% 


What is his anion gap? 
1- 16 

2- 16.8 

3- 17.2 


Dr. Khalid Yusuf El-Zohry - Sohag Teaching Hospital (01118391123) 

Ref MRCPass OE OE 2012 PasTest 2009 PassMedicine 2009 PasTest Exam ReviseMRCP 



620 


































El-zohry MRCP Questions Bank (Port 1) - 2013 


(For my personal use) 


4- 18.6 

5- 20 


Answer & Comments 

Answer: 2-16.8 

Anion gap is calculated by the formula (Na + 
K) - (Cl + [HC03-]). 135 + 4.8 - 105 - 18 = 16.8. 
Normal anion gap is 8-16. 


1- C1 

2- C2 

3- C3 

4- C4 

5- C5 


Answer & Comments 


Answer: 5- C5 


[ Q: 1549 ] MRCPass - Basic Science 

A differential diagnosis list is being 
considered for a child who is short. 

Which of the following abnormalities is 
associated with short stature? 

1- 47, XXY karyotype 

2- 47, XYY karyotype 

3- 45, XXXY karyotype 

4- 45, XO karyotype 

5- Fragile X syndrome 



Answer & Comments 

Answer: 4- 45, XO karyotype 

Turner's syndrome (45, XO karyotype) is 
associated with short stature. 


C5-9 complements are part of the membrane 
attack complex which are important in 
protection against organisms such as 
meningococci. 



[ Q: 1551 ] MRCPass - Basic Science 
Which of the following is an end 


product of porphyrins? 

1- Bilirubin 


2- Globulin 

3- Haem 

4- Glycine 

5- Lactate 


Answer & Comments 


Answer: 3- Haem 


u. 

1 

j 

X ) ( 

y 4 

VI 

5 

u 

6 

i # 

S l 

7 

it ii n 5 

N V 10 

# ii 

* M 

II 12 

«< 

13 

ii 

14 

'J 11 tl k5 

15 Ift 17 IX 

* • 

19 

20 

* * 4 % f 

51 55 \ 

m 1 1 A 

Y 


45, XO karyotype 


The first step in the biosynthesis of haem is 
the condensation of glycine with succinyl CoA 
to form delta-aminolevulinic acid. This 
reaction occurs in the mitochondria and is 
negatively regulated by haem. The reaction 
pathway then proceeds through intermediate 
products porphobilinogen, uroporphyrinogen 
III, coproporphyrinogen III, protoporphyrin IX, 
and finally haem. 


5 


1 


[ Q: 1550 ] MRCPass - Basic Science 

Which one of the following 
complement deficiencies predisposes to 
susceptibility to meningococcal infections? 


Dr. Khalid Yusuf El-Zohry - Sohag Teaching Hospital (01118391123) 

Ref MRCPass OE OE 2012 PasTest 2009 PassMedicine 2009 PasTest Exam 




































El-zohry MRCP Questions Bank (Part 1) - 2013 


(For my personal use) 


Glycine 

v_ 


H + 


C0 2 + CoA 


r 


Succinyl-CoA -4- — Pyruvate metabolism 

_' - 


Am i n o I evu I i n ate synth a s e 




H 2 0 + H + 


6-Aminolevulinate 

I 

Aminolevulinate dehydratase 


i 


Porphobilinogen 


H 2 0 


NH, 


I _ 

Porphobilinoqen deaminase 


Hydroxymethylbilane 

I 

Uro porphyrinogen-1 


h 2 0 


i 


synthase 


Uroporphyrinogen III 

Uroporphyrinogen decarboxylase 


CO, 


1 


Coproporphyrinogen III 


o 2 -^\ 


C o p ro p o rphyri n o q en oxi d a s e 

—1 


CO. 


Protoporphyrinogen IX 


O, 


H 2 0 


vl 

Protoporphyrinogen oxidase 


'1 


Protoporphyrin IX 


Fe 2+ 


. i 

Ferrochelatase 

I 


Protoheme (Heme) < -Hemoglobin digestion 


Quervain's tenosynovitis which is 
inflammation of the abductor pollicis longus 
and extensor pollicis brevis. 



Finkelstein's test 



[ Q: 1553 ] MRCPass - Basic Science 

A 20 year old man has jaundice, but 
he has normal liver function tests apart from 
a raised bilirubin. 


Porphyrin metabolism 


[ Q: 1552 ] MRCPass - Basic Science 

A 40 year old female complains of 
tenderness in the radial aspect of his wrist. 
She is a professional golf player. 

With the thumb flexed across the palm of the 
hand, movement of the wrist into flexion and 
ulnar deviation causes pain. 

What is the diagnosis? 

1- De Quervain's tenosynovitis 

2- Tennis elbow 

3- Golfer's wrist 

4- Ulner nerve lesion 

5- Radial nerve lesion 



Answer & Comments 

Answer: 1- De Quervain's tenosynovitis 

The movements described above is the 
Finkelstein's test. This is diagnostic of De 


Which one of the following enzymes catalyses 
the conjugation of bilirubin? 

1- Amylase 

2- Glucose 6 phosphatase 

3- Glucuronyl transferase 

4- Xanthine oxidase 

5- Phenylalanine hydroxylase 


Answer & Comments 

Answer: 3- Glucuronyl transferase 

Bilirubin is conjugated with glucuronic acid by 
the enzyme bilirubin uridine 5-diphosphate 
glucuronyl transferase (UGT). 

Crigler-Najjar (CN) syndrome is a congenital 
familial nonhemolytic jaundice associated 
with high levels of unconjugated bilirubin due 
to UGT1 mutation. 

Dubin-Johnson syndrome is asymptomatic 
mild jaundice due to impaired excretion of 
bilirubin. In contrast to Gilbert's syndrome, 


Dr. Khalid Yusuf El-Zohry - Sohag Teaching Hospital (01118391123) 

Ref MRCPass OE OE 2012 PasTest 2009 PassMedicine 2009 PasTest Exam ReviseMRCP 



622 





























El-zohry MRCP Questions Bank (Part 1) - 2013 


(For my personal use) 


the hyperbilirubinemia is conjugated and bile 
appears in the urine. 


Ery throcy tes 

o 0 

_ ± _ 

r Macrophages of the "'i 

^Spleen S'-. Bone Marrow J 

+ 

heme 

<1 

bilirubin 



Bilirubin (conjugated) 


Conjugation of Bilirubin catalysed by 
Glucuronyl Transferase 


[ Q: 1554 ] MRCPass - Basic Science 

A 40 year old man has an anterior 
mediastinal mass seen on CT scan. 

Which of the following is unlikely to be o 
co use for the mass? 

1- Thyroid 

2- Thymus 

3- Thoracic sarcoma 

4- Teratoma 

5- Tumour 




thorax showing a large mediastinal mass 


^ [ Q: 1555 ] MRCPass - Basic Science 

f — - J ~ * 1 2 3 4 5 - - - - 

Which one of these vitamin D 
products is formed in the liver? 

1- 1-hydroxycholecalciferol 

2- 24-hydroxycholecalciferol 

3- 25-hydroxycholecalciferol 

4- 24,25-dihydroxycholecalciferol 

5- 1,25-dihydroxycholecalciferol 


Answer & Comments 

Answer: 3- 25-hydroxycholecalciferol 

The active form of vitamin D - 1,25- 
dihydroxycholecalciferol is formed by 
regulated hydroxylations. The 25-alpha 
hydroxylation is performed in the liver, and 
the 1-alpha hydroxylation in the kidney. 


Answer & Comments 

Answer: 3- Thoracic sarcoma 

The four Ts for mediastinal masses (anterior) 
are thyroid, thymoma, teratoma and tumour 
(lymphomas). 



Dr. Khalid Yusuf El-Zohry - Sohag Teaching Hospital (01118391123) 

Ref MRCPass OE OE 2012 PasTest 2009 PassMedicine 2009 PasTest Exam ReviseMRCP 


1 







































El-zohry MRCP Questions Bank (Part 1) - 2013 


(For my personal use) 


SKIN PRODUCTION 


DIET 




^LIVER) 

Sr 


L 

et 




HO 


J 

JLch ? 

u 


OH 


25 (OH)-Vitamin D 


>CH 

HO OH 



1,25 ( 0 H) 2 Vitamin D 


[ Q: 1556 ] MRCPass - Basic Science 

In genetics , what is the meaning of 
genetic mosaicism? 

1- An activated X chromosome 

2- RNA is transcribed into DNA 

3- An individual has 2 or more genetic cell 
lines 

4- There is always the same number of 
chromosomes in all cells 

5- A form of chromosome trisomy 



Answer & Comments 

Answer: 3- An individual has 2 or more 
genetic cell lines 

A Barr body is an inactivated X chromosome. 
Mosaicism is as defined above - it can 
comprise of cells of 46 or 47 chromosomes in 
the same zygote. 


(HI DIVISION 

O O 

0 9 0 0 

66® @0606 

o NOtMAl cats «U5 WI1H MUUnON 

- 

Genetic Mosaicism 



[ Q: 1557 ] MRCPass - Basic Science 

An 20 year old woman with alpha 1 
antitrypsin deficiency attends a genetics clinic 
for advice reagarding the likelihood of her 
potential child would be affected. 

What is mode of inheritance of this disease? 

1- Autosomal dominant 

2- Autosomal recessive 

3- X linked dominant 

4- X linked recessive 

5- Polygenic 



Answer & Comments 

Answer: 2- Autosomal recessive 

The inheritance of alpha 1 antitrypsin (A1AT) 
deficiency is autosomal recessive. The alleles, 
how ever, are codominant, which means each 
allele is responsible for 50% of the circulating 
A1AT level. The production of alphal 
antiprotease is controlled by a pair of genes 
at the protease inhibitor (Pi) locus. The most 
common (90%) allele is M (PiM), and 
homozygous individuals (MM) produce 
normal amounts of alphal antiprotease 
(serum levels of 20-53 mmol/L). Deficient 
levels of alphal antiprotease are associated 
with allele Z (MZ or ZZ). Serum levels greater 
than 11 mmol/L appear to be protective 
against emphysema. Emphysema develops in 


Dr. Khalid Yusuf El-Zohry - Sohag Teaching Hospital (01118391123) 

Ref MRCPass OE OE 2012 PasTest 2009 PassMedicine 2009 PasTest Exam ReviseMRCP 



624 


































El-zohry MRCP Questions Bank (Port 1) - 2013 


(For my personal use) 


most (but not all) individuals with serum 
levels less than 9 mmol/L. 


^ [ Q: 1558 ] MRCPass - Basic Science 

/ L -- 

* An 25 year old man has Duchenne's 

muscular dystrophy. 

What chances does his daughter's sons have 
of inheriting the disease? 

1 - 100 % 

2- 50% 

3- 25% 

4- 10% 

5- 0% 


Answer & Comments 

Answer: 2- 50% 

Duchenne's muscular dystrophy is X linked. 
This patient is affected (XxY). His wife is likely 
to be unaffected (XX). 

His daughters would all have the 
chromosome (XxX)- all are carriers. Therefore 
his daughter's sons will either have the 
chromosome XxY or XY in 50% chances. 


normal Xohrorrtisoms 


SI & mi a I 
fern alt 


X chromosome wfo 
mutation 


A 


r 

x f 

j 

n* 

;; 


E 



y 


yp 

j 

u 




$ 


d* 




Hit daughters flli sons arc 
are carrier* normal 


[ Q: 1559 ] MRCPass - Basic Science 

# A 35 year old woman with a pruritic 
■ash was diagnosed as having systemic 
nastocytosis. Serum histamine was 2 ng/ml 
normal: 0.3-1.0 ng/ml). 


Which one of the following is true regarding 
mast cells? 


1- In mastocytosis there is insufficient mast 
cells 


2- Storage granules contain glucose 

3- Storage granules contain histamine and 
leukotrienes 


4- It is an IgG mediated immune response 

5- High temperature triggers mast cell release 


Answer & Comments 

Answer: 3- Storage granules contain 
histamine and leukotrienes 



Dr. Khalid Yusuf El-Zohry - Sohag Teaching Hospital (01118391123) 

Ref MRCPass OE OE 2012 PasTest 2009 PassMedicine 2009 PasTest Exam ReviseMRCP 















































































El-zohry MRCP Questions Bank (Part 1) - 2013 


(For my personal use) 


Mastocytosis is due to excessive mast cell 
stimulation. Storage granules contain 
histamine,leukotrienes, and lytic enzymes. It 
leads to anaphylactic like states - urticaria, 
flushing and also Gl symptoms such as 
diarrhoea and nausea. Mast cells are Ig E 
mediated but can be triggered by injury, 
drugs and complement activation. 



Cutaneous Mastocytosis 



[ Q: 1560 ] MRCPass - Basic Science 

Which one of the following 
hormones is increased in hypogiycaemia? 

1- Growth hormone 


2- Insulin 


3- Glucagon 

4- Somatostatin 

5- Thyroxine 


Answer & Comments 

Answer: 3- Glucagon 

Glucagon secretion from the pancreas is 
increased by amino acids arginine and 
alanine, fasting (hypogiycaemia), stressful 
stimuli. Glucagon is inhibited by 
somatostatin. 



Low Blood Sugar 


: 


Promotes 

glucagon 

release 




Insulin L Glucagon 
I™™ Feedback System 

High Blood Sugor 



Promotes 

Insulin 

release 



Pancreas 



Breakdown of 
glycogen 


Insulin 

Pormafon of 
glycogen 



▼ T 

More glucose Less glucose 

In bloodstream In bloodstream 

_I I_ 



[ Q: 1561 ] MRCPass - Basic Science 

Reverse transcriptase PCR is used to 
amplify which of the following? 


1- Proteins 


2- DNA 

3- RNA 

4- Ribosomes 

5- Plasmids 


Answer & Comments 

Answer: 3- RNA 

Reverse transcriptase PCR is a w ay to amplify 
RNA. The RNA is transcribed into 
complementary DNA (cDNA) using enzyme 
reverse transcriptase. The cDNA is then 
amplified by PCR. 


Dr. Khalid Yusuf El-Zohry - Sohag Teaching Hospital (01118391123) 

Ref MRCPass OE OE 2012 PasTest 2009 PassMedicine 2009 PasTest Exam ReviseMRCP 










































El-zohry MRCP Questions Bank (Port 1) - 2013 


(For my personal use) 


5 ' 


Tagged O/ ^ 
HCV &/ 
primer 


No priming 



(♦)-strand RNA 


a = self priming 
b = random priming 
c = false priming 



primer primer 


5’ (-)-strand cDNA 


♦ 


involves tumour suppressor genes? 

1- Neurofibromatosis 

2- Sarcoma 

3- Down's syndrome 

4- Adenomatous polyposis coli 

5- Parkinson's disease 



TAG PCR Downstream PCR 

primer primer 


(♦)-strand cDNA 


Reverse Transcriptase PCR 


Answer & Comments 


Answer: 1- Neurofibromatosis 



[ Q: 1562 ] MRCPass - Basic Science 

An 25 year old man was admitted 
with breathlessness. He has not been well for 
a few days. 

His arterial blood gases show : 
pH 7.22 
p0 2 15 kPa 
pCo 2 3.2 kPa 

bicarbonate 14.7 mmol/L 
base excess -5. 

What is this picture consistent with? 

1- Acute asthma 

2- Bulimia 

3- Paracetamol overdose 

4- Acute liver failure 

5- Diabetic ketoacidosis 


Two hits of the tumour suppressor genes are 
required for loss of regulation (e.g. loss of 
both NF-1 genes). 

NF-1 gene in neurofibromatosis, BRCA-1 in 
breast and ovarian cancer, Rb gene and the 
VHL gene involve tumour suppressor genes. 
The oncogene SRC is affected in sarcoma. 

[ Q: 1564 ] MRCPass - Basic Science 

The screening procedure in which on 
antibody directed against a desired protein , is 
used to examine bacterial transformants (for 
the presence of a specific recombinant) is 
referred to as a: 

1- Me Fadden's blot 

2- Southern blot 

3- Northern blot 

4- Western blot 

5- Eastern blot 



Answer & Comments 

Answer: 5- Diabetic ketoacidosis 

This patient has metabolic acidosis with 
respiratory compensation. The low 
bicarbonate and low pH suggests metabolic 
acidosis. In order to compensate, the body 
hyperventilates, blow ing off C02 and having 
a relatively high p02. 



[ Q: 1563 ] MRCPass - Basic Science 
Which one of the following diseases 


Answer & Comments 

Answer: 4- Western blot 

Western blot method uses antibodies to 
examine proteins immobilized to a 
membrane support. 

Southern blot uses a nucleotide sequence 
probe to examine DNA fragments separated 
by gel electrophoresis, then transferred to 
and immobilized on a membrane support. 


Dr. Khalid Yusuf El-Zohry - Sohag Teaching Hospital (01118391123) . 

Ref MRCPass OE OE 2012 PasTest 2009 PassMedicine 2009 PasTest Exam ReviseMRCP 627 




































El-zohry MRCP Questions Bank (Part 1) - 2013 


(For my personal use) 


Northern blot uses a nucleotide sequence 
probe to examine RNA immobilized to a 
membrane support. 

Eastern blot does not exist. 


Protein Blot on SDS Polyacrylamide 

Nitrocellulose Gel Electrophoresis 



Label with Specific 


Antibody 

Detect Antibody 

— 




-> 


— 




Reveals Protein 
of Interest 


* typhoid - oral 



[ Q: 1566 ] MRCPass - Basic Science 

Mutations in myosin have recently 
been found to underlie the disease familial 
hypertrophic cardiomyopathy. 


Which one of the following regarding myosin 
is correct? 


1- It is not involved in striated muscle 
contraction 

2- Form filaments in a hexameric array of 2 
heavy chains only 

3- Myosin chain mutation is not associated in 
familial hypertrophic cardiomyopathy 

4- Carney complex is not a form of myosin 
chain disorder 

5- There are sites on myosin which allow for 
ATP and actin binding 


[ Q: 1565 ] MRCPass - Basic Science 

fh - 

# A 45 year old man who has a renal 
transplant is on high dose long term steroids 
and immunosuppression. He is about to 
travel abroad and seeks vaccination advice. 


Which one of the following vaccinations is 
contraindicated in this man? 


1- Yellow fever 


Answer & Comments 

Answer: 5- There are sites on myosin which 
allow for ATP and actin binding 

Myosin is involved in striated muscle 
contraction. 

It forms filaments in a hexameric array of 2 
heavy chains and 2 pairs of light chains. 


2- Haemophilus 

3- Meningococcus 

4- Tetanus toxoid 

5- Diphtheria toxoid 


Answer & Comments 

Answer: 1- Yellow fever 

The yellow fever is a live vaccine. These are 
other examples: 

* measles, mumps and rubella 


Myosin heavy chain mutation is implicated in 
familial hypertrophic cardiomyopathy. HOCM 
and Carney complex are forms of myosin 
chain disorders. 



There are sites on myosin which allow for ATP 

and actin binding. 


* BCG 

* poliomyelitis - oral Sabin vaccine 



[ Q: 1567 ] MRCPass - Basic Science 


Dr. Khalid Yusuf El-Zohry - Sohag Teaching Hospital (01118391123) 

Ref MRCPass OE OE 2012 PasTest 2009 PassMedicine 2009 PasTest Exam ReviseMRCP 


























































El-zohry MRCP Questions Bank (Part 1) - 2013 


(For my personal use) 


A 55 year old man has been diagnosed with 
Burkitt's Lymphoma. 

Which gene mutation is associated with this 
condition? 

1- BRAF 

2- P53 

3- C-myc 

4- N-myc 

5- Bcr-Abl 


Answer & Comments 

Answer: 3- C-myc 

In Burkitt's lymphoma (associated with 
Ebstein Barr virus), genetic translocations e.g. 
t(8:14) lead to consequent c-myc 
rearrangement and overexpression. 


^ [ Q: 1568 ] MRCPass - Basic Science 

t 1 - a — 1 —- 

# A 18 year old girl has had a 3rd 
presentation with meningocccal meningitis. 
An immunologist suspects a problem with her 
immune system. 

Which of these are most likely to be 
defective? 

1- Macrophages 

2- IgG or IgM 

3- Neutrophils 

4- Lymphocytes 

5- Complement C5-9 


Answer & Comments 

Answer: 5- Complement C5-9 

Neisseria infection leading to meningococcal 
meningitis often occurs in patients with 
complement deficiencies of C5-9. 
Complements C5-C9 form the Membrane 
Atttack Complex. Deficiencies in these 
complements lead to depressed bactericidal 
activity. 



[ Q: 1569 ] MRCPass - Basic Science 

A 30 year old man was admitted 
with status epilepticus. He is given 


intravenous diazepam. 


Arterial blood gases on 15 l/min via reservoir 
bag mask showed pH 7.05, paC0 2 8 kPa, pa0 2 
15 kPa, and standard bicarbonate 16 mmol/l. 


His other results were sodium 140 mmol/l, 
potassium 4 mmol/l, and chloride 98 mmol/l. 

What is the acid-base disturbance? 


1- Hyperchloraemic metabolic acidosis 

2- Normal anion gap metabolic acidosis 

3- High anion gap metabolic acidosis 

4- Low anion gap metabolic acidosis 

5- Respiratory acidosis 


Answer & Comments 

Answer: 3- High anion gap metabolic acidosis 

This patient had acidosis with both a high 
PaC02 and a low standard bicarbonate--a 
mixed acidosis. The anion gap was 30 mmol/l 
(increased). 

The Pa02 is lower than expected because the 
patient was breathing around 70% oxygen. 
This fits with the the clinical picture: he had a 
lactic acidosis from prolonged fitting and a 
respiratory acidosis from intravenous 
diazepam. 


[ Q: 1570 ] MRCPass - Basic Science 

Which of the following tumour 
suppresor genes is a tumour suppresor gene 
involved in promoting apoptosis and 
programmed cell death? 

1- BRCA-1 

2- P53 

3- Bcl-2 

4- Ras 

5- Rb 



Dr. Khalid Yusuf El-Zohry - Sohag Teaching Hospital (01118391123) 

Ref MRCPass OE OE 2012 PasTest 2009 PassMedicine 2009 PasTest Exam ReviseMRCP 



629 






























El-zohry MRCP Questions Bank (Part 1) - 2013 


(For my personal use) 


Answer & Comments 

Answer: 2- P53 

Ras is an oncogene. Bcl-2 inhibits rather than 
promotes apoptosis. 


Normal Celia. 


p53 bi nds as a tetramer to activate 
expression of growth-inhibitory genes 



p53 binding Growth-inhibitory 
si te ge me 


Occasional Tumours: 

deletion of one or 
both p53 alleles 


Reduced expression of 
growth-inhibitory genes 


/ 



p53 binding Growth-inhibitory 
site gene 


Various tumours: 


No expression of 
growth-inhibitory genes 

/ 


Ml** 


M utati ons r es ul ti mg i n p 53 bi ndi ng G rowt h i nh(bi tor y 

t r u ncation or el teration si ^ m 


p53 tumour suppressor gene 


[ Q: 1571 ] MRCPass - Basic Science 

Which one of the following is a 
trinucleotide repeat disorder? 

1- Fragile X syndrome 

2- Duchenne's muscular dystrophy 

3- Multiple Sclerosis 

4- Klinefelter's syndrome 

5- Turner's syndrome 

Answer & Comments 

Answer: 1- Fragile X syndrome 

Fragile X syndrome, myotonic dystrophy, 
Huntington's disease and Friedrich's ataxia 
are trinucleotide repeat disorders. 




[ Q: 1572 ] MRCPass - Basic Science 

The parents of a child with cystic 
fibrosis w ant to know the risk of their next 
child being a carrier of the condition, risk of 
the child being not affected or affected. 


Which ONE of the following is the correct risk 
for the child being only a carrier? 


1 - 10 % 

2- 25% 

3- 50% 

4- 75% 

5- 100% 


Answer & Comments 

Answer: 3- 50% 

Cystic Fibrosis has autosomal recessive 
inheritance. 

As both parents are carriers (AxA,BxB) of the 
Cystic Fibrosis gene, then chances of another 
child being affected (AxBx) is 1 in 4 (25%). 

The chances of their child being free from the 
CF gene (AB) is also 1 in 4 (25%). 

The chances of a child being a carrier (AxB or 
ABx) is 1 in 2 (50%). 



Unaffected 

"Carrier" 

Father 


Unaffected 

"Carrier’' 

Mother 




■' u H f 


Unaffected ’ CARRIER * CARRIER" Affected 

l in 4 chance Lttaffected IMaffected l in 4 chance 

1 in 4 chance 1 in 4 chance 


[ Q: 1573 ] MRCPass - Basic Science 
What form of virus is the herpes 

1- Triple stranded RNA virus 

2- Single stranded RNA virus 

3- Single stranded DNA virus 

4- Double stranded RNA virus 

5- Double stranded DNA virus 



virus? 



Dr. Khalid Yusuf El-Zohry - Sohag Teaching Hospital (01118391123) 

Ref MRCPass OE OE 2012 PasTest 2009 PassMedicine 2009 PasTest Exam ReviseMRCP 


630 


















































El-zohry MRCP Questions Bank (Port 1) - 2013 


(For my personal use) 


Answer & Comments 

Answer: 5- Double stranded DNA virus 
Herpes virus is a double stranded DNA virus. 



[ Q: 1574 ] MRCPass - Basic Science 

A 50 year old man complains of 
visual loss. On examination, he has right sided 
homonymous inferior quadrantinopia. 

Which one of the following lesions is likely? 

1- Left sided temporal area 

2- Left sided parietal area 

3- Occipital lobe 

4- Optic chiasm 

5- Optic nerve 


Answer & Comments 

Answer: 2- Left sided parietal area 

A contralateral lower parietal lesion causes 
lower homonymous quadrantinopia as 
described. 



[ Q: 1575 ] MRCPass - Basic Science 
Western blotting is used to detect: 


1- DNA 

2- RNA 

3- Protein 

4- Antibodies 

5- Enzymes 


Answer & Comments 

Answer: 3- Protein 

Western blotting can be used to detect and 
quantify proteins (e.g. bovine protein). 



[ Q: 1576 ] MRCPass - Basic Science 

Which one of the following features 
is consistent with higher cortical involvement 


rather than a diagnosis of a subcortical 
lacunar stroke? 

1- Ataxia 

2- Dysarthria 

3- Dysphasia 

4- Pure motor features 

5- Pure sensory symptoms 


Answer & Comments 

Answer: 3- Dysphasia 

Evidence of higher cortical involvement, for 
example - dysphasia, dyscalculia or or 
disturbance of consciousness, would not be 
consistent with a lacunar syndrome. 



[ Q: 1577 ] MRCPass - Basic Science 

Which one of the following blood 
gas results may be an analytical error? 

1- PH 7.6, paC0 2 - 2, p0 2 -13, Bicarbonate 30 

2- PH 7.3, paC0 2 - 7, p0 2 -10, Bicarbonate 14 

3- PH 7.5, paC0 2 - 8 , p0 2 -10, Bicarbonate 14 

4- PH 7.5, paC0 2 - 6, p0 2 -12, Bicarbonate 26 

5- PH 7.2, paC0 2 - 9, p0 2 9, Bicarbonate 12 


Answer & Comments 

Answer: 3- PH 7.5, PaC02 - 8, P02 - 10, 
Bicarbonate 14 


The patient has a high C02 and low 
bicarbonate which would suggest 
uncompensated respiratory acidosis, hence 
the pH is too high to fit the scenario. 



[ Q: 1578 ] MRCPass - Basic Science 


Which of the following is amplified 
by reverse transcriptase PCR? 


1- RNA 


2- Linear DNA 

3- Circular DNA 


Dr. Khalid Yusuf El-Zohry - Sohag Teaching Hospital (01118391123) 

Ref MRCPass OE OE 2012 PasTest 2009 PassMedicine 2009 PasTest Exam ReviseMRCP 






































El-zohry MRCP Questions Bank (Part 1) - 2013 


(For my personal use) 


4- Glycoproteins 

5- Reverse transcriptase enzyme 


Answer & Comments 


Answer: 4- Lactose 


Answer & Comments 

Answer: 1- RNA 

Reverse transcriptase PCR is used to amplify 
RNA, whilst conventional PCR is used to 
amplify DNA. 

[ Q: 1579 ] MRCPass - Basic Science 

Adrenaline is a product of which 
amino acid? 

1- Arginine 

2- Glutamine 

3- Valine 

4- Lysine 

5- Tyrosine 



Lactose is broken down into glucose and 
galactose by an enzyme called lactase. 


7 


[ Q: 1581 ] MRCPass - Basic Science 

A 75 year old lady has sudden 
movements of her arm where she throws her 
arm outwards, and uncontrollably injures 
herself. 


Which of the following areas could have 
sustained an infarct? 

1- Globus pallidus 

2- Pontine nucleus 


3- Corpus callosum 

4- Subthalamic nucleus 


5- Thalamus 


Answer & Comments 


Answer & Comments 


Answer: 5- Tyrosine 

Dopamine, adrenaline and noradrenaline are 
derived from tyrosine via the intermediate 
dopa. 


Answer: 4- Subthalamic nucleus 

Hemiballismus is caused by a subthalamic 
nucleus lesion, which is commonly due to an 
infarct. 


rrr C00H 

^^OOOH 

~ HO rrv° OH 


) 


nh, 2 

HC.AA 3 * 

m 

phenylalanine 

tyrosine 

L-p-(3,4-dlhydroxy- 
pheny1)-a-alanine (Dopa) 

vX 

4 

4 


HO 

HO 



4 



dopamine 


HO ' ^ " 2 

(R>noradrenaline 



(R)-adrenaline 


? 


Adrenaline synthesis 


[ Q: 1580 ] MRCPass - Basic Science 
Which one of the following breaks 


down into Glucose and Galactose? 

1- Ribose 

2- Sucrose 

3- Fructose 





Subthalamic Nucleus 


4- Lactose 

5- Phosphofructose 


Dr. Khalid Yusuf El-Zohry - Sohag Teaching Hospital (01118391123) 

Ref MRCPass OE OE 2012 PasTest 2009 PassMedicine 2009 PasTest Exam ReviseMRCP 



632 





































El-zohry MRCP Questions Bank (Part 1) - 2013 


(For my personal use) 


[ Q: 1582 ] MRCPass - Dermatology 

{% - 

* A 30 year lady presents has 

erythema nodosum. She has a low grade 
fever. There no recent history of travel 
abroad. 

Which of the following would be the most 
appropriate investigation for this patient? 

1- Colonoscopy 

2- ESR 

3- Upper Gl endoscopy 

4- Chest x ray 

5- Coxsackie viral serology 

Answer & Comments 

Answer: 4- Chest x ray 

A chest x ray would help exclude sarcoidosis, 
as most of the other causes are unlikely due to 
the history. The causes of erythema nodosum 
are streptococcal infection, acute sarcoidosis, 
oral contraceptive pill, sulphonamides, 
penicillins, inflammatory bow el disease, TB, 
Behcet's Disease. 


prognosis for malignant melanoma are the 
tumour thickness, measured from the 
overlying granular layer of the epidermis to 
the deepest easily identifiable tumour cells - 
the Breslow thickness, and the presence of 
ulceration of the primary tumour. 



Malignant melanoma 


[ Q: 1584 ] MRCPass - Dermatology 

r% - 

0 

A 23 year old student presents with 
extensive, hypopigmented, scaly lesions on his 
back and the chest. 


^ [ Q: 1583 ] MRCPass - Dermatology 

/ I---—--- 1 

A 60 year old lady presents with a 
nodular slightly pigmented skin lesion in her 
left arm. 

Which of the following change in the feature 
suggests poor prognosis if it is malignant? 

1- Increased diameter 

2- Increased depth 

3- Color 

4- Shape 

5- Consistency 

Answer & Comments 

Answer: 2- Increased depth 

The features are suggestive of Malignant 
melanoma. The main determinants of 


The rash had been present for the last 1 year 
and had gradually become more extensive. 

He is otherwise well. 

What is the treatment of choice? 

1- Ketoconazole cream 

2- Nystatin cream 

3- Terbinafine cream 

4- Oral terbinafine 

5- Oral itraconazole 

Answer & Comments 

Answer: 1- Ketoconazole cream 

The features are suggestive of Pityriasis 
versicolor infection, a skin infection which 
often presents as patches of relatively 
depigmented skin. The cause is overgrowth of 
the yeast Malassezzia furfur. If the skin is not 


Dr. Khalid Yusuf El-Zohry - Sohag Teaching Hospital (01118391123) 

Ref MRCPass OE OE 2012 PasTest 2009 PassMedicine 2009 PasTest Exam ReviseMRCP 





























El-zohry MRCP Questions Bank (Port 1) - 2013 


(For my personal use) 


tanned, the skin appears faw n with a slight 
brawny scaling and wrinkling. It is slightly 
itchy. It is usually a disease of young adults, 
predominantly affecting the upper trunk. 
Ketoconazole (topical) is the treatment of 
choice. 




[ Q: 1585 ] MRCPass - Dermatology 

A 40 year old man presented to his 
GP because of painful blisters on the backs of 
his hands in the summer. He also had a similar 
rash on the forehead. His hands, face and 
forehead were covered with thick, wrinkled, 
hyperpigmented skin. The patient's urine was 
reddish orange. 

Whot is the likely diagnosis? 


1- Contact dermatitis 


enzyme in porphyria cutanea tarda is 
uroporphyrinogen decarboxylase. 



Porphyria cutanea tarda 



[ Q: 1586 ] MRCPass - Dermatology 

A 45 year old man presents with 
discrete, round, scaly, pigmented patches on 
the back. Pityriasis versicolor is diagnosed by 
the dermatologist. 

Whot is the appropriate treatment? 

1- Terbinafine 


2- Prednisolone 

3- Rifampicin 

4- Ultraviolet light 

5- Topical selenium sulphide 


2- Pityriasis rosea 

3- Epidermolysis bullosa 

4- Pemphigoid 

5- Porphyria cutanea tarda 

Answer & Comments 

Answer: 5- Porphyria cutanea tarda 

In porphyria cutanea tarda, the urine 
fluoresces pink to red. Porphyria cutanea 
tarda's onset is typically during the fourth or 
fifth decade of life. The disease tends to 
develop, recur, or worsen during the spring 
and summer, when exposure to sunlight is 
greatest. Photosensitivity is the hallmark of 
porphyria cutanea tarda. The deficient 


Answer & Comments 

Answer: 5- Topical selenium sulphide 

Pityriasis versicolor (also called tinea 
versicolor) is a skin infection caused by a 
fungus called Malassezia furfur. Topical 
antifungal medications - containing either 
2.5% selenium sulfide or ketoconazole cream 
can be used. Oral ketoconazole 200 mg daily 
for 7 days can also be used. Other topical 
antifungal agents such as clotrimazole, 
miconazole or terbinafine are less widely 
recommended. 

^ [ Q: 1587 ] MRCPass - Dermatology 

# A 45 year old patient had renal 
transplantation 8 years ago. He is on regular 



Dr. Khalid Yusuf El-Zohry - Sohag Teaching Hospital (01118391123) 

Ref MRCPass OE OE 2012 PasTest 2009 PassMedicine 2009 PasTest Exam ReviseMRCP 


























El-zohry MRCP Questions Bank (Port 1) - 2013 


(For my personal use) 


immunosuppressive medication. He presented 
with a large warty like growth on the back of 
his buttocks. 

What should be done next? 

1- Observation 

2- Skin biopsy 

3- Stop immunosuppressives 

4- Cryotherapy 

5- Viral PCR 


Answer & Comments 

Answer: 2- Skin biopsy 

Warts are caused by the human papilloma 
virus, and tend to develop after 4 to 5 years 
following transplantation. The risk of skin 
cancer is increased especially in sun exposed 
areas. Treatment of all warts are usually with 
cryotherapy, However with this growth a 
biopsy is necessary followed by possible 
surgery. 


[ Q: 1588 ] MRCPass - Dermatology 

A 20 year old male, born of non- 
consanguineous parents, was referred to our 
department with complaint of the insidious 
appearance of red-purple papules over the 
trunk with occasionally itching and pin-point 
bleeding, which had progressed over the past 
4 years. These papules slow ly increased in 
number and size. There was no history of 
anhidrosis, abdominal or bone pain, acral 
paresthesias, chest pain, or loss of visual 
acuity. 

On examination, Clusters of individual, 
punctate, dark red to blue-black papules were 
seen mainly over the anterior chest, back, and 
hips. 

What is the likely diagnosis? 

1- Erythema multiforme 

2- Erythema marginatum 

3- Angiokeratoderma corporis diffusum 



4- Cafe au lait spots 

5- Neurofibromas 


Answer & Comments 

Answer: 3- Angiokeratoderma corporis 

diffusum 


Systemic angiokeratoma corporis diffusum 
(Anderson-Fabry disease) is an unusual X-liked 
lysosomal disorder characterized by deficiency 
of ?-galactosidase. Onset of the disease 
usually occurs during childhood or 
adolescence with periodic crises of severe 
pain in the extremities (acroparesthesias) and 
the appearance of vascular cutaneous lesions 
(angiokeratomas), hypohidrosis or anhidrosis, 
and characteristic corneal and lenticular 
opacities. 



Angiokeratoderma corporis diffusum 


[ Q: 1589 ] MRCPass - Dermatology 

A 72 year old woman has developed 
a rash over 6 weeks. On examination, there 
were numerous fluid filled blisters over upper 
and lower limbs as well as the trunk. 

What is the likely diagnosis? 

1- Pemphigus 

2- Pemphigoid 

3- Eczema 

4- Psoriasis 

5- Erythema nodosum 



Answer & Comments 


Answer: 2- Pemphigoid 


Dr. Khalid Yusuf El-Zohry - Sohag Teaching Hospital (01118391123) 

Ref MRCPass OE OE 2012 PasTest 2009 PassMedicine 2009 PasTest Exam ReviseMRCP 



635 




























El-zohry MRCP Questions Bank (Port 1) - 2013 


(For my personal use) 


Pemphigoid is a disease of the elderly, rarely 
presenting before 50 years of age. 
Presentation is with large, tense, itchy blisters 
on an erythematous base. They usually occur 
on the upper arms and thighs and may spread 
to the trunk. Lesions may be 2-5 cm in 
diameter. They are thick walled and may last 
for several days. 



Bullous pemphigoid 


[ Q: 1590 ] MRCPass - Dermatology 

A 60 year old lady has developed an 
uncomfortable venous ulcer on leg. On 
examination, her leg is oedematous but 
peripheral pulses are palpable. Doppler 
ultrasound reveals no arterial disease. 

Which one of the following is the most 
appropriate management? 

1- Leg elevation 

2- Surgical debridement 

3- Intravenous antibiotics 

4- Compression dressing 

5- Topical steroids 



Answer & Comments 

Answer: 4- Compression dressing 

Compression helps to control painful sw elling 
from fluid buildup (edema) and is recognized 
as effective treatment for venous ulceration. 


on her arms. Examination showed an 
erythematous papular eruption with possible 
pustules in the arm. There were burrows 
between the fingers. 

What is the diagnosis? 

1- Molluscum contagiosum 

2- Scabies 

3- Dermatitis herpetiformis 

4- Impetigo 

5- Psoriasis 


Answer & Comments 

Answer: 2- Scabies 

Scabies is spread by Sarcoptes scabei. It is 
spread by contact only. Effective agents are 
benzyl benzoate, ivermectin and permethrin 
(Lyclear solution). 



Scabies Rash 



Scabies Mite 


\h. 

[ Q: 1591 ] MRCPass - Dermatology 


A 58 year old woman was seen 

fl 

[ Q: 1592 ] MRCPass - Dermatology 

because of a three day history of itchy bumps 





Dr. Khalid Yusuf El-Zohry - Sohag Teaching Hospital (01118391123) 

Ref MRCPass OE OE 2012 PasTest 2009 PassMedicine 2009 PasTest Exam ReviseMRCP 
































El-zohry MRCP Questions Bank (Port 1) - 2013 


(For my personal use) 


A 50 year old woman has had ulcerative colitis 
diagnosed 10 years ago. Over the last 5 
months, she has developed an ulcer above the 
medial malleolus. This is large has a necrotic 
base and the edges are undermined. 

How should it be managed? 

1- Foam dressings 

2- Debridement 

3- Broad spectrum antibiotics 

4- High dose steroids 

5- Emmolients 

Answer & Comments 

Answer: 4- High dose steroids 

The diagnosis is pyoderma gangrenosum 
complicating ulcerative colitis. The initial 
treatment should be with high dose steroids 
orally and topical steroids. Methotrexate and 
ciclosporin can also be used. 



Pyoderma Gangrenosum 


^ [ Q: 1593 ] MRCPass - Dermatology 

o - 

# A 35 year old man presented to his 
general practitioner with a rash beginning 
around the left ear and spreading into the 
periorbital region. He was then referred 
urgently to the hospital for assessment. The 
patient had a history of atopic eczema (since 
childhood), rhinitis, and asthma. He was taking 
inhaled corticosteroids for asthma and had 
last received a course of systemic steroids one 
year previously. There were no risk factors for 
HIV infection. 


On examination he was alert, he was pyrexial 
at 39.2°C, his pulse was 100 bpm, and his 
blood pressure was 130/70 mm Hg. There was 
a widespread erythematous rash covering his 
face, chest, and arms, which was described as 
wet with a yellow ish exudate. The rash 
extended around both eyes, which could not 
be opened. 

What is the likely diagnosis? 

1- Varicella zoster infection 

2- Pityriasis versicolor 

3- Psoriasis 

4- Pemphigus 

5- Eczema herpeticum 

Answer & Comments 

Answer: 5- Eczema herpeticum 

Eczema herpeticum represents widespread 
cutaneous HSV in patients with pre-existing 
skin disorders. Eczema herpeticum is also 
known as Kaposi's Varicelliform Eruption. The 
patient often presents with clusters of 
umbilicated vesicles appearing on abnormal or 
even apparently normal skin. The eruption 
then spreads over the following 7 to 10 days, 
and the umbilicated vesicles evolve into classic 
discrete "punched-out" small erosions. 

Typically, the patient experiences fever, 
malaise, and generalized lymphadenopathy. 
The vesicles may breakdown and coalesce into 
large erosions that have a tendency to be 
complicated by secondary infection. The 
course of this condition is usually 2 to 6 
weeks. Recurrent episodes may occur, but the 
clinical presentation is typically milder. 

^ [ Q: 1594 ] MRCPass - Dermatology 

- 

# A 50 year old woman with atopic 
eczema presented with worsening skin 
symptoms of erythema and pruritus. 

She had tried skin emmolients, and topical 
betnovate. 


Dr. Khalid Yusuf El-Zohry - Sohag Teaching Hospital (01118391123) 

Ref MRCPass OE OE 2012 PasTest 2009 PassMedicine 2009 PasTest Exam ReviseMRCP 



637 

























El-zohry MRCP Questions Bank (Port 1) - 2013 


(For my personal use) 


Which drug is most appropriate next? 

1- Retinoids 

2- Hydrocortisone 

3- Coal tar 

4- Cyclosporin 

5- Minocycline 

Answer & Comments 

Answer: 4- Cyclosporin 

The use of cyclosporin in Atopic Dermatitis has 
been fairly well studied and a high percentage 
of patients improve with therapy. 



Atopic Eczema 


[ Q: 1595 ] MRCPass - Dermatology 

A 45 year old man who has been 
exposed to the sun is complaining of a rash 
with blistering on his hands, chest and 
forehead. On examination, there were small 
areas of excoriation around the rash. 

What is the diagnosis? 

1- Pustular psoriasis 

2- Pemphigoid 

3- Dermatitis herpetiformis 

4- Porphyria cutanea tarda 

5- Granuloma annulare 



Answer & Comments 

Answer: 4- Porphyria cutanea tarda 

Patients with Porphyria Cutanea Tarda 
develop liver damage and fragile skin and 


fluid-filled blisters on the face, arms, hands 
and other skin exposed to sun. 



Blistering rash seen in porphyria cutanea tarda 


[ Q: 1596 ] MRCPass - Dermatology 

A 45 year old lady presents to the 
dermatologist with target like lesions. The 
circular lesions have a central blister. 

Which one of the following is the most 
common cause? 

1- Pemphigoid 

2- Herpes simplex 

3- Mycoplasma 

4- Vitamin C deficiency 

5- Orf 



Answer & Comments 

Answer: 2- Herpes simplex 

The lesions are erythema multiforme. The 
commonest cause is quoted to be herpes 
simplex, although all the others are also 
causes. 



Erythema Multiforme 



Dr. Khalid Yusuf El-Zohry - Sohag Teaching Hospital (01118391123) 

Ref MRCPass OE OE 2012 PasTest 2009 PassMedicine 2009 PasTest Exam ReviseMRCP 


























El-zohry MRCP Questions Bank (Part 1) - 2013 


(For my personal use) 


[ Q: 1597 ] MRCPass - Dermatology 

/ - 

* A 42 year old woman has developed 

a rash on her face over a year and a half. The 
rash is is erythematous and scaly. There are 
areas of alopecia on the scalp. 

What is the diagnosis? 

1- Lupus pernio 

2- Discoid lupus 

3- Erythroderma 

4- Dermatitis herpetiformis 

5- Scabies 


Answer & Comments 

Answer: 2- Discoid lupus 

Discoid lupus erythematosus is characterized 
by inflammation and scarring type skin lesions 
which occur on the face, ears and scalp. These 
lesions develop as an inflamed growth with, 
scaling and a warty like appearance. The 
center areas may appear lighter in color 
surrounded by an area darker than the normal 
skin. When lesions occur in hairy areas such as 
the scalp, permanent scarring and hair loss 
can occur. 



Discoid lupus 



[ Q: 1598 ] MRCPass - Dermatology 

A 42 year old man has been treated 
by the GP for 2 days for presumed psoriasis 
with topical betnovate. 


However, his rash worsened and now presents 
with severe erythema all over the body. 


What treatment is recommended? 

1- Oral steroid 

2- Cooling with air 

3- Topical tar 

4- Topical steroids 

5- Topical soft white paraffin 


Answer & Comments 

Answer: 5- Topical soft white paraffin 

A patient with severe erythroderma should be 
hospitalized for supportive care. Cooling is 
done with intravenous fluids and regulating 
room temperature. Emmolients should be 
used. Steroids and topical tar preparations 
should be avoided. 



Erythroderma 


[ Q: 1599 ] MRCPass - Dermatology 

A 62 year old woman has a rash on 
the trunk with concentric erythematous bands 
forming a wood-grain appearance. 

Which one of the following erythematous 
conditions is associated with gastric 
carcinoma? 



Dr. Khalid Yusuf El-Zohry - Sohag Teaching Hospital (01118391123) 

Ref MRCPass OE OE 2012 PasTest 2009 PassMedicine 2009 PasTest Exam ReviseMRCP 



639 




























El-zohry MRCP Questions Bank (Port 1) - 2013 


(For my personal use) 


1- Erythema multiforme 

2- Erythema ab igne 

3- Erythema gyratum repens 

4- Erythema nodosum 


Answer & Comments 

Answer: 4- Erythema nodosum 

This is a description of erythema nodosum 
related to inflammatory bow el disease. 


5- Erythroderma 

Answer & Comments 

Answer: 3- Erythema gyratum repens 

Eythema Gyratum Repens is associated with 
malignancy in as many as 80% of patients. 
Among visceral malignancies, the lung is the 
most common site, followed by the breast, 
urinary bladder, uterus and/or cervix, Gl tract 
(stomach), and prostate. 



Erythema Gyratum repens 


[ Q: 1600 ] MRCPass - Dermatology 

A 27 year old complained of 
diarrhoea, abdominal pain and fever for the 
past few days. 

On examination there was a visible mouth 
ulcer and anal skin tags. On the lower shin 
there were red painful raised skin lesions. 

Whot is the most probable diagnosis? 

1- Behcet's disease 

2- Pyoderma gangrenosum 

3- Erythema multiforme 

4- Erythema nodosum 

5- Erythema marginatum 



The fever, dirrhoea and abdominal pains may 
be due to ulcerative colitis or Crohn's disease. 



Erythema nodosum 


[ Q: 1601 ] MRCPass - Dermatology 

A 37 year old man presented to his 
general practitioner with a rash beginning 
around the right ear and spreading into the 
periorbital region. He was known to have 
eczema and also was HIV positive. 

3 days later, he was systemically unw ell, 
complaining of sweats and rigors. On 
examination he was alert, he was pyrexial at 
39.2°C, his pulse was 100 bpm, and his blood 
pressure was 128/70 mm Hg. There was a 
widespread erythematous rash covering his 
face, chest, and arms. The rash extended 
around both eyes, which could not be opened. 
On closer inspection of the rash, it was 
apparent that there were multiple small 
vesicles 1-2 mm in diameter and shallow 
ulcers over the face and periorbital region. 

What is the likely diagnosis? 

1- Dermatitis herpetiformis 

2- Smallpox 




Dr. Khalid Yusuf El-Zohry - Sohag Teaching Hospital (01118391123) 

Ref MRCPass OE OE 2012 PasTest 2009 PassMedicine 2009 PasTest Exam ReviseMRCP 
























El-zohry MRCP Questions Bank (Port 1) - 2013 


(For my personal use) 


3- Tinea versicolor 

4- Pemphigus vulgaris 

5- Eczema herpeticum 


Answer & Comments 

Answer: 5- Eczema herpeticum 

The diagnosis is consistent with eczema 
herpeticum and probable secondary bacterial 
infection. This is caused by HSV-1, and is more 
common among immunosuppressed 
individuals. Treatment is with aciclovir. 



Eczema Herpeticum 


[ Q: 1602 ] MRCPass - Dermatology 

A 60 year old man presented with a 
pruritic rash. Burrows were found on his arms 
and in the web spaces of the fingers. He was 
treated with Permethrin creams and 
improved. 3 weeks later, he came to the 
hospital again when the pruritus returned 
suddenly. 

What is the likely cause for this? 

1- Allergy to Permethrin cream 

2- Re-infection with scabies mites 

3- Type 4 hypersensitivity to mite feces 

4- Erythema multiforme 

5- Dermatitis herpetiformis 



Answer & Comments 

Answer: 3- Type 4 hypersensitivity to mite 
feces 

A delayed type IV hypersensitivity reaction to 
the mites, their eggs, or scybala (ie, packets of 
feces) occurs approximately 30 days after 
infestation. This reaction is responsible for the 
intense pruritus that is the hallmark of the 
disease. Individuals who already are sensitized 
from a prior infestation can develop 
symptoms within hours when exposed to any 
allergens. 



[ Q: 1603 ] MRCPass - Dermatology 

A 40 year old HIV-infected man 
comes in for routine care and evaluation of 
skin lesions on his face. His most recent tests 
showed a CD4 count of 38 cells/mm * 1 2 3 4 5 and HIV 
RNA of 87,000 copies/ml. He is not compliant 
on antiretroviral therapy. The patient 
describes a 2-3 month history of persistent 
papules on his fingers that have gradually 
increased in number and size. 


What is the likely diagnosis? 

1- Molluscum contagiosum 

2- Kaposi's sarcoma 

3- Herpes zoster virus infection 

4- Orf 


5- Malignant melanoma 


Answer & Comments 

Answer: 1- Molluscum contagiosum 

Molluscum contagiosum is caused by the DNA 
pox virus. Papular lesions occur first followed 
by macular lesions. Pruritus can occur, but 
erythema or inflammation is uncommon 
around the lesion. The lesion is present for 
several weeks and is self limiting. 


Dr. Khalid Yusuf El-Zohry - Sohag Teaching Hospital (01118391123) 

Ref MRCPass OE OE 2012 PasTest 2009 PassMedicine 2009 PasTest Exam 




ReviseMRCP 

641 


























El-zohry MRCP Questions Bank (Part 1) - 2013 


(For my personal use) 





Molluscum Contagiosum 


[ Q: 1604 ] MRCPass - Dermatology 


An 80 year old man complained of 
bullous skin lesions on both the upper and 
lower extremities. On examination there were 

1-2 cm blistering lesions throughout the body. 

What is the most probable diagnosis? 

1- Pemphigus vulgaris 

2- Erythema multiforme 

3- Dermatitis herpetiformis 

4- Bullous pemphigoid 

5- Insect bite 


Answer & Comments 

Answer: 4- Bullous pemphigoid 

Bullous pemphigoid is more common than 
pemphigus , occurs more commonly in later 
life (>60 years). Large bullae appears anyw 
here on the skin ,they tend to be itchy and the 
lesions are deep and mucosal involvement 
rare. The reverse is true for pemphigus 
vulgaris. 




[ Q: 1605 ] MRCPass - Dermatology 

A 30 year old woman presents with a 
rash. The rash consists of erythematous 
plaques, excoriations, and vesicles some of 
which have ruptured leaving a crust. She has 
had a past history of gluten sensitive 
enteropathy. 

What is the best treatment for the rash? 


1- Prednisolone 


2- Azathioprine 

3- Dapsone 

4- Calamine lotion 


5- E45 cream 


Answer & Comments 

Answer: 3- Dapsone 

Dapsone (diaminodiphenyl sulfone) and 
sulfapyridine are the primary medications 
used to treat Dermatitis Herpetiformis (DH). 
Dapsone often is used initially; sulfapyridine is 
substituted in patients unable to tolerate 
dapsone. 

Improvement may be dramatic; symptomatic 
improvement of skin lesions often begins 
within hours. Other, less effective, treatments 
for DH include colchicine, cyclosporine, and 
prednisone. 



Dermatitis Herpetiformis 



Dr. Khalid Yusuf El-Zohry - Sohag Teaching Hospital (01118391123) 

Ref MRCPass OE OE 2012 PasTest 2009 PassMedicine 2009 PasTest Exam ReviseMRCP 


642 
























El-zohry MRCP Questions Bank (Port 1) - 2013 


(For my personal use) 


^ [ Q: 1606 ] MRCPass - Dermatology 

tl - 

# A 14 year old girl has developed 
ulcerated lumps on the back of his legs. She 
has a previous history of tuberculosis. 

Whot is the likely diagnosis? 

1- Erythema multiforme 

2- Erythema nodosum 

3- Lupus erythematosus 

4- Erythema induratum 

5- Granuloma annulare 


2- Prednisolone 

3- UV B 

4- Oral retinoids 

5- Cryotherapy 

Answer & Comments 

Answer: 4- Oral retinoids 

Due to its anti-inflammatory activity, 
isotretinoin (retinoid) is an extremely effective 
drug if given systemically in severe forms of 
seborrhoea and acne. 


Answer & Comments 

Answer: 4- Erythema induratum 

Erythema induratum is a chronic, recurring 
panniculitis that is found predominantly on 
the legs of women with tuberculin 
hypersensitivity. 



Erythema Induratum 


[ Q: 1607 ] MRCPass - Dermatology 

A 19 year old patient with acne 
vulgaris has already been treated with a 
course of minocycline, but the condition is not 
responding well to treatment. 

Whot treatment option should be considered 
next? 

1- Erythromycin 



[ Q: 1608 ] MRCPass - Dermatology 

A 52 year old man has recently 
travelled back from South America 2 weeks 
ago on holiday. He presents to the 
dermatologist with a facial ulcer with 
granulating base. 

Whot is the likely diagnosis? 

1- Behcet's disease 

2- Cutaneous leishmaniasis 

3- Syphilis 

4- Basal cell carcinoma 

5- Squamous cell carcinoma 



Answer & Comments 

Answer: 2- Cutaneous leishmaniasis 

Cutaneous leishmaniasis is spread by female 
sandflys of the genus Phlebotomus. The 
causative agents include L. (V.) braziliensis, L. 
(L.) mexicana, L. (V.) panamensis, and related 
species. Most infections follow a bite from an 
infected sandfly and remain subclinical. 
However, in some cases, after an incubation 
period of 1-12 weeks, a papule develops that 
enlarges and ulcerates. The typical lesion are 
crusty, painless ulcers on exposed skin. 
Ulcerative lesions are usually shallow and 
circular with well-defined, raised borders and 
a bed of granulation tissue. Local 
lymphadenopathy only occurs in the presence 


Dr. Kholid Yusuf El-Zohry - Sohog Teaching Hospital (01118391123) 

Ref MRCPass OE OE 2012 PasTest 2009 PassMedicine 2009 PasTest Exam ReviseMRCP 



643 





























El-zohry MRCP Questions Bank (Port 1) - 2013 


(For my personal use) 


of bacterial superinfection. Cutaneous 
leishmaniasis is found predominantly in South 
America, Central Africa, around the 
Mediterranean Sea and India. 




[ Q: 1609 ] MRCPass - Dermatology 

A 23 year old student presents with 
extensive, hypopigmented, slightly scaly 
lesions on his back and the chest. 


itchy. It is usually a disease of young adults, 
predominantly affecting the upper trunk. 



Pityriasis Versicolor 


[ Q: 1610 ] MRCPass - Dermatology 

A 65 year old man, who received a 
renal transplant 30 years ago, was evaluated 
in clinic when he complained of a lump behind 
his ear. He has been on ciclosporin and 
tacrolimus. On physical examination he had an 
ulcerated nodule behind the ear. 

Whot is the likely diagnosis? 

1- Seborrhoeic wart 



The rash had been present for the last 1 year 
and had gradually become more extensive. 

He is otherwise well. 

What is the treatment of choice? 

1- Ketoconazole cream 


2- Actinic keratosis 

3- Squamous cell carcinoma 

4- Basal cell carcinoma 

5- Malignant melanoma 


2- Nystatin cream 

3- Terbinafine cream 

4- Oral terbinafine 

5- Oral itraconazole 

Answer & Comments 

Answer: 1- Ketoconazole cream 

The features are suggestive of Pityriasis 
versicolor infection, a skin infection which 
often presents as patches of relatively 
depigmented skin. The cause is overgrowth of 
the yeast Malassezzia furfur. If the skin is not 
tanned, the skin appears faw n with a slight 
brawny scaling and wrinkling. It is slightly 


Answer & Comments 

Answer: 3- Squamous cell carcinoma 

Patients who have had renal transplants have 
a 60 fold increased risk of squamous cell 
carcinoma. Treatment is with radiotherapy, 
surgical excision and reduction of 
immunosuppression where possible. 



Squamous cell carcinoma 



Dr. Khalid Yusuf El-Zohry - Sohag Teaching Hospital (01118391123) 

Ref MRCPass OE OE 2012 PasTest 2009 PassMedicine 2009 PasTest Exam ReviseMRCP 

























El-zohry MRCP Questions Bank (Port 1) - 2013 


(For my personal use) 



[Q: 1611] MRCPass - 
Ophthalmology 

A 65 year old woman with a chronic disease 
has asymptomatic thinning of the sclera 
through which the underlying uveal tissue can 
be seen. 

Which of the following diseases is most 
strongly associated with scleromalacia 
perforans? 

1- Psoriasis 

2- HIV 

3- SLE 

4- Rheumatoid arthritis 

5- Wegener's granulomatosis 


Answer & Comments 

Answer: 4- Rheumatoid arthritis 

Scleromalacia perforans is painless thinning of 
the sclera. Vast majority of patients are 
females with longstanding rheumatoid 
arthritis (sero positive). It is rarely seen in 
Behcet's and ulcerative colitis. 



[Q: 1612] MRCPass - 
Ophthalmology 

A 25 year old woman presents with gritty 
eyes. Her children have similar symptoms. On 
examination, the conjunctivae are red and 
swollen with a discharge which is crusting the 
eyelashes. 

What is the most likely diagnosis? 

1- Keratoconjunctivitis sicca 

2- Bacterial conjunctivitis 

3- Anterior uveitis 

4- Hay fever 

5- Herpes simplex keratitis 


Answer & Comments 


Answer: 2- Bacterial conjunctivitis 


The clinical presentation fits conjunctivitis 
best. The most common organisms are 
staphylococcus, streptococcus and 
Haemophilus influenzae. Treatment is with 
chloramphenicol eyedrops. 



[Q: 1613] MRCPass- 
Ophthalmology 


A 25 year old female presents with a history of 
headache, vomiting and blurred vision for the 
last 2 weeks. Direct questioning reveals that 
she is taking a tablet for her spots (acne). She 
has a high BMI and bilateral papilloedema on 
examination. 


Which of the following investigations is most 
likely to help confirm the diagnosis? 

1- MRI brain 


2- Copper studies 

3- Anti Ach antibodies 


4- CSF pressure measurement 

5- Iron studies 


Answer & Comments 

Answer: 4- CSF pressure measurement 

The diagnosis is Benign Intracranial 
Hypertension (Pseudotumour Cerebri). 

This is commoner in obese women of child 
bearing age. It is also associated with the OCP, 
steroid withdrawal, tetracyclines, headache, 
nausea, visual symptoms and a Vlth cranial 
nerve palsy. 

CSF pressure is typically raised when lumbar 
puncture is performed. Removal of CSF may 
relieve the pressure and improve the visual 
symptoms and headaches. 


\M 

A 30 year old man presents with a 24 hour 
history of blurred vision in left eye and mild 
left frontal headache. He had a 15 year history 


[Q: 1614] MRCPass- 
Ophthalmology 


Dr. Khalid Yusuf El-Zohry - Sohag Teaching Hospital (01118391123) 

Ref MRCPass OE OE 2012 PasTest 2009 PassMedicine 2009 PasTest Exam ReviseMRCP 

































El-zohry MRCP Questions Bank (Port 1) - 2013 


(For my personal use) 


of diabetes mellitus. Examination of the left 
eye visual field shows a central scotoma. 

What is the likely diagnosis? 


drops dilate the pupil by stimulating the 
sympathetic system and should be used with 
caution in adults with ischaemic heart disease. 


1- Central retinal artery occlusion 

2- Optic neuritis 

3- Pituitary tumour 

4- Hypertensive retinopathy 

5- Diabetic retinopathy 


Answer & Comments 


Answer: 2- Optic neuritis 


[Q: 1616] MRCPass - 
Ophthalmology 

A 60 year old man with a history of 
hypertension presents with visual loss on the 
right side. On examination he has visual loss 
affecting the temporal field of his right eye 
and the nasal field of his left eye. 

Which of the following areas is likely to show 
infarction on brain imaging? 



A painful, more gradual onset visual 1- Left occipital lobe 

impairment is consistent with optic neuritis. , n - u . .. ,, , 

r 2- Right occipital lobe 


Occlusion of central retinal artery will result in 
almost immediate loss of visual acuity in the 
involved eye, usually without pain. 


3- Right frontal lobe 

4- Optic chiasm 

5- Right temporal lobe 


?il 

1 J 


[Q: 1615] MRCPass- 



Ophthalmology 


Answer & Comments 


A 65 year old woman who has had a previous 
myocardial infarction complains of a problem 
with the vision in her left eye. 

Prior to fundoscopy, the best agent to dilate 
her pupils with is: 


Answer: 1- Left occipital lobe 

The patient has a right homonymous 
hemianopia. Lesion is therefore behind optic 
chiasm and most likely to be affecting the left 
occipital lobe. 


1- Phenylephrine 10% 

2- Phenylephrine 2.5% 

3- Cyclopentolate 1% 

4- Tropicamide 0.5% 

5- Tropicamide 1% 


Answer & Comments 

Answer: 5- Tropicamide 1% 

Tropicamide (0.5% for children, 1% for adults) 
is best for diagnostic purposes, dilating the 
pupil for two to four hours by blocking the 
parasympathetic terminals in the papillary 
constrictor muscle. 

Cyclopentolate works in a similar mode but 
lasts for six to eight hours. Phenylephrine 



[Q: 1617] MRCPass - 
Ophthalmology 

A 48 year old woman develops painful 
bloodshot eyes. 

Which one of the following features would 
make scleritis more likely than iritis? 

1- Pain which wakes her from sleep 

2- Irregular pupil 

3- Keratitis 

4- Photophobia 

5- Visual blurring 


Answer & Comments 


Answer: 1- Pain which wakes her from sleep 


Dr. Khalid Yusuf El-Zohry - Sohag Teaching Hospital (01118391123) 

Ref MRCPass OE OE 2012 PasTest 2009 PassMedicine 2009 PasTest Exam ReviseMRCP 

































El-zohry MRCP Questions Bank (Port 1) - 2013 


(For my personal use) 


Features of iritis are photophobia, visual 
blurring, keratitic precipitates (best seen with 
a slit lamp) and irregularity of the pupil. 
Severe pain suggests scleritis and is not typical 
of iritis or episcleritis (which is milder and 
affects a smaller portion of the eye). In 
scleritis, the pain can be so severe that it 
disturbs sleep. 


^ [Q: 1618] MRCPass - 

ft 

S Ophthalmology 

A 35 year old artist has painful eyes which 
have been diagnosed as iritis by an 
opthalmologist requiring steroid treatment. 

Which of the following condition is a likely 
couse? 

1- Ankylosing spondylitis 

2- Gallstones 

3- Gout 

4- Osteoarthritis 

5- Pancreatitis 


Answer & Comments 


Answer: 1- Ankylosing spondylitis 


Iritis (anterior uveitis) is found in all the 
inflammatory conditions such as Behcet's 
disease, Reiter's syndrome, ankylosing 
spondylitis and sarcoidosis. 



Iritis 



[Q: 1619] MRCPass - 
Ophthalmology 


A 70 year man presents with sudden loss of 
vision in his right eye, associated with a 
relative afferent papillary defect. He has 
poorly controlled hypertension and smokes. 


Which of the following is likely? 


1- Optic nerve compression 


2- Optic neuritis 

3- Chronic open angle glaucoma 

4- Retinal vascular occlusion 

5- Macular degeneration 


Answer & Comments 

Answer: 4- Retinal vascular occlusion 

The history of acute loss of vision affecting 
one eye suggests an acute vascular occlusion. 
This is associated with an underlying disorder 
such as glaucoma, hypertension, diabetes, 
coagulation disorders, atherosclerosis, or 
hyperlipidemia. 

The visual loss after retinal vein occlusion is 
variable. Haemorrhage may be present, and 
the patient may be at risk for developing 
glaucoma. In retinal artery occlusion, there is 
a profound visual loss. The degree of loss is 
related to the location of the occlusion. There 
is also a further risk of stroke because the 
emboli could be throw n off to other parts of 
the brain. 

^ [Q: 1620] MRCPass- 

S Ophthalmology 

A 45 year old man recently returned from a 
holiday. He presents to A&E complaining of 
disturbance in vision in his right eye. 
Examination reveals a thickened ingrowth of 
conjunctiva with prominent vessels which has 
extended to overlie the cornea. 

Which of the following is the most likely 
diagnosis? 


Dr. Kholid Yusuf El-Zohry - Sohog Teaching Hospital (01118391123) 

Ref MRCPass OE OE 2012 PasTest 2009 PassMedicine 2009 PasTest Exam ReviseMRCP 



























El-zohry MRCP Questions Bank (Port 1) - 2013 


(For my personal use) 


1- Acute closure angle glaucoma 

2- Behcet's disease of the eye 

3- Conjunctivitis 

4- Pterygium 


Leber's hereditary optic neuropathy is a 
mitochondrial disease, which affects the 
retina. Patients may also develop ataxia and 
cognitive changes. Fundoscopy is usually 
limited in value. 


5- Scleritis 


Answer & Comments 


Answer: 4- Pterygium 


Gaucher's disease causes a pigmented retina, 
Law rence Moon Biedl causes retinitis 
pigmentosa, Multiple sclerosis causes a disc 
pallor. 


Pterygium is a raised, wedge-shaped growth 
of the conjunctiva. It is most common among 
those who live in tropical climates or spend a 
lot of time in the sun. Symptoms may include 
irritation, redness, and tearing. Pterygiums are 
nourished by tiny capillaries that supply blood 
to the tissue. As the pterygium develops, it 
may alter the shape of the cornea, causing 
astigmatism. If the pterygium invades the 
central cornea, it is removed surgically. 


A 18 year old man is referred with progressive 
visual loss. He first noticed that reading had 
become difficult, and then had difficulty 
seeing further distances. On examination, 
visual acuity is reduced to 6/18 bilaterally. 
There are bilateral central scotomas 
bilaterally, with loss of colour vision. 
Fundoscopy is normal, apart from possibly 
increased tortuosity of vessels. 

What is the most likely diagnosis? 

1- Diabetic retinopathy 

2- Gaucher's disease 

3- Leber's hereditary optic neuropathy 

4- Lawrence Moon Biedl syndrome 

5- Multiple sclerosis 



[Q: 1621] MRCPass - 
Ophthalmology 


Answer & Comments 

Answer: 3- Leber's hereditary optic 

neuropathy 



[Q: 1622] MRCPass - 
Ophthalmology 


A 18 year old man progressive visual loss. On 
fundoscopy, there are multiple, dark, bone 
corpuscles across the retina and pallor of the 
optic disc. Examination of his visual fieldds 
reveals peripheral field loss. 


Which one of the following is most likely to be 
the underlying diagnosis? 


1- Diabetes 


2- Hypertension 

3- Retinoblastoma 

4- Kearn Sayre's disease 

5- Neurofibromatosis 


Answer & Comments 

Answer: 4- Kearn Sayre's disease 

The clinical picture is consistent with retinitis 
pigmentosa. Causes are: 

■ Abetaliproteinaemia 
Mitoschondrial diseases (Kearns 
Sayre, CPEO) 

■ Friedreich's and other cerbellar 
ataxias 

Law rence-Moon Syndrome (cognitive 
impairment and obesity) 

Bardet-Biedl Syndrome (syndactyly 
and high BMI) 


Dr. Khalid Yusuf El-Zohry - Sohag Teaching Hospital (01118391123) 

Ref MRCPass OE OE 2012 PasTest 2009 PassMedicine 2009 PasTest Exam ReviseMRCP 





























El-zohry MRCP Questions Bank (Part 1) - 2013 


(For my personal use) 


Refsum Disease 


^ [Q: 1623] MRCPass- 

0 Ophthalmology 

A 60 year old patient presents with a right 
sided quadrantic hemianopia. 

Which of the following conditions is likely to 
couse this presentation? 

1- A lesion of optic chiasm 

2- Chloroquine poisoning 

3- A lesion of occipital cortex 

4- Alcohol 

5- Diabetic retinopathy 


Answer & Comments 

Answer: 3- A lesion of occipital cortex 

A partial occipital cortex insult (e.g. infarct) 
may cause this. Lesions of the temporal and 
parietal parts of the optic radiation can also 
cause a quadrantic hemianopia. A lesion of 
optic chiasm would cause a bitemporal 
hemianopia. Chloroquine poisoning causes 
symmetric bilateral scotomas. 



[Q: 1624] MRCPass - 
Ophthalmology 

A 42-year-old man with a history of end-stage 
renal disease, hypertension and hepatitis C 
infection, was hospitalized with shortness of 
breath. Chest XR showed left-sided pleural 
effusion, and aspiration revealed an exudative 
process. Pleural biopsy specimens showed 
necrotic tissue with focal fibrosis, 
noncaseating granulomas, and acid-fast bacilli. 
Culture was positive for Mycobacterium 
tuberculosis. Isoniazid (300 mg) orally and 
pyrazinamide (2 g) after hemodialysis 
sessions; vitamin B6 (50 mg) and Ethambutol 
(15 mg/kg per day) were commenced. 

Visual acuity testing before initiation of 
treatment showed 20/20 vision in both eyes 
and normal color vision. Three months later, 


repeat testing showed revealed a best 
corrected visual acuity of 20/200 with the 
right eye and counting fingers at 4 feet with 
the left eye. Automated visual field testing 
results revealed a central scotoma with 
inferior temporal quadrant defects in both 
eyes. 

What is the cause of visual deterioration? 

1- Vitamin C deficiency 

2- Hypertension 

3- Ethambutol toxicity 

4- Middle cerebral artery stroke 

5- Hepatitis C 


Answer & Comments 

Answer: 3- Ethambutol toxicity 

The diagnosis is toxic optic neuropathy. The 
anterior visual pathway is susceptible to 
damage from toxins or nutritional deficiency. 
Ethambutol is one drug that commonly is 
associated with toxic optic neuropathy. The 
optic 

neuropathy that occurs is dose dependent and 
duration related. There is a reported incidence 
of 1% - 5% of optic neuropathy with 
ethambutol. Toxicity generally does not 
develop until after treatment for at least 1.5 
months. 

Isoniazid, ethylene glycol and amiodarone are 
other associated drugs. Causes of nutritional 
optic neuropathy include tobacco, ethanol, 
thiamine deficiency and vitamin B-12 
deficiency. 


A 25 year old man presents with blurring of 
vision in his right eye. Examination reveals 
visual acuity in the right eye of 6/18 and in the 
left eye 6/6. Visual fields confrontation reveal 
a right temporal visual field defect and partial 


[Q: 1625] MRCPass - 
Ophthalmology 



Dr. Khalid Yusuf El-Zohry - Sohag Teaching Hospital (01118391123) 

Ref MRCPass OE OE 2012 PasTest 2009 PassMedicine 2009 PasTest Exam ReviseMRCP 



649 




























El-zohry MRCP Questions Bank (Part 1) - 2013 


(For my personal use) 


loss of the superior part of temporal field of 
the left eye. 

Where is the likely position of lesion 
responsible this defect? 

1- Retina 

2- Optic nerve 

3- Optic chiasm 

4- Temporal lobe 

5- Frontal lobe 

Answer & Comments 

Answer: 3- Optic chiasm 

The likely localization of the lesion is around 
the optic chiasm, spreading up the right optic 
nerve. The signs indicate a bitemporal visual 
field defect with involvement of the right optic 
nerve (decreased visual acuity). An occipital 
lobe lesion causes a congruous homonymous 
hemianopia. A temporal lobe lesion causes an 
upper homonymous quantranopia. 


The diagnosis is sicca syndrome, which can be 
caused by drugs tricyclic antidepressants, high 
dose diuretics and (B-blockers. Sjorgens (anti 
Ro Ab) and sarcoidosis (serum ACE) are 
systemic causes. 

^ [Q: 1627] MRCPass- 

>- 

• Ophthalmology 

^ m 

A 30 year woman presents with 2 month's 
history of episodic, brief visual loss affecting 
the right eye. Over the last one year she had 
gained a considerable amount of weight. 
Examination reveals a BMI of 33, bilateral 
optic disc sw elling, worse on the right and 
small retinal haemorrhages on the right. 

What is the likely diagnosis? 

1- Sagittal sinus thrombosis 

2- Benign intracranial hypertension 

3- Optic neuritis 

4- Graves eye disease 

5- Pituitary tumour 


7 

Q 

A 30 year old woman has been having 
increasing dryness and discomfort affecting 
her eyes. On examination, her visual acuity is 
normal in both eyes. There are early cataracts 
visible in both lenses. The eyes appear red and 
mildly inflamed bilaterally. An Schirmer's test 
confirms that tear production is diminished. 

What is the clinical diagnosis? 

1- Hypopyon 

2- Sicca syndrome 

3- Conjunctivitis 

4- Ectropion 

5- Posterior uveitis 


[Q: 1626] MRCPass - 
Ophthalmology 


Answer & Comments 


Answer: 2- Sicca syndrome 


Answer & Comments 

Answer: 2- Benign intracranial hypertension 

Benign intracranial hypertension is raised 
intracranial pressure in the absence of a mass 
lesion or of hydrocephalus. It usually occurs in 
young obese females in their third or fourth 
decade and is often idiopathic. 

The condition appears to be due to impaired 
CSF absorption from the subarachnoid space 
across the arachnoid villi into the dural 
sinuses. Drugs such as tetracyclines and oral 
contraceptive pill are associated with the 
condition. 

[Q: 1628] MRCPass - 
Ophthalmology 

A 55 year old woman complains of severe pain 
in her right eye. There is blurring of vision and 
she feels nauseated and has vomited several 



Dr. Khalid Yusuf El-Zohry - Sohag Teaching Hospital (01118391123) 

Ref MRCPass OE OE 2012 PasTest 2009 PassMedicine 2009 PasTest Exam ReviseMRCP 





























El-zohry MRCP Questions Bank (Part 1) - 2013 


(For my personal use) 


times. Earlier in the day she has undergone 
colonoscopy for evaluation of Crohn's disease. 

What is the likely cause of her painful red eye? 

1- Conjunctivitis 

2- Episcleritis 

3- Acute angle closure glaucoma 

4- Anterior uveitis 

5- Retinitis pigmentosa 


Answer & Comments 

Answer: 4- Foscarnet 

The most frequent opportunistic infection that 
involves the eye in AIDS is CMV retinitis which 
is likely in this case. This is classically described 
as a 'cheese and tomato on pizza' appearance. 
Toxoplasma tends not to show haemorrhagic 
changes. CMV retinitis can be treated with 
ganciclovir and foscarnet. 


Answer & Comments 

Answer: 3- Acute angle closure glaucoma 

Acute angle-closure glaucoma is caused by a 
rapid increase intraocular pressure. 
Anticholinergic agents are sometimes used 
during endoscopy to cause smooth muscle 
relaxation to aid examination when difficulty 
is encountered. These agents cause pupillary 
dilatation thus precipitating acute angle 
closure glaucoma. Treatment can be with 
miotic agents (pilocarpine) which contract 
ciliary muscle, tightening the trabecular 
meshw ork and allow increased outflow of the 
aqueous. Topical beta-adrenergic receptor 
antagonists (e.g. Timolol eye drops) decrease 
aqueous humor production by the ciliary 
body. 


i_ y 

A 55 year old man known to be HIV positive 
presents with loss of vision. Fundoscopy 
reveals yellow white patches with multiple 
associated haemorrhages. In view of the likely 
diagnosis, which of the following drugs would 
be appropriate? 

1- Zidovudine 

2- Indinavir 

3- Prednisolone 

4- Foscarnet 

5- Amphotericin B 


[Q: 1629] MRCPass - 
Ophthalmology 



[Q: 1630] MRCPass - 
Ophthalmology 


A 20 year old lady develops a deterioration in 
vision in her left eye over 2 days. She 
complains of discomfort in the eye and thinks 
that difficulty with perception of colour was 
the first problem that she noticed. On 
examination, visual acuity on the left is only to 
light perception. The pupil appears dilated and 
does not constrict to light, although does 
when a torch is shone in the right eye. 


What is the most likely diagnosis? 


1- Anterior iscaemic optic neuropathy 

2- Central retinal artery occlusion 


3- Optic neuritis 

4- Retinitis pigmentosa 

5- Epiphora 


Answer & Comments 

Answer: 3- Optic neuritis 

Optic neuritis is a cause of acute vision loss 
but also is notew orthy because of its high 
association with multiple sclerosis. Of patients 
with multiple sclerosis, 15-20% initially 
present with an episode of optic neuritis. The 
classic clinical triad of optic neuritis includes 
the following: 

Loss of vision, 

Eye pain, 


Dr. Khalid Yusuf El-Zohry - Sohag Teaching Hospital (01118391123) 

Ref MRCPass OE OE 2012 PasTest 2009 PassMedicine 2009 PasTest Exam 




ReviseMRCP 

651 

























El-zohry MRCP Questions Bank (Part 1) - 2013 


(For my personal use) 


Dyschromatopsia (impairment of 
accurate color vision) 



[Q: 1631] MRCPass - 
Ophthalmology 

A 60 year old woman presents with headache, 
photophobia, nausea and vomiting. She has 
blurred vision in her left eye. On examination 
on the left the eye is red, the pupil is oval and 
fixed, and the cornea appears cloudy. 

Visual acuity is poor. 

What is the diagnosis? 

1- Acute scleritis 

2- Retinal haemorrhage 

3- Keratoconjunctivitis sicca 

4- Acute primary closed angle glaucoma 

5- Primary open angle glaucoma 


Answer & Comments 

Answer: 4- Acute primary closed angle 
glaucoma 

The clinical features are characteristic of acute 
primary closed angle glaucoma. With the 
condition, eyeball feels hard on palpation. It 
usually occurs in hypermetropic people with 
small eyeballs in whom the anterior chamber 
drainage angle is narrow . When the the iris 
becomes apposed to the lens and prevents 
the efflux of aqueous from the posterior 
chamber, the drainage angle is 


Once the acute attack has been broken, the 
definitive therapy for narrow -angle glaucoma 
is surgical. A peripheral iridotomy, surgical or 
by laser therapy, is performed. 



[Q: 1632] MRCPass - 
Ophthalmology 


A 30 year old lady has a dilated right pupil that 
is poorly responsive to light in comparison to 
the left. 


Accomodation reflex is very sluggish. There is 
no evidence of ptosis and eye movements are 
normal. Further examination reveals absent 
ankle jerks. 

The cause of her dilated pupil is most likely to 
be: 


1- Multiple Sclerosis 

2- Myotonic dystrophy 

3- A right Holmes-Adie Pupil 

4- A left Argyll-Robertson pupil 

5- Horner's syndrome 


Answer & Comments 

Answer: 3- A right Holmes-Adie Pupil 

Holmes-Adie pupil is a cause of anisocoria. It 
affects young adults (2:1 females:male). The 
affected pupil is enlarged, poorly reactive to 
light and supersensitive to 0.1% pilocarpine. 
Ability to accommodate is also impaired and 
sluggish. The clue in this scenario is that it is 
associated with the loss of tendon reflexes. 


obstructed, resulting in a rapid rise in 
intraocular pressure. 

Emergency treatment is with agents to lower 
intraocular pressure. Topical beta-adrenergic 
antagonists such as timolol and betaxolol 
decrease aqueous production. The carbonic 
anhydrase inhibitor, acetazolamide, also 
decreases aqueous production and should be 
given in conjunction. 


Argyll-Robertson pupil is seen in syphilis. 
Pupils are bilaterally small and irregular. It is 
unreactive to light but reactive to 
accommodation. 

^ [Q: 1633] MRCPass- 

* Ophthalmology 

A 20 year old man with learning difficulties has 
acute blurring of vision in his right eye. 
Examination reveals ectopia lentis. 


Dr. Kholid Yusuf El-Zohry - Sohog Teaching Hospital (01118391123) 

Ref MRCPass OE OE 2012 PasTest 2009 PassMedicine 2009 PasTest Exam ReviseMRCP 



























El-zohry MRCP Questions Bank (Port 1) - 2013 


(For my personal use) 


What is the likely diagnosis? 

1- Pseudoxanthoma elasticum 


break adhesions to the lens. Examples of pupil 
dilators are cyclopentolate and atropine. 


2- Ehlers Danlos syndrome 

3- Marfan's syndrome 

4- Metachromatic leukodystrophy 

5- Homocystinuria 


Answer & Comments 

Answer: 5- Homocystinuria 

Ectopia lentis/ subluxation of lens is 
associated with Ehlers Danlos syndrome, 
Marfan's syndrome, Refsum's disease and 
homocystinuria. The presentation fits with 
homocystinuria as the other diseases are not 
associated with mental retardation. 

Homocystinuric patients typically have fair 
skin coarse hair, osteoporosis, mental 
retardation (nearly 50%), seizure disorder, 
marfanoid habitus and increased 
thromboembolic risk. 



[Q: 1634] MRCPass - 
Ophthalmology 

A 25-year-old male who is known to have 
ankylosing spondylitis presents with a painful, 
aching, photophobic red eye. Examination 
shows cells floating in the anterior chamber 
and precipitated on the back of the cornea. 

What is the best treatment option? 

1- Local steroids 

2- Local steroids and a pupil dilator 

3- Local steroids and a pupil constrictor 

4- Chloramphenicol 

5- Oral prednisolone 


Answer & Comments 

Answer: 2- Local steroids and a pupil dilator 

The patient has anterior uveitis. Treatment 
should be with local steroids and a dilator to 



[Q: 1635] MRCPass - 
Ophthalmology 


A 25 year old man has an illness which is 
associated with visual impairment. 


Which of the following may be associated with 
optic atrophy? 


1- Anti GBM antibodies 


2- Red Ragged fibres on muscle biopsy 

3- Iron deposition on liver biopsy 

4- Anti Thyroid antibodies 

5- Anti Acetylcholinesterase antibodies 


Answer & Comments 

Answer: 2- Red Ragged fibres on muscle 
biopsy 

Most mitochondrial myopathies do have 
ragged red fibers on the muscle biopsy. 
Examples are Kearns Sayre disease, MELAS, 
MERRF and Leber's optic atrophy. Kearn 
Sayre's disease and Lebers are associated with 
optic atrophy. 


A 60 year old man is referred by a GP with 
visual loss. The patient has noticed a gradual 
deterioration in his vision over the last 3 
months. Examination shows cupping of the 
optic disc and intraocular pressure of 32 
mmHg. 

What is the diagnosis? 

1- Kearns Sayre's disease 

2- Cataract 

3- Glaucoma 

4- Episcleritis 

5- Optic neuritis 



[Q: 1636] MRCPass- 
Ophthalmology 


Dr. Khalid Yusuf El-Zohry - Sohag Teaching Hospital (01118391123) 

Ref MRCPass OE OE 2012 PasTest 2009 PassMedicine 2009 PasTest Exam 




ReviseMRCP 

653 



























El-zohry MRCP Questions Bank (Port 1) - 2013 


(For my personal use) 


Answer & Comments 

Answer: 3- Glaucoma 

Chronic (open angle) glaucoma is 
characterised by a triad of: 

-Visual field loss - typically initially supranasal, 
but gradually extending 

-Pathological cupping of the optic disc 


Which of the following arteries, if diseased ' is 
most likely to be the cause of his symptoms? 

1- Anterior Cerebral 

2- Basilar 

3- Internal Carotid 

4- Middle cerebral 

5- Vertebral 


-Raised intraocular pressure (>24 mmHg) 



[Q: 1637] MRCPass - 
Ophthalmology 


A 60 year old man is referred by a GP with 
visual loss. The patient has noticed a gradual 
deterioration in his vision over the last 3 
months. Examination shows cupping of the 
optic disc and intraocular pressure of 32 
mmHg. 


What is the diagnosis? 


1- Kearns Sayre's disease 


2- Cataract 


3- Glaucoma 

4- Episcleritis 

5- Optic neuritis 


Answer & Comments 


Answer: 3- Glaucoma 


Answer & Comments 

Answer: 3- Internal Carotid 

The clinical scenario is amaurosis fugax. 
Patients complain of a painless loss of part of 
their vision lasting for a few minutes. This is 
most commonly caused by emboli from the 
carotid artery. 


[Q: 1639] MRCPass - 
Ophthalmology 

A 50 year old man is seen annually at the 
diabetic clinic. During one appointment, he 
feels that vision has deteriorated somewhat. 

Which one of the following fundoscopic 
features, requires a referral for 

opthalmological assessment? 

1- Blot haemorrhages 

2- Dot haemorrhages 

3- New vessel formation 



Chronic (open angle) glaucoma is 
characterised by a triad of: 


4- Microaneurysms 

5- Hard exudates 


Visual field loss - typically initially 
supranasal, but gradually extending 

■ Pathological cupping of the optic disc 

■ Raised intraocular pressure (>24 
mmHg) 


Answer & Comments 

Answer: 3- New vessel formation 

The following are graded features of diabetic 
retinopathy: 


^ [Q: 1638] MRCPass- 

% Ophthalmology 

A 70 year old man gives a history of visual loss 
affecting his right eye which lasted for 
approximately 2 minutes. 


Background - Micro aneurysms, dot and blot 
haemorrhages, hard exudates Pre- 
proliferative - Cotton wool spots, venous and 
arteriolar changes in vessels, large 
haemorrhages Proliferative - New vessel 
formation Pre-proliferative retinopathy 


Dr. Khalid Yusuf El-Zohry - Sohag Teaching Hospital (01118391123) 

Ref MRCPass OE OE 2012 PasTest 2009 PassMedicine 2009 PasTest Exam ReviseMRCP 



654 































El-zohry MRCP Questions Bank (Port 1) - 2013 


(For my personal use) 


requires opthalmological assessment for early Still's disease 

signs of proliferative retinopathy. 

Reiters'syndrome 


[Q: 1640] MRCPass - 
Ophthalmology 

A tall 30 year old man has upward dislocation 
of lens on examination of his left eye. 

Which disease is he most likely to have? 

1- Homocystinuria 

2- Marfan's syndrome 

3- Myotonic dystrophy 

4- Ehlers-Danlos syndrome 

5- Pseudoxanthoma elasticum 



Answer & Comments 


Answer: 2- Marfan's syndrome 


Ankylosing spondylitis 

Behcet's disease 

Sarcoidosis 

Tuberculosis 

Leprosy 

Syphilis 



[Q: 1642] MRCPass - 
Ophthalmology 

A 30 year old woman is unable to see the 
peripheral part of the vision. 


Which one of the following causes peripheral 
visual loss? 


Upward lens dislocation occurs in Marfan's 
syndrome. downward dislocation is 
commoner in homocystinuria. 


/■—1 

3J 

A 50 year old woman presents with a 
unilateral painful red eye associated with 
blurred vision, photophobia and watering. On 
examination there are keratitic precipitates 
and pupillary irregularity. 

What is this clinical picture consistent with? 


[ Q: 1641 ] MRCPass - 
Ophthalmology 


1- Papilloedema 

2- Hereditary familial optic atrophy 

3- Syphillitic optic atrophy 

4- Retinitis pigmentosa 

5- Retrobulbar neuritis 


Answer & Comments 

Answer: 4- Retinitis pigmentosa 

Retinitis pigmentosa causes peripheral visual 
loss (tunnel vision). 


1- Ectropion 


Central Scotoma causes are: 


2- Acute closed angle glaucoma 

3- Anterior uveitis 

4- Posterior uveitis 

5- Retinitis pigmentosa 


Hereditary familial optic atrophy 
Syphillitic optic atrophy 
Papilloedema 
Retrobulbar neuritis 


Answer & Comments 

Answer: 3- Anterior uveitis 

The clinical features suggest anterior uveitis. 
Causes are : 



[ Q: 1643 ] MRCPass - 
Ophthalmology 


A 30 year old diabetic patient has intermittent 
blurring of vision. 


Dr. Khalid Yusuf El-Zohry - Sohag Teaching Hospital (01118391123) 

Ref MRCPass OE OE 2012 PasTest 2009 PassMedicine 2009 PasTest Exam 




ReviseMRCP 

655 
































El-zohry MRCP Questions Bank (Port 1) - 2013 


(For my personal use) 


What is the likely couse? 

1- Maculopathy 

2- Cataract 

3- Neovascularisation 

4- Refractory changes in the lens 

5- Vitreous haemorrhage 




7\ 

[ Q: 1645 ] MRCPass - 

Answer & Comments 

\n\ 

Ophthalmology 


Answer: 4- Refractory changes in the lens 

Hyperglycaemia can cause refractory changes 
in the lens, which is most likely to cause 
blurred vision which is intermittent. 


A 65 year old man suddenly loses all vision in 
one eye. It begins to improve 15 minutes later 
and is normal by 3 hours. 

The most likely diagnosis is: 



[Q: 1644] MRCPass - 
Ophthalmology 


A 66 year old man has a dilated left pupil, left 
sided ptosis and limited movement consistent 
with a third nerve palsy. 


1- Retinitis pigmentosa 

2- Vitreous haemorrhage 

3- Retinal arterial occlusion 

4- Retinal detachment 

5- Retinal vein occlusion 


Which one of the following is the likely couse? 

1- Motor neuron disease 

2- Crohn's disease 

3- Wilson's disease 

4- Aneurysm of the posterior communicating 
artery 

5- Cluster headache 


Answer & Comments 

Answer: 4- Aneurysm of the posterior 
communicating artery 


lllrd nerve palsy is caused by aneurysm of the 
posterior communicating artery. Opthalmic 
migraine is a rare cause of lllrd nerve palsy. 



Answer & Comments 

Answer: 3- Retinal arterial occlusion 

Complete unilateral loss of vision, even if only 
in part of the field of vision, is likely to be due 
to an arterial event. This can be in the retina, 
either amaurosis fugax or a with a CVA. 

Retinal transient ischaemic attacks are usually 
brief (<30 mins) and almost alw ays due to an 
embolic event, most typically from the 
ipsilateral carotid bifurcation. Occlusion of the 
retinal vein can present acutely, although not 
as abruptly as an arterial event, and the loss of 
vision is not transient. 

[Q: 1646] MRCPass - 
Ophthalmology 

A 35 year old woman with blurred vision and 
pain in her right eye. Examination of the 
cornea with fluorescein reveals a branching 
ulcer. 

Whot is the most likely diagnosis? 

1- Herpes zoster ophthalmicus 




Dr. Kholid Yusuf El-Zohry - Sohog Teaching Hospital (01118391123) 

Ref MRCPass OE OE 2012 PasTest 2009 PassMedicine 2009 PasTest Exam ReviseMRCP 



























El-zohry MRCP Questions Bank (Port 1) - 2013 


(For my personal use) 


2- Herpes simplex keratitis 

3- Conjunctivitis 

4- Anterior uveitis 

5- Scleritis 


Answer & Comments 


Answer: 2- Herpes simplex keratitis 


The signs are consistent a partial 3rd nerve 
palsy associated with periorbital pain. In a 
young person a posterior communicating 
artery aneurysm is a likely cause. 

It is important to realise that frequently a III 
nerve palsy is incomplete so the extent that 
individual clinical features is seen may vary 
with the following features: 


Keratitis (corneal inflammation) can result 
from a wide variety of infections. 

Inflammatory cell infiltration and oedema 
results in photophobia, pain and impaired 
visual acuity with localised 

corneal opacity. Permanent blindness can 
occur if treatment is delayed. Herpes simplex 
infection causes a characteristic branching 
(dendritic) ulcer that may be seen with 
fluorescein or rose bengal. Treatment is with 
topical acyclovir. 


[Q: 1647] MRCPass - 
Ophthalmology 

A 40 year old woman is referred for 
assessment of a painful right eye which has 
been progressive over a week. 

On examination she has a mild ptosis of the 
right eye. She was aw are of diplopia and had 
vertical image separation on looking upwards. 
She also had weakness of elevation of right 
eye. 

Which of the following is the likely diagnosis? 

1- Neurosyphillis 

2- Myasthenia Gravis 

3- Cavernous sinus thrombosis 



Ptosis 

Loss of upward, downward and medial 
movement of the affected eye 

Lifting the eyelid reveals a divergent 
strabismus and a dilated non-reactive pupil 

The eye is in a 'down and out' position 

Painful oculomotor palsy (pupil generally 
involved) can be caused by: 

1) Compression 

Intracranial aneurysm, Uncal herniation, 
Tumours (e.g. carcinomatous lesions of the 
skull base, parasellar neoplasms), Epidermoid 
cyst, Granuloma (Tolosa-Hunt, Sarcoid) 

2) Infection 

Meningitis, Encephalitis, Herpes zoster 

3) Vascular disease 

SLE, Temporal arteritis, Opthalmoplegic 
Migraine, Dural cavernous sinus fistula 

4) Infiltration 

Leptomeningeal carcinoma, Neurofibroma 

5) Demyelination 

6) Trauma 


4- Sphenoid sinusitis 

5- Posterior communicating artery aneurysm 


Answer & Comments 

Answer: 5- Posterior communicating artery 
aneurysm 


[Q: 1648] MRCPass - 
Ophthalmology 

In diabetic involvement of the eye, which one 
of the following can result in visual 
impairment? 

1- Lipaemia retinalis 



Dr. Khalid Yusuf El-Zohry - Sohag Teaching Hospital (01118391123) 

Ref MRCPass OE OE 2012 PasTest 2009 PassMedicine 2009 PasTest Exam ReviseMRCP 


























El-zohry MRCP Questions Bank (Port 1) - 2013 


(For my personal use) 


2- Rubeosis iridis 

3- Glaucoma 

4- Retinitis Pigmentosa 

5- Drusen 


Answer & Comments 

Answer: 2- Rubeosis iridis 

Retinal detachment, cataract, rubeosis iridis 
and retinal vein occlusion are diabetic eye 
involvement manifestations. Lipaemia retinalis 
is the milky appearance of retinal vessels in 
patients with hypertriglyceridaemia. It does 
not cause impaired vision. 


A 45 year old woman presents with a 24 to 48- 
hour history of a red and aching eye with 
photophobia. Vision in that eye is blurred, but 
acuity is not significantly affected. On 
examination there is sediment at the bottom 
of the anterior chamber (hypopyon). 

What is the diagnosis? 

1- Scleritis 

2- Keratitis 

3- Iritis 

4- Conjunctivitis 

5- Episcleritis 



[Q: 1649] MRCPass - 
Ophthalmology 


Answer & Comments 

Answer: 3- Iritis 

Iritis typically presents with symptoms of a 
red, aching eye with photophobia, which 
tends to worsen over hours to a few days. 
Vision may be blurred, but acuity is not 
severely affected. 

The pupil tends to be small and may be 
irregular. Hypopyon is an accumulation of 
inflammatory cells in the anterior chamber 


that produces a layered meniscus in the 
inferior anterior chamber. It can accompany 
severe iritis. 

Iritis is associated with ankylosing spondylitis, 
Reiter syndrome, sarcoidosis, inflammatory 
bow el disease, and psoriasis. 


^ [Q: 1650] MRCPass- 

* Ophthalmology 

A 35 year old Afro-Caribbean woman presents 
with a history of fatigue, widespread joint 
pains and shortness of breath with a dry 
cough. Her ears and sinuses have been 
normal, but she mentions that she has 
recently attended the local eye casualty 
department with a painful, photophobic red 
eye. This was successfully treated with 
Maxidex drops hourly during the day and 
night. 

The most likely diagnosis is: 

1- Sarcoidosis 

2- Ankylosing spondylitis 

3- Wegener's granulomatosis 

4- Ulcerative colitis 

5- Rheumatoid arthritis 


Answer & Comments 

Answer: 1- Sarcoidosis 

The presentation of a painful red eye, 
especially with photophobia, and treatment 
with frequent and potent topical 
corticosteroid (Maxidex is dexamethasone 
0.1%) together with pupillary dilatation, 
strongly suggests a slit-lamp diagnosis of acute 
iritis (anterior uveitis). 

Common causes of iritis are: 

■ Ankylosing spondylitis 

■ Reiter's syndrome 

■ Inflammatory bow el disease 
Sarcoidosis 


Dr. Khalid Yusuf El-Zohry - Sohag Teaching Hospital (01118391123) 

Ref MRCPass OE OE 2012 PasTest 2009 PassMedicine 2009 PasTest Exam ReviseMRCP 



658 


























El-zohry MRCP Questions Bank (Port 1) - 2013 


(For my personal use) 


The history and background is suggestive of 
sarcoidosis. 


2- Klinefelter's syndrome 

3- Ehler Danlos 


Wegener's granulomatosis may account for 
the non-specific systemic features, but uveitis 
is a much less common association than acute 
scleritis. 


^ [Q: 1651] MRCPass - 

( Ophthalmology 

A 65 year old diabetic woman complains that 
her reading vision has become distorted in 
one eye and the image appears smaller than 
with the other eye. She has been diabetic for 
20 years. Her glycaemic control has been 
moderate for a long time. She does not 
smoke. 

Whot is the diagnosis? 

1- Proliferative diabetic retinopathy 

2- Complication of oral hypoglycaemics 

3- Cataract 

4- Diabetic maculopathy 

5- Retinal artery occlusion 


Answer & Comments 

Answer: 4- Diabetic maculopathy 

Symptoms visual distortion and small images 
(micropsia) are typical of diabetic 
maculopathy, but not typical of proliferative 
retinopathy. The underlying pathology is due 
to photoreceptors within the deeper layers of 
the retina become irregularly spaced. 


[Q: 1652] MRCPass - 
Ophthalmology 


A 20 year old man with learning difficulties is 
brought for review by his worried parents 
after he complained of visual blurring. 
Examination with a slit lamp reveals ectopia 
lentis. 


Whot is the most likely diagnosis? 

1- Marfan's syndrome 



4- Homocystinuria 

5- Fragile X syndrome 


Answer & Comments 

Answer: 4- Homocystinuria 

Ectopia lentis/ subluxation of the lens is 
associated with Ehlers Danlos syndrome, 
Marfan's syndrome and homocystinuria. 
There is downw ards lens dislocation in 
homocystinuria. It is also more likely to be 
associated with learning difficulties than 
Marfan's syndrome. 



Lens Dislocation 


[Q: 1653] MRCPass- 
Ophthalmology 

A 60 year man with a history of Diabetes 
Mellitus and hypertension attends an eye 
clinic. On fundoscopy he is diagnosed to have 
preproliferative diabetic retinopathy. 

Which of the following is chorocteristic of 
preproliferative diabetic retinopathy? 

1- Microaneurysms 

2- Hard Exudates 

3- New vessels formation 

4- Macular Odema 

5- Venous Beading 



Answer & Comments 


Answer: 5- Venous Beading 


Dr. Khalid Yusuf El-Zohry - Sohag Teaching Hospital (01118391123) 

Ref MRCPass OE OE 2012 PasTest 2009 PassMedicine 2009 PasTest Exam ReviseMRCP 





























El-zohry MRCP Questions Bank (Part 1) - 2013 


(For my personal use) 


The stages of diabetic retinopathy are 
background, preproliferative, proliferative and 
end-stage. Microaneurysms, hard exudates 
and macular oedema suggest background 
diabetic retinopathy. Venous beading, soft 
exudates and cotton wool spots are 
associated with preproliferative diabetic 
retinopathy. 



[Q: 1654] MRCPass - 
Ophthalmology 

A 45 year old man presents to the 
opthalmologist with an acute history of pain 
and blurring in the right eye. 

Examination reveals a visual acuity of 6/36 in 
right eye and 6/6 in left eye. There is a central 
scotoma in the right eye, and a swollen optic 
disc on the right. 

What is the diagnosis? 

1- Pituitary tumour 

2- Cavernous sinus thrombosis 

3- Optic neuritis 

4- Retinal haemorrhage 

5- Retinal vein occlusion 


Answer & Comments 

Answer: 3- Optic neuritis 

The acute presentation of central scotoma, 
reduced visual acuity, a swollen optic disc is 
likely to be due to optic neuritis. This is a 
common visual presentation of a patient with 
multiple sclerosis. 



[Q: 1655] MRCPass - 
Ophthalmology 


A 40 year old woman with type 1 diabetes 
mellitus presents for assessment. 


Which one of the following features on 
fundoscopy requires an urgent referral to an 
ophthalmologist? 


1- Pigmentation 


2- Hard exudates in macular region 

3- Soft exudates 

4- Asteroid bodies 

5- Microaneurysms 


Answer & Comments 

Answer: 2- Hard exudates in macular region 

Urgent referral to an ophthalmologist (seen 
within one week) is required if there is 
proliferative retinopathy or if there evidence 
of clinically significant macular oedema (hard 
exudates at fovea). Microaneurysms signify 
background diabetic retinopathy and soft 
exudates signify preproliferative retinopathy. 


[Q: 1656] MRCPass- 
Ophthalmology 

A 50 year old man has a painful right eye. 

Which ONE of following diagnoses is 
associated acute iritis? 

1- Colorectal Cancer 

2- Pseudoxanthoma elasticum 

3- Psoriatic arthropathy 

4- Keratoconus 

5- Refsum's disease 



Answer & Comments 

Answer: 3- Psoriatic arthropathy 

Iritis is associated with conditions such as 
Reiter's syndrome, Behcet's disease. Psoriatic 
arthropathy (about 20%) and inflammatory 
bow el disease. 


[Q: 1657] MRCPass- 
Ophthalmology 

A 55 year old man with diabetes finds that the 
vision in one eye is blurred when he reads, but 
not at other times. 

The most likely diagnosis is: 



Dr. Khalid Yusuf El-Zohry - Sohag Teaching Hospital (01118391123) 

Ref MRCPass OE OE 2012 PasTest 2009 PassMedicine 2009 PasTest Exam ReviseMRCP 



660 



































El-zohry MRCP Questions Bank (Port 1) - 2013 


(For my personal use) 


1- Retinal haemorrhage 

2- Glaucoma 

3- Macular oedema 

4- Cataract 

5- Optic neuritis 


Answer & Comments 


Answer: 3- Macular oedema 


Answer & Comments 

Answer: 4- Benign intracranial hypertension 

The history of a lady with high BMI and 
papilloedema is consistent with benign 
intracranial hypertension. Vision may be 
affected with enlargement of blind spot and 
there are visual obscuration with movements 
which provoke a rise in intracranial pressure 
(eg. bending). 


The macula is the area of retina surrounding 
fovea. It is responsible for most vision. 

Visual impairment more marked for reading 
than distance is very suggestive of macular 
disease, and the likely cause of symptoms in 
this case is diabetic maculopathy, when the 
central fovea becomes affected by retinal 
oedema or frank hard exudate. Cystic macular 
oedema (CME) occurs commonly after eye 
surgery and is also called Irvine-glass 
syndrome. 



[ Q: 1659 ] MRCPass - Ophthalmology 


A 45 year old patient has small unreactive 
pupils. 

Which one of the following causes small 
pupils? 

1- 3rd nerve palsy 

2- Syphilis 

3- Retrobulbar neuritis 



[ Q: 1658 ] MRCPass - Ophthalmology 


A 20 year old girl presents with a three week 
history of headache and horizontal diplopia on 
far right lateral gaze. 


On two separate occasions she noted dimmed 
vision whilst bending forw ards. Over the last 
year she had gained 12 kilograms in weight. 
On examination, her weight was 100 kg and 
height 160 cm. Neurological examination 
reveals bilateral papilloedema and a partial 
right sixth cranial nerve palsy. 


What is the likely diagnosis? 

1- Multiple sclerosis 


2- Craniopharyngioma 


3- Graves eye disease 


4- Benign intracranial hypertension 

5- Sagittal vein thrombosis 


4- Adie syndrome 

5- Young age 


Answer & Comments 

Answer: 2- Syphilis 

Syphilis causes Argyll Robertson pupil - small 
and iregular pupils which do not react to light 
because they are already small, [mnemonic: 
ARP: Accomodation Reflex Present, PRA: 
Pupillary Reflex Absent] 

Dilated pupils occur in lllrd nerve palsy and 
Holmes Adie pupil. Holmes Adie is also called a 
myotonic pupil due to the slow reaction and is 
associated with diminished tendon reflexes. 



[ Q: 1660 ] MRCPass - Ophthalmology 


A 55 year old woman presents with a 5 hour 
history of severe pain over her left eye, 
associated with loss of vision. Examination 


Dr. Khalid Yusuf El-Zohry - Sohag Teaching Hospital (01118391123) 

Ref MRCPass OE OE 2012 PasTest 2009 PassMedicine 2009 PasTest Exam ReviseMRCP 
































El-zohry MRCP Questions Bank (Port 1) - 2013 


(For my personal use) 


reveals an oval, partly dilated unreactive pupil 
with hyperaemia of the ciliary vessels. 

Which of the following is the likely diagnosis? 

1- Pterygium 

2- Scleritis 

3- Acute (closed angle) glaucoma 

4- Grave's opthalmopathy 

5- Conjunctivitis 


Answer & Comments 

Answer: 3- Acute (closed angle) glaucoma 

Acute glaucoma is a medical emergency. It is 
usually unilateral but the other eye is often 
affected within a few weeks of initial 
presentation. The condition progresses over a 
few hours and if treatment is delayed, 
permanent blindness may result. Initial 
management is with intravenous 
acetazolamide (to reduce aqueous 
production) and pilocarpine drops (to 
constrict the pupil and so open the angle of 
the anterior chamber). Definitive treatment is 
with surgical iridectomy or laser iridotomy. 



[ Q: 1661 ] MRCPass - Ophthalmology 


A 70 year lady who a history of anxiety 
presents with a 2 day history of severe left 
temporal headache radiating from eye scalp. 
She had also experienced discomfort during 
eating. 


Which one of following drugs should be given 
this patient while awaiting diagnostic tests? 


1- Prednisolone 


Answer & Comments 

Answer: 1- Prednisolone 

The history suggests temporal arteritis (TA). 
Vision loss is an important finding. About one 
fifth of patients with TA and vision loss have 
no systemic symptoms of TA. Treatment of 
patients with TA is critical to avoid vision loss, 
and therapy should be initiated based on 
clinical suspicion, not biopsy results. The initial 
prednisolone dosage should be between 60 to 
100 mg per day. Usually, steroid therapy can 
be discontinued within one year, although 
some patients need prednisone therapy for 
years. 

An American College of Rheumatology study 
determined that highly sensitive parameters 
for diagnosis of TA are: 

age more than 50 years 

an ESR of more than 50 mm per hour 

an abnormal temporal artery biopsy 



[ Q: 1662 ] MRCPass - Ophthalmology 


A 60 year old man presents with a six week 
history of blurring of vision. His investigations 
show a fasting plasma glucose of 15. 

What is the likely cause of his blurred vision? 

1- Osmotic changes in the lens 

2- Maculopathy 

3- Retinal aretery thrombosis 

4- Cataract 

5- Proliferative diabetic retinopathy 


2- Diazepam 

3- Sumatriptan 

4- Ibuprofen 

5- Carbamazepine 


Answer & Comments 

Answer: 1- Osmotic changes in the lens 

The patient is a new ly diagnosed diabetic and 
hence proliferative retinopathy is unlikely. 
Retinal artery thrombosis would cause very 
sudden onset symptoms. With this history, 


Dr. Khalid Yusuf El-Zohry - Sohag Teaching Hospital (01118391123) 

Ref MRCPass OE OE 2012 PasTest 2009 PassMedicine 2009 PasTest Exam ReviseMRCP 






























El-zohry MRCP Questions Bank (Port 1) - 2013 


(For my personal use) 


osmotic changes due to hyperglycaemia is 
most likely to cause the visual symptoms. 



[ Q: 1663 ] MRCPass - Ophthalmology 


A35 year old woman develops an acutely 
painful left red eye which woke her from 
sleep. She is not on any medications. On 
examination, her left eye appears red and is 
watering profusely. Visual acuity is reduced to 
6/18 in the left eye, but is normal in the right. 
The pupil is contracted on the left side with a 
reduced light reflex. The lens appears cloudy 
and the iris appears hyperaemic. 


What is the most likely cause of her red eye? 

1- Maculopathy 


2- Conjunctivitis 

3- Acute iridocyclitis 

4- Acute glaucoma 

5- Corneal ulceration 


Answer & Comments 

Answer: 3- Acute iridocyclitis 

Iridocyclitis, also known as anterior uveitis, is a 
condition in which the uvea of the eye is 
inflamed. Diagnosis of iridocyclitis is suggested 
by the acute painful red eye, abnormal iris, 
contracted pupil and reduced light reflex. It 
can be effectively treated with tropane 
alkaloids or steroids. 



[ Q: 1664 ] MRCPass - Ophthalmology 


A 50 year old man was diagnosed with 
hypertension 10 years ago. He has symptoms 
of some blurred vision since 1 week ago and is 
assessed at the eye clinic. 

Which of the following fundoscopic features 
suggest a diagnosis of grade IV hypertensive 
retinopathy> 


1- New vessel formation 


2- Swollen discs 

3- Arterio-venous nipping 

4- Haemorrhages 

5- Cotton Wool spots 


Answer & Comments 

Answer: 2- Swollen discs 

Grading of Hypertensive Retinopathy can be 
done as follows: 

I - Arteriolar Attenuation 

II - AV nipping 

III - Cotton wool spots and haemorrhages 

IV - Disc Sw elling (papilloedema) 



[ Q: 1665 ] MRCPass - Ophthalmology 


A 35 year old woman reports that vision in her 
right eye has become blurred over a few days. 
Her right eye also feels painful. Examination 
reveals relative afferent papillary defect. 


The most likely diagnosis is: 


1- Ischaemic optic neuritis 

2- Retinal detachment 


3- Retinal haemorrhage 

4- Foveal oedema 


5- Demyelinating optic neuritis 


Answer & Comments 

Answer: 5- Demyelinating optic neuritis 

Painful symptoms would fit with optic neuritis. 
Reduced visual acuity (to variable degree), 
relative afferent papillary defect and 
fundoscopy may reveal a swollen optic nerve 
disc. 


Dr. Khalid Yusuf El-Zohry - Sohag Teaching Hospital (01118391123) 

Ref MRCPass OE OE 2012 PasTest 2009 PassMedicine 2009 PasTest Exam ReviseMRCP 
































El-zohry MRCP Questions Bank (Port 1) - 2013 


(For my personal use) 



[ Q: 1666 ] MRCPass - Ophthalmology 


A 46 year old man with no previous visual 
problems presents with an acute, severely 
painful red eye. 

What is the likely couse? 

1- Optic neuritis 

2- Optic atrophy 

3- Retinal detachment 

4- Vitreous haemorrhage 

5- Angle closure glaucoma 


Answer & Comments 

Answer: 5- Angle closure glaucoma 

Acute closed-angle glaucoma is an ocular 
emergency that requires immediate diagnosis 
and treatment to prevent permanent visual 
impairment and to relieve pain. There is 
sudden onset of blurred vision, eye pain and 
redness. There is elevated intraocular 
pressure of 40-80 mm Hg (normal <25 mmHg). 

Topical beta-adrenergic antagonists such as 
timolol and betaxolol decrease aqueous 
production. The carbonic anhydrase inhibitor, 
acetazolamide, also decreases aqueous 
production. A drop of pilocarpine 2-4% every 
15 minutes for the first 1-2 hours helps to 
facilitate aqueous outflow . 



Fundoscopy in glaucoma showing increased 

cup to disc ratio 



[ Q: 1667 ] MRCPass - Ophthalmology 


A 65 year old miopic woman presents with 
severe pain and reduced vision in her left eye. 
She also reports seeing 'haloes' around most 
objects. On examination, the eye is inflamed, 
the pupil is dilated and acuity is markedly 
reduced. 

Which of the following is the most appropriate 
test? 

1- MRI brain 


2- CT orbits 

3- Intraocular pressure measurement 

4- Lumbar puncture and CSF pressure 

5- Schirmer's test 


Answer & Comments 

Answer: 3- Intraocular pressure measurement 

The diagnosis is primary angle closure 
glaucoma (acute). 

The condition affects females more than 
males and long-sighted patients are more 
affected. There may be abrupt increases in 
intraocular pressure, with watering of the eye 
and loss of vision. Symptoms of eye pain, 
nausea and abdominal pain, reduced visual 
acuity, red eye and cloudy cornea can occur. 
The pupil may be oval, fixed and dilated. 

Treatment is with medical therapy 
(acetazolamide, pilocarpine and betablockers) 
followed by iridectomy. 



[ Q: 1668 ] MRCPass - Ophthalmology 


A 45 year old man reports that his vision has 
gradually become blurred, especially in the 
right eye. He is finding it difficult to read, even 
in bright light, and has great difficulty when 
driving at night because of glare from 
oncoming headlights. He had a renal 


Dr. Khalid Yusuf El-Zohry - Sohag Teaching Hospital (01118391123) 

Ref MRCPass OE OE 2012 PasTest 2009 PassMedicine 2009 PasTest Exam ReviseMRCP 



664 































El-zohry MRCP Questions Bank (Port 1) - 2013 


(For my personal use) 


transplant 10 years ago and has been on low 
dose maintenance immunosuppression and 
antihypertensives. His mother also had renal 
failure and glaucoma. 

Whot is the diagnosis? 

1- Cataract 

2- Glaucoma 

3- Scleritis 

4- Retinitis pigmentosa 

5- Visual impairment related to renal failure 

Answer & Comments 

Answer: 1- Cataract 

Cataract is relatively common in renal 
transplant patients, as risk factors include 
renal failure and long-term systemic 
corticosteroid medication. The symptoms are 
when vision becomes worse in bright light 
when the pupil constricts, confining the light 
path to the central part of the lens where it is 
thickest. This patient may have steroid 
induced cataract. 

Retinitis pigmentosa (RP) can occur in some 
types of renal failure, especially Alport's 
syndrome, associated with deafness and a 
positive family history. The main symptom in 
RP is poor night vision. Glare is less typical and 
symptoms are progressive from a relatively 
early age. Glaucoma causes gradual loss of 
visual field and is rarely symptomatic in 
middle age. 


that her reflexes are all diminished. Her eye 
movements are normal and there is no ptosis. 

Whot is the most likely couse of the pupillary 
abnormality? 

1- Right third cranial nerve palsy caused by 
posterior communicating artery aneurysm 

2- Argyll-Robertson pupil on the left 

3- Physiological state 

4- Left sided Horner's syndrome 

5- Holmes-Adie pupil on the right 

Answer & Comments 

Answer: 5- Holmes-Adie pupil on the right 

There is very slow reaction and incomplete 
reaction to light and sluggish accommodation. 
Once the pupil has constricted it remains small 
for an abnormally long time (tonic pupil). The 
presence of diminished reflexes make this 
diagnosis the most likely. Headache is not a 
common part of the syndrome, but could be 
due to problems with vision. 

Argyll-Robertson pupils are small, fail to react 
to light, constrict with near vision 
(accommodation), and are usually bilateral. 
The Argyll Robertson pupil has become a rare 
diagnostic sign of neurosyphilis. 

There are no additional features to suggest a 
third cranial nerve palsy, and absence of 
ptosis makes Horner's syndrome unlikely. 



[ Q: 1669 ] MRCPass - Ophthalmology 


A 45 year old woman visits her GP with a 12 
month history of headaches. These are 
intermittent and occur when she has 
problems with her vision. The GP has noticed 
that her right pupil is larger than the left. The 
right eye has very slow reaction to light and 
did not completely constrict. The rest of the 
neurological examination is normal except 


Dr. Khalid Yusuf El-Zohry - Sohag Teaching Hospital (01118391123) 

Ref MRCPass OE OE 2012 PasTest 2009 PassMedicine 2009 PasTest Exam 




ReviseMRCP 

665 






















El-zohry MRCP Questions Bank (Port 1) - 2013 


(For my personal use) 


[ Q: 1670 ] MRCPass - Psychiatry 

A 35 year old patient has been taking 
drugs for a psychiatric disorder. She is brought 
in by a flatmate, having had an episode where 
her eyes rolled back in the head and the 
tongue protruded involuntarily. 

Which might be the medication causing this? 

1- Procyclidine 

2- Olanzepine 

3- Haloperidol 

4- Diazepam 

5- Levetiracetam 



Answer & Comments 

Answer: 3- Haloperidol 

The patient has an oculogyric crisis. Oculogyric 
Crisis usually occurs as a side effect of 
neuroleptic drug treatment. It is one of the 
acute dystonic reactions. It is the most 
common of the ocular dystonic reactions 
(which include blepharospasm, periorbital 
twitches, and protracted staring episodes). 


5- Predisposing mental illness 


Answer & Comments 

Answer: 3- Replaying a tramatic scene in his 
mind 

Post traumatic stress disorder is a delayed 
and/or protracted response to an 
exceptionally stressful event. Symptoms 
include episodes of reliving the trauma, 
avoidance behaviour, persistent numbness, 
detachment from people. 



[ Q: 1672 ] MRCPass - Psychiatry 

A 30 year woman complains of chest 
pains and abdominal pains. On presentation 
to the hospital she is found to have numerous 
crusted, linear lesions on her forearms. A 
dermatology consultation was arranged, and a 
skin biopsy subsequently turned out to be 
normal. 


What is the likely diagnosis? 

1- Scleroderma 


2- Hereditary haemorrhagic telangiectasia 


Causes or triggering factors in oculogyric crisis 
include: neuroleptics, benzodiazepines, 

carbamazepine, chloroquine, cisplatin, 
influenza vaccine, levodopa, lithium, 
metoclopramide, nifedipine, tricyclics. 

^ [ Q: 1671 ] MRCPass - Psychiatry 

* A 40 year old man was working as a 
soldier in a war 10 years ago and has 
previously been tortured. He is having 
nightmares and mood swings. 

Which of the following features is most 
suggestive of post traumatic stress disorder? 

1- Onset usually about 3 months after the 
event 

2- More common in older men 

3- Replaying a tramatic scene in his mind 

4- Low incidence in Europe 


3- Schizophrenia 

4- Factitious disorder 

5- Depression 

Answer & Comments 

Answer: 4- Factitious disorder 

Dermatitis artefacta is a condition in which 
skin lesions are solely produced or inflicted by 
the patient's own actions. This usually occurs 
as a result or manifestation of a psychological 
problem. It could be a form of emotional 
release in situations of distress or part of an 
attention seeking behaviour. 

In very rare cases there may be an underlying 
attempt to secure an insurance claim. The 
rash described is consistent with dermatitis 
artifacta. The crusted lesions on forearms 


Dr. Khalid Yusuf El-Zohry - Sohag Teaching Hospital (01118391123) 

Ref MRCPass OE OE 2012 PasTest 2009 PassMedicine 2009 PasTest Exam ReviseMRCP 






























El-zohry MRCP Questions Bank (Port 1) - 2013 


(For my personal use) 


suggest artefact as this the commonest site of 
the lesions. 


^ [ Q: 1673 ] MRCPass - Psychiatry 

n - 

# A 35 year old homeless man 
presents with tonic clonic seizures to the 
hospital. He says that he is epileptic, but his 
GP surgery and hospital has no record of this. 
He also says he takes phenytoin but the 
plasma levels were unmeasurable. He was 
admitted for several days and further 
investigations of CT and EEG were normal. 

Whot is the diagnosis? 

1- Anxiety disorder 

2- Malingering 

3- Hypochondriasis 

4- Somatisation 

5- Korsakoff's psychosis 


Answer & Comments 

Answer: 2- Malingering 

There is a difference between malingering and 
Munchausen's syndrome. 

Malingering patients have a clear secondary 
gain, whilst there is no convincing secondary 
gain in Munchausen's syndrome (occasionally 
said to be due to attention seeking 
behaviour). 

^ [ Q: 1674 ] MRCPass - Psychiatry 

* A 55 year old man has suffered 
bereavement of his family member 8 months 
ago. He continues to be constantly tearful and 
depressed. 

Whot is the most appropriate management? 

1- Give tricyclic antidepressants 

2- Assess for clinical depression and suicide 
risk 

3- Assess for schizophrenia 

4- Reassure that he will get over it 


5- Sleeping tablets 


Answer & Comments 

Answer: 2- Assess for clinical depression and 
suicide risk 

Depression is relatively common among 
patients with chronic illnesses but can also 
occur following bereavement. A history to 
identify symptoms of clinical depression is 
important, as well as assessment of suicide 
risk. Antidepressants can be prescribed once 
the diagnosis is firm. 


[ Q: 1675 ] MRCPass - Psychiatry 

A 35 year old lady has been 
depressed and is brought to the hospital by a 
friend after slashing her wrists. 

Which of the following is a predictor that she 
may make a fatal suicide attempt? 

1- Depressed relative 

2- Personality disorder 

3- Female sex 

4- Younger age group 

5- A boyfriend who does not care 



Answer & Comments 

Answer: 2- Personality disorder 

The characteristics of patients who are likely 
to have repeated or successful suicide 
attempts are: male sex, age of 45 more, drugs, 
alcohol, personality disorder, living alone 
(divorced or widowed), criminal record, 
previous history of psychiatric treatment, 
lower social class and unemployment. 

[ Q: 1676 ] MRCPass - Psychiatry 

A 25 year old biology student is 
referred by her GP with possible UTI and 
confusion. However, she has no new urinary 
symptoms. Her MTS score is 10 /10 but she 
has difficulty answ ering questions directly. 



Dr. Khalid Yusuf El-Zohry - Sohag Teaching Hospital (01118391123) 

Ref MRCPass OE OE 2012 PasTest 2009 PassMedicine 2009 PasTest Exam ReviseMRCP 



668 































El-zohry MRCP Questions Bank (Port 1) - 2013 


(For my personal use) 


Her friend describes her as a very pleasant 
character who is very sociable in the pub. 
Recently, her friend said she had become 
quite disinhibited and and said she was going 
to become a millionaire as soon as she started 
work. She did However get annoyed easily 
when her friends questioned how she was 
going to achieve this. 

Whot is the diagnosis? 

1- Korsakoff's psychosis 

2- Hypomania 

3- Anxiety disorder 

4- Depressive psychosis 

5- Somatisation 


2- Carbamazepine 

3- Haloperidol 

4- Chlorpromazine 

5- Thioridazine 

Answer & Comments 

Answer: 1- Diazepam 

Oral or subcutaneous diazepam can be given 
to the patient to reduce the agitation and 
violence, as this is likely to be a form of 
delirium. It is short acting, hence a longer 
acting sedating agent such as haloperidol may 
be necessary second line. 


Answer & Comments 

Answer: 2- Hypomania 

Persistent mood elevation with occasional 
irritability is typical of hypomania. There is 
distinct period of persistently elevated, 
expansive, or irritable mood, sometimes 
lasting several days, that is clearly different 
from the usual non depressed mood. 

The clinical features of mania reflect a marked 
elevation of mood, characterized by euphoria, 
overactivity and disinhibition. Hypomania is 
the mild form of mania. Hypomania lasts a 
shorter time and is less severe, with no 
psychotic features and less disability. 
Hypomania can be distinguished from normal 
happiness by its persistence, non-reactivity 
(not provoked by good news and not affected 
by bad new s) and social disability. 


[ Q: 1677 ] MRCPass - Psychiatry 

A 75 year old man with history of 
stroke with residual right hemiparesis has 
become acutely confused. He is agitated and 
is trying to attack any nursing staff who are 
looking after him. 

What should be prescribed? 

1- Diazepam 




[ Q: 1678 ] MRCPass - Psychiatry 

A 45 year old man has taken an 
overdose of 25 diazepam tablets. He is 
assessed by the psychiatrist for risk for future 
suicide. 


Which of the following indicates a high risk? 
1- Overdose with alcohol 


2- Overdose with easily obtainable drugs 

3- Making plans before an overdose 

4- Previous overdoses 


5- Unemployed 


Answer & Comments 

Answer: 3- Making plans before an overdose 

Suicide intent is stronger in patients who plan 
for the suicide. The more specific the plan, the 
more serious the intent, for example - a will, 
or giving things away. 


[ Q: 1679 ] MRCPass - Psychiatry 

A 45 year old lady has been 
depressed for about year. She has early 
morning waking, anhedonia and difficulty 
sleeping. A diagnosis of clinical depression is 
made. 



Dr. Khalid Yusuf El-Zohry - Sohag Teaching Hospital (01118391123) 

Ref MRCPass OE OE 2012 PasTest 2009 PassMedicine 2009 PasTest Exam ReviseMRCP 



669 





























El-zohry MRCP Questions Bank (Port 1) - 2013 


(For my personal use) 


Which of the following forms of therapy is 
recommended? 

1- Psychoanalysis 

2- Family counselling 

3- Transactional analysis 

4- Cognitive behavioural therapy 

5- Dynamic counselling 


Answer & Comments 

Answer: 4- Cognitive behavioural therapy 

Cognitive behavioural therapy is likely to be 
effective in moderate clinical depression. 


[ Q: 1680 ] MRCPass - Psychiatry 

A 30 year old woman is 28 weeks 
pregnant. She is brought into hospital for 
assessment of recurrent vomiting. 

She mentions that she has eaten very little 
over several weeks. Examination reveals 
MMSE score of 23/30 and 

temporal disorientation, but registration is 
intact. Recall was very poor. 

What diagnosis is likely? 

1- Korsakoff's syndrome 

2- Parietal lobe syndrome 

3- Frontal lobe syndrome 

4- Creutzfeldt jakob disease 

5- Alzheimer's disease 



Answer & Comments 

Answer: 4- Creutzfeldt jakob disease 

The clinical context would fit for thiamine 
deficiency related to hyperemesis gravidarum, 
leading to Wenicke's Encephalopathy (WE). 
This can develop after just a few weeks of 
vomiting. 


Early intervention with thiamine replacement 
is typically all that is needed to prevent this 
complication. 


[ Q: 1681 ] MRCPass - Psychiatry 

A 70 year old woman has become 
increasingly confused over the past few 
months. She is commonly disorientated. 

Which one of the following is the most likely 
feature of Alzheimer's disease? 

1- Depression 

2- Paranoid delusion 

3- Impaired short term memory 

4- Diarrhea 

5- Hallucinations 



Answer & Comments 

Answer: 3- Impaired short term memory 

Alzheimer's disease is typified by short term 
memory loss which is progressive. 

There is also inability to formulate plans and 
aphasia. 


^ [ Q: 1682 ] MRCPass - Psychiatry 

I iS - 

* A 50 year old man is complaining of 
strange experiences over the past year. The 
patient hears his own thoughts being spoken 
aloud and, as a consequence, other people are 
able to hear his thoughts as well. 

Which of the following would suggest a 
psychotic disorder? 

1- Derealisation 

2- Hypnagogic hallucination 

3- Left right disorientation 

4- Thought broadcast 

5- Depersonalisation 


Women may present with the classic triad of 
symptoms, visual distrubances, confusion and 
muscular weakness. 


Answer & Comments 


Answer: 4- Thought broadcast 


Dr. Khalid Yusuf El-Zohry - Sohag Teaching Hospital (01118391123) 

Ref MRCPass OE OE 2012 PasTest 2009 PassMedicine 2009 PasTest Exam ReviseMRCP 

































El-zohry MRCP Questions Bank (Port 1) - 2013 


(For my personal use) 


Thought broadcast is an example of thought 
alienation. 

In thought broadcast, the patient's thoughts 
are either felt to be heard by someone else, or 
projected (in another media e.g. video or 
written). This suggests a psychotic disorder. 

Other forms of thought alienation are thought 
withdrawal and thought insertion. 
Depersonalisation (person feels unreal) and 
derealisation (environment feels unreal) are 
present not only in psychotic disorders, but 
also anxiety states and depression. 


muscle pain 

multi-joint pain without sw elling redness 
headaches of a new type 
pattern severity 
unrefreshing sleep 

post-exertional malaise lasting more than 24 
hours 

Low dose antidepressants are used in 
treatment of CFS, but suggested first line 
therapy should include cognitive 



[ Q: 1683 ] MRCPass - Psychiatry 

A 20 year year old university student 
presents persistent fatigue, myalgia. He has 
poor concentration in classes and mentions 
that he is less sociable because he tends to get 
irritable easily. All this started after a flu like 
illness 1 year ago. A diagnosis of chronic 
fatigue syndrome is made. 


What is the best first line management of this 
patient? 


1- Psychoanalysis 

2- Cognitive behavioural therapy 


3- Fluoxetine 


4- Chlorpromazine 

5- ECT 


Answer & Comments 

Answer: 2- Cognitive behavioural therapy 
The criteria for chronic fatigue syndrome are : 

1) severe chronic fatigue of six months longer 
duration 

2) Have four more of following symptoms: 
substantial impairment in short-term memory 
poor concentration 

sore throat 
tender lymph nodes 


behavioural therapy. 

[ Q: 1684 ] MRCPass - Psychiatry 

A 30 year old patient has been found 
wandering on the street. 

The police brought the patient for assessment 
for suspected delusions and paranoia. The 
patient is aggressive and wants to leave 
straightaway. 

What does the Section 5(2) of the Mental 
Health Act allow a physician to do if mental 
illness is suspected? 

1- Detain a patient for up to 24 hours from 
A&E 

2- Detain a patient for up to 48 hours from 
A&E 

3- Detain a patient for up to 72 hours from 
A&E 

4- Detain a patient for up to 48 hours if they 
are already being nursed in hospital 

5- Detain a patient for up to 72 hours if they 
are already being nursed in hospital 



Answer & Comments 

Answer: 5- Detain a patient for up to 72 hours 
if they are already being nursed in hospital 

Section 5(2) of the Mental Health Act allows a 
physician or surgeon to detain a patient for up 
to 72 hours if they are already being nursed in 


Dr. Khalid Yusuf El-Zohry - Sohag Teaching Hospital (01118391123) 

Ref MRCPass OE OE 2012 PasTest 2009 PassMedicine 2009 PasTest Exam ReviseMRCP 



671 

























El-zohry MRCP Questions Bank (Port 1) - 2013 


(For my personal use) 


hospital when they are suspected of, or have a 
worsening mental illness. 


[ Q: 1685 ] MRCPass - Psychiatry 

A 66 year old man was found with 
decreased consciousness. There were some 
chlorpromazine, diazepam and amitriptyline 
tablets in his pocket. He also was found with a 
half empty bottle of whisky. 

He had a temperature of 38.2°C, GCS was 
13/15 on arrival to A+E. Blood pressure was 
170/100 mmHg and there was marked muscle 
rigidity but normal reflexes and downgoing 
plantars. 

Whot is the diagnosis? 

1- Bipolar disorder 

2- Epilepsy 

3- Narcolepsy 

4- Catatonic schizophrenia 

5- Neuroleptic malignant syndrome 



Answer & Comments 

Answer: 5- Neuroleptic malignant syndrome 

Neuroleptic Malignant Syndrome is 
characterized by fever, muscular rigidity, 
altered mental status, decreased conscious 
level and autonomic dysfunction. 

Although potent neuroleptics (eg, haloperidol, 
fluphenazine) are more frequently associated 
with NMS, all antipsychotic agents, typical or 
atypical, may precipitate the syndrome. For 
example, these agents are prochlorperazine, 
promethazine , clozapine and risperidone . 


^ [ Q: 1686 ] MRCPass - Psychiatry 

- 

* An 17 year old student is very 

anxious about her A levels. She is feeling dizzy 
and may vomit several times when the exams 
come closer to date. 

Which of the following is the best 

manogement? 


1- Fluoxetine 

2- Amitriptyline 

3- ECT 

4- Counselling 

5- Diazepam 


Answer & Comments 

Answer: 4- Counselling 

There is little evidence of psychiatric illness, 
the patient has marked anxiety related to a 
stressful event and counselling should be a 
useful coping mechanism. 



[ Q: 1687 ] MRCPass - Psychiatry 

A 60 year old patient has been on an 
antipsychotic medication for a while. He 
develops a festinant gait and tremors in his 
hand. 


Which of the following antipsychotic 
medications might have this side effect? 

1- Risperidone 

2- Haloperidol 

3- Olanzepine 

4- Quetiapine 

5- Clozapine 


Answer & Comments 

Answer: 2- Haloperidol 

All of the above are newer generation 
antipsychotics. Haloperidol and trifluoperazine 
are examples of older generation 
antipsychotics which have parkinsonian side 
effects. 


[ Q: 1688 ] MRCPass - Psychiatry 

An elderly patient presents with 
fluctuating episodes of confusion, attention 
problems and visual hallucinations. 



Dr. Khalid Yusuf El-Zohry - Sohag Teaching Hospital (01118391123) 

Ref MRCPass OE OE 2012 PasTest 2009 PassMedicine 2009 PasTest Exam ReviseMRCP 



672 






























El-zohry MRCP Questions Bank (Port 1) - 2013 


(For my personal use) 


There is also a history of falls. A diagnosis of 
Lew y body dementia is suspected. In this 
patient which type of drug should be avoided? 

1- Beta blockers 

2- Neuroleptics 

3- SSRIs 

4- Anticonvulsants 

5- Benzodiazepines 


Answer & Comments 

Answer: 2- Neuroleptics 

Among patients with Lewy body dementia, 
classical neuroleptic drugs (eg haloperidol) 
have a high risk of causing an extrapyramidal 
syndrome and increase mortality. Most 
experts recommend atypical neuroleptics such 
as risperidone, olanzapine, or clozapine. 


[ Q: 1689 ] MRCPass - Psychiatry 

A 60 year man has drunk 5 pints of 
beer a day for 20 years. He had no other 
significant medical history and was not taking 
any regular medications. He presents with 
acute confusion and has an MTS score of 5/10 
on admission. 

Which of following suggests o diagnosis of 
Korsakoff's psychosis? 

1- Confabulation 




[ Q: 1690 ] MRCPass - Psychiatry 

A 27 year woman complained 
breathlessness, chest pains and severe 
abdominal pains. She has previously been 
abused by a family member. In past 10 years 
she had investigated for abdominal pains, 
without any diagnoses. 


What is the likely diagnosis? 


1- Depression 

2- Factitious disorder 


3- Somatisation disorder 

4- Anxiety disorder 

5- Personality disorder 


Answer & Comments 

Answer: 3- Somatisation disorder 

Four major somatoform disorders exist: 
conversion disorder (also known as hysteria), 
hypochondriasis, somatization disorder, and 
somatoform pain disorder. 

The list of symptoms includes: 

PAIN SYMPTOMS (4 or more) in the head, 
abdomen, back, joints, extremities, chest. 

GASTROINTESTINAL SYMPTOMS (2 or more, 
excluding pain) such as nausea, bloating, 
vomiting, diarrhea, intolerance of several 
foods. 


2- Auditory hallucinations 

3- Visual hallucinations 

4- Long term memory loss 

5- Seizures 

Answer & Comments 

Answer: 1- Confabulation 

Korsakoff's psychosis is typically associated 
with short term memory loss and then 
confabulation by patient when he is unable to 
accurately describe something. 


SEXUAL SYMPTOMS (at least 1, excluding pain) 
including indifference to sex, difficulties with 
erection or ejaculation, irregular menses, 
excessive menstrual bleeding. 

PSEUDONEUROLOGICAL SYMPTOMS (at least 
1) including impaired balance or coordination, 
weak muscles, trouble swallowing, loss of 
voice, retention of urine, hallucinations, 
numbness , double vision, blindness, deafness, 
seizures, amnesia or other dissociative 
symptoms. 

[ Q: 1691 ] MRCPass - Psychiatry 



Dr. Khalid Yusuf El-Zohry - Sohag Teaching Hospital (01118391123) 

Ref MRCPass OE OE 2012 PasTest 2009 PassMedicine 2009 PasTest Exam 




ReviseMRCP 

673 































El-zohry MRCP Questions Bank (Port 1) - 2013 


(For my personal use) 


A 23 year old woman has had rapid breathing 
attacks for 2 years. She has associated tingling 
in her fingers during these episodes which 
typically last for 5 minutes. She remains aw 
are of her surroundings during the episodes. 

What is the most likely diagnosis? 

1- Obsessive compulsive disorder 

2- Panic attacks 

3- Schizophrenia 

4- Dysmorphophobia 

5- Somatoform disorder 


A 55 old woman is referred with symptoms of 
lethargy. Medical investigations did not show 
any positive results. 

She has been diagnosed as having chronic 
fatigue syndrome. 

What treatment should be commenced? 

1- Long rest 

2- Graded exercise 

3- Benzodiazepines 

4- Neuroleptics 

5- Selective serotonin reuptake inhibitors 


Answer & Comments 


Answer: 2- Panic attacks 


Answer & Comments 


Answer: 2- Graded exercise 


The features of hyperventilation with 
subsequent paresthesiae in the fingers are 
typical of panic attacks. 



[ Q: 1692 ] MRCPass - Psychiatry 

A 50 year old man has been on 
lithium for bipolar disorder. His psychiatrist 
thinks that the dose of lithium may be too 
high due to a certain symptom. 


Which of the following symptoms are a 
feature of lithium toxicity? 


1- Abnormal eye movements 


2- Abdominal pains 

3- Breathlessness and ankle oedema 


4- Tremor and ataxia 

5- Hallucinations 


Answer & Comments 

Answer: 4- Tremor and ataxia 

Common symptoms of lithium toxicity (can 
occur at levels greater than 1.1 mmol/l) are 
nausea and vomiting, diarrhoea, 
disorientation, tremors and ataxia. 



[ Q: 1693 ] MRCPass - Psychiatry 


Graded exercise programmes and cognitive- 
behavioural therapy are the only therapies 
which have been shown to be beneficial in 
chronic fatigue syndrome. 

[ Q: 1694 ] MRCPass - Psychiatry 

# A 50 year old lady is pacing around 
the ward a lot and not able to sleep at night. 
Examination of the woman's mental status 
revealed labile mood and affect, loud and 
pressured speech. She did not have any 
delusional thoughts or hallucinations. 

What is the diagnosis? 

1- Schizophrenia 

2- Personality disorder 

3- Anxiety disorder 

4- Mania 

5- Clinical depression 


Answer & Comments 

Answer: 4- Mania 

Pressure of speech and flight of ideas (quick 
succession of thoughts) occurs in mania. In 
addition, speech rhyming and clanging (words 
chosen for sound and not meaning - I ate 


Dr. Khalid Yusuf El-Zohry - Sohag Teaching Hospital (01118391123) 

Ref MRCPass OE OE 2012 PasTest 2009 PassMedicine 2009 PasTest Exam ReviseMRCP 

































El-zohry MRCP Questions Bank (Port 1) - 2013 


(For my personal use) 


food, rude, stood) are also important 
suggestive features. 


[ Q: 1695 ] MRCPass - Psychiatry 

A 45 year man was arrested 
following abnormal behaviour in the streets. 
He was brought to the hospital for assessment 
because he was felt to have delusional 
behaviour, but then had a grand mal seizure 
during examination. 

Whot medication might hove led to the 
seizure? 

1- Amphetamine 

2- LSD 

3- Barbiturate 

4- Cocaine 

5- Chlorpromazine 



Answer & Comments 

Answer: 3- Barbiturate 

Barbiturate withdrawal in an habitual abuser a 
well recognised cause of fits. 


referred to the psychiatric outpatient 
department with a two month history of 
washing her hands and legs and complaining 
that ants were craw ling over her skin and 
biting her. 

The patient also described the insects as 
having wings, and craw ling in the bathroom. 
She would spend much time in the bathroom, 
at times repeatedly throwing water to wash 
them away, at other times leaving water for 
them and enjoying watching them drink. She 
also reported hearing rats scurrying around 
the house. 

Her husband reports that he had never seen 
any insects or rodents in the house. She had 
recently become slow and socially withdrawn. 

During further assessment , which of the 
following is o good prognostic factor? 

1- Negative symptoms 

2- Single 

3- Male 

4- Good response to medication 

5- Poorly socially adjusted 


Benzodiazepine elevates the level of an 
inhibitory neurotransmitter, GABA, therefore 
it serves as a tranquilizer. 

Commonly abused barbiturates include 
amobarbital and pentobarbital. 


Answer & Comments 

Answer: 4- Good response to medication 

The diagnosis is likely to be schizophrenia as 
the patient has hallucinations and delusions. 


These drugs depress the respiratory and 
nervous system functions; and, because 
abusers rapidly build up a tolerance to the 
effects of the drug, fatal overdose or coma can 
easily occur. 

Symptoms of withdrawal syndrome appear 
12-20 hours after the last dose. They include 
anxiety, irritability, elevated heart and 
respiration rate, muscle pains, tremors, 
confusion, and seizures. 


Negative symptoms (blunted affect, emotional 
withdrawal, apatheticness, social withdrawal, 
lack of spontaneity), male, single, poor social 
adjustment and poor response to medication 
are all poor prognostic factors in 
schizophrenia. 


[ Q: 1697 ] MRCPass - Psychiatry 


A 50 year old man has been involved 
in a car accident is admitted for assessment. 




[ Q: 1696 ] MRCPass - Psychiatry 
A 45 year old housewife was 


Since then, he has had memory loss and 
fatigue. He is told that he may have frontal 
lobe damage. 


Dr. Khalid Yusuf El-Zohry - Sohag Teaching Hospital (01118391123) 

Ref MRCPass OE OE 2012 PasTest 2009 PassMedicine 2009 PasTest Exam 




ReviseMRCP 

675 



























El-zohry MRCP Questions Bank (Port 1) - 2013 


(For my personal use) 


Which of the following might be associated 
with frontal lobe damage? 

1- Left right disorientation 

2- Homonymous hemianopia 

3- Dressing apraxia 

4- Finger agnosia 

5- Perseverance 

Answer & Comments 

Answer: 5- Perseverance 


1- Perseveration 

2- Weight loss 

3- Anhedonia 

4- Loss of libido 

5- Early morning waking 

Answer & Comments 

Answer: 1- Perseveration 

Perseveration occurs in schizophrenia and 
frontal lobe brain damage. Weight loss, 


One of the specific behavior deficits following 
frontal lobe damage is attention disorder, 
patients showing distractibility and poor 
attention. They present with poor memory, 
sometimes referred to as "forgetting to 
remember." The thinking of patients with 
frontal lobe injury tends to be concrete, and 
they may show perseveration and stereotypy 
of their responses. The perseveration, with 
inability to sw itch from one line of thinking to 
another, leads to difficulties with arithmetic 
calculations, such as serial sevens or carryover 
subtraction. 

Other features of frontal lobe syndromes 
include aphasia, reduced activity, particularly 
a diminution of spontaneous activity, lack of 
drive, inability to plan ahead, and lack of 
concern. 


[ Q: 1698 ] MRCPass - Psychiatry 

A 60 year old lady has reports of 
increasing social withdrawal, marked apathy 
tow ards all activities, hypersomnia, poor 
appetite, and decreased energy. Her family 
noted that she had frequently been confusing 
appointments where she had previously been 
quite organized and punctual. Mental status 
exam revealed a woman with depressed mood 
and blunted affect, who displayed marked 
slow ness of mentation and apathy. 

Which of the following symptoms suggests 
frontal lobe damage? 



anhedonia, loss of libido and early morning 
waking are biological features of depression. 

Confusional states, dementia, behavioral and 
mood disturbances, including irritability, 
euphoria, or depression, are also encountered 
in frontal lobe tumors / lesions. 



[ Q: 1699 ] MRCPass - Psychiatry 

A 65 year old man was admitted 
unwell and has been diagnosed with a chest 
infection. On the second day of admission, he 
becomes acutely confused and threatens to 
attack another patient. 


Which medication should be used for 
sedation? 


1- Trazadone 


2- Lorazepam 

3- Haloperidol 

4- Clozapine 

5- Chlorpromazine 


Answer & Comments 

Answer: 3- Haloperidol 

Haloperidol is longer acting than the 
benzodiazepines, and is one of the better 
options for sedation. In addition, caution 
should be used in a patient with respiratory 
problems due to the risk of respiratory 
depression. 


Dr. Khalid Yusuf El-Zohry - Sohag Teaching Hospital (01118391123) 

Ref MRCPass OE OE 2012 PasTest 2009 PassMedicine 2009 PasTest Exam ReviseMRCP 



























El-zohry MRCP Questions Bank (Port 1) - 2013 


(For my personal use) 



[ Q: 1700 ] MRCPass - Psychiatry 

A 45 year old woman has lost her 
husband in a road traffic accident 8 


years ago. She is unable to discuss the event 
without experiencing intense sadness. 


Which of the following is an abnormal grief 
reaction? 


1- Insomnia 


2- Poor appetite 

3- Thoughts of dying 

4- Feelings of hopelessness, guilt and 
worthlessness 

5- Feelings lasting 6 months from the event 


Answer & Comments 

Answer: 4- Feelings of hopelessness, guilt and 
worthlessness 

Grief reaction causes symptoms of decrease in 
appetite, weight loss, ruminations, troubled 
sleep, distractability, thoughts about dying, 
and impaired concentration. 

Symptoms last for up to 6 months. 

Feelings of hopelessness, guilt and 
worthlessness may signify depression or 
abnormal grief reaction. Abnormal grief 
reactions when symptoms persist for years 
after and the patient still grieves as if the 
event was recent. 


[ Q: 1701 ] MRCPass - Psychiatry 

A 50 year old man has a constant 
inclination to arrange all the books in his 
house, then rearranging it because he is not 
satisfied it is in the appropriate order. He has 
insight into the problem but is unable to 
control his actions. 

What is the diagnosis? 

1- Delusional behaviour 

2- Obsessive compulsive disorder 

3- Drug abuse 



4- Anxiety disorder 

5- Transient global amnesia 


Answer & Comments 

Answer: 2- Obsessive compulsive disorder 

A ritual may reduce anxiety but the behaviour 
is not classically anxiety driven. An insight into 
the obsessive behaviour but an inability to 
control it is typical of obsessive compulsive 
disorder 


[ Q: 1702 ] MRCPass - Psychiatry 

A 25 year old secretary has a history 
of depression for 2 years. She was brought 
into casualty by her sister. For a week, she had 
grandiose delusions, hyperexcitement, 
pressure of speech and flight of ideas. 

The most likely diagnosis is: 

1- Somatization 

2- Schizophrenia 

3- Bipolar disorder 

4- Schizoaffective disorder 

5- Depressive psychosis 



Answer & Comments 

Answer: 3- Bipolar disorder 

The features described would fit with mania, 
and would be most likely in conjunction with 
bipolar disorder. 


[ Q: 1703 ] MRCPass - Psychiatry 

A 40 year male presents to the 
hospital with multiple episodes of visual 
hallucinations which last several days. He is 
also vague and is not aw are of much of the 
activity around him. On examination, the 
patient is tremulous. 

Paranoid psychosis is diagnosed by the junior 
doctor but the consultant disagrees after 
seeing results of deranged liver function tests. 



Dr. Khalid Yusuf El-Zohry - Sohag Teaching Hospital (01118391123) 

Ref MRCPass OE OE 2012 PasTest 2009 PassMedicine 2009 PasTest Exam 




ReviseMRCP 

677 































El-zohry MRCP Questions Bank (Port 1) - 2013 


(For my personal use) 


What is the likely diagnosis? 

1- Dementia 

2- Paranoia 

3- Bipolar disorder 

4- Alcohol withdrawal 

5- Drug overdose 


Answer & Comments 

Answer: 4- Alcohol withdrawal 

There are hallucinations and also clouding of 
consciousness. This is suggestive of delirium 
tremens or alcohol withdrawal. Clouding of 
consciousness does not occur in dementia. It 
occurs when the patient is awake and 
functioning, but has an incomplete or 
distorted aw areness of the environment. It is 
a higher level of aw areness than stupor, in 
which the awake patient is unaw are and 
unresponsive to the environment. 


[ Q: 1704 ] MRCPass - Psychiatry 

A 55 year old man had an episode 
he cycled very hard home after stress 
at work. He developed an episode of memory 
loss during which he was able to hold a 
conversation, but seemed to lose any memory 
of what happened at work. 6 hours later he 
was able to recall everything except the 
episode of memory loss. 

What is the likely diagnosis of this event? 

1- Subarachnoid haemorrhage 

2- Frontal lobe infarct 



where 


attack lasts for hours, sometimes a whole day. 
In many cases (up to a third), TGA is preceded 

by 

physical or emotional stress, rushing around 
and personal dramas. Sex has frequently been 
implicated as well. 

A person with TGA is disorientated, though 
physically well. They can think and 
communicate, but forget what time it is or 
where they are - responding to something 
intelligently but after about a minute 
forgetting. Despite patchy memory loss about 
the recent past, distant memories remain. The 
underlying cause is proposed to be vascular 
insufficiency to the memory areas 
hippocampi. 



[ Q: 1705 ] MRCPass - Psychiatry 

A 55 year old lady has presented 
with chronic knee pain and sw elling. She has 
been examined by a rheumatologist, and 
along with knee X rays, no pathology was 
found. She mentions that she is unable to 
work due to the pain in her knee but is not 
sure why she gets knee pains despite multiple 
visits to the doctor without a diagnosis. 

What is the likely problem? 

1- Anxiety disorder 

2- Somatisation 


3- Dysmorphophobia 

4- Schizophrenia 

5- Hypochondriasis 


3- Transient global amnesia 

4- Schizophrenia 

5- Mania 

Answer & Comments 

Answer: 3- Transient global amnesia 

Transient Global Amnesia or TGA is a sudden 
failure of memory, affecting older adults. The 


Answer & Comments 

Answer: 2- Somatisation 

Patients with hypochondriasis have a high rate 
of psychiatric comorbidity. 

The core feature of hypochondriasis is not 
preoccupation with symptoms themselves, 
but rather the fear or idea of having a serious 
disease. The fear or idea is based on the 


Dr. Khalid Yusuf El-Zohry - Sohag Teaching Hospital (01118391123) 

Ref MRCPass OE OE 2012 PasTest 2009 PassMedicine 2009 PasTest Exam ReviseMRCP 



678 


























El-zohry MRCP Questions Bank (Port 1) - 2013 


(For my personal use) 


misinterpretation of bodily signs and 
sensations as evidence of disease. 

Somatization disorder is marked by multiple 
medically unexplained physical, or somatic, 
symptoms. The somatic complaints must be 
serious enough to interfere significantly with a 
person's ability to perform important 
activities, such as work or family, or lead the 
person experiencing the symptoms to seek 
medical treatment. Dysmorphophobia is a 
psychiatric condition, also termed body 
dysmorphic disorder. It is characterised by a 
fixation on an imaginary flaw in the physical 
appearance (e.g. hair or acne). 



[ Q: 1706 ] MRCPass - Psychiatry 

A 35 year patient has recently 
received a new drug for schizophrenia. He 
presented 3 weeks later a with a fever and felt 
lethargic. His investigations show : Hb 11.5 
g/dl, WCC 2.3 xl0 9 /l, Platelets 122 xl0 9 /l. 


Which is likely to be the drug which was 
commenced? 


1- Carbamazepine 

2- Haloperidol 

3- Clozapine 

4- Chlorpromazine 

5- Thioridazine 


Answer & Comments 

Answer: 3- Clozapine 

Clozapine induced agranulocytosis occurs in 
about 1% to 10% of patient who take 
clozapine. Patients who have experienced 
agranulocytosis with prior treatment of 
clozapine should not receive clozapine again. 


next year complained of breathlessness. She 
has not worked since the symptoms started. 

She does not have insight that despite 
multiple negative investigations her symptoms 
may not be organic. She has had a second 
opinion and continues to seek a different 
opinion. 

What is the diagnosis? 

1- Somatisation disorder 

2- Conversion disorder 

3- Hypochondriacal disorder 

4- Delusional disorder 

5- Depression 


Answer & Comments 

Answer: 1- Somatisation disorder 

The constant change in symptoms with 
negative clinical investigation results suggests 
somatisation. In hypochondriacal disorder the 
patients concentrate more on a disease e.g. 
diabetes or migraines, rather than the 
symptoms. 


[ Q: 1708 ] MRCPass - Psychiatry 

A 30 year old patient has admitted to 
having suicide intention to the medical staff 
on the wards. 

Which one of the following is a worrying 
feature? 

1- Number of tablets overdosed 

2- Female sex 

3- Age 30 

4- Writing a suicide note 

5- Absence of family history 



Answer & Comments 

Answer: 4- Writing a suicide note 

A 50 year old lady has had abdominal pains for 
2 years, headaches for a year and then the 



Dr. Khalid Yusuf El-Zohry - Sohag Teaching Hospital (01118391123) 

Ref MRCPass OE OE 2012 PasTest 2009 PassMedicine 2009 PasTest Exam ReviseMRCP 





























El-zohry MRCP Questions Bank (Port 1) - 2013 


(For my personal use) 


The act of writing a suicide notes indicates 
planning and intention of suicide (as opposed 
to impulsive overdose). 

Young and old ages, male sex, family history 
and planning are features of increased risk in 
deliberate self harm. 



[ Q: 1709 ] MRCPass - Psychiatry 
A 65 year old man is assessed for 


cognitive impairment. 


His wife mentions that his memory has not 
been as good lately. He has been leaving the 
front door open and the cooker on 
occasionally. 

Which of the following features suggests 
frontal lobe dysfunction? 


1- Impaired longterm recall 


2- Perserveration 


3- Sensory ataxia 

4- Finger agnosia 

5- Astereognosis 


Answer & Comments 

Answer: 2- Perserveration 

Perseveration is the act of persisting; 
continuing or repeating behavior. This can be 
associated with difficulties in planning or 
executing action. Frontal lobe changes also 
include personality change, disinhibition, 
euphoria and apathy. 


4^ [ Q: 1710 ] MRCPass - Psychiatry 

# A 50 year old woman has been 
taking lithium for more than 10 years for 
bipolar disorder. 

Which of the following is a known side effect? 

1- Polyuria 

2- Optic atrophy 

3- Weight loss 

4- Raynaud's syndrome 


5- Telangiectasia 

Answer & Comments 

Answer: 1- Polyuria 

Polyuria and polydipsia, fine tremor of the 
hands, weight gain, nausea and 
hypothyroidism are side effects of lithium. 


[ Q: 1711 ] MRCPass - Psychiatry 

A 55 year old woman with memory 
loss is considered for a diagnosis of 
Alzheimer's disese by the psychogeriatrician. 

Which of these is typical of early Alzheimer's 
disease? 

1- Impaired short term memory 

2- Urinary incontinence 

3- Seizures 

4- Cerebellar signs 

5- Auditory hallucinations 



Answer & Comments 

Answer: 1- Impaired short term memory 

Alzheimer's disease is typified early in disease 
by short term memory loss. 



[ Q: 1712 ] MRCPass - Psychiatry 

A 21 year old has come to hospital 
complaining of nausea. She also complains of 
frequent episodes of restlessness, inability to 
sleep and also paresthesiae in her hands. 
There is no relevant past medical history. 

What is the diagnosis? 

1- Somatisation 


2- Conversion disorder 


3- Personality disorder 

4- Anxiety disorder 

5- Post traumatic stress disorder 


Dr. Khalid Yusuf El-Zohry - Sohag Teaching Hospital (01118391123) 

Ref MRCPass OE OE 2012 PasTest 2009 PassMedicine 2009 PasTest Exam ReviseMRCP 



680 































El-zohry MRCP Questions Bank (Part 1) - 2013 


(For my personal use) 


Answer & Comments 


Answer: 4- Anxiety disorder 


The condition starts in early adult life, and is 
associated with depression. 


Symptoms typically associated with anxiety 
are : 

restlessness or feeling keyed up or on edge 

paresthesiae 

being easily fatigued 

difficulty concentrating or mind going blank 

irritability 

muscle tension 

sleep disturbance 


[ Q: 1713 ] MRCPass - Psychiatry 

A 35 year old woman contamination 
obsessions and washing compulsions in the 
preceding one month. 

Preoccupied with thoughts of contamination, 
she had started spending the majority of time 
washing herself or cleaning various household 
items. 

She described these thoughts as being her 
own and recognised them to be "irrational", 
but she could not resist them. 



[ Q: 1714 ] MRCPass - Psychiatry 

A 55 year old lady is constantly tired 
and is off work frequently despite previously 
having been very conscientious at work. She 
complains of mild headaches and myalgia. Her 
sleep pattern was irregular and she found it 
difficult to sleep at night. She mentions that 
she gets tired doing very little. Neurological 
examination is unremarkable. 

What is the diagnosis? 

1- Psychotic depression 

2- Anorexia nervosa 

3- Anxiety disorder 

4- Chronic fatigue syndrome 

5- Schizophrenia 



Answer & Comments 

Answer: 4- Chronic fatigue syndrome 

Chronic fatigue syndrome is defined as 
unexplained fatigue of at least six months' 
duration. 4 of the following symptoms satisfy 
the criteria: 


Which one of following is true regarding 
obsessional neurosis? 

1- It starts in the elderly 

2- There is no risk depression 

3- There is good insight 

4- The patients tend to be violent 

5- Low intelligence is typically associated 


the Joint pain 

Ineffective sleep 
Tender lymph nodes 
Myalgia 
Headache 

Subjective memory impairment 


Answer & Comments 

Answer: 3- There is good insight 

Obsessional neuroses causes rituals or 
thoughts. Although some of the thoughts may 
involve trying to hurt others, there is good 
insight and patients are not typically violent. 


Postexertional lethargy 


( 


• ( J 


[ Q: 1715 ] MRCPass - Psychiatry 


A 40 year old accountant has bipolar 
disorder. He has recently become very 
talkative and is unable to concentrate at work. 
Hypomania is diagnosed. 


Dr. Khalid Yusuf El-Zohry - Sohag Teaching Hospital (01118391123) 

Ref MRCPass OE OE 2012 PasTest 2009 PassMedicine 2009 PasTest Exam 




ReviseMRCP 

681 






























El-zohry MRCP Questions Bank (Port 1) - 2013 


(For my personal use) 


What is the recommended treatment? 

1- ECT 

2- Lithium 

3- Olanzepine 

4- Procyclidine 

5- Fluoxetine 


Answer & Comments 

Answer: 3- Olanzepine 

Lithium is used for prophylaxis in bipolar 
disorder. Olanzepine or benzodiazepines are 
more effective during episodes of hypomania. 


[ Q: 1717 ] MRCPass - Psychiatry 

A 55 year old man presents with a 10 
day history of confusion. His friend 
mentions that he drinks 15 units of alcohol a 
day. 



Which of the following strongly suggests a 
diagnosis of Korsakoff's psychosis? 


1- Absence seizures 


2- Delusional beliefs 

3- Poor long term memory 

4- Inventing recent events 

5- Auditory hallucinations 



[ Q: 1716 ] MRCPass - Psychiatry 

A 40 year old patient is assessed for 
periods of breathlessness. Although she is a 
smoker and has early bronchitis. She describes 
uncontrollable episodes where she has 
intense fear that she will die. During these 


episodes she shakes and hyperventilates. 
What is the diagnosis? 


1- Panic disorder 

2- Psychotic depression 


3- Post traumatic stress disorder 


4- Somatisation 

5- Obsessional neurosis 


Answer & Comments 

Answer: 1- Panic disorder 

One of the characteristic features of panic 
disorder is the fear that something drastic is 
going to happen during an attack, e.g. 
impending doom. The episode usually lasts for 
several minutes, and the patient may react 
with fear or try to escape the situation. A 
permanent feeling of nervousness suggests 
generalised anxiety disorder rather than panic 
disorder. 


Answer & Comments 

Answer: 4- Inventing recent events 

Korsakoff's is typified by associated short term 
memory loss and confabulation (inventing 
recent events). 


[ Q: 1718 ] MRCPass - Psychiatry 

A 66 year old man has become more 
forgetful over the last 12 months. 

His wife complains that he does not 
remember his friends' names and on several 
occasions has gone for a walk and not found 
his way back. He is also unsteady on his feet. 
On examination he has a mask like face and 
cogw heel rigidity of his upper limbs. 

The most likely diagnosis is: 

1- CJD 

2- NvCJD 

3- Vascular dementia 

4- Lewy body dementia 

5- Alzheimer's disease 



Answer & Comments 

Answer: 4- Lewy body dementia 

Lew y body dementia is characterised by 
fluctuating cognition, visual hallucinations, 


Dr. Khalid Yusuf El-Zohry - Sohag Teaching Hospital (01118391123) 

Ref MRCPass OE OE 2012 PasTest 2009 PassMedicine 2009 PasTest Exam ReviseMRCP 



682 






























El-zohry MRCP Questions Bank (Port 1) - 2013 


(For my personal use) 


parkinsonism, falls, transient loss of 
consciousness and delusions 


Answer & Comments 


Answer: 5- Obsssive compulsive disorder 


[ Q: 1719 ] MRCPass - Psychiatry 

A 23 year old man who suffers from 
schizophrenia has been on chlorpromazine. He 
complains of gynaecomastia and 
galactorrhoea. 

In this patient , chlorpromazine could be 
changed to: 

1- Haloperidol 

2- Quetiapine 

3- Thioridazine 

4- Sulpiride 

5- Risperidone 



Answer & Comments 

Answer: 2- Quetiapine 

Classical antipsychotics increase plasma 
prolactin concentrations by their blocking 
action on dopamine receptors in the 
tuberofundibular pathw ay. They can cause 
gynaecomastia and galactorrhoea and 
menstrual disturbances. A change to an 
atypical agent such as quetiapine or 
olanzapine should minimise this effect. 


The predominant feature of obsessive 
compulsive disorder is a ritual which is 
unreasonable, but essentially unstoppable. 



[ Q: 1721 ] MRCPass - Psychiatry 

A 50 year old man has recently been 
treated with amitriptyline. His brother brings 
him to the hospital following an episode 
where he went to the street telling passersby 
that he is going become the King of the 
country and conquer the world with battles. 


On examination, he was euphoric and irritable 
mood, mild grandiosity, decreased need for 
sleep, rapid and pressured speech. 


What is the likely diagnosis? 


1- Schizophrenia 


2- Somatization disorder 


3- Depressive psychosis 

4- Hypomania 

5- Anxiety disorder 


Answer & Comments 


Answer: 4- Hypomania 


[ Q: 1720 ] MRCPass - Psychiatry 

A 25 year old woman constantly 
washes her hands because of a fear of 
infection, even at times when she has not 
touched anything. 

What is the diagnosis? 

1- Personality disorder 

2- Chronic fatigue syndrome 

3- Depression 

4- Schizophrenia 

5- Obsssive compulsive disorder 



The features of disinhibition suggest 
hypomania. The diagnosis is bipolar disorder. 
A mild to moderate level of mania is called 
hypomania. Hypomania may feel good to the 
person who experiences it and may even be 
associated with good functioning. 


[ Q: 1722 ] MRCPass - Psychiatry 

A 30 year old patient presents with 
headache and numbness in her right arm. CT 
of the head is normal. There were no 
organisms seen in the CSF and protein is 
normal. 

Upon further enquiry there is a history of 
sexual abuse. 



Dr. Khalid Yusuf El-Zohry - Sohag Teaching Hospital (01118391123) 

Ref MRCPass OE OE 2012 PasTest 2009 PassMedicine 2009 PasTest Exam 




ReviseMRCP 

683 
































El-zohry MRCP Questions Bank (Port 1) - 2013 


(For my personal use) 


What is the probable diagnosis? 

1- Clinical depression 

2- Obsessive compulsive disorder 

3- Conversion disorder 

4- Anxiety disorder 

5- Acute psychosis 


Answer & Comments 

Answer: 3- Conversion disorder 

Significant stress or emotional conflicts (such 
as sexual abuse) are the most likely 
predisposing causes for conversion disorder. 


[ Q: 1723 ] MRCPass - Psychiatry 

A 65 year old man has had clinical 
depression diagnosed for 10 years. He initially 
responded to fluoxetine, but now is getting 
more episodes of depression and is suicidal. 

What is the next best management? 

1- Change to paroxetine 

2- Change to amitriptyline 

3- ECT 

4- Allow natural progression 

5- Dementia tests 



Answer & Comments 

Answer: 3- ECT 

ECT is indicated in patients who have 
responded to antidepressant medications 
before, but may not be responding now . 

Severe depression (melancholia) is the most 
frequent indication for ECT. Patients with this 
illness experience sadness and despair, have 
difficulty concentrating, lose appetite and 
weight, sleep poorly, blame themselves, are 
unable to enjoy life, and often think of suicide. 
Mania and schizophrenia are other illnesses 
that can be helped by ECT. 



[ Q: 1724 ] MRCPass - Psychiatry 

Clozapine is an atypical antipsychotic 
drug that appears to have fewer 
problems with adverse effects than older 
antipsychotics. 

The relative safety of clozapine is due to: 


1- Low affinity for 5HT receptors 


2- Decrease in prolactin levels 

3- No effect on haematopoiesis 


4- Low affinity for dopamine D2 receptors 

5- Causes sinus tachycardia 


Answer & Comments 

Answer: 4- Low affinity for dopamine D2 
receptors 

Clozapine appears to have fewer 
extrapyramidal adverse effects than older 
antipsychotics due to its relatively low affinity 
for D2 dopamine receptors. 

Unlike older antipsychotics, clozapine has 
relatively high affinity for 5HT receptors, and 
also has little effect on prolactin levels. 

Myocarditis and cardiomyopathy have been 
reported with atypical antipsychotics, and 
persistent tachycardia is an early warning sign. 
Agranulocytosis is a well-recognized 
complication of clozapine, and patients should 
be supervised under the Clozaril Patient 
Monitoring Service. 

[ Q: 1725 ] MRCPass - Psychiatry 

A patient with parkinson's disease 
has recently been commenced on an 
antipsychotic drug. He has worsening of 
rigidity and tremors. 

Which of the following is the best 
antipsychotic drug to prevent such features? 

1- Risperidone 

2- Chlorpromazine 

3- Thioridazine 



Dr. Khalid Yusuf El-Zohry - Sohag Teaching Hospital (01118391123) 

Ref MRCPass OE OE 2012 PasTest 2009 PassMedicine 2009 PasTest Exam ReviseMRCP 



684 





























El-zohry MRCP Questions Bank (Port 1) - 2013 


(For my personal use) 


4- Haloperidol 

5- Clozapine 


Answer & Comments 

Answer: 5- Clozapine 

Clozapine and quetiapine are newer, atypical 
antipsychotic drug which does not have 
parkinsonian side effects as bad as the older 
generation drugs. This is due to less dopamine 
receptor antagonism. 

[ Q: 1726 ] MRCPass - Psychiatry 

A 40 year old lady is brought to 
hospital by a friend having taken 30 tablets of 
fluoxetine 20mg all at one time. She was 
attempting to commit suicide. 

Which of the following would be consistent 
with this when she is assessed? 

1- Heart rate of 100 beats per minute 

2- Miosis 

3- Respiratory rate of 20 

4- QRS duration of 150 ms 

5- Seizures 



Answer & Comments 

Answer: 1- Heart rate of 100 beats per minute 

With fluoxetine overdose, most patients are 
asymptomatic. About 20% of patients may 
feel drow sy and have a sinus tachycardia. 

[ Q: 1727 ] MRCPass - Psychiatry 

A 20 year lady referred a year's 
history of oligomenorrhea. She is currently in 
university and has reports of poor progress 
with her work. On examination, she was found 
to have lanugo hair. 

Whot is the diagnosis? 

1- Polycystic ovary syndrome 

2- Anorexia nervosa 



3- Bulimia 

4- Anxiety disorder 

5- Depression 


Answer & Comments 

Answer: 2- Anorexia nervosa 

Lanugo hair is a fine hair that develops on the 
face, back, or arms and legs. This occurs in 
anorexia nervosa. 


[ Q: 1728 ] MRCPass - Psychiatry 

A 50 year old man has a history of 
alcohol excess, presents with a 2 week history 
of confusion. 



Which of the following suggests a diagnosis of 
Korsakoff's psychosis? 

1- Visual hallucinations 

2- Epileptic seizures 

3- Impaired longterm memory 

4- Delusional beliefs 


5- Confabulating events 


Answer & Comments 

Answer: 5- Confabulating events 

The main features of Korsakoff's psychosis is 
short term memory loss and subsequent 
compensatory confabulation by patient. Other 
symptoms may include delirium, anxiety, 
depression, confusion, delusions and 
insomnia. The treatment is with intravenous 
thiamine. 


Dr. Khalid Yusuf El-Zohry - Sohag Teaching Hospital (01118391123) 

Ref MRCPass OE OE 2012 PasTest 2009 PassMedicine 2009 PasTest Exam 




ReviseMRCP 

685 































El-zohry MRCP Questions Bank (Part 1) - 2013 


(For my personal use) 



[ Q: 1729 ] MRCPass - Statistics 

A group of 100 patients were 
involved in a study of clubbing and 
endocarditis. There were 8 patients with SBE 
and clubbing, 2 patients with SBE without 
clubbing. There were 90 patients without SBE 
and 5 of these patients had clubbing. 


Which of the following is true? 

1- Positive predictive value = 8/(8+2) 

2- Negative predictive value = 85/(85+2) 

3- Sensitivity = 8/(8+5) 

4- Sensitivity = 8/(8+85) 

5- Specificity = 85/(85+2) 


Answer & Comments 

Answer: 2- Negative predictive value = 
85/(85+2) 

This is a good exercise for calculating 
sensitivity etc. A contingency table for this 
should be drawn out. 



SBE 

No SBE 

Clubbing 

8 

5 

No Clubbing 

2 

85 


The total of all patients is 100. PPV = 8/(8+5). 
NPV = 85/(85+2). Sensitivity = 8/(8+2). 
Specificity = 85/(85+5). 


Predictive Value Positive 

-► 


Disease 


Sensitivity 




Yes 

No | 


Pos 

TP 

FP 


Test 




Neg 

FN 

TN 


Specificity 


group was 5% compared to a incidence of 8% 
in untreated group (p<0.001). 

What is the relative risk reduction ? 

1- 5% 

2 - 8 % 

3- 3% 

4- 3/8% 

5- 5/8% 


Answer & Comments 

Answer: 4- 3/8% 

The drug reduced the risk from 8% to 5%, 
hence the relative risk reduction is the 
difference divided by the original risk ie 3/8%. 


[ Q: 1731 ] MRCPass - Statistics 

In a trial of an antiplatelet therapy in 
secondary prevention of stroke, the drug was 
shown to reduce mortality from stroke, from 
8% to 4% over 10 years. 

What is the number needed to treat to prevent 
a death over 10 years? 

1- 4 

2- 5 

3- 10 

4- 25 

5- 100 



Answer & Comments 


Answer: 4- 25 


< - 

Predictive Value Negative 


[ Q: 1730 ] MRCPass - Statistics 

A study has been performed on 
preventing lung cancer with a drug Y in a 
population who smoked. The results of the 
study showed that over a 5 year period the 
incidence of lung cancer in the drug Y treated 



The drug reduced the risk of death post stroke 
by 4% over 10 years. Therefore if 100 people 
were treated we could expect the prevention 
of 4 deaths. 


100/Absolute risk reduction = Number needed to treat 


Dr. Khalid Yusuf El-Zohry - Sohog Teaching Hospital (01118391123) 

Ref MRCPass OE OE 2012 PasTest 2009 PassMedicine 2009 PasTest Exam 


ReviseMRCP 


































El-zohry MRCP Questions Bank (Port 1) - 2013 


(For my personal use) 



[ Q: 1732 ] MRCPass - Statistics 

A study examines patients with bow 
el carcinoma. The mortality rate of those 
given drug R is 7%, compared with 10% in 
those not given drug R. 

What conclusion can be drawn? 


1- The relative risk of death when given drug R 
is 1.5 


2- The relative risk of death when given drug R 
is 7/10 

3- The number needed to treat to prevent one 
death is 3 


4- The number needed to treat to prevent one 
death is 10 

5- The absolute risk reduction is 3% 


Answer & Comments 

Answer: 5- The absolute risk reduction is 3% 

The absolute risk reduction is 10 - 7 = 3%. The 
relative risk reduction is 3/10. The number 
needed to treat is 100/absolute risk reduction 
which is 33 in this case. 


100/Absolute risk reduction = Number needed to treat 


[ Q: 1733 ] MRCPass - Statistics 

A surveillance study of a new 
medication for treatment of cholesterol has 
been carried out on 1,000 subjects who had 
completed clinical trials in the postmarketing 
phase. 

Which of the following shows information 
generated from this study? 

1- Cost-effectiveness 

2- Efficacy 

3- Cost benefit analysis 

4- Effectiveness 

5- Adverse events 



Answer & Comments 

Answer: 5- Adverse events 

The postmarketing phase is phase IV. In this 
phase, the potential side effects in the large 
population is assessed under everyday 
conditions. Drug potency is usually assessed in 
phase II, and efficacy assessed in phase III. 


[ Q: 1734 ] MRCPass - Statistics 

A treatment has been shown to be 
of benefit in treating Crohn's disease following 
a phase II study in patients. 

However there were concerns regarding 
deranged liver function from this treatment in 
animal studies. 

What study should be conducted ? 

1- Case controlled study 

2- Randomised double blind placebo study 

3- Metanalysis 

4- Sequential trial 

5- Phase I study 



Answer & Comments 

Answer: 2- Randomised double blind placebo 
study 

The study has already been performed to 
phase II and hence 

should have been shown to be safe in phase I. 
The next step should be a randomised double 
blind study to determine whether there would 
be benefit, and also whether the side effects 
are significant. 



[ Q: 1735 ] MRCPass - Statistics 

The number needed to treat (NNT) is 
defined as: 

1- 0.5 divided by absolute risk reduction 

2- 0.5 divided by relative risk reduction 

3- 100 divided by absolute risk reduction 


Dr. Khalid Yusuf El-Zohry - Sohag Teaching Hospital (01118391123) 

Ref MRCPass OE OE 2012 PasTest 2009 PassMedicine 2009 PasTest Exam ReviseMRCP 



688 





























El-zohry MRCP Questions Bank (Port 1) - 2013 


(For my personal use) 


4- 100 divided by relative risk reduction 

5- The percentage of difference x patient 


Answer & Comments 

Answer: 3- 100 divided by absolute risk 
reduction 

NNT can be calculated by 100 divided by the 
number for absolute risk reduction, or 1 
divided by the percentage of absolute risk 
reduction. 


100/Absolute risk reduction = Number needed to treat 


[ Q: 1737 ] MRCPass - Statistics 

A group of investigators studied 
dizziness as a symptom of CVA. Out of 100 
patients, there were 20 patients with 
dizziness. 10 of these patients had CVA. There 
were a further 5 patients who did not have 
dizziness, who had CVA. 

What is the pretest probability? 

1 - 10/20 

2- 5/20 

3- 10/100 

4- 15/100 

5- 20/100 



[ Q: 1736 ] MRCPass - Statistics 
Which of the following defines 

variance? 

1- 2x standard deviation 

2- Square of standard deviation 

3- Square root of standard deviation 

4- Half of standard deviation 



Answer & Comments 

Answer: 4-15/100 

Pretest Probability is defined as the 
probability of the target disorder before a 
diagnostic test result is known. This is actually 
the same as the disease prevalence, in this eg 
CVA prevalence. Hence (10 + 5)/100 = 15/100. 


5- Standard deviation divided by number in 
the population 

Answer & Comments 

Answer: 2- Square of standard deviation 

Variance is the square of standard deviation. 
Standard deviation is the square root of 
variance. 


R * = I(x-x) 2 

n 


Variance 



s ='V I< r 1 

2 


Standard Deviation 


^ [ Q: 1738 ] MRCPass - Statistics 

fl 1 - 1 - 

The mean Hb value for a group of 
150 patients is 15 g/dl. The standard deviation 
is 2 g/dl. 

Which one of these conclusions is correct? 

1- 68% of patients have a Hb between 13 and 
17 g/dl 

2- 95% of patients have a Hb between 13 and 
17 g/dl 

3- 99% of patients have a Hb between 13 and 
17 g/dl 

4- Variance is 2 

5- Variance is 1 

Answer & Comments 

Answer: 1- 68% of patients have a Hb between 
13 and 17 g/dl 


Dr. Khalid Yusuf El-Zohry - Sohag Teaching Hospital (01118391123) 

Ref MRCPass OE OE 2012 PasTest 2009 PassMedicine 2009 PasTest Exam 


ReviseMRCP 
































El-zohry MRCP Questions Bank (Part 1) - 2013 


(For my personal use) 


68% of patients fall within 1 standard 
deviation, in this case, 2 less and more than 
15. 95% of patients would have a Hb between 
11 and 19. 

Standard deviation is the square root of 
variance. Hence variance is 4. 


r 2 = I(X-X) E 

n 


Variance 

s -Vlp 1 


Standard Deviation 


regarding standard error of the mean (SEM)? 

1- SEM estimates how different a median is 
compared to the population mean 

2- 75% confidence limits for a mean are the 
mean +/-1.96 SEM 

3- SEM would be halved if we doubled the 
sample size 

4- SEM would be halved if we quadrupled the 
sample size 

5- SEM increases as standard deviation 
decreases 

Answer & Comments 

Answer: 4- SEM would be halved if we 

quadrupled the sample size 


[ Q: 1739 ] MRCPass - Statistics 

A letter published in a journal 
suggests that an established antipsychotic 
drug may cause photosensitivity. The 
manufacturer wishes set up a study to 
determine rapidly and efficiently whether this 
is true. 

Which one of the following studies is most 
appropriate? 

1- Case control study 

2- Dose ranging study 

3- Double blind, randomized, placebo 
controlled study 

4- Meta-analysis 

5- Sequential trial 



Answer & Comments 

Answer: 4- Meta-analysis 

An established drug should have many trials 
published on data. Meta-analysis incorporates 
the data in order to identify effects which 
some trials may be too small to detect. 



[ Q: 1740 ] MRCPass - Statistics 
Which one of the following is correct 


Standard error of the mean estimates how 
close a study mean is compared to the 
population mean. 

The formula for Standard Error of the Mean = 
Std Dev/ Square Root of Sample Size. Hence, 
the SEM would be halved if the sample size 
were quadrupled. 

95% of observations lie 1.96 (or nearest 
estimate 2) standard deviations aw ay from 
the mean. SEM increases as the standard 
deviation increases. 

Standard error (sent): 

The standard error of the mean is 
designated as: om It is the standard 
deviation of the sampling distribution of 
the mean The formula for the standard 
error of the mean is: 



where a is the standard deviation of the 
original distribution and N is the sample 
size (the number of scores that each mean 
is based on) 


[ Q: 1741 ] MRCPass - Statistics 

Which of these is an example of a 
nonparametric test? 

1- Chi square 



Dr. Khalid Yusuf El-Zohry - Sohag Teaching Hospital (01118391123) 

Ref MRCPass OE OE 2012 PasTest 2009 PassMedicine 2009 PasTest Exam 


ReviseMRCP 




































El-zohry MRCP Questions Bank (Port 1) - 2013 


(For my personal use) 


2- Students t test 

What is the number needed to treat over 10 

3- Kaplan Meier analysis 

years to prevent 1 death ? 

4- Wilcoxon rank test 

1-5 

5- Correlation coefficient 

2- 10 


3- 15 

Answer & Comments 

4- 20 

Answer: 4- Wilcoxon rank test 

5-25 


The nonparametric tests can be used when 
distribution is not normal (skewed). The 
Wilcoxon rank sum test and Mann U Whitney 
tests are good examples of nonparametric 
tests. Other examples are the Kruskal-Wallis 
(KW) test and Friedman's test. 

[ Q: 1742 ] MRCPass - Statistics 

A publication assesses a tumour 
marker used to test for ovarian cancer. 

Which of the following would demonstrate the 
number of coses of ovarian cancer correctly 
identified by this new test , out of all the 
ovarian cancer cases? 

1- Sensitivity 

2- Specificity 

3- Positive predictive value 

4- Negative predictive value 

5- Accuracy 



Answer & Comments 


Answer: 1- Sensitivity 


Answer & Comments 

Answer: 4- 20 

NNT is defined as number needed to treat to 
prevent 1 death. The way to work this out is 
100 /(15 -10) = 20. 


100/Absolute risk reduction = Number needed to treat 


^ [ Q: 1744 ] MRCPass - Statistics 

n - 

* A study is conducted measuring 
body weight and risk of progession tow ards 
hypertension. 

Which of following best statistical test to 
evaluate results? 

1- Student's paired t test 

2- Chi squared test 

3- Student's unpaired t test 

4- Coefficient of linear regression 

5- Log rank test 


The sensitivity assess how many pickups there 
are with the test, hence the number of cases 
correctly identified out of all the cases. 

^ [ Q: 1743 ] MRCPass - Statistics 

/ l — UJ - - WL - 

* A study compared carotid 
endarterectomy with medical therapy for 
stroke prevention over 10 years. There were 
15% of patients developing in the medical 
group, and 10% in the carotid endarterectomy 
group developing a stroke. 


Answer & Comments 

Answer: 4- Coefficient of linear regression 

In such a study, a plot of weight against 
hypertension would allow regression analysis. 

A coefficient can be obtained to 
demonstrated how closely associated they are 
(the closer to a value of 1, the higher the 
coreelation). 


Dr. Khalid Yusuf El-Zohry - Sohag Teaching Hospital (01118391123) 

Ref MRCPass OE OE 2012 PasTest 2009 PassMedicine 2009 PasTest Exam 


ReviseMRCP 




























El-zohry MRCP Questions Bank (Part 1) - 2013 


(For my personal use) 



[ Q: 1745 ] MRCPass - Statistics 

The sensitivity of a new blood test 
(Blood Test X) for heart failure has been 
assessed in heart failure. The test results were 
compared with echocardiography as a gold 
standard for the diagnosis of heart failure. The 
following are the results: 



Heart Failure 

No heart 

failure 

X positive 

\ 30 

10 \ 

X negative 

5 

45 i 


What is the sensitivity of blood test X? 


1- 5/30 

2- 10/35 

3- 30/35 

4- 5/45 

5- 30/45 


There a 10% risk of myocardial infarction in 
the group receiving the drug and 20% risk of 
Ml in the placebo group. 

What number of patients are required to 
prevent one myocardial infarction? 

1- 1 

2- 5 

3- 10 

4- 20 

5- 100 


Answer & Comments 

Answer: 3-10 

The absolute risk reduction is 10% and the 
relative risk reduction is 0.5. 

The number needed to treat is 100 divided by 
the absolute risk reduction. 


Answer & Comments 

Answer: 3- 30/35 

The sensitivity is the number of positive pick 
ups out of all those who had the condition. 
Taking into account patients with heart 
failure, 30 patients had Blood test X positive 
and 5 patients had Blood test X negative. 
Hence sensitivity (expressed in %) of Blood 
Test X is 30/35. 


Predictive Value Positive 

-► 


Disease 


Sensitivity 




Yes 

No | 


Pos 

TP 

FP 


Test 



v 

Neg 

FN 

TN 


Specificity 


* - 

Predictive Value Negative 



[ Q: 1746 ] MRCPass - Statistics 

A randomised double blind 
controlled study of a blood pressure low ering 
drug assessed its effect on myocardial 
infarction. The follow up period was 10 years. 


This would be 100/10 which is 10. 

^ [ Q: 1747 ] MRCPass - Statistics 

A - 

# A set of data has been collected for 
treatment for breast cancer as follows : 



Survived 

Not Survived 

With Drug 

25 

32 

Without Drug 

20 

is ~1 


Which is the best form of analysis? 

1- Students' t test 

2- Chi square test 

3- Pearson's correlation 

4- Survival analysis 

5- Kendall's test 


Answer & Comments 

Answer: 2- Chi square test 

This is a 2x2 contingency table. A statistics 
calculator can be used to obtain a p value with 
the chi square test to determine if the 
treatment has made a significant difference. 


Dr. Khalid Yusuf El-Zohry - Sohag Teaching Hospital (01118391123) 

Ref MRCPass OE OE 2012 PasTest 2009 PassMedicine 2009 PasTest Exam 


ReviseMRCP 






































El-zohry MRCP Questions Bank (Port 1) - 2013 


(For my personal use) 


[ Q: 1748 ] MRCPass - Statistics 

A new blood test for pulmonary 
embolism has a sensitivity of 99% and a 
specificity of 20%. It has a positive predictive 
value of 30% and a negative predictive value 
of 40%. 

A test with these characteristics would be 
inappropriate in which conditions? 

1- Screening many patients to identify for 
CTPA 

2- Screening in A+E, to admit all patients who 
are positive for blood test 

3- Screening in A+E and discharging all 
patients who are negative 

4- Screening in circumstances which 
symptoms are suggestive but confirming 
with a VQscan 

5- Telling patients with a positive blood test 
they may have PE but it is not a definitive 
test 



Answer & Comments 

Answer: 3- Screening in A+E and discharging 
all patients who are negative 

The test has a high pick up rate or sensitivity, 
but is not specific enough to exclude PE. 
Hence patients who have a positive test 
should have a second test such as VQ or CTPA 
to confirm the diagnosis. PE cannot be safely 
excluded unless the negative predictive value 
is high. 


[ Q: 1749 ] MRCPass - Statistics 

A group of Turner's syndrome 
patients have their heights measured in a 
study. 

Which of the following gives a good estimate 
of the spread of heights? 

1- Standard deviation 

2- Mean 

3- Mode 



4- Median 

5- Standard error of mean 


Answer & Comments 

Answer: 1- Standard deviation 

The standard deviation would give the best 
estimate of a spread of a measurement about 
the mean, in this case, height. 


[ Q: 1750 ] MRCPass - Statistics 

In a study for a new drug, a 
researcher wants to compare the 
concentration of the drug at rest, and after 
exercise in several subjects. 

Which of the following tests is appropriate to 
assess for a difference? 

1- Pearson's correlation coefficient 

2- Paired t test 

3- Chi square test 

4- Wilcoxon rank test 

5- Unpaired t test 



Answer & Comments 

Answer: 2- Paired t test 

The drug is assessed with the same volunteers 
but with different conditions. This means that 
a paired t test can be used. 

[ Q: 1751 ] MRCPass - Statistics 

A nationw ide cross-sectional survey 
was conducted to compare the prevalence of 
asthma symptoms among high school 
students who have smoked compared to 
those who have not smoked. 

Which one of the following tests is best used 
to compare its prevalence? 

1- Mann Whitney test 

2- Logistic regression analysis 

3- Spearmann's correlation 



Dr. Khalid Yusuf El-Zohry - Sohag Teaching Hospital (01118391123) 

Ref MRCPass OE OE 2012 PasTest 2009 PassMedicine 2009 PasTest Exam 


ReviseMRCP 




























El-zohry MRCP Questions Bank (Part 1) - 2013 


(For my personal use) 


4- Chi square test 


Answer & Comments 


5- Kaplan Meier curve 


Answer: 2- Negative predictive value 


Answer & Comments 


Answer: 4- Chi square test 


The negative predictive value tells us the 
proportion of individuals who test negative 
who do not have a target condition. 


In order to compare the prevalence in two 
groups, the chi square test is most 
appropriate. 


[ Q: 1752 ] MRCPass - Statistics 

A new genetic test for Marfan's 
syndrome has recently been tested. 

Which of the following can test for the number 
of patients who would be identified by the 
test? 

1- Positive predictive value 

2- Negative predictive value 

3- Variation 

4- Sensitivity 

5- Specificity 




[ Q: 1754 ] MRCPass - Statistics 

The frequency of attendance of a 
100 medical students at lectures were 
recorded by an observer over a 3 month 
period. The students were then assessed at 
the end with a multiple choice exam with a 
test score marked out of a hundred. 


Which of these statistical methods is best used 
to analyse the effectiveness of frequency of 
attendance on higher test scores? 

1- Mann-Whitney test 

2- Spearmann correlation 

3- Chi square test 

4- Fisher's exact test 

5- Student's t test 


Answer & Comments 

Answer: 4- Sensitivity 

The sensitivity of a test is the proportion of 
people with the disease who have a positive 
test result. The higher the sensitivity, the 
greater the detection rate. 


Answer & Comments 

Answer: 2- Spearmann correlation 

Spearmann's correlation is the best method to 
determine two variables which do not follow a 
normal distribution. 


[ Q: 1753 ] MRCPass - Statistics 

A new diagnostic blood test for 
pulmonary embolus has been described. Out 
of all patients tested negative, which 
parameter measures the true numbers of 
patient who do not have pulmonary embolus? 

1- Positive predictive value 

2- Negative predictive value 

3- Sensitivity 

4- Specificity 

5- Odds ratio 



[ Q: 1755 ] MRCPass - Statistics 

A randomised, double blind, placebo 
controlled trial of an asthma drug has been 
performed. 1000 subjects are treated with 
active drug and 1000 are treated with 
placebo. They are followed up over a five year 
period. 200 individuals in placebo group 150 
in treatment group had hospitalisation due to 
asthma. 

What is the annual percentage relative risk 
reduction of asthma hospitalisation in the 
treatment group as compared to the placebo 
group? 



Dr. Khalid Yusuf El-Zohry - Sohag Teaching Hospital (01118391123) 

Ref MRCPass OE OE 2012 PasTest 2009 PassMedicine 2009 PasTest Exam ReviseMRCP 



694 




































El-zohry MRCP Questions Bank (Part 1) - 2013 


(For my personal use) 


1- 2.5% 

2- 5% 

3- 10% 

4- 25% 

5- 50% 


Answer & Comments 

Answer: 2- 5% 

The absolute risk reduction is 50 and the 
relative risk reduction is 50/200, which is 25%. 
However, the annaul percentage is required, 
hence divided by 5, it is 5%. 



[ Q: 1756 ] MRCPass - Statistics 

In a survey of 100 patients, 30 had 
ascites of which 25 had alcoholic cirrhosis. 10 
other patients who had no ascites did have 
alcoholic cirrhosis. 


Which of the following is correct? 

1- Positive predictive value is 25/(25+5) 

2- Positive predictive value is 25/(25+10) 

3- Sensitivity is 25/(25+5) 

4- Specificity is 25/(25+5) 

5- Specificity is 5/(25+5) 


Answer & Comments 

Answer: 1- Positive predictive value is 
25/(25+5) 

The table created should show : 



Cirrhosis 

No cirrhosis 

With Drug 

25 

5 ~~\ 

Without Drug 

10 J 

60 


PPV is the the no of patients with ascites 
confirmed cirrhosis / total no of patients with 
ascites, hence 25/(25+5). 


Predictive Value Positive 

-► 


Disease 


Sensitivity 




Yes 

No | 


Pos 

TP 

FP 


Test 




Neg 

FN 

TN 


Specificity 


A - 

Predictive Value Negative 



[ Q: 1757 ] MRCPass - Statistics 

A statistician wants to compare the 
significance of cholesterol levels in a group of 
200 patients before and after treatment with 
a new drug. 


What is the most appropriate statistical test? 

1- Logistic regression analysis 

2- Paired t test 


3- Chi square test 

4- Kaplan meier curve 

5- Likelihood ratios 


Answer & Comments 

Answer: 2- Paired t test 

In a clinical trial where the input variable is 
type of treatment - a nominal variable - and 
the outcome may be some clinical measure 
which is normally distributed, the required 
test is then the T-test. The paired t-test and 
the Wilcoxon signed-rank test are often 
conducted to compare two continuous 
outcomes from paired observations. An 
assumption underlying these tests is that the 
responses from pair to pair are mutually 
independent. 


^ [ Q: 1758 ] MRCPass - Statistics 

A - 

* Which test is the best of the 
following, to compare two groups of 
categorical data, e.g. developed Ml/ did not 
develop Ml when a drug or placebo is given? 

1- Pearson's correlation coefficient 

2- Students t test 


Dr. Khalid Yusuf El-Zohry - Sohag Teaching Hospital (01118391123) 

Ref MRCPass OE OE 2012 PasTest 2009 PassMedicine 2009 PasTest Exam 


ReviseMRCP 
































El-zohry MRCP Questions Bank (Part 1) - 2013 


(For my personal use) 


3- Chi square test 

4- Wilcoxon rank test 

5- Multivariate analysis 

Answer & Comments 

Answer: 3- Chi square test 

Chi-squared tests are used to compare 
percentages or proportions of categorical 
data. Data such as the above can be organised 
into a 2x2 contingency table. From the chi- 
squared value a p value is read off a statistical 
table (depends on degree of freedom) to give 
the degree of significance. 

Normally distributed data can be compared 
with a Student's t-test. Skewed continuous 
data can be compared with a Wilcoxon rank- 
sum test or a Mann-Whitney U-test. 


^ [ Q: 1760 ] MRCPass - Statistics 

f, - 

# 200 patients were in a study of 

intelligence. Mean IQ scores was 150 and 
standard deviation was 15. 

What is the coefficient of variation? 

1- 5% 

2 - 10 % 

3- 15% 

4- Square root of 15% 

5- 20% 

Answer & Comments 

Answer: 2-10% 

Coefficient of variation is expressed in %. The 
definition coefficient of variation V = 
SD/mean. 


[ Q: 1759 ] MRCPass - Statistics 

There have been many small reports 
showing that a drug used to treat cancer also 
caused hypertension. 

Which of the following studies would provide 
rapid information regarding this fact? 

1- Case control study 

2- Metanalysis 

3- Double blind trial 



In this example 15/150 = 10%. It is a statistical 
measure of the deviation of a variable from its 
mean. 


[ Q: 1761 ] MRCPass - Statistics 

A new drug for inflammatory bow el 
disease has been released. The 5 year 
mortality rate with the treatment is 50% and 
without the treatment it is 60%. 

The absolute risk reduction is : 



4- A review article 


1- 5% 


5- Sequential trial 


Answer & Comments 


Answer: 2- Metanalysis 


2 - 10 % 

3- 16% 

4- 20% 

5- 25% 


Metanalysis can be performed when there are 
several small studies, the study would take 
into account all the data from the studies and 
provide good estimate whether an effect was 
present. This would also be quicker and more 
cost effective than performing a double blind 
randomised trial. 


Answer & Comments 

Answer: 2- 10% 

The relative risk reduction is 10% of 60% (ie 
without the treatment) which is 16%. The 
absolute risk reduction is 60 - 50% which is 
10 %. 


Dr. Khalid Yusuf El-Zohry - Sohag Teaching Hospital (01118391123) 

Ref MRCPass OE OE 2012 PasTest 2009 PassMedicine 2009 PasTest Exam ReviseMRCP 



696 





























El-zohry MRCP Questions Bank (Part 1) - 2013 


(For my personal use) 


[ Q: 1762 ] MRCPass - Statistics 

* A blood test for screening heart 
failure has been introduced. Out of 300 
patients, 100 were found to have heart failure 
with echocardiography. When the blood test 
is used, 80 patients were found to have heart 
failure. 70 of these patients had heart failure 
confirmed with echocardiography but 10 did 
not. 

Which is the positive predictive value of the 
blood test? 

1- 10/80 

2- 70/80 

3- 70/100 

4- 80/100 

5- 70/300 


Answer & Comments 

Answer: 2- 70/80 

The positive predictive value of the number of 
true positives (70) out of the positives (80) 
detected by blood test. In this case, sensitivity 
would be the number of positives detected by 
the blood test (70) out of the true number of 
positives (100). 



Echo shows 

HF 

Echo does 
not show HF 

Blood test 
positive 

70 

10 

Blood test 
negative 

30 

190 


Predictive Value Positive 

-► 


Disease 


Sensitivity 




Yes 

No | 


Pos 

TP 

FP 


Test 



v 

Neg 

FN 

TN 


Specificity 


4 - 

Predictive Value Negative 


^ [ Q: 1763 ] MRCPass - Statistics 

n - 

# A professor wants to use the 
students t test to analyse data from a 
population. 

Which of these features would make the test 
inapplicable? 

1- Normal distribution 

2- Parametric distribution 

3- Large standard error 

4- Large standard deviation 

5- Skewed distribution 


Answer & Comments 

Answer: 5- Skewed distribution 

Parametric tests make the assumption that 
the data are normally distributed. The 
students t test is parametric, hence it assumes 
normal distribution, and cannot be applied if 
the distribution is not normal or skew ed. 



[ Q: 1764 ] MRCPass - Statistics 

A study of implantable cardiac 
defibrillators (ICD) against 'no treatment' 
reports that there is 5 year mortality due to 
ventricular arrhythmias of 2% in the ICD group 
compared to 4% in the 'no treatment' group. 
There was a 50% reduction in cardiac deaths 
on treatment. 


What does 50% represent? 

1- Absolute risk reduction 


2- Relative risk reduction 


3- Control event rate 

4- Experimental event rate 

5- Mortality rate 


Answer & Comments 

Answer: 2- Relative risk reduction 

The relative risk reduction in this case is 2/4 
which can be expressed as 50% or 0.5. The 


Dr. Khalid Yusuf El-Zohry - Sohag Teaching Hospital (01118391123) 

Ref MRCPass OE OE 2012 PasTest 2009 PassMedicine 2009 PasTest Exam 


ReviseMRCP 


































El-zohry MRCP Questions Bank (Part 1) - 2013 


(For my personal use) 


absolute risk reduction is 2%. The control 
event rate is 4% and experimental event rate 
is 2%. 


[ Q: 1765 ] MRCPass - Statistics 

Which of the following describes a 
type 1 statistical error in a clinical trial ? 

1- The null hypothesis is falsely accepted 

2- The null hypothesis is falsely rejected 

3- A significant value was not achieved 

4- Patients were not randomised 

5- The hypothesis was not true 



Answer & Comments 

Answer: 2- The null hypothesis is falsely 
rejected 

The null hypothesis claims that there is no 
difference between two treatments. A type 1 
error is defined as the null hypothesis being 
falsely rejected. In practice this means that 
the study claims to find a difference that does 
not really exist. 


hypothesis testing Reality 

OUTCOMES 



The NuB Hypothesis 

Is True 

The Alternative 
Hypothesis is True 


Accurate 

Type II Error 


1 - (1 

P 

The Null Hypothesis 

Is True 

© 

© 


Type 1 Error 

Accurate 

The Alternative 

(X 

1*0 

Hypothesis is True 

©© 

© 


[ Q: 1766 ] MRCPass - Statistics 

A study investigates whether a 
certain drug A is better alone with the 
addition of drug B for ulcerative colitis. 

After randomizing the patients , a few patients 
on both drug A+B drop out due to side effects. 
How should the data be analysed? 

1- Exclude the patients from statistical 
analysis 



2- Assume that the patients did not drop out 

3- Include these patient outcomes in the drug 
A+B group 

4- Recruit more patients in the A+B group 

5- Analyse the two groups separately 


Answer & Comments 

Answer: 3- Include these patient outcomes in 
the drug A+B group 

The analysis of patients dropping out of a 
study should take into account the adverse 
effects of a drug (in this case drug B) causing 
drop out. Hence even though patients have 
dropped out, the 'intention to treat' principle 
requires the patients to be analysed even if 
they did not receive the treatment. 

[ Q: 1767 ] MRCPass - Statistics 

* A researcher is trying to design a 
study to find out the cause of a rare tropical 
disease among Pima Indians in South America. 
It is not known whether this disease is caused 
by an infection or whether it is inherited. 

What study design is most appropriate? 

1- Cross sectional 

2- Case control 

3- Prospective cohort 

4- Double blind placebo 

5- Randomised controlled 


Answer & Comments 

Answer: 2- Case control 

With rare diseases and exposures, case 
control studies are the best option. Although 
cohort studies are good for rare exposures, 
they are not good for rare diseases. 

[ Q: 1768 ] MRCPass - Statistics 

A new drug has been shown to 
reduce mortality from ovarian cancer from 



Dr. Khalid Yusuf El-Zohry - Sohag Teaching Hospital (01118391123) 

Ref MRCPass OE OE 2012 PasTest 2009 PassMedicine 2009 PasTest Exam ReviseMRCP 



698 





































El-zohry MRCP Questions Bank (Part 1) - 2013 


(For my personal use) 


20% to 10% in 10 years when compared to the 
current chemotherapy used. The cost of this 
new drug is £100 per year. 

How much extra would a hospital need to 
spend to prevent one death in 10 years? 

1 - £ 20,000 

2 - £ 10,000 

3- £2000 

4- £1000 

5- £100 


Which of these conditions would ensure 
adequate randomisation? 

1- Selection from a tertiary hospital 

2- A sample of healthy volunteers 

3- Only patients who agree to the study 

4- Relatives of a family with the disease 

5- Stratified random sample 

Answer & Comments 

Answer 5- Stratified random sample 


Answer & Comments 

Answer: 2- £10,000 

The absolute risk reduction is 10%. The 
number needed to treat is 100/10 which is 10 
patients to prevent one death in 10 years. 10 
patients in 10 years would cost £100 x 10 
patients x 10 years which is £10,000. 


[ Q: 1769 ] MRCPass - Statistics 

A proposal is being made for a study 
to assess the effect of stress on coronary 
artery thrombosis. 

Which of the following is the best study? 

1- Cross over study 

2- Case control study 

3- Randomised trial 

4- Cohort study 

5- Population study 



Answer & Comments 

Answer: 2- Case control study 

In assessing for common risk factors, case 
control studies are better than cohort studies. 


r^- 

) 

disease, 

patients. 



[ Q: 1770 ] MRCPass - Statistics 

A study of a drug on a certain 
is in the first stage of selecting 


In order to study a certain disease a certain 
group needs to be identified. In a stratified 
random sample, groups of interest are 
identified first, then randomisation occurs 
within those groups need to be performed. An 
example of this is in a randomised controlled 
trial, is to stratify the groups into separate age 
groups before randomising the patients. This 
allows the study to see whether the results 
are different according to the ages. 



[ Q: 1771 ] MRCPass - Statistics 

A blood test is being evaluated for 
measuring the likelihood of heart failure (HF), 
as compared to echocardiography as a gold 
standard. In the study, the following results 
are obtained. 



Blood test positive 

negative 

Echo - has HF 

720 

10 

Echo - no HF 

30 

890 


What is the negative predictive value of the 
blood test? 


1- 30/750 

2- 30/750 

3- 890/900 

4- 890/10 

5- 890/30 


Answer & Comments 


Answer: 3- 890/900 


Dr. Khalid Yusuf El-Zohry - Sohag Teaching Hospital (01118391123) 

Ref MRCPass OE OE 2012 PasTest 2009 PassMedicine 2009 PasTest Exam 


ReviseMRCP 































El-zohry MRCP Questions Bank (Part 1) - 2013 


(For my personal use) 


Negative predictive value is the number of 
true negatives found by a negative blood test, 
in this case, 890 / (890 + 10). 

[ Q: 1772 ] MRCPass - Statistics 

Cholesterol tests were performed on 
200 patients with either type I or type II 
diabetes. 

Which method of analysis would best identify 
whether there was a difference in the two 
groups of patients? 

1- Chi square test 

2- Paired students t test 

3- Correlation coefficient 

4- Mann U Whitney test 

5- Kaplan Meier curves 



Answer & Comments 

Answer: 2- Paired students t test 

The paired students t test can be used to 
compare two groups of patients with 
parametric data (Null hypothesis being that 
any difference is due to chance). Parametric 
means that it meets certain requirements 
with respect to parameters of the population 
(for example, the data will be normal - the 
distribution parallels the normal or bell curve). 
In addition, it means that numbers can be 
added, subtracted, multiplied, and divided. 

[ Q: 1773 ] MRCPass - Statistics 

Which of the following correctly 
describes a 95% confidence interval? 

1- There is a 95% chance that the value is 
statistically significant 

2- There is a 95% chance that the true value 
falls within the confidence interval 

3- There is a 95% chance that the p value is 
<0.05 

4- There is a 95% chance that the standard 
error is correct 



5- There is a 95% chance of variability 


Answer & Comments 

Answer: 2- There is a 95% chance that the true 
value falls within the confidence interval 

The 95% Cl can be used to describe the 
chance that a value, or difference falls within 
that interval. 


[ Q: 1774 ] MRCPass - Statistics 

A study is performed to see if body 
surface area (BSA) is related to drug 
concentration. 50 subjects were injected with 
the same concentration of drug. They also had 
their BSA measured. 

Which is the most appropriate test to assess 
for a correlation ? 

1- Pearson's correlation 

2- Logistic regresssion 

3- Chi square test 

4- Unpaired t test 

5- Paired t test 



Answer & Comments 

Answer: 2- Logistic regresssion 

Logistic regression allows one to predict a 
discrete outcome, such as group membership, 
from a set of variables that may be 
continuous, discrete, dichotomous, or a mix of 
any of these. This would allow us to determine 
whether one variable is dependent on 
another, in this case whether drug 
concentration was dependent on BSA. ANOVA 
is an example of logistic regression analysis. 

[ Q: 1775 ] MRCPass - Statistics 

A large study reports that the 10 
year risk of mortality due to stroke is 10% on 
placebo and 5% on a drug called Frenzy. It was 
concluded that there was a 5% reduction in 



Dr. Khalid Yusuf El-Zohry - Sohag Teaching Hospital (01118391123) 

Ref MRCPass OE OE 2012 PasTest 2009 PassMedicine 2009 PasTest Exam ReviseMRCP 





700 





























El-zohry MRCP Questions Bank (Part 1) - 2013 


(For my personal use) 


mortality from stroke when Frenzy was 
prescribed. 

Which of these describes the value of 5%? 

1- Significance value 

2- Mortality reduction 

3- Relative risk reduction 

4- Absolute risk reduction 

5- Number needed to treat 

Answer & Comments 

Answer: 4- Absolute risk reduction 

In this example, the drug reduces the 
incidence of stroke from 10% to 5%. 

The relative risk reduction (RRR) is 50%. 

The absolute risk reduction (ARR) is 5%. 

The control event rate (CER) is 10% and the 
experimental event rate (EER) is 5%. 

The number needed to treat (NNT) is 
100%/5% = 20. 


5- Include these patient outcomes in the drug 
plus physiotherapy group 

Answer & Comments 

Answer: 5- Include these patient outcomes in 
the drug plus physiotherapy group 

This is the principle of 'intention to treat'. It is 
possible that the physiotherapy intervention 
was unpleasant and hence patients dropped 
out. 

Intention to treat helps to reduce bias by 
including the data from original allocation of 
treatment, analyzing the patient in the 
treatment group even if they did not complete 
the trial. 



[ Q: 1776 ] MRCPass - Statistics 

A study is designed to investigate 
whether a certain drug plus physiotherapy 
treatment is better than drug treatment alone 
in the management of juvenile chronic 
arthritis. 


After randomizing the patients, a small 
proportion of patients in the physiotherapy 
group decide to drop out of the study. 

What is the correct way of analysing the 
subsequent data? 

1- Record the patients as having successful 
therapy 

2- Record the patients as having failed therapy 

3- Extend the trial recruitment to make up 
numbers 


4- Exclude these patients from all analysis 


Dr. Khalid Yusuf El-Zohry - Sohag Teaching Hospital (01118391123) 

Ref MRCPass OE OE 2012 PasTest 2009 PassMedicine 2009 PasTest Exam 


ReviseMRCP 























El-zohry MRCP Questions Bank (Port 1) - 2013 


(For my personal use) 


[ Q: 1777 ] MRCPass - 2010 January 

A 35-year-old man who is known to 
have bronchiectasis has a chronic cough. A 
sputum sample is sent from the clinic. 

What organism is most likely to be isolated? 

1- Streptococcus pneumoniae 

2- Klebsiella spp. 

3- Haemophilus influenzae 

4- Moraxella 

5- Pseudomonas aeruginosa 



Answer & Comments 


Answer: 3- Haemophilus influenzae 


enzymatically converted to 5-fluorouracil in 
the tumor, where it inhibits DN synthesis and 
slows growth of tumor tissue. Pancytopenia, 
diarrhoea and hand-foot syndrome are main 
side effects. 



[ Q: 1779 ] MRCPass - 2010 January 

Which one of the following is a 
contraindication to liver biopsy? 


1- INRof 1.4 

2- ALT of 250 u/l 

3- Platelet count of 110 x 10 9 /l 

4- Obesity with BMI of 35 kg/m * 1 2 3 4 5 

5- Biliary duct dilatation on the ultrasound 


The organisms found most typically causing 
ongoing infection in bronchiectasis include 
Haemophilus species (47-55% of patients) and 
Pseudomonas species (18-26% of patients) 


Answer & Comments 

Answer: 5- Biliary duct dilatation on the 
ultrasound 


[ Q: 1778 ] MRCPass - 2010 January 

A patient with colorectal cancer has 
been commenced on a chemotherapy regime 
with Capecitabine and oxaliplatin following 
surgery. 

What is the main difference between 5 FU and 
capecitabine? 

1- Capecitabine is used orally 

2- Capecitabine has a broader indication 

3- Capecitabine causes peripheral neuropathy 

4- 5 FU has less side effects 

5- 5 FU interacts with warfarin 


The best answer here is biliary duct dilatation, 
which increases the risk of infection as there 
might be cholestasis or cholecystitis. 

Many would consider obesity, but it is not an 
absolute contraindication. 

A short list of contraindications to liver biopsy 
are: 

Prolonged (>1.6) international normalized 
ratio (INR) 

The platelet count should exceed 60xl0 A 9/l 

There should be no biliary dilatation or major 
ascites 


Answer & Comments 

Answer: 1- Capecitabine is used orally 

Capecitabine is the oral equivalent of 
intravenous 5 -FU. 


Bleeding diathesis (eg, hemophilia) 



[ Q: 1780 ] MRCPass - 2010 January 

Which is the most common site for 
primary cardiac tumours to occur in adults? 


Capecitabine (Xeloda, Roche) is an orally- 
administered chemotherapeutic agent used in 
the treatment of metastatic breast and 
colorectal cancers. It is a prodrug, that is 


1- Right ventricle 

2- Right atrium 

3- Left atrial appendage 


Dr. Khalid Yusuf El-Zohry - Sohag Teaching Hospital (01118391123) 

Ref MRCPass OE OE 2012 PasTest 2009 PassMedicine 2009 PasTest Exam 




ReviseMRCP 

703 






























El-zohry MRCP Questions Bank (Port 1) - 2013 


(For my personal use) 


4- Left atrium 

5- Left ventricle 

Answer & Comments 

Answer: 4- Left atrium 

About 75% of atrial myxomas are in the left 
atrial. 

The most common site is the fossa ovalis 
border in the left atrium. 


rash had been present for the last lyear and 
had gradually become more extensive. He is 
otherwise well. 

Whot should be the treatment of choice? 

1- Ketoconazole cream 

2- Nystatin cream 

3- Metronidazole cream 

4- Oral terbinafine 

5- Oral itraconazole 


[ Q: 1781 ] MRCPass - 2010 January 

* A 45 year old man presents with 
weight loss and breathlessness. He has a past 
medical history of heavy alcohol intake and 
intravenous drug use. On examination, he has 
right sided signs of reduced air entry in the 
lung bases. 


Answer & Comments 

Answer: 1- Ketoconazole cream 

The features are suggestive of Pityriasis 
versicolor, a skin infection which often 
presents as patches of relatively depigmented 
skin. 


A CXR shows pleural effusion on the right side. 
Chest aspiration was attempted by the senior 
house officer but this was unsuccessful. 

What is the next investigation? 

1- Lateral Chest X Ray 

2- Bronchoscopy 

3- CT of the chest 


The cause is overgrowth of the yeast 
Malassezzia furfur. 

Topical antifungal medications are the 
treatment of choice for Tinea/ Pityriasis 
versicolor. The following topical antifungal 
treatment regimens have been shown to 
produce a greater than 70% clinical response 
rate: 


4- Lung function test 

5- Ultrasound of the chest 


Answer & Comments 

Answer: 5- Ultrasound of the chest 

This patient has a failed aspiration of the 
chest. 

However, he is at high risk of empyema and so 
priority is still to get an aspirate sample 
through ultrasound guidance. 


^ [Q: 1782] MRCPass-2010 January 

* A 25 year-old man presents with 
extensive, coalescing, hypopigmented, slightly 
scaly lesions on his face and the scalp. The 


2% Ketoconazole cream and ketoconazole 
shampoo 

Selenium sulphide suspension (Selsun 
shampoo) 


[ Q: 1783 ] MRCPass - 2010 January 

A 31-year-old man who is known to 
be HIV positive presents with dyspnoea and a 
dry cough. Is last CD4 count was 150 cells/pl. 
Clinical examination reveals a respiratory rate 
of 24 / min. Chest auscultation reveals fine 
crackles bilaterally. His oxygen saturation is 
98% on room air but this desaturates to 85% 
after walking the length of the ward. A chest x 
ray shows fine infiltrates in both lower zones. 

What is the most appropriate first-line 
treatment? 



Dr. Khalid Yusuf El-Zohry - Sohag Teaching Hospital (01118391123) 

Ref MRCPass OE OE 2012 PasTest 2009 PassMedicine 2009 PasTest Exam ReviseMRCP 



704 




























El-zohry MRCP Questions Bank (Port 1) - 2013 


(For my personal use) 


1- Fluconazole 

2- Co-trimoxazole 

3- Clarithromycin 

4- Ganciclovir 

5- Sulfadiazine and pyrimethamine 


Answer & Comments 

Answer: 2- Co-trimoxazole 

Pneumocystis pneumonia (PCP) is caused by 
the yeast-like fungus previously classified as a 
protozoan Pneumocystis jirovecii. 

The risk of pneumonia due to Pneumocystis 
jirovecii increases when CD4 counts are 
lessthan 200 cells/?l. 



There is increased opacification in the lower 
lungs on both sides, characteristic of 
Pneumocystis pneumonia. 


Antipneumocystic medication (classically co- 
trimoxazole) is used with concomitant steroids 
in order to avoid inflammation. 



was 


[ Q: 1784 ] MRCPass - 2010 January 


A 55-year-old, unemployed, man 
admitted due to confusion and 


disorientation with the onset of acute 
symptoms a few days before admission. He 
had a history of alcohol dependence of 20 
years duration. He was aw ake but 
disorientated to person, location, situation, 
and time. 

Temperature was 36.5 C and BP was 120/80 
mmHg. Neurological examination revealed 
gaze-evoked nystagmus in all directions. All 
deep tendon reflexes were normal. The finger- 
nose test was ataxic. Standing and gait with 
open eyes evidenced a distinct ataxia with 
tremors of the upper extremity. The Romberg 
sign was positive. He had an MMSE score of 
26 /30 on admission. A blood glucose 
measurement was 3 mmol/I. 

Whot should be given to treat the patient? 

1- 50 ml of 50% dextrose 

2- 500 mis of 5% dextrose 

3- Thiamine infusion 

4- Lactulose 

5- Chlordiazepoxide 

Answer & Comments 

Answer: 3- Thiamine infusion 

This patient is exhibiting features of 
Wernicke's encephalopathy, which are 
nystagmus, ataxia and confusion. 

The daily thiamine requirement for healthy 
individuals is between 1 and 2 mg/day but 
both alcohol and malnutrition may interfere 
with the absorption of thiamine. Thiamine 
dependent enzymes such as transketolase and 
pyruvate dehydrogenase are essential for 
cerebral myelination and thiamine deficiency 
in alcoholism leads to Wernicke's 
encephalopathy. 

Thiamine should also be supplemented fully in 
malnourished alcoholics. The administration 
of intravenous fluids containing glucose 
without adequate thiamine supplementation 
in alcoholics could aggravate the thiamine 



Dr. Khalid Yusuf El-Zohry - Sohag Teaching Hospital (01118391123) 

Ref MRCPass OE OE 2012 PasTest 2009 PassMedicine 2009 PasTest Exam ReviseMRCP 






























El-zohry MRCP Questions Bank (Port 1) - 2013 


(For my personal use) 


deficiency leading to irreversible cerebral 
lesions. 


What is the most appropriate course of 
action ? 



[ Q: 1785 ] MRCPass - 2010 January 

A 43 year old man had a cough and 
his GP organized a chest X ray. This showed a 
pneumothorax and he was referred to the 
hospital. He does not have any symptoms of 
breathlessness or chest pains. His BP was 
110/80 mmHg, 02 sats were 99% on air & 
pulse was 80/min. CXR shows that there is a 
pneumothorax on the left with a 1 cm margin 
from the rim. 


Which is the most appropriate step? 

1- Chest drain 


2- Surgical referral 

3- CT scan of the chest 


4- Discharge with outpatient follow up 

5- needle aspiration 


Answer & Comments 

Answer: 4- Discharge with outpatient follow 
up 

The British thoracic society recommends that 
a small pneumothorax of < 2 cm rim, with no 
significant symptoms to be managed 
conservatively (discharge with outpatient 
review and advise to return if breathlessness 
occurs). 


[ Q: 1786 ] MRCPass - 2010 January 

A 60-year-old has developed an 
I hernia and is keen for surgical repair. 

The anaesthetist refers the patient for 
assessment in outpatients. His history includes 
a previous who had a drug-eluding stent 
inserted 6 months ago. His current medication 
includes aspirin, clopidogrel, atorvastatin, 
ramipril and bisoprolol. The cardiologists plan 
was to continue clopidogrel for 12 months 
following stent insertion. 



inguina 


1- Stop clopidogrel the day before the 
operation 

2- Stop clopidogrel 1 week before the 
operation 

3- Stop clopidogrel the day prior to the 
operation andstart low-molecular weight 
heparin (prophylaxis dose) 

4- Continue clopidogrel throughout the 
perioperative period 

5- Delay surgery for 6 months 


Answer & Comments 

Answer: 5- Delay surgery for 6 months 

There is a risk of stent thrombosis with 
discontinuing clopidogrel. 

For non urgent surgery this should be delayed 
till a 12 month period on clopidogrel has 
elapsed. If the patient required urgent surgery 
e.g. acute abdomen or incarcerated hernia, 
then the clopidogrel would have to be 
discontinued prior to surgery, with a 
discussion between surgeons and cardiologists 
weighing risks of bleeding during surgery vs 
stent thrombosis. 



[ Q: 1787 ] MRCPass - 2010 January 

A 70 year old lady presents with hip 
pain on the right side. She has a history of 
hypertension. On examination, she is able to 
mobilise and has normal flexion and extension 
of movement of her hip. She is However, 
tender to the palpation in the right lateral hip. 


What is the likely diagnosis? 

1- Osteoarthritis 


2- Ankylosing spondylitis 

3- Rheumatoid arthritis 


4- Trochanteric bursitis 


5- Fracture of neck of femur 


Dr. Khalid Yusuf El-Zohry - Sohag Teaching Hospital (01118391123) 

Ref MRCPass OE OE 2012 PasTest 2009 PassMedicine 2009 PasTest Exam ReviseMRCP 




























El-zohry MRCP Questions Bank (Port 1) - 2013 


(For my personal use) 


Answer & Comments 


Answer: 4- Trochanteric bursitis 


but was not a given option for this question in 
the exam. 


Trochanteric bursitis is characterized by 
painful inflammation of the bursa located just 
superficial to the greater trochanter of the 
femur. 

Patients typically complain of lateral hip pain, 
although the hip joint itself is not involved. 
The pain may radiate down the lateral aspect 
of the thigh. It may occur with trauma. Rest 
and physiotherapy are best management 
options, although steroid injection is an 
option. 



[ Q: 1788 ] MRCPass - 2010 January 

A 55-year-old woman is investigated 
for progressive shortness of breath. On 
examination a loud P2 is noted associated 
with a left parasternal heave. An ECG shows 
evidence of right ventricular strain and a 
diagnosis of pulmonary hypertension was 
suspected. 


Which one of the following is the most 
important test to confirm the diagnosis? 

1- High resolution CT thorax 

2- Chest X Ray 

3- Echocardiography 

4- Pulmonary angiography 

5- Ventilation perfusion scanning 


Answer & Comments 


Answer: 3- Echocardiography 



[ Q: 1789 ] MRCPass - 2010 January 

A 40-year-old man who was 
diagnosed with type 2 diabetes mellitus 
presents for review in the clinic. His current 
medication is metformin and gliclazide. His 
blood results were as follows: 


Total cholesterol 5.8 mmol/l 
HDL cholesterol 1.2 mmol/l 
LDL cholesterol 3.5 mmol/l 
Triglyceride 1.7 mmol/l 
HbAlc 6.6% 


According to recent NICE guidelines, what is 
the most appropriate action? 

1- Atorvastatin 40mg nocte 

2- Simvastatin 40mg nocte 

3- Dietary advice 

4- Clofibrate 


5- Nicotinic acid 


Answer & Comments 

Answer: 2- Simvastatin 40mg nocte 

NICE guidelines suggest target lipid levels of 
4:2 in diabetic patients (ie. Total cholesterol of 
< 4 and LDL of < 2). The guidelines also 
recommend the most cost effective statins to 
be commenced first (simvastatin £4 per 
month compared to £40 per month for 
atorvastatin until it comes off patent which it 
does in 2010). 


The tests are all useful, However the best 
option is echocardiography, which can provide 
an estimate of pulmonary arterial pressure, 
identify right ventricular strain and also 
exclude any congenital heart disease. 

Right heart cardiac catheterisation is most 
accurate as accurate pulmonary arterial 
pressures can be measured with a transducer, 


[ Q: 1790 ] MRCPass - 2010 January 

You are reviewing a patient's urea 
and electrolyte results. There appears to be a 
discrepancy between the serum creatinine 
and the calculated eGFR. 

Which one of the following factors is most 
likely to explain this discrepancy? 



Dr. Khalid Yusuf El-Zohry - Sohag Teaching Hospital (01118391123) 

Ref MRCPass OE OE 2012 PasTest 2009 PassMedicine 2009 PasTest Exam 




ReviseMRCP 

707 






























El-zohry MRCP Questions Bank (Port 1) - 2013 


(For my personal use) 


1- Diuretic use 

2- Pregnancy 

3- Type 1 diabetes mellitus 

4- Significant hypertension 

5- Female gender 


Answer & Comments 


Answer: 2- Pregnancy 


with positive direct sputum smears should be 
isolated in negative pressure rooms. 

[ Q: 1792 ] MRCPass - 2010 January 

On the ECG, which one of the 
following features is seen with significant 
Hypokalaemia? 

1- Flattened p wave 

2- Flattened t wave 



When a person's creatinine is stable, an 
estimated Glomerular filtration rate can be 
obtained with inputs of creatinine, age, 
gender and racial origin. 


3- Prominent u wave 

4- Prominent j wave 

5- Prominent r wave 


The eGFR estimate may be inaccurate in 
people over 70 years of age, people less than 
18 years old, pregnancy, amputees, 
malnourishment and dehydration states. 


[ Q: 1791 ] MRCPass - 2010 January 

A 35 year old man is admitted with 
fevers, cough and night sweats. 

Which one of the following test results 
suggests that he needs isolation into a side 
room in the hospital? 

1- Positive sputum culture for TB 

2- Positive sputum direct smear for TB 

3- Positive CSF culture for TB 

4- Positive urine culture for TB 

5- Positive urine direct smear for TB 



Answer & Comments 


Answer: 2- Positive sputum direct smear for 
TB 


Answer & Comments 

Answer: 3- Prominent u wave 

The prominent U wave is a component seen 
after a T wave seen in severe Hypokalaemia. 




i—-r 


Y 


n 




i 11 

ft If HP 


flVJ 


U If 


1 

V 


-Hi-1/—1r 


fTTTtYTTTf 




J J 

"Tir 1 


l | 


rH^TTHHri 


lnr—|p-f~v 


HrHr 

T 7 


U waves 


[ Q: 1793 ] MRCPass - 2010 January 

A 40 year old woman has been 
complaining of polyuria and polydipsia and 
presents to the clinic for assessment. 

Her body mass index (BMI) was 42 kg/m2. 



Stained smears of sputum specimens to detect 
the presence of acid fast bacilli (AFB) are 
useful diagnostic tools in the management of 
tuberculosis. 

Patients with tuberculosis who have negative 
sputum smears for AFB are less contagious 
than patients with positive smears. Patients 


A 75g oral glucose tolerance test shows the 
following results 

Time Plasma glucose concentration 
0 hour 5.5 mmol/I 
2 hour 15 mmol/l 

Which of the following is the appropriate next 
step in the patient’s management? 


Dr. Khalid Yusuf El-Zohry - Sohag Teaching Hospital (01118391123) 

Ref MRCPass OE OE 2012 PasTest 2009 PassMedicine 2009 PasTest Exam 




ReviseMRCP 

708 































El-zohry MRCP Questions Bank (Port 1) - 2013 


(For my personal use) 


1- Fasting glucose tolerance test 

2- Gliclazide 

3- Subcutaneous insulin 

4- Diet control 

5- Metformin 


Answer & Comments 

Answer: 5- Metformin 

Metformin is a biguanide drug used in 
diabetes. 

It improves hyperglycemia primarily through 
its suppression of hepatic glucose production 
and increases insulin sensitivity. This patient 
has type 2 diabetes (late onset) is overw eight. 
Metformin is the first-line drug of choice for 
the treatment of type 2 diabetes, particularly 
in overweight and obese people and those 
with normal kidney function. 



[ Q: 1794 ] MRCPass - 2010 January 

A 50 year old man with Type 2 
diabetes is currently on metformin 500mg tds. 
He is a muslim and prior to Ramadan, he seeks 
advice about managing his glycaemic control. 
He is concerned that he already has early 
diabetic retinopathy. 


What should be done? 


by increased dosage in the evening to combat 
hyperglycaemia and reduced dosage in early 
morning to prevent hypoglycaemia. 

Hence the best answer is for the patient to 
increase the metformin dose in the evening to 
1 g and carry on with a morning dose of 500 
mg in the morning. 


# 


[ Q: 1795 ] MRCPass - 2010 January 


A 62 year old woman has recently 
had lethargy and arthralgia. She was 
diagnosed as having influenza infection, as 
there was an outbreak in the area recently. 
She presents 1 week later with a cough and 
breathlessness. On examination, she had 
bilateral crackles audible on examination. CXR 
confirms bilateral consolidation. 


Which one of the following is most likely os o 
couse? 


1- Legionella 

2- Mycoplasma 

3- Streptococcus pneumoniae 

4- Klebsiella 


5- staphylococcus aureus 


Answer & Comments 


Answer: 5- staphylococcus aureus 


1- Advise the patient not to fast 

2- Change metformin dose to 500mg am and 
lg in the evening 

3- Change metformin to gliclazide 160 mg am 
and 80 mg in the evening 

4- Change to insulin 12 units tds 

5- Change to glargine insulin 22 units at night 


Answer & Comments 

Answer: 2- Change metformin dose to 500mg 
am and lg in the evening 

Most diabetics can safely fast but more 
medical attention to adjust their medication 


Normal incidence of staph aureus pneumonia 
is 2%, However this is significantly increased in 
iv drug users and influenzae virus infections. 

Post influenzae staph aureus pneumonia is 
characterised by rapid clinical deterioration 
with septicaemia. 


ti 


j 


[ Q: 1796 ] MRCPass - 2010 January 
A 75-year-old man on the surgical 


wards develops chest pain. He is three days 
post-op following a colectomy for colocolon 
cancer (Duke's C). He is currently on a 
prophylactic low molecular weight heparin. An 
ECG performed by the nurses shows ST 


Dr. Kholid Yusuf El-Zohry - Sohog Teaching Hospital (01118391123) 

Ref MRCPass OE OE 2012 PasTest 2009 PassMedicine 2009 PasTest Exam ReviseMRCP 































El-zohry MRCP Questions Bank (Port 1) - 2013 


(For my personal use) 


elevation in the anterior leads. Aspirin and 
oxygen have been given. 

Whot is the most appropriate treatment now? 

1- Increase low-molecular weight heparin 

2- Start unfractionated heparin 

3- Arrange for primary angioplasty 

4- arrange echocardiogram urgently to 
exclude pericardial tamponade 

5- Thrombolysis with tenecteplase 


Answer & Comments 

Answer: 2- Imatinib 

The diagnosis here is chronic myeloid 
leukaemia, which accounts for 20% of all 
leukaemias. 

It occurs mainly in middle aged and elderly 
people and is characterised by marked 
leucocytosis, a left shifted myeloid series and 
in 95% of patients, the Philadelphia 
chromosome. 


Answer & Comments 

Answer: 3- Arrange for primary angioplasty 

There is no contraindication to acute coronary 
intervention in this case, as the prognosis of a 
patient with colorectal cancer post surgery is 
good. 

Hence, primary PCI is the preferred option. 



[ Q: 1797 ] MRCPass - 2010 January 

A 60-year-old woman is investigated 
for weight loss, fatigue and anaemia. She has 
no past medical history of note. Clinical 
examination reveals splenomegaly associated 
with pallor. A full blood count is reported as 
follows: 


Hb 9.8 g/dl 
Platelets 380 x 10 9 /l 
WCC 120 x 10 9 /l 

Blood film. Demonstrates left shift with 
predominating myelocytes. Low percentage of 
blast cells. 


What is the most appropriate treatment? 

1- Chlorambucil 

2- Imatinib 


3- Thalidomide 

4- Rituximab 


Imatinib is recommended as first-line 
treatment for people with Philadelphia- 
chromosome-positive chronic myeloid 
leukaemia (CML) in the chronic phase With 
disease progression and palliative situations, 
Imatinib is used in combination with 
recombinant alpha interferon, hydroxyurea 
and busulphan. 


[ Q: 1798 ] MRCPass - 2010 January 

A 50-year-old woman presents with 
a variety of physical symptoms that have been 
present for the past 8 years, following her 
mother's death. She has been complaining of 
tremors, sensory disturbances and fits 
occurring several times a day. Numerous 
investigations and review by a variety of 
specialties have indicated no organic basis for 
her symptoms. 

What is the diagnosis? 

1- Munchausen's syndrome 

2- Hypochondrial disorder 

3- Dissociative disorder 

4- Somatisation disorder 

5- Conversion disorder 



Answer & Comments 


Answer: 5- Conversion disorder 


5- Hydroxycarbamide 


The history fits a diagnosis of conversion 
disorder as below. 


Dr. Khalid Yusuf El-Zohry - Sohag Teaching Hospital (01118391123) 

Ref MRCPass OE OE 2012 PasTest 2009 PassMedicine 2009 PasTest Exam ReviseMRCP 



710 


























El-zohry MRCP Questions Bank (Port 1) - 2013 


(For my personal use) 


Munchausen syndrome (factitious disorder): 
the patient seeks medical attention by the 
deliberate production or feigning of 
symptoms. 

The motivation for seeking attention is not 
known. 

Hypochondriasis: (somatoform disorder) the 
patient is convinced that they have a life- 
threatening illness, despite evidence to the 
contrary. The core feature of hypochondriasis 
is not preoccupation with symptoms 
themselves, but rather the fear or idea of 
having a serious disease. The fear or idea is 
based on the misinterpretation of bodily signs 
and sensations as evidence of disease. 


4- Losartan 

5- Nifedipine 

Answer & Comments 

Answer: 2- Methyldopa 

Beta blockers are safe in third trimester of 
pregnancy, but are generally not used due 
fears of IUGR. 

Generally, one would favour labetalol in these 
circumstances given there an evidence base 
its use. Nifedipine may be used by 
experienced clinicians, but currently 
unlicensed. There is good evidence that 
methyldopa effective and safe in pregnancy. 


Somatisation disorder:(somatoform disorder) 
With this a patient presents with multiple, 
medically unexplained symptoms. The 
patient's life or work are frequently affected, 
although they also might be unconcerned 
about the nature of their symptoms (thus 
appearing calm). It is not a deliberate feigning 
of symptoms. 

Conversion disorder : (somatoform 

disorder)This is a condition where a patient 
displays neurological symptoms e.g. paralysis, 
even though no neurological explanation is 
found and it is determined that the symptoms 
are due to the patient's psychological 
response to stress. 


[ Q: 1800 ] MRCPass - 2010 January 

A right- handed woman presented 
with difficulty reading. She was investigated 
with a CT brain which showed right sided 
parietal lobe infarction. 

Which of the following is likely to be 
contributing to her reading difficulty? 

1- Agraphia 

2- Acalculia 

3- Left right disorientation 

4- Visual inattention 

5- Hemianopia 



^ [ Q: 1799 ] MRCPass - 2010 January 

fit - 

# A woman who is 33 weeks pregnant 
is admitted to the obstetric ward. She has 
been monitored for the past few weeks due to 
pregnancy-induced hypertension but has now 
developed proteinuria. Her blood pressure is 
160/95 mmHg. 

Which one of the following ontihypertensives 
is most appropriate? 

1- Atenolol 

2- Methyldopa 

3- Moxonidine 


Answer & Comments 

Answer: 4- Visual inattention 

This patient is right handed, hence the left 
brain is dominant, and the right parietal lobe 
infarct is in the non dominant hemisphere. 

Tests for dominant inferior parietal lobe 
function includes right-left orientation, 
naming fingers, and calculations and 
Gerstmann syndrome describes dominant 
lobe signs. A mnemonic for the signs is ALF 
(acalculia / agraphia, left right disorientation 
and finger agnosia). 


Dr. Khalid Yusuf El-Zohry - Sohag Teaching Hospital (01118391123) 

Ref MRCPass OE OE 2012 PasTest 2009 PassMedicine 2009 PasTest Exam 




ReviseMRCP 

711 






















El-zohry MRCP Questions Bank (Port 1) - 2013 


(For my personal use) 


The non-dominant parietal lobe is important 
for visual spatial sensory tasks such as 
attending to the contralateral side of the body 
and space as well as constructional tasks such 
as drawing a face, clock or geometric figures. 
A non dominant lesion leads to visual 
inattention and dyspraxia (unable to 
coordinate motor tasks). 


[ Q: 1801 ] MRCPass - 2010 January 

A 65 year old lady has hypertension 
and is currently on with several medications. 
Over the last few months, she has been 
complaining of worsening ankle oedema and 
fatigue. 

Which drug is likely to hove coused this? 

1- Monoxidine 

2- Ramipril 

3- Amlodipine 

4- Doxazosin 

5- Indapamide 



Answer & Comments 

Answer: 3- Amlodipine 

Ankle oedema is commonly caused by calcium 
channel blockers. 


Answer & Comments 

Answer: 4- Right coronary artery 

The diagnosis is an inferior myocardial 
infarction with complete heart block, this is 
most commonly due to a RCA lesion. 



[ Q: 1803 ] MRCPass - 2010 January 

A 60-year-old man has been on 
haemodialysis for chronic kidney disease for 
the past 5 years. 


He is currently on dialysis 3 times a week. For 
a patient on haemodialysis, whot is the most 
likely eventual couse of death? 


1- Hyperkalaemia 


2- Dilated cardiomyopathy 


3- Dialysis related sepsis 

4- Ischaemic heart disease 


5- Renal carcinoma 


Answer & Comments 

Answer: 4- Ischaemic heart disease 

Cardiovascular disease, infection and 
hyperkalaemia are common causes of death, 
with ischaemic heart disease being the most 
likely. 



[ Q: 1802 ] MRCPass - 2010 January 

A 47 year old patient had chest pain. 

His ECG showed ST elevation in the leads II, III, 
aVF and he also had no correlation between p 
waves and QRS complexes. 

Which artery is likely to be affected? 

1- Left main stem 

2- Left anterior descending 

3- Circumflex 

4- Right coronary artery 

5- Diagonal 


The 10 year survival rate is 50%. 


[ Q: 1804 ] MRCPass - 2010 January 

/ - 

* A 30 year old lady has a history of 

ventricular septal defect. She wishes to 
become pregnant and is undergoing cardiac 
assessment. 

Which one of the following complications is 
associated with the highest risk? 

1- Polycythaemia 

2- High pressure gradient across VSD 

3- Pulmonary hypertension 

4- Aortic regurgitation 

5- Overriding aorta 


Dr. Khalid Yusuf El-Zohry - Sohag Teaching Hospital (01118391123) 

Ref MRCPass OE OE 2012 PasTest 2009 PassMedicine 2009 PasTest Exam ReviseMRCP 



712 
































El-zohry MRCP Questions Bank (Port 1) - 2013 


(For my personal use) 


Answer & Comments 

Answer: 3- Pulmonary hypertension 

Pulmonary hypertension with a VSD can lead 
to shunt reversal (Eisenmenger's syndrome). 

This will lead to significant hypoxia and can 
confer significant risk to oxygenation of the 
fetus during pregnancy. 


[ Q: 1805 ] MRCPass - 2010 January 

A 32-year-old female presents 
complaining of a purpuric rash on the back of 
her legs but is otherwise asymptomatic. She 
has no significant past medical history and has 
not been on any medications recently. A urine 
dipstick is normal. Her blood results are: 

Hb 11.3 g/dl 

Platelets 30 x 10 9 /l 

WCC 5.3xl0 9 /l 

PT 13 secs 

APTT 30 secs 

sodium 135 mmol/l 

potassium 4.5 mmol/l 

urea 5 mmol/l 

creatinine 100 pmol/l 

What is the most likely diagnosis? 

1- Drug-induced thrombocytopenia 

2- Henoch-Schonlein purpura 

3- Idiopathic thrombocytopenic purpura 

4- Thrombotic thrombocytopenic purpura 

5- Systemic lupus erythematosus 

Answer & Comments 

Answer: 3- Idiopathic thrombocytopenic 
purpura 

The two most likely answers are either HSP or 
ITP due to the purpuric rash. 

HSP is associated with other vasculitic 
phenomena such as renal involvement and 


abdominal vasculitis. It is also not associated 
with a low platelet count as in this case, hence 
ITP is the most likely diagnosis. 

^ [ Q: 1806 ] MRCPass - 2010 January 

* A 25 year old man presented with a 
purpuric rash. He has a history of asthma but 
otherwise was previously well. On 
examination, he had small areas of non 
blanching purpura around the arms and legs. 
Cardiac, respiratory, abdominal and 
neurological examinations were normal. The 
following results were obtained: 

Hbl2 g/dl 

WCC 8 x 10 9 /l 

Platelets 18 x 10 9 /l 

What treatment should be given? 

1- Blood transfusion 

2- Splenectomy 

3- Prednisolone 

4- Platelet transfusion 

5- Intravenous immunoglobulin 

Answer & Comments 

Answer: 3- Prednisolone 

This patient has idiopathic thrombocytopenic 
purpura. 

The underlying pathology is antibodies against 
platelets. 

Bleeding from the gums, epistaxis and 
purpuric lesions on the skin are typical 
manifestations. The platelet count is low . The 
patient should be treated with prednisolone 
(usually a high dose e.g. 60 mg) with gradual 
tapering down once the platelet count rises. If 
prednisolone does not bring a response, then 
intravenous immunoglobulin can be 
administered. Patients with recurrent 
episodes of low platelet counts despite 
steroids can be considered for splenectomy. 



Dr. Khalid Yusuf El-Zohry - Sohag Teaching Hospital (01118391123) 

Ref MRCPass OE OE 2012 PasTest 2009 PassMedicine 2009 PasTest Exam 




ReviseMRCP 

713 



























El-zohry MRCP Questions Bank (Port 1) - 2013 


(For my personal use) 


4^ [ Q: 1807 ] MRCPass - 2010 January 

* A 42-year-old man has recently 
started treatment for tuberculosis. He is 
complaining of a deterioration in his vision. On 
examination, his visual acuity in the right eye 
is 6/18 on the right and 6/12 on the left eye. 

Which one of the following drugs is most likely 
to be implicated? 

1- Rifampicin 

2- Isoniazid 

3- Pyrazinamide 

4- Ethambutol 

5- Streptomycin 


4- Anxiety disorder 

5- Obsessive compulsive disorder 

Answer & Comments 

Answer: 3- Drug induced psychosis 

It is well established that psychotic symptoms 
may follow cannabis intake. 

Patients who present with these symptoms 
may get better and be diagnosed with 
schizophrenia at a later stage. Patients can 
present with a range of symptoms including 
agitation, depression, visual and auditory 
hallucinations. 


Answer & Comments 

Answer: 4- Ethambutol 

The standard treatment for tuberculosis is a 6- 
month, four-drug initial regimen (6 months of 
isoniazid and rifampicin supplemented in the 
first 2 months with pyrazinamide and 
ethambutol). 

Ethambutol is associated with optic neuritis 
(visual deterioration) and colour blindness. 


[ Q: 1808 ] MRCPass - 2010 January 

A 16 year old student is behaving 
strangely and referred to the hospital. His 
teacher reports that he was accused the 
teacher of conspiring against him. He was also 
hearing voices asking him to cut his own 
throat. He has not been himself recently, with 
low moods according to his family. On 
examination, he looks apathetic and physical 
examination is normal. Blood tests were 
unremarkable, and urine testing showed 
traces of cannabinoids. 

Whot is the likely diagnosis? 

1- Psychotic depression 

2- Paranoid schizophrenia 

3- Drug induced psychosis 



^ [ Q: 1809 ] MRCPass - 2010 January 

/ - 

# A 22-year-old man presents with a 

painful joints. He returned 1 week ago from a 
holiday in Spain. Initially he had joint sw 
ellings in both wrists and now he has a left 
knee sw elling. There is no history of trauma 
and he has had no knee problems previously. 
On examination he has a swollen, warm left 
knee with a full range of movement. 

His ankle joints are also painful to move but 
there is no sw elling. There was also a rash on 
the soles of the feet with very small vesicles. A 
joint aspirate was done and it showed no 
organisms but had increased white cells. 

What is the most likely diagnosis? 

1- Rheumatoid arthritis 

2- Psoriatic arthritis 

3- Gout 

4- Reactive arthritis 

5- Gonococcal arthritis 


Answer & Comments 

Answer: 5- Gonococcal arthritis 

This patient with no past medical history is 
likely to have a reactive arthritis or septic 
arthritis due to gonococcal infection. 


Dr. Khalid Yusuf El-Zohry - Sohag Teaching Hospital (01118391123) 

Ref MRCPass OE OE 2012 PasTest 2009 PassMedicine 2009 PasTest Exam ReviseMRCP 





























El-zohry MRCP Questions Bank (Port 1) - 2013 


(For my personal use) 


The history above is more consistent with 
gonococcal arthritis with the bacteremic form 
(classic triad of migratory polyarthritis, 
tenosynovitis, and dermatitis). 

The patient has migratory polyarthritis with 
knee sw elling. The arthralgias are typically 
asymmetric and tend to involve the upper 
extremities more than the lower extremities. 
The wrist, elbows, ankles, and knees are most 
commonly affected. 

The ankle pains with no sw elling suggests 
tenosynovitis. The tenosynovitis of DGI is 
asymmetric and most commonly occurs over 
the dorsum of the wrist and hand, as well as 
over the metacarpophalangeal joints, ankles, 
and knees. The rash associated with the 
bacteremic form of DGI may be overlooked by 
patients because it is painless and nonpruritic 
and consists of small papular, pustular, or 
vesicular lesions. 

[ Q: 1810 ] MRCPass - 2010 January 

A 16 year old female is evaluated in 
the pediatric endocrinology clinic for primary 
amenorrhoea. 



A person with complete androgen insensitivity 
syndrome (CAIS) has a female external 
appearance despite a 46XY karyotype and 
undescended testes. This is due to the lack of 
sensitivity to androgen (testosterone) leading 
to a failure of male physical development. 

Many of these patients have a female 
phenotype. Some patients are first seen in the 
teenage years for evaluation of primary 
amenorrhea, but most are identified in the 
new born period by the presence of inguinal 
masses, which later are identified as testes 
during surgery. The patients also have a male 
level of testosterone and may have 
clitoromegaly or a micropenis. 


[ Q: 1811 ] MRCPass - 2010 January 

A statistician is advising a research 
about a study of blood pressures in a specific 
ethnic population. The researcher is 
concerned about the spread of blood 
pressures being larger than in the general 
population, hence affecting the sample mean. 

Which of these measures provides an estimate 
of this concept? 



Which one of these features is consistent with 
testicular feminisation or androgen 
insensitivity syndrome? 

1- Male phenotype with lack of hair 

2- Male phenotype with inguinal testis 

3- Male phenotype with breast development 

4- Female phenotype with clitoromegaly and 
undescended testis 

5- Female phenotype with external male 
genitals 


1- Sensitivity 

2- Specificity 

3- Positive predictive value 

4- Negative predictive value 

5- Standard error of mean 

Answer & Comments 

Answer: 5- Standard error of mean 

Standard deviation provides a measure of 
spread of observations about mean. 


Answer & Comments 

Answer: 4- Female phenotype with 

clitoromegaly and undescended testis 

Androgen insensitivity syndrome (AIS), 
formerly known as testicular feminization, is 
an X-linked recessive condition. 


It based of deviation of each observation from 
the mean value. Standard error of the mean is 
the standard deviation of the sampling 
distribution of the mean - which gives an 
estimate of how close the sample mean is to 
the true population mean. It increases with 


Dr. Khalid Yusuf El-Zohry - Sohag Teaching Hospital (01118391123) 

Ref MRCPass OE OE 2012 PasTest 2009 PassMedicine 2009 PasTest Exam 




ReviseMRCP 

715 























El-zohry MRCP Questions Bank (Port 1) - 2013 


(For my personal use) 


sample size and increases with standard 
deviation. 

The S.E.M. is the standard deviation divided by 
the square root of the sample size SEM = ? / 
?N where ? is the standard deviation of the 
original distribution and N is the sample size. 


[ Q: 1812 ] MRCPass - 2010 January 

A 40 year old Caucasian female 
presented with malaise, dysphagia and 
sclerodactyly and Raynaud's phenomenon for 
the last 3 months. On physical examination 
she was afebrile and had a supine blood 
pressure of 110/80mm Hg. Sclerodactyly and 
telangiectasia were observed in both hands. 
Blood tests revealed: 

Hb 11.5 g/dl, MCV 85 fl 

erythrocyte sedimentation rate of 80 mm/first 
hour, 

antinuclear antibody (ANA) - strongly positive 

antitopoisomerase I antibody (formerly anti 
SCL-70 antibody) positive 

normal C3 and C4 

anti-DNA, anti-centromere, anti-RNP, anti-Ro 
and La antibodies - negative 

Chest x-ray showed bilateral basilar interstitial 
infiltrates. 

What is the diagnosis? 

1- Hereditary haemorrhagic telangiectasia 

2- Sarcoidosis 

3- Wegener's granulomatosis 

4- Oesophageal carcinoma 

5- Scleroderma 



develop it during the course of their disease. 
Oesophageal dysmotility may cause reflux, 
aspiration or dysphagia. Pulmonary fibrosis 
and renal impairment are also associated. 

Antinuclear antibodies are present in about 
95% of the patients. Topoisomerase I 
antibodies (formerly Scl-70) are present in 
approximately 30% of patients with diffuse 
disease (absent in limited disease) and are 
associated with pulmonary fibrosis. 
Anticentromere antibodies are present in 
about 60-90% of patients with limited disease 
and are rare in patients with diffuse disease 
(which is more likely in this case). 


[ Q: 1813 ] MRCPass - 2010 January 

In which one of the following areas 
does polypeptide degradation take place? 

1- Golgi body 

2- Peroxisome 

3- Proteosome 

4- Endoplasmic reticulum 

5- Ribosome 



Answer & Comments 

Answer: 3- Proteosome 

The proteosome is a large complex of various 
proteases, proteins / polypeptides that break 
down other proteins in specific ways. 

The proteins are usually tagged by ubiquitin 
before they can be degraded by proteases. 

Peroxisomes participate in the breakdown of 
fatty acids. 


Answer & Comments 

Answer: 5- Scleroderma 

The clues here for scleroderma are dysphagia, 
sclerodactyly and Raynaud's phenomenon. 

70% of patients initially present with 
Raynaud's phenomenon; 95% eventually 


^ [ Q: 1814 ] MRCPass - 2010 January 

/ ■ --- 

# An 18-year-old man presents with 

lethargy, pyrexia and headaches. These 
symptoms have been present for the past 8 
days. He had not been unwell before and 
there is no recent history of travel. Clinical 
examination reveals a temperature of 37.9°C, 


Dr. Khalid Yusuf El-Zohry - Sohag Teaching Hospital (01118391123) 

Ref MRCPass OE OE 2012 PasTest 2009 PassMedicine 2009 PasTest Exam ReviseMRCP 



716 


























El-zohry MRCP Questions Bank (Port 1) - 2013 


(For my personal use) 


marked cervical lymphadenopathy and mild 
hepatomegaly. Throat examination reveals 
two small erythematous areas. A full blood 
count result shows: 

Hb 13.1 g/dl 

Platelets 225 x 10 9 /l 

WCC 17.1 x 10 9 /l 

Neut 5.2 x 1071 

Lymp 11.2 x 1071 

Blood Film Atypical lymphocytes seen 
What is the most likely diagnosis? 

1- Acute lymphoblastic leukaemia 

2- Hashimoto's thyroiditis 

3- Infectious mononucleosis 

4- HIV seroconversion 

5- Septicaemia secondary to streptococcal 
throat infection 


Answer & Comments 

Answer: 3- Infectious mononucleosis 

The history of previously being well, acute 
deterioration with lymphadenopathy, throat 
involvement and atypical lymphocytes on 
blood film are all consistent with Epstein Barr 
virus infection (glandular fever or infectious 
mononucleosis). 

Atypical lymphocytes are commonly 
associated with EBV, CMV and toxoplasma 
infection. 



Reactive lymphocytes are lymphocytes that 
become large as a result of antigen 
stimulation. 



[ Q: 1815 ] MRCPass - 2010 January 

A 40-year-old man with a history of 
bipolar disorder is admitted with acute 
confusion. On examination, had has tremors 
and was observed to have diarrhoea. He is 
suspected of having lithium toxicity. 


Which one of the following drugs is most likely 
to have precipitated this? 


1- Paracetamol 


2- Steroids 


3- Ramipril 

4- Penicillins 


5- Antimalarials 


Answer & Comments 

Answer: 3- Ramipril 

Lithium toxicity can be precipitated 
particularly by thiazide diuretics and ACE 
inhibitors. 

Other drugs which can interact are 
anticonvulsants, antidepressant (SSRI), 
phenothiazines, NSAIDS and calcium channel 
blockers drugs. 


[ Q: 1816 ] MRCPass - 2010 January 

A 62 year woman diabetes mellitus 
presented sudden onset of wild flinging 
movements of left arm which disturbed her 
during sleep at night. 

Where is the likely site of the lesion? 

1- Substantia nigra 

2- Contralateral subthalamic nucleus 

3- Non dominant parietal 

4- Caudate nucleus 

5- Cerebellar 



Answer & Comments 


Answer: 2- Contralateral subthalamic nucleus 


Dr. Khalid Yusuf El-Zohry - Sohag Teaching Hospital (01118391123) 

Ref MRCPass OE OE 2012 PasTest 2009 PassMedicine 2009 PasTest Exam 




ReviseMRCP 

717 


























El-zohry MRCP Questions Bank (Port 1) - 2013 


(For my personal use) 


The hemiballismus (swinging arm movements) 
is likely be due a vascular event in the 
subthalamic nucleus. 



[ Q: 1817 ] MRCPass - 2010 January 

A 60-year-old man is admitted to the 
resuscitation room with a GCS score of 13/ 15. 


He has a past medical history of hypertension, 
glaucoma and diabetes. He had been found 
unconscious at home.. Blood gases and blood 
test results taken on admission show the 
following: 


pH 7.22 
pC02- 3.5 kPa 
p02 -13.8 kPa 
Na+ 140 mmol/I 


K+ 4.2 mmol/l 

Chloride 110 (95-107) mmol/l 
Bicarbonate 10 (20-28) mmol/l 
Urea 2.1 mmol/l 
Creatinine 79 |imol/l 
Glucose 7.1 mmol/l 

A day following observation in the assessment 
unit, he complained of visual problems. 

Which one of the following diagnoses would 
be most consistent with these results? 


1- Addisonian crisis 

2- Pulmonary embolism 

3- Paraquat poisoning 

4- Methanol poisoning 

5- Diabetic ketoacidosis 


Answer & Comments 

Answer: 4- Methanol poisoning 

This patient has a raised anion gap and 
metabolic acidosis. 

The anion gap is = (Na + K) - (CI+HC03), 
[normal range 


10-18 mmol/L]. In this case (140 + 4.2) - (110 + 
10) = 24.2 

Methanol poisoning can cause high anion gap 
metabolic acidosis and also leads to formation 
of formic acid from methanol, which causes 
retinal toxicity and visual impairment. 

A useful mnemonic to remember causes of 
high anion gap metabolic acidosis is MUDPILES 
(methanol, uremia, DKA, propylene glycol, 
isoniazid, lactic acidosis, ethylene glycol, 
salicylates). The scenario is not consistent with 
diabetic ketoacidosis ad the glucose is normal. 


[ Q: 1818 ] MRCPass - 2010 January 

A 58 patient has been admitted with 
confusion and tremors. There was no clear 
history available, hence the patient was kept 
in hospital for observation. 

Which one of the following features supports 
the diagnosis of alcohol withdrawal? 

1- History of long term memory loss 

2- History of epilepsy 

3- Use of cannabis 

4- Seeing a dog lying next to the bed 

5- Obsessive hand washing 



Answer & Comments 

Answer: 4- Seeing a dog lying next to the bed 

Alcohol withdrawal delirium (delirium 
tremens) - this is the clinical syndrome of 
disorientation, perceptual disturbance and 
psychomotor agitation. 

Visual hallucinations are commonly 
associated, as suggested I this case where the 
patient sees a dog in the hospital. 

Korsakoff's associated short term memory 
loss, subsequent compensatory confabulation 
by patient. Other symptoms may include 
delirium, anxiety, fear, depression, confusion, 
delusions and insomnia. 


Dr. Khalid Yusuf El-Zohry - Sohag Teaching Hospital (01118391123) 

Ref MRCPass OE OE 2012 PasTest 2009 PassMedicine 2009 PasTest Exam ReviseMRCP 



718 

























El-zohry MRCP Questions Bank (Port 1) - 2013 


(For my personal use) 



[ Q: 1819 ] MRCPass - 2010 January 

A 40-year-old woman who is known 
to be HIV positive is admitted to the 
Emergency Department following a seizure. 
Her partner reports that she has been having 
headaches, night sweats and anorexia for the 
past four weeks. Blood tests and a CT head are 
arranged: 


CD4 80 cells/mm 


CT head - Single ring enhancing lesion in the 
right parietal lobe with surrounding oedema 

What is the most likely diagnosis? 

1- CNS lymphoma 

2- Tuberculosis 


3- Progressive multifocal 

leukoencephalopathy 

4- Brain abscess 


5- Cerebral toxoplasmosis 


Answer & Comments 

Answer: 1- CNS lymphoma 

The best options are either lymphoma or 
toxoplasmosis, but key to differentiating is 
whether it is single or multiple. 

With cerebral lymphoma, a single lesion that 
enhances in a nodular, homogeneous, or ring 
like pattern is observed, typically with 
surrounding cerebral oedema. 

Toxoplasmosis is the most common cause of 
brain lesions in HIV patients. 

The majority of lesions (90%) are However, 
mulltiple on presentation, so is less likely in 
the above scenario. 

Tuberculous infection is much more varied 
radiologically, with meningeal destruction and 
Granulomas. 



Cerebral Lymphoma 



[ Q: 1820 ] MRCPass - 2010 January 

A 40 year old woman is admitted to 
the Emergency Department with pleuritic 
chest pain and haemoptysis, ten days 
following surgery. She has oxygen saturations 
of 92% and is tachypnoeic. Pulmonary 
embolism was suspected and she was 
organised for a chest X ray. 


Which one of the following signs on the CXR 
suggests acute pulmonary embolism? 


1- Right heart enlargement 


2- Prominent pulmonary vessels 

3- Normal 


4- Pleural effusion 


5- Peripheral wedge shaped opacity 


Answer & Comments 

Answer: 5- Peripheral wedge shaped opacity 

This question did not ask what was the most 
common CXR finding in pulmonary embolism 
which would make the best answer 'normal'. 

There are two rare, signs which are Hampton's 
hump and Westermark sign. 

Hampton's hump is a radiologic sign seen on 
chest radiographs indicating segmental 
pulmonary infarction classically due to 
pulmonary embolism. It consists of a pleura 
based shallow wedge-shaped consolidation in 



Dr. Khalid Yusuf El-Zohry - Sohag Teaching Hospital (01118391123) 

Ref MRCPass OE OE 2012 PasTest 2009 PassMedicine 2009 PasTest Exam ReviseMRCP 

























El-zohry MRCP Questions Bank (Port 1) - 2013 


(For my personal use) 


the lung periphery with the base against the 
pleural surface. 

The Westermark Sign, is a sign that represents 
a focus of oligemia (vasoconstriction) seen 
distal to a pulmonary embolus. 


The case scenario is consistent with primary 
hyperparathyroidism. 

PTH enhances active reabsorption of calcium 
and magnesium from distal tubules and of the 
kidney. 


Right heart enlargement and prominent 
pulmonary vessels are signs of chronic 
pulmonary embolism rather than acute. 



Hampton's hump in the right basal periphery 

on this CXR 



[ Q: 1821 ] MRCPass - 2010 January 

A 25 year old woman presents with 
lethargy, polyuria and nausea. She has no past 
medical history and is currently not taking 
medications. Her blood results are: sodium 
135 mmol/l, potassium 4.3 mmol/l, urea 7 
mmol/l, creatinine 90 pmol/l, calcium 3.2 
(2.25-2.7) mmol/l, phosphate 0.3 (0.8-8) 
pmol/l, Parathyroid hormone 18 (0.8-8) 

pmol/l. 


What is the likely couse of hypercalcaemia? 

1- Hypocalciuric hypophosphataemic rickets 


2- Chronic kidney disease 

3- Hypophosphataemia 

4- Parathyroid hormone excess 

5- 1,25 (OH) vitamin D supplementation 


As bone is degraded both calcium and 
phosphate are released. It also greatly 
increases the excretion of phosphate, with a 
net loss in plasma phosphate concentration. 
By increasing the calcium:phosphate ratio 
more calcium is therefore free in the 
circulation. 

PTH enhances the absorption of calcium in the 
intestine by increasing the production of 
activated vitamin D. PTH up-regulates the 
enzyme responsible for 1-alpha hydroxylation 
of 25-hydroxy vitamin D, converting vitamin D 
to its active form (1,25-dihydroxy vitamin D). 

PTH stimulates bone resorption by 
osteoclasts. 


[ Q: 1822 ] MRCPass - 2010 January 

A blood test has been used to assess 
the likelihood of gastric cancer. The results are 
as follows: 

Cancer Diagnosed No Cancer 
Positive 60 60 
Negative 80 40 

Whot is the positive predictive value? 

1- 25% 

2- 33.3% 

3- 50% 

4- 60% 

5- 66.6% 



Answer & Comments 


Answer: 3- 50% 


Answer & Comments 


Answer: 4- Parathyroid hormone excess 


The positive predictive value of a test is the 
probability that the patient has the disease 


Dr. Kholid Yusuf El-Zohry - Sohog Teaching Hospital (01118391123) 

Ref MRCPass OE OE 2012 PasTest 2009 PassMedicine 2009 PasTest Exam ReviseMRCP 



























El-zohry MRCP Questions Bank (Port 1) - 2013 


(For my personal use) 


when restricted to those patients who test 
positive. 

This term is sometimes abbreviated as PPV. 
You can compute the positive predictive value 
as 

PPV = TP/(TP + FP) 

where TP and FP are the number of true 
positive and false positive results, 
respectively. In this case, the TP is 60, FP is 60 
and PPV is 60/120 = 50%. 

[ Q: 1823 ] MRCPass - 2010 January 

A 35-year-old man is admitted with 
left-sided pleuritic chest pains. These pains 
have been occurring for the past 2 weeks. 
Prior to the onset of the pains, he had been 
experiencing flu-like symptoms. 

What is the ECG most likely to show? 

1- SI, Q3, T3 

2- Atrial fibrillation 

3- Widespread ST elevation 

4- ST segment depression in the anterior leads 

5- Tented T waves 



Answer & Comments 

Answer: 3- Widespread ST elevation 

The diagnosis is likely to be pericarditis 
(possibly viral aetiology) and classical ECG 
changes of saddle shaped ST elevation are 
expected 



Widespread ST elevation on the ECG in 

pericarditis 



[ Q: 1824 ] MRCPass - 2010 January 
Where is the site of action of thiazide 

diuretics? 

1- Proximal part of Distal Convoluted Tubule 

2- Distal part of Distal Convoluted Tubule 

3- Proximal convoluted tubule 

4- Loop of Henle 

5- Collecting ducts 


Answer & Comments 

Answer: 1- Proximal part of Distal Convoluted 
Tubule 

Thiazide diuretics reduce the reabsorption of 
sodium and chloride in the early part of the 
distal convoluted tubule of the kidney. 

This results in the delivery of increased 
amounts of sodium to the distal tubule, where 
some of it is exchanged for potassium. The net 
result is increased excretion of sodium, 
potassium and water. 


[ Q: 1825 ] MRCPass - 2010 January 

A 23 year old male medical student 
was brought to hospital by his girlfriend who 
was concerned about his behaviour. He has 
just returned from a student elective in the 
United States. Whilst he was being assessed 
he 

appeared anxious and agitated. He was 
restless and paced up and down the corridor. 
He spoke very quickly. 

Upon questioning, he said that he was doing 
extremely well in medicine and soon was to 
become the dean of the medical school. 

What is the most likely diagnosis? 

1- Paranoid chizophrenia 

2- Anxiety disorder 

3- Hypomania 

4- Delusional disorder 

5- Obsessive compulsive disorder 



Dr. Khalid Yusuf El-Zohry - Sohag Teaching Hospital (01118391123) 

Ref MRCPass OE OE 2012 PasTest 2009 PassMedicine 2009 PasTest Exam 




ReviseMRCP 

721 











































El-zohry MRCP Questions Bank (Part 1) - 2013 


(For my personal use) 


Answer & Comments 

7} 

[ Q: 1827 ] MRCPass - 2010 January 

Answer: 3- Hvpomania 

* 1 J 

i y 

A 43-year-old South Asian man had a 


Hypomania is a mood state characterized by 
persistent and pervasive elated or irritable 
mood, and thoughts and behaviour that are 
consistent with such a mood state. 


routine blood test with his GP. He has no 
symptoms. These results were found: 

Hb 10.5 g/dl, MCV 75 fl, WCC 7 x 10 9 /l, 
platelets 220 x 10 9 /l, HbA2 level 5% (<3.5%) 


It is distinguished from mania by the absence 
of psychotic symptoms and by its lower 
degree of impact on functioning. Patients 
often have pressured speech and grandiosity. 


A blood film showed hypochromia, slight 
microcytosis and anisocytosis, a few target 
cells, and basophilic stippling. 

What is the diagnosis? 


There may be flight of ideas, lack of sleep and 
inability to slow the mind down. 


1- Sideroblastic anaemia 

2- Alpha thalassemia trait 


^ [ Q: 1826 ] MRCPass - 2010 January 

ft - 

# A 22-year-old man presents with 
rectal bleeding and pain with opening his bow 
els. He is prone to having periods 

of constipation and notices that when he 
cleans himself there is presence of blood on 
the paper. Rectal 

examination reveals no abnormality. 

What is the most likely diagnosis? 

1- Anal fissures 

2- Anogenital herpes 

3- Haemorrhoid 

4- Rectal carcinoma 

5- Rectal polyp 

Answer & Comments 

Answer: 1- Anal fissures 

An anal fissure is a crack in the wall of the anal 
mucosa so that the circular muscle layer is 
exposed. 

The peak incidence is in the 20-30 year old age 
group. There is often a history of pain of 
defaecation - often first occurs during a period 
of constipation. There is also fresh rectal 
bleeding at defaecation; the patient complains 
of 'bright red blood on the paper'. 


3- Beta thalassemia trait 

4- Acute intermittent porphyria 

5- Lead poisoning 

Answer & Comments 

Answer: 3- Beta thalassemia trait 

The difficulty here is that both lead poisoning 
and thalassemia trait, the blood film shows an 
iron deficiency picture, target cells and 
basophilic stippling. 

As this patient is asymptomatic, it fits 
thalassemia better. Below are descriptions of 
some of the options. 

thalassemia trait (minor): This trait is 
characterized by mild anemia and low RBC 
indices. This condition is typically caused by 
the deletion of 2 ? (a) genes on one 
chromosome 16 (aa/oo) or one from each 
chromosome (ao/ao). 

This condition is encountered mainly in 
Southeast Asia, the Indian subcontinent. 

thalassemia trait (minor): Patients have mild 
anemia, abnormal RBC indices, and abnormal 
Hb electrophoresis results with elevated levels 
of Hb A2, Hb F, or both. Peripheral blood film 
examination usually reveals marked 
hypochromia and microcytosis (w ithout the 
anisocytosis usually encountered in iron 


Dr. Khalid Yusuf El-Zohry - Sohag Teaching Hospital (01118391123) 

Ref MRCPass OE OE 2012 PasTest 2009 PassMedicine 2009 PasTest Exam ReviseMRCP 



























El-zohry MRCP Questions Bank (Port 1) - 2013 


(For my personal use) 


deficiency anemia), target cells, and faint 
basophilic stippling. The production of a 
chains from the abnormal allele varies from 
complete absence to variable degrees of 
deficiency. 

Sideroblastic anemias: they usually associated 
with microcytosis and hypochromia and thus 
must be distinguished from the anaemia of 
iron deficiency and thalassemia. A defect 
causes iron to accumulate in mitochondria 
ringing the red cell nucleus forming "ringed 
sideroblasts". Most sideroblastic anaemia are 
acquired and are associated with drugs 
(alcohol*, isoniazid, chloramphenicol, 
cytotoxic agents, and other Vit B6 
antagonists), heavy metals (lead), and various 
hematologic, neoplastic and inflammatory 
diseases. 

Lead poisoning: the anaemia is usually 
microcytic; with basophilic stippling; ringed 
sideroblasts; increased serum Fe, but may be 
haemolytic. Symptoms include abdominal pain 
with constipation; metabolic acidosis, and 
may lead to shock, coma and death. 



Ring sideroblast - iron accumulation in 
mitochondria in red cells 


polychrorrcfiia 


4— _^ 

dimorphic 

stippling ► 


Basophilic stipling - due to denatured RNA in 

the red blood cells 


[ Q: 1828 ] MRCPass - 2010 January 

A 42-year-old man has recently been 
diagnosed with non-Hodgkin's lymphoma. He 
has a long history of alcoholism and has 
significant alcohol-related peripheral 
neuropathy. 

Which one of the following chemotherapy 
agents should be avoided? 

1- Chlorambucil 

2- Cyclophosphamide 

3- Epirubicin 

4- Vincristine 

5- Rituximab 



Answer & Comments 

Answer: 4- Vincristine 

Vincristine (brand name, Oncovin), also known 
as leurocristine, is a vinca alkaloid. 

It works through disruption of the 
microtubules which in turns disrupts 
metaphase in mitosis. Its main uses are in 
non-Hodgkin's lymphoma as part of the 
chemotherapy regimen CHOP, Hodgkin's 
lymphoma as part of MOPP, COPP, BEACOPP. 

The main side-effects of vincristine are 
peripheral neuropathy (which can be severe), 
hyponatremia and hair loss. 


[ Q: 1829 ] MRCPass - 2010 January 

A 22-year-old man is investigated for 
weight loss and abdominal pains. He also had 
altered bow el habit with occational 
diarrhoea. A rectal biopsy is taken and 
reported as follows: 

Deep inflammatory infiltrate from the mucosa 
to the lamina propria 

Numerous non caseating granulomata were 
also seen. 

What is the most likely diagnosis? 

1- Crohn's disease 



Dr. Khalid Yusuf El-Zohry - Sohag Teaching Hospital (01118391123) 

Ref MRCPass OE OE 2012 PasTest 2009 PassMedicine 2009 PasTest Exam 




ReviseMRCP 

723 






















El-zohry MRCP Questions Bank (Port 1) - 2013 


(For my personal use) 


2- Whipple's disease 

3- Tuberculosis 

4- Laxative abuse 

5- Ulcerative colitis 


Some of the infectious causes of bloody 
diarrhoea are: 

Salmonella 

Shigella 


Answer & Comments 

Answer: 1- Crohn's disease 

Microscopically, the classical changes seen 
with Crohn's disease are non-caseating 
granulomas, transmural inflammation and 
lymphocyte infiltration. 

The terminal ileum, caecum and rectum are 
areas most commonly involved. 


[ Q: 1830 ] MRCPass - 2010 January 

A 32-year-old man who has returned 
from a holiday in Egypt presents with 
diarrhoea. For the past two days he has been 
passing frequent bloody diarrhoea associated 
with crampy abdominal pain. Abdominal 
examination demonstrates diffuse lower 
abdominal tenderness but there is no 
guarding or rigidity. His temperature is 37.7°C. 



Campylobacter jejuni 
Yersinia enterocolitica 
E. coli 

Entamoeba histolytica 


[ Q: 1831 ] MRCPass - 2010 January 

Which one of the following is 
implicated with a JAK2 mutation? 

1- Multiple myeloma 

2- Squamous cancer of the lung 

3- Polycythaemia rubra vera 

4- Haemophilia 

5- Von Willebrand's disease 



Answer & Comments 


Answer: 3- Polycythaemia rubra vera 


What is the most likely causative organism? 

1- Giardiasis 

2- Enterotoxigenic Escherichia coli 

3- Staphylococcus aureus 

4- Shigella 

5- Salmonella 


Answer & Comments 

Answer: 4- Shigella 

All are common causes of traveller's 
diarrhoea. 

However, North Africa and the Middle East (in 
particular Egypt) were also commonly 
reported regions of travel for Shigella spp 
infections. 


Janus kinase 2 (commonly called JAK2) is a 
human protein that has been implicated in 
signaling by members of the type II cytokine 
receptor family. 

These mutations have been associated with 
polycythemia vera, essential 

thrombocythemia, and other 

myeloproliferative disorders. 

[ Q: 1832 ] MRCPass - 2010 January 

A 25 year old man has recently been 
to India for a holiday, returning a week ago. 
He presents with fatigue and abdominal pains. 
He gives a history of diarrhoea for 5 days. On 
examination, he was jaundiced and had 
tender hepatomegaly. His investigations show: 



Dr. Khalid Yusuf El-Zohry - Sohag Teaching Hospital (01118391123) 

Ref MRCPass OE OE 2012 PasTest 2009 PassMedicine 2009 PasTest Exam ReviseMRCP 



724 




























El-zohry MRCP Questions Bank (Port 1) - 2013 


(For my personal use) 


sodium 135 mmol/l, potassium 4.2 mmol/l, 
urea 5 mmol/l, creatinine 100 pmol/l, ALT 
1380 (5-35) U/l, AST 1430 

(1-31) U/l, ALP 360 (20-120) U/l, GGT 320 (4- 
35) U/l, Bilirubin 35 (1-22) pmol/l, Albumin 35 
(37-49) g/l 

What is the likely diagnosis? 

1- HIV infection 

2- Leptospirosis infection 

3- Acute hepatitis A infection 

4- Acute hepatitis B infection 

5- Infectious mononucleosis 


Answer & Comments 

Answer: 3- Acute hepatitis A infection 

The abrupt onset of fever, fatigue, malaise, 
anorexia, nausea, diarrhea, jaundice and 
abdominal discomfort are consistent with 
acute hepatitis A infection (especially history 
of diarrhoea). 

Leptospirosis is less likely as there is no renal 
involvement and infectious mononucleosis 
does not usually cause diarrhoea. 


[ Q: 1833 ] MRCPass - 2010 January 

A 62-year-old man was diagnosed 
with atrial fibrillation 3 weeks ago and started 
on digoxin 125 meg od and warfarinised. 
Despite this treatment he still feels his 'heart 
race 1 regularly. An echocardiogram recently 
done showed moderate left ventricular 
dysfunction. On examination his pulse is 110 / 
min, irregularly irregular and respiratory 
examination is unremarkable. 

What is the most appropriate next step in 
management? 

1- Switch digoxin for verapamil 

2- Electrical cardioversion 



5- Flecainide 


Answer & Comments 

Answer: 4- Add bisoprolol 

It is difficult to tell if this patient has persistent 
or permanent atrial fibrillation. 

If he presents acutely and unwell, then 
cardioversion (rhythm control) should be 
considered. However, he has already gone 
down the rate control route, and NICE 
guidelines states that for patients not 
controlled with monotherapy, a combination 
of either beta blocker/calcium channel blocker 
with digoxin should be given. 

http://www.nice.org.uk/nicemedia/pdf/CG03 

6niceguideline.pdf 



[ Q: 1834 ] MRCPass - 2010 January 

A 22 year old woman presents with 
lethargy. She has no past medical history. She 
takes only an oral contraceptive pill daily. On 
examination, she has a slim build and a BMI of 
23 and the only abnormality seen is dental 
erosions. Her blood results show : 


sodium 132 mmol/l, potassium 2.8 mmol/l, 
urea 5 mmol/l, creatinine 100 pmol/l, AST 40 
(1-31) U/l, ALP 150 (20-120) 

U/l, Bilirubin 18 (1-22) |imol/l, Albumin 3 8(37- 
49) g/l, calcium 2.0 (2.25-2.7) mmol/l, 

phosphate 0.75 (0.8-8) pmol/l, 

amylase 260 (60-180) U/l. 

What is the diagnosis? 

1- Bulimia nervosa 


2- Crohn's disease 


3- Laxative abuse 


4- Diuretic use 


5- Pregnancy 


3- Add amiodarone 

4- Add bisoprolol 


Answer & Comments 


Answer: 1- Bulimia nervosa 


Dr. Khalid Yusuf El-Zohry - Sohag Teaching Hospital (01118391123) 

Ref MRCPass OE OE 2012 PasTest 2009 PassMedicine 2009 PasTest Exam 




ReviseMRCP 

725 

























El-zohry MRCP Questions Bank (Port 1) - 2013 


(For my personal use) 


Bulimia nervosa is most likely due to the 
dental erosions. 

Bulimia can produce a variety of metabolic 
and electrolytic imbalances, especially when 
combined with anorexia nervosa or 

aggravated by the abuse of laxatives, 

diuretics, thyroid hormone replacement 
medications and cathartics. Less frequent 

signs and symptoms include metabolic 

alkalosis, low levels of serum protein, zinc 
depletion, low levels of tryptophan and 
serotonin, low levels of serum calcium and 
chloride, high levels of alkaline phosphatase, 
and high levels of serum amylase. 

There is an increase in the total blood volume, 
the plasma volume and the red cell volume 
during pregnancy. The total blood volume 
increases by about 30-40% by about 34 weeks 
of pregnancy causing a haemodilution. 
However, to produce such significant changes, 
it would be a later stage of pregnancy the 
diagnosis should then be obvious in this 
patient. 

^ [ Q: 1835 ] MRCPass - 2010 January 

•Sf - 

# A 18 year old man has presented for 
investigation with haematuria. On his urine 
dipstick, there were blood ++. 

When enquired about family history, he said 
his father and older brother also had 
haematuria. An ANCA and ANA screen was 
done with negative results. An ultrasound of 
the kidney was normal and his creatinine was 
80 umol/l. 

Whot is the likely diagnosis? 

1- Polyarteritis nodosa 

2- Systemic lupus erythematosus 

3- Alport's syndrome 

4- Ig A nephropathy 

5- Exercise induced haematuria 


Answer & Comments 

Answer: 3- Alport's syndrome 

Alport's syndrome leads to a 
glomerulonephritis. 

It is a primary basement membrane disorder 
arising from mutations in genes encoding 
several members of the type IV collagen 
protein family. The disease is mainly inherited 
in the X linked form. In males, there is only 
one X chromosome, so the disease tends to 
manifest in males with the immediate family 
as suggested above. 

The clinical manifestations include recurrent 
episodes of gross hematuria, especially in 
childhood, as in the case vignette. 
Hypertension, proteinuria and sensorineural 
hearing loss can take place although those 
clues were not given in this scenario. 

^ [ Q: 1836 ] MRCPass - 2010 January 

f§ - 

0 You are performing a study of blood 
pressure readings in patients with chronic 
kidney disease. Assuming that the results are 
normally distributed, whot percentage of 
values lies above two standard deviations of 
the mean reading? 

1- 2.5 

2- 5 

3- 10 

4- 95.4 

5- 99.7 

Answer & Comments 

Answer: 1- 2.5 

The question has to be read cautiously. 

In this question regarding 2 standard 
deviations, 95% of patients will lie within 2 SD 
and 2.5% of patients will lie above 2 SD and 
2.5% of patients will lie below 2 SD. 

•68.3% of values lie within 1 SD of the mean 


Dr. Khalid Yusuf El-Zohry - Sohag Teaching Hospital (01118391123) 

Ref MRCPass OE OE 2012 PasTest 2009 PassMedicine 2009 PasTest Exam ReviseMRCP 


























El-zohry MRCP Questions Bank (Port 1) - 2013 


(For my personal use) 


•95.4% of values lie within 2 SD of the mean 
•99.7% of values lie within 3 SD of the mean 




[ Q: 1837 ] MRCPass - 2010 January 

A 47-year-old man with habitual 
smoking habit (2 packs per day) presents with 
haemoptysis. He also suffered had poor 
appetite for 6 months and body weight loss 
(loss 6 kilograms). His blood results are: 


Hb 11.5 g/dl, MCV 75 fl, WCC 6 x 10 9 /l, 
platelets 200 x 10 9 /l, sodium 125 mmol/I, 
potassium 4.5 mmol/I, urea 5 mmol/l, 
creatinine 100 |imol/l. 


Which one of the following is the most likely 
lung cancer in this patient? 


1- Squamous cell 


2- Small cell 


3- Adenocarcinoma 


4- Large cell 

5- Carcinoid 


Answer & Comments 

Answer: 2- Small cell 

Small-cell lung cancer accounts for 
approximately 20-25% of all cases of lung 
cancer. 

It is strongly associated with smoking. Small 
cell carcinomas produce ACTH and ADH, which 
can lead to SIADH, the clue in the history is 
hyponatraemia. 


[ Q: 1838 ] MRCPass - 2010 January 

A 19 year-old man presents with 
facial and ankle sw elling which has slow ly 
been developing over the past 2 weeks. A 
urine dipstick shows protein ++++. 

What is the most likely cause of this 
presentation? 

1- IgA nephropathy 

2- Focal segmental glomerulonephritis 

3- Minimal change disease 

4- Membranous glomerulonephritis 

5- Renal cell carcinoma 



Answer & Comments 

Answer: 4- Membranous glomerulonephritis 

With no haematuria, IgA nephropathy is 
unlikely. 

The most likely options are minimal change 
and membranous. Due to the age of 
presentation, membranous 

glomerulonephritis is the best answer. 

Minimal change nephropathy is responsible 
for 90% of the cases of nephrotic syndrome in 
children less than 5 years of age. The name is 
due to the fact that the only detectable 
abnormality histologically is fusion and 
deformity of the foot processes under the 
electron microscope. It also occurs in adults - 
approx 20%. Normal renal function and blood 
pressure are typical. 

Membranous glomerulonephritis often 
presents with nephrotic syndrome in males. It 
is characterized histologically by thickening of 
the capillary basement membrane secondary 
to the deposition of immune complexes. It is 
associated with SLE, drugs and malignancy. 
About 33% of patients affected go into 
spontaneous remission over five years, but 
the remainder are likely to develop 
progressive renal failure Focal segmental 
glomerulonephritis is a possibility for 


Dr. Khalid Yusuf El-Zohry - Sohag Teaching Hospital (01118391123) 

Ref MRCPass OE OE 2012 PasTest 2009 PassMedicine 2009 PasTest Exam ReviseMRCP 



727 






































El-zohry MRCP Questions Bank (Port 1) - 2013 


(For my personal use) 


nephrotic presentation but is often associated 
with HIV and malignancy, and less common. 


Answer & Comments 


Answer: 2- IV benzylpenicillin + gentamicin 


^ [ Q: 1839 ] MRCPass - 2010 January 

n - 

# A 35 year old woman who is on 
several different medications has developed 
alopecia. 

Which one of the following drugs is likely to 
couse this? 

1- Methyldopa 

2- Sodium valproate 

3- Phenytoin 

4- Metoclopramide 

5- Minoxidil 


Answer & Comments 

Answer: 2- Sodium valproate 

Dyspepsia, weight gain and alopecia are side 
effects of sodium valproate 


[ Q: 1840 ] MRCPass - 2010 January 

A 70-year-old man is admitted to the 
hospital with fevers, lethargy and night 
sweats. 

He has a history of diabetes and had a 
prosthetic mitral valve replacement 5 years 
ago. An echocardiogram is arranged which 
shows a vegetation around the prosthetic 
mitral valve. 3 sets of blood cultures were 
taken and 4/6 of the bottle grew alpha 
haemolytic streptococci, a day later. 



Alpha haemolytic streptococci suggests strep 
viridans. 

Acute endocarditis is most commonly caused 
by staph aureus, whilst subacute endocarditis 
(often with prosthetic valves) is most 
commonly caused by strep viridans. 

Standard therapy to start with (until cultures 
are know n) is intravenous benzylpenicillin and 
gentamicin unless staphylococcal is suspected 
or if there is penicillin allergy when 
vancomycin is substituted for penicillin. 

For strep viridans infection treatment is for 2 
weeks with benzylpenicillin and gentamicin. 


[ Q: 1841 ] MRCPass - 2010 January 

A 25year-old male athlete suffered 
cardiac arrest while playing football. He was 
resuscitated on the field but passed aw ay on 
the way to hospital. The patient had 
previously noticed an "irregular heart beat" on 
several occasions but no chest pain. He had no 
family history of cardiac arrhythmia or sudden 
death. 

Whot is the most likely couse of death? 

1- Arrhythmogenic right ventricular 
cardiomyopathy 

2- Brugada syndrome 

3- Massive pulmonary embolus 

4- Hypertrophic cardiomyopathy 



Whot is the most appropriate antibiotic 
therapy? 

1- IV benzylpenicillin + vancomycin 

2- IV benzylpenicillin + gentamicin 

3- IV ceftriaxone + benzylpenicillin 

4- IV flucloxacillin + gentamicin 

5- IV vancomycin + gentamicin + rifampicin 


5- Diabetic ketoacidosis 

Answer & Comments 

Answer: 4- Hypertrophic cardiomyopathy 

Although all are possibilities, under the age of 
35, the most common cause of sudden death 
among athletes is hypertrophic 
cardiomyopathy. 


Dr. Khalid Yusuf El-Zohry - Sohag Teaching Hospital (01118391123) 

Ref MRCPass OE OE 2012 PasTest 2009 PassMedicine 2009 PasTest Exam ReviseMRCP 



728 





























El-zohry MRCP Questions Bank (Port 1) - 2013 


(For my personal use) 


In hypertrophic cardiomyopathy, there is a 
thickened cardiac muscle, no evidence of 
chamber enlargement, and extensive 
myocardial scarring. 

Athletes with hypertrophic cardiomyopathy 
have an increased frequency of ventricular 
arrhythmia (a disturbance in the rhythm of the 
heartbeat). In these cases, the cardiac 
enlargement is concentric (symmetric) and 
there is no evidence of obstruction of the 
outflow tract in the left ventricle. Athletes 
with this condition often complain of cardiac 
palpitation or cardiac flutter and may have 
syncopal or fainting episodes, in which the 
athlete collapses after a strenuous workout. 


[ Q: 1842 ] MRCPass - 2010 January 

A 55-year-old man with many years 
of type 2 diabetes and multiple complications 
presents complaining of occasional episodes 
of green/blue vision. He is taking many 
medications, which is the likely drug causing 
this? 

1- Phenytoin 

2- Lithium 

3- Metformin 



4^ [ Q: 1843 ] MRCPass - 2010 January 

# A 35 year old man has presented 
with back pains and joint pains over several 
months. He has no other past medical history 
to date. He mentioned that his father had a 
history of joint problems. On examination, 
there is no evidence of skin involvement. Joint 
examination revealed evidence of synovitis in 
the metacarpophalangeal, metatarsal and 
wrist joints bilaterally limiting his range of 
joint movements. 

Laboratory testing revealed a C-reactive 
protein of 0.33 mg/dl (<0.80 mg/dl), strong 
positive CCP antibody, IgG of 148 Units (<20 
Units), and angiotensin-converting-enzyme 
(ACE) of 73 U/liter (<67 U/liter). Anti nuclear 
antibody (ANA), anti neutrophil cytoplasmic 
antibody (ANCA) and rheumatoid factor (Rh F) 
were negative. 

What it the diagnosis? 

1- Dermatomyositis 

2- Psoriatic arthropathy 

3- Rheumatoid arthritis 

4- Osteoarthritis 

5- Systemic lupus erythematosus 


4- Cisapride 

5- Sildenafil 


Answer & Comments 


Answer: 3- Rheumatoid arthritis 


Answer & Comments 

Answer: 5- Sildenafil 

Sildenafil (Viagra) is a phoshodiesterase 
inhibitor used for erectile dysfunction. 

The principal side-effects include dyspepsia, 
flushing and dizziness. The green/blue tint to 
vision is more common at the higher doses - 
and it might make it easy to remember as the 
pills are blue. The retina contains 
phosphodiesterase and inhibition of this is 
thought to be responsible for this 
phenomenon. 


Anti-citrullinated protein/peptide antibodies 
(Anti-CCP) are autoantibodies frequently 
detected in rheumatoid arthritis patients. 

It has a sensitivity of 70% as a diagnostic test. 
The positive anti-CCP antibodies and 
symmetrical polyarthropathy without skin 
involvement suggests rheumatoid arthritis. 

Rheumatoid factor is an IgM antibody against 
IgG. Some patients with rheumatoid arthritis 
will have a negative rheumatoid factor, as in 
this case. 


4% 


•I J 


[ Q: 1844 ] MRCPass - 2010 January 


Dr. Khalid Yusuf El-Zohry - Sohag Teaching Hospital (01118391123) 

Ref MRCPass OE OE 2012 PasTest 2009 PassMedicine 2009 PasTest Exam ReviseMRCP 



729 






























El-zohry MRCP Questions Bank (Port 1) - 2013 


(For my personal use) 


A patient has developed facial swelling 
following treatment with an ACE inhibitor. In 
patients with angioedema, which chemical 
mediator is mainly reponsible for causing 
vascular permeability and the tissue oedema? 

1- Histamine 

2- Serotonin 

3- Neurokinin A 

4- Bradykinin 

5- Nitric oxide 

Answer & Comments 

Answer: 4- Bradykinin 


5- Thrombolysis 

Answer & Comments 

Answer: 4- DC cardioversion 

This patient is symptomatic and unstable with 
hypotension, DC cardioversion is the best 
option as it will restore sinus rhythm and 
allow the patient a good chance of recovery. 

Amiodarone is a good option, but this scenario 
of unstable patient suggests DC cardioversion 
is quicker. As verapamil and bisoprolol are 
negatively inotropic, they are contraindicated 
due to the hypotension. 


Bradykinin is a vasoactive nonapeptide 
produced by the kinin system. 

It causes vasodilation, possibly by a direct 
effect and via prostaglandin mediation. It also 
causes an increase in vascular permeability by 
the opening of intercellular gaps in post¬ 
capillary venules. Bradykinin is inactivated by 
angiotensin converting enzyme, hence ACE 
inhibitors are often associated with 
angioedema. 


[ Q: 1846 ] MRCPass - 2010 January 

Which one of these conditions is 
associated with Human Herpes Virus (HHV) 8 
infection? 

1- Chicken pox 

2- Malignant melanoma 

3- Genital warts 

4- Kaposi's sarcoma 

5- Shingles 




[ Q: 1845 ] MRCPass - 2010 January 

A 82 year old man is admitted with 
palpitations, dizziness and shortness of 
breath. He has a history of hypertension and 
diabetes. His current medications are 
bendroflumethiazide and metformin. He has 
an ECG which showed atrial fibrillation with a 
heart rate of 150 bpm. On examination, he 
has a BP of 80/45 mmHg and JVP is raised. His 
heart sounds are normal and there are a few 
crackles in the bases of the lungs. 


Which of the following is the best 
management? 


1- IV verapamil 

2- IV Amiodarone 


3- IV Bisoprolol 

4- DC cardioversion 


Answer & Comments 

Answer: 4- Kaposi's sarcoma 

HHV 8 is associated with co-infection with HIV 
and Kaposi's sarcoma. 

Genital warts are caused by Human Papilloma 
virus. Chicken pox is caused by varicella zoster 
virus Shingles is caused by latent reactivation 
of the varicella zoster virus within a nerve. 

[ Q: 1847 ] MRCPass - 2010 January 

A 33-year-old Caucasian woman 
admitted has a history of widespread, pruritic, 
erythematous skin rash, joint pains and renal 
disease. She recently had a baby with 
congenital heart block. Laboratory 
investigations revealed mild leucopenia (w 



Dr. Khalid Yusuf El-Zohry - Sohag Teaching Hospital (01118391123) 

Ref MRCPass OE OE 2012 PasTest 2009 PassMedicine 2009 PasTest Exam ReviseMRCP 



730 





























El-zohry MRCP Questions Bank (Port 1) - 2013 


(For my personal use) 


hite cell count 3.25xl09/ml) and 
thrombocytopenia (platelets 140xl09/ml). 
Erythrocyte sedimention rate was increased 
(65 mm/h). 

Which antibody is likely to be positive? 

1- Anti Jo 1 

2- Anti double stranded DNA 


Whot is the most likely diagnosis? 

1- Amyotrophic lateral sclerosis 

2- Multiple sclerosis 

3- Progressive muscular atrophy 

4- Subacute combined degeneration 

5- Syringomyelia 


3- Anti Ro 

4- Anti centromere 

5- ANCA 


Answer & Comments 

Answer: 3- Anti Ro 

The anti Ro antibody is associated with 
Sjogren's syndrome, SLE and neonatal lupus. 

Neonatal lupus erythematosus (NLE) is a rare 
disorder caused by the transplacental passage 
of maternal autoantibodies. Only 1% of infants 
with positive maternal autoantibodies develop 
neonatal lupus erythematosus. The most 
common clinical manifestations are cardiac 
(congenital heart block), dermatologic 
(urticaria and skin desquamation), and hepatic 
(abnormal LFTs). The mother produces 
immunoglobulin G (IgG) autoantibodies 
against Ro (SSA), La (SSB), and/or Ul- 
ribonucleoprotein (Ul-RNP), and they are 
passively transported across the placenta. 
These autoantibodies can be found alone or in 
combination; However, anti-Ro is present in 
almost 95% of patients. 


[ Q: 1848 ] MRCPass - 2010 January 

A 64-year-old man presents with a 
eight-month history of generalised weakness. 
He has no previous past medical history but 
his father had a history of a similar illness at 
the age of 78. On examination he has 
fasciculation and weakness in both arms with 
absent reflexes. Examination of the lower 
limbs reveal increased tone and brisk reflexes 
with upgoing plantars. There was no sensory 
deficit. 



Answer & Comments 

Answer: 1- Amyotrophic lateral sclerosis 

Amyotrophic lateral sclerosis is a form of 
motor neurone disease (MND). 

It is the most common form of MND and 
accounts for 65% to 85% of all cases of MND. 
It is typically late onset, rapidly progressive 
and presents with a combination of upper and 
lower motor neuron signs. 

Three main types of MND are recognised and 
most patients eventually develop features of 
all three: 

progressive muscular atrophy 
amyotrophic lateral sclerosis 
bulbar palsy 



[ Q: 1849 ] MRCPass - 2010 January 

A 65-year-old female is admitted to 
the Emergency Department following an 
overdose of a long-acting atenolol 
preparation. On admission she is bradycardic 
with a pulse of 35/min and BP 90/50 mmHg. 
The bradycardia fails to respond to 3 doses of 
1 mg of atropine. 

What is the next most appropriate 

management? 

1- Temporary cardiac pacing 

2- Calcitonin 


3- Glucagon 

4- Noradrenaline infusion 


5- Salbutamol infusion 


Dr. Khalid Yusuf El-Zohry - Sohag Teaching Hospital (01118391123) 

Ref MRCPass OE OE 2012 PasTest 2009 PassMedicine 2009 PasTest Exam 




ReviseMRCP 

731 























El-zohry MRCP Questions Bank (Port 1) - 2013 


(For my personal use) 


Answer & Comments 

Answer: 3- Glucagon 

Glucagon stimulates production of cAMP 
through nonadrenergic pathw ays and can be 
given early in beta blocker poisoning. 

A dose of 3-10 mg IV bolus followed by 2-5 
mg/h infusion should be commenced. Cardiac 
pacing should be reserved for patients 
unresponsive to pharmacologic therapy or for 
those with torsade de pointes unresponsive to 
magnesium. 


themselves, but rather the fear or idea of 
having a serious disease. The fear or idea is 
based on the misinterpretation of bodily signs 
and sensations as evidence of disease. 

Somatisation disorder: (somatoform disorder) 
With this a patient presents with multiple, 
medically unexplained symptoms. The 
patient's life or work are frequently affected, 
although they also might be unconcerned 
about the nature of their symptoms (thus 
appearing calm). It is not a deliberate feigning 
of symptoms. 


[ Q: 1850 ] MRCPass - 2010 January 

A 35 year old man was investigated 
for palpitations. He was told that he had 
premature ventricular complexes on the ECG. 
However, he was convinced that he had 
cancer and presented to the surgery many 
times over the year despite reassurance. 

Whot is the diagnosis? 

1- Munchausen's syndrome 

2- Hypochondriasis 

3- Dissociative disorder 

4- Somatisation disorder 

5- Conversion disorder 



Answer & Comments 


Answer: 2- Hypochondriasis 


Conversion disorder : (somatoform 

disorder)This is a condition where a patient 
displays neurological symptoms e.g. paralysis, 
even though no neurological explanation is 
found and it is determined that the symptoms 
are due to the patient's psychological 
response to stress. 



[ Q: 1851 ] MRCPass - 2010 January 

A 45-year-old woman presents with 
weight gain and recurrent 'dizzy' episodes. 
Over the past four months she has gained 15 
kg. The episodes occur on an almost daily 
basis and are characterised by blurred vision, 
sweating and headaches. Her GP checked a 
blood sugar during one of these episodes 
which was record as being 2.0 mmol/l. 


What is the single most useful test if the 
patient presents with a further such episode? 


The history fits a diagnosis of hypochondriasis 
as below Munchausen syndrome (factitious 
disorder): the patient seeks medical attention 
by the deliberate production or feigning of 
symptoms. 

The motivation for seeking attention is not 
known. 

Hypochondriasis: (somatoform disorder) the 
patient is convinced that they have a life- 
threatening illness, despite evidence to the 
contrary. The core feature of hypochondriasis 
is not preoccupation with symptoms 


1- Glucagon level 

2- Plasma glucose 

3- Insulin + C-peptide levels 

4- Sulphonylurea level 

5- Thyroid function 

Answer & Comments 

Answer: 3- Insulin + C-peptide levels 

The ideal measurement is glucose, insulin and 
C peptide levels. 


Dr. Khalid Yusuf El-Zohry - Sohag Teaching Hospital (01118391123) 

Ref MRCPass OE OE 2012 PasTest 2009 PassMedicine 2009 PasTest Exam ReviseMRCP 



732 


























El-zohry MRCP Questions Bank (Port 1) - 2013 


(For my personal use) 


This is not given as an option, and neither is a 
72 hour fast. Only an insulin and C peptide 
level (which are elevated during one of these 
episodes) will confirm the diagnosis of a 
possible insulinoma (pancreatic insulin 
secreting tumour as suggested in the clinical 
history). 

A 72-hour fast, usually supervised in a hospital 
setting, can be done to see if insulin levels fail 
to suppress during a hypoglycaemic episode 
(glucose <2.5 mmol/l). The C peptide levels as 
proinsulin is broken down to insulin and C 
peptide. If present, then the patient is unlikely 
to be injecting insulin exogenously. 


Since drugs are approved on the basis of 
clinical trials which involve relatively small 
numbers of people who have been selected 
for this purpose, postmarketing surveillance 
can further refine the safety of a drug after it 
is used in the general population by large 
numbers of people who have a wide variety of 
medical conditions. 

Postmarketing surveillance uses a number of 
approaches to monitor the safety of licensed 
drugs, including spontaneous reporting 
databases, prescription event monitoring, 
electronic health records, patient registries 
and record linkage between health databases. 



[ Q: 1852 ] MRCPass - 2010 January 

A new drug which has been on the 
market for 2 years, has had reports of possible 
serious side effects of fulminant hepatitis. 


What is the best way of evaluate this from a 
safety perspective? 


1- Metanalysis 

2- Postmarketing surveillance 


[ Q: 1853 ] MRCPass - 2010 January 

A 42-year-old male developed neck 
pain which had worsened over several years. 
He presented to a rheumatologist with weight 
loss, back pain and stiffness. Clinical 
examination showed very restricted neck 
movement and chest expansion with no 
lumbar spinal movement. X rays of the 
patient's spine were performed. 



3- Randomised controlled trial 

4- Systematic review 

5- Case control study 

Answer & Comments 

Answer: 2- Postmarketing surveillance 

New drugs which have had regulatory 
approval may not have trial data which 
contain enough information about rare, 
serious side effects, and there may not be 
sufficiently large trials for a new drug for 
metanalysis. 

Postmarketing surveillance (PMS) is the 
practice of monitoring the safety of a 
pharmaceutical drug after it has been released 
on the market and is an important part of the 
science of pharmacovigilance. 


What feature would be expected on the x rays 
in ankylosing spondylitis? 

1- Sclerosis 

2- Syndesmophyte 

3- Lytic lesions 

4- Osteophyte 

5- Wedge shaped lesions 

Answer & Comments 

Answer: 2- Syndesmophyte 

Typical X ray changes of ankylosing spondylitis 
are the visible formation of syndesmophytes 
(bony growth originating inside a ligament) 
and abnormal bone outgrow ths similar to 
osteophytes affecting the spine. 

These changes lead to an appearance of 
'bamboo spine'. 


Dr. Khalid Yusuf El-Zohry - Sohag Teaching Hospital (01118391123) 

Ref MRCPass OE OE 2012 PasTest 2009 PassMedicine 2009 PasTest Exam 




ReviseMRCP 

733 























El-zohry MRCP Questions Bank (Port 1) - 2013 


(For my personal use) 



This lady has oligomenorrhoea with evidence 
of hyperandrogenism and the most likely 
cause is polycystic ovarian syndrome (PCOS), 
which is characterised by hyperandrogenism. 

In the complete syndrome, adipose tissue 
aromatise peripheral androgens to 
oestrogens. This suppresses FSH thus reducing 
follicullar maturation and fertility, as well as 
increases LH resulting in further androgen 
synthesis. Increased androgens results in 
hirsutism and acne and also maintain the cycle 
of peripheral aromatisation to maintain 
elevated oestrogen levels. An elevated LH:FSH 
ratio of greater than 3 is useful in confirming 
the diagnosis of PCOS but its absence does not 
exclude it. 


Syndesmophyte 


[ Q: 1854 ] MRCPass - 2010 January 

# A 40-year-old woman is referred by 
her GP with a history of oligomenorrhoea. In 
clinic, the patient also describes acne as well 
as hirsutism, such that she has to shave 
around her chin and lip region every week. 
Her body mass 

index is 35 Kg/m2 and Blood pressure was 170 
/ 95 mmHg. The following blood results were 
obtained: 

Her fasting blood glucose is 6.8mmol/l. 

testosterone 7 (1.1-6.3) pg/ml 

LH 16 (0.5-14.5) U/l 

FSH 2.8(1-11) U/l 

What is the most likely diagnosis? 

1- Congenital adrenal hyperplasia 

2- Cushing's syndrome 

3- Conn's syndrome 

4- Polycystic ovarian syndrome 

5- Uterine fibroids 


[ Q: 1855 ] MRCPass - 2010 January 

A 17-year-old man presented 
casualty complaining of difficulty breathing. 
He had brought hospital by ambulance, having 
collapsed shortly after being stung on hand by 
a bee. On examination, his blood pressure was 
80/40 mmHg, facial sw elling and pharyngeal 
oedema was noted. 

Which one of following investigations likely 
confirm anaphylaxis? 

1- Haemolytic complement (CH50) level 

2- Plasma tryptase activity 

3- Complement C3 level 

4- Total IgE level 

5- Eosinophil count 



Answer & Comments 

Answer: 2- Plasma tryptase activity 

The reaction involves preferential production 
of IgE, in response certain antigens, which in 
turn initiates a sequence of events leading to 
mast cell activation. 


Answer & Comments 


Answer: 4- Polycystic ovarian syndrome 


According to the Resuscitation council 
guidelines, the specific test to help confirm a 



Dr. Khalid Yusuf El-Zohry - Sohag Teaching Hospital (01118391123) 

Ref MRCPass OE OE 2012 PasTest 2009 PassMedicine 2009 PasTest Exam ReviseMRCP 

























El-zohry MRCP Questions Bank (Port 1) - 2013 


(For my personal use) 


diagnosis of an anaphylactic reaction is 
measurement of mast cell tryptase. 

Tryptase is the major protein component of 
mast cell secretory granules. In anaphylaxis, 
mast cell degranulation leads to markedly 
increased blood tryptase concentrations. 
Tryptase levels are useful in the follow -up of 
suspected anaphylactic reactions, not in the 
initial recognition and treatment: measuring 
tryptase levels must not delay initial 
resuscitation. Tryptase concentrations in the 
blood may not increase significantly until 30 
minutes or more after the onset of symptoms, 
and peak 1-2 hours after onset. 

The half-life of tryptase is short 
(approximately 2 hours), and concentrations 
may be back to normal within 6-8 hours, so 
timing of any blood samples is very important. 

http://www .resus.org.uk/pages/reaction.pdf 


[ Q: 1856 ] MRCPass - 2010 January 

A 35 year old man presented with a 
generalised seizure. On examination, he was 
found to have adenoma sebaceum on the 
face, two hypopigmented areas and 
subungual fibroma. He had a urine dipstick 
showing blood ++ and was organised to have 
an ultrasound of the kidneys which showed 
cystic changes. 

Whot is the likely diagnosis? 

1- Von Hippel Lindau 

2- Neurofibromatosis 

3- Vitiligo 

4- Acromegaly 

5- Tuberous sclerosis 



Answer & Comments 

Answer: 5- Tuberous sclerosis 

The diagnosis is tuberous sclerosis. 

It is an autosomal dominant condition. 
Features are epilepsy (cortical tubers in the 


brain), adenoma sebaceum on the skin, 
subungual fibroma of the nails, oval 
hypopigmented macules - ash leaf macules - 
best seen with Wood's (UV) light, retina 
phakoma, renal angiomyolipoma (causing 
cystic renal lesions) and cardiac 
rhabdomyomas. 



[ Q: 1857 ] MRCPass - 2010 January 

A patient presents with an inability 
to abduct his right shoulder. 

Which nerve supplies this muscle? 

1- Lateral cutaneous 

2- Suprascapular 

3- Musculocutaneous 

4- Axillary 

5- Median 


Answer & Comments 

Answer: 4- Axillary 

The deltoid muscle is implicated here, as the 
action is shoulder abduction. 

It is innervated by the axillary nerve (C5 and 
C6). 


[ Q: 1858 ] MRCPass - 2010 January 

A set of parents is seeking genetic 
They said that their 5-year-old boy 
with cystic fibrosis but they themselves do not 
have the disease. They also have a daughter 
who is 17 years old but not affected by the 
disease. 

Whot is the chance that she will be a carrier of 
the cystic fibrosis gene? 

1- 1 in 2 

2- 1 in 4 

3- 2 in 3 

4- 1 in 25 

5- 100% chance 



advice. 


Dr. Khalid Yusuf El-Zohry - Sohag Teaching Hospital (01118391123) 

Ref MRCPass OE OE 2012 PasTest 2009 PassMedicine 2009 PasTest Exam 




ReviseMRCP 

735 


























El-zohry MRCP Questions Bank (Port 1) - 2013 


(For my personal use) 


Answer & Comments 

Answer: 3- 2 in 3 

Inheritance of cystic fibrosis is autosomal 
recessive. 

In answering this question, the simple mistake 
is to take carriers out of total, which makes a 1 
in 2 chance. The diagram below illustrates the 
best way of working this out. 

As the daughter is not affected, there are 3 
other options, so she might be a carrier in 2 of 
the 3 scenarios (2 in 3). 


[ Q: 1859 ] MRCPass - 2010 January 

# A 45 year old lady presented to her 
GP with lesions in skin that were circular with 
an erythematous raised rim with central 
atrophy. There was scaliness, follicular 
plugging, and telangiectasia over the scalp, 
ears and face. 

This was confirmed to be discoid lupus by the 
dermatologist and she has been tried on 
betnovate steroid topical treatment but has 
not improved. 

What should be used next? 


Hydroxychloroquine is the gold standard 
treatment. Other options include 
azathioprine, dapsone, thalidomide and 
tacrolimus. 

[ Q: 1860 ] MRCPass - 2010 January 

A 44 year old type I diabetic is 
referred for renal investigations. She has been 
diagnosed with multiple myeloma 5 years ago. 
She is currently on chemotherapy. She had 
proteinuria on a urine dipstick 4+ and 
quantification with 24 hour urine collection 
revealed that she had urinary protein 3.5 
g/day. Ultrasound of the abdomen shows 
increased renal echogenicity. Investigations 
show : 

Hb 11.5 g/dl 

MCV 82 fl 

WCC 12 x 10 9 /l 

platelets 225 x 10 9 /l 

sodium 135 mmol/l 

potassium 4.5 mmol/l 

Urea 16 mmol/l 

Creat 225 umol/l 



1- Diprobase cream 

2- Tacrolimus 

3- Azathioprine 

4- Hydroxychloroquine 

5- PUVA therapy 

Answer & Comments 

Answer: 4- Hydroxychloroquine 

Discoid lupus erythematosus (DLE) is a 
chronic, scarring, atrophy producing, 
photosensitive dermatosis. 

DLE may occur in patients with systemic lupus 
erythematosus (SLE). 

Initial treatment comprises the avoidance of 
direct sunlight. Following this, 


Whot is the probable diagnosis? 

1- Minimal change glomerulonephritis 

2- Diabetic nephropathy 

3- NSAIDS induced nephropathy 

4- AL amyloidosis 

5- Crescentic glomerulonephritis 

Answer & Comments 

Answer: 4- AL amyloidosis 

Amyloidosis is a clinical disorder caused by 
extracellular and or intracellular deposition of 
insoluble abnormal amyloid fibrils that alter 
the normal function of tissues. 

AL amyloidosis can be caused by multiple 
myeloma and occurs in 5 to 15% of such 
patients. Apart from nephrotic syndrome, 


Dr. Khalid Yusuf El-Zohry - Sohag Teaching Hospital (01118391123) 

Ref MRCPass OE OE 2012 PasTest 2009 PassMedicine 2009 PasTest Exam ReviseMRCP 



736 




























El-zohry MRCP Questions Bank (Port 1) - 2013 


(For my personal use) 


cardiomyopathy, polyneuropathy and gut 
involvement are common presentations with 
AL amyloid. 

^ [ Q: 1861 ] MRCPass - 2010 January 

# A 68-year-old woman presents with 
a two month history of electric shock like 
pains on the right side of her face and jaw . 
She describes having several episodes a day 
which, each lasting for several minutes up to 2 
hours. A recent dental check was normal. 
Neurological examination is unremarkable. 

What is the most suitable medication for 
prophylaxis ? 

1- Amitriptyline 

2- Sodium valproate 

3- Carbamazepine 

4- Phenytoin 

5- Gabapentin 

Answer & Comments 

Answer: 3- Carbamazepine 

The clinical history is typical for trigeminal 
neuralgia. 


[ Q: 1862 ] MRCPass - 2010 January 

A 45 year old lady has had a renal transplant 2 
weeks ago. She is complaining of fevers, 
lethargy and diarrhea 3-4 times a day. She is 
on the following medications: cyclosporine A 
250mg bd, azathioprine 75mg od and 
prednisolone 20mg od. 

On examination, she has a blood pressure of 
115 / 75 mmHg and a temperature of 37.8 C. 
Her abdomen is soft and the area over the 
renal transplant is mildly tender. The urine 
output is normal. An ultrasound scan of the 

kidneys does not show any abnormality that 
points tow ards the cause. 

Results show : 

Hb 11.5 g/dl, 

WCC11 x 10 9 /l, 

platelets 230 x 10 9 /l 

sodium 135 mmol/l 

potassium 4.5 mmol/l 

urea 7 mmol/l 

creatinine 110 mmol/l (90 immediately post 
transplant) 

Urine dipstick : protein - 


Carbamazepine is regarded by most as the 
medical treatment of choice. The American 
Academy of Neurology published a practice 
parameter that concluded that carbamazepine 
is effective in controlling pain of patients with 
classic trigeminal neuralgia, and that 
oxcarbazepine is probably effective. Baclofen, 
lamotrigine, and pimozide were rated as 
possibly effective. The practice parameter 
stated that there was insufficient evidence to 
support or refute efficacy of clonazepam, 
gabapentin, phenytoin, tizanidine, topical 
capsaicin, or valproate for pain control in 
patients with classic trigeminal 
neuralgia 

J 
y 



Blood + 

What is the likely cause of this presentation? 

1- Allopurinol toxicity 

2- Coronary artery disease 

3- Cyclosporin toxicity 

4- CMV infection 

5- Acute rejection 


Answer & Comments 

Answer: 4- CMV infection 

CMV infection is a multifaceted phenomenon 
with a variety of direct and indirect effects in 
the organ transplant recipient. 

The symptomatology for clinical infectious 
disease (ie, fever, pneumonia, Gl ulcers, 


Dr. Khalid Yusuf El-Zohry - Sohag Teaching Hospital (01118391123) 

Ref MRCPass OE OE 2012 PasTest 2009 PassMedicine 2009 PasTest Exam ReviseMRCP 



737 

























El-zohry MRCP Questions Bank (Port 1) - 2013 


(For my personal use) 


hepatitis) ranges from the mild, subclinical 
case to life-threatening multi-organ disease. 
Most cases of symptomatic CMV infection can 
be characterized by a self-limiting syndrome 
of episodic fever spikes for a period of 3 to 4 
weeks, arthralgias, fatigue, anorexia, 
abdominal pain, and diarrhoea. Ganciclovir is 
the most commonly used agent for the 
prevention of CMV infection. 

Acute rejection is usually associated with a 
rise in creatinine and tenderness over the 
transplant graft. In this case, the systemic 
symptoms suggest that CMV infection is more 
likely than acute rejection. Cyclosporin toxicity 
is also unlikely as the creatinine and 
potassium are not significantly changed. 


[ Q: 1863 ] MRCPass - 2010 January 

Which one of the following factors is 
most useful os o predictor for determining the 
risk of sudden death in a patient with 
hypertrophic cardiomyopathy? 

1- Family history of sudden death 

2- Left ventricular outflow gradient > 30mmHg 

3- Ejection fraction 

4- Blood pressure volatility on physical 
exertion 




[ Q: 1864 ] MRCPass - 2010 January 

A 42-year-old woman is brought to 
the hospital by her husband. He 


reports that she has had an argument with 
their son which resulted in the son running aw 
ay. Since the event, she has been very 
stressed and suddenly has stopped talking 
completely. Clinical examination of her throat 
and chest is unremarkable. 


Which one of the following terms best 
describes this presentation? 

1- Akinetic mutism 


2- Depression 

3- Expressive dysphasia 

4- Schizophasia 

5- Psychogenic aphonia 


Answer & Comments 

Answer: 5- Psychogenic aphonia 

The two most likely answers are either 
akinetic mutism or psychogenic aphonia. 

Psychogenic aphonia or aphasia, is the loss of 
language due to a non-organic or psychiatric 
cause. It is the more likely answer due to the 
stressful event precipitating aphonia. 


5- Septal wall thickness > 3.0 cm 


Answer & Comments 

Answer: 5- Septal wall thickness > 3.0 cm 

Non-sustained ventricular tachycardia, 
syncope, abnormal blood pressure response 
to exercise, family history of sudden death, 
and massive left ventricular (LV) hypertrophy 
are all risk factors for sudden death in 
hypertrophic cardiomyopathy. 

However, septal wall thickness is the most 
important predictor of sudden death. The 
greater thickness of septum, more likely there 
is risk of cardiac arrhythmias (> 3 cm is 
significant). 


Akinetic mutism is a variety of stupor in which 
the patient is unable to talk or carry out 
purposeful behaviour but may 


lie with eyes open, seemingly unaw are of 
what is going on. It results from bilateral 
damage to the orbital surface of the frontal 
lobes, such as anterior cerebral artery stroke. 
The patient appears awake and has normal 
ocular movement but does not speak and has 
minimal motor response to painful 
stimulation. It can be associated with anterior 
cerebral artery stroke. 



[ Q: 1865 ] MRCPass - 2010 January 


A 75 year-old female was seen in the 
Emergency department with a 2 day history of 


Dr. Khalid Yusuf El-Zohry - Sohag Teaching Hospital (01118391123) 

Ref MRCPass OE OE 2012 PasTest 2009 PassMedicine 2009 PasTest Exam ReviseMRCP 




























El-zohry MRCP Questions Bank (Port 1) - 2013 


(For my personal use) 


headaches and fever. On examination, the 
patient had a temperature of 38.5 °C. There 
was also evidence of meningism with a 
positive Kernig's sign. Tone, power and 
reflexes were normal apart from general 
weakness. There was no sensory deficit. A 
lumbar puncture was performed. CSF showed 
100 white cells (90% lymphocytes), protein 0.9 
(<0.5) and glucose 3.3, plasma glucose 7.5. 

Whot is the diagnosis? 

1- Gullain barre syndrome 

2- Listeria meningitis 

3- Tuberculous meningitis 

4- Poliomyelitis 

5- HSV encephalitis 


3- Eastern blotting 

4- Western blotting 

5- Polymerase chain reaction 

Answer & Comments 

Answer: 1- Northern blotting 

The northern blot is a technique used in 
molecular biology research to study gene 
expression by detection of RNA in a sample. 

Northern blotting involves the use of 
electrophoresis to separate RNA samples by 
size, and detection with a hybridization probe 
(either DNA or RNA) complementary to part 
gene sequence. 


Answer & Comments 

Answer: 3- Tuberculous meningitis 

In Tuberculous meningitis, Fever, headache, 
confusion and meningism are presenting 
features. 

Meningism is absent in a fifth of patients with 
TB meningitis. 

The CSF usually has a high protein, low glucose 
and a raised number of lymphocytes as seen 
in this patient. 

Acid-fast bacilli are sometimes seen on a CSF 
smear, but more commonly, M. tuberculosis is 
grown in culture. 


^ [ Q: 1867 ] MRCPass - 2010 January 

A - 

* A 42 year old woman presents with 
breathlessness. She has a history of 
pulmonary fibrosis associated with connective 
tissue disease. Her lung function tests show : 

FVC (I) (% predicted) 2.28 (66%) 

FEV1 (I) (% predicted) 2.04 (70%) 

FEV1/FVC (%) 89 

total lung capacity (TLC) = 2.9 L (70%) 

TLCO mmol/kPa/min (% predicted) 8.5 (110) 
KCO mmol/kPa/min/l (% predicted) 4.4 (150) 
What is the diagnosis? 

1- Pulmonary haemorrhage 


In this patient, HSV is unlikely as the protein is 
high, and listeria should cause a picture with 
more polymorphonuclear white cells in the 
CSF 

[ Q: 1866 ] MRCPass - 2010 January 

Which one of the following 
technigues is used to detect RNA using a 
labelled DNA probe for hybridisation? 

1- Northern blotting 

2- Southern blotting 



2- Pulmonary embolism 

3- Pleural effusion 

4- Emphysema 

5- Diaphragmatic weakness 

Answer & Comments 

Answer: 1- Pulmonary haemorrhage 

The lung function shows a restrictive picture, 
reduced FEV1 and FVC, which are consistent 
with the underlying connective tissue disease. 


Dr. Khalid Yusuf El-Zohry - Sohag Teaching Hospital (01118391123) 

Ref MRCPass OE OE 2012 PasTest 2009 PassMedicine 2009 PasTest Exam ReviseMRCP 






























El-zohry MRCP Questions Bank (Port 1) - 2013 


(For my personal use) 


However, as well as a slightly increased TLCO, 
there is significantly increased KCO (transfer 
factor) which suggests pulmonary 
haemorrhage. 

Transfer factor for carbon monoxide (TLCO) is 
a useful investigation in alveolar 
haemorrhage. It is actually the product of 
alveolar volume and carbon monoxide 
transfer coefficient (KCO). The alveolar 
volume is mildly reduced because of alveolar 
filling with blood, and KCO is considerably 
increased because the inhaled CO reacts with 
extravascular haemoglobin. An increased 
TLCO with considerably increased KCO and 
mildly reduced alveolar volume are 
characteristic of pulmonary haemorrhage. 

^ [ Q: 1868 ] MRCPass - 2010 January 

a - 

m A 22 old student nurse complained 
of severe itching and sw elling in the hands, 
several minutes after wearing gloves. This has 
occurred before 10 years ago when he first 
put a set of gloves on. On examination, there 
was a flexural rash and sw elling in the hands. 

1/1 /hot is the diagnosis? 

1- Contact dermatitis 

2- Systemic lupus erythematosus 

3- Cl esterase inhibitor deficiency 

4- Latex allergy 

5- Anaphylaxis 

Answer & Comments 

Answer: 4- Latex allergy 

The quick reaction in this case suggests latex 
allergy, whilst in contact dermatitis the skin 
changes will be slower to develop. 

An allergy to latex (the natural rubber latex 
protein in rubber) is an immediate 
hypersensitivity (IgE mediated) reaction. 

This means that the speed of onset of features 
of a latex allergy occur over a short period of 


time (seconds+ (up to 6 hours)). 
Dermatological manifestations include itching, 
burning skin and urticaria. There may be 
associated angioedema and even anaphylaxis 
(with bronchospasm or hypotension). 

There are two types of contact dermatitis : 
allergic and irritant. Allergic contact dermatitis 
requires prior sensitization of the skin to a 
specific allergen. T lymphocytes become 
specially sensitised to the allergen, leading to 
a dermatitis reaction. It is an example of a 
type IV hypersensitivity reaction. Irritant 
contact dermatitis is a result of direct damage 
to the skin by the provoking agent. No prior 
exposure to the agent is necessary. 


[ Q: 1869 ] MRCPass - 2010 January 

# A 60-year-old man is admitted with 
chest pain to the Emergency Department. He 
has a past medical history of type 2 diabetes, 
hypertension and high cholesterol. His regular 
medications includes simvastatin, bisoprolol, 
glibenclamide and metformin. An ECG shows 
ST elevation in the anterior leads and he is 
referred for primary angioplasty. Following 
the procedure, he was transferred to the 
Coronary Care Unit (CCU). He has a blood 
glucose measurement of 15 mmol/l. 

Which drug regime should be commenced? 

1- Continue metformin and glibenclamide at 
same dose 

2- Stop metformin and increase dose of 
glibenclamide 

3- Subcutaneous insulin: basal-bolus regime 

4- Subcutaneous insulin: biphasic insulin 
regime 

5- Intravenous sliding scale insulin 

Answer & Comments 

Answer: 5- Intravenous sliding scale insulin 

It has been shown from previous trials (e.g. 
DIGAMI study) that insulin based glucose 
management leads to improved outcomes in 


Dr. Kholid Yusuf El-Zohry - Sohog Teaching Hospital (01118391123) 

Ref MRCPass OE OE 2012 PasTest 2009 PassMedicine 2009 PasTest Exam ReviseMRCP 



740 

























El-zohry MRCP Questions Bank (Port 1) - 2013 


(For my personal use) 


type 2 diabetes post myocardial infarction. 
Good glycaemic control can be achieved with 
a sliding scale insulin regime or Glucose Insulin 
Potassium (GIK) regime. 

^ [ Q: 1870 ] MRCPass - 2010 January 

n - 

# A 30 year old lady has recurrent 
episodes of lip sw elling and is suspected of 
having hereditary angioedema. 

Which of the following is deficient in patients 
with the condition? 

1- C3 

2- C6 

3- Cl esterase inhibitor 

4- Heat shock protein type 1 

5- Histamine degradation protein (HDP) 

Answer & Comments 

Answer: 3- Cl esterase inhibitor 


On examination, he was afebrile, his blood 
pressure. Blood pressure was 130/80 mm Hg 
and pulse was 72 per minute and regular. He 
had partial ptosis of his right eye, conjunctival 
congestion and miosis. His fundoscopy, visual 
acuity and eye movements were normal. The 
rest of his neurological examination was 
normal. There was no carotid bruit. There 
were no signs of meningism nor a rash. 

What investigation should be done? 

1- Skull X ray 

2- MRI of the head 

3- MRI and MRA of the neck 

4- CT head 

5- EEG 

Answer & Comments 

Answer: 3- MRI and MRA of the neck 
The diagnosis is Horner's syndrome. 


In Cl esterase inhibitor deficiency, sw elling of 
the face, stridor and limbs can develop. 

In Cl esterase inhibitor deficiency, the 
complement factors C2 and C4 are low and Cl 
esterase inhibitor level is also low . Episodes 
can be precipitated by trauma or even an 
allergen such as peanuts. 


This patient needs to have carotid artery 
dissection excluded due to the acute nature of 
the presentation and conjunctival congestion. 
However, a space occupying lesion, a brain 
stem CVA, trauma to the neck and also 
Pancoasts' tumour may all cause Horner's 
syndrome. 


[ Q: 1871 ] MRCPass - 2010 January 

A 39-year-old man presented with a 
1 week history of right sided neck pain with 
associated headache, gradual in onset and not 
associated with visual symptoms. The pain 
had no diurnal variation, was not associated 
with premonitory symptoms, fever or 
vomiting. There were no known precipitating 
factors and no history of trauma. 

His wife also mentioned that his right eye 
looked funny. There was no associated limb 
weakness or sensory symptoms. He was 
started on Sumatriptan by the general 
practitioner but it did not relieve the pains. 



[ Q: 1872 ] MRCPass - 2010 January 

A 72 years old woman was admitted 
with complaints of anorexia, nausea and 
lethargy for last 4 months. She had history of 
fall 12 months back after which she sustained 
mild compression fracture of LI vertebra. 
After the fall, she has had persistent 
backaches. On examination, she was pale, BP 
was 160/90 mmHg and bilateral pedal 
oedema. There was tenderness over the 
upper lumbar region. 

Blood tests showed: 

Hb 10.5 g/dl, WCC 7 x 10 9 /l, platelets 220 x 
10 9 /l, ESR 90 mm/hr, sodium 135 mmol/I, 
potassium 4.2 mmol/l, urea 16 mmol/I, 



Dr. Khalid Yusuf El-Zohry - Sohag Teaching Hospital (01118391123) 

Ref MRCPass OE OE 2012 PasTest 2009 PassMedicine 2009 PasTest Exam ReviseMRCP 































El-zohry MRCP Questions Bank (Port 1) - 2013 


(For my personal use) 


creatinine 240 |imol/l. IgA 3.2 (0.5-4.0) g/l , 
IgG 23 (5.0-13.0) g/l, IgM 2.3 (0.3-2.2) g/l. 

Routine urine examination showed urine 
albumin trace, urine protein/creatinine ratio 
2.7 and urinary Bence Jones protein was 
positive. 

What is the likely diagnosis? 

1- Waldenstrom's macroglobulinaemia 

2- Multiple myeloma 

3- Chronic myeloid leukaemia 

4- Acute lymphocytic leukaemia 

5- Osteoporosis 


Answer & Comments 

Answer: 2- Multiple myeloma 

The presence of unexplained anemia, kidney 
dysfunction, a high erythrocyte sedimentation 
rate (ESR) and a high serum paraprotein with 
bence jones proteinuria suggests multiple 
myeloma. 


[ Q: 1873 ] MRCPass - 2010 January 

A 20 year old woman presented 6 
hours after taking 30g of Paracetamol. 

Which of following factors is likely to predict 
an increased risk of hepatotoxicity from 
Paracetamol? 

1- Anorexia nervosa 

2- Consumption of 20 units of alcohol since 
taking Paracetamol 

3- Gilbert's disease 

4- Ingestion of Amitriptyline 

5- Smoking 20 cigarettes per day 



Answer & Comments 

Answer: 1- Anorexia nervosa 

High risk groups in paracetamol overdose 
include malnourished patients (anorexia 
nervosa/bulimia nervosa), patients taking 


enzyme inducing drugs (eg carbamazepine, 
phenytoin rifampicin St John's Wort), patients 
induced liver enzymes due chronic ethanol 
abuse HIV positive patients. 

[ Q: 1874 ] MRCPass - 2010 January 

A 50 year old patient with polycystic 
kidneys is undergoing investigations for a 
renal transplant. 

His blood group is O positive. His brother who 
is 45 year old, is being considered as a 
transplant donor. The brother has a normal 
ultrasound of the kidneys with no renal cysts 
and is blood group A positive. 

Why is he unsuitable to become a kidney 
donor? 

1- Risk of developing polycystic kidneys 

2- ABO incompatibility 

3- Rhesus incompatibility 

4- Risk of CMV reaction 

5- HLA incompatibility leading to acute 
rejection 



Answer & Comments 

Answer: 2- ABO incompatibility 

There are glycoproteins in both groups A and 
B which may stimulate antibodies tow ards 
them. 

People with blood group A cells have 
antibodies to B, a donor with group B blood is 
not compatible with a group A recipient. 
Similarly, those with blood group B have 
antibodies to group A, indicating a type A 
donor is not compatible with a type B 
recipient. Individuals with type AB cells lack 
such antibodies and are therefore compatible 
with any potential donors (with regard to 
blood type matching). 

As in the case above, a patient with blood 
group O will have antibodies against both 
those with group A and B and therefore 
require blood group O kidney donors. 


Dr. Khalid Yusuf El-Zohry - Sohag Teaching Hospital (01118391123) 

Ref MRCPass OE OE 2012 PasTest 2009 PassMedicine 2009 PasTest Exam ReviseMRCP 



742 



























El-zohry MRCP Questions Bank (Port 1) - 2013 


(For my personal use) 


Rhesus grouping is not as important in 
transplant blood group matching but a rhesus 
positive patient (particular Rhesus D 
subgroup) may cause a rhesus negative 
patient to generate antibodies and cause 
subsequent rejection. 

HLA matching is less important than ABO 
compatibility in transplantation. 


[ Q: 1875 ] MRCPass - 2010 January 

A 38-year-old man presented with 
visual symptoms which were new . He has no 
relevant past medical history. 

On examination, he had ptosis, miosis and 
anhydrosis of his right eye, conjunctival 
congestion and miosis. 

What investigation should be done? 

1- Chest X ray 

2- CT scan of the head 

3- MRI of the head 

4- Ultrasound of the neck 

5- EEG 



Answer & Comments 

Answer: 1- Chest X ray 

The diagnosis is Horner's syndrome. The first 
investigation should be a chest X ray which 
will Pancoasts' tumour or a cervical rib which 
can cause compression of the sympathetic 
nervous supply. 


Dr. Khalid Yusuf El-Zohry - Sohag Teaching Hospital (01118391123) 

OE OE 2012 PosTest 2009 PassMedicine 2009 PosTest Exom 


ReviseMRCP 


Ref MRCPoss 


743 




















El-zohry MRCP Questions Bank (Port 1) - 2013 


(For my personal use) 


[ Q: 1876 ] MRCPass - 2010 May 

A 43 year old female patient 
presented in to clinic with a generalised 
blistering rash on the arms and legs. Clinical 
examination revealed tense skin blisters with 
some generalised desquamation. 

What should be done to confirm the 
diagnosis? 

1- Skin scrapings 

2- Skin swab for microscopy and sensitivity 

3- Blister fluid to be sent for viral culture 

4- Immunofluorescence of skin biopsy 

5- Trial of steroids 



Answer & Comments 

Answer: 4- Immunofluorescence of skin biopsy 

The diagnosis is bullous pemphigoid, which is 
a chronic, autoimmune, subepidermal, 
blistering skin disease that rarely involves 
mucous membranes. 

Bullous pemphigoid is characterized by the 
presence of immunoglobulin G (IgG) 
autoantibodies in the hemidesmosomal area. 
This manifests as tense blisters. Direct 
immunofluorescence of a skin biopsy usually 
demonstrate IgG and complement C3 
deposition in a linear band at the dermal- 
epidermal junction. 


[ Q: 1877 ] MRCPass - 2010 May 

A 52-year-old female presented with 
two month history of an itchy rash which 
appeared on her wrist and on the upper arms. 
There was no relevant past medical, family or 
drug history. 

On examination there were annular flat 
violaceous lesions on the arms. 

Where else are lesions likely to be found? 



3- Plantar surface 

4- Perineal area 

5- Scalp 


Answer & Comments 

Answer: 2- Buccal mucosa 

There is an increased incidence of lichen 
planus in several diseases in which there is 
autoimmune phenomena. 

Lichen planus (LP) is a pruritic, papular 
eruption characterized by its violaceous color; 
polygonal shape; and, sometimes, fine scale. 

The initial lesion is usually located on the 
flexor surface of the limbs, such as the wrists. 

After a week or more, a generalized eruption 
develops with maximal spreading within 2-16 
weeks. Oral lesions such as on the buccal 
mucosa may be present and either 
asymptomatic or have a burning sensation, or 
they may even be painful if erosions are 
present. Characteristic fine, white lines, called 
Wickham stria, are often found on the 
papules. 


[ Q: 1878 ] MRCPass - 2010 May 

A patient was given magnesium 
sulphate for treatment of acute asthma. Given 
that the drug's half life is 4 hours, what 
proportion of the drug is left after 20 hours? 

1- 3.125% 

2- 6.25% 

3- 25% 

4- 66% 

5- 97% 



Answer & Comments 


Answer: 1- 3.125% 


1- Behind the ears 

2- Buccal mucosa 


The proportion eliminated are: 
4 hrs - 50% 


Dr. Khalid Yusuf El-Zohry - Sohag Teaching Hospital (01118391123) 

Ref MRCPass OE OE 2012 PasTest 2009 PassMedicine 2009 PasTest Exam 




ReviseMRCP 

745 




























El-zohry MRCP Questions Bank (Part 1) - 2013 


(For my personal use) 


4 hrs - 25% 

4hrs -12.5% 

4hrs - 6.25% 

4 hrs-3.125% 

It adds up to 96.875 eliminated, or 3.125% of 
the drug left. 

^ [ Q: 1879 ] MRCPass - 2010 May 

fm - 

# A patient is being worked up for 
renal transplantation. 

Which one of the following HLA compatibility 
is the most important? 

1- A 

2- B 

3- C 

4- D 

5- G 


Which one of the following is the most 
appropriate next investigation? 

1- Ankle-brachial pressure index 

2- Lower limb arteriogram 

3- Right leg venogram 

4- Venous duplex ultrasound scan 

5- Swab of the ulcer 

Answer & Comments 

Answer: 1- Ankle-brachial pressure index 

This is likely to be a venous ulcer due to 
venous insufficiency. 

However, excluding arterial disease with the 
anklebrachial pressure index is important to 
ensure that any further treatment 
(compression bandaging) will not exacerbate 
symptoms. For cases with a clinical suspicion 
of DVT then duplex ultrasound is indicated. 


Answer & Comments 

Answer: 4- D 

ABO blood group matching is the most 
important, and HLA matching is a relatively 
minor predictor of transplant outcomes. 

However, among HLA matches, DR matching 
has a greater effect than that of B or A. A 
study found that HLA-DR mismatches (and the 
number of rejection episodes) correlated with 
poor long-term survival 


[ Q: 1880 ] MRCPass - 2010 May 

A 65-year-old woman presented 
with an ulcer over the left ankle, which had 
developed over the previous 6 months. She 
had a history of right deep vein thrombosis 
five years previously. On examination she had 
a superficial slough-based ulcer, 4 cms in 
diameter, over the medial malleolus with no 
evidence of cellulitis. 




[ Q: 1881 ] MRCPass - 2010 May 

A 25 year old man has pain in his 
elbow and arm. He describes an injury whilst 
using a hammer. On examination, there is loss 
of sensation in the medial (little finger and 
half of ring finger) of the hand and loss 
sensation in medial side of the forearm, after 
elbow injury. 

What is the likely injury? 

1- Axillary nerve injury 

2- Median nerve injury 


3- Ulnar nerve injury 

4- Lateral epicondylitis 

5- Tennis Elbow 


Answer & Comments 

Answer: 3- Ulnar nerve injury 

Sensation is supplied by the ulnar nerve to the 
fifth finger and the ulnar part of the fourth 
finger. 


Dr. Khalid Yusuf El-Zohry - Sohag Teaching Hospital (01118391123) 

Ref MRCPass OE OE 2012 PasTest 2009 PassMedicine 2009 PasTest Exam ReviseMRCP 



746 




























El-zohry MRCP Questions Bank (Port 1) - 2013 


(For my personal use) 


Claw hand is typical of ulnar nerve injury. 

The axillary nerve supplies the deltoid and 
teres minor as well as the skin over the 
deltoid. 

The median nerve supplies the lateral two 
lumbricals, opponens pollicis, abductor pollicis 
brevis and flexor pollicis brevis (LOAF). Flexion 
of the fingers and thumb abduction is supplied 
by the median nerve. 

Lateral Epicondylitis, or tennis elbow is caused 
by inflammation of the common extensor 
origin, at the lateral epicondyle of the 
humerus. It causes pain in the elbow during 
movement. 



[ Q: 1882 ] MRCPass - 2010 May 

A healthcare worker gets a deep 
needlestick injury during phlebotomy. This 
was from an asymptomatic HIV positive 
patient who is not on anti-retroviral therapy. 


What is the next step? 

1- Give antiretroviral therapy with 3 drugs for 
one month 


2- Give antiretroviral therapy with 3 drugs for 
three months 


3- HIV antibody test immediately 

4- Intravenous immunoglobulins 

5- No treatment required 


Answer & Comments 

Answer: 1- Give antiretroviral therapy with 3 
drugs for one month 

In this case the risk is not well known (the viral 
load and CD4 count of the patient who was 
HIV positive). 

It is therefore best to give a basic regimen of 
post exposure prophylaxis. The basic regimen 
consists of : 4 weeks of zidovudine (600 mg/d 
in 2-3 divided doses) , lamivudine (150 mg 
twice daily) and one other agent. Exposure to 
patients with high risks such as full blow n 


AIDS or high viral loads should be with an 
extended regimen. 



[ Q: 1883 ] MRCPass - 2010 May 

A 50-year-old man with a past 
medical history of hypertension was 
investigated as an outpatient due to 
symptoms of lethargy and nausea. Blood 
results reveal : sodium 137 mmol/I, potassium 
4.5 mmol/l, urea 22 mmol/l, creatinine 340 
pmol/l. Ultrasound demonstrated a left kidney 
of 8.9 cm and right at 9.3 cm. 


What is the best investigation to perform 
next? 


1- MR angiography 

2- Renal angiography 

3- Repeat renal ultrasound 

4- Renal biopsy 

5- CT scan of the kidney 


Answer & Comments 

Answer: 1- MR angiography 

The history of hypertension and small kidneys 
suggests renovascular disease or renal artery 
stenosis. 

The best follow on tests are MR angiogram or 
isotope (MAG-3) nuclear imaging to evaluate 
renal arterial anatomy and function as these 
tests are less invasive. Once the diagnosis is 
certain, angiography with a view to 
angioplasty should then be considered. 


[ Q: 1884 ] MRCPass - 2010 May 

A 68-year-old woman was admitted 
to hospital because of facial weakness and 
limb weakness. The patient noticed weakness 
of the right arm and leg and diplopia on 
waking in the morning. She had been diabetic 
and hypertensive for the past 20 years. Her 
medications included gliclazide, metformin 
and insulin. On admission she was alert and 



Dr. Khalid Yusuf El-Zohry - Sohag Teaching Hospital (01118391123) 

Ref MRCPass OE OE 2012 PasTest 2009 PassMedicine 2009 PasTest Exam 




ReviseMRCP 

747 


























El-zohry MRCP Questions Bank (Port 1) - 2013 


(For my personal use) 


had a blood pressure of 155/75 mm/Hg. 
Cranial examination revealed a left 3rd nerve 
palsy, dilated pupil on the left side and normal 
ocular fundi. Examination of motor system 
showed right-sided spastic weakness with 
grade 4/5 power. Deep tendon reflexes were 
brisk on the right side with upgoing plantar 
response. 

Where is the lesion? 


5- Autoimmune screen 

Answer & Comments 

Answer: 2- X ray of the sacroiliac joints 

This case is most likely ankylosing spondylitis, 
which typically causes restricted spinal 
movements due to chronic inflammatory 
changes. 


1- Medial longitudinal fasciculus 

2- Posterior cerebellar 

3- Midbrain 

4- Pons 


Pain and stiffness in the lower back or 
buttocks, especially in the morning is typical of 
sacroilitis. X rays will help to confirm the 
diagnosis. 


5- Medulla 

[ Q: 1886] MRCPass-2010 May 


0 Which one of the 

following 
is global 

Answer & Comments 

nerve/roots is affected if there 

Answer: 3- Midbrain 

wasting of small muscles of hand? 


- „ . r .1 i . ^ , 1- Median nerve 

Most of the lesions causing 3rd cranial nerve 

nucleus are from dorsal midbrain infarction. 2- Radial nerve 


This patient has Weber syndrome, which 
results from a slightly more ventral lesion at 
the level of the third cranial nerve fascicles in 
the mid brain, with involvement of the 
cerebral peduncle giving rise to contralateral 
hemiplegia or hemiparesis along with 
ipsilateral third cranial nerve palsy. 


3- Ulnar nerve 

4- C7 

5- T1 


Answer & Comments 


Answer: 5- T1 


[ Q: 1885 ] MRCPass - 2010 May 

A 30 year old man has had a year's 
history of bilateral hip pains and back pains. 
There is no past medical history of trauma to 
the back. Non steroidal anti-inflammatory 
drugs helped to relieve his symptoms. He also 
has reduced chest expansion and is unable to 
touch the feet with his fingers when bending. 

What is the best investigation to confirm the 
diagnosis? 

1- CT scan of the chest 


Global wasting of hand indicate median and 
ulnar nerve lesions; probably, with damage to 
T1 root. In the hand, the median nerve 
supplies the lateral two lumbricals, opponens 
pollicis, abductor pollicis brevis, and flexor 
pollicis brevis. 

Wasting of the interossei (prominent guttering 
of the back of the hand), of the web space 
between thumb and index finger, and 
softening and flattening of the hypothenar 
eminence with sparing of abductor pollicis 
brevis indicates an ulnar nerve lesion. 


2- X ray of the sacroiliac joints 

3- Lung function tests 

4- HLA B27 testing 



[ Q: 1887 ] MRCPass - 2010 May 
A 16-year-old girl who had been 


Dr. Khalid Yusuf El-Zohry - Sohag Teaching Hospital (01118391123) 

Ref MRCPass OE OE 2012 PasTest 2009 PassMedicine 2009 PasTest Exam ReviseMRCP 



748 

































El-zohry MRCP Questions Bank (Port 1) - 2013 


(For my personal use) 


prescribed theophylline tablets 225 mg twice 
a day took an overdose of 40 tablets, 
following a stressful event. 

She presented to hospital 1 hour and 30 
minutes after the overdose. Her BP was 
100/70 and she had a sinus tachycardia with a 
heart rate of 130. 

What is the best management option? 

1- Activated charcoal 

2- Gastric lavage 

3- Whole bowel irrigation 

4- Esmolol 

5- Haemodialysis 


[ Q: 1888 ] MRCPass - 2010 May 

A 16-year-old boy was admitted to 
hospital after a blackout at dentist. His mother 
described how he looked pale and then 
blacked out as a dentist began performing a 
filling when he was sat in a chair. His arm 
jerked for a few seconds and following the 
event he was incontinent. He aw oke after a 
minute was oriented. He was not confused 
but did not recall what happened. 

What is the likely diagnosis? 

1- Complex partial seizure 

2- Pseudoseizure 

3- Stokes-Adams attack 



Answer & Comments 


Answer: 1- Activated charcoal 


4- Tonic-clonic seizure 

5- Vasovagal syncope 


It is too late for gastric lavage, hence activated 
charcoal is the best option here. 

Theophylline is absorbed rapidly and 
completely after oral administration. 
Therapeutic serum levels range from 10-20 
mcg/mL. Toxic levels are considered to be 
higher than 20 mcg/mL; However, adverse 
effects may be evident within the normal 
therapeutic range. Severe complications 
including cardiac dysrhythmias, seizures, and 
death can be observed with the levels 

of 80-100 mcg/mL. The therapeutic options 
are as below : 


Answer & Comments 

Answer: 5- Vasovagal syncope 

Vasovagal syncope is common during a 
stressful event such as dental procedures, 

There is no post event confusion and he 
recovered very quickly, hence this makes a 
genuine seizure unlikely. The description of 
pallor, brief syncope and also shaking of arms 
and limbs is consistent with vasovagal syncope 
which is commonly mistaken for a seizure. It 
can be associated with urinary incontinence 
which is not specific for epileptic seizures. 


Gastric lavage (unless contraindicated) if the 
patient has recently (<1 h) ingested a 
significant amount or a sustained-release 
preparation of theophylline. 

Multidose activated charcoal (MDAC) 
enhances elimination of theophylline. It is a 
very effective method of elimination, and it is 
considered the mainstay treatment of 
theophylline toxicity. 

Consider whole-bow el irrigation (WBI) in 
patients with exposure to sustained-release 
theophylline preparations. 



[ Q: 1889 ] MRCPass - 2010 May 

A 32 year old female patient was 
referred by the GP for assessment of unequal 
sized pupils. On examination, it was found 
that the right pupil was larger than the left. 
The pupillary reflex on the right eye was also 
sluggish both to light and accomodation. After 
performing the accommodation reflex, the 
pupil on the right was smaller than the left for 
several minutes. Eye movements and 
fundoscopy were normal. 


What is the diagnosis? 


Dr. Khalid Yusuf El-Zohry - Sohag Teaching Hospital (01118391123) 

Ref MRCPass OE OE 2012 PasTest 2009 PassMedicine 2009 PasTest Exam ReviseMRCP 



749 


























El-zohry MRCP Questions Bank (Port 1) - 2013 


(For my personal use) 


1- Argyll Robertson pupil 

2- Horner's syndrome 

3- Adie Holmes pupil 

4- Myasthenia gravis 

5- 3rd nerve palsy 


Answer & Comments 

Answer: 3- Adie Holmes pupil 

Holmes-Adie syndrome (HAS) is a neurological 
disorder affecting the pupil of the eye and the 
autonomic nervous system. 

It is characterised by one eye with a pupil that 
is larger than normal and constricts slow ly in 
bright light (tonic pupil), along with the 
absence of deep tendon reflexes, usually in 
the Achilles tendon. 

Once the pupil has constricted it remains small 
for an abnormally long time (tonic pupil), 
hence in this case after the pupils were 
constricted following the accommodation test, 
the right adie pupil remained smaller than the 
left. 


[ Q: 1890 ] MRCPass - 2010 May 

A 30-year-old Caucasian woman 
admitted has a history of widespread, pruritic, 
erythematous skin rash, joint pains and renal 
disease. She recently had a baby with 
congenital heart block. Laboratory 
investigations revealed mild leucopenia (white 
cell count 3.25xl09/ml) and 
thrombocytopenia (platelets 140xl09/ml). 
Erythrocyte sedimention rate was increased 
(65 mm/h). 

Which antibody is likely to be positive? 

1- Anti Jo 1 

2- Anti double stranded DNA 

3- Anti Ro 

4- Anti centromere 

5- ANCA 



Answer & Comments 

Answer: 3- Anti Ro 

The anti Ro antibody is associated with 
Sjogren's syndrome, SLE and neonatal lupus. 

In neonatal lupus, congenital heart block often 
occurs in babies born to women with the 
antibody (both anti Ro and anti La antibodies). 


[ Q: 1891 ] MRCPass - 2010 May 

A 52 year old patient with 
syringomyelia has a syrinx demonstrate 
unilaterally at the level of C4-T1. 

Whot is the likely manifestation for the 
patient? 

1- Loss of pinprick and vibration sense in the 
hand on the contralateral 

2- Loss of pain and temperature sensation in 
the hand on contra lateral side 

3- Winging of the scapula in the contralateral 
side 

4- Hand weakness in the contralateral side 

5- Leg weakness in the contralateral side 



Answer & Comments 

Answer: 2- Loss of pain and temperature 
sensation in the hand on contra lateral side 

Syringomyelia is a chronic disorder 
characterized by the presence of a 
longitudinal, fluid filled cavities (syrinx) within 
the spinal cord. 

The question tests know ledge of anatomical 
pathw ays. This is a one sided lesion of the 
spinal cord. Such a lesion will cause: 

a unilateral motor deficit below the lesion (the 
corticospinal tract decussates at the medulla) 

a unilateral reduction in vibration sense 
(dorsal columns decussates at the medulla) 


Dr. Khalid Yusuf El-Zohry - Sohag Teaching Hospital (01118391123) 

Ref MRCPass OE OE 2012 PasTest 2009 PassMedicine 2009 PasTest Exam ReviseMRCP 




























El-zohry MRCP Questions Bank (Port 1) - 2013 


(For my personal use) 


a contralateral pain and temperature sensory 
deficit (the spinothalamic tract decussates at 
the same level at the spinal cord). 

Winging of the scapula (C5-7) and hand 
weakness may both occur, but in this case is 
on the wrong side. 

[ Q: 1892 ] MRCPass - 2010 May 

An 18-year-old woman is admitted 
to the resuscitation room with respiratory 
distress and a GCS score of 3/ 15. 

She had been found unconscious at home. 
Blood gases and blood test results taken on 
admission show the following: 

pH 6.92 

pC02- 2.5 kPa 

p02 -16.8 kPa 

Na+ 143 mmol/I 

K+ 4.6 mmol/I 

Chloride 101 (95-107) mmol/I 

Bicarbonate 3.2 (20-28) mmol/l 

Urea 5.2 mmol/l 

Creatinine 60 |imol/l 

Glucose 7 mmol/l 

Methanol 523 mg/dl 

Whot is the best treatment option? 

1- Observation only 

2- Insulin sliding scale 

3- Haemodialysis 

4- Ethanol 

5- Fomepizole 



Answer & Comments 

Answer: 3- Haemodialysis 

In this case there is severe neurological signs 
and the patient has a high methanol level with 
metabolic acidosis, hence haemodialysis is the 
best option. 


Methanol is a commonly used organic solvent, 
the ingestion of which can cause significant 
toxicity. It is a constituent in many 
commercially available industrial solvents and 
in poorly adulterated alcoholic beverages. 
Toxicity usually results in metabolic acidosis, 
neurologic damage (ataxia, coma), blindness 
(formic acid accumulates within the optic 
nerve). 

Antidote therapy is directed tow ards delaying 
methanol metabolism until the methanol is 
eliminated from the system either naturally or 
via dialysis. This is often accomplished in 2 
ways: ethanol or fomepizole. Ethanol is also 
metabolized by ADH, and the enzyme has 10- 
20 times higher affinity for ethanol compared 
with methanol. 

Fomepizole is also metabolized by ADH; 
However, its use is limited because of high 
costs and lack of availability. 

Hemodialysis can easily remove methanol and 
formic acid. Indications include (1) greater 
than 30 mL of methanol ingested, (2) serum 
methanol level greater than 20 mg/dL, (3) 
observation of visual complications, and (4) no 
improvement in acidosis despite repeated 
sodium bicarbonate infusions. 

[ Q: 1893 ] MRCPass - 2010 May 

A 35 year old woman has 
longstanding breathlessness. Previous 
investigations have shown that she has 
multiple chronic pulmonary emboli. 

Which one of the following would be a 
consistent finding? 

1- Normal pulmonary wedge pressure 

2- Increased inspiratory reserve volume 

3- Decreased transfer factor 

4- Increased lung compliance 

5- low FEV1/FVC ratio (obstructive picture) 



Dr. Kholid Yusuf El-Zohry - Sohog Teaching Hospital (01118391123) 

Ref MRCPass OE OE 2012 PasTest 2009 PassMedicine 2009 PasTest Exam 




ReviseMRCP 

751 




















El-zohry MRCP Questions Bank (Port 1) - 2013 


(For my personal use) 


Answer & Comments 

Answer: 3- Decreased transfer factor 

Transfer factor and alveolar volume are 
significantly lower in patients with pulmonary 
emboli. 

Pulmonary wedge pressure is increased in 
pulmonary veno-occlussive disease. 

^ [ Q: 1894 ] MRCPass - 2010 May 

h - 

m A 25 year old woman presented with 
confusion and severe headache following a 
night out at a club. On examination, the 
following were found heart rate: 150 
beats/min with sinus tachycardia on the 
monitor, blood pressure: 160/90 mmHg, 
respiratory rate: 30, oxygen saturation by 
pulse oximetry was 98%. Her GCS score was 
14/15. 

Which one of the following conditions is most 
likely to couse this? 

1- Cluster headache 

2- Meningitis 

3- Paracetamol overdose 

4- Ecstasy overdose 

5- Venous sinus thrombosis 


tablets for depression and has recently been 
found to be hypertensive with a blood 
pressure of 170 / 95 mmHg. 

Whot should be the preferred drug? 

1- Verapamil 

2- Doxazosin 

3- Amlodipine 

4- Hydrochlorothiazide 

5- Ramipril 

Answer & Comments 

Answer: 3- Amlodipine 

Diuretics may increase the amounts of lithium 
in the body (hydrochlorothiazide, 
acetazolamide, furosemide). 

Other antihypertensive drugs which may 
increase or worsen the side effects of lithium 
e.g. some calcium channel blockers (i.e. 
verapamil, diltiazem) and Angiotensin 
converting enzyme inhibitors (i.e. enalapril, 
captopril, ramipril). 

Amlodipine or doxazosin can both be used, 
but amlodipine should be picked first in this 
instance. 



^ [ Q: 1896] MRCPass-2010 May 

Answer & Comments 

mi 

—1 # A 65 year old female with a history 


Answer: 4- Ecstasy overdose 

MDMA (ecstasy) is a popular drug of abuse for 
its combined properties inherent to those of 
amphetamines and hallucinogens. 


of type 2 diabetes, hyperparathyroidism and 
osteoarthritis has recurrent knee pains. On 
examination, she was apyrexial and found to 
have a right sided knee effusion. The effusion 
was aspirated. 


Symptoms of MDMA ingestion include an 
altered mental status, tachycardia, tachypnea, 
sweating and hyperthermia. Severe cases can 
progress to rhabdomyolysis, acute renal 
failure, cardiac collapse, and disseminated 
intravascular coagulation. 


Whot is the likely finding in synoviol fluid 
o no lysis? 

1- Star shaped crystals 

2- Yellow maltese cross crystals 

3- Positively birefringent rhomboid crystal 



[ Q: 1895 ] MRCPass - 2010 May 
A 45 year old man is on lithium 


4- Positively birefringent needle shaped 
crystal 

5- Negatively birefringent crystal 


Dr. Kholid Yusuf El-Zohry - Sohog Teaching Hospitol (01118391123) 

Ref MRCPass OE OE 2012 PasTest 2009 PassMedicine 2009 PasTest Exam ReviseMRCP 

































El-zohry MRCP Questions Bank (Port 1) - 2013 


(For my personal use) 


Answer & Comments 

Answer: 3- Positively birefringent rhomboid 
crystal 

In the case of an acute attack of arthritis 
(usually monoarthritis), the joint should alw 
ays be aspirated. 

The differential diagnosis lies between septic 
arthritis, gout and pseudogout. Gout and 
pseudogout are the 2 most common crystal- 
induced arthropathies. These are debilitating 
illnesses in which pain and joint inflammation 
are caused by the deposition of crystals within 
the joint space. Many cases of pseudogout are 
idiopathic, but it has also been associated with 
aging, trauma, and many different metabolic 
abnormalities like hyperparathyroidism (as in 
this case) and hemochromatosis. Pseudogout 
is an inflammation caused by the deposition of 
positively birefringent rhomboid shaped 
calcium pyrophosphate dihydrate (CPPD) 
crystals. Gout crystals are negatively 
birefringent. 


[ Q: 1897 ] MRCPass - 2010 May 

A 62 year old man fell over and lost 
consciousness for several minutes. He was 
brought to hospital and was initially alert. 
However, 2 hours later, whilst he was being 
admitted, he complained of a headache and 
his GCS deteriorated from 15 to 8. 



sudden deterioration in conscious level with 
an associated headache is a subdural 
haematoma. 

This patient needs an urgent CT scan in view 
of the deterioration. 



[ Q: 1898 ] MRCPass - 2010 May 

A 62-year-old man with a recent 
history of colorectal carcinoma presented to 
hospital with exertional dyspnea, fever and 
malaise. He had altered bow el habit for 
several weeks prior to his admission. On 
examination his temperature was 39,5°C , 
there was sinus tachycardia of 105 beats/min 
and respiratory rate was 32/minute. He had 
several splinter haemorrhages, there were 
clear breath sounds and a systolic murmur 
was heard along the apex and diastolic 
murmur at the right second intercostal area. 


Whot is the most likely infective organism? 


1- Streptococcus mitis 

2- Streptococcus bovis 

3- Streptococcus viridans 

4- Staphylococcus aureus 

5- Streptococcus milieri 


Answer & Comments 


Answer: 2- Streptococcus bovis 


Whot is the most likely diagnosis? 

1- Subarachnoid haemorrhage 

2- Subdural haematoma 

3- Extradural haematoma 

4- Diffuse axonal injury 

5- Epileptic seizure 


Answer & Comments 

Answer: 2- Subdural haematoma 

The patient who fell over may have injured his 
head and the most likely cause of a further 


Strep bovis can be found in the normal flora of 
human gastrointestinal system. 

S. bovis endocarditis is often associated with 
colonic carcinoma. It is the second common 
Streptococci causing bacterial endocarditis. 


[ Q: 1899 ] MRCPass - 2010 May 

A 70 year old man is assessed in the 
pre operative assessment clinic for a hip 
replacement. During the preoperative 
assessment was noted to have a raised 
lymphocyte count. On examination, he had 
lymphadenopathy palpable in the cervical and 



Dr. Kholid Yusuf El-Zohry - Sohog Teaching Hospital (01118391123) 

Ref MRCPass OE OE 2012 PasTest 2009 PassMedicine 2009 PasTest Exam ReviseMRCP 





































El-zohry MRCP Questions Bank (Port 1) - 2013 


(For my personal use) 


axillary area. A spleen was palpable 3 cm 
below the costal margin. 

His bloods were as follows: 

Hb 11.6 g/dl 

Pit 164 x 10 9 /l 

WBC 45.2 x 1071 

What is the likely diagnosis? 

1- Acute lymphocytic leukaemia 

2- Acute myeloblastic leukaemia 

3- Chronic lymphocytic leukaemia 

4- Chronic myeloblastic leukaemia 

5- Multiple myeloma 


Answer & Comments 

Answer: 3- Chronic lymphocytic leukaemia 

Chronic lymphocytic leukemia (CLL), is the 
most common type of leukemia and it affects 
B cell lymphocytes. 

Most (>75%) people newly diagnosed with CLL 
are over the age of 50, and the majority are 
men. Most people are diagnosed without 
symptoms as the result of a routine blood test 
that returns a high white blood cell count. 
Clinical signs of lymphadenopathy and 
splenomegaly are associated. 


[ Q: 1900 ] MRCPass - 2010 May 

A 40-year-old female patient, an ex¬ 
smoker with an 8-pack-year smoking history 
and severe pulmonary emphysema of early 
onset. Her father has a diagnosis of 1- 
antitrypsin (AAT) deficiency. Her serum 
antitrypsin levels are measured to be 
approximately 15% of the normal range. 

Which genotype is most likely? 

1- ZZ 

2- MZ 

3- SZ 

4- MM 



5- SS 


Answer & Comments 

Answer: 1- ZZ 

The commonest phenotype is Protease 
Inhibitor (Pi) MM (90% of the population have 
this). 

These individuals produce normal amounts of 
alphal-antiprotease. The most common form 
of AAT deficiency is associated with allele Z, or 
homozygous PiZ (ZZ). Serum levels of AAT in 
these patients are about 3 - 7 umol/L (10-15% 
of normal serum levels). Emphysema develops 
in most (but not all) individuals with serum 
levels less than 9 mmol/L. 


[ Q: 1901 ] MRCPass - 2010 May 

A 34 year old lady attends clinic for 
evaluation of pain or sw elling in the wrists, 
small joints of her hands or feet. 

She denied morning stiffness, subcutaneous 
nodules or sicca symptoms. Rheumatoid 
arthritis is suspected. In the process of disease 
evaluation, HLA class is considered. 

Which HLA type is associated? 

1- DR2 

2- DR4 

3- DR3 

4- B27 

5- DQ8 



Answer & Comments 

Answer: 2- DR4 

90% of patients with Rheumatoid arthritis 
have the cluster of markers known as the HLA- 
DR4/DR1 cluster, whereas only 40% of 
unaffected controls do. 

The HLA-DR4 gene, has also shown 
involvement in Lyme disease. 


Dr. Khalid Yusuf El-Zohry - Sohag Teaching Hospital (01118391123) 

Ref MRCPass OE OE 2012 PasTest 2009 PassMedicine 2009 PasTest Exam ReviseMRCP 





























El-zohry MRCP Questions Bank (Port 1) - 2013 


(For my personal use) 


Patients with HLA-DR4 are less likely to 
respond to antibiotics. 


the elbows, knees and ankles. 
Thrombocytopenia is also a common finding. 


[ Q: 1902 ] MRCPass - 2010 May 

A 52-year-old woman presents with 
lethargy and ankle oedema. She has a past 
medical history of hypertension and has 
recurrent joint pains. She currently takes 
ibuprofen and amlodipine. On examination 
she was pyrexial, had livedo reticularis. Her 
blood pressure was 175/100 mmHg. 

Investigations revealed: 

haemoglobin 12.2 g/dL (11.5-16.5) 

white cell count 8.2 x 10 9 /L (4-11) 

platelet count 83 x 10 9 /L (150-400) 

urea 5 mmol/l 

serum creatinine 155 umol/L 
urine dipstick analysis: 
blood + 
protein +++ 

Which one of following is the likely diagnosis? 

1- IgA nephropathy 

2- Rheumatoid arthritis 

3- Systemic lupus erythematosus 

4- Polyarteritis nodosa 

5- Wegener's granulomatosis 



Answer & Comments 

Answer: 3- Systemic lupus erythematosus 

This patient features of suggest Systemic lupus 
erythematosus (SLE) with possible 
antiphopholipid syndrome and nephrotic 
syndrome. 

SLE can present with many features including 
skin changes and vasculitic lesions, 
hypertension, renal involvement and alopecia. 
Livedo reticularis is a mottled, reticulated skin 
rash- usually on the lower extremities, around 



[ Q: 1903 ] MRCPass - 2010 May 

A 55-year-old man with a 10-year 
history of DM and hypertension, presents to 
the renal clinic with complaints of fatigue and 
shortness of breath. He is known to have 
chronic kidney disease and has complications 
of diabetic retinopathy and peripheral 
neuropathy. He has been gradually becoming 
more short of breath on exertion over the last 
6 months. His blood test results are: 


Hb 8.5 g/dl, MCV 85 fl, WCC 7x 10 9 /l, platelets 
220 x 10 9 /l, sodium 135 mmol/l, potassium 4.6 
mmol/l, urea 15 mmol/l, creatinine 310 
pmol/l, calcium 2.2 (2.25-2.7) mmol/l, 

phosphate 5 (0.8-8) pmol/l 

What is the best way of managing his 
symptoms? 

1- Physiotherapy to increase exercise 
tolerance 


2- Erythropoietin treatment 

3- Dialysis to improve renal function 

4- Improve glycaemic control 

5- Treat for pulmonary embolism 


Answer & Comments 

Answer: 2- Erythropoietin treatment 

This patient has anaemia secondary to chronic 
kidney disease, and patients who have 
symptomatic anaemia (in this case, shortness 
of breath limiting exercise tolerance) should 
be considered for Erythropoietin treatment. 

The renal association guidelines recommend 
that patients with CKD on ESA therapy should 
achieve haemoglobin between 10.5-12.5 g/dl. 

http://w ww 

.renal.org/Clinical/GuidelinesSection/Anaemia 
InCKD.aspx 


Dr. Khalid Yusuf El-Zohry - Sohag Teaching Hospital (01118391123) 

Ref MRCPass OE OE 2012 PasTest 2009 PassMedicine 2009 PasTest Exam 




ReviseMRCP 

755 























El-zohry MRCP Questions Bank (Port 1) - 2013 


(For my personal use) 


^ [ Q: 1904 ] MRCPass - 2010 May 

/ -1-I- 

m A 22-year-old man returns from 
holiday and presents to a clinic complaining of 
numerous scaly, hypopigmented lesions on 
the neck and upper trunk. 

What is the likely diagnosis? 

1- Chronic plaque psoriasis 


Answer & Comments 

Answer: 2- Praziquantel 

Although schistosomiasis is endemic in Africa, 
the Middle East, Latin America, and limited 
areas in Asia, the greatest risk of infection 
appears to come from exposure to freshw ater 
in east Africa. 


2- Discoid eczema 

3- Pityriasis rosea 

4- Pityriasis versicolor 

5- Seborrhoeic dermatitis 


Answer & Comments 

Answer: 4- Pityriasis versicolor 

Pityriasis versicolor is caused by a superficial 
fungal infection by pityrosporum ovale. 

It usually presents as slightly scaly 
hypopigmented lesions. Growth is encouraged 
by an increase in temperature, suntan oils and 
is commonly seen after a sunny holiday. 
Pityriasis rosea usually starts with a herald 
patch followed by small scaly lesions following 
rib lines. 


[ Q: 1905 ] MRCPass - 2010 May 

A 27 year old lady had recently 
travelled to the Uganda several months ago. 
She complained of fevers and headache. The 
GP found that she had microscopic 
haematuria on urine dipstick. A full blood 
count showed Hb 10.6g/dL, MCV 82.6fl, 
eosinophils 0.5 (0.04-0.44) x 10 9 /dL. 

What should be prescribed? 

1- Augmentin 

2- Praziquantel 

3- Fluconazole 

4- Mebendazole 

5- Caspofungin 



Infection results from penetration of intact 
skin by larvae liberated from snail 
intermediate hosts. 

Most infected travellers are asymptomatic 
and at low risk of complications because of 
low parasite burdens, although end-organ 
damage may result from egg deposition. 
Other presentations include sw immers' itch (a 
popular pruritic dermatitis at the time of 
exposure) and Katayama fever, which is 
associated with the migration of schistosome 
larvae throughout the body (symptoms 
include fever, chills, malaise, headache, cough, 
abdominal pain, diarrhoea, urticaria, and 
occasionally neurological disorders). 

Once egg deposition begins, clinical features 
include haematuria, haematospermia, 
urgency, frequency, terminal dysuria, 
salpingitis, prostatitis and genital ulcers (case 
history. More serious but rare complications 
are paraparesis, paraplegia and mass cerebral 
lesions. Immigrants and refugees from 
endemic countries are likely to have much 
heavier infection loads and may present with 
the complications of chronic fibrosis, such as 
ureteric obstruction and portal hypertension. 

The standard treatment is praziquantel (20 
nng/kg orally for two doses, four hours apart). 
About 10%-15% of patients require re¬ 
treatment. 


[ Q: 1906 ] MRCPass - 2010 May 

A 55 year old man has been 
diagnosed with Burkitt's Lymphoma. 

Which gene mutation is associated? 



1- BRAF 


Dr. Khalid Yusuf El-Zohry - Sohag Teaching Hospital (01118391123) 

Ref MRCPass OE OE 2012 PasTest 2009 PassMedicine 2009 PasTest Exam ReviseMRCP 




























El-zohry MRCP Questions Bank (Port 1) - 2013 


(For my personal use) 


2- P53 

3- C-myc 

4- N-myc 

5- BCI-ABR 


Answer & Comments 

Answer: 3- C-myc 

In Burkitt's lymphoma (associated with 
Ebstein Barr virus), genetic translocations lead 
to consequent c-myc rearrangement and 
overexpression. 

In most (approximately 90%) of the cases of 
Burkitt's lymphoma, a reciprocal translocation 
has moved the proto-oncogene c-myc from its 
normal position on chromosome 8 to a 
location close to the enhancers of the 
antibody heavy chain genes on chromosome 
141(8:14). 

[ Q: 1907 ] MRCPass - 2010 May 

A 20-year-old woman presented 
with a 1-year history of progressive hair loss in 
patches. She has a history of eczema and 
hypothyroidism. On examination, in the 
affected hair loss areas, the skin was pale and 
waxy. The hair loss did not involve other body 
sites but she had hypopigmented areas over 
the arms. Blood tests show normal full blood 
count, renal function and also negative ANA. 

What is the likely diagnosis? 

1- SLE 

2- Hypothyroidism associated hair loss 

3- Dermatitis artefacta 

4- Allergic contact dermatitis 

5- Alopecia areata 



Answer & Comments 

Answer: 5- Alopecia areata 

Alopecia areata is a form of hair loss from 
areas of the body, usually from the scalp. 


The condition affects l%-2% of the 
population. Hair loss may be diffuse or affect 
areas, and commonly affects the scalp but can 
also affect any hair growing part of the body. 
It is commonly associated with autoimmune 
diseases, such as hypothyroidism, SLE and also 
vitiligo. Steroid creams such as clobetasol, 
steroid injections or minoxidil are treatment 
options. 



[ Q: 1908 ] MRCPass - 2010 May 

A 56 year old patient with atrial 
fibrillation is on long term warfarin. He was 
recently diagnosed with pulmonary 
tuberculosis and was started on 
antituberculosis agents. He mentioned that his 
INR frequently decreased since then and the 
anticoagulant nurses had to increase the dose 
of warfarin. 


Which antituberculosis agent was most likely 
to have been responsible for the decrease in 
INR? 

1- Rifampicin 

2- Pyrazinamide 

3- Ethambutol 

4- Isoniazid 


5- Streptomicin 


Answer & Comments 

Answer: 1- Rifampicin 

Rifampicin is a liver enzyme inducer and 
isoniazid is a liver enzyme inhibitor. 

Liver enzyme inducers are likely to reduce the 
effect of warfarin, causing a decreased INR. 


[ Q: 1909 ] MRCPass - 2010 May 


Which one of the following is the 
mechanism of action of the drug , clopidogrel? 

1- Monoclonal antibody 

2- Cyclooxygenase inhibitor 

3- Glycoprotein llb/llla inhibitor 


Dr. Khalid Yusuf El-Zohry - Sohag Teaching Hospital (01118391123) 

Ref MRCPass OE OE 2012 PasTest 2009 PassMedicine 2009 PasTest Exam 




ReviseMRCP 

757 


























El-zohry MRCP Questions Bank (Port 1) - 2013 


(For my personal use) 


4- ADP antagonist 

5- Low molecular weight heparin 


Answer & Comments 


Answer: 4- ADP antagonist 


Step 3:- Add inhaled long-acting B2 agonist 
(LABA), such as salmeterol 

If control still inadequate, institute trial of 
other therapies, leukotriene receptor 
antagonist or slow release theophylline 


Clopidogrel and ticlopidine are adenosine 
diphosphate (ADP)-receptor antagonists that 
inhibit ADP-induced fibrinogen binding to 
platelets, a necessary step in the platelet 
aggregation process. 



[ Q: 1910 ] MRCPass - 2010 May 
A 23-year-old woman comes for 


review. 


She was diagnosed with asthma two years ago 
and is currently using a salbutamol inhaler 
lOOmcg PRN combined with beclometasone 
dipropionate inhaler 400mcg twice a day. She 
continues to get frequent episodes of 
wheeziness and shortness of breath with low 
peak flow readings. She has a good inhaler 
technique. 

Whot is the most appropriate next step in 
management? 

1- Switch steroid to fluticasone propionate 

2- Increase beclometasone dipropionate to 
800mcg bd 

3- Trial of monteleukast 


4- Add salmeterol 


5- Add tiotropium 


Answer & Comments 

Answer: 4- Add salmeterol 

The management of stable asthma is now well 
established with a step-w ise approach: 

Step 1:- Inhaled short-acting B2 agonist as 
required 

Step 2:- Add inhaled steroid at 200-800 
mcg/day* 


[ Q: 1911 ] MRCPass - 2010 May 

A 43 year old man presents to the 
hospital with shortness of breath and chest 
pain. 

He is a non smoker and has no previous 
medical history of respiratory problems. His 
BP was 110/80 & pulse was 80/min. CXR 
shows a large pneumothorax and the patient 
had 1.5 litres of air aspirated from the chest. A 
repeat CXR show s, a maximum of 1.5 cm 
diameter rim of air from the chest wall and 
the patient remains short of breath. 

Which is the most appropriate step? 

1- Chest drain 

2- Oxygen therapy 

3- Observation and follow up 

4- Repeat needle aspiration 

5- VATs procedure 



Answer & Comments 

Answer: 4- Repeat needle aspiration 

This patient has a symptomatic primary 
pneumothorax which has been aspirated and 
in the question the actual volume aspirated in 
the first instance was also given. 

A repeat aspiration is recommended if the 
patient is still symptomatic after the first 
aspiration (for a primary pneumothorax) and < 
2.5 I of air was aspirated in the first attempt as 
in this case. 

See pg 44 of the BTS guideline 

http://www.brit-thoracic.org.uk/clinical- 

information/pneumothorax/pneumothorax- 

guideline.aspx 


Dr. Khalid Yusuf El-Zohry - Sohag Teaching Hospital (01118391123) 

Ref MRCPass OE OE 2012 PasTest 2009 PassMedicine 2009 PasTest Exam ReviseMRCP 



758 




























El-zohry MRCP Questions Bank (Port 1) - 2013 


(For my personal use) 


[ Q: 1912 ] MRCPass - 2010 May 

A 25-year-old man presented with 
bi- temporal hemianopia. He mentioned that 
his shoe sizes were above that of his friends 
since childhood and he often had sw eaty 
episodes. 

Which one of the following tests is likely to 
confirm the diagnosis of acromegaly? 

1- Random growth hormone 

2- IGF-1 

3- Glucose tolerance test with growth 
hormone suppression 

4- MRI pituitary Synacthen test 

5- Synacthen test 

Answer & Comments 

Answer: 3- Glucose tolerance test with growth 
hormone suppression 

In Acromegaly, there is excess Growth 
hormone (GH) which is difficult to suppress. 

Because GH secretion is inhibited by glucose, 
measurement of glucose non-suppressibility is 
useful. In the glucose tolerance test, baseline 
GH levels are obtained prior to ingestion of 
100 g of oral glucose, and additional GH 
measurements are made at 30, 60, 90, and 
120 minutes following the oral glucose load. 
Patients with active acromegaly are unable to 
suppress GH concentration below 2 ng/mL. 

Random GH measurements are often not 
diagnostic because of the episodic secretion of 
GH, but IGF-I has a long half-life, and is useful 
as a screen for Acromegaly. MRI may reveal a 
pituitary tumour but it would not be specific 
for Acromegaly. 



[ Q: 1913 ] MRCPass - 2010 May 

A 69-year-old woman has lung 
carcinoma and recently underwent 
chemotherapy. She presented with shortness 
of breath and pleuritic chest pain. 



Which one of the following signs suggests a 
significant pericardial effusion? 

1- Systolic murmur 

2- Pericardial rub 

3- Rapid y descent of JVP 

4- Rise of JVP with inspiration 

5- rise in blood pressure with inspiration 


Answer & Comments 

Answer: 4- Rise of JVP with inspiration 

The question asks for which signs are present 
in cardiac tamponade due to a large 
pericardial effusion. 

The main signs are Kussmaul's sign (increase in 
JVP with inspiration) and Pulsus paradoxus 
(the inspiratory fall of aortic systolic pressure 
greater than 10 mm Hg). 



[ Q: 1914 ] MRCPass - 2010 May 

A 60 year old lady is being assessed 
for treatment of hypertension. She has a high 
blood pressure despite being on 
bendroflumethiazide. She has recently 
discontinued medications due to ankle 
oedema, gum bleeding and generalised 
lethargy. 

What medication should she be given? 


1- Atenolol 


2- Perindopril 

3- Amlodipine 

4- Verapamil 

5- Frusemide 


Answer & Comments 

Answer: 2- Perindopril 

Beta blockers may worsen lethargy and 
calcium channel blockers can cause ankle 
oedema and gum bleeding. 


Dr. Khalid Yusuf El-Zohry - Sohag Teaching Hospital (01118391123) 

Ref MRCPass OE OE 2012 PasTest 2009 PassMedicine 2009 PasTest Exam ReviseMRCP 



759 





























El-zohry MRCP Questions Bank (Port 1) - 2013 


(For my personal use) 


A thiazide diuretic has already been started, 
hence frusemide is not appropriate, hence an 
ACE inhibitor such as perindopril is the best 
option. 


[ Q: 1915 ] MRCPass - 2010 May 

A 20 year old cocaine abuser 
presents with chest pain. On admission, he 
has a blood pressure of 180 / 95 mmHg and is 
agitated. 

What serious complication is he most likely to 
develop? 

1- Hyponatraemia 

2- Hypomagnesaemia 

3- Hyperkalemia 

4- Hyperthermia 

5- Hypothermia 



Answer & Comments 

Answer: 4- Hyperthermia 
Cocaine is a strong stimulant. 


The most common acute risk is bacterial line 
related infections such as staphylococcal, 
which has been underreported. 

The important long-term risks of transmitting 
viral infection from the donor are well 
recognised and have been substantially 
reduced by screening of HIV and Hepatitis B + 
C in developed countries. 


[ Q: 1917 ] MRCPass - 2010 May 

A 35 year old woman presents with a 
history of intermittent light-headedness. 

Clinical examination and 12-lead ECG were 
normal. 

Which of the following, if present on a 24 hour 
Ho Iter ECG tracing, would be the most 
clinically important? 

1- Atrial premature beats 

2- Profound sleep-associated bradycardia 

3- Supraventricular tachycardia 

4- Transient Mobitz type 1 atrioventricular 
block 



In significant cocaine overdoses, the main 
effects are Acute coronary syndrome 
(coronary vasospasm), Agitation and delirium, 
Seizures and severe Hyperthermia. 


[ Q: 1916 ] MRCPass - 2010 May 

Which of the following types of 
infections is most commonly transmitted 
through platelet infusion? 

1- HIV 

2- Malaria 

3- Hepatitis B 

4- Staphylococcal 

5- Treponema 



Answer & Comments 


Answer: 4- Staphylococcal 


5- Ventricular premature beats 


Answer & Comments 

Answer: 3- Supraventricular tachycardia 

The most significant arrhythmia here is 
supraventricular tachycardia and 

consideration should be given to a trial of 
medication such as beta blocker. 

Both atrial and ventricular premature beats, 
sleep associated bradycardia and transient 
second degree (mobitz) heart block do not 
need intervention. 


[ Q: 1918 ] MRCPass - 2010 May 

A 48-year-old white male who is HIV 
positive presented to the emergency 
department with a 10-day history of jaundice 
and a 4-day history of right upper quadrant 
and midepigastric abdominal pain. He gives a 



Dr. Khalid Yusuf El-Zohry - Sohag Teaching Hospital (01118391123) 

Ref MRCPass OE OE 2012 PasTest 2009 PassMedicine 2009 PasTest Exam ReviseMRCP 



760 
































El-zohry MRCP Questions Bank (Port 1) - 2013 


(For my personal use) 


history of recent unprotected sexual 
intercourse. His most recent CD4+ cell count 
was 520 /|iL with a viral load of less than 400 
HIV RNA copies/mL. 

On physical examination, his abdomen was 
moderately distended, with marked 
tenderness in the right upper quadrant, and 
both his liver and spleen were 5 cm below the 
costal margin. Laboratory assessment showed: 

Bilirubin 32 (1-22) pmol/l, serum alkaline 
phosphatase, 138 IU/L (30 to 115 IU/L); serum 
aspartate and alanine transaminases, 75 and 
88 IU/L,(normal, 5 to 45 IU/L and 5 to 60 IU/L). 

What is the most likely cause of hepatitis in 
this patient? 

1- Cytomegalovirus 

2- Ebstein barr virus 

3- Hepatitis B virus 

4- Hepatitis C virus 

5- Hepatitis E virus 


Answer & Comments 

Answer: 4- Hepatitis C virus 

This questions asks what the commonest 
cause of viral co-infection causing hepatitis in 
HIV patients. 

The highest prevalence of viral co-infection 
among HIV infected patients is hepatitis C. 


[ Q: 1919 ] MRCPass - 2010 May 

A 25-year-old man presented to the 
emergency department with fevers. He drinks 
4 units of alcohol per day and admitted to 
regular intravenous drug abuse. On physical 
examination, the patient had a temperature of 
38°C, was tachypnoeic and tachycardic. Blood 
pressure was 100/85 mmHg. He had a soft 
systolic murmur heard throughout the 
precordium. 

What is the most likely infective organism? 

1- Staph viridans 



2- Staph epidermidis 

3- Staph aureus 

4- Pneumocystis carinii 

5- Streptococcus pyogenes 


Answer & Comments 

Answer: 3- Staph aureus 

In this case, due to a history of intravenous 
drug use, tricuspid valve endocarditis is most 
likely. 

Among intravenous drug users, staph aureus 
is the most common cause. In patients with 
native valve endocarditis, staph viridans is the 
most common. 


[ Q: 1920 ] MRCPass - 2010 May 

A 42-year-old woman who is known 
to be HIV positive is admitted to the 
Emergency Department following a seizure. 
Her partner reports that she has been having 
headaches, night sweats and anorexia for the 
past four weeks. Blood tests and a CT head are 
arranged: 

CD4 80 cells/mm 

CT head - Single ring enhancing lesion in the 
right parietal lobe with surrounding oedema 

What is the most likely diagnosis? 

1- CNS lymphoma 

2- Tuberculosis 

3- Progressive multifocal 

leukoencephalopathy 

4- Brain abscess 

5- Toxoplasmosis 



Answer & Comments 

Answer: 1- CNS lymphoma 

The best options are either lymphoma or 
toxoplasmosis, but key to differentiating is 
whether it is single or multiple. 


Dr. Khalid Yusuf El-Zohry - Sohag Teaching Hospital (01118391123) 

Ref MRCPass OE OE 2012 PasTest 2009 PassMedicine 2009 PasTest Exam 




ReviseMRCP 

761 





























El-zohry MRCP Questions Bank (Port 1) - 2013 


(For my personal use) 


With cerebral lymphoma, a single lesion that 
enhances in a nodular, homogeneous, or ring¬ 
like pattern is observed, typically with 
surrounding cerebral oedema. 

Toxoplasmosis is the most common cause of 
brain lesions in HIV patients. 


In this patient, rosiglitazone is contraindicated 
due to heart failure and metformin is 
relatively contraindicated due to renal 
impairment. The BMI is high and as a second 
line agent after sulphonylurea and insulin, 
exanetide should be considered. 


The majority of lesions (90%) are However, 
mulltiple on presentation, so is less likely in 
the above scenario. 

Tuberculous infection is much more varied 
radiologically, with meningeal destruction and 
Granulomas. 

^ [ Q: 1921 ] MRCPass - 2010 May 

f* - 

# A 55 year old lady is being reviewed 
in the diabetes clinic. She has type 2 diabetes 
which is poorly controlled and a history of CCF 
with moderately impaired left ventricular 
function. Her BMI is 35. She is currently on 
gliclazide 160mg bd, a long acting glargine 
insulin and short acting actrapid insulin with 
meals, frusemide, amlodipine and 
bendrofluazide. Her HbAlc value is 12 and she 
has frequently high BMS recorded. Her latest 
U&E results are urea 10 mmol/I, creatinine 
190 pmol/l. 

Whot is the best medication to odd to control 
her blood sugars? 

1- Rosiglitazone 


^ [ Q: 1922 ] MRCPass - 2010 May 

# A 64 year old man presents with an 
episode of amnesia for the second time. 2 
days ago he had an episode of confusion, 
according to his wife. He was, However, able 
to have a normal conversation despite having 
been found wandering. After 2 hours, he 
abruptly returned to normal and could not 
remember what happened. 

Whot is the most likely diagnosis? 

1- Alcoholic encephalopathy 

2- subarachnoid haemorrhage 

3- Complex partial seizure 

4- Transient ischaemic attack 

5- Transient global amnesia 

Answer & Comments 

Answer: 5- Transient global amnesia 

Transient global amnesia (TGA) is a temporary 
and isolated disorder of memory which may 
last several hours. 


2- Metformin 

3- Exanetide 

4- Glimepiride 

5- Glucagon 


Answer & Comments 

Answer: 3- Exanetide 

The newer incretin (GLP) analogues are now 
included in the guidelines by NICE CG 66 for 
patients who have not responded to insulin 
and Thiazolidinediones. 


Precipitating factors include sexual 
intercourse and heavy physical exercise, 
particularly sw imming in cold water. 


[ Q: 1923 ] MRCPass - 2010 May 

A 65-year-old comes to the clinic for 
a review of his symptoms. 

He has been followed up for aortic stenosis for 
the last 10 years. Over the past three months 
he has been complaining of fatigue and has 
lost 8 kg in weight. A full blood count was 
requested: 

Hb9.2 g/dl 



Dr. Kholid Yusuf El-Zohry - Sohog Teaching Hospital (01118391123) 

Ref MRCPass OE OE 2012 PasTest 2009 PassMedicine 2009 PasTest Exam ReviseMRCP 



762 



























El-zohry MRCP Questions Bank (Port 1) - 2013 


(For my personal use) 


MCV 65 fl 
Pit 360 x 10 9 /l 
WBC 5.0 xl0 9 /l 

Blood film Hypochromic, microcytic picture 

An upper Gl endoscopy and duodenal biopsy 
was normal. 


Answer & Comments 

Answer: 3- Aspirin 

Several large trials have demonstrated that 
aspirin improves prognosis in unstable angina 
and reduces the risks of cardiovascular events 
such as myocardial infarction. 


What is the most appropriate next 
investigation? 

1- Transthoracic echocardiogram 

2- Bone marrow biopsy 

3- Colonoscopy 

4- Faecal occult blood 

5- Mesenteric angiography 


Answer & Comments 

Answer: 3- Colonoscopy 

The patient may have angiodysplasia, which is 
associated with aortic valve disease. 

There is evidence of iron deficiency anaemia. 
Although the OGD is normal, a colonoscopy 
should be considered as it can investigate 
higher for sources of bleeding. 


The other drugs are all good antianginal drugs 
for symptomatic management. 


[ Q: 1925 ] MRCPass - 2010 May 

A 40 year old man has palpitations 
and ECG shows atrial fibrillation. This resolves 
spontaneously. The next day, the ECG was 
normal. Echocardiography reveal normal 
cardiac anatomy. The patient describes weekly 
episodes of palpitations. 

Which of the following should be started for 
maintenance of sinus rhythm? 

1- Digoxin 

2- Flecainide 

3- Amlodipine 

4- Bretylium 

5- Ramipril 



^ [ Q: 1924 ] MRCPass - 2010 May 

- 

# A 50 year old man with worsening 
angina was referred for an exercise test. The 
Bruce protocol exercise test was positive with 
recordings showing 2-3 mm ST depression in 
the anterior leads. 


Answer & Comments 

Answer: 2- Flecainide 

Flecainide is a class Ic antiarrhythmic (sodium 
channel blocker) which is useful for 
paroxysmal AF. 


Which one of the following drugs is likely to 
improve prognosis? 

1- Isosorbide mononitrate 

2- Diltiazem 

3- Aspirin 

4- Nicorandil 

5- Ivabradine 


Other helpful options are beta blockers 
(sotalol) and amiodarone. 



[ Q: 1926 ] MRCPass - 2010 May 

A 60 year-old man is admitted with 
pain, weakness and numbness in his right 
foot. He has had long standing lower back pain 
but the symptoms of weakness and numbness 
only started two weeks ago. 


On examination, there was weakness of all 
movements at the right ankle, an absent right 


Dr. Khalid Yusuf El-Zohry - Sohag Teaching Hospital (01118391123) 

Ref MRCPass OE OE 2012 PasTest 2009 PassMedicine 2009 PasTest Exam 




ReviseMRCP 

763 




























El-zohry MRCP Questions Bank (Port 1) - 2013 


(For my personal use) 


ankle jerk and sensory impairment on lateral 
aspect of the sole of the right foot. There was 
plantar flexion with the babinski reflex. 

Where is the likely site of the lesion? 

1- Femoral nerve 

2- Lumbosacral plexus 

3- Obturator nerve 

4- Sciatic nerve 


Answer & Comments 

Answer: 4- Increased sodium reaching the 
collecting ducts 

The two mechanisms of hypokalaemia cause 
by the thiazide (Bendroflumethiazide) are 
increased sodium reaching the collecting ducts 
and activation of the renin-angiotensin- 
aldosterone system 


5- SI spinal root 


Answer & Comments 

Answer: 4- Sciatic nerve 

Sciatic nerve palsy causes global weakness of 
ankle due involvement of both of its branches: 
tibial nerve (plantar flexion and inversion) and 
common peroneal nerve (dorsiflexion and 
eversion). 

The right ankle jerk is also absent due to tibial 
nerve involvement. 


[ Q: 1927 ] MRCPass - 2010 May 

A 45 year old patient who takes 
bendroflumethiazide is noted to have a 
potassium of 3.1 mmol/I. 

What is the main mechanism causing 
hypokalaemia in patients taking 
bendroflumethiazide ? 

1- Opening of potassium channels in the 
proximal convoluted tubule 

2- Decreased sodium reaching the distal 
convoluted tubule 

3- Decreased flow rate in the nephron 
resulting in a decreased potassium gradient 

4- Increased sodium reaching the collecting 
ducts 

5- Inhibition of renin-angiotensin-aldosterone 
system 



7 


[ Q: 1928 ] MRCPass - 2010 May 
A 75 year old woman presented with 


a non healing ulcer on her right foot and was 
admitted to hospital. 


She has a temperature of 37.9 C. Blood 
cultures on day 1 grew MRSA. 


What antibiotics would you consider in 
addition to vancomycin? 


1- Co amoxiclav 


2- Metronidazole 


3- Rifampicin 

4- Ciprofloxacin 

5- Linezolid 


Answer & Comments 

Answer: 3- Rifampicin 

Rifampicin and vancomycin are a good 
combination for initial treatment of MRSA. 

If this does not respond linezolid is a good 
alternative. 

[ Q: 1929 ] MRCPass - 2010 May 

A 43 year old woman was admitted 
to hospital with a several month history of 
diarrhoea, malaise and weight loss. She was in 
good health prior to the development of these 
symptoms. On examination, she had mild 
jaundice and looked thin. She had a distended 
abdomen with shifting dullness to percussion. 
Her blood tests show : 

Hb 10.5 g/dl 



Dr. Khalid Yusuf El-Zohry - Sohag Teaching Hospital (01118391123) 

Ref MRCPass OE OE 2012 PasTest 2009 PassMedicine 2009 PasTest Exam ReviseMRCP 





























El-zohry MRCP Questions Bank (Part 1) - 2013 


(For my personal use) 


MCV 82 fl 
WCC 8 x 10 9 /l 
platelets 220 x 107 L 
sodium 125 mmol/l 
potassium 4.1 mmol/l 
urea 11 mmol/l 
creatinine 160 |imol/l, 

ALT 95 (5-35) U/l 
AST 115 (1-31) U/l 
ALP 220(20-120) U/l 
Bilirubin 30 (1-22) nmol/l 
Albumin 28 (37-49) g/l 

Carcinoembryonic antigen (CEA) 3.8 <2.5 
ng/ml 

Alpha-Fetoprotein (AFP) 55 < 44 ?g/L 

CA125 - 38 (<35) U/ml 

CA 19-9-250 (< 40) U/ml 

CA 15-3-32 (< 29) U/mL 

Prostate-Specific Antigen (PSA) 2 (< 4) ng/ml 

Which one of the following is the likely primary 
tumour? 

1- Pancreas 

2- Colorectal 

3- Ovarian 

4- Liver 

5- Prostate 


He develops multiple fleshy, red nodules on 
the trunk and on the buttocks. The crops of 
lesions are flat topped and papular. These 
were treated and subsequently healed with 
scarring, but the patient noticed further 
lesions developing after a few months. 

Which one of the following is most likely? 

1- Molluscum contagiosum 

2- Kaposi's sarcoma 

3- Human papillovirus infection 

4- Herpes zoster infection 

5- herpes simplex infection 

Answer & Comments 

Answer: 2- Kaposi's sarcoma 

Kaposi's sarcoma (KS) is a tumor caused by 
Human herpesvirus 8 (HHV8) and frequently 
found in patients with HIV infection. 

Kaposi's sarcoma lesions may appear like 
bruises but are papular. With time, they 
darken. Scarring is common following 
treatment with immunosuppressive drugs. 

Molluscum contagiosum is a viral skin disease 
characterised by firm, round, translucent, 
umbilicated papules containing caseous 
matter and peculiar capsulated bodies. It is 
caused by a DNA virus of pox family. It can 
appear with crops and can be treated with 
cryotherapy. It tends to be self limiting and 
does not scar. 


Answer & Comments 



\7 

[ Q: 1931 ] MRCPass - 2010 May 

Answer: 1- Pancreas 

n.j 

A 62 year old man presents with a 


The tumour markers are not specific to one 
tumour but in this case the Ca 19.9 is highest, 
and it is most strongly associated with 
pancreatic cancer. 


cough and chest x ray confirms that he has a 
pneumonia. 

Which one of the following is a bad prognostic 
sign ? 


[ Q: 1930 ] MRCPass - 2010 May 
A 45 year old man is known to have 

HIV. 



1- Wbc >30 x 1071 

2- Urea of 9 mmol/l 

3- Temperature of 39 °C 


Dr. Khalid Yusuf El-Zohry - Sohag Teaching Hospital (01118391123) 

Ref MRCPass OE OE 2012 PasTest 2009 PassMedicine 2009 PasTest Exam 




ReviseMRCP 

765 

























El-zohry MRCP Questions Bank (Port 1) - 2013 


(For my personal use) 


4- Age of 62 

5- Consolidation on the CXR 


Answer & Comments 

Answer: 2- Urea of 9 mmol/l 

The CURB - 65 score for assessment of severity 
of pneumonia is tested here. 

The criteria are: 

■ Confusion of new onset (defined as an 
AMT of 8 or less) 

■ Urea greater than 7 mmol/l (Blood 
Urea Nitrogen > 19) 

■ Respiratory rate of 30 breaths per 
minute or greater 

■ Blood pressure systolic < 90 mmHg or 
diastolic < 60 mmHg 

age 65 or older 


[ Q: 1932 ] MRCPass - 2010 May 

A 30 year old lady is seeking genetic 
advice. She has haemochromatosis and her 
husband does not have any know ledge of 
being affected. 

What is the chance of a child carrying the 
gene? 



The HFE gene is located on short arm of 
chromosome 6 at location. Approximately one 
in ten people are carriers of the mutated 
gene. It is thought that the HFE protein 
functions to regulate iron absorption by 
regulating the interaction of the transferrin 
receptor with transferrin. 

In this case, as the mother is homozygous for 
the gene as it is autosomal recessive and she 
has the disease. This means that all of her off 
spring will be carriers of the gene. 

[ Q: 1933 ] MRCPass - 2010 May 

A 20 year old woman has been 
referred with polyuria and polydipsia. She 
mentions that she has had the symptoms for 2 
months. Upon investigation the following 
results were found. 

sodium 148 mmol/l 

potassium 4.5 mmol/l 

urea 4 mmol/l 

creatinine 78 |imol/l 

Glucose: 5.5 mmol/l 

Plasma Osmolality 308 (280-300) mmol/kgw 
ater 

Urine Osmolality . 90 (50-1200) mmol/kg 
What is the diagnosis? 

1- Addison's disease 



1 - 0 % 


2- Diabetes mellitus 


2- 1 in 50 

3- 1 in 100 

4- 1 in 400 

5- 100% 

Answer & Comments 

Answer: 5-100% 

The inheritance of the HFE gene in 
haemochromatosis is autosomal recessive. 


3- Psychogenic polydipsia 

4- Diabetes insipidus 

5- SIADH 

Answer & Comments 

Answer: 4- Diabetes insipidus 

Diabetes insipidus usually presents with thirst 
and polyuria. 

This is due to a lack of anti diuretic hormone 
(ADH) action. Despite drinking a lot, water loss 
leads to hypernatraemia and high plasma 


Dr. Khalid Yusuf El-Zohry - Sohag Teaching Hospital (01118391123) 

Ref MRCPass OE OE 2012 PasTest 2009 PassMedicine 2009 PasTest Exam ReviseMRCP 





























El-zohry MRCP Questions Bank (Port 1) - 2013 


(For my personal use) 


osmolality. The inappropriately low urine 
osmolality is due to an inability to reabsorb 
water (or concentrate urine). A lack of ADH 
production is termed cranial diabetes 
insipidus (e.g. pituitary tumour) and a lack of 
response to ADH is termed nephrogenic 
diabetes insipidus (e.g. action of drugs on the 
kidney). 

In contrast, a patient with psychogenic 
polydipsia would not have high plasma 
osmolalities and hypernatraemia (rather 
would be hyponatraemic). A patient with 
SIADH would not typically give a history of 
polydipsia and polyuria. 


[ Q: 1934 ] MRCPass - 2010 May 

A 30 year old lady was admitted to 
hospital by a concerned neighbour. She 
complains of seeing spider's crawling all over 
her body and she was disoriented. On 
examination, she was tremulous and looked 
very restless. She had a wide based, unsteady 
gait. 

What is the diagnosis? 

1- Hyponatraemic encephalopathy 

2- Schizophrenia 

3- Benzodiazepine withdrawal 

4- Delirium Tremens 

5- Wernicke's encephalopathy 



[ Q: 1935 ] MRCPass - 2010 May 

A 63 year old man presents with 
central crushing chest pain. 

Which one of the following features is most 
likely to be associated aortic dissection? 

1- Jaw pain 

2- Lower limb neurological deficit 

3- Severe hypertension 

4- Gas under the diaphragm on the chest x ray 

5- Anaemia 





severe 


Answer & Comments 

Answer: 3- Severe hypertension 

Severe hypertension, aortic aneurysm and 
connective tissue disorders (e.g. Marfan 
syndrome) are predisposing factors to aortic 
dissection. The nature of the pain can vary 
significantly - epigastric pain, tearing pains, 
back pains and even dull pains can occur. 

[ Q: 1936 ] MRCPass - 2010 May 

A 18 year old patient had a 
continuous murmur and was suspected to 
have patent ductus arteriosus. 

What is the nature of the pulse likely to be? 

1- Jerky 

2- Bisferiens 



Answer & Comments 

Answer: 4- Delirium Tremens 

This patient's history of disorientation, 
agitation and hallucination fits delirium 
tremens best. 

If she was significantly confused rather than 
having hallucinations, then alcoholic 
(Wernicke's) encephalopathy should be 
considered. 


3- Dicrotic 

4- Slow rising 

5- Collapsing 

Answer & Comments 

Answer: 5- Collapsing 

With a patent ductus arteriosus, the pulse 
may be collapsing in nature if there is a large 
shunt. 

Other signs include a continuous or machinery 
murmur is best heard at the upper left sternal 
border or left infraclavicular area, and there 


Dr. Khalid Yusuf El-Zohry - Sohag Teaching Hospital (01118391123) 

Ref MRCPass OE OE 2012 PasTest 2009 PassMedicine 2009 PasTest Exam ReviseMRCP 



767 
































El-zohry MRCP Questions Bank (Port 1) - 2013 


(For my personal use) 


may be signs of pulmonary hypertension (loud 
second heart sound). 


[ Q: 1937 ] MRCPass - 2010 May 

A 25-year-old man presented with an 
enlarged inguinal lymph node with night 
sweats, and the most likely clinical diagnosis is 
Hodgkin's lymphoma. A biopsy has been 
taken. 

Which of the following findings on histology 
confers the best prognosis in Hodgkin's 
lymphoma? 

1- Lymphocyte depleted 

2- Lymphocyte predominant 

3- Mixed cellularity 

4- Nodular sclerosing 

5- Non-Hodgkin's changes 



Answer & Comments 

Answer: 2- Lymphocyte predominant 

Hodgkin's lymphoma is a malignant 
proliferation of the lymphoid system and is 
characterised by the presence of Reed- 
Sternberg cells on histological examination. 

In order of good prognosis to worst, the 
histology types are - lymphocytic predominant 
> nodular sclerosis > mixed cellularity > 
lymphocytic depletion. 

The best prognosis is with lymphocyte 
predominant histology and lymphocyte 
depleted has the worst. 


examination, he walks with a wide based gait 
and has an MMSE score of 18 / 30. 

Of the following which is the most likely 
diagnosis? 

1- Meningovascular syphilis 

2- Normal pressure hydrocephalus 

3- Syringomyelia 

4- Wernicke-Korsakoff syndrome 

5- Alzheimer's disease 


Answer & Comments 

Answer: 2- Normal pressure hydrocephalus 

Normal pressure hydrocephalus (NPH) is a 
clinical symptom complex characterized by a 
triad of symptoms which are: abnormal gait, 
urinary incontinence, and dementia. 

Is a form of communicating hydrocephalus in 
which the intracranial pressure, as measured 
by lumbar puncture, is normal or 
intermittently raised. 



[ Q: 1939 ] MRCPass - 2010 May 

A 45 year old patient presents with 
proximal muscle weakness, particularly in the 
lower limbs. She has a heliotropic rash around 
the eyes and also has Gottron's papules. 


Which one of the following antibodies is most 
strongly associated? 


1- Sm 


2- Ro 


3- Jo-1 


[ Q: 1938 ] MRCPass - 2010 May 

A 55-year-old man has drunk six 
units of alcohol a day for most of his adult life. 

He was found wandering the street and 
appears confused. He was brought into 
hospital for assessment. A friend gives a 
history that he has worsening symptoms of 
difficulty walking, headaches and urinary 
incontinence for the past ten months. On 



4- SCL-70 

5- DsDNA 


Answer & Comments 

Answer: 3- Jo-1 

The diagnosis is dermatomyositis. 

Anti Jo-1 antibody is associated with acute 
onset myositis, particularly dermatomyositis. 


Dr. Khalid Yusuf El-Zohry - Sohag Teaching Hospital (01118391123) 

Ref MRCPass OE OE 2012 PasTest 2009 PassMedicine 2009 PasTest Exam ReviseMRCP 






























El-zohry MRCP Questions Bank (Part 1) - 2013 


(For my personal use) 


The limb girdle or proximal muscles are most 
severely affected in both polymyositis and 
dermatomyositis. 


^ [ Q: 1940] MRCPass-2010 May 

- 

* A 46 year old man is known to have 
ulcerative colitis which was diagnosed 10 
years ago. Over the last two months he has 
right upper quadrant discomfort and noticed 
to have jaundice. He was referred to the 
gastroenterology outpatients for assessment. 

Investigations performed showed these 
results: Ultrasound showed a dilated intra and 
extrahepatic ducts with beaded appearances. 

Blood tests results: 

ALT 120 (5-35) U/l 

AST 90(1-31) U/l 

ALP 850 (20-120) U/l 

GGT 250 (4-35) U/l 

Bilirubin 75 (1-22) pmol/l 

Albumin 38 (37-49) g/l 

What is the likely diagnosis? 

1- Focal nodular hyperplasia 

2- Primary sclerosing cholangitis 

3- Hemangioma 

4- Cholecystitis 

5- Hepatocellular carcinoma 


Primary sclerosing cholangitis is a disease of 
unknown aetiology characterised by chronic 
inflammation and fibrosis of the bile duct. It is 
associated with cholangiocarcinoma, a cancer 
of the biliary tree, and the lifetime risk for PSC 
sufferers is 10-15%. As many as 5% of patients 
with ulcerative colitis may progress to develop 
primary sclerosing cholangitis and 
approximately 70% of people with primary 
sclerosing cholangitis have ulcerative colitis. 


[ Q: 1941 ] MRCPass - 2010 May 

A 28 year old lady had amenorrhoea 
over the last 2 months after discontinuing use 
of an oral contraceptive pill. 

She has recently been doing vigorous exercise. 

Which one of the following is most likely? 

1- Premature ovarian failure 

2- Panhypopituitarism 

3- Adrenal tumour 

4- Polycystic ovary disease 

5- Amenorrhoea due to stopping OCP 



Answer & Comments 

Answer: 5- Amenorrhoea due to stopping OCP 

Post-pill amenorrhoea occurs in some women 
after stopping the combined oral 
contraceptive pill. 


Answer & Comments 


Answer: 2- Primary sclerosing cholangitis 


This usually settles spontaneously three 
months after discontinuing the pill. Excessive 
exercise is another cause of amenorrhoea. 


The cholestatic picture along with 
inflammatory lesion seen on the ultrasound 
hints tow ards either Sclerosing cholangitis or 
associated with ulcerative colitis. 

Narrowing of the bile duct lumen usually 
occurs throughout the biliary tree. Less 
commonly, changes are confined to the intra- 
hepatic or extra-hepatic ducts. 



[ Q: 1942 ] MRCPass - 2010 May 
Which one of the following is broken 


down to Glucose and Galactose? 

1- Maltose 


2- Sucrose 

3- Fructose 

4- Mannose 

5- Lactose 


Dr. Khalid Yusuf El-Zohry - Sohag Teaching Hospital (01118391123) 

Ref MRCPass OE OE 2012 PasTest 2009 PassMedicine 2009 PasTest Exam ReviseMRCP 



769 




























El-zohry MRCP Questions Bank (Port 1) - 2013 


(For my personal use) 


Answer & Comments 

Answer: 5- Lactose 

Lactose is broken down into glucose and 
galactose by an enzyme called lactase. 


Answer & Comments 

Answer: 3- Tumour suppressor 

The BRCA genes belongs to a class of genes 
known as tumor suppressor genes. 


[ Q: 1943 ] MRCPass - 2010 May 

A study assessed a statin tablet 
compared to placebo for stroke prevention 
over 2 years. 

There were 10% of patients developing stroke 
in the group taking a tablet and 20% in the 
carotid endarterectomy group developing a 
stroke. 

What is the number needed to treat over 1 
year to prevent 1 death? 

1- 1 



BRCA1 is expressed in the cells of breast and 
other tissue, where it helps repair damaged 
DNA, and destroy the cell when DNA can't be 
repaired. If BRCA1 itself is damaged, the 
damaged DNA can let the cell duplicate 
without control, and turn into a cancer 


[ Q: 1945 ] MRCPass - 2010 May 


I 


A 17 year old female was admitted 


with acute severe asthma. 


Which feature would make you think it is life 
threatening? 


2- 10 

3- 20 

4- 100 

5- 1000 


Answer & Comments 

Answer: 2-10 

NNT is defined as number needed to treat to 
prevent 1 death. 

The way to work this out is 1 divided by 
absolute risk reduction (Experimental event 
rate - control event rate). Hence 1 / (ARR) is 1 
/ 10% which is 10. 


[ Q: 1944 ] MRCPass - 2010 May 

Which role does the BRCA gene play 
in the DNA of tumour cells? 

1- Telomerase inhibitor 

2- Homeobox 

3- Tumour suppressor 

4- Tyrosine kinase 

5- Cyclic AMP second messenger 



1- PH of 7.40 

2- Pa02 of 8.4 

3- PaC02 of 5.5 

4- RR 30 

5- PEFR < 35% of predicted 


Answer & Comments 

Answer: 5- PEFR < 35% of predicted 

The results show a normal pH, low p02, 
normal C02 and high respiratory rate. 

The most important predictors will be 
significantly low Peak flow rate as in this case, 
or Type II respiratory failure (high C02) with 
respiratory acidosis. 


^ [ Q: 1946 ] MRCPass - 2010 May 

wm 

# A 65 year old man is admitted for 
investigation of jaundice and anorexia for 
several weeks. 6 weeks ago, he had completed 
a course of medication which had been 
prescribed by his GP. 

Investigations reveal: 

Albumin 40 g/L (37-49) 


Dr. Khalid Yusuf El-Zohry - Sohag Teaching Hospital (01118391123) 

Ref MRCPass OE OE 2012 PasTest 2009 PassMedicine 2009 PasTest Exam ReviseMRCP 



770 


































El-zohry MRCP Questions Bank (Port 1) - 2013 


(For my personal use) 


Bilirubin 260 umol/L (1-22) 

AST 80 iu/L (5-35) 

Alkaline Phosphatase 430 iu/l (50-110) 

Abdominal ultrasound reveals gallstones but 
no evidence of cholecystitis. 

Which drug is most likely to cause the 
presentation? 

1- Allopurinol 

2- Augmentin 

3- Amiodarone 

4- Pravastatin 

5- Ibuprofen 


Answer & Comments 

Answer: 2- Augmentin 

The blood results are consistent with 
cholestatic jaundice as there is significantly 
elevated bilirubin and ALP. 

Co-amoxiclav (augmentin) is a well known 
cause of this. 

Other drugs includes gold, nitrofurantoin, 
anabolic steroids, chlorpromazine, 
prochlorperazine, cimetidine, erythromycin 
and estrogen. 


[ Q: 1947 ] MRCPass - 2010 May 

A 70 year old man is admitted to 
casualty with severe agitation, tremors and 
ataxia. He is known to be on an 
antidepressant previously. Currently he 
smokes 40 cigarettes a day and drinks 20 units 
of alcohol per day. 

What drug should be given for sedation? 

1- Haloperidol 

2- Diazepam 

3- Chlorpromazine 

4- Chlordiazepoxide 

5- Propofol 



Answer & Comments 

Answer: 4- Chlordiazepoxide 

Chlordiazepoxide is a benzodiazepine used to 
control symptoms of alcohol withdrawal. 

[ Q: 1948 ] MRCPass - 2010 May 

A 19 year old female med student 
performs valsalva's maneourve for a class 
demonstration. 

During the manoeuvre , what is the initial 
physiological mechanism to occur? 

1- Reduced stroke volume 

2- Fall in cardiac output 

3- Decrease in heart rate 

4- Increase in systolic arterial blood pressure 

5- Reduced venous return 



Answer & Comments 

Answer: 4- Increase in systolic arterial blood 
pressure 

A valsalva manoeuvre is forced expiration 
against a held breath. 

There is an initial blood pressure rise (increase 
in stroke volume and cardiac output) as the 
pressure in the chest forces blood 

Following this, the increased pressure in the 
chest reduces venous return and this also 
leads to cardiac output to reduce, the heart 
rate also decreases. 

When the pressure is released (patient 
exhales) there is then a slight blood pressure 
drop The easiest way to appreciate this is the 
graph below. 


Dr. Khalid Yusuf El-Zohry - Sohag Teaching Hospital (01118391123) 

Ref MRCPass OE OE 2012 PasTest 2009 PassMedicine 2009 PasTest Exam 




ReviseMRCP 

771 



























El-zohry MRCP Questions Bank (Part 1) - 2013 


(For my personal use) 



—S ptateBP 
* f'Jlo fu-.o I 



[ Q: 1949 ] MRCPass - 2010 May 

Which of the following is most likely 
to be responsible for Ventricular tachycardia? 

1- Hyponatremia 

2- Hypokalaemia 

3- Hypomagnesaemia 

4- Hypophosphataemia 

5- Hypothermia 


Answer & Comments 

Answer: 3- Hypomagnesaemia 

Many of the options can cause VT / VF but as 
an isolated cause the most likely is 
hypomagnesaemia. As hypomagnesia is a 
common cause of VT, a stat dose magnesium 
sulphate is often given as initial treatment. 


^ [Q: 1950] MRCPass-2010 May 

* A patient presents with an inability 
to abduct his right arm and has loss of 
sensation over the right shoulder area. 

Which dermatome is affected? 


1- C3 

2- C5 

3- C7 

4- C8 

5- T1 


Answer & Comments 

Answer: 2- C5 

The deltoid muscle is implicated here, as the 
action is shoulder abduction and there is loss 
of sensation over the C5 dermatome. 

It is innervated by the axillary nerve (C5 and 
C6). 

[ Q: 1951 ] MRCPass - 2010 May 

A 53-year-old woman presents with 
periods of sweats and tremors which are 
relieved by eating. She has gained 
approximately 6 kg in weight in the last 2 
years. Her BM is 4.5. Blood tests are: Hb 13 
g/dl, MCV 78 fl, WCC 7 x 10 9 /l, platelets 200 x 
10 9 /l, sodium 135 mmol/l, potassium 4.7 
mmol/l, urea 5 mmol/l, creatinine 100 mmol/l, 
TSH - 3.3 (0.3-4) mU/l, free T4 -20 (10-24) 
pmol/l. 

Whot is the most appropriate investigation? 

1- 72 hour fast 

2- CT scan of pancreas 

3- MRI of the brain 

4- Insulin C-peptide concentration 

5- Oral glucose tolerance test 



Answer & Comments 

Answer: 1- 72 hour fast 

This patient has symptoms suggestive of 
hypoglycaemia which are relieved by 
carbohydrate. 

The likely cause is an insulinoma which is an 
insulin secreting pancreatic tumour. 

The best way of confirming the diagnosis is 
with a 72 hour fast. During the fast, the 
patient with an insulinoma may get episodes 
of hypoglycaemia with measured 
inappropriately high insulin C peptide 
(endogenous insulin). 


Dr. Khalid Yusuf El-Zohry - Sohag Teaching Hospital (01118391123) 

Ref MRCPass OE OE 2012 PasTest 2009 PassMedicine 2009 PasTest Exam ReviseMRCP 



772 










































El-zohry MRCP Questions Bank (Port 1) - 2013 


(For my personal use) 


Measurement of C-peptide is useful in 
excluding factitious hypoglycaemia from self 
injection of insulin. Insulin preparations do not 
contain C-peptide. 


pancreatitis such as pseudocysts) and ERCP 
(showing chronic ductal changes) with some 
advanced centres able to offer measurement 
of pancreatic enzyme production (protease, 
amylase, lipase). 


^ [Q: 1952] MRCPass-2010 May 

WM 

A 50 vear old ladv is assessed for 


[ Q: 1953 ] MRCPass - 2010 May 

longstanding abdominal symptoms. She 

A 50 year old man has presented 


describes chronic intermittent abdominal 
pains which were moderate in intensity, 
gripping in nature, localized in the middle of 
the abdomen. She also had increased 
frequency of stools up to 15 times a day. The 
stool was brownish yellow in color, sticky and 
oily, difficult to flush but devoid of blood or 
mucus. Typically the abdominal pain was 
relieved after passing the stool. 

What is the best investigation to confirm the 
diagnosis? 

1- Duodenal biopsy 

2- Liver biopsy 

3- Tumour markers 

4- Faecal fat 


with assessment of shortness of breath and 
wheezing. He works as a spray paint worker 
and gives a history of feeling more breathless 
during work. The symptoms improve during 
the weekend. The GP refers her to the 
respiratory clinic for assessment. 

What investigation is most appropriate to 
diagnose occupational asthma? 

1- Measure peak flows over 2 weeks 

2- Serial peak flow measurements at home 
and work 

3- Skin prick tests to isocyanates 

4- Lung function test 

5- IgE levels 


5- Faecal elastase 

Answer & Comments 

Answer: 5- Faecal elastase 

This patient has steatorrhoea and a diagnosis 
of chronic pancreatitis. 

Assays of fecal chymotrypsin and human 
pancreatic elastase 1 are useful in confirming 
advanced chronic pancreatitis with exocrine 
insufficiency. To check for pancreatic exocrine 
dysfunction, the most sensitive and specific 
test is the measurement of fecal elastase, 
which can be done with a single stool sample. 
Faecal fat analysis can also be useful but less 
convenient as it requires measurement of 
fecal fat excretion over 24hr on a lOOg fat 
diet. 

Other useful tests are CT Scan demonstrating 
calcification or complications of chronic 


Answer & Comments 

Answer: 2- Serial peak flow measurements at 
home and work 

Lung function tests and reversibility will help 
to confirm asthma. 

For diagnosis of occupational asthma, serial 
measurements of peak expiratory flow rate at 
home and at work: this is often the most 
appropriate first step. 

Measurements should be made every two 
hours from waking to sleeping for four weeks, 
keeping treatment constant and documenting 
times at work. There should be at least 3 
consecutive work days and 3 days aw ay from 
work included in the measurements, (as per 
Guidelines from the British Occupational 
Health research on Occupational Asthma). 


Dr. Khalid Yusuf El-Zohry - Sohag Teaching Hospital (01118391123) 

Ref MRCPoss OE OE 2012 PosTest 2009 PassMedicine 2009 PosTest Exom ReviseMRCP 



























El-zohry MRCP Questions Bank (Port 1) - 2013 


(For my personal use) 


[ Q: 1954 ] MRCPass - 2010 May 

A 46 year old man has been to India 
for 3 months, and returned 4 months ago. He 
presents complaining that for 6 months, he 
has had chronic diarrhoea with no blood. He 
has been previously well with no 
gastroenterological problems. 



3- Hypoglycaemia 

4- Steroid induced psychosis 

5- Depressive psychosis 


Answer & Comments 


Answer: 4- Steroid induced psychosis 


Which one of the following is the most likely 
infection? 

1- Yersinia 

2- Giardia 

3- E coli 


The incidence of steroid associated cognitive 
changes including psychosis are high 
particularly when high dose steroids (e.g. 
dexamethaxone) are used prior to or as part 
of chemotherapy. 


4- Salmonella 

5- Campylobacter 


Answer & Comments 

Answer: 2- Giardia 

Giardiasis (Giardia lamblia infection) often 
causes chronic diarrhoea which is non-bloody. 

This is prevalent in many countries. Symptoms 
might last for up to 6 weeks and are those of 
diarrhoea with some abdominal pains. 

Salmonella, shigella and Campylobacter often 
cause bloody diarrhoea. 

Yersinia infection is a type of zoonosis 
(infection from animal reservoir), although it 
can cause fevers and abdominal symptoms, 
there is no history in this case to suggest a 
zoonosis. 



[ Q: 1956 ] MRCPass - 2010 May 

A 52 year old man has had several 
previous episodes of knee joint effusions. He 
presents with a swollen right knee. He was 
apyrexial on admission. The knee was 
aspirated on admission and serous fluid was 
extracted. 


Microscopy and gram stain of the fluid 
showed no organisms. 

2 days later the patient's knee sw elled up 
again and he returned to hospital. However, 
he complained of more pain and had a 
temperature of 38°C. Blood tests revealed a 
raised white cell count and CRP. 

What should be done? 


1- Repeat aspiration 

2- Arthroscopy with washout 

3- Intravenous antibiotics 


[ Q: 1955 ] MRCPass - 2010 May 

A 63 year old man who has been 
diagnosed with a glioma is commenced on 
chemotherapy. 4 days later, he begins to 
behave strangely, and is noticed to become 
increasingly agitated. He also has suicidal 
ideation. 

Which one of the following is most likely? 

1- Vincristine encephalitis 

2- Hyponatraemia 



4- X ray of the knee 

5- MRI of knee joint 

Answer & Comments 

Answer: 1- Repeat aspiration 

Infection after knee aspiration is very 
uncommon (0.01% incidence). 

However, this patient's history of pyrexia does 
suggest the possibility, hence repeat 
aspiration and sending fluid for microscopy 


Dr. Kholid Yusuf El-Zohry - Sohog Teaching Hospital (01118391123) 

Ref MRCPass OE OE 2012 PasTest 2009 PassMedicine 2009 PasTest Exam ReviseMRCP 



774 






























El-zohry MRCP Questions Bank (Port 1) - 2013 


(For my personal use) 


and culture is sensible. If the provisional 
results suggest infection, or of there are other 
clinical suggestions of bacteraemia or sepsis 
then the patient should be treated with 
antibiotics. Septic arthritis can be treated with 
arthroscopy and washout, but usually in 
complicated cases e.g. with prosthesis or if a 
patient does not respond to medical therapy. 


[ Q: 1957 ] MRCPass - 2010 May 

A 62 year old man has discomfort in 
the left ankle for 2 days. Several weeks ago he 
had been on a course of antibiotics. He 
presented to the emergency department with 
mild, nonpitting edema of the left calf and 
ankle, and pain on plantar flexion. A Doppler 
study failed to detect a thrombus, and there 
was no radiologic evidence of a fracture. The 
patient was diagnosed with a muscle strain 
and achilles tendon rupture. 

Which drug may hove coused this? 

1- Ciprofloxacin 

2- Metronidazole 

3- Amoxicillin 

4- Nitrofurantoin 

5- Gentamicin 



2- 1 month before Surgery 

3- Several hours before Surgery 

4- 1 week after Surgery 

5- One month after surgery 


Answer & Comments 

Answer: 2-1 month before Surgery 

Pneumovax should be given at least two 
weeks before splenectomy, hence in this 
situation a month before is the best answer. 



[ Q: 1959 ] MRCPass - 2010 May 

Which ion / channel is primarily 
responsible for repolorisotion phase in the 
cardiac cycle? 

1- Sodium 


2- Sodium Calcium 

3- Phosphate 

4- Magnesium 

5- Potassium 


Answer & Comments 


Answer: 5- Potassium 


Answer & Comments 

Answer: 1- Ciprofloxacin 

Disruptions of tendons in adults, including 
rupture, have been reported in association 
with fluoroquinolones (ciprofloxacin). 

The Achilles tendon is the site most frequently 
associated with such adverse outcomes. 


[ Q: 1958 ] MRCPass - 2010 May 


A patient is about to undergo a 
splenectomy. 



Depolarisation and repolarisation refer to 
neuronal conduction or cardiac electrical 
activity. 

The action potentials produced by 
depolarization (due to Na+ ions moving into 
cell), leads to release of Ca2+ ions which lead 
to contraction of cardiac muscle, followed by 
repolarization (K+ ions moving into cell). 


[ Q: 1960 ] MRCPass - 2010 May 

A 47 year old man presented with a 
history of hemoptysis for two months and he 
had lost 5 kg of weight. 



When should pneumococcal vaccine be given 
in relation to splenectomy? 

1- 1 week before Surgery 


He gives a past history of tuberculosis 20 years 
ago which was treated with drugs for 6 
months. On physical examination, chest 


Dr. Khalid Yusuf El-Zohry - Sohag Teaching Hospital (01118391123) 

Ref MRCPass OE OE 2012 PasTest 2009 PassMedicine 2009 PasTest Exam ReviseMRCP 



775 































El-zohry MRCP Questions Bank (Port 1) - 2013 


(For my personal use) 


auscultation revealed decreased air entry on 
right upper lung field. Chest X ray showed a 
cavitating lesion with dense shadows in right 
upper lobe. Investigations show that the 
sputum culture was negative for acid fast 
bacilli and IgG antibodies for A fumigatus were 
positive. 

What is the most likely couse? 

1- Bronchogenic carcinoma 

2- Reactivation of tuberculosis 

3- Aspergilloma 

4- Invasive aspergillosis 

5- Lung abscess 


Answer & Comments 

Answer: 3- Aspergilloma 

An aspergilloma is is a tangled mass composed 
of hyphae, fibrin, and inflammatory cells, that 
lies free in the lung cavity. 

Aspergillomata may form in cavities produced 
by previous lung diseases e.g. tuberculosis, 
abscesses, or areas of pulmonary infarction. It 
is usually caused by A. fumigatus. The most 
common symptom of aspergillomas is 
haemoptysis. Aspergillus precipitin antibody 
test results (ie, for IgG) are usually positive. 


50% of the UK population are slow 
acetylators, i.e. have a deficiency of the 
enzyme N acetyltransferase. 

Acetylator status is important in determining 
drug response and drug toxicity, e.g. slow 
acetylators are more at risk of developing drug 
induced lupus, the three commondest drugs 
implicated are hydralazine, isoniazid and 
procainamide. 


^ [ Q: 1962 ] MRCPass - 2010 May 

n ~--— 

# A 35 year old male patient was 
admitted into the emergency department 
complaining of double vision and dizziness. 
There was no relevant past medical history 
but he described a coryzal illness 2 weeks ago. 
On examination, he was short of breath at 
rest. He had reduced eye movements, in 
particular abduction in both eyes. 

Visual acuity was normal and there were no 
other cranial nerve deficits. Tone was normal 
in both upper and lower limbs. Power was 
reduced in the distal modalities of the arms 
and legs. He had flaccid reflexes throughout 
and plantars were downgoing. 

Whot is the likely diagnosis? 

1- Multiple sclerosis 

2- Bilateral cerebrovascular accidents 


[ Q: 1961 ] MRCPass - 2010 May 

Which one of the following drugs is 
likely to couse on adverse effect among slow 
acetylators? 

1- Amoxicillin 

2- Trimethoprim 

3- Hydralazine 

4- Minoxidil 

5- Ramipril 



Answer & Comments 


Answer: 3- Hydralazine 


3- Miller-fisher syndrome 

4- Transverse myelitis 

5- TB meningitis 


Answer & Comments 

Answer: 3- Miller-fisher syndrome 

Miller Fisher Syndrome is a peripheral 
neurological condition (variant of acute 
demyelination or Guillain Barre syndrome), 
mostly associated with respiratory or digestive 
infections. 

The average time for neurological symptoms 
to appear after infection is 1-2 weeks. It is 
considered to be initiated by a autoimmune 


Dr. Khalid Yusuf El-Zohry - Sohag Teaching Hospital (01118391123) 

Ref MRCPass OE OE 2012 PasTest 2009 PassMedicine 2009 PasTest Exam ReviseMRCP 




























El-zohry MRCP Questions Bank (Port 1) - 2013 


(For my personal use) 


process. The typical triad for this syndrome 
includes ophthalmoplegia, ataxia, and 
arreflexia. As with Guillain Barre syndrome, IV 
Ig should be considered for treatment and the 
patient should be monitored closely for 
deterioration in respiratory function. 


[ Q: 1963 ] MRCPass - 2010 May 

A 30-year-old woman presents with 
red, raised, itchy lesions that involve her 
entire body, including her face. 

These lesions come and go at the different 
parts of the body over the last 2 days. 

What treatment should be given? 

1- Dapsone 

2- Cetirizine 

3- Prednisolone 

4- Diprobase 

5- Fluconazole 



Answer & Comments 

Answer: 2- Cetirizine 

The itchy red skin lesions which come and 
disappear are consistent with urticaria. 

The patient should be given an antihistamine 
such as cetirizine. If the reaction is severe and 
there are signs of anaphylaxis then steroids 
should be given in addition. 


Answer & Comments 

Answer: 1- Acalculia 

Lesions of parietal lobe include apraxias, 
neglect, astereognosis (unable recognise an 
object by feeling it) visual field defects 
(typically homonymous inferior 

quadrantanopia). 

They may also cause alcalculia (inability 
perform mental arithmetic). Lesions of 
temporal lobe cause visual field defects 
(typically homonymous superior 

quadrantanopia), Wernike's (receptive) 
aphasia, auditory agnosia, memory 
impairment. 


[ Q: 1965 ] MRCPass - 2010 May 

A 40-year-old female presented with 
an erythematous annular patch with central 
clearing on her left forearm following a tick 
bite. The patient mentioned she had recent 
onset of intermittent joint pains and 
asymmetry of her face. On examination, she 
was pyrexial and had bilateral 7th cranial 
nerve palsy. 

What is the preferred drug? 

1- Prednisolone 

2- Doxycycline 

3- Tazocin 

4- Ceftriaxone 



^ [ Q: 1964 ] MRCPass - 2010 May 

# A 60 year old man was found to have 
a parietal lobe stroke. 

Which one of the following is a sign which may 
be present? 

1- Acalculia 

2- Homonymous superior quadrantanopia 

3- Perseveration 

4- Primitive reflexes 

5- Wernicke's (receptive) aphasia 


5- Gentamicin 


Answer & Comments 

Answer: 4- Ceftriaxone 

Lyme Disease (LD) is a multisystem disease 
affecting the nervous system, skin, joints, and 
heart. 

Erythema migrans, the characteristic 
dermatologic lesion of LD, is described here. 
Early localized disease manifests within 3-30 
days presenting with erythema migrans, 


Dr. Khalid Yusuf El-Zohry - Sohag Teaching Hospital (01118391123) 

Ref MRCPass OE OE 2012 PasTest 2009 PassMedicine 2009 PasTest Exam ReviseMRCP 



ITT 





























El-zohry MRCP Questions Bank (Port 1) - 2013 


(For my personal use) 


myalgia, fatigue, headache, fever, 
lymphadenopathy, and arthralgia. 

This usually occurs 30 to 120 days post¬ 
infection and is characterized by erythema 
migrans , fatigue, lymphadenopathy, 
conjunctivitis, neck pain, cardiac 
abnormalities, radiculoneuritis, arthritis, and 
CNS manifestations. 


Many of the inital signs in tricyclic 
antidepressant (TCA) overdose are associated 
to the anticholinergic effects of TCAs such as 
dry mouth, blurred vision, urinary retention, 
constipation, dizziness and vomiting. 

In a patient who is acidotic who is at risk of 
cardiac arrhythmias and seizures, serum 
bicarbonate is recommended. 


First-line treatment for early disease is 
doxycycline (100 mg PO twice a day for 14 
days) or amoxicillin unless there is 
neurological involvement. 

In this case, there is neurological involvement 
(neuroborreliosis) hence the preferred 
treatment would be intravenous ceftriaxone. 
There remains some controversy about 
whether doxycycline is just as effective. 


[ Q: 1966 ] MRCPass - 2010 May 

A 25 year-old woman with a history 
of depression is brought to the hospital with 
decreased conscious level and a brief seizure. 
She had taken an overdose of tricyclic 
antidepressants 12 hours prior. Her GCS is 
12/15, she is tachycardic to 120 bpm and her 
blood pressure is 96/62 mmHg. She appears 
flushed and her skin is dry. A blood gas shows: 

pH -7.15 

p02 -13.3 kPa 

pC02 -3.5 kPa 

base excess - negative 8.5 

What should be the management? 

1- Gastric Lavage 

2- Charcoal 




[ Q: 1967 ] MRCPass - 2010 May 

A 30 year old lady ate in a Chinese 
restaurant before presenting with vomiting 
and diarrhoea 4 hours later. She continued to 
have the symptoms on the following day. 


What is the most likely organism which is 
responsible? 


1- Campylobacter 

2- Escherichia coli 


3- Bacillus cereus 

4- Salmonella 


5- Giardia lamblia 


Answer & Comments 

Answer: 3- Bacillus cereus 

B. cereus food poisoning results from the 
ingestion of preformed enterotoxins, 
producing predominantly vomiting and 
diarrhea. It is implicated in food takeaw ays 
when rice is re-heated. The vomiting form is 
most often associated with ingestion of a heat 
stable toxin from contaminated rice, while the 
diarrheal form is most often associated with 
ingestion of a heat labile toxin from 
contaminated meat or vegetables. 


3- 8.4% bicarbonate infusion 

4- Naloxone infusion 

5- Flumazenil infusion 


Answer & Comments 


Answer: 3- 8.4% bicarbonate infusion 



[ Q: 1968 ] MRCPass - 2010 May 

A 51-year-old woman with a 
longstanding medical history of rheumatoid 
arthritis was admitted to hospital because of 
nausea and leg sw elling. She said that she had 
worsening ankle sw elling over the last two 
years. 


Dr. Khalid Yusuf El-Zohry - Sohag Teaching Hospital (01118391123) 

Ref MRCPass OE OE 2012 PasTest 2009 PassMedicine 2009 PasTest Exam ReviseMRCP 



778 




























El-zohry MRCP Questions Bank (Port 1) - 2013 


(For my personal use) 


She was previously on gold injections and over 
the last few months she took regular 
ibuprofen because of worsening joint pains. 

Blood results reveal : sodium 137 mmol/I, 
potassium 4.5 mmol/I, urea 25 mmol/l, 
creatinine 290 mmol/l. Urine dipstick 

shows protein +++, blood +. 

Whot is the most likely couse of renoi failure? 

1- Interstitial nephritis 

2- Papillary necrosis 

3- Acute tubular necrosis 

4- Renal artery stenosis 

5- Amyloidosis 


1- Bundle branch block 

2- Supraventricular tachycardia 

3- Ventricular tachycardia 

4- Cerebral embolism 

5- Acute Myocardial infarction 

Answer & Comments 

Answer: 3- Ventricular tachycardia 

This patient had a syncopal event and also a 
history of ischaemic heart disease. 

The ECG changes are not specific for 
myocardial infarction, and in this case the 
clinical history suggests VT is most likely. 


Answer & Comments 

Answer: 5- Amyloidosis 

The most common renal disorders associated 
with RA are membranous nephropathy, 
secondary amyloidosis, a focal, mesangial 
proliferative glomerulonephritis, rheumatoid 
vasculitis, and analgesic nephropathy. 

NSAID nephropathy may lead to acute tubular 
necrosis, interstitial nephritis and papillary 
necrosis. However the history of ankle sw 
elling and proteinuria suggests Amyloidosis. In 
rheumatoid arthritis secondary (AA) 
Amyloidosis is associated, and it presents 
typically with impaired renal function and 
proteinuria. 


^ [ Q: 1970 ] MRCPass - 2010 May 

fi - 

# A 55 year old man presented with 
severe retrosternal chest pain. He has history 
of hypertension and diabetes. 

His ECG shows anterior wall myocardial 
infarction with ST elevation in leads VI to V4. 
He has been given Aspirin, Clopidogrel and 
Fondaparinux and is currently awaiting 
angiography. However, he continues to have 
chest pains. 

Which other drug should be given? 

1- Tissue plasminogen activator 

2- Streptokinase 

3- Statin 


^ [ Q: 1969 ] MRCPass - 2010 May 

II - 

# A 65 year old man was admitted 
following a collapse with loss of 
consciousness. A history from a friend 
revealed that he had a previous myocardial 
infarction and was currently on several cardiac 
medications. An ECG showed >2mm ST 
elevation in Leads Vl-3 with no reciprocal 
changes and there was no evidence of q 
waves. 

Whot is the most likely diagnosis? 


4- ACE-inhibitor 

5- GIIBIIIA inhibitor 

Answer & Comments 

Answer: 5- GIIBIIIA inhibitor 

This patient is unstable and should be 
considered for a GIIBIIIA inhibitor such as 
tirofibran whilst awaiting coronary 
angiography. 

Abxicimab, another GIIBIIIA inhibitor, is also 
frequently used prior to angioplasty. 


Dr. Kholid Yusuf El-Zohry - Sohog Teaching Hospital (01118391123) 

Ref MRCPass OE OE 2012 PasTest 2009 PassMedicine 2009 PasTest Exam ReviseMRCP 






























El-zohry MRCP Questions Bank (Port 1) - 2013 


(For my personal use) 


Thrombolysis is not indicated if the patient is 
going to undergo angiography because the 
risk of bleeding is high. 


omeprazole twice a day and has been 
compliant. A repeat endoscopy now shows 
two oesophageal ulcers. 



[ Q: 1971 ] MRCPass - 2010 May 

A 60 year old woman presented with 
a 10-year history of increasing stiffness and 
immobility which have led to multiple falls. 
She had had some difficulty in fine finger 
movement and urinary incontinence. On 
examination she had a lying BP of 130 / 80 
mmHg and a standing BP of 90 / 60 mmHg. 
She had a mask like facies, bradykinesia, and 
resting a tremor in both of her hands. There 
was also short-term memory loss. 


What is the diagnosis? 

1- Lewy body dementia 


What is the appropriate investigation? 

1- Barium enema 

2- insulin tolerance test 

3- H. pylori serology 

4- Colonoscopy 

5- Gastrin levels 


Answer & Comments 

Answer: 5- Gastrin levels 

Diarrhea and recurrent gastric ulceration is 
common with Zollinger Ellison syndrome 
(gastrinoma). 


2- Alzheimer's disease 

3- Parkinson's disease 

4- Multi system atrophy 

5- Progressive supranuclear palsy 


There would be demonstrable high fasting 
plasma gastrin levels. Gastrinomas may occurs 
as part of a multiple endocrine neoplasia 
syndrome type 1. 


Answer & Comments 

Answer: 4- Multi system atrophy 

Multi system atrophy (Shy drager syndrome) 
consists of a cluster of several features: 

Parkinsonism (increased tone, bradykinesia, 
resting tremor) 

Autonomic dysfunction (postural hypotension, 
urinary incontinence, constipation) 

Ataxia (leading to falls) 


^ [ Q: 1973 ] MRCPass - 2010 May 

# A 54-year-old woman with history of 
uncontrolled hypertension presented to the 
eye clinic with sudden vision loss in the right 
eye. Ophthalmologic exam showed visual 
acuity of hand motion in the right eye and 6/6 
in the left eye. Pupil exam showed sluggish 
right pupil with relative afferent papillary 
defect, and a reactive pupil on the left. 

Examination of the right eye showed retinal 
haemorrhages and extensive areas of capillary 
non-perfusion. 


Unlike Parkinson's disease, L-dopa is not What is the diagnosis? 

effective and treatment is mainly supportive. . ~, 

y 1- Glaucoma 


[ Q: 1972 ] MRCPass - 2010 May 

A 42 year old man presents with 
frequent diarrhoea and upper abdominal 
pains. 

He had a partial gastrectomy 6 months ago for 
upper Gl bleeding. He is now on high dose 



2- Retinal vasculitis 

3- Central retinal vein thrombosis 

4- Central retinal artery occlusion 

5- Malignant hypertension 


Dr. Khalid Yusuf El-Zohry - Sohag Teaching Hospital (01118391123) 

Ref MRCPass OE OE 2012 PasTest 2009 PassMedicine 2009 PasTest Exam ReviseMRCP 



780 



























El-zohry MRCP Questions Bank (Port 1) - 2013 


(For my personal use) 


Answer & Comments 

Answer: 3- Central retinal vein thrombosis 

Clinically, central retinal vein thrombosis 
presents with visual loss; the fundus may 
show retinal hemorrhages, dilated tortuous 
retinal veins, cotton-wool spots, macular 
edema, and optic disc edema. 

Major risk factors are hypertension, diabetes, 
and atherosclerosis. Other risk factors are 
glaucoma, syphilis, sarcoidosis, vasculitis, 
hyperviscosity syndromes (multiple myeloma, 
Waldenstrom's macroglobulinemia, and 
leukemia), high homocysteine levels, sickle 
cell, and HIV. 

^ [ Q: 1974 ] MRCPass - 2010 May 

£ . 

* A 36 year old Caucasian female 
presented with malaise, joint pains and 
Raynaud's phenomenon for the last 6 months. 
On physical examination she was afebrile and 
had a supine blood pressure of 110/80mm Hg 
Blood tests revealed: 

Hb 11.5 g/dl, MCV85 fl 

erythrocyte sedimentation rate of 80 mm/first 
hour 

antinuclear antibody (ANA) - strongly positive 

antitopoisomerase I antibody (formerly anti 
SCL-70 antibody) positive 

normal C3 and C4 

anti-DNA, anti-centromere, anti-RNP, anti-Ro 
and La antibodies - negative 

Whot other feature is likely to be present? 

1- Proximal myopathy 

2- Heliotrope rash 

3- Calcinosis 

4- Telangiectasia 

5- Renal involvement 

Answer & Comments 


The patient is likely to have a diffuse form of 
scleroderma. 

The limited cutaneous form of scleroderma is 
CREST syndrome (calcinosis, raynauds, 
esophageal dysmotility, sclerodactyly and 
telangiectasia). 

The diffuse form of scleroderma is more 
rapidly progressing and affects the skin 
(cutaneous scleroderma) and one or more 
internal organs, frequently the kidneys (renal 
crisis), esophagus, heart (pulmonary 
hypertension) and lungs (pulmonary fibrosis). 

In diffuse scleroderma, antinuclear antibodies 
are present in about 95% of patients. 

Topoisomerase I antibodies (formerly Scl-70) 
are present in approximately 30% of patients 
with diffuse disease (absent in limited disease) 
and are associated with pulmonary fibrosis. 

Anticentromere antibodies are present in 
about 60-90% of patients with limited disease 
and are rare in patients with diffuse disease. 

^ [ Q: 1975 ] MRCPass - 2010 May 

f\ - 

Which blood gas result may be an 
analytical error? 

1- PH 7.6, PaC02 - 2, P02 -13, Bicarbonate 30 

2- PH 7.3, PaC02 - 7, P02 -10, Bicarbonate 14 

3- PH 7.5, PaC02 - 8, P02 -10, Bicarbonate 14 

4- PH 7.5, PaC02 - 6, P02 -12, Bicarbonate 26 

5- PH 7.2, PaC02 - 9, P02 9, Bicarbonate 12 

Answer & Comments 

Answer: 3- PH 7.5, PaC02 - 8, P02 - 10, 
Bicarbonate 14 

The patient has a high C02 and low 
bicarbonate which would suggest 
uncompensated respiratory acidosis, hence 
the pH shows an alkalosis which is too high to 
fit the scenario. 


Answer: 5- Renal involvement 


Dr. Khalid Yusuf El-Zohry - Sohag Teaching Hospital (01118391123) 

Ref MRCPass OE OE 2012 PasTest 2009 PassMedicine 2009 PasTest Exam 




ReviseMRCP 

781 



























El-zohry MRCP Questions Bank (Port 1) - 2013 


(For my personal use) 



[Q: 1976] MRCPass - 2010 
September 

A 55 year old lady has known mitral valve 
stenosis from rheumatic heart disease. 


She presents with new symptoms of 
breathlessness. Upon examination, which one 
of the following signs would suggest that she 
has mitrol regurgitation? 

1- Displaced apex beat 

2- Early diastolic murmur in the pulmonary 
area 


3- Right ventricular heave 

4- V wave seen with the JVP 


5- Opening snap 


Answer & Comments 

Answer: 1- Displaced apex beat 

In mitral stenosis, the apex beat is classically 
tapping in nature and not displaced. 

Mitral regurgitation on the other hand, causes 
ventricular strain and dilatation. Most of the 
other signs described in the above options can 
occur with pulmonary hypertension due to 
significant mitral stenosis. 


A 49-year-old woman has a history of 
schizophrenia and is currently on haloperidol 
(10 mg twice a day) and risperidone (1 mg 
twice a day). 

The atypical antipsychotic effect of risperidone 
is due to its action on the: 

1- Alpha adrenergic receptor 

2- Adrenaline receptor 

3- Acetylcholine receptor 

4- Serotonin receptor 

5- Dopamine 2 receptor 



[Q: 1977] MRCPass-2010 
September 


Answer & Comments 

Answer: 4- Serotonin receptor 

Risperidone is an atypical antipsychotic drug 
which has more pronounced serotonin 
antagonism than dopamine antagonism. 

It has actions at several 5-HT (serotonin) 
receptor subtypes. The 'atypical' effects is due 
to the effectiveness via the 5-HT2A receptor, 
which reduces the extrapyramidal side effects 
(EPS) experienced with the typical 
neuroleptics due to the action on the 
dopamine receptor. 


A 40 year old man with diabetic nephropathy 
and hemodialysis-dependent renal failure is in 
clinic for evaluation of her blood results. She 
had been found to have the following : 
potassium 5.2 mmol/l, calcium 2.20 (2.25-2.7) 
mmol/I, albumin 42 g/l, phosphate 1.55 (0.8- 
8) mmol/l, parathyroid hormone (PTH) levels 
over 220 pg/mL (normal <65 pg/mL). 

What should be prescribed? 

1- Calcium acetate 

2- Cinacalcet 

3- Alucaps 

4- Alfacalcidol 

5- Sando phosphate 


[Q: 1978] MRCPass-2010 
September 



Answer & Comments 

Answer: 1- Calcium acetate 

This question refers to a patient with likely 
secondary hyperparathyroidism and 
hyperphosphataemia rather than 

hyperparathyroidism being the main problem. 

The most significant long-term complication of 
chronic uncontrolled hyperphosphatemia is 
the development of vascular calcifications. 
Patients with secondary hyperparathyroidism 


Dr. Khalid Yusuf El-Zohry - Sohag Teaching Hospital (01118391123) 

Ref MRCPass OE OE 2012 PasTest 2009 PassMedicine 2009 PasTest Exam 




ReviseMRCP 

783 































El-zohry MRCP Questions Bank (Port 1) - 2013 


(For my personal use) 


usually have a low -normal calcium and 
elevated parathyroid hormone (PTH). 

Patients with tertiary hyperparathyroidism 
may have hypercalcaemia along with a high 
PTH level. 

Phosphate binders can be used if 
hyperphosphatemia persists despite dietary 
phosphate restriction. These include calcium- 
based phosphate binders such as calcium 
carbonate, calcium acetate and non-calcium- 
based phosphate binders such as sevelamer 
hydrochloride. Calcium supplementation 
should be limited to less than 2 g/day. 

Cinacalcet is a calcimimetic which inhibits the 
release of PTH. It is recommended in patients 
with dialysis who have secondary 
hyperparathyroidism and a very high PTH level 
(800 pg/ml). 


A 60 year old lady has pain in her knees, 
shoulders, wrists and fingers. Examination of 
her hands reveals multiple symmetrical small 
joint involvement. The proximal and distal 
joints were affected. Joint X rays show the 
presence of osteophytes and 
chondrocalcinosis. She is currently on 
bendrofluazide and metformin tablets. A urate 
level on admission was 420 (<380 pmol/l). The 
rheumatoid factor was positive with a titre of 
1:30. 

What is the diagnosis? 

1- Polyarticular gout 

2- Calcium pyrophosphate deposition disease 

3- Systemic lupus erythematosus 

4- Haemochromatosis 



[Q: 1979] MRCPass - 2010 
September 


The diagnosis would fit with a subcategory of 
calcium pyrophosphate deposition disease, 
which is Pseudoosteoarthritis often involves 
the metacarpophalangeal (MCP) joints, wrists, 
elbow s, and shoulders, joints unlikely to be 
involved with primary osteoarthritis. 

It affects the knees most commonly and can 
involve the proximal interphalangeal (PIP) 
joints and spine. Osteophytes and 
chondrocalcinosis are a common radiological 
finding in patients with a pseudoosteoarthritis 
condition and usually are present along the 
second and third metacarpal heads. In 
addition, older individuals may have low -titer- 
positive rheumatoid factor as in this case. 

Pseudogout is one manifestation of calcium 
pyrophosphate deposition disease, where 
joint aspiration fluid might show rhomboid¬ 
shaped, positively birefringent crystals. This 
case is less likely to be gout as it usually 
affects the knee and big toe. In acute gout the 
uric acid level is usually normal, so an early 
urate level is not helpful. 


[Q: 1980] MRCPass-2010 
September 

A 55 year old lady with COPD presents with 
shortness of breath. She has nebulisers at 
home but not homeoxygen. On admission, her 
oxygen saturations were 88% and the nurse 
asks how you would want oxygen given. 

What should you recommend? 

1- Nasal cannulae 

2- Plain mask with non rebreathe bag 

3- Plain mask only 

4- Venturi mask 

5- Humidified oxygen mask 



5- Rheumatoid arthritis 

Answer & Comments 

Answer: 2- Calcium pyrophosphate deposition 
disease 


Answer & Comments 

Answer: 4- Venturi mask 

The venturi mask, also known as an air- 
entrainment mask, delivers a known oxygen 


Dr. Khalid Yusuf El-Zohry - Sohag Teaching Hospital (01118391123) 

Ref MRCPass OE OE 2012 PasTest 2009 PassMedicine 2009 PasTest Exam ReviseMRCP 


























El-zohry MRCP Questions Bank (Port 1) - 2013 


(For my personal use) 


concentration to patients on controlled 
oxygen therapy, this is particularly important 
in COPD patients. 

Venturi masks are considered high-flow 
oxygen therapy devices. This is because 
venturi masks are able to provide total 
inspiratory flow at a specified FI02 to patients 
therapy. The kits usually include multiple jets 
in order to set the desired FI02 which are 
usually color coded. In the UK, the colours and 
respective delivery concentrations are; Blue 
24%, White 28%, Yellow 35%, Red 40%, Green 
60%). 


A 24 year old lady was complaining of tremors 
and sw eating for 6 weeks and referred for 
assessment. She has lost 6 kg in weight. On 
examination, she has tremors in the hands 
and a palpable smooth goitre. An nuclear 
medicine scan of the neck showed increased 
and patchy uptake of radio isotope of the 
thyroid glands. Following tests, she is 
diagnosed with thyrotoxicosis. Laboratory 
data confirmed the following results: free T4- 
48 (10-24) pmol/l, TSH < 0.3 (0.3-4) mU/l, 
antithyroid antibodies negative. 

Whot is the definitive treatment? 

1- Propranolol 

2- Prednisolone 



[Q: 1981] MRCPass - 2010 
September 


or technetium isotopes. Whilst antithyroid 
drugs such as carbimazole or methimazole as 
well as propanolol can be used for short term 
treatment, the definitive treatment should be 
either radioactive iodine or thyroid surgery. 



[Q: 1982] MRCPass-2010 
September 

A 40-year-old woman complains of recurrent 
migraine type headaches. The headaches have 
been occurring every week, increasing in 
intensity and are associated with flashing 
lights. She was treated with paracetamol, 
diclofenac and sumatriptan with little 
improvement. 

Which of the following medications would you 
prescribe? 

1- Pizotifen 

2- Ergotamine 

3- Propanolol 

4- Verapamil 

5- Sodium valproate 


Answer & Comments 

Answer: 3- Propanolol 

The actual question did not specify whether 
the medication was for acute treatment or 
prophylaxis, but the way it was phrased 
suggested that it asked for prophylaxis. 


3- Thyroxine 

4- Radioactive iodine 

5- Carbimazole 


Propanolol, verapamil and sodium valproate 
can all be used for prophylaxis, and propanolol 
is the most commonly prescribed. 


Answer & Comments 

Answer: 4- Radioactive iodine 

This patient has toxic multinodular goitre with 
features of thyrotoxicosis such as sweats, 
palpitations, tremors and weight loss. 

On nuclear imaging there is usually patchy 
increased uptake of either radioactive iodine 


n 

A 25 year old lady presents with abdominal 
pains and lethargy. She has a history of 
diabetes and was on insulin. 

Clinical examination was unremarkable and 
she had an abdominal X ray which showed the 
presence of renal calculi. 


[Q: 1983] MRCPass-2010 
September 


Dr. Khalid Yusuf El-Zohry - Sohag Teaching Hospital (01118391123) 

Ref MRCPass OE OE 2012 PasTest 2009 PassMedicine 2009 PasTest Exam 




ReviseMRCP 

785 






























El-zohry MRCP Questions Bank (Port 1) - 2013 


(For my personal use) 


Investigations revealed the following: 

arterial blood pH 7.30 (7.38-7.44) 

serum bicarbonate 12.6 mmol/L (21-28 
mmol/L) 

sodium 146 (136-145 mmol/L) 
potassium 2.8 (3.5-5mmol/L) 
chloride 122 (98-106mmol/L) 

Anion gap was 15 (normal 7-16 mmol/L) 

Urine pH of 6.5 (normal range 5-9) 

Whot is the likely diagnosis? 

1- Multiple myeloma 

2- Renal tubular acidosis type 1 

3- Nephrotic syndrome 

4- Homocystinuria 

5- Porphyria 

Answer & Comments 

Answer: 2- Renal tubular acidosis type 1 

The diagnosis of Type 1 RTA is based on the 
findings of systemic acidosis, low bicarbonate, 
hypokalemia, a normal anion gap and 
relatively alkaline urine despite the acidosis. 

Type 1 RTA can be familial with autosomal 
dominant as the most common mode of 
inheritance. Typical features of type 1 RTA are 
osteomalacia and nephrocalcinosis. 


^ [ Q: 1984 ] MRCPass - 2010 

September 

A 57 year old man had a myocardial infarction 
and was transferred for coronary angiography. 
A day following this, he developed red dusky 
discoloration on the right foot and became 
pyrexial. He had a femoral bruit on the right 
side on examination. Eosinophilia was noted 
on the blood tests and he had a creatinine of 
650 umol/l. 

Which one of the following is likely? 

1- Baker's cyst 

2- Compartment syndrome 

3- Cellulitis 

4- Cholesterol embolism 

5- Anaphylactic reaction 

Answer & Comments 

Answer: 4- Cholesterol embolism 

Cholesterol embolism should be suspected in 
a patient with atherosclerotic disease who has 
fever and distal ischaemia. 

Eosinophilia strongly suggests 

atheroembolization and is present in as many 
as 80% of patients with cholesterol embolism 
syndrome. The patient is also likely to have 
cholesterol emboli to the kidneys causing 
renal impairment. 



Associated causes are Sjogren's syndrome, 
SLE, rheumatoid arthritis, renal 
transplantation and sickle cell anaemia. 

Type II RTA is associated with disorders such 
as cystinosis, galactosaemia,Wilson's disease, 
multiple myeloma and Paroxysmal nocturnal 
haemoglobinuria. The distal intercalated cells 
function normally, so the acidemia is less 
severe than dRTA and the urine can acidify to 
a pH of less than 5.3. 



[Q: 1985] MRCPass-2010 
September 


A 48-year-old Filipino man was first noted to 
have hematuria 5 years ago. He complains of 
noticing haematuria both at home and at 
work several times a week. There are no other 
symptoms. His blood pressure is 130 / 70 
mmHg. 


Urinalysis during that admission showed trace 
proteinuria and 2+ blood on the urinary 
dipstick. His blood tests show : 

sodium 135 mmol/l 


Dr. Kholid Yusuf El-Zohry - Sohog Teaching Hospital (01118391123) 

Ref MRCPass OE OE 2012 PasTest 2009 PassMedicine 2009 PasTest Exam 




ReviseMRCP 

786 























El-zohry MRCP Questions Bank (Port 1) - 2013 


(For my personal use) 


potassium 4.5 mmol/l 
urea 6 mmol/l 
creatinine 80 |imol/l 
What is the likely diagnosis? 

1- Thin basement membrane disease 

2- Focal segmental glomerulosclerosis 

3- Minimal change disease 

4- Transitional cell carcinoma 

5- Post streptococcal glomerulonephritis 


study failed to detect a thrombus, and there 
was no radiologic evidence of a fracture. The 
patient was diagnosed with a muscle strain 
Achilles tendon rupture. 

Which drug may have caused this? 

1- Ciprofloxacin 

2- Metronidazole 

3- Amoxicillin 

4- Nitrofurantoin 

5- Gentamicin 


Answer & Comments 

Answer: 1- Thin basement membrane disease 

Thin basement membrane disease (TBMD, 
also known as benign familial hematuria and 
thin basement membrane nephropathy) is, 
along with IgA nephropathy, the most 
common cause of asymptomatic hematuria. 

The only abnormal finding in this disease is a 
thinning of the basement membrane of the 
glomeruli in the kidneys. It has a benign 
prognosis. 

Most patients with thin basement membrane 
disease are incidentally discovered to have 
microscopic hematuria on urinalysis. The 
blood pressure, kidney function and the 
urinary protein excretion are usually normal. 
Mild proteinuria (less than 1.5 g/day) and 
hypertension are seen in a small minority of 
patients. Frank hematuria and loin pain should 
prompt a search for another cause, such as 
kidney stones or loin pain-hematuria 
syndrome. 

3 

A 61 year old man has discomfort in the left 
ankle was noted 2 days. Several weeks ago he 
had been on a course of antibiotics. He 
presented to the emergency department with 
mild, nonpitting oedema of the left calf and 
ankle, and pain on plantar flexion. A Doppler 


[Q: 1986] MRCPass - 2010 
September 


Answer & Comments 

Answer: 1- Ciprofloxacin 

Disruptions of tendons in adults, including 
rupture, have been reported in association 
with fluoroquinolones (ciprofloxacin). 

The Achilles tendon is the site most frequently 
associated with such adverse outcomes. 


[Q: 1987] MRCPass-2010 
September 

A 55 year old man has large spade-like hands 
and finds that he is having to buy shoes of 
increasing size in the last two years. On 
examination of his visual fields, bitemporal 
hemianopia was found. 

Which one of the following tests should be 
done? 

1- Insulin tolerance test 

2- Hydrocortisone curve 

3- CT scan of the chest 

4- Dexamethasone suppression test 

5- Oral glucose tolerance test with growth 
hormone 



Answer & Comments 

Answer: 5- Oral glucose tolerance test with 
growth hormone 

The case scenario suggests acromegaly. 


Dr. Khalid Yusuf El-Zohry - Sohag Teaching Hospital (01118391123) 

Ref MRCPass OE OE 2012 PasTest 2009 PassMedicine 2009 PasTest Exam ReviseMRCP 




























El-zohry MRCP Questions Bank (Port 1) - 2013 


(For my personal use) 


Growth hormone suppression during oral 
glucose tolerance and elevated IGF-I levels 
form the main diagnostic criteria in 
acromegaly. IGF1 provides the most sensitive 
and useful lab test for the diagnosis of 
acromegaly. A single value of the Growth 
hormone (GH) is not useful in view of its 
pulsatality (levels in the blood vary greatly 
even in healthy individuals). GH levels taken 2 
hours after a 75 or 100 gram glucose 
tolerance test are helpful in the diagnosis: GH 
levels are suppressed below 1 ?g/L in normal 
people, and levels higher than this cutoff are 
confirmatory of acromegaly. 


[Q: 1988] MRCPass - 2010 
September 

A 60 year old woman has recently had 
lethargy and arthralgia. She was diagnosed as 
having influenza infection, as there was an 
outbreak in the area recently. She presents 1 
week later with a cough and breathlessness. 
On examination, she had bilateral crackles 
audible on examination. CXR confirms bilateral 
consolidation. 

Which one of the following is most likely as a 
co use? 

1- Legionella 

2- Mycoplasma 

3- Streptococcus pneumoniae 

4- Klebsiella 

5- Staphylococcus aureus 

Answer & Comments 

Answer: 5- Staphylococcus aureus 

Normal incidence of staph aureus pneumonia 
is 2%, However this is significantly increased in 
iv drug users and influenzae virus infections. 

Post influenzae staph aureus pneumonia is 
characterised by rapid clinical deterioration 
with septicaemia. 




[Q: 1989] MRCPass-2010 
September 


A 52-year-old man was admitted because of 
progressive shortness of breath. He has been 
a smoker of 5 cigarettes a day for the past two 
years. He has a past medical history of 
seropositive rheumatoid arthritis, diabetes 
mellitus and hypertension. On examination 
the fingers were clubbed and there were 
bilateral basal crepitations. Lung function tests 
showed: 


forced expiratory volume in one second 
(FEV1) of 2.10 I (predicted 3.2 1) 

forced vital capacity (FVC) of 2.30 I (predicted 
4). 

Carbon monoxide transfer factor (TLCO) was 
reduced to 70% predicted. 

There was 5% improvement following 
salbutamol nebulisers. 


Whot is the likely diagnosis? 

1- COPD 


2- Asthma 


3- Pulmonary embolus 

4- Pulmonary fibrosis 

5- Pneumothorax 


Answer & Comments 

Answer: 4- Pulmonary fibrosis 

Although there is slight improvement with 
salbutamol, the best answer is interstitial lung 
disease/ pulmonary fibrosis. 

The FEV1 / FVC ratio is 90% which suggests 
restrictive lung disease rather than obstructive 
lung disease. There is also reduced transfer 
factor (TLCO) because pulmonary fibrosis 
thickens the alveolar walls. Both of these 
features are consistent with pulmonary 
fibrosis. 


Dr. Kholid Yusuf El-Zohry - Sohog Teaching Hospital (01118391123) 

Ref MRCPass OE OE 2012 PasTest 2009 PassMedicine 2009 PasTest Exam ReviseMRCP 


























El-zohry MRCP Questions Bank (Port 1) - 2013 


(For my personal use) 



[Q: 1990] MRCPass - 2010 
September 

A patient who recently had a traumatic car 
accident had to undergo splenectomy during 
emergency surgery. 

Which of the following is likely to be found in 
the blood film of the patient? 

1- Schistocytes 

2- Poikilocytes 

3- Howell Jolly bodies 

4- Heinz bodies 

5- Blast cells 


that he had a HBAlc of 9. On examination, he 
has a BMI of 34. Urine dipstick showed 
Glucose ++ and Ketones +. 

What is the likely diagnosis? 

1- DM typel 

2- DM type 2 

3- MODY 

4- Pancreatic failure 

5- Drug induced diabetes 

Answer & Comments 

Answer: 2- DM type 2 


Answer & Comments 


Answer: 3- Howell Jolly bodies 


This patient is unlikely to have Type 1 diabetes 
and Maturity Onset Diabetes of the Young as 
the age is > 25 years. 


Howell Jolly bodies are nuclear remnants 
which appear as basophilic (purple) spots on 
the eosinophilic (pink) erythrocyte on a 
standard H&E stained blood smear. 

These inclusions are normally pitted out by 
the spleen during erythrocyte circulation, but 
will persist in individuals with functional 
hyposplenia or asplenia. 



Howell Jolly Body 


The obese characteristic suggests type 1 
diabetes and ketonuria which is mild can 
occur in type 2 diabetes. 


3 

A 65 year old lady has epigastric pain for 
several months and is referred for endoscopy. 

A biopsy taken during the endoscopy confirms 
MALT lymphoma. 

Which one of the following associations for 
Helicobacter pylori infection and gastric 
lymphoma? 

1- Gastro oesophageal reflux disease 

2- Non ulcer forming gastritis 

3- Duodenal ulcer 

4- Pancreatic fistula 


[Q: 1992] MRCPass-2010 
September 



[Q: 1991] MRCPass-2010 
September 


A 40 year old man has recently been 
diagnosed with diabetes. He has been put on 
oral hypoglycaemic drugs. 


History revealed that he has three cousins 
who also have diabetes. Blood tests revealed 


5- Pernicious anaemia 


Answer & Comments 

Answer: 2- Non ulcer forming gastritis 

Malignancies that occur in mucosa-associated 
lymphoid tissue (MALT) are called MALT 
lymphomas (MALTomas). 


Dr. Khalid Yusuf El-Zohry - Sohag Teaching Hospital (01118391123) 

Ref MRCPass OE OE 2012 PasTest 2009 PassMedicine 2009 PasTest Exam ReviseMRCP 





























El-zohry MRCP Questions Bank (Port 1) - 2013 


(For my personal use) 


Most of the MALTomas occur in the stomach, 
and roughly 70% of gastric MALTomas are 
associated with Helicobacter pylori infection. 

H. pylori gastritis is common in individuals 
who develop gastric lymphomas. Although the 
aetiology is unknow n, this association is 
established. 



[Q: 1993] MRCPass - 2010 
September 


A couple presents to the genetic clinic for 
counselling. The male partner has haemophilia 
whilst the female partner has been screened 
and is not a carrier. 


Whot is the chance that a son would have 
haemophilia? 

1- 25% 

2- 50% 

3- 75% 

4- All sons 


5- None 


Answer & Comments 

Answer: 5- None 

Haemophilia is X linked recessive. 

In this case, if the affected X chromosome is 
designated Xa, the father is XaY and mother 
XX. The children would be carrying either XaX 
or XY. In this situation all female children will 
be carriers and all male children will not be 
affected. 


[Q: 1994] MRCPass-2010 
September 

A 65 year old woman presented with a four- 
year history of increasing stiffness and 
immobility which have led to multiple falls. 
She had had some difficulty in fine finger 
movement. Her Blood pressure was 130/90 
mmHg lying and 135/95 mmHg standing. 
Examination showed a mask like facies, 



bradykinesia, nuchal and limb rigidity. She had 
an asymmetrical tremor in her hands and 
cogw heel rigidity in the arms. She had 
diminished upward conjugate deviation of 
eyes. There was also short-term memory loss. 

What is the diagnosis? 

1- Lewy body dementia 

2- Alzheimer's disease 

3- Parkinson's disease 

4- Multi system atrophy 

5- Progressive supranuclear palsy 


Answer & Comments 

Answer: 5- Progressive supranuclear palsy 

A patient with parkinsonism and gaze palsy 
suggests a diagnosis of progressive 
supranuclear palsy. 

Progressive supranuclear palsy (PSP), is also 
known as Steele-Richardson-Olszew ski 
syndrome. Characteristics include 
supranuclear, primarily vertical, gaze 
dysfunction accompanied by extrapyramidal 
symptoms and cognitive dysfunction. The 
disease usually develops after the sixth 
decade of life. The most common symptoms 
at disease onset are postural instability and 
falls (60%); dysarthria (35%); bradykinesia 
(15%); and visual disturbances such as 
diplopia, blurred vision, burning eyes, and 
light sensitivity 


[Q: 1995] MRCPass-2010 
September 

A patient with atrial fibrillation was prescribed 
digoxin. 

What does it bind to in order to generate its 
mode of action? 

1- Calcium channel 

2- Potassium channel 

3- Beta receptors 



Dr. Khalid Yusuf El-Zohry - Sohag Teaching Hospital (01118391123) 

Ref MRCPass OE OE 2012 PasTest 2009 PassMedicine 2009 PasTest Exam ReviseMRCP 




























El-zohry MRCP Questions Bank (Part 1) - 2013 


(For my personal use) 


4- Na / K- ATPase 


4- Pneumocystis carinii pneumonia 


5- Aldosterone receptors 


5- Schistosomiasis 


Answer & Comments 

Answer 4- Na / K- ATPase 

Digoxin, also known as digitalis, is a purified 
cardiac glycoside. 

Digoxin binds to a site on the extracellular 
aspect of the ?-subunit of the Na+/K+ ATPase 
pump in the myocytes. 


[Q: 1996] MRCPass - 2010 
September 

A 36-year-old woman presented to the 
emergency department with fevers, cough, 
and dyspnoea for one week. 

She drinks 4 units of alcohol per day and 
admitted to regular intravenous drug abuse. 
On physical examination, the patient had a 
temperature of 38°C, was tachypnoeic and 
tachycardic. Blood pressure was 120/90 
mmHg. 

Examination of lungs revealed bilateral coarse 
crackles and rhonchi. She had a soft systolic 
murmur heard throughout the precordium. 
The chest X ray is shown below : 



What is the most likely diagnosis? 

1- Calcification of the lung 

2- Tuberculosis 

3- Tricuspid endocarditis 



Answer & Comments 

Answer: 3- Tricuspid endocarditis 

The best answers are either tuberculosis or 
endocarditis. 

In this case, due to a history of intravenous 
drug use the most likely scenario is tricuspid 
valve endocarditis leading to multiple lung 
abscesses (as shown on the CXR). If multiple 
cavities are present, the infection is likely due 
to haematogenous dissemination (septic 
emboli), and a source for this dissemination 
should be sought. The source could be right¬ 
sided endocarditis (usually staphylococcal) or 
infected venous thrombi. Tubercular cavities 
are usually located in the upper zone, either 
the posterior segment of the upper lobe or 
apical segment of lower lobe. 

[ Q: 1997 ] MRCPass - 2010 
September 

A 42 year old man had a cough and his GP 
organized a chest X ray. This showed a 
pneumothorax and he was referred to the 
hospital. He does not have any symptoms of 
breathlessness or chest pains. His BP was 
110/80 mmHg, 02 sats were 99% on air & 
pulse was 80/min. CXR shows that there is a 
pneumothorax on the left with calculated size 
involving 20% of the lung volume. 

Which is the most appropriate step? 

1- Observation as outpatient 

2- Observation as inpatient 

3- Needle aspiration as outpatient 

4- Needle aspiration as inpatient 

5- Discharge with outpatient follow up 



Dr. Khalid Yusuf El-Zohry - Sohag Teaching Hospital (01118391123) 

Ref MRCPass OE OE 2012 PasTest 2009 PassMedicine 2009 PasTest Exam 




ReviseMRCP 

791 























El-zohry MRCP Questions Bank (Port 1) - 2013 


(For my personal use) 


Answer & Comments 

Answer: 5- Discharge with outpatient follow 
up 

An air rim of 2 cm or more means that the 
pneumothorax occupies about 50% of the 
pleural cavity and so this patient is likely to 
have a rim of air < 2 cm. 

The British thoracic society recommends that 
a small pneumothorax of < 2 cm rim, with no 
significant symptoms to be managed 
conservatively (discharge with outpatient 
review and advise to return if breathlessness 
occurs). 

The 2010 guideline can be downloaded here: 
http://www.brit-thoracic.org.uk/Clinical- 
Information/Pneumothorax/Pneumothorax- 
Guideline.aspx 


Q 

A 33 year old man has had a 6 month history 
of bilateral hip pains and back pains. There is 
not past medical history of trauma to the 
back. Non steroidal anti-inflammatory drugs 
helped to relieve his symptoms. 

What is the likely diagnosis? 

1- Gluteus medius syndrome 

2- Osteoarthritis 

3- Avascular necrosis of the hip 

4- Sacroilitis 

5- Lumbar canal stenosis 


[Q: 1998] MRCPass - 2010 
September 


Answer & Comments 

Answer: 4- Sacroilitis 

Pain and stiffness in the lower back or 
buttocks, especially in the morning is typical of 
sacroilitis. 

It is typically helped by NSAIDS or steroids. X 
rays will help to confirm the diagnosis. It is 


associated with various inflammatory diseases 
e.g. ankylosing spondylitis, psoriatic arthritis. 

Gluteus medius syndrome can cause buttock 
pains but there should be a history of overuse 
of the gluteus muscles which make it 
consistent with the diagnosis. 


A 45 year old man presented with a history of 
wheeze, cough and fevers for over 3 months. 
On physical examination, chest auscultation 
revealed decreased air entry on right upper 
lung field. Chest X ray pulmonary infiltrates. 
Investigations show that the sputum culture 
was negative for acid fast bacilli. 

What is the best test to diagnose allergic 
bronchopulmonary aspergillosis? 

1- Lung function test 

2- Bronchoscopy 

3- Precipitin antibodies 

4- Eosinophil count 

5- CT scan of the chest 


[Q: 1999] MRCPass-2010 
September 



Answer & Comments 

Answer: 3- Precipitin antibodies 

ABPA is a hypersensitivity reaction to A 
fumigatus colonization of the 
tracheobronchial tree which eventually leads 
to bronchiectasis. 

Eosinophil counts an IgE levels are typically 
high. Lung function tests show decreased lung 
volumes and gas transfer. Skin prick testing to 
allergens from Aspergillus fumigatus are 
positive. Precipitin antibodies (IgG) are 
commonly found in the serum. Both of these 
tests are convenient to confirm a diagnosis of 
ABPA. 


Dr. Khalid Yusuf El-Zohry - Sohag Teaching Hospital (01118391123) 

Ref MRCPass OE OE 2012 PasTest 2009 PassMedicine 2009 PasTest Exam ReviseMRCP 



792 




























El-zohry MRCP Questions Bank (Part 1) - 2013 


(For my personal use) 


^ [ Q: 2000 ] MRCPass- 2010 

| | September 

A 46-year-old man presents with back pain 
and on lateral spine x rays was found to have 
vertebral collapse. His past history is 
unremarkable. He does not smoke and there 
is no significant alcohol history. A bone scan 
was organised which confirmed that he had 
osteoporosis. 

What test should be sent off to identify the 
couse of osteoporosis? 

1- Thyroid function 

2- Troponin 

3- Prolactin 

4- Testosterone 

5- Blood glucose 


Which one of following best investigation 
diagnose cause of renal failure? 

1- Urine for casts 

2- IV urogram 

3- Renal angiography 

4- Renal biopsy 

5- CT of the kidneys 

Answer & Comments 

Answer: 3- Renal angiography 

The history of hypertension, progressive renal 
impairment and small kidney size suggest 
renovascular disease. 

The best investigation in this case would be 
either renal angiography or MR angiography. 


Answer & Comments 


[ Q: 2002 ] MRCPass - 2010 

Answer: 4- Testosterone 

h_] 

September 


Osteoporosis in a young male is unusual, as it 
is usually seen in post menopausal women. 

It is known that testosterone deficiency in a 
young male and also androgen antagonists in 
older males with e.g. prostate cancer can 
cause osteoporosis. Any features of 
hypogonadism or hypercalcemia should be 
elicited. Hyperprolacinemia would causes 
hypogonadism so a testosterone level would 
be far more relevant. Hyperthyroidism would 
be present for a considerable length of time 
before producing osteoporosis. 


A 32 year old woman presents with blurred 
vision and headaches. She has described 
having had a severe headache at the back of 
the head and neck 3 weeks ago but did not 
seek help then. She has a history of 
hypertension. On examination, she had 
swollen discs with blurred disc margins 
bilaterally, and had bilateral 6th nerve palsy. 

What is the likely diagnosis? 

1- Multiple sclerosis 

2- Vertebral artery dissection 

3- Subarachnoid haemorrhage 



[Q: 2001] MRCPass-2010 
September 

A 65-year-old is referred for investigation of a 
progressive history of worsening kidney 
disease. He has a long history of hypertension. 
Dipstick analysis of urine shows Blood + and 
protein +. The Urea is 20 mmol/l and 
Creatinine 260 umol/l 100). An ultrasound of 
abdomen requested shows left sided kidney 
size of 7.5 cm and right kidney size of 10.2 cm. 


4- Benign intracranial hypertension 

5- Essential hypertension 

Answer & Comments 

Answer: 3- Subarachnoid haemorrhage 

Subarachnoid hemorrhage should alw ays be 
suspected in patients with a typical 
presentation which includes a sudden onset of 
severe headache (frequently described as the 


Dr. Khalid Yusuf El-Zohry - Sohag Teaching Hospital (01118391123) 

Ref MRCPass OE OE 2012 PasTest 2009 PassMedicine 2009 PasTest Exam ReviseMRCP 


































El-zohry MRCP Questions Bank (Port 1) - 2013 


(For my personal use) 


"w orst ever"), with nausea, vomiting, neck 
pain, photophobia, and loss of consciousness. 

Physical examination may reveal retinal 
hemorrhages, meningism, a diminished level 
of consciousness, and localizing neurologic 
signs. Localising signs usually includes third- 
nerve palsy (posterior communicating 
aneurysm), sixth-nerve palsy (increased 
intracranial pressure), bilateral low 
erextremity weakness or abulia (anterior 
communicating aneurysm), and the 
combination of hemiparesis and aphasia or 
visuospatial neglect (middle cerebral-artery 
aneurysm). 

In benign intracranial hypertension, patients 
usually present with symptoms related to 
increased intracranial pressure. These 
symptoms include headache, transient visual 
obscurations, and diplopia due to unilateral or 
bilateral sixth nerve palsy. However, in this 
case the acute nature of the headache does 
not make BIH likely. 


3 

A 58 year old man presents with dizziness and 
palpitations. The ECG shows a a broad 
complex tachycardia. 

Which of the following features favours 
ventricular tachycardia over supraventricular 
tachycardia with bundle branch block? 

1- Left bundle branch block 

2- Atrioventricular dissociation 

3- Heart rate of 180 

4- QRS complex of 120ms 

5- Saw tooth waves 


[Q: 2003] MRCPass - 2010 
September 


Answer & Comments 

Answer: 3- Heart rate of 180 
Differentiating SVT from VT 
Features that favour VT are : 


QRS of > 140ms, 

cannon a waves on JVP, fusion and/or capture 
beats 

dissociated p waves (p wave dissociation) 

history of ischaemic heart disease, 

right bundle branch block with left axis 
deviation, 

concordance of the QRS complexes in the 
chest leads 

HR >170 beats per minute. 

A patient is being assessed for renal 
transplantation. 

Which one of the following HLA compatibility 
is the most important? 

1- HLA A 

2- HLA B 

3- HLAC 

4- HLA DR 

5- HLAG 


[Q: 2004] MRCPass-2010 
September 



Answer & Comments 

Answer: 4- HLA DR 

ABO blood group matching is the most 
important, and HLA matching is a relatively 
minor predictor of transplant outcomes. 

However, among HLA matches, DR matching 
has a greater effect than that of B or A. A 
study found that HLA-DR mismatches (and the 
number of rejection episodes) correlated with 
poor long-term survival 



[Q: 2005] MRCPass-2010 
September 


A 52-year-old Asian man presented to the 
hospital with chest pain and his ECG showed 


Dr. Khalid Yusuf El-Zohry - Sohag Teaching Hospital (01118391123) 

Ref MRCPass OE OE 2012 PasTest 2009 PassMedicine 2009 PasTest Exam ReviseMRCP 






























El-zohry MRCP Questions Bank (Port 1) - 2013 


(For my personal use) 


antero lateral ST elevation myocardial 
infarction. He was thrombolysed with 
tenecteplase and his chest pains settled, two 
days later, he had worsening shortness of 
breath at rest. 

On physical exam patient was in respiratory 
distress sitting up right, blood pressure was 
150/90 mmHg, heart rate 100 bpm, regular, 
respiratory rate 28/min. Neck veins were not 
distended and he had no ankle edema. On 
examination of the cardiovascular system he 
had an apical systolic murmur. He had 
bilateral coarse crepitation all over the lungs. 

Whot is the likely cause of his deterioration? 

1- Dressler's syndrome 

2- Pericardial effusion 

3- Ventricular septal rupture 

4- Papillary muscle rupture 

5- Atrial Fibrillation 


1- Carbamazepine 

2- Chlorpropamide 

3- Fluoxetine 

4- Furosemide 

5- Lithium 

Answer & Comments 

Answer: 5- Lithium 

This patient is having drug induced 
nephrogenic Diabetes Insipidus based upon 
her polyuria, low urine osmolality, high plasma 
osmolality and high sodium. 

This is because of a lack of response of the 
renal tubules to the hormone ADH, leading to 
an inability to reabsorb water causing low 
urine osmolality and high urine sodium. The 
most likely causes of nephrogenic Diabetes 
Insipidus are Lithium and Demeclocycline. 


Answer & Comments 

Answer: 4- Papillary muscle rupture 

Papillary muscle rupture, as can be caused by 
a myocardial infarction and or ischemia, 
leading to the complication of mitral valve 
prolapse. 

The history here of myocardial infacrtion and 
also a systolic murmur of mitral regurgitation 
will fit this description. 



[Q: 2006] MRCPass - 2010 
September 


A 50 year old lady complained of having to 
pass urine frequently for several weeks. She 
was on medications for bipolar disorder, 
hypertension and diabetes. On investigation 
the following results were found: 


Serum sodium 149 mmol/l 


Plasma osmolality 304 mosmol/l (275-290) 
Urine osmolality 150 mosmol/l (350-1000) 
What drug may have caused this? 



[Q: 2007] MRCPass-2010 
September 


A 70 year old woman presents to A&E with 
decreased consciousness and weakness. Her 
neighbour has not seen her for several days 
but found her on the floor. A letter provided 
information that she had a history of 
hypothyroidism and was normally on beta 
blockers and thyroid replacement. On 
examination, her temperature is 35°C and she 
has a blood pressure of 85 / 40 mmHg. GCS is 
11 / 15. A Blood sugar was measured at 2.3 
mmol/l. 


Blood results show : 


sodium 127 mmol/l 

potassium 3.6 mmol/l 

urea 6 |imol/l 

creatinine 100 |imol/l 

What is the next best management step? 

1- 10% Dextrose solution 


2- Oral Thyroxine 


Dr. Khalid Yusuf El-Zohry - Sohag Teaching Hospital (01118391123) 

Ref MRCPass OE OE 2012 PasTest 2009 PassMedicine 2009 PasTest Exam 




ReviseMRCP 

795 

























El-zohry MRCP Questions Bank (Port 1) - 2013 


(For my personal use) 


3- IV Hydrocortisone 

4- IV T3 

5- Sliding scale insulin 


Answer & Comments 

Answer: 3- IV Hydrocortisone 

The scenario is consistent with either 
myxedema coma or Addison's disease. 

In both situations, IV hydrocortisone is 
recommended. 

Myxedema crisis occurs most commonly in 
elderly patients with long-standing 
undiagnosed or undertreated hypothyroidism 
who experience an additional significant 
stress, including cold environment, infection, 
or certain medications. However, to avoid 
adrenal crisis, thyroxine replacement for 
hypothyroidism should only be initiated after 
concomitant glucocorticoid deficiency has 
either been excluded or treated. 

Rew arming with blankets or warm fluids are 
important. Intravenous glucose and normal 
saline should be carefully administered. IV 
hydrocortisone (lOOmg) should be 
administered. An intravenous loading dose of 
200-500 meg of levothyroxine is also 
recommended if myxedema is suspected, 
followed by a daily intravenous dose of 50-100 
meg. 



[Q: 2008] MRCPass - 2010 
September 

A 56 year old man presents with a 5 day 
history of left arm pain and weakness. On 
examination, he has increased tone in the left 
arm and weakness in the biceps and triceps 
distribution. There were absent biceps and 
supinator reflexes. The triceps reflex was 
brisk. Sensation was reduced around the 
lateral part of the forearm and elbow . 
Proprioception was normal. 

Whot is the likely diagnosis? 


1- Syringomyelia 

2- C5 and C6 disc herniation 

3- Transverse myelitis 

4- Neuralgic amyotrophy 

5- Guillain barre syndrome 


Answer & Comments 

Answer: 2- C5 and C6 disc herniation 

This clinical history fits C5 / 6 disc herniation 
or cord compression. 

The weakness and distribution of abnormal 
reflexes - biceps jerk (C5, C6), triceps jerk (C6, 
C7) and supinator jerk (C5, C6) suggest that 
the areas around C5 and C6 being affected. 
There are mixed upper motor neuron 
(increased tone) and lower motor neuron 
(absent reflex). The biceps and supinator 
reflexes (C5 and C6) may be absent, with a 
brisk triceps reflex (C7). This pattern is almost 
pathognomonic of cord compression because 
of cervical spondylosis at the C5-C6 
interspace. 



[Q: 2009] MRCPass-2010 
September 


A 60 year old man presents with dysphagia 
and dysphonia. Cranial nerve examination 
revealed left-sided partial ptosis with miosis, 
left sided facial weakness, tongue deviation to 
the left, an absent gag reflex, palatal palsy on 
the left, a weak voice, and a wasted left 
sternocleidomastoid muscle. 


Which one of the following areas is 
neurological damage most likely? 

1- Pons 


2- Lateral ventricles 


3- Jugular foramen 

4- Stylomastoid foramen 

5- Cerebellopontine 


Dr. Khalid Yusuf El-Zohry - Sohag Teaching Hospital (01118391123) 

Ref MRCPass OE OE 2012 PasTest 2009 PassMedicine 2009 PasTest Exam ReviseMRCP 

























El-zohry MRCP Questions Bank (Port 1) - 2013 


(For my personal use) 


Answer & Comments 

Answer: 3- Jugular foramen 

The diagnosis is likely to be either Villaret's 
syndrome or a jugular foramen syndrome. 

Villaret's is a rare syndrome characterized by 
an ipsilateral paralysis of cranial nerves 
numbers IX ,X, XI, XII, and it can also involve 
the cervical ganglia of the sympathetic trunk. 
The jugular foramen allows passage through 
of IX, X and XI cranial nerves. 


A 36 year old man has developed 
gynaecomastia after a new medication was 
commenced for 6 months. 

Which one of the following drugs is likely to 
hove coused the gynoecomostio? 

1- Amiodarone 

2- Buserelin 

3- Thioridazine 

4- Erythromycin 

5- Ibuprofen 



[Q: 2010] MRCPass - 2010 
September 


Answer & Comments 

Answer: 2- Buserelin 

Common drugs which can cause 
gynaecomastia are digoxin, oestrogens, 
spironolactone, cimetidine, verapamil and 
nifedipine. 

Buserelin is a new one to add to the MRCP list. 

Buserelin is a Gonadotropin-releasing 
hormone agonist (GnRH agonist). Like other 
GnRH agonists, buserelin may be used in the 
treatment of hormone-responsive cancers 
such as prostate cancer or breast cancer. 
Occasional gynaecomastia (increase in breast 
size) which is usually painless, atrophy of the 
testes, decrease in libido and potency may 
occur. 


[Q: 2011] MRCPass-2010 
September 

A 24 year old health worker had a needlestick 
injury on a finger after taking blood from a 
patient who was positive for HIV. Following 
the injury, the wound on the finger was 
thoroughly cleaned under clean running 
water. 

Whot is the percentage chonce that the 
worker might contract HIV? 

1- 1 in 3 

2- 1 in 30 

3- 1 in 300 

4- 1 in 3000 

5- 1 in 30000 



Answer & Comments 

Answer: 3-1 in 300 

The rate of occupational transmission from an 
HIV-positive source is believed to be 0.3% for 
a percutaneous exposure, which is much 
lower than Hepatitis B or Hepatitis C 
transmission. 

Taking this number into account, the risk 
works out to be 1 in 300. 



[Q: 2012] MRCPass-2010 
September 


A 50- year- old male presented with sudden 
worsening of breathlessness after a severe 
episode of pneumonia was thought to have 
developed acute respiratory distress 
syndrome (ARDS). 


Which of the following features would support 
o diagnosis of ARDS? 


1- High pulmonary capillary wedge pressure 


2- High protein pulmonary oedema 


3- Hypercapnea 

4- Increased lung compliance 

5- Normal chest x- ray 


Dr. Khalid Yusuf El-Zohry - Sohag Teaching Hospital (01118391123) 

Ref MRCPass OE OE 2012 PasTest 2009 PassMedicine 2009 PasTest Exam 




ReviseMRCP 

797 































El-zohry MRCP Questions Bank (Port 1) - 2013 


(For my personal use) 


Answer & Comments 

Answer: 2- High protein pulmonary oedema 

Answer: B): high protein pulmonary oedema 

ARDS is characterised by hypoxaemia, reduced 
lung compliance (stiff lungs) and pulmonary 
infiltrates on the chest x- ray. 


Standard teaching is that urate low ering 
drugs should not be introduced during an 
acute episode as it may worsen or prolong the 
episode; furthermore initiation of 
hypouricaemic treatment may precipitate 
acute gout. Colchicine and NSAIDs can be used 
in the acute situation. 


There is also no cardiogenic cause for 
pulmonary oedema (the Pulmonary Capillary 
Wedge pressure has to be normal or less than 
18mmHg to confirm this). 

Histologically, in ARDS there is damage to the 
capillary and endothelial cell linings, resulting 
in leakage of proteins into the interstitial and 
alveolar spaces at normal pulmonary capillary 
hydrostatic pressures - hence causing 
pulmonary oedema with high protein. In 
cardiac failure the protein levels of pulmonary 
oedema fluid are low . 


A 50 year old man with a past history of 
alcohol abuse presents with a painful red and 
warm ankle which was tophaceous. Gout was 
diagnosed and he was prescribed allopurinol. 

What is allopurinol's mechanism of action? 

1- Uricosuric drug 

2- Non steroidal anti inflammatory drug 

3- Microtubule inhibitor 



[Q: 2013] MRCPass - 2010 
September 


[Q: 2014] MRCPass-2010 
September 

How does the body handle excess amounts of 
cortisol? 

1- Binds to albumin 

2- Binds to adipose tissue 

3- Inactivated in the liver 

4- Excreted as free cortisol in the urine 

5- Broken down by enzymes 



Answer & Comments 

Answer: 4- Excreted as free cortisol in the 
urine 

Most serum cortisol (all but about 4%) is 
bound to proteins, including corticosteroid 
binding globulin (CBG) and serum albumin. 

However, when there is excess cortisol there 
will be insufficient binding capacity. This 
excess free cortisol is excreted in the urine, 
which is why it is a good diagnostic test for 
Cushing's syndrome. 


4- Xanthine oxidase inhibitor 

5- Dihydrofolate reducatase inhibitor 

Answer & Comments 

Answer: 4- Xanthine oxidase inhibitor 

Hypouricaemic agents essentially comprise of 
xanthine oxidase inhibitors (for example, 
allopurinol) and uricosuric agents (for 
example, probenecid, sulphinpyrazone, 
benzbromarone or azapropazone). 


[Q: 2015] MRCPass-2010 
September 

A 54 year old man presents with acute chest 
pains 1 hour ago. On arrival to the emergency 
department, he was found to have ST 
elevation of 2-3 mm from leads VI to V4 and 
deep T wave inversion in the inferior leads. He 
was transferred immediately for coronary 
angiography. 

What is the likely finding? 



Dr. Khalid Yusuf El-Zohry - Sohag Teaching Hospital (01118391123) 

Ref MRCPass OE OE 2012 PasTest 2009 PassMedicine 2009 PasTest Exam ReviseMRCP 



798 






























El-zohry MRCP Questions Bank (Port 1) - 2013 


(For my personal use) 


1- 70% stenosis of the left anterior descending 
artery 

2- 70% stenosis of the right coronary artery 

3- Total occlusion of the left anterior 
descending artery 

4- Total occlusion of the right coronary artery 

5- Total occlusion of the right coronary artery 
and the left anterior descending artery 


Answer & Comments 

Answer: 3- Total occlusion of the left anterior 
descending artery 

The distribution of the ECG changes in the 
chest leads Vl-6 suggests that it is an anterior 
myocardial infarction and hence total 
occlusion of the left anterior descending 
artery will be a likely finding. 

The T wave inversions in the inferior leads are 
due to reciprocal changes. 



[Q: 2016] MRCPass - 2010 
September 


A 75-year-old woman presented with a 18 
month history of progressive dysphagia to 
both solids and liquids, as well as a 3 kg weight 
loss. She typically complained of heartburn 
especially when lying down and after food. On 
examination, there were no palpable masses 
or lymphadenopathy found.A chest X ray 
showed dilatation of the oesophagus with a 
fluid level. 



Achalasia is characterized by difficulty sw 
allowing, regurgitation, and sometimes chest 
pain. 

the lower esophageal sphincter fails to relax, 
leading to dilatation of the oesophagus. On a 
barium sw allow , this leads to a bird's beak 
appearance. 



[Q: 2017] MRCPass-2010 
September 


A 63 year old man presents with an episode of 
amnesia for the second time in two months. 2 
days ago he had an episode of confusion, 
according to his wife. He was, However, able 
to have a normal conversation despite having 
been found wandering. After 2 hours, he 
abruptly returned to normal and could not 
remember what happened. 


What is the most likely diagnosis? 

1- Alcoholic encephalopathy 


2- Subarachnoid haemorrhage 

3- Complex partial seizure 

4- Transient ischaemic attack 


5- Transient global amnesia 


Answer & Comments 

Answer: 5- Transient global amnesia 

Transient global amnesia (TGA) is a temporary 
and isolated disorder of memory which may 
last several hours. 


What is the most likely diagnosis? 

1- Oesophageal carcinoma 

2- Pancreatic malignancy 

3- Achalasia 

4- Gastroduodenal ulcer 

5- Zollinger Ellison syndrome 


Answer & Comments 


Answer: 3- Achalasia 


Precipitating factors include sexual 
intercourse and heavy physical exercise, 
particularly sw imming in cold water. 



[Q: 2018] MRCPass-2010 
September 


A 53-year-old man with a history of anaemia 
and abdominal discomfort is diagnosed as 
having chronic myeloid leukaemia. 


What is the mechanism of action of imatinib? 


1- EGF receptor antagonist 


Dr. Khalid Yusuf El-Zohry - Sohag Teaching Hospital (01118391123) 

Ref MRCPass OE OE 2012 PasTest 2009 PassMedicine 2009 PasTest Exam ReviseMRCP 


































El-zohry MRCP Questions Bank (Port 1) - 2013 


(For my personal use) 


2- Anti VEGF receptor antagonist 

3- Anti-CD20 monoclonal antibody 

4- P53 inhibitor 

5- Tyrosine kinase inhibitor 


Answer & Comments 

Answer: 5- Tyrosine kinase inhibitor 

Imatinib (Gleevec) is an inhibitor of the 
tyrosine kinase associated with the BCR-ABL 
defect. 

It is used in treating chronic myelogenous 
leukemia (CML) and gastrointestinal stromal 
tumors. 


A 55 year old man has multiple endocrine 
neoplasia type II. There is a history of 
parathyroid neoplasia and 

phaeochromocytoma. 

Which one of the following thyroid conditions 
is associated? 

1- Anaplastic thyroid carcinoma 

2- Follicular thyroid carcinoma 

3- Papillary thyroid carcinoma 

4- Medullary thyroid carcinoma 

5- Insulinoma 



[Q: 2019] MRCPass - 2010 
September 


Answer & Comments 

Answer: 4- Medullary thyroid carcinoma 

MEN 1 associations are: pituitary tumour, 
parathyroid hyperplasia/tumour, pancreatic 
tumours (most commonly gastrinoma / 
insulinoma). 

MEN 2a is associated with medullary thyroid 
carcinoma (MTC), parathyroid tumours (10- 
20%) and pheochromocytoma (20-50%). 

MEN 2b is associated with presentation of 
medullary thyroid carcinoma, parathyroid 


tumours and pheochromocytoma + 
ganglioneuromatosis (pathognomonic). 



[Q: 2020] MRCPass-2010 
September 


A 75-year-old woman was admitted to the 
hospital because of fever, chills, headaches 
and myalgia. 


Her husband mentions that she has vomiting 
and is very confused. She also had new 
symptoms of urinary incontinence. She has 
recently returned from a seven-day holiday in 
Kenya. On physical examination, her blood 
pressure was 105/70 mmHg, pulse rate 120 
beats/ minute and body temperature 39°C. 
Her MMSE score is 17 / 30. Except for right 
upper abdominal pain, there were no other 
abnormalities, and no hepatosplenomegaly. 
Neurological examination was normal. 
Laboratory investigations showed raised liver 
enzymes, mild renal dysfunction. Urine 
dipstick showed 1+ proteinuria and nitrite -ve. 


What is the most likely infection? 

1- Urinary Tract Infection 


2- Dengue Fever 

3- Cerebral Malaria 


4- Leptospirosis 

5- Listeria 


Answer & Comments 

Answer: 3- Cerebral Malaria 

P. falciparum infection is common in Kenya. 
The acute history and associated confusion 
and renal involvement makes malaria most 
likely. P falciparum can cause cerebral malaria 
(confusion, seizures, headaches), pulmonary 

edema, rapidly developing anemia, and renal 
problems. Proteinuria is an early sign of renal 
involvement. 

Dengue is likely to be associated with a rash 
and thrombocytopenia. Urinary tract infection 


Dr. Khalid Yusuf El-Zohry - Sohag Teaching Hospital (01118391123) 

Ref MRCPass OE OE 2012 PasTest 2009 PassMedicine 2009 PasTest Exam ReviseMRCP 
































El-zohry MRCP Questions Bank (Port 1) - 2013 


(For my personal use) 


doesn't quite fit with the scenario or travel to 
Kenya and should not cause headaches. 



[Q: 2021] MRCPass - 2010 
September 


A 80 year old woman presents with a sw elling 
over the right side of her nose. The lesion was 
noticed 4 years ago and was little changed. On 
examination it was smooth, shiny and non 
pigmented. 


Whot is the probable diagnosis? 


1- Trophic ulcer 


2- Basal cell carcinoma 


3- Lupus vulgaris 

4- Seborrhoeic warts 


5- Squamous cell carcinoma 


Answer & Comments 

Answer: 2- Basal cell carcinoma 

Basal cell carcinomas can have many different 
appearances: a red patch or irritated area; a 
smooth, shiny and waxy looking bump; a 
white or yellow scar-like area; a smooth 
reddish growth; or an open sore that won't 
heal, bleeds or oozes. 


They are slow growing as described in this 
case, and rarely metastasise. 



Basal Cell Carcinoma 



[Q: 2022] MRCPass - 2010 
September 


A 30 year old lady with pulmonary 
hypertension is prescribed bosentan. 

What is its mechanism of action? 

1- Phosphodiesterase 5 inhibitor 

2- Calcium channel antagonist 

3- Prostaglandin analogue 

4- Vasopressin analogue 

5- Endothelin receptor antagonist 


Answer & Comments 

Answer: 5- Endothelin receptor antagonist 

Bosentan is a competitive antagonist of 
endothelin-1 at the endothelin-A (ET-A) and 
endothelin-B (ET-B) receptors. 

Under normal conditions, endothelin-1 
binding of ET-A or ET-B receptors causes 
pulmonary vasoconstriction. 

By blocking this interaction, bosentan 
decreases pulmonary vascular resistance. 


[Q: 2023] MRCPass-2010 
September 

A 36 year old lady with systemic lupus 
erythematosus has features of joint arthritis 
and photosensitive skin rashes. She is being 
followed up in the rheumatology clinic. During 
assessment , which one of these tests is the 
most useful for monitoring disease activity? 

1- Antinuclear antibody 

2- C3 and C4 

3- Anti Sm antibody 

4- ESR 

5- Anti CCP antibody 



Answer & Comments 


Answer: 2- C3 and C4 


Dr. Khalid Yusuf El-Zohry - Sohag Teaching Hospital (01118391123) 

Ref MRCPass OE OE 2012 PasTest 2009 PassMedicine 2009 PasTest Exam 




ReviseMRCP 

801 































El-zohry MRCP Questions Bank (Port 1) - 2013 


(For my personal use) 


Many factors may be used to monitor disease 
activity (according to the Systemic Lupus 
Erythematosus Disease Activity Index - see link 
below). 

Falling levels of C3 and C4 may herald a lupus 
flare in patients. Antinuclear antibodies (ANA) 
are positive in more than 95% of patients with 
lupus and therefore not useful for monitoring 
disease activity. Antibodies to dsDNA may 
fluctuate with disease activity and rising 
antibodies to dsDNA and can help in 
monitoring disease activity. Other routinely 
available autoantibodies have not been 
demonstrated to be helpful as markers of 
lupus activity. 

http://www.rheumatology.org/practice/clinic 

al/indexes/sledai.asp 



[Q: 2024] MRCPass-2010 
September 


Which one of the following techniques is used 
to determine the molecular structure of 
protein? 


1- Northern blotting 


the given sample of tissue homogenate or 
extract (e.g. in HIV testing, or to detect prions 
in Bovine Spongiform Encephalopathy). 

Northern blotting detects RNA and Southern 
blotting detects DNA. 



[Q: 2025] MRCPass-2010 
September 


A 26 year old man was brought into hospital 
having drunk 2 bottles of engine coolant. He 
was found to have metabolic acidosis and was 
treated with fomepizole. 


What is its mechanism of action on alcohol 
dehydrogenase? 


1- Competitive inhibitor 


2- Non competitive inhibitor 

3- Specific Agonist 


4- Anti-metabolite 


5- Neutralisation 


Answer & Comments 


Answer: 1- Competitive inhibitor 


2- Southern blotting 

3- X ray crystallography 

4- Western blotting 

5- Polymerase chain reaction 

Answer & Comments 

Answer: 1- Northern blotting 


Fomepizole or 4-methylpyrazole is indicated 
for use as an antidote in confirmed or 
suspected methanol or ethylene glycol 
poisoning. 

It is a competitive inhibitor of alcohol 
dehydrogenase, the enzyme that catalyzes the 
initial steps in the metabolism of ethylene 
glycol and methanol to their toxic metabolites. 


X-ray crystallography is a method of 
determining the arrangement of atoms within 
a crystal, in which a beam of X-rays strikes a 
crystal and diffracts into many specific 
directions. 

The method reveals the structure of many 
biological molecules, including vitamins, 
drugs, proteins and nucleic acids such as DNA. 

The Western blot is a technique involving 
electrophoresis to detect specific proteins in 



[Q: 2026] MRCPass-2010 
September 


A 65-year-old female is brought to A&E by her 
family, who are concerned about her 
increasing confusion over the past 2 days. 
There is a history of diarrhea in the preceding 
few days. On examination she is found to be 
pyrexial at 38oC. Breath sounds are clear and 
there is mild tenderness in the lower 


Dr. Khalid Yusuf El-Zohry - Sohag Teaching Hospital (01118391123) 

Ref MRCPoss OE OE 2012 PosTest 2009 PassMedicine 2009 PosTest Exam ReviseMRCP 





























El-zohry MRCP Questions Bank (Port 1) - 2013 


(For my personal use) 


abdomen. There was no focal neurological 
signs. 

Blood tests reveal : 

Hb 9.6 g/dl 
WCC 12 x 10 9 /l 
Platelets 65 x 10 9 /l 
sodium 138 mmol/l 
potassium 4.7 mmol/l 
Urea 18.1 mmol/l 
Creatinine 210 mmol/l 

A blood film shows schistocytes and 
thrombocytopenia. 

What is the most likely diagnosis? 

1- Wegener's granulomatosis 

2- Thrombotic thrombocytopenic purpura 

3- Goodpasture?fs disease 


erythematous, swollen leg. An ultrasound scan 
confirmed the diagnosis of deep vein 
thrombosis. Upon questioning he has a family 
history of thrombophilia. 

Which one of the following is likely to couse 
resistance to activation by protein C? 

1- Anti thrombin III deficiency 

2- Protein C deficiency 

3- Protein S deficiency 

4- Factor V leiden variant 

5- Antiphospholipid syndrome 

Answer & Comments 

Answer: 4- Factor V leiden variant 

Factor V Leiden variant is the most common 
hereditary hypercoagulability disorder 
amongst Europeans. 


4- Idiopathic thrombocytopenic purpura 

5- Rapidly progressive glomerulonephritis 


Answer & Comments 

Answer: 2- Thrombotic thrombocytopenic 
purpura 

Thrombotic thrombocytopenic purpura (TTP), 
involvement of the CNS predominates in TTP 
(neurological signs) whilst in HUS there is 
mainly renal involvement. 

Most cases of TTP arise from inhibition of the 
enzyme ADAMTS13, a metalloprotease 
responsible for cleaving large multimers of 
von Willebrand factor (vWF) into smaller 
units. 

Neurologic symptoms (confusion, headaches, 
stroke), low platelet count, renal impairment 
and microangiopathic haemolytic anaemia are 
present. 


A 30 year old man presented with an 



[Q: 2027] MRCPass-2010 
September 


Leiden variant of factor V cannot be 
inactivated by activated protein C. 

To diagnose Factor V leiden deficiency, Most 
laboratories screen 'at risk' patients with 
either a snake venom (e.g. dilute Russell's 
viper venom time) based test or an aPTT 
based test. This is done by running two tests 
simultaneously, one test is run in the presence 
of activated protein C (APC) and the other, in 
the absence. A ratio is determined based on 
the two tests and the results signify to the 
laboratory whether APC is working or not. 


A 55 year old patient has undergone gastric 
bypass surgery for treatment of obesity 6 
months ago. She is not compliant with 
prescribed vitamins and now presents with 
general lethargy. 

Which of the following is commonly deficient 
in Gastric bypass surgery? 

1- Iron 

2- Folate 



[Q: 2028] MRCPass-2010 
September 


Dr. Khalid Yusuf El-Zohry - Sohag Teaching Hospital (01118391123) 

Ref MRCPoss OE OE 2012 PosTest 2009 PassMedicine 2009 PosTest Exom 




ReviseMRCP 

803 




























El-zohry MRCP Questions Bank (Port 1) - 2013 


(For my personal use) 


3- Zinc 

4- Vitamin K 

5- Vitamin C 


Answer & Comments 

Answer: 1- Iron 

Gastric acidity helps absorption of iron and 
also intrinsic factor produced in the stomach 
helps absorption of B12. 

Following gastric bypass surgery, iron and B12 
deficiency are common (about 30% of 
patients). 

As B12 isn't on the answer options Iron is the 
best answer here. Folate deficiency following 
obesity surgery is rare. 


A 27 year old man presents with a 6 month 
history of low back pain. The pain radiates to 
his buttocks. There is associated stiffness 
which is worse in the morning and after 
periods of inactivity. 

Which of the following signs is most likely to 
be present? 

1- Positive straight leg test 

2- Positive femoral stretch test 

3- Positive Trendelenburg test 

4- Sacroiliac joint tenderness 

5- Global immobile vertebrae 



[Q: 2029] MRCPass - 2010 
September 


hip and shoulder joints, and a search for signs 
of enthesitis are critical in making an early 
diagnosis of AS. Important physical findings 
include tenderness over sacroiliac joints, 
vertebral spinal processes, iliac crest, anterior 
chest wall, calcaneus ischial tuberosities, 
greater trochanters, and tibial tubercles. 

With longer disease duration and disease 
progression, the spine becomes increasingly 
stiff, leading to loss of spinal mobility in all 
planes and restricted chest expansion. 



[Q: 2030] MRCPass-2010 
September 


A 52 year old man with heavy alcohol intake 
presents with haemetemesis. On examination, 
he had signs of pallor, jaundice and ascites. He 
was transfused 4 units of blood and when his 
blood tests were available, he had significant 
liver and renal impairment which confirmed 
hepatorenal syndrome. Terlipressin was 
commenced. 


What is the mechanism of action of 
terlipressin ? 

1- Increase eprythropoietin levels 

2- Vasodilation of coronary arteries 

3- Decrease hypoxia 

4- Splanchnic vasoconstriction 

5- Decrease renal perfusion 


Answer & Comments 


Answer: 4- Splanchnic vasoconstriction 


Answer & Comments 

Answer: 4- Sacroiliac joint tenderness 

The diagnosis is ankylosing spondylitis and the 
sign is related to early disease so the best 
answer is sacroiliac joint tenderness. 

Examination of the sacroiliac joints and the 
spine (including the neck), measurement of 
chest expansion and range of motion of the 


Terlipressin (triglycyl lysine vasopressin) is a 
synthetic analogue of vasopressin, which has 
been used in the treatment of acute variceal 
hemorrhage and in hepatorenal syndrome. 

In hepatorenal syndrome, it helps by reversing 
the extreme splanchnic arterial vasodilation 
that occurs in these patients, effectively 
increasing arterial blood volume. 


Dr. Khalid Yusuf El-Zohry - Sohag Teaching Hospital (01118391123) 

Ref MRCPass OE OE 2012 PasTest 2009 PassMedicine 2009 PasTest Exam ReviseMRCP 



804 





























El-zohry MRCP Questions Bank (Port 1) - 2013 


(For my personal use) 



[Q: 2031] MRCPass - 2010 
September 

A 51-year-old female was seen in the 
Emergency department with a 2 day history of 
headaches and fever. On examination, there 
was also evidence of meningism as shown by 
presence of a positive Kernig's sign. The 
patient had a temperature of 38.5°C. A lumbar 
puncture was performed. CSF showed 
evidence of gram positive bacilli. 

Whot is the diagnosis? 

1- Hemophilus influenzae 

2- Stapylococcus aureus 

3- Listeria monocytogenes 

4- Streptococcus pneumoniae 

5- Legionella pneumophila 


Answer & Comments 

Answer: 3- Listeria monocytogenes 

This patient has signs consistent with 
meningitis. 

Gram positive bacilli (rods) include Clostridia 
(clostridium tetani), listeria monocytogenes 
and bacilli (bacillus cereus). Legionella and 
haemophilus are gram negative bacilli. 



[Q: 2032] MRCPass - 2010 
September 

A 55-year-old man with a history of Crohn's 
disease comes for review. 5 years ago he had 
abdominal surgery and has been symptom 
free until now. Recently though, he has 
started to experience discomfort around the 
stoma site from the pervious surgery. On 
examination a deep erythematous, violaceous 
ulcer is noted with a ragged edge. The 
surrounding skin is erythematous and swollen. 

Whot is the most likely diagnosis? 

1- Erythema nodosum 


4- Necrotising fasciitis 

5- Crohn's disease ulceration 


Answer & Comments 

Answer: 3- Pyoderma gangrenosum 

Pyoderma gangrenosum is associated with 
inflammatory bow el disease and may be seen 
around the stoma site. 


Treatment is usually with 

immunosuppressants. 



Pyoderma Gangrenosum 



[Q: 2033] MRCPass-2010 
September 


A 55-year-old man presented because of a 
fever (up to 40°C) that had begun 12 days 
earlier and persisted despite treatment with 
oral antibiotics and anti-inflammatory drugs. 


The fever episodes occurred every 48 hours, 
with high peaks followed by abrupt resolution. 
He had just returned from India a month ago, 
and had not received any anti-malarial 
prophylaxis a. 

On presentation, he was pyrexial and pale, 
tachycardic and had hepatosplenomegaly. 
Microscopy of peripheral blood smears 
showed trophozoites with a parasitemia of 
1.5%. Some enlarged, infected erythrocytes, 
with morphology typical of Plasmodium vivax 
parasites, were observed. 

Whot is the best ontimoloriol treatment? 


2- Contact dermatitis 


1- Quinine 


3- Pyoderma gangrenosum 


2- Chloroquine 


Dr. Kholid Yusuf El-Zohry - Sohog Teaching Hospital (01118391123) 

Ref MRCPass OE OE 2012 PasTest 2009 PassMedicine 2009 PasTest Exam 




ReviseMRCP 

805 

























El-zohry MRCP Questions Bank (Port 1) - 2013 


(For my personal use) 


3- Mefloquine 

4- Pyrimethamine and sulphadiazine 

5- Artesunate 


Answer & Comments 

Answer: 2- Chloroquine 

Plasmodium vivax is found mainly in Asia, 
Latin America, and in some parts of Africa. 

Chloroquine is the treatment of choice for 
vivax malaria, except in Indonesia's Irian Jaya 
region and Papua New Guines, where 
chloroquine resistance is common (then 
artesunate is the treatment of choice). 
Mefloquine is an alternative. 


A 34 year old African man with known HIV 
infection presents with several episodes of 
seizures. His CD4 count was 130 cells/mm 3 
when measured 1 month ago. On 
examination, he had a temperature of 38 °C 
and was confused with an MTS score of 5/10. 
There were no focal neurological signs. He had 
an MRI scan which shows multiple ring 
enhancing lesions. CSF examination showed 
an elevated protein count, lymphocytosis and 
normal 

glucose levels. The CSF cryptococcal antigen 
was negative. A chest X ray was normal. 

Whot is the likely diagnosis? 

1- Progressive multifocal 

leukoencephalopathy 

2- Cerebral toxoplasmosis 

3- Cerebral lymphoma 

4- Tuberculosis 

5- Bacterial meningitis 



[Q: 2034] MRCPass-2010 
September 


The likely diagnosis is cerebral toxoplasmosis 
as there are multiple ring enhancing lesions. 

Lymphoma usually causes single enhancing 
lesions and PML is less frequently ring 
enhancing. Tuberculosis frequently causes 
significantly low glucose levels. 



Cerebral Toxoplasmosis 


[Q: 2035] MRCPass-2010 
September 

A 23-year-old woman comes for review . She 
was diagnosed with asthma two years ago and 
is currently using a salbutamol inhaler lOOmcg 
PRN combined with beclometasone 
dipropionate inhaler 200mcg twice a day. She 
continues to get frequent episodes of 
wheeziness and shortness of breath with low 
peak flow readings. She counted that she had 
4 exacerbations in the last month. She has a 
good inhaler technique and today her PEFR is 
90% of predicted. 

Whot is the most appropriate next step in 
management? 

1- Switch steroid to fluticasone propionate 

2- Increase beclometasone dipropionate to 
lOOOmcg bd 

3- Trial of monteleukast 



Answer & Comments 


Answer: 2- Cerebral toxoplasmosis 


4- Add salmeterol 

5- Add tiotropium 


Dr. Khalid Yusuf El-Zohry - Sohag Teaching Hospital (01118391123) 

Ref MRCPoss OE OE 2012 PosTest 2009 PassMedicine 2009 PosTest Exam ReviseMRCP 






























El-zohry MRCP Questions Bank (Port 1) - 2013 


(For my personal use) 


Answer & Comments 

Answer: 4- Add salmeterol 

The management of stable asthma is now well 
established with a step-w ise approach: 

Step 1:- Inhaled short-acting B2 agonist as 
required 

Step 2:- Add inhaled steroid at 200-800 
mcg/day* 

Step 3:- Add inhaled long-acting B2 agonist 
(LABA), such as salmeterol 

If control still inadequate, institute trial of 
other therapies, leukotriene receptor 
antagonist or slow release theophylline 



[Q: 2036] MRCPass - 2010 
September 


A 63-year-old man is admitted with 
palpitations to the Emergency Department. 


contraindicated in broad complex 
tachycardias. Adenosine can be given even 
though it blocks the AV node as it is very short 
acting. 


[Q: 2037] MRCPass - 2010 
September 

A 61 year old man presented with fevers, 
lethargy and 2 month history of malaise. He 
also mentioned altered bow el habit. On 
examination, he had a temperature of 39 C 
and a soft systolic murmur in the mitral area. 
He also had several splinter haemorrhages. 
Blood culture results within 24 hours grew 
streptococcus bovis. 

What investigation will help determine the 
underlying source of infection? 

1- Abdominal x ray 

2- Colonoscopy 

3- CT scan of the chest 



An ECG on admission shows a broad complex 
tachycardia at a rate of 150 bpm. His blood 
pressure is 124/82 mmHg and there is no 
evidence of heart failure. 

Which one of the following is least appropriate 
to give? 


4- Skin biopsy 

5- Transoesophageal echocardiogram 

Answer & Comments 

Answer: 2- Colonoscopy 


1- Procainamide 

2- Lidocaine 

3- Verapamil 

4- Synchronised DC shock 

5- Adenosine 


A correlation exists between colon cancer and 
Strep, bovis proliferation in the large intestine, 
hence predisposing to endocarditis. The 
patient needs a colonoscopy which may 
identify a colorectal malignancy predisposing 
to strep bovis bacteraemia and endocarditis. 


Answer & Comments 

Answer: 3- Verapamil 

Verapamil should not be given to a patient 
with a broad complex tachycardia as it may 
precipitate ventricular fibrillation in patients 
with ventricular tachycardia. 

This is because calcium channels are 
concentrated in the sino atrial and 
atrioventricular nodes. Similarly, digoxin is 



m 

[Q: 2038] MRCPass-2010 

4 


September 


A 66 year old woman complains of fevers, 
weight loss, joint pains and diarrhoea. A 
jejunal biopsy reveals flattened mucosa 
containing periodic acid-Schiff (PAS) positive 
macrophages. 

What is the most likely diagnosis? 

1- Coeliac disease 


Dr. Khalid Yusuf El-Zohry - Sohag Teaching Hospital (01118391123) 

Ref MRCPass OE OE 2012 PasTest 2009 PassMedicine 2009 PasTest Exam 




ReviseMRCP 

807 
































El-zohry MRCP Questions Bank (Port 1) - 2013 


(For my personal use) 


2- Campylobacter infection 

3- Tropical sprue 

4- Whipple's disease 

5- Giardiasis 

Answer & Comments 

Answer: 4- Whipple's disease 

Whipple's disease, may be confirmed by small 
bow el biopsy. 


The T score is usually used to make treatment 
decisions using standard deviation (SD). 

The SD measures the difference between the 
BMD and that of a healthy young adult (the 
reference value). Every -1 SD ("minus 1 
standard deviation") equals a 10 to 12% 
decrease in bone density. T score results are 
classified as follows: 

A T score between 0 and -1 standard deviation 
(SD) is considered to be normal. 


This will show large, foamy PAS positive 
macrophages in the lamina propria. Whipple's 
disease affects mainly men aged 30 to 60. It is 
caused by an infection with Trophyrema 
whippelii. Symptoms of Whipple's disease 
include diarrhea, inflamed and painful joints, 
fever, and skin darkening. Severe 
malabsorption results in weight loss along 
with fatigue and weakness caused by anemia. 

Antibiotics such as tetracycline, co- 
trimoxazole and penicillin can be used for 
treatment (6-12 months). 


A 60 year old lady presented with a fall and 
fractured writst. She was organised to have a 
DEXA scan. This showed a T score of -2.6 in 
the hip and a score of -2.1 in the femur. 

What does this mean? 

1- Normal values on the scan 

2- Osteopenia of the hip and osteoporosis of 
the femur 

3- Osteoporosis of the hip and osteopenia of 
the femur 

4- Osteopenia of both areas 

5- Osteoporosis of both areas 


[Q: 2039] MRCPass - 2010 
September 



Answer & Comments 

Answer: 3- Osteoporosis of the hip and 
osteopenia of the femur 


A T score between -1 and -2.5 SD is classified 
as osteopenia (low bone mass). 

A T score of -2.5 SD or less is classified as 
osteoporosis (very low bone mass). 


A 30-year-old female was referred for 
evaluation of lethargy. 2 months prior to 
referral, she had developed high grade 
continuous fever of acute onset for which she 
consulted a general practitioner. She was 
administered various courses of antibiotics 
and antiviral drugs, treatment but with no 
relief. Physical examination revealed pallor 
but no other physical signs. There was no 
clubbing, cyanosis or any significant 
lymphadenopathy. 

Her blood tests results are: Hb 6.5 g/dl, MCV 
79 fl, WCC 2.4 x 10 9 /l, platelets 85 x 10 9 /l. 

What is the likely couse? 

1- Acyclovir 

2- Chloramphenicol 

3- Trimethoprim 

4- Amoxicillin 

5- Erythromycin 


[Q: 2040] MRCPass-2010 
September 


Answer & Comments 

Answer: 2- Chloramphenicol 

From the investigations, this patient is likely to 
have aplastic anaemia. 


Dr. Kholid Yusuf El-Zohry - Sohog Teaching Hospital (01118391123) 

Ref MRCPass OE OE 2012 PasTest 2009 PassMedicine 2009 PasTest Exam ReviseMRCP 



808 




























El-zohry MRCP Questions Bank (Port 1) - 2013 


(For my personal use) 


Aplastic anemia is sometimes associated with 
exposure to substances such as benzene, 
radiation, or to the use of drugs, including 
chloramphenicol, sulphonamides, 

carbamazepine, phenytoin, quinine, and 
phenylbutazone (Some candidates answ ered 
trimethoprim, but it is the incorrect choice as 
it is the sulphonamide component that causes 
aplastic anaemia rather than trimethoprim in 
examples such as co-trimoxazole) 

^ [ Q: 2041 ] MRCPass - 2010 

0 September 

Which one of the following is the most 
common underlying mechanism leading to 
long QTsyndrome? 

1- Opening of calcium channels 

2- Opening of potassium channels 

3- Opening of sodium channels 

4- Blocking sodium channels 

5- Blocking potassium channels 

Answer & Comments 

Answer: 5- Blocking potassium channels 

Most candidates answ ered either blockage of 
sodium or potassium channels. 

Although it can be caused by blockage of 
sodium, potassium or calcium channels, 
around 90% of inherited long QT syndrome 
are due to defects in potassium channels 

^ [ Q: 2042 ] MRCPass - 2010 

I September 

A 41 year old man presented to his GP 
because of painful blisters on the backs of his 
hands in the summer. He also had a similar 
rash on the forehead. His face and forehead 
were covered with thickly wrinkled, 
hyperpigmented skin. The patient's urine was 
reddish orange. 

What is the likely diagnosis? 


1- Contact dermatitis 

2- Pityriasis rosea 

3- Epidermolysis bullosa 

4- Pemphigoid 

5- porphyria cutanea tarda 

Answer & Comments 

Answer: 5- porphyria cutanea tarda 

In porphyria cutanea tarda, the urine 
fluoresces pink to red. 

Porphyria cutanea tarda's onset is typically 
during the fourth or fifth decade of life. The 
disease tends to develop, recur, or worsen 
during the spring and summer, when exposure 
to sunlight is greatest (ie photosensitivity). 
Though blisters are the most common skin 
manifestations of PCT, other skin 
manifestations like hyperpigmentation and 
hypertrichosis (mainly on top of the cheeks) 
also occur. 

The most common photocutaneous 
manifestations of porphyria cutanea tarda are 
due to increased mechanical fragility after 
sunlight exposure; erosions and blisters form 
painful indolent sores that heal with milia 
(cysts), dyspigmentation, and scarring. The 
deficient enzyme in porphyria cutanea tarda is 
uroporphyrinogen decarboxylase. 



Porphyria Cutanea Tarda 


Dr. Khalid Yusuf El-Zohry - Sohag Teaching Hospital (01118391123) 

Ref MRCPass OE OE 2012 PasTest 2009 PassMedicine 2009 PasTest Exam ReviseMRCP 



809 
























El-zohry MRCP Questions Bank (Port 1) - 2013 


(For my personal use) 


[Q: 2043] MRCPass - 2010 
September 

A 47 year old male presented to hospital with 
a five day history of jaundice, fever, poor 
appetite and rigors. He has recently been 
treated for an infection and had been on a 
course of antibiotics. 

On examination, he was jaundiced. His blood 
pressure was 130/70, pulse rate was 80 beats 
per minute, heart sounds were normal. There 
was no evidence of organomegaly or 
tenderness on abdominal examination. 

Bloods showed: 

albumin 30 g/L 

alkaline phosphatase 1866 U/L 
gamma glutamyl transferase of 336 U/L 
alanine transaminase 138 U/L 
bilirubin 80 umol/L. 

Which drug was likely to have been given? 

1- Cefotaxime 

2- Paracetamol 

3- Amoxicillin 

4- Flucloxacillin 

5- Tramadol 



Answer & Comments 

Answer: 4- Flucloxacillin 

Augmentin (Co-amoxiclav), flucloxacillin 
cefotaxime and tetracyclines can cause 
cholestatic jaundice. 

Amoxicillin on its own can also cause 
cholestatic jaundice but less commonly so 
than flucloxacillin. 



[Q: 2044] MRCPass - 2010 
September 


A 35 year old woman has pleuritic chest pains. 
If she had pericarditis , on her ECG, the ST- 
segment is: 


1- Concave upwards 

2- Convex upwards 

3- Concave downwards 

4- Convex downwards 

5- Straight 


Answer & Comments 

Answer: 1- Concave upwards 

The shape of the ST elevation is typically 
described as concave upwards in acute 
pericarditis. 

In myocardial infarction, the ST elevation is 
described as convex upwards. 

http://www.aafp.org/afp/980215ap/marinell. 

html 


A 75 year old woman was referred with a two- 
month history of generalised weakness, fever, 
and weight loss. 

There was no other relevant past medical 
history. There was no palpable splenomegaly. 
The Hb was 9.5 g/dl, 

white cell count 5.3 x 10 9 /l, MCV 112 fl and 
platelet count was 89 x 10 9 /l. The reticulocyte 
count was <0.0001%. 

There was anisocytosis and poikilocytosis was 
seen in the blood film. 

What is the likely diagnosis? 

1- Aplastic anaemia 

2- Parvovirus infection 

3- Myelodysplasia 

4- Multiple myeloma 

5- Waldenstrom's macroglobulinaemia 


[Q: 2045] MRCPass-2010 
September 



Answer & Comments 


Answer: 3- Myelodysplasia 


Dr. Khalid Yusuf El-Zohry - Sohag Teaching Hospital (01118391123) 

Ref MRCPass OE OE 2012 PasTest 2009 PassMedicine 2009 PasTest Exam ReviseMRCP 


































El-zohry MRCP Questions Bank (Port 1) - 2013 


(For my personal use) 


The myelodysplastic syndromes (MDS, 
formerly known as "preleukemia") are a 
diverse collection of hematological medical 
conditions that involve ineffective production 
of the myeloid cell lines. 

The median age at diagnosis of a MDS is 
between 60 and 75 years. 

There is often pancytopenia as seen in this 
case. A high MCV is common and a blood film 
often shows poikilocytosis (which is in itself 
non specific). 

In this question, the answer is not aplastic 
anaemia as the MCV is high. 



[Q: 2046] MRCPass-2010 
September 


A 50 year old lady with chronic loin pains was 
organised to have a intravenous urogram. This 
revealed staghorn calculi. She had two 
episodes or urinary tract infection in the last 3 
years with Proteus identified in the urine 
culture. 


What is the predominant composition of these 
calculi? 


1- Uric acid 

2- Manganese 

3- Magnesium ammonium phosphate 

4- Cystine 

5- Calcium oxalate 


Answer & Comments 

Answer: 3- Magnesium ammonium phosphate 

Upper urinary tract stones that involve the 
renal pelvis and extend into at least 2 calyces 
are classified as staghorn calculi. 

Although all types of urinary stones can 
potentially form staghorn calculi, 

approximately 75% are composed of a 
struvite-carbonate-apatite matrix. Struvite 
(ammonium magnesium phosphate) is a 
phosphate mineral with formula: 


((NH4)MgP04*6H20). Struvite stones are 
caused by bacterial infection that hydrolyzes 
urea to ammonium and raises urine pH to 
neutral or alkaline values. Urea-splitting 
organisms include Proteus, Pseudomonas, 
Klebsiella, Staphylococcus, and Mycoplasma. 


A 65 year old man presents with shortness of 
breath. He has a history of COPD and has 
home oxygen and nebulisers. The ambulance 
crew reported that they had given him high 
flow oxygen and he is currently on 60% 
oxygen. On examination, he is short of breath 
but is able to have a conversation at present. 
He has a respiratory rate of 20 and wheezing 
throughout the lung. His arterial blood gases 
on examination are: 

pH 7.15 (7.36-7.44) 

p02 15 (11.0-13.5) kPa 

pC02 10 (3-6.0) kPa 

HC03 28 (22-28) 

BE-7 (-1 to 1) 

What should be done? 

1- Reduce to 24% oxygen 

2- Continue at 60% oxygen 

3- Intravenous theophylline 

4- Non invasive ventilation 

5- Intubation and ventilation 



[Q: 2047] MRCPass-2010 
September 


Answer & Comments 

Answer: 1- Reduce to 24% oxygen 

This COPD patient has had too much oxygen 
and is starting to C02 retain, causing a 
respiratory acidosis. 

The correct action is to reduce to 24% oxygen 
in the first instance and initiate therapy for 
COPD exacerbation, including steroids and 
nebulisers. If he does not improve or 
continues to worsen, then non invasive 


Dr. Khalid Yusuf El-Zohry - Sohag Teaching Hospital (01118391123) 

Ref MRCPoss OE OE 2012 PosTest 2009 PassMedicine 2009 PosTest Exom ReviseMRCP 




























El-zohry MRCP Questions Bank (Port 1) - 2013 


(For my personal use) 


ventilation (BIPAP) or intubation should be 
considered. 



[Q: 2048] MRCPass - 2010 
September 


A 26-year-old presents a three month history 
of amenorrhea. She had no other symptoms 
but takes medication for contraception. Her 
BMI is 23 and blood pressure is 120 / 80 
mmHg. Examination reveals slight 
galactorrhoea expression from both breasts 
but otherwise normal. Blood tests revealed a 
result for Prolactin 890 mll/L (< 450). 


An MRI Scan was organised and this showed a 
microprolactinoma with a 7 mm diameter. 

Which one of the following hormones is likely 
to be deficient? 


1- Oestrogen 

2- Growth hormone 



[Q: 2049] MRCPass - 2010 
September 


A 70 year old man presents with left sided 
facial drop and left hemiparesis. A CT scan 
showed that he had a right middle cerebral 
artery infarct and he was commenced on 
dipyridamole. 


What is dipyridamole's mechanism of action? 


1- Phosphodiesterase inhibitor 

2- Cyclooxygenase inhibitor 

3- Glycoprotein llb/llla inhibitor 


4- ADP antagonist 

5- Low molecular weight heparin 


Answer & Comments 

Answer: 1- Phosphodiesterase inhibitor 
The action of dipyridamole is on platelets. 


3- Antidiuretic hormone 

4- Thyroid hormone 

5- Cortisol 


Answer & Comments 

Answer: 1- Oestrogen 

Prolactinomas are the most common 
hormone-secreting pituitary tumors. 

Based on size, a prolactinoma can be classified 
as a microprolactinoma (<10 mm diameter) or 
macroprolactinoma (>10 mm diameter). 
Prolactin inhibits gonadotrophin secretion. 
The major effect of increased prolactin is a 
decrease in levels of sex hormones - estrogen 
in women and testosterone in men. 
Symptoms due to a Prolactinoma include 
amenorrhea, galactorrhea, loss of axillary and 
pubic hair, hypogonadism and 
gynecomastia(in males). 


Inhibition of phosphodiesterase by 
dipyridamole elevates platelet cAMP levels by 
inhibiting its breakdow n. The high cAMP 
levels lead to a reduction in intracellular Ca2+ 
and this inhibitsevents leading to platelet 
activation and granule excretion. 


I ft 

A 36 year old man has injured his leg whilst 
playing football. 

Which one of the following is consistent with a 
common peroneal nerve injury due to fibula 
neck injury? 

1- Loss of ankle jerk 

2- loss of sensation over the lateral part of the 
leg 

3- Loss of dorsiflexion 

4- Loss of sensation to the medial part of the 
thigh 

5- Loss of inversion 


[Q: 2050] MRCPass-2010 
September 


Dr. Khalid Yusuf El-Zohry - Sohag Teaching Hospital (01118391123) 

Ref MRCPass OE OE 2012 PasTest 2009 PassMedicine 2009 PasTest Exam ReviseMRCP 




























El-zohry MRCP Questions Bank (Port 1) - 2013 


(For my personal use) 


Answer & Comments 

Answer: 3- Loss of dorsiflexion 

There is foot drop, loss of dorsiflexion and 
eversion of the foot in common peroneal 
nerve injury. 

Inversion and plantar flexion are normal. 
There is usually sensory loss over the lower 
lateral part of the leg and dorsum of the foot. 



[Q: 2051] MRCPass - 2010 
September 


A 40 year old man is seeking advice. He has 
adult polycystic kidney disease and has a 
young son. He wants to know what is the 
chance that his son has the disease. 


In what percentage is the son likely to be 
affected? 

1 - 0 % 


2- 25% 

3- 50% 

4- 75% 

5- 100% 


Answer & Comments 


Answer: 3- 50% 


There is no organomegaly in the abdomen. A 
chest X ray shows hyperinflated lungs. Blood 
results show : Hb 18.5 g/dl, WCC 14 x 10 9 /l, 
platelets 350 x 10 9 /l. Haematocrit is 55% 
(<48%). 

What is the likely cause of this picture? 

1- Primary polycythemia 

2- Secondary polycythemia due to smoking 

3- Polycythemia rubra vera 

4- Methaemoglobinaemia 

5- Congenital heart disease 


Answer & Comments 

Answer: 2- Secondary polycythemia due to 
smoking 

This patient is a heavy smoker and probably 
has COPD. 

Polycythaemia can be due to smoking can 
cause plethora and a clinical picture as 
described above. 

There is no splenomegaly, which suggest that 
polycythaemia rubra vera is unlikely. The 
haematocrit (proportion of red cell mass to 
plasma volume) is elevated in both primary 
and secondary polycythaemias. 


The patient has autosomal dominant 
polycystic kidney disease. 

This means he is likely to have one gene 
affected and one unaffected. The likelihood of 
the son inheriting one of these genes is 50%. 

[Q: 2052] MRCPass - 2010 
September 

A 60 year old man has been complaining of 
breathlessness. He is a long standing smoker 
of 25 cigarettes a day. 

On examination, he has a plethoric facies. 
Cardiovascular examination is normal and the 
breath sounds are clear. 



[Q: 2053] MRCPass-2010 
September 

A 42 year old type I diabetic is referred for 
renal investigations. She has been suffering 
from Rheumatoid arthritis for the last 20 
years. She is currently on insulin injections, 
ibuprofen and penicillamine. She had 
proteinuria on a urine dipstick and 
quantification with 24 hour urine collection 
revealed that she had urinary protein > 4.5 
g/day. 

Ultrasound of the abdomen shows increased 
renal echogenicity. Investigations show : 

Hb 11.5 g/dl 

MCV 82 fl 



Dr. Khalid Yusuf El-Zohry - Sohag Teaching Hospital (01118391123) 

Ref MRCPass OE OE 2012 PasTest 2009 PassMedicine 2009 PasTest Exam 




ReviseMRCP 

813 




























El-zohry MRCP Questions Bank (Port 1) - 2013 


(For my personal use) 


WCC 12 x 1071 
platelets 225 x 10 9 /l 
sodium 135 mmol/I 
potassium 4.5 mmol/I 
Urea 14 mmol/l 
Creat 215 umol/l 

A renal biopsy shows eosinophilic deposits 
within the mesangium on light microscopy. 
The basement membrane and epithelial space 
is normal. 

Whot is the probable diagnosis? 

1- Minimal change nephropathy 

2- Membranous nephropathy 

3- Diabetic nephropathy 

4- NSAIDS induced nephropathy 

5- Amyloidosis 


Answer & Comments 

Answer: 5- Amyloidosis 

Diabetic nephropathy is unlikely to cause such 
heavy proteinuria, hence amyloidosis due to 
rheumatoid arthritis is the most likely 
diagnosis. 

Amyloidosis is a clinical disorder caused by 
extracellular and or intracellular deposition of 
insoluble abnormal amyloid fibrils that alter 
the normal function of tissues. AA amyloidosis 
can be caused by rheumatoid arthritis. Up to 
5% of patients with long-standing RA can 
develop systemic amyloidosis that usually 
presents as nephrotic syndrome. The biopsy 
shows eosinophilic deposits in the mesangium 
and capillary walls, which can be stained pink 
with Congo Red. 

Membranous nephropathy can present 
similarly, but is more commonly associated 
with autoimmune diseases (e.g. SLE), 
infections (e.g. hepatitis B) and malignancy 
(e.g. lymphoma).The drugs for rheumatoid 
arthirits ie NSAIDS, penicillamine gold can 
cause membranous nephropathy. The renal 


biopsy will show small subepithelial deposits 
in the glomeruli which can also lead to spikes 
or thickening of the basement membrane but 
the mesangium is typically normal. 

This case is also unlikely to be minimal change 
disease - (age of onset usually younger), the 
histology shows in minimal change shows a 
normal glomerulus and fusion of epithelial 
foot process will be seen only on electron 
microsocopy. 

? 

1 _ s 

A 23 year old man has been bitten by a dog on 
the thigh whilst walking in a park. 2 days later 
he develops erythema around the site and a 
purulent wound. 

What is best antibiotic? 

1- Trimethoprim 

2- Metronidazole 

3- Flucloxacillin 

4- Ciprofloxacin 

5- Augmentin 


[Q: 2054] MRCPass-2010 
September 


Answer & Comments 

Answer: 5- Augmentin 

Pasteurella (canis or multicoda) species are 
the most frequent isolates from both dog 
bites. 

Other common aerobes included streptococci, 
staphylococci, moraxella, and neisseria. 
Augmentin is the recommended antibiotic, 
along with tetanus injection for prophylaxis. If 
there is evidence of cellulites as in this case, 
then fluclox and benpen should be prescribed. 



[Q: 2055] MRCPass-2010 
September 


A 20-year-old lady with polycystic ovary 
syndrome was prescribed Metformin. 


How does Metformin acts in this situation? 


Dr. Khalid Yusuf El-Zohry - Sohag Teaching Hospital (01118391123) 

Ref MRCPoss OE OE 2012 PosTest 2009 PassMedicine 2009 PosTest Exam ReviseMRCP 


























El-zohry MRCP Questions Bank (Port 1) - 2013 


(For my personal use) 


1- Increasing oestradiol levels 

2- Increasing luteinising hormone levels 

3- Increasing gluconeogenesis 

4- Increasing insulin levels 

5- Increasing peripheral glucose uptake 


Answer & Comments 

Answer: 5- Increasing peripheral glucose 
uptake 

Metformin is being used increasingly in 
polycystic ovary syndrome (PCOS) and non¬ 
alcoholic steatohepatitis, two diseases that 
feature insulin resistance. 

Metformin improves insulin sensitivity by 
increasing peripheral glucose uptake and 
utilization. The reduction of hormonal 
imbalance and treatment of insulin resistance 
helps to restore the ovulatory cycles and 
fertility in PCOS. 


1 *I 

A 75 year old lady presents with visual 
problems and poor coordination. She 
mentions that over the last two days, she 
experienced a mild headache and visual 
blurring. She found it difficult to dress herself. 
There is a history of hypertension and type 2 
diabetes. On examination, she has left 
homonymous hemianopia. Tone and power 
were normal, but she had a left sided loss of 
sensation to the upper and lower limbs. 

What is the likely site of lesion? 

1- Posterior cerebral artery 

2- Posterior inferior cerebellar artery 

3- Inferior cerebral artery 

4- Middle cerebral artery 

5- Anterior cerebral artery 


[Q: 2056] MRCPass-2010 
September 


Answer & Comments 

Answer: 1- Posterior cerebral artery 

The better answer is posterior cerebral artery, 
as it the distribution affected by a stroke can 
cause homonymous hemianopia, hemisensory 
loss and also parietal lobe signs such as 
apraxia. 

In a middle cerebral artery lesion more 
significant signs such as motor weakness 
would be expected. 



[Q: 2057] MRCPass-2010 
September 


A 57-year-old man with a history of 
hypertension, diabetes, bipolar disorder and 
chronic obstructive pulmonary disease. 


He has bloods taken in clinic with the 
following results: 

sodium 119 mmol/l 

potassium 3.8 mmol/l 

Bicarbonate 26 mmol/l 

Urea 3.7 mmol/l 

Creatinine 92 pmol/l 

Plasma osmolality 270 mosmol/l (275-290) 
Urine osmolality 400 mosmol/l (350-1000) 


Which one of the following medications is 
most likely to be responsible? 

1- Metformin 

2- Lithium 

3- Carbamazepine 

4- Carbimazole 


5- Pioglitazone 


Answer & Comments 

Answer: 3- Carbamazepine 

This patient has hyponatraemia which is most 
likely to be due to drug induced SIADH causes. 

The reason the diagnosis is SIADH, is because 
the urine osmolality is inappropriately high 


Dr. Khalid Yusuf El-Zohry - Sohag Teaching Hospital (01118391123) 

Ref MRCPoss OE OE 2012 PosTest 2009 PassMedicine 2009 PosTest Exom 




ReviseMRCP 

815 

























El-zohry MRCP Questions Bank (Port 1) - 2013 


(For my personal use) 


due to excess ADH causing concentrated 
urine, despite hyponatraemia. An easy way to 
remember certain drugs causing SIADH are 
those starting with C: 

■ carbamazepine 

■ chlorpromazine 

■ chlorpropramide 
cyclophosphamide 



[Q: 2058] MRCPass - 2010 
September 


A 30 year old lady is 32 weeks pregnant. This is 
her second pregnancy, the first pregnancy was 
uneventful. She has pruritus and on 
examination, was mildly jaundiced. Liver 
function tests showed: 


ALT 75 (5-35) U/l 
AST 70(1-31) U/l 
ALP 350 (20-120) U/l 
Bilirubin 70 (1-22) |imol/l 
Albumin 38 (37-49) g/l 

Gamma glutamyl transpeptidase 120 (<50) 
U/L 

What is the likely diagnosis? 

1- Primary biliary cirrhosis 


extremities, palms, and soles. The itching may 
be severe, and it is often worse at night. 
Jaundice develops in 20 to 60 percent of 
women one to four weeks after the onset of 
itching. The features of obstructive jaundice, 
including pale stools and dark urine, may be 
present, but patients do not have 
constitutional symptoms. Intrahepatic 
cholestasis is associated with an increased risk 
of prematurity and stillbirth. Women with 
intrahepatic cholestasis should be treated at 
centers capable of caring for premature 
infants. Cholestyramine, given in divided 
doses totalling 10 to 12 g per day, may help 
relieve pruritus. 


A 25 year old man person attacks his girlfriend 
and shows no remorse. His friend says that of 
late he has become very aggressive. His 
girlfriend says that he hasn't slept for 2 days. 
On examination he is aggressive and is pacing 
around a lot. He says he cannot be punished 
as he has contacts with high level police 
officials. 

What is the diagnosis? 

1- Paranoid schizophrenia 

2- Mania 



[Q: 2059] MRCPass-2010 
September 


2- Gallstones 

3- Cholangiocarcinoma 

4- Intrahepatic cholestasis of pregnancy 

5- Viral hepatitis 

Answer & Comments 

Answer: 4- Intrahepatic cholestasis of 
pregnancy 

Intrahepatic cholestasis of pregnancy usually 
presents during the third trimester, at a mean 
of 30 weeks of gestation. 

The characteristic symptom is itching (pruritus 
gravidarum), which involves the trunk, 


3- Psychotic depression 

4- Conversion disorder 

5- Anxiety disorder 

Answer & Comments 

Answer: 2- Mania 

This man demonstrates increased levels of 
activity, aggressiveness and restlessness 
consistent with mania. 

He also has abnormal unrealistic beliefs but 
not psychotic features such as delusions or 
hallucinations. 


Dr. Khalid Yusuf El-Zohry - Sohag Teaching Hospital (01118391123) 

Ref MRCPass OE OE 2012 PasTest 2009 PassMedicine 2009 PasTest Exam ReviseMRCP 


























El-zohry MRCP Questions Bank (Port 1) - 2013 


(For my personal use) 



[Q: 2060] MRCPass - 2010 
September 


A 55-year-old man with a history of epilepsy is 
seen in the neurology clinic. Over the last few 
months, he has experienced a 'numbness' of 
his hands and feet. On examination he has 
reduced sensation in a gloveand-stocking 
distribution associated with a reduced ankle 
reflex. On examination, he has 
lymphadenopathy in the cervical and inguinal 
region and some bleeding around the gums. 


Which one of the following medications is he 
most likely to hove been taking? 


1- Carbamazepine 

2- Phenytoin 

3- Topiramate 

4- Sodium valproate 


5- Lamotrigine 


Answer & Comments 

Answer: 2- Phenytoin 

Phenytoin side effects include gingival 
hypertrophy, megaloblastic anaemia, 
lymphadenopathy and peripheral neuropathy. 



[Q: 2061] MRCPass-2010 
September 


A 65 year old man presents with a three 
month history of fever, malaise, anorexia, 
twenty-five pound weight loss, diffuse 
myalgias and night sweats, and more recently 
hemoptysis. 


He had a past medical history of hypertension, 
and described episodes of haematuria. 
Physical examination showed that he had 
diffuse lower extremity muscle tenderness, 
crepitations in the lungs and a rash on the 
trunks. Chest x-ray showed bilateral diffuse 
pulmonary infiltrates and also 2 areas of 
cavitation. 


Investigations showed: 
urine protein 1+ 


urine sediment - many red blood cell and 
granular casts 

sodium 135 mmol/I 

potassium 5.2 mmol/l 

urea 14 mmol/l 

creatinine 220 [irnol/l 

Whot investigation should be organised next? 

1- Urine culture 

2- Anti neutrophil cytoplasmic antibody 

3- Renal biopsy 

4- CT of kidney, ureter, bladder 

5- MR angiogram of the kidneys 


Answer & Comments 

Answer: 2- Anti neutrophil cytoplasmic 
antibody 

The diagnosis fits a pulmonary renal syndrome 
such as Wegener's, Churg Strauss or 
Goodpasture's syndrome. 

Apart from renal failure, there may be 
pulmonary haemorrhage, haemoptysis, 
infiltrates on the CXR as well as cavitation. 

The least invasive method initially to confirm a 
vasculitis is to request an ANCA. 



[Q: 2062] MRCPass-2010 
September 


A trial assessed a statin tablet compared to 
placebo for stroke prevention over 1 year. 
There were 10% of patients developing stroke 
in the group taking a tablet and 20% in the 
carotid endarterectomy group developing a 
stroke over the 1 year. 


What is the number needed to treat over 1 
year to prevent 1 death? 


1- 1 


2- 10 

3- 20 

4- 100 


Dr. Khalid Yusuf El-Zohry - Sohag Teaching Hospital (01118391123) 

Ref MRCPass OE OE 2012 PasTest 2009 PassMedicine 2009 PasTest Exam 




ReviseMRCP 

817 





























El-zohry MRCP Questions Bank (Port 1) - 2013 


(For my personal use) 


5- 1000 


Answer & Comments 

Answer: 2-10 

NNT is defined as number needed to treat to 
prevent 1 death. 

The way to work this out is 1 divided by 
absolute risk reduction (Experimental event 
rate - control event rate). Hence 1 / (ARR) is 1 
/ 10% which is 10. 


[Q: 2063] MRCPass - 2010 
September 

A 60 year old man with no previous symptoms 
had a routine ECG. The ECG shows left bundle 
branch block. The patient is currently taking 
Aspirin 75 mg od. He has a family history of 
myocardial infarction. He smokes 20 
cigarettes a day. The GP is concerned and 
refers the patient for further investigation. On 
examination, BP is 120/70 mHg and there are 
no findings during cardiovascular exam. 

Which of the following investigations is 
indicated? 



appropriate to conduct a myocardial perfusion 
scan. 



[Q: 2064] MRCPass - 2010 
September 


A 50 year old lady is being reviewed in the 
diabetes clinic. She has type 2 diabetes which 
is poorly controlled and a history of CCF with 
moderately impaired left ventricular function. 
Her BMI is 35. She is currently on gliclazide 


160mg bd, a long acting glargine insulin and 
short acting actrapid insulin with meals, 
frusemide, amlodipine and bendrofluazide. 
Her HbAlc value is 12 and she has frequently 
high BMS recorded. Her latest U&E results are 
urea 10 mmol/l, creatinine 190 pmol/l. 

What is the best medication to add to control 
her blood sugars? 


1- Rosiglitazone 


2- Metformin 


3- Exanetide 


4- Glimepiride 

5- Glucagon 


1- Exercise ECG 

2- CT of coronary arteries 

3- Myocardial perfusion imaging 

4- Coronary angiography 

5- Cardiac MRI 


Answer & Comments 

Answer: 3- Myocardial perfusion imaging 

For a patient with moderate likelihood of 
coronary artery disease (this patient is not 
symptomatic so is not in the high risk 
category), non invasive testing such as 
Exercise tolerance tests or myocardial 
perfusion tests should be performed. 

An exercise test will be difficult to interpret 
due to the ECG changes of LBBB. Thus its most 


Answer & Comments 

Answer: 3- Exanetide 

The newer incretin (GLP) analogues such as 
exanetide and liraglutide are now included in 
the guidelines by NICE CG 66 for patients who 
have not responded to insulin and 
Thiazolidinediones. 

In this patient, rosiglitazone is contraindicated 
due to heart failure and metformin is 
relatively contraindicated due to renal 
impairment. The BMI is high and as a second 
line agent after sulphonylurea and insulin, 
exanetide should be considered. 


A 45 year old man is investigated for 



[Q: 2065] MRCPass-2010 
September 


Dr. Khalid Yusuf El-Zohry - Sohag Teaching Hospital (01118391123) 

Ref MRCPass OE OE 2012 PasTest 2009 PassMedicine 2009 PasTest Exam ReviseMRCP 


































El-zohry MRCP Questions Bank (Port 1) - 2013 


(For my personal use) 


hypertension. Despite being on 3 different 
drugs, his blood pressure is consistently above 
180/100 mmHg. Blood results show : 

PH 7.5 

P02 -13 kPa 

PC02 - 4 kPa 

bicarbonate 32 (20-28) mmol/l 

sodium 138 mmol/l 

potassium 2.7 mmol/l 

urea 6 pmol/l 

creatinine 100 |imol/l 

Whot investigation should be requested? 

1- 24 hour urine catecholamines 

2- 24 hour urine HIAA 

3- Renin aldosterone ratio 

4- MRI of the adrenal glands 

5- Selective venous sampling 



[Q: 2066] MRCPass - 2010 
September 


A clinical trial studied patients' outcomes 
before and after they were given an 
antihypertensive drug. 


Which one of these factors would invalidate 
the use of a paired t test? 


1- Not being in normal distribution 


2- Small sample size 


3- An underpowered study 

4- Study bias 


5- Loss to follow up 


Answer & Comments 

Answer: 1- Not being in normal distribution 

The students t test can be used to compare 
two groups of data (paired T test being an 
example) with parametric data. 


Answer & Comments 

Answer: 3- Renin aldosterone ratio 

Hypokalaemic alkalosis with refractory 
hypertension as in this case suggests primary 
hyperaldosteronism (Conn's syndrome). 


Parametric means that the data will be of 
normal distribution and parallels the normal 
or bell curve). In this case if the data was not 
of normal distribution, t tests cannot be used 
and a non parametric test such as Kruskal 
Wallis or Wilcoxon test should be used 
instead. 


Secondary hyperaldosteronism would also be 
possible but tends to cause hypertension 
which is easier to control. As Conn's is caused 
by high aldosterone due to an aldosterone 
secreting tumour, the high aldosterone 
enhances exchange of sodium for potassium 
in the kidney so there is hypernatremia and 
hypokalemia. 

The sodium retention leads to plasma volume 
expansion and elevated blood pressure. The 
increased blood pressure will lead to 
increased glomerular filtration rate and cause 
a decrease in renin release from the granular 
cells of the juxtaglomerular apparatus in the 
kidney. Usually, renin levels are suppressed, 
leading to a very low renin-aldosterone ratio 
(<0.0005). 


[Q: 2067] MRCPass - 2010 
September 

A 41-year-old female patient, an ex-smoker 
with an 8-pack-year smoking history and 
severe pulmonary emphysema of early onset, 
received a diagnosis of 1-antitrypsin (AAT) 
deficiency. Regarding alpha 1 antitrypsin 
phenotypes, which of the following is most 
strongly associated with emphysema? 

1- ZZ 

2- MZ 

3- SZ 

4- MM 

5- SS 



Dr. Khalid Yusuf El-Zohry - Sohag Teaching Hospital (01118391123) 

Ref MRCPass OE OE 2012 PasTest 2009 PassMedicine 2009 PasTest Exam ReviseMRCP 



























El-zohry MRCP Questions Bank (Port 1) - 2013 


(For my personal use) 


Answer & Comments 

Answer: 1- ZZ 

The commonest phenotype is Protease 
Inhibitor (Pi)MM (90% of the population have 
this). 

These individuals produce normal amounts of 
alphal-antiprotease. The most common form 
of alpha 1 antitrypsin deficiency is associated 
with allele Z, or homozygous PiZ (ZZ). Serum 
levels of AAT in these patients are about 3 - 7 
umol/L (10-15% of normal serum levels). 
Emphysema develops in most (but not all) 
individuals with serum levels less than 9 
mmol/L. 

PiMM: 100% (normal) 

PiMS: 80% of normal serum level of A1AT 

PiSS: 60% of normal serum level of A1AT 

PiMZ: 60% of normal serum level of A1AT 

PiSZ: 40% of normal serum level of A1AT 

PiZZ: 10-15% (severe alpha 1-antitrypsin 
deficiency) 


3- Central retinal artery thrombosis 

4- Central retinal vein thrombosis 

5- Diabetic retinopathy 

Answer & Comments 

Answer: 2- Anterior ischaemic optic 

neuropathy 

The diagnosis is Temporal arteritis (Giant Cell 
Arteritis). 

Around 50% of patients with GCA eventually 
experience visual symptoms (eg, transient 
visual blurring, diplopia, eye pain, sudden 
vision loss). Transient repeated episodes of 
blurred vision are usually reversible, but 
sudden loss of vision is an ominous sign and is 
almost always permanent. The most common 
cause of vision loss is anterior ischemic optic 
neuropathy (AION). This results from ischemia 
of the optic nerve head, supplied mainly by 
the posterior ciliary arteries. Examination of 
the fundus may reveal optic disc edema, with 
or without splinter hemorrhages along the 
disc margin. 


[Q: 2068] MRCPass - 2010 
September 

A 70-year-old female presented with sudden 
onset loss of vision in the left eye. Her past 
medical history includes hypertension, 
diabetes, polymyalgia and hypothyroidism. 
She had a headache on the left. On 
examination, she had temporal artery 
tenderness on the left. The patient's 
erythrocyte sedimentation rate (ESR) was 120 
and her blood pressure is 150/90 mmHg. 
Fundoscopy revealed optic disc oedema with 
splinter haemorrhages around the optic disc 
on the left. 

What is the most likely couse of her visual 
loss ? 

1- Hypertensive retinopathy 

2- Anterior ischaemic optic neuropathy 




[Q: 2069] MRCPass - 2010 
September 


Whot is the likely physiological action of 
Gastrin? 


1- Luminal peptides stimulates its release in 
the gastric antrum 

2- Somatostatin stimulates its release in the 
gastric antrum 

3- Acts on G cells in antrum 


4- reduces pancreatic bicarb secretion 

5- Reduces gastric blood flow 


Answer & Comments 

Answer: 1- Luminal peptides stimulates its 
release in the gastric antrum 


Dr. Khalid Yusuf El-Zohry - Sohag Teaching Hospital (01118391123) 

Ref MRCPass OE OE 2012 PasTest 2009 PassMedicine 2009 PasTest Exam ReviseMRCP 






























El-zohry MRCP Questions Bank (Port 1) - 2013 


(For my personal use) 


In humans, gastrin is a hormone that 
stimulates secretion of gastric acid by parietal 
cells in the stomach. 

It is released by G cells in the stomach antrum 
and duodenum (it doesn't act on G cells). 

Gastrin release is stimulated by: 

stomach distension 
vagal stimulation 

the presence of partially digested 
proteins especially amino acids 

Gastrin release is inhibited by: 

Increased acidity 
Somatostatin 



[Q: 2070] MRCPass - 2010 
September 


A 28 year old female took 40 tablets of 
Paracetamol and was admitted to hospital. 
She is seen the following day and needs 
assessment of her medical condition. 


Which of the following is the best investigation 
to assess prognosis after 26 hours for a 
paracetamol overdose? 

1- Prothrombin time 

2- AST 


3- Paracetamol level 


4- Urea and creatinine 

5- Bilirubin 


Answer & Comments 

Answer: 1- Prothrombin time 

Although all of the tests may be abnormal, the 
INR / prothrombin time measurement is the 
most important in predicting prognosis (part 
of the child pugh criteria for liver failure) after 
a paracetamol overdose. 


[Q: 2071] MRCPass-2010 
September 

A 55-year-old man presented with a 10 -year 
history of an intermittent rash and pruritus 
associated with sweating from exertion. 

For 2 years he had noted pruritus and 
erythema mainly in the hands and feet, 
occurring on exposure to cool weather and 
resolving promptly on warming. He has a past 
medical history of Investigations showed a 
normal full blood count and mildly deranged 
liver function tests. Cryoglobulin levels were 
elevated. 

Which one of the following is likely to be 
positive? 

1- Rheumatoid factor 

2- Antinuclear antibody 

3- Anti neutrophil cytoplasmic antibody 

4- Anti centromere antibody 

5- Anti Ro and La antibody 



Answer & Comments 

Answer: 1- Rheumatoid factor 

Cryoglobulinaemia occurs when there are 
large amount of proteins that become 
insoluble at reduced temperatures. 

Type I is most commonly encountered in 
patients with multiple myeloma.Types II and 
III are strongly associated with infection by the 
hepatitis C virus. 

Types II and III have Rheumatoid Factor 
activity and bind to polyclonal 
immunoglobulins. 

Cryoglobulins may also be present in 
mycoplasma pneumonia, leukemias, primary 
macroglobulinemia, and some autoimmune 
diseases, such as systemic lupus 
erythematosus and rheumatoid arthritis. 


Dr. Khalid Yusuf El-Zohry - Sohag Teaching Hospital (01118391123) 

Ref MRCPass OE OE 2012 PasTest 2009 PassMedicine 2009 PasTest Exam 




ReviseMRCP 

821 























El-zohry MRCP Questions Bank (Port 1) - 2013 


(For my personal use) 



[Q: 2072] MRCPass - 2010 
September 

A 30-year-old man presents with sever 
abdominal pains. His blood tests and 
abdominal examination were normal. 

He had recently had an ultrasound of the 
abdomen which did not identify any 
pathology. It was noticed that he had multiple 
scars on his upper limb. He also kept asking 
for opiate drugs for pain relief. 

What is the likely diagnosis? 

1- Malingering 

2- Hypochondriasis 

3- Factitious disorder 


medically unexplained symptoms. The 
patient's life or work are frequently affected, 
although they also might be unconcerned 
about the nature of their symptoms (thus 
appearing calm). It is not a deliberate feigning 
of symptoms. 

Conversion disorder : (somatoform 

disorder)This is a condition where a patient 
displays neurological symptoms e.g. paralysis, 
even though no neurological explanation is 
found and it is determined that the symptoms 
are due to the patient's psychological 
response to stress. Malingering: the patient 
know ingly fabricates a medical illness for 
known gain. 


4- Somatisation disorder 

5- Conversion disorder 


Answer & Comments 

Answer: 1- Malingering 

The different common psychiatric diagnoses 
are below . 

In this case it seems that the patient is 
malingering in order to obtain narcotic drugs 
(evidence of intravenous injection marks on 
the limbs). 

Munchausen syndrome (factitious disorder): 
the patient seeks medical attention by the 
deliberate production or feigning of 
symptoms. The motivation for seeking 
attention is not known. 

Hypochondriasis: (somatoform disorder) the 
patient is convinced that they have a life- 
threatening illness, despite evidence to the 
contrary. The core feature of hypochondriasis 
is not preoccupation with symptoms 
themselves, but rather the fear or idea of 
having a serious disease. The fear or idea is 
based on the misinterpretation of bodily signs 
and sensations as evidence of disease. 

Somatisation disorder:(somatoform disorder) 
With this a patient presents with multiple, 



[Q: 2073] MRCPass-2010 
September 


A 50 year old lady with fever, fatigue and 
anaemia has been diagnosed as having acute 
myeloid leukaemia. On examination, she is 
pale and has mild splenomegaly. 


Which one of the following is the strongest 
determinant of prognosis? 


1- White cell count 


2- Number of blast cells with bone marrow 

3- Morphology of cells 

4- Size of spleen 

5- Karyotype 


Answer & Comments 

Answer: 5- Karyotype 

Acute myeloid leukemia (AML), also known as 
acute myelogenous leukemia, is the most 
common acute leukemia affecting adults. 

Anaemia, fever, weight loss, bleeding 
(thrombocytopenia) and also infections can be 
presenting symptoms. According to the widely 
used WHO criteria, the diagnosis of AML is 
established by demonstrating involvement of 
more than 20% of the blood and/or bone 


Dr. Khalid Yusuf El-Zohry - Sohag Teaching Hospital (01118391123) 

Ref MRCPass OE OE 2012 PasTest 2009 PassMedicine 2009 PasTest Exam ReviseMRCP 



























El-zohry MRCP Questions Bank (Part 1) - 2013 


(For my personal use) 


marrow by leukemic myeloblasts. There are 8 
subtypes. 

Patients with AML can have high, normal, or 
low WBC counts. 

The single most important prognostic factor in 
AML is cytogenetics, or the chromosomal 
structure of the leukemic cell. Cytogenetic 
karypotypes e.g. t(15;17), t(8;21) and 

inv/del/t(16) are associated with good 
prognosis. Because acute promyelocytic 
leukemia (APL) has the highest curability and 
requires a unique form of treatment, it is 
important to quickly establish the diagnosis, 
particularly the t(15;17) translocation. 

A number of other cytogenetic abnormalities 
are known to associate with a poor prognosis: 
5, -7, del(5q), Abnormal 3q, Complex 

cytogenetics. 

Age >60 years and elevated lactate 
dehydrogenase level are also associated with 
poorer outcomes in AML 



[Q: 2074] MRCPass-2010 
September 


A 37-year-old patient presents with a painless 
skin lesion on his left finger, first observed 5 
days before admission. On examination, a 
purulent looking pustule of 1 cm diameter was 
seen. The patient reported that he was 
working on a sheep farm. 


What is the diagnosis? 


1- Staphylococcal furuncle 

2- Cutaneous anthrax 


3- Chickenpox 

4- Leprosy 

5- Orf 


Answer &. Comments 

Answer: 5- Orf 

Orf is an exanthemous disease caused by a 
parapox virus and it also known as Ecthyma 


contagiosum. 

It is a zoonosis usually transmitted to humans 
from affected sheep or goat through direct 
contact or contaminated fomites. There are 
typically no systemic symptoms. 

The papule may persist for 7 to 10 weeks and 
spontaneously resolves. Whilst treatment is 
mainly conservative, some cases have 
improved with topical antiviral agents. 



ORF 


A 35 year old turkish woman presents with a 2 
month history of intermittent fevers 
associated with cutaneous pallor, weight loss, 
vomiting and anorexia. Physical examination 
revealed an enlarged liver (4.5 cm from the 
right costal margin) with smooth borders, a 
soft palpable spleen (6 cm from the left costal 
margin). The following results were obtained: 

Hb 7.5 g/dl 

MCV 75 fl 

WCC 3 x 10 9 /l 

platelets 54 x 10 9 /l 

sodium 135 mmol/l 

potassium 4.5 mmol/l 

urea 5 mmol/l 

creatinine 85 mmol/l 

What is the likely diagnosis? 

1- Malaria 

2- Non Hodgkin's lymphoma 


[Q: 2075] MRCPass-2010 
September 


Dr. Khalid Yusuf El-Zohry - Sohag Teaching Hospital (01118391123) 

Ref MRCPoss OE OE 2012 PosTest 2009 PassMedicine 2009 PosTest Exom 




ReviseMRCP 

823 
























El-zohry MRCP Questions Bank (Port 1) - 2013 


(For my personal use) 


3- Tuberculosis 

4- Visceral leishmaniasis 

5- Schistosomiasis 


Answer & Comments 

Answer: 4- Visceral leishmaniasis 

There are several forms of leishmaniasis, 
cutaneous and visceral are commonly quoted. 

In this case, fever and hepatosplenomegaly 
would be consistent with visceral 
leishmaniasis. Bone marrow infiltration may 
cause anemia, thrombocytopenia, and 
leukopenia. The gold standard for diagnosis is 
visualisation of the amastigotes in the bone 
marrow aspirate. Leishmania donovani can be 
spread by the sandfly. The traditional 
treatment is with pentavalent antimonials 
such as sodium stibogluconate (many resistant 
cases in India), but increasingly amphotericin 
B is the preferred treatment in depending on 
geography. 


Dr. Kholid Yusuf El-Zohry - Sohog Teaching Hospital (01118391123) 

Ref MRCPass OE OE 2012 PasTest 2009 PassMedicine 2009 PasTest Exam ReviseMRCP 


824 
















El-zohry MRCP Questions Bank (Port 1) - 2013 


(For my personal use) 


[ Q: 2076 ] MRCPass - 2011 January 

A 55-year-old man, known to have 
hepatitis C, was admitted with lethargy and 
diffuse petechial rash of the lower extremities. 
The rash had been present for the previous 2 
months, starting as a macular rash in both 
lower extremities and progressing to involve 
the trunk and upper extremities. The following 
investigation results were obtained: 

Urinalysis showed microscopic hematuria with 

3-6 coarse granular casts, and proteinuria +++. 

Hb 8.5 g/dl 

MCV 85 fl 

WCC 8 x 10 9 /l 

platelets 170 x 10 9 /l 

sodium 135 mmol/I 

potassium 4.5 mmol/I 

urea 24 mmol/l 

creatinine 355 |imol/l 

C3 = 52 mg/dL (79-152) 

C4 = 3.5 mg/dL (16-38) 

Rheumatoid factor was 150 lU/mL (normal: 0- 
20 ) 

Whot is the diagnosis? 

1- Haemolytic uraemic syndrome 

2- Crescentic glomerulonephritis 

3- Autoimmune haemolytic anaemia 

4- Henoch Schonlein purpura 

5- Cryoglobulinemia 



Answer & Comments 

Answer: 5- Cryoglobulinemia 

Cryoglobulins are serum proteins that 
precipitate in the cold. 

It is classified into three types (I, II and III), 
type II and III usually associated with hepatitis 
C virus (HCV) infection. In chronic HCV 
infection, cryoglobulins are found in 80% of 
patients of whom 10% develop 


cryoglobulinemic vasculitis. The clinical 
manifestations are as described in this 
scenario, with cutaneous vasculitis (skin 
lesions are usually palpable purpura of the 
lower extremities) and renal impairment 
(cryoglobulin deposits in the renal glomeruli 
leading to proteinuria) in the form of 
glomerulonephritis. 



[ Q: 2077 ] MRCPass - 2011 January 

A 31 year old man with a 15 year 
history of type 1 diabetes presents with a 4 
month history of pain and stiffness of the left 
shoulder. On examination, he has limited 
internal rotation and abduction of the 
shoulder. 


What is the likely diagnosis? 

1- Osteoarthritis 

2- Rheumatoid arthritis 


3- Brachial neuropathy 

4- Adhesive capsulitis 

5- Glenohumeral menisceal tear 


Answer & Comments 

Answer: 4- Adhesive capsulitis 

The features of reduced internal rotation and 
abduction are typical of frozen shoulder / 
adhesive capsulitis. 

In this condition, the connective tissue 
surrounding the glenohumeral joint of the 
shoulder, becomes inflamed and stiff leading 
to limited movement and pain. The movement 
of the shoulder is severely restricted and the 
pain is usually constant, worse at night. 
Treatment is with analgesia (NSAIDS) and 
physiotherapy. In more severe cases, steroid 
injections or surgery (capsular release) may be 
necessary. 

[ Q: 2078 ] MRCPass - 2011 January 

A 53 year old man presented to the 
emergency department with a collapse. He 



Dr. Khalid Yusuf El-Zohry - Sohag Teaching Hospital (01118391123) 

Ref MRCPass OE OE 2012 PasTest 2009 PassMedicine 2009 PasTest Exam ReviseMRCP 



825 




























El-zohry MRCP Questions Bank (Port 1) - 2013 


(For my personal use) 


was found at home unwell by his wife. He had 
a past medical history of TIA, a type II 
diabetes, and depression. His medications 
were gliclazide and aspirin. On examination he 
was afebrile and his BM was 9.0. His GCS was 
8 (E2, V3, M3). He had no neck stiffness. His 
pupils were pinpoint and there was a lack of 
horizontal gaze. He had bilateral upgoing 
plantar reflexes, increased tone throughout, 
and brisk reflexes. 

What is the diagnosis? 

1- Posterior inferior cerebellar stroke 

2- Demyelination 

3- Pontine haemorrhage 

4- Middle cerebral artery lesion 

5- Posterior communicating artery aneurysm 


Answer & Comments 

Answer: 3- Pontine haemorrhage 

Large pontine haemorrhage lesions can 
Quadriplegia, coma, small reactive pupils and 
bilateral paralysis of horizontal conjugate gaze 
are typical sequelae. 

There is often downward eye movements - 
ocular bobbing - imply preservation of rostral 
brainstem gaze centres. 

Smaller unilateral basal pontine lesions can 
present with contralateral hemiparesis, often 
with ataxia in the limbs affected. 


[ Q: 2079 ] MRCPass - 2011 January 

A 30 year old man has had Hepatitis 
B testing. He mentioned that he previously 
had Hepatitis B immunisation. 

Which one of the following is likely to be 
positive indicating effective immunisation? 

1- Hep B s Antigen 

2- Hep B c Antigen 

3- Hep B e Antigen 

4- Hep B s Antibody 



5- Hep B c Antibody 

Answer & Comments 

Answer: 4- Hep B s Antibody 

A chronic carrier has positive Hep B s Ag and 
positive HepB c Ab 

A patient with previous immunization has Hep 
B s Ag negative and negative Hep B c Ab and 
positive HepBs Antibody, (note that the core 
antibody is non specific and not elevated post 
immunisation) 

A patient with previous Hep B infection who is 
not a chronic carrier, has negative Hep B s Ag 
and positive HepBc 

Antibody and positive HepBs Antibody (both 
antibodies are positive) 


[ Q: 2080 ] MRCPass - 2011 January 

A 57 year old man was admitted to 
another hospital with frequent bloody 
diarrhea (10-15 bow el motions/day), 
abdominal pain, and fever. On admission, 
physical examination showed a temperature 
of 37.5°C; the abdomen was soft, but there 
was moderate tenderness in the lower 
abdomen. Laboratory results showed mild 
anemia (haemoglobin, llg/dl), albumin 32 
(37-49) g/l, elevated C-reactive protein 
57mg/dl. A colonoscopy was performed. 

This showed aggregates of yellow -whitish, 
well defined plaque in the ascending colon to 
the caecum. There was uniform congestion, 
oedema and irregular shallow ulcers, crypt 
abscesses and depletion of goblet cells. 

What is the diagnosis? 

1- Celiac disease 

2- Crohn's disease 

3- Ulcerative colitis 

4- Clostridium difficile colitis 

5- Enteroviral colitis 



Dr. Khalid Yusuf El-Zohry - Sohag Teaching Hospital (01118391123) 

Ref MRCPass OE OE 2012 PasTest 2009 PassMedicine 2009 PasTest Exam ReviseMRCP 


























El-zohry MRCP Questions Bank (Port 1) - 2013 


(For my personal use) 


Answer & Comments 

Answer: 3- Ulcerative colitis 

The history of bloody diarrhea is typical of 
ulcerative colitis. 

The biopsy specimen showing uniform areas 
of inflammation rather than skip lesions, and 
also crypt abscesses (these are crypts 
infiltrated with leukocytes) are typical. 
Typically, ulcerative colitis involves only the 
mucosa, with the formation of crypt abscesses 
and a coexisting depletion of goblet cell 
mucin. 



[ Q: 2081 ] MRCPass - 2011 January 

A 36-year-old woman was evaluated 
in outpatients following a complaint of visual 
dificulties and an ophthalmologic evaluation 
showed bitemporal hemianopia. An urgent 
MRI scan was requested. 


Where is the likely lesion? 


1- Optic nerve 

2- Optic chiasm 

3- Optic radiation 



[ Q: 2082 ] MRCPass - 2011 January 

A 25 year old man has presented 
with a primary pneumothorax for the first 
time. He smokes 5 cigarettes a day. 


The pneumothorax was aspirated and has 
resolved when the CXR was repeated. He is 
seeking advice about further management. 


Which one of the following should he avoid? 


1- No restrictions 


2- Travelling by plane for 3 months 

3- Travelling by plane indefinitely 

4- Diving for 3 months 

5- Diving indefinitely 


Answer & Comments 

Answer: 5- Diving indefinitely 

These guidance is within the British Thoracic 
society guidelines for pneumothorax. 

Commercial airlines and BTS guidelines 
currently advise that there should be a 6 week 
interval between having a pneumothorax and 
travelling by air. 


4- Occipital lobe 

5- Oculomotor nerve 


Answer & Comments 

Answer: 2- Optic chiasm 

Bitemporal hemianopia is typically caused by 
an optic chiasm lesion. 

Since the adjacent structure is the pituitary 
gland, some common tumors causing 
compression are pituitary adenomas and 
craniopharyngiomas. Another relatively 
common neoplastic etiology is meningiomas. 
In addition, an anterior communicating artery 
aneurysm which arises superior to the chiasm 
can enlarge, and compress it from above. 


After a pneumothorax, diving should be 
discouraged permanently unless a very secure 
definitive prevention strategy such as surgical 
pleurectomy has been performed. 


[ Q: 2083 ] MRCPass - 2011 January 

A man presents to the hospital with 
severe vomiting and dirrhoea. On 
examination, he was dehydrated, and his face 
was flushed. He gives a history of drinking 
wine and eating tuna as a main course in a fish 
restaurant. 

What is the likely cause? 

1- Clostridium difficile toxin 

2- Cholera toxin 

3- Scrombrotoxin 

4- Alfatoxin 



Dr. Khalid Yusuf El-Zohry - Sohag Teaching Hospital (01118391123) 

Ref MRCPass OE OE 2012 PasTest 2009 PassMedicine 2009 PasTest Exam 




ReviseMRCP 

827 






























El-zohry MRCP Questions Bank (Port 1) - 2013 


(For my personal use) 


5- Celiac disease 


Answer & Comments 


Answer & Comments 

Answer: 3- Scrombrotoxin 

Scombroid food poisoning is a foodborne 
illness that results from eating spoiled 
(decayed) fish. 


Answer: 2- Common bile duct 

This patient demonstrates obstructive 
jaundice and pancreatitis, and hence is likely 
to have a form of obstruction lower down in 
the biliary tree which is likely to be due to 
gallstones. 


Scrombotoxin is produced by oily fish such as 
mackerel, tuna, mahi-mahi, bonito, sardines, 
anchovies. Unlike many types of food 
poisoning, this form is not brought about by 
ingestion of a bacterium or virus. Histidine 
exists naturally in many types of fish, and at 
temperatures above 16°C (60°F) on air contact 
it is converted to the biogenic amine 
histamine via the enzymes in the fish. The 
effects of histamine lead to nausea, diarrhoea 
and flushing. Symptoms usually last for 4 -6 



Esophagus 


Liver left lobe 

Liver nght Lobe 





R>gtt8,u#l 

Hepatic duct 


Cystic 

duct 


Gall 

1 

bladder 




Falciform 

ligament 


Stomach 


— Common 
Hepatic duct 


Pancreatic 

duct 




... 

Pancreas 


Common bile 
duct 


hours. 



Biliary Anatomy 

I A 

[ Q: 2084 ] MRCPass - 2011 January 


[ Q: 2085 ] MRCPass - 2011 January 

M 

0 

A 55-year-old female patient was 

w 

0 

A 45 year old man presents with 


referred to the emergency department with a 
one-w eek history of epigastric pain, poor food 
intake and pains radiating to her back, nausea 
and vomiting. She also complained of dark 
urine and jaundice, and her faeces become 
pale. The following blood results were 
obtained: 


ALT: 127 U/L ( 5-40 U/L) 


painful groin area and fevers. 

He had no known past medical history but was 
a frequent traveller. A month ago, he had 
returned from India and following a month's 
trip. He describes having a painless penile 
ulcer 2 weeks ago. On examination, he had 
palpable painful inguinal lymph nodes. The 
blood results are: 


alkaline phosphatase: 1300 U/l ( 98-290 U/L) 

Bilirubin 85 (1-22) |imol/l 

serum amylase: 2100 U/L (<220U/I) 

Where is the likely site of obstruction leading 
to jaundice? 

1- Cystic duct Left hepatic duct 

2- Common bile duct 

3- Left hepatic duct 

4- Right hepatic duct 

5- Pancreatic duct 


Hb 11.5 g/dl 
MCV 82 fl 
WCC 8 x 10 9 /l 
platelets 330 x 10 9 /l 
Blood cultures - no growth 
Filarial serology- negative 
Chlamydia serology- positive 
Monospot test - negative 
What is the likely diagnosis? 
1- HIV infection 


Dr. Khalid Yusuf El-Zohry - Sohag Teaching Hospital (01118391123) 

Ref MRCPoss OE OE 2012 PosTest 2009 PassMedicine 2009 PosTest Exam ReviseMRCP 



828 








































El-zohry MRCP Questions Bank (Port 1) - 2013 


(For my personal use) 


2- Primary syphillis 


potassium 5.8 mmol/l 


3- Lymphogranuloma venereum 


urea 6 mmol/l 


4- Chancroid 

5- Mycobacterium leprae infection 


Answer & Comments 

Answer: 3- Lymphogranuloma venereum 

Lymphogranuloma venereum (LGV) is 
primarily an infection of the lymphactic 
system. 

Chlamydia trachomatis is the bacteria 
responsible for LGV. It gains entrance through 
breaks in the skin, or the mucous membranes. 
LGV may begin as a self-limited painless 
genital ulcer that occurs at the contact site 3- 
12 days after infection. This is followed by the 
secondary stage most often occurs 10-30 days 
later. The infection spreads to the lymph 
nodes through lymphatic pathw ays causing 
lymphangitis and painful lymphandenopathy. 
Diagnosis is made with Chlamydia serology or 
PCR of aspirate / pus samples. Treatment is 
with tetracyclines or erythromycin. 

The chancroid (a sexually transmitted disease) 
can also present with painful inguinal 
lymphadenopathy and genital ulcers. It does 
not fit the scenario as Haemophilus ducreyi is 
the infective organism. Treatment is with a 
macrolide (erythromycin) or ceftriaxone. 


[ Q: 2086 ] MRCPass - 2011 January 

A 35 year old woman was admitted 
to hospital with profuse diarrhoea over 
several weeks. She had symptoms which 
started 6 months ago. The diarrhoea was 
watery and did not have mucus. The 
symptoms also included nausea, poor appetite 
and lethargy. On examination, she had a blood 
pressure of 95 /60 mmHg. She looked 
relatively well. There was no positive 
examination findings. Blood test results were: 

sodium 135 mmol/l 



creatinine 90 |imol/l 
ALT 30 (5-35) U/l 
AST 25 (1-31) U/l 
ALP (20-120) U/l 
Bilirubin 13 (1-22) pmol/l 
Albumin 32 (37-49) g/l 
Glucose 5.5 mmol/l 
What is the likely diagnosis? 

1- Celiac disease 

2- Adrenal insufficiency 

3- Crohn's disease 

4- Norovirus infection 

5- Anorexia nervosa 


Answer & Comments 

Answer: 2- Adrenal insufficiency 

This patient is likely to be Addisonian due to 
features of hypotension, lethargy, diarrhoea 
and mild hyperkalaemia (due to low 
aldosterone levels). 

[ Q: 2087 ] MRCPass - 2011 January 

A 75 year old woman was referred 
with a two-month history of generalised 
weakness, fever, and weight loss. 

There was no other relevant past medical 
history. 

On examination she had multiple petechiae 
present on observation. There was no 
palpable splenomegaly. 

The blood results showed: 

Hb was 9.5 g/dl, white cell count 5.3 x 10 9 /l, 
MCV 112 fl and platelet count was 89 x 10 9 /l. 
The reticulocyte count was <0.0001%. There 
was anisocytosis and poikilocytosis was seen 
in the blood film. 

What is the likely diagnosis? 



Dr. Khalid Yusuf El-Zohry - Sohag Teaching Hospital (01118391123) 

Ref MRCPass OE OE 2012 PasTest 2009 PassMedicine 2009 PasTest Exam ReviseMRCP 



829 



























El-zohry MRCP Questions Bank (Port 1) - 2013 


(For my personal use) 


1- Aplastic anaemia 

2- Parvovirus infection 

3- Myelodysplasia 

4- Multiple myeloma 

5- Waldenstrom's macroglobulinaemia 


Answer & Comments 

Answer: 3- Myelodysplasia 

The myelodysplastic syndromes (MDS, 
formerly known as "preleukemia") are a 
diverse collection of hematological medical 
conditions that involve ineffective production 
of the myeloid cell lines. 

The median age at diagnosis of a MDS is 
between 60 and 75 years. 

There is often pancytopenia as seen in this 
case. A high MCV is common and a blood film 
often shows poikilocytosis (which is in itself 
non specific). 

In this question, although the picture of the 
blood tests is of a pancytopenia, the answer is 
unlikely to be aplastic anaemia as the MCV is 
high. 


7 


[ Q: 2088 ] MRCPass - 2011 January 

A 40 year old man presented with 
epigastric pains and indigestion. He had an 
upper Gl endoscopy which showed 
oesophagitis and gastritis. A biopsy was taken 
which showed positive H pylori result and he 
was commenced on 1 week of triple therapy 
for eradication. 


How should the eradication be monitored 
now? 


1- Repeat OGD for surveillance 

2- Repeat OGD with biopsy 

3- 14 C Breath test 

4- H pylori serology 

5- No further monitoring 


Answer & Comments 

Answer: 3-14 C Breath test 

With the breath test, patients sw allow urea 
labelled with an uncommon isotope, 
radioactive carbon-14. 

In the subsequent 10-30 minutes, the 
detection of isotope-labelled carbon dioxide in 
exhaled breath indicates that the urea was 
split; this indicates that urease (H. pylori uses 
to metabolize urea) is present in the stomach. 
This is a useful non invasive test for 
monitoring whether eradication is successful. 

[ Q: 2089 ] MRCPass - 2011 January 

A 34-year-old man presents with a 
dry cough and shortness of breath. He has 
typically experienced these dry cough 
episodes twice a week. He has worked as a 
spray paint worker in a car factory for 5 years. 
He does not smoke nor drink any alcohol. On 
examination, the patient was febrile (39°C), 
and a clinical lung examination showed diffuse 
crackles. A chest radiograph showed 
interstitial infiltrates and bilateral hilar 
adenopathy. 

Pulmonary function tests were performed. 

The results were (predicted percentages): 

Forced vital capacity (FVC) 1.8 L (60%) 

Forced expiratory volume (FEV1)1.7 L (70%) 

FEV1/FVC ratio - 90% 

Forced expiratory time (FET) 9.2 sec 

Residual volume (RV) 0.8 L (40%) 

Total lung capacity (TLC) 2.7 L (50%) 

Diffusing capacity (DLCO) Hg 8.7 mL/min/mm 
Hg (38%) 

What is the likely diagnosis? 

1- Asthma 

2- Chronic obstructive pulmonary disease 

3- Bronchiolitis obliterans organising 
pneumonia 



Dr. Khalid Yusuf El-Zohry - Sohag Teaching Hospital (01118391123) 

Ref MRCPass OE OE 2012 PasTest 2009 PassMedicine 2009 PasTest Exam ReviseMRCP 



830 

























El-zohry MRCP Questions Bank (Part 1) - 2013 


(For my personal use) 


4- Sarcoidosis 

5- Hypersensitivity pneumonitis 


Answer & Comments 

Answer: 5- Hypersensitivity pneumonitis 

Extrinsic allergic alveolitis (hypersensitivity 
pneumonitis) results from hypersensitivity 
immune reactions to the repeated inhalation 
or ingestion of various antigens derived from 
fungal, bacterial, animal protein, or reactive 
chemical sources such as dust or paint. 


are often used in the staging of non-Hodgkin's 
lymphoma and Hodgkin's lymphoma. 



[ Q: 2091 ] MRCPass - 2011 January 


A 20 year old lady came to you with 
a history of intermittent diarrhea. 


A biopsy taken during colonoscopy revealed 
melanosis coli. 


What is the most likely cause for the biopsy 
finding? 

1- Crohn's disease Celiac disease 


They can also present subacutely with 
recurrent pneumonia or chronically with 
exertional dyspnea, productive cough, and 
weight loss. CXR and High resolution CT often 
show upper zone pulmonary fibrosis. Lung 
function tests often show a restrictive picture, 
with decreased lung capacity and increased 
FEV1/FVC ratio as demonstrated in this case. 
The treatment is to reduce exposure to the 
allergenic component, in this case paint 
(isocyanates), and exacerbations can be 
improved with a course of steroids. 


2- Celiac disease 

3- Ferrous sulphate use 

4- Laxative overuse 

5- Ulcerative colitis 

Answer & Comments 

Answer: 4- Laxative overuse 

Melanosis coli is a disorder of pigmentation of 
the wall of the colon, identified at the time of 
colonoscopy. 



[ Q: 2090 ] MRCPass - 2011 January 

A 74-year-old man with a history of 
non hodgkin's lymphoma of the nodular 
sclerosing type has previously had 
radiotherapy. He is complaining of lethargy. 

Which one of the following symptoms 
indicates poor prognosis? 

1- Weight gain 

2- Palpitations 

3- Anxiety 

4- Night sweats 

5- Lethargy 

Answer & Comments 

Answer: 4- Night sweats 

The B symptoms - fevers, night sweats and 
weight loss - indicate a poorer prognosis and 


It is benign, and has no significant correlation 
with disease. The name is considered a 
misnomer: the brow n pigment seen in the 
colon is actually lipofuscin, not melanin as the 
name implies. 

The most common cause of melanosis coli is 
the surreptitious use of laxatives, and 
commonly anthraquinone containing laxatives 
such as Senna and other plant glycosides. 
Endoscopically, the mucosa shows a brownish 
discoloration. 


[ Q: 2092 ] MRCPass - 2011 January 

A 47-year-old man presents with 
palpitations which he has had for 3 months. 
He feels his 'heart race' regularly. 

On examination his pulse is 110 / min, 
irregularly irregular and respiratory 
examination is unremarkable 
confirms atrial fibrillation. 



Dr. Khalid Yusuf El-Zohry - Sohag Teaching Hospital (01118391123) 

Ref MRCPass OE OE 2012 PasTest 2009 PassMedicine 2009 PasTest Exam ReviseMRCP 


An 

ECG 




831 































El-zohry MRCP Questions Bank (Port 1) - 2013 


(For my personal use) 


What is the most appropriate next step in 
management? 

1- Digoxin 

2- Electrical cardioversion 

3- Amiodarone 

4- Metoprolol 

5- Flecainide 


Answer & Comments 

Answer: 4- Metoprolol 

In uncomplicated atrial fibrillation a beta 
blocker should be used first line for rate 
control and maintenance of sinus rhythm if 
the patient has paroxysmal AF. 

Flecainide would be a good second line option 
if AF wasn't adequately controlled. 

http://www.nice.org.uk/nicemedia/pdf/CG03 

6niceguideline.pdf 


[ Q: 2093 ] MRCPass - 2011 January 

A 45 year old lady presents with 
lethargy and nausea. Her blood results are 
listed below : 

Hb 12.5 g/dl 

MCV 75 fl 

WCC 8 x 10 9 /l 

platelets 215 x 10 9 /l 

sodium 135 mmol/I 

potassium 4.5 mmol/I 

urea 8 mmol/I 

creatinine 90 pmol/l 

calcium 1.7 (2.25-2.7) mmol/I 

phosphate 0.9 (0.8-8) pmol/l 

magnesium 0.8(0.67-0.96) mmol/I. 

Which one of the following findings on her ECG 
is probable? 

1- Tall T wave 

2- ST depression 



3- Prolonged QT 

4- Short PR interval 

5- Reciprocal changes 


Answer & Comments 

Answer: 3- Prolonged QT 

Hypocalcaemia is the main biochemical 
finding on the blood results in this patient. 

The electrolyte causes of prolonged QT are 
hypokalaemia, hypocalcaemia and 
hypomagnesaemia. 


[ Q: 2094 ] MRCPass - 2011 January 

A 35 year old lady presents with 
palpitations. She has no significant past 
medical history and does not take regular 
medications. Her BP was 120 / 70 mmHg and 
she had a heart rate of 180 bpm. An ECG 
confirms a narrow complex tachycardia. She 
was given 6 mg of adenosine with no clinical 
effect to the heart rate. 

What should be administered next? 

1- DC cardioversion 

2- Intravenous amiodarone 

3- Adenosine 12 mg 

4- Bisoprolol 5 mg 

5- Digoxin 125 meg 



Answer & Comments 

Answer: 3- Adenosine 12 mg 

The patient with rapid narrow complex 
tachycardia is likely to have supra ventricular 
tachycardia. 

This may be cardioverted by adenosine which 
blocks conduction at the AV node. The dose of 
adenosine in this case has to be optimal 
before trying a second option. 6 mg is unlikely 
to be adequate and a step up to 12 mg or 
even 18 mg may be necessary to cardiovert 
the patient from the SVT to sinus rhythm. 


Dr. Khalid Yusuf El-Zohry - Sohag Teaching Hospital (01118391123) 

Ref MRCPass OE OE 2012 PasTest 2009 PassMedicine 2009 PasTest Exam ReviseMRCP 



832 




























El-zohry MRCP Questions Bank (Port 1) - 2013 


(For my personal use) 



[ Q: 2095 ] MRCPass - 2011 January 

A 46 year old woman presents with 
fevers, night sweats, arthralgia. The urine 
dipstick showed blood ++ and protein +. On 
examination, her BP was 110 / 70 mmHg. She 
had a systolic and diastolic murmur in the 
aortic area and endocarditis was suspected. 
An urgent echocardiogram confirmed 
bacterial vegetations on the aortic valve. 


Which one of the following is on indicotion for 
urgent surgery? 


1- Aortic regurgitation 

2- Cardiac failure 


3- Prolongation of PR interval on the ECG 

4- Pyrexia 

5- Renal impairment 


Answer & Comments 

Answer: 3- Prolongation of PR interval on the 
ECG 

In aortic valve endocarditis, prolongation of 
the PR interval or AV dissociation is a feature 
of possible aortic root abscess (due to the 
position of the cardiac conduction system). 

This is an indication for urgent surgery. 


Answer & Comments 

Answer: 5- Co-amoxiclav 

Pasteurella (canis or multicoda) species are 
the most frequent isolates from both dog 
bites. 

Other common aerobes included streptococci, 
staphylococci, moraxella, and neisseria. 
Augmentin is the recommended antibiotic, 
along with tetanus injection for prophylaxis. If 
there is evidence of cellulites as in this case, 
then fluclox and benpen should be prescribed. 


[ Q: 2097 ] MRCPass - 2011 January 

An 20 year old patient is suspected 
presents with a 6 month history of lethargy 
and weight gain. She has a BMI of 30. On 
examination, she looked obese and has 
abdominal striae. Her Blood pressure is 125 / 
90 mmHg. Blood tests show these results: 
sodium 135 mmol/I, potassium 3.4 mmol/l, 
urea 5 mmol/l, creatinine 100 pmol/l. 

Whot is the investigation of choice? 

1- Low dose dexamethasone suppression test 

2- High dose dexamethasone suppression test 

3- CRH test 

4- Serum ACTH levels 



[ Q: 2096 ] MRCPass - 2011 January 

A 23 year old man has been bitten by 
a dog on the thigh whilst walking in a park. 2 
days later he develops erythema around the 
site and a purulent wound. 



5- Inferior petrosal sinus sampling 


Answer & Comments 

Answer: 1- Low dose dexamethasone 
suppression test 


Whot is best antibiotic to prescribe? 
1- Trimethoprim 


This patient is likely to have Cushing's 
syndrome. 


2- Metronidazole 

3- Flucloxacillin 

4- Ciprofloxacin 

5- Co-amoxiclav 


Failure of suppression of cortisol with low 
dose dexamethasone over 2 days confirms 
Cushing's syndrome. The high dose 
suppression test is often used to determine 
likelihood of pituitary dependent Cushing's 
disease. 


Dr. Kholid Yusuf El-Zohry - Sohog Teaching Hospital (01118391123) 

Ref MRCPass OE OE 2012 PasTest 2009 PassMedicine 2009 PasTest Exam 




ReviseMRCP 

833 






























El-zohry MRCP Questions Bank (Port 1) - 2013 


(For my personal use) 


[ Q: 2098 ] MRCPass - 2011 January 

/ - 

# A 60-year-old, man was admitted to 

a hospital for the evaluation of a left adrenal 
mass detected by abdominal ultrasonography. 
He had no known history of hypertension, 
diabetes mellitus, and other systemic 
diseases. 

Abdominal magnetic resonance imaging 
showed a 54 x 50 x 46 mm mixed cystic and 
solid mass arising from the left adrenal gland. 
The tumor was suspected as a 
pheochromocytoma because 24-hour urine 
metanephrine and vanillymandellic acid levels 
were found to be 5 mg (0-1 mg) and 9.2 mg (0- 
8 mg).The patient's blood pressure was 
220/130 mmHg and HR 105/min. 

Which drug should be used to treat the 
hypertension ? 

1- Bisoprolol 

2- Lisinopril 

3- Phenoxybenzamine 

4- Bendrofluazide 

5- Indapamide 

Answer & Comments 

Answer: 3- Phenoxybenzamine 


Physical examination showed that he had 
diffuse lower extremity muscle tenderness, 
crepitations in the lungs and a rash on the 
trunks. Chest x-ray showed bilateral diffuse 
pulmonary infiltrate. 

Investigations showed: 

urine protein 1+ 

urine sediment - many red blood cell and 
granular casts 

erythrocyte sedimentation rate (ESR) was 65 
mm/hr 

anti-nuclear antibody (ANA) - borderline 
positive 1:40 

anti-streptolysin O (ASO) antibody -< 1:40 

c-ANCA - positive at a titer of 1:320, PR 3 
positive 

p - ANCA - negative 

anti-double stranded DNA titer < less than 1:2. 
What is the diagnosis? 

1- Goodpasture's syndrome 

2- Systemic lupus erythematosis 

3- Microscopic polyangiitis 

4- Wegener's granulomatosis 

5- Sarcoidosis 


In a patient with phaeochromocytoma, 
irreversible alpha adrenoceptor blocker such 
as phentolamine or phenoxybenzamine is 
recommended as first line agents to treat 
hypertension (Irreversible blockade is 
important because a massive release of 
catecholamines from the tumor may 
overcome a reversible blockade). 

[ Q: 2099 ] MRCPass - 2011 January 

A 50 year old man presents with a 
three month history of weight loss, diffuse 
myalgia, epistaxis and hemoptysis. 

He had a past medical history of hypertension, 
and described episodes of haematuria. 


Answer & Comments 

Answer: 4- Wegener's granulomatosis 

The diagnosis fits Wegener's best because of 
the pulmonary and renal involvement. 

In Wegener's, there is often pulmonary 
haemorrhage, haemoptysis, infiltrates on the 
CXR as well as cavitation. It is also associated 
with rapidly progressive segmental necrotising 
glomerulonephritis (75%) which may lead to 
renal failure. 

A diffuse, cytoplasmic pattern of staining 
results from binding of ANCA to antigen 
targets throughout the neutrophil cytoplasm, 
the most common protein target being 
proteinase 3 (PR3). PR3 is the most common 


Dr. Khalid Yusuf El-Zohry - Sohag Teaching Hospital (01118391123) 

Ref MRCPass OE OE 2012 PasTest 2009 PassMedicine 2009 PasTest Exam ReviseMRCP 

























El-zohry MRCP Questions Bank (Port 1) - 2013 


(For my personal use) 


antigen target of ANCA in patients with 
Wegener's granulomatosis. 


C-ANCA 


cANCA - cytoplasmic staining 


P-ANCA 


p ANCA - perinuclear staining 


[ Q: 2100 ] MRCPass - 2011 January 

A 41 year old woman presented with 
rectal bleeding. She has no past medical 
history. Over the last two weeks, she noticed 
rectal bleeding, and alternating constipation 
and diarrhoea. Physical examination revealed 
brown black pigmentation on the oral lips, the 
buccal mucosa, about 1-3 mm in diameter. 
She mentioned that these lesions were 
longstanding. There were no palpable lymph 
nodes palpable, no abdominal mass, 
hepatosplenomegaly, or ascites. 

Whot is the likely diagnosis? 

1- Angiodysplasia 

2- Colon carcinoma 

3- Crohn's disease 

4- Haemorrhoids 



5- Ulcerative colitis 


Answer & Comments 

Answer: 2- Colon carcinoma 

The pigmented oral lesions point tow ards a 
diagnosis of Peutz-Jegher's syndrome, with 
the cause of rectal bleeding being a rectal or 
colonic carconima. 

Peutz-Jeghers syndrome is an autosomal 
dominant disease characterized by the 
development of benign hamartomatous 
polyps in the gastrointestinal tract and 
hyperpigmented macules on the lips and oral 
mucosa. There is a significant association with 
increased risk of carcinoma. Cumulative 
lifetime cancer risk begins to rise in middle 
age. Gastrointestinal cancers and pancreatic 
cancer are associated. 



[ Q: 2101 ] MRCPass - 2011 January 

A 45-year-old man was admitted 
with acute-onset lower back and diarrhoea 
occurring 7-8 times a day. Five weeks earlier, 
he had a permanent pacemaker insertion for 
symptomatic bradycardia. On examination he 
was pyrexial. He had restricted lumbar spine 
movement with pain at this site. 

Investigations showed a normochromic 
normocytic anaemia (haemoglobin 9.6 g/l), 
and a white cell count of 18.0x109/1 (4.0- 
11.0x109/1), urea 8.0 mmol/I (2.5-7.5 mmol/I), 
creatinine of 160 |imol/l (60-120 |imol/l), 
erythrocyte sedimentation rate (ESR) of 108 
mm/h and CRP of 210 mg/I (<20 mg/I). 




Dr. Kholid Yusuf El-Zohry - Sohog Teaching Hospital (01118391123) 

Ref MRCPass OE OE 2012 PasTest 2009 PassMedicine 2009 PasTest Exam ReviseMRCP 


835 

























El-zohry MRCP Questions Bank (Port 1) - 2013 


(For my personal use) 


What is the likely diagnosis? 

1- Ankylosing spondylitis 

2- Crohn's diseae 

3- Clostridium difficile infection 

4- Dysfunctional pacemaker 

5- Staphylococcal discitis 


Dapsone 

Nitrofurantoin 

Sulphonamides (co trimoxazole or septrin) 
Quinolones (ciprofloxacin) 

Primaquine (as an anti malarial drug) 


Answer & Comments 

Answer: 5- Staphylococcal discitis 

This is a case of septic discitis (infection of the 
spinal disc) due to staphylococcus, and it is 
likely to be due to the recent pacemaker 
insertion. 

Back pain, fever and neurological signs are 
often present in septic discitis. 40% of cases 
are due to staph aureus, and blood cultures 
are often positive. MRI of the spine will help 
to confirm the diagnosis. 



[ Q: 2102 ] MRCPass - 2011 January 

A 30 year old patient has previously 
presented with jaundice and diagnosed to 
have G6PD deficiency. She seeks advice about 
travelling to Africa because she wants to take 
malaria prophylaxis. 

Which drug should she avoid taking? 


1- Quinine 


2- Chloroquine 

3- Artesunate 


4- Primaquine 

5- Doyxycycline 


[ Q: 2103 ] MRCPass - 2011 January 

A 53-year-old man was driving in a 
car with his wife when they had an accident. 
He suffered concussion during the car 
accident. 6 months following this incident he 
consults his GP mentioning that is tearful, 
agitated, and unable to concentrate at work. 
He has avoided visiting the site of the 
accident, and also seems to deny the incident. 
His wife's belongings are left untouched since 
the day of the accident, and he still has visions 
of his wife. 

What is the most likely cause? 

1- Anxiety disorder 

2- Post traumatic stress disorder 

3- Grief reaction 

4- Post concussion 

5- Psychotic depression 



Answer & Comments 

Answer: 3- Grief reaction 

A grief reaction can last for up to 12 months, 
but can vary within different cultures. 

The average is probably around six months. 
Symptoms of grief reaction are: 


Answer & Comments 

Answer: 4- Primaquine 

A patient who is known to have G6PD 
deficiency is prone to having haemolytic 
jaundice due to drugs. 

Drugs with risks of causing haemolysis in G6PD 
deficiency are: 


disbelief, shock, numbness and feelings of 
unreality feelings of guilt 

sadness and tearfulness 

preoccupation with the deceased 

disturbed sleep and appetite and, 
occasionally, weight loss 

seeing or hearing the voice of the deceased 


Dr. Khalid Yusuf El-Zohry - Sohag Teaching Hospital (01118391123) 

Ref MRCPass OE OE 2012 PasTest 2009 PassMedicine 2009 PasTest Exam ReviseMRCP 



836 




























El-zohry MRCP Questions Bank (Port 1) - 2013 


(For my personal use) 


[ Q: 2104 ] MRCPass - 2011 January 

A 30 year old patient has been on 
warfarin for atrial fibrillation. His INR is 2.8. 

Which one of the following is inhibited? 

1- Factor V 

2- Factor VII 

3- Factor VIII 

4- Platelet 

5- Fibrinogen 



Answer & Comments 

Answer: 2- Factor VII 

Warfarin is an antagonist of vitamin K, a 
necessary element in the synthesis of clotting 
factors II, VII, IX and X. 



[ Q: 2105 ] MRCPass - 2011 January 

A 60 year-old woman presents with 
a history of acute leukemia of the Ml 
phenotype.Initial treatment consisted of 2 
cycles of induction therapy and 1 cycle of 
high-dose cytarabine/mitoxantrone. 


1 week later she became very unwell. She was 
admitted to hospital and had a temperature of 
39.5 C. She was treated with teicoplanin and 
gentamicin intravenous, but 24 hours later she 
worsened with high fevers. 


What should be added to the treatment 
regime? 


1- Vancomycin 

2- Amphotericin 

3- Aciclovir 

4- Flucloxacillin 


5- Meropenem 


Answer & Comments 

Answer: 2- Amphotericin 

Patients with AML who undergo 
chemotherapy are at increased risk of fungal 


infections, most commonly candidiasis and 
aspergillosis. 

This patient may have invasive fungal 
infection. Amphotericin B therapy is the 
treatment option for invasive candidiasis and 
is also a treatment option for invasive 
aspergillosis. 

[ Q: 2106 ] MRCPass - 2011 January 

A 40 year old African man had been 
home for a holiday but this was 6 months ago. 
He now presents with fever and intermittent 
rigors. There has been no other travel history 
and he has no past medical illnesses. The 
patient gave the history of intermittent high 
fever for the past 5 days accompanied by chills 
and rigors, body-ache and vomiting. 

What is the likely diagnosis? 

1- Plasmodium malaria 

2- Plasmodium falciparum 

3- Plasmodium ovale 

4- Dengue 

5- Trypanosomiasis 



Answer & Comments 

Answer: 3- Plasmodium ovale 

Due to the long period where the patient has 
been well, the most likely cause is 
plasmodium ovale or vivax infection. 

The hypnozoites can develop into mature 
schizonts and release merozoites into the 
blood stream causing clinical symptoms of 
malaria (relapsing malaria) even many months 
after the primary infection. 

Relapses can occur up to 5 years after 
infection. Plasmodium vivax infection may 
present similarly. 


[ Q: 2107 ] MRCPass - 2011 January 


A 62-year-old female is brought to 
A&E by her family, who are concerned about 



Dr. Khalid Yusuf El-Zohry - Sohag Teaching Hospital (01118391123) 

Ref MRCPass OE OE 2012 PasTest 2009 PassMedicine 2009 PasTest Exam 




ReviseMRCP 

837 






























El-zohry MRCP Questions Bank (Port 1) - 2013 


(For my personal use) 


her increasing confusion over the past 2 days. 
There is a history of diarrhea in the preceding 
few days. On examination she is found to be 
pyrexial at 38oC. Breath sounds are clear and 
there is mild tenderness in the lower 
abdomen. There was no focal neurological 
signs. 

Blood tests reveal : 

Hb 9.6 g/dl 
WCC 12 x 10 9 /l 
Platelets 65 x 10 9 /l 
sodium 138 mmol/I 
potassium 4.7 mmol/l 
Urea 23.1 mmol/l 
Creatinine 360 mmol/l 
A blood film shows schistocytes. 

What is the most likely organism? 

1- Streptococcus faecalis 

2- E coli 

3- Staphylococcus aureus 

4- Mycoplasma pneumoniae 

5- Bacteroides 

Answer & Comments 

Answer: 2- E coli 


presents with a history of shortness of breath, 
anorexia, low -grade fever, and severe 
anemia. Her investigation results showed: 

Hb 4.5 g/dl 

MCV 75 fl 

WCC 16 x 1071 (60% blasts, 30% polymorphs) 

platelets 90 x 1071 

Bone marrow biopsy showed: 

hypercellularity, altered myeloid to erythroid 
(M:E) ratio, decreased megakaryocytosis, 
mixed erythropoiesis. 

There were blasts 65%, promyelocytes 61%, 
myelocytes 02%, metamyelocytes 0.1%, band 
cells 0.1%, polymorphs 19%, eosinophils 4%, 
lymphocytes 35%. 

There were occasional blasts with Auer rods, 
many with azurophilic granules, suggestive of 
acute myeloblastic leukaemia. 

The patient was assessed for prognosis. 

Which one of the following is a marker of a 
good prognosis in acute promyelocytic 
leukaemia? 

1- Female sex 

2- T(15:17) 

3- Age of >60 years 

4- Elevated LDH 


The diagnosis here is Hemolytic uremic 
syndrome (HUSIt is characterized by the triad 
of microangiopathic hemolytic anemia, 
thrombocytopenia, and acute renal failure. 

Diarrhea and upper respiratory infection are 
the most common precipitating factors. The 
most common cause of HUS is a toxin 
produced by Escherichia coli serotype 
0157:H7. 

Additional agents include Shigella, Salmonella, 
Yersinia, and Campylobacter species. 

[ Q: 2108 ] MRCPass - 2011 January 
A 20 year old female patient 


5- Philadelphia chromosome t(9:22) positive 

Answer & Comments 

Answer: 2- T (15:17) 

Acute myeloid leukemia (AML), also known as 
acute myelogenous leukemia, is the most 
common acute leukemia affecting adults. 

Anaemia, fever, weight loss, bleeding 
(thrombocytopenia) and also infections can be 
presenting symptoms. According to the widely 
used WHO criteria, the diagnosis of AML is 
established by demonstrating involvement of 
more than 20% of the blood and/or bone 



Dr. Khalid Yusuf El-Zohry - Sohag Teaching Hospital (01118391123) 

Ref MRCPass OE OE 2012 PasTest 2009 PassMedicine 2009 PasTest Exam ReviseMRCP 























El-zohry MRCP Questions Bank (Part 1) - 2013 


(For my personal use) 


marrow by leukemic myeloblasts. There are 8 
subtypes. 


of the child pugh criteria for liver failure) after 
a paracetamol overdose. 


Patients with AML can have high, normal, or 
low WBC counts. 

The single most important prognostic factor in 
AML is cytogenetics, or the chromosomal 
structure of the leukemic cell. Cytogenetic 
karypotypes e.g. t(15;17), t(8;21) and 

inv/del/t(16) are associated with good 
prognosis. Because acute promyelocytic 
leukemia (APL) has the highest curability and 
requires a unique form of treatment, it is 
important to quickly establish the diagnosis, 
particularly the t(15;17) translocation. 

A number of other cytogenetic abnormalities 
are known to associate with a poor prognosis: 
5, -7, del(5q), Abnormal 3q, Complex 
cytogenetics. 

Age >60 years and elevated lactate 
dehydrogenase level are also associated with 
poorer outcomes in AML 


[ Q: 2109 ] MRCPass - 2011 January 

A 29 year old female took 40 tablets 
of Paracetamol and was admitted to hospital. 
She is seen the following day and needs 
assessment of her medical condition. 

Which of the following is the best investigation 
to assess prognosis after 26 hours for a 
paracetamol overdose? 

1- Prothrombin time 

2- AST 

3- Paracetamol level 

4- Urea and creatinine 

5- Bilirubin 



[ Q: 2110 ] MRCPass - 2011 January 

- 

* A 41 year old Caucasian female 
presented with malaise, dysphagia and 
sclerodactyly and Raynaud's phenomenon for 
the last 3 months. 

On physical examination she was afebrile and 
had a supine blood pressure of 110/80mm Hg. 
Sclerodactyly and telangiectasia were 
observed in both hands. Blood tests revealed: 

Hb 11.5 g/dl, MCV 85 fl 

erythrocyte sedimentation rate of 80 mm/first 
hour, antinuclear antibody (ANA) - strongly 
positive antitopoisomerase I antibody 
(formerly anti SCL-70 antibody) positive 

normal C3 and C4 


anti-DNA, anti-centromere, anti-RNP, anti-Ro 
and La antibodies - negative 

Chest x-ray showed bilateral basilar interstitial 
infiltrates. 

What is the diagnosis? 

1- Hereditary haemorrhagic telangiectasia 

2- Sjogren's syndrome 

3- Wegener's granulomatosis 

4- Oesophageal carcinoma 

5- Limited systemis sclerosis 


Answer & Comments 

Answer: 5- Limited systemis sclerosis 

The clues here for limited systemic sclerosis 
(limited scleroderma) are dysphagia, 
sclerodactyly and Raynaud's phenomenon. 


Answer & Comments 

Answer: 1- Prothrombin time 

Although all of the tests may be abnormal, the 
INR / prothrombin time measurement is the 
most important in predicting prognosis (part 


70% of patients initially present with 
Raynaud's phenomenon; 95% eventually 
develop it during the course of their disease. 
Oesophageal dysmotility may cause reflux, 
aspiration or dysphagia. Pulmonary fibrosis 
and renal impairment are also associated. 


Dr. Khalid Yusuf El-Zohry - Sohag Teaching Hospital (01118391123) 

Ref MRCPass OE OE 2012 PasTest 2009 PassMedicine 2009 PasTest Exam ReviseMRCP 



839 


























El-zohry MRCP Questions Bank (Port 1) - 2013 


(For my personal use) 


Antinuclear antibodies are present in about 
95% of the patients. Topoisomerase I 
antibodies (formerly Scl-70) are present in 
approximately 30% of patients with diffuse 
disease (absent in limited disease) and are 
associated with pulmonary fibrosis. 
Anticentromere antibodies are present in 
about 60-90% of patients with limited disease 
and are rare in patients with diffuse disease 
(which is more likely in this case). 


showed oesophageal varices which were not 
bleeding. 

Which one of the following drugs should the 
patient be commenced on for prophylaxis of 
bleeding? 

1- Terlipressin 

2- Propanolol 

3- Lansoprazole 

4- Mesalazine 


[ Q: 2111 ] MRCPass - 2011 January 

A 40 year old lady has been 
commenced on docetaxel as part of 
chemotherapy for ovarian carcinoma. 

What is its mechanism of action? 

1- Pyrimidine analogue 

2- Purine analogue 

3- Inhibition of thymidylate synthase 

4- Binds and inhibits microtubules 



5- Prednisolone 


Answer & Comments 

Answer: 2- Propanolol 

For oesophageal varices, the non-selective a- 
blockers (e.g., propranolol) and nitrates (e.g. 
isosorbide mononitrate) have a role in 
prophylaxis of bleeding. Terlipressin 
(vasopressin analogue) is used in acute 
bleeding as it is vasoactive. 


5- Anthracycline intercalating DNA 


Answer & Comments 

Answer: 4- Binds and inhibits microtubules 

Docetaxel is a clinically well established anti 
mitotic chemotherapy drug used mainly for 
the treatment of breast, ovarian, and non¬ 
small cell lung cancer. 

Docetaxel binds to microtubules reversibly 
with high affinity with the end result of 
reducing mitotic division of cells. 


[ Q: 2112 ] MRCPass - 2011 January 

A 50-year-old man presented to the 
emergency department with complaints of 
fatigue, nausea and vomiting for several 
months. He had a 20 pack-year smoking 
history and alcohol intake consisted of a 20 
units a day. On examination, he was jaundiced 
and had palpable hepatomegaly. He was 
organised to have upper Gl endoscopy, which 



^ [ Q: 2113 ] MRCPass - 2011 January 

f| - 

# A 60-year-old woman is investigated 
for weight loss, fatigue and anaemia. These 
symptoms have been present for 1 year. She 
has a history of hypertension and transient 
ischaemic attack 2 years ago. Clinical 
examination reveals splenomegaly palpable 5 
cm below the costal margin. A full blood count 
is reported as follows: 

Hb 9.8 g/dl 

Platelets 380 x 10 9 /l 

WCC 120 x 10 9 /l 

Blood film. Demonstrates left shift with 
predominating myelocytes. Low percentage of 
blast cells 

What is the likely diagnosis? 

1- Chronic myeloid leukaemia 

2- Acute lymphoblastic leukaemia 

3- Polycythaemic rubra vera 

4- Waldenstrom's macroglobulinaemia 


Dr. Khalid Yusuf El-Zohry - Sohag Teaching Hospital (01118391123) 

Ref MRCPass OE OE 2012 PasTest 2009 PassMedicine 2009 PasTest Exam ReviseMRCP 



840 
































El-zohry MRCP Questions Bank (Port 1) - 2013 


(For my personal use) 


5- Myelodysplasia 

Answer & Comments 

Answer: 1- Chronic myeloid leukaemia 

The diagnosis here is chronic myeloid 
leukaemia, which accounts for 20% of all 
leukaemias. 

It is one of the forms of myeloproliferative 
leukaemias. It occurs mainly in middle aged 
and elderly people and is characterised by 
marked leucocytosis. CML is often suspected 
when the blood count shows increased 
granulocytes of all types, typically including 
mature myeloid cells. A left shift (presence of 
immature cells) myeloid series is present . In 
95% of patients, there is the presence of the 
chromosomal translocation, the Philadelphia 
chromosome. 


[ Q: 2114 ] MRCPass - 2011 January 

A 85 year old man was hospitalised 
after having found on the floor at home. He 
had a history of ischaemic heart disease, 
diabetes and previous surgery for prostate 
carcinoma. Initial investigations suggested 
that he had a urinary tract infection. During 
the admission, he suddenly became very 
agitated at night, and was lashing out at any 
nursing staff who approached him to feed and 
administer medications. 

Which one of the following medications should 
be used for sedation? 

1- Propofol 

2- Haloperidol 

3- Olanzepine 

4- Chlorpromazine 

5- Temazepam 



Answer & Comments 


Answer: 2- Haloperidol 


Pharmacologic management is necessary in 
more severe cases of agitation in which 
patients are a danger to themselves or others, 
or are impeding medical evaluation and care. 

The ideal agent for undifferentiated acutely 
agitated geriatric patients would be effective 
with a rapid onset of action and would be safe 
with minimal side effects. Pharmacologic 
options include the benzodiazepines and the 
typical and atypical antipsychotics agents. The 
typical antipsychotic Haloperidol is commonly 
used for the treatment of acute agitation 
because of its lower incidence of respiratory 
depression, hypotension and anticholinergic 
effects. Chlorpromazine and thioridazine are 
also sedating antipsychotics, but they have 
low potency. Olanzepine is an atypical 
antipsychotic which may be used for chronic 
agitation. Benzodiazepines may also be used 
for acute agitation, lorazepam and midazolam 
are more commonly used - However they are 
associated with increased risk of falls, hypoxia 
and respiratory depression. 



[ Q: 2115 ] MRCPass - 2011 January 

A 30-year-old man gradually noticed 
that he had jaundice and dark urine since 
being started on two medications by the GP. 
He had no other symptoms; in particular there 
was no itching, fever or bleeding, and he was 
not previously taking any drugs. On 
examination, he was anaemic and jaundiced. 


His blood results are: Hb 5.5 g/dl, WCC 7 x 
10 9 /l, platelets 200 x 10 9 /l. The blood film 
showed polychromasia with nucleated red 
cells and spherocytes; the reticulocyte count 
was 9%. His serum bilirubin (47mmol/l), 
aspartate transaminase (90iu/l) and lactate 
dehydrogenase levels (5721iu/l) were raised. 


Which one of the following tests will reveal 
autoimmune haemolytic anaemia? 


1- Cold agglutinins 


2- Direct antiglobulin test 

3- HAM's test 


Dr. Khalid Yusuf El-Zohry - Sohag Teaching Hospital (01118391123) 

Ref MRCPass OE OE 2012 PasTest 2009 PassMedicine 2009 PasTest Exam ReviseMRCP 



841 

























El-zohry MRCP Questions Bank (Port 1) - 2013 


(For my personal use) 


4- Haptoglobins 

5- G6PD levels 


Answer & Comments 

Answer: 2- Direct antiglobulin test 

Antibody Autoimmune Hemolytic Anemia 
(AIHA) is the most common of the 
autoimmune hemolytic diseases. 

The most common antibody involved in warm 
antibody AIHA is IgG. Diagnosis is made by a 
positive direct Coombs test. 

The direct Coombs test is used to detect red 
blood cells sensitized with IgG antibody, and 
complement proteins. It detects antibodies 
bound to the surface of red blood cells in vivo. 

Corticosteroids and immunoglobulins are two 
commonly used treatments for warm 
antibody AIHA. Initial treatment consists of 
prednisolone. If ineffective, splenectomy 
should be considered. 

Cold agglutinins are found in mycoplasma 
infection and infectious mononucleosis. 

[ Q: 2116 ] MRCPass - 2011 January 

A 22 year old woman presented to 
the Emergency Department because of 
progressively severe unilateral knee pain of 24 
hours duration. There was no preceding injury 
or unusual physical activity was identified. She 
did not have any significant past medical 
history, but has been on a trip around Europe 
over the last month. Plain radiographs of the 
knee were unremarkable. On examination, the 
left knee, right knee, right wrist and elbows 
were erythematous and swollen. She did not 
have a skin rash. A joint aspirate was done and 
it showed no organisms but had increased 
white cells, predominantly neutrophils. Her 
ESR Is 60 mm/hr. A urethral swab was taken 
and this did not show any cultures. 

What is the most likely couse? 

1- Lyme disease 



2- Reactive arthritis 

3- Septic arthritis 

4- Psoriatic arthritis 

5- Gonococcal arthritis 

Answer & Comments 

Answer: 2- Reactive arthritis 

This young lady with no past medical history is 
likely to have a reactive arthritis or septic 
arthritis. 

The arthralgia of gonococcal infection is most 
often an asymmetric polyarthralgia. In this 
patient However, the urethral cultures are 
negative hence a reactive arthritis to a 
different infection (e.g. Chlamydia, salmonella, 
shigella, Campylobacter) which occurred 
during the recent travel is more likely. 



[ Q: 2117 ] MRCPass - 2011 January 

A 26 year old woman was admitted 
24 hours after taking 20 tablets of 50mg of 
amitriptyline. She had no history of diabetes 
or any other illnesses. On admission she had a 
low conscious level and restless, the pulse was 
130 beats per minute and her blood pressure 
was 110/70mmHg. The heart sounds were 
normal . The pupils were dilated but reactive 
to light. The electrocardiogram (ECG) revealed 
sinus tachycardia and the QRS duration was 
wide. 


Whot drug should be given? 

1- Diazepam 

2- Naloxone 


3- Flumazenil 


4- Sodium chloride 

5- Sodium bicarbonate 


Answer & Comments 


Answer: 5- Sodium bicarbonate 


Dr. Kholid Yusuf El-Zohry - Sohog Teaching Hospital (01118391123) 

Ref MRCPass OE OE 2012 PasTest 2009 PassMedicine 2009 PasTest Exam ReviseMRCP 




























El-zohry MRCP Questions Bank (Port 1) - 2013 


(For my personal use) 


In amitryptiline overdoses, there are risks of 
tachyarrhythmias which are best treated by 
correction of hypoxia and acidosis. 

Even in the absence of acidosis, sodium 
bicarbonate should be given by intravenous 
infusion to adults with arrhythmias or 
clinically significant QRS prolongation on the 
ECG. 


[ Q: 2118 ] MRCPass - 2011 January 

A 62 year old man presents with 
dysphagia and dysphonia. Cranial nerve 
examination revealed left-sided partial ptosis 
with miosis, tongue deviation to the left, an 
absent gag reflex, palatal palsy on the left, a 
weak voice, and a wasted left 
sternocleidomastoid muscle. 



the cervical ganglia of the sympathetic trunk, 
leading to Horner's syndrome. 


[ Q: 2119 ] MRCPass - 2011 January 

A 75-year old woman has presented 
to the clinic for assessment. She has a past 
medical history of palpitations and is 
suspected to have congenital long QT 
syndrome from evidence of her ECG findings. 

Which one of these drugs should be avoided? 

1- Amoxycillin 

2- Thyroxine 

3- Sertraline 

4- Co-proxamol 

5- Morphine 



Which one of the following areas is 
neurological damage most likely? 

1- Pons 

2- Lateral ventricles 

3- Jugular foramen 

4- Stylomastoid foramen 

5- Cerebellopontine 

Answer & Comments 

Answer: 3- Jugular foramen 

The jugular foramen syndrome is also known 
as Vernet's syndrome. 

The jugular foramen allows passage through 
of IX, X and XI cranial nerves. In jugular 
foramen syndrome, patients present with 
difficulty in phonation and aspiration and 
ipsilateral motor paralysis of the vocal cord, 
soft palate (curtain sign), superior pharyngeal 
constrictor, sternocleidomastoid, and 
trapezius. 

This syndrome may be caused by multiple 
etiologies including trauma or tumor growth 
for example, slow growing vascular tumours 
such as glomus tumours. It can also involve 


Answer & Comments 

Answer: 3- Sertraline 

Common drugs which cause long QT 
syndrome are: 

■ tricyclic antidepressants (e.g. 
sertraline) 

antiarrhythmics: 
o quinidine 
o disopyramide 
o procainamide 
o amiodarone 
o sotalol 

■ non-sedative antihistamine toxicity 

o terfenadine 

■ antimalarials 

o halofantrine 

■ antipsychotics 

o notably haloperidol and 
thioridazine 
cisapride 


Dr. Khalid Yusuf El-Zohry - Sohag Teaching Hospital (01118391123) 

Ref MRCPass OE OE 2012 PasTest 2009 PassMedicine 2009 PasTest Exam 




ReviseMRCP 

843 























El-zohry MRCP Questions Bank (Port 1) - 2013 


(For my personal use) 


methadone 



[ Q: 2120 ] MRCPass - 2011 January 

A 66 year old man presented with a 
sudden onset severe headache. There is a 
history of hypertension and asthma. When he 
was brought into hospital, he was found to 
have a GCS score of 14. On examination, his 
BP was 180/70 mmHg. He had a CT scan which 
showed subarachnoid haemorrhage and he 
was commenced on nimodipine and 
monitored as an inpatient. On day 5, he 
became more drow sy and confused. He did 
not complain of a headache and had no focal 
neurological signs. His GCS score dropped to 
9 . 


tolerance test and urinalysis were both 
normal. During a water deprivation test 
confirmed diabetes insipidus. Her serum 
osmolality was 366 mOsm/kg, while her urine 
osmolality was 156 mOsm/kg. She was given 

l-desmino-8D-arginine-vasopressin (DDAVP), 
and the results revealed suspected 
nephrogenic diabetes insipidus. In this 
condition, which one of these receptors is 
defective? 

1- Calcium 

2- Potassium 

3- Aquaporin 2 

4- Erythropoietin 

5- Cytosome 


What is the most likely complication? 

1- Subdural haematoma 

2- Hydrocephalus 

3- Cerebral infarct 

4- Sagittal sinus thrombosis 

5- Herniation of the brainstem 


Answer & Comments 

Answer: 3- Aquaporin 2 

Aquaporins selectively conduct water 
molecules in and out of the cell, while 
preventing the passage of ions and other 
solutes. 


Answer & Comments 

Answer: 2- Hydrocephalus 

After the first 24 hours have passed, 
rebleeding risk remains around 40% over the 
subsequent four weeks. 

The use of calcium channel blockers, thought 
to be able to prevent the spasm of blood 
vessels by preventing vasospasm. The oral 
calcium channel blocker nimodipine improves 
outcome if administered between the fourth 
and tw enty-first day after the hemorrhage. 
Hydrocephalus may complicate SAH in both 
the short- and long term and may lead to a 
drop in the GCS score. 


There are several types of aquaporin 
receptors, and aquaporin 2 absorbs water in 
response to antidiuretic hormone. 

Nephrogenic diabetes insipidus arises from 
defective or absent receptor sites at the 
cortical collecting duct segment of the 
nephron or defective or absent aquaporin, the 
protein that transports water at the collecting 
duct (autosomal recessive, locus 12ql3). 


[ Q: 2122 ] MRCPass - 2011 January 

A patient with hemophilia A was 
given DDAVP prior to dental extraction. 

What is its mechanism of action in this 
situation? 



[ Q: 2121 ] MRCPass - 2011 January 


A 17-year-old woman complained of 
polydipsia, polyuria, and nocturia. Her glucose 



1- Blocks anti diuretic hormone action 

2- Release stored factor VIII 

3- Increases ristocetin co factor activity 

4- Promotes antithrombin activity 


Dr. Khalid Yusuf El-Zohry - Sohag Teaching Hospital (01118391123) 

Ref MRCPass OE OE 2012 PasTest 2009 PassMedicine 2009 PasTest Exam ReviseMRCP 





























El-zohry MRCP Questions Bank (Port 1) - 2013 


(For my personal use) 


5- Blocks the intrinsic pathway 

Answer & Comments 

Answer: 2- Release stored factor VIII 

Desmopressin can be used to promote the 
release of von willebrand factor and factor VIII 
in patients with disorders such as von 
Willebrand disease and mild hemophilia A 
(factor VIII deficiency). 

Although the mechanisms are not well 
understood, desmopressin causes vw F and 
factor VIII to be released from storage sites 
such as vascular endothelium. 


the scapula. He has a history of hypertension 
and diabetes. He takes aspirin, gliclazide and 
metformin. On examination, he has absent 
pulses on the right arm and an irregularly 
irregular heart beat. The blood pressure is 160 
/ 100 mmHg. He had left sided arm and leg 
weakness compared to the right. 

What is the most likely diagnosis? 

1- Thromboembolic CVA 

2- Takayasu's arteritis 

3- Aortic dissection 

4- Left ventricular aneurysm 

5- Ventricular septal defect 


ft 

[ Q: 2123 ] MRCPass - 2011 January 

Answer & Comments 

• ! tj 

A 25 year old man was admitted to 

Answer: 3- Aortic dissection 


the Psychiatric Ward with symptoms of 
aggressiveness, inappropriate behaviour and 
cyclothymia. On examination there was 
evidence of psychomotor agitation, pressure 
of speech and flight of ideas. A drug screen 
was negative. A head CT was within normal 
limits. A diagnosis of mania was made. 


The patient has chest pain mimicking the 
clinical history of myocardial infarction but has 
two other features (absent pulses unilaterally 
and hemiparesis) which could be 
manifestations of occlusion of vascular supply 
from the aorta. 


What drug should be commenced? 

1- Fluoxetine 

2- Amitriptyline 

3- Diazepam 

4- Lithium 

5- Clozapine 


Answer & Comments 

Answer: 4- Lithium 

Lithium has traditionally been the first line of 
treatment for mania since the 1970s and 
evidence continues to show its effectiveness. 

It can also be used as a mood stabiliser. 

[ Q: 2124 ] MRCPass - 2011 January 

A 66 year old man presents with 
central chest pains radiating to the arm and 



In aortic dissection, if the dissection flap 
occludes the blood supply to the right arm 
(subclavian artery) then an absent pulse may 
occur and if the flap occludes the blood supply 
to the brain (carotid arteries) a hemiparesis 
can occur. 

[ Q: 2125 ] MRCPass - 2011 January 

A 62-year-old man is examined in the 
cardiology clinic. 

During cardiac examination it is noted that the 
pulmonary component of the second heart 
sound occurs before the aortic component. 

Which one of the following is associated with 
this finding? 

1- Pulmonary stenosis 

2- Left bundle branch block 

3- Right bundle branch block 



Dr. Khalid Yusuf El-Zohry - Sohag Teaching Hospital (01118391123) 

Ref MRCPass OE OE 2012 PasTest 2009 PassMedicine 2009 PasTest Exam 




ReviseMRCP 

845 



























El-zohry MRCP Questions Bank (Part 1) - 2013 


(For my personal use) 


4- Atrial septal defect 

5- Deep inspiration 


Answer & Comments 

Answer: 2- Left bundle branch block 

This patient has reversed splitting of the 
second heart sound. 

Left bundle branch block causes a reversed 
split second heart sound as it results in a delay 
in the aortic component. 



[ Q: 2126 ] MRCPass - 2011 January 

A 62 year old man presents with 
worsening breathlessness, confusion and 
headache. Since three months ago, he began 
to have progressive exertional dyspnea, 
aching in the legs, and pain in the left arm 
(without chest discomfort) after 50 yards. 
Upon further investigation, the results were 
obtained: 


IgA 2.8 (0.5-4.0) g/l 
IgG 7 (5.0-13.0) g/l 
IgM 24 (0.3-2.2) g/l 
ESR 90 mm/hr 

Which one of the following is likely to be 
associated? 


1- Hypercalcaemia 

2- Hyperviscosity 

3- Chronic renal failure 


4- Diarrhoea 


5- Demyelination 


Answer & Comments 

Answer: 2- Hyperviscosity 

In this scenario, there is likely to be a 
pulmonary embolus but the underlying 
diagnosis is likely to be Waldenstrom's 
macroglobulinemia due to the high IgM levels. 


Waldenstrom's macroglobulinemia is a 
malignant tumor of lymphocytic and 
plasmacytic cells that secrete IgM. Patients 
often present with hepatosplenomegaly and 
lymphadenopathy. Most of the clinical 
manifestations are due to the hyperviscosity 
syndrome. Common presentations are: 
fatigue, serum hyperviscosity - causing 
mucosal and gastrointestinal bleeding, and 
retinal haemorrhage); due to engorged vessels 
and platelet dysfunction, purpura , 
hepatosplenomegaly and lymphadenopathy, 
neurologic symptoms - alterations in 
consciousness, peripheral neuropathy, visual 
disturbance, nausea and vertigo. 

[ Q: 2127 ] MRCPass - 2011 January 

A 71 year old man presents with 
shortness of breath on exertion. He has a 
history of working in a dockyard. He has 
smoked 15 cigarettes a day for the past 20 
years. He had lost more than 7 kg in weight 
over the past two months. On examination, 
tar stained fingers and grade 3 finger clubbing 
are noted. Auscultation and percussion of the 
chest are suggestive of right sided pleural 
effusion. A chest X ray confirms that he has a 
right sided pleural effusion and also pleural 
plaques. Mesothelioma is suspected. 

What is the best way of confirming the 
diagnosis? 

1- MRI scan of the chest 

2- Closed lung biopsy 

3- Bronchoscopy 

4- Fine needle aspiration 

5- Video assisted thoracoscopy (VATS) biopsy 



Answer & Comments 

Answer 5- Video assisted thoracoscopy (VATS) 
biopsy 

Pleural mesothelioma usually begins as 
discrete plaques and nodules that coalesce to 
produce a sheetlike neoplasm. 


Dr. Khalid Yusuf El-Zohry - Sohag Teaching Hospital (01118391123) 

Ref MRCPass OE OE 2012 PasTest 2009 PassMedicine 2009 PasTest Exam ReviseMRCP 



846 




























El-zohry MRCP Questions Bank (Port 1) - 2013 


(For my personal use) 


Subcutaneous malignant seeding of the 
needle tract is a well-known complication of 
percutaneous fine-needle aspiration biopsy 
despite the fact that it may be less invasive. 
Thus for sake of accuracy, a CT guided or 
thoracoscopically guided biopsy (VATS) should 
be performed if mesothelioma is suggested, 
and results are diagnostic in 98% of cases. 


[ Q: 2128 ] MRCPass - 2011 January 

A 43 year old woman presents with 
breathlessness and chest pains. She has a 
history of pulmonary fibrosis associated with 
connective tissue disease. She takes 
prednisolone, n- acetylcysteine, salbutamol 
and atrovent nebulisers. A lung function test 
was organise to investigate the cause of 
breathlessness and results are: 

FVC (I) (% predicted) 2.28 (66%) 

FEV1 (I) (% predicted) 2.04 (70%) 

FEV1/FVC (%) 89 

total lung capacity (TLC) = 2.9 L (70%) 

TLCO mmol/kPa/min (% predicted) 8.5 (110) 
KCO mmol/kPa/min/l (% predicted) 4.4 (150) 
Whot is the diagnosis? 

1- Pulmonary haemorrhage 

2- Pulmonary embolism 

3- Pleural effusion 

4- Emphysema 

5- Diaphragmatic weakness 



Answer & Comments 


Answer: 1- Pulmonary haemorrhage 


Transfer factor for carbon monoxide (TLCO) is 
a useful investigation in alveolar 
haemorrhage. It is actually the product of 
alveolar volume and carbon monoxide 
transfer coefficient (KCO). The alveolar 
volume is mildly reduced because of alveolar 
filling with blood, and KCO is considerably 
increased because the inhaled CO reacts with 
extravascular haemoglobin. An increased 
TLCO with considerably increased KCO and 
mildly reduced alveolar volume are 
characteristic of pulmonary haemorrhage. 


[ Q: 2129 ] MRCPass - 2011 January 

A 65-year-old was brought into 
hospital for assessment of a progressive 
memory loss and unsteadiness. A friend gives 
a history that he has worsening symptoms of 
difficulty walking, headaches and urinary 
incontinence for the past ten months. On 
examination, he walks with a wide based gait 
and has an MMSE score of 18 / 30. An MRI 
scan was organised, and this showed 
significantly dilated ventricles with no 
identifiable obstructive lesion. 

What should be done? 

1- CT angiogram 

2- Lumbar puncture with CSF drainage 

3- Trial of levodopa 

4- Burr hole surgery 

5- Intravenous methylprednisolone 



Answer & Comments 

Answer: 2- Lumbar puncture with CSF 
drainage 


The lung function shows a restrictive picture, 
reduced FEV1 and FVC, which are consistent 
with the underlying connective tissue disease. 

However, as well as a slightly increased TLCO, 
there is significantly increased KCO (transfer 
factor) which suggests pulmonary 
haemorrhage. 


This patient has Normal pressure 
hydrocephalus (NPH) is a clinical symptom 
complex characterized by a triad of symptoms 
which are: abnormal gait, urinary 

incontinence, and dementia. 

Is a form of communicating hydrocephalus in 
which the intracranial pressure, as measured 
by lumbar puncture, is normal or raised. 


Dr. Khalid Yusuf El-Zohry - Sohag Teaching Hospital (01118391123) 

Ref MRCPass OE OE 2012 PasTest 2009 PassMedicine 2009 PasTest Exam ReviseMRCP 



847 























El-zohry MRCP Questions Bank (Port 1) - 2013 


(For my personal use) 


Removal of CSF is a good diagnostic test as 
symptoms often improve after a significant 
volume of CSF is drained. A good response 
suggests that ventriculo-peritoneal shunting 
should be considered as definitive treatment. 



[ Q: 2130 ] MRCPass - 2011 January 

A 36-year-old white woman 
presented with a 5 year history of plaques 
over both shins. She had noted the gradual 
development of fine blood vessels on her 
shins. On examination, 10- to 15-cm, yellow , 
atrophic, centrally scarred plaques with 
multiple telangiectases around the borders 
were seen over both shins. 


What should be checked? 

1- Thyroid stimulating hormone (TSH) 

2- Cortisol 

3- Fasting blood glucose 

4- Platelet count 


5- Erythrocyte Sedimentation Rate 


the scalp, face, or extremities) and erythema 
nodosum (there are no telangiectasia). 


[ Q: 2131 ] MRCPass - 2011 January 

A 32-year-old woman presented 
with a 2-day history of severe headache. She 
was currently 30 weeks pregnant. She had a 
history of hypertension, was a non-smoker 
and was not diabetic. 

On examination, she had a red right eye with 
mild exophthalmos and clinically obvious 
palsies of the right oculomotor, trochlear, 
ophthalmic, and abducens nerves. There was 
loss of pinprick sensation over the forehead. 

What is the diagnosis? 

1- Orbital cellulitis 

2- Cavernous sinus thrombosis 

3- Midbrain infarct 

4- Posterior inferior cerebellar infarct 

5- Grave's disease of the eye 



Answer & Comments 

Answer: 3- Fasting blood glucose 

The diagnosis is likely to be Necrobiosis 
lipoidica which is associated with diabetes, 
hence the necessity to check the blood 
glucose. 

The lesions most commonly are located on the 
shins but may occur on the face, trunk, and 
arms. 

They typically start as erythematous papules 
over the pretibial areas and slow ly enlarge 
and evolve to well-demarcated, atrophic, 
shiny, yellow -brow n telangiectatic plaques. 
They usually are multiple and bilateral. The 
clinical appearance of necrobiosis lipoidica is 
distinctive. The differential diagnostic 
considerations include granuloma annulare 
(typically found on the dorsa of hands, fingers, 
and feet), sarcoidosis (red-brown papules on 


Answer & Comments 

Answer: 2- Cavernous sinus thrombosis 

Cavernous sinus thrombosis is a very rare, 
typically septic, thrombosis of the cavernous 
sinus, usually caused by bacterial sinusitis. 

Symptoms and signs include pain, proptosis, 
ophthalmoplegia, vision loss, papilledema, and 
fever. 

There is often pain in the eye and forehead - 
ophthalmic division of V may be affected, 
exophthalmos and occasionally, papilloedema 
are common and there could also be cranial 
nerve palsies - III, IV, VI. Pregnancy is a risk 
factor for venous sinus thrombosis, and 
cavernous sinus thrombosis is included in this 
category. Treatment is with antibiotics for any 
underlying infection, and anticoagulation with 
heparin (controversial). 


Dr. Khalid Yusuf El-Zohry - Sohag Teaching Hospital (01118391123) 

Ref MRCPass OE OE 2012 PasTest 2009 PassMedicine 2009 PasTest Exam ReviseMRCP 



848 


























El-zohry MRCP Questions Bank (Port 1) - 2013 


(For my personal use) 


^ [ Q: 2132 ] MRCPass - 2011 January 

t - ------- 

# A 36-year-old male, had a 

splenectomy performed after medical 

treatment for immune thrombocytopenic 

purpura failed. At present, he is asymptomatic 

and wants to know if any further action is 

necessary for the future. He is concerned 

about infections. 

Which one of the following organisms is he 
susceptible to? 

1- E coli 

2- Enterococcus 

3- Klebsiella 

4- Pneumococcus 

5- Legionella 


5- Thyroid hormone 

Answer & Comments 

Answer: 4- Dehydroepiandrosterone 

Addison's disease can lead to a deficiency of 
androgen hormones including testosterone, 
dehydroepiandrosterone (DHEA), and DHEA 
sulphate. 

In women, androgens are produced in the 
adrenal glands and the ovaries. In women, 
adrenal androgens promote the development 
of secondary sex characteristics such as 
underarm and pubic hair. 

These hormones may also be important for 
women's libido (sex drive). 


Answer & Comments 


[ Q: 2134 ] MRCPass - 2011 January 

Answer: 4- Pneumococcus 

•I 

A 21-year-old man without any 


Although any encapsulated organism can 
cause infection post splenectomy, 
pneumococcus (Strep. Pneumoniae) is the 
organism in more than 60% of cases. 
According to guidelines published in the 
Journal of Royal College of Physicians in 2002, 
the standard of care for postsplenectomy 
patients includes immunization with 
pneumococcal vaccine (pneumovax), H. 
influenza vaccine, and meningococcal vaccine 
within 2 weeks of splenectomy. 


[ Q: 2133 ] MRCPass - 2011 January 

A 26 year old lady has been 
diagnosed with addison's disease and recently 
had developed secondary amenorrhoea as 
well as loss of libido. 



significant past history presented to his 
physician with swelling of his hands and feet, 
progressive dyspnoea and weight gain in 
excess of 10 kg in the week prior to admission. 
He also noted decreased frequency and 
quantity of urine during this period. On 
examination he had a blood pressure of 
180/80 mmHg, heart rate of 90 and 
respiratory rate of 16/min. There were no skin 
lesions or lymphadenopathy, and all pulses 
were palpable. There was 2+ pitting pedal in 
the upper and lower limbs. He was also found 
to be in acute renal impairment with a 
creatinine of 250. Urine dipstick showed 
proteinuria 4+. Blood tests results are as 
below : 

sodium 135 mmol/I 
potassium 4.5 mmol/l 


Which hormone deficiency is likely to lead to 
the loss of libido? 

1- Aldosterone 

2- Cortisol 

3- 17alpha OH progesterone 

4- Dehydroepiandrosterone 


urea 5 mmol/l 
creatinine 100 |imol/l 
ALT 32 (5-35) U/l 
AST 25 (1-31) U/l 
ALP 86 (20-120) U/l 


Dr. Kholid Yusuf El-Zohry - Sohog Teaching Hospital (01118391123) 

Ref MRCPass OE OE 2012 PasTest 2009 PassMedicine 2009 PasTest Exam ReviseMRCP 



























El-zohry MRCP Questions Bank (Port 1) - 2013 


(For my personal use) 


Albumin 22 (37-49) g/l 

Total cholesterol 12 (< 5 ) mmol/L 

The patient has been commenced on 
prednisolone but remains hypertensive. 

Whot should be commenced? 

1- Rituximab 

2- Azathioprine 

3- Simvastatin 

4- Ramipril 

5- Penicillamine 


Answer & Comments 

Answer: 4- Ramipril 

The patient has nephrotic syndrome as 
indicated with oedema, hypoalbuminaemia 
proteinuria and hypercholesterolaemia. 

This is likely to be associated with a 
glomerulophritis such as minimal change or 

membranous nephropathy. 

Prednisolone is the main treatment. However, 
blood pressure control is also important with 
an ACE inhibitor to reduce disease progression 
and have an effect in reducing proteinuria in 
the long term. 



[ Q: 2135 ] MRCPass - 2011 January 

A 55 year old man has a long history 
of emphysema. He smoked 20 cigarettes a day 
till the age of 45 but has discontinued now . 
He had large bullae in the lung confirmed by 
CT scans. He was referred for bullectomy. 


Which one of the following is likely to occur 
after surgery? 


1- Increase in FEV1 


2- Increase in intrathoracic gas volume 

3- Decreased FEV1/FVC ratio 

4- Decreased Vital capacity 

5- Decreased DLCO 


Answer & Comments 

Answer: 1- Increase in FEV1 

In patients with giant bullous emphysema, 
bullectomy is the treatment of choice. 

Indeed, patients have reported early 
improvement of dyspnea, hypoxemia and 
hypercapnia usually improve. In addition, 
there is a rise in FEV1, FEV1/FVC ratio, and 
diffusing capacity of the lung for carbon 
monoxide (DLCO). The KCO (corrected 
diffusion capacity) should remain the same. 
The intrathoracic gas volume is decreased 
after bullectomy. 


[ Q: 2136 ] MRCPass - 2011 January 

A 42-year-old has a history of dry 
cough and fever for 2 months, and weight loss 
and night sweats for 1 month. 

He had a history of hypertension and was on 
bendroflumethiazide. Several sputum samples 
were sent and results revealed numerous 
acid-fast bacilli. He was commenced on 
quadruple TB therapy, two months later, the 
patient complained of significant joint pains all 
over the body. 

Which one of the following drugs is likely to 
couse this side effect? 

1- Rifampicin 

2- Isoniazid 

3- Pyrazinamide 

4- Ethambutol 

5- Bendroflumethiazide 



Answer & Comments 

Answer: 3- Pyrazinamide 

The most common (approximately 1%) side 
effect of pyrazinamide is arthralgia. 

Other side effects are hepatitis, vomiting and 
sideroblastic anaemia. 


Dr. Kholid Yusuf El-Zohry - Sohog Teaching Hospital (01118391123) 

Ref MRCPass OE OE 2012 PasTest 2009 PassMedicine 2009 PasTest Exam ReviseMRCP 



850 




























El-zohry MRCP Questions Bank (Port 1) - 2013 


(For my personal use) 


^ [Q: 2137] MRCPass-2011 January 

^ [Q: 2138] MRCPass- 

2011 January 

mft 

A 55-year-old woman presents with 

A 51-year-old man 

has a sudden 


weakness and pains in her limbs. She was 
prescribed simvastatin for 

hypercholesterolaemia 6 months ago. She has 

recently been taking fruit juices as her 
neighbour recommended it for health 

reasons. 

On admission, she had muscle weakness and a 
raised serum creatine kinase of 8000 IU/1. 
There was return of muscle enzymes to 

normal shortly after stopping simvastatin. 

Whot substance was likely to have interacted 
with simvastatin to cause rhabdomyolysis? 

1- Cranberry juice 


attack of ataxia and weakness which required 
admission. He has a history hypertension and 
diabetes. He was a non-smoker and rarely 
drank alcohol. 

On examination, he had a left Horner's 
syndrome with horizontal nystagmus, absent 
left gag reflex. His speech was dysarthric. 
There was also dysdiadochokinesia of the left 
arm and leg and normal motor strength. Upon 
sensory examination, there was decreased 
pinprick sensation over the left side of the 
face and entire right side of the body. 

What is the most likely diagnosis? 


2- Apple juice 

3- Grapefruit juice 

4- Blackcurrant juice 

5- Orange juice 


1- Multiple sclerosis 

2- Brown sequard syndrome 

3- Anterior spinal artery thrombosis 

4- Pontine infarct 


5- Posterior inferior cerebellar artery infarct 


Answer & Comments 

Answer: 3- Grapefruit juice 

Simvastatin and atorvastatin are all 
metabolized by CYP3A4 enzyme and have the 
potential to interact with CYP3A4 substrates 
and inhibitors. 

Commonly quoted inhibitors include 
erythromycin /clarithromycin, fluoxetine, 
cyclosporine and grapefruit juice. 

Consumption of grapefruit juice inhibits the 
metabolism of statins-furanocoumarins in 
grapefruit juice inhibit the cytochrome P450 
enzyme CYP3A4, which is involved in the 
metabolism of most statins and some other 
medications (it had been thought that 
flavonoids were responsible). This increases 
the levels of the statin, increasing the risk of 
dose-related adverse effects (including 
myopathy/rhabdomyolysis). 


Answer & Comments 

Answer: 5- Posterior inferior cerebellar artery 
infarct 

Lateral Medullary Syndrome (Wallenberg's 
syndrome) is of an infarct of the posterior 
inferior cerebellar artery (or basilar artery). 

The clinical features of lateral medullary 
syndrome can be divided into those resulting 
from brainstem or cerebellar dysfunction: 

Cerebellar features: 

■ ipsilateral limb ataxia 

nystagmus to the side of the lesion 

Brain stem features: 

sudden onset of dizziness and 
vomiting 


dysphagia and dysarthria 


ipsilateral Horner's syndrome 


Dr. Khalid Yusuf El-Zohry - Sohag Teaching Hospital (01118391123) 

Ref MRCPoss OE OE 2012 PosTest 2009 PassMedicine 2009 PosTest Exom 




ReviseMRCP 

851 






















El-zohry MRCP Questions Bank (Port 1) - 2013 


(For my personal use) 


ipsilateral facial sensory loss - pain 
and temperature 

ipsilateral pharyngeal and laryngeal 
paralysis - cranial IX and X palsies 
contralateral sensory loss - pain and 


day history of urethral discharge. Mid stream 
urine is negative. A swab was sent and was it 
positive for chlamydia. 

Which one of the following should be 
prescribed? 

1- Penicillin V 


temperature of the limbs and trunk 


^ [ Q: 2139 ] MRCPass - 2011 January 

f| 

# A 45 year old patient has end stage 
renal failure. He has haemodialysis 3 times a 
week at the renal unit. He has become unwell 
over 24 hours ago with erythematous line 
insertion site. His blood pressure is 90/60 
mmHg and he has a temperature of 39 C. 
Dialysis line infection was suspected. Blood 
cultures were taken but results are not 
available yet. 


2- Trimethoprim 

3- Metronidazole 

4- Doxycycline 

5- Ciprofloxacin 


Answer & Comments 

Answer: 4- Doxycycline 

The diagnosis is non gonococcal urethritis. 

This is commonly due to Chlamydia. 
Treatment of choice is doxycycline. 


Which antibiotic should be commenced? 

1- Amoxicillin 

2- Gentamicin 

3- Vancomycin 

4- Linezolid 

5- Teicoplanin 


Answer & Comments 

Answer: 3- Vancomycin 

The likely organism is staphylococcus aureus 
causing dialysis line infection. 

In renal dialysis patients who are ill, 
vancomycin should be started first as there is 
a possibility of MRSA infection. If the blood 
cultures subsequently grow staph aureus 
which are sensitivie to methicillin then the 
antibiotic can be changed to flucloxacillin to 
complete a longer course of treatment eg. 2 
weeks. 



[ Q: 2140 ] MRCPass - 2011 January 
A 19 year old female has a several 


[ Q: 2141 ] MRCPass - 2011 January 

A 70 year man presents with 
complaints of visual changes. He is a type II 
diabetic with a history of autonomic 
neuropathy, hypertension and atrial 
fibrillation. There is a history of sexual 
dysfunction. Recently, he has been put on 
sildenafil. 

Which one of these is a recognised side effect? 

1- Retinitis pigmentosa 

2- Optic neuritis 

3- Tunnel vision 

4- Nystagmus 

5- Bluish vision 



Answer & Comments 

Answer: 5- Bluish vision 

Some sildenafil users have complained of 
seeing everything tinted blue (cyanopsia). 

Other most common side effects of sildenafil 
use include headache, flushing, dyspepsia, 
nasal congestion and impaired vision, 


Dr. Kholid Yusuf El-Zohry - Sohog Teaching Hospital (01118391123) 

Ref MRCPass OE OE 2012 PasTest 2009 PassMedicine 2009 PasTest Exam ReviseMRCP 



852 






























El-zohry MRCP Questions Bank (Port 1) - 2013 


(For my personal use) 


including photophobia and blurred vision. 
Digoxin causes yellow discolouration of vision 
(xanthopsia). 


2- Lisinopril 

3- Amlodipine 

4- Atenolol 


[ Q: 2142 ] MRCPass - 2011 January 

* A 36 year old lady has recently 
presented with weight loss and anaemia. 
Investigations confirmed that she had colon 
carcinoma. Upon review , she said she that 
both her parents had colon carcinoma. She 
enquires about risks of other cancers. 

Which one of the following is she most at risk 
of developing? 

1- Pancreatic carcinoma 

2- Endometrial carcinoma 

3- Small cell carcinoma of the lung 

4- Squamous cell carcinoma of the lung 

5- Breast carcinoma 


Answer & Comments 


Answer: 2- Endometrial carcinoma 


5- Doxazosin 


Answer & Comments 

Answer: 3- Amlodipine 

According to the British Hypertension Society 
guidelines, Patients who are > 55 in age or 
black should be on either a calcium channel 
blocker (C) or thiazide diuretic (D). 

Amlodipine is a calcium channel blocker hence 
the best option here. Both thiazides and ACE 
inhibitors can increase lithium concentration 
levels. 


[ Q: 2144 ] MRCPass - 2011 January 

A 50 year old man presents with 
severe crushing chest pains. His ECG showed 
dominant R waves in VI and V2 leads. The T 
waves were also tall. 



The case scenario refers to the patient having 
Hereditary nonpolyposis colorectal cancer 
(HNPCC ) is an autosomal dominang condition. 

Associated conditions apart from which has 
colon cancer are cancers of the endometrium, 
ovary, stomach, hepatobiliary tract and 
urinary tract. Women with HNPCC have a 80% 
lifetime risk of endometrial cancer. The 
average age of diagnosis of endometrial 
cancer is about 46 years. 


[ Q: 2143 ] MRCPass - 2011 January 

A 71 year old woman has been 
referred for management of a blood pressure 
of 190/100 mmHg. She has a history of bipolar 
disorder and peripheral vascular disease. She 
is currently on aspirin and lithium. 

Which one of the following is the best 
antihypertensive agent to commence? 

1- Valsartan 



Which artery is most likely to be occluded? 

1- Right coronary artery 

2- Septal branch of left anterior descending 
artery 

3- Circumflex artery 

4- Posterior descending artery 

5- Left main artery 


Answer & Comments 

Answer: 3- Circumflex artery 

Posterior myocardial infarction usually results 
from occlusion of the left circumflex coronary 
artery but the anatomy can vary a little. 

Occlusion of the right coronary artery may 
also result in a posterior Ml. 

The changes of posterior myocardial infarction 
are seen indirectly in the anterior precordial 
leads. Leads VI to V3 face the endocardial 


Dr. Khalid Yusuf El-Zohry - Sohag Teaching Hospital (01118391123) 

Ref MRCPass OE OE 2012 PasTest 2009 PassMedicine 2009 PasTest Exam ReviseMRCP 



853 





























El-zohry MRCP Questions Bank (Port 1) - 2013 


(For my personal use) 


surface of the posterior wall of the left 
ventricle. As these leads record from the 
opposite side of the heart instead of directly 
over the infarct, the changes of posterior 
infarction are reversed in these leads. The R 
waves increase in size, becoming broader and 
dominant, and are associated with ST 
depression and upright T waves. 



[ Q: 2145 ] MRCPass - 2011 January 

A 35-year old man was referred with 
a two-month history of generalised weakness, 
fever, and weight loss. 


There was no other relevant past medical 
history. Physical examination revealed a 
moderately wasted young man with severe 
pallor and pyrexia of 39°C. The spleen was 
palpable 6 cm below the left costal margin. 


Blood results showed: 


Hb was 9 g/dl 

haematocrit was 18% 

white cell count 5.3 x 10 9 /l 

platelet count was 89 x 10 9 /I 

His reticulocyte count was <0.0001%. 

There were some tear drop erythrocytes in his 
blood film with 4 normoblasts per 100 
leucocytes interspersed by myelocytes. The 
bone marrow biopsy showed replacement of 
normal haemopoietic elements by early 
fibrosis. 


What is the likely diagnosis? 

1- Chronic myeloid leukaemia 

2- Essential thrombocythaemia 

3- Myelofibrosis 

4- Multiple myeloma 

5- Waldenstrom's macroglobulinaemia 


Myelofibrosis is a chronic, progressive 
myeloproliferative disease. 

It is characterised by prominent bone marrow 
stromal reaction including collagen fibrosis 
and osteosclerosis. 

Clinical features include lethargy, 
constitutional symptoms, transfusion 
dependent anaemia, splenomegaly, tear drop 
poikilocytosis, and a leucoerythroblastic blood 
film. A leucoerythroblastic picture on blood 
film is commonly seen in conditions with 
marrow infiltration. Immature cells 
(myelocytes and normoblasts) are also seen 
on the blood film. 

[ Q: 2146 ] MRCPass - 2011 January 

A 55-year-old woman presents for 
evaluation of a chronic cough, productive of 
very thick, yellow sputum that sometimes 
becomes blood-tinged. She has experienced 
recurrent episodes of fever associated with 
pleuritic chest pain. Over the last 5 years, she 
has developed shortness of breath with 
exertion. A CT scan was performed and it 
revealed that she had bronchiectasis. A recent 
result for sputum culture sent by the GP 
showed the presence of pseudomonas, 
although sensitivities are not known. 

What antibiotic should be commenced? 

1- Amoxicillin 

2- Clarithromycin 

3- Cephradine 

4- Ciprofloxacin 

5- Vancomycin 



Answer & Comments 


Answer: 4- Ciprofloxacin 


Answer & Comments 


Answer: 3- Myelofibrosis 


Bronchiectasis is an uncommon condition that 
is characterized by irreversible dilation of the 
bronchi. 


Dr. Khalid Yusuf El-Zohry - Sohag Teaching Hospital (01118391123) 

Ref MRCPass OE OE 2012 PasTest 2009 PassMedicine 2009 PasTest Exam ReviseMRCP 



854 


























El-zohry MRCP Questions Bank (Port 1) - 2013 


(For my personal use) 


Chronic pulmonary infections and airw ay 
inflammation cause bronchial damage through 
destruction of the muscular and elastic layer 
of the bronchial wall, leading to 
bronchiectasis. Antimicrobial therapy should 
target the following common pathogens 
depending on the patient specific risk factors: 
Haemophilus influenza, Pseudomonas 
aeruginosa, Staphylococcus aureus, and 
Streptococcus pneumoniae. 

P. aeruginosa usually becomes a chronic 
infection and is rarely eradicated, despite the 
use of intravenous antibiotic therapy. 
Fluoroquinolones such as ciprofloxacin or 
levofloxacin are reasonable outpatient 
antibiotics in patients with severe symptoms 
for 7-14 days. Other antibiotics which are 
effective are gentamicin, tobramycin and 
ceftazidime. 


[ Q: 2147 ] MRCPass - 2011 January 

A 25 year old lady returned from 
Indonesia 2 weeks ago and now feels unwell. 
Whilst she was there she was bitten by 
mosquitos. She is lethargic, has significant 
myalgia and complained of fevers. Her 
temperature was 39.6 C and her blood 
pressure was 85 / 60 mmHg . There was a 
generalized petechial rash in the lower part of 
the body. Blood tests showed raised 
inflammatory markers and a malarial film was 
negative on admission. 

What treatment should be commenced? 

1- Morphine 

2- Intravenous quinine 

3- Intravenous fluids 

4- Intravenous ceftriaxone 

5- Intravenous prednisolone 



Answer & Comments 

Answer: 3- Intravenous fluids 

This patient is likely to have dengue fever, as 
she has just returned from South East Asia. 


She also exhibits features of a purpuric rash, 
myalgia, fevers (and also frequently a 
thrombocytopenia). Treatment is supportive, 
as the disease is self limiting. 



[ Q: 2148 ] MRCPass - 2011 January 

A 20 year old man was referred for 
pink discolouration of his urine to the hospital. 
He had no previous relevant medical history. 3 
days ago he complained of a sore throat and 
was given a course of amoxicillin and 
ibuprofen by the GP but those symptoms have 
resolved now . On examination, he looked 
well. His blood pressure was 120/70 mmHg, 
temperature 36 C. There were normal 
abdominal examination and he had no 
palpable organomegaly. Urine dipstick showed 
blood ++, Protein +, nitrites negative. 

Whot is the most likely diagnosis? 


1- Crescentic glomerulonephritis 


2- Wegener's granulomatosis 

3- IgA nephropathy 

4- Post streptococcal glomerulonephritis 

5- Goodpasture's syndrome 


Answer & Comments 

Answer: 3- IgA nephropathy 

IgA nephropathy is the most common 
glomerulonephritis and is characterized by 
deposition of the IgA antibody in the 
glomerulus. 

The classic presentation (in 40-50% of the 
cases) is episodic frank hematuria which 
usually starts within a day or two of a non¬ 
specific upper respiratory tract infection. The 
common differential is post-streptococcal 
glomerulonephritis which typically occurs 
weeks after initial infection. The gross 
hematuria resolves after a few days, though 
microscopic hematuria may persist. Renal 
function usually remains normal. Mild 
proteinuria can also be associated. 


Dr. Khalid Yusuf El-Zohry - Sohag Teaching Hospital (01118391123) 

Ref MRCPass OE OE 2012 PasTest 2009 PassMedicine 2009 PasTest Exam 




ReviseMRCP 

855 























El-zohry MRCP Questions Bank (Port 1) - 2013 


(For my personal use) 


[ Q: 2149 ] MRCPass - 2011 January 

A 50 year old man with heartburn 
was referred by the GP for endoscopy. He has 
had the endoscopy done, it showed Barrett's 
oesophagus. A histology specimen showed 
that there was no H pylori infection and 
dysplastic features. 

What should be done next? 

1- No medication required 

2- Start triple therapy 

3- Start PPI and discharge 

4- Start PPI and follow up with repeat 
endoscopy in 2 years 

5- Start PPI and repeat endoscopy in 8 weeks 



Answer & Comments 

Answer: 5- Start PPI and repeat endoscopy in 
8 weeks 

Barrett's oesophagus (columnar-lined 
oesophagus[CLO] ) is an oesophagus in which 
any portion of the normal squamous lining has 
been replaced by a metaplastic columnar 
epithelium which is visible macroscopically. 


has a temperature of 38.5C. A lumbar 
puncture was performed. Results showed: 

CSF pressure: 12 cm 

glucose - 3.7 mmol/l 

protein < 0.55 g/l 

white cells 290 (95% lymphocytes) 

An MRI scan showed high signal in the 
temporal lobes including hippocampal 
formations and parahippogampal gyrae and 
right inferior frontal gyrus. 

What is the likely diagnosis? 

1- Pneumococcal meningitis 

2- Guillain Barre syndrome 

3- TB meningitis 

4- Poliomyelitis 

5- Herpes simplex virus encephalitis 


Answer & Comments 

Answer: 5- Herpes simplex virus encephalitis 

In Herpes simplex virus (HSV) encephalitis, a 
presentation with fevers, confusion or a 
change in personality is common. 


CLO represents the extreme end of the 
pathophysiological spectrum of gastro- 
oesophageal reflux disease. 

If the endoscopy shows no dysplastic features, 
surveillance should be discussed with the 
patient and it is recommended that it should 
be performed every 2 years. 


The CSF white cell count is elevated with 
lymphocytosis. The majority of cases of herpes 
encephalitis are caused by herpes simplex 
virus-1 (HSV-1) . The MRI typically shows high 
signal changes in the T2 weighted images in 
the temporal lobe areas, in HSV encephalitis. 
Treatment is with iv acyclovir. 


If there were features of dysplasia, then the 
patient should be managed firstly by extensive 
re-biopsy after intensive acid suppression for 
8-12 weeks. If the features of dysplasia 
persist, surveillance should be six monthly and 
if the features progress surgery may be 
recommended. 



[ Q: 2150 ] MRCPass - 2011 January 

A 62 year old woman presents with 
confusion, headache and neck stiffness. She 


^ [ Q: 2151 ] MRCPass - 2011 January 

- 

* A 45-year-old woman presents with 
weight gain and recurrent 'dizzy' episodes. 
Over the past four months she has gained 10 
kg. The episodes occur on an almost daily 
basis and are characterised by blurred vision, 
sweating and headaches. Her GP checked a 
blood sugar during one of these episodes 
which was record as being 2.0 mmol/l. 


Dr. Khalid Yusuf El-Zohry - Sohag Teaching Hospital (01118391123) 

Ref MRCPass OE OE 2012 PasTest 2009 PassMedicine 2009 PasTest Exam ReviseMRCP 



856 



























El-zohry MRCP Questions Bank (Port 1) - 2013 


(For my personal use) 


What is the most useful test to confirm the 
diagnosis? 

1- Insulin tolerance test 

2- Oral glucose tolerance test 

3- 72 hour fast with Insulin + C-peptide levels 

4- Sulphonylurea level 

5- Thyroid function 


2- Asymmetrical bradykinesia 

3- Intention tremor 

4- Ataxic gait 

5- Sensory ataxia 

Answer & Comments 

Answer: 2- Asymmetrical bradykinesia 


Answer & Comments 

Answer: 3- 72 hour fast with Insulin + C- 
peptide levels 

A 72-hour fast, usually supervised in a hospital 
setting, measuring any hypoglycaemia(glucose 
<2.5 mmol/l), insulin and C peptide level 
(which are elevated during one of these 
episodes) will confirm the diagnosis of a 
possible insulinoma (pancreatic insulin 
secreting tumour as suggested in the clinical 
history). 


Idiopathic Parkinson's disease is characterised 
by tremor, rigidity and bradykinesia (which is 
typically asymmetrical). 

There are also features of postural instability, 
a mask like face and a shuffling gait. 



[ Q: 2153 ] MRCPass - 2011 January 
A 70 year old lady presents with 


visual problems. 


On examination, she had a homonymous 
superior left homonymous quadrantinopia. 


The C peptide levels are useful as proinsulin is 
broken down to insulin and C peptide. 


Whot is the likely site of lesion? 

1- Frontal lobe 


If present, then the patient is unlikely to be 
injecting insulin exogenously (which is among 
the possibilities in patients with unexplained 
hypoglycaemic episodes). 

[ Q: 2152 ] MRCPass - 2011 January 

A 60 year old woman presents with a 
four-year history of increasing stiffness and 
immobility which have led to multiple falls. 
She had had some difficulty in fine finger 
movement. Her Blood pressure was 130/90 
mmHg lying and 135/95 mmHg standing. 
Examination showed a mask like facies, 
bradykinesia, nuchal and limb rigidity. She had 
an asymmetrical tremor in her hands and 
cogw heel rigidity in the arms. There was also 
short-term memory loss. 

Which one of the following features most 
likely suggests Parkinson's disease? 

1- Flaccidity 



2- Parietal lobe 

3- Occipital lobe 

4- Temporal lobe 

5- Cerebellum 


Answer & Comments 

Answer: 4- Temporal lobe 

Inferior quadrantinopia is a sign of a parietal 
lobe lesion, whilst a superior quadrantinopia is 
a sign of a temporal lobe lesion. 

[ Q: 2154 ] MRCPass - 2011 January 

A 35 year old man was found to have 
a heart murmur on a routine check up and his 
GP refers him to the general medical clinic. 

He has a past medical history of asthma. On 
examination, blood pressure was 110/ 60 mm 
Hg. The JVP is not elevated. Cardiac 
auscultation evidenced a grade III pan systolic 



Dr. Khalid Yusuf El-Zohry - Sohag Teaching Hospital (01118391123) 

Ref MRCPass OE OE 2012 PasTest 2009 PassMedicine 2009 PasTest Exam ReviseMRCP 



857 






























El-zohry MRCP Questions Bank (Port 1) - 2013 


(For my personal use) 


murmur at both the cardiac apex and left 
sternal border. A thrill was palpable at the left 
parasternal area. 

Whot is the likely diagnosis? 

1- Mitral regurgitation 

2- Aortic stenosis 

3- Tricuspid regurgitation 

4- Atrial septal defect 

5- Ventricular septal defect 

Answer & Comments 

Answer: 5- Ventricular septal defect 

A small VSD could present with few symptoms 
until a heart murmur is picked up. 


Answer & Comments 

Answer: 2- Medullary thyroid carcinoma 

This lady has hypertension and thyroid 
nodules. 

The symptoms and hypertension suggest an 
underlying phaeochromocytoma. The thyroid 
nodules with normal thyroid function tests is 
consistent with multiple endocrine neoplasia 
(MEN), in this case type 2 fits where the 
patient may have medullary thyroid 
carcinoma. 

MEN 1 associations are: pituitary tumour, 
parathyroid hyperplasia/tumour, pancreatic 
tumours (most commonly gastrinoma / 
insulinoma). 


A palpable thrill and pan systolic mumur in the 
left parasternal region is typical. 

[ Q: 2155 ] MRCPass - 2011 January 

A 25 year old woman presents with 
irritability, diarrhoea, sw eatiness and 
palpitations. She has no past medical history 
and is currently not taking medications. She 
has a family history of her maternal uncle 
having primary hyperparathyroidism at the 
age of 35 yrs and his son also had a similar 
diagnosis. On examination, her BP is 190 / 110 
mmHg. She had palpable thyroid nodules. Her 
blood results are: 

sodium 135 mmol/I, potassium 4.3 mmol/l, 
urea 5 mmol/l, creatinine 100 pmol/l,calcium 
3.2 (2.25-2.7) mmol/l, phosphate 0.7 (0.8-8) 
pmol/l. free T4 18 (10-24) pmol/l, TSH 3.2 
(0.3-4) mll/l. 

Which one of the following thyroid conditions 
is associated? 

1- Papillary thyroid carcinoma 

2- Medullary thyroid carcinoma 

3- Hashimoto's disease 

4- Grave's disease 

5- Iodine deficiency 



MEN 2a is associated with medullary thyroid 
carcinoma (MTC), parathyroid tumours (10- 
20%) and pheochromocytoma (20-50%). 

MEN 2b is associated with presentation of 
medullary thyroid carcinoma, parathyroid 
tumours and pheochromocytoma + 
ganglioneuromatosis (pathognomonic). 



[ Q: 2156 ] MRCPass - 2011 January 

A 30 year old lady has severe 
bleeding gums for 6 months and is referred to 
a haematologist who organised the following 
tests with these results: 


Hemoglobin 12 g/dl (10.5-13.5) 
Hematocrit 37% (33.0-39.0) 


WBC 7.9 x 10 9 /L (6.0-17.5) 
Platelets 330 x 10 9 /L (156-369) 

PT 11.3 s (10.0-12.8) 

APTT 49s (28.0-38.0) 

FACTOR VIII 0.15 U/ml (0.60-1.50) 
FACTOR IX 0.82 U/ml (0.60-1.50) 
THROMBIN TIME 18s (16.0-22.0) 
RCOF <0.10 U/ml (0.50-1.50) 


What is the diagnosis? 


Dr. Khalid Yusuf El-Zohry - Sohag Teaching Hospital (01118391123) 

Ref MRCPass OE OE 2012 PasTest 2009 PassMedicine 2009 PasTest Exam ReviseMRCP 



858 


























El-zohry MRCP Questions Bank (Port 1) - 2013 


(For my personal use) 


1- Hemophilia A 

2- Hemophilia B 

3- Von Willebrand's disease 

4- DDAVP deficiency 

5- Factor V leiden deficiency 

Answer & Comments 

Answer: 3- Von Willebrand's disease 

Von Willebrand's disease is caused by 
congenital deficiency (or dysfunction) of vWF, 
a protein cofactor essential for normal platelet 
adhesion and for the transport of Factor VIII. 

Bleeding time is prolonged, platelets show 
reduced adhesion and levels of Factor VIII are 
low . Inheritance is usually autosomal 
dominant, but in the severe forms it may be 
recessive. 

The disease is phenotypically classified into 
three broad categories: 

Type 1 (partial quantitative deficiency, most 
common type) 

Type 2 (qualitative defect) 


which was bloody. Ischaemic colitis was 
diagnosed following a surgical review . 

Where is the most commmon site for the 
condition? 

1- Hepatic flexure 

2- Splenic flexure 

3- Caecum 

4- Sigmoid 

5- Rectum 

Answer & Comments 

Answer: 2- Splenic flexure 

The colon receives blood from both the 
superior and inferior mesenteric arteries. 

The blood supply from these two major 
arteries overlap, with abundant collateral 
circulation. However, there are weak points, 
or "watershed" areas, at the borders of the 
territory supplied by each of these arteries, 
such as the splenic flexure and the transverse 
portion of the colon. These watershed areas 
are most vulnerable to ischemia, thus leading 
to ischaemic colitis. 


Type 3 (total deficiency) 

Useful tests for diagnosing von Willebrand's 
Disease are: 

increased template bleeding time 


[ Q: 2158 ] MRCPass - 2011 January 

Which one of the hormones listed 
below is under a state of continuous 
inhibition? 



low factor VIII level (vWF protect FVIII from 
degradation) 

reduced levels of vWF: Antigen ristocetin 
cofactor activity RCOF - ristocetin fails to 
induce platelet aggregation(because of lack of 
vWF:R - a cofactor for ristocetin) 


1- Prolactin 

2- Growth hormone 

3- Adrenocorticotrophic hormone (ACTH) 

4- Thyroid stimulating hormone (TSH) 

5- Anti diuretic hormone (ADH) 


[ Q: 2157 ] MRCPass - 2011 January 

A 72-year-old male presents to the 
emergency department with sudden-onset, 
diffuse abdominal pain that began 18 hours 
ago. He has not been vomiting, but he has had 
several episodes of diarrhoea, the last of 


Answer & Comments 

Answer: 1- Prolactin 

Prolactin release from the pituitary is inhibited 
(under negative control) by dopamine (from 
hypothalamus). Dopamine produced by 
neurons in the hypothalamus is secreted into 


Dr. Kholid Yusuf El-Zohry - Sohog Teaching Hospital (01118391123) 

Ref MRCPass OE OE 2012 PasTest 2009 PassMedicine 2009 PasTest Exam ReviseMRCP 



859 





























El-zohry MRCP Questions Bank (Port 1) - 2013 


(For my personal use) 


the hypothalamo-hypophysial blood vessels 
which supply the pituitary gland. 

The lactotrope cells that produce prolactin 
and in the absence of dopamine, would 
secrete prolactin continuously. 

[ Q: 2159 ] MRCPass - 2011 January 

A 48 year old man is known to have 
alcoholic liver cirrhosis. He drinks 60 units of 
alcohol per week. He presents unwell and 
complains of abdominal distension and 
abdominal pain. Temperature is 38 C and 
blood pressure is 96/50 mmHg. Abdominal 
palpation reveals hepatomegaly and ascites 
with shifting dullness. 

What should be done next? 

1- Albumin infusion 

2- Ascitic fluid cytology 

3- Ascitic fluid microscopy 

4- Liver biopsy 

5- ultrasound of the abdomen 



Answer & Comments 

Answer: 3- Ascitic fluid microscopy 

There is a high chance of spontaneous 
bacterial peritonitis (SBP) in this patient with 
cirrhotic liver disease. 

When analysis of ascitic fluid reveals a white 
blood cell count of more than 250 cells/cc, 
SBP is likely. Cefotaxime should be 
commenced after a tap is done and blood 
cultures are sent. 

^ [ Q: 2160 ] MRCPass - 2011 January 

# A 30-year-old man presented with a 
pruritic rash, which he has had for 3 months. 
The rash is present on the arm, elbows, 
buttocks and thigh. He has no significant past 
medical history but mentioned that he had 
frequent episodes of loose stool which he had 
accepted was normal for him. The GP 


prescribed betnovate creams which have not 
helped the lesions. On examination, he was 
noted to have many papular, vesicular lesions, 
of average 0.5 cm each. 

Whot is the likely diagnosis? 

1- Erythema marginatum 

2- Erythema multiforme 

3- Guttate psoriasis 

4- Dermatitis herpetiformis 

5- Pityriasis versicolor 


Answer & Comments 

Answer: 4- Dermatitis herpetiformis 

Dermatitis herpetiformis is characterized by 
intensely itchy chronic papulovesicular 
eruptions, usually distributed symmetrically 
on extensor surfaces (buttocks, back of neck, 
scalp, elbows, knees, back). Dermatitis 
herpetiformis 

symptoms typically first appear in the early 
years of adulthood between 20 and 30 years 
of age. There is a strong association with 
gluten intolerance (celiac disease) 

Diagnosis is confirmed by a skin biopsy which 
reveals IgA deposits in the dermal papillae, 
revealed by direct immunofluorescence. 
Dapsone is an effective treatment for most 
patients. A gluten free diet also often leads to 
symptom improvement. 

[ Q: 2161 ] MRCPass - 2011 January 

A 32 year old man complained of 
severe pain in the feet which are burning and 
painful typically at night. He has type 1 
diabetes and is on lantus and novorapid 
insulin. He has poor glycaemic control and a 
HbAlc of 9.5 %. He took diclofenac but the 
pain was not relieved completely. 

What is the best treatment for this man? 

1- Physiotherapy 

2- Gabapentin 



Dr. Khalid Yusuf El-Zohry - Sohag Teaching Hospital (01118391123) 

Ref MRCPass OE OE 2012 PasTest 2009 PassMedicine 2009 PasTest Exam ReviseMRCP 



860 



























El-zohry MRCP Questions Bank (Port 1) - 2013 


(For my personal use) 


3- Morphine 

4- Fluoxetine 

5- Bed rest 


Answer & Comments 

Answer: 2- Gabapentin 

This patient has peripheral neuropathy 
causing pain. 


1- Hydroxyurea 

2- Aspirin 

3- Intravenous immunoglobulin 

4- Abxicimab 

5- Prednisolone 


Answer & Comments 


Answer: 1- Hydroxyurea 


Although they can provide real relief, NSAIDs 
have a "ceiling effect" - that is, there's a limit 
to how much pain they can control. 

Neuropathic agensts such as tricyclic 
antidepressants ( nortriptyline or amitriptyline 
) anticonvulsant drugs, such as gabapentin, 
also may be prescribed for chronic pain. They 
may help by blocking pain messages to the 
brain or by enhancing the production of 
endorphins, body's natural painkillers. 


[ Q: 2162 ] MRCPass - 2011 January 

A 65-year-old white man was 
referred following several significant episodes 
of epistaxis. Physical examination 
demonstrated no lymphadenopathy, no 
bruises, and no purpura. His liver was 
palpated 1 cm below the right costal margin. 
The spleen was not palpable. 

Blood tests results showed: 

hemoglobin level of 9 g/dL 

mean corpuscular volume, 84 x 10-15 L (80- 
96) 

white blood cell count, 10 x 10 3 /|il_ 
platelets, 900 x 10 3 /|iL. 

Bone marrow biopsy showed marked 
megakaryocytic hyperplasia, morphologically 
abnormal megakaryocytes with nuclear 
pleomorphism, and clustering of 
megakaryocytes 

A diagnosis of essential thrombocythemia was 
suspected. 

What drug should be commenced? 



Essential thrombocythemia is a chronic 
myeloproliferative disorder characterized by 
sustained thrombocytosis in the blood ( 
peripheral blood platelet count greater than 
600 ?~ 109/L) and increased numbers of large, 
mature megakaryocytes in the bone marrow . 

In some cases this disorder may be 
progressive, and rarely may evolve into acute 
myeloid leukemia or myelofibrosis. Not all 
patients will require treatment at 
presentation. In those who are at increased 
risk of thrombosis or bleeding (older age, prior 
history of bleeding or thrombosis, or very high 
platelet count), reduction of the platelet count 
to the normal range can be achieved using 
hydroxyurea (also known as 

hydroxycarbamide), interferon-f? or 
anagrelide (phosphodiesterase inhibitor). Low 
-dose aspirin is also widely used to reduce the 
risk of thrombosis. 


[ Q: 2163 ] MRCPass - 2011 January 

A 26 year old lady presents with 
abdominal pains and lethargy. She has a 
history of diabetes and was on insulin. 

Clinical examination was unremarkable and 
she had an abdominal X ray which showed the 
presence of renal calculi. 

Investigations revealed the following: 

arterial blood pH 7.30 (7.38-7.44) 

serum bicarbonate 12.6 mmol/L (21-28 
mmol/L) 

sodium 146 (136-145 mmol/L) 



Dr. Kholid Yusuf El-Zohry - Sohog Teaching Hospital (01118391123) 

Ref MRCPass OE OE 2012 PasTest 2009 PassMedicine 2009 PasTest Exam 




ReviseMRCP 

861 
























El-zohry MRCP Questions Bank (Port 1) - 2013 


(For my personal use) 


potassium 2.8 (3.5-5mmol/L) 
chloride 122 (98-106mmol/L) 

Anion gap was 15 (normal 7-16 mmol/L) 
Urine pH of 6.5 (normal range 5-9) 

Whot is the likely diagnosis? 

1- Multiple myeloma 

2- Renal tubular acidosis type 1 

3- Nephrotic syndrome 

4- Homocystinuria 

5- Porphyria 


Answer & Comments 

Answer: 2- Renal tubular acidosis type 1 

The diagnosis of Type 1 RTA is based on the 
findings of metabolic acidosis, low 
bicarbonate, -hypokalemia, a normal anion 
gap (the anion gap here is Na + K - Cl - HC03 = 
14.6 which is normal) and relatively alkaline 
urine despite the acidosis. 

Type 1 RTA can be familial with autosomal 
dominant as the most common mode of 
inheritance. 

Typical features of type 1 RTA are 
osteomalacia and nephrocalcinosis. 
Associated causes are Sjogren's syndrome, 
SLE, rheumatoid arthritis, renal 
transplantation and sickle cell anaemia. 

Type II RTA is associated with disorders such 
as cystinosis, galactosaemia,Wilson's disease, 
multiple myeloma and Paroxysmal nocturnal 
haemoglobinuria. The distal intercalated cells 
function normally, so the acidemia is less 
severe than dRTA and the urine can acidify to 
a pH of less than 5.3. 



[ Q: 2164 ] MRCPass - 2011 January 

A 35 year old man presented with 
severe retrosternal chest pain. He has history 
of frequent cocaine use. His blood pressure in 
200/110 mmHg and his ECG shows anterior 


wall myocardial infarction with ST elevation in 
leads VI to V4. He has been given Aspirin by 
the ambulance crew. 

Whot is the likely couse of the presentation? 

1- Atherosclerosis 

2- Vasculitis 

3- Coronary vasospasm 

4- Cardiac arrhythmia 

5- Vessel thrombosis 


Answer & Comments 

Answer: 3- Coronary vasospasm 

This man has cocaine induced coronary 
vasospasm. 

Whilst this is frequently transient and can be 
relieved by vasodilators, it can lead to an ST 
elevation myocardial infarction. Thrombolytics 
and anticoagulation will be of minimal effect 
as it is not a thrombotic event like most other 
myocardial infarction cases. 

[ Q: 2165 ] MRCPass - 2011 January 

A 32 year old woman has presented 
with symptoms of haemoptysis for 6 months. 
Urine dipstick showed blood ++ and she was 
noted to have renal impairment with a 
creatinine of 160 umol/l. A Chest XR showed 
upper lobe infiltrates. A renal biopsy was 
performed, and this showed crescentic 
accumulation of cells, deposition of IgG in 
glomerular basement membrane and 
extracellular material in the urinary space of a 
glomeruli. 

Which one of the following tests is likely to be 
positive? 

1- Anti centromere antibody 

2- P ANCA 

3- C ANCA 

4- Anti smooth muscle antibody 

5- Anti GBM antibody 



Dr. Kholid Yusuf El-Zohry - Sohog Teaching Hospital (01118391123) 

Ref MRCPass OE OE 2012 PasTest 2009 PassMedicine 2009 PasTest Exam ReviseMRCP 



862 

























El-zohry MRCP Questions Bank (Port 1) - 2013 


(For my personal use) 


Answer & Comments 

Answer: 5- Anti GBM antibody 

A patient with pulmonary haemorrhage and 
crescentic glomerulonephritis (anti GBM 
positive) or microscopic polyangitis (p ANCA 
positive and anti myeloperoxidase antibody 
positive). 

In Goodpasture's syndrome, the renal biopsy 
typically shows linear IgG deposition along the 
basement membrane. If the patient had a 
wheeze and eosinophilia, then the description 
fits a patient with Churg Strauss syndrome 
(also anti myeloperoxidase antibody positive). 


[ Q: 2166 ] MRCPass - 2011 January 

A 52-year-old man presented with 
an insidious 12-month history of a sensation 
of food getting stuck in his throat, and 
regurgitation of small amounts of previously 
eaten food, some hours after a meal. He also 
mentioned that his family mentioned 
significant halitosis. He said he had lost 1 
stone over the last year. On examination, he 
had a normal oral cavity and sw allowing 
ability, but halitosis was noted. 

What is the likely diagnosis? 



underlying mucosa through this weakness in 
the muscles. 

The most common clinical findings of a 
pharyngeal pouch include dysphagia, or the 
feeling of a lump in the throat. 

Regurgitation of food and mucus a few hours 
after eating may occur, in addition to halitosis 
as a result of food being trapped in the pouch. 


[ Q: 2167 ] MRCPass - 2011 January 

A 61 year old man presents with 
bradykinesia and mask like facies. He was 
found to have cogw heeling and bradykinesia. 
His gait is shuffling in nature. 

Which one of the following drugs is most likely 
to help the bradykinesia? 

1- Amantadine 

2- Benzhexol 

3- Bromocriptine 

4- Levodopa 

5- Selegiline 



Answer & Comments 


Answer: 4- Levodopa 


1- Gastroesophageal reflux 

2- Pharyngeal pouch 

3- Oesophageal adenocarcinoma 

4- Hiatus hernia 

5- Gastric ulcer 

Answer & Comments 

Answer: 2- Pharyngeal pouch 

A pharyngeal pouch is a pulsion diverticulum 
of the pharyngeal mucosa through a 
weakening between the cricopharyngeus and 
thyropharyngeus muscles at the top of the 
oesophagus. 

The pressure generated by swallowing can 
over time result in an out-pouching of the 


The primary pathology in Parkinson's disease 
is loss of dopaminergic action in the substantia 
nigra, leading to rigidity, bradykinesia and 
tremors. 

Bradykinesia results from a failure of basal 
ganglia output to reinforce the cortical 
mechanisms that prepare and execute the 
commands to move. The first line treatment is 
with L-dopa which is the metabolic precursor 
of L-dopa. Benzhexol is an anticholinergic drug 
(used to alleviate tremors in parkinson's 
disease), and is not effective against 
bradykinesia. 

Dopamine agonists (bromocriptine) and MAO 
inhibitors (selegiline) are used as adjuncts to 
patients who have motor fluctuations on L- 
dopa. 


Dr. Khalid Yusuf El-Zohry - Sohag Teaching Hospital (01118391123) 

Ref MRCPass OE OE 2012 PasTest 2009 PassMedicine 2009 PasTest Exam ReviseMRCP 































El-zohry MRCP Questions Bank (Port 1) - 2013 


(For my personal use) 



[ Q: 2168 ] MRCPass - 2011 January 

A 40 year old lady has had a renal 
transplant 4 years ago. She has end stage 
renal failure due to diabetes and 
hypertension. She is on the following 
medications: tacrolimus , azathioprine, 

prednisolone , lansoprazole and insulin 
novomix . Her baseline creatinine has been 
100 for the last two years. She came to clinic 
for blood tests which show these results: 


Hb 11.0 g/dl, WCC 12 x 10 9 /l platelets 250 x 

10 9 /l 

sodium 136 mmol/I, potassium 6.6 mmol/I 

urea 6 mmol/I 

creatinine 105 |imol/l 

Whot is the likely cause of hyperkalaemia? 

1- Tacrolimus 

2- Azathioprine 

3- Prednisolone 


4- Lansoprazole 

5- Insulin 


Answer & Comments 

Answer: 1- Tacrolimus 

Tacrolimus suppresses renin release, leading 
to decreased aldosterone synthesis and 
decreased potassium secretion in collecting 
duct leading to hyperkalaemia. 


[ Q: 2169 ] MRCPass - 2011 January 

A 25 year old woman has a history of 
recurrent urinary tract infection as a child. 

Her mother has a history of hypertension and 
was told that her kidneys were 'damaged'. An 
ultrasound of the patient showed scarring in 
both kidneys. 

What is the most likely diagnosis? 

1- Autosomal dominant polycystic kidney 
disease 

2- Reflux nephropathy 



3- Renal cell carcinoma 

4- Diabetic nephropathy 

5- IgA nephropathy 


Answer & Comments 

Answer: 2- Reflux nephropathy 

Urine reflux is the most common cause of 
chronic pyelonephritis and can lead to 
nephropathy. 

The risk factors include a personal or family 
history of reflux. Ultrasound can identify renal 
scarring if the degree is moderate to severe. 


[ Q: 2170 ] MRCPass - 2011 January 

A 30 year old patient has been 
diagnosed as having likely breast cancer on a 
mammogram. 

Which one of the following blood tests is 
useful as a prognostic marker for this patient? 

1- Ca 125 

2- Ca 19-9 

3- Ca 15-3 

4- Alpha feto protein 

5- Human Chorionic Gonadotrophin 



Answer & Comments 

Answer: 3- Ca 15-3 

CA 15-3 (also known as MUC1) is the most 
widely used serum marker in breast cancer. 

Studies have shown that patients with high 
preoperative levels of CA 15-3 (>30.4 U/mL) 
had a worse outcome than patients with low 
levels of the marker. 



[ Q: 2171 ] MRCPass - 2011 January 

A 46-year-old male with no previous 
medical history was admitted to hospital with 
a severe, dull chest pain, radiating to both 
shoulders. The chest pain is worse with 


Dr. Khalid Yusuf El-Zohry - Sohag Teaching Hospital (01118391123) 

Ref MRCPass OE OE 2012 PasTest 2009 PassMedicine 2009 PasTest Exam ReviseMRCP 



864 
































El-zohry MRCP Questions Bank (Port 1) - 2013 


(For my personal use) 


inspiration. A few days before, he had 
suffered a minor cold. He was stable 
haemodynamically and cardiac examination 
showed a nondisplaced point of maximal 
intensity and normal SI and S2. There were no 
extra heart sounds or cardiac murmurs. The 
ECG showed wide spread saddle shaped ST 
elevation. Full blood count, erythrocyte 
sedimentation rate, blood electrolytes and 
serum creatinine were normal. Troponin 
result was positive. 

Whot is the diagnosis? 

1- Myocardial infarction 

2- Musculoskeletal chest pain 

3- Pulmonary embolus 

4- Costochondritis 

5- Pericarditis 


Answer & Comments 

Answer: 5- Pericarditis 

The diagnosis is acute pericarditis, which is 
likely to be due to a viral infection e.g. 
coxsackie virus. The mainstay of therapy is 
nonsteroidal anti-inflammatory drugs 
(NSAIDs). Aspirin, indomethacin, naproxen 
and diclofenac are examples. Corticosteroids 
should be reserved for patients whose 
symptoms are refractory to NSAID therapy. 


[ Q: 2172 ] MRCPass - 2011 January 

A 36-year-old woman presented 
with haemorrhage, peri-rectal bleeding and 
easy bruising.She has a history of chronic 
lymphocytic leukaemia and has been on 
Fludarabine, chlorambucil, cyclophosphamide 
and rituximab treatment. Physical 
examination revealed multiple bruses, right 
cervical lymphadenopathy, and 

hepatosplenomegaly. Laboratory data showed 
haemoglobin 5.9 g/dL, while blood cell count 
(WBC) 92 x 10 9 /L with circulating blasts, and 
platelet count 34 x 10 9 /L. Her blood group is 0 
Rh Negative. 



Whot type of blood should be prescribed? 

1- Irradiated blood 

2- Group A blood 

3- Group B blood 

4- Rh negative 

5- CMV negative 


Answer & Comments 

Answer: 1- Irradiated blood 

Irradiated blood reduces the risk of Grave 
versus Host disease, and is recommended for 
patients who have had stem cell 
transplantation or haematological malignancy 
with immunosuppression due to 
chemotherapy. 

Patients with blood group 0 can only receive 
blood of Group 0. 


[ Q: 2173 ] MRCPass - 2011 January 

A 22-year-old woman presents with 
a fall and was found at home. She had a past 
history of coronary artery bypass grafting, 
stroke and diabetes. Her usual drugs include 
ramipril, aspirin and atenolol. On admission, 
she had a blood pressure of 95/60 mmHg and 
temperature of 34 C. She was very weak and 
unable to get out of bed. 

Cardiovascular, respiratory and abdominal 
examination were unremarkable. Urine 
dipstick shows protein ++, blood +++, white 
cells +. 

Whot test should be done? 

1- Magnesium 

2- Creatine kinase 

3- International normalised ratio 

4- Troponin 

5- Brain natriuretic peptide 



Answer & Comments 


Answer: 2- Creatine kinase 


Dr. Kholid Yusuf El-Zohry - Sohog Teaching Hospital (01118391123) 

Ref MRCPass OE OE 2012 PasTest 2009 PassMedicine 2009 PasTest Exam ReviseMRCP 






























El-zohry MRCP Questions Bank (Port 1) - 2013 


(For my personal use) 


This patient is likely to have been on the floor 
due to weakness and may have 
rhabdomyolysis. 

The urine dipstick may demonstrate blood, 
although the true test is of myoglobin levels. A 
significantly elevated creatine kinase enzyme 
would be a reasonable indicated of 
rhabdomyolysis, and the patient should be 
kept well hydrated. 

Monitoring of renal function and urine output 
would be important. 


What is the mechanism of action of 
carbimazoie? 

1- Thyroperoxidase enzyme inhibitor 

2- Thyroid stimulating hormone inhibitor 

3- Thyroxine binding action 

4- Inhibits iodination of thyroxine 

5- Inhibition of enzyme 5'-deiodinase 

Answer & Comments 

Answer: 4- Inhibits iodination of thyroxine 


[ Q: 2174 ] MRCPass - 2011 January 

A 20 year old man has noticed some 
lesions around the genital area and seeks 
consultation. On examination, there are 
several popular lesions around the shaft of the 
penis and scrotum. Genital warts are 
confirmed as the diagnosis. 

What treatment should be offered? 

1- Augmentin 

2- Podophyllotoxin 

3- Acyclovir 

4- Ketoconazole 

5- Permethrin 



Carbimazoie is an aitithyroid agent that 
decreases the uptake and concentration of 
inorganic iodine by the thyroid. 

It prevents the thyroid peroxidase enzyme 
from coupling and iodinating the tyrosine 
residues on thyroglobulin, hence reducing the 
production of the thyroid hormones T3 and 
T4. 

Propylthiouracil inhibits the thyroperoxidase 
enzyme 


Answer & Comments 

Answer: 2- Podophyllotoxin 

Genital warts are caused by human 
papillovirus infection. 

First-line treatment (0.15% cream) for soft 
warts in accessible sites, e.g. vaginal introitus, 
under foreskin includepodophyllotoxin and 
cryotherapy. 

^ [ Q: 2175 ] MRCPass - 2011 January 

#1 - 

# A 28 year old lady was prescribed 

carbimazoie as she was diagnosed with 
Grave's disease. 


Dr. Khalid Yusuf El-Zohry - Sohag Teaching Hospital (01118391123) 

Ref MRCPass OE OE 2012 PasTest 2009 PassMedicine 2009 PasTest Exam ReviseMRCP 



866 

























El-zohry MRCP Questions Bank (Port 1) - 2013 


(For my personal use) 


[ Q: 2176 ] MRCPass - 2011 May 

A 46 year old lady presented to her 
physician with complaints of weakness and 
headaches for several months. 

At presentation, she was found to have severe 
hypertension with blood pressure 190/110 
mmHg. 

Blood results are: 

sodium 149 mmol/l 

potassium 2.9 mmol/l 

urea 7 mmol/l 

creatinine 100 |imol/l 

Renin 4.1 ng/L (13.6 -70) 

Plasma aldosterone 2170 pmol/L (110-800) 

Urine Adrenaline 52 (<80 nmol/24 hours) 

Urine Noradrenaline 650 (<780 nmol/24 
hours) 

Urine Dopamine 2100 (<3500 nmol/24 hours) 

Urine Cortisol 210 (100-300) nmol/24 hours 

An MRI abdomen showed a well defined 1 cm 
x 1.5 cm mass in the right supra-renal gland. 

What is the likely diagnosis? 

1- Phaeochromocytoma 

2- Multiple endocrine neoplasia I 

3- Renal artery stenosis 

4- Cushing's syndrome 

5- Conn's syndrome 



Answer & Comments 

Answer: 5- Conn's syndrome 

The diagnosis is Conn's syndrome. 

Conn syndrome is characterized by increased 
aldosterone secretion from the adrenal 
glands, suppressed plasma renin activity 
(PRA), hypertension, and hypokalemia as seen 
in the case above. 

Routine laboratory studies can show 
hypernatremia, hypokalemia, and metabolic 


alkalosis resulting from the action of 
aldosterone on the distal tubule of the kidney. 

^ [Q: 2177] MRCPass-2011 May 

A - 

* A 41 year old, chronic hemodialysis 
patient received a living donor kidney 
transplant from an HLA-identical sibling. 

The transplant recipient had no antibodies to 
CMV at the time of transplantation, whereas 
the donor was CMV positive. The 
posttransplant immunosuppression therapy 
included tacrolimus 3 mg twice a day, 
mycophenolate mofetil 1000 mg twice a day, 
and prednisone 20 mg every day. 

On posttransplant day 39, the patient was 
admitted to the inpatient transplant service, 
complaining of abdominal flank pain, nausea 
and experiencing malaise. She had been 
having intermittent fevers for the last week. 
At that time, the patient's white blood cell 
count was 5.8 k/uL and her platelet count was 
within normal limits. Her creatinine rose from 
pre transplant 100 ul/l to 145 ul/l. A renal 
biopsy showed acute rejection. Subsequent to 
that biopsy result, her CMV viral load (PCR) 
was also found to be positive (8870 
copies/mL). 

What should be done? 

1- Increase dose of tacrolimus 

2- Increase dose of prednisolone 

3- Commence ribavirin 

4- Commence ganciclovir 

5- Perform an OGD 


Answer & Comments 

Answer: 4- Commence ganciclovir 

The timing of events and CMV serology 
suggests that the deterioration is due to CMV 
infection. 

CMV infection is a multifaceted phenomenon 
with a variety of direct and indirect effects in 
the organ transplant recipient. The 


Dr. Kholid Yusuf El-Zohry - Sohog Teaching Hospital (01118391123) 

Ref MRCPass OE OE 2012 PasTest 2009 PassMedicine 2009 PasTest Exam ReviseMRCP 


























El-zohry MRCP Questions Bank (Port 1) - 2013 


(For my personal use) 


symptomatology for clinical infectious disease 
(ie, fever, pneumonia, Gl ulcers, hepatitis) 
ranges from the mild, subclinical case to life- 
threatening multi-organ disease. Most cases 
of symptomatic CMV infection can be 
characterized by a self-limiting syndrome of 
episodic fever spikes for a period of 3 to 4 
weeks, arthralgia, fatigue, anorexia, 
abdominal pain, and diarrhoea. Ganciclovir is 
the most commonly used agent for the 
prevention and also treatment of CMV 
infection. The treatment dose is 5 mg/kg 
intravenously every 12 hours. 

Hyperacute rejection of the renal allograft 
happens within hours of the transplant, and it 
occurs when circulating, preformed, cytotoxic, 
antidonor antibodies directed to the ABO 
blood group antigens or to the donor HLA 
class I antigens are present. No treatment 
exists, and nephrectomy is indicated. 

Accelerated acute rejection is a very early, 
rapidly progressive, aggressive rejection 
reaction dependent on T cells. 


had had no abdominal pain and had started to 
vomit the day before presentation. On 
examination he was dehydrated and mildly 
disorientated. He had a tachycardia of 
140/min, and a blood pressure of 90/70 
mmHg, with a postural drop to 60/30 mmHg. 
His abdomen was soft and non-tender, with 
tinkling bow el sounds. 

Sodium 140 mmol/l (135-145 mmol/l), 
Potassium 7.6 mmol/l (3.5-5.0 
mmol/l),Chloride 83 mmol/l (95-110 mmol/l) 
Bicarbonate 16 mmol/l (22-26 mmol/l) Urea 
32 mmol/l (2.8-8.9 mmol/l) Creatinine 360 
[irnol/l (75-115 mmol/l) Glucose 4.9 mmol/l 
(3.9-6.1 mmol/l) pH 7.20 (7.35-7.45 pH units). 
An ECG shows broad complexes and tall T 
waves. 

Whot should be given next? 

1- Frusemide 

2- Haemodialysis 

3- Sodium bicarbonate 

4- Insulin and dextrose 


It can occur within the first week of 
transplantation. Immediate therapy with anti- 
T-cell antibodies and pulse corticosteroids 
may reverse the process. Approximately 50% 
of cases can be salvaged. Chronic rejection is a 
slow and progressive deterioration in renal 
function characterized by histologic changes 
involving the renal tubules, capillaries, and 
interstitium. The precise mechanism of this 
disease is poorly defined and is an area of 
intense study. Application of conventional 
antirejection agents, such as corticosteroids or 
anti-T-cell antibodies, does not appear to alter 
the progressive course. Unfortunately, this is a 
major cause of kidney allograft loss, occurring 
later than 2 years posttransplantation. 


[ Q: 2178 ] MRCPass - 2011 May 

A 71-year-old man was admitted 
with a 3-w eek history of general debility and 
diarrhoea, passing copious amounts of watery, 
brow n stool through the day and night. He 



5- Calcium gluconate 


Answer & Comments 

Answer: 5- Calcium gluconate 

The important management step is to provide 
cardioprotection in the form of calcium 
gluconate, particularly when there are ECG 
changes. 

Following this the patient should have insulin 
and dextrose, as well as consideration for 
haemodialysis if the potassium does not 
improve. Sodium bicarbonate therapy should 
be reserved for the treatment of severe 
metabolic acidosis (i.e., pH below 7.2 or a 
bicarbonate level below 10 to 15 mmol per L) 
with or without associated hyperkalemia. 


Dr. Kholid Yusuf El-Zohry - Sohog Teaching Hospital (01118391123) 

Ref MRCPass OE OE 2012 PasTest 2009 PassMedicine 2009 PasTest Exam ReviseMRCP 





















El-zohry MRCP Questions Bank (Port 1) - 2013 


(For my personal use) 


^ [ Q: 2179 ] MRCPass - 2011 May 

/ - - - — - — - 

# A 53-year-old female presented with 

two month history of an itchy rash which first 

appeared on her wrist and gradually her neck 

and upper arms. She also complained of a sore 

tongue and lethargy which had become 

increasingly severe over the past 5 months. 

There was no relevant past medical, family or 

drug history. 

On examination the mucous membranes were 
pale, the tongue appeared smooth and shiny 
with several superficial erosions on the upper 
surface. There were annular atrophic 
pigmented lesions on the abdomen, lumbar 
region, back of the neck and scattered over 
other areas on the trunk. 

What is the most likely diagnosis? 

1- Guttate psoriasis 

2- Granuloma annulare 

3- Pityriasis versicolor 

4- Pityriasis rosea 

5- Lichen planus 

Answer & Comments 

Answer: 5- Lichen planus 

There is an increased incidence of lichen 
planus in several diseases in which there is 
autoimmune phenomena. 

Lichen planus (LP) is a pruritic, papular 
eruption characterized by its violaceous color; 
polygonal shape; and, sometimes, fine scale. 


hyperkeratosis, onycholysis, and longitudinal 
melanonychia can result from LP. 



Oral lichen planus 



Lichen Planus on flexural surface 


The initial lesion is usually located on the 
flexor surface of the limbs, such as the wrists. 

After a week or more, a generalized eruption 
develops with maximal spreading within 2-16 
weeks. Oral lesions may be asymptomatic or 
have a burning sensation, or they may even be 
painful if erosions are present. 

Characteristic fine, white lines, called 
Wickham's striaw , are often found on the 
papules. Hyperpigmentation, subungual 


[ Q: 2180 ] MRCPass - 2011 May 

A 60 year old man presents with leg 
weakness and breathlessness. He had bilateral 
lower limb paraesthesias followed by bilateral 
progressive leg weakness and difficulty in 
walking. The symptoms have worsened over 
the last 5 days. He reported an episode of an 
upper respiratory tract infection 3 weeks prior 
to the onset of the neurological symptoms. A 
lumbar puncture was performed. 




Dr. Kholid Yusuf El-Zohry - Sohog Teaching Hospital (01118391123) 

Ref MRCPass OE OE 2012 PasTest 2009 PassMedicine 2009 PasTest Exam ReviseMRCP 


869 
























El-zohry MRCP Questions Bank (Port 1) - 2013 


(For my personal use) 


Cerebrospinal fluid revealed increased protein 
but normal white blood cell count. 

How should his respiratory function be 
monitored? 

1- PEFR 

2- Vital capacity 

3- Tidal volume 

4- Arterial blood gases 

5- Chest expansion 


Answer & Comments 

Answer: 2- Vital capacity 

The history here of ascending limb weakness 
following a recent infection is typical of 
guillain barre syndrome (GBS). 

A typical GBS patient presents with rapidly 
ascending symmetrical weakness, which may 
progress to respiratory failure in 30% of 
patients. Vital capacity measurements are 
important in Guillain Barre syndrome. 

Patients with an FVC less than 15-20 mL/kg 
indicate compromise and need to be observed 
for the need for ventilation. 


Transferrin saturation (TS) is the most 
effective and inexpensive screening test for Fe 
overload. 

If the TS is > 45%, the test should be repeated, 
together with serum ferritin. If TS (w ith or 
without high ferritin) is raised, HFE genes 
should be determined. 

[ Q: 2182 ] MRCPass - 2011 May 

A 18-year-old woman presented 
feeling unwell to the accident and emergency 
department of a district general hospital. She 
has been attending a summer camp teaching 
children basic outdoor skills over the last 
week. She complained of a widespread rash, 
dry cough and sore eyes. She has no relevant 
past medical history and does not take regular 
medications. On examination, the 
temperature is 38 C. She has a widespread 
maculopapular rash with confluence. Her eyes 
are red and he had bilateral crepitations in the 
chest. 

What is the likely diagnosis? 

1- HIV seroconversion 

2- Measles 




[ Q: 2181 ] MRCPass - 2011 May 


A 26 year old patient presents to the 
clinic for advice. His brother has recently been 
diagnosed with haemochromatosis. 


Which one of the following is the most useful 
screening test? 


1- Ferritin 

2- HFE gene analysis 

3- Ultrasound of the liver 


4- Liver iron levels 

5- Transferrin saturation 


Answer & Comments 


Answer: 5- Transferrin saturation 


3- Mumps 

4- Typhoid fever 

5- Chickenpox 

Answer & Comments 

Answer: 2- Measles 

Measles is a highly communicable acute 
disease that is caused by the airborne 
transmission of a paramyxovirus. 

In its classical form it is characterized by high 
fever, dry cough, coryza and conjunctivitis. 
Koplik's spots are rarely seen but are 
pathognomonic of disease. The characteristic 
rash appears several days after the onset of 
fever. The rash is maculopapular and 
erythematous, which spreads from the head 
to the torso and the extremities. 


Dr. Khalid Yusuf El-Zohry - Sohag Teaching Hospital (01118391123) 

Ref MRCPass OE OE 2012 PasTest 2009 PassMedicine 2009 PasTest Exam ReviseMRCP 



870 



























El-zohry MRCP Questions Bank (Port 1) - 2013 


(For my personal use) 



[ Q: 2183 ] MRCPass - 2011 May 

A 60 year old lady has investigations 
for renal impairment (creatinine 220 umol/l). 
She has a previous history of hypertension, 
peripheral vascular disease and osteoarthritis. 
She has been taking diclofenac for 6 years and 
penicillamine for the past 2 years (both drugs 
were stopped 6 months ago). Her blood 
pressure is 100/60 mmHg and her estimated 
GFR is 6 mls/min/1.73m2. Results are: 

Urine : protein +, blood -ve 

Ultrasound of kidneys : right 8.6 cm, left 9.4 
cm in length. 

What is the likely diagnosis? 

1- Ischaemic nephropathy 

2- Analgesic nephropathy 

3- Interstitial nephritis 

4- Minimal change nephropathy 

5- Diabetic nephropathy 



Answer & Comments 

Answer: 1- Ischaemic nephropathy 

The likely diagnosis is renovascular disease 
due to the small kidney size. 

There is only 1+ Proteinuria, which makes 
glomerulonephritis less likely. The other clues 
are the history of hypertension and the 
vascular risk factors for renovascular disease. 


[ Q: 2184 ] MRCPass - 2011 May 

A 35 year old lady has started 
working in a factory 2 months ago. She 
complains of breathlessness and wheeziness. 
The GP refers her to the respiratory clinic for 
assessment. 

What investigation is most appropriate to 
diagnose occupational asthma? 

1- Measure peak flows over 2 weeks 

2- Serial peak flow measurements at home 
and work 

3- Skin prick tests 

4- Lung function test 

5- Vitalogram 



Answer & Comments 

Answer: 2- Serial peak flow measurements at 
home and work 

Lung function tests and reversibility will help 
to confirm asthma. 

For diagnosis of occupational asthma, serial 
measurements of peak expiratory flow rate at 
home and at work: this is often the most 
appropriate first step. 

Measurements should be made every two 
hours from waking to sleeping for four weeks, 
keeping treatment constant and documenting 
times at work. There should be at least 3 
consecutive work days and 3 days away from 
work included in the measurements, (as per 
Guidelines from the British Occupational 
Health research on Occupational Asthma). 


[ Q: 2185 ] MRCPass - 2011 May 

A 42 year old man presented to his 
GP because of painful blisters on the backs of 
his hands in the summer. He also had a similar 
rash on the forehead. His face and forehead 
were covered with thickly wrinkled, 
hyperpigmented skin. The patient's urine was 
reddish orange. 



Dr. Khalid Yusuf El-Zohry - Sohag Teaching Hospital (01118391123) 

Ref MRCPass OE OE 2012 PasTest 2009 PassMedicine 2009 PasTest Exam 




ReviseMRCP 

871 






























El-zohry MRCP Questions Bank (Port 1) - 2013 


(For my personal use) 


What is the likely diagnosis? 

1- Porphyria cutanea tarda 

2- Pemphigoid 

3- Contact dermatitis 

4- Pityriasis rosea 

5- Epidermolysis bullosa 


Answer & Comments 

Answer: 1- Porphyria cutanea tarda 

In porphyria cutanea tarda, the urine 
fluoresces pink to red. 



Porphyria Cutanea Tarda 


Porphyria cutanea tarda's onset is typically 
during the 

fourth or fifth decade of life. The disease 
tends to develop, recur, or worsen during the 
spring and summer, when exposure to 
sunlight is greatest (ie photosensitivity). 
Though blisters are the most common skin 
manifestations of PCT, other skin 
manifestations like hyperpigmentation and 
hypertrichosis (mainly on top of the cheeks) 
also occur. 

The most common photocutaneous 
manifestations of porphyria cutanea tarda are 
due to increased mechanical fragility after 
sunlight exposure; erosions and blisters form 
painful indolent sores that heal with milia 
(cysts), dyspigmentation, and scarring. The 
deficient enzyme in porphyria cutanea tarda is 
uroporphyrinogen decarboxylase. 


[ Q: 2186 ] MRCPass - 2011 May 

A 31-year old woman has a history of 
3 previous pregnancies all of which ended in 
spontaneous abortion between 18 and 24 
weeks. The last pregnancy was complicated by 
episodes of purpura and thrombocytopenia 
with platelet counts of around 20 x 10 9 /l. A 
diagnosis of systemic lupus erythematosus 
(SLE) has been diagnosed on the basis of a 
positive anti-nuclear factor and anti ds DNA 
binding of 70%. Her symptoms were 
occasional attacks of arthralgia and rash. She 
is being seen in clinic for evaluation. 

Which one of the following antibody tests is 
most appropriate for diagnosing the cause of 
her abortions? 

1- Anti La 

2- Anti Ro 

3- Anti cardiolipin 

4- Anti Scl-70 

5- Anti centromere 



Answer & Comments 

Answer: 3- Anti cardiolipin 

Cardiolipin is a phospholipid to which the 
Anticardiolipin reacts with. 

The lupus anticoagulant is an immunoglobulin, 
IgG or IgM, which also binds to phospholipids. 


Dr. Khalid Yusuf El-Zohry - Sohag Teaching Hospital (01118391123) 

Ref MRCPass OE OE 2012 PasTest 2009 PassMedicine 2009 PasTest Exam ReviseMRCP 



























El-zohry MRCP Questions Bank (Port 1) - 2013 


(For my personal use) 


Patients with systemic lupus erythematosus 
(SLE) are more likely to develop a lupus 
anticoagulant than the general population as 
in the case above. 

Both lupus anticoagulant and anticardiolipin 
antibody are associated with each of the 
clinical manifestations of the antiphospholipid 
syndrome, which is associated with arterial 
and venous thrombosis, and recurrent 
spontaneous abortions. 


There was also pain and temperature 
sensation loss in the left thigh area. 

What is the most likely diagnosis? 

1- Motor neuron disease 

2- Multiple sclerosis 

3- Friedrich's ataxia 

4- Poliomyelitis 

5- Spinal meningioma 


[ Q: 2187 ] MRCPass - 2011 May 

A 40 year old severe asthmatic is 
assessed for hip pain. He takes salbutamol and 
atrovent inhalers, and has been on multiple 
courses of high dose prednisolone. He is 
complaining of hip pain for 8 weeks and is 
unable to weight bear. He gives no history of 
trauma. 

What is the most likely diagnosis? 

1- Osteomyelitis 

2- Osteoporosis with fracture 

3- Avascular necrosis 

4- Sickle cell crisis 



Answer & Comments 

Answer: 5- Spinal meningioma 

Brow n-Sequard syndrome is an incomplete 
spinal cord lesion characterized by a clinical 
picture reflecting hemisection of the spinal 
cord. 

The clinical features are: 

-Unilateral upper motor neuron weakness 
below the level of the lesion 

-Ipsilateral loss of tactile discrimination, 
vibratory, and position sensation below the 
level of the lesion 


5- Osteoarthritis 


Answer & Comments 

Answer: 3- Avascular necrosis 

A painful limb for no apparent reason would 
fit avascular necrosis, which is predisposed to 
by high dose steroids. 

[ Q: 2188 ] MRCPass - 2011 May 

A 45-year-old woman has presented 
with weakness in right lower limb that had 
gradually progressed over the previous 8 
months. 

She has had difficulty in walking for the last 2 
months. On examination, she has increased 
tone, weakness of knee and ankle flexion and 
extension, and upgoing plantar on the right. 



-Contralateral loss of pain and temperature 
sensation. 



[ Q: 2189 ] MRCPass - 2011 May 

A 75 year old man presents with a 
sw elling over the right side of the temple. The 
lesion was noticed 4 years ago and was little 
changed. On examination it was smooth, shiny 
and non pigmented. 


What is the probable diagnosis? 

1- Trophic ulcer 


2- Basal cell carcinoma 


3- Lupus vulgaris 

4- Sebaceous cyst 

5- Squamous cell carcinoma 


Dr. Khalid Yusuf El-Zohry - Sohag Teaching Hospital (01118391123) 

Ref MRCPass OE OE 2012 PasTest 2009 PassMedicine 2009 PasTest Exam 




ReviseMRCP 

873 

























El-zohry MRCP Questions Bank (Port 1) - 2013 


(For my personal use) 


Answer & Comments 

Answer: 2- Basal cell carcinoma 

Basal cell carcinomas can have many different 
appearances: a red patch or irritated area; a 
smooth, shiny and waxy looking bump; a 
white or yellow scar-like area; a smooth 
reddish grow th; or an open sore that won't 
heal, bleeds or oozes. 


They are slow growing as described in this 
case, and rarely metastasise. 



Basal Cell Carcinoma 


^ [ Q: 2190 ] MRCPass - 2011 May 

fi - 

# A 35-year-old woman had initially 
presented with cold hands. Her complaint 
usually occurred on cold days or in cold water, 
initially presenting as pale and cold digits 
which progressed to painful, purple finger tips. 
In the last 5 years, she complained of 
dysphagia and heartburn. The patient has a 
history of hypertension. 

On physical examination, her blood pressure 
was 150/90 mmHg, and her heart rate was 90 
/min. Skin hardness and thickening were 
palpated at the fingers and toes. X rays of 
bilateral hand and foot showed cutaneous 
calcifications, especially near the fingertips. 

Which one of the following tests is likely to be 
correlated with the diagnosis? 

1- ANA 

2- DsDNA 

3- Anti centromere antibody 


4- Anti smooth muscle antibody 

5- Anti Ul ribonucleoprotein (RNP) 

Answer & Comments 

Answer: 3- Anti centromere antibody 

The limited cutaneous form of systemic 
scleroderma (IcSSc) is often referred to as 
CREST syndrome. 

"CREST" is an acronym for the five main 
features: Calcinosis, Raynaud's syndrome, 
Esophageal dysmotility, Sclerodactyly, 
Telangiectasia. The anti-centromere antibody 
is more specific for the CREST syndrome than 
diffuse progressive systemic sclerosis 
(scleroderma). 

The Anti Ul ribonucleoprotein (RNP) is a 
marker for mixed connective tissue disease 
(MCTD) which is characterized by the 
presence of high titers of a distinct 
autoantibody in combination with clinical 
features commonly seen in systemic lupus 
erythematosus (SLE), scleroderma, and 
polymyositis (referred to as overlap 

syndrome) 

[ Q: 2191 ] MRCPass - 2011 May 

A 19 year old lady had a 5 kg weight 
loss recently which made her family 
concerned about her. She moved from school 
to college recently and had difficulty coping . 
She was having amenorrhoea for the last 3 
months. 

Which one finding is likely to be present? 

1- Acanthosis nigricans 

2- Fine hair on the face & body 

3- Hirsutism 

4- Increased LH/FSH ratio 

5- Hyperthermia 



Dr. Khalid Yusuf El-Zohry - Sohag Teaching Hospital (01118391123) 

Ref MRCPass OE OE 2012 PasTest 2009 PassMedicine 2009 PasTest Exam ReviseMRCP 



874 



























El-zohry MRCP Questions Bank (Part 1) - 2013 


(For my personal use) 


Answer & Comments 

Answer: 2- Fine hair on the face & body 

The features are suggestive of Anorexia 
nervosa. 

Possible physical and biochemical features 
are: 

emaciation 

■ hair changes e.g. lanugo; hair loss 
from the scalp 

■ amenorrhoea 
gonadotrophins - low FSH, LH 


few months and presents to the neurology 
clinic for review . Otherwise she is only on oral 
contraceptive pill. 

Which of the following is the best alternative? 

1- Carbamazepine 

2- Phenytoin 

3- Topiramate 

4- Lamotrigine 

5- No treatment 

Answer & Comments 

Answer: 1- Carbamazepine 


T3 - low ; T4, TSH - normal 


[ Q: 2192 ] MRCPass - 2011 May 

A 19-year-old female was admitted 
after severe paracetamol overdose. She was 
treated with IV N-acetylcysteine. 

N-acetylcysteine acts by replenishing which 
compound? 

1- Arginine 

2- Cysteine 

3- Lysine 

4- Glutathione 

5- Methionine 



Answer & Comments 

Answer: 4- Glutathione 

Treatment of paracetamol overdose with N- 
acetylcysteine is well established. 

The effect is to enhance glutathione stores 
and to promote the elimination of 
paracetamol metabolites. 


[ Q: 2193 ] MRCPass - 2011 May 

A 21 year old female has been on 
sodium valproate 200mg od for epilepsy. She 
noticed weight gain and ataxia over the past 



Sodium valproate is the drug of first choice for 
primary generalized epilepsy, valproate has a 
very wide spectrum and is effective in most 
seizure types, including myoclonic seizures. 

It has multiple mechanisms of anticonvulsant 
effects, including increasing gamma- 
aminobutyric acid (GABA) levels in brain as 
well as T-type calcium channel activity.This 
older antiepileptic drug is used as a second- 
choice agent along with phenytoin, but 
phenytoin is not favoured in the long term 
due to side effects of osteopenia and 
cerebellar ataxia. 

Both sodium valproate and carbamazepine are 
older generation antiepileptics. If both of 
these have been tried and contraindicated 
due to side effects, then newer generation 
antiepileptics such as topirimate and 
lamotrigine should then be used. 


^ [ Q: 2194 ] MRCPass - 2011 May 

f* - 

# An 18 year old man who has had 
acne for 1 year has been on minocycline 
orally. The case has been getting worse 
despite the drug. He currently still has 
pustules and scarring on the face. 

Which treatment should be started next? 

1- Oxytetracycline 

2- Isotretinoin 


Dr. Khalid Yusuf El-Zohry - Sohag Teaching Hospital (01118391123) 

Ref MRCPass OE OE 2012 PasTest 2009 PassMedicine 2009 PasTest Exam 




ReviseMRCP 

875 































El-zohry MRCP Questions Bank (Port 1) - 2013 


(For my personal use) 


3- UV light 

4- Prednisolone 

5- Dithranol 


Answer & Comments 

Answer: 2- Isotretinoin 

Patients who are on long term tetracyclines 
and not responding should also have 
isotretinoin tablets.Oral isotretinoin is 
marketed under various trade names, the 
most common ones being Roaccutane. 

Isotretinoin is used only after other acne 
treatments fail to produce results. Treatment 
of acne usually begins with topical 
medications (e.g., benzoyl peroxide), followed 
by oral antibiotics (or a combination) and 
finally isotretinoin therapy, because other 
treatments, while less effective than 
isotretinoin, are thought to be associated with 
fewer adverse effects and lower cost. 


Idiopathic Parkinson's disease is characterised 
by tremor, rigidity and bradykinesia (which is 
typically asymmetrical). 

There are also features of postural instability, 
a mask like face and a shuffling gait. 

Autonomic dysfunction is often associated 
(mild postural hypotension) 



[ Q: 2196 ] MRCPass - 2011 May 

A 50 year old patient with 
Parkinson's disease has been established on 
ropinirole. 


Whot is its mechanism of action? 


1- Dopamine agonist 

2- Monoamine oxidase inhibitor 


3- Antiviral drug 

4- Catechol - o - methyl transferase inhibitor 

5- Anticholinergic drug 


[ Q: 2195 ] MRCPass - 2011 May 

A 60 year old woman presented with 
a 10-year history of increasing stiffness and 
immobility which have led to multiple falls. On 
examination, she had a lying BP of 130 / 80 
mmHg and a standing BP of 125 / 70 mmHg. 
She had a mask like facies, bradykinesia, and 
resting a tremor in both of her arms. The 
tremors and rigidity was worse on the right 
side. There was also short-term memory loss. 



Answer & Comments 

Answer: 1- Dopamine agonist 

--careldopa (contains L dopa) is often the first 
medication used to increase dopaminergic 
activity in the basal ganglia. 

-Benztropine is an anticholinergic drug, used 
typically when there are tremors. 

-Entecapone is a Catechol - o - methyl 
transferase inhibitor. 


What is the diagnosis? 
1- Lewy body dementia 


-Selegiline is an MAO inhibitor (potentiates 
dopamine) 


2- Alzheimer's disease 

3- Parkinson's disease 

4- Multi system atrophy 

5- Progressive supranuclear palsy 


-Amantadine is an antiviral drug (also 
potentiates dopamine) used as a second line 
drug. 

-Apomorphine is a dopamine agonist used for 
on-off fluctuations. 


Answer & Comments 


Answer: 3- Parkinson's disease 


-Ropinirole is a dopamine agonist (D2,D3, D4) 
which is used in conjunction or in place with 
levodopa. 


Dr. Khalid Yusuf El-Zohry - Sohag Teaching Hospital (01118391123) 

Ref MRCPass OE OE 2012 PasTest 2009 PassMedicine 2009 PasTest Exam ReviseMRCP 



876 




























El-zohry MRCP Questions Bank (Port 1) - 2013 


(For my personal use) 


[ Q: 2197 ] MRCPass - 2011 May 

Which one of the following is a 
marker of bod prognosis in acute 
lymphoblastic leukaemia? 

1- Pre-B phenotype 

2- Age of < 20 years 

3- Initial white cell count of 18 x 10 9 /l 

4- Female sex 

5- BCR-Abl gene 



Answer & Comments 

Answer: 5- BCR-Abl gene 

Acute lymphoblastic leukaemia (ALL) is most 
common in childhood with a peak incidence at 
4-5 years of age, and another peak in old age. 

Some prognostic factors are: 


inhibitors (such as imatinib and sunitinib) are 
important drugs against a variety of cancers 
including in CML, and sometimes in Ph- 
positive acute lymphoblastic leukemia 
(Ph+ALL) 

[ Q: 2198 ] MRCPass - 2011 May 

A 54-year-old woman presents with 
periods of sweats and tremors which are 
relieved by eating. She has gained 
approximately 6 kg in weight in the last 2 
years. Her BM is 4.5. Blood tests are: Hb 13 
g/dl, MCV 78 fl, WCC 7 x 10 9 /l, platelets 200 x 
10 9 /l, sodium 135 mmol/l, potassium 4.7 
mmol/I, urea 5 mmol/l, creatinine 100 pmol/l, 
TSH - 3.3 (0.3-4) mU/l, free T4 -20 (10-24) 
pmol/l. 

What is the most appropriate investigation? 

1- 72 hour fast 



Sex: females tend to fare better than males. 


2- CT scan of pancreas 


Age at diagnosis: children between 1-10 years 
of age are most likely to develop ALL and to be 
cured of it. 

Cytogenetics: Philadelphia translocation, 

t(9;22) is a bad prognostic factor. (Philadelphia 
translocation, t(9;22) - good prognosis in CML, 
poor prognosis in AML + ALL) The exact 
chromosomal defect in Philadelphia 
chromosome is a translocation. Parts of two 
chromosomes, 9 and 22, sw ap places. The 
result is that a fusion gene is created by 
juxtapositioning the Abll gene on 
chromosome 9 (region q34) to a part of the 
BCR ("breakpoint cluster region") gene on 
chromosome 22 (region qll). The result of the 
translocation is the oncogenic BCR-ABL gene 
fusion. Because the Abl gene expresses a 
membrane-associated protein, a tyrosine 
kinase, the BCR-Abl transcript is also 
translated into a tyrosine kinase, adding a 
phosphate group to tyrosine. Although the 
BCR region also expresses serine/threonine 
kinases, the tyrosine kinase function is very 
relevant for drug therapy. Tyrosine kinase 


3- MRI of the brain 

4- Insulin C-peptide concentration 

5- Oral glucose tolerance test 

Answer & Comments 

Answer: 1- 72 hour fast 

This patient has symptoms suggestive of 
hypoglycaemia which are relieved by 
carbohydrate. 

The likely cause is an insulinoma which is an 
insulin secreting pancreatic tumour. 

The best way of confirming the diagnosis is 
with a 72 hour fast. During the fast, the 
patient with an insulinoma may get episodes 
of hypoglycaemia with measured 
inappropriately high insulin C peptide 
(endogenous insulin). 

Measurement of C-peptide is useful in 
excluding factitious hypoglycaemia from self 
injection of insulin. Insulin preparations do not 
contain C-peptide. 


Dr. Khalid Yusuf El-Zohry - Sohag Teaching Hospital (01118391123) 

Ref MRCPass OE OE 2012 PasTest 2009 PassMedicine 2009 PasTest Exam 




ReviseMRCP 

877 























El-zohry MRCP Questions Bank (Port 1) - 2013 


(For my personal use) 



[ Q: 2199 ] MRCPass - 2011 May 

A 65-year-old female is brought to 
A&E by her family, who are concerned about 
her increasing lethargy and confusion over the 
past 3 days. There is a history of diarrhea in 
the preceding few days. On examination she is 
found to be pyrexial at 38°C. Breath sounds 
are clear and there is mild tenderness in the 
lower abdomen. 


Blood tests results show : 

Hb 8.6 g/dl 

WCC 12 x 10 9 /l 

Platelets 65 x 10 9 /l 

sodium 138 mmol/I 

potassium 4.7 mmol/I 

Urea 22.1 mmol/l 

Creatinine 366 mmol/l 

Whot is the likely causative organism? 

1- Staphylococcus aureus 

2- Neisseria Meningitidis 

3- Legionella 

4- Leptospira 

5- E coli 0157 


Answer & Comments 

Answer: 5- E coli 0157 

The patient has haemolytic uraemic 
syndrome. 

It is characterized by the triad of 
microangiopathic hemolytic anemia, 
thrombocytopenia, and acute renal failure. 
Diarrhea (E coli 0157) and upper respiratory 
infection are the most common precipitating 
factors. The hallmark of HUS in the peripheral 
smear is the presence of schistocytes 
(fragmented, deformed red cell fragments) 
and helmet-shaped RBCs. 


[ Q: 2200 ] MRCPass - 2011 May 

A 45 year old lady, presented with a 
history of pain in the upper abdomen and an 
ultrasound showed gallstones. 



She underwent a laparoscopic 
cholecystectomy. Initially she felt well but 
started to develop frequent diarrhoea. In view 
of the history, what treatment is she likely to 
need? 


1- Cholestyramine 

2- Azathioprine 

3- Bromocriptine 

4- Pancreatin 

5- Amitriptyline 


Answer & Comments 

Answer: 1- Cholestyramine 

The term postcholecystectomy syndrome 
(PCS) describes the presence of symptoms 
after cholecystectomy. 

Two types of problems may occur. The first 
problem is continuously increased bile flow 
into the upper Gl tract, which may contribute 
to esophagitis and gastritis. The second 
consequence is related to the lower Gl tract, 
where diarrhea and colicky lower abdominal 
pain may result which is described in the case 
above. Cholestyramine has been of help for 
patients with diarrhea. It is a bile acid 
sequestrant, which binds bile in the 
gastrointestinal tract to prevent its 
reabsorption. 

It has been used in cases of pruritus due to 
jaundice. Cholestyramine is also used to 
prevent diarrhea in Crohn's disease patients 
who have undergone post-ileal resection. The 
terminal portion of the small bow el (ileum) is 
where bile acids are reabsorbed. 

[ Q: 2201 ] MRCPass - 2011 May 
An 81 year old woman was found to 



Dr. Khalid Yusuf El-Zohry - Sohag Teaching Hospital (01118391123) 

Ref MRCPass OE OE 2012 PasTest 2009 PassMedicine 2009 PasTest Exam ReviseMRCP 





























El-zohry MRCP Questions Bank (Port 1) - 2013 


(For my personal use) 


be anaemic by the GP following complaints of 
feeling generally unwell. She was referred to 
the haematology department, where 
subsequent investigations found that she had 
idiopathic myelofibrosis. 

Whot is the commonest presenting symptom 
for myelofibrosis? 

1- Haemoptysis 

2- Splenomegaly 

3- Diarrhoea 

4- Fatigue 

5- Rectal bleeding 


Answer & Comments 

Answer: 4- Fatigue 

The most common symptoms at presentation 
for myelofibrosis include fatigue, fever, 
bruising, and shortness of breath. 

Splenomegaly is associated, but not the most 
common presentation. 


^ [ Q: 2202 ] MRCPass-2011 May 

/ - 

* A 42 year old woman presents with 

confusion, headache and neck stiffness. She 
has no previous past medical history of note. 
When she arrived at the admissions unit she 
was witnessed to have a generalised seizure 
which spontaneously resolved. She has a 
temperature of 38.5 C. A lumbar puncture was 
performed. Results showed: 

CSF pressure: 12 cm 

glucose - 3.7 mmol/l 

protein < 0.55 g/l 

white cells 290 (95% lymphocytes) 

An MRI scan showed high signal in the 
temporal lobes including hippocampal 
formations and parahippogampal gyrae and 
right inferior frontal gyrus. 

Whot is the likely diagnosis? 

1- Pneumococcal meningitis 


2- Guillain Barre syndrome 

3- TB meningitis 

4- Poliomyelitis 

5- Herpes simplex virus encephalitis 


Answer & Comments 

Answer: 5- Herpes simplex virus encephalitis 

In Herpes simplex virus (HSV) encephalitis, a 
presentation with fevers, confusion or a 
change in personality is common. 

The CSF white cell count is elevated with 
lymphocytosis. The majority of cases of herpes 
encephalitis are caused by herpes simplex 
virus-1 (HSV-1). The MRI typically shows high 
signal changes in the T2 weighted images in 
the temporal lobe areas, in HSV encephalitis. 
Treatment is with iv acyclovir. 



HSV encephalitis 



[ Q: 2203 ] MRCPass - 2011 May 

A 60 year old lady presents with 
blurred vision acutely. Further questioning 
revealed that she had episodes of slurred 
speech suggesting that she had 3 episodes of 
transient ischaemic attacks in the last 2 weeks. 
She has a history of hypertension. An ECG 
showed sinus rhythm. Carotid dopplers 
showed: right sided carotid artery normal, left 
sided 50% occlusion. 


Dr. Kholid Yusuf El-Zohry - Sohog Teaching Hospital (01118391123) 

Ref MRCPass OE OE 2012 PasTest 2009 PassMedicine 2009 PasTest Exam ReviseMRCP 



























El-zohry MRCP Questions Bank (Port 1) - 2013 


(For my personal use) 


What is the best monogement? 

1- Right sided carotid endarterectomy 

2- Left sided carotid endarterectomy 

3- Bilateral carotid endarterectomy 

4- Carotid angioplasty 

5- Thrombolysis 


Answer & Comments 

Answer: 2- Left sided carotid endarterectomy 

Indications for carotid endarterectomy are: 
Asymptomatic patients with greater than 70% 
stenosis or symptomatic patients with > 50% 
stenosis. 


A 39-year-old male, was diagnosed 
with seropositive rheumatoid arthritis at the 
age of 20. 

In addition to prednisolone therapy, several 
different disease-modifying drugs were given 
over the following years, including 
sulfasalazine, oral gold, resochine, 
methotrexate, and TNF-inhibitors. He smokes 
15 cigarettes a day. 

H presented to the hospital with shortness of 
breath and intermittent nausea. On clinical 
examination, he showed signs of extensive 
rheumatoid arthritis, most marked on hand, 
foot, and shoulder joints as well as rheumatic 
nodules on both elbow s. His heart rate was 
regular at 105/min, blood pressure was 
120/70 mm Hg. He had a raised JVP of 6 cm 
and a left parasternal heave. He also had a 
palpable liver and moderate ankle oedema. 
His second heart sound (P2) was loud. The 
breath sounds are clear and heart size is 
normal on the chest x ray. 

An echocardiogram revealed normal left 
ventricular systolic function, and both atria 
were dilated. 

What is the most likely diagnosis? 

1- Pulmonary embolus 



[ Q: 2204 ] MRCPass - 2011 May 


2- Pulmonary fibrosis 

3- Constrictive pericarditis 

4- Aortic dissection 

5- Aortic aneurysm 


Answer & Comments 

Answer: 3- Constrictive pericarditis 

Constrictive pericarditis, an extra-articular 
complication in RA patients, is predominantly 
seen in males with active, seropositive 
disease. 

Echocardiography often reveals normal 
ventricular function and atrial dilatation. Heart 
catheterization reveals equalization of 
elevated ventricular diastolic pressures and 
normal systolic function. There may be signs 
of right sided heart failure, as in the above 
case. 



[ Q: 2205 ] MRCPass - 2011 May 

A 17 year old student is behaving 
strangely and referred to the hospital. His 
teacher reports that he was accused the 
teacher of conspiring against him. He was also 
hearing voices asking him to cut his own 
throat. He has not been himself recently, with 
low moods according to his family. On 
examination, he looks apathetic and physical 
examination is normal. Blood tests were 
unremarkable, and urine testing showed 
traces of cannabinoids. 


What is the likely diagnosis? 

1- Psychotic depression 

2- Paranoid schizophrenia 

3- Drug induced psychosis 

4- Anxiety disorder 

5- Obsessive compulsive disorder 


Answer & Comments 


Answer: 3- Drug induced psychosis 


Dr. Khalid Yusuf El-Zohry - Sohag Teaching Hospital (01118391123) 

Ref MRCPass OE OE 2012 PasTest 2009 PassMedicine 2009 PasTest Exam ReviseMRCP 



880 





























El-zohry MRCP Questions Bank (Port 1) - 2013 


(For my personal use) 


It is well established that psychotic symptoms 
may follow cannabis intake. 

Patients who present with these symptoms 
may get better and be diagnosed with 
schizophrenia at a later stage. Patients can 
present with a range of symptoms including 
agitation, depression, visual and auditory 
hallucinations. 


[ Q: 2206 ] MRCPass - 2011 May 

A 55 year homeless man was found 
collapsed at home and brought to the 
emergency department. He was unable to give 
a history. On examination, he smelled of 
alcohol. Observations in A&E reveal a 
temperature of 34 C, a pulse of 45 bpm 
pressure of 110/80 mmHg. Dipstick urine 
analysis shows Blood +++, protein +, glucose - 
ve. 

Some of his investigation results are listed 
below: 

sodium 135 mmol/I 
potassium 4.5 mmol/I 
urea 5 mmol/l 
creatinine 300 |imol/l 
AST 320 (1-31) U/l 
LDH 800 U/L (110-230) U/L 



As patient was found unconscious and 
hypothermic he was likely to have sustained 
muscle injury. The positive urinalysis caused 
by myoglobin a muscle protein released 
during muscle damage. High amounts of 
myoglobin damages the renal tubules which 
then leads to acute kidney injury. 


[ Q: 2207 ] MRCPass - 2011 May 

A 36-year-old woman presented 
with an 8 week history of weight gain, acne, 
hirsutes and weakness. 

Examination confirmed florid Cushingoid 
features, with truncal obesity, striae, buffalo 
hump, acne, hypertension and proximal 
myopathy. Biochemical investigations are 
summarised in the table. Gross 
hypercortisolaemia (unsuppressed by 
dexamethasone), hyperglycaemia and 
hypokalaemia suggested an ectopic ACTH 
syndrome. 

Computed tomographic (CT) imaging showed 
normal pituitary and hypothalamus, mild 
bilateral adrenal hyperplasia. 

What is the most likely couse? 

1- Pituitary adenoma 

2- Adrenal adenoma 

3- Small cell carcinoma 



Whot likely couse of the raised serum 
creatinine concentration? 

1- Chronic renal failure 

2- Dehydration 

3- Hypothermia 

4- Paracetamol poisoning 

5- Rhabdomyolysis 

Answer & Comments 

Answer: 5- Rhabdomyolysis 

The elevated serum creatinine likely to be due 
to rhabdomyolsis. 


4- Bronchial carcinoid 

5- Colon carcinoma 

Answer & Comments 

Answer: 3- Small cell carcinoma 

The presentation of ectopic ACTH syndrome is 
usually with cushingoid states such as weight 
gain, oedema, diabetes and proximal muscle 
weakness. 

Ectopic ACTH (not from the pituitary) is usually 
associated with a small cell carcinoma of the 
bronchus. Less common causes include thymic 
tumours, pancreatic adenocarcinoma and 
bronchial carcinoid 


Dr. Khalid Yusuf El-Zohry - Sohag Teaching Hospital (01118391123) 

Ref MRCPass OE OE 2012 PasTest 2009 PassMedicine 2009 PasTest Exam ReviseMRCP 



881 

























El-zohry MRCP Questions Bank (Port 1) - 2013 


(For my personal use) 


^ [ Q: 2208 ] MRCPass - 2011 May 

/ ■ —- *■ - ■ - 

# A 76 year old lady develops diarrhea 

following a course of antibiotics. 

Which of the following antibiotics is most 
commonly associated with pseudo¬ 
membranous colitis? 

1- Quinolones 

2- Cephalosporins 

3- Macrolides 


Answer & Comments 

Answer: 2- Check compliance of patient 

This is a new ly diagnosed patient who was 
started on thyroxine. 

Levels show a low thyroxine level and TSH 
remains high. This suggests undertreatment 
but compliance of the treatment should be 
evaluated before increasing the dose of 
thyroxine. 


4- Folate antagonists 

5- Aminoglycosides 


Answer & Comments 

Answer: 2- Cephalosporins 

Penicillins, cephalosporins (cefuroxime) and 
clindamycin are most commonly associated 
with pseudomembranous colitis. 


[ Q: 2209 ] MRCPass - 2011 May 

A 24 year old lady was complaining 
of tremors and sw eating for 6 weeks and 
referred for assessment. 

She has lost 6 kg in weight. A nuclear medicine 
scan of the neck showed increased and patchy 
uptake of radio isotope of the thyroid glands. 
Following tests, she is diagnosed with 
thyrotoxicosis and she was started on 
thyroxine replacement. 2 months later, she 
had blood tests which showed the following 
results: free T4- 11 (10-24) pmol/l, TSH 8 (0.3- 
4) mU/l. 

What should be done next? 

1- T3 replacement 

2- Check compliance of patient 

3- Check thyroid hormone resistance 

4- Check pituitary hormone profile 

5- Repeat radioisotope scan 



[ Q: 2210 ] MRCPass - 2011 May 

A 71 year old man who is a heavy 
smoker has been brought into hospital, with a 
history of cough and breathlessness. He has a 
history of COPD which has been managed with 
home nebulisers but not oxygen. He was given 
60% oxygen by the ambulance crew . He 
appears confused when he was brought into 
hospital. GCS is 14/15. An arterial blood gas 
shows: 

pH-7.21 (7.36-7.44) 
p02 -18 (11.0-13.5) kPa 
pC02 -10 (4.6-6.0) kPa 
What should be done? 

1- Stop oxygen 

2- Give 24% oxygen 

3- Continue 60% oxygen 

4- Non invasive ventilation 

5- Intubation and ventilation 



Answer & Comments 

Answer: 2- Give 24% oxygen 

The best answers here are either give 24% 
oxygen or non invasive ventilation. 

If the patient is not too unwell and is alert, it is 
safe to turn down the oxygen to 24% (1 L) and 
assess the degree of hypoxia and C02 
retention. With steroids and salbutamol 
nebulisers, the patient may improve and not 
require NIV. A 02 of 18 is too high for a 


Dr. Khalid Yusuf El-Zohry - Sohag Teaching Hospital (01118391123) 

Ref MRCPass OE OE 2012 PasTest 2009 PassMedicine 2009 PasTest Exam ReviseMRCP 






























El-zohry MRCP Questions Bank (Part 1) - 2013 


(For my personal use) 


patient with COPD, and when patients are 
admitted oxygen therapy should be delivered 
to achieve a 02 of about 8-10 on the blood 
gas. 


[ Q: 2211 ] MRCPass - 2011 May 

A set of parents comes to seek 
genetic advice at the clinic. They said that 
their 5-year-old boy has cystic fibrosis but they 
themselves do not have the disease. They also 
have a daughter who is 17 years old but not 
affected by the disease. 

What is the chance that she will be a carrier of 
the cystic fibrosis gene? 

1- 1 in 2 

2- 1 in 4 

3- 2 in 3 

4- 1 in 25 

5- 100% chance 



Answer & Comments 

Answer: 3- 2 in 3 

Inheritance of cystic fibrosis is autosomal 
recessive. 

In answering this question, the simple mistake 
is to take carriers out of total, which makes a 1 
in 2 chance. The diagram below illustrates the 
best way of working this out. 

As the sister is not affected, there are 3 other 
options, so she might be a carrier in 2 of the 3 
scenarios (2 in 3). 




Unaffected 



CF carrier 


* 

CF sufferer 



[ Q: 2212 ] MRCPass - 2011 May 

A 31 year old female patient seeks 
medical help for infertility and has 
subsequently been diagnosed as having 
polycystic ovarian disease on ultrasound. 


Which one of the features is likely to be 
present? 


1- Low androgen levels 


2- Weight loss 

3- Alopecia 

4- Decreased visual acuity 


5- Increased insulin resistance 


Answer & Comments 

Answer: 5- Increased insulin resistance 

Common symptoms of Polycystic ovary 
syndrome (PCOS) include: 

Menstrual disorders - oligomenorrhea or 
amenorrhea 

Infertility,-generally resulting from chronic 
anovulation 

Hirsutism and symptoms of 
hyperandrogenism, such as acne 

Metabolic syndrome- characterised by central 
obesity, insulin resistance and other 
symptoms. 

Metformin is being used increasingly in 
polycystic ovary syndrome (PCOS) and non¬ 
alcoholic steatohepatitis, two diseases that 
feature insulin resistance. 

Metformin improves insulin sensitivity by 
increasing peripheral glucose uptake and 
utilization. 



[ Q: 2213 ] MRCPass - 2011 May 

A 26-year-old man presented with 
bi- temporal hemianopia. He mentioned that 
his shoe sizes were above that of his friends 


Dr. Khalid Yusuf El-Zohry - Sohag Teaching Hospital (01118391123) 

Ref MRCPass OE OE 2012 PasTest 2009 PassMedicine 2009 PasTest Exam ReviseMRCP 



883 
































El-zohry MRCP Questions Bank (Port 1) - 2013 


(For my personal use) 


since childhood and he often had sw eaty 
episodes. 

Which one of the following tests is likely to 
confirm the diagnosis of acromegaly? 

1- Random growth hormone 

2- IGF-1 

3- Glucose tolerance test with growth 
hormone suppression 

4- Synacthen test 

5- MRI pituitary 


Answer & Comments 

Answer: 3- Glucose tolerance test with growth 
hormone suppression 

In Acromegaly, there is excess Growth 
hormone (GH) which is difficult to suppress. 

Because GH secretion is inhibited by glucose, 
measurement of glucose non-suppressibility is 
useful. In the glucose tolerance test, baseline 
GH levels are obtained prior to ingestion of 
100 g of oral glucose, and additional GH 
measurements are made at 30, 60, 90, and 
120 minutes following the oral glucose load. 
Patients with active acromegaly are unable to 
suppress GH concentration below 2 ng/mL. 

Random GH measurements are often not 
diagnostic because of the episodic secretion of 
GH, but IGF-I has a long half-life, and is useful 
as a screen for Acromegaly. MRI may reveal a 
pituitary tumour but it would not be specific 
for Acromegaly. 



[ Q: 2214 ] MRCPass - 2011 May 

A 76-year-old male presented with a 
4-year history of mild cognitive decline. He has 
a 10-year history of hypertension and type 2 
diabetes. According to his family, he had 
become more forgetful, yet he was able to 
carry out simple tasks independently.. His 
short-term memory was impaired, as was his 
ability to concentrate. His gait was slow and 
he was unsteadyHe leaned backward when he 


walked and fell often, especially when trying 
to turn to the left or right. He experienced 
urinary frequency, nocturia, and urinary 
incontinence at least once a 

day. 

What is the most likely diagnosis? 

1- Alzheimer's disease 

2- Transient ischaemic attack 

3- Lewy body dementia 

4- Pick's disease 

5- Normal pressure hydrocephalus 


Answer & Comments 

Answer: 5- Normal pressure hydrocephalus 

Normal pressure hydrocephalus (NPH) is a 
clinical symptom complex characterized by 
abnormal gait, urinary incontinence, and 
dementia. 

The CT scan often shows evidence of 
hydrocephalus (distended ventricles), but the 
CSF pressure is normal on lumbar puncture. 
The treatment is to remove CSF by lumbar 
puncture (normally 50 mis). 


[ Q: 2215 ] MRCPass - 2011 May 

A 25 year old man is investigated for 
infertility. He also had a history of delayed 
pubertal development. Physical examination 
revealed a slim tall man. There was testicular 
atrophy bilaterally and also gynaecomastia. 
There was also sparse body hair. 

What test is likely to reveal the diagnosis? 

1- Polymerase chain reaction 

2- Southern Blotting 

3- Chromosomal analysis 

4- ELISA 

5- VDRL 



Dr. Khalid Yusuf El-Zohry - Sohag Teaching Hospital (01118391123) 

Ref MRCPass OE OE 2012 PasTest 2009 PassMedicine 2009 PasTest Exam ReviseMRCP 



884 


























El-zohry MRCP Questions Bank (Port 1) - 2013 


(For my personal use) 


Answer & Comments 

Answer: 3- Chromosomal analysis 

Klinefelter's syndrome (XXY) causes testicular 
atrophy, which commonly leads to 
gynecomastia and infertility. 

Diagnosis is by chromosomal analysis (shown 
below). 

Androgen deficiency causes eunuchoid body 
proportions; 

sparse or absent, facial, axillary, pubic, or 
body hair; decreased muscle mass and 
strength; feminine distribution of adipose 
tissue; gynecomastia; small testes and penis. 

Androgen therapy is the most important 
aspect of treatment. Testosterone 
replacement should begin at puberty to 
correct androgen deficiency 



4 i 

H 

II 

P m 

4? ■ 

il 

ir ir 

* 4 ft | 

11 

S 1 

% 7 

f 1 

1 i 

a 

7 

ft 

4 1U 

11 

17 

u 

|t 

ft 4 

|| 

VI 

it 

n 

14 


1§ 

fl 

11 

* * 

11 

■ * 


41 f| 

- ft * 
111 


t* 

JC 


71 27 

X ¥ 



[ Q: 2216 ] MRCPass - 2011 May 

A 65-year-old woman presented 
with a history of painful, red left eye for 3 
months. The patient's past medical history 
includes rheumatoid arthritis, atrial fibrillation 
and diabetes. She has arthritis involving both 
the knee joints and hands but symptoms are 
well controlled at present. 

On examination, visual acuities were 6/9 on 
the right and 6/60 on the left eye. Slit lamp 
examination of the right eye showed a 
sectoral nodular changes superotemporally on 



the left. There were trace cells in the anterior 
chambers of both the eyes. Dilated fundus 
examination of both the eyes showed clear 
vitreous, cup disc ratio of 0.3 bilaterally and 
no retinal lesions were noted. 

What is the diagnosis? 

1- Scleritis 

2- Conjunctivitis 

3- Acute closed angle glaucoma 

4- Uveitis 

5- Iritis 


Answer & Comments 

Answer: 1- Scleritis 

The two principal eye manifestations in 
rheumatoid arthritis are episcleritis, which is 
usually mild and transient, and scleritis, which 
involves the deeper layers of the eye and is a 
more serious inflammatory condition. 

Rheumatoid Scleritis is most common in the 
sixth decade of life, affects women more 
frequently than men, and is often bilateral. 

Although it maybe an initial sign of 
rheumatoid disease it typically presents many 
years after the onset of RA and often at the 
time when joint inflammation is in remission. 
Scleritis typically causes redness, pain (unlike 
episcleritis) and loss of vision. 



Scleritis 



[ Q: 2217 ] MRCPass - 2011 May 
A 56 year man complained of tremor 


Dr. Khalid Yusuf El-Zohry - Sohag Teaching Hospital (01118391123) 

Ref MRCPass OE OE 2012 PasTest 2009 PassMedicine 2009 PasTest Exam ReviseMRCP 




























El-zohry MRCP Questions Bank (Port 1) - 2013 


(For my personal use) 


which was more severe on right. The tremor 
persisted while he was moving. There was 
mild head nodding. On examination, cranial 
nerves were normal. He has normal tone, 
power and reflexes in the upper and lower 
limbs. 

Whot is the treatment of choice? 

1- Tetrabenazine 

2- Benzhexol 

3- Co-careldopa 

4- Propranolol 

5- Ropinorole 

Answer & Comments 

Answer: 4- Propranolol 

The features of Benign essential tremor are: 

aggravation by posture and movement, relief 
by rest and alcohol (improvement lasting 
about two hours), most affected are the arms, 
head and vocal apparatus. 


oedema. There was tenderness over the 
upper lumbar region. 

Blood tests showed: 

Hb 10.5 g/dl, WCC 7 x 10 9 /l, platelets 220 x 
10 9 /l, sodium 135 mmol/I, potassium 4.2 
mmol/l, urea 16 mmol/l, creatinine 220 
|imol/l, ALT 22 (5-35) U/l, Bilirubin 13(1-22) 
|imol/l, Albumin 32 (37-49) g/l, calcium 2.8 
(2.25-2.7) mmol/l, phosphate 0.70 (0.8-8) 
pmol/l. 

Routine urine examination showed urine 
albumin trace, urine protein/creatinine ratio 
2.7 and urinary Bence Jones protein was 
positive. 

What test should be carried out to confirm the 
diagnosis? 

1- Bone marrow aspiration 

2- Plasma electrophoresis 

3- MRI of the spine 

4- Renal ultrasound 

5- Blood film 


There is often no rigidity, no cerebellar signs, 
it is a familial condition. 

Management options include: Blockers - 
including propranolol, atenolol and sotalol 

primidone 

topiramate 

botulinum A toxin-haemagglutinin complex 

[ Q: 2218 ] MRCPass - 2011 May 

A 57 years old woman was admitted 
with complaints of anorexia, nausea and 
episodic vomiting for last 2 months, decreased 
urine output for last 15 days and sw elling of 
feet, face and upper limbs for last 15 days. She 
had history of fall 12 months back after which 
she sustained mild compression fracture of LI 
vertebra. After the fall, she has had persistent 
backaches. On examination, she was pale, BP 
was 160/90 mmHg and bilateral pedal 



Answer & Comments 

Answer: 1- Bone marrow aspiration 

The pathological fractures, renal impairment 
and hypercalcaemia point tow ards multiple 
myeloma. 

The best answers are either plasma 
electrophoresis or bone marrow aspiration. 
Serum electrophoresis is better as a screening 
tool and bone marrow aspirate and biopsy 
samples to calculate the percent of plasma 
cells in the aspirate (reference range, <3%) will 
be the most diagnostic test. 


^ [ Q: 2219 ] MRCPass - 2011 May 

* ^ j Which one of the following enzymes 
is inhibited by alpha 1 antitrypsin? 

1- Neutral alpha glucosidase 

2- Peroxisome catalase 


Dr. Khalid Yusuf El-Zohry - Sohag Teaching Hospital (01118391123) 

Ref MRCPass OE OE 2012 PasTest 2009 PassMedicine 2009 PasTest Exam ReviseMRCP 



886 




























El-zohry MRCP Questions Bank (Port 1) - 2013 


(For my personal use) 


3- Lymphocyte 5 neucleotidase 

4- Lactate dehydrogenase 

5- Neutrophil elastase 


Answer & Comments 


Answer: 5- Neutrophil elastase 


discharged with a follow up appointment to 
reassess in 2-4 weeks. 

A repeat aspiration is recommended if the 
patient is still symptomatic after the first 
aspiration (for a primary pneumothorax) and < 
2.5 I of air was aspirated in the first attempt as 
in this case. 


Alpha 1-antitrypsin (A1AT) is produced in the 
liver, and one of its functions is to protect the 
lungs from the neutrophil elastase enzyme, 
which can disrupt connective tissue. 

Smokers develop increased levels of elastase 
enzymes and thus are more at risk of 
emphysema in alpha 1 antitrypsin deficiency. 



[ Q: 2220 ] MRCPass - 2011 May 

A 44 year old man is referred to the 
hospital by the GP as he had a routine chest x 
ray which was abnormal. The patient does not 
complain of any symptoms. He is a non 
smoker and has no previous medical history of 
respiratory problems. His BP was 110/80 & 
pulse was 80/min. CXR shows a pneumothorax 
with a 1.5 cm diameter rim of air from the 
chest wall. 


Which is the most appropriate step? 

1- Chest drain 


2- Oxygen therapy 

3- Observation and follow up 

4- Needle aspiration 

5- VATs procedure 


Answer & Comments 

Answer: 3- Observation and follow up 

This patient has a asymptomatic primary 
pneumothorax. 


BTS Pneumothorax 2010 Guideline. 


[ Q: 2221 ] MRCPass - 2011 May 

A 26-year-old left-handed woman 
was referred for evaluation of pain in the left 
wrist. 6 months previously, she started to 
develop numbness involving the lateral three 
digits of the left hand and pain in the left 
wrist. There was weakness of thumb 
abduction and mild wasting of the thenar 
muscles. Muscle stretch reflexes were normal. 

Sensation to pinprick was diminished on the 
volar surface of the second and third digits of 
the left hand. On her left side, Phalen's sign 
was present, and Tinel's sign was elicited over 
the median nerve at the wrist. 

What is the likely diagnosis? 

1- Brachial plexus neuropathy 

2- Ulnar nerve neuropathy 

3- Radial nerve neuropathy 

4- Axillary nerve neuropathy 

5- Carpal tunnel syndrome 



Answer & Comments 

Answer: 5- Carpal tunnel syndrome 

Carpal tunnel syndrome occurs when the 
median nerve is compressed at the wrist, 
leading to pain, paresthesia, and muscle 
weakness in the forearm and hand. 


The latest BTS guidelines (2010) continues to 
distinguish between a large and small 
pneumothorax as defined by a rim of > 2cm or 
< 2 cm from the chest wall. For patients who 
are asymptomatic, the patient can be 


There may be loss of sensation to the area of 
the thumb, index, middle, and radial half of 
the ring finger motor wasting and weakness 
lead to thenar wasting and weakened 
abduction of the thumb 


Dr. Khalid Yusuf El-Zohry - Sohag Teaching Hospital (01118391123) 

Ref MRCPass OE OE 2012 PasTest 2009 PassMedicine 2009 PasTest Exam ReviseMRCP 



887 




























El-zohry MRCP Questions Bank (Port 1) - 2013 


(For my personal use) 


Phalen's test is used in carpal tunnel 
syndrome where forcible palmar flexion of the 
wrist causes venous engorgement of the canal 
and an exacerbation of the symptoms. Tinel's 
test is performed by tapping over the carpal 
tunnel, it causes tingling in the thumb and 
radial two and a half fingers. 



Tinel's test 


[ Q: 2222 ] MRCPass - 2011 May 

A 61 year old man is unwell having 
ingested a bottle of dye. On examination, he is 
afebrile but has tachypnea, cyanosis, and 
drow siness. He is given 100% oxygen but does 
not improve. A lab test confirms 
methaemoglobin levels >70% hence 
confirming the diagnosis. In 
methaemoglobinaemia, what is the underlying 
mechanism? 

1- Reductive stress 



2- Oxidation of Fe2+ to Fe3+ 

3- Increase in NADP levels 

4- Increase in NADPH levels 

5- Increase in gluthathione system 


Answer & Comments 

Answer: 2- Oxidation of Fe2+ to Fe3+ 

Methaemoglobinaemia is haemoglobin with 
an oxidised ferric state Fe3+ instead of the 
ferrous state Fe2+. 

It is commonly due to oxidative stress, caused 
by drugs or exogenous substances e.g. 
sulphonamides, trimethoprim or dyes, 
chlorates, bromates, nitrates (fertilisers). It 
causes cyanosis when metHb >1.5 g/dl. 
Chemicals which are oxidising agents may 
cause this e.g. aniline dyes, chlorates, nitrates, 
nitrophenols, primaquine and sulphonamides. 

Treatment is with methylene blue if 
methaemoglobin >3.0g/dL. 

Mechanisms to counteract oxidative stress: 
NADH methemoglobin reductase 

(cytochrome-b5 reductase) (major pathw ay), 
NADPH methemoglobin reductase (minor 
pathw ay), the ascorbic acid and glutathione 
enzyme systems are usually overw helmed. 


[ Q: 2223 ] MRCPass - 2011 May 

A 31 year old man presents with 
fevers, malaise and a cough. There was 
associated lethargy. He is a type 1 diabetic and 
is on insulin. He works in a water purifier 
factory, and legionella infection is suspected. 

Which of the following tests is most practical 
for confirming the diagnosis? 

1- Serum Immuno Fluorescent Antibody 

2- Sputum Immuno Fluorescent Antibody 

3- Sputum microscopy and culture 

4- Urinary antigen 

5- PCR for legionella DNA 



Dr. Khalid Yusuf El-Zohry - Sohag Teaching Hospital (01118391123) 

Ref MRCPass OE OE 2012 PasTest 2009 PassMedicine 2009 PasTest Exam 




ReviseMRCP 


888 


i 




































El-zohry MRCP Questions Bank (Port 1) - 2013 


(For my personal use) 


Answer & Comments 

Answer: 4- Urinary antigen 

The urine antigen test is a rapid, relatively 
inexpensive, and practical test for the 
detection of Legionella 

pneumophila antigen excreted in the urine or 
present in pleural fluid. 

Direct fluorescent antibody (DFA) staining is a 

rapid test that can be performed on 
respiratory samples and tissue and requires 
only 2-4 hours for results. It is 

very specific but not sensitive, hence a 
negative result does not rule out legionella 
infection. 

[ Q: 2224 ] MRCPass - 2011 May 

A 26 year old nurse has known latex 
allergy and currently avoids using latex gloves 
at work. One day after 



This is because some of the proteins in latex 
show cross-reactivity, perhaps 

because of resemblance to a latex protein 
component. 



[ Q: 2225 ] MRCPass - 2011 May 

A 30-year-old man who has returned 
from a holiday in Egypt presents with 
diarrhoea. He had been on a cruise at the Nile 
river. For the past two days he has been 
passing frequent bloody diarrhoea associated 
with crampy abdominal pain. Abdominal 
examination demonstrates diffuse lower 
abdominal tenderness but there is no 
guarding or rigidity. His temperature is 37.8°C. 

What is the most likely causative organism? 


1- Giardiasis 


2- Enterotoxigenic Escherichia coli 

3- Staphylococcus aureus 

4- Shigella 


lunch, she developed itching and flushing to 
her face with difficulty in breathing. She 
mentioned that she only had 

a salad and some fruit. 

Which fruit is most likely to be associated with 
latex allergy? 

1- Banana 

2- Orange 

3- Apple 

4- Pear 


5- Salmonella 

Answer & Comments 

Answer: 4- Shigella 

All are common causes of traveller's 
diarrhoea. 

However, North Africa and the Middle East (in 
particular Egypt) were also commonly 
reported regions of travel for Shigella spp 
infections. 


5- Grape 

Answer & Comments 

Answer: 1- Banana 

Some people with latex allergy have allergic 
reactions when eating particular foods 
including banana, avocado, 

chestnut, tomato, peach or kiw ifruit. 


Some of the infectious causes of bloody 
diarrhoea are: 

■ Salmonella 

■ Shigella 

■ Campylobacter jejuni 

■ Yersinia enterocolitica 

■ E. coli 

Entamoeba histolytica 


Dr. Khalid Yusuf El-Zohry - Sohag Teaching Hospital (01118391123) 

Ref MRCPass OE OE 2012 PasTest 2009 PassMedicine 2009 PasTest Exam ReviseMRCP 































El-zohry MRCP Questions Bank (Port 1) - 2013 


(For my personal use) 


[ Q: 2226 ] MRCPass - 2011 May 

A 25 year old man was admitted to 
the with the complaints of generalised 
weakness of whole body. Upon further 
enquiry, he described easily being tired, 
difficulty with his speech and drooping of both 
the eyelids for the past 6 months. All his 
symptoms are worse on exertion and in the 
evening. He mentioned that in the evenings 
he occasionally developed double vision. He 
does not have any past medical history and 
does not take regular medication. On 
examination he has a BP of 120/70 mmHg and 
02 saturations of 99% on air. He had bilateral 
ptosis, reduced power in all the muscles of the 
four limbs with normal reflexes, bilateral 
flexor planters and normal sensation. 

What is the likely diagnosis? 

1- Motor neuron disease 

2- Multiple sclerosis 

3- Myasthenia gravis 

4- Paraneoplastic syndrome 

5- Parkinson's syndrome 



Answer & Comments 

Answer: 3- Myasthenia gravis 

Myasthenia Gravis is an autoimmune 
neuromuscular disease leading to fluctuating 
muscle weakness and fatiguability. 

It is an autoimmune disorder, in which 
weakness is caused by circulating antibodies 
that block acetylcholine receptors at the 
postsynaptic neuromuscular junction. 


directly improve muscle function and 
immunosuppressant (prednisone, cyclosporin, 
mycophenolate and azathioprine. 

Thymectomy is essential in cases of suspected 
thymoma in view of the potential neoplastic 
effects of the tumor. 

[ Q: 2227 ] MRCPass - 2011 May 

A 76 year-old female was seen in the 
Emergency department with a 2 day history of 
headaches and fever. On examination, the 
patient had a temperature of 38.5 °C. There 
was also evidence of meningism with a 
positive Kernig's sign. Tone, power and 
reflexes were normal apart from general 
weakness. There was no sensory deficit. A 
lumbar puncture was performed. CSF showed 
100 white cells (90% lymphocytes), protein 0.9 
(<0.5) and glucose 3.3, plasma glucose 7.5. 
The patient was commenced on rifampicin, 
isoniazid, pyrazinamide and ethambutol. 

What other drug should be added? 

1- Fusidic acid 

2- Streptomycin 

3- Prednisolone 

4- Linezolid 

5- Rituximab 



Answer & Comments 

Answer: 3- Prednisolone 

In Tuberculous meningitis, Fever, headache, 
confusion and meningism are presenting 
features. 


The classic feature of myasthenia gravis is 
fatiguability. Muscles become progressively 
weaker during periods of activity and improve 
after periods of rest. Muscles that control 
ocular movements, facial expression, chewing, 
talking, and sw allowing are affected. Ptosis 
and diplopia are common presentations. 

Medication consists mainly of cholinesterase 
inhibitors (neostigmine, pyridostigmine) to 


The CSF usually has a high protein, low glucose 
and a raised number of lymphocytes as seen 
in this patient. 

Acid-fast bacilli are sometimes seen on a CSF 
smear, but more commonly, M. tuberculosis is 
grown in culture. 

Quadruple Tuberculous therapy is 
recommended in cases of pulmonary TB, but 


Dr. Khalid Yusuf El-Zohry - Sohag Teaching Hospital (01118391123) 

Ref MRCPass OE OE 2012 PasTest 2009 PassMedicine 2009 PasTest Exam ReviseMRCP 



890 

























El-zohry MRCP Questions Bank (Port 1) - 2013 


(For my personal use) 


for TB meningitis, prednisolone is added and 
treatment is continued for at least 1 year. 


Answer & Comments 


Answer: 2- 50% of all children 



[ Q: 2228 ] MRCPass - 2011 May 

A patient who is on haemodialysis is 
on several medications. 

Which one of the following is the most likely 
reason for a drug to be ineffectively removed 
by dialysis? 

1- High water solubility 

2- High first pass metabolism 

3- High plasma protein binding 

4- High volume of distribution 

5- Low bioavailability 


Answer & Comments 

Answer: 3- High plasma protein binding 

Because the primary binding proteins for most 
drugs (albumin, ?l-acid glycoprotein) are of 
large molecular size, the drug-protein complex 
is often unable to cross the dialysis 
membrane, especially the hemodialysis 
membrane. 

Drugs that are highly protein-bound (e.g., 
phenytoin, valproate) are difficult to remove 
from the systemic circulation by hemodialysis. 

[ Q: 2229 ] MRCPass - 2011 May 

A couple presents to the clinic for 
genetic counselling. The male partner has 
haemophilia whilst the female partner has 
been screened and shown to be a carrier of 
the gene causing haemophilia A. 

What is the chance that a child would have 
haemophilia? 

1- 25% if male child, 0% if female child 

2- 50% of all children 

3- 100% if male child, 0% if female child 

4- 50% if male child, 0% if female child 

5- 25% of all children 



Hemophilia A is X linked recessive, which 
means that females are commonly carriers 
and males are affected if they inherit the 
affected X chromosome. 

In this case, if the affected X chromosome is 
designated Xa, the father is XaY and mother 
XaX. The children would be either XaXa, XaX, 
XaY and XY. In this situation 50% (XaXa and 
XaY) of all children will be affected. It would 
also be 50% of male children, and 50% of 
female children affected (refer to top half of 
diagram below). 


Father Children 



hemophiliac 


[ Q: 2230 ] MRCPass - 2011 May 

A 25 year old patient has been 
diagnosed with adult polycystic kidney disease 
recently. The family came to the clinic for 
advice. In particular, his brother and sister 
would like to know the likelihood of them 
developing the disease. 

Which one of the following is the most 
appropriate screening test? 

1- Renal ultrasound of the brother and sister 

2- Renal ultrasound scan of all first degree 
relatives 

3- Renal MR angiography of brother and sister 

4- Renal MR angiography of all first degree 
relatives 

5- Renal function test for all first degree 
relatives 




Dr. Khalid Yusuf El-Zohry - Sohag Teaching Hospital (01118391123) 

Ref MRCPass OE OE 2012 PasTest 2009 PassMedicine 2009 PasTest Exam ReviseMRCP 





































El-zohry MRCP Questions Bank (Port 1) - 2013 


(For my personal use) 


Answer & Comments 

Answer: 2- Renal ultrasound scan of all first 
degree relatives 

Although the brother and sister are 
enthusiastic for screening, the whole family 
(anyone aged >20) should be screened with 
ultrasound. 


[ Q: 2231 ] MRCPass - 2011 May 

An 80 year old lady undergoes 
preoperative assessment for colorectal 
surgery but is found to be unwell. She is on 
analgesic medications only. On examination, 
she looks pale and there is evidence of vitiligo. 
A spleen was palpable at 2 finger breadths 
below the costal margin. Investigation of 
blood results showed: 

Hb 7.5 g/dl 

MCV 106 fl 

WCC 3 x 10 9 /l 

platelets 85 x 10 9 /l 

sodium 138 mmol/I 

potassium 4.5 mmol/l 

urea 6 mmol/l 

creatinine 68 |imol/l 

ALT 30 (5-35) U/l 

ALP 110 (20-120) U/l 

Bilirubin 35 (1-22) |imol/l 

Lactate dehydrogenase 550. (110-230) U/L 

What is the diagnosis? 

1- Pernicious anaemia 

2- Myelodysplasia 

3- Autoimmune haemolytic anaemia 

4- Chronic myeloid leukaemia 

5- Polycythaemia rubra vera 



Answer & Comments 


Answer: 1- Pernicious anaemia 


Pernicious anaemia is usually associated with 
atrophic gastritis, the autoimmune 
destruction of gastric parietal cells leads to a 
lack of intrinsic factor. 

Typically, patients with pernicious anemia are 
described as having a stereotypic appearance. 
Patients have a lemon-yellow waxy pallor with 
premature whitening of the hair. A splenic tip 
is palpable in about 20% of patients. Vitiligo 
which was seen here is an autoimmune 
disorder which may be associated with other 
autoimmune diseases, such as pernicious 
anemia, rheumatoid arthritis, type 1 diabetes, 
alopecia areata, and diseases of the thyroid 
gland. 

In pernicious anaemia, the peripheral blood 
usually shows a macrocytic anemia with a mild 
leukopenia and thrombocytopenia. The mean 
cell volume (MCV) and mean cell hemoglobin 
(MCH) are increased. The bilirubin level may 
be elevated because pernicious anemia is a 
hemolytic disorder associated with increased 
turnover of bilirubin. 

The serum lactic dehydrogenase (LDH) 
concentration usually is markedly increased. 


[ Q: 2232 ] MRCPass - 2011 May 

A 75 year old man has palpitations 
and is admitted to hospital. He has a past 
medical history of previous myocardial 
infarction and hypertension. An ECG shows 
atrial fibrillation and he was monitored on the 
ward. The next day, the ECG was repeated and 
it showed sinus rhythm. 

Which of the following should be started for 
maintenance of sinus rhythm? 

1- Digoxin 

2- Flecainide 

3- Amlodipine 

4- Sotalol 

5- Ramipril 



Dr. Khalid Yusuf El-Zohry - Sohag Teaching Hospital (01118391123) 

Ref MRCPass OE OE 2012 PasTest 2009 PassMedicine 2009 PasTest Exam ReviseMRCP 



892 



























El-zohry MRCP Questions Bank (Port 1) - 2013 


(For my personal use) 


Answer & Comments 

Answer: 4- Sotalol 

Flecainide is a class Ic antiarrhythmic (sodium 
channel blocker) which is useful for 
paroxysmal AF. 

However, the CAST trial showed that patients 
who had ischaemic heart disease had a worse 
outcome when treated with flecainide, so it is 
generally not used in patients with established 
history of ischemic heart disease. Other 
helpful options to maintain sinus rhythm are 
beta blockers (sotalol) and amiodarone. In this 
scenario sotalol is the best option. 


Since drugs are approved on the basis of 
clinical trials which involve relatively small 
numbers of people who have been selected 
for this purpose, postmarketing surveillance 
can further refine the safety of a drug after it 
is used in the general population by large 
numbers of people who have a wide variety of 
medical conditions. 

Postmarketing surveillance uses a number of 
approaches to monitor the safety of licensed 
drugs, including spontaneous reporting 
databases, prescription event monitoring, 
electronic health records, patient registries 
and record linkage between health databases. 


[ Q: 2233 ] MRCPass - 2011 May 

A drug which has been on the 
market for 2 years, has had reports of possible 
serious side effects of fulminant hepatitis. 



Whot is the best way of evaluate this from a 
safety perspective? 

1- Metanalysis 

2- Postmarketing surveillance 


3- Randomised controlled trial 


4- Systematic review 

5- Case control study 


Answer & Comments 

Answer: 2- Postmarketing surveillance 

New drugs which have had regulatory 
approval may not have trial data which 
contain enough information about rare, 
serious side effects, and there may not be 
sufficiently large trials for a new drug for 
metanalysis. 

Postmarketing surveillance (PMS) is the 
practice of monitoring the safety of a 
pharmaceutical drug after it has been released 
on the market and is an important part of the 
science of pharmacovigilance. 


[ Q: 2234 ] MRCPass - 2011 May 

A 53 year man enquired about 
whether it was advisable to have vaccination 
prior a holiday abroad. He had treated asthma 
and had been on long term steroids regularly. 

Which one of the following is a live vaccine 
which should be used with caution in this 
man? 

1- Yellow fever 

2- Diphtheria toxoid 

3- Hepatitis B 

4- Meningococcus 

5- Tetanus toxoid 



Answer & Comments 

Answer: 1- Yellow fever 
The live vaccines are: 

■ BCG 

■ Mumps 

■ Measles 

■ Rubella 

■ Yellow fever 
Smallpox 


Dr. Khalid Yusuf El-Zohry - Sohag Teaching Hospital (01118391123) 

Ref MRCPass OE OE 2012 PasTest 2009 PassMedicine 2009 PasTest Exam 




ReviseMRCP 

893 




























El-zohry MRCP Questions Bank (Port 1) - 2013 


(For my personal use) 


^ [ Q: 2235 ] MRCPass - 2011 May 

- 

0 In the cell cycle, which phase does 
chromotin condense into chromosomes? 

1- Prophase 

2- Metaphase 

3- Anaphase 

4- Telophase 

5- Cytokinesis 


Answer & Comments 

Answer: 1- Prophase 

The relatively brief M phase consists of 
nuclear division (karyokinesis). 

The M phase has been broken down into 
several distinct phases, sequentially known as: 

Prophase, metaphase, anaphase, telophase, 
cytokinesis. 

Prophase, is a stage of mitosis in which the 
chromatin condenses (it becomes shorter and 
fatter) into a highly ordered structure called a 
chromosome in which the chromatin becomes 
visible. 




Nuclear Plasma Chromosome, consisting 
envelope membrane of two sister chromatids 


G 2 OF INTERPHASE 

Centrosomes Chromatin 

(with centriole pairs) (duplicated) 


PROPHASE 
Early mitotic 


PROMETAPHASE 



Kinetochore 

microtubule 


METAPHASE 


ANAPHASE 


TELOPHASE AND CYTOKINESIS 


Meta phase 
plate 


Cleavage 


Nucleolus 





Daughter 

chromosomes 



envelope 

forming 


[ Q: 2236 ] MRCPass - 2011 May 

A 76 year old white male with a past 
I history of duodenal ulcers presents to 
the emergency department complaining of 



medica 


nausea, vomiting, flushing, diarrhea and a dry 
cough for two months. The patient stated the 
pain did not change with food or position and 
was not accompanied by any fevers, chills, 
jaundice, diarrhea or blood per rectum. The 
patient's social history included occasional 
alcohol and tobacco use. On examination, 
there was hepatomegaly. Carcinoid syndrome 
was suspected. Investigations showed that a 
24 hour urine collection for 5- 
hydroxyindoleacetic acid was dramatically 
elevated at 400 milligrams. An octreotide scan 
revealed increased uptake in the lung 
confirming the diagnosis of a carcinoid lung 
tumor. 

Which one of the following is the most early 
symptom in carcinoid syndrome? 

1- Diarrhoea 

2- Facial flushing 

3- Vomiting 

4- Haemoptysis 

5- Nausea 


Answer & Comments 

Answer: 2- Facial flushing 

Patients with carcinoids have commonly been 
diagnosed with irritable bow el syndrome or 
idiopathic flushing. 

The syndrome is characterized by 
hepatomegaly, diarrhea, and flushing in 80% 
of patients; right heart valvular disease in 
50%; and asthma in 25%. Cutaneous flushing 
is a common manifestation (~80% of patients) 
and is often the earliest sign of the syndrome. 
Flushing can occur spontaneously, typically in 
the head and neck. It may be triggered by 
excitement, exercise, some types of food, or 
alcohol. Flushing is mediated by the 
vasoactive peptides secreted by the tumor. 
Diarrhea is also very common. 


Dr. Khalid Yusuf El-Zohry - Sohag Teaching Hospital (01118391123) 

Ref MRCPass OE OE 2012 PasTest 2009 PassMedicine 2009 PasTest Exam ReviseMRCP 




























El-zohry MRCP Questions Bank (Port 1) - 2013 


(For my personal use) 


[ Q: 2237 ] MRCPass - 2011 May 

A 41 year old lady presents with a 1 
year history of pain in the right hand 
progressing to involve the entire right upper 
limb up to the scapular and pectoral regions. 
On examination, there is decreased pinprick 
and temperature sensation in the hand. There 
were absent biceps and supinator reflexes and 
there is muscle wasting in the forearm. 

The likely diagnosis is: 

1- Brachial plexus infiltration 

2- Cervical spondylosis 

3- Syringomyelia 

4- Subacute combined degeneration of the 
cord 

5- Motor neuron disea 

Answer & Comments 

Answer: 3- Syringomyelia 


12 days earlier and persisted despite 
treatment with oral antibiotics and anti¬ 
inflammatory drugs. The fever episodes 
occurred every 48 hours, with high peaks 
followed by abrupt resolution. He had just 
returned from India a month ago, and had not 
received any anti-malarial prophylaxis a. 

On presentation, he was pyrexial and pale, 
tachycardic and had hepatosplenomegaly. 
Microscopy of peripheral blood smears 
showed trophozoites with a parasitemia of 
1.5%. Some enlarged, infected erythrocytes, 
with morphology typical of Plasmodium vivax 
parasites, were observed. 

What is the best antimalarial treatment? 

1- Quinine 

2- Chloroquine 

3- Mefloquine 

4- Pyrimethamine and sulphadiazine 

5- Artesunate 


The clinical features are consistent with 
syringomyelia. 

Syringomyelia is a chronic disorder 
characterized by the presence of a 
longitudinal, fluid filled cavities (syrinx) within 
the spinal cord. Syrinx interrupts the 
decussating spinothalamic fibers that mediate 
pain and temperature. When the cavity 
enlarges to involve the posterior columns, 
position and vibration senses in the feet are 
lost (hence positive Rombergs). Syrinx 
extension into the anterior horns of the spinal 
cord damages motor neurons (lower motor 
neuron) and causes diffuse muscle atrophy 
that begins in the hands and progresses 
proximally to include the forearms and 
shoulder girdles. Impaired bowel and bladder 
functions usually occur as a late manifestation 
of autonomic problems. 


[ Q: 2238 ] MRCPass - 2011 May 

A 56-year-old man presented 
because of a fever (up to 40°C) that had begun 



Answer & Comments 

Answer: 2- Chloroquine 

Plasmodium vivax is found mainly in Asia, 
Latin America, and in some parts of Africa. 

Chloroquine is the treatment of choice for 
vivax malaria, except in Indonesia's Irian Jaya 
region and Papua New Guines, where 
chloroquine resistance is common (then 
artesunate is the treatment of choice). 
Mefloquine is an alternative. 


^ [ Q: 2239 ] MRCPass - 2011 May 

mil 

* A 48 year old patient had chest pain. 
His ECG showed ST elevation in the leads II, III, 
aVF and he also had no correlation between p 
waves and QRS complexes. 

Which artery is likely to be affected? 

1- Left main stem 


2- Left anterior descending 

3- Circumflex 


Dr. Khalid Yusuf El-Zohry - Sohag Teaching Hospital (01118391123) 

Ref MRCPass OE OE 2012 PasTest 2009 PassMedicine 2009 PasTest Exam 




ReviseMRCP 

895 


























El-zohry MRCP Questions Bank (Port 1) - 2013 


(For my personal use) 


4- Right coronary artery 

5- Diagonal 


The clues for Behcet's disease here are oro- 
genital ulceration, eye involvement, Turkish 
origin and also erythema nodosum. 


Answer & Comments 

Answer: 4- Right coronary artery 

The diagnosis is an inferior myocardial 
infarction with complete heart block, this is 
most commonly due to a RCA lesion. 



Inferior Ml (sinus rhythm) 


[ Q: 2240 ] MRCPass - 2011 May 

A 34 year old white Turkish woman 
presented with a swollen left leg which was 
painful. 

She has a history of conjunctivitis, joint pains, 
oral and genital ulceration and livedo 
reticularis. 

On presentation she was afebrile, with 
cervical lymphadenopathy. The patient was 
noted to have oral ulcers and a genital ulcer. 
She had a rash on both her legs. The leg on 
the left was erythematous, painful and tender 
to compression. 

What is the likely diagnosis? 

1- Wegener's granulomatosis 

2- Henoch Schonlein purpura 

3- Behcet's disease 

4- Non Hodgkin's lymphoma 

5- Kawasaki's disease 



Answer & Comments 


Answer: 3- Behcet's disease 


The usual presentation in Behcet's disease is 
of mouth ulcers, sore genitals and eye 
inflammation, and arthritis in older patients. 

Patients must have must have mouth ulcers 
(any shape, size or number at least 3 times in 
any 12 months), and 2 out of the next 4 
"hallmark" symptoms: 

- genital ulcers (including anal ulcers and spots 
in the genital region and swollen testicles or 
epididymitis in men), -skin lesions (papulo¬ 
pustules, folliculitis, erythema nodosum, acne 
in post-adolescents not on corticosteroids) 

- eye inflammation (iritis, uveitis, retinal 
vasculitis, cells in the vitreous), pathergy 
reaction (papule >2 mm dia. 24-48 hrs or more 
after needle-prick). 

There is an increased thrombotic tendency 
among Behcet's disease patients, in this case 
the patient has a clinical presentation 
consistent with a deep vein thrombosis in the 
leg. 



Oral Ulceration in Behcets 


[ Q: 2241 ] MRCPass - 2011 May 

An 81-year-old man with a history of 
schizoaffective disorder presented to hospital 
with increasing auditory hallucinations, 
persecutory delusions and depressive 
symptoms, including suicidal ideation. He was 
admitted to hospital and given haloperidol for 
his psychotic symptoms. 



Dr. Khalid Yusuf El-Zohry - Sohag Teaching Hospital (01118391123) 

Ref MRCPass OE OE 2012 PasTest 2009 PassMedicine 2009 PasTest Exam ReviseMRCP 
























































El-zohry MRCP Questions Bank (Port 1) - 2013 


(For my personal use) 


Three days later he became mildly confused. 
His temperature was elevated (38.3°C), and 
although normotensive (blood pressure 
124/84 mm Hg) he had tachycardia (heart rate 
128 beats/min) and exhibited Parkinsonian 
signs. 

Which one of the following suggests 
neuroleptic malignant syndrome? 

1- Muscular rigidity 

2- Visual blurring 

3- Diarrhoea 

4- Constipation 

5- Erythematous rash 

Answer & Comments 

Answer: 1- Muscular rigidity 

Neuroleptic malignant syndrome (NMS) refers 
to the combination of hyperthermia, rigidity, 
and autonomic dysregulation (labile blood 
pressure) that can occur as a serious 
complication of the use of antipsychotic drugs. 

The most widely accepted mechanism by 
which antipsychotics cause NMS is that of 
dopamine D2 receptor antagonism. 

This leads to increased muscle rigidity and 
tremor via extrapyramidal pathw ays. 


[ Q: 2242 ] MRCPass - 2011 May 

Within which part of the nephron 
does the anti diuretic hormone work on? 

1- Proximal convoluted tubule 

2- Descending limb Loop of Henle 

3- Ascending limb Loop of Henle 

4- Distal convoluted tubule 

5- Cortical collecting duct 



Answer & Comments 


Answer: 5- Cortical collecting duct 


Though the collecting duct particularly the 
outer medullary and cortical collecting ducts 
are normally impermeable to water, but they 
becomes permeable under the actions of 
antidiuretic hormone (ADH). 

As much as three-fourths of the water from 
urine can be reabsorbed as it leaves the 
collecting duct by osmosis. 



Proximal 

convoluted 

tubule t Bowman's 

capsule 


Distal 

convoluted 

tubule 


Glomerulus 


Cortex 


Medulla 


Loop of Henle 


Collecting 


tubule 


to ureter 


[ Q: 2243 ] MRCPass - 2011 May 

A 70-year-old woman presented to 
the emergency room with an acute history of 
progressive exertional chest pain. The pain 
was sharp and was associated with shortness 
of breath. Physical activity made it worse and 
improvement was noted with sublingual 
nitroglycerin. On arrival to the department her 
blood pressure was 105/62 mmHg, pulse was 
98 beats per minute. Cardiac examination 
revealed a regular heart with no murmur, rubs 
or gallop. The ECG showed sinus rhythm with 
low voltage, left axis deviation with ST, lateral 
T wave abnormalities and elevated cardiac 
enzymes Her Troponin T levels peaked at 
5ng/ml (< O.lOng/ml). The following day, her 
chest pains subsided but she developed 
fevers, myalgia and a mottled discoloration of 
both her legs. 

What other finding is likely? 




Dr. Khalid Yusuf El-Zohry - Sohag Teaching Hospital (01118391123) 

Ref MRCPass OE OE 2012 PasTest 2009 PassMedicine 2009 PasTest Exam ReviseMRCP 






























El-zohry MRCP Questions Bank (Port 1) - 2013 


(For my personal use) 


1- Anaemia 

2- Polycythaemia 

3- Thrombocytopenia 

4- Eosinophilia 

5- Haemolysis 


Answer & Comments 

Answer: 4- Eosinophilia 

This patient had a myocardial infarction and 
has developed cholesterol emboli probably 
due to severe atherosclerosis of the aorta. 

The symptoms experienced in cholesterol 
embolism are fever, muscle ache and weight 
loss. Embolism to the legs causes a mottled 
appearance and purple discoloration of the 
toes, small infarcts and areas of gangrene due 
to tissue death that usually appear black, and 
areas of the skin that assume a marbled 
pattern known as livedo reticularis. The full 
blood count may show particularly high 
numbers eosinophils (more than 0.5 x 10 9 /l); 
this occurs in 60-80% of cases. 


[ Q: 2244 ] MRCPass - 2011 May 

A 26-year-old presents with a six 
weeks history of galactorrhoea. She has no 
other symptoms but takes medication for 
contraception, indigestion and headaches. She 
was found to have a Prolactin level of 850 
mU/L (< 450). 

Which one of the following drugs may be 
responsible? 

1- Codeine phosphate 

2- Metoclopramide 

3- Omeprazole 

4- Oral contraceptive pill 

5- Sumatriptan 



Answer & Comments 


Answer: 2- Metoclopramide 


Metoclopramide acts as a dopamine 
antagonist. 

Dopamine inhibits the release of Prolactin 
from the anterior Pituitary gland. Therefore, 
metoclopramide can predispose to 
hyperprolactinaemia, which in turn causes 
galactorrhoea. 


^ [ Q: 2245 ] MRCPass - 2011 May 

# A 46 year old lady presented to her 
GP with lesions in skin that were circular with 
an erythematous raised rim with central 
atrophy. There was scaliness, follicular 
plugging, and telangiectasia over the scalp, 
ears and face. 

This was confirmed to be discoid lupus by the 
dermatologist and she has been tried on 
betnovate steroid topical treatment but has 
not improved. 

What should be used next? 

1- Diprobase cream 

2- Tacrolimus 

3- Azathioprine 

4- Hydroxychloroquine 

5- PUVA therapy 


Answer & Comments 

Answer: 4- Hydroxychloroquine 

Discoid lupus erythematosus (DLE) is a 
chronic, scarring, atrophy producing, 
photosensitive dermatosis. 

DLE may occur in patients with systemic lupus 
erythematosus (SLE). 

Initial treatment comprises the avoidance of 
direct sunlight. Following this, 
Hydroxychloroquine is the gold standard 
treatment. Other options include 
azathioprine, dapsone, thalidomide and 
tacrolimus. 


Dr. Kholid Yusuf El-Zohry - Sohog Teaching Hospital (01118391123) 

Ref MRCPass OE OE 2012 PasTest 2009 PassMedicine 2009 PasTest Exam ReviseMRCP 



898 



























El-zohry MRCP Questions Bank (Port 1) - 2013 


(For my personal use) 



[ Q: 2246 ] MRCPass - 2011 May 

A 41-year-old woman with a past 
history of epilepsy complains of recurrent 
migraine type headaches. The headaches have 
been occurring daily, and are associated with 
flashing lights. She was previously treated 
with paracetamol, aspirin and diclofenac with 
little improvement. 


What of the following medications should you 
then prescribe? 


1- Pizotifen 


2- Meperidine 

3- Sumatriptan 

4- Ergotamine 

5- Propranolol 


Answer & Comments 

Answer: 3- Sumatriptan 

Sumatriptan is structurally similar to 
serotonin, and is a 5-HT1D agonist, which is 
one of the receptors to which serotonin binds. 


4- Diabetic nephropathy 

5- IgA nephropathy 


Answer & Comments 

Answer: 2- Reflux nephropathy 

Urine reflux is the most common cause of 
chronic pyelonephritis and can lead to 
nephropathy. 

The risk factors include a personal or family 
history of reflux. Ultrasound can identify renal 
scarring if moderate to severe in degree 


[ Q: 2248 ] MRCPass - 2011 May 

A 45 year old lady presented to 
hospital complaining of lesions on the skin. 
The patient is Brazilian and has been in the 
country for several months. The skin lesions 
were circular with an erythematous raised rim 
with central atrophy. There was scaliness, 
follicular plugging, and telangiectasia over the 
face. There was also evidence of loss of hair 
around the eyebrows. 



Low serotonin levels in the brain may lead to a 
process of constriction and dilation of the 
blood vessels which trigger a migraine. 
Triptans activate serotonin receptors to stop a 
migraine attack, and are recommended for 
treating acute migraine headaches. They take 
15-30 minutes to work. 


What is the diagnosis? 

1- Cutaneous leishmaniasis 

2- Discoid lupus erythematosus 

3- Leprosy 

4- Lyme's disease 

5- Pityriasis versicolor 


^ [ Q: 2247 ] MRCPass - 2011 May 

# A 31 year old woman has a history of 

recurrent urinary tract infections as a child. 
Her mother has a history of hypertension and 
was told that her kidneys were 'damaged'. An 
ultrasound of the patient showed scarring in 
both kidneys. 

What is the most likely diagnosis? 

1- Autosomal dominant polycystic kidney 
disease 

2- Reflux nephropathy 

3- Renal cell carcinoma 


Answer & Comments 

Answer: 2- Discoid lupus erythematosus 

Discoid lupus erythematosus (DLE) is a 
chronic, scarring, atrophy producing, 
photosensitive dermatosis. 

DLE may occur in patients with systemic lupus 
erythematosus (SLE). Skin lesions are typically 
localized above the neck, with favored sites 
being the scalp, bridge of nose, cheeks, lower 
lip, and ears. The primary lesion is an 
erythematous papule or plaque with slight-to- 


Dr. Khalid Yusuf El-Zohry - Sohag Teaching Hospital (01118391123) 

Ref MRCPass OE OE 2012 PasTest 2009 PassMedicine 2009 PasTest Exam ReviseMRCP 



899 





























El-zohry MRCP Questions Bank (Port 1) - 2013 


(For my personal use) 


moderate scaling. As the lesion progresses, 
the scale may thicken and become adherent, 
and pigmentary changes may develop, with 
hypopigmentation in the central or inactive 
area and hyperpigmentation at the active 
border. Hydroxychloroquine can be used for 
treatment. 

The other possible answer here is cutaenous 
leishmaniasis. The typical lesions are crusty, 
painless ulcers on exposed skin. Ulcerative 
lesions are usually shallow and circular with 
well-defined, raised borders and a bed of 
granulation tissue. However, the eyebrow 
alopecia for this patient makes discoid lupus 
more likely. 



Discoid Lupus 



[ Q: 2249 ] MRCPass - 2011 May 

A 62 year old man presented with 
fevers, lethargy and 2 month history of 
malaise. He also mentioned altered bow el 
habit. On examination, he had a temperature 
of 39 C and a soft systolic murmur in the 
mitral area. He also had several splinter 
haemorrhages. Blood culture results within 24 
hours grew streptococcus bovis. 


What investigation will help determine the 
underlying source of infection? 

1- Abdominal x ray 


2- Colonoscopy 

3- CT scan of the chest 


4- Skin biopsy 


5- Transoesophageal echocardiogram 

Answer & Comments 

Answer: 2- Colonoscopy 

A correlation exists between colon cancer and 
Strep. 

bovis proliferation in the large intestine, 
hence predisposing to endocarditis. The 
patient needs a colonoscopy which may 
identify a colorectal malignancy predisposing 
to strep bovis bacteraemia and endocarditis. 


[ Q: 2250 ] MRCPass - 2011 May 

A 44 year old female patient 
presented in to clinic with a generalised 
blistering rash on the arms and legs. Clinical 
examination revealed tense skin blisters with 
some generalised desquamation. There was 
no involvement of the mucous membranes. 

What is the diagnosis? 

1- Pemphigus vulgaris 

2- Erythema multiforme 

3- Dermatitis herpetiformis 

4- Bullous pemphigoid 

5- Insect bite 



Answer & Comments 

Answer: 4- Bullous pemphigoid 

Bullous pemphigoid is more common than 
pemphigus , occurs more commonly in later 
life (>60years). 

It is a chronic, autoimmune, subepidermal, 
blistering skin disease that rarely involves 
mucous membranes. Large bullae appears 
anyw here on the skin ,they tend to be itchy 
and the lesions are deep and mucosal 
involvement rare. Bullous 

pemphigoid is characterized by the presence 
of immunoglobulin G (IgG) autoantibodies in 
the hemidesmosomal area. 


Dr. Khalid Yusuf El-Zohry - Sohag Teaching Hospital (01118391123) 

Ref MRCPass OE OE 2012 PasTest 2009 PassMedicine 2009 PasTest Exam ReviseMRCP 



900 

























El-zohry MRCP Questions Bank (Port 1) - 2013 


(For my personal use) 


These manifest as tense blisters. Direct 
immunofluorescence of a skin biopsy usually 
demonstrate IgG and complement C3 
deposition in a linear band at the dermal- 
epidermal junction. 



Bullous Pemphigoid 


[ Q: 2251 ] MRCPass - 2011 May 

A 35-year-old, female secretary 
presents to the emergency department with 7 
days of bloody diarrhea and lower abdominal 
cramping. A flexible sigmoidoscopy and biopsy 
showed evidence of inflammation consistent 
with ulcerative colitis. Over the next few 
months, she continued to have several 
episodes of such presentations which required 
steroid treatment. 

What should be prescribed now? 

1- Infliximab 

2- Azathioprine 

3- Chlorpromazine 

4- Amitriptyline 

5- Low dose prednisolone 



Answer & Comments 

Answer: 2- Azathioprine 

Azathioprine may be used in patients with 
Crohn's disease or ulcerative colitis that are 
steroid dependent or steroid resistant. 


This is a very effective, safe and well-tolerated 
drug with no definite associated risk of cancer. 

Azathioprine has a slow onset of action and 
requires continuing steroid cover for 8-12 
weeks. The principle side-effects of 
azathioprine are idiosyncratic acute 
pancreatitis and bone-marrow suppression. 



[ Q: 2252 ] MRCPass - 2011 May 

A 70-year-old woman has lung 
carcinoma and recently underwent 
chemotherapy. She presented with shortness 
of breath and pleuritic chest pain. 


Which one of the following signs suggests a 
significant pericardial effusion? 

1- Systolic murmur 


2- Pericardial rub 


3- Rapid y descent of JVP 

4- Pulsus paradoxus 

5- Pulses alternans 


Answer & Comments 

Answer: 4- Pulsus paradoxus 

The question asks for which signs are present 
in cardiac tamponade due to a large 
pericardial effusion. 

The main signs are Kussmaul's sign (increase in 
JVP with inspiration) and Pulsus paradoxus 
(the inspiratory fall of aortic systolic pressure 
greater than 10 mm Hg). 


[ Q: 2253 ] MRCPass - 2011 May 

A 58 year old man has presented 
with chest pain and pulmonary oedema. He is 
managed as acute coronary syndrome. He did 
not tolerate a GTN infusion as his blood 
pressure was 85/60 but improved with 
frusemide and his blood pressure stabilized. 
An ACE-inhibitor was held off due to renal 
impairment. He is coincidentally found to have 
a chest infection, his blood cultures grew 



Dr. Khalid Yusuf El-Zohry - Sohag Teaching Hospital (01118391123) 

Ref MRCPass OE OE 2012 PasTest 2009 PassMedicine 2009 PasTest Exam ReviseMRCP 


































El-zohry MRCP Questions Bank (Port 1) - 2013 


(For my personal use) 


streptococci and he was treated with 
augmentin. His blood tests on admission show 
a creatinine of 145 pmol/l rising up to 190 
pmol/l and then 250 pmol/l the day after. 

What is the likely cause of acute kidney injury? 

1- Urinary tract infection 

2- Acute tubular necrosis 

3- Interstitial nephritis due to augmentin 

4- Post streptococcal glomerulonephritis 

5- Renal artery stenosis 


prolapse. He also had pectus excavatum, 
arachnodactyly and arm span greater than 
height. Slit lamp examination revealed had 
upward dislocation of the lens in the eye. 

The gene defect is: 

1- Actin 

2- Myosin 

3- Fibrillin 

4- Retinoblastoma 

5- Elastin 


Answer & Comments 


Answer: 2- Acute tubular necrosis 


Answer & Comments 


Answer: 3- Fibrillin 


A rapid rise in creatinine following periods of 
hypotension is most commonly due to acute 
tubular necrosis. 

Acute tubular necrosis or (ATN) involves the 
death of tubular cells that form the tubule 
that transports urine to the ureters while 
reabsorbing 99% of the water. Tubular cells 
continually replace themselves and if the 
cause of ATN is removed then recovery is 
likely. 



A renal biopsy shows renal medulla, which is 
composed mainly of renal tubules. Patchy or 
diffuse denudation of the renal tubular cells is 
observed, suggesting acute tubular necrosis 

(ATN) 



[ Q: 2254 ] MRCPass - 2011 May 

A 21 year old man is tall compared to 
his peers. On examination, he was found to 
have aortic incompetence and mitral valve 


The fibrillin gene defect is the basis of 
Marfan's syndrome. 

It is an autosomal dominant disorder 
characterised by arachnodactyly, upward lens 
dislocation, tall habitus and flat feet. Aortic 
aneurysms and aortic regurgitation are also 
associated. 


^ [ Q: 2255 ] MRCPass - 2011 May 

f - 

A 44 year old woman was admitted 
to hospital with a several month history of 
diarrhoea, malaise and weight loss. She was in 
good health prior to the development of these 
symptoms. On examination, she had mild 
jaundice and looked thin. She had a distended 
abdomen with shifting dullness to percussion. 
Her blood tests show : 

Hb 10.5 g/dl 

MCV 82 fl 

WCC 8 x 10 9 /l 

platelets 220 x 10 9 /l 

sodium 125 mmol/l 

potassium 4.1 mmol/l 

urea 11 mmol/l 

creatinine 160 |imol/l 

ALT 95 (5-35) U/l 


Dr. Khalid Yusuf El-Zohry - Sohag Teaching Hospital (01118391123) 

Ref MRCPass OE OE 2012 PasTest 2009 PassMedicine 2009 PasTest Exam ReviseMRCP 

























El-zohry MRCP Questions Bank (Part 1) - 2013 


(For my personal use) 


AST 115 (1-31) U/l 
ALP 220(20-120) U/l 
Bilirubin 30 (1-22) nmol/l 
Albumin 28 (37-49) g/l 

Carcinoembryonic antigen (CEA) 3.8 <2.5 
ng/ml 

Alpha-Fetoprotein (AFP) 55 < 44 ?g/L 

CA 125 - 38 (<35) U/m 

CA 19.9 - 250 (< 40) U/ml 

CA 15.3 - 32 (< 29) U/mL 

Prostate-Specific Antigen (PSA) 2 (< 4) ng/ml 

Which one of the following is the likely primary 
tumour? 

1- Pancreas 

2- Colorectal 

3- Ovarian 

4- Liver 

5- Prostate 


Answer & Comments 

Answer: 1- Pancreas 

The tumour markers are not specific to one 
tumour but in this case the Ca 19.9 is highest, 
and it is most strongly associated with 
pancreatic cancer. 

[ Q: 2256 ] MRCPass - 2011 May 
Which is the site of action of thiazide 

diuretics? 

1- Proximal part of Distal Convoluted Tubule 

2- Distal part of Distal Convoluted Tubule 

3- Proximal convoluted tubule 

4- Loop of Flenle 

5- Collecting duct 



Answer &. Comments 

Answer: 1- Proximal part of Distal Convoluted 
Tubule 


Thiazide diuretics reduce the reabsorption of 
sodium and chloride in the early part of the 
distal convoluted tubule of the kidney. 


This results in the delivery of increased 
amounts of sodium to the distal tubule, where 
some of it is exchanged for potassium. The net 
result is increased excretion of sodium, 
potassium and water. 



SPIRONOLACTONE 


INHIBITORS Of 
M«*.Cr SYMPORTER 

THIAZIDES 


ALDOSTERONE 

INHIBITORS 


Hm* CHANNEL 
BLOCKERS 

TRIAMTERINE 
AMI LOR IOC 


DIURETICS 


INHIBITORS OF 

Hm*. K*. acr SYMPORTER 


FUROSEMIDE 
BUMETANIOE 
ETHACRYNC ACID 


INHIBITORS OF 
CARBONIC ANHYDRASE 


ACETAZOLAMIDE 

I-► 


[ Q: 2257 ] MRCPass - 2011 May 

A 62 year old man presents with 
lethargy and weight loss which has occurred 
over a period of 4 months. He has a history of 
hypertension and diabetes. He was admitted 
to hospital and investigated for the symptoms. 
A CT scan of the abdomen confirmed that he 
had sigmoid colon carcinoma which was 
localised. His blood results showed that he 
had a urea of 15 mmol/l and creatinine 180 
umol/l. Urine dip stick showed protein ++++ 
and blood +. A renal biopsy was performed. 
Shortly after this, the patient underw ent a 
colectomy as an inpatient. He recovered well 
and was discharged. A follow up appointment 
was arranged to review the renal biopsy result 
it was observed that the proteinuria 
improved. 

What is the likely histology of the glomerular 
lesion ? 

1- Minimal change glomerulonephritis 

2- Membranoproliferative glomerulonephritis 



Dr. Khalid Yusuf El-Zohry - Sohag Teaching Hospital (01118391123) 

Ref MRCPass OE OE 2012 PasTest 2009 PassMedicine 2009 PasTest Exam 




ReviseMRCP 

903 





























El-zohry MRCP Questions Bank (Port 1) - 2013 


(For my personal use) 


3- Membranous nephropathy 

4- Focal segmental glomerulosclerosis 

5- IgA nephropathy 

Answer & Comments 

Answer: 3- Membranous nephropathy 

The scenario shows nephrotic range 
glomerular disease, as this resolved after the 
colectomy it suggests an association which is 
most likely membranous nephropathy 
associated with carcinoma. 

Membranous nephropathy is caused by 
circulating immune complexes which are 
formed by binding of antibodies to antigens in 
the glomerular basement membrane. 

85 % of membranous nephropathy cases are 
idiopathic. 

About 15% of cases are secondary due to: 

autoimmune conditions (e.g., systemic lupus 
erythematosus) 

infections (e.g., syphilis, malaria, hepatitis B) 

drugs (e.g., captopril, NSAIDs, gold, mercury, 
penicillamine, probenecid). 

tumors, frequently solid tumors of the lung 
and colon; hematological malignancies such as 
chronic lymphocytic leukemia are less 
common. 

Normal glomerular Membranous glomerulopathy 
capillaries with thin walls with thick capillary walls 



^ [ Q: 2258 ] MRCPass - 2011 May 

m A 62 year woman has presented with 
new sudden jerky movements of her arm. She 
has a history of diabetes mellitus and is on 
insulin. She describes sudden onset of wild 
flinging movements of left arm which are 
worse when she is doing physical work. When 
she rests the movements appear to lessen. 
This movement can occur up to every few 
minutes. 

Where is the likely site of the lesion? 

1- Substantia nigra 

2- Contralateral subthalamic nucleus 

3- Non dominant parietal 

4- Caudate nucleus 

5- Cerebellar 

Answer & Comments 

Answer: 2- Contralateral subthalamic nucleus 

The hemiballismus (swinging arm movements) 
is likely be due a lesion in the subthalamic 
nucleus.The movements are often violent and 
have wide amplitudes of motion. 

They can be continuous and random. The 
facial muscles, arms and legs can be involved. 
The more a patient is active, the more the 
movements increase. With relaxation comes a 
decrease in movements. 

The subthalamic nucleus innervates other 
structures within the basal ganglia, including a 
very important connection to the inside of the 
globus pallidus. The subthalamic nucleus 
essentially provides the excitement needed to 
drive the globus pallidus. Injury to the 
subthalamus or its efferent or afferent 
connections can induce hemiballismus. 
Although traditionally thought that the 
disorder was only caused by injury to the 
subthalamic nucleus, new studies show that 
damage to other areas of the brain can also be 
responsible for causing this disorder. 


Dr. Kholid Yusuf El-Zohry - Sohog Teaching Hospital (01118391123) 

Ref MRCPass OE OE 2012 PasTest 2009 PassMedicine 2009 PasTest Exam ReviseMRCP 
























El-zohry MRCP Questions Bank (Port 1) - 2013 


(For my personal use) 


Causes can be due to a vascular lesion 
(stroke), trauma, neurodegeneration (motor 
neuron disease) and demyelination. 



Subthalamic Nucleus 


[ Q: 2259 ] MRCPass - 2011 May 

A 51 year old man with a past history 
of alcohol abuse presents with a painful red 
and warm ankle which was tophaceous. Gout 
was diagnosed and he was prescribed 
allopurinol. 

Whot is allopurinol's mechanism of action? 

1- Uricosuric drug 

2- Non steroidal anti inflammatory drug 

3- Microtubule inhibitor 

4- Xanthine oxidase inhibitor 

5- Dihydrofolate reducatase inhibitor 



Answer & Comments 

Answer: 4- Xanthine oxidase inhibitor 

Hypouricaemic agents essentially comprise of 
xanthine oxidase inhibitors (for example, 
allopurinol) and uricosuric agents (for 
example, probenecid, sulphinpyrazone or 
azapropazone). 

Standard teaching is that urate lowerring 
drugs should not be introduced during an 
acute episode as it may worsen or prolong the 


episode; furthermore initiation of 
hypouricaemic treatment may precipitate 
acute gout. Colchicine and NSAIDs can be used 
in the acute situation. 



[ Q: 2260 ] MRCPass - 2011 May 

A 53 year old patient e was referred 
to the outpatient clinic with a history of 
recurrent pruritic lesions of 9 months' 
duration, which were initially located on the 
elbow s, periumbilical areas, and subsequently 
on the back and shoulders. The patient 
reported that milky-w hite areas of skin had 
appeared 4 years previously, and these areas 
had remained unchanged. He had not sought 
any treatment for this condition. A 
dermatologist suspects dermatitis 

herpetiformis. 

What should be done to confirm this? 

1- Anti gliadin antibody 

2- Anti endomysial antibody 

3- Small bowel biopsy of the intestine 

4- immunofluorescence of perilesional skin for 
IgA 


5- Trial of steroids 


Answer & Comments 

Answer: 4- immunofluorescence of 

perilesional skin for IgA 

In dermatitis herpetiformis, the biopsy sample 
should be taken from the edge of a lesion for 
hematoxylin and eosin staining and from 
normal-appearing perilesional skin for direct 
immunofluorescence staining. 

Granular IgA deposits in dermal papillae of 
perilesional skin observed by direct 
immunofluorescence is the criterion standard 
of diagnosis. 


Dr. Khalid Yusuf El-Zohry - Sohag Teaching Hospital (01118391123) 

Ref MRCPass OE OE 2012 PasTest 2009 PassMedicine 2009 PasTest Exam 




ReviseMRCP 

905 























El-zohry MRCP Questions Bank (Port 1) - 2013 


(For my personal use) 



Direct immunofluorescence with linear IgA 
deposits along the dermal epidermal junction 


[ Q: 2261 ] MRCPass - 2011 May 

A 42 year old type I diabetic is 
referred for renal investigations. She has been 
suffering from Rheumatoid arthritis for the 
last 20 years. She is currently on insulin 
injections, ibuprofen and penicillamine. She 
had 4 + proteinuria on a urine dipstick and 
quantification with 24 hour urine collection 
revealed that she had urinary protein > 4.5 
g/day. 

Ultrasound of the abdomen shows increased 
renal echogenicity. 

Investigations : 

Hb 11.5 g/dl 

MCV 82 fl 

WCC 12 x 10 9 /l 

platelets 225 x 10 9 /l 

sodium 135 mmol/l 

potassium 4.5 mmol/l 

Urea 14 mmol/l 

Creat 215 umol/l 

A renal biopsy shows eosinophilic deposits 
within the mesangium on light microscopy. 
The basement membrane and epithelial space 
is normal. 

Whot is the probable diagnosis? 

1- Minimal change nephropathy 

2- Membranous nephropathy 

3- Diabetic nephropathy 



4- NSAIDS induced nephropathy 

5- Amyloidosis 


Answer & Comments 

Answer: 5- Amyloidosis 

Diabetic nephropathy is unlikely to cause such 
heavy proteinuria, hence amyloidosis due to 
rheumatoid arthritis is the most likely 
diagnosis. 

Amyloidosis is a clinical disorder caused by 
extracellular and or intracellular deposition of 
insoluble abnormal amyloid fibrils that alter 
the normal function of tissues. AA amyloidosis 
can be caused by rheumatoid arthritis. Up to 
5% of patients with long-standing RA can 
develop systemic amyloidosis that usually 
presents as nephrotic syndrome. The biopsy 
shows eosinophilic deposits in the mesangium 
and capillary walls, which can be stained pink 
with Congo Red. 

Membranous nephropathy can present 
similarly, but is more commonly associated 
with autoimmune diseases (e.g. 

SLE), infections (e.g. hepatitis B) and 
malignancy (e.g. lymphoma).The drugs for 
rheumatoid arthirits ie NSAIDS, penicillamine 
gold can cause membranous nephropathy. 
The renal biopsy will show small subepithelial 
deposits in the glomeruli which can also lead 
to spikes or thickening of the basement 
membrane but the mesangium is typically 
normal. 

This case is also unlikely to be minimal change 
disease - (age of onset usually younger), the 
histology shows in minimal change shows a 
normal glomerulus and fusion of epithelial 
foot process will be seen only on electron 
microsocopy. 


Dr. Khalid Yusuf El-Zohry - Sohag Teaching Hospital (01118391123) 

Ref MRCPass OE OE 2012 PasTest 2009 PassMedicine 2009 PasTest Exam ReviseMRCP 



906 























El-zohry MRCP Questions Bank (Part 1) - 2013 


(For my personal use) 


• • • 


A • 


• Tubules j % 

\ V.% ; V * 

; ' >V? A* 


v 


a 







/ ‘ V k Glomeruli with^* ,\I ^ ^ J */ 

amyloid deposits •* ‘ nt V^r/U 



[ Q: 2262 ] MRCPass - 2011 May 

A 60 year old man presented with 
back pain. On examination, he had normal 
tone bilaterally. There was absent ankle jerk 
reflex on the right. He also had decreased 
touch and pain sensation on the lateral side of 
the right. The plantar reflex was normal on the 
left and upgoing on the right. The anal tone 
was normal and there was no evidence of 
saddle anaesthesia. 

What it the likely diagnosis? 

1- Peripheral neuropathy 

2- Lumbar spinal canal stenosis 

3- Cauda equina 

4- Posterolateral disc prolapse at L5-S1 



If the prolapse is very large and presses on the 
spinal cord or the cauda equina syndrome 
may occur. Compression of the cauda equina 
can cause permanent nerve damage or 
paralysis. In cauda equina syndrome, the 
nerve damage can result in bilateral leg 
weakness and pains, saddle anaesthesia and 
loss of bow el and bladder control as well as 
sexual dysfunction. 


^ [ Q: 2263 ] MRCPass - 2011 May 

ft - 

A 18 year woman with mild Von 
Willebrand's disease was scheduled for dental 
extraction. A previous dental extraction 
resulted in bleeding and had required two 
units of blood for transfusion. 

What is the appropriate treatment prior to 
dental surgery? 

1- Cryoprecipitate 

2- DDAVP 

3- Fresh frozen plasma 

4- High purity factor VIII concentrate 

5- Recombinant factor VIII concentrate 


Answer & Comments 


Answer: 2- DDAVP 


5- Subacute combined degeneration of the 
cord 


Answer & Comments 

Answer: 4- Posterolateral disc prolapse at L5- 
S1 

A prolapsed intervertebral disc typically 
causes lower back pain and pain radiating 
down the legs (sciatica). 

The L5/S1 disc is the disc most commonly 
damaged. With a posterolateral herniation, 
disc will affect the nerve corresponding to the 
lower level in this case SI. The absent ankle 
reflex and lateral foot sensory loss is 
consistent with an SI root lesion. 


DDAVP is the treatment of choice for mild 
disease, which would include Type I, majority 
of Type II. 

It of limited use in Type III - severe Von 
Villebrand's disease. The history indicates that 
she has mild disease. For severe disease one 
would use a Von Willebrand factor 
concentrate, factor VIII concentrate. 


[ Q: 2264 ] MRCPass - 2011 May 

A 63- year-old male was admitted 
with a history of severe recurrent attacks of 
central abdominal pain radiating to both 
hypocondria, occasionally associated with 
vomiting. He used to be a heavy smoker and 
admitted to a moderate alcohol consumption. 



Dr. Khalid Yusuf El-Zohry - Sohag Teaching Hospital (01118391123) 

Ref MRCPass OE OE 2012 PasTest 2009 PassMedicine 2009 PasTest Exam ReviseMRCP 



907 
























El-zohry MRCP Questions Bank (Port 1) - 2013 


(For my personal use) 


On examination, he had no signs of chronic 
liver disease and there were no palpable 
masses. 

What is the investigation of choice to confirm 
suspected chronic pancreatitis? 

1- Endoscopic ultrasound 


Alcohol withdrawal delirium (delirium 
tremens) is the clinical syndrome of 
disorientation, perceptual disturbance and 
psychomotor agitation. 

Visual hallucinations are commonly 
associated. 


2- CT scan of the abdomen 

3- Barium enema 

4- Colonoscopy 

5- Flexible sigmoidoscopy 


Answer & Comments 

Answer: 2- CT scan of the abdomen 

A secretin simulation test is considered the 
gold standard functional test for diagnosis of 
chronic pancreatitis. 


Korsakoff's psychosis is associated short term 
memory loss, subsequent compensatory 
confabulation by patient. 

Other symptoms may include delirium, 
anxiety, fear, depression, confusion, delusions 
and insomnia. 


[ Q: 2266 ] MRCPass - 2011 May 

What is the mode of inheritance of 
the disease Hereditary Haemorrhagic 
Telangiectasia? 



CT scan of the abdomen is useful because 1- Autosomal recessive 

longstanding inflammation often causes 2- Autosomal dominant 

calcification of the pancreas. , , , , 

3- X-linked dominant 



[ Q: 2265 ] MRCPass - 2011 May 

A 58 patient has been admitted with 
confusion. The patient's relatives describe that 
he does not remember the events that 
occurred in the last day but recognised them. 
He also seemed to make up events which he 
did not remember. On examination, he had an 
MMSE score of 22 / 30. He was tremulous in 
both his hands and had an ataxic gait. Tone, 
power and reflexes were normal in both upper 
and lower limbs. 


What is the likely diagnosis? 


4- X-linked recessive 

5- Polygenic inheritance 


Answer & Comments 

Answer: 2- Autosomal dominant 

There are four forms of Hereditary 
Hemorrhagic Telangiectasia. 

Inheritance is autosomal dominant. HHT 1 is 
associated with a mutation in the endoglin 
gene, and HHT 2 is associated with a mutation 
in the alkl gene. 


1- Alzheimer's dementia 

2- Lewy body disease 

3- Korsakoff's psychosis 

4- Creudsfeldt Jakob disease 

5- Normal pressure hydrocephalus 


HHT is characterised by telangiectasia (small 
vascular malformations on the skin and 
mucosal linings (as shown in the diagram 
below ), epistaxis (nosebleeds), and 
arteriovenous malformations (AVMs) in 
various internal organs. 


Answer & Comments 


Answer: 3- Korsakoff's psychosis 


Dr. Khalid Yusuf El-Zohry - Sohag Teaching Hospital (01118391123) 

Ref MRCPass OE OE 2012 PasTest 2009 PassMedicine 2009 PasTest Exam 




ReviseMRCP 

908 

























El-zohry MRCP Questions Bank (Port 1) - 2013 


(For my personal use) 



[ Q: 2267 ] MRCPass - 2011 May 

A 16 year old boy presents with 
discoloured urine. He describes having had a 
sore throat 5 days ago but has recovered from 
the symptoms. The urine dipstick shows blood 
+++, protein +. Renal function was normal on 
the blood tests. A renal biopsy is likely to show 
which of the following on light microscopy? 

1- Crescents 






tj Increased cellularity 
y in glomeruli 


Reduced 
Bowman spaces 





[ Q: 2268 ] MRCPass - 2011 May 

An 18 year old girl with 
meningococcal meningitis has further 
investigations as this is her 3rd episode of 
meningitis. It was found that she had low 
properdin levels measured by the ELISA test. 
How is this likely to have been inherited? 



2- Collapsed glomeruli 

3- Normal tissue 


1- Autosomal dominant 

2- Autosomal recessive 


4- Segmental glomerulosclerosis 

5- Mesangial hypercellularity 

Answer & Comments 

Answer: 5- Mesangial hypercellularity 

The age , sex and almost simultaneous 
presentation of sore throat with haematuria 
suggests IgA nephropathy. 

It is a common cause of macroscopic 
haematuria in a child. It also occurs commonly 
in young adults. Episodes of haematuria are 
often simultaneous with periods of viral 
infection (sore throat) and flank pain. The 
urine may be frankly bloody or may be cola 
colour. There are clots in urine. It usually 
resolves spontaneously within 

4-7days.Renal biopsy will show mesangial IgA 
deposition on immunofluorescence, light 
microscopy will show mesangial 
hypercellularity with matrix expansion. 


3- X linked recessive 

4- X linked dominant 

5- Mitochondrial inheritance 

Answer & Comments 

Answer: 3- X linked recessive 

Complement deficiencies are relatively rare 
worldw ide, and estimates of prevalence are 
based on results from screening high-risk 
populations. 

Retrospective studies of persons with 
frequent meningococcal infections report 

varying prevalence based on geographic 
location. In populations with recurrent 
meningococcal infection, the prevalence rate 
is as high as 30%. 

The majority of complement deficiencies are 
inherited in an autosomal recessive patter. An 
exception to the autosomal pattern of 
inheritance is properdin deficiency, which is 
an X-linked trait. 



Dr. Kholid Yusuf El-Zohry - Sohog Teaching Hospital (01118391123) 

Ref MRCPass OE OE 2012 PasTest 2009 PassMedicine 2009 PasTest Exam ReviseMRCP 


























El-zohry MRCP Questions Bank (Port 1) - 2013 


(For my personal use) 


The pathw ays include the classic pathway 
(Clqrs, C2, C4), the alternative pathway (C3, 
factor B, properdin). 

Properdin stabilizes the C3 convertase (C3bBb) 
of the alternative pathw ay, involved in 
opsonisation. Meningococcal disease is a 
prominent manifestation in a significant 
fraction of reported cases in all clinical 
patterns of complement deficiency, 
particularly those where opsonisation is 
defective. Properdin deficiency states are X- 
linked recessive, while other genetic defects 
within the complement system appear to be 
transmitted as autosomal recessive traits. 

[ Q: 2269 ] MRCPass - 2011 May 

A 20 year old female is referred to 
the hospital. Her family are concerned about 
her because she has been very agitated in the 
last 2 days. She has no past medical history of 
note. The sister said that she was hearing 
voices. 

On examination, she looks restless and 
physical examination is normal. 

Whot is the likely couse of this presentation? 

1- Alcohol 

2- Barbiturates 

3- Diazepam 

4- Amphetamines 

5- Opiates 



Answer & Comments 

Answer: 4- Amphetamines 

This patient is likely to have taken a stimulant 
which has made her restless and agitated with 
some psychotic features. 

The most common stimulants taken as 
recreational drugs are amphetamines and 
cocaine. 


[ Q: 2270 ] MRCPass - 2011 May 

A 70 year old lady has longstanding 
lethargy. On physical examination she had a 
plethoric face & conjunctival injection. 
Investigations revealed: 

hemoglobin 17.5 g/dl (12-16 g/dl) 

red cell count 8.1 M/?l (3.5 to 6 M/?l) 

Hct. 58.% (37 to 52%) 

MCV 65.7fL (67 to 96 fL) 

MCH 19.6 pg (27 to 32 pg) 

WBC 13.1 x 10 9 /l 

ESR 2 mm/hour 

Blood film showed erythrocytosis, 
thrombocytosis and leucocytosis. 

With the diagnosis in mind, which one of the 
following is likely to be associated? 

1- JAK2 mutation 

2- P53 mutation 

3- EGFR mutation 

4- NPAT mutation 

5- MYD88 mutation 



Answer & Comments 

Answer: 1- JAK2 mutation 

The diagnosis in this case is polycythaemia 
rubra vera. 

Janus kinase 2 (commonly called JAK2) is a 
human protein 

that has been implicated in signaling by 
members of the type II cytokine receptor 
family. These mutations have been associated 
with polycythemia vera, essential 
thrombocythemia, and other 

myeloproliferative disorders. 



[ Q: 2271 ] MRCPass - 2011 May 

A 55 year old lady presents with 
sudden onset bilateral lower limb weakness 
half a day ago. On examination there was loss 


Dr. Kholid Yusuf El-Zohry - Sohog Teaching Hospital (01118391123) 

Ref MRCPass OE OE 2012 PasTest 2009 PassMedicine 2009 PasTest Exam ReviseMRCP 




























El-zohry MRCP Questions Bank (Port 1) - 2013 


(For my personal use) 


of pain and temperature sensation (T12 
downw ards) but joint position sense and 
vibration sense was preserved bilaterally. Both 
the lower limbs were hyperreflexic with 
upgoing plantars. 

What is the diagnosis? 

1- Severe combined degeneration of the cord 

2- Anterior spinal artery infarction 

3- Multiple sclerosis 

4- Brown Sequard Syndrome 

5- Motor neuron disease 

Answer & Comments 

Answer: 2- Anterior spinal artery infarction 

The typical clinical features of spinal artery 
infarction include: 


Answer & Comments 

Answer: 2- Southern blotting 

A Southern blot is a method routinely used in 
molecular biology for detection of a specific 
DNA sequence in DNA samples. 

Southern blotting combines transfer of 
electrophoresis-separated DNA fragments to a 
filter membrane and subsequent fragment 
detection with a hybridisation probe. 

The northern blot is a technique used in 
molecular biology research to study gene 
expression by detection of RNA in a sample. 
Northern blotting involves the use of 
electrophoresis to separate RNA samples by 
size, and detection with a hybridization probe 
(either DNA or RNA) complementary to part 
gene sequence. 


-sudden radicular pain and para- or quadra- 
paresis 

-limbs initially flaccid but within days, become 
spastic 


The Western blot is a technique involving 
electrophoresis to detect specific proteins in 
the given sample of tissue homogenate or 
extract (e.g. in HIV testing, or to detect prions 
in Bovine Spongiform Encephalopathy). 


-reflexes initially absent but within days, 
become hyper-reflexic with extensor plantars 

-loss of pain and temperature sensation up to 
the level of cord damage - due to involvement 
of anterolateral 

spinothalamic tracts 

-urinary and faecal incompetence from lesions 
in the lower lumbar / sacral areas 


[ Q: 2272 ] MRCPass - 2011 May 

Which one of the following 
techniques is used to detect DNA using a 
labelled probe for hybridisation? 



[ Q: 2273 ] MRCPass - 2011 May 

A 30 year old Caucasian man 
presented with 2 weeks of gradually 
worsening vision in his left eye with 
alterations in seeing colours. The patient also 
describes pressure and tightness with left eye 
movement for the past week. The patient 
reported his vision in the left eye worsened 
when show ering. On examination, there was 
no ptosis. His pupils were reactive to light, and 
there was a left afferent pupillary defect. Slit 
lamp exam was normal. Goldmann visual 
fields and showed a central scotoma on the 
left. 



1- Northern blotting 

2- Southern blotting 

3- Eastern blotting 

4- Western blotting 

5- Polymerase chain reaction 


What is the likely diagnosis? 

1- Acute glaucoma 

2- Optic neuritis 

3- Retinitis pigmentosa 

4- Anterior uveitis 


Dr. Khalid Yusuf El-Zohry - Sohag Teaching Hospital (01118391123) 

Ref MRCPass OE OE 2012 PasTest 2009 PassMedicine 2009 PasTest Exam 




ReviseMRCP 

911 

























El-zohry MRCP Questions Bank (Port 1) - 2013 


(For my personal use) 


5- Retinal vein thrombosis 


Answer & Comments 

Answer: 2- Optic neuritis 

Optic neuritis typically presents with a triad of 
symptoms: loss of vision, colour changes and 
eye pain. 

The initial attack is unilateral in 70% of adult 
patients and bilateral in 30%. The mean age of 
onset of optic neuritis is in the third decade of 
life, but can occur FROM the first to the 
seventh decades.Associated visual symptoms 
are reduced perception of light intensity and 
Uhthoff's symptom (visual deficit induced by 
exercise or increased body temperature). The 
visual loss may be subtle or profound. 

The most common etiology is multiple 
sclerosis. Up to 50% of patients with MS will 
develop an episode of optic neuritis. Some 
other causes of optic neuritis include infection 
( Syphilis, Lyme disease, herpes zoster), 
autoimmune disorders (SLE) and drugs (e.g. 
chloramphenicol, Ethambutol). 


surface of the plasma membrane and 
transmembrane receptors of hormones. 



[ Q: 2274 ] MRCPass - 2011 May 

G-proteins coupled receptors are 
located in which part of the cell? 

1- Cell membrane 

2- Mitochondrion 


3- Nucleus 


4- Cytoplasm 

5- Nuclear membrane 


Answer & Comments 

Answer: 1- Cell membrane 

G proteins are so-called because they bind the 
guanine nucleotides GDP and GTP. 

They are heterotrimers (i.e., made of three 
different subunits) associated with the inner 


Dr. Kholid Yusuf El-Zohry - Sohog Teaching Hospital (01118391123) 

Ref MRCPass OE OE 2012 PasTest 2009 PassMedicine 2009 PasTest Exam ReviseMRCP 
























El-zohry MRCP Questions Bank (Port 1) - 2013 


(For my personal use) 



[Q: 2275] MRCPass - 2011 
September 

A 55-year-old woman presents with periods of 
sweats and tremors which are relieved by 
eating. She has gained approximately 6 kg in 
weight in the last 2 years. Her BM is 4.5. Blood 
tests are: Hb 13 g/dl, MCV 78 fl, WCC 7 x 10 9 /l, 
platelets 200 x 10 9 /l, sodium 135 mmol/l, 
potassium 4.7 mmol/l, urea 5 mmol/l, 
creatinine 100 |imol/l, TSH - 3.3 (0.3-4) mU/l, 
free T4 -20 (10-24) pmol/l. 

Whot is the most appropriate investigation? 

1- 72 hour fast 

2- CT scan of pancreas 

3- MRI of the brain 

4- Insulin C-peptide concentration 

5- Oral glucose tolerance test 

Answer & Comments 

Answer: 1- 72 hour fast 

This patient has symptoms suggestive of 
hypoglycaemia which are relieved by 
carbohydrate. 

The likely cause is an insulinoma which is an 
insulin secreting pancreatic tumour. 


pupil was larger than the left. The pupillary 
reflex on the right eye was also sluggish both 
to light and accomodation. Eye movements 
and fundoscopy were normal. 

What is the diagnosis? 

1- Argyll Robertson pupil 

2- Horner's syndrome 

3- Holmes Adie pupil 

4- Myasthenia gravis 

5- 3rd nerve palsy 

Answer & Comments 

Answer: 3- Holmes Adie pupil 

Holmes-Adie syndrome (HAS) is a neurological 
disorder affecting the pupil of the eye and the 
autonomic nervous system. 

It is characterised by one eye with a pupil that 
is larger than normal and constricts slow ly in 
bright light (tonic pupil), along with the 
absence of deep tendon reflexes, usually in 
the Achilles tendon. HAS is thought to be the 
result of a viral or bacterial infection that 
causes inflammation and damage to neurons 
in the ciliary ganglion. It is more common in 
women than men. 


The best way of confirming the diagnosis is 
with a 72 hour fast. During the fast, the 
patient with an insulinoma may get episodes 
of hypoglycaemia with measured 
inappropriately high insulin C peptide 
(endogenous insulin). 

Measurement of C-peptide is useful in 
excluding factitious hypoglycaemia from self 
injection of insulin. Insulin preparations do not 
contain C-peptide. 


1 g i 

A 31 year old female patient was referred by 
the GP for assessment of unequal sized pupils. 
On examination, it was found that the right 


[Q: 2276] MRCPass-2011 
September 



[Q: 2277] MRCPass-2011 
September 


A 41-year-old female patient presents with a 
1-year history of recurrent episodes of vertigo. 
The vertigo spells are described as a sensation 
of the room spinning that lasts from 20 
minutes to a few hours and may be associated 
with nausea and vomiting. The spells are 
incapacitating and are accompanied by 
dizziness, vertigo, and disequilibrium, which 
may last for days. 


The patient also reports tinnitus, and hearing 
loss in the right ear that is more pronounced 
around the time of her vertigo spells. Physical 
examination of the head and neck is normal. 
There was no nystagmus. She is unable to 


Dr. Khalid Yusuf El-Zohry - Sohag Teaching Hospital (01118391123) 

Ref MRCPass OE OE 2012 PasTest 2009 PassMedicine 2009 PasTest Exam ReviseMRCP 



913 


























El-zohry MRCP Questions Bank (Port 1) - 2013 


(For my personal use) 


maintain her position during Romberg's 
testing. She turns tow ards the right side and 
she is unable to walk tandem. Her cerebellar 
function tests are normal. 

What is the diagnosis? 

1- Cerebellar disorder 

2- Vestibular neuronitis 

3- Meniere's disease 

4- Labyrinthine disease 

5- Internuclear opthalmoplegia 


Answer & Comments 

Answer: 3- Meniere's disease 

Meniere's disease is a disorder of the inner 
ear that can affect hearing and balance to a 
varying degree. 

It is characterized by episodes of vertigo and 
tinnitus and progressive hearing loss, usually 
in one ear. Nystagmus can occur, but is not 
typical. Vestibular neuronitis and larynthine 
disease do not typically cause deafness. 


A 26 year-old woman with a history of 
depression is brought to the hospital with 
decreased conscious level and a brief seizure. 
She had taken an overdose of tricyclic 
antidepressants 12 hours prior. Her GCS is 
12/15, she is tachycardic to 120 bpm and her 
blood pressure is 96/62 mmHg. She appears 
flushed and her skin is dry. A blood gas shows: 

pH -7.15 

p02 -13.3 kPa 

pC02 -3.5 kPa 

base excess - negative 8.5 

What should be given? 

1- Gastric Lavage 

2- Charcoal 

3- 8.4% bicarbonate infusion 



[Q: 2278] MRCPass-2011 
September 


4- Naloxone infusion 

5- Flumazenil infusion 


Answer & Comments 

Answer: 3- 8.4% bicarbonate infusion 

Many of the inital signs in tricyclic 
antidepressant (TCA) overdose are associated 
to the anticholinergic effects of TCAs such as 
dry mouth, blurred vision, urinary retention, 
constipation, dizziness and vomiting. 

In a patient who is acidotic who is at risk of 
cardiac arrhythmias and seizures, serum 
bicarbonate is recommended. 



[Q: 2279] MRCPass-2011 
September 


A 56 year old man with schizophrenia has 
been prescribed olanzepine. 

Which one of the following is a side effect? 

1- Hirsutism 

2- Impotence 

3- Diarrhoea 

4- Weight gain 

5- Malignancy 


Answer & Comments 

Answer: 4- Weight gain 

Olanzepine is an antipsychotic which is said to 
block serotonin receptors. 

Antagonism of dopamine receptors is 
associated with extrapyramidal effects such as 
tardive dyskinesia. Antagonizing HI histamine 
receptors causes sedation and may cause 
weight gain (90% of users). Dry mouth, 
sedation and urinary retention are other side 
effects. 


Dr. Khalid Yusuf El-Zohry - Sohag Teaching Hospital (01118391123) 

Ref MRCPoss OE OE 2012 PosTest 2009 PassMedicine 2009 PosTest Exam ReviseMRCP 



914 


























El-zohry MRCP Questions Bank (Port 1) - 2013 


(For my personal use) 



[Q: 2280] MRCPass - 2011 
September 


A 25 year old lady returned from Hong Kong 
(she had been there for 2 months). 2 weeks 
after she returned to the UK and she 
complained of fever, chills, severe myalgias 
and arthalgia. Past medical, surgical, family 
history and review of systems were 
unremarkable. On examination, temperature 
was 39.6 C. There was a generalized petechial 
rash in the lower part of the body. Her blood 
tests revealed the following results: 


Hb 11.0 g/dl, 
WCC 12 x 10 9 /l, 


platelets 250 x 10 9 /l 


aspartate transaminase (AST) of 319 (1-31) 
U/L 


alanine transaminase (ALT) of 198 (5-35) U/L 
alkaline phosphatase of 74 (20-120) U/L 
What is the most likely diagnosis? 

1- Dengue fever 

2- Tyhoid fever 


3- Lassa virus 


4- Acute HIV infection 


5- p falciparum malaria 


Answer & Comments 

Answer: 1- Dengue fever 

Fever, thrombocytopenia and petechial rash 
are typical of dengue fever. 


fever. Rash, gastrointestinal pain, and diarrhea 
are common manifestations of the syndrome. 

The fever typically lasts for 5-7 days. It is often 
self limiting. Upon resolution of the fever, 
patients may run the risk of hemorrhagic 
symptoms (epistaxis, ecchymoses, and 
gastrointestinal bleeding) and plasma leakage 
syndrome (hemoconcentration, ascites, or 
pleural effusions) called dengue hemorrhagic 
fever. Dengue hemorrhagic fever may have a 
mortality of 5% in those untreated. Treatment 
is supportive ( e.g. iv fluids) 


n 

A 63 year old man presents with an episode of 
amnesia for the second time in two months. 2 
days ago he had an episode of confusion, 
according to his wife. He was, However, able 
to have a normal conversation despite having 
been found wandering. After 2 hours, he 
abruptly returned to normal and could not 
remember what happened. 

What is the most likely diagnosis? 

1- Alcoholic encephalopathy 

2- Subarachnoid haemorrhage 

3- Complex partial seizure 

4- Transient ischaemic attack 

5- Transient global amnesia 

Answer & Comments 

Answer: 5- Transient global amnesia 


[Q: 2281] MRCPass-2011 
September 


Dengue fever is caused by a 
mosquitotransmitted flavivirus that is endemic 
throughout much of the tropical world. 
Symptoms arise 4-7 days after the bite of a 
mosquito, but can be as short as 3 days or a 
long as 14 days following insect exposure. The 
clinical syndrome of classic dengue fever is 
characterized initially by headache, retro- 
orbital eye pain, and severe myalgias and 
arthalgias originally termed "breakbone" 


Transient global amnesia (TGA) is a syndrome 
in clinical neurology whose key defining 
characteristic is temporary but almost total 
disruption of short-term memory with a range 
of problems accessing older memories. 

A person in a state of TGA exhibits no other 
signs of impaired cognitive functioning but 
recalls only the last few moments of 
consciousness plus deeply-encoded facts of 
the individual's past, such as his or her own 


Dr. Khalid Yusuf El-Zohry - Sohag Teaching Hospital (01118391123) 

Ref MRCPass OE OE 2012 PasTest 2009 PassMedicine 2009 PasTest Exam ReviseMRCP 



915 



























El-zohry MRCP Questions Bank (Port 1) - 2013 


(For my personal use) 


name. It may last several hours. The most 
commonly cited precipitating events include 
vigorous exercise (including sexual 
intercourse), sw imming in cold water or 
enduring other temperature changes, and 
emotionally traumatic or stressful events. 



[Q: 2282] MRCPass - 2011 
September 


A 67 year old lady has epigastric pain for 
several months and is referred for endoscopy. 
Biopsy confirms MALT lymphoma. 

Whot is the treatment of choice? 

1- Chemotherapy 

2- Radiotherapy 

3- Interferon 

4- Surgery 

5- H.pylori eradication 


forced expiratory volume in one second 
(FEV1) of 2.09 I (predicted 3.18 I) 

forced vital capacity (FVC) of 2.33 I (predicted 
4.04). 

Carbon monoxide transfer factor (TLCO) was 
reduced to 67% predicted. 

There was 5% improvement following 
salbutamol nebulisers. 

Whot is the likely diagnosis? 

1- COPD 

2- Asthma 

3- Pulmonary embolus 

4- Interstitial lung disease 

5- Pneumothorax 


Answer & Comments 


Answer: 4- Interstitial lung disease 


Answer & Comments 

Answer: 5- H.pylori eradication 

Most cases of MALT lymphoma affecting the 
stomach are associated with infection by a 
bacterium called Helicobacter pylori (often 
abbreviated to H. 

pylori). If tests confirm its presence then a 
course of intensive antibiotic treatment will 
sometimes lead to a remission of the 
lymphoma. 



[Q: 2283] MRCPass-2011 
September 

A 50-year-old woman was admitted because 
of progressive shortness of breath. She has 
been a smoker of 5 cigarettes a day for the 
past two years. She has a past medical history 
of seropositive rheumatoid arthritis, 

diabetes mellitus and hypertension. On 
examination the fingers were clubbed and 
there were bilateral basal crepitations. 

Lung function tests showed : 


Although there is slight improvement with 
salbutamol, the best answer is interstitial lung 
disease/ pulmonary fibrosis. 

The FEV1 / FVC ratio is 90% which suggests 
restrictive lung disease rather than obstructive 
lung disease. 

There is also reduced transfer factor. This is 
likely to be due to the history of rheumatoid 
arthritis in this case. 



[Q: 2284] MRCPass-2011 
September 


A 36 year old Caucasian female presented with 
malaise, joint pains , Raynaud's phenomenon 
and shortness of breath for the last 6 months. 
On physical examination she was afebrile and 
had a supine blood pressure of 110/80mm Hg. 
Her apex beat was displaced, she had a loud 
P2 and a right ventricular heave. There were 
fine basal crepitations in both lung bases on 
auscultation. Blood tests revealed: 


Hb 11.5 g/dl, MCV 85 fl 


Dr. Kholid Yusuf El-Zohry - Sohog Teaching Hospital (01118391123) 

Ref MRCPass OE OE 2012 PasTest 2009 PassMedicine 2009 PasTest Exam ReviseMRCP 



916 



























El-zohry MRCP Questions Bank (Port 1) - 2013 


(For my personal use) 


erythrocyte sedimentation rate of 80 mm/first 
hour 

sodium 135 mmol/l 
potassium 4.5 mmol/l 
urea 18 mmol/l 
creatinine 285 |imol/l 

antinuclear antibody (ANA) - strongly positive 

antitopoisomerase I antibody (formerly anti 
SCL-70 antibody) positive 

normal C3 and C4 

anti-DNA, anti-centromere, anti-RNP, anti-Ro 
and La antibodies - negative 

Whot is the likely diagnosis? 

1- Hereditary haemorrhagic telangiectasia 

2- Sjogren's syndrome 

3- Wegener's granulomatosis 

4- Oesophageal carcinoma 



[Q: 2285] MRCPass - 2011 
September 


A blood test has been used to screen for the 
likelihood of gastric cancer. The results are as 
follows: 



Cancer Diagnosed 

No Cancer 

Positive 

60 

60 

Negative 

80 

40 


What is the positive predictive value? 


1- 25% 

2- 33.3% 

3- 50% 

4- 60% 

5- 66.6% 


Answer & Comments 


Answer: 3- 50% 


5- Diffuse systemic sclerosis 

Answer & Comments 

Answer: 5- Diffuse systemic sclerosis 

The patient is likely to have a diffuse form of 
scleroderma. 


The positive predictive value of a test is the 
probability that the patient has the disease 
when restricted to those patients who test 
positive. 

This term is sometimes abbreviated as PPV. 
You can compute the positive predictive value 
as 


The limited cutaneous form of scleroderma is 
CREST syndrome (calcinosis, raynauds, 
esophageal dysmotility, sclerodactyly and 
telangiectasia). 

The diffuse form of scleroderma is more 
rapidly progressing and affects the skin 
(cutaneous scleroderma) and one or more 
internal organs, frequently the kidneys (renal 
crisis), esophagus, heart (pulmonary 
hypertension) and lungs (pulmonary fibrosis). 

In diffuse scleroderma, antinuclear antibodies 
are present in about 95% of patients. 

Topoisomerase I antibodies (formerly Scl-70) 
are present in approximately 30% of patients 
with diffuse disease (absent in limited disease) 
and are associated with pulmonary fibrosis. 


PPV = TP/(TP + FP) 

where TP and FP are the number of true 
positive and false positive results, 
respectively. In this case, the TP is 60, FP is 60 
and PPV is 60/120 = 50%. 


A 42-year-old female presented with an 
erythematous annular patch with central 
clearing on her left foot. She has returned 
from a walking holiday 1 week ago. The 
patient mentioned that the rash has gotten 
progressively larger and spreading up the leg 
over the last 3 weeks and she has had a recent 
onset of intermittent joint pains. 



[Q: 2286] MRCPass - 2011 
September 


Dr. Khalid Yusuf El-Zohry - Sohag Teaching Hospital (01118391123) 

Ref MRCPass OE OE 2012 PasTest 2009 PassMedicine 2009 PasTest Exam ReviseMRCP 
































El-zohry MRCP Questions Bank (Port 1) - 2013 


(For my personal use) 


On examination, there is a large area of 
erythema with a central clearing over the calf 
and the foot. 

What test is likely to confirm the diagnosis? 

1- Borrelia burgdoferi 

2- Wuchereria bancrofti 

3- Spirochaetes 

4- Echinococcus granulosus 

5- Trypanosomiasis cruzi 


Answer & Comments 

Answer: 1- Borrelia burgdoferi 

Lyme Disease (LD) is a multisystem disease 
affecting the nervous system, skin, joints, and 
heart. 

It is endemic in the 

temperate regions of the northern 
hemisphere (United States, Europe, Canada). 
Erythema migrans (EM), the characteristic 
dermatologic lesion of LD, is an expanding red 
papule or macule with central clearing, often 
found in the axilla, midriff, or popliteal areas. 

B. burgdorferi is transmitted by the tick 
Ixodes, a hard bodied tick found in wooded 
areas. Lyme disease occurs in 3 stages: early 
localized, early disseminated, and late. If left 
untreated, each stage progresses to the next. 
Early localized disease manifests within 3-30 
days presenting with erythema migrans (EM), 
myalgia, fatigue, headache, fever, 
lymphadenopathy, and arthralgia. Early 
disseminated disease occurs 30 to 120 days 
post-infection and is characterized by EM 
(single or multiple), fatigue, 
lymphadenopathy, conjunctivitis, neck pain, 
cardiac abnormalities, radiculoneuritis, 
arthritis, and CNS manifestations. Late disease 
manifests from 4 months to 1 year, presenting 
with fatigue, chronic arthritis, CNS 
manifestations, and encephalopathy. 

First-line treatment for early disease is 
doxycycline (100 mg PO twice a day for 14 to 


21 days) or amoxicillin (500 mg PO three times 
a day for 14 to 21 days). 


3 

A 18 year old man has presented for 
investigation with haematuria. On his urine 
dipstick, there were blood ++. 

When enquired about family history, he said 
his father and older brother also had 
haematuria. An ANCA and ANA screen was 
done with negative results. An ultrasound of 
the kidney was normal and his creatinine was 
80 umol/l. 

What is the likely diagnosis? 

1- Polyarteritis nodosa 

2- Systemic lupus erythematosus 

3- Alport's syndrome 

4- Ig A nephropathy 

5- Exercise induced haematuria 


[Q: 2287] MRCPass-2011 
September 


Answer & Comments 

Answer: 3- Alport's syndrome 

Alport's syndrome leads to a 
glomerulonephritis. 

It is a primary basement membrane disorder 
arising from mutations in genes encoding 
several members of the type IV collagen 
protein family. The disease is mainly inherited 
in the X linked form. In males, there is only 
one X chromosome, so the disease tends to 
manifest in males with the immediate family 
as suggested above. 

The clinical manifestations include recurrent 
episodes of gross hematuria, especially in 
childhood, as in the case vignette. 
Hypertension, proteinuria and sensorineural 
hearing loss can take place although those 
clues were not given in this scenario. 


Dr. Khalid Yusuf El-Zohry - Sohag Teaching Hospital (01118391123) 

Ref MRCPoss OE OE 2012 PosTest 2009 PassMedicine 2009 PosTest Exam ReviseMRCP 























El-zohry MRCP Questions Bank (Port 1) - 2013 


(For my personal use) 



[Q: 2288] MRCPass - 2011 
September 

A 28 year old girl develops sudden onset left 
sided weakness and dysarthria which resolves 
fairly promptly. She has just returned from 
Australia 2 days previously. Physical 
examination is normal. 

What test is likely to identify the underlying 
couse? 

1- Transthoracic echocardiogram 

2- Transoesophageal echocardiogram 

3- Carotid dopplers 

4- MRI of the brain 

5- EEG 


Answer & Comments 

Answer: 2- Transoesophageal echocardiogram 

This patient had a transient ischaemic attack. 

As she is young, she is most likely to have a 
patent foramen ovale which will be diagnosed 
with a transoesophageal echocardiogram 
(usually with a bubble contrast study) 


The purine analogues azathioprine and 
mercaptopurine are effective in inducing and 
maintaining remission in patients with 
ulcerative colitis and Crohn's disease. 

Azathioprine is a prodrug which is converted 
to mercaptopurine and the enzyme thiopurine 
methyltransferase (TPMT) breaks down 
mercaptopurine. Deficiency of the enzyme is 
associated with a greater risk of 
myelosuppression. 


[Q: 2290] MRCPass - 2011 
September 

A 43-year-old man has recently been 
diagnosed with non-Hodgkin's lymphoma. He 
has a long history of alcoholism and has 
significant alcohol-related peripheral 
neuropathy. 

Which one of the following chemotherapy 
agents should be avoided? 

1- Chlorambucil 

2- Cyclophosphamide 

3- Epirubicin 

4- Vincristine 




[Q: 2289] MRCPass-2011 
September 


A 36 year old lady has Crohn's disease with 
poorly controlled symptoms. She also has a 
history of chronic anaemia. 


Which enzyme should be checked before 
starting the drug azathioprine? 


1- Thiopurine methyltransferase 


2- Glycogen phosphorylase 

3- Creatinine kinase 


4- Myeloperoxidase 

5- Acetylcholinesterase 


Answer & Comments 


Answer: 1- Thiopurine methyltransferase 


5- Rituximab 


Answer & Comments 

Answer: 4- Vincristine 

Vincristine (brand name, Oncovin), also known 
as leurocristine, is a vinca alkaloid. 

It works through disruption of the 
microtubules which in turns disrupts 
metaphase in mitosis. Its main uses are in 
non-Hodgkin's lymphoma as part of the 
chemotherapy regimen CHOP, Hodgkin's 
lymphoma as part of MOPP, COPP, BEACOPP. 

The main side-effects of vincristine are 
peripheral neuropathy (which can be severe), 
hyponatremia and hair loss. 


Dr. Khalid Yusuf El-Zohry - Sohag Teaching Hospital (01118391123) 

Ref MRCPass OE OE 2012 PasTest 2009 PassMedicine 2009 PasTest Exam ReviseMRCP 
































El-zohry MRCP Questions Bank (Port 1) - 2013 


(For my personal use) 


^ [ Q: 2291 ] MRCPass - 2011 

S September 

A 60 year old lady presented with a fall but did 
not sustain a fracture. She experienced 
menopause in her early 50s and initiated 
hormone therapy (HT) with 
es-trogen/progestin for her menopausal 
symptoms. She has polymyalgia rheumatica 
and has been on prednisolone for the last 1 
year. She was organised to have a DEXA scan. 
This showed a T score of -2.6 in the hip. 

Whot should be prescribed? 

1- Vitamin D 


He also seemed to make up events which he 
did not remember. On examination, he had an 
MMSE score of 22 / 30. He was tremulous in 
both his hands and had an ataxic gait. Tone, 
power and reflexes were normal in both upper 
and lower limbs. 

What is the likely diagnosis? 

1- Alzheimer's dementia 

2- Lewy body disease 

3- Korsakoff's psychosis 

4- Creudsfeldt Jakob disease 

5- Normal pressure hydrocephalus 


2- Calcichew 

3- Teriparatide 

4- Bisphosphonates 

5- Raloxifene 


Answer & Comments 

Answer: 4- Bisphosphonates 

This patient is postmenopausal and is likely to 
have steroid related osteoporosis. 

Bisphosphonates are thought to inhibit the 
activation and function of osteoclasts, and are 
the drug of choice in this scenario. Examples 
are Alendronate and risedronate, which are 
licensed for the treatment of osteoporosis in 
post-menopausal women, and the prevention 
of osteoporosis in those post-menopausal 
women considered to be at risk. The NICE 
guidelines recommend that bisphosphonates 
are used as treatment for preventing bone 
fractures in postmenopausal women who 
have had osteoporosis diagnosed but have not 
had a fracture. 


A 59 patient has been admitted with 
confusion. The patient's relatives describe that 
he does not remember the events that 
occurred in the last day but recognised them. 



[Q: 2292] MRCPass - 2011 
September 


Answer & Comments 

Answer: 3- Korsakoff's psychosis 

Alcohol withdrawal delirium (delirium 
tremens) is the clinical syndrome of 
disorientation, perceptual disturbance and 
psychomotor agitation. 

Visual hallucinations are commonly 
associated. 

Korsakoff's psychosis is associated short term 
memory loss, subsequent compensatory 
confabulation by patient. 

Other symptoms may include delirium, 
anxiety, fear, depression, confusion, delusions 
and insomnia. 


[Q: 2293] MRCPass-2011 
September 

A 61 year old lady has pain in her knees, 
shoulders, wrists and fingers. Examination of 
her hands reveals multiple symmetrical small 
joint involvement. The proximal and distal 
joints were affected. Joint X rays show the 
presence of osteophytes and 
chondrocalcinosis. She is currently on 
bendrofluazide and metformin tablets. A urate 
level on admission was 420 (<380 |imol/l). The 
rheumatoid factor was positive with a titre of 
1:30. 



Dr. Khalid Yusuf El-Zohry - Sohag Teaching Hospital (01118391123) 

Ref MRCPass OE OE 2012 PasTest 2009 PassMedicine 2009 PasTest Exam ReviseMRCP 



920 



























El-zohry MRCP Questions Bank (Port 1) - 2013 


(For my personal use) 


What is the diagnosis? 

1- Polyarticular gout 

2- Pseudogout 

3- Systemic lupus erythematosus 

4- Haemochromatosis 


4- Post streptococcal glomerulonephritis 

5- Goodpasture's syndrome 


Answer & Comments 


Answer: 3- IgA nephropathy 


5- Rheumatoid arthritis 


Answer & Comments 

Answer: 2- Pseudogout 

The diagnosis would fit with a subcategory of 
calcium pyrophosphate deposition disease. 

Pseudogout is one manifestation of calcium 
pyrophosphate deposition disease, where 
joint aspiration fluid might show 
rhomboidshaped, positively birefringent 
crystals. It affects the knees most commonly 
and can involve the proximal interphalangeal 
(PIP) joints and spine. Osteophytes and 
chondrocalcinosis are a common radiological 
finding . In addition, older individuals may 
have low -titer-positive rheumatoid factor as 
in this case. 


[Q: 2294] MRCPass - 2011 
September 

A 21 year old man was referred for pink 
discolouration of his urine to the hospital. He 
had no previous relevant medical history. 3 
days ago he complained of a sore throat and 
was given a course of amoxicillin and 
ibuprofen by the GP but those symptoms have 
resolved now . On examination, he looked 
well. His blood pressure was 120/70 mmHg, 
temperature 36 C. There were normal 
abdominal examination and he had no 
palpable organomegaly. Urine dipstick showed 
blood ++, Protein +, nitrites negative. 

What is the most likely diagnosis? 

1- Crescentic glomerulonephritis 

2- Wegener's granulomatosis 

3- IgA nephropathy 



IgA nephropathy is the most common 
glomerulonephritis and is characterized by 
deposition of the IgA antibody in the 
glomerulus. 

The classic presentation (in 40-50% of the 
cases) is episodic frank hematuria which 
usually starts within a day or two of a non¬ 
specific upper respiratory tract infection. The 
common differential is post-streptococcal 
glomerulonephritis which typically occurs 
weeks after initial infection. The gross 
hematuria resolves after a few days, though 
microscopic hematuria may persist. Renal 
function usually remains normal. Mild 
proteinuria can also be associated. 


[Q: 2295] MRCPass-2011 
September 

A 43 year old man is known to have HIV. He 
develops multiple fleshy, red nodules on the 
trunk which were diagnosed as Kaposi's 
sarcoma. 

What is the aetiological agent? 

1- Cytomegalovirus 

2- Ebstein barr virus 

3- Coronavirus 

4- HHV 8 

5- Parvovirus 



Answer & Comments 

Answer: 4- HHV 8 

HHV-8, is a gammaherpesvirus found only in 
humans. 

Kaposi's sarcoma (KS) is caused by Human 
herpesvirus 8 (HHV8) and frequently found in 


Dr. Khalid Yusuf El-Zohry - Sohag Teaching Hospital (01118391123) 

Ref MRCPass OE OE 2012 PasTest 2009 PassMedicine 2009 PasTest Exam 




ReviseMRCP 

921 



























El-zohry MRCP Questions Bank (Port 1) - 2013 


(For my personal use) 


patients with HIV infection. Kaposi's sarcoma 
lesions may appear like bruises but are 
papular. With time, they darken. Scarring is 
common following treatment with 
immunosuppressive drugs. 


A 18 year old man has a a history of 2 months 
of bumps in the left axilla. Initially there was 
just 1 lesion but now the man has 6 lesions. 
Some of the areas have been inflamed, and 
the man has pruritus, which keeps him up at 
night. One of his cousins, with whom he swam 
with regularly, may have such lesions as well. 
He develops multiple fleshy, red nodules on 
the trunk. These were treated but healed with 
mild scarring. 

Which one of the following is most likely? 

1- Molluscum contagiosum 

2- Kaposi's sarcoma 

3- Human papillovirus infection 

4- Herpes zoster infection 

5- Herpes simplex infection 



[Q: 2296] MRCPass - 2011 
September 


Answer & Comments 

Answer: 1- Molluscum contagiosum 

Molluscum contagiosum is a viral skin disease 
characterised by firm, round, translucent, 
umbilicated papules containing caseous 
matter and peculiar capsulated bodies. 

It is caused by a DNA virus of pox family. This 
common viral disease has a higher incidence 
in children, sexually active adults, and those 
who are immunodeficient. It can appear with 
crops and can be treated with cryotherapy. It 
tends to be self limiting. 



[Q: 2297] MRCPass-2011 
September 


What is the mechanism of action of the drug 
aspirin ? 


1- Monoclonal antibody 

2- Cyclooxygenase inhibitor 

3- Glycoprotein llb/llla inhibitor 

4- ADP antagonist 

5- Low molecular weight heparin 


Answer & Comments 

Answer: 2- Cyclooxygenase inhibitor 

Aspirin also known as acetylsalicylic acid is a 
salicylate drug. 

Aspirin is classified under nonsteroidal anti¬ 
inflammatory drugs (NSAIDs), but differs from 
them in the mechanism of action. Like other 
NSAIDS, it inhibits the same enzyme 
cyclooxygenase (COX), However it affects 
more the COX-1 variant than the COX-2 
variant of the enzyme 



[Q: 2298] MRCPass-2011 
September 


A 40-year-old man presented with an initial 
complaint of dyspnea on exertion that had 
developed five years prior and had progressed 
to shortness of breath while walking up one 
flight of stairs. He has a history of 
osteoarthritis. He experienced significant 
inorganic dust exposure while working as a 
builder. He does not smoke and drinks 2 units 
of alcohol per day. 


Physical examination revealed pulmonary 
auscultation that was remarkable for a 
prolonged expiratory phase without wheezes 
or rhonchi. The remainder of his physical 
examination was unremarkable. A chest x ray 
showed hyperinflated lungs with large bullae 


Pulmonary function testing at presentation 
demonstrated : 


forced expiratory volume in one second 
(FEV1) was 2.10 litres (61% of the predicted 
value) 


Dr. Khalid Yusuf El-Zohry - Sohag Teaching Hospital (01118391123) 

Ref MRCPass OE OE 2012 PasTest 2009 PassMedicine 2009 PasTest Exam ReviseMRCP 



922 


























El-zohry MRCP Questions Bank (Port 1) - 2013 


(For my personal use) 


forced vital capacity (FVC) was 3.60 litres (81% 
of the predicted value) the FEV1:FVC ratio was 
0.58 

total lung capacity (TLC) was 6.40 litres (93% 
of the predicted value) 

residual lung volume (RV) was 2.91 litres 
(123% of the predicted value) 


Stool weight chart (fasting - 850 mis / day) 
Faecal stool osmolality 295 mosmol/kg 
What is the likely diagnosis? 

1- Laxative abuse 

2- VIPoma 

3- Giardiasis 


There was minimal response to an inhaled 
bronchodilator. 

What is the likely diagnosis? 

1- Smoking related COPD 

2- Bronchiectasis 


4- Irritable bowel syndrome 

5- Crohn's disease 


Answer & Comments 


Answer: 2- VIPoma 


3- Alpha 1 antitrypsin deficiency 

4- Silicosis 

5- Usual interstitial pneumonitis 


Answer & Comments 

Answer: 3- Alpha 1 antitrypsin deficiency 

Alpha 1-antitrypsin deficiency is an autosomal 
recessive genetic disorder caused by defective 
production of alpha 1-antitrypsin (A1AT), 
leading to decreased A1AT activity in the 
lungs, and deposition of excessive abnormal 
A1AT protein in liver cells. 

Severe A1AT deficiency causes panacinar 
emphysema or COPD in adult life. The case 
above demonstrated severe emphysema, and 
in a non smoker with young age onset the 
patient is likely to have an inherited cause 
such as A1AT deficiency. 


A 28 year old lady complained of watery 
diarrhoea for the past few weeks. On 
investigation of blood tests and stool 
collections, the following results were 
obtained: 

Na 138 mmol/l 

K 2.5 mmol/l 


[Q: 2299] MRCPass - 2011 
September 



VIPoma (Verner Morrison syndrome) is a rare 
(1 per 10,000,000 per year) endocrine tumor, 
usually (about 90%) originating in the 
pancreas, that produces vasoactive intestinal 
peptide (VIP). 

The massive amounts of VIP in turn cause 
profound and chronic watery diarrhea and 
resultant dehydration, hypokalemia, 
achlorhydria, acidosis, vasodilation (flushing 
and hypotension), hypercalcemia and 
hyperglycemia. 

Clinical diagnosis is based on a history of 
approximately 10 watery stools per day. 
Fasting stool volume > 750 to 1000 mL/day) is 
diagnostic. Fecal losses while fasting are at 
least 20 mL/kg/d but exceed 50 mL/kg/d in 
most cases. 

Fecal osmolality is entirely accounted for by 
twice the sum of the concentrations of sodium 
and potassium, indicating the electrolyte loss. 
Patients may complain about colicky 
abdominal pain or pain in the upper 
abdominal area radiating to the back. 



[Q: 2300] MRCPass-2011 
September 


A 46 year old woman experienced visual 
disturbance. The patient reported blurred 
vision and oscillopsia on downgaze, but did 
not complain of double vision. 


Dr. Khalid Yusuf El-Zohry - Sohag Teaching Hospital (01118391123) 

Ref MRCPass OE OE 2012 PasTest 2009 PassMedicine 2009 PasTest Exam ReviseMRCP 



























El-zohry MRCP Questions Bank (Port 1) - 2013 


(For my personal use) 


Ophthalmological examination showed 
decreased near visual acuity. Examination of 
eye motility showed bilateral weakness of the 
inferior and lateral rectus muscles. 

Downbeat nystagmus on down and lateral 
gaze was seen clinically. 

What is the likely diagnosis? 

1- Grave's eye disease 

2- Cerebellar tumour 

3- Arnold Chiari malformation 

4- Sagittal meningioma 

5- Horner's syndrome 


Answer & Comments 

Answer: 3- Arnold Chiari malformation 

In the Arnold-Chiari malformation a part of 
the brainstem and the cerebellum are 
herniated into the cervical vertebral canal. 

The cerebellar tonsils are elongated and 
pushed down through the opening of the 
foramen magnum, blocking the flow of 
cerebrospinal fluid. Clinical findings include 
oscillopsia, impaired smooth pursuit, and 
OKN, and in many cases downbeat nystagmus. 
Patients may experience no symptoms or 
remain asymptomatic until early adulthood, at 
which point they will often experience severe 
headaches and neck pain. Fatigue, dizziness, 
vertigo, neuropathic pain, visual disturbances, 
difficulty sw allowing, ringing in the ears may 
also occur. 



[Q: 2301] MRCPass - 2011 
September 


A 35 year old woman with anxiety is 
hyperventilating acutely after hearing stressful 
news. 


What would be expected on the arterial blood 
gas? 


1- Low P02 


2- High PC02 


3- Normal pH 

4- Low bicarbonate 

5- Low H+ ion 


Answer & Comments 

Answer: 5- Low H+ ion 

In a patient who is hyperventilating, C02 is 
blown off. 

Since carbon dioxide is carried as bicarbonate 
in the blood, the loss of carbon dioxide will 
drive bicarbonate to combine with hydrogen 
ions (protons) to form more carbon dioxide. 
The loss of hydrogen ions results in the blood 
becoming alkaline, i.e. the blood pH value 
rises. This is known as a respiratory alkalosis. If 
the hyperventilation continues, then a drop in 
bicarbonate levels may occur due to renal 
clearance after several hours, but the question 
refers to the acute situation where H+ is low , 
but not bicarbonate. 


A 25 year old known asthmatic was admitted 
with an acute exacerbation of her problem. 
She was given lOOmg of IV Hydrocortisone 
and 2 doses of salbutamol and ipratropium 
nebulization. Her PEFR was measured on 
admission as 200 (Predicted 550). After 2 
doses of salbutamol and Ipratropium 
nebulization the peak flow did not improve 
beyond 210 and she was still dyspnoeic. 

What should be the next step in the 
management? 

1- Salbutamol 

2- Aminophylline 

3- Na cromoglycate 

4- IV Magnesium 

5- Antibiotics 


[Q: 2302] MRCPass-2011 
September 



Dr. Khalid Yusuf El-Zohry - Sohag Teaching Hospital (01118391123) 

Ref MRCPass OE OE 2012 PasTest 2009 PassMedicine 2009 PasTest Exam ReviseMRCP 



924 
























El-zohry MRCP Questions Bank (Port 1) - 2013 


(For my personal use) 


Answer & Comments 

Answer: 4- IV Magnesium 

A single dose of IV magnesium sulphate has 
been shown to be safe and effective in acute 
severe asthma who did not had a good initial 
response to inhaled bronchodilator therapy in 
life threatening or near fatal asthma. 


[Q: 2303] MRCPass-2011 
September 

A 62 year old man with a history of excessive 
alcohol use presented with a fall and lost 
consciousness for several minutes. 

He was brought to hospital and was initially 
alert. However, whilst he was awaiting 
assessment he complained of a headache and 
his GCS deteriorated from 15 to 8. He also 
became acutely confused. 

What is the most likely diagnosis? 

1- Subarachnoid haemorrhage 

2- Subdural haemorrhage 

3- Diffuse axonal injury 

4- Epileptic seizure 

5- Somatization 



Answer & Comments 

Answer: 2- Subdural haemorrhage 

The patient who fell over may have injured his 
head and the most likely cause of a further 
sudden deterioration in conscious level with 
an associated headache is a subdural 
haemorrhage or haematoma. 

This patient needs an urgent CT scan in view 
of the deterioration. 




[Q: 2304] MRCPass-2011 
September 


A 72 year old woman has been referred for 
management of a blood pressure of 190/100 
mmHg. She has a history of bipolar disorder 


and peripheral vascular disease. She is 
currently on aspirin and lithium. 

Which one of the following is the best 
antihypertensive agent to commence? 

1- Valsartan 

2- Lisinopril 

3- Amlodipine 

4- Atenolol 

5- Doxazosin 


Answer & Comments 

Answer: 3- Amlodipine 

According to the British Hypertension Society 
guidelines, Patients who are > 55 in age or 
black should be on either a calcium channel 
blocker (C) or thiazide diuretic (D). 

Amlodipine is a calcium channel blocker hence 
the best option here. Both thiazides and ACE 
inhibitors can increase lithium concentration 
levels. 


[Q: 2305] MRCPass-2011 
September 

A 19 year old lady presents with amenorrhoea 
and is investigated in clinic. She is sexually 
active and had normal periods up till 1 year 
ago. Her pregnancy tests across 6 months 
were negative. The patient has a body mass 
index of 28. She has normal stature and 
cardiac, respiratory examination are normal. 

Laboratory evaluation reveals the following: 

prolactin level of 215 (50-450) ng/mL 

LH 22 (0.5-14.5) IU/L 

FSH 44 (1-11) IU/L 

(3-HCG - negative 

Which one of the following is most likely? 

1- Premature ovarian failure 

2- Panhypopituitarism 

3- Adrenal tumour 



Dr. Khalid Yusuf El-Zohry - Sohag Teaching Hospital (01118391123) 

Ref MRCPoss OE OE 2012 PosTest 2009 PassMedicine 2009 PosTest Exom 




ReviseMRCP 

925 































El-zohry MRCP Questions Bank (Port 1) - 2013 


(For my personal use) 


4- Polycystic ovary disease 

5- Turner's syndrome 

Answer & Comments 

Answer: 1- Premature ovarian failure 

Premature ovarian failure is defined as 
menopause occurring in women prior to the 
age of 40 years. 

Diagnosis requires elevated gonadotrophins - 
FSH above 40 IU per litre together with raised 
LH and low oestradiol (less than 100 pmol per 
litre) on at least two occasions. Ultrasound 
usually reveals small ovaries, a small uterus 
and a thin endometrium in premature ovarian 
failure. 


Cure rates for appropriately managed 
(including both medical and surgical therapies) 
native valve endocarditis are as follows: 

• For S viridans and S bovis infection, the rate 
is 98%. 

• For enterococci and S aureus infection in 
individuals who abuse intravenous drugs, the 
rate is 90%. 

• For community-acquired S aureus infection 
in individuals who do not abuse intravenous 
drugs, the rate is 60-70%. 

• For infection with aerobic gram-negative 
organisms, the rate is 40-60%. 

• For infection with fungal organisms, the rate 
is lower than 50%. 



[Q: 2306] MRCPass - 2011 
September 


A 62-year-old man with exertional dyspnea, 
fever and malaise. He had altered bow el habit 
for several weeks prior to his admission. On 
examination his temperature was 39.5 C , 
there was sinus tachycardia of 105 beats/min 
and respiratory rate was 32/minute. There 
was a systolic murmur along the apex and 
diastolic murmur at the right second 
intercostal area. He was arranged to have 
urgent trans-thoracic echocardiography which 
revealed vegetations on both the mitral 
valves. Blood cultures were taken. 


Which of the following organisms, if cultured 
confers the best prognosis? 

1- Streptococcus mitis 

2- Enterococcus 


3- Streptococcus viridans 

4- Staphylococcus aureus 

5- Streptococcus milieri 


[Q: 2307] MRCPass - 2011 
September 

A 58-year-old non alcoholic patient was 
admitted to hospital for investigation of 
hematemesis and melena. 

On admission, he looked pale and had a Hb of 
8 g/ dl. Urgent endoscopy was organised and 
this showed grade 2 oesophageal varices. 

What treatment should be undertaken? 

1- Intravenous octreotide 

2- Intravenous terlipressin 

3- Intravenous fluids 

4- Banding of varices 

5- Oral propanolol 



Answer & Comments 

Answer: 4- Banding of varices 

This patient has bleeding oesophageal varices 
and thus banding should be undertaken. 


Answer & Comments 


Answer: 3- Streptococcus viridans 


Oesophageal varices are graded according to 
their size, as follows: 

Grade 1 - Small, straight esophageal 
varices 


Dr. Khalid Yusuf El-Zohry - Sohag Teaching Hospital (01118391123) 

Ref MRCPass OE OE 2012 PasTest 2009 PassMedicine 2009 PasTest Exam ReviseMRCP 



926 


























El-zohry MRCP Questions Bank (Port 1) - 2013 


(For my personal use) 


Grade 2 - Enlarged, tortuous 

esophageal varices occupying less 

than one third of the lumen 
■ Grade 3 - Large, coil-shaped 

esophageal varices occupying more 
than one third of the lumen 



[Q: 2308] MRCPass - 2011 
September 

A 41-year-old man is investigated for 
deterioration in his liver function tests. 

It is decided to perform a liver biopsy. 

Which one of the following is o 
contraindication to liver biopsy? 

1- INRof 1.4 

2- ALT of 250 u/l 

3- Platelet count of 110 x 10 9 /l 

4- Obesity with BMI of 35 kg/m * 1 2 3 4 

5- Biliary duct dilatation on the ultrasound 


More information can be found in this 
document: 

http://gut.bmj.eom/content/45/suppl_4/IVl.f 

ull 



[Q: 2309] MRCPass-2011 
September 


A 16 year old girl presented with a 
polyarthritis and haematuria. She has noticed 
a discolouration of her urine and a rash in the 
legs. On examination, she has a non blanching 
purpuric rash in both her shins. Blood results 
show that the urea is 16 mmol/I and 
creatinine 210 umol/l. 


What is the most likely outcome of the renal 
involvement? 


1- High probability of relapse 

2- Complete renal recovery 

3- Persistent hypertension 

4- Development of nephrotic syndrome 

5- Long-term corticosteroids required 


Answer & Comments 

Answer: 5- Biliary duct dilatation on the 
ultrasound 

The best answer here is biliary duct dilatation, 
which increases the risk of infection as there 
might be cholestasis or cholecystitis. 

Many would consider obesity, but it is not an 
absolute contraindication. 

A short list of contraindications to liver biopsy 
are: 

Prolonged (>1.6) international normalized 
ratio (INR) 

The platelet count should exceed 60xl0 A 9/l 

There should be no biliary dilatation or major 
ascites 

Bleeding diathesis (eg, hemophilia) 


Answer & Comments 

Answer: 2- Complete renal recovery 

This patient is likely to have Henoch-Schonlein 
purpura (HSP), which is a self-limited systemic 
vasculitis. 

It is suspected to be triggered by an IgA- 
mediated response to an antigen. It is 
characterized by 4 clinical syndromes: 

1. Palpable purpura in the absence of 
thrombocytopenia or coagulopathy. Develops 
in 100% of patients. 

2. Arthritis/arthralgia in 45-75% of patients. 
Second most common manifestation of HSP. 

3. Abdominal pain in 50%, Gl bleeding (often 
occult) in 20-30% of patients. 

4. Renal disease in 20-50%. 

Full renal recovery is the commonest outcome 
(90%) in HSP. 


Dr. Khalid Yusuf El-Zohry - Sohag Teaching Hospital (01118391123) 

Ref MRCPass OE OE 2012 PasTest 2009 PassMedicine 2009 PasTest Exam 




ReviseMRCP 

927 























El-zohry MRCP Questions Bank (Part 1) - 2013 


(For my personal use) 


[Q: 2310] MRCPass - 2011 
September 

A 36 year old female who has been on thyroid 
replacement therapy has routine thyroid 
function tests. On examination, she appeared 
clinically euthyroid with no abnormal findings. 
Her blood tests showed: 

TSH 3.8 mU/L (0. 35 - 5.0) 

Total T4 18 nmol/L (55 -144) 

free T4 5.2 pmol/L (9 - 24) 

Total T3 2.4 nmol/L (0. 9 - 2.5) 

Which one of the following is the likely 
scenario? 

1- She is taking thyroid supplements 
unnecessarily 

2- She has secondary hypothyroidism 

3- She has sick euthyroid syndrome 

4- Her thyroid hormone replacement is 
adequate 

5- She should have a short synacthen test 



Answer & Comments 

Answer: 4- Her thyroid hormone replacement 
is adequate 

In hypothyroidism TSH provides a good 
measure of treatment adequacy and the 
picture is consistent with adequate 
replacement despite the low T4 levels. 


fl 

A 30 year old man presents with persistent 
diarrhoea and fevers. 

The diarrhoea did not improve despite fluids 
and ciprofloxacin treatment. He has known 
HIV infection and his most recent CD4 count 
was 45. 

Which one of the following is the likely 
pathogen? 

1- Salmonella 


[Q: 2311] MRCPass-2011 
September 


2- Mycobacterium avium intracellulare 

3- Shigella 

4- Campylobacter 

5- Rotavirus 


Answer & Comments 

Answer: 2- Mycobacterium avium 

intracellulare 

MAC rarely causes disease in individuals with a 
normal immune system. 

In patients with AIDS (CD4 count < 50), 
However, it is one of the most common 
serious opportunistic infections. Patients most 
commonly report persistent fever, night 
sweats, fatigue, weight loss, and anorexia. 
Abdominal pain or chronic diarrhea may result 
from involvement of retroperitoneal lymph 
nodes or gut mucosa, respectively. 



[Q: 2312] MRCPass-2011 
September 


A 42-year-old man has recently started 
treatment for pulmonary tuberculosis. He has 
a history of diabetes and osteoarthritis. Prior 
to starting treatment, which one of the 
following tests should be done? 


1- Full blood count 


2- Urea and electrolytes 

3- ESR 

4- Coagulation screen 

5- Liver function test 


Answer & Comments 

Answer: 5- Liver function test 

The standard treatment for tuberculosis is a 6- 
month, four-drug initial regimen (6 months of 
isoniazid and rifampicin supplemented in the 
first 2 months with pyrazinamide and 
ethambutol). 


Dr. Khalid Yusuf El-Zohry - Sohag Teaching Hospital (01118391123) 

Ref MRCPass OE OE 2012 PasTest 2009 PassMedicine 2009 PasTest Exam ReviseMRCP 
































El-zohry MRCP Questions Bank (Port 1) - 2013 


(For my personal use) 


Anti-tuberculosis chemotherapy is associated 
with abnormalities in liver function tests in 10- 
25% of patients. Several anti-tuberculosis 
agents have been implicated as being 
hepatotoxic. Isoniazid (particularly in 
association with rifampicin) and pyrazinamide 
cause hepatic dysfunction more frequently 
than ethambutol and streptomycin. 


A 38-year-old man went for a holiday in Belize. 
He presented with a non-healing ulcer on the 
nasal area after 6 weeks. The lesion started as 
an itchy red papule which slow ly enlarged 
into an ulcerated plaque. He remembered 
being bitten by sandflies during his stay in 
Belize. There were no systemic symptoms. The 
ulcer failed to heal despite several courses of 
systemic antibiotics. There was no relevant 
past medical or drug history of note. 

On examination, he had a was noted to have a 
2 cm x 1.8 cm crusted, ulcerated plaque on 
the upper, inner aspect of the left nasal area. 
There were no regional or generalized 
lymphadenopathy and no muco-cutaneous 
changes. 

What is the likely diagnosis? 

1- Behcet's disease 

2- Cutaneous leishmaniasis 

3- Syphilis 

4- basal cell carcinoma 



[Q: 2313] MRCPass - 2011 
September 


sandfly and remain subclinical. However, in 
some cases, after an incubation period of 1-12 
weeks, a papule develops that enlarges and 
ulcerates. The typical lesion are crusty, 
painless ulcers on exposed skin. 

Ulcerative lesions are usually shallow and 
circular with well-defined, raised borders and 
a bed of granulation tissue. Local 
lymphadenopathy only occurs in the presence 
of bacterial superinfection. Cutaneous 
leishmaniasis is found predominantly in South 
America, Central Africa, around the 
Mediterranean Sea and India 



[Q: 2314] MRCPass-2011 
September 


A 61 year old lady is being assessed for 
treatment of hypertension. She has a high 
blood pressure despite being on 
bendroflumethiazide. She has recently 
discontinued medications due to ankle 
oedema, gum bleeding and generalised 
lethargy. 


What medication should she be given? 


1- Atenolol 


2- Perindopril 

3- Amlodipine 

4- Verapamil 

5- Frusemide 


Answer & Comments 


5- Squamous cell carcinoma 


Answer: 2- Perindopril 


Answer & Comments 

Answer: 2- Cutaneous leishmaniasis 

Cutaneous leishmaniasis is spread by female 
sandflys of the genus Phlebotomus. 

The causative agents include L. (V.) 
braziliensis, L. (L.) mexicana, L. (V.) 
panamensis, and related species. Most 
infections follow a bite from an infected 


Beta blockers may worsen lethargy and 
calcium channel blockers can cause ankle 
oedema and gum bleeding. 

A thiazide diuretic has already been started, 
hence frusemide is not appropriate, hence an 
ACE inhibitor such as perindopril is the best 
option. 


Dr. Khalid Yusuf El-Zohry - Sohag Teaching Hospital (01118391123) 

Ref MRCPass OE OE 2012 PasTest 2009 PassMedicine 2009 PasTest Exam ReviseMRCP 



929 



























El-zohry MRCP Questions Bank (Part 1) - 2013 


(For my personal use) 



[Q: 2315] MRCPass - 2011 
September 


Which organ listed below is with direct contact 
with left kidney? 

1- Liver 

2- Duodenum 


output from the a-cells of the pancreas, which 
results in decreased hepatic glucose output. 

Gliptins are recommended as an option in 
type 2 diabetes for patients with inadequate 
glycaemic control despite being on metformin, 
sulphonylureas and glitazones. 


3- Small intestine 

4- Pancreas 

5- Colon 


Answer & Comments 

Answer: 4- Pancreas 

The left kidney is anatomically next to the 
spleen and pancreas. 


A type 2 diabetic with poor glycaemic control 
on metformin and rosiglitazone was then 
started on sitagliptin. 

What is the mechanism of action of gliptins? 

1- Binds to PPAR gamma 

2- Sulphonylurea secretagogue 

3- Alpha glucosidase inhibitor 

4- Glucagon like peptide 

5- Dipeptidyl peptidase 4 inhibitor 


[Q: 2316] MRCPass-2011 
September 



Answer & Comments 

Answer: 5- Dipeptidyl peptidase 4 inhibitor 

Dipeptidyl peptidase-4 inhibitors (DPP-4s), 
also commonly called gliptins, are a relatively 
new class of drugs for the treatment of type 2 
diabetes. 

Examples are vitagliptin and sitagliptin. These 
agents work in a unique way to improve 
insulin secretion from the Beta-cells of the 
pancreas in response to an increase in blood 
sugar and simultaneously decrease glucagon 


[Q: 2317] MRCPass-2011 
September 

A 41 year old man is admitted with fevers and 
a cough productive of green sputum. His 
friends had similar presenting symptoms. On 
investigation the blood results showed 
macrocytosis and chest x ray shows a right 
upper lobe cavitating lesion. 

What is the most likely cause? 

1- Haemophilus influenzae 

2- Klebsiella 

3- Legionella 

4- Streptococcus pneumoniae 

5- Mycoplasma 



Answer & Comments 

Answer: 2- Klebsiella 

Klebsiella infection most common in men over 
40 years of age and is most frequently found 
in alcoholics. 

Other predisposing factors are heart or lung 
disease, diabetes and malignancy. There is 
often sudden with severe systemic upset (high 
fever, rigors) and pleuritic pain. The sputum is 
purulent, gelatinous or blood-stained 
(haemoptysis occurs more often than in most 
bacterial pneumonias) Consolidation is usually 
seen in the upper lobes and may be extensive 
- sw elling of the infected lobe may result in 
bulging of the fissures on the lateral chest X- 
ray. 


Dr. Khalid Yusuf El-Zohry - Sohag Teaching Hospital (01118391123) 

Ref MRCPass OE OE 2012 PasTest 2009 PassMedicine 2009 PasTest Exam ReviseMRCP 






























El-zohry MRCP Questions Bank (Part 1) - 2013 


(For my personal use) 


t 

A 17 year old female is evaluated in the 
pediatric endocrinology clinic for primary 
amenorrhoea. 

Which one of these features is consistent with 
testicular feminisation or androgen 
insensitivity syndrome? 

1- Male phenotype with lack of hair 

2- Male phenotype with inguinal testis 

3- Male phenotype with breast development 

4- Female phenotype with clitoromegaly and 
undescended testis 

5- Female phenotype with external male 
genitals 

Answer & Comments 

Answer: 4- Female phenotype with 

clitoromegaly and undescended testis 

Androgen insensitivity syndrome (AIS), 
formerly known as testicular feminization, is 
an X-linked recessive condition. 


eek history of a progressive malaise. He had 
been commenced 5 days of antibiotic 
treatment by his GP no improvement was 
noted. On examination he had several 
palpable cervical and axillary lymph nodes. He 
did relate a history of exposure to cats at 
home and had scratch mark on his chest. 

What is the likely organism? 

1- Bartonella hensalae 

2- Pasteurella multicoda 

3- Pseudomonas 

4- Staph aureus 

5- Candida 

Answer & Comments 

Answer: 1- Bartonella hensalae 

The genus Bartonella (formerly Rochalimaea) 
is a member of the rickettsial order of 
bacteria, and it is found worldwide. 

The most likely mode of bacterial transmission 
is an infected cat, hence the term 'Cat Scratch' 
disease. 


[Q: 2318] MRCPass - 2011 
September 


A person with complete androgen insensitivity 
syndrome (CAIS) has a female external 
appearance despite a 46XY karyotype and 
undescended testes. This is due to the lack of 
sensitivity to androgen (testosterone) leading 
to a failure of male physical development. 

Many of these patients have a female 
phenotype. Some patients are first seen in the 
teenage years for evaluation of primary 
amenorrhea, but most are identified in the 
new born period by the presence of inguinal 
masses, which later are identified as testes 
during surgery. The patients also have a male 
level of testosterone and may have 
clitoromegaly or a micropenis. 


An 18 year old man was referred with a 6-w 



[Q: 2319] MRCPass-2011 
September 


Cat-to-cat transmission is believed to be 
attributable to fleas. 


[Q: 2320] MRCPass-2011 
September 

A 36-year-old man is admitted with left-sided 
pleuritic chest pains. These pains have been 
occurring for the past 2 weeks. Prior to the 
onset of the pains, he had been experiencing 
flu-like symptoms. 

What is the ECG most likely to show ? 

1- SI, Q3, T3 

2- Atrial fibrillation 

3- Widespread ST elevation 

4- ST segment depression in the anterior leads 

5- Tented T waves 



Dr. Khalid Yusuf El-Zohry - Sohag Teaching Hospital (01118391123) 

Ref MRCPass OE OE 2012 PasTest 2009 PassMedicine 2009 PasTest Exam 




ReviseMRCP 

931 




























El-zohry MRCP Questions Bank (Port 1) - 2013 


(For my personal use) 


Answer & Comments 

Answer: 3- Widespread ST elevation 

The diagnosis is likely to be pericarditis 
(possibly viral aetiology) and classical ECG 
changes of saddle shaped ST elevation are 
expected 



[Q: 2321] MRCPass - 2011 
September 


What is the mode of inheritance of vitamin D 
resistant rickets? 

1- Autosomal recessive 

2- Autosomal dominant 

3- X linked Recessive 

4- X linked dominant 

5- Sporadic 


Answer & Comments 

Answer: 4- X linked dominant 

X-linked hypophosphatemic or Vitamin D 
resistant Rickets is an X-linked dominant 
disorder characterized by growth retardation, 
rachitic and osteomalacic bone disease, 
hypophosphatemia, and renal defects in 
phosphate reabsorption and vitamin D 
metabolism 


A 34-year-old Caucasian woman admitted has 
a history of widespread, pruritic, 
erythematous skin rash, joint pains and renal 
disease. 

She recently had a baby with congenital heart 
block. Laboratory investigations revealed mild 
leucopenia (white cell count 3.25 x 10 9 /ml) 
and thrombocytopenia (platelets 140 x 
10 9 /ml). Erythrocyte sedimention rate was 
increased (65 mm/h). 

Which antibody is likely to be positive? 

1- Anti Jo 1 



[Q: 2322] MRCPass - 2011 
September 


2- Anti double stranded DNA 

3- Anti Ro 

4- Anti centromere 

5- ANCA 


Answer & Comments 

Answer: 3- Anti Ro 

The anti Ro antibody is associated with 
Sjogren's syndrome, SLE and neonatal lupus. 

Neonatal lupus erythematosus (NLE) is a rare 
disorder caused by the transplacental passage 
of maternal autoantibodies. Only 1% of infants 
with positive maternal autoantibodies develop 
neonatal lupus erythematosus. The most 
common clinical manifestations are cardiac 
(congenital heart block), dermatologic 
(urticaria and skin desquamation) , and 
hepatic (abnormal LFTs). The mother produces 
immunoglobulin G (IgG) autoantibodies 
against Ro (SSA), La (SSB), and/or Ul- 
ribonucleoprotein (Ul-RNP), and they are 
passively transported across the placenta. 
These autoantibodies can be found alone or in 
combination; However, anti-Ro is present in 
almost 95% of patients. 


^ [ Q: 2323 ] MRCPass - 2011 

• September 

A 55 year old man presented with severe 
retrosternal chest pain. His ECG shows ST 
depression in leads VI to V4. He has been 
given Fondaparinux. 

What is the drug's mechanism of action? 

1- Tissue plasminogen activator 

2- GIIBIIIA inhibitor 

3- Factor X a inhibitor 

4- Low molecular weight heparin 

5- Antithrombin III inhibitor 


Answer & Comments 


Answer: 3- Factor X a inhibitor 


Dr. Khalid Yusuf El-Zohry - Sohag Teaching Hospital (01118391123) 

Ref MRCPass OE OE 2012 PasTest 2009 PassMedicine 2009 PasTest Exam ReviseMRCP 



932 
































El-zohry MRCP Questions Bank (Port 1) - 2013 


(For my personal use) 


Fondaparinux is a synthetic pentasaccharide 
Factor Xa inhibitor. 

It is used for the prevention of deep vein 
thrombosis , pulmonary embolism and for 
management of acute coronary syndrome. 
One potential advantage of fondaparinux over 
LMWH or unfractionated heparin is that the 
risk for heparin-induced thrombocytopenia 
(HIT) is substantially lower. 


[Q: 2324] MRCPass - 2011 
September 

An 18 year man whose brother had 
hypertrophic cardiomyopathy was referred for 
a cardiological assessment. 

His echocardiogram confirmed the above 
condition. 

Which one of following echocardiographic 
features is an important risk factor for sudden 
cardiac death? 

1- Gradient of 30 mmHg across left ventricular 
outflow tract 

2- Septal wall thickness of > 3 cm 

3- An enlarged left atrium 

4- Systolic anterior motion of mitral valve 

5- The presence of mitral regurgitation 



Answer & Comments 

Answer: 2- Septal wall thickness of > 3 cm 

Patients die of hypertrophic obstructive 
cardiomyopathy by obstructing left ventricular 
outflow tract (LVOT), usually during exercise. 

The greater thickness of septum, more likely 
there is risk of cardiac arrhythmias (> 3 cm is 
significant). 



[Q: 2325] MRCPass - 2011 
September 


Which one of following cells in lung 
parenchyma produces surfactant? 


1- Alveolar macrophage 


2- Endothelial cell 

3- Goblet Cell 

4- Type I pneumocyte 

5- Type II pneumocyte 


Answer & Comments 

Answer: 5- Type II pneumocyte 

Surfactant is produced by type II 
pneumocytes. 

Type II pneumocytes also called great alveolar 
cells or septal cells are granular and roughly 
cuboidal in shape. Type II pneumocytes are 
typically found at the alveolar-septal junction. 



[Q: 2326] MRCPass-2011 
September 


A 25 year old man has had behavioural 
disturbance recently. His parents mentioned 
that his brother has been investigated for liver 
problems recently. On examination, he has a 
MMSE score of 28/30. He has a mask like face 
and was noticed to have hypersalivation. 
When the investigations are complete, which 
drug is most likely to be used for treatment? 


1- Desferrioxamine 


2- Co careldopa 

3- Penicillamine 


4- Interferon alpha 

5- Chlorpromazine 


Answer & Comments 

Answer: 3- Penicillamine 

The likely diagnosis is Wilson's disease. 

Most patients who present with 
neuropsychiatric manifestations have 
cirrhosis. The most common presenting 
neurologic feature is asymmetric tremor, 
occurring in approximately half of individuals 
with Wilson disease. Frequent early symptoms 
include difficulty speaking, excessive 


Dr. Khalid Yusuf El-Zohry - Sohag Teaching Hospital (01118391123) 

Ref MRCPass OE OE 2012 PasTest 2009 PassMedicine 2009 PasTest Exam ReviseMRCP 

































El-zohry MRCP Questions Bank (Port 1) - 2013 


(For my personal use) 


salivation, ataxia, masklike facies, clumsiness 
with the hands, and personality changes. The 
disease is autosomal recessive. 

Penicillamine is used as a copper chelator. 


A 22 year old man is tall compared to his 
peers. On examination, he was found to have 
aortic incompetence and mitral valve 
prolapse. He also had pectus excavatum, 
arachnodactyly and arm span greater than 
height. Slit lamp examination revealed had 
upward dislocation of the lens in the eye. 

The gene defect is: 

1- Actin 

2- Myosin 

3- Fibrillin 

4- Retinoblastoma 

5- Elastin 



[Q: 2327] MRCPass-2011 
September 


Answer & Comments 

Answer: 3- Fibrillin 

The fibrillin gene defect is the basis of 
Marfan's syndrome. 

It is an autosomal dominant disorder 
characterised by arachnodactyly, upward lens 
dislocation, tall habitus and flat feet. Aortic 
aneurysms and aortic regurgitation are also 
associated. 



[Q: 2328] MRCPass-2011 
September 


A 20-year-old female college student presents 
with increased polydipsia and polyuria since 
childhood. She has alw ays feels thirsty and 
has a frequency of micturition of 20 to 25 
times in a 24-hour period. She mentions that 
she has had the symptoms for 2 months. Upon 
investigation the following results were found. 


sodium 122 mmol/I 


potassium 4.5 mmol/I 
urea 4 mmol/l 
creatinine 78 |imol/l 
Glucose: 5.5 mmol/l 

Plasma Osmolality 270 (280-300) 

mosm/kgwater 

Urine Osmolality 90 (50-1200) mosm/kg 

A water deprivation test was conducted and 
the urine osmolality increased to 300 
mosm/kg and the serum osmolality increased 
to 290 mosm/kg after 6 hours. 

Whot is the diagnosis? 

1- Addison's disease Diabetes mellitus 

2- Diabetes mellitus 

3- Psychogenic polydipsia 

4- Diabetes insipidus 

5- SIADH 


Answer & Comments 

Answer: 3- Psychogenic polydipsia 

Primary polydipsia or psychogenic polydipsia 
is usually associated with a patient's 
increasing fluid intake due to the sensation of 
having a dry mouth. 

The test of choice to distinguish primary 
polydipsia from diabetes insipidus is by fluid 
restriction (water deprivation test). 

In primary polydipsia, the urine osmolality 
should increase and stabilize at above 280 
Osm/kg. Stabilization in this test means, more 
specifically, when the hourly increase in 
osmolality is less than 30 Osm/kg per hour for 
at least 3 hours. A stabilization at an 
osmolality of less than 280 Osm/kg indicates 
diabetes insipidus. 



[Q: 2329] MRCPass-2011 
September 


A 60 year old woman has recently been 
diagnosed with small cell carcinoma of the 


Dr. Kholid Yusuf El-Zohry - Sohog Teaching Hospital (01118391123) 

Ref MRCPoss OE OE 2012 PosTest 2009 PassMedicine 2009 PosTest Exam ReviseMRCP 




























El-zohry MRCP Questions Bank (Port 1) - 2013 


(For my personal use) 


lung. She has a 45 pack year smoking history. 
She comes to the urgent care clinic today 
complaining of a cough and was found to be 
confused. The blood tests show : 

sodium 119 mmol/l, 

potassium 4.2 mmol/l 

urea 6 mmol/l 

creatinine 80 |imol/l 

Plasma Osmolality 260 (280-300) mmol/kgw 
ater 

Urine Osmolality 380 mmol/kg 
Whot is the diagnosis? 

1- Addison's disease 

2- Diabetes mellitus 

3- Psychogenic polydipsia 

4- Diabetes insipidus 

5- SIADH 

Answer & Comments 

Answer: 5- SIADH 

The syndrome of inappropriate antidiuretic 
hormone hypersecretion (SIADH) is 
characterized by excessive release of 
antidiuretic hormone (ADH or vasopressin) 
from the posterior pituitary gland or another 
source. 

The result is hyponatremia . It is associated 
with small-cell carcinoma of the lung, 
pneumonia, brain tumors, head trauma, 
stroke, meningitis, and encephalitis. In 
general, increased ADH causes water 
retention and extracellular fluid volume 
expansion without edema or hypertension, 
owing to natriuresis (the excretion of sodium 
by the kidneys). The water retention and 
sodium loss both cause hyponatremia, which 
is a key feature in SIADH. Hyponatremia and 
concentrated urine (UOsm >300 mOsm) are 
seen, as well as no signs of edema or 
dehydration. Severe hyponatraemia can lead 
to cerebral oedema and hence . 


^ [ Q: 2330 ] MRCPass - 2011 

| I September 

Which one of the following is a bod prognostic 
marker in acute lymphoblastic leukaemia? 

1- Pre-B phenotype 

2- Age of < 20 years 

3- Initial white cell count of 18 x 10 9 /l 

4- Female sex 

5- BCR-Abl gene 

Answer & Comments 

Answer: 5- BCR-Abl gene 

Acute lymphoblastic leukaemia (ALL) is most 
common in childhood with a peak incidence at 
4-5 years of age, and another peak in old age. 

Some prognostic factors are: 

Sex: females tend to fare better than males. 

Age at diagnosis: children between 1-10 years 
of age are most likely to develop ALL and to be 
cured of it. 

Cytogenetics: Philadelphia translocation, 

t(9;22) is a bad prognostic factor. (Philadelphia 
translocation, t(9;22) - good prognosis in CML, 
poor prognosis in AML + ALL) The exact 
chromosomal defect in Philadelphia 
chromosome is a translocation. Parts of two 
chromosomes, 9 and 22, sw ap places. The 
result is that a fusion gene is created by 
juxtapositioning the Abll gene on 
chromosome 9 (region q34) to a part of the 
BCR ("breakpoint cluster region") gene on 
chromosome 22 (region qll). The result of the 
translocation is the oncogenic BCR-ABL gene 
fusion. Because the Abl gene expresses a 
membrane-associated protein, a tyrosine 
kinase, the BCR-Abl transcript is also 
translated into a tyrosine kinase, adding a 
phosphate group to tyrosine. Although the 
BCR region also expresses serine/threonine 
kinases, the tyrosine kinase function is very 
relevant for drug therapy. Tyrosine kinase 


Dr. Khalid Yusuf El-Zohry - Sohag Teaching Hospital (01118391123) 

Ref MRCPass OE OE 2012 PasTest 2009 PassMedicine 2009 PasTest Exam ReviseMRCP 
























El-zohry MRCP Questions Bank (Port 1) - 2013 


(For my personal use) 


inhibitors (such as imatinib and sunitinib) are 
important drugs against a variety of cancers 
including in CML, and sometimes in Ph- 
positive acute lymphoblastic leukemia 
(Ph+ALL) 


A 73-year-old male presents to the emergency 
department with sudden-onset, diffuse 
abdominal pain that began 18 hours ago. He 
has not been vomiting, but he has had several 
episodes of diarrhoea, the last of which was 
bloody. Ischaemic colitis was diagnosed 
following a surgical review. 

Where is the most commmon site for the 
condition? 

1- Hepatic flexure 

2- Splenic flexure 

3- Caecum 

4- Sigmoid 

5- Rectum 



[Q: 2331] MRCPass - 2011 
September 


Answer & Comments 

Answer: 2- Splenic flexure 

The colon receives blood from both the 
superior and inferior mesenteric arteries. 

The blood supply from these two major 
arteries overlap, with abundant collateral 
circulation. However, there are weak points, 
or "watershed" areas, at the borders of the 
territory supplied by each of these arteries, 
such as the splenic flexure and the transverse 
portion of the colon. These watershed areas 
are most vulnerable to ischemia, thus leading 
to ischaemic colitis. 



[Q: 2332] MRCPass - 2011 
September 


A 27-year-old man presented with bi¬ 
temporal hemianopia. He mentioned that his 


shoe sizes were above that of his friends since 
childhood and he often had sw eaty episodes. 

Which one of the following tests is likely to 
confirm the diagnosis of acromegaly? 

1- Random growth hormone 

2- IGF-1 

3- Glucose tolerance test with growth 
hormone suppression 

4- Synacthen test 

5- MRI pituitary 


Answer & Comments 

Answer: 3- Glucose tolerance test with growth 
hormone suppression 

In Acromegaly, there is excess Growth 
hormone (GH) which is difficult to suppress. 

Because GH secretion is inhibited by glucose, 
measurement of glucose non-suppressibility is 
useful. In the glucose tolerance test, baseline 
GH levels are obtained prior to ingestion of 
100 g of oral glucose, and additional GH 
measurements are made at 30, 60, 90, and 
120 minutes following the oral glucose load. 
Patients with active acromegaly are unable to 
suppress GH concentration below 2 ng/mL. 

Random GH measurements are often not 
diagnostic because of the episodic secretion of 
GH, but IGF-I has a long half-life, and is useful 
as a screen for Acromegaly. MRI may reveal a 
pituitary tumour but it would not be specific 
for Acromegaly. 


[Q: 2333] MRCPass-2011 
September 

A 53-year-old woman presented to the 
emergency department with complaints of 
intermittent fatigue, nausea and vomiting for 
several months. She complains of having a 
constant dry mouth. She had a past medical 
history of hypothyroidism and pernicious 
anaemia. She had a 20 pack-year smoking 
history and alcohol intake consisted of a 



Dr. Khalid Yusuf El-Zohry - Sohag Teaching Hospital (01118391123) 

Ref MRCPass OE OE 2012 PasTest 2009 PassMedicine 2009 PasTest Exam ReviseMRCP 



























El-zohry MRCP Questions Bank (Port 1) - 2013 


(For my personal use) 


maximum of 4 units a day. On examination, 
she was jaundiced and had palpable 
hepatomegaly. She was noted to have 
excoriation marks on the skin. 

Blood results are: Hb 12.5 g/dl, WCC 7 x 10 9 /l, 
platelets 235 x 10 9 /l, sodium 136 mmol/I, 
potassium 4.5 mmol/I, urea 6 mmol/l, 
creatinine 110 pmol/l, ALT 88 (5-35) U/l, AST 
55 (1-31) U/l, ALP 520 (20-120) U/l, GGT 85 (4- 
35) U/l, Bilirubin 125 (1-22) [irnol/l, Albumin 
38 (37-49) g/l. 

Whot is the likely diagnosis? 

1- Hepatitis B infection 


4- Inhibition of sodium channels 

5- Inhibition of potassium channels 

Answer & Comments 

Answer: 5- Inhibition of potassium channels 

Most candidates answ ered either inhibition of 
sodium or potassium channels. 

Although it can be caused by inhibition of 
sodium, potassium or calcium channels, 
around 90% of inherited long QT syndrome 
are due to defects in potassium channels 


2- Ulcerative colitis 

3- Crohn's disease 

4- Primary biliary cirrhosis 

5- Gilbert's syndrome 


Answer & Comments 

Answer: 4- Primary biliary cirrhosis 

The patient has a cholestatic picture (high 
alkaline phosphatase and bilirubin) in the liver 
function tests, and hence out of all the 
options, primary biliary cirrhosis is most likely. 

There is an association with autoimmune 
diseases such as scleroderma, autoimmune 
thyroiditis. Fatigue is the first reported 
symptom, pruritus is also a common 
symptom. 

Examination findings usually include 
hepatomegaly and xanthelasmata. 


«J 

A 65 year old man had recently retired and 
over the last month has developed symptoms 
of trouble concentrating, early morning 
waking and poor interest in daily acivities. He 
lost his retirement pension in the recession. 
He reported current suicidal ideation without 
a specific plan but these thoughts were not 
recurrent. He hated his life and cried 
constantly. He reported that he felt hopeless 
and unaw are of future opportunities or 
choices in life. He felt "stuck" and "lost." His 
wife said that he was often withdrawn. 

What is the diagnosis? 

1- Mania 

2- Depression 

3- Schizophrenia 

4- Conversion disorder 


[Q: 2335] MRCPass-2011 
September 


[Q: 2334] MRCPass-2011 
September 

Which one of the following is the most 
common underlying physiological mechanism 
causing prolongation of the QT segment? 

1- Opening of calcium channels 

2- Opening of potassium channels 

3- Opening of sodium channels 


5- Anxiety disorder 

Answer & Comments 

Answer: 2- Depression 

Depression is a state of low mood where 
patients may feel sad, anxious, hopeless and 
worthless. 

They may lose interest in activities that once 
were pleasurable, experience loss of appetite, 



Dr. Khalid Yusuf El-Zohry - Sohag Teaching Hospital (01118391123) 

Ref MRCPoss OE OE 2012 PosTest 2009 PassMedicine 2009 PosTest Exom ReviseMRCP 






























El-zohry MRCP Questions Bank (Port 1) - 2013 


(For my personal use) 


or problems concentrating; and may 
contemplate or attempt suicide. Insomnia, 
excessive sleeping, fatigue, loss of energy are 
additional symptoms which may be present. 


A 36 year old man presented with a 
generalised seizure. On examination, he was 
found to have adenoma sebaceum on the 
face, two hypopigmented areas and 
subungual fibroma. He had a urine dipstick 
showing blood ++ and was organised to have 
an ultrasound of the kidneys which showed 
cystic changes. 

What is the likely diagnosis? 

1- Von Hippel Lindau 

2- Neurofibromatosis 

3- Vitiligo 

4- Acromegaly 

5- Tuberous sclerosis 


[Q: 2336] MRCPass - 2011 
September 



Answer & Comments 

Answer: 5- Tuberous sclerosis 

The diagnosis is tuberous sclerosis. 

It is an autosomal dominant condition. 
Features are epilepsy (cortical tubers in the 
brain), adenoma sebaceum on the skin, 
subungual fibroma of the nails, oval 
hypopigmented macules - ash leaf macules - 
best seen with Wood's (UV) light, retina 
phakoma, renal angiomyolipoma (causing 
cystic renal lesions) and cardiac 
rhabdomyomas. 



[Q: 2337] MRCPass - 2011 
September 


A 25 year old woman complained of voices 
which told her to cut rhis arms, and that she 
had heard these voices over the last few days. 
She was unemployed and lived with two 


friends. Her friends mentioned that she often 
takes drugs. 

Which one of the following is the most likely 
cause of her symptoms? 

1- Alcohol 

2- Amphetamines 

3- Diazepam 

4- Gamma-hydroxybutyrate 

5- Morphine 


Answer & Comments 

Answer: 2- Amphetamines 

The patient is likely to be having delusions and 
amphetamines are the most likely cause. 

It tends to occur after large doses and chronic 
use. 


A 75 year old man with sever COPD was 
admitted to hospital with acute shortness of 
breath. He was initially alert and the admitting 
doctor asked him about escalation of 
treatment if he should become unwell. He 
understood the question and could repeat the 
information back to the doctor. He agreed to 
have antibiotics, nebulisers and non invasive 
ventilation but refused to have intubation and 
ventilation. 

An hour later, whilst on non invasive 
ventilation, he developed type II respiratory 
failure, worsened and became acutely 
confused. His GCS dropped to 6 / 15. His 
family members became anxious and asked 
what was being done for him. 

What should be done? 

1- Continue antibiotics and nebulisers only 

2- Intubate the patient in his best interest 

3- Obtain consent from next of kin to intubate 
patient 


[Q: 2338] MRCPass-2011 
September 



Dr. Khalid Yusuf El-Zohry - Sohag Teaching Hospital (01118391123) 

Ref MRCPass OE OE 2012 PasTest 2009 PassMedicine 2009 PasTest Exam ReviseMRCP 



938 



























El-zohry MRCP Questions Bank (Port 1) - 2013 


(For my personal use) 


4- Increase settings of non invasive ventilation 

5- Stop all treatment completely 

Answer & Comments 

Answer: 1- Continue antibiotics and nebulisers 
only 

The patient has worsened on non invasive 
ventilation (NIV) and is too unwell to tolerate 
NIV due to the low GCS. 


All are common causes of traveller's 
diarrhoea. 

However, North Africa and the Middle East (in 
particular Egypt) were also commonly 
reported regions of travel for Shigella spp 
infections. 

Some of the infectious causes of bloody 
diarrhoea are: 

Salmonella 


The patient had expressed clear wishes not for 
intubation when he had capacity, so it is 
inappropriate to obtain consent from next of 
kin to go against his wishes. 

The best option is to manage conservatively, 
hence the patient can have medications but to 
explain to the family that he is extremely 
unwell and may not survive. Stopping all 
treatment abruptly may distress the family at 
this point. 



[Q: 2339] MRCPass-2011 
September 


A 31-year-old man who has returned from a 
holiday in Egypt presents with diarrhoea. He 
had been on a cruise at the Nile river. For the 
past two days he has been passing frequent 
bloody diarrhoea associated with crampy 
abdominal pain. Abdominal examination 
demonstrates diffuse lower abdominal 
tenderness but there is no guarding or rigidity. 
His temperature is 37.8°C. 


What is the most likely causative organism? 


1- Giardiasis 


2- Enterotoxigenic Escherichia coli 

3- Staphylococcus aureus 

4- Shigella 

5- Salmonella 


Shigella 

Campylobacter jejuni 
Yersinia enterocolitica 
E. coli 

Entamoeba histolytica 



[Q: 2340] MRCPass-2011 
September 


A 77-year-old male presented with a 4-year 
history of mild cognitive decline. He has a 10- 
year history of hypertension and type 2 
diabetes. According to his family, he had 
become more forgetful, yet he was able to 
carry out simple tasks independently.. His 
short-term memory was impaired, as was his 
ability to concentrate. His gait was slow and 
he was unsteady. He leaned backward when 
he walked and fell often, especially when 
trying to turn to the left or right. He 
experienced urinary frequency, nocturia, and 
urinary incontinence at least once a day. 


What is the most likely diagnosis? 

1- Alzheimer's disease 


2- Transient ischaemic attack 


3- Lewy body dementia 

4- Pick's disease 


5- Normal pressure hydrocephalus 


Answer & Comments 


Answer: 4- Shigella 


Answer & Comments 


Answer: 5- Normal pressure hydrocephalus 


Dr. Khalid Yusuf El-Zohry - Sohag Teaching Hospital (01118391123) 

Ref MRCPoss OE OE 2012 PosTest 2009 PassMedicine 2009 PosTest Exom ReviseMRCP 






























El-zohry MRCP Questions Bank (Port 1) - 2013 


(For my personal use) 


Normal pressure hydrocephalus (NPH) is a 
clinical symptom complex characterized by 
abnormal gait, urinary incontinence, and 
dementia. 

The CT scan often shows evidence of 
hydrocephalus (distended ventricles), but the 
CSF pressure is normal on lumbar puncture. 
The treatment is to remove CSF by lumbar 
puncture (normally 50 mis). 

[Q: 2341] MRCPass - 2011 
September 

A 38 year old man was investigated for 
infertility. He has a history of hypertension, 
diabetes and Crohn's disease. He takes several 
medications. 

Which one of the following drugs is most likely 
to couse this? 


unable to get out of bed. Cardiovascular, 
respiratory and abdominal examination were 
unremarkable. Urine dipstick shows protein 
++, blood +++, white cells +. 

Whot test should be done? 

1- Magnesium 

2- Creatine kinase 

3- International normalised ratio 

4- Troponin 

5- Brain natriuretic peptide 

Answer & Comments 

Answer: 2- Creatine kinase 

This patient is likely to have been on the floor 
due to weakness and may have 
rhabdomyolysis. 



1- Mesalazine 

2- Sulfasalazine 

3- Aspirin 

4- Azathioprine 

5- Cyclosporin 


Answer & Comments 

Answer: 2- Sulfasalazine 

Sulfasalazine, anabolic steroids, 

cyclophosphamide, chlorambucil, busulfan 
and cisplatin are drugs which cause 
azoospermia. 



[Q: 2342] MRCPass-2011 
September 

A 22-year-old woman presents with a fall and 
was found at home with a low conscious level. 

She has no significant medical history. A friend 
who came with her mentioned that she was 
an intravenous drug user and drank 8 - 10 
pints of lager a day. On admission, she had a 
blood pressure of 95/60 mmHg and 
temperature of 34 C. She was very weak and 


The urine dipstick may demonstrate blood, 
although the true test is of myoglobin levels. A 
significantly elevated creatine kinase enzyme 
would be a reasonable indicated of 
rhabdomyolysis, and the patient should be 
kept well hydrated. 

Monitoring of renal function and urine output 
would be important. 


[Q: 2343] MRCPass-2011 
September 

A 43 year old man complained of abdominal 
pain, weight loss and diarrhoea for the past 
few months. On examination, he had a soft 
abdomen with inguinal lymphadenopathy. A 
colonoscopy was performed and the small 
intestinal biopsy showed PAS stained 
macrophages. 

Whot is the probable diagnosis? 

1- Intestinal lymphoma 

2- Whipple's disease 

3- Hepatitis 

4- Celiac disease 

5- Tropical Sprue 



Dr. Kholid Yusuf El-Zohry - Sohog Teaching Hospital (01118391123) 

Ref MRCPass OE OE 2012 PasTest 2009 PassMedicine 2009 PasTest Exam ReviseMRCP 



























El-zohry MRCP Questions Bank (Port 1) - 2013 


(For my personal use) 


Answer & Comments 

Answer: 2- Whipple's disease 

Whipple's disease is caused by the organism 
Tropheryma whipplei. 

It primarily causes malabsorption but may 
affect any part of the body including the heart, 
lungs, brain, joints, skin, and the eyes. The 
clinical features of Whipple's disease include: 
malabsorption, weight loss, abdominal pain, 
lymphadenopathy, migratory polyarthritis, 
sacroiliitis, increased skin pigmentation, 
pyrexia, neurological abnormalities. 

The disease is regarded as extremely rare, 
with an incidence of one case per million 
people. The patients are predominantly male 
and in their age 40s. T. whipplei appears to be 
an environmental organism that is commonly 
present in the gasterointestinal tract but 
remains asymptomatic . Patients who have 
Whipple's disease are thought to have an 
immunological defect. 


Neut 5.2 xl0 9 /l 
Lymp 11.2 x 10 9 /l 

Blood Film Atypical lymphocytes seen 
Whot is the most likely diagnosis? 

1- Acute lymphoblastic leukaemia 

2- Hashimoto's thyroiditis 

3- Infectious mononucleosis 

4- HIV seroconversion 

5- Septicaemia secondary to streptococcal 
throat infection 

Answer & Comments 

Answer: 3- Infectious mononucleosis 

The history of previously being well, acute 
deterioration with lymphadenopathy, throat 
involvement and atypical lymphocytes on 
blood film are all consistent with Epstein Barr 
virus infection (glandular fever or infectious 
mononucleosis). 


The small intestinal biopsy in Whipple's 
disease reveals: 

minimal villous atrophy 

PAS positive staining foamy macrophages with 
intracellular bacilli (Tropheryma whippelii) 



[Q: 2344] MRCPass - 2011 
September 


A 20-year-old man presents with lethargy, 
pyrexia and headaches. These symptoms have 
been present for the past 8 days. He had not 
been unwell before and there is no recent 
history of travel. Clinical examination reveals a 
temperature of 37.9°C, marked cervical 
lymphadenopathy and mild hepatomegaly. 
Throat examination reveals two small 
erythematous areas. A full blood count result 
shows: 


Hb 13.1 g/dl 
Platelets 225 x 10 9 /l 
WCC 17.1x1071 


Atypical lymphocytes are commonly 
associated with EBV, CMV and toxoplasma 
infection. 



Reactive lymphocytes are lymphocytes that 
become large as a result of antigen 
stimulation. 


[U 

A 51-year-old male presented with sudden 
worsening of breathlessness after a severe 
episode of pneumonia was thought to have 


[Q: 2345] MRCPass-2011 
September 


Dr. Khalid Yusuf El-Zohry - Sohag Teaching Hospital (01118391123) 

Ref MRCPass OE OE 2012 PasTest 2009 PassMedicine 2009 PasTest Exam ReviseMRCP 






























El-zohry MRCP Questions Bank (Port 1) - 2013 


(For my personal use) 


developed acute respiratory distress 
syndrome (ARDS). 

Which of the following features would support 
a diagnosis of ARDS? 

1- High pulmonary capillary wedge pressure 

2- High protein pulmonary oedema 

3- Hypercapnea 

4- Increased lung compliance 

5- Normal chest x- ray 

Answer & Comments 

Answer: 2- High protein pulmonary oedema 

Answer: B): high protein pulmonary oedema 

ARDS is characterised by hypoxaemia, reduced 
lung compliance (stiff lungs) and pulmonary 
infiltrates on the chest x- ray. 

There is also no cardiogenic cause for 
pulmonary oedema (the Pulmonary Capillary 
Wedge pressure has to be normal or less than 
18mmHg to confirm this). 


WCC 12 x 10 9 /l 
Platelets 65 x 10 9 /l 
sodium 138 mmol/l 
potassium 4.7 mmol/l 
Urea 22.1 mmol/l 
Creatinine 366 pmol/l 

Blood film : fragmented red cells and helmet 
shaped red cells 

What is the likely causative organism? 

1- Staphylococcus aureus 

2- Neisseria Meningitidis 

3- Legionella 

4- Leptospira 

5- E coli 0157 

Answer & Comments 

Answer: 5- E coli 0157 

The patient has haemolytic uraemic 
syndrome. 


Histologically, in ARDS there is damage to the 
capillary and endothelial cell linings, resulting 
in leakage of proteins into the interstitial and 
alveolar spaces at normal pulmonary capillary 
hydrostatic pressures - hence causing 
pulmonary oedema with high protein. In 
cardiac failure the protein levels of pulmonary 
oedema fluid are low . 



[Q: 2346] MRCPass-2011 
September 

A 65-year-old female is brought to A&E by her 
family, who are concerned about her 
increasing lethargy and confusion over the 
past 3 days. There is a history of diarrhea in 
the preceding few days. On examination she is 
found to be pyrexial at 38oC. Breath sounds 
are clear and there is mild tenderness in the 
lower abdomen. 


Blood tests results show : 
Hb 8.6 g/dl 


It is characterized by the triad of 
microangiopathic hemolytic anemia, 
thrombocytopenia, and acute renal failure. 
Diarrhea (E coli 0157) and upper respiratory 
infection are the most common precipitating 
factors. The hallmark of HUS in the peripheral 
smear is the presence of schistocytes 
(fragmented, deformed red cell fragments) 
and helmet-shaped RBCs. 


[Q: 2347] MRCPass-2011 
September 

A 61-year-old man is admitted with chest pain 
to the Emergency Department. He has a past 
medical history of type 2 diabetes, 
hypertension and high cholesterol. His regular 
medications includes simvastatin, bisoprolol, 
glibenclamide and metformin. An ECG shows 
ST elevation in the anterior leads and he is 
referred for primary angioplasty. Following 
the procedure, he was transferred to the 



Dr. Khalid Yusuf El-Zohry - Sohag Teaching Hospital (01118391123) 

Ref MRCPoss OE OE 2012 PosTest 2009 PassMedicine 2009 PosTest Exam ReviseMRCP 



942 




























El-zohry MRCP Questions Bank (Port 1) - 2013 


(For my personal use) 


Coronary Care Unit (CCU). He has a blood 

glucose measurement of 15 mmol/l. 

Which drug regime should be commenced? 

1- Continue metformin and glibenclamide at 
same dose 

2- Stop metformin and increase dose of 
glibenclamide 

3- Subcutaneous insulin: basal-bolus regime 

4- Subcutaneous insulin: biphasic insulin 
regime 

5- Intravenous sliding scale insulin 


Answer & Comments 

Answer: 5- Intravenous sliding scale insulin 

It has been shown from previous trials (e.g. 
DIGAMI study) that insulin based glucose 
management leads to improved outcomes in 
type 2 diabetes post myocardial infarction. 
Good glycaemic control can be achieved with 
a sliding scale insulin regime or Glucose Insulin 
Potassium (GIK) regime. 


_i\__ 


1 — 



ii m 

Tti—^ 


—A 


jJi 


TTiHTT 


4 


frrfftrrrtjy rti~t 

U waves 



[Q: 2349] MRCPass - 2011 
September 


A 73 year old woman complained of pain at 
the base of her right thumb. 


On examination, there was tenderness and 
swelling of the right first carpo-metacarpal 
joint. 


What is the most likely diagnosis? 

1- Avascular necrosis of the scaphoid 


2- De Quervain's tenosynovitis 

3- Osteoarthritis 


4- Psoriatic arthritis 


^ [ Q: 2348 ] MRCPass - 2011 

M I L J 

S September 

On a ECG, which one of the following features 
is seen with significant Hypokalaemia? 

1- Flattened p wave 

2- Prominent r wave 

3- Flatttened t wave 

4- Prominent u wave 

5- Prominent j wave 

Answer & Comments 

Answer: 4- Prominent u wave 


5- Rheumatoid arthritis 

Answer & Comments 

Answer: 3- Osteoarthritis 

The tenderness at the carpometacarpal joint is 
most likely due to osteoarthritis in a patient of 
this age. 

Approximately 40% of post-menopausal 
females have radiographic changes at the base 
of the thumb. Abnormal loads across the joint 
cause the articular cartilage to wear out. Bony 
sw elling is apparent in advanced cases and 
local palpation at the base of the thumb is 
tender. 


The prominent U wave is a component seen 
after a T wave seen in severe Hypokalaemia. 



[Q: 2350] MRCPass-2011 
September 


A 14 year-old male presents with fever and 
headache, and within hours he felt neck pain 
when moving his head. 


Dr. Khalid Yusuf El-Zohry - Sohag Teaching Hospital (01118391123) 

Ref MRCPass OE OE 2012 PasTest 2009 PassMedicine 2009 PasTest Exam 




ReviseMRCP 


943 
































El-zohry MRCP Questions Bank (Port 1) - 2013 


(For my personal use) 


The following day, his blood pressure fell to 
100/70 mmHg, and as a result, he was rushed 
to the hospital where he exhibited signs of 
confusion. Lumbar puncture was performed, 
and the cerebrospinal fluid (CSF) cultured 
Neisseria meningitides. He has a history of 2 
such previous presentations which were also 
diagnosed as meningococcal meningitis. 

Whot is the most likely underlying problem? 

1- Hypogammaglobulinaemia 

2- HIV infection 

3- Recurrent steroid use 

4- Macrophage dysfunction 

5- Complement deficiency 


4- Attention deficit hyperactivity disorder 

5- Acute dystonia 

Answer & Comments 

Answer: 3- Tourette's syndrome 

Tourette's syndrome is an inherited 
neuropsychiatric disorder with onset in 
childhood, characterized by multiple physical 
(motor) tics and at least one vocal (phonic) tic; 
these tics characteristically wax and wane. 

The tics are sudden, repetitive, stereotyped, 
nonrhythmic movements (motor tics) and 
utterances (phonic tics) that involve discrete 
muscle groups. 


Answer & Comments 

Answer: 5- Complement deficiency 

Deficiencies in terminal pathway 
complements(C5-C9), particularly C8 
complements are implicated in susceptibility 
to meningococcal infection. 

The lack of membrane attack complex 
formation results in severe recurrent infection 
by Neisseria gonorrhoeae or Neisseria 
meningitidis. 



[Q: 2351] MRCPass - 2011 
September 


An 18 year old man has had longstanding 
symptoms of vocal and motor tics. He 
displayed peculiar physical tics as well as 
repetitive throat clearings-despite the fact 
that he rarely had mucus or anything else 
bothering his throat. His physical tics 
consisted mainly of accentuated blinking and 
clenching/extending movements of the hands 
and sometimes feet. 


What is the diagnosis? 

1- Wallenberg's syndrome 

2- Huntington's disease 

3- Tourette's syndrome 


Motor tics are movement-based tics, while 
phonic tics are involuntary sounds produced 
by moving air through the nose, mouth, or 
throat. In the above case, the patient has 
hemifacial spasms causing blinking and 
athetoid movements of the hands and feet. 

[Q: 2352] MRCPass-2011 
September 

A 55-year-old man presented with a 10 -year 
history of an intermittent rash and pruritus 
associated with sw eating from exertion. For 2 
years he had noted pruritus and erythema 
mainly in the hands and feet, occurring on 
exposure to cool weather and resolving 
promptly on warming. He has a past medical 
history of Investigations showed a normal full 
blood count and mildly deranged liver 
function tests. Cryoglobulin levels were 
elevated. 

Which one of the following is likely to be a 
cause? 

1- Hepatitis C 

2- Staph aureus pneumonia 

3- Colorectal carcinoma 

4- Breast carcinoma 

5- Osteoarthritis 



Dr. Khalid Yusuf El-Zohry - Sohag Teaching Hospital (01118391123) 

Ref MRCPass OE OE 2012 PasTest 2009 PassMedicine 2009 PasTest Exam ReviseMRCP 


























El-zohry MRCP Questions Bank (Part 1) - 2013 


(For my personal use) 


Answer & Comments 

Answer: 1- Hepatitis C 

Cryoglobulinaemia occurs when there are 
large amount of proteins that become 
insoluble at reduced temperatures. 

Type I is most commonly encountered in 
patients with multiple myeloma.Types II and 
III are strongly associated with infection by the 
hepatitis C virus. 


frontal lobe had infarcted acutely. 

Which one of the following is a test of frontal 
lobe dysfunction? 

1- Inability to draw a clock face 

2- Inability to generate a list rapidly 

3- Intention tremor with finger nose testing 

4- Sensory inattention 

5- Visual field defects 


Types II and III have Rheumatoid Factor 
activity and bind to polyclonal 
immunoglobulins. 

Cryoglobulins may also be present in 
mycoplasma pneumonia, leukemias, primary 
macroglobulinemia, and some autoimmune 
diseases, such as systemic lupus 
erythematosus and rheumatoid arthritis. 


B-type natriuretic peptide (BNP) is used as a 
serum marker for cardiac failure. 

Where is it predominantly secreted from? 

1- Kidney 

2- Liver 

3- Brain 



[Q: 2353] MRCPass - 2011 
September 


Answer & Comments 

Answer: 2- Inability to generate a list rapidly 

An inability to draw a clock face is associated 
with constructional apraxia in parietal lobe 
damage. 

A failure to generate a list rapidly is a test of 
frontal lobe (e.g. name animals in 60 seconds 
with words beginning from letter F etc). 
Intention tremor is a cerebellar sign 

Sensory inattention is a manifestation of 
parietal lobe dysfunction. 

Visual field defect is a manifestation of 
occipital lobe (homonymous hemianopia), 
temporal lobe (superior quantranopia) or 
parietal lobe (inferior quantranopia) 
pathology. 


4- Cardiac ventricle 

5- Pancreas 


Answer & Comments 

Answer: 4- Cardiac ventricle 

BNP is secreted mainly from the left ventricle 
in normal adult humans (as well as in patients 
with left ventricular dysfunction), whilst ANP 
is secreted from atria. 



[Q: 2354] MRCPass - 2011 
September 


A 60-year-old male presents with a stroke and 
the CT scan shows a large territory of the 



[Q: 2355] MRCPass-2011 
September 


A 66 year old man presents with a three 
month history of fever, malaise, anorexia, 
twenty-five pound weight loss, diffuse 
myalgias and night sweats, and more recently 
hemoptysis. He had a past medical history of 
hypertension, and described episodes of 
haematuria. Physical examination showed 
that he had diffuse lower extremity muscle 
tenderness, crepitations in the lungs and a 
rash on the trunks. Chest x-ray showed 
bilateral diffuse pulmonary infiltrates and also 
2 areas of cavitation. 


Investigations showed: 


Dr. Khalid Yusuf El-Zohry - Sohag Teaching Hospital (01118391123) 

Ref MRCPass OE OE 2012 PasTest 2009 PassMedicine 2009 PasTest Exam 




ReviseMRCP 

945 






























El-zohry MRCP Questions Bank (Port 1) - 2013 


(For my personal use) 


urine protein 1+ 

urine sediment - many red blood cell and 
granular casts 

sodium 135 mmol/l 

potassium 5.2 mmol/l 

urea 14 mmol/l 

creatinine 220 [imol/l 

What investigation should be organised next? 

1- Urine culture 

2- Anti neutrophil cytoplasmic antibody 

3- Renal biopsy 

4- CT of kidney, ureter, bladder 

5- MR angiogram of the kidneys 

Answer & Comments 

Answer: 2- Anti neutrophil cytoplasmic 
antibody 

The diagnosis fits a pulmonary renal syndrome 
such as Wegener's, Churg Strauss or 
Goodpasture's syndrome. 

Apart from renal failure, there may be 
pulmonary haemorrhage, haemoptysis, 
infiltrates on the CXR as well as cavitation. 

The least invasive method initially to confirm a 
vasculitis is to request an ANCA. 


2- Tacrolimus 

3- Azathioprine 

4- Hydroxychloroquine 

5- PUVA therapy 

Answer & Comments 

Answer: 4- Hydroxychloroquine 

Discoid lupus erythematosus (DLE) is a 
chronic, scarring, atrophy producing, 
photosensitive dermatosis. 

DLE may occur in patients with systemic lupus 
erythematosus (SLE). Skin lesions are typically 
localized above the neck, with favored sites 
being the scalp, bridge of nose, cheeks, lower 
lip, and ears. The primary lesion is an 
erythematous papule or plaque with slight-to- 
moderate scaling. As the lesion progresses, 
the scale may thicken and become adherent, 
and pigmentary changes may develop, with 
hypopigmentation in the central or inactive 
area and hyperpigmentation at the active 
border. 

Initial treatment comprises the avoidance of 
direct sunlight. Following this, 
Hydroxychloroquine is the gold standard 
treatment. Other options include 
azathioprine, dapsone, thalidomide and 
tacrolimus. 


\?\ 

[Q: 2356] MRCPass - 2011 

1 ? 1 

[Q: 2357] MRCPass-2011 

• l 1 

/ 

i 

September 

•u 

_ y 

September 


A 47 year old lady presented to her GP with 
lesions in skin that were circular with an 
erythematous raised rim with central atrophy. 
There was scaliness, follicular plugging, and 
telangiectasia over the scalp, ears and face. 

This was confirmed to be discoid lupus by the 
dermatologist and she has been tried on 
betnovate steroid topical treatment but has 
not improved. 


A 41 year old lady has been seen for 
symptoms of joint stiffness, sw elling and 
pains. On examination, she has features of 
wrist subluxation, ulnar deviation of her hands 
and rheumatoid nodules. She is investigated 
with X rays of her hand. 

Which one of the following features is 
expected in rheumatoid arthritis? 

1- Subchondral sclerosis 


What should be used next? 
1- Diprobase cream 


2- Marginal osteophyte formation 

3- Subchondral cyst 


Dr. Khalid Yusuf El-Zohry - Sohag Teaching Hospital (01118391123) 

Ref MRCPoss OE OE 2012 PosTest 2009 PassMedicine 2009 PosTest Exam ReviseMRCP 

























El-zohry MRCP Questions Bank (Port 1) - 2013 


(For my personal use) 


4- Periarticular osteopenia 

5- Compression fractures 


Answer & Comments 

Answer: 4- Periarticular osteopenia 

Findings on XRay consistent with 
Rheumatoid Arthritis are: 

Narrowing of joint space 

■ Periarticular osteopenia 
Juxtaarticular bony Erosions 
Subluxation and gross deformity 
Periarticular soft tissue sw elling 

- XRay Findings in Osteoarthritis 

■ Non-uniform narrowing of joint space 

■ Subchondral bony sclerosis 
Marginal osteophyte formation 
Subchondral Cyst formation 



[Q: 2358] MRCPass-2011 
September 


A 23 year old male medical student was 
brought to hospital by his girlfriend who was 
concerned about his behaviour. 


He has just returned from a student elective in 
the United States. Whilst he was being 
assessed he appeared anxious and agitated. 
He was restless and paced up and down the 
corridor. He spoke very quickly. 

Upon questioning, he said that he was doing 
extremely well in medicine and soon was to 
become the dean of the medical school. 

Whot is the most likely diagnosis? 

1- Paranoid chizophrenia 

2- Anxiety disorder 

3- Hypomania 

4- Delusional disorder 


5- Obsessive compulsive disorder 


Answer & Comments 

Answer: 3- Hypomania 

Hypomania is a mood state characterized by 
persistent and pervasive elated or irritable 
mood, and thoughts and behaviour that are 
consistent with such a mood state. 

It is distinguished from mania by the absence 
of psychotic symptoms and by its lower 
degree of impact on functioning. Patients 
often have pressured speech and grandiosity. 

There may be flight of ideas, lack of sleep and 
inability to slow the mind down. 

[Q: 2359] MRCPass-2011 
September 

A 26 year old woman presents with lethargy, 
polyuria and nausea. She has no past medical 
history and is currently not taking 
medications. Her blood results are: sodium 
135 mmol/I, potassium 4.3 mmol/l, urea 7 
mmol/l, creatinine 90 pmol/l, calcium 3.2 
(2.25-2.7) mmol/l, phosphate 0.3 (0.8-8) 
pmol/l, Parathyroid hormone 18 (0.8-8) 

pmol/l. 

What is the likely cause of hypercaicaemia? 

1- Chronic kidney disease 

2- Hypophosphataemia 

3- Primary hyperparathyroidism 

4- 1,25 (OH) vitamin D supplementation 

5- Hypocalciuric hypophosphataemic rickets 



Answer & Comments 

Answer: 3- Primary hyperparathyroidism 

The case scenario is consistent with primary 
hyperparathyroidism. 

PTH enhances active reabsorption of calcium 
and magnesium from distal tubules and of the 
kidney. As bone is degraded both calcium and 
phosphate are released. It also greatly 


Dr. Khalid Yusuf El-Zohry - Sohag Teaching Hospital (01118391123) 

Ref MRCPoss OE OE 2012 PosTest 2009 PassMedicine 2009 PosTest Exom 




ReviseMRCP 

947 





























El-zohry MRCP Questions Bank (Port 1) - 2013 


(For my personal use) 


increases the excretion of phosphate, with a 
net loss in plasma phosphate concentration. 

By increasing the calcium:phosphate ratio 
more calcium is therefore free in the 
circulation. 

PTH enhances the absorption of calcium in the 
intestine by increasing the production of 
activated vitamin D. PTH 

up-regulates the enzyme responsible for 1- 
alpha hydroxylation of 25-hydroxy vitamin D, 
converting vitamin D to its active form (1,25- 
dihydroxy vitamin D). 

PTH stimulates bone resorption by 
osteoclasts. 



[Q: 2361] MRCPass - 2011 
September 


A 56 year old man with insulin dependent 
diabetes has routine follow up in the clinic. On 
examination, neovascularization was found on 
fundoscopy. Blood pressure was 146/92mm 
Hg. 


What is the treatment of choice? 


1- Better glycaemic control 

2- Follow up after 3 months 


3- Photocoagulation 

4- Better pressure control 


5- Statin 



[Q: 2360] MRCPass - 2011 
September 


A 71 year old man has a diagnosis of non small 
cell lung tumour, and has completed a set of 
investigations. 


Which one of the following is a 
controindicotion to lung surgery? 


1- FEV1 of 1.7 L (50% predicted) 


2- Horner's syndrome 

3- History of myocardial infarction 


4- Hypercalcaemia 

5- Peripheral neuropathy 


Answer & Comments 

Answer: 2- Horner's syndrome 

An FEV1 of < 1.1 L is a contraindication for 
most cardiothoracic surgical procedures. 

A malignant pleural effusion, distant 
metastases, contralateral mediastinal lymph 
node spread, vocal cord paralysis, phrenic 
nerve paralysis, Horner's syndrome, and SVC 
syndrome are contraindications to surgery in 
lung cancer. 


Answer & Comments 

Answer: 3- Photocoagulation 

Treatment is directed at regressing 
neovascularisation using Argon laser pan- 
retinal photocoagulation. 

The new vessels themselves are not targeted 
but photocoagulation is spread over a wide 
area in order to destroy ischaemic retina and 
remove the vasoproliferative stimulus. There 
is variable visual loss and night blindness. 


[Q: 2362] MRCPass-2011 
September 

A 56 year old lady has known mitral valve 
stenosis from rheumatic heart disease. She 
presents with new symptoms of 
breathlessness. 

Upon examination, which one of the following 
signs would suggest that she has mitral 
regurgitation? 

1- Displaced apex beat 

2- Early diastolic murmur in the pulmonary 
area 

3- Right ventricular heave 

4- V wave seen with the JVP 

5- Opening snap 



Dr. Khalid Yusuf El-Zohry - Sohag Teaching Hospital (01118391123) 

Ref MRCPass OE OE 2012 PasTest 2009 PassMedicine 2009 PasTest Exam ReviseMRCP 

































El-zohry MRCP Questions Bank (Port 1) - 2013 


(For my personal use) 


Answer & Comments 

Answer: 1- Displaced apex beat 

In mitral stenosis, the apex beat is classically 
tapping in nature and not displaced. 


pyrexial at 38 C. Breath sounds are clear and 
there is mild tenderness in the lower 
abdomen. There was no focal neurological 
signs. 

Blood tests reveal : 


Mitral regurgitation on the other hand, causes 
ventricular strain and dilatation. Most of the 
other signs described in the above options can 
occur with pulmonary hypertension due to 
significant mitral stenosis. 



[Q: 2363] MRCPass-2011 
September 


A 30 year old woman has been investigated 
for hypertension and weight gain. The 25 hour 
urine cortisol and dexamethasone suppression 
tests confirm significant Cushing's syndrome 
with excessively high cortisol levels. 


Which one of the following is the most like 
odd bose finding? 


1- Respiratory acidosis 

2- Respiratory alkalosis 

3- Metabolic alkalosis 

4- Metabolic acidosis 


5- Neutral pH 


Answer & Comments 

Answer: 3- Metabolic alkalosis 

With severe hypercortisolism, hypokalemic 
metabolic alkalosis may occur. 

Hypokalemic metabolic alkalosis may occur in 
patients with urinary free cortisol (UFC) levels 
higher than 1500 mcg/24-h. 


[Q: 2364] MRCPass-2011 
September 

A 66-year-old female is brought to A&E by her 
family, who are concerned about her 
increasing confusion over the past 2 days. 
There is a history of diarrhea in the preceding 
few days. On examination she is found to be 



Hb 9.6 g/dl 
WCC 12 x 10 9 /l 
Platelets 65 x 10 9 /l 
sodium 138 mmol/l 
potassium 4.7 mmol/l 
Urea 18.1 mmol/l 
Creatinine 210 pmol/l 

A blood film shows schistocytes and 
thrombocytopenia. 

Whot is the most likely diagnosis? 

1- Wegener's granulomatosis 

2- Thrombotic thrombocytopenic purpura 

3- Goodpasture's disease 

4- Idiopathic thrombocytopenic purpura 

5- Rapidly progressive glomerulonephritis 


Answer & Comments 

Answer: 2- Thrombotic thrombocytopenic 
purpura 

Thrombotic thrombocytopenic purpura (TTP), 
involvement of the CNS predominates in TTP 
(neurological signs) whilst in HUS there is 
mainly renal involvement. 

Most cases of TTP arise from inhibition of the 
enzyme ADAMTS13, a metalloprotease 
responsible for cleaving large multimers of 
von Willebrand factor (vWF) into smaller 
units. 

Neurologic symptoms (confusion, headaches, 
stroke), low platelet count, renal impairment 
and microangiopathic haemolytic anaemia are 
present. 


Dr. Kholid Yusuf El-Zohry - Sohog Teaching Hospital (01118391123) 

Ref MRCPoss OE OE 2012 PosTest 2009 PassMedicine 2009 PosTest Exom ReviseMRCP 



949 





























El-zohry MRCP Questions Bank (Part 1) - 2013 


(For my personal use) 


[Q: 2365] MRCPass-2011 
September 

A 36 year old lady upper middle class white 
lady who used to be an athlete has been 
referred for investigation due to tiredness. 
She mentions fatigue which is chronic and 
occurs even after minor physical work. This 
has been going on for 3 years. Investigations 
including CK, ESR, EMGs and muscle biopsy 
have revealed no obvious medical cause for 
this. 

Which of the following is the best treatment? 

1- Erythropoietin injection 

2- Cognitive behavioural therapy 

3- Graded exercise programme 

4- Codeine 

5- Fluoxetine 



Answer & Comments 

Answer: 3- Graded exercise programme 

Chronic Fatigue syndrome is defined by 
symptoms and not signs. 

The clinical profile of an individual with CFS is 
of a high-achieving student or athlete who 
usually is female (80%), white, and middle- 
class to upper middle-class. 

Treatment is largely supportive and 
responsive to symptomatology. This includes 
physical therapy and modest aerobic or 
anaerobic exercise (if possible) to avoid 
cardiovascular deconditioning. Sleep may be 
addressed with medication; often, melatonin 
or night-time amitriptyline is helpful. If 
present and severe, pain often is addressed in 
a pain clinic. 


L2J 

A 50 year-old woman developed symptoms 
decreased sleep and increased anxiety over 2 
months. She was constantly worried about 


[Q: 2366] MRCPass-2011 
September 


contracting bird flu. Although there was no 
epidemic at the moment, she refused to go 
out to the garden to collect the washing as she 
felt that there were birds which would cause 
flu transmission. 

Her husband witnessed that she was hearing 
voices when there was no one speaking. 

What is the diagnosis? 

1- Mania 

2- Psychotic depression 

3- Paranoid schizophrenia 

4- Conversion disorder 

5- Anxiety disorder 


Answer & Comments 

Answer: 3- Paranoid schizophrenia 

This lady has delusions and paranoid 
symptoms consistent with paranoid 
schizophrenia. 

Schizophrenia most commonly manifests as 
auditory hallucinations, paranoid or bizarre 
delusions, or disorganized speech and 
thinking, and it is accompanied by significant 
social or occupational dysfunction. . Delusions 
are persecutory and/or grandiose, but in 
addition to these, other themes such as 
jealousy, religiosity, or somatization may also 
be present. 


A 46-year-old woman presented with a chief 
complaint of heavy smoking and a desire to 
quit. She has a history of diabetes, 
hypertension, migraines and epilepsy. The 
patient's social histories revealed a 3 pack per 
year smoking history, minimal alcohol use, 
and no elicit drug use. Given her desire to quit 
smoking, the patient planned to be started on 
bupropion 100 mg daily for 1 week, with goal 
titration to 150 mg twice daily by the end of 
week. 



[Q: 2367] MRCPass-2011 
September 


Dr. Khalid Yusuf El-Zohry - Sohag Teaching Hospital (01118391123) 

Ref MRCPoss OE OE 2012 PosTest 2009 PassMedicine 2009 PosTest Exam 




ReviseMRCP 

950 




























El-zohry MRCP Questions Bank (Port 1) - 2013 


(For my personal use) 


Which one of the following is o 
contraindication ? 

1- Diabetes 

2- Hypertension 

3- Ischaemic heart disease 

4- Epilepsy 

5- Migraines 

Answer & Comments 

Answer: 4- Epilepsy 

Buproprion is an atypical antidepressant that 
acts as a norepinephrine and dopamine 
reuptake inhibitor, and nicotinic antagonist. 

Initially researched and marketed as an 
antidepressant, bupropion was subsequently 
found to be effective as a smoking cessation 
aid. 


There were some tear drop erythrocytes in his 
blood film with 4 normoblasts per 100 
leucocytes interspersed by myelocytes. The 
bone marrow biopsy showed replacement of 
normal haemopoietic elements by early 
fibrosis. 

What is the likely diagnosis? 

1- Chronic myeloid leukaemia 

2- Essential thrombocythaemia 

3- Myelofibrosis 

4- Multiple myeloma 

5- Waldenstrom's macroglobulinaemia 

Answer & Comments 

Answer: 3- Myelofibrosis 

Myelofibrosis is a chronic, progressive 
myeloproliferative disease. 


The manufacturers, GSK, advises that 
bupropion should not be prescribed to 
individuals with epilepsy or other conditions 
that lower the seizure threshold, such as 
alcohol or benzodiazepine discontinuation. 



[Q: 2368] MRCPass-2011 
September 

A 36-year old man was referred with a two- 
month history of generalised weakness, fever, 
and weight loss. 

There was no other relevant past medical 
history. Physical examination revealed a 
moderately wasted young man with severe 
pallor and pyrexia of 39°C. The spleen was 
palpable 6 cm below the left costal margin. 

Blood results showed: 

Hb was 9 g/dl 

haematocrit was 18% 

white cell count 5.3 x 10 9 /l 

platelet count was 89 x 10 9 /I 

His reticulocyte count was <0.0001%. 


It is characterised by prominent bone marrow 
stromal reaction including collagen fibrosis 
and osteosclerosis. 

Clinical features include lethargy, 
constitutional symptoms, transfusion 
dependent anaemia, splenomegaly, tear drop 
poikilocytosis, and a leucoerythroblastic blood 
film. A leucoerythroblastic picture on blood 
film is commonly seen in conditions with 
marrow infiltration. Immature cells 
(myelocytes and normoblasts) are also seen 
on the blood film. 


[Q: 2369] MRCPass-2011 
September 

A 14 year old male patient has long standing 
history of difficulty gaining weight, polyuria 
and recurrent infections. 

On examination, he had a blood pressure of 
120 / 70 mmHg, abdomen was soft, non 
tender and there were no localizing signs on 
neurological examination. Investigations 
showed a metabolic alkalosis. The patient was 
diagnosed with Bartter's syndrome. 



Dr. Khalid Yusuf El-Zohry - Sohag Teaching Hospital (01118391123) 

Ref MRCPoss OE OE 2012 PosTest 2009 PassMedicine 2009 PosTest Exom 




ReviseMRCP 

951 
























El-zohry MRCP Questions Bank (Port 1) - 2013 


(For my personal use) 


What other abnormality is likely to be present? 

1- Hyponatraemia 

2- Hypernatraemia 

3- Hypokalaemia 

4- Hyperchloraemia 

5- Hypermagnesaemia 

Answer & Comments 

Answer: 3- Hypokalaemia 

The clues here are hypokalaemia and normal 
blood pressure pointing tow ards Bartter's 
syndrome. 

Both Conn's and Cushing's syndrome are 
associated with hypertension. Bartter's 
syndrome (autosomal recessive) is an 
inherited renal tubular disorder characterized 
by hypokalemia, hypochloremic metabolic 
alkalosis, hyperreninemia, 

hyperprostaglandinism, normal blood 
pressure, with increased urinary loss of 
sodium, chloride, potassium, calcium. 

Vomiting, diarrhoea polyuria and poor growth 
are presenting symptoms. 

Different forms of renal tubular defects are 
found in Bartter's syndrome, and the Na-K-2CI 
transporter is a common defect. Failure to 
reabsorb chloride results in a failure to 
reabsorb sodium and leads to excessive 
sodium and chloride (salt) delivery to the 
distal tubules, leading to excessive salt 
(including calcium) and water loss from the 
body. The renin-angiotensin-aldosterone 
system is activated due to hypovolaemic state, 
but helps to maintain a normal blood 
pressure. 

[Q: 2370] MRCPass - 2011 
September 

A 20 year-old man presents with facial and 
ankle sw elling which has slow ly been 
developing over the past 2 weeks. There were 
no urinary symptoms and no family history of 


renal disease. On examination, he was pale 
and thin with ankle oedema and a blood 
pressure of 130/80. 

Investigations showed normal haemoglobin 
and white cell count and an erythrocyte 
sedimentation rate (ESR) of 32mm/hr. His 
blood urea was 9.1mmol/l (2.5-7.5), serum 
albumin 26g/l and a creatinine clearance of 
106ml/min. 

His serum immunoglobulin IgM and IgA, C3 
and C4 levels were normal. Antinuclear 
antibodies and hepatitis B surface antigen 
were not detected, and the antistreptolysin O 
titre was not raised. A urine dipstick shows 
protein ++++ and 24 hour urine collection 
demonstrated urinary protein loss of 
7.8g/day. 

What is the most likely cause of this 
presentation? 

1- IgA nephropathy 

2- Focal segmental glomerulonephritis 

3- Minimal change disease 

4- Membranous glomerulonephritis 

5- Renal cell carcinoma 

Answer & Comments 

Answer: 4- Membranous glomerulonephritis 

With no haematuria, IgA nephropathy is 
unlikely. 

The most likely options are minimal change 
and membranous. Due to the age of 
presentation, membranous 

glomerulonephritis is the best answer. 

Minimal change nephropathy is responsible 
for 90% of the cases of nephrotic syndrome in 
children less than 5 years of age. The name is 
due to the fact that the only detectable 
abnormality histologically is fusion and 
deformity of the foot processes under the 
electron microscope. It also occurs in adults - 
approx 20%. Normal renal function and blood 
pressure are typical. 



Dr. Khalid Yusuf El-Zohry - Sohag Teaching Hospital (01118391123) 

Ref MRCPass OE OE 2012 PasTest 2009 PassMedicine 2009 PasTest Exam ReviseMRCP 



952 






















El-zohry MRCP Questions Bank (Port 1) - 2013 


(For my personal use) 


Membranous glomerulonephritis often 
presents with nephrotic syndrome in males. It 
is characterized histologically by thickening of 
the capillary basement membrane secondary 
to the deposition of immune complexes. It is 
associated with SLE, drugs and malignancy. 
About 33% of patients affected go into 
spontaneous remission over five years, but 
the remainder are likely to develop 
progressive renal failure 

Focal segmental glomerulonephritis is a 
possibility for nephrotic presentation but is 
often associated with HIV and malignancy, and 
less common. 



[Q: 2371] MRCPass - 2011 
September 


A 35 year old lady has a past history of two 
episodes of deep vein thombosis and two 
miscarriages. During investigation she was 
found to have positive anti cardiolipin 
antibody. 


What is the best treatment? 


1- Clopidogrel 

2- Warfarin 3 months 


3- Long term low molecular weight heparin 

4- Aspirin and Warfarin 

5- Lifelong warfarin 


Answer & Comments 


Answer: 5- Lifelong warfarin 


Which one of the following drugs is 
contraindicated for use with sildenafil? 

1- Doxazosin 

2- Aspirin 

3- Nicorandil 

4- Atenolol 

5- Ramipril 


Answer & Comments 

Answer: 3- Nicorandil 

The combination of nitrates (and drugs such 
as nicorandil) with sildenafil is 
contraindicated. 

This combination must be avoided as it can 
produce significant hypotension and is 
potentially fatal. 



[Q: 2373] MRCPass-2011 
September 


A 18-year-old man presented casualty 
complaining of difficulty breathing. He had 
brought hospital by ambulance, having 
collapsed shortly after being stung on hand by 
a bee. On examination, his blood pressure was 
80/40 mmHg, facial sw elling and pharyngeal 
oedema was noted. 


Which one of following investigations likely 
confirm anaphylaxis? 

1- Haemolytic complement (CH50) level 

2- Serum tryptase activity 


This patient with recurrent DVTs has the 
presence of lupus anticoagulant. 

She requires lifelong warfarin treatment. 


3- Complement C3 level 

4- Total IgE level 

5- Eosinophil count 



[Q: 2372] MRCPass-2011 
September 


A 51-year-old man who had a large myocardial 
infarction of the anterior wall 20 years ago is 
on several cardiac medications. 


Answer & Comments 

Answer: 2- Serum tryptase activity 

The reaction involves preferential production 
of IgE, in response certain antigens, which in 
turn initiates a sequence of events leading to 
mast cell activation. 


Dr. Khalid Yusuf El-Zohry - Sohag Teaching Hospital (01118391123) 

Ref MRCPass OE OE 2012 PasTest 2009 PassMedicine 2009 PasTest Exam ReviseMRCP 































El-zohry MRCP Questions Bank (Part 1) - 2013 


(For my personal use) 


According to the Resuscitation council 
guidelines, the specific test to help confirm a 
diagnosis of an anaphylactic reaction is 
measurement of mast cell tryptase. 

Tryptase is the major protein component of 
mast cell secretory granules. In anaphylaxis, 
mast cell degranulation leads to markedly 
increased blood tryptase concentrations 
(Figure 4). Tryptase levels are useful in the 
follow -up of suspected anaphylactic 
reactions, not in the initial recognition and 
treatment: measuring tryptase levels must not 
delay initial resuscitation. Tryptase 
concentrations in the blood may not increase 
significantly until 30 minutes or more after the 
onset of symptoms, and peak 1-2 hours after 
onset. 

The half-life of tryptase is short 
(approximately 2 hours), and concentrations 
may be back to normal within 6-8 hours, so 
timing of any blood samples is very important. 

http://www.resus.org.uk/pages/reaction.pdf 



[Q: 2374] MRCPass-2011 
September 


A 41 year old patient with previous tonic 
clonic seizures is on sodium valproate 400mg 
bd. 


Which of the following is a common side-effect 
of sodium valproate? 

1- Gum hypertrophy 

2- Weight loss 

3- Hirsutism 

4- Tremor 

5- Thrombocytosis 


Answer & Comments 

Answer: 4- Tremor 

Side effects of sodium valproate are tremor, 
weight gain, transient hair loss and 
thrombocytopenia. 


Dr. Khalid Yusuf El-Zohry - Sohag Teaching Hospital (01118391123) 

Ref MRCPoss OE OE 2012 PosTest 2009 PassMedicine 2009 PosTest Exam ReviseMRCP 






















El-zohry MRCP Questions Bank (Port 1) - 2013 


(For my personal use) 


*+ [ Q: 2375 ] MRCPass - 2012 January 

/ - 

# A 55 year old man was brought to 
the A&E with sw elling of the face 
bronchospasm and urticaria after he took 
some unknown food items. 

On examination Blood pressure is 90/60mm 
of Hg. He gave a past history of food allergy. 

What is the route of administration of 
adrenaline which should be given? 


3- Penicillin 

4- Linezolid 

5- Ceftazidime 

Answer & Comments 

Answer: 2- Vancomycin 

This is a case of septic discitis (infection of the 
spinal disc) due to staphylococcus. 


1- Intravenous 

2- Intramuscular 

3- Subcutaneous 

4- Oral 

5- Inhaled 


Answer & Comments 

Answer: 2- Intramuscular 

Adrenaline 1 ml of a 1:1000 solution (1 mg) 
should be injected intramuscularly to treat 
anaphylaxis. 


[ Q: 2376 ] MRCPass - 2012 January 

A 40-year-old man was admitted 
with acute-onset lower back. 

On examination he was pyrexial. He had 
restricted lumbar spine movement with pain 
at this site. He is currently on 
antihypertensive drugs and is allergic to 
penicillin. 



Back pain, fever and neurological signs are 
often present in septic discitis. 40% of cases 
are due to staph aureus, and blood cultures 
are often positive. MRI of the spine will help 
to confirm the diagnosis. Treatment options 
include vancomycin, gentamicin and 
ceftazidime. The best first line option here 
would be vancomycin. 


[ Q: 2377 ] MRCPass - 2012 January 

A 65-year-old man is brought to 
A&E by his family, who are concerned him 
being tired and unwell. He has a previous 
history of myocardial infarction. He takes 
ramipril and bendrofluazide tablets. There 
was a history of diarrhea in the preceding few 
days. On examination his breath sounds are 
clear and there is mild tenderness in the 
lower abdomen. 

Blood tests reveal : 

Hb 9.4 g/dl 

WCC 12 x 10 9 /l 



Investigations showed a white cell count of 

22.0x1071 (4.0-11.0x1071), urea 8.0 mmol/l 

(2.5-7.5 mmol/l), creatinine of 160 [imol/l 
(60-120 pmol/l), erythrocyte sedimentation 
rate (ESR) of 108 mm/h and CRP of 210 mg/I 
(<20 mg/I). 

Blood cultures grew Methicillin sensitive 
staph aureus. 

What antibiotics should be commenced? 

1- Clarithromycin 

2- Vancomycin 


Platelets 65 x 1071 

sodium 138 mmol/l 

potassium 4.7 mmol/l 

Urea 23.1 mmol/l 

Creatinine 366 pmol/l 

What is the blood film likely to show ? 

1- Target cells 

2- Howell jolly body 

3- Red cell casts 


Dr. Khalid Yusuf El-Zohry - Sohag Teaching Hospital (01118391123) . 

Ref MRCPass OE OE 2012 PasTest 2009 PassMedicine 2009 PasTest Exam ReviseMRCP 955 






























El-zohry MRCP Questions Bank (Part 1) - 2013 


(For my personal use) 


4- Fragmented red blood cells 

5- Spherocytes 


Answer & Comments 

Answer: 4- Fragmented red blood cells 

The diagnosis is Hemolytic uremic syndrome 
(HUS). 

It is characterized by the triad of 
microangiopathic hemolytic anemia, 
thrombocytopenia, and acute renal failure. 
Diarrhea and upper respiratory infection are 
the most common precipitating factors. The 
most common cause of HUS is a toxin 
produced by Escherichia coli serotype 
0157:H7. 

Additional agents include Shigella, 
Salmonella, Yersinia, and Campylobacter 
species. 

Although the vascular lesions are identical in 
HUS and Thrombotic thrombocytopenic 
purpura (TTP), involvement of the CNS 
predominates in TTP (neurological signs) 
whilst in HUS there is mainly renal 
involvement. 

The hallmark of HUS in the peripheral smear 
is the presence of schistocytes. These consist 
of fragmented, deformed, irregular, or 
helmet-shaped RBCs 



[ Q: 2378 ] MRCPass - 2012 January 

A 22 year old woman presents 
bilateral leg weakness following an episode of 
diarrhoea 2 weeks ago. She is suspected of 
having Guillain-Barre syndrome. 


Which of the following test results is likely on 
the neurological tests? 

1- Fibrillations on the EMG 


2- Slow conduction velocities in the nerve 
conduction study 

3- Decreased visual evoked potential 


4- Increased amplitude in the nerve 

conduction study 

5- Shortened latencies in the nerve 

conduction study 


Answer & Comments 

Answer: 2- Slow conduction velocities in the 
nerve conduction study 

In Guillain Barre syndrome, Electromyography 
(EMG) and nerve conduction studies (NCS) 
may show prolonged distal latencies, 
conduction slowing, conduction block, and 
temporal dispersion of compound action 
potential in demyelinating cases. 


[ Q: 2379 ] MRCPass - 2012 January 

A 23 year old man with learning 
difficulties is brought for review by his 
worried parents after he complained of visual 
blurring. 

Examination with a slit lamp reveals ectopia 
lentis. 

What is the most likely diagnosis? 

1- Marfan's syndrome 

2- Klinefelter's syndrome 

3- Ehler Danlos 

4- Homocystinuria 

5- Fragile X syndrome 



Answer & Comments 

Answer: 4- Homocystinuria 

Ectopia lentis/ subluxation of the lens is 
associated with Ehlers Danlos syndrome, 
Marfan's syndrome and homocystinuria. 

There is downwards lens dislocation in 
homocystinuria. 


[ Q: 2380 ] MRCPass - 2012 January 


A 62 year old lady presented with a 
fall and fractured writst. She was organised to 



Dr. Khalid Yusuf El-Zohry - Sohag Teaching Hospital (01118391123) 

Ref MRCPass OE OE 2012 PasTest 2009 PassMedicine 2009 PasTest Exam ReviseMRCP 



956 






























El-zohry MRCP Questions Bank (Port 1) - 2013 


(For my personal use) 


have a DEXA scan. This showed a T score of - 
2.6 in the hip and a score of -2.1 in the femur. 

What does this mean? 

1- Normal values on the scan 

2- osteopenia of the hip and osteoporosis of 
the femur 

3- Osteoporosis of the hip and osteopenia of 
the femur 

4- Osteopenia of both areas 

5- Osteoporosis of both areas 

Answer & Comments 

Answer: 3- Osteoporosis of the hip and 
osteopenia of the femur 

The T score is usually used to make treatment 
decisions using standard deviation (SD). 

The SD measures the difference between the 
BMD and that of a healthy young adult (the 
reference value). Every -1 SD ("minus 1 
standard deviation") equals a 10 to 12% 
decrease in bone density. T score results are 
classified as follows: 


potassium 2.4 mmol/l, 

calcium 1.6 (2.25-2.7) mmol/l 

phosphate 0.8 (0.8-8) pmol/l, 

chloride 86 (95-107) mmol/l 

bicarbonate 33 (20-28) mmol/l 

What other abnormality is likely to be found? 

1- Hyperthyroidism 

2- Hyperaldosteronism 

3- Hypoglycaemia 

4- Hyperlipidaemia 

5- Hypomagnesaemia 

Answer & Comments 

Answer: 5- Hypomagnesaemia 

The laboratory findings of hypokalaemia, 
hypocalcaemia, hypomagnesaemia and 
metabolic alkalosis can occur in severe 
alcoholic patients. 

Correction of these electrolyte abnormalities 
with cautious refeeding is important. 


A T score between 0 and -1 standard 
deviation (SD) is considered to be normal. 

A T score between -1 and -2.5 SD is classified 
as osteopenia (low bone mass). 


[ Q: 2382 ] MRCPass - 2012 January 

A 43-year-old woman is found to be 
hypertensive but referred by the GP due to 
poorly controlled hypertension. 



A T score of -2.5 SD or less is classified as 
osteoporosis (very low bone mass). 

^ [ Q: 2381 ] MRCPass - 2012 January 

/ - 

# A 58-year-old Caucasian male 

presents with weight loss and severe 
muscular weakness. He was a heavy smoker 
and drinks 20 units of alcohol per day. His 
medical history included hypertension and 
gastrointestinal reflux. On examination, he 
had a blood pressure of 120 / 70 mmHg, 
abdomen was soft, non tender. 

Investigations showed: 

sodium 136 mmol/l 


The patient received 3 antihypertensive 
medications including a beta-blocker, diuretic 
and a calcium channel blocker. Her blood 
pressure is consistenly above 150 mmHg 
systolic. There is no family history of 
hypertension. 

Her renal function is normal but urine dipstick 
testing shows + blood. A renal ultrasound 
shows kidney sizes of 7.5 in the right and 7 
cm in the left. 

What is the most likely cause of this patient's 
hypertension ? 

1- Autosomal dominant polycystic kidney 
disease 


Dr. Khalid Yusuf El-Zohry - Sohag Teaching Hospital (01118391123) . 

Ref MRCPass OE OE 2012 PasTest 2009 PassMedicine 2009 PasTest Exam ReviseMRCP 957 

























El-zohry MRCP Questions Bank (Part 1) - 2013 


(For my personal use) 


2- Conn's syndrome 

3- Essential hypertension 

4- Fibromuscular dysplasia 

5- Cushing's syndrome 

Answer & Comments 

Answer: 4- Fibromuscular dysplasia 

Fibromuscular dysplasia (FMD) is a 
nonatherosclerotic angiopathy of unknown 
etiology. 


4- L5/S1 disc prolapse 

5- Common peroneal nerve injury 

Answer & Comments 

Answer: 4- L5/S1 disc prolapse 

Ankle dorsiflexion is generally supplied by 
L4/L5 and plantar flexion supplied by S1/S2. 

The history of back pain and neurological 
signs is consistent with an L5/S1 disc 
prolapse. 


Medial FMD represents the most common 
type and is characterized by the classic "string 
of beads" appearance. FMD usually affects 
females between 15 and 50 years of age, 
frequently involves the mid or/and distal 
segments of the renal artery and is bilateral in 
2/3 of the patients. Often, hypertension tends 
to be refractory to simple drug therapy. 

Percutaneous transluminal renal angioplasty 
(PTRA) should be considered in well-defined 
groups of patients: those with a recent onset 
of hypertension (in particular patients 
younger than 50 who are less likely to have 
underlying atherosclerotic disease) in whom 
the goal is to cure the hypertension. 


^ [ Q: 2383 ] MRCPass - 2012 January 

t - u - 

# A 43 year old man presents to A+E 
complaining of severe lower back pain 
following carpentry work. The pain radiates 
to his left buttock and thigh. On examination, 
he was able to straight leg raise to 45 degrees 
only on the left side. The sciatic stretch test is 
positive. He has difficulty plantar flexing his 
left ankle and has abnormal sensation on the 
plantar aspect of the foot. The left ankle 
reflex was difficult to elicit. 

What is the diagnosis? 

1- Cauda equina syndrome 

2- L2/L3 disc prolapse 

3- L4/L5 disc prolapse 


[ Q: 2384 ] MRCPass - 2012 January 

A 55-year-old female presented 
with dyspnea, orthopnea, lower extremities 
sw elling, palpitations. Physical examination 
revealed an irregular pulse 80 beats/min, 
blood pressure of 100/60 mmHg and bibasal 
crackles on chest auscultation. The jugular 
veins were distended and there was evidence 
of CV waves. She had a left parasternal heave. 
There was a systolic murmur at the upper left 
border of the sternum which was louder with 
inspiration. The second heart sounds were 
fixed, wide and split. 

What is the most likely diagnosis? 

1- Aortic stenosis 

2- Mitral regurgitation 

3- Atrial septal defect 

4- Pulmonary stenosis 

5- Pulmonary regurgitation 



Answer & Comments 

Answer: 3- Atrial septal defect 

In an ASD, there may be an ejection systolic 
murmur that is due to the increased flow of 
blood through the pulmonic valve rather than 
any structural abnormality of the valve 
leaflets. 

In unaffected individuals, there are 
respiratory variations in the splitting of the 


Dr. Khalid Yusuf El-Zohry - Sohag Teaching Hospital (01118391123) 

Ref MRCPass OE OE 2012 PasTest 2009 PassMedicine 2009 PasTest Exam ReviseMRCP 



958 




























El-zohry MRCP Questions Bank (Part 1) - 2013 


(For my personal use) 


second heart sound (S2). In individuals with 
an ASD, there is a fixed splitting of S2. 

This does not occur with pulmonary stenosis. 
There is also evidence of right heart strain 
(tricuspid regurgitation, right ventricular 
hypertrophy causing left parasternal heave). 


^ [ Q: 2385 ] MRCPass - 2012 January 

fi - 

# A 35-year-old Caucasian woman 
admitted has a history of widespread, 
pruritic, erythematous skin rash, joint pains 
and renal disease. She recently had a baby 
with congenital heart block. Laboratory 
investigations revealed mild leucopenia (w 
hite cell count 3.25xl09/ml) and 
thrombocytopenia (platelets 140xl09/ml). 
Erythrocyte sedimention rate was increased 
(65 mm/h). 

Which antibody is likely to be positive? 

1- Anti Jo 1 


placenta. These autoantibodies can be found 
alone or in combination; However, anti-Ro is 
present in almost 95% of patients. 


^ [ Q: 2386 ] MRCPass - 2012 January 

ft - 

* A 30 year old lady presented with 
joint pains which has been present for 2 years 
but were getting more severe. 

After clinical examination, blood tests and x 
rays, she has been diagnosed with 
rheumatoid arthritis. The rheumatoid factor 
was negative. Hand X rays showed osteopenia 
and joint erosions. 

Which one of the following carries the poorest 
prognostic factor? 

1- Rheumatoid factor negative 

2- Joint erosions 

3- Female sex 

4- Insidious onset 


2- Anti double stranded DNA 


5- Age of 30 years 


3- Anti Ro 

4- Anti centromere 

5- ANCA 


Answer & Comments 

Answer: 3- Anti Ro 

The anti Ro antibody is associated with 
Sjogren's syndrome, SLE and neonatal lupus. 

Neonatal lupus erythematosus (NLE) is a rare 
disorder caused by the transplacental passage 
of maternal autoantibodies. Only 1% of 
infants with positive maternal autoantibodies 
develop neonatal lupus erythematosus. The 
most common clinical manifestations are 
cardiac (congenital heart block), dermatologic 
(urticaria and skin desquamation) , and 
hepatic (abnormal LFTs). The mother 
produces immunoglobulin G (IgG) 
autoantibodies against Ro (SSA), La (SSB), 
and/or Ul-ribonucleoprotein (Ul-RNP), and 
they are passively transported across the 


Answer & Comments 

Answer: 2- Joint erosions 

Poor prognostic factors in Rheumatoid 
arthritis include persistent synovitis, early 
erosive disease, extra-articular findings 
(including subcutaneous rheumatoid 
nodules), positive serum RF findings, positive 
serum anti-CCP autoantibodies, carrier of 
HLA-DR4 alleles, family history of RA, poor 
functional status, socioeconomic factors, 
elevated acute phase response ESR, CRP, and 
increased clinical severity. 



[ Q: 2387 ] MRCPass - 2012 January 

A 42-year-old man with end-stage 
renal disease (ESRD) who was receiving 
hemodialysis was admitted with fevers and 
rigors. 


He has been having dialysis via a tunnelled 
subclavian central venous cathether for 


Dr. Khalid Yusuf El-Zohry - Sohag Teaching Hospital (01118391123) 

Ref MRCPass OE OE 2012 PasTest 2009 PassMedicine 2009 PasTest Exam ReviseMRCP 




























El-zohry MRCP Questions Bank (Part 1) - 2013 


(For my personal use) 


several weeks. The patient was febrile and 
had a WBC count of 18 x 10 3 /|iL. 

What is the most likely cause of septicaemia 
in this patient? 

1- Escherichia coli 

2- Listeria monocytogenes 

3- Staphylococcus aureus 

4- Staphylococcus epidermidis 

5- Streptococcus pyogenes 


Answer & Comments 

Answer: 2- Small cell 

Small-cell lung cancer accounts for 
approximately 20-25% of all cases of lung 
cancer. 

It is strongly associated with smoking. Small 
cell carcinomas produce ACTH and ADH, 
which can lead to SIADH, the clue in the 
history is hyponatraemia. 


Answer & Comments 

Answer: 3- Staphylococcus aureus 

Staph aureus and pseudomonas are common 
organisms causing sepsis in dialysis patients 
with central venous lines. 

Coagulase negative staph (staph epidermidis) 
are common commensal organisms but do 
not usually lead to sepsis in these patients. 


[ Q: 2389 ] MRCPass - 2012 January 

A 20-year-old man presents with 
discrete scaly papules affecting his trunk and 
upper arms. The eruption developed 2 weeks 
after an episode of acute exudative tonsillitis. 
He was treated with a course of penicillin for 
5 days. On examination, there were small, 
drop-like, salmon-pink, 1 mm to 10 mm 
papules with a fine scale on the trunk and 
arms. 



[ Q: 2388 ] MRCPass - 2012 January 

A 47-year-old man with habitual 
smoking habit (2 packs per day) presents with 
haemoptysis. He also suffered had poor 
appetite for 6 months and body weight loss 
(loss 6 kilograms). A chest X ray shows a 
suspicious lesion in the left lung. His blood 
results are: 

Hb 11.5 g/dl, MCV 75 fl, WCC 6 x 10 9 /l, 
platelets 200 x 10 9 /l, sodium 125 mmol/l, 
potassium 4.5 mmol/l, urea 5 mmol/l, 
creatinine 100 |imol/l. 

Which one of the following is the most likely 
lung cancer in this patient? 

1- Squamous cell 



What is the diagnosis? 

1- Erythema multiforme 

2- Secondary syphilis 

3- Guttate psoriasis 

4- Dermatitis herpetiformis 

5- Pityriasis versicolor 


Answer & Comments 

Answer: 3- Guttate psoriasis 

Guttate psoriasis refers to a distinctive, acute 
clinical presentation of an eruption 
characterized by small, droplike, 1-10 mm in 
diameter, salmon-pink papules, usually with a 
fine scale. 


2- Small cell 

3- Adenocarcinoma 

4- Large cell 

5- Carcinoid 


It is more common in individuals younger 
than 30 years, a history of upper respiratory 
infection secondary to group A beta- 
hemolytic streptococci (eg, Streptococcus 
pyogenes) often precedes the eruption by 2-3 
weeks. Although recurrent episodes may 
occur, especially those due to pharyngeal 


Dr. Khalid Yusuf El-Zohry - Sohag Teaching Hospital (01118391123) 

Ref MRCPass OE OE 2012 PasTest 2009 PassMedicine 2009 PasTest Exam ReviseMRCP 



960 





























El-zohry MRCP Questions Bank (Part 1) - 2013 


(For my personal use) 


carriage of streptococci, isolated bouts are 
known to occur. Usually, the psoriasis 
spontaneously disappears in a few weeks 
without treatment. Simple reassurance and 
emollients may be sufficient care. 


[ Q: 2390 ] MRCPass - 2012 January 

Which one of the following HLA 
groups is genetically associated with 
rheumatoid arthritis? 

1- HLA B27 

2- HLA B6 

3- HLA DR3 

4- HLA DR4 

5- HLA A8 



Answer & Comments 

Answer 4- HLA DR4 

Rheumatoid arthritis has a genetic link with 
HLA-DR4 and related allotypes of MHC Class II 
and the T cell-associated protein PTPN22. 


[ Q: 2391 ] MRCPass - 2012 January 

A 30 year old woman is admitted to 
hospital due to concerns from her relatives 
with lack of sleep and agitation. 

When giving a history, she says that she is the 
Queen of England and continues talking 
despite interruption by the doctor. 

What is this feature? 

1- Circumstantiality 

2- Flight of ideas 

3- Pressure of speech 

4- Paranoia 

5- Thought intrusion 



Answer & Comments 


Answer: 3- Pressure of speech 


Patients with mania often have pressured 
speech and grandiosity. 

There may be flight of ideas, lack of sleep and 
inability to slow the mind down. Pressure of 
speech is a tendency to speak rapidly and 
frenziedly, as if motivated by an urgency not 
apparent to the listener. 


[ Q: 2392 ] MRCPass - 2012 January 

A new blood test is being evaluated 
for measuring the likelihood of heart failure 
(HF), as compared to echocardiography as a 
gold standard. In the study, the following 
results are obtained: 



Blood test 
positive 

Blood test 
negative 

Echo-HF 

720 

10 "1 

Echo - no HF 

30 1 

f 890 


What is the negative predictive value of the 
blood test? 

1- 30/750 

2- 30/780 

3- 890/900 

4- 890/10 

5- 890/30 



Answer & Comments 

Answer: 3- 890/900 

Negative predictive value is the number of 
true negatives found by a negative blood test, 
in this case, 890 / (890 + 10). 


[ Q: 2393 ] MRCPass - 2012 January 

A 55 year old man presented with 
retrosternal chest pain. 

He has history of hypertension and diabetes. 

His ECG shows anterior wall myocardial 
infarction with ST elevation in leads VI to V4. 
He has been given Aspirin, Clopidogrel and 



severe 


Dr. Khalid Yusuf El-Zohry - Sohag Teaching Hospital (01118391123) 

Ref MRCPass OE OE 2012 PasTest 2009 PassMedicine 2009 PasTest Exam ReviseMRCP 







































El-zohry MRCP Questions Bank (Part 1) - 2013 


(For my personal use) 


continues to have chest pains. Tirofibran is 
then commenced. 

What is tirofibran's mechanism of action? 

1- Tissue plasminogen activator 

2- Streptokinase 

3- Statin 

4- Direct Thrombin inhibitor 

5- GIIBIIIA inhibitor 


Answer & Comments 

Answer: 5- GIIBIIIA inhibitor 

An unstable patient with coronary arterial 
disease can be considered for a GIIBIIIA 
inhibitor such as tirofibran whilst awaiting 
coronary angiography. 

Abxicimab, another GIIBIIIA inhibitor, is also 
frequently used prior to angioplasty. 


[ Q: 2394 ] MRCPass - 2012 January 


A 72 years old woman was admitted 
with complaints of anorexia, nausea and 
lethargy for last 4 months. She had history of 
fall 12 months back after which she sustained 
mild compression fracture of LI vertebra. 
After the fall, she has had persistent 
backaches. On examination, she was pale, BP 
was 160/90 mmHg and bilateral pedal 
oedema. There was tenderness over the 
upper lumbar region. 

Blood tests showed: 


Hb 10.5 g/dl, WCC 7 x 10 9 /l, platelets 220 x 
10 9 /l, ESR 90 mm/hr, sodium 135 mmol/I, 
potassium 4.2 mmol/l, urea 16 mmol/I, 
creatinine 240 pmol/l, calcium 2.9 (2.25-2.7) 
mmol/l, phosphate 0.6 (0.8-8) pmol/l. IgA 3.2 
(0.5-4.0) g/l , IgG 23 (5.0-13.0) g/l, IgM 2.3 
(0.3-2.2) g/l. 

Routine urine examination showed urine 
albumin trace, urine protein/creatinine ratio 
2.7 and urinary Bence Jones protein was 
positive. 


What is the likely diagnosis? 

1- Waldenstrom's macroglobulinaemia 

2- Multiple myeloma 

3- Chronic myeloid leukaemia 

4- Acute lymphocytic leukaemia 

5- Osteoporosis 


Answer & Comments 

Answer: 2- Multiple myeloma 

The presence of unexplained anemia, kidney 
dysfunction, a high erythrocyte 
sedimentation rate (ESR), hypercalcaemia and 
a high serum paraprotein with bence jones 
proteinuria suggests multiple myeloma. 


^ [ Q: 2395 ] MRCPass - 2012 January 

/ --— 

* A 30 year old female took 40 tablets 
of Paracetamol and was admitted to hospital. 

She is seen the following day and needs 
assessment of her medical condition. 

Which of the following is the best 
investigation to assess prognosis after 26 
hours for a paracetamol overdose? 

1- Prothrombin time 

2- AST 

3- Paracetamol level 

4- Urea and creatinine 

5- Bilirubin 


Answer & Comments 

Answer: 1- Prothrombin time 

Although all of the tests may be abnormal, 
the INR / prothrombin time measurement is 
the most important in predicting prognosis 
(part of the child pugh criteria for liver failure) 
after a paracetamol overdose. 


[ Q: 2396 ] MRCPass - 2012 January 
A 70 year old man presents with a 



Dr. Khalid Yusuf El-Zohry - Sohag Teaching Hospital (01118391123) 

Ref MRCPass OE OE 2012 PasTest 2009 PassMedicine 2009 PasTest Exam ReviseMRCP 




























El-zohry MRCP Questions Bank (Part 1) - 2013 


(For my personal use) 


history of breathlessness. 

He has a past medical history of hypertension 
and a myocardial infarction 5 years ago. He 
smokes 20 cigarettes and drinks 2 units of 
alcohol per day. On examination, BP was 
120/70, 02 sats 95% on air. Cardiovascular 
and respiratory examination was 
unremarkable. 

Neurological examination revealed a right 
sided eye ptosis and miosis. Eye movements 
were normal. 

What investigation should be done to confirm 
the diagnosis? 

1- MRI of the brain 

2- Chest X ray 

3- ECG 

4- Fundoscopy 

5- CT scan of the abdomen 


Answer & Comments 

Answer: 2- Chest X ray 

This patient who is a heavy smoker also has 
signs of horner's syndrome on the right side. 

This suggests a possible Pancoast tumour of 
the lung. A space occupying lesion, a brain 
stem CVA, trauma to the neck and also 
Pancoasts' tumour may all cause Horner's 
syndrome. 


^ [ Q: 2397 ] MRCPass - 2012 January 

# I 

Which ion / channel is primarily 
responsible for repolarisation phase in the 
cardiac cells? 

1- Sodium 

2- Calcium 

3- Phosphate 

4- Magnesium 

5- Potassium 


Answer & Comments 

Answer: 5- Potassium 

Depolarisation and repolarisation refer to 
neuronal conduction or cardiac electrical 
activity. 

The action potentials produced by 
depolarization (due to Na+ ions moving into 
cell), leads to release of Ca2+ ions which lead 
to contraction of cardiac muscle, followed by 
repolarization (K+ ions moving into cell). 



[ Q: 2398 ] MRCPass - 2012 January 

A 32 year old woman has presented 
with recurrent sinusitis and symptoms of 
haemoptysis for 6 months. Urine dipstick 
showed blood ++ and she was noted to have 
renal impairment with a creatinine of 160 
umol/l. A Chest XR showed upper lobe 
infiltrates. 


Investigations showed: 

urine sediment - many red blood cell and 
granular casts 

erythrocyte sedimentation rate (ESR) was 50 
mm/hr 

anti-nuclear antibody (ANA) - borderline 
positive 1:40 

anti-streptolysin O (ASO) antibody -< 1:40 

c-ANCA - positive at a titer of 1:320, PR 3 
positive 

p - ANCA - negative 

What is a renal biopsy likely to show ? 

1- Crescentic glomerulonephritis 

2- Minimal change glomerulonephritis 

3- IgA nephropathy 

4- Post streptococcal glomerulonephritis 

5- Goodpasture's syndrome 


Answer & Comments 


Answer: 1- Crescentic glomerulonephritis 


Dr. Khalid Yusuf El-Zohry - Sohag Teaching Hospital (01118391123) 

Ref MRCPass OE OE 2012 PasTest 2009 PassMedicine 2009 PasTest Exam ReviseMRCP 




























El-zohry MRCP Questions Bank (Part 1) - 2013 


(For my personal use) 


Crescentic glomeurloneprhtis, also known as 
Rapidly progressive glomerulonephritis 
(RPGN) is characterized by a rapid loss of 
renal function, (usually a 50% decline in the 
glomerular filtration rate (GFR) within 3 
months) with glomerular crescent formation 
seen in at least 50% of glomeruli seen on 
kidney biopsies. 

In 50% of cases, RPGN is associated with an 
underlying disease such as Goodpasture 
syndrome, systemic lupus erythematosus, or 
Wegener granulomatosis; the remaining 
cases are idiopathic. Regardless of the 
underlying cause, RPGN involves severe injury 
to the kidneys' glomeruli, with many of the 
glomeruli containing characteristic glomerular 
crescents (crescent-shaped scars). 


[ Q: 2399 ] MRCPass - 2012 January 

A 30 year old man has been on a 
recent trip on a holiday visiting his family. He 
has not previously been immunized with BCG. 
He now feels unwell with fevers and lethargy 
but did not have a productive cough. He also 
mentioned he was recently in contact with a 
friend who had active tuberculosis infection 
confirmed through positive sputum culture. 

What should be done for him now? 

1- Mantoux test 

2- Quantiferon test 

3- BCG immunisation 

4- Isoniazid prophylaxis 

5- Start quadruple TB therapy 



Answer & Comments 


Answer: 1- Mantoux test 


intradermal injection of tuberculin purified 
protein derivative (PPD). If the mantoux test 
showed a strongly positive response then the 
patient is likely to have active tuberculosis 
and will need treatment. The best diagnostic 
tool in suspected pulmonary TB is sputum 
stain for AFB but this patient does not have a 
productive cough. A quantiferon test is a 
blood test which is helpful but can be non 
specific and should not be used as a primary 
diagnostic tool. 


[ Q: 2400 ] MRCPass - 2012 January 

A 55 year woman presents with a 
red eye on the left. It only began last night 
and she complained of pain and blurred 
vision. On examination, there was left corneal 
edema, and dilated pupils. 

Which one of the following is most likely? 

1- Central retinal vein occlusion 

2- Closed angle glaucoma 

3- Vitreous haemorrhage 

4- Partial retinal detachment 

5- Cataract 



Answer & Comments 

Answer: 2- Closed angle glaucoma 

In acute closed angle glaucoma, intra-ocular 
pressure is increased as a result of an 
impairment of aqueous outflow. 

The closure is due to contact of the peripheral 
iris with the trabecular meshw ork at the 
entrance to Schlemm's canal. 

The commonest presentation is with a red 
eye. 


As this patient has not been previously 
immunised, a mantoux text is helpful because 
it is likely to be negative if he did not have TB 
infection. 

The Mantoux Test (MT) is a classical delayed- 
type hypersensitivity (DTH) response to the 


[ Q: 2401 ] MRCPass - 2012 January 

A 46 year old man is known to have 
ulcerative colitis which was diagnosed 10 
years ago. Over the last two months he has 
right upper quadrant discomfort and noticed 



Dr. Khalid Yusuf El-Zohry - Sohag Teaching Hospital (01118391123) 

Ref MRCPass OE OE 2012 PasTest 2009 PassMedicine 2009 PasTest Exam ReviseMRCP 



964 



























El-zohry MRCP Questions Bank (Part 1) - 2013 


(For my personal use) 


to have jaundice. He was referred to the 
gastroenterology outpatients for assessment. 
Investigations performed showed these 
results: Ultrasound showed a dilated intra 
and extrahepatic ducts with beaded 
appearances. Blood tests results: 

ALT 120 (5-35) U/l 

AST 90(1-31) U/l 

ALP 750 (20-120) U/l 

Bilirubin 80 (1-22) pmol/l 

Albumin 38 (37-49) g/l 

amylase 250 (60-180) U/l 

What test should be done to confirm the 
diagnosis? 

1- Chest X Ray 

2- OGD 

3- CT scan of abdomen 

4- Gamma GT 

5- Magnetic resonance 

cholangiopancreatography 


Answer & Comments 

Answer: 5- Magnetic resonance 

cholangiopancreatography 

The cholestatic picture along with 
inflammatory lesion seen on the ultrasound 
hints tow ards either Sclerosing cholangitis or 
associated with ulcerative colitis. 

Narrowing of the bile duct lumen usually 
occurs throughout the biliary tree. Less 
commonly, changes are confined to the intra- 
hepatic or extra-hepatic ducts. Primary 
sclerosing cholangitis is a rare disease of 
unknown aetiology characterised by chronic 
inflammation and fibrosis of the bile duct. 
Primary sclerosing cholangitis is associated 
with cholangiocarcinoma, a cancer of the 
biliary tree, and the lifetime risk for PSC 
sufferers is 10-15%. As many as 5% of 
patients with ulcerative colitis may progress 
to develop primary sclerosing cholangitis. 


The diagnosis is by imaging of the bile duct, 
usually in the setting of endoscopic 
retrograde cholangiopancreatography (ERCP) 
which shows "beading" (both strictures and 
dilation) of the intrahepatic and extrahepatic 
bile ducts. Another option is magnetic 
resonance cholangiopancreatography 

(MRCP), where magnetic resonance imaging 
is used to visualise the biliary tract. 


[ Q: 2402 ] MRCPass - 2012 January 

A 56-year-old, unemployed, man 
was admitted due to confusion and 
disorientation with the onset of acute 
symptoms a few days before admission. He 
had a history of alcohol dependence of 20 
years duration. He was aw ake but 
disorientated to person, location, situation, 
and time. 

Temperature was 36.5 °C and BP was 120/80 
mmHg. Neurological examination revealed 
gaze-evoked nystagmus in all directions. All 
deep tendon reflexes were normal. The 
finger-nose test was ataxic. Standing and gait 
with open eyes evidenced a distinct ataxia 
with tremors of the upper extremity. The 
Romberg sign was positive. He had an MMSE 
score of 26 /30 on admission. A blood glucose 
measurement was 3 mmol/I. 

What should be given to treat the patient? 

1- 50 ml of 50% dextrose 

2- 500 mis of 5% dextrose 

3- Thiamine infusion 

4- Lactulose 

5- Chlordiazepoxide 



Answer & Comments 

Answer: 3- Thiamine infusion 

This patient is exhibiting features of 
Wernicke's encephalopathy, which are 
nystagmus, ataxia and confusion. 


Dr. Khalid Yusuf El-Zohry - Sohag Teaching Hospital (01118391123) . 

Ref MRCPass OE OE 2012 PasTest 2009 PassMedicine 2009 PasTest Exam ReviseMRCP 965 























El-zohry MRCP Questions Bank (Part 1) - 2013 


(For my personal use) 


The daily thiamine requirement for healthy 
individuals is between 1 and 2 mg/day but 
both alcohol and malnutrition may interfere 
with the absorption of thiamine. Thiamine 
dependent enzymes such as transketolase 
and pyruvate dehydrogenase are essential for 
cerebral myelination and thiamine deficiency 
in alcoholism leads to Wernicke's 
encephalopathy. 

Thiamine should also be supplemented fully 
in malnourished alcoholics. The 
administration of intravenous fluids 
containing glucose without adequate 
thiamine supplementation in alcoholics could 
aggravate the thiamine deficiency leading to 
irreversible cerebral lesions. 


[ Q: 2403 ] MRCPass - 2012 January 

A 30 year old man has had Hepatitis 
B testing. The results show: 

Anti-HBc Ab positive 

Anti-Hbs Ab positive 

HBs Ag negative 

Which one of the following is likely to be the 
correct scenario? 

1- Acute Hepatitis B infection 

2- Chronic hepatitis B carrier 

3- Previous hepatitis B immunisation 

4- Previous hepatitis B infection but not a 
carrier 

5- Not infected with hepatitis B 



Answer & Comments 

Answer: 4- Previous hepatitis B infection but 
not a carrier 

A chronic carrier has positive Hep B s Ag and 
positive HepB c Ab 

A patient with previous immunization has 
Hep B s Ag negative and negative Hep B c Ab 
and positive Hep B s Antibody, (note that the 


core antibody is non specific and not elevated 
post immunisation) 

A patient with previous Hep B infection who 
is not a chronic carrier, has negative Hep B s 
Ag and positive Hep B c 

Antibody and positive Hep B s Antibody (both 
antibodies are positive) 


4^ [ Q: 2404 ] MRCPass - 2012 January 

* A 56-year-old woman presents with 
periods of sweats and tremors which are 
relieved by eating. She has gained 
approximately 6 kg in weight in the last 2 
years. Her BM is 4.5. Blood tests are: Hb 13 
g/dl, MCV 78 fl, WCC 7 x 10 9 /l, platelets 200 x 
10 9 /l, sodium 135 mmol/I, potassium 4.7 
mmol/I, urea 5 mmol/I, creatinine 100 
pmol/l, TSH - 3.3 (0.3-4) mU/l, free T4 -20 
(10-24) pmol/l. 

What is the most appropriate investigation? 

1- 72 hour fast 


2- CT scan of pancreas 

3- MRI of the brain 


4- Insulin C-peptide concentration 

5- Oral glucose tolerance test 


Answer & Comments 

Answer: 1- 72 hour fast 

This patient has symptoms suggestive of 
hypoglycaemia which are relieved by 
carbohydrate. 

The likely cause is an insulinoma which is an 
insulin secreting pancreatic tumour. 

The best way of confirming the diagnosis is 
with a 72 hour fast. During the fast, the 
patient with an insulinoma may get episodes 
of hypoglycaemia with measured 
inappropriately high insulin C peptide 
(endogenous insulin). 


Dr. Khalid Yusuf El-Zohry - Sohag Teaching Hospital (01118391123) 

Ref MRCPass OE OE 2012 PasTest 2009 PassMedicine 2009 PasTest Exam ReviseMRCP 



966 

























El-zohry MRCP Questions Bank (Part 1) - 2013 


(For my personal use) 


Measurement of C-peptide is useful in 
excluding factitious hypoglycaemia from self 
injection of insulin. Insulin preparations do 
not contain C-peptide. 



[ Q: 2405 ] MRCPass - 2012 January 

A 61-year-old man is admitted to 
the resuscitation room with a GCS score of 
13/ 15. He has a past medical history of 
hypertension, glaucoma and diabetes. He had 
been found unconscious at home. Blood 
gases and blood test results taken on 
admission show the following: 


pH 7.22 
pC02- 3.5 kPa 
p02 -13.8 kPa 
Na+ 140 mmol/l 


K+ 4.2 mmol/l 

Chloride 110 (95-107) mmol/l 
Bicarbonate 10 (20-28) mmol/l 
Urea 2.1 mmol/l 
Creatinine 79 |imol/l 
Glucose 7.1 mmol/l 

A day following observation in the 
assessment unit, he complained of visual 
problems. 

Which one of the following diagnoses would 
be most consistent with these results? 


1- Addisonian crisis 

2- Pulmonary embolism 

3- Paraquat poisoning 

4- Methanol poisoning 


which causes retinal toxicity and visual 
impairment. 

Methanol is a commonly used organic 
solvent, the ingestion of which can cause 
significant toxicity. It is a constituent in many 
commercially available industrial solvents and 
in poorly adulterated alcoholic beverages. 



[ Q: 2406 ] MRCPass - 2012 January 

A 34-year-old man who has 
returned from a holiday in Egypt presents 
with diarrhoea. For the past two days he has 
been passing frequent bloody diarrhoea 
associated with crampy abdominal pain. 
Abdominal examination demonstrates diffuse 
lower abdominal tenderness but there is no 
guarding or rigidity. His temperature is 
37.7 °C. 


What is the most likely causative organism? 

1- Giardiasis 


2- Enterotoxigenic Escherichia coli 

3- Staphylococcus aureus 

4- Shigella 

5- Salmonella 


Answer & Comments 

Answer: 4- Shigella 

All are common causes of traveller's 
diarrhoea. 

However, North Africa and the Middle East (in 
particular Egypt) were also commonly 
reported regions of travel for Shigella spp 
infections. 


5- Diabetic ketoacidosis 

Answer & Comments 

Answer: 4- Methanol poisoning 

Methanol poisoning can cause high anion gap 
metabolic acidosis and also leads to 
formation of formic acid from methanol, 


Some of the infectious causes of bloody 
diarrhoea are: 

Salmonella 

Shigella 

Campylobacter jejuni 
Yersinia enterocolitica 


Dr. Khalid Yusuf El-Zohry - Sohag Teaching Hospital (01118391123) 

Ref MRCPass OE OE 2012 PasTest 2009 PassMedicine 2009 PasTest Exam ReviseMRCP 

























El-zohry MRCP Questions Bank (Part 1) - 2013 


(For my personal use) 


E. coli 

Entamoeba histolytica 


[ Q: 2407 ] MRCPass - 2012 January 

A 60 year old man has squamous 
cell lung cancer and presents with shortness 
of breath. He has a history of ischaemic heart 
disease. A pleural effusion was tapped and it 
showed a transudate. 

Which one of the following is a 
contraindication tow ards lung resection 
surgery? 

1- FEV1 of 1.8 L 

2- Pleural effusion 

3- Hypercalcaemia 

4- Superior vena cava obstruction 

5- Ischaemic heart disease 



Answer & Comments 

Answer: 4- Superior vena cava obstruction 

SVC obstruction is a contraindication. 

With a pleural effusion, there should be 
malignant cells demonstrated on cytology to 
contraindicate surgery. 

Contraindications for surgical resection of 
bronchial lung tumour: 

FEV1 is less than 1 L 

Mediastinal LN >lcm 

Staging >IIIB (extrathoracic metastatic 
disease) 

Nerve involvement is present (rec laryngeal 
nerve, vocal cord paralysis) 

Malignant pleural effusion is present (positive 
cytology) 

Horner's syndrome 
SVC obstruction 


[ Q: 2408 ] MRCPass - 2012 January 

A - 

# A 33 year old woman presents with 
tender, erythematous lesions on both legs 
and is offered a mantoux test. 

Which one of the following molecules 
mediates the action in a mantoux test? 

1- Interleukin 2 

2- TNF alpha 

3- HIF1 alpha 

4- Beta interferon 

5- CCL1 


Answer & Comments 

Answer: 2- TNF alpha 

The Mantoux Test (MT) is a classical delayed- 
type hypersensitivity (DTH) response to the 
intradermal injection of tuberculin purified 
protein derivative (PPD). 

It represents a cutaneous T cell mediated 
memory recall immune response. 

The presence of IL-1 alpha, IL-1 beta, IL-6, 
interferon-gamma (IFN-gamma) and tumour 
necrosis factor-alpha (TNF-alpha) are found in 
the skin of tuberculin-purified protein 
derivative (PPD)-induced reactions. 


[ Q: 2409 ] MRCPass - 2012 January 

A 70-year-old man with multiple 
myeloma presented with sudden onset gross 
hematuria to hospital. He had recently been 
on chemotherapy and treatment with 
predonisolone, cyclophosphamide and 
zoledronic acid. He also has a history of 
rheumatoid arthritis and hypertension. 

Which of the following is the likely cause of 
haematuria? 

1- Bendroflumethiazide 

2- Prednisolone 

3- Cyclophosphamide 

4- Zoledronic acid 



Dr. Khalid Yusuf El-Zohry - Sohag Teaching Hospital (01118391123) 

Ref MRCPass OE OE 2012 PasTest 2009 PassMedicine 2009 PasTest Exam ReviseMRCP 



968 



























El-zohry MRCP Questions Bank (Part 1) - 2013 


(For my personal use) 


5- Infliximab 


Answer & Comments 

Answer: 3- Cyclophosphamide 

High dose cyclophosphamide is associated 
with haemorrhagic cystitis (severe 
haematuria and bladder pains). 

Discontinuation of the drug and analgesics 
are essential. 

[ Q: 2410 ] MRCPass - 2012 January 

A 32 year old man presents with 
cough and purulent sputum. 

He has had several friends with similar 
symptoms. 

On examinations, there were crepitations in 
the left base of the lungs. Chest X ray shows 
patchy left lung shadow s. Investigations 
show : 

Hb 10.5 g/dl 

WCC 13 x 10 9 /l 

platelets 270 x 10 s /\, 

Neutrophil 9 (2 to 7.5) x 10 s /\, 

Reticulocyte count 5 (0.5-1.5) % 

Which one of the following is the most likely 
causative organism? 

1- Legionella 

2- Staph aureus 

3- Streptococcus pyogenes 

4- Mycoplasma 

5- Pneumocystis 



Answer & Comments 

Answer: 4- Mycoplasma 

There is likely to be cold agglutinins 
associated with mycoplasma infection. 

Transient acute hemolysis (reticulocytes) may 
occur secondary to certain infectious 


diseases, such as Mycoplasma pneumoniae 
infection and infectious mononucleosis (eg, 
EBV) infections. 



[ Q: 2411 ] MRCPass - 2012 January 

A 37-year-old woman presented 
with a 2-year history of progressive dysphagia 
to both solids and liquids, as well as a 5 kg 
weight loss. She typically complained of 
heartburn especially when lying down and 
after food. A chest X ray showed dilatation of 
the oesophagus with a fluid level. 

What is the most likely diagnosis? 

1- Oesophageal diverticulum 

2- Pancreatic malignancy 

3- Achalasia 

4- Gastroduodenal ulcer 


5- Zollinger Ellison syndrome 


Answer & Comments 

Answer: 3- Achalasia 

Achalasia is characterized by difficulty 
swallowing, regurgitation, and sometimes 
chest pain. 

The lower esophageal sphincter fails to relax, 
leading to dilatation of the oesophagus. On a 
barium swallow , this leads to a bird's beak 
appearance. 


[ Q: 2412 ] MRCPass - 2012 January 

An 18-year-old man presented 
casualty complaining of difficulty breathing. 
He had brought hospital by ambulance, 
having collapsed shortly after being stung on 
hand by a bee. On examination, his blood 
pressure was 80/40 mmHg, facial sw elling 
and pharyngeal oedema was noted. 

Which one of following investigations likely 
confirm anaphylaxis? 

1- Haemolytic complement (CH50) level 

2- Plasma tryptase activity 



Dr. Khalid Yusuf El-Zohry - Sohag Teaching Hospital (01118391123) 

Ref MRCPass OE OE 2012 PasTest 2009 PassMedicine 2009 PasTest Exam ReviseMRCP 






























El-zohry MRCP Questions Bank (Part 1) - 2013 


(For my personal use) 


3- Complement C3 level 

4- Total IgE level 

5- Eosinophil count 

Answer & Comments 

Answer: 2- Plasma tryptase activity 

The reaction involves preferential production 
of IgE, in response certain antigens, which in 
turn initiates a sequence of events leading to 
mast cell activation. 

According to the Resuscitation council 
guidelines, the specific test to help confirm a 
diagnosis of an anaphylactic reaction is 
measurement of mast cell tryptase. 

Tryptase is the major protein component of 
mast cell secretory granules. In anaphylaxis, 
mast cell degranulation leads to markedly 
increased blood tryptase concentrations 
(Figure 4). Tryptase levels are useful in the 
follow-up of suspected anaphylactic 
reactions, not in the initial recognition and 
treatment: measuring tryptase levels must 
not delay initial resuscitation. Tryptase 
concentrations in the blood may not increase 
significantly until 30 minutes or more after 
the onset of symptoms, and peak 1-2 hours 
after onset. 

The half-life of tryptase is short 
(approximately 2 hours), and concentrations 
may be back to normal within 6-8 hours, so 
timing of any blood samples is very 
important. 


Physical examination revealed mild wheezes 
on auscultation. The remainder of his physical 
examination was unremarkable. A chest x ray 
showed hyperinflated lungs with large bullae. 

Pulmonary function testing at presentation 
demonstrated : 

forced expiratory volume in one second 
(FEV1) was 2.10 litres (61% of the predicted 
value) 

forced vital capacity (FVC) was 3.60 litres 
(81% of the predicted value) 

the FEV1:FVC ratio was 0.59 

total lung capacity (TLC) was 6.40 litres (93% 
of the predicted value) 

residual lung volume (RV) was 2.91 litres 
(123% of the predicted value) 

There was minimal response to an inhaled 
bronchodilator. 

What is the likely diagnosis? 

1- Asthma 

2- Bronchiectasis 

3- Emphysema 

4- Silicosis 

5- Usual interstitial pneumonitis 

Answer & Comments 

Answer: 3- Emphysema 

There is evidence of obstructive picture 
(reduced FEV1/FVC ratio) and hyperinflation 
on lung function tests. 


http://www.resus.org.uk/pages/reaction.pdf 



[ Q: 2413 ] MRCPass - 2012 January 

A 50-year-old man presented with 
an initial complaint of dyspnea on exertion 
that had developed five years prior and had 
progressed to shortness of breath while 
walking up one flight of stairs. He smokes 20 
cigarettes and drinks 2 units of alcohol per 
day. 


This is in keeping with smoking related 
emphysema of the lungs. 


[ Q: 2414 ] MRCPass - 2012 January 

A 69-year-old man was admitted for 
evaluation of a new blistering rash. He has a 
history of hypertension and epilepsy. He 
normally takes bendroflumethiazide and 
phenytoin. On the day of admission, 
complained of soreness in his mouth and that 



Dr. Khalid Yusuf El-Zohry - Sohag Teaching Hospital (01118391123) 

Ref MRCPass OE OE 2012 PasTest 2009 PassMedicine 2009 PasTest Exam ReviseMRCP 



970 


























El-zohry MRCP Questions Bank (Part 1) - 2013 


(For my personal use) 


his hands were sore. His conjunctivae were 
mildly erythematous, and both upper eyelids 
had superficial erosions. His oral cavity 
revealed multiple shallow , 3 mm erosions 
with hemorrhagic crusting extending from the 
lips to the soft palate. His right chest revealed 
a 3 cm full-thickness erosion. Both palms had 
full-thickness, necrotic erosions with 
surrounding, partially blanching erythema of 
the digits. The soles of both feet were 
erythematous. 

What is the likely diagnosis? 

1- Toxic epidermal necrolysis 

2- Erythrodermic psoriasis 

3- Eczema herpeticum 

4- Bullous pemphigoid 

5- Erythema multiforme 

Answer & Comments 

Answer: 1- Toxic epidermal necrolysis 

Toxic epidermal necrolysis (TEN) is a 
potentially life-threatening dermatologic 
disorder characterized by widespread 
erythema, necrosis, and bullous detachment 
of the epidermis and mucous membranes, 
resulting in exfoliation and possible sepsis 
and/or death (see the image below). 

Mucous membrane involvement can result in 
gastrointestinal hemorrhage, respiratory 
failure, ocular abnormalities, and 

genitourinary complications. 

TEN is most commonly drug induced. 
However, the disorder has other potential 
etiologies, including infection, malignancy, 
and vaccinations. Antibiotics such as 

macrolides and sulphonamides are 
associated. Anticonvulsants such as 
phenytoin and carbamazepine are also 
associated. TEN in patients taking 

anticonvulsants has most often been 
reported within 2 months of starting the 
drug. 


4^ [ Q: 2415 ] MRCPass - 2012 January 

# A 21-year-old man presents with 
lethargy, pyrexia and headaches. These 
symptoms have been present for the past 8 
days. He had not been unwell before and 
there is no recent history of travel. Clinical 
examination reveals a temperature of 37.9°C, 
marked cervical lymphadenopathy and mild 
hepatomegaly. Throat examination reveals 
two small erythematous areas. A full blood 
count result shows: 

Hb 13.1 g/dl 

Platelets 225 x 10 9 /l 

WCC 17.1xl0 9 /l 

Neut 5.2 xll0 9 l 

Lymp 11.2 x 10 9 /l 

Blood Film Atypical lymphocytes seen 
What is the most likely diagnosis? 

1- Acute lymphoblastic leukaemia 

2- Hashimoto's thyroiditis 

3- Infectious mononucleosis 

4- HIV seroconversion 

5- Septicaemia secondary to streptococcal 
throat infection 

Answer & Comments 

Answer: 3- Infectious mononucleosis 

The history of previously being well, acute 
deterioration with lymphadenopathy, throat 
involvement and atypical lymphocytes on 
blood film are all consistent with Epstein Barr 
virus infection (glandular fever or infectious 
mononucleosis). 

Atypical lymphocytes are commonly 
associated with EBV, CMV and toxoplasma 
infection. 


Dr. Khalid Yusuf El-Zohry - Sohag Teaching Hospital (01118391123) . 

Ref MRCPass OE OE 2012 PasTest 2009 PassMedicine 2009 PasTest Exam ReviseMRCP 971 






















El-zohry MRCP Questions Bank (Part 1) - 2013 


(For my personal use) 



Reactive lymphocytes are lymphocytes that 
become large as a result of antigen 
stimulation. 



[ Q: 2417 ] MRCPass - 2012 January 

In a dehydrated patient, 
compensatory water retention takes place. 
Despite this, in which part of the nephron 
does water absorption not take place? 


1- Proximal convoluted tubule 


2- Descending limb Loop of Henle 

3- Ascending limb Loop of Henle 

4- Distal convoluted tubule 


5- Collecting duct 


[ Q: 2416 ] MRCPass - 2012 January 

A 35 year old man has has been 
investigated for liver cirrhosis and been 
diagnosed as having Wilson's disease. 

His wife has had no symptoms but knows that 
there is a single copy of the gene is present in 
1 in 100 people. He asks what is the chance 
that his children might have Wilson's disease. 

What is the risk of each child having the 
disease? 

1- None 

2- 1 in 50 

3- 1 in 100 

4- 1 in 200 


Answer & Comments 

Answer: 3- Ascending limb Loop of Henle 

Unlike the descending limb, the ascending 
limb of Henle's loop is impermeable to water, 
a critical feature of the countercurrent 
exchange mechanism employed by the loop. 

The ascending limb actively pumps sodium 
out of the filtrate, generating the hypertonic 
interstitium that drives countercurrent 
exchange. Though the collecting duct is 
normally impermeable to water, it becomes 
permeable under the actions of antidiuretic 
hormone (ADH). As much as three-fourths of 
the water from urine can be reabsorbed as it 
leaves the collecting duct by osmosis. 


5- 1 in 1000 


Answer & Comments 

Answer: 4-1 in 200 

Wilson's disease is inherited in an autosomal 
recessive pattern. 

A single abnormal copy of the gene is present 
in 1 in 100 people. In this case since the man 
already has both copies of the gene and the 
wife has a 1 in 100 chance of being a carrier, 
thus each child will has half the risk because if 
the wife was a carrier she has one normal and 
one gene for Wilson's disease. Half of 1 in 100 
chance is 1 in 200. 



[ Q: 2418 ] MRCPass - 2012 January 

A 20 year old female is referred to 
the hospital. Her family are concerned about 
her because she has been very agitated in the 
last 2 days. She has no past medical history of 
note. On examination, she looks restless and 
has significantly dilated pupils. 

What is the likely cause of mydriasis? 

1- Alcohol 


2- Barbiturates 


3- Diazepam 

4- Cocaine 


5- Opiates 


Dr. Khalid Yusuf El-Zohry - Sohag Teaching Hospital (01118391123) 

Ref MRCPass OE OE 2012 PasTest 2009 PassMedicine 2009 PasTest Exam ReviseMRCP 



972 





























El-zohry MRCP Questions Bank (Part 1) - 2013 


(For my personal use) 


Answer & Comments 

Answer: 4- Cocaine 

This patient is likely to have taken a stimulant 
which has made her restless and agitated. 

The most common stimulants taken as 
recreational drugs are amphetamines and 
cocaine. They also often cause dilated pupils 
(mydriasis). 



[ Q: 2419 ] MRCPass - 2012 January 

A 63-year-old man is examined in 
the cardiology clinic. During cardiac 
examination it is noted that the pulmonary 
component of the second heart sound occurs 
before the aortic component. 


Which one of the following is associated with 
this finding? 


1- Pulmonary stenosis 

2- Left bundle branch block 


3- Right bundle branch block 

4- Atrial septal defect 

5- Deep inspiration 


Answer & Comments 

Answer: 2- Left bundle branch block 

This patient has reversed splitting of the 
second heart sound. 

LBBB and left heart strain in HCM and aortic 
stenosis can cause reversal of A2P2 second 
heart sounds. Also, in type B wolf parkinson 
white syndrome, early activation of the right 
ventricle through an accessory pathway can 
cause P2 to close prematurely. Patent ductus 
arteriosus is another cause. 


[ Q: 2420 ] MRCPass - 2012 January 

A 40-year-old woman has presented 
with an erythematous annular patch with 
central clearing on her left foot. 



She likes walking and has been hiking on hills 
recently. The patient mentioned that the rash 
has gotten progressively larger and spreading 
up the leg over the last 3 weeks and she has 
had a recent onset of intermittent joint pains. 
On examination, there is a large area of 
erythema with a central clearing over the left 
leg. 

What is the likely diagnosis? 

1- Staphylococcal infection 

2- Lyme disease 

3- Pastereulla infection 

4- Necrotising fasciitis 

5- Gonorrhoea 


Answer & Comments 

Answer: 2- Lyme disease 

Lyme Disease (LD) is a multisystem disease 
affecting the nervous system, skin, joints, and 
heart. 

Erythema migrans (EM), the characteristic 
dermatologic lesion of LD, is an expanding red 
papule or macule with central clearing, often 
found in the axilla, midriff, or popliteal areas. 

B. burgdorferi is transmitted by the tick 
Ixodes, a hard bodied tick found in wooded 
areas. Lyme disease occurs in 3 stages: early 
localized, early disseminated, and late. If left 
untreated, each stage progresses to the next. 
Early localized disease manifests within 3-30 
days presenting with erythema migrans (EM), 
myalgia, fatigue, headache, fever, 
lymphadenopathy, and arthralgia. Early 
disseminated disease occurs 30 to 120 days 
post-infection and is characterized by EM 
(single or multiple), fatigue, 
lymphadenopathy, conjunctivitis, neck pain, 
cardiac abnormalities, radiculoneuritis, 
arthritis, and CNS manifestations. Late 
disease manifests from 4 months to 1 year, 
presenting with fatigue, chronic arthritis, CNS 
manifestations, and encephalopathy. 


Dr. Khalid Yusuf El-Zohry - Sohag Teaching Hospital (01118391123) 

Ref MRCPass OE OE 2012 PasTest 2009 PassMedicine 2009 PasTest Exam ReviseMRCP 


























El-zohry MRCP Questions Bank (Part 1) - 2013 


(For my personal use) 


First-line treatment for early disease is 
doxycycline (100 mg PO twice a day for 14 to 
21 days) or amoxicillin (500 mg PO three 
times a day for 14 to 21 days). 


[ Q: 2421 ] MRCPass - 2012 January 

With which one of the following 
carcinomas, is the RET oncogene associated ? 

1- Anaplastic thyroid carcinoma 

2- Follicular thyroid carcinoma 

3- Papillary thyroid carcinoma 

4- Medullary thyroid carcinoma 

5- Parathyroid carcinoma 



Answer & Comments 

Answer: 4- Medullary thyroid carcinoma 

The RET proto-oncogene encodes a receptor 
tyrosine kinase. 


sodium 135 mmol/l 
potassium 4.5 mmol/l 
urea 5 mmol/l 
creatinine 90 |imol/l 
IgA 0.2(0.5-4.0) g/l 
IgG 0.35 (5.0-13.0) g/l 
IgM 0.2 (0.3-2.2) g/l 
What is the diagnosis? 

1- Cystic fibrosis 

2- Kartagener's syndrome 

3- HIV infection 

4- Common variable immunodeficiency 

5- Tuberculosis infection 


Answer & Comments 

Answer: 4- Common variable 

immunodeficiency 


RET dysfunction mutations are associated 
with the development of medullar thyroid 
carcinoma and multiple endocrine neoplasias 
type II and III (formerly types 2A and 2B) and 
Hirschsprung's disease. 


[ Q: 2422 ] MRCPass - 2012 January 

n - 

* A 39-year-old male was admitted 
hospital with a two week history of cough and 
diarrhoea. He has a history of recurrent 
respiratory infections, otitis media, and 
sinusitis starting from adolescence. He also 
had a severe episode of meningoencephalitis 
and inguinal abscesses. A chest X ray showed 
patchy consolidation and he went to have a 
CT scan of the chest which showed bilateral 
changes consistent with bronchiectasis. 

Blood test results are: 

Hb 12.5 g/dl 

MCV 75 fl 

WCC 6 x 10 9 /l 

platelets 180 x 10 9 /l 


Common variable immunodeficiency (CVID) is 
a group of approximately 150 primary 
immunodeficiencies which have a common 
set of features (including 

hypogammaglobulinemia) but which have 
different underlying causes. 

Recurring infections involving the ears, eyes, 
sinuses, nose, bronchi, lungs, skin, Gl tract, 
joints, bones, CNS, parotid glands are a 
feature. 

Another feature is hypogammaglobulinaemia 
with low levels of IgA, IgG and IgM. 


[ Q: 2423 ] MRCPass - 2012 January 

A study was conducted to 
investigate the effect of a new antiplatelet 
drug. It compared the number of survivors of 
myocardial infarction with those who died 
after a 2 year period of being on the drug 
versus a placebo. 

Which one of the following tests is best used 
to compare the effects of the drug on 
survival? 



Dr. Khalid Yusuf El-Zohry - Sohag Teaching Hospital (01118391123) 

Ref MRCPass OE OE 2012 PasTest 2009 PassMedicine 2009 PasTest Exam ReviseMRCP 



974 




























El-zohry MRCP Questions Bank (Port 1) - 2013 


(For my personal use) 


1- Mann Whitney test 

2- Logistic regression analysis 

3- Spearmann's correlation 

4- Chi square test 

5- Kaplan Meier curve 


Answer & Comments 

Answer: 4- Chi square test 

The chi square test is most appropriate here 
for comparing drug treatment and placebo 
groups and survival. 

Survived Not Survived 

With Drug A B 

Without Drug C D 



[ Q: 2424 ] MRCPass - 2012 January 

A 75 year man was found collapsed 
at home and brought to the emergency 
department. 


He has a past history of coronary artery 
bypass grafting, stroke and diabetes. He was 
unable to give a history. On examination, he 
was covered with multiple bruises. A dipstick 
urine analysis shows Blood ++++, protein +, 
glucose -ve. 

Some of his investigation results are listed 
below: 


sodium 135 mmol/l 
potassium 4.5 mmol/l 
urea 5 mmol/l 
creatinine 300 pmol/l 
AST 320 (1-31) U/l 

What likely cause of the raised serum 
creatinine concentration? 


1- Urinary tract infection 

2- Glomerulonephritis 

3- Nephrotic syndrome 


4- Nephritic syndrome 


5- Myoglobinuria 


Answer & Comments 

Answer: 5- Myoglobinuria 

The elevated serum creatinine likely to be 
due to rhabdomyolsis. 

The positive urinalysis is caused by myoglobin 
a muscle protein released during muscle 
damage. High amounts of myoglobin 
damages the renal tubules which then leads 
to acute kidney injury. 




t\ \ 


[ Q: 2425 ] MRCPass - 2012 January 


A 29 year old man presented to the 


emergency deparment with a severe 
exacerbation of asthma. He responded poorly 
to aggressive therapy with systemic 
corticosteroids (prednisolone 40 mg) 100% 
oxygen and continuous inhaled beta 
agonist/anticholinergic therapy. He had a 
history of poorly controlled asthma, for which 
he had been hospitalized six months ago. He 
is drow sy, confused and lethargic. His PC02 8 
kPa and his Sa0 2 (oxygen saturation) has 
fallen to 85%, despite the oxygen therapy. 

Which one of the following is an indication for 
intubation? 

1- Not completing sentences 

2- Using accessory muscles 

3- Elevated C02 levels 

4- Wheezing in the chest 

5- Pneumothorax 


Answer & Comments 

Answer: 3- Elevated C02 levels 

Asthmatic patients who respond poorly to 
aggressive, inhaled bronchodilator therapy 
and intravenous steroids should be carefully 
monitored for progressive type II respiratory 
failure. 


Dr. Khalid Yusuf El-Zohry - Sohag Teaching Hospital (01118391123) 

Ref MRCPass OE OE 2012 PasTest 2009 PassMedicine 2009 PasTest Exam ReviseMRCP 






























El-zohry MRCP Questions Bank (Part 1) - 2013 


(For my personal use) 


A sign of type II respiratory failure in asthma 
patients who are young and otherwise 
healthy is a normal or increasing partial 
pressure of carbon dioxide (PC02). 

It is an indication for intubation. 

[ Q: 2426 ] MRCPass - 2012 January 

A 68-year-old woman presents with 
a vesicular rash in the trigeminal distribution 
on the left side of her face. This was 
diagnosed as herpes zoster infection and she 
recovered. Several weeks following the 
episode, she describes having severe pains 
several episodes a day which, each lasting for 
several minutes up to 2 hours. 

The drug did not respond to nonsteroidal anti 
inflammatory drugs and paracetamol. 

What drug should be commenced? 

1- Oramorph 

2- Sodium valproate 

3- Carbamazepine 

4- Phenytoin 

5- Tramadol 



Answer & Comments 

Answer: 3- Carbamazepine 

The clinical history is typical for post herpetic 
neuralgia. 

Neuropathic agents such as carbamazepine 
and gabapentin are recommended for its 
treatment. 



[ Q: 2427 ] MRCPass - 2012 January 

A 66-year-old woman comes to the 
clinic for a review of symptoms. Over the past 
three months she has been complaining of 
fatigue and a full blood count was requested: 


Hb 8.4 g/dl 
MCV 69 fl 
Pit 362 x 10 9 /l 


WBC 5.0 xl07l 

Blood film Hypochromic, microcytic picture 

An upper Gl endoscopy was reported as 
normal. 

What is the most appropriate next 
investigation? 

1- Barium swallow 

2- Faecal occult blood 

3- Colonoscopy 

4- Schilling test 

5- Reticulocyte count 


Answer & Comments 

Answer: 3- Colonoscopy 

There is evidence of iron deficiency anaemia. 

Although the OGD is normal, a colonoscopy 
should be considered to exclude sources of 
bleeding e.g. angiodysplasia or colorectal 
cancer 


[ Q: 2428 ] MRCPass - 2012 January 

A 40 year old man has a tetanus 
injection at the GP surgery. 3 hours following 
this he presents unwell to A&E. 

His blood pressure was 75/50 mmHg, he has a 
pulse rate of 115 and 02 saturation of 98% on 
air. There was erythema around the site of 
the tetanus injection on his arm. 

Which type of hypersensitive reaction is this? 

1- Type 1 

2- Type 2 

3- Type 3 

4- Type 4 

5- Type 5 



Answer & Comments 


Answer: 1- Type 1 


Dr. Khalid Yusuf El-Zohry - Sohag Teaching Hospital (01118391123) 

Ref MRCPass OE OE 2012 PasTest 2009 PassMedicine 2009 PasTest Exam ReviseMRCP 



976 






























El-zohry MRCP Questions Bank (Port 1) - 2013 


(For my personal use) 


This is an allergic or anaphylactic reaction due 
to the hypotension and erythema. 

Classifications for hypersensitivity are: 

Type 1: allergy - asthma, anaphylaxis 

Type 2: cytotoxic - autoimmune haemolytic 
anaemia, goodpasture's syndrome 

Type 3: immune complex - SLE 

Type 4: delayed hypersensitivity - Mantoux 
test, multiple sclerosis 


Type 5: autoimmune - Grave's disease, 
myasthenia gravis 



[ Q: 2429 ] MRCPass - 2012 January 


A 30 year old woman has just 
returned from south east asia and was bitten 
by mosquitoes. She has a fever and a malarial 
film is positive. She was given a course of 
chloroquine and primaquine. 


Why is primaquine used? 


1- To reduce multiplication of parasites in 
blood stream 


2- To eradicate hepatic forms of parasite 

3- To reduce reaction to chloroquine 

4- To prevent repeat infection 

5- To treat falciparum malaria 


Answer & Comments 

Answer: 2- To eradicate hepatic forms of 
parasite 

Primaquine (or primaquine phosphate) is a 
medication used to treat the P. vivax or P. 
ovale malaria. Once the parasite has been 
eliminated from the bloodstream, the 
remaining hypnozoites must be removed 
from the liver and this is done by 
administering a 14 day course of primaquine. 
This process is called a radical cure. 


[ Q: 2430 ] MRCPass - 2012 January 

A 75 year old patient presents with 
dizziness and an episode of collapse. The ECG 
on admission shows complete heart block. 
The patient has a single lead pacemaker 
inserted. 

What anatomical site should the pacemaker 
wire tip be inserted? 

1- Right atrium 

2- Right ventricular apex 

3- Left ventricular apex 

4- Coronary sinus 

5- Atrioventricular node 



Answer & Comments 

Answer: 2- Right ventricular apex 

The most basic form of a pacemaker is a VVI 
pacemaker where the tip of the pacemaker 
lead is inserted through the superior vena 
cava then through the tricuspid valve and into 
the right ventricular apex. 

This is also the case where a temporary wire 
which needs to be inserted such that the tip 
sits in the right ventricular apex. 


[ Q: 2431 ] MRCPass - 2012 January 

A 70 year old lady has longstanding 
lethargy. On physical examination she had a 
plethoric face & conjunctival injection. 
Investigations revealed: 

hemoglobin 18.5 g/dl (12-16 g/dl) 

red cell count 8.1 M/?l (3.5 to 6 M/?l) 

Hct. 58.% (37 to 52%) 

MCV 65.7fL (67 to 96 fL) 

MCH 19.6 pg (27 to 32 pg) 

Platelet 720 (150-400) x 10 9 /l WBC 13.1 x 

10 9 /l 

ESR 2 mm/hour 



Dr. Khalid Yusuf El-Zohry - Sohag Teaching Hospital (01118391123) . 

Ref MRCPass OE OE 2012 PasTest 2009 PassMedicine 2009 PasTest Exam ReviseMRCP 977 




























El-zohry MRCP Questions Bank (Part 1) - 2013 


(For my personal use) 


Blood film showed erythrocytosis, 
thrombocytosis and leucocytosis. 

With the diagnosis in mindl which drug should 
be commenced? 

1- Clopidogrel 

2- Bortezomib 

3- Infliximab 

4- Cyclosporin 

5- Hydroxycarbamide 

Answer & Comments 

Answer: 5- Hydroxycarbamide 


weight loss, back pain and stiffness. Clinical 
examination showed very restricted neck 
movement and chest expansion with no 
lumbar spinal movement. X rays of the 
patient's spine were performed. 

What feature would be expected on the x rays 
in ankylosing spondylitis? 

1- Osteosclerosis 

2- Syndesmophyte 

3- Lytic lesions 

4- Osteophyte 

5- Wedge shaped lesions 


The diagnosis in this case is polycythaemia 
rubra vera. 

Janus kinase 2 (commonly called JAK2) is a 
human protein that has been implicated in 
signaling by members of the type II cytokine 
receptor family. These mutations have been 
associated with polycythemia vera, essential 
thrombocythemia, and other 

myeloproliferative disorders. 

Low dose aspirin is indicated if there are no 
contraindications. 

Short, intermittent courses of cytoreductive 
therapy be administered to patients who 
have had thrombotic episodes or in whom 
platelet count continues to rise despite 
repeated phlebotomy. 

Interferon is first-line cytoreductive therapy 
in the younger patients (< 40 years old). 

Hydroxycarbamide is used as first line 
cytoreductive therapy in patients 40-75 years 
old. 

Anagrelide is a second line cytoreductive 
therapy in these age groups. 



[ Q: 2432 ] MRCPass - 2012 January 

A 44-year-old male developed neck 
pain which had worsened over several years. 
He presented to a rheumatologist with 


Answer & Comments 

Answer: 2- Syndesmophyte 

Typical X ray changes of ankylosing 
spondylitis are the visible formation of 
syndesmophytes (bony growth originating 
inside a ligament) and abnormal bone 
outgrow ths similar to osteophytes affecting 
the spine. 

These changes lead to an appearance of 
'bamboo spine'. 



Syndesmophyte 



Dr. Khalid Yusuf El-Zohry - Sohag Teaching Hospital (01118391123) 

Ref MRCPass OE OE 2012 PasTest 2009 PassMedicine 2009 PasTest Exam ReviseMRCP 

































El-zohry MRCP Questions Bank (Port 1) - 2013 


(For my personal use) 



[ Q: 2433 ] MRCPass - 2012 January 

An 80 year old woman is brought in 
after being found collapsed. She had not been 
seen by her neighbor overnight and may have 
been on the floor for many hours. Her 
temperature on admission was 33 C. 


Which feature may be a sign of hypothermia 
on the ECG? 


1- Short PR interval 

2- Right axis deviation 

3- Tented T waves 

4- Reversed Tick sign 

5- Prolonged QT 


Lithium level 6.5 mmol/l (0.4 to 1.0) mmol/L 

Na+ 136 mmol/l 

K+ 4.6 mmol/l 

Urea 6.1 mmol/l 

Creatinine 92 pmol/l 

Bicarbonate 25 mmol/l 

What is the most appropriate management? 

1- Intravenous magnesium 

2- Intravenous bicarbonate 

3- Intravenous normal saline 

4- Arrange for haemodialysis 

5- Arrange for plasma exchange 


Answer & Comments 

Answer: 5- Prolonged QT 

The following are characteristic of the ECG 
changes in the hypothermic patient: 

■ atrial fibrillation 

J-waves - which are pathognomonic 
of hypothermia 
PR elongation 

■ QRS widening 
QT elongation 

4+ [ Q: 2434 ] MRCPass - 2012 January 

# A 41-year-old man with a history of 
bipolar disorder is admitted with acute 
confusion. 

Whilst being transferred to hospital he had 
generalised seizure which terminated 
spontaneously after around 30 seconds. On 
arrival in the Emergency Department his GCS 
is 142/15 and he is noted to have a 
generalised tremor. A diagnosis of lithium 
toxicity is suspected. Intravenous access is 
obtained, bloods are taken and a saline 
infusion is started. Blood results reveal the 
following: 


Answer & Comments 

Answer: 4- Arrange for haemodialysis 

The high lithium level and reduced GCS are an 
indication for haemodialysis in this patient. 

Haemodialysis should be considered in any 
cases of lithium level > 4 mmol/l. 

[ Q: 2435 ] MRCPass - 2012 January 

£ - 

A 35-year old woman complained of 

visual symptoms and presents for 
assessment. She complained of pain, reduced 
visual acuity, visual disturbance and blurred 
vision in the left eye. Her left visual acuity was 
6/20. Light reflex in the left eye was reduced. 
The relative afferent pupillary defect (RAPD) 
was positive in the left eye where a central 
scotoma was present. Fundoscopy revealed a 
pale disc on the left retina. 

What is the clinical diagnosis? 

1- Retinitis pigmentosa 

2- 3rd nerve palsy 

3- Optic neuritis 

4- Acute glaucoma 

5- Retinal artery thrombosis 


Dr. Khalid Yusuf El-Zohry - Sohag Teaching Hospital (01118391123) . 

Ref MRCPass OE OE 2012 PasTest 2009 PassMedicine 2009 PasTest Exam ReviseMRCP 979 


























El-zohry MRCP Questions Bank (Part 1) - 2013 


(For my personal use) 


Answer & Comments 

Answer: 3- Optic neuritis 

The afferent pupillary defect suggest that the 
left eye optic nerve is affected, and a central 
scotoma suggests optic neuritis. 

Common clinical features of optic neuritis are 
variable loss of central vision which usually 
recovers within 2-6 weeks and dull aching 
pain in the region of the eye. The visual field 
defect is most commonly a central scotoma. 

There may be loss of colour vision and 
relative afferent pupillary defect. Multiple 
sclerosis is the most common cause of optic 
neuritis accounting for 50% of cases. Other 
causes include toxic amblyopia, 
toxoplasmosis, diabetes, syphilis, herpes 
zoster and infectious mononucleosis. 

[ Q: 2436 ] MRCPass - 2012 January 

A 62-year-old woman is investigated 
for weight loss, fatigue and anaemia. She has 
no past medical history of note. Clinical 
examination reveals splenomegaly associated 
with pallor. A full blood count is reported as 
follows: 

Hb 9.8 g/dl 

Platelets 380 x 10 9 /l 

WCC 120 x 10 9 /l 

Blood film. Demonstrates left shift with 
predominating myelocytes. Low percentage 
of blast cells 

What is the most appropriate treatment? 

1- Chlorambucil 

2- Imatinib 

3- Thalidomide 

4- Rituximab 

5- Hydroxycarbamide 



Answer & Comments 


Answer: 2- Imatinib 


The diagnosis here is chronic myeloid 
leukaemia, which accounts for 20% of all 
leukaemias. 

It occurs mainly in middle aged and elderly 
people and is characterised by marked 
leucocytosis, a left shifted myeloid series and 
in 95% of patients, the Philadelphia 
chromosome. 

Imatinib is recommended as first-line 
treatment for people with Philadelphia- 
chromosome-positive chronic myeloid 
leukaemia (CML) in the chronic phase With 
disease progression and palliative situations, 
Imatinib is used in combination with 
recombinant alpha interferon, hydroxyurea 
and busulphan. 


[ Q: 2437 ] MRCPass - 2012 January 

/ ---— 

# A 71-year-old woman presented to 

the emergency room with an acute history of 

progressive exertional chest pain. The pain 

was sharp and was associated with shortness 

of breath. Physical activity made it worse and 

improvement was noted with sublingual 

nitroglycerin. On arrival to the department 

her blood pressure was 105/62 mmHg, pulse 

was 98 beats per minute. Cardiac 

examination revealed a regular heart with no 

murmur, rubs or gallop. The ECG showed 

sinus rhythm with low voltage, left axis 

deviation with ST, lateral T wave 

abnormalities and elevated cardiac enzymes 

Her Troponin T levels peaked at 5ng/ml (< 

O.lOng/ml). The following day, her chest 

pains subsided but she developed fevers, 

myalgia and a mottled discoloration of both 

her legs. 

What other finding is likely? 

1- Anaemia 

2- Polycythaemia 

3- Thrombocytopenia 

4- Eosinophilia 

5- Haemolysis 


Dr. Khalid Yusuf El-Zohry - Sohag Teaching Hospital (01118391123) 

Ref MRCPass OE OE 2012 PasTest 2009 PassMedicine 2009 PasTest Exam ReviseMRCP 


























El-zohry MRCP Questions Bank (Part 1) - 2013 


(For my personal use) 


Answer & Comments 

Answer: 4- Eosinophilia 

This patient had a myocardial infarction and 
has developed cholesterol emboli probably 
due to severe atherosclerosis of the aorta. 

The symptoms experienced in cholesterol 
embolism are fever, muscle ache and weight 
loss. Embolism to the legs causes a mottled 
appearance and purple discoloration of the 
toes, small infarcts and areas of gangrene due 
to tissue death that usually appear black, and 
areas of the skin that assume a marbled 
pattern known as livedo reticularis. The full 
blood count may show particularly high 
numbers eosinophils (more than 0.5 x 
10 A 9/I); this occurs in 60-80% of cases. 


[ Q: 2438 ] MRCPass - 2012 January 

A 70-year-old man presented to the 
hospital with sudden onset of double vision 
and a 1 month history of chronic headache. 
On examination, he had a left sided 3rd nerve 
palsy and a dilated pupil. His blood tests show 
no abnormality. Magnetic resonance imaging 
was organised. 



on the oculomotor nerve or ischemia. 


^ [ Q: 2439 ] MRCPass - 2012 January 

fit - 

# A 46 year old lady presented to her 
physician with complaints of weakness and 
headaches for several months. 

At presentation, she was found to have 
severe hypertension with blood pressure 
180/110 mmHg. Her blood test results are: 

sodium 149 mmol/l 

potassium 2.9 mmol/l 

urea 7 mmol/l 

creatinine 100 |imol/l 

What test should be done next? 

1- Thyroid function test 

2- Renin:aldosterone ratio 

3- Abdominal MRI 

4- Intrapetrosal venous sampling 

5- 24 hour urine for catecholamines 


Answer & Comments 


Answer: 2- Renin:aldosterone ratio 


What is this likely to show? 

1- Encephalitis 


The clinical diagnosis for this case is Conn's 
syndrome. 


2- Meningioma in the frontal lobe 

3- Brainstem glioma 

4- Posterior communicating artery aneurysm 

5- Occipital territory infarct 


Answer & Comments 

Answer: 4- Posterior communicating artery 
aneurysm 

Compression of the oculomotor nerve 
resulting in third nerve palsy is most 
commonly caused by posterior 
communicating artery aneurysm. 

In these cases, third nerve palsy typically 
develops in response to direct pressure 


Conn syndrome is characterized by increased 
aldosterone secretion from the adrenal 
glands, suppressed plasma renin activity 
(PRA), hypertension, and hypokalemia as 
seen in the above case. Routine laboratory 
studies can show hypernatremia, 
hypokalemia, and metabolic alkalosis 
resulting from the action of aldosterone on 
the distal tubule of the kidney. 
Renin:aldosterone ratio is a good screening 
test for the patient. If this suggested Conn's 
syndrome, then an abdominal MRI to identify 
an adrenal lesion should be done. 



[ Q: 2440 ] MRCPass - 2012 January 
A 30-year-old woman with a history 


Dr. Khalid Yusuf El-Zohry - Sohag Teaching Hospital (01118391123) 

Ref MRCPass OE OE 2012 PasTest 2009 PassMedicine 2009 PasTest Exam ReviseMRCP 




























El-zohry MRCP Questions Bank (Part 1) - 2013 


(For my personal use) 


of migraine for 2 years presented to the 
emergency ward with complaints of visual 
disturbances. She had noted an increase in 
frequency of headache over the past few 
months. The patient's past medical history 
was unremarkable. She was taking NSAIDs for 
headaches. She was a non-smoker. 

On physical examination, the patient's blood 
pressure level was 120/70 mm Hg. On 
neurologiccal examination, there was no focal 
limb weakness and mental test function was 
normal. She had a bilateral homonymous 
superior quadrantanopia. 

What is the most likely cause? 

1- Craniopharyngioma 

2- Pituitary macroadenoma 

3- Sagittal meningioma 

4- Glioma 

5- Subdural haematoma 


Answer & Comments 

Answer: 2- Pituitary macroadenoma 

Pituitary tumours often enlarge upwards, 
hence compressing the optic chiasm. 

the visual field defect will first appear as 
bitemporal superior quadrantanopia and 
eventually leading to bitemporal hemianopia. 

If originating superior to the optic chiasm, 
more commonly in a craniopharyngioma of 
the pituitary stalk, the visual field defect will 
first appear as bitemporal inferior 
quadrantanopia. 



[ Q: 2441 ] MRCPass - 2012 January 

A 53-year-old woman presented to 
the emergency department with complaints 
of intermittent fatigue, nausea and itching for 
several months. She complains of having a 
constant dry mouth. She had a past medical 
history of hypothyroidism. She had a 20 pack- 
year smoking history and alcohol intake 
consisted of a maximum of 4 units a day. On 


examination, she was jaundiced and had 
palpable hepatomegaly. She was noted to 
have excoriation marks on the skin and 
xerostomia. 

Blood results are: Hb 12.5 g/dl, WCC 7 x 10 9 /l, 
platelets 235 x 10 9 /l, sodium 136 mmol/l, 
potassium 4.5 mmol/l, urea 6 mmol/l, 
creatinine 110 pmol/l, ALT 90 (5-35) U/l, AST 
68 (1-31) U/l, ALP 650 (20-120) U/l, GGT 90 
(4-35) U/l, Bilirubin 125 (1-22) |imol/l, 
Albumin 38 (37-49) g/l. 

What test should be sent to help confirm the 
diagnosis? 

1- Antinuclear antibody 

2- Anti parietal cell antibody 

3- Anti gliadin antibody 

4- Anti smooth muscle antibody 

5- Anti mitochondrial antibody 


Answer & Comments 

Answer: 5- Anti mitochondrial antibody 

The patient has a cholestatic picture (high 
alkaline phosphatase and bilirubin) in the 
liver function tests, and hence out of all the 
options, primary biliary cirrhosis is most 
likely. 

Antimitochondrial antibody is often positive. 
Fatigue is the first reported symptom, 
pruritus is also a common symptom. 
Examination findings usually include 
hepatomegaly and xanthelasmata. 


[ Q: 2442 ] MRCPass - 2012 January 

A 26-year-old man presented to the 
hospital with sw elling of his hands and feet, 
progressive dyspnoea and weight gain in 
excess of 10 kg in the week prior to 
admission. On examination he had a blood 
pressure of 180/80 mmHg, heart rate of 90 
and respiratory rate of 16/min. There were 
no skin lesions or lymphadenopathy, and all 
pulses were palpable. There was 3+ pitting 



Dr. Khalid Yusuf El-Zohry - Sohag Teaching Hospital (01118391123) 

Ref MRCPass OE OE 2012 PasTest 2009 PassMedicine 2009 PasTest Exam ReviseMRCP 



982 

























El-zohry MRCP Questions Bank (Part 1) - 2013 


(For my personal use) 


pedal oedema to the upper thighs and 
periorbital oedema. Urine dipstick showed 
proteinuria 4+. 


immunosuppressants is given. The choice of 
immunosuppressants includes 

cyclophosphamide and chlorambucil. 


A percutaneous renal biopsy showed many 
glomeruli with normal appearance. Electron 
microscopy showed areas of visceral 
epithelial podocyte effacement and fusion 
without evidence of immune deposits, and 
minimal change glomerulonephritis is 
diagnosed. 

What treatment is most likely to reduce 
proteinuria? 

1- Cyclophosphamide 

2- Aciclovir 

3- Lisinopril 

4- Prednisolone 

5- Intravenous immunoglobulin 


Answer & Comments 


Answer: 4- Prednisolone 


Angiotensin converting enzyme inhibitors and 
angiotensin II receptor blockers, alone or in 
combination should be used with a goal of 
reducing the proteinuria. Blood pressure and 
renal function should be monitored closely in 
patients on angiotensin converting enzyme 
inhibitors and angiotensin II receptor 
blockers. 



[ Q: 2443 ] MRCPass - 2012 January 

A 63 year old woman has recently 
had lethargy and arthralgia. She was 
diagnosed as having influenza infection, as 
there was an outbreak in the area recently. 
She presents 1 week later with a cough and 
breathlessness. On examination, she had 
bilateral crackles audible on examination. CXR 
confirms bilateral consolidation and 
infiltrates. 


The patient has minimal change disease 
(MCD). 

It is postulated that MCD is a disorder of T 
cells, which release a cytokine that injures the 
glomerular epithelial foot processes. 
Oedema, hypertension and proteinuria are 
common presentations. The renal biopsy 
often shows normal appearances but on 
electron microscopy, there is diffuse loss of 
visceral epithelial cells (podocyte) foot 
processes. 

Corticosteroids are the treatment of choice, 
leading to complete remission of proteinuria 
in most cases. 

Approximately 90% of children respond 
within 2 weeks to prednisone at a dose of 60 
mg/msq/d. Adults respond more slow ly than 
children. A response in up to 80-90% has 
been recorded in adolescents and adults. 
However, the time to remission is up to 16 
weeks. If patients are steroid-resistant or they 
relapse frequently, a trial of 


Which one of the following is most likely as a 
cause? 

1- Legionella 

2- Mycoplasma 

3- Streptococcus pneumoniae 

4- Klebsiella 

5- Staphylococcus aureus 

Answer & Comments 

Answer: 5- Staphylococcus aureus 

Normal incidence of staph aureus pneumonia 
is 2%, However this is significantly increased 
in iv drug users and influenzae virus 
infections. 

Post influenzae staph aureus pneumonia is 
characterised by rapid clinical deterioration 
with septicaemia. 


Dr. Khalid Yusuf El-Zohry - Sohag Teaching Hospital (01118391123) 

Ref MRCPass OE OE 2012 PasTest 2009 PassMedicine 2009 PasTest Exam ReviseMRCP 























El-zohry MRCP Questions Bank (Part 1) - 2013 


(For my personal use) 


[ Q: 2444 ] MRCPass - 2012 January 

A 25 year old man was admitted to 
hospital with frequent bloody diarrhea (10-15 
bow el motions/day), abdominal pain, and 
fever for a period of 3 months. There is no 
history of recent travel and the patient did 
not respond to courses of antibiotics given by 
the GP. On admission, physical examination 
showed a temperature of 37.5°C; the 
abdomen was soft, but there was moderate 
tenderness in the lower abdomen. 


A blood film showed hypochromia, slight 
microcytosis and anisocytosis, a few target 
cells, and basophilic stippling. 

What is the diagnosis? 

1- Sideroblastic anaemia 

2- Alpha thalassemia trait 

3- Beta thalassemia trait 

4- Acute intermittent porphyria 

5- Lead poisoning 


A colonoscopy was performed. This showed 
large areas of continuous severe 
inflammatory changes of congestion, oedema 
and irregular abscesses around the sigmoid 
colon with diverticulae seen. 

What is the diagnosis? 

1- Diverticulitis 

2- Crohn's disease 

3- Ulcerative colitis 

4- Inflammatory colitis 

5- Ischaemic colitis 

Answer & Comments 

Answer: 3- Ulcerative colitis 

The history of bloody diarrhea is typical of 
ulcerative colitis. 


Answer & Comments 

Answer: 3- Beta thalassemia trait 

The blood film shows an iron deficiency 
picture, target cells and basophilic stippling. 

Additionally the patient is asymptomatic and 
there are elevated HBA2 levels, hence it fits a 
thalassemia trait. Below are descriptions of 
some of the options. 

(3 thalassemia trait (minor): This trait is 
characterized by mild anemia and low RBC 
indices. This condition is typically caused by 
the deletion of 2 ? (a) genes on one 
chromosome 16 (aa/oo) or one from each 
chromosome (ao/ao). 

This condition is encountered mainly in 
Southeast Asia, the Indian subcontinent. 


The biopsy specimen showing severe areas of 
inflammation are typical of colitis. In this 
case, although there are diverticulae, 
diverticulitis should not cause such large 
areas of inflammatory changes. 


[ Q: 2445 ] MRCPass - 2012 January 

A 44-year-old South Asian man had 
a routine blood test with his GP. He has no 
symptoms. These results were found: 

Hb 10.5 g/dl, MCV 75 fl, WCC 7 x 10 9 /l, 
platelets 220 x 10 9 /l, HbA2 5% (<3.5%) 



a thalassemia trait (minor): Patients have mild 
anemia, abnormal RBC indices, and abnormal 
Hb electrophoresis results with elevated 
levels of Hb A2, Hb F, or both. Peripheral 
blood film examination usually reveals 
marked hypochromia and microcytosis 
(without the anisocytosis usually encountered 
in iron deficiency anemia), target cells, and 
faint basophilic stippling. The production of a 
chains from the abnormal allele varies from 
complete absence to variable degrees of 
deficiency. 



[ Q: 2446 ] MRCPass - 2012 January 
A 21-year-old lady with polycystic 


Dr. Khalid Yusuf El-Zohry - Sohag Teaching Hospital (01118391123) 

Ref MRCPass OE OE 2012 PasTest 2009 PassMedicine 2009 PasTest Exam ReviseMRCP 



984 



























El-zohry MRCP Questions Bank (Part 1) - 2013 


(For my personal use) 


ovary syndrome was prescribed Metformin. 
How does Metformin acts in this situation? 

1- Increasing oestradiol levels 

2- Increasing luteinising hormone levels 

3- Increasing gluconeogenesis 

4- Increasing insulin levels 

5- Increasing peripheral glucose utilisation 


Thoracic outlet syndrome is a syndrome 
involving compression at the superior 
thoracic outlet involving compression of a 
neurovascular bundle. It can affect the 
brachial plexus and/or the subclavian artery. 

The compression may be positional (caused 
by movement of the clavicle and shoulder 
girdle on arm movement), a first rib fixation 
and a cervical rib. 


Answer & Comments 

Answer: 5- Increasing peripheral glucose 
utilisation 

Metformin is being used increasingly in 
polycystic ovary syndrome (PCOS) and non¬ 
alcoholic steatohepatitis, two diseases that 
feature insulin resistance. 

metformin improves insulin sensitivity by 
increasing peripheral glucose uptake and 
utilization 



[ Q: 2447 ] MRCPass - 2012 January 

A 25-year-old man presents with sw 
elling and pain in his left arm after exercising 
in the gym. Symptoms started 75 minutes 
after the exercises. The arm turned reddish, 
and he described it as 'feeling different than it 
ever had before'. On examination there was 
no supraclavicular tenderness but when he 
lifted his arm up the brachial pulse on the left 
arm was absent. 


What is the likely diagnosis? 

1- Lower trunk brachial plexus lesion 

2- Syringomyelia 

3- Thoracic outlet syndrome 

4- Neuralgic amyotrophy 

5- Takayasu's arteritis 


Answer & Comments 


Answer: 3- Thoracic outlet syndrome 



[ Q: 2448 ] MRCPass - 2012 January 

A 32-year-old female presents 
complaining of a purpuric rash on the back of 
her legs and her buttocks. She has been 
complaining of a sore throat and a productive 
cough a week ago. She also has mild pains in 
her elbow and knee joints. She has no 
significant past medical history and has not 
been on any medications recently. A urine 
dipstick shows blood ++ proteins +. Her blood 
results are: 


Hb 11.3 g/dl 
Platelets 155 x 10 9 /l 
WCC 5.3 x 10 9 /l 
PT 13 secs 


APTT 30 secs 

sodium 135 mmol/l 

potassium 4.5 mmol/l 

urea 5 mmol/l 

creatinine 100 [imol/l 

What is the most likely diagnosis? 

1- Drug-induced thrombocytopenia 

2- Henoch-Schonlein purpura 

3- Idiopathic thrombocytopenic purpura 

4- Thrombotic thrombocytopenic purpura 

5- Systemic lupus erythematosus 


Answer & Comments 


Answer: 2- Henoch-Schonlein purpura 


Dr. Khalid Yusuf El-Zohry - Sohag Teaching Hospital (01118391123) 

Ref MRCPass OE OE 2012 PasTest 2009 PassMedicine 2009 PasTest Exam ReviseMRCP 



























El-zohry MRCP Questions Bank (Part 1) - 2013 


(For my personal use) 


This patient is likely to have Henoch- 
Schonlein purpura (HSP), which is a self- 
limited systemic vasculitis. It is suspected to 
be triggered by an IgA-mediated response to 
an antigen. It is characterized by 4 clinical 
syndromes: 

1. Palpable purpura in the absence of 
thrombocytopenia or coagulopathy. Develops 
in 100% of patients. 

2. Arthritis/arthralgia in 45-75% of patients. 
Second most common manifestation of HSP. 

3. Abdominal pain in 50%, Gl bleeding (often 
occult) in 20-30% of patients. 

4. Renal disease in 20-50%. 


ethnic population. The researcher is 
concerned about the spread of blood 
pressures being larger than in the general 
population, hence affecting the sample mean. 

Which of these measures provides an 
estimate of this concept of uncertainty? 

1- Sensitivity 

2- Specificity 

3- Positive predictive value 

4- Negative predictive value 

5- Standard error of mean 

Answer & Comments 

Answer: 5- Standard error of mean 


[ Q: 2449 ] MRCPass - 2012 January 

A 20 year old man drinks a litre of 
beer and has polyuria. 

Which one of these mechanisms leads to 
polyuria? 

1- Decreased glomerular filtration rate 

2- Decreased Aquaporin action 

3- Increased ADH secretion 



Standard deviation provides a measure of 
spread of observations about mean. It based 
of deviation of each observation from the 
mean value. Standard error of the mean is 
the standard deviation of the sampling 
distribution of the mean - which gives an 
estimate of how close the sample mean is to 
the true population mean. It increases with 
sample size and increases with standard 
deviation. 


4- Increased atrial natriuretic peptide 

5- Increased sodium absorption 


Answer & Comments 

Answer: 2- Decreased Aquaporin action 

Aquaporins selectively conduct water 
molecules in and out of the cell, while 
preventing the passage of ions and other 
solutes. There are several types of aquaporin 
receptors, and aquaporin 2 absorbs water in 
response to antidiuretic hormone and in this 
case, the effect of Aquaporin 2 is decreased 
leading to polyuria. 


[ Q: 2450 ] MRCPass - 2012 January 


A statistician is advising a research 
about a study of blood pressures in a specific 



The S.E.M. is the standard deviation divided 
by the square root of the sample size SEM = 
f? / ??N where f? is the standard deviation of 
the original distribution and N is the sample 
size. 

[ Q: 2451 ] MRCPass - 2012 January 

A 36-year-old woman was 
commenced on nasogastric feeding following 
significant weight loss. She has a history of 
severe alcohol abuse. Three days later she 
becomes very confused. On examination she 
was apyrexial, appeared appropriately 
hydrated, a pulse of 98 bpm pressure 96/60 
mmHg. 

Which one of the following investigations 
should be done? 

1- Bicarbonate 



Dr. Khalid Yusuf El-Zohry - Sohag Teaching Hospital (01118391123) 

Ref MRCPass OE OE 2012 PasTest 2009 PassMedicine 2009 PasTest Exam ReviseMRCP 



986 





























El-zohry MRCP Questions Bank (Part 1) - 2013 


(For my personal use) 


2- Phosphate 

3- Calcium 

4- Magnesium 

5- Potassium 


Answer & Comments 

Answer: 2- Phosphate 

The patient has refeeding syndrome. 
Refeeding malnourished patients increases 
basal metabolic rate. This anabolic response 
causes intracellular movement of minerals, 
serum phosphate levels may fall significantly, 
leading to cardiorespiratory compromise, 
confusion and lethargy. 

[ Q: 2452 ] MRCPass - 2012 January 

A 50 year old man presents with 
hearing loss and difficulty with balance and 
gait. He is examined carefully by the 
neurologist. 

Which one of the following signs is consistent 
with early presentation of acoustic neuroma? 

1- Bitemporal hemianopia 

2- Decrease in visual acuity 

3- Loss of corneal reflex 

4- Tongue deviation 

5- Loss of gag reflex 



reported tinnitus (most often a unilateral 
high-pitched ringing, sometimes a machinery¬ 
like roaring or hissing sound, like a steam 
kettle). Additionally, the classic description of 
early trigeminal involvement with an acoustic 
neuroma is loss of the corneal reflex. 

[ Q: 2453 ] MRCPass - 2012 January 

A 60 year old lady presents with 
lethargy. On investigation the following 
results were obtained: 

Hb 7.8g/dl 

WBC 3 x 10 9 /l 

Platelet 90 x 10 9 /l 

MCV 109 fl 

Anti parietal antibody positive. 

An endoscopy was performed. 

Which area is most likely to sure abnormal 
biopsy to confirm the diagnosis? 

1- Proximal stomach 

2- Distal stomach 

3- Oesophagus 

4- Duodenum 

5- Terminal ileum 



Answer & Comments 


Answer: 1- Proximal stomach 


Answer & Comments 

Answer: 3- Loss of corneal reflex 

Acoustic neuroma is a benign primary 
intracranial tumor of the myelin-forming cells 
of the vestibulocochlear nerve (CN VIII).The 
earliest symptoms of acoustic neuromas 
include ipsilateral sensorineural hearing 
loss/deafness, disturbed sense of balance and 
altered gait, vertigo with associated nausea 
and vomiting, and pressure in the ear, all of 
which can be attributed to the disruption of 
normal vestibulocochlear nerve function. 
Additionally more than 80% of patients have 


The diagnosis is likely to be pernicious 
anaemia because of the macrocytic anaemia 
and positive anti parietal antibody. The 
question refers to where atrophic gastritis 
occurs. There are two types of atrophic 
gastritis. 

Type A gastritis primarily affects the 
body/fundus (proximal) of the stomach, and 
is more common with pernicious anemia. 

Type B gastritis (most common overall) 
primarily affects the antrum (distal), and is 
more common with H. pylori infection. 


Dr. Khalid Yusuf El-Zohry - Sohag Teaching Hospital (01118391123) . 

Ref MRCPass OE OE 2012 PasTest 2009 PassMedicine 2009 PasTest Exam ReviseMRCP 987 



























El-zohry MRCP Questions Bank (Part 1) - 2013 


(For my personal use) 


[ Q: 2454 ] MRCPass - 2012 January 

A 36 year old man has presented 
with back pains and joint pains over several 
months. He has no other past medical history 
to date. He mentioned that his father had a 
history of joint problems. On examination, 
there is no evidence of skin involvement. 
Joint examination revealed evidence of 
synovitis in the metacarpophalangeal, 
metatarsal and wrist joints bilaterally limiting 
his range of joint movements. 

Laboratory testing revealed a C-reactive 
protein of 0.33 mg/dl (<0.80 mg/dl), strongly 
positive CCP antibody, IgG of 148 Units (<20 
Units), and angiotensin-converting-enzyme 
(ACE) of 73 U/liter (<67 U/liter). Anti nuclear 
antibody (ANA), anti neutrophil cytoplasmic 
antibody (ANCA) and rheumatoid factor (Rh 
F) were negative. 

What is the diagnosis? 

1- Dermatomyositis 

2- Psoriatic arthropathy 

3- Rheumatoid arthritis 

4- Osteoarthritis 

5- Systemic lupus erythematosus 



Answer & Comments 

Answer: 3- Rheumatoid arthritis 

Anti-citrullinated protein/peptide antibodies 
(Anti-CCP) are autoantibodies frequently 
detected in rheumatoid arthritis patients. It 
has a sensitivity of 70% as a diagnostic test. 
The positive anti-CCP antibodies and 
symmetrical polyarthropathy without skin 
involvement suggests rheumatoid arthritis. 

Rheumatoid factor is an IgM antibody against 
IgG. Some patients with rheumatoid arthritis 
will have a negative rheumatoid factor, as in 
this case. 


palpitations which have been going on for 2 
days. He has a past medical history of 
hypertension and currently takes aspirin and 
amlodipine. On examination he has a regular 
pulse rate of 150. 

The ECG shows a narrow complex 

tachycardia. 

What is the likely arrhythmia? 

1- Atrial fibrillation 

2- Atrial flutter 

3- Atroventricular re-entry tachycardia 

4- Atroventricular nodal re-entry tachycardia 

5- Idioventricular rhythm 


Answer & Comments 

Answer: 2- Atrial flutter 

The heart rate of 150 which is regular, narrow 
QRS complex tachycardia is suggestive of 
atrial flutter with 2:1 conduction. The atrial 
flutter circuit usually runs at a rate of 300 per 
minute and can present with 2:1, 3:1 or 4:1 
conduction. 


[ Q: 2456 ] MRCPass - 2012 January 

A 28 year old patient presents to the 
clinic for advice. His brother has recently 
been diagnosed with haemochromatosis. 



Which one of the following is the most useful 
screening test? 


1- Ferritin 


2- HFE gene analysis 

3- Ultrasound of the liver 

4- Liver iron levels 


5- Transferrin saturation 


Answer & Comments 


Answer: 5- Transferrin saturation 



[ Q: 2455 ] MRCPass - 2012 January 
A 55 year old man presents with 


Transferrin saturation (TS) is the most 
effective and inexpensive screening test for 


Dr. Khalid Yusuf El-Zohry - Sohag Teaching Hospital (01118391123) 

Ref MRCPass OE OE 2012 PasTest 2009 PassMedicine 2009 PasTest Exam ReviseMRCP 



988 































El-zohry MRCP Questions Bank (Part 1) - 2013 


(For my personal use) 


Fe overload. If the TS is > 45%, the test should 
be repeated, together with serum ferritin. If 
TS (w ith or without high ferritin) is raised, 
HFE genes should be determined. 


showed Polymorphic VT with constant 
varying axis (Torsade de pointes). 

What should be administered? 

1- Flecainide 



[ Q: 2457 ] MRCPass - 2012 January 

A 46 year old lady has had moderate 
redness in her right eye which is painless. She 
has a long standing history of changes in her 
hands and elbows consistent with 
rheumatoid arthritis. Her ocular condition 
had improved within a few days without 
treatment. 


2- Magnesium sulphate 

3- Amiodarone 

4- DC cardioversion 

5- Na bicarbonate 


Answer & Comments 


Answer: 2- Magnesium sulphate 


What is the likely diagnosis? 
1- Glaucoma 


Torsade is defined as a polymorphous VT in 
which the morphology of the QRS complexes 
varies from beat to beat. 


2- Scleritis 

3- Episcleritis 

4- Keratoconjunctivitis sicca 

5- Optic neuritis 


Answer & Comments 

Answer: 3- Episcleritis 

Episcleritis is a relatively common benign 
inflammatory condition of the eye. The 
symptoms of the disease are usually self- 
limiting, presenting in part with acute onset 
of red eye. Episcleritis is often acute and 
painless, whilst scleritis has more gradual 
onset and very painful. Although most of the 
cases are idiopathic, there is often an 
underlying disease including rheumatoid 
arthritis, polyarteritis nodosa, systemic lupus 
erythematosus, sarcoidosis and Wegener's 
granulomatosis. 



with 


[ Q: 2458 ] MRCPass - 2012 January 

A 55 year old gentleman presents 
dizziness. He was on various 


medications. On examination his Blood 
pressure was 110/68mm of Hg. The ECG 


This was symbolically termed torsade de 
pointes, or "tw isting of the point" about the 
isoelectric axis. 

The electrolyte disturbances that have been 
reported to precipitate torsade include 
hypokalemia and hypomagnesemia. 

Torsade also is subject to degeneration into 
ventricular fibrillation so therapy should be 
started as soon as the rhythm clearly fulfills 
the criteria for torsade. Treat hypokalemia if 
it is the precipitating factor and administer 
magnesium sulfate in a dose of 2-4 g 
intravenously (IV) initially. Magnesium is 
usually very effective, even in the patient 
with a normal magnesium level. 


^ [ Q: 2459 ] MRCPass - 2012 January 

#1 - 

0 A 21-year-old man presented to 

A&E with fever for one day. On arrival, the 
patient's BP was 109/50 mmHg, pulse rate 
was 115 and temperature was 38.1°C. On 
examination, there were generalised maculo- 
papular rash. 

Investigations showed a raised white cell 
count of 18 with neutrophilia, raised CSF 
protein of 5.45 g/L and decreased CSF glucose 
of 0.1 mmol/L. CSF was turbid with 


Dr. Khalid Yusuf El-Zohry - Sohag Teaching Hospital (01118391123) . 

Ref MRCPass OE OE 2012 PasTest 2009 PassMedicine 2009 PasTest Exam ReviseMRCP 989 



























El-zohry MRCP Questions Bank (Part 1) - 2013 


(For my personal use) 


predominating polymorphs and CSF culture 
grew Neisseria meningitides. The patient has 
close contact with 2 other housemates. 

What prophylaxis should be given? 

1- Penicillin 

2- Flucloxacillin 

3- Rifampicin 

4- Isoniazid 

5- Gentamicin 


Answer & Comments 

Answer: 3- Rifampicin 

Those who have close contact with a patient 
with meningococcal infection should have 
prophylaxis. The current regime for 
chemoprophylaxis includes oral rifampicin 
600 mg every 12 hours for 2 days, a single 
oral dose of ciprofloxacin 500 mg or 
ceftriaxone 250 mg intramuscularly. 



[ Q: 2460 ] MRCPass - 2012 January 

A 65 year old patient with colorectal 
cancer has been commenced on a 
chemotherapy regime with Capecitabine and 
oxaliplatin following surgery. 


What is the main difference between 5 FU 
and capecitabine? 


1- Capecitabine is used orally 

2- Capecitabine has a broader indication 


cancers. It is a prodrug, that is enzymatically 
converted to 5-fluorouracil in the tumor, 
where it inhibits DNA synthesis and slows 
growth of tumor tissue. Pancytopenia, 
diarrhoea and hand-foot syndrome are main 
side effects. 



[ Q: 2461 ] MRCPass - 2012 January 

A 61 year old lady presents with a 
flu like illness and cough. She had not 
improved despite amoxicillin prescribed by 
the GP over the last week. On examination, 
she had a temperature of 38 C and was 
tachycardic. Respiratory examination and 
chest X ray confirmed right middle lobe 
consolidation. Antibodies to mycoplasma 
pneumonia was sent and repeated a week 
later, demonstrating increasing titres. 

She is allergic to macrolides so she could not 
be given erythromycin. 

Which one of the following antibiotics is a 
good alternative? 

1- Ceftriaxone 


2- Tazocin 


3- Gentamicin 

4- Linezolid 

5- Doxycycline 


Answer & Comments 


Answer: 5- Doxycycline 


3- Capecitabine causes peripheral neuropathy 

4- 5 FU has less side effects 

5- 5 FU interacts with warfarin 


Answer & Comments 

Answer: 1- Capecitabine is used orally 

Capecitabine is the oral equivalent of 
intravenous 5 -FU. Capecitabine (Xeloda, 
Roche) is an orally-administered 
chemotherapeutic agent used in the 
treatment of metastatic breast and colorectal 


The patient may have developed antibiotic 
related diarrhoea. Mycoplasma infection can 
be treated with macrolides (erythromycin), 
ciprofloxacin and doxycycline. An antibiotic 
with narrower spectrum of action here is 
doxycycline. 


[ Q: 2462 ] MRCPass - 2012 January 

A 22 year old woman presents 
bilateral leg weakness and numbness 
following an episode of diarrhoea 2 weeks 
ago. On examination, she has distal weakness 



Dr. Khalid Yusuf El-Zohry - Sohag Teaching Hospital (01118391123) 

Ref MRCPass OE OE 2012 PasTest 2009 PassMedicine 2009 PasTest Exam ReviseMRCP 



990 





























El-zohry MRCP Questions Bank (Port 1) - 2013 


(For my personal use) 


in the hands and legs. Supinator and ankle 
reflexes were difficult to elicit. 

Which of the following should be used to 
monitor her condition? 

1- Postural blood pressure 

2- Chest x ray 


Tests for dominant inferior parietal lobe 
function includes right-left orientation, 
naming fingers, and calculations and 
Gerstmann syndrome describes dominant 
lobe signs. A mnemonic for the signs is ALF 
(acalculia / agraphia, left right disorientation 
and finger agnosia). 


3- Vital capacity 

4- PEFR 

5- FEV1/FVC ratio 


Answer & Comments 

Answer: 3- Vital capacity 

The patient has distal motor and sensory 
nerve involvement following infection, hence 
the likely diagnosis is acute inflammatory 
demyelinating polyneuropathy (Guillain Barre 
Syndrome). Vital capacity is used to monitor 
for respiratory distress due to neuromuscular 
weakness in Guillain Barre syndrome. Normal 
values are usually 3-6 litres, varying with age, 
gender and height. 


[ Q: 2463 ] MRCPass - 2012 January 

A 45 year old right- handed woman 
presented with difficulty reading. She was 
investigated with a CT brain which showed 
right sided parietal lobe infarction. 



The non-dominant parietal lobe is important 
for visual spatial sensory tasks such as 
attending to the contralateral side of the 
body and space as well as constructional tasks 
such as drawing a face, clock or geometric 
figures. A non dominant lesion leads to visual 
inattention and dyspraxia (unable to 
coordinate motor tasks). 

[ Q: 2464 ] MRCPass - 2012 January 

A 36 year old African man with 
known HIV infection presents with several 
episodes of seizures. His CD4 count was 130 
cells/mm * 1 2 3 4 when measured 1 month ago. On 
examination, he had a temperature of 38 °C 
and was confused with an MTS score of 5/10. 
There were no focal neurological signs. He 
had an MRI scan which shows multiple ring 
enhancing lesions. CSF examination showed 
an elevated protein count, lymphocytosis and 
normal glucose levels. The CSF cryptococcal 
antigen was negative. A chest X ray was 
normal. 



Which of the following is likely to be 
contributing to her reading difficulty? 

1- Agraphia 

2- Amnesia 

3- Left right disorientation 

4- Visual inattention 


Whot is the likely diagnosis? 

1- Progressive multifocal 

leukoencephalopathy 

2- Cerebral toxoplasmosis 

3- Cerebral lymphoma 

4- Tuberculosis 


5- Hemianopia 


5- Bacterial meningitis 


Answer & Comments 

Answer: 4- Visual inattention 

This patient is right handed, hence the left 
brain is dominant, and the right parietal lobe 
infarct is in the non dominant hemisphere. 


Answer & Comments 

Answer: 2- Cerebral toxoplasmosis 

The likely diagnosis is cerebral toxoplasmosis 
as there are multiple ring enhancing lesions. 
Lymphoma usually causes single enhancing 


Dr. Khalid Yusuf El-Zohry - Sohag Teaching Hospital (01118391123) . 

Ref MRCPass OE OE 2012 PasTest 2009 PassMedicine 2009 PasTest Exam ReviseMRCP 991 



























El-zohry MRCP Questions Bank (Part 1) - 2013 


(For my personal use) 


lesions and PML is less frequently ring 
enhancing. Tuberculosis frequently causes 
significantly low glucose levels. 



Cerebral Toxoplasmosis 


[ Q: 2465 ] MRCPass - 2012 January 

A 36 year old woman lives alone. 
She has been unable to work because she 
begins to fear that the other co-w orkers are 
plotting against her. She also fears leaving the 
home, due to embarrassment. She has 
become increasingly anxious and withdrawn, 
and refused to eat with anyone else. Then she 
began receiving mental messages when she 
watched television, so she withdrew from the 
living area as well. She was finally hospitalized 
after she barricaded herself in her bedroom. 

What is the diagnosis? 

1- Obsessive compulsive disorder 

2- Personality disorder 

3- Paranoid schizophrenia 

4- Bipolar disorder 

5- Hypochondriasis 



Answer & Comments 


Answer: 3- Paranoid schizophrenia 


This patient has paranoid feelings that others 
are plotting against her and has emotional 
embarrassment which is out of context. She 
also has delusions, hence fits the diagnosis of 
schizophrenia. 


[ Q: 2466 ] MRCPass - 2012 January 

A 60-year-old man has a history of 
aortic stenosis. He developed symptoms of 
shortness of breath and was referred into 
hospital. On examination, he had a raised JVP, 
bilateral basal crepitations in the lung and 
moderate ankle oedema. He had a harsh 
systolic murmur in the aortic area and a soft 
second heart sound. The ECG showed sinus 
rhythm with left ventricular hypertrophy. An 
echocardiogram showed severe aortic 
stenosis and heavy valvular calcification. 
There was mild aortic regurgitation. Left 
ventricular ejection fraction was 45%. 

Which one of the following is the most 
significant indicator of poor prognosis? 

1- Aortic valve calcification 

2- LVH on the ECG 

3- Aortic regurgitation 

4- Left ventricular failure 

5- Atrial fibrillation 



Answer & Comments 

Answer: 4- Left ventricular failure 

There are many factors to consider in the 
assessment of aortic stenosis, the valve 
gradient on echocardiogram, left ventricular 
function, pulse character, and symptoms of 
the patient. In the list of features above, the 
most significant is the development of left 
ventricular failure which suggests that the 
patient is no longer able to compensate for 
the severe aortic stenosis. 



[ Q: 2467 ] MRCPass - 2012 January 
A 40 year old man is reviewed 4 


Dr. Khalid Yusuf El-Zohry - Sohag Teaching Hospital (01118391123) 

Ref MRCPass OE OE 2012 PasTest 2009 PassMedicine 2009 PasTest Exam ReviseMRCP 



992 

























El-zohry MRCP Questions Bank (Part 1) - 2013 


(For my personal use) 


weeks post renal transplantation. He has 
been on ciclosporin and prednisolone. 

What is the mechanism of action of 
ciclosporin? 

1- Inhibits prostaglandins 

2- Inhibits action of IL-2 

3- Antagonises tetrahydrofolate reductase 

4- Phosphodiesterase 5 inhibition 

5- Podophyllin inhibitor 


Answer & Comments 

Answer: 3- Acute hepatitis A infection 

The abrupt onset of fever, fatigue, malaise, 
anorexia, nausea, diarrhea, jaundice and 
abdominal discomfort are consistent with 
acute hepatitis A infection (especially history 
of diarrhoea). Leptospirosis is less likely as 
there is no renal involvement and infectious 
mononucleosis does not usually cause 
diarrhoea. 


Answer & Comments 

^ [ Q: 2469 ] MRCPass - 2012 January 

Answer: 2- Inhibits action of IL-2 

mm 

A 16 year old lady presented to the 


Ciclosporin is a calcineurin inhibitor. It binds 
to the cytosolic protein cyclophilin 
(immunophilin) of T-lymphocytes. This 
complex of ciclosporin and cyclophilin inhibits 
calcineurin, which is responsible for activating 
the transcription of IL-2. 


[ Q: 2468 ] MRCPass - 2012 January 

A 26 year old man has recently been 
to India for a holiday, returning a week ago. 
He presents with fatigue and abdominal 
pains. He gives a history of diarrhoea for 5 
days. On examination, he was jaundiced and 
had tender hepatomegaly. 



cardiology clinic for assessment. She had 
been diagnosed with Turner syndrome. She 
had normal early developmental milestones 
but had multiple ear infections in childhood 
and presented with primary amenorrhea 
later. On examination, she had short stature, 
cubitus valgus and a webbed neck. Cardiac 
examination revealed a soft ejection systolic 
murmur in the aortic area. 

What is the most likely cause of the murmur? 

1- Ventricular septal defect 

2- Bicuspid aortic valve 

3- Aortic stenosis 

4- Pulmonary stenosis 


His investigations show: 

sodium 135 mmol/I, potassium 4.2 mmol/l, 
urea 5 mmol/l, creatinine 100 pmol/l, ALT 
1380 (5-35) U/l, AST 1430 (1-31) U/l, ALP 360 
(20-120) U/l, GGT 320 (4-35) U/l, Bilirubin 35 
(1-22) pmol/l, Albumin 35 (37-49) g/l 

What is the likely diagnosis? 

1- HIV infection 

2- Leptospirosis infection 

3- Acute hepatitis A infection 

4- Acute hepatitis B infection 

5- Infectious mononucleosis 


5- Partial anomalous venous drainage 

Answer & Comments 

Answer: 2- Bicuspid aortic valve 

Normal females have two X chromosomes, 
but in Turner syndrome, one of those sex 
chromosomes is missing or has other 
abnormalities (XO). The most commonly 
observed are congenital obstructive lesions of 
the left side of the heart, leading to reduced 
flow on this side of the heart. This includes 
bicuspid aortic valve and coarctation 
(narrowing) of the aorta.15% of adults with 
Turner's syndrome have bicuspid aortic 


Dr. Khalid Yusuf El-Zohry - Sohag Teaching Hospital (01118391123) . 

Ref MRCPass OE OE 2012 PasTest 2009 PassMedicine 2009 PasTest Exam ReviseMRCP 993 





























El-zohry MRCP Questions Bank (Part 1) - 2013 


(For my personal use) 


valves. 10% of patients with Turner's 
syndrome have coarctation of aorta. 


[ Q: 2470 ] MRCPass - 2012 January 

# A 60-year-old man has developed 
symptoms of shortness of breath and was 
referred into hospital. On examination, he 
had a hyperdynamic pulse, prominent carotid 
pulsations and a displaced apex beat. He had 
a grade 3 early diastolic murmur in the lower 
left sternal edge. 

What is the likely diagnosis? 

1- Mitral stenosis 

2- Mitral regurgitation 

3- Atrial septal defect 

4- Aortic stenosis 

5- Aortic regurgitation 


The organisms found most typically causing 
ongoing infection in bronchiectasis include 
Haemophilus species (47-55% of patients) 
and Pseudomonas species (18-26% of 
patients) 



[ Q: 2472 ] MRCPass - 2012 January 

Which one of these methods allows 
the study of the 3D structure of protein? 

1- Northern blotting 

2- Southern blotting 

3- Western blotting 

4- X ray crystallography 

5- Gel electrophoresis 


Answer & Comments 


Answer: 4- X ray crystallography 


Answer & Comments 

Answer: 5- Aortic regurgitation 

A displaced apex beat, hyperdynamic and 
collapsing pulse, prominent carotid pulsations 
(Corrigan's sign) and early diastolic murmur 
heard best in the lower left sternal edge are 
signs of aortic regurgitation. 


[ Q: 2471 ] MRCPass - 2012 January 

A 36-year-old man who is known to 
have bronchiectasis has a chronic cough. A 
sputum sample is sent from the clinic. 

What organism is most likely to be isolated? 

1- Streptococcus pneumoniae 

2- Klebsiella spp. 

3- Haemophilus influenzae 

4- Moraxella 

5- Pseudomonas aeruginosa 



Answer & Comments 


Answer: 3- Haemophilus influenzae 


X-ray crystallography is a method of 
determining the arrangement of atoms within 
a crystal, in which a beam of X-rays strikes a 
crystal and diffracts into many specific 
directions. The method can reveal the 3D 
structure of many biological molecules, 
including vitamins, drugs, proteins and 
nucleic acids such as DNA. 

The northern blot is a technique used in 
molecular biology research to study gene 
expression by detection of RNA in a sample. 
Northern blotting involves the use of 
electrophoresis to separate RNA samples by 
size, and detection with a hybridization probe 
(either DNA or RNA) complementary to part 
gene sequence. The Western blot is a 
technique involving electrophoresis to detect 
specific proteins in the given sample of tissue 
homogenate or extract (e.g. in HIV testing, or 
to detect prions in Bovine Spongiform 
Encephalopathy). 

Northern blotting detects RNA and Southern 
blotting detects DNA. 


Dr. Khalid Yusuf El-Zohry - Sohag Teaching Hospital (01118391123) 

Ref MRCPass OE OE 2012 PasTest 2009 PassMedicine 2009 PasTest Exam ReviseMRCP 






























El-zohry MRCP Questions Bank (Part 1) - 2013 


(For my personal use) 



[ Q: 2473 ] MRCPass - 2012 January 

A 41-year-old lady was referred to 
an endocrinologist for evaluation of a lump in 
the neck. The patient reported a recent 5-kg 
weight gain and difficulty sleeping and 
lethargy. She mentioned that her mother has 
a problem with hypothyroidism and her son 
who is aged 10 has type I diabetes and is on 
insulin. 


On examination, blood pressure was 110/78 
mm Hg, and resting pulse was 76 beats per 
minute. Upon neck examination, a firm, 
tender thyroid gland was felt, approximately 
twice normal size, with the right lobe slightly 
larger than the left. A thyroid uptake scan 
showed a 24-h thyroid uptake of 32% and a 
homogenous (but asymmetric) distribution of 
isotope on thyroid scan, the right lobe 
containing more isotope than the left. 

Blood results: Free T3 was 4.5 (4.0-8.0) 
pmol/L, Serum T4 was 5.4 ?g/dL (normal, 5- 
12), serum TSH was 9.5 mU/L (normal, 0.4- 
4.6). High titres of antithyroid peroxidase 
antibodies were found. A fine needle 
aspiration of the nodule was performed, and 
it revealed a mixed population of lymphoid 
cells, mainly small round and small cleaved 
lymphocytes, with numerous plasma cells: 

What is the diagnosis? 

1- Papillary thyroid carcinoma 

2- Post partum thyroiditis 

3- Multinodular goiter 


patient more likely to have Hashimoto's 
thyroiditis. 

A diagnosis of Hashimoto's thyroiditis is 
usually made when positive antithyroid 
peroxidase antibodies are present or 
spontaneous hypothyroidism develops and 
also low thyroglobulin antibody levels. 
Hashimoto's thyroiditis is an autoimmune 
disorder characterised by extensive 
infiltration of the thyroid parenchyma by 
lymphocytes and plasma cells, with the 
formation of germinal centres. Classically, 
presentation is with a goitre or 
hypothyroidism or both. It is the most 
common cause of sporadic goitrous 
hypothyroidism in non-iodine deficient areas. 
The thyroid may be barely palpable or greatly 
enlarged. Characteristically, it is firm, and well 
defined with an enlarged pyramidal lobe and 
palpable neighbouring lymph nodes. 

[ Q: 2474 ] MRCPass - 2012 January 

A 36-year-old woman was admitted 
to hospital with complaints of weakness, back 
pain and difficulty walking. 

She had been treated for epilepsy for 20 
years, and was being on a combination 
therapy of carbamazepine and valproic acid, 
which she had been taking for approximately 
10 years. The seizures were well controlled 
with these drugs. She had a normal diet, was 
independent and worked in a restaurant. She 
had a normal social life. 



4- Grave's disease 

5- Hashimoto's thyroiditis 

Answer & Comments 

Answer: 5- Hashimoto's thyroiditis 

In view of the family history of autoimmune 
disease, the diagnosis here is likely to be 
either Grave's disease or autoimmune 
thyroiditis (Hashimoto's thyroiditis). The 
associated hypothyroid picture makes this 


Physical examination indicated that the 
patient was well-built and had no skeletal 
deformity. Neck and lumbosacral movements 
were slightly limited. In laboratory 
examinations, full blood cell count and renal 
function were normal. 

Biochemical analyses showed: serum alkaline 
phosphatase (AP) 2670 U/L (normal 15-270), 
calcium 2.10 (2.25-2.7) mol/l, albumin 4.6 
g/dL (3.5-5.0), and phosphate 0.75 (0.8-8) 
pmol/l. Serum 25-hydroxy (OH)-vitamin D 
concentration was 7 ng/mL (10-40). 


Dr. Khalid Yusuf El-Zohry - Sohag Teaching Hospital (01118391123) 

Ref MRCPass OE OE 2012 PasTest 2009 PassMedicine 2009 PasTest Exam ReviseMRCP 























El-zohry MRCP Questions Bank (Part 1) - 2013 


(For my personal use) 


Parathormone (PTH) concentration was 1371 
pg/mL (12-72). X rays revealed minimal 
narrowing and irregularity of the sacroiliac 
joint, with marked osteoporosis and 
biconcave vertebrae. 

What is the underlying cause? 

1- Dietary vitamin D deficiency 

2- Decreased sun exposure 

3- Chronic inactivity 

4- Chronic renal failure 

5- Drug induced osteomalacia 

Answer & Comments 

Answer: 5- Drug induced osteomalacia 

Based on the history of prolonged 
anticonvulsant use, low serum calcium, and 
25-OH-vitamin D concentrations with high 
PTH, the most likely diagnosis is 
anticonvulsant-induced osteomalacia. 


atrophy bilaterally and also gynaecomastia. 
There was also sparse facial and body hair. A 
series of blood tests were conducted 
revealing the following results; 

prolactin concentration 320 mu/I (50-450) 

testosterone 4 (11-36) nmol/L 

LH 6 (0.5-9) IU/L 

FSH 3 (1-8 ) IU/L 

Early morning cortisol 500 (130-690) nmol/L 
What is the likely diagnosis? 

1- Klinefelter's syndrome 

2- Hypopituitarism 

3- Kallman's syndrome 

4- Achondroplasia 

5- Turner's syndrome 

Answer & Comments 

Answer: 1- Klinefelter's syndrome 


A normal social life and employment suggests 
normal amounts of sun exposure. 
Hypocalcemia, increased serum alkaline 
phosphatase, and decrease in the 25-OH- 
vitamin D concentrations are hallmarks. 
Phenobarbitone, phenytoin, carbamazepine 
and also sodium valproate can cause 
osteomalacia. 

25-OH-vitamin D (alphacalcidol 1.5 ?g/d) and 
calcium (elementary calcium 2 g/d) 
supplementations should be initiated. 
l,25(OH)2 vitamin D is more difficult and 
expensive to measure than 25(OH)D; 
moreover, it is not a good measure of vitamin 
D status. 


[ Q: 2475 ] MRCPass - 2012 January 

A 25 year old man has presented for 
evaluation for right sided gynaecomastia. He 
has had a mastectomy on the left side 2 years 
ago and is concerned that it is now recurring. 
Physical examination revealed a slim tall man 
with a height of 1.86 m. There was testicular 



Klinefelter's syndrome (XXY) causes testicular 
atrophy, which commonly leads to 
gynecomastia and infertility. 

Androgen deficiency (in this case low 
testosterone) causes eunuchoid body 
proportions; sparse or absent. 

facial, axillary, pubic, or body hair; decreased 
muscle mass and strength; feminine 
distribution of adipose tissue; gynecomastia; 
small testes and penis. Most 47,XXY males 
have normal intelligence. 

The blood tests show normal levels of LH and 
FSH hence making this unlikely to be 
Kallman's syndrome (hypothlamic 
gonadotrophin releasing hormone deficiency) 


Dr. Khalid Yusuf El-Zohry - Sohag Teaching Hospital (01118391123) 

Ref MRCPass OE OE 2012 PasTest 2009 PassMedicine 2009 PasTest Exam ReviseMRCP 



996